Está en la página 1de 740

Problem Books in Mathematics

VladimirV.Tkachuk

A Cp-Theory
Problem
Book
Functional Equivalencies

Problem Books in Mathematics


Series Editor:
Peter Winkler
Department of Mathematics
Dartmouth College
Hanover, NH 03755
USA

More information about this series at http://www.springer.com/series/714

Vladimir V. Tkachuk

A Cp-Theory Problem Book


Functional Equivalencies

123

Vladimir V. Tkachuk
Departamento de Matematicas
Universidad Autonoma Metropolitana-Iztapal
Mexico, Mexico

ISSN 0941-3502
ISSN 2197-8506 (electronic)
Problem Books in Mathematics
ISBN 978-3-319-24383-2
ISBN 978-3-319-24385-6 (eBook)
DOI 10.1007/978-3-319-24385-6
Library of Congress Control Number: 2015958805
Mathematics Subject Classification (2010): 54C35
Springer Cham Heidelberg New York Dordrecht London
Springer International Publishing Switzerland 2016
This work is subject to copyright. All rights are reserved by the Publisher, whether the whole or part of
the material is concerned, specifically the rights of translation, reprinting, reuse of illustrations, recitation,
broadcasting, reproduction on microfilms or in any other physical way, and transmission or information
storage and retrieval, electronic adaptation, computer software, or by similar or dissimilar methodology
now known or hereafter developed.
The use of general descriptive names, registered names, trademarks, service marks, etc. in this publication
does not imply, even in the absence of a specific statement, that such names are exempt from the relevant
protective laws and regulations and therefore free for general use.
The publisher, the authors and the editors are safe to assume that the advice and information in this book
are believed to be true and accurate at the date of publication. Neither the publisher nor the authors or
the editors give a warranty, express or implied, with respect to the material contained herein or for any
errors or omissions that may have been made.
Printed on acid-free paper
Springer International Publishing AG Switzerland is part of Springer Science+Business Media (www.
springer.com)

Preface

This is the fourth and the last volume of the series of books of problems in Cp theory entitled A Cp -Theory Problem Book, i.e., this book is a continuation of
the three volumes subtitled Topological and Function Spaces, Special Features of
Function Spaces and Compactness in Function Spaces. The series was conceived as
an introduction to Cp -theory with the hope that each volume could also be used as
a reference guide for specialists.
The first volume provides a self-contained introduction to general topology and
Cp -theory and contains some highly nontrivial state-of-the-art results. For example,
Section 1.4 presents Shapirovskys theorem on existence of a point-countable base in any compact space of countable tightness, and Section 1.5 brings the reader
to the frontier of the modern knowledge about realcompactness in the context of
function spaces.
The second volume covers a wide variety of topics in Cp -theory and general
topology at the professional level bringing the reader to the frontiers of modern
research. It presents, among other things, a self-contained introduction to Descriptive Set Theory and Advanced Set Theory providing a basis for working with most
popular axioms independent of ZFC.
The third volume introduces the reader to compactness and its generalizations in
the context of function spaces. It continues dealing with topology and Cp -theory at
a professional level. The main objective is to develop from the very beginning the
theory of compact spaces most used in functional analysis, i.e., Corson compacta,
Eberlein compacta, and Gulko compacta.
This volume presents a reasonably complete and up-to-date information on
preservation of topological properties by homeomorphisms of function spaces. An
exhaustive theory of t -equivalent, u-equivalent and l-equivalent spaces is developed
from scratch. Since the policy of the author is to make this book self-contained,
the reader will find here an introduction to the theory of uniform spaces, the theory
of locally convex spaces as well as to the theory of inverse systems and dimension
theory. The above-mentioned policy also made it necessary to include Kolmogorovs
solution of Hilberts Problem 13 since it is needed for the presentation of the theory
of l-equivalent spaces.
v

vi

Preface

The authors intention was to include in this volume all classical results on
functional equivalencies. In particular, we present the famous theorem of Gulko
and Khmyleva on non-preservation of compactness by t -equivalence as well as
Okunevs results on t -invariance of spread, hereditary density, hereditary Lindelf
number and  -compactness. Of course, it was impossible to omit the fundamental
result of Gulko on preservation of the dimension dim by u-equivalence, a deep
theorem of Marciszewski which states that I and I! are not t -equivalent as well as
the Bouziads result on preservation of the Lindelf number by l-equivalence.
We apply here all topological methods developed in the first three volumes, so we
refer to their problems and solutions when necessary; the abbreviation for the first
volume is TFS, and we will use the expressions SFFS and CFS to refer to the second
and third volumes, respectively. For example, TFS-273 refers to Problem 273 of
the book TFS. The references to the solutions are not that straightforward: the
abbreviation S.115 means solution of Problem 115 of the book TFS, while T.025
stands for solution of Problem 025 of the book SFFS. The expression U.249
abbreviates the phrase solution of Problem 249 of the book CFS, and, finally,
V.411 denotes the solution of Problem 411 of this volume. The author did his best
to keep every solution as independent as possible, so a short argument could be
repeated several times in different places.
The author wants to emphasize that if a postgraduate student mastered the
material of the first three volumes, it will be more than sufficient to understand
every problem and solution of this book. However, for a concrete topic much less
might be needed. Finally, the author outlines some points which show the potential
usefulness of this work.
The only background needed is some knowledge of set theory and real numbers;
any reasonable course in calculus covers everything needed to understand this
book.
The student can learn all of general topology required without recurring to any
textbook or papers; the amount of general topology is strictly minimal and is
presented in such a way that the student works with the spaces Cp .X / from the
very beginning.
What is said in the previous paragraph is true as well if a mathematician
working outside of topology (e.g., in functional analysis) wants to use results
or methods of Cp -theory; he (or she) will find them easily in a concentrated
form or with full proofs if there is such a need.
The material we present here is up to date and brings the reader to the frontier
of knowledge in a reasonable number of important areas of Cp -theory.
This book seems to be the first self-contained introduction to Cp -theory.
Although there is an excellent textbook written by Arhangelskii (1992a), it
heavily depends on the readers good knowledge of general topology.
Mexico City, Mexico

Vladimir V. Tkachuk

Contents

Properties Preserved by Homeomorphisms


of Function Spaces . . . . . . . . . . . . . . . . . . . . . . . . . . . . . . . . . . . . . . . . . . . . . . . . . . . . . . . . . . .
1.1 Equivalences that arise from homeomorphisms of Cp .X/ . . . . . . . . . . . .
1.2 Uniformities, Dimension, and u-Equivalence . . . . . . . . . . . . . . . . . . . . . . . . .
1.3 Linear Topological Spaces and l-Equivalence . . . . . . . . . . . . . . . . . . . . . . . . .
1.4 Metrizable Spaces and l-Equivalence . . . . . . . . . . . . . . . . . . . . . . . . . . . . . . . . . .
1.5 The Last-Minute Updates. Yet More on l-Equivalence . . . . . . . . . . . . . . .
1.6 Bibliographic notes to Chapter 1 . . . . . . . . . . . . . . . . . . . . . . . . . . . . . . . . . . . . . . .

1
4
13
25
37
47
61

Solutions of problems 001500 . . . . . . . . . . . . . . . . . . . . . . . . . . . . . . . . . . . . . . . . . . . . .

63

Bonus results: Some Hidden Statements . . . . . . . . . . . . . . . . . . . . . . . . . . . . . . . . .


3.1 Standard spaces . . . . . . . . . . . . . . . . . . . . . . . . . . . . . . . . . . . . . . . . . . . . . . . . . . . . . . . . .
3.2 Compact spaces and their generalizations . . . . . . . . . . . . . . . . . . . . . . . . . . . . .
3.3 Properties of continuous maps. . . . . . . . . . . . . . . . . . . . . . . . . . . . . . . . . . . . . . . . . .
3.4 Cardinal invariants and set theory . . . . . . . . . . . . . . . . . . . . . . . . . . . . . . . . . . . . . .
3.5 Locally Convex Spaces and Homotopies . . . . . . . . . . . . . . . . . . . . . . . . . . . . . .
3.6 Zero-dimensional Spaces and Connected Spaces . . . . . . . . . . . . . . . . . . . . .
3.7 Raznoie (Unclassified results) . . . . . . . . . . . . . . . . . . . . . . . . . . . . . . . . . . . . . . . . . .

637
639
641
642
643
644
645
646

Open problems . . . . . . . . . . . . . . . . . . . . . . . . . . . . . . . . . . . . . . . . . . . . . . . . . . . . . . . . . . . . . . .
4.1 Mappings which involve Cp -spaces . . . . . . . . . . . . . . . . . . . . . . . . . . . . . . . . . . . .
4.2 Properties preserved by t-equivalence . . . . . . . . . . . . . . . . . . . . . . . . . . . . . . . . . .
4.3 Properties preserved by u-equivalence . . . . . . . . . . . . . . . . . . . . . . . . . . . . . . . . .
4.4 Properties preserved by l-equivalence . . . . . . . . . . . . . . . . . . . . . . . . . . . . . . . . .
4.5 Generalizations of functional equivalences . . . . . . . . . . . . . . . . . . . . . . . . . . . .
4.6 Fuzzy questions . . . . . . . . . . . . . . . . . . . . . . . . . . . . . . . . . . . . . . . . . . . . . . . . . . . . . . . . .
4.7 Raznoie (unclassified questions) . . . . . . . . . . . . . . . . . . . . . . . . . . . . . . . . . . . . . . .

649
650
652
655
656
658
661
663

Bibliography . . . . . . . . . . . . . . . . . . . . . . . . . . . . . . . . . . . . . . . . . . . . . . . . . . . . . . . . . . . . . . . . . . . . . . 665
List of special symbols . . . . . . . . . . . . . . . . . . . . . . . . . . . . . . . . . . . . . . . . . . . . . . . . . . . . . . . . . . . 717
Index . . . . . . . . . . . . . . . . . . . . . . . . . . . . . . . . . . . . . . . . . . . . . . . . . . . . . . . . . . . . . . . . . . . . . . . . . . . . . . . 721
vii

Detailed Summary of Exercises

1.1. Equivalencies That Arise From Homeomorphisms


of Cp .X/.
Easy facts on t -invariance . . . . . . . . . . . . . . . . . . . . . . . . . . . . . . . . . Problems 001015.
Compactness is not preserved by t -equivalence . . . . . . . . . . . . . . . Problems 016027.
On t -equivalence of X and Y . . . . . . . . . . . . . . . . . . . . . . . . . . . . Problems 028029
Monolithity and stability are t -invariant . . . . . . . . . . . . . . . . . . . . . . . . . . . Problem 030
Functionally perfect spaces and t -equivalence . . . . . . . . . . . . . . . . Problems 031033
Okunevs method for proving t -invariance . . . . . . . . . . . . . . . . . . . Problems 034046
Zero-dimensional spaces; homeomorphisms of Cp .X; D/ . . . . . Problems 047051
A couple of facts on Cp .P/ . . . . . . . . . . . . . . . . . . . . . . . . . . . . . . . . . Problems 052053
Compactness is preserved by a-equivalence . . . . . . . . . . . . . . . . . . . . . . . Problem 054
t -invariance of spread, hl and hd . . . . . . . . . . . . . . . . . . . . . . . . . . . Problems 055070
Tightness and sequentiality in compact spaces . . . . . . . . . . . . . . . . Problems 071077
Tightness in compact spaces is preserved by t -equivalence . . . . . . . . . . Problem 078
Sequentiality in compact spaces and t -equivalence . . . . . . . . . . . . . . . . . Problem 079
Metrizable t -equivalent spaces . . . . . . . . . . . . . . . . . . . . . . . . . . . . . . Problems 080083
A first glance at simplexes . . . . . . . . . . . . . . . . . . . . . . . . . . . . . . . . . Problems 084087
Sperners lemma . . . . . . . . . . . . . . . . . . . . . . . . . . . . . . . . . . . . . . . . . . . . . . . . Problem 088
Browers fixed point theorem and its applications . . . . . . . . . . . . . Problems 089090
Homotopies and Mushroom lemma . . . . . . . . . . . . . . . . . . . . . . . . . Problems 091093
Partitions in finite-dimensional cubes and in Hilbert cube . . . . . . Problems 094095
The Hilbert cube is not t -equivalent to any In . . . . . . . . . . . . . . . . Problems 096099
The space Cp .X / need not be homeomorphic to its square . . . . . . . . . . Problem 100

1.2. Uniformities, Dimension, and u-Equivalence.


Basic facts about uniformities . . . . . . . . . . . . . . . . . . . . . . . . . . . . . . Problems 101134
ix

Detailed Summary of Exercises

Cp .X / is complete as a uniform space iff X is discrete . . . . . . . . . . . . . Problem 135


Preservation of pseudocompactness by u-equivalence . . . . . . . . . . . . . . Problem 136
u-equivalence is strictly stronger than t -equivalence . . . . . . . . . . . . . . . . Problem 137
Preservation of compactness by u-equivalence . . . . . . . . . . . . . . . . . . . . . Problem 138
u-equivalence implies a-equivalence . . . . . . . . . . . . . . . . . . . . . . . . . . . . . . Problem 139
Basic facts about the dimension dim . . . . . . . . . . . . . . . . . . . . . . . . Problems 140164
Basic facts about inverse systems . . . . . . . . . . . . . . . . . . . . . . . . . . . Problems 165175
Preservation of the dimension dim by u-equivalence . . . . . . . . . . Problems 176182
Absolute Borel sets and u-equivalence . . . . . . . . . . . . . . . . . . . . . . . Problems 183198
Infinite countable compact spaces are u-equivalent . . . . . . . . . . . . Problems 199200

1.3. Linear Topological Spaces and l-Equivalence.


General facts about linear topological spaces . . . . . . . . . . . . . . . . . Problems 201232
The topologies of Cp .X / and Lp .X / are weak . . . . . . . . . . . . . . . . . . . . Problem 233
General properties of Lp .X / . . . . . . . . . . . . . . . . . . . . . . . . . . . . . . . Problems 234250
R-quotient maps and contracting a set to a point . . . . . . . . . . . . . . Problems 251255
Constructing pairs of l-equivalent spaces . . . . . . . . . . . . . . . . . . . . Problems 256262
Non-preservation of some properties by l-equivalence . . . . . . . . . . . . . . Problem 263
One-point compactifications of discrete spaces . . . . . . . . . . . . . . . . . . . . . Problem 264
Function spaces of discrete unions and l-equivalence . . . . . . . . . . . . . . . Problem 265
Finite discrete unions of intervals and finite sets . . . . . . . . . . . . . . . . . . . Problem 266
Alexandroff double and l-equivalence . . . . . . . . . . . . . . . . . . . . . . . Problems 267268
Some more properties that are not l-invariant . . . . . . . . . . . . . . . . Problems 269271
l-equivalence of products . . . . . . . . . . . . . . . . . . . . . . . . . . . . . . . . . . Problems 272276
Hereditary normality is not preserved by l-equivalence . . . . . . . . . . . . . Problem 277
Souslin property is not l-invariant . . . . . . . . . . . . . . . . . . . . . . . . . . . . . . . . Problem 278
Continuous linear surjections of function spaces . . . . . . . . . . . . . . Problems 279282
-spaces and l-equivalence . . . . . . . . . . . . . . . . . . . . . . . . . . . . . . . . Problems 283286

Cech-completeness
and hyperspaces . . . . . . . . . . . . . . . . . . . . . . . . . . . . . . Problem 287

Cech-completeness
and l-equivalence . . . . . . . . . . . . . . . . . . . . . . . Problems 288289
Condensations on  -compact spaces and l-equivalence . . . . . . . . . . . . . Problem 290

Cech-completeness
in countable spaces . . . . . . . . . . . . . . . . . . . . . . . . . . . Problem 291
A criterion of l-equivalence for metrizable spaces . . . . . . . . . . . . . . . . . . Problem 292
Second countable spaces l-equivalent to standard ones . . . . . . . . Problems 293300

1.4. Metrizable Spaces and l-Equivalence.


More properties not preserved by l-equivalence . . . . . . . . . . . . . . Problems 301307

On Cech-complete
l-equivalent spaces . . . . . . . . . . . . . . . . . . . . . . Problems 308310
Basic facts about L.X / . . . . . . . . . . . . . . . . . . . . . . . . . . . . . . . . . . . . Problems 311316

Detailed Summary of Exercises

xi

bf -spaces and Cb .X / . . . . . . . . . . . . . . . . . . . . . . . . . . . . . . . . . . . . . . Problems 317319


A characterization of the topology of L.X / . . . . . . . . . . . . . . . . . . . . . . . Problem 320
Continuous linear functionals on Ck .X / . . . . . . . . . . . . . . . . . . . . . . . . . . Problem 321
Some properties of barreled spaces . . . . . . . . . . . . . . . . . . . . . . . . . . Problems 322326
Linear maps between Cb .X / and Cp .X / . . . . . . . . . . . . . . . . . . . . . . . . . . Problem 327
bf -spaces, l-equivalence, and L-equivalence . . . . . . . . . . . . . . . . . Problems 328331
Weaker metrizable topology on X and L.X / . . . . . . . . . . . . . . . . . . . . . . Problem 332
Spaces l-equivalent to metrizable ones . . . . . . . . . . . . . . . . . . . . . . Problems 333335
Hemicompactness, k-property, and l-equivalence . . . . . . . . . . . . . Problems 336340
General properties of @0 -spaces and l-equivalence . . . . . . . . . . . . Problems 341351
BanachMazur game, category, and Baire property . . . . . . . . . . . Problems 352353
GruenhageMa game and moving off property . . . . . . . . . . . . . . . Problems 354358
q-spaces and l-equivalence . . . . . . . . . . . . . . . . . . . . . . . . . . . . . . . . . Problems 359361
l-equivalence to metrizable spaces . . . . . . . . . . . . . . . . . . . . . . . . . . Problems 362367
l-equivalence in first countable spaces . . . . . . . . . . . . . . . . . . . . . . . Problems 368369
Linear factors of Cp .X / . . . . . . . . . . . . . . . . . . . . . . . . . . . . . . . . . . . Problems 370376
Universality in dimension n and l-equivalence . . . . . . . . . . . . . . . Problems 377378
l-equivalence to finite-dimensional cubes . . . . . . . . . . . . . . . . . . . . Problems 379388
Linear factors of linear topological spaces . . . . . . . . . . . . . . . . . . . Problems 389392
l-equivalence to a finite power of the real line . . . . . . . . . . . . . . . . Problems 393394
Linear factors of Cp .X / and dimension dim . . . . . . . . . . . . . . . . . Problems 395396
Open Mapping theorem for Banach spaces . . . . . . . . . . . . . . . . . . . . . . . . Problem 397
Closed Graph theorem for Banach spaces . . . . . . . . . . . . . . . . . . . . . . . . . Problem 398
Adding an isolated point to a space and l-equivalence . . . . . . . . . Problems 399400

1.5. The Last-Minute Updates. Yet More on l-Equivalence.


Order of -bases in Cp .X / . . . . . . . . . . . . . . . . . . . . . . . . . . . . . . . . . Problems 401412
General properties of the point-open game . . . . . . . . . . . . . . . . . . . Problems 413416
Point-open game in X and the W -property in Cp .X / . . . . . . . . . . . . . . . Problem 417
A W -space Cp .X / without a point-countable -base . . . . . . . . . . . . . . . Problem 418
General properties of d -separable spaces . . . . . . . . . . . . . . . . . . . . Problems 419421
d -separability in Cp .X / . . . . . . . . . . . . . . . . . . . . . . . . . . . . . . . . . . . Problems 422429
Continuous images of dense subspaces of products . . . . . . . . . . . Problems 430437
-cosmic and strongly -cosmic spaces . . . . . . . . . . . . . . . . . . . . . . Problems 438442
Compact continuous images of Cp .X / . . . . . . . . . . . . . . . . . . . . . . Problems 443450
Condensations of Cp .X / onto compact spaces . . . . . . . . . . . . . . . Problems 451457
Condensations of Cp .X / onto  -compact spaces . . . . . . . . . . . . . Problems 458461
Metalindelfness of Cp .X / . . . . . . . . . . . . . . . . . . . . . . . . . . . . . . . . Problems 462463
Monotonically monolithic spaces and D-spaces . . . . . . . . . . . . . . Problems 464466
Monotone -monolithity vs monotone -stability . . . . . . . . . . . . . . . . . . Problem 467
Monotone monolithity in Cp .X / . . . . . . . . . . . . . . . . . . . . . . . . . . . . Problems 468469
Monotonically -monolithic spaces and D-property . . . . . . . . . . Problems 470472

xii

Detailed Summary of Exercises

Corson compacta and monotone !-monolithity . . . . . . . . . . . . . . . Problems 473474


Monotone !-monolithity and caliber !1 . . . . . . . . . . . . . . . . . . . . . . . . . . Problem 475
Monotonically retractable/Sokolov spaces . . . . . . . . . . . . . . . . . . . Problems 476487
Closure-preserving local bases in Cp .X / . . . . . . . . . . . . . . . . . . . . . . . . . . Problem 488
 -discrete networks in Cp .X / . . . . . . . . . . . . . . . . . . . . . . . . . . . . . . Problems 489491
Hereditary Baire property of Cp .! / . . . . . . . . . . . . . . . . . . . . . . . . . . . . . . Problem 492
l-equivalence of ! and ! implies ! ' ! . . . . . . . . . . . . . . . . . . . . . . Problem 493
Countable compact spaces and l-equivalence . . . . . . . . . . . . . . . . . . . . . . Problem 494
Linear surjections of Cp and zero-dimensionality . . . . . . . . . . . . . . . . . . Problem 495
No continuous linear surjection of Cp .I/ onto Cp .I! / . . . . . . . . . . . . . . Problem 496
A characterization of l-equivalence to the Cantor set . . . . . . . . . . . . . . . Problem 497
Kolomogorovs solution of Hilberts Problem 13 . . . . . . . . . . . . . . . . . . . Problem 498
Linear surjections of Cp .I/ onto Cp .X / for compact X . . . . . . . . . . . . . Problem 499
Lindelf number is preserved by l-equivalence . . . . . . . . . . . . . . . . . . . . Problem 500

Introduction

The term Cp -theory was invented to abbreviate the phrase the theory of function
spaces endowed with the topology of pointwise convergence. The credit for the
creation of Cp -theory must undoubtedly be given to Alexander Vladimirovich
Arhangelskii. The author is proud to say that Arhangelskii also was the person
who taught him general topology and directed his PhD thesis. Arhangelskii was
the first to understand the need to unify and classify a bulk of heterogeneous results
from topological algebra, functional analysis and general topology. He was the first
to obtain crucial results that made this unification possible. He was also the first
to formulate a critical mass of open problems which showed this theorys huge
potential for development.
Later, many mathematicians worked hard to give Cp -theory the elegance and
beauty it boasts nowadays. The author hopes that the work he presents for the
readers judgement will help to attract more people to this area of mathematics.
The main text of this volume consists of 500 statements formulated as problems;
it constitutes Chapter 1. These statements provide a gradual development of many
popular topics of Cp -theory to bring the reader to the frontier of the present-day
knowledge. A complete solution is given to every problem of the main text.
The material of Chapter 1 is divided into five sections with 100 problems in each
one. The sections start with an introductory part where the definitions and concepts
to be used are given. The introductory part of any section never exceeds two pages
and covers everything that was not defined previously. Whenever possible, we try
to save the reader the effort of ploughing through various sections, chapters and
volumes, so we give the relevant definitions in the current section not caring much
about possible repetitions.
Chapter 1 ends with some bibliographical notes to give the most important
references related to its results. The selection of references is made according to
the authors preferences and by no means can be considered complete. However,
a complete list of contributors to the material of this book can be found in our
bibliography of 800 items. It is the authors pleasant duty to acknowledge that
he consulted the paper of Arhangelskii (1998a) to include quite a few of its 375
references in his bibliography.
xiii

xiv

Introduction

Sometimes, as we formulate a problem, we use without reference definitions and


constructions introduced in other problems. The general rule is to try to find the
relevant definition not more than ten problems before.
The complete solutions of all problems of Chapter 1 are given in Chapter 2.
Chapter 3 begins with a selection of 80 statements which were proved as auxiliary
facts in the solutions of the problems of the main text. This material is split into 7
sections to classify the respective results and make them easier to find. Chapter 4
consists of 100 open problems presented in 7 sections with the same idea: to classify
this bulk of problems and make the readers work easier.
Chapter 4 also witnesses an essential difference between the organization of our
text and the book by Arhangelskii and Ponomarev (1974): we never put unsolved
problems in the main text as is done in their book. All problems formulated in
Chapter 1 are given complete solutions in Chapter 2, and the unsolved ones are
presented in Chapter 4.
There is little to explain about how to use this book as a reference guide. In this
case the methodology is not that important, and the only thing the reader wants
is to find the results he (or she) needs as fast as possible. To help with this, the
titles of chapters and sections give the first approximation. To better see the material
of a chapter, one can consult the second part of the Contents section where a
detailed summary is given; it is supposed to cover all topics presented in each
section. Besides, the index can also be used to find necessary material.
To sum up the main text, the author believes that the coverage of Cp -theory
will be reasonably complete and many of the topics can be used by postgraduate
students who want to specialize in Cp -theory. Formally, this book can also be used
as an introduction to general topology. However, it would be a somewhat biased
introduction, because the emphasis is always given to Cp -spaces and the topics are
only developed when they have some applications in Cp -theory.
To conclude, let the author quote an old saying which states that the best way
for one to learn a theorem is to prove it oneself. This text provides a possibility
to do this. If the readers wish is to read the proofs, there they are concentrated
immediately after the main text.

Chapter 1

Properties Preserved by Homeomorphisms


of Function Spaces

The reader who has found his (or her) way through the first fifteen hundred problems
of this book is fully prepared to enjoy working professionally in Cp -theory. Such a
work implies choosing a topic, reading the papers with the most recent progress
thereon, and attacking the unsolved problems. Now the first two steps are possible
without doing heavy library work, because Chapter 1 provides information on
the latest advances in all areas of Cp -theory, where functional equivalencies are
concerned. Here, many ideas, results, and constructions came from functional
analysis and the theory of uniform spaces giving a special flavor to this part of
Cp -theory, but at the same time making it more difficult to master. I must warn the
reader that most topics, outlined in the forthcoming bulk of 500 problems, constitute
the material of important research papersin many cases very difficult ones. The
proofs and solutions, given in Chapter 2, are complete, but sometimes they require a
very high level of understanding of the matter. The reader should not be discouraged
if some proofs seem to be unfathomable. We still introduce new themes in general
topology and formulate, after a due preparation, some nontrivial results which might
be later used in Cp -theory.
This volume presents a very popular line of research in Cp -theory. The objective
here is to find common features of the spaces X and Y knowing that Cp .X / and
Cp .Y / are similar in some way. Theorem of Nagata (Problem TFS-200) gives
a complete solution if the rings Cp .X / and Cp .Y / are topologically isomorphic
because in this case the spaces X and Y must be homeomorphic. It turns out that the
existence of a linear homeomorphism between Cp .X / and Cp .Y / need not imply
that the spaces X and Y are homeomorphic. Nevertheless, X and Y have to share
quite a few important properties even if there exists a nonlinear homeomorphism
between Cp .X / and Cp .Y /.
This volume requires a much broader vision of general topology than the
previous ones because the general scheme of research here is to take some property
(which might come from any area of general topology or even from outside) and
check whether it is shared by the spaces X and Y in case Cp .X / is (linearly or
Springer International Publishing Switzerland 2016
V.V. Tkachuk, A Cp-Theory Problem Book, Problem Books in Mathematics,
DOI 10.1007/978-3-319-24385-6_1

1 Properties Preserved by Homeomorphisms of Function Spaces

uniformly) homeomorphic to Cp .Y /. As a consequence, we had to introduce some


theories which could be the subject of a separate book. In particular, we had to
develop the basic techniques of dealing with uniformities, linear topological spaces,
dimension theory, and topological games. Once accomplished, this made it possible
to present in a self-contained way almost all important results in the area.
If nothing is said about the separation axioms of a space X then X is assumed
to be Tychonoff. Section 1.1 deals with pairs of spaces X; Y for which Cp .X / is
homeomorphic to Cp .Y / (such spaces are called t -equivalent). One of the outstanding results presented here is Gulko and Khmylevas theorem on non-preservation
of compactness by t -equivalence (Problem 027). The theorems on invariance
of spread, hereditary density, hereditary Lindelf number, and  -compactness
(Problems 043, 068070) constitute a breakthrough due to Okunev. Another gem
of this collection is a result of W. Marciszewski which states that I and I! are not
t -equivalent (Problem 099).
Section 1.2 is devoted to the study of pairs of spaces X; Y for which Cp .X /
is uniformly homeomorphic to Cp .Y / (such spaces are called u-equivalent). We
give the reader a glimpse of the theory of uniform spaces. Inverse systems and
dimension theory are also developed to some extent to make it possible to present a
famous result of S. Gulko on invariance of the dimension dim under u-equivalence
(Problem 180). Another important result is a theorem of W. Marciszewski on
preservation of absolute Borel classes by u-equivalence (Problems 197198). We
conclude this section with another beautiful result of S. Gulko: if X and Y are
infinite countable compact spaces then they are u-equivalent (Problem 200).
Section 1.3 starts with a short introduction to linear topological spaces. The
results that follow are intended to give information on pairs of spaces X; Y for which
Cp .X / is linearly homeomorphic to Cp .Y / (such spaces are called l-equivalent). We
present a general method of Okunev for constructing pairs of l-equivalent spaces
(Problem 257) and a classification of spaces l-equivalent to some standard ones
(Problems 293, 295, 297300).
Section 1.4 presents another portion of deep results on l-equivalence. A fact
which could not be omitted here is a theorem of Baars, de Groot, and Pelant

on preservation of Cech-completeness
by l-equivalence in metrizable spaces
(Problem 366). It is a nontrivial theorem of Dranishnikov that any nonempty
open subspace of Rn is l-equivalent to Rn (Problem 394); this Section concludes
with a very difficult example of Marciszewski of an infinite compact space K
for which there exists no continuous linear surjection of Cp .K/ onto Cp .K/  R
(Problem 400).
This book was in preparation for almost ten years and six years passed in the
process of publishing the first three volumes; so quite a few new results in Cp -theory
emerged during this period. The author included them in the book where it was
possible to avoid violation of the existing classification scheme. However, at the
moment of writing Section 1.5 (which was originally planned to cover what was
left from the theory of l-equivalent spaces) many new fundamental results appeared
and they did not fit into any classification at all. That is why Section 1.5 stands
completely apart: it contains the most recent results which could not be left out and
continues the study of functional equivalences.

1 Properties Preserved by Homeomorphisms of Function Spaces

One of the main results here is Bouziads theorem on preservation of the


Lindelf number by l-equivalence (Problem 500). Another one is a theorem of
Leiderman, Levin, and Pestov stating that Cp .I/ can be linearly and continuously mapped onto Cp .X / for any finite-dimensional metrizable compact space
X (Problem 499). Problem 494 gives a complete classification (due to Gulko
and Oskin) of l-equivalent countable compact spaces. A fundamental result of
Marciszewski and Pol states that if X is a linearly orderable separable compact
space or a separable dyadic compact space, then Cp .X / has a  -discrete network
(Problems 484 and 485).

1 Properties Preserved by Homeomorphisms of Function Spaces

1.1 Equivalences that arise from homeomorphisms


of Cp .X/
All spaces are assumed to be Tychonoff. Given a space X , the family .X / is its
topology,  .X / D .X /nf;g and .x; X / D fU 2 .X / W x 2 U g for any x 2 X .
The expression X ! Y says that the space X can be embedded in the space Y .
If we write X ' Y , this means that X is homeomorphic to Y . The spaces X
t
and Y are called t -equivalent (which is denoted by X  Y ), if Cp .X / ' Cp .Y /.
A topological property P (a cardinal invariant ') is called t -invariant if it is
t
preserved by t -equivalence, i.e., if X ` P (or '.X /  ) and X  Y imply
Y ` P (or '.Y /   respectively). A subspace Y of a space X is t -embedded
(l-embedded) in X , if there is a (linear) continuous map ' W Cp .Y / ! Cp .X /
such that '.f /jY D f for any f 2 Cp .Y /. If X is a space, then 0X 2 Cp .X / is
the function equal to zero at all points of X . The space I is the set 1; 1
with the
topology inherited from R. A set F  Cp .X; I/ is called D-separating, if 0X 2 F
and, given " > 0, a finite A  X and a closed P  X nA, there is f 2 F such
that f .A/  ."; "/ and f .P /  34 ; 1
. Now, if F  Cp .X; I/ is any set which
contains 0X , let ZF .X / D f' W F ! I W '.0X / D 0 and '.V /   12 ; 12
for some
V 2 .0X ; F /g.
Given a cardinal invariant ', we define '  .X / D supf'.X n / W n 2 Ng. Call a
class of spaces C complete if C is invariant under finite products, countable unions,
closed subspaces, and continuous images. Only in this Section, we denote by K.X /
the smallest complete class which contains the space X and all compact spaces.
Any homeomorphism f W X ! X is called autohomeomorphism. A trivial
example of an autohomeomorphism is the map idX W X ! X defined by
idX .x/ D x for any x 2 X . Given maps f; g 2 C.X; Y /, we say that f and g
are homotopic if there exists a continuous map F W X  0; 1
! Y such that
F .x; 0/ D f .x/ and F .x; 1/ D g.x/ for any x 2 X . The map F is called
the homotopy which connects the mappings f and g. If F W X  0; 1
! Y
is a homotopy, it generates the family fFt W t 2 0; 1
g of functions defined by
Ft .x/ D F .x; t / for any x 2 X and t 2 0; 1
. By exp.X / we denote the family of
all subsets of a set X . If  is a cardinal which can be finite, we will need the families
X
 D fA  X W jAj D g and X
 D fA  X W jAj  g together with
X
< D fA  X W jAj < g. In particular, X
<! is the family of all finite subsets
of X .
If X is a space denote by F.X / the family of all closed subsets of X . For every
open subset U of the space X consider the families S
P .U / D fF 2 F.X / W F  U g
and Q.U / D fF 2 F.X / W F \ U ;g; let S D fP .U / [ Q.U / W U 2 .X /g.
The topology V .X / generated by S as a subbase is called the Vietoris topology on
the family of all closed subset of X . Let CL.X / D .F.X /; V .X //; if G  F.X /,
by the Vietoris topology on G we mean the topology on G inherited from CL.X /.
Given sets X and Y , we say that p is a multi-valued map from X to Y if p W
X ! exp.Y /. The map p is called finite-valued if p.X /  Y
<! and singlevalued if jp.x/j D 1 for any x 2 X . Any single-valued map p W X ! exp.Y / is
identified with the map p 0 W X ! Y such that p.x/ D fp 0 .x/g for any x 2 X . Now,

1.1 Equivalences that arise from homeomorphisms of Cp .X/

if X and Y are spaces, a map p W X ! exp.Y / is called lower semicontinuous


if pl1 .U / D fx 2 X W p.x/ \ U ;g is open in X for any open U  Y .
The map p is called upper semicontinuous if pu1 .U / D fx 2 X W p.x/  U g
is open in X for any open U  Y . Given two maps p; q W X ! exp.Y /, we
say that q is lower semicontinuous with respect to p if, for any x 2 X , we have
p.x/  q.x/ and ql1 .U / is a neighborhood of pl1 .U / for any open U  Y .
A finite-valued map p W X ! exp.Y / is called almost lower semicontinuous if there
exists a space Z  Y and a finite-valued map q W X ! exp.Z/ such that q is lower
semicontinuous with respect to p. We say that a single-valued map p W X ! Y has
defect  n if there exists a space Z  Y and a map q W X ! Z
nC1 which is
lower semicontinuous with respect to p. The map p has finite defect if it has defect
 n for some n 2 !.
Given a natural
number n and set-valued maps pi W Xi ! exp.Yi / for every
Q
i < n let . i<nQ
pi /.x0 ; : : : ; xn1
.xn1 / for any point
Q/ D p0 .x
Q0 /  : : :  pn1Q
.x0 ; : : : ; xn1 / 2 i<n Xi . Thus, i<n pi W i<n Xi ! exp. i<n Yi /.
Let  .A/ D fx 2 RA W for any " > 0, the set fa 2 A W jx.a/j  "g is finiteg.
Given x 2  .A/, let jjxjj D supfjx.a/j W a 2 Ag. The space D is the two-point
set f0; 1g endowed with the discrete topology. The symbol P denotes the irrational
numbers with the usual topology. If X is a space and A; B  X are disjoint closed
subsets of X , we say that a (closed) set P  X is a partition between A and B if
there are open U; V  X such that A  U; B  V; U \V D ; and U [V D X nP .
For every n 2 !, the set S n D fx 2 RnC1 W .x.0//2 C: : :C.x.n//2 D 1g with the
topology induced from RnC1 is called the n-dimensional sphere. A set F  In is a
face of the cube In if F D fx 2 In W x.i / D 1g or F D fx 2 In W x.i / D 1g
for some i < n. A set fa0 ; : : : ; am g  Rn is called independent if, for any
0 ; : : : ; m 2 R such that 0 C : : : C m D 0 and 0 a0 C : : : C m am D 0 we have
i D 0 for all i  m; here 0 is the vector of Rn whose all coordinates are zeros. If
fa0 ; : : : ; am g  Rn is an independent set, let a0 ; : : : ; am
D f 0 a0 C: : :C m am W
i  0 for all i  m and 0 C : : : C m D 1g. The set a0 ; : : : ; am
is called an
m-dimensional simplex with the vertices a0 ; : : : ; am .
It is easy to see that if x 2 a0 ; : : : ; am
then there are unique 0 .x/; : : : ; m .x/
such that x D 0 .x/ a0 C : : : C m .x/ am , where i .x/  0 for all i  m
and 0 .x/ C : : : ; C m .x/ D 1. The numbers 0 .x/; : : : ; m .x/ are called the
barycentric coordinates of the point x. A face of the simplex a0 ; : : : ; am
is any
simplex ai0 ; : : : ; aik
for which aij ail if j l. The barycenter of a simplex
1
1
S D a0 ; : : : ; am
is the point b.S / D mC1
a0 C : : : C mC1
am . A simplicial
subdivision
of
a
simplex
S
is
a
finite
family
P
D
fS
W
0

i
<
kg of simplexes
i
S
such that P D S , for any i; j < k, all faces of Si belong to P and Si \Sj is either
empty or is a common face of the simplexes Si and Sj . The mesh of a simplicial
subdivision P of a simplex S is the number .P/ D maxfdiam.T / W T 2 Pg.
For any  2 !1 the class M (or A ) of absolute Borel sets of multiplicative
(additive) class  consists of metrizable spaces X such that for any embedding of X
into a metrizable space Y the set X belongs to 0 .Y / (or to 0 .Y / respectively).

1 Properties Preserved by Homeomorphisms of Function Spaces

001. Prove that cardinality, network weight, i -weight, as well as density are
t -invariant.
002. Prove that if Cp .Y /! Cp .X / then nw.Y /  nw.X /; d.Y /  d.X / and
jY j  jX j. Give an example showing that the inequality i w.Y /  i w.X / is
not necessarily true.
003. Prove that p.Y /  p.X / whenever Cp .Y /! Cp .X /. As a consequence,
point-finite cellularity is t -invariant.
004. Suppose that X and Y are t -equivalent Baire spaces. Prove that c.X / D c.Y /.
In particular, the Souslin numbers of t -equivalent pseudocompact spaces
coincide.
005. Let  be a caliber of X . Knowing that Cp .Y / embeds in Cp .X /, prove that 
is a caliber of Y . In particular, calibers are t -invariant.
006. Suppose that Cp .Y / embeds into Cp .X /. Prove that l  .Y /  l  .X /. As a
consequence, l  is t -invariant.
007. Suppose that Cp .Y / embeds into Cp .X /. Prove that '.Y /  '.X / for any
' 2 fhl  ; hd  ; s  g and hence ' is t -invariant.
t
008. Prove that, if X  Y then tm .X / D tm .Y /. Give an example of spaces X and
Y such that Cp .Y / embeds into Cp .X / and tm .Y / > tm .X /.
t
009. Prove that X  Y implies q.X / D q.Y /. In particular, realcompactness is
t -invariant.
010. Give an example of spaces X and Y such that Cp .Y / embeds into Cp .X /, the
space X is realcompact and Y is not realcompact.
t
011. Suppose that X is a P -space and X  Y . Prove that Y is also a P -space.
012. Prove that discreteness is t -invariant.
013. Suppose that X and Y are compact spaces such that Cp .Y /! Cp .X /. Prove
t
that Y is scattered whenever X is scattered. In particular, if X  Y then X is
scattered if and only if so is Y .
t
014. Suppose that X n is a Hurewicz space for each n 2 N and X  Y . Prove that
Y n is also a Hurewicz space for each n 2 N.
t
015. Suppose that X  Y and X is a  -compact space with a countable network.
Prove that Y is also  -compact. As a consequence, if X is a metrizable
t
compact space and X  Y then Y is  -compact.
016. Given an arbitrary number " > 0 prove that there exists a homeomorphism u W R   .!/ !  .!/ for which we have the inequality
j jju.r; x/jj  jjxjj j  " for any r 2 R and x 2  .!/.
017. Prove that  .!/ is homeomorphic to R!   .!/.
018. Suppose that X is a pseudocompact space. Given any function f 2 Cp .X /,
let jjf jj D supfjf .x/j W x 2 X g. Prove that C .X / ' C .X /  .Cp .X //! ,
where C .X / D f' 2 .Cp .X //! W jj'.n/jj ! 0g.
019. Let X be a pseudocompact space. As usual, for any f 2 Cp .X /, we
define jjf jj D supfjf .x/j W x 2 X g. Supposing that the space Cp .X / is
homeomorphic to C .X / D f' 2 .Cp .X //! W jj'.n/jj ! 0g, prove that
Cp .X / ' .Cp .X //! .

1.1 Equivalences that arise from homeomorphisms of Cp .X/

020. Prove that  .!/ is homeomorphic to . .!//! .


021. Prove that, for every infinite space X , the space R! embeds into Cp .X / as a
closed subspace.
022. Prove that a space X is not pseudocompact if and only if R! embeds in Cp .X /
as a linear subspace.
023. Prove that, if either X or Cp .X / is Lindelf, then R!1 does not embed into
Cp .X /.
024. Prove that there exists a space X such that c.X / D ! and R!1 embeds in
Cp .X / as a closed linear subspace.
025. Prove that if ! C 1! X then Cp .X / ' Cp .X /  R! . Deduce from this
fact that pseudocompactness, countable compactness, and compactness are not
t -invariant.
026. Prove that Cp .R/! ' Cp .R/ and Cp .I/! ' Cp .I/.
027. Prove that R is t -equivalent to 0; 1
.
t
t
028. Prove that X  Y whenever X  Y . Give an example which shows that
t
t
X  Y does not necessarily imply X  Y .
t
029. Give an example of spaces X and Y such that X ' Y (and hence X  Y )
while X is not t -equivalent to Y .
030. Prove that -monolithity and -stability are t -invariant for any infinite
cardinal .
t
031. Given spaces X and Y such that X  Y prove that X is functionally perfect if
and only if so is Y .
032. Give an example of spaces X and Y such that X is functionally perfect, Cp .Y /
embeds into Cp .X / while Y is not functionally perfect.
033. Suppose that compact spaces X and Y are t -equivalent. Prove that X is
Eberlein (Corson or Gulko) compact if and only if so is Y .
034. Suppose that F  Cp .X; I/ is a D-separating set (and hence 0X 2 F ). For
each x 2 X , let ex .f / D f .x/ for any f 2 F . Prove that XQ D fex W x 2 X g
is a closed subset of the space ZF .X / D f' W F ! I W '.0X / D 0
and '.V /   12 ; 12
for some V 2 .0X ; F /g, and the map x ! ex
Q In other words, X is canonically
is a homeomorphism between X and X.
homeomorphic to a closed subset of ZF .X /.
035. Knowing that 0X 2 F  Cp .X; I/ and 0Y 2 G  Cp .Y; I/, suppose that
there is an embedding i W G ! F with i.0Y / D 0X . Prove that ZF .X / maps
continuously onto ZG .Y /.
036. Given a space X prove that if 0X 2 F  Cp .X; I/ then ZF .X / belongs to
the class K.X /.
037. Let G be a D-separating subspace of Cp .Y /. Prove that, if G embeds into
Cp .X / then Y 2 K.X /.
038. Given spaces X; Y and a subspace Z  Y suppose that the space Cp .ZjY / D
ff 2 Cp .Z/ W f D gjZ for some g 2 Cp .Y /g embeds in Cp .X /. Prove that
Z 2 K.X /.
039. Let X be a  -compact space. Prove that any space Y 2 K.X / is also
 -compact.

1 Properties Preserved by Homeomorphisms of Function Spaces

040. Let X be a Lindelf -space. Prove that any Y 2 K.X / is also a Lindelf
-space.
041. Let X be a K-analytic space. Prove that any Y 2 K.X / is also a K-analytic
space.
042. Prove that ext  .Y /  ext  .X / for any Y 2 K.X /.
043. Suppose that Cp .Y / embeds into Cp .X /. Prove that
(1)
(2)
(3)
(4)

if X is  -compact then Y is  -compact.


if X is Lindelf -space then Y is Lindelf .
if X is K-analytic then Y is also K-analytic.
ext  .Y /  ext  .X /.
t

044.

045.
046.

047.
048.

049.
050.
051.
052.

053.
054.
055.

As a consequence, if X  Y then ext  .X / D ext  .Y / and, for any property


P 2 f -compactness, Lindelf -property, K-analyticityg, we have X ` P
if and only if Y ` P.
Suppose that X is an analytic space and Cp .Y / embeds into Cp .X /. Prove
t
that Y is also analytic. In particular, analyticity is t -invariant, i.e., if X  Y
then X is analytic if and only if so is Y .
t
Suppose that X  Y . Prove that X is  -bounded if and only if so is Y .
Given a zero-dimensional space Y , suppose that Cp .Y; D/ embeds in Cp .X /.
Prove that, for any property P 2 f -compactness, Lindelf -property,
analyticity, K-analyticityg, we have Y ` P whenever X ` P.
Let X be a zero-dimensional space. Prove that l  .X / D t .Cp .X; D//.
Let X and Y be zero-dimensional spaces with Cp .X; D/ ' Cp .Y; D/. Prove
that X is pseudocompact if and only if so is Y . Deduce from this fact that X
is compact if and only if so is Y .
Prove that there exist zero-dimensional spaces X and Y such that Cp .X / is
homeomorphic to Cp .Y / and Cp .X; D/ is not homeomorphic to Cp .Y; D/.
Prove that .Cp .X; D// D w.Cp .X; D// D jX j for any zero-dimensional
space X .
Prove that a zero-dimensional compact space X is scattered if and only if
Cp .X; D/ is FrchetUrysohn.
Suppose that X is not  -compact and w.X /  . Prove that there is a subspace
Z  Cp .X / such that jZj   and Z is not embeddable into Cp .Y / for any
 -compact Y . In particular, there is a countable subspace of Cp .P/ which
cannot be embedded into Cp .Y / for any  -compact Y .
Prove that, for any X , the space Cp .X / embeds in Cp .P/ if and only if Cp .X /
is homeomorphic to a linear subspace of Cp .P/.
Suppose that a space X is compact and there exists a homeomorphism ' W
RX ! RY such that '.Cp .X //  Cp .Y /. Prove that Y is compact.
Suppose that p; q W X ! exp.Y / are finite-valued mappings such that q
is lower semicontinuous with respectS
to p. Prove that, for any nonempty set
A  X , the map qjA W AS ! exp. q.A// is lower semicontinuous with
respect to pjA W A ! exp. q.A//.

1.1 Equivalences that arise from homeomorphisms of Cp .X/

056. Suppose that p W X ! exp.Y / and q W X ! exp.Y / are finite-valued


mappings such that q is lower semicontinuous with respect to p. Given an
open set U  Y , let pU .x/ D p.x/ \ U and qU .x/ D q.x/ \ U for every
x 2 X . Prove that the map qU W X ! exp.U / is lower semicontinuous with
respect to pU W X ! exp.U /.
057. Given a number n 2 N and spaces Xi ; Yi ; for every i < n suppose that pi ; qi W
Xi ! exp.Yi / are set-valued
Q maps such that qi is lower semicontinuous with
respect
to
p
.
Prove
that
i
i<n qi is lower semicontinuous with respect to
Q
p
.
As
a
consequence,
any
finite product of almost lower semicontinuous
i
i<n
maps is an almost lower semicontinuous map.
058. Suppose that, for finite-valued mappings p; r; q W X ! exp.Y /, we have
p.x/  r.x/  q.x/ for any x 2 X . Prove that, if q is lower semicontinuous
with respect to r then q is lower semicontinuous with respect to p.
059. Suppose that X is a nonempty space and
S p W X ! exp.Y / is an almost
lower semicontinuous map such that
p.X / D Y . Prove that Y is a
countable union of images of subspaces of X under single-valued almost lower
semicontinuous maps of finite defect.
060. Given nonempty spaces X; Z and k 2 N, suppose that we have maps
p W X ! Z and q W X ! exp.Z/ such that p.x/ 2 q.x/ and jq.x/j  k for
each x 2 X while q.x/ \ q.y/ D ; if x y and q is lowerS
semicontinuous
with respect to p. Prove that, if p.X / is discrete then X D j <k Dj where
every Dj  X is discrete and hence there exists a discrete subspace S  X
such that jS j D jX j.
061. Given nonempty spaces X; Z and k 2 N, suppose that we have maps
p W X ! Z and q W X ! exp.Z/ such that p.x/ 2 q.x/ and jq.x/j  k for
each x 2 X while q.x/ \ q.y/ D ; if x y and q is lower semicontinuous
S
with respect to p. Prove that, if p.X / is left-separated then X D j <k Dj
where every Dj  X is left-separated and hence there exists a left-separated
subspace S  X such that jS j D jX j.
062. Given nonempty spaces X; Z and k 2 N, suppose that we have maps
p W X ! Z and q W X ! exp.Z/ such that p.x/ 2 q.x/ and jq.x/j  k for
each x 2 X while q.x/ \ q.y/ D ; if x y and q is lower semicontinuous
S
with respect to p. Prove that, if p.X / is right-separated then X D j <k Dj
where every Dj  X is right-separated and hence there exists a rightseparated subspace S  X such that jS j D jX j.
063. Suppose that an infinite space Y is an image of a space X under a finitevalued almost lower semicontinuous map, i.e., there exists an
S almost lower
semicontinuous map p W X ! exp.Y / such that Y D
p.X /. Prove
that for each n 2 N, we have s.Y n /  s.X n /, hl.Y n /  hl.X n / and
hd.Y n /  hd.X n /.
064. Let h W Cp .Y / ! Cp .X / be an embedding such that h.0Y / D 0X . Given a
point x 2 X and " > 0, a point y 2 Y is called "-inessential for x if there is
U 2 .y; Y / such that jh.g/.x/j  " whenever g.Y nU /  f0g. The point y
is "-essential for x if it is not "-inessential for x. Denote by supp" .x/ the set
of all points which are "-essential for x. Prove that supp" .x/ is finite for any
x 2 X and " > 0.

10

1 Properties Preserved by Homeomorphisms of Function Spaces

065. Let h W Cp .Y / ! Cp .X / be an embedding such that h.0Y / D 0X . Denote


by supp
S " .x/ the set of all points which are "-essential for x. Prove that
Y D fsupp1=n .x/ W x 2 X; n 2 Ng.
066. Let h W Cp .Y / ! Cp .X / be an embedding such that h.0Y / D 0X . Denote
by supp" .x/ the set of all points which are "-essential for x. Prove that, for
any " > 0, the finite-valued map supp" W X ! exp.Y / is almost lower
semicontinuous.
067. Suppose that Cp .Y / embeds in Cp .X /. Prove that Y is a countable union of
images of X under finite-valued almost lower semicontinuous maps.
068. Suppose that Cp .Y / embeds in Cp .X /. Prove that, for any n 2 N, we have
s.Y n /  s.X n /. In particular, if X and Y are t -equivalent then s.Y n / D s.X n /
for any n 2 N.
069. Suppose that Cp .Y / embeds in Cp .X /. Prove that, for any n 2 N, we have
hd.Y n /  hd.X n /. In particular, if the spaces X and Y are t -equivalent then
hd.Y n / D hd.X n / for any n 2 N.
070. Suppose that Cp .Y / embeds in Cp .X /. Prove that, for any n 2 N, we have
hl.Y n /  hl.X n /. In particular, if the spaces X and Y are t -equivalent then
hl.Y n / D hl.X n / for any n 2 N.
071. Suppose that f W X ! Y is a closed continuous onto map such that t .Y /  
and t .f 1 .y//   for any y 2 Y . Prove that t .X /  . Deduce from this
fact that, for every infinite compact space X we have t .X n / D t .X / for any
n 2 N.
072. Given countably compact sequential spaces X1 ; : : : ; Xm prove that the space
X1  : : :  Xm is countably compact and sequential.
073. Assuming that a space X maps continuously onto a compact space K prove
that t .K/  l.X / t .X /.
S
074. Suppose that K is a compact space and K D n2! Kn where every Kn is
a sequential closed subspace of K. Prove that if either Martins Axiom or
Luzins Axiom (2!1 > 2! ) holds then the space K is sequential.
075. Suppose that we have a space X , a compact space K and a S
compact-valued
upper semicontinuous map p W X ! exp.K/ such that
p.X / D K.
Knowing that l.X / t .X /   and t .p.x//   for any x 2 X prove that
t .K/  .
076. Suppose that K is a compact sequential space and L is a compact space for
which there
S exists a finite-valued upper semicontinuous map p W K ! exp.L/
such that p.K/ D L. Prove that L is also sequential.
077. Suppose that there exists an open continuous map of a subspace of Cp .X / onto
Cp .Y /. Prove that, for any n 2 N there
S Sis a finite-valued upper semicontinuous
map 'n W X n ! exp.Y / such that f 'n .X n / W n 2 Ng D Y .
078. Suppose that there is an open continuous map of a subspace of Cp .X /
onto Cp .Y /. Prove that t .K/  t  .X / l  .X / for any compact K  Y .
Deduce from this fact that if K and L are t -equivalent compact spaces then
t .K/ D t .L/.

1.1 Equivalences that arise from homeomorphisms of Cp .X/

11

079. Suppose that X is a compact sequential space and Y is a compact space such
that there is an open continuous map of some subspace of Cp .X / onto Cp .Y /.
Prove that Y is a countable union of its compact sequential subspaces. As
a consequence, under Martins Axiom if K and L are compact t -equivalent
spaces and K is sequential then L is also sequential.
080. S
Let X and Y be metrizable t -equivalent spaces. Prove that we have Y D
fYn W n 2 !g, where each Yn is a G -subspace of Y , homeomorphic to some
G -subspace of X .
081. Let XSand Y be metrizable spaces such that Cp .X / ' Cp .Y /. Prove that
Y D fYn W n 2 !g, where each Yn is a G -subspace of Y , homeomorphic to
some G -subspace of X .
082. Let X and Y be metrizable t -equivalent spaces. Prove that X is a countable
union of zero-dimensional subspaces if and only if so is Y .
083. Let X and Y be metrizable t -equivalent spaces. Prove that X is a countable

union of its Cech-complete


subspaces if and only if so is Y .
084. Suppose that fa0 ; : : : ; am g  Rn is an independent set. Prove that
(i) the simplex S D a0 ; : : : ; am
is a compact subset of Rn ;
(ii) any two m-dimensional simplexes are homeomorphic;
(iii) the barycentric coordinates are continuous functions from S to 0; 1
.
085. Let S be a simplex in Rn and suppose that S0  S1  : : :  Sk are distinct
faces of S . Prove that the points fb.S0 /; : : : ; b.Sk /g are independent. Here
b.Si / is the barycenter of the simplex Si for all i  k.
086. For a simplex S D a0 ; ; : : : ; am
, consider the family B.S / of all simplexes
of the form b.S0 /; : : : ; b.Sk /
, where S0  S1  : : :  Sk are distinct faces
of S . Prove that P is a simplicial subdivision of S such that any .m  1/dimensional simplex T 2 B.S / is a face of one or two m-dimensional
members of B.S / depending on whether T is contained in an .m  1/dimensional face of S . The subdivision B.S / is called the barycentric
subdivision of the simplex S .
087. Given a simplex S , let B1 .S / be the barycentric subdivision B.S / of the
simplex S . IfSwe have a simplicial subdivision Bn .S / of the simplex S , let
BnC1 .S / D fB.T / W T 2 Bn .S /g. The family Bn .S / is called the n-th
barycentric subdivision of the simplex S . Prove that, for any simplex S and
any " > 0, there exists a natural number n such that the mesh of the n-th
barycentric subdivision of the simplex S is less than ".
088. (Sperners lemma). Given a number l 2 N and an m-dimensional simplex
a0 ; : : : ; am
let V be the set of all vertices of simplexes in Bl .a0 ; : : : ; am
/.
Suppose that, for a function h W V ! f0; 1; : : : ; mg, we have h.v/ 2
fi0 ; : : : ; ik g whenever v 2 ai0 ; : : : ; aik
. Prove that the family of simplexes
in Bl .a0 ; : : : ; am
/, on vertices of which h takes all values from 0 to m, has
an odd cardinality.
089. (Browers fixed-point theorem). Prove that, for any n 2 N, if S is an
n-dimensional simplex and f W S ! S is a continuous function then there
exists a point x 2 S such that f .x/ D x.

12

1 Properties Preserved by Homeomorphisms of Function Spaces

090. Prove that, for any n 2 N, there is no retraction of the cube In onto its boundary
@In D fx 2 In W jx.i /j D 1 for some i < ng.
091. Given spaces X and Y and functions f; g 2 C.X; Y /, let f  g denote the
fact that f and g are homotopic. Prove that  is an equivalence relation on
C.X; Y /.
092. Given a space X , let f; g W X ! @In be continuous maps such that the points
f .x/ and g.x/ belong to the same face of In for any x 2 X . Prove that f and
g are homotopic.
093. (Mushroom lemma). Let X be a normal countably paracompact space. Suppose that F  X is closed and we have continuous homotopic mappings f0 ; f1
of F to the n-dimensional sphere S n D fx 2 RnC1 W x.0/2 C: : :Cx.n/2 D 1g.
Prove that, if there exists a continuous map g0 W X ! S n with g0 jF D f0
then there is a continuous map g1 W X ! S n such that g1 jF D f1 and g1 is
homotopic to g0 .
094. For each i < n, consider the faces Fi D fx 2 In W x.i / D 1g and Gi D fx 2
In W x.i / D 1g of the n-dimensional
cube In . Prove that, if Ci is a partition
T
between Fi and Gi then fCi W i < ng ;.
095. For each i 2 !, consider the subsets Fi D fx 2 I! W x.i / D 1g and
Gi D fx 2 I! W x.i / DT1g of the cube I! . Prove that, if Ci is any partition
between Fi and Gi then fCi W i 2 !g ;.
096. Prove that, for any n 2 N, the space In is the finite union of its zerodimensional subspaces.
097. Prove that, for any n 2 N, the space In cannot be represented as the union of
 n-many of its zero-dimensional subspaces.
098. Prove that the cube I! cannot be represented as the countable union of its
zero-dimensional subspaces. Prove (inSZFC) that there exist zero-dimensional
spaces fX W < !1 g such that I! D fX W < !1 g.
099. Prove that, for any n 2 N, the spaces In and I! are not t -equivalent.
100. Suppose that X is one of the spaces !1 or !1 C 1. Prove that, for any distinct
m; n 2 N, the spaces .Cp .X //n and .Cp .X //m are not homeomorphic. In
particular, X is not t -equivalent to X X .

1.2 Uniformities, Dimension, and u-Equivalence

13

1.2 Uniformities, Dimension, and u-Equivalence


Given a set X and A  X 2 , let A1 D f.x; y/ 2 X  X W .y; x/ 2 Ag. The set
A is symmetric if A1 D A. If A; B  X  X , then A B D f.x; y/ 2 X 2 W
there exists a point z 2 X such that .x; z/ 2 B and .z; y/ 2 Ag. Here, as usual,
D X D f.x; x/ W x 2 X g. For any point x 2 X and U S
 X 2 let U.x/ D
2
fy 2 X W .x; y/ 2 U g; if A  X and U  X , then U.A/ D fU.x/ W x 2 Ag.
A nonempty family U  exp.X X / is a uniformity on a set X if it has the following
properties:
T
(U1)
U D , besides, if U; V 2 U then U 1 2 U and U \ V 2 U ;
(U2) if U 2 U then W 2 U whenever U  W ; besides, V V  U for
some V 2 U .
The pair .X; U / is called a uniform space. A family B  U is called a base of the
uniformity U if, for any U 2 U , there is V 2 B such that V  U . A family S  U
is called a subbase of the uniformity U if the family of all finite intersections of the
elements of S is a base for U . As usual, if f W X ! Y then f f W X X ! Y Y
is defined by .f  f /.x1 ; x2 / D .f .x1 /; f .x2 //.
If .X; U / is a uniform space, let U D f;g [ fO  X W for any x 2 O there
is U 2 U such that U.x/  Og. The family U is a topology; it is called the
topology generated by the uniformity U . If a topological notion (closure, interior,
base, continuity, etc.) is mentioned for a uniform space .X; U /, the topology in
question is always U . Given uniform spaces .X; U / and .Y; V/, a map f W X ! Y
is called uniformly continuous with respect to U and V if, for any V 2 V, we
have U D .f  f /1 .V / D f.x; y/ 2 X 2 W .f .x/; f .y// 2 V g 2 U . If the
uniformities U and V are clear, we will just say that f is uniformly continuous.
The mapping f is called a uniform isomorphism (or uniform homeomorphism) if it
is a bijection and both maps f and f 1 are uniformly continuous. If there exists
a uniform isomorphism f W X ! Y , the spaces .X; U / and .Y; V/ are called
uniformly isomorphic (or uniformly homeomorphic). It is impossible to distinguish
two uniformly isomorphic uniform spaces because their properties, which can be
expressed in terms of their uniformities, are identical.
An arbitrary function d W X  X ! R is called a pseudometric on the set X
if d.x; x/ D 0; d.x; y/  0 for any x; y 2 X and d.x; z/  d.x; y/ C d.y; z/
for any x; y; z 2 X . If .X; U / is a uniform space and Y  X , then the family
UYX D fU \ .Y  Y / W U 2 U g is a uniformity on Y ; the pair .Y; UYX / is called a
uniform subspace of the uniform space .X; U / and UYX is the uniformity induced
on Y from X . If X is a set, .Y; V/ is a uniform space and f W X ! Y then
f 1 .V/ D f.f  f /1S
.V / W V 2 Vg. If we have a family f.Xt ; Ut / W t 2 T g
of uniform spaces then fpt1
Q.Ut / W t 2 T g is a subbase of a (uniquely defined)
uniformity U on the set X D fXt W t 2 T g (check it, please). The uniform space
.X; U / is called the uniform product of the uniform spaces f.Xt ; Ut / W t 2 T g.
Given a uniform space .X; U /, a family F  exp.X / is Cauchy if, for any U 2 U ,
we have F  F  U for some F 2 F. The uniform space .X; U / is called complete

14

1 Properties Preserved by Homeomorphisms of Function Spaces

T
if, for any Cauchy filter F on X , we have fA W A 2 Fg ;. A uniform space
.X; U / is called totally bounded if, for any U 2 U , there is a finite P  X such that
U.P / D X .
Given a linear topological space L let G.U / D f.x; y/ 2 L  L W x  y 2 U g for
any U 2 .0; L/ where 0 is the zero vector of L. Then fG.U / W U 2 .0; L/g
is a base for the linear uniformity on the set L. From now on, if a uniformity
notion is used in a linear topological space L (in particular, when L D R or
L D Cp .X /), then the relevant uniformity is assumed to be linear. If X and Y
u
are Tychonoff spaces, they are called u-equivalent (which is denoted by X  Y ),
if Cp .X / is uniformly isomorphic to Cp .Y /. A set E  Cp .X / is a QS-algebra
for X if, given x 2 X and closed F  X with x F , there is f 2 E with
f .x/ D 1 and f .F /  f0g and, besides, for any f; g 2 E and q 2 Q, we have
f C g 2 E; f g 2 E and q f 2 E.
A set P  X is called functionally closed (open) if there exists f 2 C.X / and
a closed (open) Q  R such that P D f 1 .Q/. A family C of sets is called closed
(open, functionally open, functionally closed) if all elements of C are closed (open,
functionally open, functionally closed). If U  exp.X /, we say that the order of U
is  k (denoting it by ord.U /  k), if every x 2 X belongs to at most k elements
of U . If A D fAs W s 2 S g  exp.X /, a family fBs W s 2 S g  exp.X / is
called a swelling of A if As  Bs for each s 2 S and BS
s0 \ : : : Bsm D ; if
and only if As0 \ : : : Asm D ; for any s0 ; : : : sm 2 S . If A D X , a family
B DS
fBs W s 2 S g  exp.X / is called a shrinking of A if Bs  As for each s 2 S
and B D X . Given X 2 T3 1 , let dim X D 1 if and only if X D ;. If X ;,
2
we say that dim X  n 2 ! if any finite functionally open cover U of the space
X has a finite functionally open refinement V with ord.V/  n C 1. It is said that
dim X D n if dim X  n and dim X  n  1 does not hold. Finally, dim X D 1 if
dim X  n is false for all n 2 !.
A partial order  on a set T is called a direction on T if, for any s; t 2 T , there
is w 2 T such that s  w and t  w. A set with a direction is called a directed set.
A subset T 0 of a directed set T is cofinal in T if, for any t 2 T , there is t 0 2 T 0
such that t  t 0 . Suppose that, for every element t of a directed set T , we have a
topological space Xt and, for every pair t; s 2 T with s  t , there is a continuous
map st W Xt ! Xs . If, for all r; s; t 2 T with r  s  t , we have tt D idXt and
rs st D rt then the family S D fXt ; st W t; s 2 T g is called an inverse system and
the maps st are called projections. An inverse system Q
S D fXm ; nm W m; n 2 !g
is called an inverse sequence. An element x 2 X D fXt W t 2 T g is called a
thread of S if st .x.t // D x.s/ whenever t; s 2 T and s < t . The set lim S of all

threads of S (with the topology inherited from X ) is called the limit of the inverse
system S. The natural projection pt W X ! Xt , restricted to lim S, is called the limit


projection and is denoted by t . If we have inverse systems S D fXt ; st W t; s 2 T g


and T D fYt ; pst W t; s 2 T g with the same directed set T , a family fft W t 2 T g is a
topological isomorphism (embedding) between S and T (of S in T ), if ft W Xt ! Yt
is a homeomorphism (a topological embedding) such that pst ft D fs st for any
s; t 2 T with s  t .

1.2 Uniformities, Dimension, and u-Equivalence

15

101. Prove that a nonempty family B  exp.X  X / is a base for some uniformity
on X if and only if it has the following properties:
T
(1)
B D ;
(2) for any U 2 B, there is V 2 B such that V 1  U ;
(3) for any U 2 B, there is V 2 B such that V V  U ;
(4) if U; V 2 B then there is W 2 B such that W  U \ V .
102. Suppose that a nonempty family S  exp.X X / has the following properties:
T
(1)
S D ;
(2) for any U 2 S, there is V 2 S such that V 1  U ;
(3) for any U 2 S, there is V 2 S such that V V  U .
Prove that S is a subbase for some uniformity on X . As a consequence, the
union of any family of uniformities on X is a subbase of some uniformity
on X .
103. Let .X; U / be a uniform space. Prove that
(1) Int.A/ D fx W U.x/  A for some U 2 U g for any set A  X ; in
particular, x 2 Int.U.x// for any U 2 U ;
(2) if B is a base of the uniformity U then, for any x 2 X and O 2 .x; X /,
there is B 2 B such that B.x/  O. In particular, the family fInt.B.x// W
B 2 Bg is a local base of the space X at x.
(3) if S is a subbase of U then,
T for any x 2 X and O 2 .x; X /, there is a
finite S 0  S such that fS.x/ W S 2 S 0 g  O.
(4) for any U 2 U , the interior (in X  X ) of the set U also belongs to U . As
a consequence, the family of all open symmetric elements of U is a base
of U ;T
(5) A D TfU.A/ W U 2 U g for any A  X ;
(6) B D fU B U W U 2 U g for any B  X  X ;
(7) the family of all closed symmetric elements of U is a base of U .
104. Given uniform spaces .X; U / and .Y; V/, prove that every uniformly continuous map f W X ! Y is continuous. In particular, every uniform isomorphism
is a homeomorphism.
105. Suppose
Q that .Xt ; Ut / is a uniform space for every t 2 T and consider the set
X D t2T Xt . Let pt W X ! Xt be the natural projection for every t 2 T ;
prove that
S
(1) the family S D fpt1 .Ut / W t 2 T g is a subbase of a unique uniformity
on X , i.e., the uniform product .X; U / of the spaces f.Xt ; Ut / W t 2 T g is
well defined;
(2) every map pt W .X; U / ! .Xt ; Ut / is uniformlyQcontinuous;
(3) U coincides with the topology of the product f.Xt ; Ut / W t 2 T g.
106. Let .X; U / be the uniform product of the family f.Xt ; Ut / W t 2 T g. Given a
uniform space .Y; V/, prove that a map f W Y ! X is uniformly continuous
if and only if pt f W Y ! Xt is uniformly continuous for any t 2 T . Here,
as always, the map pt W X ! Xt is the natural projection.

16

1 Properties Preserved by Homeomorphisms of Function Spaces

107. Prove that, for any uniform space .X; U /, a pseudometric d W X  X ! R is


uniformly continuous with respect to the uniform square of .X; U / if and only
if the set Or D f.x; y/ 2 X  X W d.x; y/ < rg belongs to U for any r > 0.
Such pseudometrics will be called uniformly continuous on .X; U /.
108. Suppose that a sequence fUn W n 2 !g of subsets of X  X has the following
properties:
(1) U0 D X  X and  Un for any n 2 !;
(2) the set Un is symmetric and UnC1 UnC1 UnC1  Un for any n 2 !.

109.

110.
111.

112.
113.
114.

Prove that there exists a pseudometric d on the set X such that, for any n 2 N,
we have Un  f.x; y/ W d.x; y/  2n g  Un1 .
Given a uniform space .X; U / and U 2 U , prove that there is a uniformly
continuous pseudometric  on .X; U / such that f.x; y/ 2 X  X W .x; y/ <
1g  U .
Prove that a topological space X is Tychonoff if and only if there exists a
uniformity U on the set X such that .X / D U .
Given a metric  on a set X and a number r > 0, consider the set Ur D
f.x; y/ 2 X X W .x; y/ < rg. Prove that the family B D fUr W r > 0g forms
a base of some uniformity U on the set X . The uniformity U is called the
uniformity induced by the metric . A uniform space .X; U / is called uniformly
metrizable if U D U for some metric  on the set X .
Prove that a uniform space .X; U / is uniformly metrizable if and only if U has
a countable base.
Prove that every uniform space is uniformly isomorphic to a subspace of a
product of uniformly metrizable spaces.
Given a uniform space .X; U / prove that the following conditions are
equivalent:
(i) the space .X; U / is complete;
T
(ii) if C is a centered Cauchy family of closed subsets of X then T
C ;;
(iii) if C is a centered Cauchy family of subsets of X then
fC W
C 2 Cg ;;
T
(iv) if B is a Cauchy filter base on X then fB W B 2 Bg T;;
(v) if B is a Cauchy filter base of closed subsets of X then B ;;
(vi) any Cauchy filter F on X converges to a point x 2 X , i.e.,
.x; X /  F;
T
(vii) if E is a Cauchy ultrafilter on X then fE W E 2 Eg ;;
(viii) if E is a Cauchy ultrafilter on X then it converges to a point x 2 X .

115. Prove that any closed uniform subspace of a complete uniform space is
complete.
116. Let .X; U / be a uniform space such that some Y  X is complete considered
as a uniform subspace of .X; U /. Prove that Y is closed in X .
117. Prove that, for any family A D f.Xt ; Ut / W t 2 T g of complete uniform spaces,
the uniform product .X; U / of the family A is complete.

1.2 Uniformities, Dimension, and u-Equivalence

17

118. Let .X; U / be a uniform space such that the uniformity U is induced by a
metric . Prove that .X; U / is complete if and only if the metric space .X; /
is complete.
119. Let A be a dense subspace of a uniform space .X; U /. Suppose that
f W A ! Y is a uniformly continuous map of .A; UAX / to a complete uniform
space .Y; V/. Prove that there is a uniformly continuous map F W X ! Y
such that F jA D f .
120. Let .X; U / and .Y; V/ be complete uniform spaces. Suppose that A is a dense
subspace of X and B is a dense subspace of Y . Prove that every uniform
isomorphism between the uniform spaces .A; UAX / and .B; VBY / is extendable
to a uniform isomorphism between .X; U / and .Y; V/.
121. Prove that every uniform space .X; U / is uniformly isomorphic to a dense
subspace of a complete uniform space .X  ; U  /. The space .X  ; U  / is called
the completion of the space .X; U /. Prove that the completion of .X; U / is
unique in the sense that, if .Y; V/ is a complete uniform space such that .X; U /
is a dense uniform subspace of .Y; V/ then there is a uniform isomorphism
W X  ! Y such that .x/ D x for any x 2 X .
122. Let X be a paracompact Tychonoff space. Prove that the family of all neighborhoods of the diagonal of X is a uniformity on X which generates .X /.
123. Suppose that X is a Tychonoff space such that the family of all neighborhoods
of the diagonal of X is a uniformity on X which generates .X /. Prove that
X is collectionwise normal.
124. Give an example of a Tychonoff countably compact non-compact (and hence
non-paracompact) space X such that the family of all neighborhoods of the
diagonal of X is a uniformity on X which generates .X /.
125. Prove that, for any compact uniform space .X; U /, the uniformity U coincides
with the family of all neighborhoods of the diagonal of X .
126. Suppose that .X; U / a compact uniform space. Prove that, for any uniform
space .Y; V/, any continuous map f W X ! Y is uniformly continuous.
127. Let .X; U / be a uniform space such that the uniformity U is induced by a
metric . Prove that .X; U / is totally bounded if and only if the metric space
.X; / is totally bounded.
128. Prove that a uniform space .X; U / is totally bounded if and only if every
ultrafilter on X is a Cauchy family with respect to U .
129. Prove that any uniform product of totally bounded uniform spaces is totally
bounded.
130. Prove that a uniform space is compact if and only if it is complete and totally
bounded. Deduce from this fact that a uniform space is totally bounded if and
only if its completion is compact.
131. Prove that a Tychonoff space X is pseudocompact if and only if every
uniformity U on the set X with U D .X / is totally bounded.
132. For any Tychonoff space X let UX be
Sthe family of all uniformities on the set
X which generate .X /. Note that UX can be considered a subbase of a
uniformity NX (called the universal uniformity) on the set X . Prove that

18

1 Properties Preserved by Homeomorphisms of Function Spaces

(i) the topology generated by NX coincides with .X / and hence NX 2 UX ;


(ii) if Y is a Tychonoff space and f W X ! Y is a continuous map then the
map f W .X; NX / ! .Y; V/ is uniformly continuous for any uniformity
V 2 UY .
133. Let X be a Tychonoff space. Prove that the following are equivalent:
(i) there exists a complete uniformity U on the set X such that U D .X /;
(ii) the universal uniformity on the space X is complete;
(iii) the space X is Dieudonn complete.
134. For any linear topological space L denote by 0L its zero vector and let G.U / D
f.x; y/ 2 L  L W x  y 2 U g for any U 2 .0L ; L/. Prove that
(i) the family BL D fG.U / W U 2 .0L ; L/g forms a base for a uniformity
UL on the set L (called the linear uniformity on L).
(ii) If M is a linear subspace of L then the linear uniformity UM on the set
M coincides with the subspace uniformity induced on M from L.
(iii) If L0 is a linear topological space then a map f W L ! L0 is uniformly
continuous if and only if, for any U 0 2 .0L0 ; L0 / there exists U 2
.0L ; L/ such that f .x/  f .y/ 2 U 0 for any x; y 2 L with x  y 2 U .
(iv) If L0 is a linear topological space then any linear continuous map f W
L ! L0 is uniformly continuous if L and L0 are considered with their
linear uniformities. In particular, any linear isomorphism between L and
L0 is a uniform isomorphism.
135. Prove that the linear uniformity of RX coincides with the uniform product
of the respective family of real lines. Deduce from this fact that RX is the
completion of Cp .X / for any space X so Cp .X / is complete as a uniform
space if and only if X is discrete.
136. Prove that Cp .X / is  -totally bounded as a uniform space if and only if X is
pseudocompact. More formally, X is pseudocompact if and only
S if there exists
a family fCn W n 2 !g  exp.Cp .X // such that Cp .X / D fCn W n 2 !g
and each Cn is totally bounded considered as a uniform subspace of Cp .X /.
In particular, if Cp .X / is uniformly isomorphic to Cp .Y / then the space X is
pseudocompact if and only if so is Y .
u
t
137. Observe that X  Y implies X  Y . Given an example of t -equivalent spaces
X and Y which are not u-equivalent.
138. Suppose that Cp .X / is uniformly isomorphic to Cp .Y /. Prove that X is
compact if and only if so is Y .
139. Suppose that the spaces X and Y are u-equivalent. Prove that there exists a
homeomorphism ' W RX ! RY such that '.Cp .X // D Cp .Y /.
140. Let F D fF1 ; : : : ; Fk g be a family of functionally closed subsets of a
Tychonoff space X . Suppose that fU1 ; : : : ; Uk g is a family of functionally
open subsets of X such that Fi  Ui for each i . Prove that the family F has a
functionally open swelling fW1 ; : : : ; Wk g such that Fi  Wi  W i  Ui for
each i  k.

1.2 Uniformities, Dimension, and u-Equivalence

19

141. Let F D fF1 ; : : : ; Fk g be a family of closed subsets of a normal space X .


Suppose that fU1 ; : : : ; Uk g is a family of open subsets of X such that Fi  Ui
for each i  k. Prove that the family F has an open swelling fW1 ; : : : ; Wk g
such that Fi  Wi  W i  Ui for each i  k.
142. Let U D fU1 ; : : : ; Uk g be a functionally open cover of a Tychonoff
space X . Prove that U has shrinkings F D fF1 ; : : : ; Fk g and W D
fW1 ; : : : ; Wk g such that F is functionally closed, W is functionally open
and Fi  Wi  W i  Ui for every i  k.
143. Let U D fU1 ; : : : ; Uk g be an open cover of a normal space X . Prove that U has
shrinkings F D fF1 ; : : : ; Fk g and W D fW1 ; : : : ; Wk g such that F is closed,
W is open, and Fi  Wi  W i  Ui for every i  k.
144. Prove that, for any Tychonoff space X , the following conditions are
equivalent:
(i) dim X  n;
(ii) every finite functionally open cover of X has a finite functionally closed
refinement of order  n C 1;
(iii) every finite functionally open cover of X has a functionally closed
shrinking of order  n C 1;
(iv) every finite functionally open cover of X has a functionally open
shrinking of order  n C 1.
145. Prove that, for any normal X , the following conditions are equivalent:
(i) dim X  n;
(ii) every finite open cover of X has a finite open refinement of order  nC1;
(iii) every finite open cover of X has a finite closed refinement of order
 n C 1;
(iv) every finite open cover of X has a closed shrinking of order  n C 1;
(v) every finite open cover of X has an open shrinking of order  n C 1.
146. Suppose that X is a Tychonoff space and Y is a C  -embedded subset of X .
Prove that dim Y  dim X . In particular, if X is normal then dim F  dim X
for any closed F  X .
147. Prove that dim X D dim X for any Tychonoff space X . Deduce from this
fact that dim X D dim Y for any Y with X  Y  X .
148. Prove that a Tychonoff space X is strongly zero-dimensional if and only if X
is normal and dim X D 0. Give an example of a Tychonoff space X such that
dim X D 0 while X is not strongly zero-dimensional.
149. Prove that dim X D 0 implies that X is zero-dimensional. Give an example of
a zero-dimensional space Y such that dim Y > 0.
150. (The countable sum theorem for normal spaces). Given n 2 !, suppose that
a normal space X has a countable closed cover F such that dim F  n for
every F 2 F. Prove that dim X  n.
151. (General countable sum theorem). Given n 2 !, suppose that we have a closed
countable cover F of a Tychonoff space X such that

20

1 Properties Preserved by Homeomorphisms of Function Spaces

(i) every F 2 F is C  -embedded in X ;


(ii) dim F  n for each F 2 F.

152.
153.
154.

155.

156.

157.

158.

159.

160.
161.

162.

163.

Prove that dim X  n; give an example


of a Tychonoff non-normal space Y
S
such that dim Y > 0 and Y D fYi W i 2 !g, where Yi is closed in Y and
dim Yi D 0 for every i 2 !.
Give an example of a compact (and hence normal) space X such that
dim X D 0 while dim Y > 0 for some Y  X .
Give an example of a Tychonoff space X such that dim X D 0 and dim Y > 0
for some closed Y  X .
Let X be a normal space with dim X  n. Given a subspace Y  X , suppose
that, for every open U  Y , there exists an F -set P such that Y  P  U .
Prove that dim Y  n.
Prove that, for any perfectly normal space X , we have dim Y  dim X for
any Y  X . In particular, dim Y  dim X for any subspace Y of a metrizable
space X .
Given n 2 ! and a Tychonoff space X , prove that dim X  n if and only if,
for any family f.A0 ; B0 /; : : : ; .An ; Bn /g of n C 1 pairs of disjoint functionally
closed sets, it is possible to choose, for each i  n, a functionally closed
partition Ci between Ai and Bi in such a way that L0 \ : : : \ Ln D ;.
Given a natural n  0 and a normal space X , prove that dim X  n if and only
if, for any family f.A0 ; B0 /; : : : ; .An ; Bn /g of nC1 pairs of disjoint closed sets,
it is possible to choose, for each i  n, a partition Ci between Ai and Bi in
such a way that L0 \ : : : \ Ln D ;.
Let X be a normal space. Prove that dim X  n if and only if, for any closed
F  X and any continuous map f W F ! S n , there exists a continuous
map g W X ! S n such that gjF D f . Here S n D f.x0 ; : : : ; xn / 2 RnC1 W
x02 C : : : C xn2 D 1g is the n-dimensional sphere with the topology inherited
from RnC1 .
Prove that dim.In / D dim.Rn / D dim.S n / D n for any n 2 N. Here S n D
f.x0 ; : : : ; xn / 2 RnC1 W x02 C : : : C xn2 D 1g is the n-dimensional sphere with
the topology inherited from RnC1 .
Given n 2 N prove that, for any set X  Rn , we have dim X D n if and only
if the interior of X in Rn is nonempty.
Prove that, for any Tychonoff space X and n 2 !, we have dim X  n if and
only if, for any second countable space Y and any continuous f W X ! Y ,
there exists a second countable space M and continuous maps g W X ! M;
h W M ! Y such that dim M  n and f D h g.
Prove that, for any n 2 !, there exists a compact second countable space Un
such that dim Un  n and any second countable X with dim X  n can be
embedded in Un .
Suppose that X is a second countable space, Y  X and dim Y  n. Prove
that there exists a G -set Y 0 of the space X such that Y  Y 0 and dim Y 0  n.

1.2 Uniformities, Dimension, and u-Equivalence

21

164. Given n 2 ! and a second countable Tychonoff space X prove that dim X  n
if and only there exist X0 ; : : : ; Xn  X such that X D X0 [ : : : [ Xn and
dim Xi  0 for each i  n.
of Hausdorff topological
165. Let S D fXt ; st W t; s 2 T g be an inverse system
Q
spaces. Prove that the set lim S is closed in fXt W t 2 T g. Therefore the

limit of an inverse system of Hausdorff compact spaces is a Hausdorff compact
space.
166. Suppose that S D fXt ; st W t; s 2 T g is an inverse system in which Xt is a
nonempty compact Hausdorff space for each t . Prove that lim S ;.


167. Let S D fXt ; st W t; s 2 T g be an inverse system. Suppose that, for a space
Y , a continuous map ft W Y ! Xt is given for every t 2 T and, besides,
st ft D fs for any s; t 2 T with s  t . Prove that the diagonal product
f D t2T ft maps Y continuously into lim S.

168. Let X be a topological space. Suppose that, for a nonempty directed set T ,
a subspace Xt  X is given for each t 2 T in such a way that Xt  Xs
whenever s  t . Given s; t 2 T with s  t , let st .x/ D x for each x 2 Xt .
Prove that the inverse system STD fXt ; st W t; s 2 T g is well defined and the
limit of S is homeomorphic to fXt W t 2 T g.
169. Give an example of an inverse sequence S D fXn ; mn W n; m 2 !g such that
every Xn is a nonempty second countable Tychonoff space while lim S D ;.


170. Given an inverse system S D fXt ; st W t; s 2 T g of topological spaces, prove


that the family B D ft1 .U / W t 2 T; U 2 .Xt /g is a base of the space
L Dlim S. Here t W L ! Xt is the limit projection for every t 2 T .


171. Suppose that S D fXt ; st W t; s 2 T g is an inverse system of topological


spaces. Prove that, for any closed set F  lim S, the subspace F is the limit of


the inverse system SF D ft .F /; st jt .F / W t; s 2 T g. Here t W lim S ! Xt



is the limit projection for every t 2 T .
172. Given an inverse system S D fXt ; st W t; s 2 T g and a cofinal set T 0  T ,
prove that the limit of the inverse system S 0 D fXt ; st W t; s 2 T 0 g is
homeomorphic to lim S.


173. Suppose that an inverse system S D fXt ; st W t; s 2 T g consists of compact


Hausdorff spaces Xt and all projections st are onto. Prove that all limit
projections t are also surjective maps.
174. Suppose that n 2 ! and S D fXt ; st W t; s 2 T g is an inverse system in
which all spaces Xt are compact and Hausdorff. Knowing additionally that
dim Xt  n for each t 2 T prove that dim.lim S/  n.


175. Suppose that n 2 ! and S D fXl ; ml W l; m 2 !g is an inverse sequence in


which every Xl is a Lindelf -space. Knowing additionally that dim Xl  n
for every l 2 ! prove that dim.lim S/  n.


176. Prove that if X is second countable and A  Cp .X / is countable then there


exists a countable QS-algebra E  Cp .X / such that A  E.

22

1 Properties Preserved by Homeomorphisms of Function Spaces

177. Let K D fx0 ; : : : ; xn g  X be a finite subset of a space X . Suppose


additionally that U 2 .K; X / and q0 ; : : : ; qn 2 Q. Prove that, for any
QS-algebra E on a space X , there is f 2 E such that f .X nU /  f0g and
f .xi / D qi for each i  n.
178. Given second countable Tychonoff spaces X and Y , suppose that some
QS-algebras E.X / and E.Y / are chosen in Cp .X / and Cp .Y / respectively.
Prove that, if E.X / is uniformly homeomorphic to E.Y / then X is representable as a countable union of closed subspaces each one of which embeds
in Y .
179. Given second countable Tychonoff spaces X and Y , suppose that some
QS-algebras E.X / and E.Y / are chosen in Cp .X / and Cp .Y / respectively.
Prove that, if E.X / is uniformly homeomorphic to E.Y / then dim X D
dim Y .
180. Suppose that X and Y are Tychonoff spaces such that Cp .X / is uniformly
homeomorphic to Cp .Y /. Prove that dim X D dim Y .
181. Let X be a zero-dimensional compact space. Prove that Y is also a zerou
dimensional compact space whenever Y  X .
u
182. Suppose that X is a zero-dimensional Lindelf space and Y  X . Prove that
Y is also zero-dimensional.
183. Given a countable ordinal   1, prove that a metrizable space X is an
absolute Borel set of multiplicative class  (i.e., X 2 M ) if and only if there
exists a completely metrizable space Z such that X is homeomorphic to some
Y 2 0 .Z/.
184. Given a countable ordinal   2, prove that a metrizable space X is an
absolute Borel set of additive class  (i.e., X 2 A ) if and only if there
exists a completely metrizable space Z such that X is homeomorphic to some
Y 2 0 .Z/.
185. Suppose that n 2 N and a space Xi is metrizable for every i  n. Prove that,
for any countable ordinal   2,
(i) if Xi 2 A for all i  n then X1  : : :  Xn 2 A ;
(ii) if Xi 2 M for all i  n then X1  : : :  Xn 2 M .
186. Given ordinals ; 2 !1 such
S that  2 and < suppose that X is a
metrizable space and X D fXn W n 2 !g where Xn 2 0 .X / \ M for
every n 2 !. Prove that X 2 M .
187. Prove that a metrizable space X is a Borel set of absolute additive class   2
(i.e., X 2S
A ) if and only if there exists a sequence fn W n 2 !g   such
that X D fXn W n 2 !g and Xn 2 Mn for every n 2 !.
188. Given a countable ordinal   2, let M be the class of absolute Borel sets of
multiplicative class . Prove that the following conditions are equivalent for
any metrizable X :
(i) the space X belongs to M ;
(ii) there is a complete sequence fUn W n 2 !g of  -discrete covers of X
such that, for any n 2 !, there is n <  with Un  0n .X /;

1.2 Uniformities, Dimension, and u-Equivalence

23

(iii) there is a complete sequence fVn W n 2 !g of  -discrete


covers of X such
S
that, for any n 2 !, there is n <  with Vn  f0 .X / W < n g.
189. Given a countable ordinal   2 prove that the following conditions are
equivalent for any second countable X :
(i) the space X belongs to M ;
(ii) there is a complete sequence fUn W n 2 !g of countable covers of X such
that, for any n 2 !, there is n <  with Un  0n .X /;
(iii) there is a complete sequence fVn W n 2 !g of countable
covers of X such
S
that, for any n 2 !, there is n <  with Vn  f0 .X / W < n g.
190. Prove that any analytic space has a complete sequence of countable covers.
Show that in metrizable spaces the converse is also true, i.e., a metrizable
space X is analytic if and only if there exists a complete sequence of countable
covers of X .
191. For any metrizable space X and n 2 N define a map e W X n ! X
n by
e..x1 ; : : : ; xn // D fx1 ; : : : ; xn g for every .x1 ; : : : ; xn / 2 X n . Prove that there
exists an F -set G in the space X n such that e.G/ D X
n and the map ejG W
G ! X
n is a bijection.
192. Given a metrizable space X and n 2 N consider the set X
n together with
its Vietoris topology. Prove that there exists
S a family fYm W m 2 !g of
closed subsets of X
n such that X
n D fYm W m 2 !g and every Ym is
homeomorphic to some closed subspace of X n .
193. Suppose that there exists a uniformly continuous surjection of Cp .X / onto
Cp .Y /. Prove that if X is pseudocompact then Y is also pseudocompact.
Deduce from this fact that if X is a metrizable compact space and there exists a
uniformly continuous surjection of Cp .X / onto Cp .Y / then Y is also compact.
Give an example of a (non-metrizable!) compact space X such that there is a
non-compact space Y and a uniformly continuous surjection of Cp .X / onto
Cp .Y /.
194. Assume that X and Y are metrizable spaces and there exists either a uniformly
continuous surjection of Cp .X / onto Cp .Y / or a uniformly continuous
surjection of Cp .X / onto Cp .Y /. Prove S
that there exists a family fYn W n 2 !g
of closed subspaces of Y such that Y D n2! Yn and each Yn can be perfectly
mapped onto a closed subspace of X
kn (with the Vietoris topology) for some
kn 2 N.
195. Let P be a class of metrizable spaces with the following properties:
(1) P contains all complete metrizable spaces;
(2) P is invariant under finite products and closed
S subspaces;
(3) if M is a metrizable space with M D fMn W n 2 !g, where Mn is
closed in M and Mn 2 P for each n 2 !, then M 2 P.
Suppose that X 2 P and Y is a metrizable space. Prove that, if there exists
a uniformly continuous surjection of Cp .X / onto Cp .Y / (or Cp .X / onto
Cp .Y /), then Y 2 P.

24

1 Properties Preserved by Homeomorphisms of Function Spaces

196. Let P be a class of second countable spaces such that


(1) every compact metrizable space belongs to P;
(2) P is invariant under finite products and closedSsubspaces;
(3) if M is a second countable space with M D fMn W n 2 !g, where Mn
is closed in M and Mn 2 P for each n 2 !, then M 2 P.
Suppose that X 2 P and Y is a metrizable space. Prove that, if there exists
a uniformly continuous surjection of Cp .X / onto Cp .Y / (or Cp .X / onto
Cp .Y /), then Y 2 P.
197. Given a countable ordinal , let M be the class of absolute Borel sets of
multiplicative class . Suppose that X is a metrizable space such that X 2 M
for some  2. Let Y be a metrizable space such that Cp .Y / (or Cp .Y /) is
a uniformly continuous image of Cp .X / (or Cp .X / respectively). Prove that
u

Y 2 M . In particular, if X  Y then X belongs to M if and only if so


does Y .
198. Given a countable ordinal , let A be the class of absolute Borel sets of
additive class . Suppose that X is a metrizable space such that X 2 A
for some  2. Let Y be a metrizable space such that Cp .Y / (or Cp .Y /) is
a uniformly continuous image of Cp .X / (or Cp .X / respectively). Prove that
u

Y 2 A . In particular, if X  Y then X belongs to A if and only if so does Y .


199. Prove that every nonempty countable compact space X is homeomorphic to
the space C 1 D f W  g for some countable ordinal . Here, as usual,
the set C 1 is considered with the topology generated by the well ordering
on C 1.
u
200. Let X and Y be infinite countable compact spaces. Prove that X  Y , i.e., the
spaces Cp .X / and Cp .Y / are uniformly homeomorphic.

1.3 Linear Topological Spaces and l-Equivalence

25

1.3 Linear Topological Spaces and l-Equivalence


Given a space X , consider the family L of all continuous maps of X into locally
convex linear topological spaces of cardinality not exceeding jX j 2! . If ' 2 L, we
denote by L' the relevant linear Q
topological space. The map i D f' W ' 2 Lg
is an embedding of X into L D fLf W f 2 Lg. The linear span L.X / of i.X /
in L is called the free locally convex space of the space X . We will identify X and
L
i.X /. A space X is said to be L-equivalent to a space Y (or X  Y ), if L.X / is
l

linearly homeomorphic to L.Y /. The spaces X and Y are l-equivalent (or X  Y )


if Cp .X / is linearly homeomorphic to Cp .Y /. A topological property P (a cardinal
invariant ') is called l-invariant if it is preserved by l-equivalence, i.e., if X ` P
l

(or '.X /  ) and X  Y imply Y ` P (or '.Y /   respectively).


The space Lp .X / is theQlinear span of X in Cp .Cp .X //. If Lt is a linear space
for all t 2 T , the set L D fLt W t 2 T g carries a natural structure of linear space:
let .x C y/.t / D x.t / C y.t / and .x/.t / D x.t / for any t 2 T; x; y 2 L and
2 R. The set L with the operations defined above is called the product of linear
spaces fLt W t 2 T g. If each Lt is a linear topological space, the linear space L is
always considered with the respective product topology.
If f W X ! Y is a map of a space XSonto a set Y , consider the families  D
fU  Y W f 1 .U / 2 .X /g and  D ffg 1 .U / W U 2 .R/g W g 2 RY and
g f 2 C.X /g. The family  is called the quotient topology induced by f . The
map f W X ! .Y; / is quotient and the space .Y; / may fail to be a Tychonoff
space. The topology  0 generated by  as a subbase is called the R-quotient topology
induced by the map f . The map f W X ! .Y;  0 / is R-quotient and .Y;  0 / is a
completely regular (but, maybe, not Tychonoff) space. If X is a space and F is a
nonempty closed subset of X , let XF D .X nF / [ fF g. For any y 2 X nF , let
pF .y/ D y and pF .x/ D F for any x 2 F . The set XF with the R-quotient
topology induced by the contraction map pF W X ! XF is called the R-quotient
space XF . Given two retractions r; s W X ! X in a space X , we say that r is
parallel to s if r s D r and s r D s. Two retracts of a space X are called parallel
if they are images of X under parallel retractions. If  exp.X / and Y  X then
jY D fU \ Y W U 2 g.
A subset B of a linear space L is balanced if tB D ftx W x 2 Bg  B for any
t 2 I. The set B is absorbing if, for any x 2 L, there exists > 0 such that tx 2 B
for all t 2 .; /. Recall that 0L denotes the zero vector of L. If L is clear, we write
0 instead of 0L . The set B is linearly bounded or l-bounded if, for any U 2 .0; L/,
there exists s > 0 such that B  t U for all t  s. A subset A of a space X is called
bounded if f .A/ is bounded in R for any f 2 C.X /. If L and M are linear spaces, a
map f W L ! M is called linear if f .x C y/ D f .x/ C f .y/ for all x; y 2 L
and ; 2 R. The linear topological spaces M and L are linearly homeomorphic if
there exists a linear map f W L ! M which is a homeomorphism. Given a linear
space L, a function p W L ! R is a seminorm on L, if p.x Cy/  p.x/Cp.y/ and
p.x/ D jjp.x/ for all x; y 2 L and 2 R. If, additionally, p.x/ 0 for any

26

1 Properties Preserved by Homeomorphisms of Function Spaces

x 0, then p is called a norm; in this case we write jjxjj instead of p.x/. A family
P of seminorms on a linear space L is called separating if, for any x 0, there
is p 2 P such that p.x/ 0. Given an absorbing set A in a linear space L, the
Minkowski functional A is defined on L as follows: A .x/ D infft > 0 W xt 2 Ag
for any x 2 L.
If L is a linear topological space then H  L is a Hamel basis of L if H is
linearly independent (i.e., for any distinct x1 ; : : : ; xn 2 H and 1 ; : : : ; n 2 R, the
equality 1 x1 C : : : C n xn D 0 implies i D 0 for all i  n) and the linear
span of H is equal to L. Now, L D ff 2 C.L/ W f is a linear functionalg and
L0 D ff 2 RL W f is a linear functionalg; the space L is called the dual of L. The
topology on L with the subbase f' 1 .U / W U 2 .R/; ' 2 L g is called the weak
topology of L. The set L with the topology inherited from Cp .L/ is the weak dual
of L. Given a closed linear subspace N  L, let .x/ D x C N for any x 2 L.
Denote the set f.x/ W x 2 Lg by L=N . For any x; y 2 L let .x/ C .y/ D
.x C y/ and .x/ D .x/. Then .x/ D .x 0 / and .y/ D .y 0 / imply
.x/ C .y/ D .x 0 / C .y 0 / and .x/ D .x 0 /; thus L=N , with the zero
element N D .0/, is a linear space called the quotient space of L over N . If L=N
is dealt with as a topological space, it is assumed to carry the quotient topology
 D fU  L=N W  1 .U / 2 .L/g, induced by .
A linear topological space L is (completely) normable if there exists a norm jj jj
on L such that the metric d.x; y/ D jjx  yjj (is complete and) generates .L/.
The space L is barreled if any convex closed balanced and absorbing subset of L
is a neighborhood of 0 in L. Suppose that X is a topological space and .Y; U / is a
uniform space. A family F  C.X; Y / is called equicontinuous at a point x 2 X if,
for any U 2 U , there exists an open V  X such that x 2 V and .f .x/; f .y// 2 U
for any y 2 V and f 2 F. If the family F is equicontinuous at every x 2 X , it
is called equicontinuous. Note that a set F  C.X / is equicontinuous at x 2 X if
and only if, for any " > 0, there is U 2 .x; X / such that jf .y/  f .x/j < " for
any y 2 U and f 2 F. If L and M are linear topological spaces and F is a set of
linear continuous maps from L to M , then F is equicontinuous if and only if, for
any U 2 .0M ; M /, there exists V 2 .0L ; L/ such that f .V /  U for any f 2 F.
Suppose that X is a space and C is a cover of X . If " > 0 and P1 ; : : : ; Pn 2
C then P1 ; : : : ; Pn ; "
D ff 2 C.X / W f .Pi /  ."; "/ for all i  ng. The
family C D f;g [ fU  C.X / W for any f 2 U , there are n 2 N; " > 0 and
P1 ; : : : ; Pn 2 C such that f C P1 ; : : : ; Pn ; "
 U g is a topology on C.X / and, if
every P 2 C is bounded in X , then .C.X /; C / is a linear topological space. The
topology C is called the topology of uniform convergence on the elements of C. If
C D fK  X W K is compactg then .C.X /; C / is denoted by Ck .X / and C is called
the compact-open topology. Denote by BDX the family of all bounded subsets of X .
The set C.X / with the topology of uniform convergence on the elements of BDX
is denoted by Cb .X /. A function f W X ! R is called b-continuous if, for any
B 2 BDX , there is g 2 C.X / with gjB D f . The space X is called a bf -space
if every b-continuous function on X is continuous. A set P  C.X / is pointwise
bounded if the set ff .x/ W f 2 P g is bounded in R for every x 2 X .

1.3 Linear Topological Spaces and l-Equivalence

27

201. Prove that the topology of any linear topological T0 -space is Tychonoff.
202. Let L be a linear topological Tychonoff space. Prove that, for any local base
B of the space L at 0, the following properties hold:
(1)
(2)
(3)
(4)
(5)

for any U; V 2 B, there is W 2 B such


T that W  U \ V ;
every B 2 B is an absorbing set and B D f0g;
for any U 2 B, there exists V 2 B such that V C V  U ;
for any U 2 B and x 2 U , there exists V 2 B such that x C V  U ;
for any U 2 B and " > 0 there is V 2 B such that V  U for any
2 ."; "/.

Prove that, if L is a linear space without topology and B is a family of subsets


of L which has the properties (1)(5) then there exists a unique Tychonoff
topology on L such that .L; / is a linear topological space and B is a local
base of at 0.
203. Let L be a linear topological space. Prove that
T
(1) for any local base B of L at 0 and any A  L, we have A D fA C V W
V 2 Bg;
(2) for any A; B  L, we have A C B  A C B;
(3) if M is a linear subspace of L then M is also a linear subspace of L;
(4) if C is a convex subset of L then the sets C and Int.C / are also convex;
(5) if B is a balanced subset of L then B is also balanced; if, additionally, we
have 0 2 Int.B/ then Int.B/ is also balanced;
(6) if E is an l-bounded subset of L then E is also l-bounded.
204. Let L be a linear topological space. Prove that
(1) every neighborhood of 0 contains an open balanced neighborhood of 0;
(2) every convex neighborhood of 0 contains an open convex balanced
neighborhood of 0.
Deduce from (2) that any locally convex linear topological space has a local
base B at 0 such that each U 2 B is convex and balanced.
205. Let L be a linear topological space. Given a nontrivial linear functional
f W L ! R, prove that the following properties are equivalent:
(i)
(ii)
(iii)
(iv)

f is continuous;
f 1 .0/ is closed in L;
f 1 .0/ is not dense in L;
there exists U 2 .0; L/ such that f .U / is a bounded subset of R.

206. Suppose that L is a locally convex linear topological space which has a
countable local base at 0. Prove that there exists a metric d on the set L with
the following properties:
(i) d generates the topology of L;
(ii) all d -open balls are convex and all balls with the center at 0 are balanced;
(iii) the metric d is invariant, i.e., d.xCz; yCz/ D d.x; y/ for all x; y; z 2 L.
As a consequence, a locally convex space is metrizable if and only if it has
countable character.

28

1 Properties Preserved by Homeomorphisms of Function Spaces

207. Let p be a seminorm on a linear space L. Prove that


(1)
(2)
(3)
(4)

p.0/ D 0 and p.x/  0 for any x 2 L;


jp.x/  p.y/j  p.x  y/ for any x; y 2 L;
fx 2 L W p.x/ D 0g is a linear subspace of L;
the set B D fx W p.x/ < 1g is convex, balanced, absorbing, and p D B .

208. Let A be a convex absorbing set in a linear space L. Prove that


(1)
(2)
(3)
(4)

A .x C y/  A .x/ C A .y/ for any x; y 2 L;


A .tx/ D tA .x/ for any x 2 L and t  0;
if A is balanced then A is a seminorm;
if B D fx 2 L W A .x/ < 1g and C D fx 2 L W A .x/  1g then
B  A  C and A D B D C .

209. Given a locally convex linear topological space L, take any local base B at 0
such that all elements of B are convex and balanced. Prove that fV W V 2 Bg
is a separating family of continuous seminorms on L.
210. Let P be a separating family of seminorms on a linear space L. Given p 2 P
and n 2 N, let O.p; n/ D fx 2 L W p.x/ < n1 g. Prove that the family
B D fO.p1 ; n/ \ : : : \ O.pn ; n/ W n 2 N; p1 ; : : : ; pn 2 Pg is a convex
balanced local base at 0 for some topology on L such that .L; / is a locally
convex space in which all elements of P are continuous and any E  L is
l-bounded if and only if p.E/ is bounded for any p 2 P.
211. Prove that a linear topological space is normable if and only if it has a convex
l-bounded neighborhood of zero.
212. Let N be a closed subspace of a linear topological space L. Prove that
(1) the quotient topology of L=N makes L=N a linear topological space;
(2) the quotient map  W L ! L=N is linear, open, and continuous;
(3) If P 2 fmetrizability, local convexity, normability, complete normabilityg
and L has P then L=N also has P.
213. Prove that any product of locally convex spaces is a locally convex space.
214. Suppose that L and M are linear topological spaces and is an equicontinuous family of linear maps from L to M . Prove that, for any l-bounded set
A  L there is an l-bounded set B  M such that f .A/  B for all f 2 .
215. Suppose that L and M are linear topological spaces and is a family of linear
continuous maps from L to M . Let .x/ D ff .x/ W f 2 g for every x 2 L
and assume that the set B D fx 2 L W .x/ is l-bounded in M g is of second
category in L. Prove that B D L and the family is equicontinuous.
216. (HahnBanach theorem) Let L be a linear space (without topology). Suppose
that we are given a map p W L ! R such that p.x C y/  p.x/ C p.y/ and
p.tx/ D tp.x/ for all x; y 2 L and t  0. Prove that, for any linear subspace
M of the linear space L and any linear functional f W M ! R such that
f .x/  p.x/ for any x 2 M , there exists a linear functional F W L ! R such
that F jM D f and p.x/  F .x/  p.x/ for any x 2 L.

1.3 Linear Topological Spaces and l-Equivalence

29

217. Let L be a linear space (without topology). Suppose that we are given a
seminorm p W L ! R, a linear subspace M  L and a linear functional
f W M ! R such that jf .x/j  p.x/ for any x 2 M . Prove that there exists
a linear functional F W L ! R such that F jM D f and jF .x/j  p.x/ for
any x 2 L.
218. Given a linear topological space L prove that any nontrivial continuous linear
functional f W L ! R is an open map.
219. Let L be a linear topological space and suppose that A and B are nonempty
disjoint convex subsets of L and A is open. Prove that there exists a continuous
linear functional f W L ! R such that, for some r 2 R, we have
f .x/ < r  f .y/ for any x 2 A and y 2 B.
220. Let L be a locally convex linear topological space. Suppose that A and B are
disjoint convex subsets of L such that A is compact and B is closed. Prove
that there exists a continuous linear functional f W L ! R such that, for some
r; s 2 R, we have f .x/ < r < s < f .y/ for any x 2 A and y 2 B.
221. Let L be a locally convex linear topological space. Prove that L separates the
points of L.
222. Let M be a linear subspace of a locally convex linear topological space L and
x0 M . Prove that there exists f 2 L such that f .x0 / D 1 and f(M)={0}.
223. Let B be a closed convex balanced subset of a locally convex space L. Prove
that, for any x 2 LnB, there exists a continuous linear functional f W L ! R
such that f .B/  1; 1
and f .x/ > 1.
224. Let L be a locally convex linear topological space. Given a linear subspace M
of the linear space L and a continuous linear functional f W M ! R, prove
that there exists a functional g 2 L such that gjM D f .
225. Given a linear space L (without topology) denote by L0 the family of all linear
functionals on L. Suppose that M  L0 is a linear subspace of L0 (i.e., f C
g 2 M whenever f; g 2 M and ; 2 R) and M separates the points of L;
let  be the topology generated by the set M . Then LM D .L; / is a locally
convex space and .LM / D M . Deduce from this fact that if L is a locally
convex space and Lw is the set L with the weak topology of the space L then
Lw is a locally convex space such that .Lw / D L .
226. Let E be a convex subset of a locally convex space L. Prove that the closure
of E in L coincides with the closure of E in the weak topology of L.
227. Let V be a neighborhood of 0 in a locally convex space L. Prove that the set
P .V / D ff 2 L W f .V /  1; 1
g is compact if considered with the
topology induced from Cp .L/.
228. Given n 2 N suppose that L is a linear topological space and M is a linear
subspace of L of linear dimension n. Prove that M is closed in L and every
linear isomorphism ' W Rn ! M is a homeomorphism.
229. Given a linear topological space L prove that the following conditions are
equivalent:
(i) L has a finite Hamel basis, i.e., the linear dimension of L is finite;
(ii) dim L  n for some n 2 N;
(iii) L is locally compact.

30

1 Properties Preserved by Homeomorphisms of Function Spaces

230. Suppose that L is a finite-dimensional linear topological space. Prove that


any linear functional f W L ! R is continuous on L, i.e., L0 D L .
Give an example of an infinite-dimensional locally convex space M such that
M 0 D M .
231. Let L be a locally convex space. Denote by L0  RL the set of all (not
necessarily continuous) linear functionals on L with the topology induced
from RL . Prove that L is dense in L0 .
232. Given a linear space L let L0  RL be the set of all linear functionals on L
with the topology induced from RL . Prove that L0 is linearly homeomorphic
to RB for some B.
233. For any linear topological space L denote by w .L/ the weak topology of the
space L. Prove that
(1) if w .L/ D .L/ and M is a linear subspace of L then w .M / D .M /;
(2) for any space X , the topology of Cp .X / coincides with its weak topology;
(3) for any space X , the topology of Lp .X / coincides with its weak topology.
234. Suppose that L is a locally convex space with its weak topology and X  L
is a Hamel basis in L. Prove that the following conditions are equivalent:
(i) there exists a linear homeomorphism h W L ! Lp .X / such that
h.x/ D x for all x 2 X ;
(ii) for every f 2 C.X / there exists a continuous linear functional
' W L ! R such that 'jX D f ;
(iii) for every continuous map f W X ! M from X to a locally convex
space M with its weak topology, there exists a continuous linear map
W L ! M such that jX D f .
235. Given a space X let .Lp .X // D f' 2 Cp .Lp .X // W ' is a linear functional
on Lp .X /g and consider the restriction map  W .Lp .X // ! Cp .X /.
Prove that .Lp .X // is a closed linear subspace of Cp .Lp .X // and  is a
linear homeomorphism. Deduce from this fact that the operation of extending
continuous real-valued functions on X to continuous linear functionals on
Lp .X / is also a linear homeomorphism between Cp .X / and .Lp .X // .
236. Prove that any Lp .X / is homeomorphic to a dense subspace of RA for some A.
Deduce from this fact that every uncountable regular cardinal is a precaliber
of Lp .X /. In particular, c.Lp .X // D ! for any space X .
237. Given spaces X and Y prove that
(i) there exists a linear continuous map of Cp .X / onto Cp .Y / if and only if
Lp .Y / is linearly homeomorphic to a linear subspace of Lp .X /;
(ii) there exists a linear continuous open map of Cp .X / onto Cp .Y / if and
only if Lp .Y / is linearly homeomorphic to a closed linear subspace of
Lp .X /;
(iii) the space Cp .X / linearly condenses onto Cp .Y / if and only if Lp .Y / is
linearly homeomorphic to a dense linear subspace of Lp .X /;
(iv) Cp .X / is linearly homeomorphic to Cp .Y / if and only if Lp .Y / is
linearly homeomorphic to Lp .X /.

1.3 Linear Topological Spaces and l-Equivalence

31

238. Given spaces X and Y prove that


(i) there is a linear continuous map of Cp .X / onto Cp .Y / if and only if Y
is homeomorphic to a subspace Y 0  Lp .X / such that every f 2 C.Y 0 /
extends to a continuous linear functional on Lp .X /;
(ii) the space Cp .X / linearly condenses onto Cp .Y / if and only if Y is
homeomorphic to a subspace Y 0  Lp .X / such that every f 2 C.Y 0 /
extends to a uniquely determined continuous linear functional on Lp .X /;
(iii) Cp .X / is linearly homeomorphic to Cp .Y / if and only if Y is homeomorphic to some Y 0  Lp .X / whose linear hull coincides with Lp .X /
and every f 2 C.Y 0 / extends to a continuous linear functional on
Lp .X /.
239. Let P be a class of spaces which have the following properties:
(1) if Y 2 PSand Z is a continuous image of Y then Z 2 P;
(2) if Y D fYi W i 2 !g; Yi  YiC1 ; Yi 2 P and Yi closed in Y for every
i 2 !, then Y 2 P;
(3) if Y 2 P and n 2 N then Y n  Rn 2 P;
Prove that if a space X belongs to P then Lp .X / 2 P.
240. Prove that i w.Lp .X // D .Lp .X // D .Lp .X // D i w.X / for any space
X ; show that we also have nw.X / D nw.Lp .X // and d.X / D d.Lp .X //.
241. Prove that, for any space X , we have the following equalities.
(i) s  .X / D s.Lp .X // D s  .Lp .X //;
(ii) hl  .X / D hl.Lp .X // D hl  .Lp .X //;
(iii) hd  .X / D hd.Lp .X // D hd  .Lp .X //.
242. Given a space X prove that l  .X / D l.Lp .X // D l  .Lp .X // and
ext  .X / D ext .Lp .X // D ext  .Lp .X //.
243. Prove that an uncountable regular cardinal  is a caliber of X if and only if 
is a caliber of Lp .X /.
244. Denote by L the following collection of classes of Tychonoff spaces: fanalytic
spaces, K-analytic spaces,  -compact spaces, Lindelf -spaces, realcompact spacesg. Prove that, for any class P from the list L, a space X belongs to
P if and only if Lp .X / belongs to P.
245. Given w D 1 x1 C : : : C n xn 2 Lp .X /, where x1 ; : : : ; xn 2 X and
1 ; : : : ; n 2 Rnf0g, let supp.w/ D fx1 ; : : : ; xn g. If w D 0, then supp.w/ D ;.
Say that a set B  Lp .X / is weakly bounded if .B/ is a bounded subset of
R for any continuous linear functional  W Lp .X / ! R. Observe that any
bounded subset of Lp .X / is weaklySbounded and prove that, for any weakly
bounded set B  Lp .X /, the set fsupp.w/ W w 2 Bg is bounded in the
space X .
246. Prove that, for any Dieudonn complete space X , if A is a bounded subset of
Lp .X / then A is compact.
247. Suppose that a space X has a weaker metrizable topology and A is a bounded
subset of Lp .X /. Prove that A is compact and metrizable.

32

1 Properties Preserved by Homeomorphisms of Function Spaces

248. Prove that, for any infinite pseudocompact space X , there exists an infinite
closed discrete set D in the space Lp .X / which is weakly bounded in Lp .X /.
Therefore, even for a metrizable compact space X , the closure of a weakly
bounded subset of Lp .X / can fail to be compact.
249. Give an example of a space X in which all compact subspaces are metrizable
while there are non-metrizable compact subspaces in Lp .X /.
250. Given spaces X and Y and a continuous map ' W X ! Y observe that
there exists a unique continuous linear map u' W Lp .X / ! Lp .Y / such
that u' jX D '. Prove that the following conditions are equivalent for any
continuous onto map ' W X ! Y .
(i)
(ii)
(iii)
(iv)

The map ' is R-quotient.


The map u' is R-quotient.
The map u' is quotient.
The map u' is open.

251. Let f W X ! Y be an R-quotient map. Prove that, for any open U  Y , the
map f j.f 1 .U // W f 1 .U / ! U is also R-quotient.
252. Let X be a Tychonoff space. Prove that, for any nonempty closed set F  X,
the R-quotient space XF is also Tychonoff and if pF W X ! XF is the
contraction map then pF j.X nF / W X nF ! XF nfF g is a homeomorphism.
253. Suppose that X is a space and F is a nonempty closed subspace of X ; in the
R-quotient space XF denote by aF the point represented by the set F . Say that
F is deeply inside a set U 2 .X / if there exists a zero-set G in the space X
such that F  G  U . For the family U D fU W U is a cozero subset of X
and F is deeply inside the set U g prove that V D ffaF g [ .U nF / W U 2 U g is
a local base of the space XF at the point aF .
254. Suppose that X is a normal space and F is a nonempty closed subspace of
X ; in the R-quotient space XF denote by aF the point represented by the
set F . Prove that U 2 .aF ; XF / if and only if .U nfaF g/ [ F is an open
neighborhood of F in the space X .
255. Suppose that X is a space and K is a nonempty compact subspace of X ; in the
R-quotient space XK denote by aK the point represented by the set K. Prove
that U 2 .aK ; XK / if and only if .U nfaK g/ [ K is an open neighborhood of
K in the space X .
256. Given a nonempty space X prove that closed sets P; Q  X are parallel
retracts of X if and only if there exists a retraction r W X ! P such that
rjQ W Q ! P is a homeomorphism.
257. (Okunevs method of constructing l-equivalent spaces). Suppose that P and Q
are parallel retracts of a nonempty space X . Prove that the completely regular
quotient spaces XP and XQ are l-equivalent.
258. Suppose that K is a nonempty l-embedded subspace of a space X and fix a
point a X . Prove that the spaces X fag and XK K are l-equivalent.
l

Deduce from this fact that if K is a retract of the space X then X fag  XK
K. Here XK is the R-quotient space obtained by contracting K to a point.

1.3 Linear Topological Spaces and l-Equivalence

33

259. Given a space Xi and a point xi 2 Xi for any i D 1; : : : ; n consider the space
X D X1 : : : Xn and the set F D fx1 ; : : : ; xn g  X . The R-quotient
space XF is denoted by .X1 ; x1 / _ : : : _ .Xn ; xn / and called a bunch of spaces
X1 ; : : : ; Xn with respect to the points x1 ; : : : ; xn . Prove that if we choose any
point yi 2 Xi for every i D 1; : : : ; n then the spaces .X1 ; x1 / _ : : : _ .Xn ; xn /
and .X1 ; y1 / _ : : : _ .Xn ; yn / are l-equivalent.
260. Let K be a retract of a nonempty space X and fix any point z 2 K. Denote by
aK the point of the space XK represented by the set K. Prove that the space X
is l-equivalent to the bunch .XK ; aK / _ .K; z/ of the spaces XK and K with
respect to the points aK and z.
261. Assume that K and L are retracts of a nonempty space X and there exists
a retraction r W X ! L such that r.K/ D K \ L D fag for some point
a 2 L; let M D K [ L. Prove that the space X is l-equivalent to the bunch
.XM ; c0 / _ .K; c1 / _ .L; c2 / where the points c0 2 XM ; c1 2 K and c2 2 L
are chosen arbitrarily.
262. Given spaces Y and Z consider the space X D Y  Z; choose arbitrary points
y0 2 Y; z0 2 Z and let M D .Y  fz0 g/ [ .fy0 g  Z/. Prove that, for any
x0 2 XM , the space X is l-equivalent to the bunch .XM ; x0 /_.Y; y0 /_.Z; z0 /.
1
263. Let a D 0 and an D nC1
for all n 2 !; then S D fan W n 2 !g [ fag is a
faithfully indexed convergent sequence with limit a. Given an infinite cardinal
 consider the discrete space D./ of cardinality  and let E D D./  S .
Observe that F D D./  fag is a retract of E; as usual let EF be the
R-quotient space obtained by contracting F to a point. The space EF will
be denoted by V ./; it is often called the FrchetUrysohn -fan. The space
V .!/ is called the FrchetUrysohn fan. Prove that V ./ is l-equivalent to
D./  S for any infinite cardinal . Deduce from this fact that
(i) there exist l-equivalent spaces X and Y with w.X / w.Y / and
.X / .Y /;
(ii) metrizability is not preserved by l-equivalence;

(iii) a space l-equivalent to a locally compact space need not be Cechcomplete.


264. Given infinite cardinals 1 ; : : : ; n prove that A.1 / : : : A.n / is
l-equivalent to the space A./ where  D maxf1 ; : :L
: ; n g.
265. Given a family of spaces fXt W t 2QT g let X D
t2T Xt and prove that
Cp .X / is linearly homeomorphic to t2T Cp .Xt /. Deduce from this fact that
L
l
l
if Xt  Yt for any t 2 T then X  Y D t2T Yt .
266. Suppose
L that a space Ji is homeomorphic to I for any i D 1; : : : ; n and let
J D fJi W 1  i  ng. Prove that the space J D is l-equivalent to I for
any finite space D. Deduce from this fact that
(i) connectedness is not preserved by l-equivalence;
(ii) for any cardinal  there exist l-equivalent spaces X and Y such that X
has no isolated points and Y has -many isolated points.

34

1 Properties Preserved by Homeomorphisms of Function Spaces

267. Let X be a compact space with jX j D   !. Prove that AD.X / is


l-equivalent to X A./. Here AD.X / is the Alexandroff double of the
space X and A./ is the one-point compactification of a discrete space of
cardinality .
268. Let X be a compact space such that jX j D   !. Prove that AD.X / is
l-equivalent to AD.X / A./. Here AD.X / is the Alexandroff double of the
space X .
269. Prove that there exist l-equivalent compact spaces X and Y such that
.X / .Y /. As a consequence, pseudocharacter is not l-invariant.
270. Prove that there exist l-equivalent compact spaces X and Y such that X has
nontrivial convergent sequences and Y does not have any.
271. Prove that for any uncountable regular cardinal  with its usual order topology
l

we have the equivalencies   A./ and .C1/ .C1/A./. Therefore


there exist l-equivalent spaces X and Y such that all compact subspaces of X
are metrizable while Y has non-metrizable compact subspaces.
272. Suppose that compact spaces X and Y are l-equivalent. Prove that X  Z is
l-equivalent to Y  Z for any space Z.
l

273. Given a family fX1 ; : : : ; Xn g of compact spaces assume that Xi  Yi for all
i 2 f1; : : : ; ng. Prove that the spaces X D X1 : : :Xn and Y D Y1 : : :Yn
are l-equivalent.
274. Give an example of l-equivalent spaces X and Y such that X  Z is not
t -equivalent to Y  Z for some space Z.
275. Give an example of l-equivalent spaces X and Y such that X  X is not
t -equivalent to Y  Y .
276. Given infinite cardinals 1 ; : : : ; n prove that A.1 /: : :A.n / is l-equivalent
to the space A./ where  D maxf1 ; : : : ; n g.
277. Prove that there exist l-equivalent spaces X and Y such that X is hereditarily
paracompact while Y is not hereditarily normal.
278. Prove that Lp .D/ is l-equivalent to Lp .D/ D for any infinite discrete space
D. Deduce from this fact that the Souslin property is not l-invariant.
279. Given spaces X and Y suppose that ' W Cp .X / ! Cp .Y / is a continuous
linear surjection. Prove that,
(i) for any point y 2 Y , there exist uniquely determined n D n.y/ 2 N,
distinct points x1 .y/; : : : ; xn .y/ 2 X and numbers 1 .y/; : : : ; n .y/ 2
Pn.y/
Rnf0g such that '.f /.y/ D iD1 i .y/f .xi .y// for any f 2 Cp .X /;
for further reference denote the set fx1 .y/; : : : ; xn .y/g by supp.'; y/.
(ii) if '  W Cp .Cp .Y // ! Cp .Cp .X // is the dual map of ' then '  embeds
Lp .Y / in Lp .X / and '  .y/ D 1 .y/x1 .y/ C : : : C n.y/ .y/xn.y/ .y/ for
any y 2 Y .
280. Suppose that ' W Cp .X / ! Cp .Y / is a continuous linear surjection and let
.y/ D supp.'; y/ for any y 2 Y . Prove that the map  W Y ! exp.X / is
lower semicontinuous.

1.3 Linear Topological Spaces and l-Equivalence

35

281. Given a continuous linear surjection ' W Cp .X / ! CpS


.Y / prove that, for any
bounded subset A of the space Y , the set supp.A/ D fsupp.'; y/ W y 2 Ag
is bounded in X .
282. Suppose that ' W Cp .X / ! Cp .Y / is a continuous linear surjection. Prove
that, for any bounded subset B  X , the set C D fy 2 Y W supp.'; y/  Bg
is bounded in Y .
283. Say that X is a -space if A is compact for any bounded set A  X . Prove
that X is a -space if and only if Lp .X / is a -space. As a consequence,
-property is preserved by l-equivalence.
284. Given spaces X and Y assume that X is a -space and there exists a linear
surjection ' W Cp .X / ! Cp .Y / which is an R-quotient map. Prove that
Y is also a -space. Give an example of a compact space X (which is,
automatically, a -space) such that there exists a continuous linear surjection
of Cp .X / onto Cp .Y / for some Y which is not a -space.
285. Given -spaces X and Y , let ' W Cp .X / ! Cp .Y / be a continuous linear
surjection. Prove that, if X is compact then Y is also compact. Observe that
the same conclusion about Y may be false if Y is not a -space.
286. Given -spaces X and Y , let ' W Cp .X / ! Cp .Y / be a continuous linear
surjection. Prove that, if X is  -compact then Y is also  -compact. Observe
that the same conclusion about Y may be false if Y is not a -space.
287. For any space X let K.X / be the family of all compact subspaces of X .

Prove that a second countable space X is Cech-complete


if and only if there
exists a Polish space M and a map ' W K.M / ! K.X / such that, for any
F; G 2 K.M / the inclusion F  G implies '.F /  '.G/ and, for any
P 2 K.X /, there exists F 2 K.M / such that '.F /  P .
288. Let X and Y be second countable spaces for which there is a continuous linear

surjection of Cp .X / onto Cp .Y /. Prove that, if X is Cech-complete


then Y is

also Cech-complete. In particular, if two second countable spaces X and Y are

l-equivalent then X is Cech-complete


if and only if so is Y .
289. Give an example of second countable l-equivalent spaces X and Y such that
X is pseudocomplete and Y is not Baire. As a consequence, having a dense

Cech-complete
subspace is not an l-invariant property in the class of second
countable spaces.
290. Prove that there exist l-equivalent  -compact second countable spaces X and
Y such that X can be condensed onto a compact space and Y doesnt have
such a condensation.
291. Prove that a countable second countable space is scattered if and only if it is

Cech-complete.
Deduce from this fact that if X and Y are countable second
countable l-equivalent spaces then X is scattered if and only if Y is scattered.
292. Let X and Y be metrizable spaces such that Cp .X / is linearly homeomorphic
to Cp .X /  Cp .X / and Cp .Y / is linearly homeomorphic to Cp .Y /  Cp .Y /.
Prove that if X embeds in Y as a closed subspace and Y embeds in X as a
closed subspace then X and Y are l-equivalent.

36

1 Properties Preserved by Homeomorphisms of Function Spaces

293. Let X be a countable second countable space. Prove that the following
properties are equivalent:
(i)
(ii)
(iii)
(iv)

X
X
X
X

is l-equivalent to Q;
is not scattered;
has a subspace homeomorphic to Q;
has a closed subspace homeomorphic to Q.

294. Prove that, for any infinite cardinal  there exist l-equivalent spaces X and Y
such that X is dense-in-itself and Y has a dense set of -many isolated points.

295. Suppose that a second countable space X is Cech-complete


and has a closed
!
subspace homeomorphic to R . Prove that X is l-equivalent to R! .
296. Let K be an uncountable metrizable compact space. Prove that K is
l-equivalent to K E for any metrizable zero-dimensional compact space E.
297. Prove that a compact space X is l-equivalent to the Cantor set K if and
only if X is metrizable, zero-dimensional, and uncountable. As a consequence, any two zero-dimensional metrizable uncountable compact spaces are
l-equivalent.
298. Prove that a second countable space X is l-equivalent to space P of the
irrational numbers if and only if X is non- -compact, zero-dimensional, and

Cech-complete.
299. Given any n 2 N prove that a compact set K  Rn is l-equivalent to In if
l

and only if In embeds in K. Deduce from this fact that K  In if and only if
dim K D n.
300. Prove that a space X is l-equivalent to I! if and only if X is compact,
metrizable and I! embeds in X .

1.4 Metrizable Spaces and l-Equivalence

37

1.4 Metrizable Spaces and l-Equivalence


All topological spaces are assumed to be Tychonoff. Given a space X , let  .X / D
.X /nf;g and X C D X f0g; by K.X / we denote the family of all compact subsets
of X . If L and M are linear topological spaces, the expression L
M says that L
is linearly homeomorphic to M ; the space M is a linear topological factor of L if
there exists a linear topological space N such that L
M  N . If, for a compact
X , we treat C.X / as a Banach space, then the respective norm is defined by jjf jj D
supff .x/ W x 2 X g for any f 2 C.X /. A sequence fxn g  L is called linearly
Cauchy if, for any U 2 .0; L/, there is m 2 ! such that xn  xk 2 U for any
n; k  m. Given spaces X and Y , a map f W X ! Y is called compact-covering if,
for any compact K  Y , there is a compact K 0  X such that f .K 0 / D K.
A space X is called hemicompact if there exists a sequence fKn W n 2 !g of
compact subsets of X such that, for any compact K  X , we have K  Kn for some
n 2 !. The space X is an @0 -space if there exists a countable family N  exp.X /
which is a network for K.X /, i.e., for any compact K  X and any open U  K,
we have K  P  U for some P 2 N . The space X is a q-space if, for any x 2 X ,
there is a sequence fUn W n 2 !g  .x; X / such that the sequence fxn W n 2 !g
has an accumulation point whenever xn 2 Un for each n 2 !. A space X is called a
-space if A is compact for any bounded A  X . A space X is  -metacompact if
any open cover of X has a  -point-finite open refinement, i.e., a refinement which
is a countable union of point-finite families.
A family K of subsets of a space X has a discrete open expansion if there is a
discrete family fUK W K 2 Kg  .X / such that K  UK for any K 2 K. A family
K of nonempty compact subsets of X is called a moving off collection if, for any
compact L  X , there is K 2 K such that K \ L D ;. A space X has the moving
off property if every moving off collection contains an infinite subcollection which
has a discrete open expansion. For a locally compact non-compact X , let .X /
be its one-point compactification. A family A of subsets of X is T1 -separating if,
for any distinct x; y 2 X , there are A; B 2 A such that A \ fx; yg D fxg and
B \ fx; yg D fyg.
The GruenhageMa game is played on a space X by players I and II . For
every n 2 N the n-th move of player I is to choose a set Kn 2 K.X / while the
player II responds with a set Ln 2 K.X nKn /. The player I wins if S
the collection
fLn W n 2 Ng chosen by II has a discrete open expansion. Let G D f.K.X //n W
n 2 Ng; a strategy of player II in GruenhageMa game on a space X is a map
s W G ! K.X /, such that s.K1 ; : : : ; Kn / 2 K.X nKn / for every .K1 ; : : : ; Kn / 2 G.
A play fKi ; Li W i 2 Ng of a GruenhageMa game is said to have been played by II
applying a strategy s if Ln D s.K1 ; : : : ; Kn / for all n 2 N. A strategy s of player
II in GruenhageMa game is called winning on a space X if II wins in every play
on X , in which he (or she!) applies the strategy s.
A BanachMazur game on a space X is a two-person game in which players E
(for empty) and NE (for nonempty) take turns picking a nonempty open subset of X
contained in the opponents previous move (if any). Thus, if U1 2  .X / is the first

38

1 Properties Preserved by Homeomorphisms of Function Spaces

move of E, player NE has to respond with a set V1 2  .U1 /. The second move of
E has to be some U2 2  .V1 /. Then NE has to choose V2 2  .U2 / and so on. The
BanachMazur game as described above, where E makes the first move, is called
an E-game. Now, if V1 2  .X / is the first move of NE, player E has to respond
with a set U1 2  .V1 /. The second move of NE has to be some V2 2  .U1 /. Then
E has to choose U2 2  .V2 / and so on. The BanachMazur game, where player
NE makes the first move, is called an NE-game. In both games player E wins after
! moves if the intersection of the moves is empty. Otherwise the winner is NE. The
sequence of moves is called a play of the relevant game.
A strategy of player E in E-game on a space X is a map s defined inductively as
follows. First we have to choose a set U1 D s.;/ 2  .X /. If the strategy s is defined
for first n moves then an n-tuple .V1 ; : : : ; Vn / 2 .  .X //n is called admissible if
V1  U1 and Vi  Ui D s.V1 ; : : : ; Vi1 / for any i 2 f2; : : : ; ng. For any admissible
n-tuple .V1 ; : : : ; Vn / we have to choose a set UnC1 D s.V1 ; : : : ; Vn / 2  .Vn /.
We say that E applies the strategy s in a play fUi ; Vi W i 2 Ng of an E-game if
U1 D s.;/ and UkC1 D s.V1 ; : : : ; Vk / for all k 2 N. A strategy s of player E in
E-game is winning on a space X if E wins in every play on X , in which he (or
she!) applies s.
To define inductively a strategy s of player E in NE-game on a space X we
have to choose a set s.V1 / 2  .V1 / for any V1 2  .X /. If the strategy s is defined
for the first n moves then say that an .n C 1/-tuple .V1 ; : : : ; VnC1 / 2 .  .X //nC1
is admissible if ViC1  s.V1 ; : : : ; Vi / for all i D 1; : : : ; n. If .V1 ; : : : ; VnC1 / 2
.  .X //nC1 is admissible then we have to choose s.V1 ; : : : ; VnC1 / 2  .VnC1 /. We
say that E applies the strategy s in a play fVi ; Ui W i 2 Ng of an NE-game if
Un D s.V1 ; : : : ; Vn / for all n 2 N. A strategy s of player E in NE-game is winning
on X if E wins in every play on X , in which he (or she!) applies s.
Given a linear space L and a norm jj jj on L let d.x; y/ D jjx  yjj for any
x; y 2 L. It is easy to see that d is a metric on L; if is the topology generated
by the metric d then is called the topology generated by the norm jj jj. A linear
topological space is normable if its topology is generated by a norm.

1.4 Metrizable Spaces and l-Equivalence

39

301. Prove that there exist l-equivalent spaces X and Y such that X is hereditarily
paracompact while Y is not collectionwise normal.
302. Prove that there exist l-equivalent spaces X and Y such that X is collectionwise normal while Y is not normal.
303. Prove that there exist l-equivalent spaces X and Y such that X is hereditarily
normal while Y is not normal.
304. Prove that -weight is not preserved by l-equivalence neither in the class of
compact spaces nor in the class of countable spaces.
305. Give an example of l-equivalent spaces X and Y with ext .X / ext .Y /.
306. Prove that there exist l-equivalent spaces X and Y such that X is Frchet
Urysohn while t .Y / > ! and there is a non-closed set A  Y such that B \ A
is finite whenever B is a bounded subset of Y . As a consequence, Frchet
Urysohn property, k-property, sequentiality, and countable tightness are not
l-invariant.
307. Show that FrchetUrysohn property is not preserved by l-equivalence in the
class of compact spaces.
308. Let Y be a space in which every closed subspace has the Baire property.
Suppose that Y is l-equivalent to a space X and a nonempty set Z  X
also has the Baire property. Prove that there is a nonempty W  Z which is
open in Z and homeomorphic to a subspace of Y .

309. Let X and Y be Cech-complete


l-equivalent spaces. Prove that every
nonempty subspace of X has a -base whose elements are embeddable in

Y . Deduce from this fact that if X and Y are nonempty Cech-complete


l-equivalent spaces and X is scattered then Y is also scattered.

310. Let X and Y be Cech-complete


l-equivalent spaces such that .X /  . Prove
that Y has a dense open subspace D such that .x; Y /   for each x 2 D.
In particular, .D/  .
311. Prove that X is a closed Hamel basis of L.X / for any space X .
312. Prove that, for any space X and any continuous map f W X ! L of X to
a locally convex space L, there exists a unique continuous linear map f W
L.X / ! L such that f jX D f . Observe that this makes it possible to
consider that L.X /, as a linear space, coincides with Lp .X /  C.Cp .X //
while the topology of L.X / is stronger than .Lp .X //. In all problems that
follow we use this observation identifying the underlying set of L.X / with
Lp .X /.
313. Suppose that L is a locally convex space such that X is embedded as a Hamel
basis in L. Prove that the following conditions are equivalent:
(i) there exists a linear homeomorphism i W L ! L.X / such that i.x/ D x
for all x 2 X ;
(ii) every continuous function f W X ! M of the space X to a locally
convex space M can be extended to a continuous linear functional f W
L ! M.

40

1 Properties Preserved by Homeomorphisms of Function Spaces

314. Prove that, for any space X , the set .L.X // coincides with the set .Lp .X // .
Deduce from this fact that the weak topology of the space L.X / coincides
with the topology of Lp .X /.
315. Given a space X let E be the weak dual of L.X /, i.e., E D .L.X // and
the topology of E is induced from Cp .L.X //. For every f 2 E let .f / D
f jX , i.e.,  W E ! Cp .X / is a restriction map. Prove that  is a linear
homeomorphism and hence E is linearly homeomorphic to Cp .X /.
316. Observe that l-equivalence implies u-equivalence, i.e., for any spaces X and
l

317.
318.

319.

320.

321.

322.

323.

324.
325.

Y , if X  Y then X  Y . Prove that L-equivalence implies l-equivalence.


Prove that Cb .X / is complete (as a uniform space with its linear uniformity)
if and only if X is a bf -space.
Given a space X call a set P  C.X / equicontinuous at a point x 2 X if, for
any " > 0 there exists U 2 .x; X / such that f .U /  .f .x/"; f .x/C"/ for
any f 2 P . The family P is called equicontinuous if it is equicontinuous at
every point x 2 X . Say that P is pointwise bounded if the set ff .x/ W f 2 P g
is bounded in R for any x 2 X . Prove that, for any equicontinuous pointwise
bounded set P  C.X /, the closure of P in the space Cb .X / is compact.
In particular, if X is pseudocompact and P  C.X / is equicontinuous and
pointwise bounded then the closure of P in Cu .X / is compact.
Prove that, for any bf -space X , a set P  C.X / is equicontinuous and
pointwise bounded if and only if the closure of P in the space Cb .X / is
compact.
Given a space X , let P; "
D f' 2 L.X / W '.P /  ."; C"/g for every
P  C.X / and " > 0. Prove that a set U  L.X / is open in L.X / if and
only if, for any  2 U there exists an equicontinuous pointwise bounded set
P  C.X / and " > 0 such that CP; "
 U . In other words, the topology of
L.X / coincides with the topology of uniform convergence on equicontinuous
pointwise bounded subsets of C.X /.
Given a space X and P  X let IP D ff 2 C.X / W f .P /  f0gg. Prove that
for any linear continuous functional ' W Ck .X / ! R which is not identically
zero on Ck .X /, there exists a compact subspace K  X (called the support of
' and denoted by supp.'/) such that '.IK / D f0g and '.IK 0 / f0g whenever
K 0 is a proper compact subset of K.
Recall that a set B is a barrel in a locally convex space L if B is closed,
convex, balanced, and absorbing in L. The space L is barreled if any barrel
in L is a neighborhood of 0. Prove that a locally convex space L is barreled
whenever it has the Baire property.
Prove that Ck .X / is a barreled space if and only if X is a -space, i.e., the
closure of any bounded subspace of X is compact. Deduce from this fact that
Ck .X / is barreled for any realcompact space X .
Prove that Cp .X / is barreled if and only if all bounded subspaces of X are
finite.
Give an example of a space X such that Cp .X / barreled but does have the
Baire property.

1.4 Metrizable Spaces and l-Equivalence

41

326. Given a point z in a space Z, say that a family B of subsets of Z is a local


base of neighborhoods of Z at the point z, if z 2 Int.U / for any U 2 B, and
z 2 V 2 .Z/ implies U  V for some U 2 B. Prove that the family of
all barrels in Ck .X / constitutes a local base of neighborhoods of 0 in Cb .X /.
Deduce from this fact that the family of all barrels in Cp .X / is also a local
base of neighborhoods of 0 in Cb .X /.
327. Let ' W Ck .X / ! Cp .Y / be a linear continuous map. Prove that ' is
continuous considered as a map from Cb .X / to Cb .Y /.
328. Assuming that a space Y is l-equivalent to a bf -space X prove that Y is also
a bf -space. In other words, bf -property is l-invariant.
329. Let ' W Lp .X / ! Lp .Y / be a linear homeomorphism. Prove that, if X is a
bf -space then ' is a linear homeomorphism of L.X / onto L.Y /.
330. Let X and Y be spaces one of which is a bf -space. Prove that X is
L-equivalent to Y if and only if X and Y are l-equivalent.
331. Prove that there exist l-equivalent spaces which are not L-equivalent.
332. Prove that a space X has a weaker metrizable topology if and only if L.X / has
a weaker metrizable topology. In particular, if X and Y are L-equivalent and
X can be condensed onto a metrizable space then Y can also be condensed
onto a metrizable space.
333. Suppose that X and Y are l-equivalent spaces. Prove that, if X is metrizable,
then Y can be condensed onto a metrizable space.
334. Say that a space X is  -metrizable if X is the countable union of its closed
metrizable subspaces. Prove that a space X is  -metrizable and paracompact
if and only if L.X / is  -metrizable and paracompact.
335. Suppose that a space X is l-equivalent to a metrizable space. Prove that X is
 -metrizable and paracompact.
336. For an arbitrary space X , prove that X is hemicompact if and only if Ck .X /
is first countable.
337. Prove that hemicompactness is preserved by l-equivalence.
338. Given a space X prove that
(i) if X is a k-space then a sequence ffn W n 2 !g  Ck .X / is convergent
whenever it is linearly Cauchy;
(ii) for a hemicompact space X the converse is true, i.e., if any linearly
Cauchy sequence in Ck .X / is convergent then X is a k-space.
339. Let X be an arbitrary space. Prove that X is a hemicompact space with
k-property if and only if Ck .X / is metrizable by a complete metric.
340. Let X and Y be l-equivalent spaces. Prove that X is a hemicompact k-space
if and only if Y is a hemicompact k-space.
341. Prove that any subspace of an @0 -space is an @0 -space and any countable
product of @0 -spaces is an @0 -space.
342. Observe that a compact-covering continuous image of an @0 -space is an
@0 -space. Prove that a space X is an @0 -space if and only if X is a compactcovering continuous image of a second countable space.

42

1 Properties Preserved by Homeomorphisms of Function Spaces

343. Prove that a space X is an @0 -space with the k-property if and only if it is a
quotient image of a second countable space.
344. Prove that any @0 -space of countable character is second countable.
345. Let ' W X ! Y be a continuous map. Recall that the dual map '  W Ck .Y / !
Ck .X / is defined by the formula '  .f / D f ' for every f 2 Ck .Y /.
Assuming that ' is compact-covering, prove that '  is an embedding.
346. Given a compact subspace K of a space X let v.f; x/ D f .x/ for every
f 2 Ck .X / and x 2 K. Prove that the map v W Ck .X /K ! R is continuous.
347. Prove that the following properties are equivalent for any space X :
(i) X is an @0 -space;
(ii) Ck .X / is an @0 -space.
(iii) Ck .X / has a countable network.
348. Let X and Y be l-equivalent spaces. Prove that X is an @0 -space if and only
if so is Y . In particular, if some space Z is l-equivalent to a second countable
space then Z is an @0 -space. Deduce from this fact that any first countable
space l-equivalent to a second countable space must be second countable.
349. Suppose that a space X has a countable network and Y is an @0 -space. Prove
that Cp .X; Y / has a countable network.
350. Given spaces X; Y and a function u W X  Y ! R let ux .y/ D u.x; y/ for
all y 2 Y ; then ux W Y ! R for every x 2 X . Analogously, let uy .x/ D
u.x; y/ for all x 2 X ; then uy W X ! R for every y 2 Y . Say that the
function u is separately continuous if the functions ux and uy are continuous
(on Y and X respectively) for all x 2 X and y 2 Y . Let Cps .X  Y / be
the set of all separately continuous functions on X  Y with the topology
induced from RXY . Observe that Cps .X  Y / is a locally convex space and
let .'/.x; y/ D '.x/.y/ for any continuous function ' W X ! Cp .Y /. Prove
that .'/ 2 Cps .X Y / for every ' 2 Cp .X; Cp .Y // and  W Cp .X; Cp .Y // !
Cps .X  Y / is a linear homeomorphism.
351. Prove that the space Cp .X; Cp .X // has a countable network if and only if X
is countable. Deduce from this fact that Cp .X / is an @0 -space if and only if X
is countable.
352. Prove that a space X is of second category in itself if and only if the player E
has no winning strategy in the BanachMazur NE-game on X .
353. Prove that a space X has the Baire property if and only if the player E has no
winning strategy in the BanachMazur E-game on X .
354. Prove that
(i) any pseudocompact space with the moving off property is compact.
(ii) any paracompact locally compact space has the moving off property.
355. Let X be a q-space. Prove that, if X has the moving off property then it is
locally compact.

1.4 Metrizable Spaces and l-Equivalence

43

356. Prove that the following conditions are equivalent for any space X ;:
(i) X has the moving off property;
(ii) given a sequence fKi W i 2 !g of moving off collections in X , we can
choose Ki 2 Ki for each i 2 !, such that the family fKi W i 2 !g has a
discrete open expansion;
(iii) the player II has no winning strategy in the GruenhageMa game on X .
357. Prove that, if Ck .X / has the Baire property then the space X has the moving
off property.
358. Prove that, for any q-space X , the following conditions are equivalent:
(i) Ck .X / has the Baire property;
(ii) X has the moving off property;
(iii) the player II has no winning strategy in GruenhageMa game on the
space X .
359. Let X be a paracompact q-space. Prove that Ck .X / has the Baire property if
and only if X is locally compact.
360. Let X be a paracompact q-space. Prove that, if X is l-equivalent to a locally
compact paracompact space then X is also locally compact. In particular,
any first countable paracompact space l-equivalent to a locally compact
paracompact space is locally compact. Deduce from this fact that
(i) if X and Y are metrizable l-equivalent spaces then X is locally compact
if and only if Y is locally compact;
(ii) if a first countable space X is l-equivalent to a second countable locally
compact space then X is also locally compact and second countable.
361. Suppose that a q-space X is l-equivalent to a locally compact metrizable
space. Prove that X is metrizable and locally compact.

362. Suppose that X is l-equivalent to a metrizable space, Y  X and Y is Cech


complete. Prove that Y is metrizable. In particular, if a Cech-complete space
X is l-equivalent to a metrizable space then X is metrizable.
363. Suppose that X is l-equivalent to a metrizable space. Prove that A is an
@0 -space for any countable set A  X .
364. For any space X let dc.X / D supfjU j W U   .X / and U is a discrete
familyg. Prove that if X is l-equivalent to a metrizable space then nw.X / D
dc.X /. In particular, if X is l-equivalent to a metrizable space then both the
Souslin property of X and ext .X /  ! imply that X has a countable network.
365. Given a space X and a first countable space Y assume that there exists a
continuous linear surjection ' W Cp .X / ! Cp .Y /. For any y 2PY there exist
x1 ; : : : ; xn 2 X and 1 ; : : : ; n 2 Rnf0g such that '.f /.y/ D niD1 i f .xi /
for any f 2 Cp .X /; denote the set fx1 ; : : : ; xn g by supp.y/. Suppose that U is
a locally finite open cover of X and let T .U / D fy 2 Y W supp.y/ \ U ;g
for every U 2 U . Prove that the family fT .U / W U 2 U g is a locally finite
open cover of Y .

44

1 Properties Preserved by Homeomorphisms of Function Spaces

366. Let X and Y be metrizable spaces for which there exists a continuous linear

surjection of Cp .X / onto Cp .Y /. Prove that, if X is Cech-complete


then Y is

also Cech-complete.

367. Prove that if a metrizable space X is l-equivalent to a Cech-complete


space

then X is also Cech-complete. As a consequence, if X and Y are l-equivalent


metrizable spaces then X is metrizable by a complete metric if and only if so
is Y .
368. Show that there exist first countable l-equivalent spaces X and Y such that X
is locally compact and Y is not locally compact.
369. Given a space Z let Z 0 be the set of non-isolated points of Z. Suppose
that X and Y are normal first countable l-equivalent spaces. Prove that if
X 0 is countably compact then Y 0 is also countably compact. Show that this
statement can be false if we omit first countability of X and Y .
370. Given a nonempty closed subspace F of a normal space X suppose that F is
a retract of some neighborhood of F . Prove that F is l-embedded in X and
hence Cp .X /
Cp .F /  I , where I D ff 2 Cp .X / W f .F / D f0gg.
371. Given a space X assume that X0 and X1 are closed subspaces of X such
that X D X0 [ X1 and the set F D X0 \ X1 is l-embedded in X ;
suppose additionally that Cp .F /
Cp .F /  Cp .F /. Prove that Cp .X /

Cp .X0 /  Cp .X1 /.
372. Suppose that a space X has a nontrivial convergent sequence. Prove that X is
l

l-equivalent to X .! C 1/ and X C  X . Deduce from this fact that X C is


l-equivalent to X for every infinite metrizable space X .
373. Suppose that a space X has a nontrivial convergent sequence and Y is
l-embedded in X . Prove that X is l-equivalent to XY Y . Consequently,
l

for any infinite metrizable space X , if Y is closed in X then X  XY Y .


Here XY is the R-quotient space obtained from X by contracting Y to a point.
374. Suppose that X is a compact space and F is l-embedded in X . Prove that X C
is l-equivalent to F .X nF /. In particular, if X is an infinite metrizable
l

compact space then X  F .X nF / for any closed F  X .


375. Let X and Y be metrizable compact spaces. Suppose that F and G are
l

closed subspaces of X and Y respectively such that F  G and X nF is


homeomorphic to Y nG. Prove that X is l-equivalent to Y .
376. Let X and Y be nonempty compact metrizable spaces such that either Y 
.! C 1/ or .Y  !/ embeds in X . Prove that Cp .X /
Cp .X /  .Cp .Y //n
for every n 2 N.
377. Say that a metrizable compact space K is universal in the dimension n 2 ! if
dim K D n and any metrizable compact space of dimension at most n embeds
in K. Prove that if X and Y are metrizable compact spaces universal in the
l

dimension n, then X  Y .
378. Suppose that a space X is l-equivalent to Y and Y is a metrizable compact
space universal in the dimension n. Prove that X is also a metrizable compact
space universal in the dimension n.

1.4 Metrizable Spaces and l-Equivalence

45

379. Prove that, for any n1 ; : : : ; nk 2 N, the space Cp .In1 /: : :Cp .Ink / is linearly
homeomorphic to Cp .In / where n D maxfn1 ; : : : ; nk g.
380. Given n 2 N and a closed subset F of the space In such that ; F In
prove that .In /F is l-equivalent to In . Here .In /F is the R-quotient image of In
obtained by contracting F to a point.
381. Prove that, for any n 2 N, if U is a nonempty open subset of the space Rn ,
then the space .U / is l-equivalent to In .
382. Given a space X with dim X D n 2 N assume that X is homeomorphic to a
finite union
S of Euclidean cubes, i.e., there is a finite family F of subsets of X
such that F D X and every F 2 F is homeomorphic to Ik for some k 2 N.
Prove that X is l-equivalent to In .
383. For any n 2 N prove that both spaces In .!C1/ and .In !/ are l-equivalent
to In .
384. Prove that In  D! is not l-equivalent to In for any n 2 N.
385. Suppose that K is a compact space and there exists a continuous bijective map
of 0; C1/ onto K. Prove that K is l-equivalent to I. Deduce from this fact
that if there is a continuous bijection of R onto a compact space L then L is
also l-equivalent to I.
386. Assume that X is a second countable S
space and dim X D n 2 !. Let O D
fU 2 .X / W dim U < ng and O D O. The set K.X / D X nO is called
the dimensional kernel of X . Prove that dim O < n and dim W D n for any
nonempty open subset W of the space K.X /.
387. Prove that if n 2 N and a space X is l-equivalent to In then the dimensional
kernel K.X / of the space X is also l-equivalent to In .
388. Call a second countable space Y weakly n-Euclidean if dim Y D n and
every n-dimensional subspace of Y has nonempty interior and contains a
homeomorphic copy of In . Prove that a compact space X is l-equivalent to
In if and only if its dimensional kernel K.X / has a nonempty open weakly
n-Euclidean subspace and every U 2  .K.X // contains a subset which is
l-equivalent to X .
389. Given (linear) topological spaces X and Y the expression X  Y says that
they are (linearly) homeomorphic. Suppose that X !  X and there exist
(linear) topological spaces E and F such that X  Y  F and Y  X  E.
Prove that X  Y .
390. Suppose that L is a linear topological space, M is a linear subspace of L and
there exists a linear retraction r W L ! M . Prove that L
M  r 1 .0/.
Deduce from this fact that for any linear topological spaces L and E there
exists a linear topological space N such that L
E  N (i.e., E is a linear
topological factor of L) if and only if there exists a linear retract E 0 of the
space L such that E 0
E.
391. Given linear topological spaces L; M , and N prove that L
M  N if
and only if there exist linear subspaces M 0 and N 0 of the space L for which
M 0
M; N 0
N and there exist linear retractions r W L ! M 0 and
s W L ! N 0 such that r.x/ C s.x/ D x for any x 2 L.

46

1 Properties Preserved by Homeomorphisms of Function Spaces

392. For any linear topological space L denote by L the set of all continuous
linear functionals on L with the topology inherited from Cp .L/. Prove that,
for any locally convex spaces M and N , we have .M  N /
M   N  . In
particular, Cp .Y / is a linear topological factor of Cp .X / if and only if Lp .Y /
is a linear topological factor of Lp .X /.
393. Suppose that X is a second countable non-compact S
space, n 2 N and there
exists a locally finite cover I of the space X such that fInt.I / W I 2 Ig D X
and every I 2 I is homeomorphic to In . Prove that X is l-equivalent to In !.
394. Prove that, for any n 2 N, every nonempty open subspace of Rn is l-equivalent
l

395.
396.
397.

398.

399.
400.

to In  !. In particular, if U 2  .Rn / then U  Rn .


Given spaces X and Y assume that nw.X / D ! and Cp .Y / is a linear
topological factor of Cp .X /. Prove that dim Y  dim X .
Assuming that X is a compact space and Cp .Y / is a linear topological factor
of Cp .X / prove that dim Y  dim X .
(Open Mapping Theorem) Suppose that L is a Banach space, M is a linear
topological space which is of second category in itself, and f W L ! M is
a surjective continuous linear map. Prove that f is open. In particular, any
continuous linear onto map between two Banach spaces is open.
(Closed Graph Theorem) Suppose that L and M are Banach spaces and f W
L ! M is a linear map such that its graph G D f.x; f .x// W x 2 Lg is closed
in L  M . Prove that the map f is continuous.
Prove that, for any nonempty space X , there exists a continuous surjection of
the space Cp .X / onto Cp .X /  R.
Prove that there exists an infinite compact space K for which there is no linear
continuous surjection of Cp .K/ onto Cp .K/  R. In particular, K C is not
l-equivalent to the space K.

1.5 The Last-Minute Updates. Yet More on l-Equivalence

47

1.5 The Last-Minute Updates. Yet More on l-Equivalence


All spaces are assumed to be Tychonoff; the space D is the doubleton f0; 1g with the
discrete topology. Given an infinite cardinal , say that a set P  X is an F -subset
of X if P is a union of   closed subsets of X ; if Q is the intersection of at most
-many open subsets of X then Q is called a G -subset of X . For any space X let
C
.X / D minf W there is no continuous surjective map of X onto I g. The cardinal
.X / is called the dyadicity index of the space X . Let  .X / D minf  ! W every
closed subset of X is a G -set in X g. If X is a space then x 2 X is called a P -point
in X if x 2 Int.F / for any G -subset F of the space X such that x 2 F .
Given a point  2 !n! let U D fU \ ! W U 2 .; !/g; it is an easy exercise
to see that U is an ultrafilter on !. We will follow the usual practice to identify
 with U for any  2 !n!. In particular, the elements of !n! will be called
ultrafilters on !. Observe that, for each ultrafilter  2 !n! and any set A  ! we
have  2 A if and only if A 2 ; let ! be the set ! [ fg endowed with the topology
inherited from !. Recall that  .RA / D fx 2 RA W jfa 2 A W x.a/ 0gj < !g for
any set A.
Given an infinite cardinal , a space X is called -cosmic if nw.X /  ; the
space X is stronglyS
-cosmic if there exists a family fX W < g of subspaces of
X such that X D fX W < g and nw.X / <  for any < . If P  X
then a set A  X is said to be concentrated around P if AnU is countable for
any U 2 .P; X /. If X is a space and n > 1 is a natural number then the set
n .X / D fx 2 X n W x.i / D x.j / for some distinct i; j < ng is called the
n-diagonal of X . For technical reasons it is convenient to define the 1-diagonal
1 .X / to be the empty
Tfor any space X . If A D fAn W n 2 !g is a sequence of
S set
sets then lim A D n2! in Ai .
A game of two players E and NE on a nonempty space X is called a Choquet
game if at the n-th move E chooses an open set Un and a point xn 2 Un . The
player NE responds by choosing an open set Vn 3 xn with Vn  Un . At the move
.n C 1/ the player E takes a point xnC1 2 Vn and an open set UnC1 such that
xnC1 2 UnC1  Vn . The player NE has to respond with a set VnC1 2 .X / such that
xnC1 2 VnC1  UnC1 . The gameT
ends after the moves .xn ; Un /; Vn are made for all
n 2 ! and the player E wins if fUn W n 2 !g D ;; otherwise, player NE is the
winner. In the point-open game PO on a nonempty space X at the n-th move the
player P chooses a point xn and the player O takes an open set Un which contains
xn . TheS
game ends after the moves xn ; Un are made for all n 2 ! and the player P
wins if n2! Un D X ; otherwise O is the winner.
A neighborhood assignment on a space X is any function
S N W X ! .X / such
that x 2 N.x/ for every x 2 X ; if A  X then N.A/ D fN.x/ W x 2 Ag. Say that
X is a D-space if for any neighborhood assignment N on X there exists a closed
discrete set D  X such that N.D/ D X .

48

1 Properties Preserved by Homeomorphisms of Function Spaces

If is an ordinal then a family fA W < g of subsets of a space X is called


increasing if <  < implies A  A . If X is a space and P  X say that a
family N  exp.X / is an external network (base) of P in X if (N  .X / and)
for any x 2 P and U 2 .x; X / there exists N 2 N such that x 2 N  U .
Given sets X and Y and an infinite cardinal  suppose that we have families
A  exp.X /, B  exp.Y / and a map ' W A ! B. Say that ' is -monotone if
(1) j'.A/j  maxfjAj; !g whenever A 2 A and jAj  ;
(2) if A  B and A; B 2 A, then '.A/  '.B/;
0
0
(3) if   is an infinite cardinal,
SfA W < g
S A and < implies A  A
then we have the equality '. < A / D < '.A /.
Given an infinite cardinal , say that a space X is (strongly) monotonically
-monolithic if, for any A  X with jAj  , there exists an external network
(base) O.A/ of the set A in X such that the assignment A ! O.A/ is -monotone.
If X is a space and f W X ! Y is a continuous map, then a family N  exp.X /
is a network for f if for any x 2 X and U 2 .f .x/; Y / there exists N 2 N
such that x 2 N and f .N /  U . Say that a space X is monotonically -stable
if to any set B  Cp .X / with jBj   we can assign a network N .B/ for the
map eB W X ! Cp .B/ in such a way that the correspondence B ! N .B/ is
-monotone. Recall that eB .x/.f / D f .x/ for any f 2 B.
A space X is (strongly) monotonically monolithic if, for any A  X , there exists
an external network (base) O.A/ of the set ASin X such that jO.A/j
S  maxfjAj; !g
while A  B implies O.A/  O.B/ and O. fA W < g/D fO.A / W < g
whenever is an ordinal and fA W < g is an increasing family of subsets of X .
Say that a space X is monotonically retractable if we can assign to any countable
A  X a retraction rA W X ! X and a countable network N .A/ for the map rA
such that A  rA .X / and the assignment N is !-monotone. Say that a space X is
monotonically Sokolov if we can assign to any countable family F of closed subsets
of X a continuous retraction rF W X ! X and a countable external network N .F/
for rF .X / in X such that rF .F /  F for each F 2 F and the assignment N is
!-monotone.
Say that X is a  -space if the space X has a  -discrete network. A spaceS
X is a
strong -space if there exists a family C of compact subsets of X such that C D
X and there exists a  -discrete family N  exp.X / which is a network modulo C,
i.e., for any C 2 C and U 2 .C; X / there exists N 2 N with C  N  U . Call
a space Z hereditarily Baire if every closed subspace of Z has the Baire property.
A space X is called d -separable if it has a dense  -discrete subspace.
Given any ordinal let C 0 D ; if is an ordinal and we defined C then
C . C 1/ D . C / C 1. If is a limit ordinal and we defined C  for any
 < then C D supf C  W  < g. This defines an ordinal C for any
ordinals and . If is an ordinal then let 0 D 0. If is an ordinal and we have
defined then . C 1/ D C . If is a limit ordinal and we defined 
for every  < then D supf  W  < g. This defines an ordinal for

1.5 The Last-Minute Updates. Yet More on l-Equivalence

49

any ordinals and . For each ordinal let 0 D 1. If is an ordinal and we have
defined then C1 D . / . If is a limit ordinal and we defined  for every
 < then D supf  W  < g. This defines an ordinal for any ordinals
and .
Say that a space X is finite-dimensional
if dim X  n for some n 2 !. The space
S
X is  -zero-dimensional if X D i2! Xi and every Xi is zero-dimensional.

50

1 Properties Preserved by Homeomorphisms of Function Spaces

401. Suppose that  is an infinite cardinal and X is a Lindelf -space such that
t .X /   and  .X /  . Prove that X has a -base of order  . In
particular, if X is a Lindelf -space with t .X / D  .X / D ! then X has
a point-countable -base. Deduce from this fact that any Lindelf -space X
with .X /   has a -base of order  . In particular, any first countable
Lindelf -space has a point-countable -base.
402. Given a space X and an infinite cardinal  suppose that  .X /   and
d.X /   C . Prove that X has a -base of order  . In particular, if
 .X /  ! and d.X /  !1 then X has a point-countable -base.
403. Assuming CH prove that
(i) any Lindelf first countable space has a point-countable -base;
(ii) any space X with .X / D c.X / D ! has a point-countable -base;
(iii) if !1 is a caliber of X and .X /  ! then X is separable.
404. Give an example of a first countable space which has no point-countable
-base.
405. Let X be a space for which we can find a family of sets fAm W m 2 !g and a
sequence fkm W m 2 !g  Nnf1g such that supfjAm j W m 2 !g D jX j while
Am  X km n km .X / and Am is concentrated around km .X / for every m 2 !.
Prove that the space Cp .X / has a point-countable -base. In particular, the
space Cp .X / has a point-countable -base if there is a set A with jAj D jX j
such that either A  X n n n .X / and A is concentrated around n .X / or
A  X and A is concentrated around some point of X .
406. Prove that
(a) Cp ./ has a point-countable -base whenever is an ordinal with its
order topology;
(b) Cp .AD.X // has a point-countable -base for any countably compact
space X . Here AD.X / is the Alexandroff double of X .
407. Prove that the following conditions are equivalent for any infinite space X
with l  .X /  !:
(i) Cp .X / has a point-countable -base;
(ii) there is a family of sets fAm W m 2 !g and a sequence fkm W m 2 !g 
Nnf1g such that supfjAm j W m 2 !g D jX j while Am  X km n km .X /
and Am is concentrated around km .X / for every m 2 !.
408. Given a space X suppose that the cardinality of X is regular and uncountable
while l  .X / D !, i.e., all finite powers of X are Lindelf. Prove that the
space Cp .X / has a point-countable -base if and only if there exists a natural
number n > 1 such that some set A  X n n n .X / is concentrated around
n .X / and jAj D jX j.
409. Given a metrizable space X prove that Cp .X / has a point-countable -base if
and only if X is countable.

1.5 The Last-Minute Updates. Yet More on l-Equivalence

51

410. Suppose that X is an infinite space with l  .X / D !. Prove that if Cp .X / has


a point-countable -base then jX j D .X /. Here .X / D minf W 2 .X / is
a G -subset of X  X g is the diagonal number of the space X . Deduce from
this fact that if X is compact and Cp .X / has a point-countable -base then
w.X / D jX j.
411. Prove that if K is a scattered Corson compact space then Cp .K/ has a pointcountable -base.
412. Prove that if Cp .X / embeds in a -product of first countable spaces then it
has a point-countable -base.
413. Say that a space X is P -favorable (for the point-open game) if the player P
has a winning strategy in the point-open game on X . Prove that
(i)
(ii)
(iii)
(iv)
(v)

any countable space is P -favorable;


a continuous image of a P -favorable space is P -favorable;
the countable union of P -favorable space is P -favorable;
any nonempty closed subspace of a P -favorable space is P -favorable;
any nonempty Lindelf scattered space is P -favorable.

414. Prove that


(a) if X is P -favorable for the point-open game and .X /  ! then X is
countable;
(b) a compact space X is P -favorable for the point-open game if and only if
X is scattered.
415. Given a nonempty space X define a game PO0 on the space X as follows: at
the n-th move the player P chooses a finite set Fn  X and the player O takes
a set Un 2 .X / such that Fn  Un . The gameSends after the moves Fn ; Un
are made for all n 2 !. The player P wins if n2! Un D X ; otherwise the
victory is assigned to O. Prove that the player P has a winning strategy in the
game PO0 on a space X if and only if X is P -favorable for the point-open
game.
416. Given a nonempty space X define a game PO00 on the space X as follows: at
the n-th move the player P chooses a finite set Fn  X and the player O takes
a set Un 2 .X / such that Fn  Un . After all moves fFn ; Un W n 2 !g are
made let U D fUn W n 2 !g. The player P wins if lim U D X ; otherwise the
victory is assigned to O. Prove that the player P has a winning strategy in the
game PO00 on a space X if and only if X is P -favorable for the point-open
game.
417. Prove that a space X is P -favorable for the point-open game if and only if
Cp .X / is a W -space.
418. Assuming that 2! D !1 and 2!1 D !2 prove that there exists a scattered
Lindelf P -space X such that hd  .X / D !1 and jX j D !2 . Show that Cp .X /
is a W -space with no point-countable -base. In particular, Cp .X / is a W space which cannot be embedded into a -product of first countable spaces.

52

1 Properties Preserved by Homeomorphisms of Function Spaces

419. Prove
that if Xt is a d -separable space for each t 2 T then the product space
Q
X
t is d -separable. In other words, any product of d -separable spaces
t2T
must be d -separable.
420. Given an infinite cardinal  and a space X prove that X  is d -separable if
and only if there exists a family D D fDn W n 2 !g of discrete subspaces of
X  such that supfjDn j W n 2 !g  d.X /. In particular, if X  has a discrete
subspace of cardinality d.X / then X  is d -separable. Deduce from this fact
that X d.X/ is d -separable for any space X .
421. Prove that
(a) if K is a compact space then K ! is d -separable;
(b) there exists a compact space K such that K n is not d -separable for any
n 2 N. Thus K ! is d -separable but no finite power of K is d -separable.
422. Prove that if Cp .X / is d -separable then there is a discrete subspace D 
Cp .X / such that jDj D d.Cp .X //.
423. Given a space X and n 2 N say that a discrete subspace D  X n is essential if
D \ n .X / D ; and jDj D i w.X /. Prove that if, for some n 2 N, there exists
an essential discrete set D  X n then Cp .X / is d -separable. In particular,
if there exists a discrete subspace of X of cardinality i w.X / then Cp .X / is
d -separable.
424. Assume that X is a space for which there exists a discrete subspace D 
X  X such that jDj D i w.X /. Prove that Cp .X / is d -separable.
425. Let X be a space such that the cardinal  D i w.X / has uncountable cofinality.
Prove that the following conditions are equivalent:
(i)
(ii)
(iii)
(iv)

.Cp .X //n is d -separable for all n  2;


.Cp .X //n is d -separable for some n  2;
Cp .X /  Cp .X / is d -separable;
for some m 2 N there is a discrete set D  X m with jDj D .

426. Prove that


(a) if supfs.X n / W n 2 Ng > i w.X / then Cp .X / is d -separable;
(b) if K is a Corson compact space then Cp .K/ is d -separable;
(c) if X is a metrizable space then Cp .X / is d -separable.
427. Prove that if Cp .X / is a Lindelf -space then it is d -separable.
428. Prove that if Cp .X / is a Lindelf -space then the space X must be
hereditarily d -separable.
429. Prove that under CH,
(a) there exists a compact space K such that Cp .K/ is not d -separable;
(b) there exists a space X such that X  X is d -separable while X is not
d -separable.
430. Suppose that  is an infinite cardinal, T ; is a set, and Nt is a space such
that nw.Nt /   for all t 2 T . Assume that D is a dense subspace of N D
Q
t2T Nt and f W D ! K is a continuous map of D onto a compact space K.

1.5 The Last-Minute Updates. Yet More on l-Equivalence

431.

432.

433.

434.

435.

436.

437.

438.

439.

53

Prove that if .K/   then w.K/  . Deduce from this fact that if a compact
space K is a continuous image of a dense subspace of a product of cosmic
spaces then w.K/ D t .K/ D .K/.
Suppose that  is a cardinal of uncountable cofinality, T ; is a set, and Nt
is a space such that
Q nw.Nt /  ! for all t 2 T . Assume that D is a dense
subspace of N D t2T Nt and f W D ! K is a continuous map of D onto a
compact space K with w.K/ D . Prove that K maps continuously onto I .
Given an infinite
cardinal  suppose that nw.Nt /   for any t 2 T and
Q
C  N D t2T Nt is a dense subspace of N . Assume additionally that we
have a continuous (not necessarily surjective) map ' W C ! L of C into a
compact space L. Prove that if y 2 C 0 D '.C / and h .y; L/   then
.y; C 0 /  . Here the cardinal h .y; L/ is the hereditary -character of
the space L at the point y, i.e., h .y; L/ D supf .y; Z/QW y 2 Z  Lg.
Suppose that C is a dense subspace of a product N D t2T Nt such that
nw.Nt /   for each t 2 T . Assume that K is a compact space with
t .K/   and ' W C ! K is a continuous (not necessarily surjective) map; let
C 0 D '.C /. Prove that every closed subspace of C 0 is a G -set; in particular,
.C 0 /  .
Q
Suppose that C is a dense subspace of a product N D t2T Nt such that
nw.Nt /   for each t 2 T . Assume additionally that l.C /   and K
is a compact space with t .K/   such that there exists a continuous (not
necessarily surjective) map ' W C ! K. Prove that if C 0 D '.C / then
hl.C 0 /  .
Prove that if C is a dense subspace of a product of cosmic spaces and K
is a compact space then, for any continuous map ' W C ! K, we have
 .'.C //  t .K/.
Suppose that C is a dense subspace of a product of cosmic spaces and ' W
C ! K is a continuous (not necessarily surjective) map into a compact space
K of countable tightness; let Y D '.C /. Prove that Y is a perfect space and
w.Y /  !. In particular, if ' W Cp .X / ! K is a continuous map of Cp .X /
into a compact space K with t .K/  ! then '.Cp .X // is a perfect space of
countable -weight.
Suppose that C is a dense Lindelf -subspace of a product of cosmic
spaces and ' W C ! K is a continuous (not necessarily surjective) map
into a compact space K of countable tightness. Prove that nw.'.C //  !.
In particular, if Cp .X / is a Lindelf -space and ' W Cp .X / ! K is a
continuous map of Cp .X / into a compact space K of countable tightness then
'.Cp .X // is cosmic.
Given an infinite cardinal  observe that the property P of being strongly
-cosmic is stronger than being -cosmic. Besides, P is preserved by
subspaces and continuous images. Prove
Q that if  < cf./ and X is strongly
-cosmic for any <  then X D < X is also strongly -cosmic.
Given an infinite cardinal  prove that D is not strongly -cosmic and hence
no strongly -cosmic space can be continuously mapped onto I .

54

1 Properties Preserved by Homeomorphisms of Function Spaces

440. Assume that  is an infinite cardinal and K is a strongly -cosmic compact


space. Prove that there exists x 2 K such that .x; K/ < .
441. Suppose that  is an uncountable cardinal and X is a space such that w.X /  
and l.X / < cf./. Prove that Cp .X / is strongly -cosmic. In particular, if
cf./ > ! and X is a Lindelf space with w.X / D  then Cp .X / is strongly
-cosmic.
442. Let  be a cardinal of uncountable cofinality. Prove that if X is a Lindelf
-space with nw.X /   then Cp .X / is strongly -cosmic.
443. Given an uncountable cardinal  suppose that a space X is strongly
-monolithic, i.e., w.A/   for every A  X with jAj   and, additionally,
l.X / < cf./. Prove that w.K/ <  for any compact continuous image K
of the space Cp .X /. Deduce from this fact that if X is a Lindelf strongly
!1 -monolithic space (in particular, if l.X / D ! and w.X /  !1 ) then every
compact continuous image of Cp .X / is metrizable.
444. Given a cardinal  with cf./ > ! assume that X is a -monolithic Lindelf
-space. Prove that if K is a compact continuous image of Cp .X / then
w.K/ < . In particular, if X is an !1 -monolithic Lindelf -space then
every compact continuous image of Cp .X / is metrizable.
445. Let  be a cardinal and denote by D a discrete space of cardinality . For the
compact space K D D prove that Cp .K/ maps continuously onto I .
446. Prove that every one of the following statements is equivalent to Luzins axiom
(2!1 > c/:
(i) for every separable compact space K any compact continuous image of
Cp .K/ is metrizable;
(ii) any compact continuous image of Cp .!/ is metrizable;
(iii) any compact continuous image of Cp .Ic / is metrizable;
(iv) for every compact space K with w.K/  c, any compact continuous
image of Cp .K/ is metrizable.
447. Under Luzins Axiom prove that
(a) if X is Lindelf and w.X /  c then every compact continuous image of
Cp .X / is metrizable.
(b) if X is Lindelf and first countable then every compact continuous image
of Cp .X / is metrizable.
448. Prove that, for every hereditarily Lindelf first countable space X , any
compact continuous image of Cp .X / is metrizable. In particular, for each
perfectly normal compact space X , any compact continuous image of Cp .X /
is metrizable.
449. Suppose that X is a space such that l.Cp .X // D t .Cp .X // D !. Prove that
every compact continuous image of Cp .X / is metrizable.
450. Prove that if X is a space such that Cp .X / is a Lindelf -space then every
compact continuous image of Cp .X / is metrizable.
451. For the double arrow space K prove that the space Cp .K/ can be condensed
onto I! .

1.5 The Last-Minute Updates. Yet More on l-Equivalence

55

452. Prove that Cp .Dc / condenses onto I! .


453. Suppose that X is a nonempty  -compact second countable space. Prove that
Cp .X / condenses onto a compact space.
454. Prove that there exists a set X  R such that Cp .X / does not condense onto
an analytic space.
455. Given a nonempty space X such that jCp .X /j < 2d.X/ prove that Cp .X /
cannot be condensed onto a compact space. Deduce from this fact that neither
one of the spaces Cp .!n!/ and Cp ..Dc // condenses onto a compact space.
Here .Dc / D fx 2 Dc W jx 1 .1/j  !g is the -product of Dc .
456. Prove that if 2! D !1 and 2!1 > !2 then Cp .D!2 / does not condense onto a
compact space.
457. Suppose that 2! < 2!1 and Cp .X / condenses onto a compact space. Prove
that X is separable if and only if jCp .X /j  c.
458. Suppose that l.X n / D ! for all n 2 N and Cp .X / is Lindelf. Observe that
if Cp .X / condenses onto a compact space K then K is metrizable and X is
separable. Prove that if Cp .X / condenses onto a  -compact space Y then X is
separable and .Y / D !. Deduce from this fact that if X is a non-metrizable
Corson compact space then Cp .X / does not condense onto a  -compact space.
459. Given a space X prove that the space Cp .X / condenses onto a space
embeddable in a compact space of countable tightness if and only if Cp .X /
condenses onto a second countable space.
460. Assuming MAC:CH prove that if K is a compact space such that Cp .K/ is
Lindelf and condensable onto a  -compact space then K is metrizable.
461. Assume that Cp .X / is a Lindelf -space and there exists a condensation of
Cp .X / onto a  -compact space Y . Prove that nw.X /  ! and nw.Y /  !.
462. Prove that, for any compact space X such that w.X /  !1 , the space Cp .X /
is hereditarily metalindelf.
463. Given an uncountable discrete space X prove that the space Cp .X; D/ is not
metalindelf; therefore Cp .X / is not metalindelf either.
464. Prove that
(i) every strong -space is a D-space. In particular, any Lindelf -space
is a D-space;
(ii) every space with a point-countable base is a D-space.
465. Prove that,
(i) for any infinite cardinal , if a space is monotonically -monolithic then
it is -monolithic;
(ii) for any infinite cardinal , if X is a monotonically -monolithic space
and Y  X then Y is also monotonically -monolithic;
(iii) if  is an infinite cardinal, then any countable product of monotonically
-monolithic spaces is monotonically -monolithic;
(iv) for any infinite cardinal , every closed continuous image of a monotonically -monolithic space is monotonically -monolithic;

56

1 Properties Preserved by Homeomorphisms of Function Spaces

(v) a space X is monotonically monolithic if and only if X is monotonically


-monolithic for any infinite cardinal ;
(vi) for any infinite cardinal , a space X is monotonically -monolithic if
and only if to any finite set F  X we can assign a countable family
O.F / 
Sexp.X / in such a way that for any set A  X with jAj  , the
family fO.F / W F 2 A
<! g is an external network at all points of A;
(vii) a space X is monotonically monolithic if and only if to any finite set
F  X we can assign a countable family
S O.F /  exp.X / in such
a way that for any A  X , the family fO.F / W F 2 A
<! g is an
external network at all points of A;
(viii) if X is a space and the set X 0 of non-isolated points of X has a countable
network then X is monotonically monolithic. In particular, if a space X
is cosmic or has a unique non-isolated point then X is monotonically
monolithic;
(ix) given an infinite cardinal , if a space X is monotonically -monolithic
and t .X /  , then X is monotonically monolithic;
(x) for any infinite cardinal , every  -product of monotonically monolithic spaces is monotonically -monolithic. In particular, any
 -product of monotonically monolithic spaces is monotonically
monolithic.
466. Prove that,
(i) a space X is strongly monotonically monolithic if and only if X is
strongly monotonically !-monolithic;
(ii) every space with a point-countable base must be strongly monotonically
monolithic;
(iii) if a space X is strongly monotonically monolithic, then it must be
strongly monolithic and monotonically monolithic;
(iv) every subspace of a strongly monotonically monolithic space is strongly
monotonically monolithic;
(v) any countable product of strongly monotonically monolithic spaces must
be strongly monotonically monolithic;
(vi) if X is strongly monotonically monolithic and f W X ! Y is an open
map such that d.f 1 .y//  ! for any y 2 Y then Y is strongly
monotonically monolithic;
(vii) every countably compact strongly monotonically monolithic space is
compact and metrizable.
467. For any infinite cardinal  prove that
(a) a space X is monotonically -stable if and only if Cp .X / is monotonically
-monolithic.
(b) a space X is monotonically -monolithic if and only if Cp .X / is
monotonically -stable.

1.5 The Last-Minute Updates. Yet More on l-Equivalence

57

468. Prove that if X is a Lindelf -space then Cp .X / is monotonically monolithic. In particular, if Cp .Y / is a Lindelf -space then Y is monotonically
monolithic.
469. Prove that if X is a Lindelf -space then Cp .X / is monotonically !monolithic. In particular, if .Cp .X // is a Lindelf -space then Cp .X / is
monotonically !-monolithic. Deduce from these facts that if X is pseudocompact then Cp .X / is monotonically !-monolithic.
470. Let X be a monotonically -monolithic space with ext .X / D  . Prove
that
(i) if <  then l.X /  ;
(ii) if D  and t .X /   then l.X /  .
471. Prove that every subspace of a monotonically monolithic space is a D-space.
As a consequence,
(i) if X is a Lindelf -space then every subspace of Cp .X / is a D-space.
(ii) Observe that ext .Y / D l.Y / for any D-space Y . Therefore (i) generalizes Baturovs theorem (SFFS-269).
472. Given an infinite cardinal , suppose that a space X is monotonically monolithic and nw.X /   C . Prove that every subspace of X is a D-space.
Deduce from this fact that for any pseudocompact space Z with nw.Z/  !1 ,
every subspace of the space Cp .Z/ is a D-space.
473. Prove that any countably compact monotonically !-monolithic space is
Corson compact.
474. Give an example of a Corson compact space that is not monotonically !monolithic.
475. Observe that if X is a monotonically !-monolithic compact space and !1
is a caliber of X , then the space X is metrizable. Prove that there exists a
monotonically !1 -monolithic (and hence monotonically !-monolithic) noncompact pseudocompact space X such that !1 is a caliber of X .
476. Suppose that X is a monotonically Sokolov space. Prove that
(a) X is monotonically !-monolithic;
(b) any F -subset of X is monotonically Sokolov.
477. Suppose that a space X is monotonically retractable and fix, for any countable
set A  X , a retraction rA W X ! X and a network N .A/ for the map rA that
witness monotone retractability of X . Prove that, for any countable family
G of closed subsets of X we can find a countable set P .G/  X such that
rP .G/ .G/  G for any G 2 G and the assignment G ! P .G/ is !-monotone.
478. Given a space
SY denote by CL.Y / the family of all closed subsets of Y and let
CL .Y / D fCL.Y n / W n 2 Ng. Prove that, for any space X , the following
conditions are equivalent:
(a) X is monotonically retractable:
(b) for any countable family G of closed subsets of X , there exists a retraction
sG W X ! X and a countable network O.G/ for the map sG such that
s.G/  G for any G 2 G and the assignment O is !-monotone;

58

1 Properties Preserved by Homeomorphisms of Function Spaces

(c) for any countable family G  CL .X / there exists a retraction tG W X !


X and a countable network P.G/ for the map tG such that, for any n 2 N,
we have the inclusion tGn .G/  G for any G 2 G \ CL.X n / and the
assignment P is !-monotone.
479. Given a monotonically retractable space X , prove that
(a) every F -subset of X is monotonically retractable;
(b) the space X is Sokolov; in particular, X is !-stable, collectionwise
normal, and ext .X /  !;
(c) Cp .X / is a Lindelf D-space.
480. Prove that
(a) any  -product of monotonically retractable spaces is
retractable;
(b) any -product of monotonically retractable spaces is
retractable and hence every countable product of
retractable spaces is monotonically retractable;
(c) any closed subspace of a -product of cosmic spaces is
retractable.

monotonically
monotonically
monotonically
monotonically

481. For any space X prove that


(a) X is monotonically retractable if and only if Cp .X / is monotonically
Sokolov.
(b) X is monotonically Sokolov if and only if Cp .X / is monotonically
retractable.
482. Prove that
(a) any monotonically Sokolov space is Lindelf and Sokolov;
(b) any countable product of monotonically Sokolov spaces is monotonically
Sokolov;
(c) every R-quotient image of a monotonically Sokolov space is monotonically Sokolov;
(d) every R-quotient image of a monotonically retractable space X must be
monotonically retractable;
(e) a monotonically retractable space must be !-monolithic but it can fail to
be monotonically !-monolithic.
483. Prove that a compact space X is monotonically retractable if and only if X is
Corson compact.
484. Prove that a monotonically retractable space X is monotonically Sokolov if
and only if X is monotonically !-monolithic. In particular, a compact space
is monotonically Sokolov if and only if it is monotonically !-monolithic.
485. Suppose that X is a first countable countably compact subspace of an ordinal
with its order topology. Prove that X is monotonically retractable and hence
Cp .X / is a Lindelf D-space.

1.5 The Last-Minute Updates. Yet More on l-Equivalence

59

486. Prove that


(a) the set E of all ordinals < !2 of uncountable cofinality is monotonically
!-monolithic but is neither a D-space nor monotonically !1 -monolithic;
(b) for set Y of all ordinals < !2 of countable cofinality, the space X D Y [
f!2 g is countably compact and ext .Cp .X // D ! while l.Cp .X // D !2 .
In particular, X is a countably compact space such that Cp .X / is not a
D-space.
487. Suppose that X and Cp .X / are Lindelf -spaces. Prove that both X and
Cp .X / must be monotonically retractable and monotonically Sokolov. In
particular, X and Cp .X / have the Sokolov property.
488. Prove that
(a) any Eberlein compact space X has a  -closure-preserving local base at
every point, i.e.,S
for any x 2 X there exists a local base Bx at the point x
such that Bx D n2! Bxn and every Bxn is closure-preserving;
(b) there exists an Eberlein compact space K that does not have a closurepreserving local base at some point;
(c) there exists a non-metrizable compact space Y such that Cp .Y / has a
closure-preserving local base at every point.
489. Given a compact space K let jjf jj D supfjf .x/j W x 2 Kg for any f 2 C.K/;
if f; g 2 C.K/ then K .f; g/ D jjf  gjj. As usual, Cu .K/ is the set C.K/
with the topology generated by the metric K . If A  Cp .K/ and A ;
then diam.A/ D supfjjf  gjj W f; g 2 Ag. For each " > 0 say that a family
A of subsets of C.K/ is "-small if diam.A/ < " for any A 2 A. The space
Cp .K/ is said to have the property JNR if for every " > 0 we can findSa family
fMn W n 2 !g of closed subsets of Cp .K/ such that Cp .K/ D n2! Mn
and,
S for every n 2 ! there exists an "-small family Un  .Mn / such that
Un D Mn . Prove that the following conditions are equivalent:
(i) the space Cp .K/ has the property JNR;
(ii) there exists a  -discrete family N in the space Cp .K/ such that N is a
network in both spaces Cp .K/ and Cu .K/.
In particular, if Cp .K/ has the property JNR then it has a  -discrete network.
490. Prove that
(a) if L is a linearly ordered separable compact space then Cp .L/ is a  space, i.e., it has a  -discrete network;
(b) for any separable dyadic compact space K, the space Cp .K/ has a  discrete network.
491. Prove that there exists a scattered compact space K with a countable dense set
of isolated points such that Cp .K; D/ is not perfect. Deduce from this fact that
Cp .!; D/ is not perfect and, in particular, the space Cp .!/ does not have a
 -discrete network.

60

1 Properties Preserved by Homeomorphisms of Function Spaces

492. Prove that, for any ultrafilter  2 !n!, the following conditions are
equivalent:
(i)  is not a P -point in !n!;
(ii)  .R! / is homeomorphic to a closed subspace of Cp .! /;
(iii) Cp .! / is not hereditarily Baire.
493. Let ;  2 !n!. Prove that the spaces ! and ! are l-equivalent if and only
if there exists a bijection b W ! ! ! such that b./ D fb.U / W U 2 g D .
In particular, ! and ! are l-equivalent if and only if they are homeomorphic.
494. Suppose that and are ordinals such that !   < !1 . Prove that
. C 1/ is l-equivalent to . C 1/ if and only if < ! . Deduce from
this fact that there exist countable compact u-equivalent spaces which are not
l-equivalent.
495. Suppose that X is a metrizable zero-dimensional compact space and there
exists a continuous linear surjection of Cp .X / onto Cp .Y /. Prove that Y is
a metrizable compact zero-dimensional space. In particular, there exists no
continuous linear surjection of Cp .K/ onto Cp .I/. Here, as usual, K is the
Cantor set and I D 1; 1
 R.
496. Suppose that a space X is metrizable, compact, and  -zero-dimensional. Prove
that if there exists a continuous linear surjection of Cp .X / onto Cp .Y / then Y
is also a compact metrizable  -zero-dimensional space. In particular, if X is
compact, metrizable, and finite-dimensional then there is no continuous linear
surjection of Cp .X / onto Cp .I! /.
497. Prove that a space K is l-equivalent to the Cantor set K if and only if
there exists a continuous linear surjection of Cp .K/ onto Cp .K/ as well as
a continuous linear surjection of Cp .K/ onto Cp .K/.
498. Let I be the closed interval 0; 1
 R. Given any n 2 ! suppose that K is a
metrizable compact space and dim K  n. Prove that there exists a compact
subspace K 0  I 2nC1 with the following properties:
(i) K 0 is homeomorphic to K;
(ii) for any function ' 2 C.K 0 /, we can choose ' 1 ; : : : ; ' 2nC1 2
C.I / such that '.y/ D ' 1 .y1 / C : : : C ' 2nC1 .y2nC1 / for any
y D .y1 : : : ; y2nC1 / 2 K 0 .
Deduce from this fact that if X is a second countable space and dim X  n
then X embeds in I2nC1 .
499. Prove that, for any finite-dimensional metrizable compact space K, there exists
a surjective continuous linear mapping of Cp .I/ onto Cp .K/.
500. Prove that l.X / D l.Y / for any l-equivalent spaces X and Y . In particular, if
l

X  Y then X is Lindelf if and only if so is Y .

1.6 Bibliographic notes to Chapter 1

61

1.6 Bibliographic notes to Chapter 1


The material of Chapter 1 consists of problems of the following types:
(1)
(2)
(3)
(4)

textbook statements which give a gradual development of some topic;


folkloric statements that might not be published but are known by specialists;
famous theorems cited in textbooks and well-known surveys;
comparatively recent results which have practically no presence in textbooks.

We will almost never cite the original papers for statements of the first three
types. We are going to cite them for a very small sample of results of the fourth
type. The selection of theorems to cite is made according to the preferences of the
author and does not mean that all statements of the fourth type are mentioned. I
bring my apologies to readers who might think that I did not cite something more
important than what is cited. The point is that a selection like that has to be biased
because it is impossible to mention all contributors. As a consequence, there are
quite a few statements of the main text, published as results in papers, which are
never mentioned in our bibliographic notes. A number of problems of the main
text cite published or unpublished results of the author. However, those are treated
exactly like the results of others: some are mentioned and some arent. On the other
hand, the Bibliography contains (to the best knowledge of the author) the papers and
books of all contributors to the material of this book.
Chapter 1 seems to be the first complete and systematic presentation of the material on functional equivalences. Section 1.1 contains many results on t -invariance
which are simple consequences of previously proved results on duality. Gulko and
Khmyleva were the first ones to prove in (1986) that compactness is not t -invariant
(Problems 016027). Okunev proved in (1990) that  -compactness and similar
properties are preserved by t -equivalence (Problems 034045). The t -invariance
of spread, hereditary density, and hereditary Lindelf number (Problems 055070)
was proved in Okunevs paper (1997a). Dobrowolski et al. established in (1990) that
all metrizable countable non-discrete spaces are t -equivalent. Since they applied
very deep results and methods of infinite-dimensional topology, it was impossible
to present their theorem in this book. Another fundamental result of Section 1.1
is Marciszewskis theorem (2000) which states that I is not t -equivalent to I!
(Problem 099).
Section 1.2 gives a brief introduction to the theory of uniform spaces. This material is best covered in Engelkings book (1977). Some knowledge of uniformities
is necessary to tackle Gulkos theorem (1993) on u-invariance of the dimension
dim (Problems 176180). There are also some deep results on the behavior of the
dimension dim, which are used in Gulkos proof. This made it necessary to develop
some methods used in dimension theory and the theory of inverse systems. The
reader can find an exhaustive information on the mentioned topics in Engelkings
books (1977) and (1978). The u-invariance of absolute Borel sets (Problems 197
and 198) was proved by Marciszewski and Pelant in (1997). The last group of
problems of Section 1.2 presents another theorem of Gulko (1988) which states
that all infinite countable compact spaces are u-equivalent (Problem 200).

62

1 Properties Preserved by Homeomorphisms of Function Spaces

Section 1.3 develops the most important tools to deal with linear topological
spaces. A good reference is W. Rudins book (1973). Okunevs method of constructing pairs of l-equivalent spaces (Problems 257263) was published in (1986)
in a stronger form.
Section 1.4 presents quite a few nontrivial results on l-equivalence. Uspenskijs
theorem on spaces which are l-equivalent to metrizable ones (Problems 329
335) was proved in (1983a). The theorem of Baars, de Groot, and Pelant on

l-invariance of Cech-completeness
in metrizable spaces was established in (1993)
(Problems 365367). A very difficult example of an infinite compact space which
is not l-equivalent to itself with an added isolated point was constructed by
Marciszewski in (1997a) (Problem 400).
Section 1.5 contains some more material on l-equivalence and the latest results
in Cp -theory which did not appear in previous parts of the book. Marciszewski and
Pol constructed in (2009) examples of non-cosmic spaces X such that the space
Cp .X / has a  -discrete network (Problems 489490). Gruenhage proved in (2012)
that there exists a Corson compact space which is not monotonically !-monolithic
(Problem 474). The exhaustive classification of countable compact l-equivalent
spaces (Problem 494) was obtained in a paper of Gulko and Oskin (1975). The first
readable text of the solution of Hilberts Problem 13 (Problem 498) was published
in Kolmogorov (1957). Leiderman et al. (1997) used this result to prove that Cp .I/
can be linearly and continuously mapped onto Cp .X / for any finite-dimensional
metrizable compact X (Problem 499). Velichko proved l-invariance of the Lindelf
property in (1998a). Later Bouziad established in (2001) that Lindelf number is
l-invariant (Problem 500).

Chapter 2

Solutions of problems 001500

It took the author six long years to come to finally start writing this last portion of
his work. In this period he had a sabbatical stay, celebrated the arrival of the new
century and the new millennium, changed his citizenship, became a grandfather, and
published about thirty papers. Also, he understood much better the material of this
book.
Since it could take just as much (or even more) time to read this book and/or to
solve its problems, it is now almost impossible for the author to write the standard
phrase: the reader who mastered the previous material is now able to : : :. How
many readers will repeat this accomplishment? Well, at the present moment there is
at least one (who, evidently, coincides with the author), so, formally, the previous
three volumes show that it is possible to solve the problems of this book at least,
when one already knows the solutions.
So, left behind are 1500 solutions and about 700 statements proved as auxiliary
facts; some of these facts are quite famous and highly nontrivial theorems. As in the
previous volume, the treatment of topology and Cp -theory is professional. When
you read a solution of a problem of the main text, it has more or less the same level
of exposition as a published paper on a similar topic.
The author hopes, however, that reading our solutions is more helpful than
ploughing through the proofs in published papers; the reason is that we are not
so constrained by the amount of the available space as a journal contributor, so we
take much more care about all details of the proof. It is also easier to work with the
references in our solutions than with those in research papers because in a paper
the author does not need to bother about whether the reference is accessible for the
reader whereas we only refer to what we have proved in this book apart from some
very simple facts of calculus and set theory.
This volume has the same policy about the references as the third one; we use
the textbook facts from general topology without giving a reference to them. This
book is self-contained, so all necessary results are proved in the previous volumes
but the references to standard things have to stop sometime. This makes it difficult
Springer International Publishing Switzerland 2016
V.V. Tkachuk, A Cp-Theory Problem Book, Problem Books in Mathematics,
DOI 10.1007/978-3-319-24385-6_2

63

64

2 Solutions of problems 001500

for a beginner to read this volumes results without some knowledge of the previous
material. However, a reader who mastered the first four chapters of R. Engelkings
book (1977) will have no problem with this.
We also omit references to some standard facts of Cp -theory. The reader can
easily find the respective proofs using the index. Our reference omission rule can
be expressed as follows: we omit references to textbook results from topology and
Cp -theory proved in the previous volumes. There are quite a few phrases like it is
easy to see or it is an easy exercise; the reader should trust the authors word and
experience that the statements like that are really easy to prove as soon as one has the
necessary background. On the other hand, the highest percentage of errors comes
exactly from omissions of all kinds, so my recommendation is that, even though you
should trust the authors claim that the statement is easy to prove or disprove, you
shouldnt take just his word for the truthfulness of any statement. Verify it yourself
and if you find any errors communicate them to me to correct the respective parts.
V.001. Prove that cardinality, network weight, i -weight, as well as density are
t -invariant.
Solution. Suppose that X is a space and Cp .X / is homeomorphic to Cp .Y /. Then
jX j D w.Cp .X // D w.Cp .Y // D jY j (see TFS-169); this proves that cardinality
is t -invariant. Furthermore, nw.X / D nw.Cp .X // D nw.Cp .Y // D nw.Y /
(see TFS-172) and hence network weight is also t -invariant. Applying TFS-174
we conclude that the equalities i w.X / D d.Cp .X // D d.Cp .Y // D i w.Y / show
that i -weight is t -invariant as well. Finally observe that d.X / D i w.Cp .X // D
i w.Cp .Y // D d.Y / (see TFS-173), so density is t -invariant too.
V.002. Prove that if Cp .Y /! Cp .X / then nw.Y /  nw.X /; d.Y /  d.X / and
jY j  jX j. Give an example showing that the inequality i w.Y /  i w.X / is not
necessarily true.
Solution. We have nw.Y / D nw.Cp .Y //  nw.Cp .X // D nw.X / (see TFS159 and TFS-172) which settles the first inequality. It follows from TFS-159 and
TFS-173 that we have d.Y / D i w.Cp .Y //  i w.Cp .X // D d.X /. Now, TFS-159
together with TFS-169 imply that jY j D w.Cp .Y //  w.Cp .X // D jX j.
Finally, consider the spaces X D D.c/ and Y D A.c/; the space X is discrete
and jX j D jY j, so X maps continuously onto Y and hence Cp .Y /! Cp .X / by
TFS-163. Since jIj D c, any bijection between X and I is a condensation of X
onto I, so i w.X / D !. The space Y being compact we have i w.Y / D w.Y / D
c > ! D i w.X /, so X and Y is a pair of spaces such that Cp .Y /! Cp .X / while
i w.Y / > i w.X /.
V.003. Prove that p.Y /  p.X / whenever Cp .Y /! Cp .X /. As a consequence,
point-finite cellularity is t -invariant.
Solution. We have p.Y / D a.Cp .Y //  a.Cp .X // D p.X / by Problem
TFS-178; as a consequence, p.Y /  p.X /. If X is t -equivalent to Y then
Cp .X /! Cp .Y / and Cp .Y /! Cp .X / so p.X / D p.Y / and hence point-finite
cellularity is t -invariant.

2 Solutions of problems 001500

65

V.004. Suppose that X and Y are t -equivalent Baire spaces. Prove that
c.X / D c.Y /. In particular, the Souslin numbers of t -equivalent pseudocompact
spaces coincide.
Solution. The spaces X and Y being Baire we can apply TFS-282 to see that
we have p.X / D c.X / and p.Y / D c.Y /. It follows from TFS-178 and our
observation that c.X / D p.X / D a.Cp .X // D a.Cp .Y // D p.Y / D a.Y /
and hence c.X / D c.Y /. Any pseudocompact space is Baire (see TFS-274), so the
Souslin numbers of t -equivalent pseudocompact spaces coincide.
V.005. Let  be a caliber of X . Knowing that Cp .Y / embeds in Cp .X /, prove that
 is a caliber of Y . In particular, calibers are t -invariant.
Solution. Apply SFFS-290 to see that the diagonal of Cp .X / is -small; it follows
from Cp .Y /! Cp .X / that the diagonal of Cp .Y / is -small as well (it is an easy
exercise to prove that having a -small diagonal is a hereditary property). Applying
SFFS-290 again we conclude that  is a caliber of Y . If Cp .X / ' Cp .Y / then
Cp .X /! Cp .Y / and Cp .Y /! Cp .X / which shows that  is a caliber of X if
and only if it is a caliber of Y ; therefore calibers are t -invariant.
V.006. Suppose that Cp .Y / embeds into Cp .X /. Prove that l  .Y /  l  .X /. As a
consequence, l  is t -invariant.
Solution. We have l  .Y / D t .Cp .Y //  t .Cp .X // D l  .X / (see TFS-149 and
t
TFS-159). If X  Y then Cp .Y /! Cp .X / and Cp .X /! Cp .Y / so l  .X / D

l .Y / and hence l  is t -invariant.
V.007. Suppose that Cp .Y / embeds into Cp .X /. Prove that '.Y /  '.X / for any
' 2 fhl  ; hd  ; s  g and hence ' is t -invariant.
Solution. We have hl  .Y / D hd  .Cp .Y //  hd  .Cp .X // D hl  .X / (see SFFS026), so the promised inequality is true for ' D hl  . Apply SFFS-027 to convince
ourselves that hd  .Y / D hl  .Cp .Y //  hl  .Cp .X // D hd  .X / which shows
that '.Y /  '.X / also in case when ' D hd  . Besides, s  .Y / D s  .Cp .Y // 
s  .Cp .X // D s  .X / by SFFS-025 and hence the inequality '.Y /  '.X / is true
for ' D s  as well. An immediate consequence is that any ' 2 fhd  ; hl  ; s  g is
t -invariant.
t

V.008. Prove that, if X  Y then tm .X / D tm .Y /. Give an example of spaces X


and Y such that Cp .Y / embeds into Cp .X / and tm .Y / > tm .X /.
Solution. The spaces Cp .X / and Cp .Y / being homeomorphic we can apply TFS429 to conclude that tm .X / D q.Cp .X // D q.Cp .Y // D tm .Y /. Now if X is a
discrete space of cardinality !1 and Y D L.!1 / is the one-point lindelfication of
X then any bijection between X and Y maps X continuously onto Y , so Cp .Y /
embeds in Cp .X / by TFS-163. Observe that tm .X /  t .X / D ! (see TFS-419)
and let a 2 Y be the unique non-isolated point of Y . It is easy to see that a 2 Y nfag
and a B for any countable B  Y nfag. This, together with Fact 1 of S.419 shows
that tm .Y / > !. Therefore Cp .Y /! Cp .X / and tm .Y / > tm .X /.

66

2 Solutions of problems 001500


t

V.009. Prove that X  Y implies q.X / D q.Y /. In particular, realcompactness is


t -invariant.
Solution. The spaces Cp .X / and Cp .Y / being homeomorphic we can apply
TFS-434 to conclude that q.X / D tm .Cp .X // D tm .Cp .Y // D q.Y /. Since a
space X is realcompact if and only if q.X / D ! (see TFS-401), realcompactness is
a t -invariant property.
V.010. Give an example of spaces X and Y such that Cp .Y / embeds into Cp .X /,
the space X is realcompact and Y is not realcompact.
Solution. Let X be a discrete space of cardinality c; if Y is a Mrowka space (see
TFS-142) then Y is pseudocompact and non-compact which implies that Y is not
realcompact (see TFS-407). Any bijection between X and I is a condensation of
X onto a second countable space, so i w.X /  c and hence X is realcompact by
TFS-446. Furthermore, it follows from jY j  c that X maps continuously onto Y
and hence Cp .Y / embeds in Cp .X / by TFS-163.
t

V.011. Suppose that X is a P -space and X  Y . Prove that Y is also a P -space.


Solution. Every countable subset of X is closed and C -embedded in X by Fact 1 of
S.479, so Cp .X / is pseudocomplete by TFS-485. Since Cp .Y / ' Cp .X /, the space
Cp .Y / is also pseudocomplete and hence Cp .Y; I/ is pseudocompact by TFS-476.
Fact 1. If Z is a P -space then A  Cp .Z/ for any countable A  Cp .Z/ (the bar
denotes the closure in RZ ).
Proof. Take a countable A  Cp .Z/ and f 2 A. Given any z 2 Z, we will prove
that f is continuous at the point
T z. Note that the set fh.z/g is a G -set in the space
I for any h 2 A, so W D fh1 .h.z// W h 2 Ag is a G -set in Z. Since Z is a
P -space, the set W is an open neighborhood of z; we claim that f .W / D ff .z/g.
To see this, suppose that w 2 W and jf .w/  f .z/j > " for some " > 0. Since
f 2 A, there is h 2 A such that jh.z/  f .z/j < 2" and jh.w/  f .w/j < 2" .
However, h.w/ D h.z/ and hence
jf .w/  f .z/j  jf .w/  h.w/j C jh.z/  f .z/j <

"
"
C D"
2
2

which is a contradiction. Therefore, f .W / D ff .z/g  .f .z/  "; f .z/ C "/, for


any " > 0, i.e., f is continuous at z. Therefore f 2 Cp .Z/ and Fact 1 is proved.
Returning to our solution fix a homeomorphism ' W Cp .Y / ! Cp .X / and
assume that K D Cp .Y; I/ is not countably compact. Then there is a countably
infinite closed discrete set B  K; it is evident that B is also closed in Cp .Y /, so
A D '.B/ is closed and discrete in Cp .X /. The set F D '.K/ is pseudocompact,
so L D F is also pseudocompact by Fact 18 of S.351 (the bar denotes the closure
in RX ). However, every closed pseudocompact subspace of RX is compact (see
TFS-401 and TFS-415), so L is compact. Since A  L, the set A has to be compact;

2 Solutions of problems 001500

67

however, A  Cp .X / by Fact 1, so A is an infinite closed discrete subspace of a


compact space A. This contradiction shows that Cp .Y; I/ is countably compact and
hence Y is a P -space by TFS-397.
V.012. Prove that discreteness is t -invariant.
t

Solution. Suppose that X is a discrete space and Y  X . Then Cp .X / D RX is


pseudocomplete (see TFS-470 and TFS-467), so Cp .Y / is pseudocomplete as well.
Besides, Cp .X / is realcompact by TFS-401, so the space Cp .Y / is also realcompact
and hence tm .Y / D q.Cp .Y // D ! (see TFS-429). If some point y 2 Y is not
isolated in Y then we can apply Fact 1 of S.419 to see that there is a countable
set A  Y nfyg such that y 2 A and hence y 2 AnA. However, every countable
subset of Y is closed in Y by TFS-485; this contradiction shows that all points of Y
are isolated, i.e., Y is discrete.
V.013. Suppose that X and Y are compact spaces such that Cp .Y /! Cp .X /.
t
Prove that Y is scattered whenever X is scattered. In particular, if X  Y then X is
scattered if and only if so is Y .
Solution. If X is scattered then Cp .X / is a FrchetUrysohn space by SFFS-134.
Therefore the space Cp .Y /! Cp .X / also has the FrchetUrysohn property;
t
applying SFFS-134 once more we conclude that Y is scattered. If X  Y then
Cp .X /! Cp .Y / and Cp .Y /! Cp .X / so X is scattered if and only if so is Y .
t

V.014. Suppose that X n is a Hurewicz space for each n 2 N and X  Y . Prove


that Y n is also a Hurewicz space for each n 2 N.
Solution. Apply CFS-057 to see that vet .Cp .X // D !; since Cp .Y / ' Cp .X /,
we also have vet .Cp .Y // D !, so we can apply CFS-057 once more to conclude
that Y n is a Hurewicz space for any n 2 N.
t

V.015. Suppose that X  Y and X is a  -compact space with a countable network.


Prove that Y is also  -compact. As a consequence, if X is a metrizable compact
t
space and X  Y then Y is  -compact.
S
Solution. We have X D
n2! Kn where every set Kn is compact; since also
nw.Kn / D !, the space Kn is metrizable (see Fact 4 of S.307) and hence analytic
for any n 2 !. It follows from SFFS-337 that X is analytic; since Cp .Y / ' Cp .X /,
we can apply Fact 12 of T.250 to see that the space Y is K-analytic. Furthermore,
nw.Y / D nw.Cp .Y // D nw.Cp .X // D nw.X /  !, so the space Y is analytic by
SFFS-346.
It is an easy exercise that every  -compact space is Hurewicz; since every finite
power of X is  -compact, the space X n is Hurewicz for any n 2 N and therefore
vet .Cp .X // D ! (see CFS-057). As a consequence, vet .Cp .Y // D ! and hence Y
is Hurewicz as well. Thus Y is analytic and Hurewicz, so we can apply CFS-053 to
conclude that Y is  -compact.

68

2 Solutions of problems 001500

V.016. Given an arbitrary number " > 0 prove that there exists a homeomorphism
u W R   .!/ !  .!/ such that j jju.r; x/jj  jjxjj j  " for any r 2 R and
x 2  .!/.
Solution. Denote by I the subspace 0; 1
of the real line R. Given a space X , the
function idX W X ! X is the identity on X , i.e., idX .x/ D x for any x 2 X .
A family fft W t 2 I g is called an autoisotopy of X if, for any t 2 I , the map
ft W X ! X is a homeomorphism and both maps fC ; f W I  X ! X defined by
fC .t; x/ D ft .x/ and f .t; x/ D ft 1 .x/ respectively for any t 2 I and x 2 X are
continuous.
Fact 1. Fix a number a > 0 and let an D .1  2n /a for any n 2 N. Given a point
z D .z0 ; z1 / 2 R2 let jjzjj D maxfjz0 j; jz1 jg. Then, for any n 2 N, there exists an
autoisotopy fft n W t 2 I g of the plane R2 with the following properties:
(1)
(2)
(3)
(4)

f0n .z/ D z for any z 2 R2 ;


ft n .z/ D z for any z 2 an ; an
 R and t 2 I ;
f1n ..am ; am /  R/ D R  .1; 12 am / for any m > n;
jjft n .z/jj D jjzjj for any z 2 R2 and t 2 I .

Proof. To obtain the property (1) we have to define f0n to be the identity on R2 , so
let f0n .z/ D z for any z 2 R2 . To define the homeomorphism f1n W R2 ! R2 we
will need the set Am D 0; am /  R for all m 2 N. To satisfy (2) let f1n .z/ D z for
any z 2 An ; we will next define f1n on the set an ; C1/  R meaning to extend it
symmetrically over the whole plane. First, let f1n .z/ D z for every z D .z0 ; z1 / 2
an ; C1/R such that z0 < z1 . Thus we defined f1n on the set B D An [f.z0 ; z1 / 2
R2 W an  z0 < z1 g. To complete our construction of f1n on the half-plane P0 D
0; C1/  R, we must define
S it on the set Q D f.z0 ; z1 / W an  z0 and z1  z0 g.
Observe first that Q D fQb W b 2 .1; an
g where this union is disjoint and
Qb D .an ; b
 fbg/ [ .fbg  b; b
/ for any b  an . In other words, every
set Qb is the union of two line segments: the horizontal segment an ; b
 fbg and
the vertical one fbg  b; b
. Observe that jjzjj D jbj for any z 2 Qb , so, to
satisfy the condition (4), it suffices to assure that f1n .Qb /  Qb for every b  an .
Another requirement we must meet is not to move the endpoints of the set Qb ,
i.e., f1n ..an ; b// D .an ; b/ and f1n ..b; b// D .b; b/.
Fix any number b  an ; to define the function f1n jQb we will consider
an auxiliary interval Jb D an ; 3b
 R which, intuitively, is obtained by
straightening up the vertical part of Qb . We will construct a piecewise linear
increasing function hb W Jb ! Jb which gives f1n jQb if we identify Qb and Jb .
If we construct the function f1n according to the above plan then, to meet the
condition (3), it suffices to guarantee that f1n ..am ; b// D .b; 12 am / whenever
an < am  b. If b is sufficiently large then a 2 .an ; b
; in this case we
will also require that f1n ..a; b// D .b; 12 a/. In the terms of the function hb this is
equivalent to saying that hb .an / D an , hb .3b/ D 3b while hb .am / D 2bC 12 am
for any m with an < am  b. Besides, hb .a/ D 2b C 12 a whenever a  b.
It is easy to see that these conditions uniquely determine every function hb for all
b  anC1 . To visualize it better, take in consideration the key points in the square

2 Solutions of problems 001500

69

Jb  Jb which belong to its graph b . It is obligatory for the points Cn D .an ; an /


and Db D .3b; 3b/ to belong to b . If Cmb D .am ; 2b C 12 am / for every m > n
then Cmb 2 b whenever am  b. Finally, the point C b D .a; 2b C 12 a/ also
belongs to b if a  b.
If anC2 < b  anC1 then the set b consists of two line segments: the
b
one which connects the points Cn and CnC1
and the second one which connects
b
the points CnC1
and Db . In the case when b  anC2 , the set b is the union of
three line segments because the set fC b g [ fCmb W m > ng is contained in the straight
b
b
line determined by the points CnC1
and CnC2
.
To construct hb for anC1 < b  an observe that the function han has to
be an identity on the interval an ; 3an
and hence hanC1 has to be continuously
transformed into identity as b runs from anC1 to an . To do this consider the
function .x/ D hanC1 .x/  x on the set an ; 3anC1
. Then hanC1 .x/ D x C .x/
and .an / D .3anC1 / D 0.
Given b 2 anC1 ; an
let .b; x/ be the unique linear function such that
n
.b; an / D an and .b; 3b/ D 3anC1 . If .b; x/ D x C anbCa
. .b; x//
anC1
then .anC1 ; x/ D x C .x/ D hanC1 .x/ for any x 2 an ; 3anC1
. It is also
easy to see that .an ; x/ D x for any x 2 an ; 3an
. The only problem with the
function .b; x/ could be its range, so let hb .x/ D nnfmaxf.b; x/; an g; 3bg.
Then the function hb maps the interval an ; 3b
onto itself while hb .an / D an and
hb .3b/ D 3b.
Consider the set E D f.b; x/ W b  an and an  x  3bg  R2 ; it follows
from our construction of the family fhb W b  an g that the function H W E ! E
defined by H..b; x// D .b; hb .x// is continuous. Given b  an define f1n jQb by
applying hb on fbg  Jb and bending back the interval fbg  Jb . More formally,
for any point z D .b; x/ 2 E let '.z/ D z if x  b and '.z/ D .b; x C 2b/
otherwise. Then ' W E ! Q is a homeomorphism, so the map ' H ' 1 W Q ! Q
is continuous; let f1n jQ D 'H ' 1 . This consistently defines the function f1n jP0 .
The sets F D .0; an
 R/ [ f.x0 ; x1 / W an  x0  x1 g and Q are closed in P0 and
P0 D F [ Q. The function f1n jF is continuous being an identity on F ; we already
saw that f1n jQ is also continuous, so f1n jP0 is also continuous by Fact 2 of T.354.
Since f1n jP0 is an identity on the y-axis, its symmetric extension over the whole
plane gives us a continuous map f1n W R2 ! R2 (to see this apply again Fact 2 of
T.354) for which the conditions (1)(4) are satisfied.
Let Kn DSfx 2 R2 W jjxjj  ng for any n 2 N. It is evident that every Kn is
compact and fInt.Kn / W n 2 Ng D R2 . Since f1n .Kn / D Kn and f1n is a bijection,
the map .f1n /1 jKn W Kn ! Kn is continuous. Next apply Fact 1 of S.472 to see
that .f1n /1 is continuous and hence the map f1n is a homeomorphism.
Fix t 2 I ; as before, we will first construct the map ft n on the half-plane P0 .
First of all declare ft n to be an identity on the set B. To define ft n jQ we will again
guarantee that ft n .Qb / D Qb for any b  an .
To this end consider, for any b  an , the function htb .x/ D x C t .hb .x/ 
x/ which maps the interval Jb into itself. Since hb is an increasing function, the
function htb is increasing as well.

70

2 Solutions of problems 001500

It easily follows from our definition of the family fhtb W b  an ; t 2 I g that
the function G W I  E ! E defined by G.t; .b; x// D .b; htb .x// is continuous.
Define a map Gt W E ! E by Gt ..b; x// D G.t; .b; x// D .b; htb .x// for any t 2 I
and .b; x/ 2 E. Then the mapping ft n jQ D ' Gt ' 1 W Q ! Q is continuous.
This gives us a function ft n jP0 . As before, P0 D F [ Q and the function ft n jF is
continuous being an identity; the function ft n jQ is continuous by our construction,
so ft n jP0 is continuous.
Since every ft n j.f0g  R/ is an identity, we can symmetrically extend it over the
whole plane obtaining a continuous map ft n W R2 ! R2 for which the condition
(4) is satisfied.
To show that every ft n is a homeomorphism replace f1n by ft n in the proof for
t D 1. Now that we have the family fft n W t 2 I g of autohomeomorphisms of R2
let fCn .t; x/ D ft n .x/ and fn .t; x/ D .ft n /1 .x/ for any t 2 I and x 2 R2 . To see
that the map fCn W I  R2 ! R2 is continuous observe that the symmetry of our
situation and Fact 2 of T.354 show that it suffices to prove that fCn j.I  P0 / ! P0
is continuous. The fact that fCn j.I  F / is an identity which does not depend on the
first coordinate shows that it suffices to prove that fCn j.I  Q/ W I  Q ! Q is
continuous. To do this note that fCn .t; z/ D ft n .z/ D '.G.t; ' 1 .z/// for any t 2 I
and z 2 Q which, together with continuity of G and ', implies that fCn j.I  Q/ is
continuous and hence the function fCn is, indeed, continuous.
The proof that the function fn is continuous is analogous if we consider the
map G W I  E ! E defined by G .t; .b; x// D .b; .htb /1 .x// for any
t 2 I and .b; x/ 2 E. Once we observe that G is continuous and fn .t; x/ D
'.G .t; ' 1 .z/// for any t 2 I and z 2 Q, we can make the necessary changes in
the proof of continuity of fCn to convince ourselves that fn is continuous and hence
the family fft n W t 2 I g is an autoisotopy on R2 with the properties (1)(4). Fact 1
is proved.
Fact 2. Given spaces X and Y suppose that f't W t 2 I g is an autoisotopy on
X and s W Y ! I is a continuous function. If u.x; y/ D .'s.y/ .x/; y/ for any
.x; y/ 2 X  Y then the map u W X  Y ! X  Y is an autohomeomorphism of
1
the space X  Y and u1 .x; y/ D .'s.y/
.x/; y/ for all x 2 X and y 2 Y .
Proof. It is straightforward from the definition that both maps uu1 and u1 u are
identities on X  Y , so u is a bijection and u1 is its inverse. Let C W I  X ! X
and  W I  X ! X be defined by C .t; x/ D 't .x/ and  .t; x/ D .'t /1 .x/
respectively for any t 2 I and x 2 X . Both maps C and  are continuous
because f't W t 2 I g is an isotopy on X . The map  D s  idX W Y  X ! I  X is
continuous and hence so is the map C . Since u is the diagonal product of C 
and idY , it has to be continuous. Analogously, the map u1 is continuous being the
diagonal product of   and idY . This shows that u is a homeomorphism, so
Fact 2 is proved.
Fact 3. Given " > 0 suppose that fGn W n 2 !g is an increasing sequence of open
subsets of  .!/ and fvn W n 2 !g is a set of autohomeomorphisms of  .!/ with
the following properties:

2 Solutions of problems 001500

71

(1) v0 jG0 D idG0Sand vnC1 jGnC1 D vn jGnC1 for any n 2 !;


(2) vn .GnC1 /  1knC1 fx 2  .!/ W jx.k/j < "g for all n 2 !;
(3) jjvn .x/jj D jjxjj for all x 2  .!/ and n 2 !.
S
Then, for every x 2 G D n2! Gn the sequence fvn .x/ W n 2 !g converges to a
point v.x/ 2  .!/ and the respective map v W G !  .!/ is a homeomorphism
between G and  .!/ such that jjv.x/jj D jjxjj for any x 2 G.
Proof. Using the property (1) it is easy to prove by induction that
. / for any m 2 !, if n  m then vn jGm D vm jGm .
If x 2 G then there is m 2 ! such that x 2 Gm . It follows from . / that vn .x/ D
vm .x/ for any n  m and hence the sequence fvn .x/ W n 2 !g is convergent because
it is eventually constant. Thus the element v.x/ D limn!1 vn .x/ is well defined for
any x 2 G and
. / vjGm D vm jGm for any m 2 !.
Given distinct points x; y 2 G there is m 2 ! with x; y 2 Gm which shows that
v.x/ D vm .x/ and v.y/ D vm .y/. The map vm being a homeomorphism, we have
v.x/ D vm .x/ vm .y/ D v.y/, i.e., v is injective. If y 2  .!/ then there is k 2
N such that jy.k/j < "; the property (2) shows that y 2 vk .GkC1 /. Take x 2 GkC1
such that vk .x/ D y; by (1) and . /, we have v.x/ D vkC1 .x/ D vk .x/ D y and
hence the map v is a bijection between G and  .!/.
Fact 1 of S.472 and the property . / easily imply that v is a continuous map.
To prove that v 1 is also continuous fix a point y 2  .!/ and x 2 G with
v.x/ D y. There is m 2 ! such that x 2 Gm and therefore v.x/ D vm .x/. The
map vm being a homeomorphism, the set vm .Gm / is open in  .!/ and it follows
1
from . / that v 1 jvm .Gm / D vm
jvm .Gm /. Thus fvn .Gn / W n 2 !g is an open
cover of the space  .!/ such that v 1 jvn .Gn / is continuous for any n 2 !; this
makes it possible to apply Fact 1 of S.472 again to conclude that v 1 is continuous
and hence v W G !  .!/ is a homeomorphism.
Finally, take any x 2 G; there is m 2 ! with x 2 Gm , so we can apply . / once
more to convince ourselves that v.x/ D vm .x/ and hence jjv.x/jj D jjvm .x/jj D
jjxjj by (3). This proves that jjv.x/jj D jjxjj for any x 2 G, so Fact 3 is proved.
Fact 4. Suppose that a > 0 and let G D fx 2  .!/ W jx.0/j < ag. Then there
exists a homeomorphism ' W G !  .!/ such that jj'.x/jj D jjxjj for any x 2 G.
Proof. Let an D .1  2n1 /a and Gn D fx 2  .!/ W jx.0/j < an g for
every n 2 !;
S then fGn W n 2 !g is an increasing sequence of open subsets of
 .!/ and n2! Gn D G. We are going to construct a sequence fvn W n 2 !g of
autohomeomorphisms of  .!/ which satisfy the premises of Fact 3.
To that end we will produce a sequence fun W n 2 !g of autohomeomorphisms
of  .!/ such that vn D un : : : u0 for any n 2 !. Besides, every map un will be
two-dimensional, i.e., it will change at most two coordinates of any point of  .!/.
Let s0 .x/ D 1 for all x 2  .!/; to construct the functions sn for n > 0 we
will need the function zn defined by zn .x/ D nnfx.1/; : : : ; x.n/g for any n 2 N and

72

2 Solutions of problems 001500

x 2  .!/. It is clear that zn W  .!/ ! R is a continuous function, so the sets


Zn0 D fx 2  .!/ W zn .x/  12 an g and Zn1 D fx 2  .!/ W zn .x/  12 anC1 g
are closed and disjoint in  .!/. Take a continuous function sn0 W R ! I such
that sn0 ..1; 12 an
/ D f0g and sn0 . 12 anC1 ; C1// D f1g and let sn D sn0 zn . Then
sn .Zn0 / D f0g and sn .Zn1 / D f1g.
For any n 2 ! and x 2  .!/ let un .x/.i / D x.i / for all i 2 !nf0; n C 1g;
this guarantees that the map un can only change the coordinates 0 and n C 1.
Fact 1 provides an autoisotopy fft nC1 W t 2 I g of the plane R2 ; observe that
every map ft nC1 W R2 ! R2 has its two components, i.e., ft nC1 ..a; b// D
nC1
nC1
.ft;0
.a; b/; ft;1
.a; b// for any .a; b/ 2 R2 . To finish our definition of the
mapping un let
.x.0/; x.n C 1// and un .x/.n C 1/ D fsnC1
.x.0/; x.n C 1//:
un .x/.0/ D fsnC1
n .x/;0
n .x/;1
To see that every un is an autohomeomorphism of  .!/ observe that we can
consider that  .!/ D X  Y where X is the plane determined by the coordinates
0 and n C 1 and Y D  .!nf0; n C 1g/. The function sn .x/ is formally defined on
the whole space  .!/ but it does not depend on the coordinates 0 and n C 1, so
we can consider that it is defined on Y . Thus we can apply Fact 2 to conclude that
un is an autohomeomorphism for any n 2 !.
An immediate consequence of Fact 1 is that jjun .x/jj D jjxjj for any x 2  .!/
and n 2 !. Let vn D un : : : u0 for any n 2 !. We will prove by induction that
SnC1
(i) vn .Gm / D Gm [ . kD1
fx 2  .!/ W x.k/ < 12 am g/ for any n 2 ! and m > n.
If n D 0 then v0 D u0 and the property (i) is an immediate consequence of
Fact 1. Assume that (i) is proved for n D l and consider the set vlC1 .GmS
/ for some
nC1
m > l C 1. Observe that vlC1 .Gm / D ulC1 .vl .Gm // and let L D kD1
fx 2
1
 .!/ W x.k/ < 2 am g. The map ulC1 only changes coordinates 0 and l C 2 while
L is determined by the coordinates from 1 to l C 1; this implies ulC1 .L/ D L.
Now, if x 2 Gm nL then zlC1 .x/  12 am  12 alC2 and therefore slC1 .x/ D 1. Thus
lC2
slC1 .x/ D 1 for any point x 2 Gm nL, so ulC1 .x/.n C 1/ D f1;1
.x.0/; x.l C 2//.
lC2
(see Fact 1), it is easy to see that
Recalling the properties of the function f1
ulC1 .Gm nL/ D .Gm [ fx 2  .!/ W xlC2 < 12 am g/nL. This shows that the
formula (i) is true if we substitute l by l C 1, so (i) is proved.
An immediate consequence of (i) is that our autohomeomorphisms vn have the
property (2) of Fact 3 for " D 14 a because 12 am  12 a0 D 14 a for all m 2 !. It is
evident that we also have the property (3) of Fact 3, so the last thing we must check
is the property (1). The property (2) of Fact 1 shows that we have the equalities
v0 jG0 D u0 jG0 D idG0 . Now, take n 2 !; we must show that vnC1 jGnC1 D
vn jGnC1 , so fix any point x 2 GnC1 . If y D vn .x/ 2 GnC1 then jx.0/j < anC1 , so
unC1 .y/ D y, i.e., vnC1 .x/ D unC1 .y/ D y D vn .x/ by the property (2) of Fact 1.
If y GnC1 then we can apply (i) for m D n C 1 to conclude that y.k/ < 12 am
for some number k 2 f1; : : : ; n C 1g and hence znC1 .y/  12 anC1 which implies
snC1 .y/ D 0. Recalling the definition of unC1 and the condition (1) of Fact 1, we
conclude that again unC1 .y/ D y and hence vnC1 .x/ D vn .x/.

2 Solutions of problems 001500

73

Thus all premises of Fact 3 are satisfied for our sequences fGn W n 2 !g and
fvn W n 2 !g, so Fact 3 is applicable to see that there is a homeomorphism ' W G !
 .!/ such that jj'.x/jj D jjxjj for any x 2 G. Fact 4 is proved.
Returning to our solution, fix a homeomorphism h W R ! ."; "/. Given a point
.t; x/ 2 R   .!/ let .t; x/.0/ D h.t / and .t; x/.n/ D x.n  1/ for any n 2 N.
It is straightforward that, for the set G D fx 2  .!/ W jx.0/j < "g, the map
 W R   .!/ ! G is a homeomorphism such that j jj.t; x/jj  jjxjj j  " for
any t 2 R and x 2  .!/. By Fact 4 there is a homeomorphism ' W G !  .!/
such that jj'.x/jj D jjxjj for any x 2 G. Therefore the map u D '  is a
homeomorphism between R   .!/ and  .!/ such that j jju.t; x/jj  jjxjj j  ",
so our solution is complete.
V.017. Prove that  .!/ is homeomorphic to R!   .!/.
Solution. Fix
n W n 2 !g of infinite subsets of ! such
S a disjoint family !A D fA
that ! D
A and let n W R ! RAn be the natural projection of R! onto is
face RAn for any n 2 !. We will need the space En D  .An / for any n 2 !; let
jjxjjn D supfjx.k/j W k 2 An g for each x 2 En . It is easy to see that there is a
homeomorphism hn W En !  .!/ such that jjh.x/jj D jjxjjn for any x 2 En .
ConsiderQthe set E D fx 2 R! W n .x/ 2 En for any n 2 !g. It is straightforward
that E ' n2! En and  .!/ D fx 2 E W the sequence fjjn .x/jjn W n 2 !g
converges to zerog. Apply Problem 016 to find a homeomorphism 'n W REn ! En
such that
(1) j jj'n .s; y/jjn  jjyjjn j  2n for any s 2 R; y 2 En and n 2 !.
Given t 2 R! and x 2 E let '.t; x/ be the unique point of E such that
n .'.t; x// D 'n .t .n/;Qn .x// forQevery n 2 !. It follows
from Fact 1 of S.271 that
Q
the product map ' 0 D n2! 'n W n2! .R  En / ! n2! En is aQhomeomorphism.
It is Q
easy to see that there exist homeomorphisms W R!  E ! n2! .R  En / and
 W n2! En ! E such that ' D  ' 0 . Therefore the map ' W R!  E ! E
is a homeomorphism.
If t 2 R! and x 2 E then n .x/ 2 En for any n 2 !; it follows from (1) that
the sequence fjjn .x/jjn W n 2 !g converges to zero if and only if the sequence
fjj'n .t .n/; n .x//jjn W n 2 !g converges to zero. An immediate consequence is that
'.R!   .!// D  .!/ and therefore 'j.R!   .!// is a homeomorphism
between R!   .!/ and  .!/.
V.018. Suppose that X is a pseudocompact space. Given any function f 2 Cp .X /,
let jjf jj D supfjf .x/j W x 2 X g. Prove that C .X / ' C .X /  .Cp .X //! , where
C .X / D f' 2 .Cp .X //! W jj'.n/jj ! 0g.
Solution. Fix
n W n 2 !g of infinite subsets of ! such
S a disjoint family !A D fA
that ! D
A and let n W R ! RAn be the natural projection of R! onto is
face RAn for any n 2 !. We will need the space En D  .An / for any n 2 !;
let jjxjjn D supfjx.k/j W k 2 An g for each x 2 En . It is easy to see that there is
a homeomorphism hn W En !  .!/ such that jjh.x/jj D jjxjjn for any x 2 En .

74

2 Solutions of problems 001500

From now on the norm symbol jj jj is applied only to the functions on X with the
meaning defined in the formulation of this problem.
ConsiderQthe set E D fx 2 R! W n .x/ 2 En for any n 2 !g. It is straightforward
that E ' n2! En and  .!/ D fx 2 E W the sequence fjjn .x/jjn W n 2 !g
converges to zerog. Apply Problem 016 to find a homeomorphism un W REn ! En
such that
(1) j jjun .s; y/jjn  jjyjjn j  2n for any s 2 R; y 2 En and n 2 !.
Given t 2 R! and x 2 E let u.t; x/ be the unique point of E such that
n .u.t; x// D un .t .n/;Qn .x// forQevery n 2 !. It follows
from Fact 1 of S.271 that
Q
the product map u0 D n2! un W n2! .R  En / ! n2! En is aQhomeomorphism.
It isQ
easy to see that there exist homeomorphisms W R!  E ! n2! .R  En / and
 W n2! En ! E such that u D  u0 . Therefore the map u W R!  E ! E is
a homeomorphism.
For any n 2 ! let pn W R! ! R be the natural projection of R! onto the
n-th factor of R! ; recall that pn .x/ D x.n/ for any x 2 R! and n 2 !. Let
q0 W R!  E ! R! and q1 W R!  E ! E be the natural projections. Consider the
sets H D ff 2 Cp .X; R!  E/ W jjpn .q1 f /jj ! 0g and G D ff 2 Cp .X; E/ W
jjpn f jj ! 0g and define a map ' W Cp .X; R! E/ ! Cp .X; E/ by '.f / D uf
for any f 2 Cp .X; R!  E/. Since u is a homeomorphism, it follows easily from
TFS-091 that the map ' is also a homeomorphism. Our next step is to establish that
(2) '.H / D G and hence the spaces H and G are homeomorphic.
To prove (2) take any f 2 H ; since the sequence fjjpk .q1 f /jj W k 2 !g
converges to zero, there exists a 2  .!/ such that jq1 .f .x//.k/j  ja.k/j for any
x 2 X and k 2 !. If rn D supfja.k/j W k 2 An g then the sequence frn W n 2 !g
converges to zero and jjn .q1 .f .x///jjn  rn for every n 2 !.
The property (1) and the equality n .u.f .x/// D un .q0 .f .x//.n/; n .q1 .f .x////
imply that j jjn .q1 .f .x///jjn  jjn .u.f .x///jjn j  2n and hence we have the
inequality jjn .u.f .x///jjn  rn C2n for any point x 2 X and n 2 !. Given k 2 !
let sk D rn C 2n if n 2 ! is the unique number with k 2 An . It is straightforward
that the sequence fsk W k 2 !g converges to zero and ju.f .x//.k/j  sk for any
x 2 X and k 2 !. Therefore jjpk .'.f //jj ! 0 which shows that '.f / 2 G. This
proves that '.H /  G.
Now assume that g D '.f / D u f 2 G; by the definition of G we can choose
an element a 2  .!/ such that ju.f .x//.k/j  ja.k/j for any x 2 X and k 2 !.
If sn D supfja.k/j W k 2 An g then the sequence fsn W n 2 !g converges to zero and
jjn .u.f .x///jjn  sn for every n 2 !.
The property (1) and the equality n .u.f .x/// D un .q0 .f .x//.n/; n .q1 .f .x////
imply j jjn .q1 .f .x///jjn  jjn .u.f .x///jjn j  2n , so jjn .q1 .f .x///jjn 
sn C 2n for any point x 2 X and n 2 !. Given k 2 ! let rk D sn C 2n if n 2 ! is
the unique number with k 2 An . It is straightforward that the sequence frk W k 2 !g
converges to zero and jq1 ..f .x//.k/j  rk for any x 2 X and k 2 !. Therefore
jjpk q1 f jj ! 0 which shows that f 2 H . This proves that '.H /  G and
hence '.H / D G, i.e., (2) is proved.

2 Solutions of problems 001500

75

Consider the map 0 W Cp .X; R!  E/ ! Cp .X; R! / defined by the formula


0 .f / D q0 f for any f 2 Cp .X; R!  E/. If we let 1 .f / D q1 f for each
f 2 Cp .X; R!  E/ then we obtain a map 1 W Cp .X; R!  E/ ! Cp .X; E/.
It follows from TFS-091 that 0 and 1 are continuous and it is left to the reader
as an exercise (consisting in the extraction of the relevant part of the proof of TFS112) that the map  D 0 1 W Cp .X; R!  E/ ! Cp .X; R! /  Cp .X; E/ is a
homeomorphism.
It is straightforward that .H / D Cp .X; R! /G, so we can apply (2) to see that
Cp .X; R! /  G ' G. Observe that E  R! , so G  Cp .X; R! /. For any n 2 !
let n .f / D pn f for any f 2 Cp .X; R! /; then n W Cp .X; R! / ! Cp .X / is a
continuous map. It is an easy exercise (being again an extraction the relevant part of
the proof of TFS-112) to show that the map D n2! n W Cp .X; R! / ! .Cp .X //!
is a homeomorphism. We also omit a simple checking that .G/ D C .X / and
hence G ' C .X /. We already saw that Cp .X; R! / ' .Cp .X //! (another way to
see it is to apply TFS-112 directly), so .Cp .X //!  G ' .Cp .X //!  C .X / '
C .X / which shows that our solution is complete.
V.019. Let X be a pseudocompact space. As usual, for any f 2 Cp .X /, we define
jjf jj D supfjf .x/j W x 2 X g. Supposing that the space Cp .X / is homeomorphic to
C .X / D f' 2 .Cp .X //! W jj'.n/jj ! 0g, prove that Cp .X / ' .Cp .X //! .
Solution. It follows from Problem 018 that C .X / ' C .X /  .Cp .X //! . Since
we also have Cp .X / ' C .X /, we conclude that Cp .X / ' Cp .X /  .Cp .X //! '
.Cp .X //! .
V.020. Prove that  .!/ is homeomorphic to . .!//! .
Solution. Given any function f 2 Cp .!C1/ let jjf jj D supfjf .x/j W x 2 .!C1/g.
TakeS
a disjoint family A D fAn W n 2 !g of countably infinite subsets of ! such
that A D ! and fix a bijection n W .! C 1/ ! An for every n 2 !. For any
x 2  .An / let jjxjjn D supfjx.k/j W k 2 An g.
For each f 2 Cp .! C 1/ let 'n .f /.n .m// D f .m/  f .!/ for any m 2 ! and
'n .f /.n .!// D f .!/. It is easy to see that 'n .f / 2  .An / for all f 2 Cp .!C1/
and 'n W Cp .! C 1/ !  .An / is a homeomorphism. Besides,
jj  jj'n .f /jjn  2jjf jj for any f 2 Cp .! C 1/ and n 2 !.
Q
Q
The map ' D n2! 'n W .Cp .! C 1//! ! n2!  .An / is a homeomorphism.
If f 2 .Cp .! C 1//! then it follows from (1) that the sequence fjjf .n/jj W n 2 !g
converges to zero if and only if the sequence fjj'n .f .n//jjn W n 2 !g converges to
zero.
Q
For any x 2 P D n2!  .An / let .x/.k/ D x.n/.k/ where n 2 ! is
the unique number for which k 2 An . Again, it is straightforward that  W P !
.P /  R! is a homeomorphism and .x/ 2  .!/ if and only if jjx.n/jjn ! 0.
This shows that, for any f 2 .Cp .! C 1//! , the sequence fjjf .n/jj W n 2 !g
converges to zero if and only if the sequence f.'.f //.n/ W n 2 !g converges
to zero. In other words, .'.C .! C 1/// D  .!/ and therefore C .! C 1/ '
 .!/. Since .! C 1/ ' A.!/, we can apply CFS-105 to convince ourselves that
(1)

1
jjf
2

76

2 Solutions of problems 001500

C .! C 1/ '  .!/ ' Cp .! C 1/. This makes it possible to apply Problem 019 to
conclude that Cp .! C 1/ ' .Cp .! C 1//! and hence . .!//! ' .Cp .! C 1//! '
Cp .! C 1/ '  .!/.
V.021. Prove that, for every infinite space X , the space R! embeds into Cp .X / as
a closed subspace.
Solution. If X is not pseudocompact then R! embeds in Cp .X / as a closed
subspace by Fact 6 of T.132. If X is pseudocompact apply Fact 7 of T.132 to find
a function ' 2 C.X / such that '.X /  R is infinite; let K D '.X /. The function
' W X ! K is R-quotient (see Fact 3 of 6.154), so the space Cp .K/ embeds in
Cp .X / as a closed subspace.
The space K is infinite, compact, and metrizable, so there exists a subspace S 
K with S ' .! C 1/. Now apply Fact 2 of U.216 to see that Cp .! C 1/ embeds
in Cp .K/ as a closed subspace. Furthermore, Cp .! C 1/ '  .!/ '  .!/ 
R! (see CFS-105 and Problem 017) which shows that R! embeds in Cp .! C 1/
as a closed subspace. Therefore R! embeds in Cp .K/ as a closed subspace; an
immediate consequence is that R! also embeds in Cp .X / as a closed subspace.
V.022. Prove that a space X is not pseudocompact if and only if R! embeds in
Cp .X / as a linear subspace.
Solution. We will need some notions of the theory of linear topological spaces. If L
is a linear space and A; B  L then A C B D fx C y W x 2 A; y 2 Bg; if A D fxg
then we write x C B instead of fxg C B. If L is a linear topological space and 0 2 L
is its zero vector, a set K  L is called totally bounded if for any U 2 .0; L/
there is
Sa finite A  L such that K  A C U ; a set P  L is  -totally bounded if
P D n2! Kn where Kn is totally bounded for any n 2 !. A set P  L is called
symmetric if P D P D fx W x 2 P g.
Fact 1. Suppose that L is a linear topological space and M  L is a linear subspace
of L. If a set K  L is totally bounded in L then K 0 D K \ M is totally bounded
in M .
Proof. Let U 0 2 .0; M / and take a set U 2 .L/ such that U \ M D U 0 . Using
continuity of the addition operation in the space L it is easy to find a symmetric set
V 2 .0; L/ such that V C V  U ; let V 0 D V \ M . The set K being totally
bounded in L there is a finite set A  L such that K  A C V . For any a 2 A
choose a point .a/ 2 .a C V / \ M if .a C V / \ M ;. If .a C V / \ M D ;
then let .a/ D 0. It is evident that A0 D f.a/ W a 2 Ag is a finite subset of M .
Fix a point x 2 K 0 ; there is a 2 A such that x 2 .a C V / and, in particular,
.a C V / \ M ;. Therefore b D .a/ 2 A0 , so there is v0 2 V with b D a C v0 .
Besides, there is v1 2 V such that x D a C v1 ; an immediate consequence is that
x D b  v0 C v1 . Observe that w D v0 C v1 2 V C V  U and, since also
w D x  b, the point w belongs to M because M is a linear subspace of L. Thus
w 2 U \ M D U 0 and we proved that x D b C w 2 b C U 0  A0 C U 0 . The
point x 2 K 0 was chosen arbitrarily, so we established that K 0  A0 C U 0 , i.e., K 0
is totally bounded in M . Fact 1 is proved.

2 Solutions of problems 001500

77

Fact 2. Suppose that L and L0 are linear topological spaces and f W L ! L0 is a


continuous linear map. If a set K  L is totally bounded in L then K 0 D f .K/ is
totally bounded in L0 .
Proof. If U 0 is an open neighborhood of zero in L0 then U D f 1 .U 0 / is an open
neighborhood of zero in L, so there exists a finite A  L with K  A C U . It is
straightforward that A0 D f .A/  L0 is a finite set such that K 0  A0 C U 0 , so K 0
is totally bounded in L0 and hence Fact 2 is proved.
Fact 3. Suppose that L is a linear topological space and K  L is totally bounded
in L. Then
(i) any K 0  K is totally bounded;
(ii) the set K is totally bounded;
(iii) the set z C K is totally bounded for any z 2 L.
Proof. Since (i) is evident, let us prove (ii). Fix a set U 2 .0; L/; using continuity
of the addition operation in L it is easy to find a symmetric set V 2 .0; L/ such
that V C V  U . Since K is totally bounded, we can find a finite set A  L with
K  A C V . If x 2 K then W D x C V 2 .x; L/, so there is y 2 W \ K; this
means that there is v0 2 V such that y D x C v0 . Since y 2 A C V , there are a 2 A
and v1 2 V for which y D a C v1 .
Thus x D a C v1  v0 2 .a C V C V /  a C U  A C U . Since the point
x 2 K was chosen arbitrarily, we proved that K  A C U and hence K is totally
bounded. This settles (ii).
To prove (iii) take a set U 2 .0; L/; since K is totally bounded, there is a finite
set A0  L such that K  A0 C U . The set A D z C A0 is finite and it is easy to
see that .z C K/  A C U , so z C K is totally bounded; this shows that (iii) is also
true, so Fact 3 is proved.
Fact 4. The space R! is not  -totally bounded (as a linear topological space).
S
Proof. Suppose that R! D n2! Kn and every Kn is totally bounded. Fact 3 shows
that every Fn D K n is totally bounded as well; the space R! has the Baire property,
so there is n 2 ! for which the interior V of the set K n is nonempty. Apply again
Fact 3 to see that V is totally bounded.
!
Any nonempty open subset of R! contains a standard open
Q subset of R!nn, so we

can fix n 2 ! and Wi 2 .R/ for any i < n such that W D . i<n Wi /R
V.
Let  W R! ! R be the projection of R! onto its n-th factor. Then  is a continuous
linear map; since W is totally bounded (here we used Fact 3 once more), we can
apply Fact 2 to see that the set .W / D R is totally bounded in R.
However, R is not totally bounded in itself because, for the open neighborhood
H D .1; 1/ of zero in R there is no finite A  R with A C H D R (this is an
easy exercise which can be left to the reader). This contradiction shows that R! is
not  -totally bounded and hence Fact 4 is proved.
Fact 5. A space Z is pseudocompact if and only if the space Cp .Z/ (considered as
a linear topological space) is  -totally bounded.

78

2 Solutions of problems 001500

Proof. If Z is not pseudocompact then there exists a countably infinite discrete


family of nonempty open subsets of Z; an easy consequence of Fact 1 of T.217 is
that R! is linearly homeomorphic to a closed linear subspace of Cp .Z/. If Cp .Z/ is
 -totally bounded then it follows from Fact 1 that R! is also  -totally bounded
which, together with Fact 4, gives a contradiction. This shows that if Cp .Z/ is
 -totally bounded then the space Z is pseudocompact,
S i.e., we proved sufficiency.
Now, if Z is pseudocompact then Cp .Z/ D
n2! Cp .Z; n; n
/. For any
n 2 ! the set n; n
Z is compact and hence totally bounded in RZ . The space
Cp .Z/ is a linear subspace of RZ , so we can apply Fact 2 to convince ourselves that
Cp .Z; n; n
/ D n; n
Z \ Cp .Z/ is totally bounded in Cp .Z/ for any n 2 !.
Thus Cp .Z/ is  -totally bounded and hence we established necessity. Fact 5 is
proved.
Returning to our solution observe that if the space X is not pseudocompact then
there exists a countably infinite discrete family of nonempty open subsets of X ; an
easy consequence of Fact 1 of T.217 is that R! is linearly homeomorphic to a closed
linear subspace of Cp .X /. This proves necessity.
Now, assume that X is pseudocompact and hence Cp .X / is  -totally bounded
by Fact 5. If R! embeds in Cp .X / as a linear subspace then it follows from Fact 1
that R! is also  -totally bounded which contradicts Fact 4. Thus R! does not
embed in Cp .X / as a linear subspace; this proves sufficiency and makes our solution
complete.
V.023. Prove that, if either X or Cp .X / is Lindelf, then R!1 does not embed
into Cp .X /.
Solution. If Z is a space and Y  Z then e.Y; Z/ (called the relative extent of
Y in Z) is the supremum of cardinalities of discrete subspaces of Y which are
closed in Z (and hence in Y ). In the same spirit, l.Y; Z/ (called the relative Lindelf
number of Y in Z) is the minimal cardinal number  such that every open cover of
Z contains a subfamily of cardinality at most  which covers Y .
Fact 1. Given an infinite cardinal , if hd.Y /   and w.Z/   then
hd.Y  Z/  . In particular, the product of a second countable space and a
hereditarily separable space is hereditarily separable.
Proof. Fix a base B in the space Z such that jBj   and assume, toward a
contradiction, that some P D fp W <  C g  Y  Z is left-separated, i.e.,
every set P D fp W < g is closed in P . For any <  C there are q 2 Y
and r 2 Z such that p D .q ; r /; choose B 2 B and U 2 .Z/ such that
q 2 U ; r 2 B and .U  B / \ P D ;. There exist L   C and B 2 B such
that jLj D  C and B D B for all 2 L.
Suppose that ; 2 L and < . If q 2 U then the point p D .q ; r /
belongs to the set U  B D U  B D U  B which is a contradiction with the
choice of the sets B and U . Consequently, q U whenever < and therefore
fq W 2 Lg is a left-separated subspace of Y of cardinality  C ; this contradiction

2 Solutions of problems 001500

79

with hd.Y /   (see SFFS-004) shows that there are no left-separated subspaces
of Y  Z of cardinality  C , so hd.Y  Z/  , i.e., Fact 1 is proved.
Fact 2. Suppose that X is a space with l  .X /  !. If X can be perfectly mapped
onto a space M with hd  .M /  ! then l.Y; Z/ D ext .Y; Z/ for any Y  Z 
Cp .X /.
Proof. Suppose that we have spaces Z; T and a map u W Z ! T . Given any n 2 N
let un .z1 ; : : : ; zn / D .u.z1 /; : : : ; u.zn // for any z D .z1 ; : : : ; zn / 2 Z n . Thus un W
Z n ! T n . If P is a set then Fin.P / is the family of all nonempty finite subsets
of P . For the family O D f.a; b/ W a < b; a; b 2 Qg and every n 2 N let
On D fO1  : : :  On W Oi S
2 O for any i D 1; : : : ; ng. Choose some enumeration
fOk W k 2 !g of the family fOn W n 2 Ng. Thus, for each k 2 ! there is mk 2 N
and O1k ; : : : ; Omk k 2 O such that Ok D O1k : : :Omk k . For any x D .x1 ; : : : ; xmk / 2
mk
k
mk
X mk let x;
SOk
D ff 2 Cp .X / W f .x/ 2 O g. If Bk D fx; Ok
W x 2 X g
then B D fBk W k 2 !g is a base of the space Cp .X /.
Fix a perfect map p W X ! M ; then pk D p mk maps X mk perfectly onto the
space M mk for every k 2 ! (see Fact 4 of S.271).
To prove that ext .Y; Z/ D l.Y; Z/ is suffices to show that l.Y; Z/  ext .Y; Z/,
so assume the contrary; then there
S is an infinite cardinal   ext .Y; Z/ and a family
U  .Cp .X // such that Z  U and no subfamily of U of cardinality   covers
Y . It is easy to see that, without loss of generality, we can assume that U  B. We
will need the set Ak D fx 2 X mk W x; Ok
2 U g for any k 2 !; observe that the
fact that U covers Z implies that
(1) for any f 2 Z there is k 2 ! and x 2 Ak such that f mk .x/ 2 Ok .
On the other hand, no subfamily of U of cardinality   covers Y , so we have
(2) if Bk  Ak and jBk j   for any k 2 ! then there is f 2 Y such that
f mk .Bk / \ Ok D ; for every k 2 !.
Choose a function f0 2 Y arbitrarily and let B.k; 0/ D ; for all k 2 !. Assume
that 0 < <  C and we have chosen a set ff W < g  Y and a family
fB.k; / W < ; k 2 !g with the following properties:
(3) B.k; /  Ak and jB.k; /j   for all k 2 ! and < ;
(4) if  < < then B.k;  /  B.k; / for every k 2 !;
(5) for any < ; k 2 ! and any H 2 Fin.ff W  < g/ the set uH .B.k; // is
dense in uH .Ak / where uH D pk . ff mk W f 2 H g/ W X ! M mk  Rmk jH j ;
(6) fmk .B.k; // \ Ok D ; for all < and k 2 !.
To get f , let F D ff W < g and fix any k 2 !; for every H 2 Fin.F /
let uH D pk . ff mk W f 2 H g/ W X ! M mk  Rmk jH j . The space uH .Ak /
being hereditarily separable by Fact 1, there is a countable B.H; k/  Ak such that
uH .B.H; k// is dense in uH .Ak /. The set
[
[
fB.k; / W < g/ [ . fB.H; k/ W H 2 Fin.F /g/

B.k; / D .

80

2 Solutions of problems 001500

has cardinality  . Once we have a set B.k; / for every k 2 ! apply (2) to find a
function f 2 Y such that fmk .B.k; //\Ok D ; for all k 2 !. It is immediate that
the properties (3)(6) still hold for the set ff W  g and the family fB.k; / W
 ; k 2 !g and therefore our inductive construction can be continued to give us
a set D D ff W <  C g and a family fB.k; / W <  C ; k 2 !g such that the
properties (3)(6) hold for all <  C .
Assume that < <  C ; it follows from (5) that .pk fmk /.B.k; // is dense
in .pk fmk /.Ak / and hence fmk .B.k; // is dense in fmk .Ak / for all k 2 !. The
property (1) shows that fmk .Ak / \ Ok ; and therefore fmk .B.k; // \ Ok ;
for some k 2 !. On the other hand, fmk .B.k; // \ Ok D ; for all k 2 ! by the
property (6). Consequently, f f and therefore jDj D  C .
Our purpose is to prove that D is closed and discrete in Z, so assume, toward
a contradiction, that g is an accumulation point in Z for the set D. Recall that
t .Cp .X // D ! by TFS-149; so g is also an accumulation point for some countable
subset of D and hence the ordinal D nnf <  C W g is an accumulation point for
F g is well defined. It is evident that is a limit ordinal. There is k 2 ! and y 2 Ak
such that g 2 y; Ok
; it is evident that
T g is also an accumulation point for the set
G D F \ y; Ok
. The set K D f.f mk /1 .f mk .y// W f 2 Gg is nonempty
because y 2 K.
Let W D .g mk /1 .Ok / and assume that KnW ;. Take any x 2 KnW and
observe that g mk .x/ Ok while g mk .y/ 2 Ok and therefore g mk .x/ g mk .y/.
On the other hand, f mk .x/ D f mk .y/ for all f 2 G which contradicts g 2 G. We
proved that the case KnW ; is impossible, i.e., K  W .
Let Kf D .f mk /1 .f mk .y// for all f 2 G; the set N D pk1 .pk .y// is compact
because the map pk T
is perfect. Therefore N \ Kf is a nonempty compact set for
all f 2 G and y 2 fN \ Kf W f 2 Gg  K  W . Now we canTapply Fact 1
of S.326 to conclude that there is a finite H  G such that Q D fN \ Kf W
f 2 H g  W . Observe that, for the map uH D pk . ff mk W f 2 H g/ we
have Q D u1
H .uH .y//. Now, if Y D uH .X / then the map uH W X ! Y is perfect
because pk is perfect (see Fact 1 of T.266).
Therefore Fact 1 of S.226 is applicable to conclude that there is U 2 .Y / such
that uH .y/ 2 U and u1
H .U /  W . Let  D maxf W f 2 H g. Then  <  D
 C 1 < because is a limit ordinal. We have H 2 Fin.F / and therefore
uH .B.k; // is dense uH .Ak / by (5). Furthermore, uH .y/ 2 U \ uH .Ak / which
shows that U \ uH .Ak / is a nonempty open subset of uH .Ak /. The set uH .B.k; //
being dense in uH .Ak / by (5), we have uH .B.k; // \ U ; and therefore there
is z 2 B.k; / for which u1
H .uH .z//  W and, in particular, z 2 W . This implies
that g mk .z/ 2 Ok .
On the other hand, the condition (6) implies that fmk .B.k; // \ Ok D ;; the
conditions (4) and (6) show that, for any ordinal with  < < we have
fmk .B.k; // \ Ok  fmk .B.k; // \ Ok D ;. Consequently, fmk .z/ Ok
whenever   < which shows that g GnF and hence g is an accumulation
point for the set F which is a contradiction with  < and the choice of .

2 Solutions of problems 001500

81

This contradiction proves that D is a closed discrete subspace of Z. We already


saw that jDj D  C > ext .Y; Z/; this final contradiction shows that l.Y; Z/ 
ext .Y; Z/ and hence Fact 2 is proved.
Fact 3. Suppose that X is a space such that Cp .X / is Lindelf. Given a set
A  X assume that every countable subset of A is C -embedded in X . Then A
is C -embedded in X .
Proof. Fix a continuous function ' W A ! R; for any countable B  A the set
FB D ff 2 Cp .X / W f jB D 'g is nonempty and closed in Cp .X /. It is easy to
see that the family F D fFB W B is a countableT
subset of Ag is countably centered,
T
so the Lindelf property of Cp .X / implies that F ;. Any element of F is a
continuous extension of ' over the whole space X , so A is C -embedded in X and
hence Fact 3 is proved.
Fact 4. Given spaces X; Y and a map ' W X ! Y suppose that D  X is
discrete, 'jD is injective and D 0 D '.D/ is a discrete subspace of Y . If D 0
is C -embedded (or C  -embedded) in Y then the set D is C -embedded (or C  embedded respectively) in the space X .
Proof. Suppose that f W D ! R is a (bounded) function (observe that f is
automatically continuous because D is discrete). Then g D f .'jD/1 W D 0 ! R
is a continuous (bounded) function because D 0 is also discrete. The set D 0 being
C -embedded (C  -embedded) in Y , there is a continuous (bounded) function h W
Y ! R such that hjD 0 D g. It is straightforward that f1 D h ' W X ! R is a
continuous (bounded) function such that f1 jD D f , so Fact 4 is proved.
Fact 5. If D  Cp .!1 C 1/ is closed, discrete, and jDj  ! then D is C -embedded
in Cp .!1 C 1/.
Proof. Let  W Cp .!1 C 1/ ! Cp .!1 / be the restriction map. Since !1 is countably
compact and !1 C 1 D !1 (see TFS-314), the space !1 C 1 is the Hewitt
extension of !1 (see TFS-417 and Fact 3 of S.309). Therefore jA W A ! .A/
is a homeomorphism for any countable A  Cp .!1 C 1/ (see TFS-437). As a
consequence, the set D 0 D .D/ is closed and discrete in Cp .!1 /. The space
Cp .!1 / being Lindelf (see TFS-316), the set D 0 is C -embedded in Cp .!1 /, so
we can apply Fact 4 to see that D is also C -embedded in Cp .!1 C 1/. Fact 5 is
proved.
Fact 6. Given an infinite cardinal  if a set Z  Cp . C C 1/ separates the points
of  C C 1 then there is a discrete subspace D  Z which is closed in Cp . C C 1/
and jDj D  C .
Proof. For each <  C fix rational numbers s ; t and a function f 2 Z such
that f ./ < s < t < f . C / or f ./ > s > t > f . C /. Since each f
is continuous, there exists < such that f . / < s or f . / > s for every
 2 . ;
.
The map r W  C !  C defined by r./ D for all <  C satisfies the
hypothesis of Fact 3 of U.074, so there is <  C and a stationary set R   C such

82

2 Solutions of problems 001500

that D for all 2 R. There is a set R0  R with jR0 j D  C for which there
are s; t 2 Q such that s D s and t D t for all 2 R0 ; let E D ff W 2 R0 g.
Passing, if necessary, to a subset of E of cardinality  C , we can assume that either
f ./ < s < t < f . C / or f ./ > s > t > f . C / for all 2 R0 . Since these
two cases are analogous, we will only consider the first one.
For every function f 2 Cp . C C 1/ let Of D Cp . C C 1/nE if f E. Then
Of is an open neighborhood of f in Cp . C C 1/ such that Of \ E D ; and hence
Bf D f 2 R0 W f 2 Of g D ;. If f 2 E then f . C /  t because g. C / > t for
all g 2 E. Choose any s 0 2 .s; t / and observe that, by continuity of f , there is  >
such that f . / > s 0 > s. The set Of D fg 2 Cp . C C 1/ W g. / > s 0 g is an open
neighborhood of f in Cp . C C 1/. If >  and 2 R0 then  2 .;
D . ;

which implies, by the choice of , that f . / < s < s 0 whence f Of . Thus


Of \ E  ff W   g and therefore, for the set Bf D f 2 R0 W f 2 Of g, we
have jBf j  j j  .
The family U D fOf W f 2 Zg is an open cover of the spaceSCp . C C 1/ such
that jU S
\ Ej   for any U 2 U . If U 0  U and jU 0 j   then j. U 0 / \ Ej S
, so
the set U 0 does not cover the set E  Z and hence Z is not contained in U 0 .
This shows that l.Z; Cp . C C 1//   C , so we have e.Z; Cp . C C 1//   C (see
Fact 2) and hence there exists a discrete D  Z which is closed in Cp . C C 1/ and
jDj D  C . Fact 6 is proved.
Fact 7. If X is a space and !1 C 1 embeds in Cp .X / then there is a closed discrete
uncountable D  X such that every countable A  D is C -embedded in X .
Proof. Let  Cp .X / be a subspace homeomorphic to !1 C 1. For any x 2 X and
f 2 let '.x/.f / D f .x/. Then ' W X ! Cp ./ is a continuous map such that
Y D '.X / separates the points of . By Fact 6, there is an uncountable discrete
D 0  Y which is closed in Cp ./. For any y 2 D 0 choose a point a.y/ 2 ' 1 .y/;
then the set D D fa.y/ W y 2 D 0 g  X is uncountable, discrete, and closed in X .
Since the map 'jD is an injection, we can apply Fact 4 and Fact 5 to conclude that
every countable A  D is C -embedded in X , so Fact 7 is proved.
Fact 8. If Cp .X / is Lindelf then !1 does not embed in Cp .X /.
Proof. Assume that there is a subspace Z  Cp .X / which is homeomorphic to !1 .
Since Z is pseudocompact and Lindelf, it has to be compact, so K D Z is a
compact extension of Z. Observe that Z is not closed in Cp .X / because !1 is not
Lindelf. Since !1 C 1 is canonically homeomorphic to !1 (see TFS-314), there
is a continuous onto map ' W .!1 C 1/ ! Z such that 'j!1 W !1 ! Z is a
homeomorphism.
Apply Fact 3 of S.261 to see that '..!1 C 1/n!1 / D ZnZ and hence ZnZ
is a singleton. An immediate consequence is that ' is a bijection and hence
homeomorphism. Thus K is a subspace of Cp .X / homeomorphic to !1 C 1. By
Fact 7, there is a closed discrete uncountable D  X such that every countable
A  D is C -embedded in X . Apply Fact 3 to conclude that D is C -embedded in X .
If  W Cp .X / ! Cp .D/ is the restriction map then .Cp .X // D Cp .D/ D RD .

2 Solutions of problems 001500

83

This, together with the Lindelf property of Cp .X /, implies that RD is Lindelf


which is a contradiction (see e.g., Fact 2 of S.215). Thus !1 cannot be embedded in
Cp .X /, so Fact 8 is proved.
Returning to our solution observe that if X is Lindelf then !1 C1 does not embed
in Cp .X / by Fact 1 of U.089. Since !1 C 1 embeds in R!1 , the space R!1 cannot be
embedded in Cp .X / either. Now, if Cp .X / is Lindelf then even the space !1 is not
embeddable in Cp .X / by Fact 8; as before, this implies that R!1 is not embeddable
in Cp .X / as well.
V.024. Prove that there exists a space X such that c.X / D ! and R!1 embeds in
Cp .X / as a closed linear subspace.
Solution. All spaces in this solution are considered to be nonempty. We will often
use without explicit reference the fact that any linear space has a Hamel basis (see
Fact 1 of S.489).
Fact 1. Suppose that Z is a space and H is a Hamel basis in Cp .Z/. If A  H is
a finite nonempty set then, for any u 2 RA , there is a continuous linear functional
' W Cp .Z/ ! R such that 'jA D u.
Proof. For any x 2 Z let ex .f / D f .x/ for any f 2 Cp .Z/; then ex is a
continuous linear functional on Cp .Z/ (see TFS-196).
We will show first by induction on n 2 N that
(1) for any f1 ; : : : ; fn 2 H there is a set fx1 ; : : : ; xn g  Z such that the family
f.fi .x1 /; : : : ; fi .xn // W i D 1; : : : ; ng of vectors of Rn is linearly independent.
If n D 1 then it follows from f1 2 H that f1 is not identically zero, so there
is x1 2 Z such that f1 .x1 / 0. It is clear that the vector .f1 .x1 // forms an
independent family in R, so (1) is proved for n D 1.
Assume that the property (1) is proved for all n  m and fix any functions
f1 ; : : : ; fmC1 2 H . By the induction hypothesis there is a set fx1 ; : : : ; xm g  Z
such that the family f.fi .x1 /; : : : ; fi .xm // W i D 1; : : : ; mg of vectors of Rm
is linearly independent; let ai D .fi .x1 /; : : : ; fi .xm // for all i  m C 1. It is
clear that the family fai W i  m C 1g cannot be linearly independent in
Rm , so we can find 1 ; : : : ; m 2 R such that amC1 D 1 a1 C : : : C m am .
However, the functions f1 ; : : : ; fmC1 are linearly independent, so fmC1 1 f1 C
: : : C m fm ; therefore there is a point xmC1 2 Z such that fmC1 .xmC1 /
1 f1 .xmC1 / C : : : C m fm .xmC1 /. We leave it to the reader to verify that the family
f.fi .x1 /; : : : ; fi .xmC1 // W i  m C 1g of vectors of RmC1 is linearly independent,
so (1) is proved.
Now choose a faithful enumeration ff1 ; : : : ; fn g of the set A and let bi D u.fi /
for any i D 1; : : : ; n. Apply the property (1) to find a set fx1 ; : : : ; xn g  Z such
that the family f.fi .x1 /; : : : ; fi .xn // W i  ng of vectors of the space Rn is linearly
independent. A well-known theorem of algebra shows that there exist 1 ; : : : ; n 2
R such that 1 fi .x1 / C : : : C n fi .xn / D bi for each i D 1; : : : ; n. It is evident that
' D 1 ex1 C : : : C n exn is a continuous linear functional on Cp .Z/ and '.fi / D bi
for every i  n, i.e., 'jA D u, so Fact 1 is proved.

84

2 Solutions of problems 001500

Fact 2. Suppose that Z is a space and H  Cp .Z/ is a Hamel basis in Cp .Z/.


Let J D f.a; b/ W a; b 2 Q and a < bg. Given functions f1 ; : : : ; fn 2 Cp .Z/ and
O1 ; : : : ; On 2 J let f1 ; : : : ; fn I O1 ; : : : ; On
D f' 2 Lp .Z/ W '.fi / 2 Oi for all
i  ng. Then the family O D ff1 ; : : : ; fn I O1 ; : : : ; On
W n 2 N; f1 ; : : : ; fn 2 H
and O1 ; : : : ; On 2 J g is a base in the space Lp .Z/.
Proof. It is clear that all elements of O are open in Lp .Z/. To prove that O is
a base in Lp .Z/ take a point ' 2 Lp .Z/ and a set W 2 .'; Lp .Z//. There are
g1 ; : : : ; gk 2 Cp .Z/ and " > 0 such that ' 2 V D fu 2 Lp .Z/ W ju.gi /'.gi /j < "
for each i  kg  W .
The set H being a Hamel basis in the space Cp .Z/ there are f1 ; : : : ; fn 2 H
such that every function gi is a linear combination of f1 ; : : : ; fn . Thus for any i 2
f1; : : : ; kg there are i1 ; : : : ; in 2 R such that gi D i1 f1 C : : : C in fn . For the
number D maxfj ij j W i  k; j  ngC1 choose > 0 such that n < " and take
a set Oi D .a; b/ 2 J such that '.fi / 2 Oi  .'.fi /; '.fi /C/ for every i  n.
The set U D f1 ; : : : ; fn I O1 ; : :P
: ; On
belongs to O and ' 2 U . Given any u 2 U
observe that ju.gi /'.gi /j D j nj D1 ij .u.fj /'.fj //j  n < " for any i  k
and therefore u 2 V . This proves that U  V and hence ' 2 U  V  W . Thus
O is a base in Lp .Z/ and Fact 2 is proved.
Fact 3. The space Lp .Z/ has the Souslin property for any space Z.
Proof. If c.Lp .Z// > ! then there exists a disjoint family U   .Lp .Z// with
jU j D !1 . Since the family O from Fact 2 is a base in Lp .Z/, we can assume,
without loss of generality, that U  O. For any U D f1 ; : : : ; fn I O1 ; : : : ; On
2 U
let kU D n, supp.U / D ff1 ; : : : ; fn g and O.U / D .O1 ; : : : ; On / 2 J n . The
family J being countable we can consider, without loss of generality (passing
to an appropriate uncountable family of U if necessary), that there are n 2 N
and O1 ; : : : ; On 2 J such that, for any U 2 U , we have kU D n and U D
f1U ; : : : ; fnU I O1 ; : : : ; On
for some f1U ; : : : ; fnU 2 H .
Apply the Delta-lemma (SFFS-038) to see that there exists an uncountable
U 0  U and a finite set D  H such that supp.U / \ supp.V / D D for
any distinct U; V 2 U 0 . If ff1 ; : : : ; fm g is a faithful enumeration of D then
changing the respective order in the sets ff1U ; : : : ; fnU g and fO1 ; : : : ; On g if
necessary, we can assume, without loss of generality, that for any U 2 U 0 we have
U
V
; : : : ; fnU I O1 ; : : : ; On
while the sets ffmC1
; : : : ; fnV g and
U D f1 ; : : : ; fm ; fmC1
U
U
0
ffmC1 ; : : : ; fn g are disjoint whenever U; V 2 U and U V .
Take distinct U; V 2 U 0 and apply Fact 1 to find a linear continuous functional
' W Cp .Z/ ! R such that '.fi / 2 Oi for all i  m (observe that the set D
can be empty; in this case m D 0) while '.fiU / 2 Oi and '.fiV / 2 Oi for all
i 2 fm C 1; : : : ; ng. It is immediate that ' 2 U \ V ; this contradiction shows that
c.Lp .Z// D ! and hence Fact 3 is proved.
Returning to our solution let D be a discrete space with jDj D !1 and let X D
Lp .D/. There exists an l-embedding of D in X by CFS-466, so Cp .D/ D RD '
R!1 embeds in Cp .X / as a closed linear subspace by CFS-448. Finally, apply Fact 3
to see that c.X / D ! and hence our solution is complete.

2 Solutions of problems 001500

85

V.025. Prove that if ! C 1! X then Cp .X / ' Cp .X /  R! . Deduce from


this fact that pseudocompactness, countable compactness, and compactness are not
t -invariant.
Solution. Let K  X be a subspace homeomorphic to ! C1 and consider the space
I D ff 2 Cp .X / W f .K/ D f0gg. The set K is l-embedded in X (see CFS-482)
and hence Cp .X / ' I  Cp .K/ by CFS-448. Furthermore, Cp .K/ '  .!/ (see
CFS-105), so we can apply Problem 017 to conclude that Cp .X / ' I   .!/ '
I   .!/  R! ' Cp .X /  R! .
We proved, in particular, that Cp .! C 1/ ' Cp .! C 1/  R! ' Cp ..! C
t
1/ !/ and therefore .! C 1/ .! C 1/ !. The space ! C 1 is compact while
.! C 1/ ! is not even pseudocompact. This shows that pseudocompactness,
countable compactness, and compactness are not t -invariant.
V.026. Prove that Cp .R/! ' Cp .R/ and Cp .I/! ' Cp .I/.
Solution. It follows from TFS-177 that Cp .R/ ' .Cp .R//! . To prove that the
space .Cp .I//! is homeomorphic to Cp .I/ it suffices to establish the same for the
space I D 0; 1
because I ' I.
The space K D f0g [ f2n W n 2 !g  I is homeomorphic to ! C 1; since it
is l-embedded in I (see CFS-482), for the set M D ff 2 Cp .I / W f .K/ D f0gg,
we have Cp .I / ' M  Cp .K/. Recalling that Cp .K/ '  .!/ (see CFS-105) we
conclude that Cp .I / ' M   .!/. Let an D 2n1 ; bn D 2n and In D an ; bn
;
given a function f 2 Cp .In / let jjf jjn D fsup jf .x/j W x 2 In g be its usual norm
in the space Cp .In / for any n 2 !.
We will also need the space Mn D ff 2 Cp .In / W f .an / D f .bn / D 0g; let
n W Cp .I / ! Cp .In / be the restriction map and observe that n .M / D Mn for
each n 2 !. We claim that
Q
(1) the map  D n2! n W M ! M 0 D .M /  n2! Mn is a homeomorphism.
Q
Let pn W n2! Mn ! Mn be the projection for every n 2 !. It is an easy exercise
that  is a continuous bijection; to see that the map  1 W M 0 ! M is continuous,
let qx .f / D f .x/ for any x 2 I and f 2 M . We also need an analogous map in
every space Cp .In /: let qxn .f / D f .x/ for any x 2 In and f 2 Cp .In /. If x 2 K
then .qx  1 /.g/ D 0 for any g 2 M 0 , so the map qx  1 is continuous. If
x 2 I nK then fix the unique n 2 ! with x 2 In . Given a function g 2 M 0 observe
that qx . 1 .g// D g.n/.x/; this shows that qx  1 D qxn pn is continuous
being the composition of two continuous maps. Therefore we can apply TFS-102 to
see that  1 is continuous and hence
Q  is, indeed, a homeomorphism.
Next observe that M 0 D ff 2 n2! Mn W jjf .n/jjn ! 0g. If 'n W In ! I is a
homeomorphism then the dual map 'n W Cp .I / ! Cp .In / is a homeomorphism

which preserves the norm, i.e., jj'
any Q
f 2 Cp .I /. As
Qn .f /jjn D jjf jj for
!
a consequence, the map ' D
'
W
.C
.I
//
!
pQ
n2! n
n2! Cp .In / is a
homeomorphism such that E D '.C .I // D ff 2 n2! Cp .In / W jjf .n/jjn ! 0g
(see Problem 018 for the definition of C .X / for any space X ).

86

2 Solutions of problems 001500

For any n 2 ! we will need an extender en from the doubleton Sn D fan ; bn g


.an /
to In . Define en by letting en .f /.t / D f .bbnn/f
.t an /Cf .an / for any f 2 RSn
an
and t 2 In . In other words, e.f / is the linear function whose graph is obtained
connecting the points .an ; f .an // and .bn ; f .bn // by a line segment. It is easy to
check that the map e W RSn ! Cp .In / is a linear continuous extender such that
jje.f /jj D maxfjf .an /j:jf .bn /jg for any f 2 RSn .
Let rn0 W RSn  Mn ! RSn and rn1 W RSn  Mn ! Mn be the natural projections
for each n 2 !. It follows from CFS-448 that every map un W RSn  Mn ! Cp .In /
defined by un .f; g/ D e.f / C g for any .f; g/ 2 RSn  Mn , is a homeomorphism;
denote by vn its inverse. By the choice of our extender e we can see that
(2) jjun .f; g/jjn  maxfjf .an /j; jf .bn /jg C jjgjjn for any .f; g/ 2 RSn  Mn .
Besides, for any h 2 Cp .In / we have
(3) jjrn1 .vn .h//jjn  2jjhjjn and maxfjrn0 .vn .h//.an /j; jrn0 .vn .h//.bn /jg  jjhjjn .
Q
Q
Q
Sn
The product map v D n2! vn W n2! Cp .In / ! Q
P D n2! .RQ
 Mn /
Sn
is, evidently, a homeomorphism. Let w W P ! R D . n2! R /  . n2! Mn /
be the homeomorphism obtained by an evident coordinate
Q permutation. It follows
from the properties (2) and (3) that QD w v W
n2! Cp .In / ! R is a
homeomorphism such that, for any f 2 n2! Cp .In /, if .f / D .g; h/ 2 R then
the sequence fjjf .n/jjn W n 2 !g converges to zero if and only if both sequences
S g
D fmaxfjg.n/.an /j; jg.n/.bn /jg W n 2 !g and fjjh.n/jjn W n 2 !g converge
to zero.
Q
In other words, .E/ D A  M 0 where A D fg 2 n2! RSn W S g
! 0g
and hence E ' A  M 0 . It is evident that A '  .!/, so E ' M 0   .!/. We
already saw that E ' C .I / and M ' M 0 , so C .I / ' M 0   .!/ ' Cp .I /.
This makes it possible to apply Problem 019 to conclude that Cp .I / ' .Cp .I //!
and hence our solution is complete.
V.027. Prove that R is t -equivalent to 0; 1
.
Solution. For any n 2 Z let In D n; n C 1
 R and Sn D fn; n C 1g; we will
also need the set M D ff 2 Cp .R/ W f .Z/ D f0gg. It is easy to see that the
family f.n  13 ; n C 13 / W n 2 Zg   .R/ is discrete so Cp .R/ is homeomorphic
to the space RZ  M ' R!  M (see Fact 1 of T.217). Every restriction map
n W Cp .R/ ! Cp .In / is continuous; let Mn D ff 2 Cp .In / W f .Sn / D f0gg for
each n 2 Z.
Q
The diagonal product map  D fn W n 2 Zg W Cp .R/ !
Q fCp .In / W n 2 Zg
is continuous; it is straightforward
to check that .M / D fMn W n 2 Zg and
Q
the map jM W M ! fMn W n 2 Zg is a homeomorphism. Q
As a consequence,
!
 fMn W n 2 Zg;
the space Cp .R/ is homeomorphic to the space H D RQ
an evident permutation of coordinates shows that H ' fR2  Mn W n 2 Zg.
Furthermore, R2  Mn ' CpQ
.In / (see Fact 1 of S.409) for any n 2 Z, so the space
Cp .R/ is homeomorphic to fCp .In / W n 2 Zg; recalling that every Cp .In / is
homeomorphic to Cp .I/ and Z is countable, we can see that Cp .R/ ' .Cp .I//! .
Finally, observe that .Cp .I//! ' Cp .I/ by Problem 026 and I ' 0; 1
, so Cp .R/ '
Cp .I/ ' Cp .0; 1
/ which shows that R is t -equivalent to 0; 1
.

2 Solutions of problems 001500


t

87
t

V.028. Prove that X  Y whenever X  Y . Give an example which shows that


t
t
X  Y does not necessarily imply X  Y .
Solution. If Z is a space and   exp.Z/ then jA D fM \ A W M 2 g for
any A  Z; let M D f W is a Tychonoff topology on Z and jA D .Z/jA for
any countable
S A  Zg. The family M is nonempty because .Z/ 2 M. Therefore
the family M can be considered a subbase for a topology Z on the set Z. It is
easy to see that Z 2 M and, if Z ' Y then the spaces .Z; Z / and .Y; Y / are
homeomorphic.
t
Suppose that X  Y and hence Cp .X / ' Cp .Y /; since the restriction map
 W Cp .X / ! Cp .X / is a condensation, we can identify the sets C.X / and
C.X / and consider that the topology of Cp .X / is given on the set C.X /.
Reformulating TFS-437 we can see that the topology of Cp .X / on C.X / coincides
with Cp .X/ . Analogously, the topology of Cp .Y / on C.Y / coincides with Cp .Y / .
As a consequence, Cp .X / ' .C.X /; Cp .X/ / ' .C.Y /; Cp .Y // / ' Cp .Y /
which shows that the spaces X and Y are t -equivalent.
Next observe that the spaces X D A.!/ and Y D A.!/ ! are t -equivalent
by Problem 017 and CFS-105. We claim that the spaces X D X and Y are
not t -equivalent. Indeed, it easily follows from Fact 2 of S.451 that the space Y
contains the space ! and therefore nw.Y /  nw.!/ D w.!/ > ! (see TFS368). However, nw.X / D nw.A.!// D !, so nw.X / nw.Y / and hence the
spaces X and Y cannot be t -equivalent by Problem 001.
V.029. Give an example of spaces X and Y such that X ' Y (and hence
t
X  Y ) while X is not t -equivalent to Y .
Solution. Let X D !1 and Y D !1 C 1; then Y D Y ' X (see TFS-314,
Fact 3 of S.309 and TFS-417). However, X is not t -equivalent to Y because Cp .X /
is Lindelf and Cp .Y / is not (see TFS-316 and TFS-320).
V.030. Prove that -monolithity and -stability are t -invariant for any infinite
cardinal .
t

Solution. If X  Y and X is -monolithic then Cp .X / is -stable (see SFFS152) and hence so is Cp .Y /. Applying SFFS-152 again we conclude that Y is
-monolithic. This proves that -monolithity is t -invariant.
t
Now, if X  Y and X is -stable then Cp .X / is -monolithic (see SFFS-154)
and hence Cp .Y / is -monolithic. Applying SFFS-154 again we conclude that Y is
-stable. This proves that -stability is also t -invariant.
t

V.031. Suppose that X  Y . Prove that X is functionally perfect if and only if so


is Y .
Solution. If X is functionally perfect, then Cp .X / has a dense  -compact subspace
(see CFS-301); therefore Cp .Y / also has a dense  -compact subspace, so we can
apply CFS-301 once more to conclude that Y is functionally perfect. Analogously, if
Y is functionally perfect then so is X and hence the space X is functionally perfect
if and only if Y is functionally perfect.

88

2 Solutions of problems 001500

V.032. Give an example of spaces X and Y such that X is functionally perfect,


Cp .Y / embeds into Cp .X / while Y is not functionally perfect.
Solution. Let X be a discrete space of cardinality !1 and Y D !1 C 1. Then Y is
not an Eberlein compact because t .Y / > !. Therefore Y is not functionally perfect.
However, X is functionally perfect (see CFS-316) and X maps continuously onto
Y ; this implies that Cp .Y / embeds in Cp .X / (see TFS-163).
V.033. Suppose that compact spaces X and Y are t -equivalent. Prove that X is
Eberlein (Corson or Gulko) compact if and only if so is Y .
Solution. If X is Eberlein compact then X is functionally perfect, so Y is also
functionally perfect by Problem 031 and hence Y is Eberlein compact. Analogously,
if Y is Eberlein compact then so is X .
Now, if X is Corson compact then Cp .X / is primarily Lindelf (see CFS-150),
so Cp .Y / is also primarily Lindelf and hence Y is Corson compact. Analogously,
if Y is Corson compact then so is X .
Finally, if X is Gulko compact then Cp .X / is a Lindelf -space so Cp .Y /
is also a Lindelf -space and hence Y is Gulko compact. Analogously, if Y is
Gulko compact then so is X .
V.034. Suppose that F  Cp .X; I/ is a D-separating set (and hence 0X 2 F ). For
e D fex W x 2 X g is a
each x 2 X , let ex .f / D f .x/ for any f 2 F . Prove that X
closed subset of the space ZF .X / D f' W F ! I W '.0X / D 0 and '.V /   12 ; 12

for some V 2 .0X ; F /g, and the map x ! ex is a homeomorphism between


e In other words, X is canonically homeomorphic to a closed subset of
X and X.
ZF .X /.
Solution. For an arbitrary space Z and p 2 Z let D.Z; p/ D ff 2 IZ W f .p/ D 0
and there is U 2 .p; Z/ such that f .U /   12 ; 12
g. We consider that D.Z; p/ is
a space with the topology induced from IZ .
Given any point x 2 X let e.x/ D ex ; then e.x/ 2 Cp .F; I/ and the map
e W X ! Cp .F; I/ is continuous (see TFS-166). Since e.x/.0X / D 0X .x/ D 0
for any x 2 X and e.x/ is continuous at 0X , we have e.x/ 2 D.F; 0X / for any
x 2 X , i.e., e.X /  D.F; 0X /. It is an evident consequence of the fact that F is
D-separating that F separates the points and closed sets, i.e., for any x 2 X and
any closed P  X with x P , there is f 2 F such that f .x/ f .P /. Therefore
e D e.X / is closed in
e is an embedding by TFS-166 and we only must prove that X
ZF .X / D D.F; 0X /.
Take any element ' 2 D.F; 0X /ne.X /. There exists O 2 .0X ; F / such that
'.O/   12 ; 12
. By definition of the pointwise convergence topology there is a
finite K  X and " > 0 such that 0X 2 W D ff 2 F W f .K/  ."; "/g  O and
hence '.W /   12 ; 12
. Since ' e.K/, there is U 2 .K; X / such that ' e.U /.
The family F being D-separating, there is g 2 F such that g.K/  ."; "/ and
g.X nU /  34 ; 1
and, in particular, g 2 W . This implies e.x/.g/ D g.x/ 2 34 ; 1

for every point x 2 X nU while we have '.g/ 2 '.W /   12 ; 12


. Consequently,
the set G D f 2 D.F; 0X / W .g/ < 34 g is an open neighborhood of ' in D.F; 0X /

2 Solutions of problems 001500

89

such that G \ e.X nU / D ; and therefore ' e.X nU /. It is easy to see that this
implies ' e.U / [ e.X nU / D e.X /. The function ' 2 D.F; 0X /ne.X / was
e D e.X / is closed in the space D.F; 0X / D ZF .X /.
chosen arbitrarily, so X
V.035. Knowing that 0X 2 F  Cp .X; I/ and 0Y 2 G  Cp .Y; I/, suppose that
there is an embedding i W G ! F with i.0Y / D 0X . Prove that ZF .X / maps
continuously onto ZG .Y /.
Solution. For an arbitrary space Z and p 2 Z let D.Z; p/ D ff 2 IZ W f .p/ D 0
and there is U 2 .p; Z/ such that f .U /   12 ; 12
g. We consider that D.Z; p/ is
a space with the topology induced from IZ .
Let H D i.G/; for any f 2 IH the function i  .f / D f i belongs to IG and
it is easy to see, using TFS-163, that the map i  W IH ! IG is a homeomorphism.
Besides, it follows from i.0Y / D 0X that i  .D.H; 0X // D D.G; 0Y /. Since 0X 2
H  F , the restriction map  W IF ! IH maps D.F; 0X / onto D.H; 0X / by
Fact 8 of T.250. Therefore i   maps ZF .X / D D.F; 0X / continuously onto
ZG .Y / D D.G; 0Y /.
V.036. Given a space X prove that if 0X 2 F  Cp .X; I/ then ZF .X / belongs to
the class K.X /.
Solution. This was proved in Fact 10 of T.250.
V.037. Let G be a D-separating subspace of Cp .Y /. Prove that, if G embeds into
Cp .X / then Y 2 K.X /.
Solution. It is easy to see that there exists an embedding e W Cp .X / ! Cp .X; I/
such that e.0X / D 0X . The space Cp .X / being homogeneous (i.e., for any functions
f; g 2 Cp .X / there is a homeomorphism ' W Cp .X / ! Cp .X / such that '.f / D g
(see TFS-079)), there is an embedding w W G ! Cp .X / such that w.0Y / D 0X .
Therefore i D e w embeds G in Cp .X; I/ in such a way that i.0Y / D 0X .
This shows that, for the set F D Cp .X; I/, the space ZG .Y / is a continuous
image of ZF .X / by Problem 035. The space ZF .X / belongs to the class K.X /
by Problem 036 and hence ZG .Y / also belongs to K.X /. The space Y embeds in
ZG .Y / as a closed subspace (see Problem 034), so Y 2 K.X /.
V.038. Given arbitrary spaces X; Y and a subspace Z  Y suppose that the space
Cp .ZjY / D ff 2 Cp .Z/ W f D gjZ for some g 2 Cp .Y /g embeds in Cp .X /.
Prove that Z 2 K.X /.
Solution. Suppose that A  Z is finite, F  Z is closed in Z and A \ F D ;.
Then G D clY .F / is closed in Y and A \ G D ;. By the Tychonoff property of
Y , for any a 2 A, there is a function
Q fa 2 C.Y; 0; 1
/ such that fa .a/ D 1 and
fa .G/  f0g. The function g D a2A .1  fa / 2 C.Y; 0; 1
/ is equal to zero on
A and g.G/  f1g. Consequently, h D gjZ 2 Cp .ZjY / while h.F /  f1g and
h.A/  f0g. This proves that the set E D ff jZ W f 2 Cp .Y; I/g is D-separating.
Since Cp .ZjY / embeds in Cp .X /, the space E  Cp .ZjY / also embeds in Cp .X /,
so we can apply Problem 037 to conclude that Z belongs to the class K.X /.

90

2 Solutions of problems 001500

V.039. Let X be a  -compact space. Prove that any space Y 2 K.X / is also
 -compact.
Solution. It is easy to see that the class SK of  -compact spaces is complete and
contains all compact spaces. Therefore X 2 SK implies K.X /  SK because
K.X / is the minimal complete class which contains X and all compact spaces.
Thus every Y 2 K.X / belongs to SK, i.e., Y is  -compact.
V.040. Let X be a Lindelf -space. Prove that any Y 2 K.X / is also a Lindelf
-space.
Solution. It is easy to see that the class LS of Lindelf -spaces is complete and
contains all compact spaces. Therefore X 2 LS implies K.X /  LS because K.X /
is the minimal complete class which contains X and all compact spaces. Thus every
Y 2 K.X / belongs to LS, i.e., Y is a Lindelf -space.
V.041. Let X be a K-analytic space. Prove that any space Y 2 K.X / is also
K-analytic.
Solution. It is easy to see that the class KA of K-analytic spaces is complete and
contains all compact spaces. Therefore X 2 KA implies K.X /  KA because
K.X / is the minimal complete class which contains X and all compact spaces.
Thus every Y 2 K.X / belongs to KA, i.e., Y is a K-analytic space.
V.042. Prove that ext  .Y /  ext  .X / for any Y 2 K.X /.
Solution. Assume that ext  .X / D  and consider the class E of spaces Z such
that ext  .Z/  . We leave to the reader a simple verification of the fact that E is a
complete class and all compact spaces are in E. Therefore K.X /  E by minimality
of the class K.X /, so any Y 2 K.X / belongs to E, i.e., ext  .Y /   D ext  .X /.
V.043. Suppose that Cp .Y / embeds into Cp .X /. Prove that
(1)
(2)
(3)
(4)

if X is  -compact then Y is  -compact;


if X is Lindelf -space then Y is Lindelf .
if X is K-analytic then Y is also K-analytic.
ext  .Y /  ext  .X /.
t

As a consequence, if X  Y then ext  .X / D ext  .Y / and, for any property


P 2 f -compactness, Lindelf -property, K-analyticityg, we have X ` P if and
only if Y ` P.
Solution. It follows from Cp .Y /! Cp .X / that Cp .Y; I/! Cp .X /; since
Cp .Y; I/ is a D-separating subset of Cp .Y /, we can apply Problem 037 to see
that Y 2 K.X /.
Now, the statement (1) follows from Problem 039, the property (2) is an
immediate consequence of Problem 040, the property (3) is implied by Problem 041
and (4) can be deduced from Problem 042.

2 Solutions of problems 001500

91

V.044. Suppose that X is an analytic space and Cp .Y / embeds into Cp .X /. Prove


t
that Y is also analytic. In particular, analyticity is t -invariant, i.e., if X  Y then X
is analytic if and only if so is Y .
Solution. The space X being K-analytic (see SFFS-346), it follows from Problem 043 that Y is also K-analytic. Besides, nw.X /  ! because X is analytic and
network weight is not increased by continuous images. Therefore
nw.Y / D nw.Cp .Y //  nw.Cp .X // D nw.X /  !
and hence we can apply SFFS-346 again to conclude that Y is also analytic.
t

V.045. Suppose that X  Y . Prove that X is  -bounded if and only if so is Y .


Solution. The space X is  -bounded if and only if X is  -compact (see TFS-416);
t
since also X  Y (see Problem 028), the space Y is  -compact if and only if
the space X is  -compact (see Problem 043). Finally, Y is  -compact if and only
if Y is  -bounded (here we used TFS-416 again). This shows that X is  -bounded
if and only if so is Y .
V.046. Given a zero-dimensional space Y , suppose that Cp .Y; D/ embeds in
Cp .X /. Prove that, for any property P 2 f -compactness, Lindelf -property,
analyticity, K-analyticityg, we have Y ` P whenever X ` P.
Solution. We leave it to the reader to verify that the set Cp .Z; D/ is D-separating in
Cp .Z/ for any zero-dimensional space Z. Thus it follows from Cp .Y; D/! Cp .X /
that Y 2 K.X / (see Problem 037). Now, if X is  -compact then so is Y by
Problem 039. If X is Lindelf then Y is Lindelf by Problem 040. If X is
K-analytic then Y is also K-analytic by Problem 041.
If the space X is analytic then Y is K-analytic by our above observations and
nw.Cp .Y; D//  nw.Cp .X // D !. Since Cp .Y; D/ is D-separating, it separates the
points from closed sets in Y , so Y embeds in Cp .Cp .Y; D// by TFS-166. Therefore
nw.Y /  nw.Cp .Cp .Y; D/// D nw.Cp .Y; D//  ! which shows that we can apply
SFFS-346 to conclude that Y is analytic.
V.047. Let X be a zero-dimensional space. Prove that l  .X / D t .Cp .X; D//.
Solution. Denote by Fin(X) the family of all finite subsets of the space X and
observe that t .Cp .X; D//  t .Cp .X // D l  .X / (see TFS-149), so we only have
to prove that l  .X /   D t .Cp .X; D//. To do so fix an open !-cover U of the
space X . For any finite F  X choose a set UF 2 U with F  UF ; since X is
zero-dimensional, we can find a clopen set OF such that F  OF  UF . Let hF
be the characteristic function of the set OF , i.e., hF .x/ D 1 for all x 2 OF and
hF .x/ D 0 if x 2 X nOF .
The function u 2 Cp .X; D/ with u.x/ D 1 for all x 2 X belongs to the closure of
the set fhF W F 2 Fin.X /g  Cp .X; D/, so there is a family F  Fin.X / such that
jFj   and u 2 fhF W f 2 Fg; let U 0 D fUF W F 2 Fg. Given a finite K  X the

92

2 Solutions of problems 001500

set G D ff 2 Cp .X; D/ W f .K/  f1gg is an open neighborhood of u in Cp .X; D/,


so there is F 2 F with hF 2 G; this, evidently, implies K  OF  UF 2 U 0 , so
we proved that U 0 is an !-cover of X of cardinality  . Thus every !-cover of X
has an !-subcover of cardinality at most , so we can apply TFS-148 to conclude
that l  .X /   and hence l  .X / D t .Cp .X; D//.
V.048. Let X and Y be zero-dimensional spaces with Cp .X; D/ ' Cp .Y; D/. Prove
that X is pseudocompact if and only if so is Y . Deduce from this fact that X is
compact if and only if so is Y .
Solution. As usual, given a space Z and A  Z, the characteristic function A of
the set A is defined by A .x/ D 1 for all x 2 A and A .x/ D 0 for all x 2 ZnA.
Suppose that X is pseudocompact and take a countable set A  Cp .X; D/. Since
the space Cp .X / is !-monolithic (see Fact 9 of S.351), we have nw.A/  !. For
any x 2 X let '.x/.f / D f .x/ for any f 2 B D A. Then ' W X ! Cp .B; D/ is a
continuous map (see TFS-166); let X 0 D '.X /. We have
nw.X 0 /  nw.Cp .B; D//  nw.Cp .B// D nw.B/ D !I
this, together with pseudocompactness of X 0 , shows that the space X 0 is compact
and metrizable.
For any u 2 B let .u/.f / D f .u/ for any f 2 X 0 ; since X 0 generates the
topology of B (see TFS-166), the map  embeds B in Cp .X 0 ; D/. Apply Fact 1
of U.077 to conclude that jBj  jCp .X 0 ; D/j D ! and therefore the closure of
every countable subset of Cp .X; D/ is countable. Since Cp .Y; D/ ' Cp .X; D/, we
conclude that
(1) the closure of every countable subset of Cp .Y; D/ is countable.
Assume toward a contradiction that Y is not pseudocompact and fix a discrete
family fUn W n 2 !g   .Y /; pick a point xn 2 Un , a clopen set On such that xn 2
On  Un and let fn be the characteristic function ofP
On for any n 2 !. Let D D
fxn W n 2 !g; if g 2 DD then the function e.g/ D ffn W n 2 g 1 .1/g belongs
to Cp .Y; D/. Apply Fact 5 of T.132 to see that the map e W DD ! Cp .Y; D/ is
continuous; it is straightforward that e is injective, so K D e.DD / is homeomorphic
to DD ' D! .
The space K being second countable, we can choose a countable dense A  K.
Since the set K D A is uncountable, we obtain a contradiction with (1) which
shows that Y has to be pseudocompact. This shows that pseudocompactness of X
implies pseudocompactness of Y ; changing the roles of X and Y in the above proof
we can derive pseudocompactness of X from pseudocompactness of Y . Thus X is
pseudocompact if and only if so is Y .
Finally, assume that the space X is compact; then Y is pseudocompact by the
above result. Furthermore, t .Cp .X; D// D l  .X / D ! (see Problem 047) and hence
l  .Y / D t .Cp .Y; D// D !. This shows that Y is compact being pseudocompact
and Lindelf. Therefore compactness of X implies compactness of Y . Analogously,
compactness of Y implies compactness of X , so X is compact if and only if so is Y .

2 Solutions of problems 001500

93

V.049. Prove that there exist zero-dimensional spaces X and Y such that Cp .X / is
homeomorphic to Cp .Y / and Cp .X; D/ is not homeomorphic to Cp .Y; D/.
Solution. Let X D ! C 1 and Y D .! C 1/ !. It is trivial that X and Y
are zero-dimensional; it follows from CFS-105 and Problem 017 that Cp .X / is
homeomorphic to Cp .Y /. However, Cp .X; D/ is not homeomorphic to Cp .Y; D/
because X is compact and Y is not even pseudocompact (see Problem 048).
V.050. Prove that .Cp .X; D// D w.Cp .X; D// D jX j for any zero-dimensional
space X .
Solution. We have .Cp .X; D//  w.Cp .X; D//  w.Cp .X // D jX j (see
TFS-169), so it suffices to show that jX j   D .Cp .X; D//. To do this let
u 2 Cp .X; D/ be the function which is identically zero on X and fix a local base B at
the point u in the space Cp .X; D/ such that jBj  . Taking smaller neighborhoods
of u if necessary we can assume that all elements of B belong to the standard base
of the space Cp .X; D/, i.e., for any B 2 B there is a finite set KB  X such that
B D ff 2 Cp .X;SD/ W f .KB /  f0gg.
The set Y D fKB W B 2 Bg has cardinality at most ; if Y X then pick a
point x 2 X nY . The set U D ff 2 Cp .X; D/ W f .x/ D 0g is an open neighborhood
of u in Cp .X; D/. Therefore there is B 2 B with B  U ; however, x KB ,
so zero-dimensionality of X shows that there is a clopen set G  X such that
x 2 G  X nKB . Let f .y/ D 1 for all y 2 G and f .y/ D 0 whenever y 2 X nG.
It is straightforward that f 2 BnU ; this contradiction shows that Y D X and hence
jX j  . We already saw that this implies .Cp .X; D// D w.Cp .X; D// D jX j.
V.051. Prove that a zero-dimensional compact space X is scattered if and only if
Cp .X; D/ is FrchetUrysohn.
Solution. Recall that an !-cover of a space Z is a family U  exp.Z/ such that,
for any finite A  Z, there is U 2 U with A  U . If Bn 2 exp.Z/ for each n 2 !
then Bn ! Z says that, for any z 2 Z, there is m 2 ! such that z 2 Bn for all
n  m.
If the space X is scattered then Cp .X / is FrchetUrysohn (see SFFS-134) and
hence Cp .X; D/  Cp .X / is also FrchetUrysohn; this proves necessity.
To prove sufficiency denote by Fin.X / the family of all finite subsets of X and
assume that Cp .X; D/ is a FrchetUrysohn space; take an open !-cover U of the
space X . For any finite K  X fix a set UK 2 U with K  UK and choose, using
zero-dimensionality of X , a clopen set OK such that K  OK  UK ; let hK .x/ D 1
for all x 2 OK and hK .x/ D 0 if x 2 X nOK . We will also need the function
u W X ! R such that u.x/ D 1 for all x 2 X ; it is straightforward that u 2 Cp .X; D/
belongs to the closure of the set P D fhK W K 2 Fin.X /g  Cp .X; D/. The space
Cp .X; D/ being FrchetUrysohn, we can choose a sequence fKn W n 2 !g such
that the sequence fhKn W n 2 !g converges to u; let Un D UKn for each n 2 !.
Given a point x 2 X the set G D ff 2 Cp .X; D/ W f .x/ D 1g is an open
neighborhood of u in Cp .X; D/, so there is m 2 ! such that hKn 2 G for all n  m.
This implies x 2 OKn  Un for all n  m and hence Un ! X . It turns out that

94

2 Solutions of problems 001500

any open !-cover U of X has a subfamily fUn W n 2 !g  U with Un ! X . This


implies that Cp .X / is a FrchetUrysohn space (see TFS-144), so we can apply
SFFS-134 once more to conclude that X is scattered and complete our solution.
V.052. Suppose that X is not  -compact and w.X /  . Prove that there is a
subspace Z  Cp .X / such that jZj   and Z is not embeddable into Cp .Y /
for any  -compact Y . In particular, there is a countable subspace of Cp .P/ which
cannot be embedded into Cp .Y / for any  -compact Y .
Solution. Choose a base B of the space X such that jBj   and call a pair p D
.U; V / 2 B  B adequate if U  V and there is a function fp 2 C.X; 0; 1
/ for
which fp .U /  f0g and fp .X nV /  f1g; let Wp D V and denote by P the set of
all adequate pairs of elements of B.
It is evident that the family P D fQ W Q  P is finiteQand the collection
fWq W q 2 Qg is disjointg has cardinality at most ; let hQ D ffq W q 2 Qg for
any Q 2 P. The set Z D fhQ W Q 2 Pg [ f0X g also has cardinality  ; we claim
that Z is D-separating in Cp .X /.
To prove this take a finite K  X and a closed set F  X nK. It is easy to find
a finite disjoint family B 0 D fBx W x 2 Kg  B such that x 2 Bx  X nF for
every x 2 K. Using the Tychonoff property of X choose, for any x 2 K, a function
gx 2 C.X; 0; 1
/ for which gx .x/ D 0 and gx .X nBx /  f1g; pick a set Cx 2 B
such that x 2 Cx  gx1 .0; 12 //. There exists a continuous function ' W 0; 1
!
0; 1
such that '.0; 12
/ D f0g and '.1/ D 1. Consequently, .' gx /.Cx /  f0g
and .' gx /.X nBx /  f1g which shows that every px D .Cx ; Bx / is an adequate
pair and hence the set Q D fpx W x 2 Kg belongs to P.
Since fpx .X nBx /  f1g and F  X nBx , we have fpx .F /  f1g for any x 2 K;
as a consequence, hQ .F /  f1g. Furthermore, fpx .x/ D 0 for any x 2 K, so
hQ .K/  f0g; recalling that hQ 2 Z we convince ourselves that Z is indeed, a
D-separating family. If Z embeds in the space Cp .Y / for some  -compact Y then
X 2 K.Y / (see Problem 037) and hence X is  -compact by Problem 039 which is
a contradiction. Thus Z  Cp .X / is a set of cardinality at most  which cannot be
embedded in Cp .Y / for any  -compact space Y .
V.053. Prove that, for any X , the space Cp .X / embeds in Cp .P/ if and only if
Cp .X / is homeomorphic to a linear subspace of Cp .P/.
Solution. Since sufficiency is evident, assume that Cp .X /! Cp .P/. Then X is
analytic by Problem 044, so let ' W P ! X be a continuous onto map. The dual
map '  W Cp .X / ! Cp .P/ is an embedding (see TFS-163) and it is straightforward
that '  .Cp .X // is a linear subspace of Cp .P/.
V.054. Suppose that a space X is compact and there exists a homeomorphism ' W
RX ! RY such that '.Cp .X //  Cp .Y /. Prove that Y is compact.
Solution. It follows from '.Cp .X //  Cp .Y / that Cp .Y / is embeddable in the
space Cp .X /, so Y is  -compact by Problem 043. The space X being compact,
there exists a  -compact P  RX such that Cp .X /  P (see CFS-204); then
P 0 D '.P /  RY is a  -compact space such that Cp .Y /  P 0 . Apply CFS-204
again to conclude that Y is pseudocompact and hence compact.

2 Solutions of problems 001500

95

V.055. Suppose that p; q W X ! exp.Y / are finite-valued mappings such that q is


lower semicontinuous withSrespect to p. Prove that, for any nonempty set A  X ,
the map qjA
S W A ! exp. q.A// is lower semicontinuous with respect to pjA W
A ! exp. q.A//.
S
Solution. Let Z D
q.A/ and denote the mappings pjA W A ! exp.Z/ and
qjA W A ! exp.Z/ by pA and qA respectively. Given a set U 2 .Z/ there is
U 0 2 .Y / such that U D U 0 \ Z; observe that, for any a 2 A, we have q.a/  Z,
so q.a/ \ U 0 ; implies q.a/ \ U ;. The set ql1 .U 0 / being a neighborhood
of the set pl1 .U 0 / in X , for any a 2 A with p.a/ \ U 0 ;, there is Ua 2 .a; X /
such that q.x/ \ U 0 ; for any x 2 Ua .
Now, if we are given a point a 2 .pA /1
l .U / then pA .a/ \ U D p.a/ \ U ;
and hence p.a/ \ U 0 ;. The set Va D Ua \ A is an open neighborhood of a in A;
for any point x 2 Va we have q.x/\U 0 ; and hence qA .x/\U D q.x/\U ;
1
by the above observation.
This shows that Va  .qA /1
l .U / for any a 2 .pA /l .U /
S
1
and hence the set fVa W a 2 .pA /l .U /g is an open neighborhood (in A) of
1
.pA /1
l .U / contained in .qA /l .U /, i.e., qA is lower semicontinuous with respect
to pA .
V.056. Suppose that p W X ! exp.Y / and q W X ! exp.Y / are finite-valued
mappings such that q is lower semicontinuous with respect to p. Given an open
set U  Y , let pU .x/ D p.x/ \ U and qU .x/ D q.x/ \ U for every x 2 X .
Prove that the map qU W X ! exp.U / is lower semicontinuous with respect to
pU W X ! exp.U /.
Solution. Take a set V 2 .U /; then V is open in Y , so the set ql1 .V / is
a neighborhood of the set pl1 .V /. This shows that, for any x 2 X such that
Ox .
p.x/ \ V ;, there
S is Ox 2 .x; X / such that q.y/ \ V ; for each y 2 1
The set O D fOx W x 2 .pU /1
.V
/g
is
an
open
neighborhood
of
.p
/
U l .V /.
l
If y 2 O then y 2 Ox for some x 2 .pU /1
.V
/;
we
have
p.y/
\
V
D
pU .y/ \
l
V ;, so q.y/ \ V ; and hence qU .y/ \ V ; because V  U . Thus
1
1
O  .qU /1
l .V /, i.e., .qU /l .V / is a neighborhood of .pU /l .V / which shows
that qU is lower semicontinuous with respect to pU .
V.057. Given a number n 2 N and spaces Xi ; Yi ; for every i < n suppose that
pi ; qi W Xi ! exp.Yi / are set-valued
maps such that qi is lower semicontinuous
Q
with
respect
to
p
.
Prove
that
q
i
i<n i is lower semicontinuous with respect to
Q
p
.
As
a
consequence,
any
finite
product of almost lower semicontinuous maps
i
i<n
is an almost lower semicontinuous map.
Q
Q
Q
Solution. Let p D i<n pi and q D i<n qi ; fix an open set U  i<n Yi and a
point a D .a0 ; : : : ; an1 / 2 pl1 .U /. Since p.a/ \ U ;, we can find Ui 2 .Yi /
for any i < n such that U 0 D U0 : : : Un1  U and p.a/\U 0 ;. This implies
that, for every i < n, we have p.ai /\Ui ; and hence there exists Wi 2 .ai ; Xi /
such that qi .y/ \ Ui ; for any y 2 Wi .
Now, if W .a/ D W0  : : :  Wn1 then, for any b D .b0 ; : : : ; bn1 / 2 W .a/,
the set q.b/ meets U 0  U and hence q.b/ \ U ; which shows that W .a/ 

96

2 Solutions of problems 001500

S
ql1 .U /. Thus the set W D fW .a/ W a 2 pl1 .U /g is an open Q
neighborhood of
pl1 .U /; since also W  ql1 .U /, we
proved
that
the
map
q
D
i<n qi is lower
Q
semicontinuous with respect to p D i<n pi .
V.058. Suppose that, for finite-valued mappings p; r; q W X ! exp.Y /, we have
p.x/  r.x/  q.x/ for any x 2 X . Prove that, if q is lower semicontinuous with
respect to r then q is lower semicontinuous with respect to p.
Solution. If U 2 .Y / then ql1 .U / is a neighborhood of the set rl1 .U /; since
also pl1 .U /  rl1 .U /, the set ql1 .U / is a neighborhood of pl1 .U /, so q is lower
semicontinuous with respect to p.
V.059. Suppose that X is a nonempty spaceS and p W X ! exp.Y / is an
almost lower semicontinuous map such that
p.X / D Y . Prove that Y is a
countable union of images of subspaces of X under single-valued almost lower
semicontinuous maps of finite defect.
Solution. Fix a space Z  Y and a finite-valued map q W X ! exp.Z/ such
that q is lower semicontinuous with respect to p. For any m; n 2 S
N consider the
set Xmn S
D fx 2 X W jp.x/jSD m and jq.x/j D ng; if Ymn D
p.Xmn / and
Zmn D q.Xmn / then Y D fYmn W m; n 2 Ng. Apply Problem 055 to see that,
for any m; n 2 N, the map qjXmn W Xmn ! exp.Zmn / is lower semicontinuous with
respect to pjXmn W Xmn ! exp.Ymn /, so it suffices to prove our statement for every
pjXmn . Thus we can assume, without loss of generality, that there exist m; n 2 N
such that jp.x/j D m and jq.x/j D n for each x 2 X .
Fix a faithful enumeration fp 1 .x/; : : : ; p m .x/g of every set p.x/; this gives a
single-valued map p i W X ! Y ; let Yi D p i .X / for any i 2 f1; : : : ; mg.
By Problem 058, the map q is lower semicontinuous with respect to every p i W
X ! Yi ; since jq.x/j D n for any point xS
2 X , the defect of the map p i does not
exceed n  1. Finally, it follows from Y D fYi W i 2 f1; : : : ; mgg that we obtained
the promised representation of the space Y .
V.060. Given nonempty spaces X; Z and k 2 N, suppose that we have maps p W
X ! Z and q W X ! exp.Z/ such that p.x/ 2 q.x/ and jq.x/j  k for each
x 2 X while q.x/ \ q.y/ D ; if x y and q is S
lower semicontinuous with respect
to p. Prove that, if p.X / is discrete then X D j <k Dj where every Dj  X is
discrete and hence there exists a discrete subspace S  X such that jS j D jX j.
Solution. If Z is a space then let Z .0/ D Z and I0 .Z/ D ;. If n 2 ! and we have
a set Z .n/  Z, let InC1 .Z/ be the set of all isolated points of the space Z .n/ and
Z .nC1/ D Z .n/ nInC1 .Z/. If Z .n/ D ; for some n 2 ! then we say that dispersion
index of Z does not exceed n. If the dispersion index of the space Z does not exceed
n then, evidently, Z can be represented as the union of at most n discrete subspaces.
If X is finite then it is discrete and hence the sets D0 D : : : D Dk1 D S D X
do the job; this shows that we can assume, without loss of generality, that X is
infinite. Given an open set U  X , a point x 2 U is isolated in U if and only it is
isolated in X . Using this fact it is easy to prove by induction that U .n/ D X .n/ \ U
for any n 2 !. This implies that

2 Solutions of problems 001500

97

(1) if U \ X .nC1/ ; for some n 2 ! then the set U \ X .n/ is infinite.


For any x 2 X fix a set Ox 2 .p.x/; Z/ such that O x \ p.X / D fp.x/g.
It suffices to show that X .k/ D ;, so assume toward a contradiction, that there is a
point x0 2 X .k/ . Let V0 D Ox0 ; since q is lower semicontinuous with respect to p,
there is a set U0 2 .x0 ; X / such that q.y/ \ V0 ; for any y 2 U0 . Proceeding
by induction suppose that m < k and we have points x0 ; : : : ; xm 2 X and sets
U0 ; V0 ; : : : ; Um ; Vm with the following properties:
(2)
(3)
(4)
(5)
(6)

xi 2 X .ki/ nfx0 ; : : : ; xi1 g for any i  m;


Vi 2 .p.xi /; Z/ and V i \ p.X / D fp.xi /g for any i  m;
the family fV 0 ; : : : ; V m g is disjoint;
if i  m then Ui 2 .xi ; X / and q.y/ \ Vi ; for any y 2 Ui ;
U0  : : :  Um .

It follows from the properties (1) and (2) that the set Um \ X .km1/ is infinite,
so we can choose a point xmC1 2 X .km1/ nfx0 ; : : : ; xm g. The map p is easily
seen to be injective, so it follows from (3) that p.xmC1 / G D V 0 [ : : : [ V m .
Choose a set VmC1 2 .p.xmC1 /; Z/ such that VmC1  OxmC1 and V mC1 \ G D ;.
The map q being lower semicontinuous with respect to p, we can choose a set
UmC1 2 .xmC1 ; X / such that UmC1  Um and q.y/\VmC1 ; for any y 2 UmC1 .
It is straightforward that the conditions (2)(6) are still satisfied if we substitute m
by mC1, so our inductive procedure can be continued to construct points x0 ; : : : ; xk
and sets U0 ; V0 ; : : : ; Uk ; Vk for which the properties (2)(6) hold for any i  k.
It follows from (5) and (6) that q.xk / \ Vi ; for any i D 0; : : : ; k; an
immediate consequence of (4) is that jq.xk /j  k C 1; this contradiction shows
that X .k/ D ;, so we can let Dj D Ij .X / for any j < k. Since X is infinite, some
Dj has the same cardinality as X ; letting S D Dj we obtain a discrete subspace
S  X such that jS j D jX j.
V.061. Given nonempty spaces X; Z and k 2 N, suppose that we have maps p W
X ! Z and q W X ! exp.Z/ such that p.x/ 2 q.x/ and jq.x/j  k for each
x 2 X while q.x/ \ q.y/ D ; if x y and q is lower
S semicontinuous with respect
to p. Prove that, if p.X / is left-separated then X D j <k Dj where every Dj  X
is left-separated and hence there exists a left-separated subspace S  X such that
jS j D jX j.
Solution. If Z is a space and < is a well-order on Z say that a point z 2 Z is
left-separated in Z if there exists U 2 .z; Z/ such that z is the minimal point
of U , i.e., x  y for any y 2 U . Let Z .0/ D Z and L0 .Z/ D ;. If n 2 !
and we have a set Z .n/  Z, let LnC1 .Z/ D fz 2 Z .n/ W z is left-separated in
Z .n/ g and Z .nC1/ D Z .n/ nLnC1 .Z/. If Z .n/ D ; for some n 2 ! then we say
that left-separation index of Z does not exceed n. It is clear that every Li .Z/ is a
left-separated space with respect to the order inherited from Z; this shows that if the
left-separation index of the space Z does not exceed n then Z can be represented as
the union of at most n left-separated subspaces.

98

2 Solutions of problems 001500

If the space X is finite then it is discrete and hence left-separated, so the sets
D0 D : : : D Dk1 D S D X do the job; this shows that we can assume,
without loss of generality, that X is infinite. Let < be a well-order on p.X / which
witnesses that p.X / is left-separated. It is straightforward that p W X ! p.X / is a
bijection, so we obtain a well-order on X if we declare that x y if and only if
p.x/ < p.y/.
Given an open set U  X , a point x 2 U is left-separated in U if and only it
is left-separated in X . Using this fact it is easy to prove by induction that U .n/ D
X .n/ \ U for any n 2 !. This implies that
(1) if n 2 ! and x 2 U \ X .nC1/ then the set fy 2 U \ X .n/ W y xg is infinite.
For any x 2 X fix a set Ox 2 .p.x/; Z/ such that O x \ p.X / does not meet the
set fy 2 p.X / W y < p.x/g and hence p.x/ is the minimal element of O x \ p.X /.
It suffices to show that X .k/ D ;, so assume toward a contradiction, that there is a
point x0 2 X .k/ . Let V0 D Ox0 ; since q is lower semicontinuous with respect to p,
there is a set U0 2 .x0 ; X / such that q.y/ \ V0 ; for any y 2 U0 . Proceeding
by induction suppose that m < k and we have points x0 ; : : : ; xm 2 X and sets
U0 ; V0 ; : : : ; Um ; Vm with the following properties:
(2) x0 : : : xm and xi 2 X .ki/ for any i  m;
(3) Vi 2 .p.xi /; Z/ and p.xi / is the minimal element of V i \ p.X / for any
i  m;
(4) the family fV 0 ; : : : ; V m g is disjoint;
(5) if i  m then Ui 2 .xi ; X / and q.y/ \ Vi ; for any y 2 Ui ;
(6) U0  : : :  Um .
It follows from (1) and (2) that the set fy 2 Um \ X .km1/ W y xm g is infinite,
so we can choose a point xmC1 2 X .km1/ \ Um such that xmC1 xm . Since p
is injective, it follows from the property (3) that p.xmC1 / G D V 0 [ : : : [ V m .
Choose a set VmC1 2 .p.xmC1 /; Z/ such that VmC1  OxmC1 and V mC1 \ G D
;. Since q is lower semicontinuous with respect to p, we can find a set UmC1 2
.xmC1 ; X / such that UmC1  Um and q.y/ \ VmC1 ; for any y 2 UmC1 . It is
straightforward that the conditions (2)(6) are still satisfied if we substitute m by
m C 1, so our inductive procedure can be continued to construct points x0 ; : : : ; xk
and sets U0 ; V0 ; : : : ; Uk ; Vk for which the properties (2)(6) hold for any i  k.
It follows from (5) and (6) that q.xk / \ Vi ; for any i D 0; : : : ; k; an
immediate consequence of (4) is that jq.xk /j  k C 1; this contradiction shows
that X .k/ D ;, so we can let Dj D Lj .X / for any j < k. Since X is infinite,
some Dj has the same cardinality as X ; letting S D Dj we obtain a left-separated
subspace S  X such that jS j D jX j.
V.062. Given nonempty spaces X; Z and k 2 N, suppose that we have maps p W
X ! Z and q W X ! exp.Z/ such that p.x/ 2 q.x/ and jq.x/j  k for each
x 2 X while q.x/ \ q.y/ D ; if x y and q is lower semicontinuous
with
S
respect to p. Prove that, if p.X / is right-separated then X D j <k Dj where
every Dj  X is right-separated and hence there exists a right-separated subspace
S  X such that jS j D jX j.

2 Solutions of problems 001500

99

Solution. If Z is a space and < is a well-order on Z say that a point z 2 Z is


right-separated in Z if there exists U 2 .z; Z/ such that z is the maximal point
of U , i.e., y  x for any y 2 U . Let Z .0/ D Z and M0 .Z/ D ;. If n 2 !
and we have a set Z .n/  Z, let MnC1 .Z/ D fz 2 Z .n/ W z is right-separated in
Z .n/ g and Z .nC1/ D Z .n/ nMnC1 .Z/. If Z .n/ D ; for some n 2 ! then we say
that right-separation index of Z does not exceed n. It is clear that every Mi .Z/
is a right-separated space with respect to the order inherited from Z; this shows
that if the right-separation index of the space Z does not exceed n then Z can be
represented as the union of at most n right-separated subspaces.
If the space X is finite then it is discrete and hence right-separated, so the sets
D0 D : : : D Dk1 D S D X do the job; this shows that we can assume, without
loss of generality, that X is infinite. Let < be a well-order on p.X / which witnesses
that p.X / is right-separated. It is straightforward that p W X ! p.X / is a bijection,
so we obtain a well-order on X if we declare that x y if and only if p.x/ <
p.y/.
Given an open set U  X , a point x 2 U is right-separated in U if and only it
is right-separated in X . Using this fact it is easy to prove by induction that U .n/ D
X .n/ \ U for any n 2 !. This implies that
(1) if n 2 ! and x 2 U \ X .nC1/ then the set fy 2 U \ X .n/ W x yg is infinite.
For any x 2 X fix a set Ox 2 .p.x/; Z/ such that O x \ p.X / does not meet the
set fy 2 p.X / W p.x/ < yg and hence p.x/ is the maximal element of O x \ p.X /.
It suffices to show that X .k/ D ;, so assume toward a contradiction that there is a
point x0 2 X .k/ . Let V0 D Ox0 ; since q is lower semicontinuous with respect to p,
there is a set U0 2 .x0 ; X / such that q.y/ \ V0 ; for any y 2 U0 . Proceeding
by induction suppose that m < k and we have points x0 ; : : : ; xm 2 X and sets
U0 ; V0 ; : : : ; Um ; Vm with the following properties:
(2) x0 : : : xm and xi 2 X .ki/ for any i  m;
(3) Vi 2 .p.xi /; Z/ and p.xi / is the maximal element of V i \ p.X / for any
i  m;
(4) the family fV 0 ; : : : ; V m g is disjoint;
(5) if i  m then Ui 2 .xi ; X / and q.y/ \ Vi ; for any y 2 Ui ;
(6) U0  : : :  Um .
It follows from (1) and (2) that the set fy 2 Um \ X .km1/ W xm yg is
infinite, so we can choose a point xmC1 2 X .km1/ \ Um such that xm xmC1 .
Since p is injective, it follows from the property (3) that p.xmC1 / G D V 0 [
: : : [ V m . Choose a set VmC1 2 .p.xmC1 /; Z/ such that VmC1  OxmC1 and
V mC1 \ G D ;. Since q is lower semicontinuous with respect to p, we can find
a set UmC1 2 .xmC1 ; X / such that UmC1  Um and q.y/ \ VmC1 ; for any
y 2 UmC1 . It is straightforward that the conditions (2)(6) are still satisfied if we
substitute m by m C 1, so our inductive procedure can be continued to construct
points x0 ; : : : ; xk and sets U0 ; V0 ; : : : ; Uk ; Vk for which the properties (2)(6) hold
for any i  k.

100

2 Solutions of problems 001500

It follows from (5) and (6) that q.xk / \ Vi ; for any i D 0; : : : ; k; an


immediate consequence of (4) is that jq.xk /j  k C 1; this contradiction shows
that X .k/ D ;, so we can let Dj D Mj .X / for any j < k. Since X is infinite,
some Dj has the same cardinality as X ; letting S D Dj we obtain a right-separated
subspace S  X such that jS j D jX j.
V.063. Suppose that an infinite space Y is an image of a space X under a
finite-valued almost lower semicontinuous map, i.e., there
S exists an almost lower
semicontinuous map p W X ! exp.Y / such that Y D p.X /. Prove that for each
n 2 N, we have s.Y n /  s.X n /, hl.Y n /  hl.X n / and hd.Y n /  hd.X n /.
Solution. Assume for a moment that
. / for any infinite spaces Z and T , ifS
there exists an almost lower semicontinuous
map ' W Z ! exp.T / such that '.Z/ D T then s.T /  s.Z/, hd.T / 
hd.Z/ and hl.T /  hl.Z/.
Q
For any n 2 N and x D .x0 ; : : : ; xn1 / 2 X n let p n .x/ D i<n p.xi /. Then
pn W X n S
! exp.Y n / is an almost lower semicontinuous map (see Problem 057)
such that p n .X n / D Y n . This shows that, letting Z D X n ; T D Y n ; ' D p n
and applying . /, we obtain the promised inequalities for every n 2 N. Therefore it
suffices to establish . /.
Since spread, hereditary density, and hereditary Lindelf number are hereditary
and countably additive, we can apply Problem 059 to see that we can assume,
without loss of generality, that ' is a single-valued almost lower semicontinuous
map of finite defect. Thus there exists a space H  T , a number n 2 N and a finitevalued map q W Z ! exp.H / such that jq.z/j  n for any z 2 Z and q is lower
semicontinuous with respect to '.
To deal with spread assume that s.Z/ D  and s.T / > ; therefore we can find
a discrete subspace D  T such that jDj D  C . It is easy to find a set E  Z
such that 'jE W E ! D is a bijection. Apply the Delta-lemma (SFFS-038) to
see that there exists a set E 0  E and a set K  Z such that jE 0 j D  C and
q.x/ \ q.y/ D K for any distinct x; y 2 E 0 .
Throwing away a finite subset of E 0 if necessary, we can assume, without loss
of generality, that '.E 0 / \ K D ;. Let r.x/ D q.x/nK for any x 2 E 0 ; it follows
from Problems 055 and 056 that r W E 0 ! Z is still lower semicontinuous with
respect to 'jE 0 . Since we also have r.x/ \ r.y/ D ; for distinct x; y 2 E 0 , we can
apply Problem 060 to conclude that there is a discrete subspace A  E 0 such that
jAj D jE 0 j D  C . This contradiction with s.Z/   shows that s.T /   D s.Z/.
To prove our inequality for hereditary density, assume that hd.Z/ D  and
hd.T / > ; then there exists a left-separated subspace D  T such that jDj D  C
(see SFFS-004). It is easy to find a set E  Z such that 'jE W E ! D is a bijection.
Apply the Delta-lemma to see that there exists a set E 0  E and a set K  Z such
that jE 0 j D  C and q.x/ \ q.y/ D K for any distinct x; y 2 E 0 .
As in the previous case, we can assume that '.E 0 /\K D ;. Let r.x/ D q.x/nK
for any x 2 E 0 ; it follows from Problems 055 and 056 that r W E 0 ! Z is still
lower semicontinuous with respect to 'jE 0 . Since we also have r.x/ \ r.y/ D ;

2 Solutions of problems 001500

101

for distinct x; y 2 E 0 , we can apply Problem 061 to conclude that there is a leftseparated subspace A  E 0 such that jAj D jE 0 j D  C . The obtained contradiction
with hd.Z/   shows that hd.T /   D hd.Z/.
To prove our inequality for hereditary Lindelf number assume that hl.Z/ D 
and hl.T / > ; therefore we can find a right-separated subspace D  T such that
jDj D  C (see SFFS-005). It is easy to find a set E  Z such that 'jE W E ! D
is a bijection. Apply the Delta-lemma to see that there exists a set E 0  E and a set
K  Z such that jE 0 j D  C and q.x/ \ q.y/ D K for any distinct x; y 2 E 0 .
As before, we can assume, without loss of generality, that '.E 0 / \ K D ;. Let
r.x/ D q.x/nK for any x 2 E 0 ; then r W E 0 ! Z is still lower semicontinuous
with respect to 'jE 0 . Since we also have r.x/ \ r.y/ D ; for distinct x; y 2 E 0 , we
can apply Problem 062 to conclude that there is a right-separated subspace A  E 0
such that jAj D jE 0 j D  C which is a contradiction with hl.Z/  . This shows
that hl.T /   D hl.Z/ finishing the proof of . / and completing our solution.
V.064. Let h W Cp .Y / ! Cp .X / be an embedding such that h.0Y / D 0X . Given
x 2 X and " > 0, a point y 2 Y is called "-inessential for x if there is U 2 .y; Y /
such that jh.g/.x/j  " whenever g.Y nU /  f0g. The point y is "-essential for
x if it is not "-inessential for x. Denote by supp" .x/ the set of all points which are
"-essential for x. Prove that supp" .x/ is finite for any x 2 X and " > 0.
Solution. Given a set T let Fin.T / be the family of all finite subsets of T . If Z
is a space and A  Z then OZ .A; "/ D ff 2 Cp .Z/ W jf .z/j < " for any
z 2 Ag. It is evident that family fOZ .A; "/ W A 2 Fin.Z/; " > 0g is a local base
of Cp .Z/ at 0Z . If A D fx1 ; : : : ; xn g then we write OZ .x1 ; : : : ; xn ; "/ instead of
OZ .fx1 ; : : : ; xn g; "/.
Fact 1. Given spaces Z and T suppose that ' W Cp .T / ! Cp .Z/ is an embedding
such that '.0T / D 0Z . Assume also that z 2 Z; " > 0; K 2 Fin.T / and
'.OT .K; //  OZ .z; "/ for some > 0. Then supp" .z/  K and hence the set
supp" .z/ is finite.
Proof. If t 2 T nK then U D T nK is an open neighborhood of the point z; assume
that f .T nU / D f .K/  f0g for some f 2 Cp .T /. Then f 2 OT .K; / and hence
'.f / 2 OZ .z; "/, i.e., j'.f /.z/j < ". Thus the set U witnesses that t is "-inessential
for z. Since t 2 T nK was chosen arbitrarily, we established that no point of T nK
can be "-essential for z, so supp" .z/  T and Fact 1 is proved.
Returning to our solution, fix a point x 2 X and " > 0. Since OX .x; "/ is an open
neighborhood of 0X in Cp .X /, by continuity of h, there is a finite K  Y and > 0
such that h.OY .K; //  OX .z; "/. Now apply Fact 1 to see that supp" .x/  K and
hence supp" .x/ is finite.
V.065. Let h W Cp .Y / ! Cp .X / be an embedding such that h.0Y / D 0X .
Denote
S by supp" .x/ the set of all points which are "-essential for x. Prove that
Y D fsupp1=n .x/ W x 2 X; n 2 Ng.

102

2 Solutions of problems 001500

Solution. Given a set T let Fin.T / be the family of all finite subsets of T . If Z
is a space and A  Z then OZ .A; "/ D ff 2 Cp .Z/ W jf .z/j < " for any z 2
Ag. It is evident that family fOZ .A; "/ W A 2 Fin.Z/; " > 0g is a local base
of Cp .Z/ at 0Z . If A D fx1 ; : : : ; xn g then we write OZ .x1 ; : : : ; xn ; "/ instead of
OZ .fx1 ; : : : ; xn g; "/.
Fix y 2 Y ; since h.OY .y; 1// is an open neighborhood of 0X in h.Cp .Y //, there
is K 2 Fin.X / and " > 0 such that OX .K; "/ \ h.Cp .Y //  h.OY .y; 1//. Choose
n 2 N such that n1 < "; we claim that y 2 supp1=n .x/ for some x 2 K.
Indeed, if this is not so then, for any x 2 K,
T there is Ux 2 .y; Y / which
witnesses that y is n1 -inessential for x; let U D fUx W x 2 Kg. Take a function
f 2 Cp .Y / such that f .y/ D 1 and f .Y nU /  f0g. Then f .Y nUx /  f0g and
hence jh.f /.x/j  n1 < " for any x 2 K. This shows that h.f / 2 OX .K; "/ \
h.Cp .Y // and hence h.f / 2 h.OY .y; 1//. The map h being injective we have
f 2 OY .y; 1/, i.e., jf .y/j < 1 which is a contradiction. Thus y is n1 -essential
for
S some x 2 K; since the point y 2 Y was chosen arbitrarily, we proved that
fsupp1=n .x/ W x 2 X g D Y .
V.066. Let h W Cp .Y / ! Cp .X / be an embedding such that h.0Y / D 0X . Denote
by supp" .x/ the set of all points which are "-essential for x. Prove that, for any
" > 0, the finite-valued map supp" W X ! exp.Y / is almost lower semicontinuous.
Solution. Given a set T let Fin.T / be the family of all finite subsets of T . If Z
is a space and A  Z then OZ .A; "/ D ff 2 Cp .Z/ W jf .z/j < " for any z 2
Ag. It is evident that family fOZ .A; "/ W A 2 Fin.Z/; " > 0g is a local base
of Cp .Z/ at 0Z . If A D fx1 ; : : : ; xn g then we write OZ .x1 ; : : : ; xn ; "/ instead of
OZ .fx1 ; : : : ; xn g; "/.
Fix " > 0; for any x 2 X there is a finite set Kx  Y and x > 0 such that
h.OY .Kx ; x //  OX .x; "/. Let q.x/ D Kx for any x 2 X ; then q W X ! exp.Y /
is a finite-valued map. By Fact 1 of V.064, we have supp" .x/  Kx D q.x/ for
any x 2 X , so we only need to prove that q is lower semicontinuous with respect to
supp" .
To do so take an arbitrary set U 2 .Y / and consider a point x 2 X such that
supp" .x/ \ U ;. Pick a point y 2 supp" .x/ \ U ; since y is "-essential for x,
there is a function f 2 Cp .Y / such that f .Y nU /  f0g and jh.f /.x/j > ". The
set Wx D fz 2 X W jh.f /.z/j > "g is an open neighborhood of x in X . If z 2 Wx
and q.z/ \ U D ; then f .q.z//  f0g; recalling that q.z/ D Kz we conclude
that f 2 OY .Kz ; z / and therefore h.f / 2 OX .z; "/, i.e., jh.f /.z/j < " which
contradicts z 2 Wx . Thus q.z/ \ U ; and hence z 2 ql1 .U / for any z 2 Wx
which shows that Wx  ql1 .U /. An immediate consequence is that
.supp" /1
l .U / 

[
1
fWx W x 2 .supp" /1
l .U /g  ql .U /I

this proves that ql1 .U / is a neighborhood of .supp" /1


l .U /, i.e., q is lower semicontinuous with respect to supp" and therefore supp" is almost lower semicontinuous.

2 Solutions of problems 001500

103

V.067. Suppose that Cp .Y / embeds in Cp .X /. Prove that Y is a countable union


of images of X under finite-valued almost lower semicontinuous maps.
Solution. Using homogeneity of the space Cp .X / it is easy to find an embedding
h W Cp .Y / ! Cp .X / such that h.0Y / D 0X . For any " > 0 and x 2 X let supp" .x/
be the set of points of the space Y which are "-essential for x. Given n 2 N let
pn .x/ D supp1=n .x/ for any x 2 X . Then every pn W X ! exp.Y / is an almost
lower semicontinuous
finite-valued map (see Problems 064 and S
066) and the space
S
Yn D pn .X /  Y is the image of X under pn . Since Y D fYn W n 2 Ng by
Problem 065, the space Y is a countable union of images of X under finite-valued
almost lower semicontinuous maps.
V.068. Suppose that Cp .Y / embeds in Cp .X /. Prove that, for any n 2 N, we have
s.Y n /  s.X n /. In particular, if X and Y are t -equivalent then s.Y n / D s.X n / for
any n 2 N.
Solution. Fix n 2 N and let s.X n / D ; there is a sequence fpm W m 2 !g
such that S
every
S pm W X ! exp.Y / is a finite-valued almost lower semicontinuous
S
map and f pm .X / W m 2 !g D Y (see Problem 067). Let Ym D
pm .X /
for each m 2 !; for any n-tuple  D .m0 ; : : : ; mn1 / 2 ! n consider the space
Y D Ym0 Q: : :  Ymn1 and a finite-valued map q W X n ! exp.Y / defined
by q .x/ D i<n pmi .xi / for any x D .x0 ; : : : ; xn1 / 2SX n . Every q is almost
lower semicontinuous by Problem 057; since also Y D
q .X n /, we can apply
n
Problem 063 to see that s.Y /   for any  2 ! . Finally observe that Y n D
S
fY W  2 ! n g; the spread being countably additive, we conclude that s.Y n / 
 D s.X n /.
V.069. Suppose that Cp .Y / embeds in Cp .X /. Prove that, for any n 2 N, we have
hd.Y n /  hd.X n /. In particular, if the spaces X and Y are t -equivalent then
hd.Y n / D hd.X n / for any n 2 N.
Solution. Fix n 2 N and let hd.X n / D ; there is a sequence fpm W m 2 !g
such that S
every
S pm W X ! exp.Y / is a finite-valued almost lower semicontinuous
S
map and f pm .X / W m 2 !g D Y (see Problem 067). Let Ym D
pm .X /
for each m 2 !; for any n-tuple  D .m0 ; : : : ; mn1 / 2 ! n consider the space
Y D Ym0 Q: : :  Ymn1 and a finite-valued map q W X n ! exp.Y / defined
by q .x/ D i<n pmi .xi / for any x D .x0 ; : : : ; xn1 / 2SX n . Every q is almost
lower semicontinuous by Problem 057; since also Y D
q .X n /, we can apply
n
Problem 063 to see that hd.Y /   for any  2 ! . Finally observe that Y n D
S
fY W  2 ! n g; the hereditary density being countably additive, we conclude that
hd.Y n /   D hd.X n /.
V.070. Suppose that Cp .Y / embeds in Cp .X /. Prove that, for any n 2 N, we
have hl.Y n /  hl.X n /. In particular, if the spaces X and Y are t -equivalent then
hl.Y n / D hl.X n / for any n 2 N.
Solution. Fix n 2 N and let hl.X n / D ; there is a sequence fpm W m 2 !g
such that every pm W X ! exp.Y / is a finite-valued almost lower semicontinuous

104

2 Solutions of problems 001500

S S
S
map and f pm .X / W m 2 !g D Y (see Problem 067). Let Ym D
pm .X /
for each m 2 !; for any n-tuple  D .m0 ; : : : ; mn1 / 2 ! n consider the space
Y D Ym0 Q: : :  Ymn1 and a finite-valued map q W X n ! exp.Y / defined
by q .x/ D i<n pmi .xi / for any x D .x0 ; : : : ; xn1 / 2SX n . Every q is almost
lower semicontinuous by Problem 057; since also Y D
q .X n /, we can apply
n
Problem 063 to see that hl.Y /   for any  2 ! . Finally observe that Y n D
S
fY W  2 ! n g; the hereditary Lindelf number being countably additive, we
conclude that hl.Y n /   D hl.X n /.
V.071. Suppose that f W X ! Y is a closed continuous onto map such that
t .Y /   and t .f 1 .y//   for any y 2 Y . Prove that t .X /  . Deduce from
this fact that, for every infinite compact space X , we have t .X n / D t .X / for any
n 2 N.
Solution. Given a space Z say that a set A  Z is -closed in Z if B  A for any
B  A with jBj  . It is straightforward that
(1) if A is -closed in Z then A \ F is -closed both in Z and in F for any closed
F  Z.
Now assume that A is a -closed subset of X . Let P D f .A/ and take any
Q  P such that jQj  . It is easy to find a set B  A for which jBj  
and f .B/ D Q. The map f being closed, we have Q D f .B/  f .A/ D P
and hence P is -closed in Y ; since t .Y /  , the set P is closed in Y , so we
proved that
(2) if A is -closed in X then f .A/ is closed in Y .
Suppose that a set A  X is -closed but not closed in X ; fix a point x 2 AnA
and let F D f 1 .f .x//. The set E D F \ A is -closed in F by (1); recalling
that t .F /  , we conclude that E is closed in F and hence in X . Choose a set
U 2 .x; X / such that U \ E D ;; then A0 D A \ U is -closed in X while
A0 \ F D ; and x 2 A0 . Finally observe that the set P D f .A0 / is closed in Y
by (2); since also x 2 A0 , we have f .x/ 2 P D P which is a contradiction with
A0 \ f 1 .f .x// D ;. This contradiction shows that every -closed subset of X is
closed in X , so t .X /   by Lemma from S.162.
Finally, assume that X is an infinite compact space and t .X /  . To prove by
induction that t .X n /  for any n 2 N suppose that k 2 N and we proved that
t .X k /  . Let  W X kC1 ! X be the projection onto the first factor of X kC1 . For
any x 2 X , the space  1 .x/ is homeomorphic to X k , so t . 1 .x//  by the
induction hypothesis; since the map  is closed, we can apply our above result to
see that t .X kC1 /  . Therefore t .X n /  t .X / for any n 2 N; since X embeds
in every X n , we also have the opposite inequality and hence t .X n / D t .X / for any
n 2 N.
V.072. Given countably compact sequential spaces X1 ; : : : ; Xm prove that the
space X1  : : :  Xm is countably compact and sequential.

2 Solutions of problems 001500

105

Solution. A space Z is called sequentially compact if every sequence in Z has a


convergent subsequence. Say that a set A  Z is sequentially closed in the space Z
if S  A for any convergent sequence S  A. It is straightforward that
(1) if A is sequentially closed in Z then A \ F is sequentially closed both in Z and
in F for any closed F  Z.
Fact 1. Any sequentially compact space is countably compact; if Z is countably
compact and sequential then Z is sequentially compact.
Proof. If a space Y is not countably compact then there is a countably infinite closed
discrete D  Y ; taking a faithful enumeration fdn W n 2 !g of the set D we obtain
a sequence in Y which has no convergent subsequence. This proves the first part of
our Fact.
To prove the second statement, suppose that Z is countably compact and
sequential and take a sequence S  Z; if the set S is finite then it is easy to extract a
constant (and hence convergent) subsequence from S . If S is infinite then, applying
Fact 4 of S.382 and passing, if necessary, to a subsequence of S we can assume,
without loss of generality, that S is an infinite discrete subspace of Z. Since Z is
countably compact, the set S cannot be closed; the space Z being sequential, there
is a sequence S 0  S which converges to a point x 2 ZnS . It is clear that S 0 is the
promised convergent subsequence of S , so Fact 1 is proved.
Fact 2. Any finite product of sequentially compact spaces is sequentially compact
and hence countably compact.
Proof. Suppose that the spaces Z1 ; : : : ; Zn are sequentially compact and take a
sequence S  Z D Z1  : : :  Zn . Denote by i W Z ! Zi the projection of
Z onto its i -th factor for every i  n. Use sequential compactness of the factors
to pass n times to a subsequence of S to finally obtain a sequence S 0  S such
that i .S 0 / converges to a point zi 2 Zi for any i 2 f1; : : : ; ng. It is easy to check
that the sequence S 0 converges to the point z D .z1 ; : : : ; zn /; this proves that Z is
sequentially compact and hence countably compact by Fact 1, so Fact 2 is proved.
Returning to our solution observe that an easy induction argument shows that
there is no loss of generality to assume that m D 2; thus it suffices to prove that
X D X1  X2 is countably compact and sequential. Let i W X ! Xi be the natural
projection of X onto Xi for every i 2 f1; 2g. Fact 1 and Fact 2 imply that the space
X is sequentially compact and hence countably compact.
Take a sequentially closed set A  X1  X2 ; if B D 1 .A/ is not sequentially
closed in X1 then fix a point t 2 X1 nB and a sequence T D ftn W n 2 !g 
B which converges to t . Choose a point sn 2 11 .tn / \ A for each n 2 !; by
sequential compactness of X some subsequence S 0 of the sequence S D fsn W n 2
!g converges to a point x 2 X . Then T 0 D 1 .S 0 / has to converge to 1 .x/; since
T 0 is a subsequence of T , it converges to t and therefore 1 .x/ D t . Consequently,
x A and hence S 0  A is a sequence which converges to a point outside of A.
This contradiction with the set A being sequentially closed proves that

106

2 Solutions of problems 001500

(2) the set 1 .A/ is sequentially closed and hence closed in X1 for any sequentially
closed A  X .
Now assume that A  X is a sequentially closed non-closed set and fix a point
x 2 AnA. The subspace F D 11 .1 .x// is closed in X and sequential being
homeomorphic to X2 , so the set E D A \ F is sequentially closed in F by (1); an
immediate consequence is that E is closed in F and hence in X .
Therefore we can take U 2 .x; X / such that U \ E D ;. Apply (1) again to see
that the set A0 D A \ U is sequentially closed in X ; it is evident that A0 \ F D ;.
Furthermore, we have x 2 A0 and hence t D 1 .x/ 2 1 .A0 /. However, 1 .A0 / is
closed in X1 by (2) which shows that 1 .x/ 2 1 .A0 / and hence 11 .1 .x// \ A0
;. This contradiction shows that every sequentially closed subset of X is closed
and hence X is sequential; we already saw that X is countably compact, so our
solution is complete.
V.073. Assuming that a space X maps continuously onto a compact space K prove
that t .K/  l.X / t .X /.
Solution. Given a space Z and a cardinal  say that a set A  Z is -closed in Z
if B  A for any B  A with jBj  . For the cardinal  D t .X / l.X / we must
prove that t .K/  ; fix a continuous onto map f W X ! K.
Fact 1. Suppose that Z is a space and is a cardinal such that l.Z/  and A is
compact for any A  Z with jAj  . Then Z is compact.
Proof. To obtain a contradiction assume that U is an open cover of Z which has
no finite subcover. It follows from l.Z/  that we can assume, without
S loss of
generality, that jU j  . For any finite V  U pick a point z.V/ 2 Zn. V/; then
the set A D fz.V/ W V is a finite subfamily of U g has cardinality at most and hence
F D A is compact. The
Sfamily U being an open cover
S of F , there exists a finite
V  U such that F  V. However, z.V/ 2 F n. V/; this contradiction shows
that Z is compact so Fact 1 is proved.
Returning to our solution, assume toward a contradiction that t .K/ > ; then
there is a non-closed set A  K which is -closed in K (see Lemma from S.162) and
hence P D clA .P / is compact for any P  A with jP j  . It is straightforward
that B D f 1 .A/ is -closed in X ; since t .X /  , the set B is closed in X . An
immediate consequence is that l.B/  l.X /  ; the set A being a continuous
image of B, we have l.A/  . This makes it possible to apply Fact 1 to conclude
that A is compact and hence closed in K; this contradiction shows that t .K/   D
l.X / t .X /.
S
V.074. Suppose that K is a compact space and K D n2! Kn where every Kn is
a sequential closed subspace of K. Prove that if either Martins Axiom or Luzins
Axiom (2!1 > 2! ) holds then the space K is sequential.
Solution. Given a space Z and A  Z the set A is sequentially closed in Z if
S  A for any convergent sequence S  A. It is easy to see that Z is sequential if
and only if every sequentially closed subset of Z is closed in Z.

2 Solutions of problems 001500

107

Assume toward a contradiction that there exists a sequentially closed non-closed


set A  K; observe that every set AnSD A \ Kn is sequentially closed and hence
closed in Kn . Therefore the set A D n2! An is  -compact. We assumed that A is
not compact, so it cannot be countably compact; fix a countably infinite set D  A
which is closed and discrete in A. The set F D D is compact and G D F nD 
KnA.
Since F is separable, we can apply Fact 2 of S.368 to see that w.F /  c. If Fn D
F \ Kn then w.Fn /  w.F /  c, so we can take a set Hn  Fn such that jHn j  c
and Fn D H n for any n 2 !. The space Kn  Fn being sequential, we have
jFn j  c (see Fact 2 of T.041) for each n 2 !, so jF j  ! c D c.
Now assume that Luzins Axiom holds. If .x; G/  !1 for any point x 2 G then
we can apply TFS-330 to see that jGj  2!1 > c which is contradiction. Therefore,
there is a point x 2 G with .x; G/  !. Since G is a G -subset of F , the point
x is also a G -subset of F (see Fact 2 of S.358), so .x; F /  ! by TFS-327. An
immediate consequence is that there is a sequence S  D which converges to x;
since x A, we obtained a contradiction which proves that K is sequential.
Finally assume that Martins Axiom takes place; since Luzins Axiom is already
taken care of and CH implies Luzins Axiom, we can assume that MAC:CH holds.
If .x; G/  c for any point x 2 G then we can apply TFS-330 to see that jGj 
2c > c which is contradiction. Therefore, there is a point x 2 G with .x; G/ < c.
Since .G; F / D .G; F /  !, we can apply Fact 2 of S.358 again to conclude
that .x; F / D .x; F / < c. Consequently, .x; fxg [ D/ < c and hence there is
a sequence S  D which converges to x (see SFFS-054); since x A, we again
obtained a contradiction which shows that K is sequential.
V.075. Suppose that we have a space X , a compact space
S K, and a compact-valued
upper semicontinuous map p W X ! exp.K/ such that p.X / D K. Knowing that
l.X / t .X /   and t .p.x//   for any x 2 X prove that t .K/  .
Solution. For the space Y D X  K let X W Y ! X and
S K W Y ! K be the
natural projections. We will next prove that the set Z D ffxg  p.x/ W x 2 X g
is closed in Y . To do so take an arbitrary point u D .a; b/ Z; then b p.a/
and hence there exist disjoint sets U 2 .p.a/; K/ and V 2 .b; K/. By upper
semicontinuity of p, the set W D fx 2 X W p.x/  U g is an open neighborhood of
a in X and hence O D W  V 2 .u; Y /. If .x; y/ 2 O then p.x/  U and y 2 V
which shows that y p.x/ and hence .x; y/ Z. Thus O \ Z D ; and we proved
that every u 2 Y nZ has an open neighborhood O in Y with O \ Z D ;. Therefore
Z is closed in Y .
Let H D fx 2 X W p.x/ D ;g; by upper semicontinuity of p the set H is open
in X , so X0 D X nH is closed in X and hence l.X0 /  l.X /  . The projection
X is perfect by Fact 3 of S.288, so the map ' D X jZ W Z ! X0 is perfect as well.
We have ' 1 .x/ D fxg  p.x/ ' p.x/, so t .' 1 .x//   for any x 2 X0 . Besides,
t .X0 /  t .X /  , so we can apply Problem 071 to see that t .Z/  . We also
have l.Z/   by Fact 5 of S.271; since the map K jZ W Z ! K is continuous and
onto, we can apply Problem 073 to conclude that t .K/  .

108

2 Solutions of problems 001500

V.076. Suppose that K is a compact sequential space and L is a compact space for
which
Sthere exists a finite-valued upper semicontinuous map p W K ! exp.L/ such
that p.K/ D L. Prove that L is also sequential.
Solution. Given a space Z say that A  Z is sequentially closed if S  A for any
convergent sequence S  A.
It turns out that the set F D f.a; b/ 2 K  L W b 2 p.a/g  K  L is closed
in K  L and hence compact. To see this take any point z D .a; b/ 2 .K  L/nF ;
then b p.a/ and hence there exist disjoint sets U 2 .b; L/ and V 2 .p.a/; L/.
By upper semicontinuity of p, the set W D fx 2 K W p.x/  V g is an open
neighborhood of a in K, so z 2 W  U 2 .z; K  L/. If t D .x; y/ 2 W  U then
y 2 U and p.x/  V which shows that y p.x/; this proves that .W V /\F D ;
and hence every point z 2 .K  L/nF has an open neighborhood disjoint from F .
This implies that the set .K  L/nF is open in K  L, so F is, indeed, closed in
K  L. Let K W K  L ! K and L W K  L ! L be the natural projections.
Our next step is to prove that
(1) for any sequentially closed set A  F the set K .A/ is sequentially closed and
hence closed in K.
Indeed, if the set B D K .A/ is not closed in K then there is a faithfully indexed
sequence S D fsn W n 2 !g  B which converges to a point x 2 KnB; let
S 0 D S [ fxg and choose a set On 2 .sn ; K/ such that On \ S D fsn g for
every n 2 !. Take a point tn 2 K1 .sn / \ A for every n 2 !. The sequence
T D ftn W n 2 !g is an infinite discrete subspace of F , so it cannot be closed
in F . Observe that Z D K1 .S 0 / \ F is a closed subset of F ; since K1 .s/ \ F
is finite for any s 2 S 0 , the set Z is countable. Being compact and countable, Z is
metrizable so there is a sequence T 0  T which converges to a point y 2 ZnT .
Since K1 .On / \ T D ftn g, the sequence T 0 cannot converge to a point of K1 .sn /
for any n 2 !. Thus K .y/ 2 SnS D fxg, i.e., K .y/ D x; as a consequence,
K .y/ K .A/ and therefore y 2 F nA. This gives a contradiction with the set A
being sequentially closed, so (1) is proved.
Now assume that A  F is sequentially closed and not closed in F ; fix a point
z 2 AnA. The set B D K .A/ is closed in K by (1). We have K .z/ 2 B D B, so
z 2 K1 .B/. Since z cannot be in the closure of the set E D K1 .K .z// \ A we
can find W 2 .z; K  L/ such that W \ E D ;. It is straightforward that the set
A0 D A\W is also sequentially closed; besides, z 2 A0 and hence K .z/ 2 K .A0 /.
However, K .A0 / is closed in K by (1); since K .z/ K .A0 /, it is impossible that
K .z/ 2 K .A0 /. This contradiction shows that every sequentially closed subset of
F is closed in F , i.e.,
S F is a sequential space.
It follows from p.K/ D L that L .F / D L, so L is a continuous (and hence
closed) image of the compact space K. Applying Fact 1 of S.224 we conclude that
L is also sequential, so our solution is complete.
V.077. Suppose that there exists an open continuous map of a subspace of Cp .X /
onto Cp .Y /. Prove that, for any n 2SNS
there is a finite-valued upper semicontinuous
map 'n W X n ! exp.Y / such that f 'n .X n / W n 2 Ng D Y .

2 Solutions of problems 001500

109

Solution. The following easy fact will be useful for this solution and future
references.
Fact 1. Given spaces Z; T and an open continuous onto map f W Z ! T , the map
fA D f jf 1 .A/ W f 1 .A/ ! A is open for any A  T .
Proof. It is clear that the map fA is continuous and onto. If U is an open subset of
f 1 .A/ then take U 0 2 .Z/ such that U 0 \ f 1 .A/ D U ; it is straightforward that
fA .U / D f .U / D f .U 0 / \ A, so fA .U / is open in A. Fact 1 is proved.
Returning to our solution observe that, by homogeneity of Cp .X / and Cp .Y /,
there exists a subspace C0  Cp .X / for which there is a continuous open onto map
0 W C0 ! Cp .Y / such that 0X 2 C0 and 0 .0X / D 0Y . Let C D 01 .Cp .Y; I//;
then the map  D 0 jC W C ! Cp .Y; I/ is open by Fact 1 and .0X / D 0Y . For any
function f 2 Cp .Y; I/ there is fQ 2 Cp .Y; I/ such that fQjY D f . Given points
x1 ; : : : ; xk 2 X and " > 0 let OX .x1 ; : : : ; xk ; "/ D ff 2 C W jf .xi /j < " for each
i  kg. It is clear that the family fOX .x1 ; : : : ; xk ; n1 / W k; n 2 N; x1 ; : : : ; xk 2 X g
is a local base at 0X in the space C .
Analogously, let OY .y1 ; : : : ; yk ; "/ D ff 2 Cp .Y; I/ W jfQ.yi /j < " for all i  kg
for any y1 ; : : : ; yk 2 Y and " > 0. A set OY .y1 ; : : : ; yk ; "/ is not necessarily open
in Cp .Y; I/; however, the family fOY .y1 ; : : : ; yk ; n1 / W k; n 2 N and y1 ; : : : ; yk 2 Y g
is a local base at 0Y in Cp .Y; I/. For any points y1 ; : : : ; yk 2 Y and " > 0 let
HY .y1 ; : : : ; yk ; "/ D ff 2 Cp .Y; I/ W jfQ.yi /j  " for all i  kg.
Consider, for every point x D .x1 ; : : : ; xn / 2 X n , the set
1
1
'n .x/ D fy 2 Y W .OX .x1 ; : : : ; xn ; //  HY .y; /g:
n
2
Our first observation is that
(1) the set 'n .x/ is finite and contained in Y for any x 2 X n .
To prove (1) recall that the map  is open, so there are y1 ; : : : ; ym 2 Y and " > 0
such that .OX .x1 ; : : : ; xn ; n1 //  OY .y1 ; : : : ; ym ; "/. If y 2 Y nfy1 ; : : : ; ym g then
there exists a function g 2 Cp .Y; I/ such that g.y/
Q
D 1 and g.yi / D 0 for each
i  m. Pick a function f 2 OX .x1 ; : : : ; xn ; n1 / with .f / D g. It follows from
g HY .y; 12 / that .OX .x1 ; : : : ; xn ; n1 // is not contained in HY .y; 12 / and hence
y 'n .x/. Thus 'n .x/  fy1 ; : : : ; ym g is a finite subset of Y , so (1) is proved.
To see that the map 'n W X n ! exp.Y / is upper semicontinuous fix a set U 2
.Y / and x D .x1 ; : : : ; xn / 2 X n with 'n .x/  U . Choose U 0 2 .Y / such
that U 0 \ Y D U . For any y 2 Y nU 0 we have y 'n .x/, so there is a function
fy 2 OX .x1 ; : : : ; xn ; n1 / such that j.fy /.y/j > 12 ; let gy D .fy / and Fy D
T
.gQ y /1 . 12 ; 12
/. Since y Fy for any y 2 Y nU 0 , the set fFy W y 2 Y nU 0 g
is contained in U 0 . By Fact 1 of S.326, there are y1 ; : : : ; yk 2 U nU 0 such that
Fy1 \ : : : \ Fyk  U 0 .
The set Wx D fz D .z1 ; : : : ; zn / 2 X n WSjfyi .zi /j < n1 for any i  kg is an open
neighborhood of x in X n ; we claim that f'n .z/ W z 2 Wx g  U . Indeed, take

110

2 Solutions of problems 001500

any z D .z1 ; : : : ; zn / 2 Wx ; if y 2 Y nU 0 then there is i 2 f1; : : : ; kg such that


y Fyi and hence jgQ yi .y/j > 12 . An immediate consequence is that the function
gyi D .fyi / 2 .OX .z1 ; : : : ; zn ; n1 //nHY .y; 12 / witnesses that y 'n .z/ and hence
0
'n .z/  U 0 ; the property (1) shows that 'S
n .z/  Y , so 'n .z/  Y \ U D U . Thus
1
n
.'n /u .U / D fx 2 X W 'n .x/  U g D fWx W x 2 U g is an open set and hence
the map 'n is upper semicontinuous.
Finally, for any y 2 Y the set OY .y; 12 / is an open neighborhood of 0Y in
Cp .Y; I/; the map  is continuous and .0X / D 0Y , so there are z1 ; : : : ; zm 2 X
and " > 0 such that .OX .z1 ; : : : ; zm ; "//  OY .y; 12 /. Take n 2 N such that
m  n and n1 < ". If x D .x1 ; : : : ; xn / 2 X n and fx1 ; : : : ; xn g  fz1 ; : : : ; zm g
thenSOX .x1 ; : : : ; xn ; n1 /  OX .z1 ; : : : ; zm ; "/ and hence y 2 'n .x/. This proves that
S
f 'n .X n / W n 2 Ng D Y and shows that our solution is complete.
V.078. Suppose that there is an open continuous map of a subspace of Cp .X / onto
Cp .Y /. Prove that t .K/  t  .X / l  .X / for any compact K  Y . Deduce from
this fact that if K and L are t -equivalent compact spaces then t .K/ D t .L/.
Solution. Let  D t  .X / l  .X /; there exists a subspace C  Cp .X / and an open
continuous onto map ' W C ! Cp .Y /. Given a compact subspace K  Y let  W
Cp .Y / ! Cp .K/ be the restriction map; since  is open continuous and surjective,
the map  ' W Cp .X / ! Cp .K/ is also open continuous and onto. Apply
Problem 077 to see that we can choose for any n 2 S
N,S
an upper semicontinuous
finite-valued map pn W X n ! exp.K/ such that K D f pn .X /L
W n 2 Ng.
e D fX n W n 2 Ng.
Identify every X n with the respective clopen subspace of X
e then there is a unique n 2 N such that x 2 X n ; let p.x/ D pn .x/.
If x 2 X
e ! exp.K/ is a finite-valued map and it follows from the choice of the
Then p W X
S
e D K.
sequence fpn W n 2 Ng that p.X/
n
.p.x/; K/; by upper semicontinuity of pn
Fix a point x 2 X and a set U 2S
e
thereSis a set V S
2 .x; X n / such that pn .V /  U . The set V is also open in X
and p.V / D pn .V /  U . This, together with Fact 1 of T.346, shows that the
e l.X
e / D t  .X / l  .X / D , we can
map p is upper semicontinuous. Since t .X/
apply Problem 075 to conclude that t .K/  .
Finally, if K and L are compact t -equivalent spaces then Cp .L/ is homeomorphic to Cp .K/ and hence it is an open continuous image of the space Cp .K/,
so t .L/  t  .K/ l  .K/ D t  .K/ D t .K/ (see Problem 071). Analogously,
t .K/  t .L/, so t .K/ D t .L/.
V.079. Suppose that X is a compact sequential space and Y is a compact space
such that there is an open continuous map of some subspace of Cp .X / onto Cp .Y /.
Prove that Y is a countable union of its compact sequential subspaces. As a
consequence, under Martins Axiom if K and L are compact t -equivalent spaces
and K is sequential then L is also sequential.
Solution. Apply Problem 077 to fix, for any n 2 N,
San
Supper semicontinuous finitevalued map pn S
W X n ! exp.Y / such that Y D f pn .X / W n 2 Ng. Consider
the space Yn D pn .X /; it is easy to see that pn W X n ! exp.Yn / is also an upper

2 Solutions of problems 001500

111

semicontinuous map, so we can apply SFFS-241 to convince ourselves that Yn is


compact
S for any n 2 N. Apply Problem 076 to see that every Yn is sequential, so
Y D fYn W n 2 Ng is a countable union of compact sequential subspaces.
If K and L are compact t -equivalent spaces then Cp .L/ is homeomorphic to
Cp .K/ and hence it is an open continuous image of the space Cp .K/. The space
K being sequential, L is a countable union of compact sequential subspaces which,
together with Martins Axiom and Problem 074, implies that L is sequential.
V.080. Suppose
that X and Y are metrizable t -equivalent spaces. Prove that we
S
have Y D fYn W n 2 !g, where each Yn is a G -subspace of Y , homeomorphic to
some G -subspace of X .
Solution. For any set P denote by Fin.P / the family of all finite subsets of P ; if
n 2 N then P
n D fB  P W jBj D ng. Given a space Z the map idZ W Z ! Z
is the identity, i.e., idZ .z/ D z for any z 2 Z. If A is a finite subset of Z and n 2 N
then OZ .A; n/ D ff 2 Cp .Z/ W jf .x/j < n1 for any x 2 Ag. For all z 2 Z and
n 2 N let GZ .z; n/ D ff 2 Cp .Z/ W jf .z/j  n1 g.
Choose metrics X and Y which generate the topologies of the spaces X and
Y respectively; the spaces Cp .X / and Cp .Y / being homogeneous, it is easy to
convince ourselves that there exists a homeomorphism W Cp .X / ! Cp .Y / such
that .0X / D 0Y . Fix m; n 2 N and consider, for every point y 2 Y , the family
FY .y; m; n/ D fA 2 Fin.X / W .OX .A; m//  GY .y; n/g; if FY .y; m; n/ ;
then we can define am;n .y/ D nnfjAj W A 2 FY .y; m; n/g. Likewise, for all
x 2 X let FX .x; m; n/ D fB 2 Fin.Y / W 1 .OY .B; m//  GX .x; n/g; if
FX .x; m; n/ ; then let bm;n .x/ D nnfjBj W B 2 FX .x; m; n/g. In what
follows, when we deal with am;n .y/ and/or bm;n .x/ we assume, without mentioning
it explicitly, that the respective family FY .y; m; n/ and/or FX .x; m; n/ is nonempty.
Observe that OX .;; m/ D Cp .X / and hence .OX .;; m// D Cp .Y / 6 G.y; n/
for any y 2 Y ; this shows that if FY .y; m; n/ ; then all elements of the family
FY .y; m; n/ are nonempty and hence am;n .y/ 2 N for all m; n 2 N; let
EY .y; m; n/ D fA 2 FY .y; m; n/ W jAj D am;n .y/g:
The same reasoning demonstrates that if FX .x; m; n/ ; then all elements of
FX .x; m; n/ are nonempty; let EX .x; m; n/ D fB 2 FX .x; m; n/ W jBj D bm;n .x/g.
Fact 1. Suppose that M and L are metrizable spaces and we have A  M and
B  L which are G -sets in M and L respectively. Assume additionally that there
are continuous maps f W A ! L and g W B ! A such that f g D idB . Then the
map g W B ! g.B/ is a homeomorphism and g.B/ is a G -subset of M .
Proof. Since f jg.B/ is a continuous inverse of g, the map g W B ! g.B/ is a
homeomorphism. To see that g.B/ is a G -subset of X take complete metric spaces
f and e
f and L  e
M
L such that M  M
L. Apply Fact 2 of T.333 to see that there are
f and e
G -sets A0 and B 0 in M
L respectively such that g.B/  A0 ; B  B 0 and there
is a homeomorphism h W B 0 ! A0 with hjB D g; note that A \ A0 is a G -subset

112

2 Solutions of problems 001500

of M . Since h1 jg.B/ D f jg.B/, the set E D fx 2 A \ A0 W h1 .x/ D f .x/g


contains g.B/; it is straightforward that E is a G -subset of A \ A0 , so it is also a
G -subset of M . If E 0 D f .E/ then f0 D f jE W E ! E 0 is a homeomorphism;
since E 0  L, the subspace B  E 0 is a G -subset of E 0 , so g.B/ D f01 .B/ is a
G -subset in E and hence in M . Fact 1 is proved.
Fact 2. Suppose that M and L are metrizable spaces, n 2 N and p W M ! L
n is
a lower semicontinuous
map. Then there exists a family U D fUi W i 2 !g  .M /
S
such that U D M and, for any i 2 !, there are continuous maps f1i ; : : : ; fni such
that fji W Ui ! L for every j  n and p.x/ D ff1i .x/; : : : ; fni .x/g for any x 2 Ui .
Proof. Fix a point x 2 M and let fy1 ; : : : ; yn g be an enumeration of p.x/. Choose
disjoint sets W1 ; : : : ; Wn 2 .L/ such that yi 2 Wi for all i  n. By lower
semicontinuity of p, the set Ox D fz 2 M W p.z/ \ Wi ; for every i  ng
is an open neighborhood of x in M and p.z/ \ Wi is a singleton for any z 2 Ox
and i  n. For any z 2 Ox let hxi .z/ be the unique point of the set p.z/ \ Wi ; it is
an easy exercise that the map hxi W Ox ! Wi is continuous for any i  n. Besides,
p.z/ D fhx1 .z/; : : : ; hxn .z/g for any z 2 Ox .
The space M being metrizable, we can find a  -discrete refinement U of the
cover fOx W x 2 M
S g. Pick a sequence fUi W i 2 !g of discrete subfamilies of
U such that U D n2! Ui . Fix i 2 !; for every U 2 Ui there is x.U / 2 M
x.U /
such that U  Ox.U / . Let fji .z/ D hj .z/ for any z 2 U and j  n. This
S
defines a continuous map fji on the set Ui D Ui for any j  n. It is evident that
S
M D n2! Ui and the maps f1i ; : : : ; fni are as promised for any i 2 !, so Fact 2 is
proved.
Fact 3. Suppose that m; n; k 2 N and a sequence fyi W i 2 !g  Y converges to
a point y 2 Y . If Ai 2 FY .yi ; m; n/ and jAi j D k for any i 2 ! then there is
a subsequence fAij W j 2 !g of the sequence A D fAi W i 2 !g and a number
j
j
q  k for which we can choose a faithful enumeration fa1 ; : : : ; ak g of every set Aij
j
in such a way that the sequence fap W j 2 !g converges to a point xp 2 X for all
p  q and A D fx1 ; : : : ; xq g 2 FY .y; m; n/.
Proof. We will pass several times to a subsequence of the sequence A D fAi W
i 2 !g; since our aim is to find a certain subsequence of A, at each step we will
identify the obtained subsequence with A considering that all elements of A have
the property we have found in a subsequence.
The first step is to use Fact 2 of U.337 to choose a subsequence A0  A for
which there is a set D D fd1 ; : : : ; dr g  X such that A \ A0 D D for distinct
A; A0 2 A0 (observe that it is possible that r D 0 in which case D D ;). According
to the above-mentioned politics we can consider that, for any i 2 !, we have Ai D
i
fd1 ; : : : ; dr ; a1i ; : : : ; akr
g and the family fAi nD W i 2 !g is disjoint.
An evident property of metric spaces is that any sequence contains either a
convergent subsequence or an infinite closed discrete subspace. This makes it
possible to pass to a subsequence of A once more to guarantee that, for any

2 Solutions of problems 001500

113

j 2 f1; : : : ; k  rg, the sequence Sj D faji W i 2 !g is either convergent


or constitutes a closed discrete subspace of X . If Sj is convergent then denote
by xj its limit. Renumbering every Ai if necessary we can assume that Ai D
i
i
i
fd1 ; : : : ; dr ; a1i ; : : : ; ali ; alC1
; : : : ; akr
g while the set Q D falCj
W i 2 !; 1 
j  k  r  lg is closed and discrete in X and the sequence Sj converges to xj for
any j 2 f1; : : : ; lg.
It turns out that the set B D fd1 ; : : : ; dr ; x1 ; : : : ; xl g belongs to FY .y; m; n/.
To prove this assume toward a contradiction that there is a function f 2 Cp .X /
such that f .B/  . m1 ; m1 / and j.f /.y/j > n1 . By continuity of there is a finite
set E  B and > 0 such that,
(1) for any g 2 Cp .X /, if jg.x/  f .x/j < for all x 2 E then j.g/.y/j > n1 .
It is easy to find W 2 .B; X / such that W \ Q is finite and f .W /  . m1 ; m1 /.
Every sequence Sj is eventually in W , so there is p 2 N such that aji 2 W for
all j  l and i  p. Therefore we can pass to a subsequence of A once more to
assume, without loss of generality, that Sj  W for all j  l and .W [E/\Q D ;.
Now we can find a function g 2 Cp .X / such that gj.W [ E/ D f j.W [ E/ and
g.Q/  f0g. Then g.Ai /  . m1 ; m1 / and hence j.g/.yi /j  n1 for every i 2 !.
The sequence fyi W i 2 !g converges to y, so j.g/.y/j  n1 by continuity of .g/.
However gjE D f jE, so (1) implies that j.g/.y/j > n1 ; this contradiction shows
that B 2 FY .y; m; n/. Letting q D l C r and xlCi D di for all i 2 f1; : : : ; rg we
obtain the promised set A D fx1 ; : : : ; xq g 2 FY .y; m; n/; now it is evident how to
enumerate the sets Ai to obtain the convergence we need, so Fact 3 is proved.
Fact 4. For any k; m; n 2 N, the set CY .k; m; n/ D fy 2 Y W am;n .y/  kg is
closed in Y . Analogously, every set CX .k; m; n/ D fx 2 X W bm;n .x/  kg is
closed in X .
Proof. We have a symmetric situation with CX .k; m; n/ and CY .k; m; n/, so it
suffices to show that CY .k; m; n/ is closed in Y . To do so suppose that yi 2
CY .k; m; n/ for every i 2 ! and the sequence S D fyi W i 2 !g converges to
a point y 2 Y . Fix a set Ai 2 FY .yi ; m; n/ such that jAi j  k for each i 2 !.
Passing to a subsequence of S if necessary, we can assume that there is k0  k such
that jAi j D k0 for all i 2 !. It follows from Fact 3 that there is k1  k0 and a
set A 2 FY .y; m; n/ with jAj  k1 . Consequently, am;n .y/  k1  k and hence
y 2 CY .k; m; n/. This shows that CY .k; m; n/ is, indeed, closed in Y , so Fact 4 is
proved.
S
S
Fact 5. We have Y D k;m2N CY .k; m; n/ and X D k;m2N CX .k; m; n/ for any
n 2 N.
Proof. Fix any n 2 N; again, by the symmetry of the situation, it suffices to prove
the first equality. To do so, take any point y 2 Y . Since the set O D ff 2 Cp .Y / W
jf .y/j < n1 g  G.y; n/ is an open neighborhood of 0Y , there is a set A 2 Fin.X /
and m 2 N such that .OX .A; m//  O. We have A 2 FY .y; m; n/; if jAj D k
then am;n .y/  k which shows that y 2 CY .k; m; n/. Fact 5 is proved.

114

2 Solutions of problems 001500

Fact 6. For any m; n 2 N and y 2 Y the family EY .y; m; n/ is finite. Analogously,


every family EX .x; m; n/ is finite.
Proof. Once more we have a symmetric situation, so it suffices to show that
the family EY .y; m; n/ is finite. Assume the contrary, let k D am;n .y/ and fix an
infinite family A  EY .y; m; n/. Apply Fact 2 of U.337 to find a set D  X and an
infinite subfamily A0  A such that A \ A0 D D for any distinct A; A0 2 A0 ; since
also jAj D jA0 j D k for any A; A0 2 A0 , we have AnD ; for all A 2 A0 and
hence jDj < k.
We claim that D 2 FY .y; m; n/; to prove it suppose that there is a function
f 2 OX .D; m/ such that j.f /.y/j > n1 . By continuity of , there is a finite set
E  D and > 0 such that
(2) if jg.x/  f .x/j < for all x 2 E then j.g/.y/j > n1 .
The family fAnD W A 2 A0 g being disjoint and infinite, there is A 2 A0 such that
.AnD/ \ E D ;. Pick h 2 Cp .X / such that h.AnD/ D f0g and hjE D f jE. Then
h 2 OX .A; m/; this, together with A 2 FY .y; m; n/ implies that j.h/.y/j  n1 .
On the other hand, it follows from f jE D hjE and (2) that j.h/.y/j > n1 which
is a contradiction. Therefore the family A is finite and hence Fact 6 is proved.
Fact 7. Fix m; n 2 N; we will need the set DY .1; m; n/ D CY .1; m; n/; if k > 1
then let DY .k; m; n/ D CY .k; m; n/nCY .k  1; m; n/. Now fix k 2SN and define
a map 'm;n W DY .k; m; n/ ! exp.X / by the equality 'm;n .y/ D
EY .y; m; n/
for all y 2 DY .k; m; n/. Next, for any p; q 2 N such that p  k consider the set
EY .k; m; n; p; q/ D fy 2 DY .k; m; n/ W j'm;n .y/j D p and X .x; x 0 /  q1 for
distinct x; x 0 2 'm;n .y/g. Then every EY .k; m; n; p; q/ is a G -subset of Y and the
map 'm;n jEY .k; m; n; p; q/ W EY .k; m; n; p; q/ ! X
p is lower semicontinuous.
Proof. The set DY .k; m; n/ is open in the closed set CY .k; m; n/; an immediate
consequence is that DY .k; m; n/ is a G -subset of Y . For fixed p  k and q 2 N
consider the set K.p; q/ D fy 2 DY .k; m; n/ W there exists a set A  'm;n .y/ such
that jAj D p and X .x; x 0 /  q1 for any distinct x; x 0 2 Ag.
To see that the set K.p; q/ is closed in DY .k; m; n/ suppose that a sequence
fyi W i 2 !g  K.p; q/ converges to a point y 2 DY .k; m; n/ and fix, for any
i 2 !, a set Ai  'm;n .yi / such that jAi j D p and X .x; x 0 /  q1 for any distinct
x; x 0 2 Ai .
S
Take an arbitrary point zi 2 Ai ; since Ai  EY .yi ; m; n/, we can pick a set
Bi 2 EY .yi ; m; n/ with zi 2 Bi for every i 2 !. Apply Fact 3 to find a subsequence
fBij W j 2 !g of the sequence fBi W i 2 !g for which we can choose an enumeration
j
j
fb1 ; : : : ; bk g of every Bij in such a way that there is k0  k for which every
j
sequence fbl W j 2 !g converges to a point bl and the set fb1 ; : : : ; bk0 g belongs
j
j
to FY .y; m; n/. However, am;n .y/ D k, so k0 D k and hence zij 2 fb1 ; : : : ; bk0 g
and therefore the sequence fzij W j 2 !g converges to a point of 'm;n .y/.
This shows that, passing several times to a subsequence of fAi W i 2 !g if
necessary, we can assume that there is an enumeration fa1i ; : : : ; api g of every Ai

2 Solutions of problems 001500

115

such that the sequence fali W i 2 !g converges to a point al 2 'm;n .y/ for any
l  p. Given distinct numbers l; l 0 2 f1; : : : ; pg we have X .ali ; ali 0 /  q1 for any
i 2 !; this, evidently, implies that X .al ; al 0 /  q1 and hence the set fa1 ; : : : ; ap g 
'm;n .y/ witnesses that the point y belongs to K.p; q/. This proves that every set
K.p; q/ is closed in a G -set DY .k; m; n/, so it is a G -set
S in Y as well. Now, it
follows from the equality EY .k; m; n; p; q/ D K.p; q/n. fK.p C 1; r/ W r 2 Ng/
that EY .k; m; n; p; q/ is a G -subset of Y .
To finally see that the map ' D 'm;n jEY .k; m; n; p; q/ is lower semicontinuous
let E D EY .k; m; n; p; q/ and fix a set U 2 .X / such that '.y/ \ U ; for some
y 2 E. It suffices to show that there is a set V 2 .y; E/ such that '.z/ \ U ;
for any z 2 V . If such a set V does not exist then it is easy to find a sequence
S D fyi W i 2 !g  E such that S ! y and '.yi /\U D ; for any i 2 !. Using the
observation in the third paragraph of this proof we can pass to a subsequence of S if
necessary to be able to assume, without loss of generality, that '.yi / D fzi1 ; : : : ; zip g
for every i 2 ! and the sequence fzil W i 2 !g is convergent to a point zl 2 '.y/nU
for any l  p. Given distinct numbers l; l 0 2 f1; : : : ; pg we have X .zil ; zil 0 /  q1
for any i 2 !; this, evidently, implies that X .zl ; zl 0 /  q1 and hence zl zl 0 .
Thus the set fz1 ; : : : ; zp g  '.y/nU has cardinality p; since also '.y/ \ U ;,
we conclude that j'.y/j  p C 1 which is a contradiction. Therefore the map ' is
lower semicontinuous and Fact 7 is proved.
Fact 8. Fix m; n 2 N; we will need the set DX .1; m; n/ D CX .1; m; n/; if k > 1
then let DX .k; m; n/ D CX .k; m; n/nCX .k  1; m; n/. Now fix k 2SN and define
a map m;n W DX .k; m; n/ ! exp.Y / by the equality m;n .x/ D
EX .x; m; n/
for all x 2 DX .k; m; n/. Next, for any p; q 2 N such that p  k consider the set
EX .k; m; n; p; q/ D fy 2 DX .k; m; n/ W jm;n .x/j D p and Y .y; y 0 /  q1 for
distinct y; y 0 2 m;n .x/g. Then every EX .k; m; n; p; q/ is a G -subset of X and the
map m;n jEX .k; m; n; p; q/ W EX .k; m; n; p; q/ ! Y
p is lower semicontinuous.
Proof. Left as an exercise to the reader; one only needs to substitute the notions we
used for Y by their analogues for X and vice versa.
Fact 9. If k; m 2 N and y 2 DY .k; m; 1/ then there are numbers l; s 2 N and a
point x 2 'm;1 .y/ \ DX .s; l; m C 1/ such that y 2 l;mC1 .x/.
Proof. Assume the contrary; by definition of
S'm;1 we can find a set A  'm;1 .y/
such that .OX .A; m//  GY .y; 1/. Since fCX .p; q; m C 1/ W p; q 2 Ng D X
by Fact 5, for each x 2 A there are p; qx 2 N with x 2 CX .p; qx ; m C 1/. If px D
nnfp 2 N W x 2 CX .p; qx ; m C 1/g S
then x 2 DX .px ; qx ; m C 1/.
By our assumption, y P D fqx ;mC1 .x/ W x 2 Ag, so we can take f 2
Cp .Y / such that f .y/ D 2 and f jP  0. Every qx ;mC1 .x/ contains a set Bx such
that 1 .OY .Bx ; qx //  GX .x; m C 1/. It is evident that f 2 OY .Bx ; qx / for any
1
x 2 A, so j 1 .f /.x/j  mC1
< m1 which shows that g D 1 .f / 2 OX .A; m/
and therefore f D .g/ 2 GY .y; 1/, i.e., jf .y/j  1 which is a contradiction. As
a consequence, y 2 qx ;mC1 .x/ for some x 2 'm;1 .y/ \ DX .px ; qx ; m C 1/, so the
numbers s D px and l D qx are as promised. Fact 9 is proved.

116

2 Solutions of problems 001500

S
Returning to our solution observe that Y D k;m2N CY .k;
Sm; 1/ by Fact 5; it
follows from the definition of the sets DY .k; m; S
1/ that Y D k;m2N DY .k; m; 1/.
Another easy observation is that DY .k; m; 1/ D fEY .k; m; 1; p; q/ W p  k; q 2
Ng which shows that Y is a countable union of its G -subspaces EY .k; m; 1; p; q/,
so it suffices to show that every set EY .k; m; 1; p; q/ is a countable union of
subspaces homeomorphic to their respective G -subspaces of X . To that end fix
k; m; p; q 2 N with k  p.
S
Apply Fact 7 and Fact 2 to see that EY .k; m; 1; p; q/ D r2N Gr where every Gr
is open in EY .k; m; 1; p; q/ (and hence is a G -subset of Y ) and there are continuous
functions f1 ; : : : ; fp W Gr ! X such that 'm;1 .y/ D ff1 .y/; : : : ; fp .y/g for any
point y 2 Gr . It suffices to show that every Gr is a countable union of its G -subsets
which are homeomorphic to their respective G -subspaces of X , so we fix r 2 N
and f1 ; : : : ; fp as above.
S
0
0
Analogously, X D fEX .k 0 ; m0 ; mC1; p 0 ; q 0 / W k 0 ; m0 ; p 0 ; q 0 2 N;
S p  k g, so
we can apply Fact 2 and Fact 7 again to convince ourselves that X D fHs W s 2 Ng
where every Hs is a G -subset of X , contained in some EX .k 0 ; m0 ; m C 1; p 0 ; q 0 /,
for which there exist continuous maps g1s ; : : : ; glss W Hs ! Y such that ls D p 0 and
m0 ;mC1 .x/ D fg1s .x/; : : : ; glss .x/g for any x 2 Hs .
Applying Fact 9 we conclude that
(3) for any y 2 Gr with fi .y/ 2 Hsi for all i  p there exist i  p and j  lsi
such that gjsi .fi .y// D y.
Given any  D .s1 ; : : : ; sp / 2 Np the set Gr 
D fy 2 GS
r W fi .y/ 2 Hsi for
each i  pg is a G -subspace of Y ; it is evident that Gr D fGr 
W  2 Np g
and hence again it suffices to show that every set Gr 
is the countable union of its
G -subsets which are homeomorphic to their respective G -subspaces of X , so we
fix an arbitrary  D .s1 ; : : : ; sp / 2 Np .
S
S
The property (3) implies that Gr 
D ip j ls fy 2 Gr 
W gjsi .fi .y// D
i
yg; every set Q.i; j / D fy 2 Gr 
W gjsi .fi .y// D yg is a G -subset of Y being
closed in Gr 
; therefore Q.i; j / is homeomorphic to a G -subspace of X by
Fact 1. This completes the desired representation of Y as the countable union of
its G -subspaces each one of which is homeomorphic to a G -subspace of X .
V.081.S Let X and Y be metrizable spaces such that Cp .X / ' Cp .Y /. Prove that
Y D fYn W n 2 !g, where each Yn is a G -subspace of Y , homeomorphic to some
G -subspace of X .
Solution. For any set P denote by Fin.P / the family of all finite subsets of P ; if
n 2 N then P
n D fB  P W jBj D ng. If A is a finite subset of Z and n 2 N then
OZ .A; n/ D ff 2 Cp .Z/ W jf .x/j < n1 for any x 2 Ag. For all z 2 Z and n 2 N
let GZ .z; n/ D ff 2 Cp .Z/ W jf .z/j  n1 g.
Choose metrics X and Y which generate the topologies of the spaces X and
Y respectively; the spaces Cp .X / and Cp .Y / being homogeneous, it is easy to
convince ourselves that there exists a homeomorphism W Cp .X / ! Cp .Y / such
that .0X / D 0Y . Fix m; n 2 N and consider, for every point y 2 Y , the family
FY .y; m; n/ D fA 2 Fin.X / W .OX .A; m//  GY .y; n/g; if FY .y; m; n/ ;

2 Solutions of problems 001500

117

then we can define am;n .y/ D nnfjAj W A 2 FY .y; m; n/g. Likewise, for all x 2 X
let FX .x; m; n/ D fB 2 Fin.Y / W 1 .OY .B; m//  GX .x; n/g; if FX .x; m; n/
; then let bm;n .x/ D nnfjBj W B 2 FX .x; m; n/g. In what follows, when we deal
with am;n .y/ and/or bm;n .x/ we assume, without mentioning it explicitly, that the
respective family FY .y; m; n/ and/or FX .x; m; n/ is nonempty.
Observe that OX .;; m/ D Cp .X / and hence .OX .;; m// D Cp .Y / 6 G.y; n/
for any y 2 Y ; this shows that if FY .y; m; n/ ; then all elements of the family
FY .y; m; n/ are nonempty and hence am;n .y/ 2 N for all m; n 2 N; let
EY .y; m; n/ D fA 2 FY .y; m; n/ W jAj D am;n .y/g:
The same reasoning demonstrates that if FX .x; m; n/ ; then all elements of
FX .x; m; n/ are nonempty; let EX .x; m; n/ D fB 2 FX .x; m; n/ W jBj D bm;n .x/g.
Fact 1. Suppose that m; n; k 2 N and a sequence fyi W i 2 !g  Y converges to
a point y 2 Y . If Ai 2 FY .yi ; m; n/ and jAi j D k for any i 2 ! then there is
a subsequence fAij W j 2 !g of the sequence A D fAi W i 2 !g and a number
j
j
q  k for which we can choose a faithful enumeration fa1 ; : : : ; ak g of every set Aij
j
in such a way that the sequence fap W j 2 !g converges to a point xp 2 X for all
p  q and A D fx1 ; : : : ; xq g 2 FY .y; m; n/.
Proof. We will pass several times to a subsequence of the sequence A D fAi W
i 2 !g; since our aim is to find a certain subsequence of A, at each step we will
identify the obtained subsequence with A considering that all elements of A have
the property we have found in a subsequence.
The first step is to use Fact 2 of U.337 to choose a subsequence A0  A for
which there is a set D D fd1 ; : : : ; dr g  X such that A \ A0 D D for distinct
A; A0 2 A0 (observe that it is possible that r D 0 in which case D D ;). According
to the above-mentioned politics we can consider that, for any i 2 !, we have Ai D
i
fd1 ; : : : ; dr ; a1i ; : : : ; akr
g and the family fAi nD W i 2 !g is disjoint.
An evident property of metric spaces is that any sequence contains either a
convergent subsequence or a closed discrete subspace. This makes it possible to
pass to a subsequence of A once more to guarantee that, for any j 2 f1; : : : ; k  rg,
the sequence Sj D faji W i 2 !g is either convergent or constitutes a closed discrete
subspace of X . If Sj is convergent then denote by xj its limit. Renumbering every
i
i
Ai if necessary we can assume that Ai D fd1 ; : : : ; dr ; a1i ; : : : ; ali ; alC1
; : : : ; akr
g
i
while the set Q D falCj W i 2 !; 1  j  k  r  lg is closed and discrete in X
and the sequence Sj converges to xj for any j 2 f1; : : : ; lg.
It turns out that the set B D fd1 ; : : : ; dr ; x1 ; : : : ; xl g belongs to FY .y; m; n/.
To prove this assume toward a contradiction that there is a function f 2 Cp .X /
such that f .B/  . m1 ; m1 / and j.f /.y/j > n1 . By continuity of there is a finite
set E  B and > 0 such that,
(1) for any g 2 Cp .X /, if jg.x/  f .x/j < for all x 2 E then j.g/.y/j > n1 .

118

2 Solutions of problems 001500

It is easy to find W 2 .B; X / such that W \ Q is finite and f .W /  . m1 ; m1 /.


Every sequence Sj is eventually in W , so there is p 2 N such that aji 2 W for
all j  l and i  p. Therefore we can pass to a subsequence of A once more to
assume, without loss of generality, that Sj  W for all j  l and .W [E/\Q D ;.
Now we can find a function g 2 Cp .X / such that gj.W [ E/ D f j.W [ E/ and
g.Q/  f0g. Then g.Ai /  . m1 ; m1 / and hence j.g/.yi /j  n1 for every i 2 !.
The sequence fyi W i 2 !g converges to y, so j.g/.y/j  n1 by continuity of .g/.
However gjE D f jE, so (1) implies that j.g/.y/j > n1 ; this contradiction shows
that B 2 FY .y; m; n/. Letting q D l C r and xlCi D di for all i 2 f1; : : : ; rg we
obtain the promised set A D fx1 ; : : : ; xq g 2 FY .y; m; n/; now it is evident how to
enumerate the sets Ai to obtain the convergence we need, so Fact 1 is proved.
Fact 2. For any k; m; n 2 N, the set CY .k; m; n/ D fy 2 Y W am;n .y/  kg
is closed in Y . Analogously, every set CX .k; m; n/ D fx 2 X W bm;n .x/  kg
is closed in X .
Proof. We have a symmetric situation with CX .k; m; n/ and CY .k; m; n/, so it
suffices to show that CY .k; m; n/ is closed in Y . To do so suppose that yi 2
CY .k; m; n/ for every i 2 ! and the sequence S D fyi W i 2 !g converges to
a point y 2 Y . Fix a set Ai 2 FY .yi ; m; n/ such that jAi j  k for each i 2 !.
Passing to a subsequence of S if necessary, we can assume that there is k0  k such
that jAi j D k0 for all i 2 !. It follows from Fact 1 that there is k1  k0 and a
set A 2 FY .y; m; n/ with jAj  k1 . Consequently, am;n .y/  k1  k and hence
y 2 CY .k; m; n/. This shows that CY .k; m; n/ is, indeed, closed in Y , so Fact 2 is
proved.
S
S
Fact 3. We have Y D k;m2N CY .k; m; n/ and X D k;m2N CX .k; m; n/ for any
n 2 N.
Proof. Fix any number n 2 N; again, by the symmetry of the situation, it suffices
to prove only the first equality. To do so, take any point y 2 Y . Since the set O D
ff 2 Cp .Y / W jf .y/j < n1 g  G.y; n/ is an open neighborhood of 0Y , there is a set
A 2 Fin.X / and m 2 N such that .OX .A; m//  O. We have A 2 FY .y; m; n/;
if jAj D k then am;n .y/  k which shows that y 2 CY .k; m; n/. Fact 3 is proved.
Fact 4. For any m; n 2 N and y 2 Y the family EY .y; m; n/ is finite. Analogously,
every family EX .x; m; n/ is finite.
Proof. Once more we have a symmetric situation, so it suffices to show that
the family EY .y; m; n/ is finite. Assume the contrary, let k D am;n .y/ and fix an
infinite family A  EY .y; m; n/. Apply Fact 2 of U.337 to find a set D  X and an
infinite subfamily A0  A such that A \ A0 D D for any distinct A; A0 2 A0 ; since
also jAj D jA0 j D k for any A; A0 2 A0 , we have AnD ; for all A 2 A0 and
hence jDj < k.
We claim that D 2 FY .y; m; n/; to prove it suppose that there is a function
f 2 OX .D; m/ such that j.f /.y/j > n1 . By continuity of , there is a finite set
E  D and > 0 such that
(2) if jg.x/  f .x/j < for all x 2 E then j.g/.y/j > n1 .

2 Solutions of problems 001500

119

The family fAnD W A 2 A0 g being disjoint and infinite, there is A 2 A0 such


that .AnD/ \ E D ;. Pick h 2 Cp .X / such that h.AnD/ D f0g and hjE D f jE.
Then h 2 OX .A; m/; this, together with A 2 FY .y; m; n/ implies j.h/.y/j  n1 .
On the other hand, it follows from f jE D hjE and (2) that j.h/.y/j > n1 which
is a contradiction. Therefore the family A is finite and hence Fact 4 is proved.
Fact 5. Fix m; n 2 N; we will need the set DY .1; m; n/ D CY .1; m; n/; if k > 1
then let DY .k; m; n/ D CY .k; m; n/nCY .k  1; m; n/. Now fix k 2SN and define
a map 'm;n W DY .k; m; n/ ! exp.X / by the equality 'm;n .y/ D
EY .y; m; n/
for all y 2 DY .k; m; n/. Next, for any p; q 2 N such that p  k consider the set
EY .k; m; n; p; q/ D fy 2 DY .k; m; n/ W j'm;n .y/j D p and X .x; x 0 /  q1 for
distinct x; x 0 2 'm;n .y/g. Then every EY .k; m; n; p; q/ is a G -subset of Y and the
map 'm;n jEY .k; m; n; p; q/ W EY .k; m; n; p; q/ ! X
p is lower semicontinuous.
Proof. The set DY .k; m; n/ is open in the closed set CY .k; m; n/; an immediate
consequence is that DY .k; m; n/ is a G -subset of Y . For fixed p  k and q 2 N
consider the set K.p; q/ D fy 2 DY .k; m; n/ W there exists a set A  'm;n .y/ such
that jAj D p and X .x; x 0 /  q1 for any distinct x; x 0 2 Ag.
To see that the set K.p; q/ is closed in DY .k; m; n/ suppose that a sequence
fyi W i 2 !g  K.p; q/ converges to a point y 2 DY .k; m; n/ and fix, for any
i 2 !, a set Ai  'm;n .yi / such that jAi j D p and X .x; x 0 /  q1 for any distinct
x; x 0 2 Ai .
S
Take an arbitrary point zi 2 Ai ; since Ai  EY .yi ; m; n/, we can pick a set
Bi 2 EY .yi ; m; n/ with zi 2 Bi for every i 2 !. Apply Fact 1 to find a subsequence
fBij W j 2 !g of the sequence fBi W i 2 !g for which we can choose an enumeration
j
j
fb1 ; : : : ; bk g of every Bij in such a way that there is k0  k for which every
j
sequence fbl W j 2 !g converges to a point bl and the set fb1 ; : : : ; bk0 g belongs
j
j
to FY .y; m; n/. However, am;n .y/ D k, so k0 D k and hence zij 2 fb1 ; : : : ; bk0 g
and therefore the sequence fzij W j 2 !g converges to a point of 'm;n .y/.
This shows that, passing several times to a subsequence of fAi W i 2 !g if
necessary, we can assume that there is an enumeration fa1i ; : : : ; api g of every Ai
such that the sequence fali W i 2 !g converges to a point al 2 'm;n .y/ for any
l  p. Given distinct numbers l; l 0 2 f1; : : : ; pg we have X .ali ; ali 0 /  q1 for any
i 2 !; this, evidently, implies that X .al ; al 0 /  q1 and hence the set fa1 ; : : : ; ap g 
'm;n .y/ witnesses that the point y belongs to K.p; q/. This proves that every set
K.p; q/ is closed in a G -set DY .k; m; n/, so it is a G -set
S in Y as well. Now, it
follows from the equality EY .k; m; n; p; q/ D K.p; q/n. fK.p C 1; r/ W r 2 Ng/
that EY .k; m; n; p; q/ is a G -subset of Y .
To finally see that the map ' D 'm;n jEY .k; m; n; p; q/ is lower semicontinuous
let E D EY .k; m; n; p; q/ and fix a set U 2 .X / such that '.y/ \ U ; for some
y 2 E. It suffices to show that there is a set V 2 .y; E/ such that '.z/ \ U ;
for any z 2 V . If such a set V does not exist then it is easy to find a sequence
S D fyi W i 2 !g  E such that S ! y and '.yi / \ U D ; for any i 2 !. Using
the observation in the third paragraph of this proof we can pass to a subsequence

120

2 Solutions of problems 001500

of S if necessary to be able to assume, without loss of generality, that '.yi / D


fzi1 ; : : : ; zip g for every i 2 ! and the sequence fzil W i 2 !g is convergent to a point
zl 2 '.y/nU for any l  p. Given distinct numbers l; l 0 2 f1; : : : ; pg we have
X .zil ; zil 0 /  q1 for any i 2 !; this, evidently, implies that X .zl ; zl 0 /  q1 and
hence zl zl 0 . Thus the set fz1 ; : : : ; zp g  '.y/nU has cardinality p; since also
'.y/ \ U ;, we conclude that j'.y/j  p C 1 which is a contradiction. Therefore
the map ' is lower semicontinuous and Fact 5 is proved.
Fact 6. Fix m; n 2 N; we will need the set DX .1; m; n/ D CX .1; m; n/; if k > 1
then let DX .k; m; n/ D CX .k; m; n/nCX .k  1; m; n/. Now fix k 2SN and define
a map m;n W DX .k; m; n/ ! exp.Y / by the equality m;n .x/ D
EX .x; m; n/
for all x 2 DX .k; m; n/. Next, for any p; q 2 N such that p  k consider the set
EX .k; m; n; p; q/ D fy 2 DX .k; m; n/ W jm;n .x/j D p and Y .y; y 0 /  q1 for
distinct y; y 0 2 m;n .x/g. Then every EX .k; m; n; p; q/ is a G -subset of X and the
map m;n jEX .k; m; n; p; q/ W EX .k; m; n; p; q/ ! Y
p is lower semicontinuous.
Proof. Left as an exercise to the reader; one only needs to substitute the notions we
used for Y by their analogues for X and vice versa.
Fact 7. If k; m 2 N and y 2 DY .k; m; 1/ then there are numbers l; s 2 N and a
point x 2 'm;1 .y/ \ DX .s; l; m C 1/ such that y 2 l;mC1 .x/.
Proof. Assume the contrary; by definition of
S'm;1 we can find a set A  'm;1 .y/
such that .OX .A; m//  GY .y; 1/. Since fCX .p; q; m C 1/ W p; q 2 Ng D X
by Fact 3, for each x 2 A there are p; qx 2 N with x 2 CX .p; qx ; m C 1/. If px D
nnfp 2 N W x 2 CX .p; qx ; m C 1/g S
then x 2 DX .px ; qx ; m C 1/.
By our assumption, y P D fqx ;mC1 .x/ W x 2 Ag, so we can take f 2
Cp .Y / such that f .y/ D 2 and f jP  0. Every qx ;mC1 .x/ contains a set Bx such
that 1 .OY .Bx ; qx //  GX .x; m C 1/. It is evident that f 2 OY .Bx ; qx / for any
1
x 2 A, so j 1 .f /.x/j  mC1
< m1 which shows that g D 1 .f / 2 OX .A; m/
and therefore f D .g/ 2 GY .y; 1/, i.e., jf .y/j  1 which is a contradiction. As
a consequence, y 2 qx ;mC1 .x/ for some x 2 'm;1 .y/ \ DX .px ; qx ; m C 1/, so the
numbers s D px and l D qx are as promised. Fact 7 is proved.
S
Returning to our solution observe that Y D k;m2N CY .k;
Sm; 1/ by Fact 3; it
follows from the definition of the sets DY .k; m; S
1/ that Y D k;m2N DY .k; m; 1/.
Another easy observation is that DY .k; m; 1/ D fEY .k; m; 1; p; q/ W p  k; q 2
Ng which shows that Y is a countable union of its G -subspaces EY .k; m; 1; p; q/,
so it suffices to show that every set EY .k; m; 1; p; q/ is a countable union of
subspaces homeomorphic to their respective G -subspaces of X . To that end fix
k; m; p; q 2 N with k  p.
S
Apply Fact 5 and Fact 2 of V.080 to see that EY .k; m; 1; p; q/ D r2N Gr
where every Gr is open in EY .k; m; 1; p; q/ (and hence is a G -subset of Y )
and there are continuous functions f1 ; : : : ; fp W Gr ! X such that 'm;1 .y/ D
ff1 .y/; : : : ; fp .y/g for any point y 2 Gr . It suffices to show that every Gr is
a countable union of its G -subsets which are homeomorphic to their respective
G -subspaces of X , so we fix r 2 N and f1 ; : : : ; fp as above.

2 Solutions of problems 001500

121

S
Analogously, X D fEX .k 0 ; m0 ; m C 1; p 0 ; q 0 / W k 0 ; m0 ; p 0 ; q 0 2 N; p 0 
k 0 g, soS
we can apply Fact 5 and Fact 2 of V.080 again to convince ourselves that
X D fHs W s 2 Ng where every Hs is a G -subset of X , contained in some
EX .k 0 ; m0 ; mC1; p 0 ; q 0 /, for which there exist continuous maps g1s ; : : : ; glss W Hs !
Y such that ls D p 0 and m0 ;mC1 .x/ D fg1s .x/; : : : ; glss .x/g for any x 2 Hs .
Applying Fact 7 we conclude that
(3) for any y 2 Gr with fi .y/ 2 Hsi for all i  p there exist i  p and j  lsi
such that gjsi .fi .y// D y.
Given any  D .s1 ; : : : ; sp / 2 Np the set Gr 
D fy 2 GS
r W fi .y/ 2 Hsi for
each i  pg is a G -subspace of Y ; it is evident that Gr D fGr 
W  2 Np g
and hence again it suffices to show that every set Gr 
is the countable union of its
G -subsets which are homeomorphic to their respective G -subspaces of X , so we
fix an arbitrary  D .s1 ; : : : ; sp / 2 Np .
S
S
The property (3) implies that Gr 
D ip j ls fy 2 Gr 
W gjsi .fi .y// D
i
yg; every set Q.i; j / D fy 2 Gr 
W gjsi .fi .y// D yg is a G -subset of Y being
closed in Gr 
; therefore Q.i; j / is homeomorphic to a G -subspace of X by Fact 1
of V.080. This completes the desired representation of Y as the countable union of
its G -subspaces each one of which is homeomorphic to a G -subspace of X .
V.082. Let X and Y be metrizable t -equivalent spaces. Prove that X is a countable
union of zero-dimensional subspaces if and only if so is Y .
Solution. Say that a space Z belongs to the class P if Z is the countable union of
its zero-dimensional subspaces. Since every subspace of a zero-dimensional space
is zero-dimensional, we conclude that
(1) if Z 2 P then each Y  Z also belongs to P.
Suppose that X is the countable union of its zero-dimensional subspaces, i.e.,
X 2 P. By Problem
S 080, there exists a family F D fYn W n 2 !g of subspaces of
Y such that Y D F and every Yn is homeomorphic to a subspace of X . Apply
(1) to see that Yn 2 P for any n 2 !; the class P is, evidently,  -additive, so
Y 2 P. Analogously, if Y 2 P then X 2 P, so X is the countable union of its
zero-dimensional subspaces if and only if so is Y .
V.083. Let X and Y be metrizable t -equivalent spaces. Prove that X is a countable

union of its Cech-complete


subspaces if and only if so is Y .
Solution. Say that a space Z belongs to the class P if Z is the countable union of

its Cech-complete
subspaces. Since every G -subspace of a Cech-complete
space is

Cech-complete
(see TFS-260), we conclude that
(1) if Z 2 P then each G -subspace Y  Z also belongs to P.

Suppose that X is the countable union of its Cech-complete


subspaces, i.e.,
X 2 P. By Problem S
080, there exists a family F D fYn W n 2 !g of subspaces
of Y such that Y D
F and every Yn is homeomorphic to a G -subspace of X .

122

2 Solutions of problems 001500

Apply (1) to see that Yn 2 P for any n 2 !; the class P is, evidently,  -additive,
so Y 2 P. Analogously, if Y 2 P then X 2 P, so X is the countable union of its

Cech-complete
subspaces if and only if so is Y .
V.084. Suppose that fa0 ; : : : ; am g  Rn is an independent set. Prove that
(i) the simplex S D a0 ; : : : ; am
is a compact subset of Rn ;
(ii) any two m-dimensional simplexes are homeomorphic;
(iii) the barycentric coordinates are continuous functions from S to 0; 1
.
Solution. Consider the set m D f D . 0 ; : : : ; m / 2 RmC1 W i  0 for all
i  m and 0 C : : : C m D 1g. The i -th projection pi W RmC1 ! R defined
by pi . / D i for any D . 0 ; : : : ; m / 2 RmC1 is continuous for any i  m; an
easy consequence is that the set m is closed in RmC1 . Since also 0  i  1 for
any D . 0 ; : : : ; m / 2 m and i  m, we have m  ImC1 , so m is compact
being closed in the compact space ImC1 .
For any D . 0 ; : : : ; m / 2 m let '. / D 0 a0 C : : : C m am ; since
Rn is a linear topological space, the map ' W m ! Rn is continuous. Besides,
S D '. m /, so S is compact and hence (i) is proved.
Suppose that D . 0 ; : : : ; m /,  D .0 ; : : : ; m /; ;  2 m and '. / D
'./. If 0 2 Rn is the zero vector then . 0  0 / a0 C : : : C . m  m / am D 0
and . 0  0 / C : : : C . m  m / D 0 which implies, together with independency
of fa0 ; : : : ; am g, that i D i for every i  m, i.e., D ; this shows that the map
' is injective. Since '. m / D S , the map ' is a homeomorphism, i.e., we proved
that any m-dimensional simplex is homeomorphic to m ; it follows that any two
m-dimensional simplexes are homeomorphic, so we verified (ii). Finally observe
that if bi W S ! R is the i -th barycentric coordinate function then bi D pi ' 1 , so
every bi is continuous; this settles (iii) and completes our solution.
V.085. Let S be a simplex in Rn and suppose that S0  S1  : : :  Sk are distinct
faces of S . Prove that the points fb.S0 /; : : : ; b.Sk /g are independent. Here b.Si / is
the barycenter of the simplex Si for all i  k.
Solution. Choose an enumeration fa0 ; : : : ; am g of the vertices of S in such a way
that if T0 D a0 ; : : : ; am
; : : : ; Ti D ai ; : : : ; am
; : : : ; Tm D am
then every simplex
Si is listed in the sequence fT0 ; : : : ; Tm g. Since any subset of an independent set is
independent, it is sufficient to prove that the set fb.T0 /; : : : ; b.Tm /g is independent.
n
Let
P0m 2 R be the zero vector and suppose that we are given 0 ; : : : ; m 2 R such
that iD0 i D 0 and 0 b.T0 / C : : : C m b.Tm / D 0. Recalling that, for any i  m,
Pm
Pm
Pm
i
1
we have b.Ti / D miC1
j Di aj , we conclude that
iD0 miC1
j Di aj D 0
1
1
0 C : : : C mC1i
i
which shows that 0 a0 C : : : C m am D 0 where i D mC1
for every i P
m.
1
1
 D .mC1/ . mC1
0 /C: : :C.mC1i / . mC1i
i /C: : :C m
Note that m
Pm iD0 i Pm
and hence iD0 i D iD0 i D 0 which, together with independency of the set
fa0 ; : : : ; am g, implies that i D 0 for each i  m. An immediate consequence
is that i D 0 for all i  m and hence the set H D fb.T0 /; : : : ; b.Tm /g is
independent. We already saw that this implies that the set fb.S0 /; : : : ; b.Sk /g  H
is also independent.

2 Solutions of problems 001500

123

V.086. For a simplex S D a0 ; ; : : : ; am


, consider the family B.S / of all simplexes
of the form b.S0 /; : : : ; b.Sk /
, where S0  S1  : : :  Sk are distinct faces of S .
Prove that P is a simplicial subdivision of S such that any .m  1/-dimensional
simplex T 2 B.S / is a face of one or two m-dimensional members of B.S /
depending on whether T is contained in an .m  1/-dimensional face of S . The
subdivision B.S / is called the barycentric subdivision of the simplex S .
Solution. For any x 2 S we denote by i .x/ its i -th barycentric coordinate in S
for all i D 0; : : : ; m. Take a permutation P D fai0 ; : : : ; aim g of the set fa0 ; : : : ; am g
and let T0 D ai0 ; ai1 ; : : : ; aim
; : : : ; Tl D ail ; ailC1 ; : : : ; aim
; : : : ; Tm D aim
. Say
that the sequence T D fT0 ; : : : ; Tm g and the simplex C D b.T0 /; : : : ; b.Tm /
are
generated by the permutation P.
Pm
Given a point x 2 C there are nonnegative 0 ; : : : ; m such that
iD0 i D 1
P
m
1
and 0 b.T0 / C : : : C m b.Tm / D x. Recalling that b.Tl / D mlC1
j Dl aij for any
Pm
Pm
l
l  m, we conclude that lD0 mlC1 j Dl aij D x; an immediate consequence is
1
1
0 C : : : C mC1l
l for every l  m.
that x D 0 ai0 C : : : C m aim where l D mC1
Pm
1
1
l / C
Note that lD0 l D .m C 1/ . mC1 0 / C : : : C .m C 1  l/ . mC1l
Pm
Pm
: : : C m and hence lD0 l D lD0 l D 1 which shows that every l is the il -th
barycentric coordinate of x in S and, in particular, x 2 S . Thus
(1) for any permutation P D fai0 ; : : : ; aim g of the set fa0 ; : : : ; am g if the sequence
fT0 ; : : : ; Tm g and the simplex C D b.T0 /; : : : ; b.Tm /
are generated by P then
C  S and, for any x 2 C , if 0 ; : : : ; m are the barycentric coordinates of the
1
1
point x in C then il .x/ D mC1
0 C : : : C mC1l
l for every l  m.
Now take any B 2 B.S /; there are distinct faces S0  S1  : : :  Sk
of the simplex S such that B D b.S0 /; : : : ; b.Sk /
. It is easy to find a
permutation P D fai0 ; : : : ; aim g of the set fa0 ; : : : ; am g such that every Si is
listed in the sequence fT0 ; : : : ; Tm g generated by P and hence B  C D
b.T0 /; : : : ; b.Tm /
. Applying (1) we conclude that B  C 
S S . Since the
simplex B 2 B.S / was chosen arbitrarily, we proved that
B.S /  S .
To verify the rest of the properties of B.S / let us show that
(2) if P D fai0 ; : : : ; aim g is a permutation of the set fa0 ; : : : ; am g and C is the
element of B.S / generated by P then, a point x 2 S belongs to C if and only
if i0 .x/  i1 .x/  : : :  im .x/.
If x 2 C then it is easy to see that (1) implies the inequalities in (2). If, on the
other hand, we have a point x 2 S such that i0 .x/  i1 .x/  : : :  im .x/
then let 0 D .m C 1/ i0 .x/ and l D .m C 1  l/. il .x/  il1P.x// for any
l 2 f1; : : : ;P
mg. It is easy to check that l  0 for all l  m and m
lD0 i D 1

b.T
/
which
shows
that
x
2
C
and

is
the
l-th
barycentric
while x D m
l
l
lD0 l
coordinate of the point x for every l  m, so (2) is proved.
Now, if x 2 S then choose a permutation P D fai0 ; : : : ; aim g of the set
fa0 ; : : : ; am g such that i0 .x/  i1 .x/  : : :  im .x/ and apply (2) to see that
x belongsS
to the simplex C generated
S by P. Since C 2 B.S /, we established
that S  B.S / and hence S D B.S /. It is immediate from the definition
of B.S / that if B 2 B.S / then any face of B also belongs to B.S /.

124

2 Solutions of problems 001500

Take a permutation P D fai0 ; : : : ; aim g of the set fa0 ; : : : ; am g and let C be


the simplex generated by P. We have C D fx 2 S W i0 .x/  i1 .x/ 
: : :  im .x/g by the property (2). For any x 2 C let 0 .x/; : : : ; m .x/ be
the barycentric coordinates of x in C . If B is a face of C then there is a set
E  f0; : : : ; mg such that B D fx 2 C W l .x/ D 0 for all l 2 Eg. The
property (1) shows that 0 .x/ D 0 is equivalent to i0 .x/ D 0; if l > 0 then
l .x/ D 0 is equivalent to il .x/ D il1 .x/. Letting i1 .x/ D 0 for any
x 2 S we conclude that B D fx 2 C1 W il .x/ D il1 .x/ for all l 2 Eg.
Therefore
(3) a simplex B is a face of C if and only if there is a set E  f0; : : : ; mg such that
B D fx 2 C W il .x/ D il1 .x/ for any l 2 Eg.
Now assume that P1 D fai0 ; : : : ; aim g and P2 D faj0 ; : : : ; ajm g are permutations of the set fa0 ; : : : ; am g. If Ci is the simplex generated by Pi for every
i 2 f1; 2g then C1 D fx 2 S W il1 .x/  il .x/ for each l  mg and
C2 D fx 2 S W jl1 .x/  jl .x/ for each l  mg.
Given a face B2 of the simplex C2 , an immediate consequence of (3) is that
there exists a set E 0  f0; : : : mg for which B2 D fx 2 C2 W jl1 .x/ D jl .x/
for all l 2 E 0 g.
Fix a point x 2 C1 and l  m and take u; v  m with jl1 D iu and jl D iv ; it
is clear that u v. If u < v then jl1 .x/  jl .x/ is fulfilled automatically;
the property jl1 .x/ D jl .x/ holds if and only if il1 D il for all l 2
fu C 1; : : : ; vg.
If v < u then both conditions jl1 .x/  jl .x/ and jl1 .x/ D jl .x/ are
equivalent to the equalities il1 D il for all l 2 fv C 1; : : : ; ug. Carrying out
this procedure for every l  m we conclude that
(4) there is a set E  f0; : : : ; mg such that C1 \ B2 D fx 2 C1 W il .x/ D il1 .x/
for all l 2 Eg and, in particular, C1 \ B2 is a face of C1 .
For the general case suppose that B1 ; B2 2 B.S / have nonempty intersection
and choose permutations P1 D fai0 ; : : : ; aim g and P2 D faj0 ; : : : ; ajm g of the set
fa0 ; : : : ; am g such that Bi is a face of the simplex Ci generated by Pi for each
i 2 f1; 2g. The condition (2) implies that C1 D fx 2 S W i0 .x/  : : :  im .x/g.
Apply the property (4) to see that we can find a set E2  f0; : : : ; mg such that
C1 \ B2 D fx 2 C1 W il .x/ D il1 .x/ for all l 2 E2 g.
The property (3) demonstrates that there exists a set E1  f0; : : : ; mg for which
B1 D fx 2 C1 W il .x/ D il1 .x/ for all l 2 E1 g. It is straightforward to check that
B1 \ B2 D fx 2 C1 W il .x/ D il1 .x/ for all l 2 E1 [ E2 g, so we can apply (3) to
conclude that B1 \ B2 is a face of C1 . Since C1 and C2 are in a symmetric situation,
the set B1 \ B2 is also a face of C2 . It is evident that any face of Ci contained in Bi
is a face of Bi for every i 2 f1; 2g, so B1 \ B2 is a common face of B1 \ B2 . This
proves that B.S / is a simplicial subdivision of S .
To check the last promise for the family B.S / take an .m  1/-dimensional T 2
B.S / such that T D b.S0 /; : : : ; b.Sm1 /
for some faces S0  : : :  Sm1 of the
simplex S ; fix a permutation fai0 ; : : : ; aim g of the set fa0 ; : : : ; am g such that every
Sl is listed in the sequence fail ; : : : ; aim
W l 2 f0; : : : ; mgg

2 Solutions of problems 001500

125

If S0 S then Sl D ailC1 ; : : : ; aim


for every l 2 f0; : : : ; m  1g; in particular,
T is contained in S0 which is an .m  1/-dimensional face of S . Since no simplex
other than S can have S0 as its proper face, T is a face of the unique m-dimensional
simplex B D b.S /; b.S0 /; : : : ; b.Sm1 /
of the family B.S /.
Now, if S0 D S then b.S / D b.S0 / 2 T ; it is easy to see that b.S / does not
belong to any .m  1/-dimensional face of S , so the simplex T is not contained in
any .m  1/-dimensional face of S . We have two cases to consider.
1) Sm1 D am1 ; am
and hence Sl D ail ; : : : ; aim
for any l  m  1. It is clear
that the simplexes b.S0 /; : : : ; b.Sm1 /; b.am1
/
and b.S0 /; : : : ; b.Sm1 /;
b.am
/
are two distinct m-dimensional elements of B.S / such that T is their
common face. It is straightforward that there are no other possibilities to add to
the sequence T D fS0 ; : : : ; Sm1 g a face F of S in such a way that the sequence
T [ F be decreasing.
2) There exists a number l 2 f0; : : : ; m  2g such that Sl D ail ; : : : ; aim

and SlC1 D ailC2 ; : : : ; aim


. If S 0 D ail ; ailC2 ; : : : ; aim
and S 00 D
ailC1 ; ailC2 ; : : : ; aim
then it is evident that the simplexes b.S0 /; : : : ; b.Sl /;
b.S 0 /; b.SlC1 /; : : : ; b.Sm1 /
and b.S0 /; : : : ; b.Sl /; b.S 00 /; b.SlC1 /; : : : ;
b.Sm1 /
are the m-dimensional elements of B.S / whose common face is T .
It is easy to see that there are no other possibilities to add to the sequence
T D fS0 ; : : : ; Sm1 g a face F of S in such a way that the sequence T [ F be
decreasing.
This proves that an .m  1/-dimensional simplex T 2 B.S / is contained in an
.m  1/-dimensional face of S (we saw that this happens if and only if the first
element of the sequence that determines T is distinct from S ) if and only if T is
a face of exactly one m-dimensional element of B.S /. If S is not contained in an
.m  1/-dimensional face of S (we saw that this happens if and only if the first
element of the sequence that determines T coincides with S ) then T is the common
face of exactly two distinct elements of B.S /.
V.087. Given a simplex S , let B1 .S / be the barycentric
subdivision of S . If Bn .S /
S
is a simplicial subdivision of S , let BnC1 .S / D fB1 .T / W T 2 Bn .S /g. The family
Bn .S / is called the n-th barycentric subdivision of the simplex S . Prove that, for
any simplex S and any " > 0, there exists a natural number n such that the mesh of
the n-th barycentric subdivision of the simplex S is less than ".
q
Solution. For any n 2 N and x D .x1 ; : : : ; xn / 2 Rn let jjxjjn D x12 C : : : C xn2 .
If n is clear we will write jjxjj instead of jjxjjn . Given x; y 2 Rn , it is evident
that jjx  yjjn is the distance between the points x and y. For any set A  Rn let
diam.A/ D supfjjx  yjjn W x; y 2 Ag. If T is a simplex then b.T / is its barycenter.
Fact 1. Let T D a0 ; : : : ; am
 Rn be an m-dimensional simplex. Then for any
points x; y such that x 2 Rn and y 2 T there is k  m for which jjx  yjj 
jjx  ak jj. In particular, diam.T / D maxfjjai  aj jj W i; j 2 f0; : : : ; mgg.

126

2 Solutions of problems 001500

Proof. Take k 2 f0; : : : ; mg such that jjx  aiP


jj  jjx  ak jj for any P
i  m and
m
m
choose numbers 0 ; : : :P
; m 2 0; 1
such
that

D
1
and
y
D
i
iD0
iD0 i ai .
Pm
Pm
m
Then jjx  yjj D jjx 

a
jj
D
jj
.
x


a
/jj



jjx
 ai jj;
i
i
i
i
i
i
iD0
iD0
P iD0
P
m
therefore jjx  yjj  m

jjx

a
jj


jjx

a
jj

jjx

a
jj,
so
k is as
i
i
i
k
k
iD0
iD0
promised. Applying twice this part of our Fact we conclude that, for any x; y 2 T ,
there are k; l 2 f0; : : : ; mg such that jjx  yjj  jjal  ak jj; as a consequence,
diam.T /  d D maxfjjai  aj jj W i; j 2 f0; : : : ; mgg. The opposite inequality is
evident, so diam.T / D d and hence Fact 1 is proved.
Fact 2. Given an m-dimensional simplex T D a0 ; : : : ; am
 Rn , the mesh of its
m
barycentric subdivision B.T / does not exceed mC1
diam.T /.
Proof. Take any simplex B 2 B.T /; there are faces T0  : : :  Tk of the simplex T
such that B D b.T0 /; : : : ; b.Tk /
. Fix j; l 2 f0; : : : ; kg with j < l. Since Tl  Tj ,
there is a permutation fai0 ; : : : ; aim g of the set fa0 ; : : : ; am g and p; q 2 f0; : : : ; mg
such that p < q while Tl D ai0 ; : : : ; aip
and Tj D ai0 ; : : : ; aiq
.
1
For any r  p the equality jjair  b.Tj /jj D jjair  qC1
.ai0 C : : : C aiq /jj shows
P
q
1
1
that jjair b.Tj /jj D qC1 jj hD0 .air aih /jj  qC1 q diam.T / (the last inequality
holds because the summand for h D r is equal to zero). This shows that, for any
m
r  p, we have jjair  b.Tj /jj  mC1
diam.T /. Now apply Fact 1 to see that there
m
is r  q such that jjb.Tj /  b.Tl /jj  jjair  b.Tl /jj  mC1
diam.T /. Since j; l 2
f0; : : : ; kg were chosen arbitrarily, we can apply Fact 1 once more to conclude that
m
diam.B/  mC1
diam.T /. Thus every element of B.T / has diameter not exceeding
m
m

diam.T
/
and
hence the mesh of B.T / does not exceed mC1
diam.T /, i.e.,
mC1
Fact 2 is proved.
Returning to our solution suppose that S D a0 ; : : : ; am
and " > 0. It is a
m n
/ diam.T / for
consequence of Fact 2 that the mesh of Bn .S / does not exceed . mC1
m n
any n 2 N. Since . mC1 / diam.T / ! 0 as n ! 1, there exists n 2 N such that
m n
. mC1
/ diam.T / < " and hence the mesh of Bn .S / is less than ".
V.088 (Sperners lemma). Given a number l 2 N and an m-dimensional simplex
a0 ; : : : ; am
let V be the set of all vertices of simplexes in Bl .a0 ; : : : ; am
/. Suppose
that, for a function h W V ! f0; 1; : : : ; mg, we have h.v/ 2 fi0 ; : : : ; ik g whenever
v 2 ai0 ; : : : ; aik
. Prove that the family of simplexes in Bl .a0 ; : : : ; am
/, on vertices
of which h takes all values from 0 to m, has an odd cardinality.
Solution. All simplexes are subspaces of some Rp ; in most cases, the power p
is not mentioned since we make no use of it. The symbol 0 is used to denote the
respective zero vector of Rp . Given a simplex S , the point b.S / is its barycenter
and B.S / is the barycentric subdivision
of S . We also let B1 .S / D B.S / and, if
S
Bk .S / is defined, then BkC1 .S / D fB.T / W T 2 Bk .S /g.
Fact 1. Assume that S D d0 ; : : : ; dn
is an n-dimensional simplex. If a simplex
T D c0 ; : : : ; ck
is contained in S then k  n.
Proof. Suppose that k > n and let i0 ; : : : ; in be the barycentric coordinates of ci
in S for any i  k. Considering the .n C 1/-tuple vi D . i0 ; : : : ; in / to be a vector

2 Solutions of problems 001500

127

of the space RnC1 for every i  k, observe that the family M D fvi W i  kg has
k C 1 vectors; since k C 1 > n C 1, the family
independent,
P M cannot be linearly
P
so we can choose 0 ; : : : ; k 2 R such that kiD0 i2 0 and kiD0 i vi D 0.
P
P
P
Thus kiD0 i ij D 0 for every j  n and hence A D nj D0 kiD0 i ij D 0.
Pn
P
Pk
It is immediate that A D iD0 i . j D0 ij / D kiD0 i D 0 (we used the fact
P
that nj D0 ij D 1 for every i  k), so the set fc0 ; : : : ; ck g is not independent; since
only independent sets can span a simplex, we obtained a contradiction which shows
that Fact 1 is proved.
Fact 2. Given an arbitrary simplex S D d0 ; : : : ; dn
and a point x 2 S , denote
by i .x/ the i -th barycentric coordinate of x in S for all i D 0; : : : ; n. Fix a
permutation P D fdi0 ; : : : ; din g of the set fd0 ; : : : ; dn g and consider the simplexes
T0 D di0 ; di1 ; : : : ; din
; : : : ; Tl D dil ; dilC1 ; : : : ; din
; : : : ; Tn D din
. Then the
simplex C D b.T0 /; : : : ; b.Tn /
is contained in S and, for any x 2 C , if 0 ; : : : ; n
1
are the barycentric coordinates of the point x in C then il .x/ D nC1
0 C : : : C
1
for every l  n. We will say that the sequence fT0 ; : : : ; Tn g and the simplex
nC1l l
C are generated by the permutation P.
Pn
Proof. Given a point x 2 C there are nonnegative 0 ; : : : ; n such that
iD0 i D 1
P
n
1
and 0 b.T0 / C : : : C n b.Tn / D x. Recalling that b.Tl / D nlC1
d
j Dl ij for any
Pn
P
l
d
D
x;
an
immediate
consequence
is
l  n, we conclude that nlD0 nlC1
j Dl ij
1
1
that x D 0 di0 C : : : C n din where l D nC1 0 C : : : C nC1l l for every l  n.
P
1
1
0 / C : : : C .n C 1  l/ . nC1l
l / C : : : C n
Note that nlD0 l D .n C 1/ . nC1
Pn
Pn
and hence lD0 l D lD0 l D 1 which shows that l D il .x/ for every l  n,
so x 2 S and hence Fact 2 is proved.
Fact 3. Given a simplex S D d0 ; : : : ; dn
and a point x 2 S , denote by i .x/ the
i -th barycentric coordinate of x in S for all i D 0; : : : ; n. If P D fdi0 ; : : : ; din g is
a permutation of the set fd0 ; : : : ; dn g and C is the element of B.S / generated by P
then a point x 2 S belongs to C if and only if i0 .x/  i1 .x/  : : :  in .x/.
Proof. If x 2 C then it is easy to see that Fact 1 implies our inequalities. If, on
the other hand, we have a point x 2 S such that i0 .x/  i1 .x/  : : :  in .x/
then let 0 D .n C 1/ i0 .x/ and l D .n C 1  l/. il .x/  il1
P.x// for any
l 2 f1; : : : ;P
ng. It is easy to check that l  0 for all l  n and nlD0 i D 1
while x D nlD0 l b.Tl / which shows that x 2 C and l is the l-th barycentric
coordinate of the point x in C for every l  n, so Fact 3 is proved.
Fact 4. Given a simplex S D d0 ; : : : ; dn
and a point x 2 S , denote by i .x/
the i -th barycentric coordinate of x in S for all i D 0; : : : ; n. Take a permutation
P D fdi0 ; : : : ; din g of the set fd0 ; : : : ; dn g and let C be the simplex generated by P.
Then a simplex B is a face of C if and only if there is a set E  f0; : : : ; ng such that
B D fx 2 C W il .x/ D il1 .x/ for any l 2 Eg; here i1 .x/ D 0 for any x 2 S .
Proof. Fact 3 shows that C D fx 2 S W i0 .x/  i1 .x/  : : :  in .x/g. For
any point x 2 C let 0 .x/; : : : ; n .x/ be the barycentric coordinates of x in C .

128

2 Solutions of problems 001500

A set B is a face of C if and only if there exists a set E  f0; : : : ; ng such that
B D fx 2 C W l .x/ D 0 for all l 2 Eg. It follows from Fact 2 that 0 .x/ D 0 is
equivalent to i0 .x/ D 0; if l > 0 then l .x/ D 0 is equivalent to il .x/ D il1 .x/.
Therefore B D fx 2 C W il .x/ D il1 .x/ for all l 2 Eg and Fact 4 is proved.
Fact 5. Given a simplex S D d0 ; : : : ; dn
and B 2 B.S / there exists a permutation
P D fdi0 ; : : : ; din g of the set fd0 ; : : : ; dn g such that B is a face of the simplex
generated by P. In particular, every element of B.S / is a face of an n-dimensional
element of B.S /.
Proof. By definition of the family B.S / there exist faces S0  : : :  Sk of the
simplex S such that B D b.S0 /; : : : ; b.Sk /
. Take a permutation P D fdi0 ; : : : ; din g
of the set fd0 ; : : : ; dn g such that every simplex Si is listed in the sequence
T0 D di0 ; di1 ; : : : ; din
; : : : ; Tl D dil ; dilC1 ; : : : ; din
; : : : ; Tn D din
:
It is clear that B is a face of the n-dimensional simplex C generated by the
permutation P, so Fact 5 is proved.
Fact 6. For any simplex S , suppose that B 2 B.S / and S 0 is a face of S such that
B \ S 0 ;; then B \ S 0 is a face of B and hence B \ S 0 2 B.S /.
Proof. By Fact 5, there is a permutation P D fdi0 ; : : : ; din g of the set fd0 ; : : : ; dn g
such that B is a face of the simplex C generated by P. By Fact 4 we can find a set
E  f0; : : : ng for which B D fx 2 C W il1 .x/ D il .x/ for all l 2 Eg.
There is a set E1  f0; : : : ; ng such that S 0 D fx 2 S W il .x/ D 0 for all l 2
E1 g. If p is the maximal element of E1 then B \ S 0 D fx 2 C W il .x/ D il1 .x/
for all l 2 f0; : : : ; pg [ Eg. Applying Fact 4 again we conclude that B \ S 0 is a face
of C ; any face of C contained in B is, evidently, a face of B, so Fact 6 is proved.
Fact 7. For any simplex S and k 2 N, the family Bk .S / is a simplicial subdivision
of S .
Proof. For k D 1 this was proved in Problem 086. Proceeding inductively,
assume
S
that Bk .S / of S is simplicial. It is immediate from the definition that BkC1 .S / D
S and, for any B 2 BkC1 .S /, all faces of B also belong to BkC1 .S /.
Now, if B1 ; B2 2 BkC1 .S / and B1 \ B2 ; then there are P1 ; P2 2 Bk .S / such
that Bi 2 B.Pi / for each i 2 f1; 2g. If P1 D P2 then B1 \ B2 is a common face of
B1 and B2 by Problem 086. If P1 P2 then P D P1 \ P2 is a common face of
P1 and P2 by the induction hypothesis. It follows from Fact 6 that Bi \ P 2 B.P /
and Bi \ P is a face of Bi for every i D 1; 2. Therefore we can apply Problem 086
once more to see that .B1 \ P / \ .B2 \ P / D B1 \ B2 is a common face of
B1 \ P and B2 \ P . It is clear that the common face of B1 \ P and B2 \ P is also a
common face of B1 \B2 , so we proved that BkC1 .S / is also a simplicial subdivision
of S . Therefore Bk .S / is a simplicial subdivision of S for any k 2 N, i.e., Fact 7 is
proved.

2 Solutions of problems 001500

129

Fact 8. If S is a simplex and k 2 N then for any face S 0 of the simplex S and any
P 2 Bk .S /, the set P \ S 0 is a face of P and hence P \ S 0 2 Bk .S 0 /.
Proof. Fact 6 says that our statement is true for k D 1. Proceeding by induction
assume that we proved (6) for any k  l and take a simplex P 2 BlC1 .S / and a
face S 0 of the simplex S . There is B 2 Bl .S / such that P 2 B.B/. By the induction
hypothesis, the set B 0 D B \ S 0 is a face of B, so we can apply Fact 6 again to see
that P \ S 0 D P \ B 0 is a face of P . An immediate consequence of Fact 7 is that
P \ S 0 2 BlC1 .S /, so our induction procedure shows that our statement is true for
any k 2 N, i.e., Fact 8 is proved.
Fact 9. Given k 2 N and a simplex S if a simplex B 2 Bk .S / is contained in
an .n  1/-dimensional face of S then there is a unique n-dimensional simplex
C 2 Bk .S / such that B is a face of C .
Proof. For k D 1 this statement was proved in Problem 086. Proceeding inductively
assume that we proved our Fact for all k  l and take an .n1/-dimensional simplex
B 2 BlC1 .S / which is contained in an .n  1/-dimensional face S 0 of the simplex
S . By Fact 5, there is an n-dimensional simplex E 2 Bl .S / such that B 2 B.E/.
By Fact 8, the set E 0 D E \ S 0 is a face of E; since B  E 0 , it follows from Fact 1
that the dimension of E 0 is at least n  1. Apply Fact 1 again to see that the simplex
E 0 cannot be n-dimensional because S 0 is .n  1/-dimensional and E 0  S 0 .
Therefore E 0 is an .n  1/-dimensional face of E; by the induction hypothesis
it is not a face of any other n-dimensional element of Bl .S /. Since B  E 0 , we
can apply Problem 086 to convince ourselves that there is a unique n-dimensional
simplex B 0 2 B.E/ such that B is a face of B 0 . We must show that B cannot be a
face of any other n-dimensional element of BlC1 .S /.
Assume, toward a contradiction, that there exists an n-dimensional simplex C 2
BlC1 .S /nfB 0 g such that B is a face of C . We saw already that C B.E/, so
there is an n-dimensional simplex D 2 Bl .S /nfEg such that C 2 B.D/. As a
consequence, ; B  D \ E and hence the simplex D 0 D D \ E is the common
.n  1/-dimensional face of D and E (see Fact 7). Since E 0 is not a face of D, the
simplexes D 0 and E 0 are distinct. We have ; B  E 0 \ D 0 , so E 0 \ D 0 is the
common face of E 0 and D 0 by Fact 7. Apply Fact 1 once more to see that E 0 \ D 0
is an .n  1/-dimensional face of both E 0 and D 0 . Since the unique p-dimensional
face of a p-dimensional simplex is the simplex itself, we have D 0 D E 0 which is a
contradiction. Thus B is a face of a unique n-dimensional element of BlC1 .S /, so
the induction step is accomplished and hence Fact 9 is proved.
Fact 10. Given k 2 N and a simplex S if a simplex B 2 Bk .S / is not contained
in any .n  1/-dimensional face of S then there are exactly two n-dimensional
simplexes C1 ; C2 2 Bk .S / such that B is a common face of C1 and C2 .
Proof. For k D 1 this statement was proved in Problem 086. Proceeding inductively
assume that we proved our Fact for all k  l and take an .n  1/-dimensional
simplex B 2 BlC1 .S / which is not contained in any .n  1/-dimensional face S 0 of

130

2 Solutions of problems 001500

the simplex S . By Fact 5, there is an n-dimensional simplex E 2 Bl .S / such that


B 2 B.E/. We have two possible cases.
Case 1. The simplex B is contained in an .n  1/-dimensional face E 0 of the
simplex E. By Problem 086, there is a unique n-dimensional simplex C1 2
B.E/ such that B is a face of C1 . The simplex E 0  B is not contained in any
.n  1/-dimensional face of S , so we can apply the induction hypothesis to see
that it is a common face of exactly two n-dimensional elements of Bl .S /. Fix
an n-dimensional simplex D 2 Bl .S /nfEg such that E 0 is also a face of D.
Since our simplex B is also contained in the .n  1/-dimensional face E 0 of the
simplex D, there is a unique n-dimensional simplex C2 2 B.D/ such that B is a
face of C2 . Observe that C1 C2 because otherwise C1 D C1 \ C2  E 0 which
contradicts Fact 1. Thus C1 and C2 are distinct n-dimensional elements of BlC1 .S /
such that B is the common face of C1 and C2 .
Assume, toward a contradiction, that there exists an n-dimensional simplex
C3 2 BlC1 .S /nfC1 ; C2 g such that B is also a face of C3 . We saw already that
C B.E/ [ B.D/, so there is an n-dimensional simplex F 2 Bl .S /nfE [ Dg such
that C3 2 B.F /. As a consequence, ; B  F \ E and hence F 0 D F \ E is the
common .n  1/-dimensional face of F and E (see Fact 7 and Fact 1). Analogously,
F 00 D F \ D is the common .n  1/-dimensional face of F and D. It follows
from B  F 0 \ F 00 and Fact 1 that F 0 \ F 00 is a common .n  1/-dimensional
face of F 0 and F 00 . Therefore F 0 D F 0 \ F 00 D F 00 which shows that F 0 is the
common .n  1/-dimensional face of the three distinct simplexes E; D; F 2 Bl .S /
which is a contradiction with the induction hypothesis. Thus our induction step is
accomplished if Case 1 takes place.
Case 2. The simplex B is not contained in any .n  1/-dimensional face of the
simplex E. By Problem 086, there are exactly two n-dimensional simplexes
C1 2 B.E/ such that B is the common face of C1 and C2 .
Assume, toward a contradiction, that there exists an n-dimensional simplex C3 2
BlC1 .S /nfC1 ; C2 g such that B is also a face of C3 . We saw already that C3 B.E/,
so there exists an n-dimensional simplex F 2 Bl .S /nfEg such that C3 2 B.F /.
As a consequence, ; B  F \ E and hence F 0 D F \ E is the common
.n  1/-dimensional face of F and E (see Fact 7 and Fact 1). It turns out that the
simplex B  F 0 is contained in an .n  1/-dimensional face F 0 of the simplex E;
this contradiction shows that our induction step is accomplished in Case 2 as well,
so Fact 10 is proved.
Returning to our solution let S D a0 ; : : : ; am
; assume first that m D 1 and
hence S D a0
D fa0 g. Then Bl .S / D fS g; the simplex S has the unique vertex
a0 and h.a0 / D a0 by our assumption about the function h. Therefore the relevant
number of simplexes is 1, i.e., our solution is carried out for m D 1.
Proceeding by induction assume that we proved our statement for all m < n and
take an n-dimensional simplex S D a0 ; : : : ; an
and a function h with values in
f0; : : : ; ng defined on the set V of all vertices of Bl .S / such that h.v/ 2 fi0 ; : : : ; ik g
whenever v 2 ai0 ; : : : ; aik
.

2 Solutions of problems 001500

131

Let S D fS1 ; : : : ; Sp g be a faithfully enumerated family of all n-dimensional


simplexes from Bl .S /. For any number i 2 N let Mi D f1; : : : ; i g; for any simplex
T 2 Bl .S / let V .T / be the set of its vertices. Call a simplex T 2 S marked if
h.V .T // D f0; : : : ; ng. We must prove that the cardinality q of the family of marked
simplexes from S is an odd number.
We will say that an .n  1/-dimensional simplex T 2 Bl .S / is adequate if
h.V .T // D f0; : : : ; n  1g. Let T be the family of all .n  1/-dimensional simplexes
from Bl .S /. If we consider the face S 0 D a0 ; : : : ; an1
of the simplex S then it
takes a trivial induction to prove that Bi .S 0 /  Bi .S / for any i 2 N, so Bl .S 0 / 
Bl .S /. It follows from Fact 8 that if T 2 Bl .S / and T  S 0 then T 2 Bl .S 0 /. Thus
the family T 0 D fT 2 T W T  S 0 g consists of all .n  1/-dimensional elements
of Bl .S 0 /.
Let g be the restriction of our function h to the set W of all vertices of the
simplexes from Bl .S 0 /. Then g W W ! f0; : : : ; n  1g and g.v/ 2 fi0 ; : : : ; ik g
whenever v 2 ai0 ; : : : ; aik
. Therefore we can apply the induction hypothesis to see
that the cardinality s of the family E D fT 2 T 0 W g.V .T // D f0; : : : ; n  1gg is an
odd number.
Given i 2 Mp let wi be the number of adequate faces of Si . If Si is marked
then it has only one adequate face, i.e., wi D 1. If a non-marked simplex Si has
an adequate face then we can consider that Si D d0 ; : : : ; dn
and the enumeration
fd0 ; : : : ; dn g of the vertices of Si is chosen in such a way that h.di / D i for any
i  n  1. The simplex Si being non-marked, we have h.dn / D j 2 f0; : : : ; n  1g
and hence the face d0 ; : : : ; dj 1 ; dj C1 ; : : : ; dn
of the simplex Si is also adequate.
It is easy to see that Si has no other adequate faces, so wi D 2. This proves that
(1) if a simplex Si is marked then wi D 1; if Si is not marked then either wi D 0
or wi D 2.
Pp
An immediate consequence of (1) is that the parity of the number w D iD1 wi
coincides with the parity of q, i.e., w  q D 2t for some t 2 N.
Next observe that it follows from our assumption about the function h that the
unique .n  1/-dimensional face of S that can contain an adequate simplex is S 0 .
Therefore an adequate simplex T is either contained in S 0 or not contained in any
.n  1/-dimensional face of S . In the first case T is a face of a unique element of S
by Fact 9; in the second case T is the common face of exactly two simplexes from
S (see Fact 10).
Let u be the number of adequate simplexes T 2 T which are not contained in S 0 .
For any i 2 Mp let w0i be the number of the faces
Pp of Si which belong to E. It is clear
that either w0i D 1 or w0i D 0 and hence s D iD1 w0i . SinceP
every adequate simplex
p
T 2 T nE belongs to exactly two elements of S, in the sum iD1 .wi  w0i / D w  s
every adequate T 2 T nE is counted twice, so w  s D 2u. Therefore q D w  2t D
s C 2.u  t / is an odd number. This accomplishes the induction step of our proof
and completes our solution.

132

2 Solutions of problems 001500

V.089 (Browers fixed-point theorem). Prove that, for any n 2 N, if S is an


n-dimensional simplex and f W S ! S is a continuous function then there exists a
point x 2 S such that f .x/ D x.
Solution. All simplexes are subspaces of some Rp with the respective metric; the
power p is not mentioned since we make no use of it. If T is a simplex then V .T /
is the set of vertices of T and B.T / is the barycentric subdivision
of T . We also let
S
B1 .T / D B.T / and, if Bk .T / is defined, then BkC1 .T / D fB.T / W T 2 Bk .T /g.
Fact 1. Given a simplex T D a0 ; : : : ; am
suppose that Fi is a closed subset of
T for any i D 0; : : : ; m. If, additionally, ai0 ; : : : ; aik
 Fi0 [ : : : [ Fik for any
i0 ; : : : ; ik 2 f0; : : : ; mg then F0 \ : : : \ Fm ;.
Proof. Consider the set Ui D T nFi for any i  m; if F0 \ : : : \ Fn D ;
then the family U D fUi W i  mg is an open cover of the compact space T
(see Problem 084), so we can apply TFS-244 to find a number > 0 such that
any set of diameter not exceeding is contained in one of the elements of U .
Apply Problem 087 to find k 2 N such that every element of the k-th barycentric
subdivision Bk .T / of the simplex T has diameter less than . Let V be the set of
vertices of the elements of Bk .T /.
For each v 2 V let Tv be the intersection of all faces of T which contain v. It is
evident that Tv is still a face of T ; therefore there are ai0 ; : : : ; ail 2 fa0 ; : : : ; am g
such that Tv D ai0 ; : : : ; ail
. Since Tv  Fi0 [ : : : [ Fil , there is j  l such that
v 2 Fij ; let h.v/ D ij .
This gives us a map h W V ! f0; : : : ; mg such that v 2 Fh.v/ and ah.v/ is one of
the vertices of Tv for any v 2 V . Consequently, if T 0 is a face of T and v 2 T 0 then
Tv  T 0 and hence ah.v/ 2 V .Tv / is also a vertex of T 0 . This shows that we can
apply Problem 088 to find a simplex B 2 Bk .T / such that h.V .B// D f0; : : : ; mg.
For any i  m there is v 2 V .B/ such that h.v/ D i and hence v 2 Fi ; therefore
B \ Fi ; and hence B is not contained in Ui for each i  m. This contradiction
with the choice of shows that F0 \ : : : \ Fm ;, i.e., Fact 1 is proved.
Returning to our solution let a0 ; : : : ; an be the vertices of the simplex S ; then
S D a0 ; : : : ; an
. Denote by i .x/ the i -th barycentric coordinate of x in S for any
i  n. The set Fi D fx 2 S W i .f .x//  i .x/g is closed in S for any i  n; let
us check that the family fF0 ; : : : ; Fn g satisfies the premises of Fact 1.
Indeed,
0 for all k P
fi0 ; : : : ; im g and
P if x 2 ai0 ; : : : ; aim
then k .x/PD
m
m
hence m
j D0 ij .x/ D 1 which shows that
j D0 ij .f .x// 
j D0 ij .x/ and
hence there exists j  m such that ij .f .x//  ij .x/, i.e., x 2 Fij . This proves
that ai0 ; : : : ; aim
 Fi0 [ : : : [ Fim and therefore we can apply Fact 1 to conclude
that F D F0 \ : : : \ Fn ;. IfP
x 2 F then iP
.f .x//  i .x/ for any i  n;
n
this, together with the equalities niD1 i .x/ D
iD1 i .f .x// D 1 shows that
i .x/ D i .f .x// for every i  n, so f .x/ D x and hence the point x 2 S is as
promised.
V.090. Prove that, for any n 2 N, there is no retraction of the cube In onto its
boundary @In D fx 2 In W jx.i /j D 1 for some i < ng.

2 Solutions of problems 001500

133

Solution. For any m 2 N denote by 0m the zero point of Rm and let eim 2 Rm
be the point defined by eim .i / D 1 and eim .j / D 0 for all j < m with j i .
m
We
qPconsider the space R with the usual metric dm defined by dm .x; y/ D
m1
m
m
2
iD0 .x.i /  y.i // for any points x; y 2 R . Besides, Bm .x; s/ D fy 2 R W
dm .x; y/ < sg is the ball of radius s centered at x.
P
The set Em D fx 2 Rm W x.i /  0 for all i < m and m1
iD0 x.i /  1g
m
is easily seen to be the simplex spanned by the points 0m ; e0m ; : : : ; em1
. For any
m-dimensional simplex S we denote by @S the union of all .mP
 1/-dimensional
faces of S . Then @Em D fx 2 Em W x.0/ : : : x.m  1/ D 0 or m1
iD0 x.i / D 1g.
Fact 1. For any m 2 N there exists a homeomorphism h W Em ! Im such that
h.@Em / D @Im .
Proof. Define a point v0 2 Rm by requiring that v0 .i / D 1 for all i < m; the map
f W Rm ! Rm , defined by f .x/ D 2x C v0 for any x 2 Rm , is a homeomorphism.
m
Let I D fx 2 RQ
W 0  x.i /  1 for all i < mg. Then f .I / D Im and, for the set
m1
@I D fx 2 I W iD0 x.i /.x.i /  1/ D 0g, we have f .@I / D @Im . Thus it suffices
to construct a homeomorphism g W E m ! I such that g.@Em / D @I .
The geometric idea of construction of g is very simple: for any point u 2 Rm
with u.i /  0 for all i < m we define g on the ray Ru D ft u W t 2 R; t  0g as
the unique linear map for which g.Au / D Bu where Au and Bu are the intersection
points of Ru with @Em and @I respectively.
P
To do it formally, let s.x/ D
i<m x.i / and m.x/ D maxi<m x.i / for any
x 2 Rm . Now, define g by requiring that g.0m / D 0m and, if x 2 Em nf0m g then
s.x/
g.x/ D m.x/
x. It is trivial that g W Em ! I is a bijection which is continuous
at every point x 2 Em nf0m g and g.@Em / D @I . However, the map g is also
continuous at 0m because g.Bm .0m ; ms //  Bm .0m ; s/ for any s > 0. Thus g is
a homeomorphism, so h D f g W Em ! Im is the promised homeomorphism.
Fact 1 is proved.
Fact 2. Given a simplex S D a0 ; : : : ; am
denote by Si the .m  1/-dimensional
face of S with ai Si and let Fi be
S a closed subspace of S such that Fi \Si D ; for
any i  m. If, additionally, S D fFi W i 2 f0; : : : ; mgg then F0 \ : : : \ Fm ;.
Proof. Take a face B D ai0 ; : : : ; aik
of the simplex S . If Fi \ B ; then we
must have i 2 fi0 ; : : : ; ik g because otherwise B  Si and hence B \ Fi D ;. As
a consequence, B  Fi0 [ : : : [ Fik , so we can apply Fact 1 of V.089 to conclude
that F0 \ : : : \ Fm ; and hence Fact 2 is proved.
Fact 3. Given a simplex S D a0 ; : : : ; am
there is no retraction r W S ! @S .
Proof. Suppose that r W S ! @S is a retraction, i.e., r is continuous and r.x/ D x
for any x 2 @S . For any i 2 f0; : : : ; mg denote by Si the .m  1/-dimensional face
of S for which ai Si . It is easy to see that S0 \ : : : \ Sm D ;, so the family
f@S nSi W i  mg is an open cover of @S ; since also S0 [ : : : [ Sm D @S , we have
T
1
im .@S nSi / D ;. Let Ui D r .@S nSi / for every i  m; then fU0 ; : : : ; Um g is
an open cover of S such that U0 \ : : : \ Um D ;. The map r being a retraction, the
equality Ui \ Si D ; holds for any i  m.

134

2 Solutions of problems 001500

Apply S
Fact 2 of S.226 to choose a family F0 ; : : : ; Fm of closed subsets of S such
that S D fFi W i  mg and Fi  Ui for every i  m. Thus Fi \Si 
TUi \Si D ;
for every i T
 m, so we can apply Fact 2 to convince ourselves that im Fi ;
and hence im Ui ;; this contradiction shows that Fact 3 is proved.
Returning to our solution suppose that there is a retraction r W In ! @In . Apply
Fact 1 to find a simplex S and a homeomorphism h W S ! In such that h.@S / D @In .
It is straightforward that r0 D h1 r h W S ! @S is a retraction; this contradiction
with Fact 3 shows that there exists no retraction of In onto @In .
V.091. Given spaces X and Y and functions f; g 2 C.X; Y /, let f  g denote
the fact that f and g are homotopic. Prove that  is an equivalence relation
on C.X; Y /.
Solution. For any function f 2 C.X; Y / let F .x; t / D f .x/ for any t 2 0; 1
. It
is clear that F W X  0; 1
! Y is continuous; since F .x; 0/ D F .x; 1/ D f .x/
for any x 2 X , we proved that f  f .
Now assume that f; g 2 C.X; Y / and f  g; there exists a continuous map
F W X  0; 1
! Y such that F .x; 0/ D f .x/ and F .x; 1/ D g.x/ for any x 2 X .
Define a function G W X  0; 1
! Y by the equality G.x; t / D F .x; 1  t / for any
x 2 X and t 2 0; 1
. It is immediate that G is continuous while G.x; 0/ D g.x/
and G.x; 1/ D f .x/ for any x 2 X . Therefore g  f .
Finally assume that f; g; h 2 C.X; Y / and f  g  h. There exists continuous
functions F; G W X  0; 1
! Y such that F .x; 0/ D f .x/; F .x; 1/ D G.x; 0/ D
g.x/ and G.x; 1/ D h.x/ for any x 2 X . Define a map H1 W X  0; 12
! Y by the
formula H1 .x; t / D F .x; 2t / for any x 2 X and t 2 0; 12
. We will also need the
map H2 W X  12 ; 1
! Y defined by H2 .x; t / D G.x; 2t  1/ for any x 2 X and
t 2 12 ; 1
. It is clear that both maps H1 and H2 are continuous.
Observe that H1 .x; 12 / D H2 .x; 12 / D g.x/ for any x 2 X ; this, together with
Fact 2 of T.354 shows that the map H D H1 [ H2 is continuous. Recall that the
map H W X  0; 1
! Y is defined by H.x; t / D H1 .x; t / if x 2 X; t  12
and H.x; t / D H2 .x; t / whenever x 2 X and t  12 . Since H.x; 0/ D f .x/
and H.x; 1/ D G.x; 1/ D h.x/ for any x 2 X , we proved that f  h, so being
homotopic is an equivalence relation.
V.092. Given a space X , let f; g W X ! @In be continuous maps such that the
points f .x/ and g.x/ belong to the same face of In for any x 2 X . Prove that f
and g are homotopic.
Solution. Let F .x; t / D .1  t /f .x/ C tg.x/ for any x 2 X and t 2 0; 1
. It is
clear that the map F W X  0; 1
! Rn is continuous while F .x; 0/ D f .x/ and
F .x; 1/ D g.x/ for any x 2 X . Now, if x 2 X then there is a face B of In such that
ff .x/; g.x/g  B. Therefore there is i < n such that either f .x/.i / D g.x/.i / D 1
or f .x/.i / D g.x/.i / D 1. In both cases
F .x; t /.i / D .1  t /f .x/.i / C tg.x/.i / D f .x/.i /.1  t C t / D f .x/.i /

2 Solutions of problems 001500

135

which shows that F .x; t / 2 B for any t 2 0; 1


. Therefore F W .X  0; 1
/  @In ,
so the map F W X  0; 1
! @In witnesses the fact that the functions f and g are
homotopic.
V.093 (Mushroom lemma). Let X be a normal countably paracompact space.
Suppose that F  X is closed and we have continuous homotopic mappings f0 ; f1
of F to the n-dimensional sphere S n D fx 2 RnC1 W x.0/2 C : : : C x.n/2 D 1g.
Prove that, if there exists a continuous map g0 W X ! S n with g0 jF D f0 then
there is a continuous map g1 W X ! S n such that g1 jF D f1 and g1 is homotopic
to g0 .
p
Solution. If m 2 N then jxjm D .x.0//2 C : : : C .x.m  1//2 for any x 2 Rm .
Fact 1. Given a number m 2 N, if Y is a normal space, G is closed in Y and we
have a continuous map f W G ! S m then there exists a set U 2 .G; Y / and a
continuous map g W U ! S m such that gjG D f .
Proof. We can consider that f maps G into RmC1 ; let i .x/ D x.i / for any x 2
RmC1 and i 2 f0; : : : ; mg. The map i f W Y ! R is continuous, so there exists a
continuous function pi W Y ! R such that pi jG D i f for any i  m.
Letting p.y/.i / D pi .y/ for any index i  m we obtain a continuous map
p W Y ! RmC1 such that pjG D f . The set O D fx 2 RmC1 W jxjmC1 0g is
open in RmC1 ; since S m  O, the set U D p 1 .O/ is open in Y and G  U .
y
For any point y 2 O let r.y/ D jyjmC1
; it is evident that the map r W O ! S m
is continuous and r.y/ D y whenever y 2 S m . Thus the map g D r .pjU / W
U ! S m is continuous as well. Given an arbitrary point y 2 G we have p.y/ D
f .y/ 2 S m which shows that g.y/ D r.p.y// D r.f .y// D f .y/ and hence g is
the desired continuous extension of f . Fact 1 is proved.
Returning to our solution fix a continuous map f W F  0; 1
! S n such that
f .x; 0/ D f0 .x/ and f .x; 1/ D f1 .x/; let h.x; 0/ D g0 .x/ for any x 2 X . Since
g0 W X ! S n is a continuous map, the map h W X  f0g ! S n is also continuous.
Furthermore, hj.F  f0g/ D f j.F  f0g/, so the map q D f [ h is continuous by
Fact 2 of T.354.
The space X  0; 1
is normal (see TFS-288) and the set P D .X  f0g/ [ .F 
0; 1
/ is closed in X 0; 1
, so we can apply Fact 1 to find a set U 2 .P; X 0; 1
/
and a continuous map g W U ! S n such that gjP D q. Since 0; 1
is compact,
the projection  W X  0; 1
! X is a perfect map (see Fact 3 of S.288), so
H D ..X  0; 1
/nU / is a closed subset of X disjoint from F . Therefore the set
V D X nH is an open neighborhood of F in X and  1 .V / D V  0; 1
 U .
By normality of X there is a continuous function ' W X ! 0; 1
such that
'.x/ D 1 for any x 2 F and '.x/ D 0 whenever x 2 X nV . Let .x; t / D
g.x; t '.x// for any .x; t / 2 X  0; 1
. It is easy to see that is well defined on
X  0; 1
and W X  0; 1
! S n is a continuous map. Besides, x 2 F implies
t '.x/ D t and hence .x; t / D g.x; t / D f .x; t / for any t 2 0; 1
. This shows
that j.F  0; 1
/ D f ; observe that also .x; 0/ D g.x; 0/ D g0 .x/ for every
x 2 X.

136

2 Solutions of problems 001500

Finally, let g1 .x/ D .x; 1/ for any x 2 X ; then g1 W X ! S n is a continuous


function and witnesses that g0  g1 . Since also g1 .x/ D .x; 1/ D f .x; 1/ D
f1 .x/ for any x 2 F , the function g1 is a continuous extension of f1 , so our solution
is complete.
V.094. For each i < n, consider the faces Fi D fx 2 In W x.i / D 1g and
n
Gi D fx 2 In W x.i / D 1g of the
T n-dimensional cube I . Prove that, if Ci is a
partition between Fi and Gi then fCi W i < ng ;.
Solution. If m 2 N then um 2p
Rm is the zero point of Rm , i.e., um .i / D 0 for every
i < m. Furthermore, jxjm D .x.0//2 C : : : C .x.m  1//2 for any x 2 Rm ; the
set Bm D fx 2 ImC1 W jx.i /j D 1 for some i  mg is the boundary of the .m C 1/dimensional cube ImC1 .
Fact 1. Given m 2 N let '.x/ D
' W Bm ! S m is a homeomorphism.

x
jxjmC1

for any point x 2 Bm . Then the map

Proof. It is clear that the map ' is continuous and '.Bm /  S m . If y 2 S m then t D
maxfjy.i /j W i  mg > 0 and the point x D yt belongs to Bm . It is straightforward
that '.x/ D y, so ' is an onto map.
jx j
If x0 ; x1 2 Bm and '.x0 / D '.x1 / then x1 D tx0 where t D jx10 jmC1
. Assume
mC1
first that t < 1; since jx0 .i /j  1, we have jtx0 .i /j < 1 for all i  m. Therefore
tx0 Bm and hence tx0 x1 . If t > 1 then take a coordinate i  m such that
jx0 .i /j D 1; then jtx0 .i /j > 1 and hence tx0 Bm which shows that x1 tx0 . This
proves that t D 1 and therefore x0 D x1 , i.e., ' is a bijection; the space Bm being
compact, ' is a homeomorphism, so Fact 1 is proved.
T
Returning to our solution assume that fCi W i < ng D ; and fix any i < n.
Take disjoint sets Ui ; Vi 2 .In / such that Fi  Ui ; Gi  Vi and In n.Ui [Vi / D Ci .
The space In being metrizable we can choose continuous functions ai and bi from In
to I D 0; 1
such that Fi D ai1 .0/ and In nUi D bi1 .0/. Analogously, there exist
continuous functions ci ; di W In ! I such that Gi D ci1 .0/ and In nVi D di1 .0/.
di
i
 ai bCb
is continuous, gi W In ! I
It is easy to check that the function gi D ci Cd
i
i
1
while gi .Fi / D f1g and gi .Gi / D f1g; besides, gi .0/ D Ci . Let g.x/.i / D
gi .x/ for any i < n and x 2 In ; then the function g W In ! In is continuous being
the diagonal product of continuous functions g0 ; : : : ; gn1 . It is straightforward that
(1) g.Fi /  Fi and g.Gi /  Gi for any i < n.
For any x 2 In nfun g let m.x/ D maxfjx.0/j; : : : ; jx.n  1/jg; then the point
x
p.x/ D m.x/
is well defined and belongs to Bn1 . It is evident that the map p W
n
I nfun g ! Bn1 is continuous. If g.x/TD un for some x 2 In then gi .x/ D 0 and
hence x 2 Ci for any i < n, i.e., x 2 i<n Ci which is a contradiction. Therefore
g.In /  In nfun g and hence D p g W In ! Bn1 is a continuous map. The
function p being an identity on Bn1 , it follows from (1) that
(2) .Fi /  Fi and .Gi /  Gi for any i < n.

2 Solutions of problems 001500

137

Let id W Bn1 ! Bn1 be the identity map, i.e., id.x/ D x for any point
x 2 Bn1 . It follows from (2) that h D jBn1 W Bn1 ! Bn1 ; apply Problem 092
to see that the maps id and h are homotopic. For any m 2 N the spaces S m and Bm
are homeomorphic by Fact 1, so Mushroom lemma (Problem 093) is applicable for
the respective maps into Bm . Since W In ! Bn1 is a continuous extension of h,
we can apply Problem 093 to the maps h and id to conclude that there exists a map
r W In ! Bn1 such that rjBn1 D id, i.e., r is a retraction of In onto Bn1 ; this
contradiction with Problem 090 shows that our solution is complete.
V.095. For each i 2 !, consider the subsets Fi D fx 2 I! W x.i / D 1g and
Gi D fx 2 I! W T
x.i / D 1g of the cube I! . Prove that, if Ci is any partition between
Fi and Gi then fCi W i 2 !g ;.
Solution. For any m 2 ! let m W I! ! Im be the projection of I! onto its face Im .
Say that a set U  I! is standard if there is m 2 ! and a set V 2 .Im / such that
m1 .V / D U ; inTthis case let j.U / D m.
AssumeTthat i<! Ci D ;; it follows from compactness of I! that there is m 2 !
such that fCi W i < mg D ;. Thus fI! nCi W i < mg is an open cover of the
space I! . The standard sets form a base in I! , so we can choose, for any x 2 I! a
!
!
standard set
x  I nCi for some i < m. Fix a finite set A  I
SOx 3 x such that O
!
for which fOx W x 2 Ag D I .P
It is easy to see that, for n D fj.Ox / W x 2 Ag C m, we have
T
(1) fCi W i < ng D ; and n1 .n .Ox // D Ox for any x 2 A.
Consider the sets Fi0 D fx 2 In W x.i / D 1g and Gi0 D fx 2 In W x.i / D 1g
for every i < n. It turns out that
(2) the set Di D n .Ci / is a partition between Fi0 and Gi0 for any i < n.
To prove this, fix a number i < n and choose disjoint sets Ui ; Vi 2 .I! / such
that Fi  Ui ; Gi  Vi and I! n.Ui [ Vi / D Ci . The map n being closed the sets
Ui0 D In nn .I! nUi / and Vi0 D In nn .I! nVi / are open in In ; it is straightforward
that Ui0 \ Vi0 D ; while Fi0  Ui0 and Gi0  Vi0 . Given a point x 2 In nDi , the space
n1 .x/  Ui [ Vi is connected because n1 .x/ ' I!nn (see Fact 1 of U.493), so
n1 .x/ cannot meet both sets Ui and Vi ; thus either n1 .x/  Ui or n1 .x/  Vi .
This shows that x 2 Ui0 [ Vi0 , i.e., In n.Ui0 [ Vi0 / D Di , so (2) is proved.
Given a point y 2 In take any z 2 I! such that n .z/ D y. There is x 2 A
such that z 2 Ox ; pick i < n for which Ox  I! nCi . It follows fromT(1) that
n1 .y/ D n1 .n .z//  Ox  I! nCi and hence y Di . Therefore
y i<n Di ;
T
the point y 2 In was chosen arbitrarily, so we proved that i<n Di D ;. Since Di
is a partition in the cube In between its faces Fi0 and Gi0 for any i < n, we obtained
a contradiction with Problem 094.
V.096. Prove that, for any n 2 N, the space In is the finite union of its zerodimensional subspaces.
Solution. Let Z0 D Q \ I and Z1 D InZ0 . It is clear that Z0 and Z1 are zerodimensional
Q subspaces of I such that I D Z0 [ Z1 . Now if n > 1 then consider the
set P D fZ.i/ W i < ng for any  2 Dn . Every space P is zero-dimensional by

138

2 Solutions of problems 001500

S
SFFS-302; it is straightforward that In D fP W  2 Dn g, so fP W  2 Dn g is a
finite family of zero-dimensional subspaces of In whose union is In .
V.097. Prove that, for any n 2 N, the space In cannot be represented as the union
of  n-many of its zero-dimensional subspaces.
Solution. A space is cosmic if has a countable network. Given a space X we say
that sets A; B  X are separated in X if A \ B D A \ B D ;.
Fact 1. Suppose that X is a cosmic space such that X nfag is zero-dimensional for
some point a 2 X . Then X is zero-dimensional. In other words, adding a point to a
cosmic zero-dimensional space gives a zero-dimensional space.
Proof. Since .a;
T X /  nw.X / D !, there exists a family fUn W n 2 !g  .a; X /
such that fag D fUn W n 2 !g. It follows from hl.X nfag/ D ! that any subspace
of X nfag is strongly zero-dimensional (see SFFS-301 and SFFS-306); this implies
that the set Fn D X nUn  X nfag is closed in X and strongly zero-dimensional for
any n 2 !. The family F D ffagg[fFn W n 2 !g is countable and consists of
Sclosed
strongly zero-dimensional subsets of X ; the space X is normal and X D F, so
we can apply SFFS-311 to conclude that X is strongly zero-dimensional and hence
zero-dimensional. Fact 1 is proved.
Fact 2. Suppose that X is a second countable space and F; G  X are disjoint
closed subsets of X . Then, for any zero-dimensional Z  X , there is a partition C
between the sets F and G such that C \ Z D ;.
Proof. Observe first that
(1) there exists a second countable compact space K and an embedding e W X ! K
for which e.F / \ e.G/ D ;.
Indeed, the space X is normal, so there is a continuous function f W X ! I such
that f .F /  f0g and f .G/  f1g. If ' W X ! I! is an arbitrary embedding
then the map e D ' f W X ! K D I!  I is still an embedding such that
g.F / \ g.G/ D ;.
To simplify the notation we will identify X and e.X /; thus X  K and the sets
F 0 D F and G 0 D G are disjoint (the bar denotes the closure in K). Call a set
U 2 .K/ adequate if U \ Z D U \ Z and, in particular, the set U \ Z is
clopen in Z. We claim that
(2) adequate sets form a base in K.
To prove it fix a point x 2 K and U 2 .x; K/. Since fxg[Z is zero-dimensional
by Fact 1, there is a clopen subset A of the space fxg [ Z such that x 2 A 
U \ .fxg [ Z/. The sets A and B D ZnA are disjoint and clopen in fxg [ Z; an
easy consequence is that they are separated in K.
Apply Fact 1 of S.291 to find disjoint sets VA ; VB 2 .K/ such that A  VA
and B  VB . We leave to the reader a simple verification of the fact that the set
V D VA \ U is adequate; since also x 2 V  U , the property (2) is proved.

2 Solutions of problems 001500

139

Now apply (2) to find, for any point x 2 F 0 , an adequate set Ox 2 .x; K/ such
0
D ;. The set F 0 being compact, there is a finite P  F 0 for which
that O x \ GS
0
F  O D fOx W x 2 P g. It is evident that O is adequate so C 0 D OnO is a
closed set such that C 0 \ Z D ;. The sets O  F 0 and W D KnO witness that C 0
is a partition between F 0 and G 0 ; an immediate consequence is that C D C 0 \ X is
the promised partition in X between the sets F and G. Fact 2 is proved.
S
Returning to our solution assume that In D fZi W i < ng and every Zi is zerodimensional. The faces Fi D fx 2 In W x.i / D 1g and Gi D fx 2 In W x.i / D 1g
are disjoint and closed in In ; by Fact 2 there is a partition CiTbetween the sets Fi
and Gi such that Ci \ Zi D ; for any i < n. The set C D i<n Ci is nonempty
by Problem 094 and C \ Zi D ; for any i < n; therefore no point of C belongs to
S
fZi W i < ng D In . This contradiction shows that the above representation of In
is impossible and hence our solution is complete.
V.098. Prove that the cube I! cannot be represented as the countable union of
its zero-dimensional subspaces. ProveS(in ZFC) that there exist zero-dimensional
spaces fX W < !1 g such that I! D fX W < !1 g.
S
Solution. Assume that I! D fZi W i < !g and every Zi is zero-dimensional.
The faces Fi D fx 2 I! W x.i / D 1g and Gi D fx 2 I! W x.i / D 1g are disjoint
and closed in I! ; by Fact 2 of V.097 there is a partition CiTbetween the sets Fi and
Gi such that Ci \ Zi D ; for any i < !. The set C D i<! Ci is nonempty by
Problem 095 and C \ Zi D ; for any i < !; therefore no point of C belongs to
S
fZi W i < !g D I! . This contradiction shows that the above representation of I!
is impossible.
Next observe that jc  !1 j D c, so there is a bijection ' W c ! c  !1 . Let
A D ' 1 .c  fg/ for any < !1 . Then
(1) the family
A D fA W < !1 g is disjoint, consists of nonempty subsets of c
S
and A D c.
It is easy to see that
(2) if Y  I and InY is dense in I then Y is zero-dimensional.
Apply Fact 1 of S.480 to find a disjoint family E D fE W S
< cg of subsets of
R such that every E is dense in R. Let H0 D .E0 \ I/ [ .In E/; if 0 < < c
then let H D E \ I.
SThen H D fH W < cg is a disjoint family of dense
subsets of I suchSthat H D I.
Now, if G D fH W 2 A g for any < !1 then it follows from (1)
that S
G D fG W < !1 g is still a family of disjoint dense subsets
of I such
S
that G D I. The property (2) shows that the space P D fG W < g
is zero-dimensional because InP contains a dense set G for any < !1 .
Therefore
(3) the family P DSfP W < !1 g is increasing, consists of zero-dimensional
subsets of I and P D I.
Finally let X D .P /! for every < !1 . Then fX W < !1 g is a family
of zero-dimensional subsets of I! (see SFFS-302). Given a point x 2 I! , for any

140

2 Solutions of problems 001500

i 2 !, there is i < !1 with x.i / 2 Pi . If > supfi W i < !gSthen x.i / 2 P for
any i < ! and hence x 2 .P /! D X . This proves that I! D fX W < !1 g, so
our solution is complete.
V.099. Prove that, for any n 2 N, the spaces In and I! are not t -equivalent.
Solution. Take any n 2 N and assume that In is t -equivalent to I! . Since the space
In is representable as the finite union of its zero-dimensional subspaces (see Problem 096), we can apply Problem 082 to conclude that I! is also the countable
union of its zero-dimensional subspaces. This, however, gives a contradiction with
Problem 098 and shows that the spaces In and I! are not t -equivalent.
V.100. Suppose that X is one of the spaces !1 or !1 C 1. Prove that, for any
distinct m; n 2 N, the spaces .Cp .X //n and .Cp .X //m are not homeomorphic. In
particular, X is not t -equivalent to X X .
Solution. Let I D 0; 1
 R and P D RnQ; for any set X the map idX W X ! X
is the identity on X , i.e., idX .x/ D x for any x 2 X . Given spaces X and Y , a
map h W X ! Y is called constant if there is a point b 2 Y such that f .x/ D b
for any x 2 X ; if we have maps f; g 2 C.X; Y / then the expression f  g says
that f and g are homotopic. Say that spaces X and Y are homotopically equivalent
if there exist maps f W X ! Y and g W Y ! X such that f g  idY and
g f  idX . If n 2 N say that a space X has the n-partition property if for
any family f.F0 ; G0 /; : : : ; .Fn1 ; Gn1 /g of pairs of disjoint closed subsets of X ,
for
T every i < n, there exists a partition Ci between the sets Fi and Gi such that
fCi W i < ng D ;.
If L is a linear topological space then a family f'1 ; : : : ; 'n g of continuous
linear
P
functionals on L is called independent if, for any a1 ; : : : ; an 2 R with niD1 ai2 0
the functional a1 '1 C : : : C an 'n is not identically zero. A linear subspace L0  L is
said to have codimension n 2 N in L if there is an independent family f'1 ; : : : ; 'n g
of continuous linear functionals on L such that L0 D '11 .0/ \ : : : \ 'n1 .0/. Given
ordinals and the interval ;
consists of the ordinals  such that    ;
analogously, .; / D f W < <  g.
Fact 1. Suppose that we have spaces X; Y; Z and maps f; g; h with f; g 2 C.X; Y /
and h 2 C.Y; Z/. If, additionally, f  g then h f  h g.
Proof. Let F W X  I ! Y be a continuous map such that F .x; 0/ D f .x/ and
F .x; 1/ D g.x/ for any x 2 X . Then G D h F W X  I ! Z is a continuous
map such that G.x; 0/ D h.F .x; 0// D .h f /.x/ and G.x; 1/ D h.F .x; 1// D
.h g/.x/ for any x 2 X , i.e., G witnesses that h f and h g are homotopic.
Fact 1 is proved.
Fact 2. Suppose that we have spaces X; Y; Z and maps f; g; h with f; g 2 C.Y; Z/
and h 2 C.X; Y /. If, additionally, f  g then f h  g h.
Proof. Since f is homotopic to g, we can find a continuous map F W Y  I ! Z
such that F .y; 0/ D f .y/ and F .y; 1/ D g.y/ for any y 2 Y . Let H.x; t / D
.h.x/; t / for any x 2 X and t 2 I ; then H W X  I ! Y  I is a continuous map.

2 Solutions of problems 001500

141

It is evident that the map G D F H W X  I ! Z is also continuous and


G.x; 0/ D F .h.x/; 0/ D .f h/.x/ and G.x; 1/ D F .h.x/; 1/ D .g h/.x/ for
any x 2 X , i.e., G witnesses that f h and g h are homotopic. Fact 2 is proved.
Fact 3. Homotopical equivalence is an equivalence relation on the class of topological spaces.
Proof. It is clear from the definition that any space is homotopically equivalent to
itself and X is homotopically equivalent to Y if and only if Y is homotopically
equivalent to X . Now assume that X is homotopically equivalent to Y and Y is
homotopically equivalent to Z. To witness homotopical equivalence of X and Y
choose continuous maps f W X ! Y and g W Y ! X such that f g  idY and
g f  idX . Analogously, there exist continuous maps u W Y ! Z and v W Z ! Y
such that u v  idZ and v u  idY .
Then h D u f W X ! Z and w D g v W Z ! X are continuous maps; it
follows from Fact 1 and Fact 2 that h w D u f g v  u idY v D u v  idZ .
Analogously, w h D g v u f  g idY f D g f D idX and hence the
maps h and w witness homotopical equivalence of X and Z. Fact 3 is proved.
Fact 4. If a second countable space is representable as the union of at most n-many
zero-dimensional subspaces then it has the n-partition property.
S
Proof. Take a second countable space X such that X D fZi W i < ng and every
Zi is zero-dimensional. Assume that Fi and Gi are disjoint closed subsets of X for
any i < n. By Fact 2 of V.097 there is a partition C
Ti between the sets Fi and Gi
such
that
C
\
Z
D
;
for
any
i
<
n.
Then
C
D
i
i
i<n Ci does not meet the set
S
i<n Zi D X which shows that C D ; and hence Fact 4 is proved.
Fact 5. Suppose that X is a normal space. If F  X is closed and U 2 .F; X /
then there exists a cozero set V such that F  V  V  U .
Proof. By normality of X there is a continuous function f W X ! I such that
f .F /  f0g and f .X nU /  f1g. Then V D f 1 .0; 12 // is a cozero set by Fact 1
of T.252. We have F  V  V  f 1 .0; 12
/  U , so Fact 5 is proved.
Fact 6. Suppose that X is a normal space and fU1 ; : : : ; Un g is an open cover of X
for some n 2 N. If Fi is a closed subset of X with Fi  Ui for any i  n then there
exists a cozero set Vi such that Fi  Vi  V i  Ui for each i  n and fV1 ; : : : ; Vn g
is a cover of X .
Proof. It is evident that the set F D F1 [ .X n.U2 [ : : : [ Un // is closed in Z
and F  U1 ; besides F [ U2 [ : : : [ Un D X . Since X is normal, we can apply
Fact 5 to find a cozero set V1 such that F  V1  V 1  U1 ; it is clear that
V1 [ U2 [ : : : [ Un D X .
Assume that 1  k < n and we have constructed cozero sets V1 ; : : : ; Vk such
that Fi  Vi  V i  Ui and V1 [ : : : [ Vi [ UiC1 [ : : : [ Un D X for any i  k.
The set F D FkC1 [ .X n.V1 [ : : : [ Vk [ UkC2 [ : : : [ Un // is closed in X and
F  UkC1 . Since X is normal, we can apply Fact 5 to find a cozero set VkC1 such

142

2 Solutions of problems 001500

that F  VkC1  V kC1  UkC1 ; it is clear that FkC1  VkC1  V kC1  UkC1 and
V1 [: : :[VkC1 [UkC2 [: : :[Un D X , so our inductive construction can be continued
to obtain a family fV1 ; : : : ; Vn g of cozero subsets of X such that Fi  Vi  V i  Ui
for every i  n and V1 [ : : : [ Vn D X . Fact 6 is proved.
Fact 7. Suppose that, for some n 2 !, a normal countably paracompact space X
has the .n C 1/-partition property. Then for any closed set F  X and any function
f 2 C.F; S n / there is a continuous map h W X ! S n such that hjF D f .
Proof. Denote by u the zero point of RnC1 , i.e., u.i / D 0 for every i  n. As usual,
the set Bn D fx 2 InC1 W jx.i /j D 1 for some i  ng is the boundary of the .n C 1/dimensional cube InC1 . Since Bn ' S n by Fact 1 of V.094, it suffices to prove our
Fact replacing S n with Bn , so we consider from now on that f W F ! Bn . For any
i  n, let Fi0 D fx 2 Bn W x.i / D 1g and Gi0 D fx 2 Bn W x.i / D 1g; the sets
Fi D f 1 .Fi0 / and Gi D f 1 .Gi0 / are disjoint and closed in F and hence
T in X , so
we can find a partition Ci0 in X between the sets Fi and Gi such that in Ci0 D ;.
For every partition Ci0 take disjoint sets Ui0 ; Vi0 2 .X / such that Fi  Ui0 ; Gi  Vi0
and X n.Ui0 [ Vi0 / D Ci0 .
The family fU00 ; V00 ; : : : ; Un0 ; Vn0 g is an open cover of the space X . By Fact 6 there
are cozeroS
sets U0 ; V0 ; : : : ; Un ; Vn such that Fi  Ui  Ui0 ; Gi  Vi  Vi0 for every
i  n and fUi [Vi W i  ng D X . Therefore Ci D X n.Ui [Vi / is a zero-set which
is a partition between Fi andTGi for every i  n. The family fU0 ; V0 ; : : : ; Un ; Vn g
being a cover of X , we have in Ci D ;.
Fix i  n and apply Fact 1 of S.499 and Fact 1 of T.252 to find continuous
functions ai and bi from X to I such that ai .Fi /  f0g; ai .X nUi /  f1g and
X nUi D bi1 .0/. Analogously, there exist functions ci ; di 2 Cp .X; I / such that
ci .Gi /  f0g; ci .X nVi /  f1g and X nVi D di1 .0/. It is easy to check that the
di
i
 ai bCb
is continuous, gi W X ! I while gi .Fi /  f1g and
function gi D ci Cd
i
i
1
gi .Gi /  f1g; besides, gi .0/ D Ci .
Let g.x/.i / D gi .x/ for any i  n and x 2 X ; then the function g W X ! InC1
is continuous being the diagonal product of continuous functions g0 ; : : : ; gn . It is
straightforward that
. / g.Fi /  Fi0 and g.Gi /  Gi0 for any i  n.
For any x 2 InC1 nfug, let m.x/ D maxfjx.0/j; : : : ; jx.n/jg; then the point
x
p.x/ D m.x/
is well defined and belongs to Bn . It is evident that the mapping
nC1
p W I nfug ! Bn is continuous. If g.x/
T D u for some x 2 X then gi .x/ D 0 and
hence x 2 Ci for any i  n, i.e., x 2 in Ci which is a contradiction. Therefore
g.X /  InC1 nfug and hence D p g W X ! Bn is a continuous map. The
function p being an identity on Bn , it follows from . / that
. / .Fi /  Fi0 and .Gi /  Gi0 for any i  n.
S
Given x 2 F , it follows from in .Fi0 [ Gi0 / D Bn that f .x/ 2 Fi0 [ Gi0 for
some i  n. If f .x/ 2 Fi0 then x 2 Fi and hence .x/ 2 Fi0 by . /; analogously,
if f .x/ 2 Gi0 then .x/ 2 Gi0 which shows that .x/ and f .x/ belong to the

2 Solutions of problems 001500

143

same face of Bn for any x 2 F . Apply Problem 092 to see that the maps f and
jF are homotopic. For any m 2 N the spaces S m and Bm are homeomorphic by
Fact 1 of V.094, so Mushroom lemma (Problem 093) is applicable for the respective
maps into Bm . Since W X ! Bn is a continuous extension of jF , we can apply
Problem 093 to the maps jF and f to conclude that there is a continuous map
h W X ! S n with hjF D f , so Fact 7 is proved.
Fact 8. For any n 2 N both spaces Rn and S n are representable as the union of at
most .n C 1/-many of their zero-dimensional subspaces.
Proof. Consider the set Z.k; m/ D fx 2 Rm W jfi < m W x.i / 2 Pgj D kg for every
m 2 N and k  m. We will show that every Z.k; m/ is zero-dimensional. To do this,
fix k  m and let H.r1 ; : : : ; rmk ; i1 ; : : : ; imk / D fx 2 Z.k; m/ W x.ij / D rj for
all j  m  kg for any r1 ; : : : ; rmk 2 Q and distinct i1 ; : : : ; imk 2 f0; : : : ; m  1g.
Every set H.r1 ; : : : ; rmk ; i1 ; : : : ; imk / is zero-dimensional being homeomorphic to Pk . The family H D fH.r1 ; : : : ; rmk ; i1 ; : : : ; imk / W ri 2 Q for each
i S
m  k and i1 ; : : : ; imk are distinct elements of f0; : : : ; m  1gg is countable
and H D Z.k; m/. It turns out that every set H D H.r1 ; : : : ; rmk ; i1 ; : : : ; imk /
is closed in Z.k; m/.
Indeed, if x 2 Z.k; m/ is an accumulation point of H then x.ij / D rj for
any j  m  k. Therefore x.i / 2 P for any i fi1 ; : : : ; imk g which shows
that x 2 H , i.e., H is closed in Z.k; m/. Finally, apply SFFS-311 and SFFS-306
to conclude that Z.k; m/ is zero-dimensional for every k  m. We have Rn D
Z.0; n/ [ : : : [ Z.n; n/, so Rn as the union of .n C 1/-many zero-dimensional
subspaces.
Recall that the boundary Bn D fx 2 InC1 W jx.i /j D 1 for some i  ng of the
.n C 1/-dimensional cube InC1 is homeomorphic to S n by Fact 1 of V.094, so it
suffices to show that Bn is representable as the union of at most .n C 1/-many of its
zero-dimensional subspaces. Now, RnC1 D Z.0; nC1/[: : :[Z.nC1; nC1/; since
every point of Bn has a rational coordinate (either 1 or 1), the set Bn does not meet
Z.nC1; nC1/ and therefore Bn D .Z.0; nC1/\Bn /[: : :[.Z.n; nC1/\Bn / is the
desired representation of Bn as the union of at most .n C 1/-many zero-dimensional
spaces. Fact 8 is proved.
Fact 9. If n < m then any continuous function f W S n ! S m is homotopic to a
constant map.
P
Proof. Fix a map f 2 C.S n ; S m / and let W D fx 2 RnC1 W niD0 .x.i //2  1g
be the .n C 1/-dimensional ball in RnC1 whose boundary is S n . There are zerodimensional spaces Z0 ; : : : ; ZnC2 such that RnC1 D Z0 [ : : : [ ZnC2 (see Fact 8);
0
every Zi0 D Zi \W is zero-dimensional and W D Z00 [: : :[ZnC2
which shows that
the metrizable space W has the .n C 2/-partition property by Fact 4. This, together
with n C 2  m C 1 implies that W also has .m C 1/-partition property, so we
can apply Fact 7 to the space W , its closed subspace S n and the map f to obtain a
continuous map g W W ! S m such that gjS n D f .

144

2 Solutions of problems 001500

Now let F .x; t / D g.tx/ for any x 2 S n and t 2 I . The map F W S n  I ! S m


is continuous, F .x; 1/ D g.x/ D f .x/ and F .x; 0/ D g.0/ for any x 2 S n which
shows that the map f is homotopic to a constant map h W S n ! fg.0/g and hence
Fact 9 is proved.
Fact 10. For any n 2 ! and p 2 RnC1 the spaces RnC1 nfpg and S n are
homotopically equivalent.
p
Proof. For any x 2 RnC1 let jxj D .x.0//2 C : : : C .x.n//2 and denote by u
the zero point of RnC1 . The space RnC1 nfug and RnC1 nfpg are easily seen to be
homeomorphic, so it suffices to show that RnC1 nfug is homotopically equivalent to
S n (see Fact 3). Define a map g W S n ! RnC1 nfug by g.x/ D x for any x 2 S n ; it
is evident that g is continuous.
x
Letting f .x/ D jxj
for any x 2 RnC1 nfug we also obtain a continuous map f W
nC1
n
R nfug ! S . To see that the maps f and g witness homotopical equivalence of
RnC1 nfug and S n observe first that f g D idS n because f is a retraction.
For any x 2 RnC1 nfug and t 2 I let F .x; t / D .1  t /x C tf .x/; it is clear that
F .x; 0/ D x and F .x; 1/ D f .x/ for any x 2 RnC1 nfug. If F .x; t / D u then x.1 
t
t
t C jxj
/ D u which implies that 1  t C jxj
D 0 and hence jxj.1  t / C t D 0. Since
the last equality cannot hold for t 2 I and x u, we have obtained a contradiction;
therefore F W .RnC1 nfug/  I ! RnC1 nfug is a continuous map which witnesses
that g f D f is homotopic to idRnC1 nfug . Fact 10 is proved.
Fact 11. For any distinct n; m 2 N the spaces S n and S m are not homotopically
equivalent.
Proof. There is no loss of generality to assume that n < m; suppose that the spheres
S n and S m are homotopically equivalent. There exist continuous maps f W S n !
S m and g W S m ! S n such that f g is homotopic to idS m . There is a point a 2 S m
such that the map f is homotopic to the constant map d W S n ! fag (see Fact 9).
The map d g is also constant and d g  f g by Fact 2. Being homotopic is
a transitive relation (see Problem 091), so idS m is homotopic to a constant map. It
is easy to see that this implies that S m is homotopically equivalent to a one-point
space.
Let Bm D fx 2 ImC1 W jx.i /j D 1 for some i  mg be the boundary of
the .m C 1/-dimensional cube ImC1 . It was proved in Fact 1 of V.094 that Bm
is homeomorphic to S m , so Bm is also homotopically equivalent to a one-point
space (see Fact 3). Let E be a one-point space for which there are (automatically
continuous) maps r W Bm ! E and s W E ! Bm such that s r  idBm .
The map w D s r W Bm ! Bm is constant, so let b 2 Bm be the point such that
w.Bm / D fbg. Letting q.x/ D b for any x 2 ImC1 we obtain a continuous extension
q W ImC1 ! Bm of the map w. The space Bm being homeomorphic to S m , we can
apply Mushroom lemma (Problem 093) to the space ImC1 , its closed subset Bm and
the pair of homotopic maps w; idBm W Bm ! Bm . Since q is a continuous extension
of the map w, there exists a continuous map  W ImC1 ! Bm such that jBm D idBm ,
i.e.,  is a retraction of ImC1 onto Bm . This, however, contradicts Problem 090 and
shows that Fact 11 is proved.

2 Solutions of problems 001500

145

Fact 12. Assume that P and Q are spaces with the following properties:
S
S
(i) P D fP W < !1 g and Q D fQ W < !1 g;
(ii) the sets P and Q are countable for any < !1 ;
(iii) P  P
S and Q  Q wheneverS < < !1 ;
(iv) P D fP W < g and Q D fQ W < g if < !1 is a limit ordinal.
Suppose additionally that sets R  Cp .P / and S  Cp .Q/ are dense in Cp .P /
and Cp .Q/ respectively and there exists a homeomorphism ' W R ! S . Then
the set A D f < !1 W for any f; g 2 R we have f jP D gjP if and only if
'.f /jQ D '.g/jQ g is closed and unbounded in !1 .
Proof. Let p W R ! Cp .P / and q W S ! Cp .Q / be the restriction maps for
any < !1 . Suppose that fn W n 2 !g  A and n < nC1 for any n 2 !. Let
D supn2! n and take any f; g 2 R.
Observe that the properties (iii) and (iv) imply that f jP D gjP if and only
if f jPn D gjPn for any n 2 !. By our choice of A this happens if and only if
'.f /jQn D '.g/jQn for any n 2 ! and the last statement is equivalent (due to
(iii) and (iv) again) to '.f /jQ D '.g/jQ . Therefore 2 A, so we proved that
the set A is closed in !1 .
To see that the set A is unbounded take an arbitrary ordinal 2 !1 and let
0 D . Then q0 ' W R ! Cp .Q0 / is a continuous map of R to a second
countable space, so we can apply TFS-299 together with (i) and (iii) to find an
ordinal 0 2 .0 ; !1 / such that p0 .f / D p0 .g/ implies q0 .'.f // D q0 .'.g//
for any f; g 2 R. Analogously, there is an ordinal 1 2 .0 ; !1 / such that the
equality q1 .f / D q1 .g/ implies p0 .' 1 .f // D p0 .' 1 .g// for any f; g 2 S .
Continuing this construction inductively, we obtain sequences fn W n 2 !g and
fn W n 2 !g of countable ordinals with the following properties:
(1) D 0 and n < n < nC1 for any n 2 !;
(2) for any n 2 ! and f; g 2 R, if pn .f / D pn .g/ then qn .'.f // D qn .'.g//;
(3) for any n 2 ! and f; g 2 S , if qnC1 .f / D qnC1 .g/ then pn .' 1 .f // D
pn .' 1 .g//;
It is evident that D supn2! n D supn2! n > ; to prove that 2 A take any
f; g 2 R. If p .f / D p .g/ then pn .f / D pn .g/ and hence we have the equality
qn .'.f // D qn .'.g// (see (2)) for any n 2 !. It follows from the properties
(iii) and (iv) that q .'.f // D q .'.g//.
Now let f0 D '.f /; g0 D '.g/ and assume that q .f0 / D q .g0 /. Then we have
qnC1 .f0 / D qnC1 .g0 / and hence pn .' 1 .f0 // D pn .' 1 .g0 // for any n 2 ! by
(3). Apply (iv) to conclude that p .f / D p .' 1 .f0 // D p .' 1 .g0 // D p .g/,
so p .f / D p .g/ if and only if q .'.f // D q .'.g// and therefore 2 A. This
shows that A is unbounded and finishes the proof of Fact 12.
Fact 13. Assume that R and S are spaces with the following properties:
S
S
(i) R D fR W < !1 g and S D fS W < !1 g;
(ii) R is a separable closed subspace of R and S is a separable closed subspace
of S for any < !1 ;

146

2 Solutions of problems 001500

(iii) R  R and S  S whenever < < !1 ;


S
S
(iv) R D fR W < g and S D fS W < g if < !1 is a limit ordinal.
Suppose additionally that ' W R ! S is a homeomorphism. Then B D f <
!1 W '.R / D S g is closed and unbounded in !1 .
Proof. Take a sequence fn W n 2 !g  B such that n < nC1 for any n 2 ! and
let D supn2!
a homeomorphism, it is closed, so we have
S n . The map ' being
S
S
'.R / D '
fRn W n 2 !g D f'.Rn / W n 2 !g D fSn W n 2 !g D S
and hence 2 B, i.e., the set B is closed in !1 .
To see that B is unbounded take any < !1 and let 0 D . The space S0 being
separable, it follows from (i) and (iii) that there is an ordinal 0 > 0 such that
'.R0 / \ S0 is dense in S0 ; since '.R0 / is closed in S , we have S0  '.R0 /.
Analogously, there is an ordinal 1 > 0 such that R0  ' 1 .S1 /. Continuing
this construction inductively we obtain sequences fn W n 2 !g and fn W n 2 !g
with the following properties:
(4) D 0 and n < n < nC1 for any n 2 !;
(5) Sn  '.Rn / for any n 2 !;
(6) Rn  ' 1 .SnC1 / and hence '.Rn /  SnC1 for any n 2 !.
If D supn2! n D supn2! n then it follows from the properties (4) and (5)
that
'.R / D '

[

 [
[
fRn W n 2 !g D
f'.Rn / W n 2 !g 
fSn W n 2 !g D S ;

so '.R /  S . Analogously, it follows from the properties (4) and (6) that
'.R / D '

[
 [
[
fRn W n 2 !g D
f'.Rn / W n 2 !g 
fSnC1 W n 2 !g D S ;

so '.R / D S and hence 2 B. Thus B is also unbounded, i.e., Fact 13 is proved.


Fact 14. Suppose that L is a linear space and f0 ; : : : ; fn1 are linear functionals
on L. Then a linear functional f W L ! R is a linear combination of f0 ; : : : ; fn1
1
if and only if f01 .0/ \ : : : \ fn1
.0/  f 1 .0/.
Proof. If there are 0 ; : : : ; n1 2 R such that f D 0 f0 C : : : C n1 fn1 then
1
for any point x 2 K D f01 .0/ \ : : : \ fn1
.0/ we have fi .x/ D 0 for all i < n, so
1
f .x/ D 0, i.e., x 2 f .0/ and therefore K  f 1 .0/. This proves necessity.
Now assume that K  f 1 .0/ and consider the map W L ! Rn defined by
.x/ D .f0 .x/; : : : ; fn1 .x// for any x 2 L. It is straightforward that is a linear
map; let ei .j / D 0 for any distinct i; j < n and ei .i / D 1 whenever i < n. Then
e0 ; : : : ; en1 is a basis in Rn ; we claim that
(7) there exists a linear functional h W Rn ! R such that h D f .
To prove this observe first that if u is the zero vector of Rn then 1 .u/ D K;
given any y 2 .L/ take any x 2 1 .y/ and let h.y/ D f .x/. This definition

2 Solutions of problems 001500

147

must be consistent, of course, so take any x0 ; x1 2 1 .y/. Then .x0 / D .x1 /


which shows that .x0 x1 / D u and hence x0 x1 2 K; an immediate consequence
is that x0 x1 2 f 1 .0/ and therefore f .x0 / D f .x1 /, i.e., this definition is, indeed,
consistent. So far, the function h is defined on .L/ which is a linear subspace of
Rn ; besides, h is a linear functional on .L/. Using Fact 1 of S.489 and Fact 3 of
S.489 it is easy to see that h can be extended linearly over the whole Rn , so (7) is
proved.
Finally let i D h.ei / for each i < n and take any point x 2 L. We have
.x/ D f0 .x/e0 C : : : C fn1 .x/en1 , so we can apply the property (7) to conclude
that
f .x/ D h..x// D f0 .x/h.e0 / C : : : C fn1 .x/h.en1 / D 0 f0 .x/ C : : : C n1 fn1 .x/

which shows that f D 0 f0 C : : : C n1 fn1 , so Fact 14 is proved.


Fact 15. Suppose that L is a linear topological space and K  L is a linear
subspace of L of codimension n. Then LnK is homotopically equivalent to S n1 .
Proof. Denote by u the zero vector of the space L and fix independent
continuous
Tn
1
linear functionals f1 ; : : : ; fn on the space
L
such
that
K
D
f
iD1 i .0/. Then,
T
for any i  n there is a vector xi 2 ffj1 .0/ W j 2 f1; : : : ; ngnfi gg such that
fi .xi / 0 (see Fact 14); now, if i D f .x1 i / xi then fi .j / D 0 if i j while
fi .i / D 1 for any i  n.
The vectors 1 ; : : : ; n are easily seen to be linearly independent, so their linear
hull M is an n-dimensional linear subspace of L. Let r.x/ D f1 .x/1 C : : : C
fn .x/n for any x 2 L. Then r W L ! M is a continuous linear map (this is an
easy exercise left to the reader) such that r.LnK/ D M nfug and r.x/ D x for any
x 2 M . Let r0 D rj.LnK/ W LnK ! M nfug.
Define a map e W M nfug ! L to be the identity, i.e., e.x/ D x for any point
x 2 M nfug. It is clear that r0 e D idM nfug . Let F .x; t / D .1  t /x C t r0 .x/ for
any x 2 LnK and t 2 I . Then F W .LnK/  I ! LnK is a continuous map such
that F .x; 0/ D x and F .x; 1/ D r0 .x/ for any x 2 LnK; as a consequence, the
map e r0 D r0 is homotopic to idLnK , i.e., the maps e and r0 witness homotopic
equivalence of LnK and M nfug.
For any i  n and a 2 R let i .a/ D ai ; it follows from the axioms of linear
topological space that the map i W R ! L is continuous. Consequently, the map
 W Rn ! L defined by .a1 ; : : : ; an / D a1 1 C: : :Can n for any .a1 ; : : : ; an / 2 Rn
is continuous as well. It is clear that, actually,  W Rn ! M ; now if we define a
map W M ! L by .x/ D .f1 .x/; : : : ; fn .x// then is continuous because so
is every fi . Since is the inverse of , the spaces M and Rn are homeomorphic
and hence M nfug ' Rn nfpg where p D .u/ is the zero point of Rn . Thus LnK
is homotopically equivalent to Rn nfpg by Fact 3; since Rn nfpg is homotopically
equivalent to S n1 by Fact 10, we can apply Fact 3 once more to conclude that
LnK is homotopically equivalent to S n1 , so Fact 15 is proved.

148

2 Solutions of problems 001500

Fact 16. Let L and L0 be linear topological spaces and suppose that K  L
and K 0  L0 are linear subspaces of L and L0 respectively. Assume also that K
is of codimension m in L while K 0 is of codimension n in L0 . If there exists a
homeomorphism h W L ! L0 such that h.K/ D K 0 then m D n.
Proof. Since h.LnK/ D L0 nK 0 , the spaces LnK and L0 nK 0 are homeomorphic
and hence homotopically equivalent. By Fact 15, the space LnK is homotopically
equivalent to S m1 while L0 nK 0 is homotopically equivalent to S n1 . By Fact 3, the
spaces S m1 and S n1 are homotopically equivalent so m  1 D n  1 by Fact 11.
This, of course, implies m D n, so Fact 16 is proved.
Returning to our solution let X D !1 and fix distinct m; n 2 N. Since
.Cp .X //k ' Cp .X  k/ for any k 2 N, it suffices to show that the spaces
Cp .X  n/ and Cp .X  m/ are not homeomorphic. Let u and v be the zero functions
on X  n and X  m respectively. Assume toward a contradiction that there is a
homeomorphism ' W Cp .X  n/ ! Cp .X  m/. It follows from homogeneity of
the spaces Cp .X  n/ and Cp .X  m/ that we can assume that '.u/ D v.
Let P D  n and Q D  m for any < !1 . The families fP W < !1 g
and fQ W < !1 g satisfy the premises of Fact 12 if P D X  n and Q D X  m,
so there is a closed unbounded A  !1 such that,
(8) for any 2 A and f; g 2 Cp .X  n/ we have f jP D gjP if and only if
'.f /jQ D '.g/jQ .
For any 2 !1 denote by E the set of functions from Cp .X  n/ which are
constant starting from , i.e., E D ff 2 Cp .X  n/ W f ..; i // D f ..; i //
for any  and i < ng. Analogously, F D ff 2 Cp .X  m/ W f ..; i // D
f ..; i // for any  and i < mg.
It is straightforward to check that every set E is closed in Cp .X  n/ and F is
closed in Cp .X  m/; besides,
S E  E and F  F whenever < < !1 . It
follows
from
TFS-314
that
fE W < !1 g D Cp .X  n/ and we have the equality
S
fF W < !1 g D Cp .X  m/.
Fix < !1 and let  W Cp .X  n/ ! Cp .PC1 / be the restriction map. Then
p D  jE W E ! Cp .PC1 / is a condensation. For any O1 ; : : : ; Ok 2 .R/ and
x1 ; : : : ; xk 2 X n let x1 ; : : : ; xk I O1 ; : : : ; Ok
D ff 2 Cp .X n/ W f .xi / 2 Oi for
any i  kg. The family B D fx1 ; : : : ; xk I O1 ; : : : ; Ok
W k 2 N; x1 ; : : : ; xk 2 X n
and O1 ; : : : ; Ok 2 .R/g is a base in the space Cp .X n/. It is evident that the family
B 0 D fx1 ; : : : ; xk I O1 ; : : : ; Ok
2 B W .; i / 2 fx1 ; : : : ; xk g for any i < ng is also a
base in Cp .X  n/.
Observe that, for any U D x1 ; : : : ; xk I O1 ; : : : ; Ok
2 B 0 , the set p .U / is open
in Cp .PC1 / because p .U / D ff 2 Cp .PC1 / W f .xij / 2 Oij ; j D 1; : : : ; lg
where fxi1 ; : : : ; xil g D fx1 ; : : : ; xk g \ PC1 . Thus p is a homeomorphism being
an open condensation (see TFS-155 and Fact 2 of S.491).
This proves that every E is second countable and hence separable. Analogously,
the space
< !1 . We leave it to the reader to check that
S F is separable for anyS
E D fE W < g and F D fF W < g for any limit ordinal . Therefore
we can apply Fact 13 to the families fE W < !1 g and fF W < !1 g to conclude

2 Solutions of problems 001500

149

that there exists a closed unbounded set A0  !1 such that '.E / D F for any
2 A0 . The set B D A \ A0 is also closed and unbounded in !1 .
Fix an ordinal 2 B and let C D nnf 2 B W < g; consider the sets L D
fx 2 EC W xjP D ujP g and M D fy 2 FC W yjQ D vjQ g. An immediate
consequence of (8) is that '.L / D M for any 2 B; besides, both L and M are
closed linear subspaces of Cp .X  n/ and Cp .X  m/ respectively. For any i < n
define a linear functional di W L ! R by the equality di .f / D f .. C ; i // for
any f 2 L . Analogously, let ei .g/ D g.. C ; i // for any i < m and f 2 M .
Then fdi W i < ng and fei W i < mg are independent
of continuous
T families
1
linear functionals
on
L
and
M
respectively;
let
K
D
f.d
/
.0/
W i < ng and

i
T
N D f.ei /1 .0/ W i < mg.
Since n m, we can apply Fact 16 to see that '.K / N and hence either
'.K / 6 N or ' 1 .N / 6 K for any 2 B. Thus either '.K / 6 N or
' 1 .N / 6 K for uncountably many . These two cases being analogous, we
can assume, without loss of generality, that there is an uncountable B 0  B such
that '.K / 6 N for all 2 B 0 . Thus there is a function f 2 K such that
j'.f /.. C ; j //j > 0 for any 2 B 0 .
There exist: " > 0, an uncountable set B 00  B 0 and j < m such that j D j and
j'.f /.. C ; j //j  " for any 2 B 00 . Choose a sequence fi W i 2 !g  B 00 such
that iC1 > iC for any i 2 !. Since f1
.Rnf0g/  i ; iC
 n for any i 2 !,
i
1
the family ffi .Rnf0g/ W i 2 !g is disjoint and hence the sequence ffi W i 2 !g
converges to u in L . Every '.fi / is constant on each !1  fkg starting from iC ,
so if > supfiC W i 2 !g then j'.fi /..; j //j  " for every i 2 !. Therefore the
sequence f'.fi / W i 2 !g does not converge to zero which is a contradiction with
continuity of '. Thus the spaces Cp .X  n/ and Cp .X  m/ are not homeomorphic.
To settle the case when X D !1 C 1 we will first prove that
(9) the space Cp .X / is linearly isomorphic to the subspace D ff 2 Cp .!1 / W
there exists < !1 such that f ./ D 0 for any  g of the space Cp .!1 /.
The space !1 C 1 being homeomorphic to 1; !1
it suffices to construct a linear
homeomorphism between and Cp .1; !1
/. To do so take any function f 2
and let .f /./ D f .0/ C f ./ for any 2 1; !1
; then .f / 2 Cp .1; !1
/ and
the map  W ! Cp .1; !1
/ is a linear homeomorphism (this is an easy exercise
which is left to the reader), so (9) is proved.
Now assume that there is a homeomorphism  W n ! m ; by homogeneity
of n and m we can consider, without loss of generality, that .u/ D v. We
consider that n and m are the respective subsets of Cp .!1  n/ and Cp .!1  m/.
Since is dense in Cp .!1 /, the sets n and m are dense in Cp .!1  n/ and
Cp .!1  m/ respectively. We use the same notation as in the first part of our proof
and, in particular, P D  n and Q D  m for any < !1 . Since Fact 12
is also applicable to the homeomorphism  and the families fP W < !1 g and
fQ W < !1 g, we conclude that there exists a closed unbounded set J  !1
such that

150

2 Solutions of problems 001500

(10) for any 2 J and f; g 2 n the equality f jP D gjP holds if and only if
.f /jQ D .g/jQ .
Recall that E D ff 2 Cp .!1  n/ W f ..; i // D f ..; i // for any  and
i < ng and F Dff 2 Cp .!1  m/ W f ..; i //Df ..; i // for any  and i < mg
for every < !1 . We saw that E and F are closed second countable subsets of
Cp .!1  n/ and Cp .!1  m/ respectively; therefore E0 D E \ n and F0 D
F \ m are second countable closed subsets of n and m respectively. It is easy
to check that, for the families fE0 W < !1 g and fF0 W < !1 g, the premises of
Fact 13 are satisfied, so there exists a closed unbounded subset J 0  !1 such that
.E0 / D F0 for all 2 J 0 .
The set J \ J 0 being also closed and unbounded in !1 , we can find a limit
ordinal 2 J \ J 0 . Consider the sets K D ff 2 n W f ..; i // D 0 for all
i < ng and M D fg 2 m W g..; i // D 0 for all i < mg. It is easy to see that K
is a linear subspace of n of codimension n and M is a linear subspace of m of
codimension m. Therefore .K/ M by Fact 16.
Now fix any function f 2 K and let qf ..; i // D f ..; i // for any < . If
 then let qf ..; i // D 0 for all i < n. It follows from qf 2 E0 and 2 J 0
that h D .qf / 2 F0 and, in particular, h..; i // D 0 for any i < m. We also have
qf j  n D f j  n, so 2 J implies that hj  m D .f /j  m. The ordinal
is a limit while the functions h and .f / are continuous on  n; this, evidently,
implies that .f /..; i // D h..; i // D 0 for any i < m and therefore .f / 2 M .
An identical proof shows that 1 .g/ 2 K for any g 2 M , so .K/ D M ; this
contradiction proves that the spaces .Cp .!1 C 1//n and .Cp .!1 C 1//m are not
homeomorphic and hence our solution is complete.
V.101. Prove that a nonempty family B  exp.X X / is a base for some uniformity
on X if and only if it has the following properties:
T
(1)
B D ;
(2) for any U 2 B, there is V 2 B such that V 1  U ;
(3) for any U 2 B, there is V 2 B such that V V  U ;
(4) if U; V 2 B then there is W 2 B such that W  U \ V .
Solution.
T Suppose that .X; U / is a uniform space and B  U is a base of U . Then
 B by (U1); now if z 2 X 2 n then there is U 2 U such that z U . Since B
is a base of U , there
T is B 2 B
Twith B  U . Therefore z B and hence no point of
X 2 n belongs to B, i.e., B D , so we proved that B has (1).
Now, if U 2 B then V 0 D U 1 2 U ; choose V  V 0 with V 2 B. Then
1
V  .V 0 /1 D U , so (2) is proved. Analogously, if U 2 B then there is V 0 2 U
such that V 0 V 0  U ; choose V  V 0 with V 2 B. Then V V  V 0 V 0  U
and hence we also have (3).
To see that (4) holds too, assume that U; V 2 B; then U \ V 2 U by (U1), so
there is W 2 B such that W  U \ V ; this finishes the proof of necessity.
To establish sufficiency, take a nonempty B  exp.X  X / with the properties
(1)(4) and consider the family U D fU  X  X W there is B 2 B such that

2 Solutions of problems 001500

151

B T
U g. Given U 2 U , we have U
T  B T for some B 2TB and therefore
 U . It is clear that B  U , so U  B D and hence U D .
Next, take any U 2 U and fix B 2 B with B  U . The property (2) shows that
there is V 2 B such that V 1  B. This implies that V D .V 1 /1  B 1  U 1
and hence U 1 2 U . Analogously, if U; V 2 U then there are B; C 2 B with
B  U and C  V . It follows from (4) that we can find a set D 2 B such that
D  B \ C  U \ V ; thus U \ V 2 U and hence the condition (U1) is satisfied
for the family U .
Given U 2 U and V  X  X with V  U there is B 2 B such that B  U ;
then B  V and hence V 2 U . This proves that the first part of (U2) holds for U .
Finally, take any U 2 U and pick B 2 B with B  U . The property (3) shows
that there is a set V 2 B such that V V  B; then V 2 U and V V  U , so
(U2) is also fulfilled for U and hence U is a uniformity on X . Our definition of U
implies that B is a base of U , so we established sufficiency and hence our solution
is complete.
V.102. Suppose that a nonempty family S  exp.X  X / has the following
properties:
T
(1)
S D ;
(2) for any U 2 S, there is V 2 S such that V 1  U ;
(3) for any U 2 S, there is V 2 S such that V V  U .
Prove that S is a subbase for some uniformity on X . As a consequence, the union
of any family of uniformities on X is a subbase of some uniformity on X .
Solution. Let B be the family of all finite intersections of the elements of S. Then
B T
; and S
T  B. It is clear that TB for any B 2 B and hence we have
 B  S D which shows that B D .
Take any B 2 B and fix S1 ; : : : ; Sn 2 S such that B D S1 \ : : : \ Sn . It follows
from (2) that there exist T1 ; : : : ; Tn 2 S with Ti1  Si for any i  n. Then
C D T1 \ : : : \ Tn 2 B and C 1 D T11 \ : : : \ Tn1  S1 \ : : : \ Sn D B; this
proves that the properties (1) and (2) of Problem 101 are fulfilled for B. It is clear
that U \ V 2 B whenever U; V 2 B, so the condition (4) of Problem 101 is also
satisfied for B.
To prove that the property (3) of Problem 101 holds as well take any B 2 B and
pick S1 ; : : : ; Sn 2 S for which B D S1 \ : : : \ Sn . Our property (3) guarantees
existence of T1 ; : : : ; Tn 2 S such that Ti Ti  Si for any i  n. The set C D
T1 \ : : : \ Tn belongs to B; if .x; y/ 2 C C then there exists z 2 X such that
.x; z/ 2 C and .z; y/ 2 C . As a consequence, .x; z/ 2 Ti andT.z; y/ 2 Ti , i.e.,
.x; y/ 2 Ti Ti  Si for each i  n. This proves that .x; y/ 2 in Si D B and
hence C C  B, so the condition (3) of Problem 101 is satisfied.
Thus we can apply Problem 101 to the family B to conclude that there is a
uniformity U on X such that B is a base of U ; it is evident that S is a subbase
of U . Finally, observe that our properties (1)(3) are fulfilled for any uniformity and
hence for any union of uniformities. Therefore any union of uniformities on X is a
base of some uniformity of X .

152

2 Solutions of problems 001500

V.103. Let .X; U / be a uniform space. Prove that


(1) Int.A/ D fx W U.x/  A for some U 2 U g for any set A  X ; in particular,
x 2 Int.U.x// for any U 2 U ;
(2) if B is a base of the uniformity U then, for any x 2 X and O 2 .x; X / there
is B 2 B such that, B.x/  O. In particular, the family fInt.B.x// W B 2 Bg
is a local base of the space X at x.
(3) if S is a subbase of
T U then, for any x 2 X and O 2 .x; X /, there is a finite
S 0  S such that fS.x/ W S 2 S 0 g  O.
(4) for any U 2 U , the interior (in X  X ) of the set U also belongs to U . As a
consequence,
the family of all open symmetric elements of U is a base of U ;
T
(5) A D TfU.A/ W U 2 U g for any A  X ;
(6) B D fU B U W U 2 U g for any B  X  X ;
(7) the family of all closed symmetric elements of U is a base of U .
Solution. To prove (1) let OA D fx W U.x/  A for some U 2 U g. If x 2 Int.A/
then, by definition of the topology generated by U , there is U 2 U such that x 2
U.x/  Int.A/  A, so x 2 OA . This proves that Int.A/  OA .
Now, if x 2 OA then fix U; V 2 U for which U.x/  A and V V  U . Take
a point y 2 V .x/; if z 2 V .y/ then it follows from .x; y/ 2 V and .y; z/ 2 V that
.x; z/ 2 V V , so z 2 .V V /.x/  U.x/  A. Thus V .y/  A for any y 2 V .x/
which shows that V .x/  OA . It turns out that, for any x 2 OA , there is V 2 U
such that V .x/  OA ; therefore OA  A is an open set and hence OA  Int.A/.
We checked that OA D Int.A/, so (1) is proved.
To deal with (2) observe that, by definition of the topology generated by U , there
is a set U 2 U with U.x/  O; pick B 2 B such that B  U . Then B.x/ 
U.x/  O; since also x 2 Int.B.x//  B.x/  O (see (1)), we proved that the
family fInt.B.x// W B 2 Bg is a local base at x.
As to (3), take a set U 2 U for which U.x/  O. The family S being a subbase
of U there are n 2 ! and S1 ; : : : ; Sn 2 S such that S D S1 \ : : : \ Sn  U . If
y 2 S1 .x/\: : :\Sn .x/ then .x; y/ 2 Si for any i  n and therefore .x; y/ 2 S  U
which implies that y 2 U.x/  O. Thus S1 .x/ \ : : : \ Sn .x/  O, so the family
S 0 D fS1 ; : : : ; Sn g is as promised.
To settle (4) take any U 2 U ; there exists a set V0 2 U such that V0 V0  U .
Applying the property (U2) again we can find V1 2 U with V1 V1  V0 . Therefore
.V1 V1 / .V1 V1 /  V0 V0  U . It follows from  V1 that V1  V1 V1 , so
V1 .V1 V1 /  .V1 V1 / .V1 V1 /  U . It is easy to see that the composition is
associative, i.e., A .B C / D .A B/ C for any A; B; C  X  X ; this makes it
possible to omit parenthesis in the expressions that involve composition, so we will
write V1 V1 V1 instead of V1 .V1 V1 /.
The set V D V1 \ .V1 /1 is a symmetric element of U and it follows from
V  V1 that V V V  V1 V1 V1  U . If .x; y/ 2 V then x 2 G D Int.V .x//
and y 2 H D Int.V .y// (see (1)). The set Q D G  H is open in X  X and
.x; y/ 2 Q. If .z; t / 2 Q then .x; z/ 2 V and .y; t / 2 V ; the set V being symmetric
we have .z; x/ 2 V ; this, together with .x; y/ 2 V and .y; t / 2 V implies that
.z; t / 2 V V V  U .

2 Solutions of problems 001500

153

The point .z; t / 2 Q was taken arbitrarily, so Q  U and hence the point
.x; y/ 2 Q  U belongs to the interior of U . Thus V  Int.U / which shows
that Int.U / 2 U . This proves that open elements of U form a base of U . Now, if
U 2 U then V D Int.U / is open in X  X ; it is an easy exercise that V 1 is open in
X  X as well. Therefore W D V \ V 1 is an open symmetric element of U with
W  U , so open symmetric elements of U also constitute a base of U , i.e., (4) is
proved.
For (5), fix a set A  X ; given U 2 U , it follows from (U2) and (4) that there is
a symmetric V 2 U with V  U . For any x 2 A, the set V .x/ is a neighborhood
of x by (1), so V .x/ \ A ;. Therefore there is a 2 A with .x; a/ 2 V and hence
.a; x/ 2 V , i.e., x 2 V .a/  U.a/ which shows that
T x 2 U.A/. The point x 2 A
and the set U 2 U were chosen arbitrarily, so A  fU.A/ W U 2 U g.
To establish the opposite inclusion take any point x 2 X nA. There is U 2 U with
U.x/  X nA; apply (U2) and (4) to choose a symmetric V 2 U such that V  U .
If x 2 V .a/ for some a 2 A then .a; x/ 2 V and hence .x; a/ 2 V ; this implies
a 2 V .x/ while V T
.x/  U.x/  X nA. This contradiction
shows that x V .A/
T
and therefore x fU.A/ W U 2 U g whence A D fU.A/ W U 2 U g, i.e., (5) is
proved.
T
Passing to (6), fix a set B  X  X and let P D fU B U W U 2 U g.
Given .x; y/ 2 B fix any set U 2 U and apply (4) to choose a symmetric set V 2 U
such that V  U . Then V .x/  V .y/ is a neighborhood of .x; y/ (see (1)), so
there is .z; t / 2 .V .x/  V .y// \ B. The set V being symmetric, it follows from
.y; t / 2 V that .t; y/ 2 V which implies, together with .x; z/ 2 V and .z; t / 2 B,
that .x; y/ 2 V B V  U B U . This shows that B  U B U for any
U 2 U , i.e., B  P .
To obtain the opposite inclusion take any point .x; y/ 2 .X  X /nB. It follows
from (2) and (U1) that there is U 2 U such that .U.x/  U.y// \ B D ;. Take a
symmetric V 2 U with V  U (see (4)). If .x; y/ 2 V B V then there are z; t 2 X
for which .x; z/ 2 V; .z; t / 2 B and .t; y/ 2 V . The set V being symmetric we
have t 2 V .y/ and therefore .z; t / 2 .V .x/  V .y// \ B which is a contradiction.
Thus P  B and hence P D B, i.e., (6) is proved.
To prove (7) take any U 2 U and find a set V0 2 U with V0 V0  U . Apply (U2)
again to obtain a set V1 2 U such that V1 V1  V0 . It is easy to see that V1 V1 V1 
U . It follows from (4) that there is a symmetric V 2 U with V  V1 ; consequently,
V V V  V1 V1 V1  U . Now, (6) implies that V  V V V  U . To see that V
 1
is symmetric, it suffices to show that V
 V , so assume toward a contradiction
that .x; y/ 2 V and .y; x/ V . It follows from (6) that there is W 0 2 U such
that .y; x/ W 0 V W 0 . Choose a symmetric W 2 U such that W  W 0 ; it
is straightforward that R D W V W is a symmetric set for which .x; y/ 2 R
and .y; x/ R; this contradiction shows that G D V is a closed symmetric set
contained in U . Finally, observe that G 2 U because V  G and V 2 U . Thus
every U 2 U contains a closed symmetric G 2 U , so closed symmetric elements of
U constitute a base of U ; this proves (7) and makes our solution complete.

154

2 Solutions of problems 001500

V.104. Given uniform spaces .X; U / and .Y; V/, prove that every uniformly continuous map f W X ! Y is continuous. In particular, every uniform isomorphism is a
homeomorphism.
Solution. Fix an arbitrary point x 2 X and let y D f .x/. To prove that f is
continuous at x take a set O 2 .y; Y /. There is V 2 V with V .y/  O;
by uniform continuity of f , there exists a set U 2 U such that .x; y/ 2 U
implies .f .x/; f .y// 2 V . The set G D Int.U.x// is open in X and x 2 G
(see Problem 103). If t 2 G then t 2 U.x/ and hence .x; t / 2 U ; consequently,
.y; f .t // 2 V which shows that f .t / 2 V .y/  O. The point t 2 G was chosen
arbitrarily, so f .G/  V .x/  O, i.e., the set G witnesses continuity of the map f
at the point x.
V.105. Suppose
that .Xt ; Ut / is a uniform space for every t 2 T and consider the
Q
set X D t2T Xt . Let pt W X ! Xt be the natural projection for every t 2 T ;
prove that
S
(1) the family S D fpt1 .Ut / W t 2 T g is a subbase of a unique uniformity on X ,
i.e., the uniform product .X; U / of the spaces f.Xt ; Ut / W t 2 T g is well defined;
(2) every map pt W .X; U / ! .Xt ; Ut / is uniformlyQcontinuous;
(3) U coincides with the topology of the product f.Xt ; Ut / W t 2 T g.
Solution. Let D .X / be the diagonal of X . Denote by t the diagonal .Xt /
of theTspace Xt and let St D pt1 .Ut /; it follows from  .pt  pt /1 . t / that
 St for every t 2 T .
Fix t 2 T , a set U 2 St and V 2 Ut such that U D .pt  pt /1 .V /. A point
.x; y/ from X  X belongs to the set U 1 if and only if .y; x/ 2 U which happens
if and only if .pt .y/; pt .x// 2 V which occurs if and only if .pt .x/; pt .y// 2 V 1 .
This shows that U 1 D .pt  pt /1 .V 1 / and hence
T
(i)  St and, for any U 2 St , the set U 1 also belongs to St .
Next, choose a set V 0 2 Ut such that V 0 V 0  V . The set W D .pt  pt /1 .V 0 /
belongs to St . If .x; z/ 2 W W then there is y 2 X such that .x; y/ 2 W and
.y; z/ 2 W . This implies that .pt .x/; pt .y// 2 V 0 and .pt .y/; pt .z// 2 V 0 and
therefore .pt .x/; pt .z// 2 V 0 V 0  V . Thus .x; z/ 2 U for any .x; z/ 2 W W ,
i.e., W W  U . This proves that
(ii) for any U 2 St there is W 2 St such that W W  U .
S
It is straightforward that the family S D t2T St also has the properties (i) and
(ii). Given a point .x; y/ 2 .X  X /n we have x y, so there is t 2 T such
that pt .x/ pt .y/ and hence .pt .x/; pt .y// 2 .Xt  Xt /n t . The family Ut being
a uniformity on Xt , there is V 2 Ut such that .pt .x/; pt .y// V and therefore
.x; y/ .pt  pt /1 .V / 2 St  S. This implies that
T
(iii) fS W S 2 Sg D ,
so the family S satisfies the premises of Problem 102 and hence there exists a
uniformity for which S is a subbase. The family B of all finite intersections of

2 Solutions of problems 001500

155

the elements of S has to be a base of any uniformity U for which S is a subbase.


Besides, U D fE  X  X W there is B 2 B with B  Eg, so U is uniquely
determined by B and hence by S. This settles (1).
Given V 2 Ut the set .pt  pt /1 .V / belongs to St and hence to U . This shows
that every pt is uniformly continuous, i.e., we Q
established (2).
Denote by  the topology of the product t2T .Xt ; Ut / and let e W X ! X
be the identity map. To see that e W .X; U / ! .X; / is continuous, observe that
every map pt e D pt W .X; U / ! .Xt ; Ut / is uniformly continuous being the
natural projection of the uniform product .X; U / onto the factor .Xt ; Ut / (see (2));
thus pt is continuous by Problem 104. Therefore e is continuous by TFS-102; as a
consequence, e 1 .V / D V 2 U for any V 2 , i.e.,   U .
To show that e W .X; / ! .X; U / is also continuous fix a point x 2 X and a
set O 2 U with x 2 O. There is U 2 U such that U.x/  O; the family S being a
subbase of .X; UT
/ we can choose a finite S  T and a set Ut 2 Ut for any t 2 S in
such a way that f.pt  pt /1 .Ut / W t 2 S g  U .
For every t 2 T let St D .pt  pt /1 .Ut /; it follows from Problem
103 that there
T
is Vt 2 Ut such that pt .x/ 2 Vt  Ut .pt .x//. The set V D fpt1 .Vt / W t 2 S g
belongs to  and x 2 V . If y 2 V then pt .y/ 2 Vt ; thus .p
Tt .x/; pt .y// 2 Ut and
hence .x; y/ 2 St for any t 2 S . This shows that .x; y/ 2 fSt W t 2 S g  U ; an
immediate consequence is that y 2 U.x/  O. Since the point y 2 V was chosen
arbitrarily, we proved that V D e.V /  O and hence the map e W .X; / ! .X; U /
is continuous at every point x 2 X . Finally, observe that e 1 .V / D V 2  for any
V 2 U , so U   and hence U D , i.e., (3) is proved, so our solution is
complete.
V.106. Let .X; U / be the uniform product of the family f.Xt ; Ut / W t 2 T g. Given a
uniform space .Y; V/, prove that a map f W Y ! X is uniformly continuous if and
only if pt f W Y ! Xt is uniformly continuous for any t 2 T . Here, as always,
the map pt W X ! Xt is the natural projection.
Solution. It is an easy exercise that a composition of two uniformly continuous
maps is a uniformly continuous map. Therefore, if f W Y ! X is uniformly
continuous then so is every map pt f (see Problem 105). This proves necessity.
To establish sufficiency assume that the map pt f isS
uniformly continuous for
any t 2 T and fix a set U 2 U . Since the family S D fpt1 .Ut / W t 2 T g is a
subbase
of U , there exists a finite S  T and a set Ut 2 Ut for every t 2 S such that
T
f.pt  pt /1 .Ut / W t 2 S g  U ; let St D .pt  pt /1 .Ut / for each t 2 S . The set
Vt D .f  f /1 .St / D ..pt f /  .pt f //1 .Ut / belongs to V for every
Tt 2 S
because the map pt f is uniformly continuous. Consequently, the set V D t2S Vt
belongs to V; if .z; t / 2 V then .z; t / 2 Vt and hence .f .z/; f .t // 2 St for every
t 2 S.
T
This implies that .f .z/; f .t // 2 t2S St  U for any .z; t / 2 V and therefore
.f  f /.V /  U , i.e., V  .f  f /1 .U / which shows that .f  f /1 .U / 2 V.
The set U 2 U was chosen arbitrarily so the map f is uniformly continuous and
hence we completed the proof of sufficiency.

156

2 Solutions of problems 001500

V.107. Prove that, for any uniform space .X; U /, a pseudometric d W X  X ! R


is uniformly continuous with respect to the uniform square of .X; U / if and only if
the set Or D f.x; y/ 2 X  X W d.x; y/ < rg belongs to U for any r > 0. Such
pseudometrics will be called uniformly continuous on .X; U /.
Solution. Let .X  X; V/ be the uniform square of the space .X; U /. For any point
z D .x; y/ 2 X  X let p0 .z/ D x and p1 .z/ D y. The maps p0 ; p1 W X  X ! X
are the natural projections of X  X onto its factors so the family p01 .U / [ p11 .U /
is a subbase of .X  X; V/.
Assume that the pseudometric d is uniformly continuous and fix r > 0. There
exists V 2 V such that jd.z1 /  d.z2 /j < r for any point .z1 ; zT
We can find
2/ 2 V . T
1
finite families S0  p01 .U / and
S

p
.U
/
such
that
S
D
.
S
/\.
S1 /  V .
0
1
T 1
T
It is easy to see that S0 D
S0 2 p01 .U / and S1 D
S1 2 p11 .U /, so fix
W0 ; W1 2 U for which .p0  p0 /1 .W0 / D S0 and .p1  p1 /1 .W1 / D S1 .
Since W D W0 \ W1 2 U , the set G D .p0  p0 /1 .W / \ .p1  p1 /1 .W /
belongs to V; besides, G  S0 \ S1  V and therefore jd.z0 /  d.z1 /j < r for any
.z0 ; z1 / 2 G. Now, if z D .x; y/ 2 W then let z0 D z and z1 D .y; y/. It is clear that
.x; y/ D .p0 .z0 /; p0 .z1 // 2 W and .y; y/ D .p1 .z0 /; p1 .z1 // 2 W , so .z0 ; z1 / 2 G
and therefore jd.x; y/  d.y; y/j D d.x; y/ < r; an immediate consequence is that
G  Or and hence Or 2 U . Thus Or 2 U for any r > 0, i.e., we have established
necessity.
For sufficiency, assume that Or 2 U for any r > 0 and fix " > 0. Then W D
O"=2 2 U and hence V D .p0  p0 /1 .W / \ .p1  p1 /1 .W / 2 V. Take any point
.z0 ; z1 / 2 V ; then z0 D .x0 ; y0 / and z1 D .x1 ; y1 / where f.x0 ; x1 /; .y0 ; y1 /g  W ,
so it follows from the triangle inequality that
d.x0 ; y0 /  d.x0 ; x1 /Cd.x1 ; y1 /Cd.y1 ; y0 / <

"
"
Cd.x1 ; y1 /C D d.x1 ; y1 /C";
2
2

so d.x0 ; y0 /  d.x1 ; y1 / < ". Analogously,


d.x1 ; y1 /  d.x1 ; x0 /Cd.x0 ; y0 /Cd.y0 ; y1 / <

"
"
Cd.x0 ; y0 /C D d.x0 ; y0 /C";
2
2

which shows that d.x1 ; y1 /d.x0 ; y0 / < " and therefore jd.x1 ; y1 /d.x0 ; y0 /j < ".
We proved that jd.z0 /  d.z1 /j < " for any point .z0 ; z1 / 2 V , so the function
d W .X  X; V/ ! R is uniformly continuous, i.e., we settled sufficiency.
V.108. Suppose that a sequence fUn W n 2 !g of subsets of X X has the following
properties:
(1) U0 D X  X and  Un for any n 2 !;
(2) the set Un is symmetric and UnC1 UnC1 UnC1  Un for any n 2 !.
Prove that there exists a pseudometric d on the set X such that, for any n 2 N,
we have Un  f.x; y/ W d.x; y/  2n g  Un1 .

2 Solutions of problems 001500

157

Solution. Given x; y 2 X , a set C D fx0 ; x1 ; : : : ; xn g  X is a chain which


connects the points x and y if x0 D x and xn D y; we will say that the number of
links of C is equal to n. It is an easy exercise, that for any A  X  X with  A,
. / a point .x; y/ 2 X  X belongs to A A A if and only if there exist z; t 2 X
such that f.x; z/; .z; t /; .t; y/g  A.
An immediate consequence of . / is that UnC1  Un for each
T n 2 !. Given
any point .x; y/ 2 X  X let  .x; y/ D 0 if .x; y/ 2 U D fUn W n 2 !g.
If the point .x; y/ belongs to .X  X /nU then there is a unique n 2 ! such that
.x; y/ 2 P
Un nUnC1 ; let  .x; y/ D 2n . For each chain C D fx0 ; : : : ; xn g the number
l.C / D n1
iD0  .xi ; xiC1 / can be thought of as the length of the chain C . If x; y 2
X then the formula d.x; y/ D inffl.C / W C is a chain that connects x and yg
defines a function d W X  X ! 0; C1/; we will show that d is the promised
pseudometric.
Since fx; yg is a chain which connects x and y, we have d.x; y/   .x; y/ for
any x; y 2 X . Therefore d.x; x/   .x; x/ D 0 and hence d.x; x/ D 0 for any
x 2 X.
To prove that d is symmetric observe that  .x; y/ D  .y; x/ for any x; y 2 X
because every Un is a symmetric set. Now if x; y 2 X and C D fx0 ; x1 ; : : : ; xn g is a
chain which connects x and y then the chain C 0 D fxn ; xn1 ; : : : ; x1 ; x0 g connects
y and x; it follows from the equalities  .xi ; xiC1 / D  .xiC1 ; xi /; i D 0; : : : ; n  1,
that l.C / D l.C 0 /. Thus, for every chain C connecting x and y there is a chain C 0
which connects y and x such that l.C 0 / D l.C /. An evident consequence is that
d.x; y/ D d.y; x/ for any x; y 2 X .
To see that the triangle inequality also holds for d take any points x; y; z 2 X
and " > 0. There exist chains C 0 D fx0 ; : : : ; xn g and C 00 D fy0 ; : : : ; yk g such that
l.C 0 / < d.x; y/ C 2" and l.C 00 / < d.y; z/ C 2" while C 0 connects x and y and C 00
connects y and z. The set C D fx0 ; : : : ; xn ; y0 ; : : : ; yk g is a chain connecting the
points x and z, so d.x; z/  l.C / D l.C 0 / C l.C 00 / < d.x; y/ C d.y; z/ C ". Thus
d.x; z/ < d.x; y/Cd.y; z/C" for any " > 0 and hence d.x; z/  d.x; y/Cd.y; z/,
i.e., we indeed have the triangle inequality for d , so d is a pseudometric on the set X .
Our next step is to establish that
 d.x; y/   .x; y/ for any x; y 2 X .
We have already seen that the rightmost inequality in (i) is true, so let us prove
that d.x; y/  12  .x; y/ for any x; y 2 X . It suffices to show that,
(ii) for any chain C which connects the points x and y, we have l.C /  12  .x; y/.
(i)

1
 .x; y/
2

We will prove (ii) by induction on the number m of links of C . If m D 1 then


C D fx; yg, so l.C / D  .x; y/  12  .x; y/. Now assume that m > 1 and we
have established that l.C /  12  .x; y/ whenever C is a chain with < m-many links
connecting x and y.
To accomplish the inductive step take any chain C D fx0 ; : : : ; xm g with x0 D x
and xm D y. Letting a0 D 0 and ak D  .x0 ; x1 / C : : : C  .xk1 ; xk / for any
k 2 f1; : : : ; mg we obtain a sequence a0  : : :  am in which a D am is equal to the
length of the chain C . It is easy to see that there exists a number k 2 f0; : : : ; m  1g
such that ak  a2 and akC1  a2 .

158

2 Solutions of problems 001500

If .x; y/ 2 U then  .x; y/ D 0, so l.C /  12  .x; y/. If the point .x; y/ does not
belong to U then .x; y/ 2 Un nUnC1 for some n 2 ! and hence  .x; y/ D 2n . If
H D f.x; xk /; .xk ; xkC1 /; .xkC1 ; y/g  UnC2 then .x; y/ 2 UnC2 UnC2 UnC2 
UnC1 (see . /) which is a contradiction. Thus one of the elements of H does not
belong to UnC2 .
If .xk ; xkC1 / UnC2 then l.C /   .xk ; xkC1 /  2n1 D 12  .x; y/. In the case
when .x; xk / UnC2 apply the induction hypothesis to the chain C 0 D fx0 ; : : : ; xk g
to see that ak D l.C 0 /  12  .x; xk /  12 2n1 D 2n2 . By our choice of k, we
have l.C / D a  2ak  2n1 D 12  .x; y/. Finally, if .xkC1 ; xm / UnC2 then we
can apply the induction hypothesis to the chain C 00 D fxkC1 ; : : : ; xm g. This gives
1
 .xkC1 ; xm /  l.C 00 / D l.C /  akC1  a2 , so it follows from  .xkC1 ; xm / 
2
n1
that a2  2n2 , i.e., a  2n1 D 12  .x; y/, so we proved (ii). The leftmost
2
inequality of (i) is an immediate consequence of (ii), so (i) is proved as well.
Given an arbitrary n 2 !, we have  .x; y/  2n for any .x; y/ 2 Un , so
d.x; y/   .x; y/  2n ; this shows that Un  f.x; y/ W d.x; y/  2n g. Now
if d.x; y/  2n then  .x; y/  2n1 by the property (i), so .x; y/ 2 Un1 ; this
implies that f.x; y/ W d.x; y/  2n g  Un1 and hence our solution is complete.
V.109. Given a uniform space .X; U / and U 2 U , prove that there is a uniformly
continuous pseudometric  on .X; U / such that f.x; y/ 2 X  X W .x; y/ < 1g
 U.
Solution. We will first prove the following easy statement.
(i) Suppose that A; B; C  X  X and  A \ B \ C . If B B  A and
C C  B then C C C  A.
To show that (i) holds take a point .x; y/ 2 C C C ; there exist z; t 2 X
such that f.x; z/; .z; t /; .t; y/g  C . As a consequence, .x; t / 2 C C  B; it
follows from f.t; y/; .y; y/g  C that .t; y/ 2 C C  B, so f.x; t /; .t; y/g 
B and therefore .x; y/ 2 B B  A, so (i) is proved.
It follows from Problem 103 that there exists a symmetric set U1 2 U with U1 
U . The property (i) and Problem 103 make it possible to construct a sequence
fUn W n 2 Nnf1gg of symmetric elements of U such that UnC1 UnC1 UnC1 
Un for any n 2 N. Letting U0 D X  X we obtain a sequence fUn W n 2 !g
which satisfies the premises of Problem 108. Thus there exists a pseudometric
d W X  X ! 0; C1/ such that
(ii) Un  f.x; y/ 2 X  X W d.x; y/  2n g  Un1 for any n 2 N.
It is straightforward that  D 4d is also a pseudometric on X . If .x; y/ < 1
then d.x; y/ < 14 D 22 , so (ii) shows that .x; y/ 2 U1  U . Therefore we have
the inclusion f.x; y/ 2 X  X W d.x; y/ < 1g  U .
Finally, take any r > 0 and fix n 2 ! such that 2n2 < r. If .x; y/  2n then
d.x; y/  14 2n D 2n2 < r which shows, together with (ii), that
Un  f.x; y/ 2 X  X W d.x; y/  2n g  f.x; y/ 2 X  X W d.x; y/ < rg.

2 Solutions of problems 001500

159

An immediate consequence is that f.x; y/ 2 X  X W d.x; y/ < rg 2 U for any


r > 0, so the pseudometric  is uniformly continuous on .X; U / by Problem 107.
V.110. Prove that a topological space X is Tychonoff if and only if there exists a
uniformity U on the set X such that .X / D U .
Solution. Suppose first that there exists a uniformity U on the set X such that U D
.X /. Given x 2 X pick any y 2 X nfxg; the point .y; x/ does not belong to
the diagonal of the space X , so there is U 2 U such that .y; x/ U and therefore
x U.y/. It follows from Problem 103 that V D Int.U.y// 2 .y; X /; since
x V , we proved that every y 2 X nfxg has a neighborhood V  X nfxg. Thus
X nfxg is open for every x 2 X , i.e., X is a T1 -space.
Now fix a point a 2 X and O 2 .a; X /. There is a set U 2 U such that
U.a/  O; by Problem 109 there exists a uniformly continuous pseudometric  W
X  X ! R such that G D f.x; y/ W .x; y/ < 1g  U . Since  is continuous on
X  X by Problem 104, the function g W X ! R, defined by g.x/ D .a; x/ for
any x 2 X , is continuous as well.
It is clear that g.a/ D 0; given x 2 X nO, it follows from U.a/  O that
.a; x/ U and hence g.x/ D .a; x/  1. Thus the function f W X ! R defined
by the formula f .x/ D 1  nnfg.x/; 1g for any x 2 X , is continuous on X and
f .a/ D 1 while f .X nO/  f0g. This shows that X is a Tychonoff space, i.e., we
proved sufficiency.
To establish necessity, suppose that X is a Tychonoff space and consider the set
O.f; r/ D f.x; y/ 2 X  X W jf .x/  f .y/j < rg for each f 2 Cp .X /. Clearly,
every element of the family S D fO.f; r/ W f 2 Cp .X /; r > 0g contains . If
.x; y/ then x y, so there is f 2 Cp .X / for which f .x/
T D 0 and f .y/ D 1;
an immediate consequence is that .x; y/ O.f; 1/ 2 S, so S D .
Suppose that U D O.f; r/ 2 S; then V D O.f; 2r / 2 S. If f.x; y/; .y; z/g  V
then jf .x/  f .z/j  jf .x/  f .y/j C jf .y/  f .z/j < 2r C 2r D r and hence
.x; z/ 2 U . This proves that V V  U , so the family S satisfies the conditions (1)
and (3) of Problem 102. Since every element of S is a symmetric subset of X  X ,
the condition (2) is satisfied as well, so there is a uniformity U on the set X such
that S is a subbase of U .
Take any O 2 .X / and x 2 O. There is f 2 Cp .X / such that f .x/ D 1 and
f .X nO/  f0g. The set U D O.f; 1/ belongs to the family U ; if y 2 U.x/ then
jf .y/  f .x/j D jf .y/  1j < 1 which shows that f .y/ > 0 and hence y 2 O.
Therefore U.x/  O, i.e., we proved that, for every x 2 O there is U 2 U for
which U.x/  O. Consequently, O 2 U , so we proved that .X /  U .
Finally, fix a set O 2 U ; given a point x 2 O there is U 2 U with U.x/  O.
The family S being a subbase of U we can find k 2 N and V1 ; : : : ; Vk 2 S for
which V D V1 \ : : : \ Vk  U . Choose fi 2 Cp .X / and ri > 0 such that Vi D
O.fi ; ri / for all i  k. The function fi is continuous so the set Vi .x/ D fy 2
X W jfi .y/  fi .x/j < ri g is open in X for every i 2 f1; : : : ; kg; thus V .x/ D
V1 .x/ \ : : : \ Vk .x/ 2 .x; X / and V .x/  U.x/  O. It turns out that, for
every point x 2 O, there is a set V .x/ 2 .x; X / with V .x/  O; this shows
that O 2 .X / and proves that U  .X / and hence U D .X /, i.e., we settled
necessity.

160

2 Solutions of problems 001500

V.111. Given a metric  on a set X and a number r > 0, consider the set Ur D
f.x; y/ 2 X  X W .x; y/ < rg. Prove that the family B D fUr W r > 0g forms a
base of some uniformity U on the set X . The uniformity U is called the uniformity
induced by the metric . A uniform space .X; U / is called uniformly metrizable if
U D U for some metric  on the set X .
Solution. It is evident that the diagonal of the space X is contained in every
element of B. If .x; y/ 2 .X
T  X /n then x y and hence r D .x; y/ > 0; it is
clear that .x; y/ Ur , so B D .
An immediate consequence of the definition is that every Ur is a symmetric
subset of X  X . Now if U; V 2 B then U D Ur and V D Us for some positive
numbers r and s. If t D nnfr; sg then U \ V D Ut 2 B, so the properties (1), (2),
and (4) of Problem 101 are fulfilled for B.
To see that the property (3) of Problem 101 also holds for B take any U 2 B; then
U D Ur for some r > 0. The set V D Ur=2 also belongs to B; if f.x; y/; .y; z/g  V
then .x; z/  .x; y/C.y; z/ < 2r C 2r D r, so .x; z/ 2 Ur D U . Thus V V  U ,
so all conditions (1)(4) of Problem 101 are satisfied for B and hence there exists a
uniformity U such that B is a base of U .
V.112. Prove that a uniform space .X; U / is uniformly metrizable if and only if U
has a countable base.
Solution. Assume that the space .X; U / is uniformly metrizable and hence there
exists a metric  such that the family B D fUr W r > 0g is a base for U ; here
Ur D f.x; y/ 2 X  X W .x; y/ < rg for any r > 0. The family B0 D fUr W r > 0
and r 2 Qg is countable and it is straightforward that B is a base for U ; this proves
necessity.
Now assume that there exists a countable base B for the uniformity U and let
fBn W n 2 Ng be an enumeration of B. Using Problems 101 and 103 it is easy to
construct by induction a family fUn W n 2 !g of symmetric elements of U with the
following properties:
(i) U0 D X  X and Un  Bn for each n 2 N;
(ii) UnC1 UnC1 UnC1  Un for any n 2 !.
By Problem 108, there exists a pseudometric d on the set X such that
(iii) Un  f.x; y/ W d.x; y/  2n g  Un1 for every n 2 N.
Given distinct x; y 2 X the point .x; y/ does not belong to the diagonal of the
space X , so there is U 2 U with .x; y/ U . The family B being a base of U we
can find n 2 N for which Bn  U and therefore .x; y/ Bn . It follows from (i) that
.x; y/ Un and hence d.x; y/ > 2n1 > 0 by (iii). Thus d is actually a metric on
X ; let us show that U is induced by d .
Let Or D f.x; y/ 2 X  X W d.x; y/ < rg for any r > 0; we must prove that the
family O D fOr W r > 0g is a base of U . If r > 0 then there is n 2 ! with 2n < r.
The property (iii) implies that Un  f.x; y/ 2 X  X W d.x; y/  2n g  Or , so
Or 2 U for any r > 0, i.e., O  U .

2 Solutions of problems 001500

161

Finally, if U 2 U then pick n 2 N with Bn  U . Then Un  U by (i); it


follows from the property (iii) that, for r D 2n1 > 0 we have the inclusions
Or  f.x; y/ 2 X  X W d.x; y/  rg  Un  U , so O is, indeed, a base of U , i.e.,
U is induced by the metric d . This proves sufficiency and completes our solution.
V.113. Prove that every uniform space is uniformly isomorphic to a subspace of a
product of uniformly metrizable spaces.
Solution. The following statement will help us to show that, for any uniform space,
there is a sufficiently large family of uniformly continuous mappings of this space
into uniformly metrizable spaces.
Fact 1. If .X; U / is a uniform space and d is a uniformly continuous pseudometric
on .X; U / then there exists a metric space .Y; / and a map ' W X ! Y such that
d.x; y/ D .'.x/; '.y// for any x; y 2 X . In particular, the map ' is uniformly
continuous if we consider Y with the uniformity V induced by the metric .
Proof. For any x; y 2 X , let x  y if and only if d.x; y/ D 0. Then  is an
equivalence relationship on X . Indeed, it follows from d.x; x/ D 0 that x  x;
now, if x  y then d.y; x/ D d.x; y/ D 0, so y  x. To check transitivity,
observe that x  y and y  z imply that d.x; y/ D d.y; z/ D 0, so d.x; z/ 
d.x; y/ C d.y; z/ D 0 which shows that x  z. Let Px D fy 2 X W y  xg for any
x 2 X.
The space X is the disjoint union of the classes of equivalence with respect to ,
i.e., for the family fPx W x 2 X g we have either
Px D Py or Px \ Py D ; for any
S
x; y 2 X . Choose a set Y  X such that fPy W y 2 Y g D X and Py \ Pz D ;
for distinct y; z 2 Y .
Given y; z 2 Y let .y; z/ D d.y; z/; the function  W Y  Y ! R is a metric
on Y . To prove it, assume that .y; z/ D 0 for some y; z 2 Y . If y z then
Py \ Pz D ; and hence z Py which shows that .y; z/ D d.y; z/ > 0. Therefore
.y; z/ D 0 implies y D z; the other axioms of metric are fulfilled for  because it
coincides with the pseudometric d on Y . Thus .Y; / is metric space.
For each x 2 X there is a unique y 2 Y with x 2 Py ; let '.x/ D y. To see that
the map ' W X ! Y has the promised properties take x; y 2 X . If a D '.x/ and
b D '.y/ then x 2 Pa and y 2 Pb ; therefore d.x; a/ D d.y; b/ D 0. Consequently,
.a; b/ D d.a; b/  d.a; x/ C d.x; y/ C d.y; b/ D d.x; y/. On the other hand,
we have the inequality d.x; y/  d.x; a/ C d.a; b/ C d.b; y/ D d.a; b/ D .a; b/
which implies that .a; b/ D d.x; y/, i.e., d.x; y/ D .'.x/; '.y// for any
x; y 2 X.
Let V be the uniformity induced on Y by the metric . To see that the map
' W .X; U / ! .Y; V/ is uniformly continuous take any V 2 V. There is r > 0
such that Wr D f.y; z/ 2 Y  Y W .y; z/ < rg  V . It is straightforward that
.'  '/1 .Wr /  Or D f.x; y/ 2 X  X W d.x; y/ < rg. The pseudometric d
being uniformly continuous, we have Or 2 U (see Problem 107), so it follows from
the equality U D .'  '/1 .V /  Or that U 2 U , so the map ' is uniformly
continuous and Fact 1 is proved.

162

2 Solutions of problems 001500

Returning to our solution fix a uniform space .X; U / and apply Problem 109 to
construct, for any U 2 U , a uniformly continuous pseudometric dU on the space
.X; U / such that OU D f.x; y/ 2 X  X W dU .x; y/ < 1g  U . Fact 1 guarantees
us that there exists a metric space .YU ; U / and a map 'U W X ! YU such that
dU .x; y/ D U .'U .x/; 'U .y// for any x; y 2 X and, in particular, 'U is uniformly
continuous if we
Q consider the set Y with the uniformity VU induced by U .
Let Y D
fYU W U 2 U g be the uniform product of the family fYU W
U 2 U g; since every YU is uniformly metrizable, it suffices to construct a uniform
isomorphism of .X; U / onto a subspace of Y . For every x 2 X let '.x/.U / D
'U .x/ for each U 2 U ; then ' W X ! Y is the diagonal product of the family
f'U W U 2 U g.
To show that the map ' W X ! X 0 D '.X / is a uniform isomorphism consider
the natural projection pU W Y ! YU for each U 2 U . It is straightforward that
pU ' D 'U , so the map pU ' is uniformly continuous for every U 2 U . It
follows from Problem 106 that the map ' is uniformly continuous.
Given distinct points x; y 2 X there is a set U 2 U such that .x; y/ U and
hence dU .x; y/  1. Then U .'U .x/; 'U .y//  1 which implies that 'U .x/
'U .y/, so '.x/ '.x/, i.e., the map ' is a bijection.
To finally see that ' 1 W X 0 ! X is uniformly continuous fix a set U 2 U . Then
W D f.s; t / 2 YU  YU W U .s; t / < 1g belongs to the uniformity VU and hence the
set V D f.a; b/ 2 X 0  X 0 W .pU .a/; pU .b// 2 W g is an element of the uniformity
V 0 induced on X 0 from Y . It is an easy exercise that .' 1  ' 1 /.V / coincides with
the set OU  U , so .' 1  ' 1 /1 .U / D .'  '/.U /  .'  '/.OU / D V
belongs to V 0 and therefore the map ' 1 is uniformly continuous. This proves that
' W X ! X 0 is a uniform isomorphism and completes our solution.
V.114. Given a uniform space .X; U / prove that the following conditions are
equivalent:
(i)
(ii)
(iii)
(iv)
(v)
(vi)
(vii)
(viii)

the space .X; U / is complete;


T
if C is a centered Cauchy family of closed subsets of XTthen C ;;
if C is a centered Cauchy family of subsets
T of X then fC W C 2 Cg ;;
if B is a Cauchy filter base on X then fB W B 2 Bg T;;
if B is a Cauchy filter base of closed subsets of X then B ;;
any Cauchy filter F on X converges to
Ta point x 2 X , i.e., .x; X /  F;
if E is a Cauchy ultrafilter on X then fE W E 2 Eg ;;
if E is a Cauchy ultrafilter on X then it converges to a point x 2 X .

Solution. Assume that .X; U / is complete and fix a centered Cauchy family C of
closed subsets of X . There exists a filter F  exp.X / such T
that C  F (see TFS117). It is evident
that
F
is
still
a
Cauchy
family,
so
P
D
fF W F 2 Fg ;.
T
T
T
Then P  fC W C 2 Cg D C, so C ;, i.e., we proved that (i)H)(ii).
Now, if the property (ii) holds and C  exp.X / is a centered Cauchy family then
C 0 D fC W C 2 Cg is a centered family of closed subsets of X ; take any U 2 U . By
Problem 103, there is a closed set V  X  X such that V 2 U and V  U . The
family C being Cauchy there is C 2 C with C C  V . An immediate consequence

2 Solutions of problems 001500

163

is that C  C  V DT
V  U ; this proves that C 0 is also a Cauchy family. Applying
(ii) we conclude that C 0 ;, i.e., (iii) holds and hence we settled (ii)H)(iii).
Every Cauchy filter base on X is a centered Cauchy family of subsets of X , so
(iii)H)(iv).
T
T If (iv) holds and B is a Cauchy filter base of closed subsets of X then
B D fB W B 2 Bg ; by (iv); this shows that (iv)H)(v).
Next assume that (v) holds and take a Cauchy filter F on the set X . Given F; G 2
F we have F \ G  F \ G; this implies that the family F 0 D fF W F 2 Fg is a
filter base of closed subsets of X . Take any U 2 U ; by Problem 103, there is a closed
set V  X  X such that V 2 U and V  U . The family F being Cauchy there is
F 2 F with F F  V . An immediate consequence is that F F  V D V  U ;
this proves that F 0 is also a CauchyTfamily.
Thus we can pick a point x 2 F 0 ; we claim that the filter F converges to x.
To see this take an arbitrary set O 2 .x; X /; there exists U 2 U with U.x/  O.
Apply Problem 103 to find a closed V  X  X such that V  U and V 2 U . The
family F being Cauchy, we can find F 2 F with F  F  V . Then F  F  V D
V  U ; given any y 2 F it follows from fx; yg  F that .x; y/ 2 F  F  U , so
y 2 U.x/. Thus F  U.x/  O and hence O 2 F; this proves that .x; X /  F
and settles the implication (v)H)(vi).
To establish (vi)H)(vii), assume that (vi) is true and take a Cauchy ultrafilter
E  exp.X /. Since any ultrafilter is a filter, we can apply (vi) to find a point x 2 X
such that .x; X /  E. If x E for some E 2 E then O D X nE 2 .x; X / and
hence
TO 2 E which, together with O \ E D ; gives a contradiction. Therefore
x 2 fE W E 2 Eg, so the implication (vi)H)(vii) is proved.
For (vii)H)(viii) suppose that (vii) is T
fulfilled and take any Cauchy ultrafilter
E  exp.X /; there exists a point x 2
fE W E 2 Eg. If O E for some
O 2 .x; X / then E D X nO 2 E (see TFS-117), so x E D E which is a
contradiction. Thus the ultrafilter E converges to x and (vii)H)(viii) is proved.
Finally assume that (viii) is satisfied and take any Cauchy filter F on the
set X . There exists an ultrafilter E  exp.X / with F  E (see TFS-117); it is
straightforward that E is also Cauchy, so it converges to a point x 2 X . If there
is F 2 F with x F then O D X nF 2 .x; X / which shows that O 2 E
which,Ttogether with F 2 E and O \ F D ;, gives a contradiction. Therefore
x 2
fF W F 2 Fg; this proves that (viii)H)(i) and hence our solution is
complete.
V.115. Prove that any closed uniform subspace of a complete uniform space is
complete.
Solution. Suppose that .X; U / is a complete uniform space and F is a closed
subspace of X ; denote by V the uniformity induced on F from .X; U /. Take any
centered family C of closed subsets of F which is Cauchy in .F; V/; all elements
of C are also closed in X . If U 2 U then V D U \ .F  F / 2 V; the family C
being Cauchy in .F; V/ there is C 2 C such that C  C  V 
T U . An immediate
consequence is that C is also a Cauchy family in .X; U /; thus C ; and hence
the space .F; V/ is complete by Problem 114.

164

2 Solutions of problems 001500

V.116. Let .X; U / be a uniform space such that some Y  X is complete


considered as a uniform subspace of .X; U /. Prove that Y is closed in X .
Solution. Let V be the uniformity induced on Y from .X; U /. If Y is not closed in
X then fix a point y 2 Y nY and let B D fO 0 \Y W O 0 2 .y; X /g. Obviously, B is a
filter base of subsets of Y . Given a set V 2 V fix U 2 U such that U \ .Y  Y / D V
and choose a symmetric W 2 U for which W W  U .
Apply Problem 103 to find an open neighborhood O 0 of the point y in X such that
0
O  W .y/; then O D O 0 \ Y 2 B. If z; t 2 O then f.y; z/; .y; t /g  W ; the set W
being symmetric, we have f.z; y/; .y; t /g  W and therefore .z; t / 2 W W  U .
This shows that O  O  U \ .Y  Y / D V , so B is a Cauchy
T family in .Y; V/.
The space .Y; V/ is complete, so there exists a point x 2 fclY .O/ W O 2 Bg;
the points x and y are distinct, so we can choose disjoint sets G; H 2 .X / such
that x 2 G and y 2 H . Then O D G \ Y 2 B and x clY .O/; this contradiction
shows that Y is closed in X .
V.117. Prove that, for any family A D f.Xt ; Ut / W t 2 T g of complete uniform
spaces, the uniform product .X; U / of the family A is complete.
Solution. Let pt W X ! Xt be the natural projection for any t 2 T . Suppose
that F is a Cauchy filter on the space .X; U /. It is easy to verify that the family
Ft D fpt .F / W F 2 Fg is a Cauchy filter on .Xt ; Ut /; by completeness of .Xt ; Ut /
we can apply Problem 114 to fix a point at 2 Xt with .at ; Xt /  Ft for every
t 2 T.
Define a point x 2 X by letting x.t / D at for each t 2 T and assume that
there exists a set F 2 F with x F . Then there isT
a finite S  T and a set
Ot 2 .at ; Xt / for any t 2 S such that the set O D fpt1 .Ot / W t 2 S g does
not meet F . However, Ot 2 Ft , so there is Wt 2 F such that pt .Wt / D Ot ; an
immediate consequence is that Wt  pt1 .Ot / and hence pt1 .Ot / 2 F for any
t 2 S . Therefore
O and F are
T
T two disjoint elements of F; this contradiction shows
that x 2 fF W F 2 Fg, so fF W F 2 Fg ; for any Cauchy filter F on .X; U /,
i.e., the space .X; U / is complete.
V.118. Let .X; U / be a uniform space such that the uniformity U is induced by a
metric . Prove that .X; U / is complete if and only if the metric space .X; / is
complete.
Solution. Assume first that the space .X; U / is complete and take a sequence F D
fFn W n 2 !g of nonempty closed subsets of X such that FnC1  Fn for any n 2 !
and diam.Fn / ! 0. It is clear that F is a filter base in X . Given U 2 U there exists
r > 0 for which Or D f.x; y/ 2 X  X W .x; y/ < rg  U . Pick n 2 ! such
that d D diam.Fn / < r; then .x; y/  d < r for any x; y 2 Fn which implies
that Fn  Fn  Or  U . Thus F is aTCauchy family; by completeness of .X; U /
we can apply Problem 114 to see that F ;. Therefore the metric space .X; /
is complete by TFS-236; this proves necessity.
Now assume that the metric space .X; / is complete and take a centered Cauchy
family F of closed subsets of X . Given any r > 0, the set Or=2 belongs to U , so

2 Solutions of problems 001500

165

there exists F 2 F such that F  F  Or=2 ; then d.x; y/ < 2r for any x; y 2 F
and hence diam.F /  2r < r. This shows that the family F hasTsets of arbitrarily
small diameter, so we can apply TFS-236 once more to see that F ;. Finally,
apply Problem 114 to conclude that .X; U / is complete; this settles sufficiency.
V.119. Let A be a dense subspace of a uniform space .X; U /. Suppose that f W
A ! Y is a uniformly continuous map of .A; UAX / to a complete uniform space
.Y; V/. Prove that there is a uniformly continuous map F W X ! Y such that
F jA D f .
Solution. If x 2 X then the set O \ A is nonempty for any O 2 .x; X /, so the
family Ox D fO \ A W O 2 .x; X /g is a filter base of subsets of A. An easy
consequence is that the family Ux D ff .P / W P 2 Ox g is a filter base of subsets of
Y for every x 2 X .
Fix a point x 2 X and V 2 V; by uniform continuity of f there is a set U 0 2 U
such that .f .a/; f .b// 2 V whenever .a; b/ 2 U 0 \ .A  A/. Apply Problem 103
to find a symmetric U 2 U for which U U  U 0 . The set O D Int.U.x// is an
open neighborhood of x in X by Problem 103, so P D O \ A 2 Ox and therefore
Q D f .P / 2 Ux . Now if z; t 2 Q then pick a; b 2 P with f .a/ D z; f .b/ D t .
Since fa; bg  U.x/, we have f.x; a/; .x; b/g  U and hence .a; x/ 2 U by the
symmetry of U . Consequently, .a; b/ 2 U U  U 0 which shows that .z; t / D
.f .a/; f .b// 2 V . The points z; t 2 Q were chosen arbitrarily, so Q  Q  V
and T
hence Ux is a Cauchy filter base in .Y; V/. By Problem 114 there is a point
y 2 fQ W Q 2 Ux g; let F .x/ D y. This defines a map F W X ! Y . T
Given a 2 A observe that f .a/ 2 Q for any Q 2 Ua , so f .a/ 2 fQ W Q 2
Ua g. If z 2 Y nff .a/g then there is V 2 V with .f .a/; z/ V ; by Problem 103,
we can find a closed W 2 V such that W  V . The family Ua being Cauchy,
there is Q 2 Ua for which Q  Q  W . The set W being closed we also have
Q  Q TW  V which shows that .f .a/; z/ Q  Q, i.e., z Q. Therefore
z R D fQ W Q 2 Ua g and hence f .a/ is the unique element of R; this proves
that F .a/ D f .a/ for each a 2 A, i.e., F jA D f .
To finally see that the map F W .X; U / ! .Y; V/ is uniformly continuous fix any
V 2 V and apply Problem 103 to find a closed set V0 2 V with V0  V . The map
f being uniformly continuous, we can find U 2 U such that .f .a/; f .b// 2 V0
whenever .a; b/ 2 U \ .A  A/. Apply Problem 103 to find a symmetric set W 2 U
with W W W  U . Now, if .x; y/ 2 W then H D Int.W .x// \ A 2 Ox
and G D Int.W .y// \ A 2 Oy , so Q0 D f .H / 2 Ux and Q1 D f .G/ 2 Uy .
If a 2 H and b 2 G then f.a; x/; .x; y/; .y; b/g  W which shows that .a; b/ 2
W W W  U and therefore .f .a/; f .b// 2 V0 . This proves that Q0  Q1  V0 ;
the set V0 being closed, we conclude that Q0  Q1  V0  V . By definition of F ,
we have F .x/ 2 Q0 and F .y/ 2 Q1 . Consequently, .F .x/; F .y// 2 Q0 Q1  V ;
the point .x; y/ 2 W was chosen arbitrarily so .F  F /.W /  V and hence F is a
uniformly continuous map.
V.120. Let .X; U / and .Y; V/ be complete uniform spaces. Suppose that A is a
dense subspace of X and B is a dense subspace of Y . Prove that every uniform
isomorphism between the uniform spaces .A; UAX / and .B; VBY / is extendable to a
uniform isomorphism between .X; U / and .Y; V/.

166

2 Solutions of problems 001500

Solution. Let idX W X ! X and idY W Y ! Y be the identity maps on the


spaces X and Y respectively and assume that a map ' W .A; UAX / ! .B; VBY / is a
uniform isomorphism. It is straightforward that the maps ' W .A; UAX / ! .Y; V/ and
' 1 W .B; VBY / ! .X; U / are uniformly continuous, so we can apply Problem 119
to find uniformly continuous maps W .X; U / ! .Y; V/ and  W .Y; V/ ! .X; U /
such that jA D ' and  jB D ' 1 .
Since .  /.y/ D '.' 1 .y// D y for any y 2 B, the continuous maps idY
and  coincide on a dense set B of the space Y . By Fact 0 of S.351, we have
 D idY . Analogously, . /.x/ D ' 1 .'.x// D x for any x 2 A, i.e.,
the continuous maps  and idX coincide on a dense subset A of the space X .
Therefore  D idX which shows that  D 1 and hence both maps and 
are uniform isomorphisms.
V.121. Prove that every uniform space .X; U / is uniformly isomorphic to a dense
subspace of a complete uniform space .X  ; U  /. The space .X  ; U  / is called the
completion of the space .X; U /. Prove that the completion of .X; U / is unique in the
sense that, if .Y; V/ is a complete uniform space such that .X; U / is a dense uniform
subspace of .Y; V/ then there is a uniform isomorphism W X  ! Y such that
.x/ D x for any x 2 X .
Solution. By Problem 113, Q
we can assume that X is a uniform subspace of the
uniform product .M; U / D t2T .Mt ; Ut / of some family f.Mt ; Ut / W t 2 T g of
uniformly metrizable spaces. Let t W Mt  Mt ! R be a metric which induces the
uniformity Ut for every t 2 T . It follows from TFS-237 that there exists a complete
metric space .Nt ; dt / such that Mt is a dense subspace of Nt and dt j.Mt Mt / D t
for each t 2 T .
If Wt is the uniformity on Nt induced by the metric dt then it is easy to see that
the uniformity induced by Wt on
Q Mt coincides with Ut for any t 2 T . Another easy
exercise is that if .N; W/ D t2T .Nt ; Wt / is the uniform product of the family
f.Nt ; Wt / W t 2 T g then the uniformity W induces the uniformity U on the set M .
Thus .X; U / is a uniform subspace of the space .N; W/; apply Problem 117
to see that the space .N; W/ is complete. Let X  be the closure of X in N and
denote by U  the uniformity induced on X  from .N; W/. Clearly, .X; U / is a dense
uniform subspace of the space .X  ; U  /; it follows from Problem 115 that .X  ; U  /
is complete.
Finally, assume that .X; U / is a dense uniform subspace of some complete
uniform space .Y; V/. Letting '.x/ D x for any x 2 X we obtain a uniform
isomorphism of .X; U / onto itself. Considering ' to be an isomorphism between the
respective dense subspaces of .X  ; U  / and .Y; V/ (both of which coincide with X ),
we can apply Problem 120 to find a uniform isomorphism W .X  ; U  / ! .Y; V/
such that jX D ', i.e., .x/ D x for every x 2 X .
V.122. Let X be a paracompact Tychonoff space. Prove that the family of all
neighborhoods of the diagonal of X is a uniformity on X which generates .X /.

2 Solutions of problems 001500

167

Solution. Let U be the family of all neighborhoods of the diagonal of the


space X . Since X is Tychonoff, any point .x; y/ is closed in X  X , so
T the set
.X  X /nf.x; y/g belongs to U for any .x; y/ 2 .X  X /n . Thus D U .
The map ' W X X ! X X defined by '.x; y/ D .y; x/ for any .x; y/ 2 X X
is easily seen to be a homeomorphism for which '. / D . Therefore, for any
U 2 U , the set '.U / D U 1 is still a neighborhood of , so U 1 2 U . It is evident
that the intersection of two neighborhoods of is still a neighborhood of , so the
axiom (U1) is fulfilled for U .
If U 2 U then U is a neighborhood of , so every W  U is also a neighborhood
of , i.e., W 2 U . To check the second part of (U2) fix a set U 2 U ; then U0 D
Int.U /  . For each x 2 X fix a set Ox 2 .x; X / such that Ox  Ox  U0 ;
then O D fOx W x 2 X g is an open cover of the space X . By paracompactness of
X weScan find an open star refinement H of the cover O (see TFS-230). The set
V D fH  H W H 2 Hg is an open neighborhood of , so V 2 U .
Next, assume that f.a; b/; .b; c/g  V for some a; b; c 2 X ; then there are
H; G 2 H with .a; b/ 2 H  H and .b; c/ 2 G  G. This implies fa; bg  H and
fb; cg  G which shows that b 2 H \ G; by our choice of the cover H there is
x 2 X for which G [ H  Ox . Thus .a; c/ 2 Ox  Ox  U0  U ; this proves that
V V  U0  U , so the property (U2) also holds for U . Therefore the family U of
all neighborhoods of the diagonal of X is a uniformity on X .
To see that U generates the topology of X take any U 2 U and fix U0 2 .X X /
with  U0  U . Given any x 2 X the set O D U0 \.fxgX / is open in fxgX ;
there is an evident homeomorphism between fxg  X and X which takes O onto
U0 .x/, so U0 .x/  U.x/ is open in X . Therefore U.x/ is a neighborhood of x for
any x 2 X and U 2 U . If H 2 U then, for every x 2 H , there is U 2 U with
U.x/  H ; we already saw that every U.x/ is a neighborhood of x, so H is open
in X and hence we proved that U  .X /.
Finally, if O 2 .X / and x 2 O then pick a set W 2 .x; X / with W  O and
let G D X nW ; the set U D .O  O/ [ .G  G/ is an open neighborhood of the
diagonal, i.e., U 2 U . If .x; y/ 2 U then .x; y/ G  G, so y 2 O and hence
U.x/  O. This shows that O 2 U ; since we have chosen O 2 .X / arbitrarily,
we proved that .X /  U and hence .X / D U , i.e., the topology generated by U
coincides with .X /.
V.123. Suppose that X is a Tychonoff space such that the family of all neighborhoods of the diagonal of X is a uniformity on X which generates .X /. Prove that
X is collectionwise normal.
Solution. Let U be the family of all neighborhoods of the diagonal of the
space X . Given a discrete family F of closed subsets of X , we can fix a set
Ox 2 .x; X / whichS
intersects at most one element of F for every point x 2 X .
It is clear that U D fOx  Ox W x 2 X g is an element of U , so we can find a
symmetric set V 2 U with V V  U .
S
For every F 2 F, apply Problem 103 to see that OF D fV .x/ W x 2 F g is a
neighborhood of the set F . Suppose that F and G are distinct elements of F and
a 2 OF \ OG . There are y 2 F and z 2 G such that a 2 V .y/ \ V .z/; it follows

168

2 Solutions of problems 001500

from f.y; a/; .a; z/g  V that .y; z/ 2 V V  U . As a consequence, there exists
x 2 X with .y; z/ 2 Ox  Ox and hence fy; zg  Ox , i.e., Ox intersects both F and
G which is a contradiction. Thus the family fInt.OF / W F 2 Fg is disjoint, so we
can apply Fact 1 of S.302 to conclude that X is collectionwise normal.
V.124. Give an example of a Tychonoff countably compact non-compact (and
hence non-paracompact) space X such that the family of all neighborhoods of the
diagonal of X is a uniformity on X which generates .X /.
Solution. Our space X will be the ordinal !1 with its order topology; given ordinals
;  !1 let .;
D f W <   g. We will prove that the family
O D fO  X  X W O is a neighborhood of X in X  X g
is a uniformity on X which generates .X /. Observe first that the space X is
countably compact and non-compact being a proper dense subspace of the space
K D !1 C 1 (see TFS-314). The space K being compact, it follows from
Problem 122 that the family V of all neighborhoods of the diagonal K in the space
K  K is a uniformity on K which generates .K/. Therefore U D fV \ .X  X / W
V 2 Vg is a uniformity on X which generates the topology of X . It is evident that
every U 2 U is a neighborhood of the diagonal X in the space X  X , i.e., U  O.
To see that O  U fix any O 2 O. For any limit ordinal 2 X there is './ <
such that the set W D .'./;
 .'./;
is contained in O; let W D f.; /g
for any successor ordinal . By Pressing-Down Lemma (SFFS-067) there is  < !1
and an uncountable
S A  X such that './ D  for every S2 A.
The set H D fW W   g is open in K while G D fW W 2 Ag is easily
seen to coincide with ..; !1
.; !1
/\.X X /. The set V D H [..; !1
.; !1
/
belongs to the family V being a neighborhood of the set K in the space K  K.
Since V \.X X /  H [G  O, we conclude that O 2 U ; this proves that O D U
and hence the family O of all neighborhoods of X in X  X is a uniformity which
generates .X /.
V.125. Prove that, for any compact uniform space .X; U /, the uniformity U
coincides with the family of all neighborhoods of the diagonal of X .
Solution. Let O be the family of all neighborhoods in X  X of the diagonal of
the space X . It follows from Problem 103 that every U 2 U is a neighborhood of
, i.e., U  O.
To see that O  U fix a set O 2 O. Apply Problem 103 again to observe
T that the
family F D fU 2 U W U is closed in X X g is a base of U and therefore F D .
Since all elements of F are compact,T
we can apply Fact 1 of S.326 to conclude that
there is a finite F 0  F with U D F 0  O. The set U  O is an element of
U , so O 2 U ; this proves that O  U and hence O D U , i.e., the uniformity U
coincides with the family of all neighborhoods of in X  X .
V.126. Suppose that .X; U / a compact uniform space. Prove that, for any uniform
space .Y; V/, any continuous map f W X ! Y is uniformly continuous.

2 Solutions of problems 001500

169

Solution. Denote by X and Y the diagonals of the spaces X and Y respectively.


Fix a set V 2 V; by Problem 103 we can find W 2 . Y ; Y  Y / with W  V .
Since f is continuous, the map f  f is continuous as well, so the set U D .f 
f /1 .W / is open in X  X and X  U . The space X being compact, we can
apply Problem 125 to see that U 2 U ; it follows from U  .f  f /1 .V / that
.f  f /1 .V / 2 U , so the map f is uniformly continuous.
V.127. Let .X; U / be a uniform space such that the uniformity U is induced by a
metric . Prove that .X; U / is totally bounded if and only if the metric space .X; /
is totally bounded.
Solution. Assume first that the space .X; U / is totally bounded; given " > 0, the set
U D f.x; y/ 2 X  X W .x; y/ < "g belongs to U , so there is a finite set P  X
such that U.P / D X . Thus, for
S any x 2 X , there is p 2 P with x 2 U.p/ and
hence d.x; p/ < ". Therefore fB .p; "/ W p 2 P g D X which shows that .X; /
is totally bounded, i.e., we established necessity.
Now, if .X; / is totally bounded, then take any U 2 U ; there is " > 0 such that
V D f.x;
S y/ 2 X  X W .x; y/ < "g  U . There exists a finite set P  X for
which fB .p; "/ W p 2 P g D X . Given any point x 2 X there is p 2 P with
.x; p/ < ", so .p; x/ 2 V  U and hence x 2 U.p/. This proves that X D U.P /
and hence .X; U / is totally bounded, i.e., we settled sufficiency.
V.128. Prove that a uniform space .X; U / is totally bounded if and only if every
ultrafilter on X is a Cauchy family with respect to U .
Solution. Suppose first that .X; U / is totally bounded and take an ultrafilter F on
the set X . Fix a set U 2 U and apply Problem 103 to find a symmetric V 2 U such
that V V  U . There exists a finite set A  X with V .A/ D X .
Given a point x 2 A withTV .x/ F we have X nV .x/ 2 F, so if V .x/ F
for every x 2 A then ; D fX nV .x/ W x 2 Ag 2 F which is a contradiction.
Therefore V .a/ 2 F for some a 2 A. The set V being symmetric, for any points
x; y 2 V .a/ we have f.x; a/; .a; y/g  V and hence .x; y/ 2 V V  U ; this
shows that V .a/  V .a/  U . Recalling that V .a/ 2 F we conclude that F is a
Cauchy ultrafilter on X , i.e., we proved necessity.
Now assume that every ultrafilter on X is a Cauchy family with respect to U and
fix a set U 2 U . If U.A/ X for any finite A  X then the family E D fX nU.x/ W
x 2 X g is centered, so there exists an ultrafilter F on the set X such that E  F
(see TFS-117). By our assumption, F is a Cauchy family, so there is F 2 F with
F  F  U.
Fix an arbitrary point a 2 F and observe that, for any x 2 F , it follows from
.a; x/ 2 F  F  U that x 2 U.a/; this proves that F  U.a/. An immediate
consequence is that U.a/ 2 F; since also X nU.a/ 2 F by our choice of F, we
obtained a contradiction which shows that U.A/ D X for some finite A  X and
hence .X; U / is totally bounded. This settles sufficiency.
V.129. Prove that any uniform product of totally bounded uniform spaces is totally
bounded.

170

2 Solutions of problems 001500

Solution. Suppose that a uniform space .Xt ; Ut / is totally bounded for any t 2 T
and let .X; U / be the uniform product of the family f.Xt ; Ut / W t 2 T g. Then X D
Q
t2T Xt ; let pt W X ! Xt be the natural projection for any t 2 T .
Fix an ultrafilter F on the set X and any U 2 U ; by definition of the uniform
product, there isTa finite S  T and a family fWt W t 2 S g such that Wt 2 Ut for
each t 2 S and f.pt  pt /1 .Wt / W t 2 S g  U .
It is straightforward that Ft D fpt .F / W F 2 Fg is an ultrafilter on the set Xt ;
the space .Xt ; Ut / being totally bounded, Ft is a Cauchy family with respect to Ut
for every t 2 S . Consequently,
T there is Ft 2 F such that pt .Ft /  pt .Ft /  Wt for
each t 2 S . The set F D t2S Ft belongs to F; take any x; y 2 F .
We have pt .F /  pt .F /  pt .FT
t /  pt .Ft /  Wt , so .pt .x/; pt .y// 2 Wt for
every t 2 S and therefore .x; y/ 2 f.pt  pt /1 .Wt / W t 2 S g  U . The points
x; y 2 F were taken arbitrarily, so we proved that F  F  U and hence F is a
Cauchy ultrafilter on X . Finally apply Problem 128 to conclude that .X; U / is totally
bounded.
V.130. Prove that a uniform space is compact if and only if it is complete and totally
bounded. Deduce from this fact that a uniform space is totally bounded if and only
if its completion is compact.
Solution. If a uniform space .X; U / is compact then each centered family of closed
subsets of X (which does not even need to be a Cauchy family) has a nonempty
intersection; this, together with Problem 114 implies that X is complete. Given any
U 2 U , the family fInt.U.x// W x 2 X g is S
an open cover of X by Problem 103.
Therefore there is a finite A  X with X D fInt.U.x// W x 2 Ag; an immediate
consequence is that U.A/ D X and hence X is totally bounded, i.e., we proved
necessity.
Now assume that a uniform space .X; U / is complete and totally bounded. Given
an ultrafilter F on the
T set X it is a Cauchy family by Problem 128; by completeness
of .X; U / we have fF W F 2 Fg ; (see Problem 114). Finally, apply TFS118 to conclude that X is compact; this settles sufficiency and shows that a uniform
space is compact if and only if it is complete and totally bounded.
Next suppose that .X; U / is a uniform space such that its completion .X  ; U  /
is compact. Fix an arbitrary ultrafilter F on the set X and U 2 U . Since F can be
considered a filter base on X  , there exists an ultrafilter F  on the set X  such that
F  F  . Choose a set V 2 U  with V \.X X / D U . The uniform space .X  ; U  /
being totally bounded, the ultrafilter F  is a Cauchy family by Problem 128, so there
is F  2 F  with F   F   V . Observe that X 2 F  , so F D F  \ X 2 F  .
If F F then G D X nF 2 F and hence G 2 F  , so F and G are two disjoint
elements of F  . This contradiction shows that F 2 F; besides, F  F  .F  
F  / \ .X  X /  V \ .X  X / D U . Thus every ultrafilter on X is a Cauchy
family and hence .X; U / is totally bounded by Problem 128.
Finally assume that .X; U / is totally bounded and let .X  ; U  / be the completion
of .X; U /. Fix an arbitrary set V 2 U  and apply Problem 103 to find a closed
W 2 U  with W  V . The set G D W \ .X  X / belongs to U , so there is a finite
A  X such that G.A/ D X .

2 Solutions of problems 001500

171

For any a 2 A the set Pa D W \ .fag  X  / is closed in fag  X  ; an evident



homeomorphism from fag  X  onto
in
S X takes Pa onto W .a/, so W .a/ is closed

X for any a 2 A. Thus W .A/ D fW .a/ W a 2 Ag is a closed subset of X  such
that X D G.A/  W .A/. The set X being dense in X  , we have W .A/ D X  ; this
proves that .X  ; U  / is totally bounded. Since .X  ; U  / is also complete, it has to
be compact, so we have finally established that a space .X; U / is totally bounded if
and only if its completion .X  ; U  / is compact.
V.131. Prove that a Tychonoff space X is pseudocompact if and only if every
uniformity U on the set X with U D .X / is totally bounded.
Solution. Suppose that X is pseudocompact and some uniformity U generates the
topology of X . Given U 2 U suppose that U.A/ X for any finite A  X and find
a symmetric set V 2 U such that V V V V  U (see Problem
103). Construct
S
by induction a set Y D fxn W n 2 !g  X such that xnC1 fU.xi / W i  ng for
every n 2 !.
To see that the family fV .xi / W i 2 !g is discrete fix a point x 2 X and suppose
that there are distinct i; j 2 ! such that V .x/ \ V .xi / ; and V .x/ \ V .xj / ;;
we can assume, without loss of generality, that i < j . If y 2 V .x/ \ V .xi / and
z 2 V .x/ \ V .xj / then f.xi ; y/; .y; x/; .x; z/; .z; xj /g  V (we used the fact that
V is symmetric). Thus .xi ; xj / 2 V V V V  U , i.e., xj 2 U.xi / which is
a contradiction with the choice of the set Y . This shows that Int.V .x// is an open
neighborhood of the point x (see Problem 103) which intersects at most one element
of the family fV .xi / W i 2 !g. Therefore fInt.V .xi // W i 2 !g is an infinite discrete
family of nonempty open subsets of X . This contradiction with pseudocompactness
of X proves that there is a finite A  X with U.A/ D X , i.e., .X; U / is totally
bounded and hence we established necessity.
For sufficiency, assume that every uniformity on the set X is totally bounded
whenever it generates the topology of X . The space X being Tychonoff, we can
fix a uniformity U on the set X such that U D .X /. If X is not pseudocompact
then there exists a continuous unbounded function f W X ! R; consider the set
Or D f.x; y/ 2 X  X W jf .x/  f .y/j < rg for each r > 0. It is clear that
Or D Or1 and Or Or  O2r for any r > 0. An easy consequence is that
the family U [ fOr W r > 0g generates a uniformity V on X as a subbase (see
Problem 102); let O D fOr W r > 0g.
Given a set W 2 V fix a point x 2 W ; there is V 2TV with VT
.x/  W . There
0
0
0
exist finite families
U

U
and
O

O
such
that
.
U
/
\
.
O/  V . It is
T
T
clear that U D U 0 2 U and O D O0 2 O, so fix r > 0 such that O D Or .
It follows from continuity of the function f that the set Or .x/ is open in X ; thus
the set Int.U.x// \ Or .x/ is an open neighborhood of x contained in V .x/  W .
The point x 2 W was chosen arbitrarily, so we proved that every point of W has
a neighborhood contained in W ; thus W is open in X , i.e., V  .X /. It follows
from U  V that .X / D U  V , so we have V D .X / and therefore the space
.X; V/ is totally bounded.

172

2 Solutions of problems 001500

Since O1 2 V, there exists a finite set A  X such that O1 .A/ D X ; let q D


maxfjf .x/j W x 2 Ag C 2. Given any y 2 X there is x 2 A with y 2 O1 .x/ and
hence jf .x/  f .y/j < 1; an immediate consequence is that jf .y/j  jf .x/j C 1 
q. It turns out that jf .y/j  q for any y 2 X , i.e., the function f is bounded. This
contradiction shows that X is pseudocompact, so we settled sufficiency.
V.132. For any Tychonoff space X let US
X be the family of all uniformities on the
set X which generate .X /. Note that UX can be considered a subbase of a
uniformity NX (called the universal uniformity) on the set X . Prove that
(i) the topology generated by NX coincides with .X / and hence NX 2 UX ;
(ii) if Y is a Tychonoff space and f W X ! Y is a continuous map then the map
f W .X; NX / ! .Y; V/ is uniformly continuous for any uniformity V 2 UY .
Solution. ItSis straightforward that all conditions of Problem 102 are satisfied for
the family UX , so it can, indeed, be considered a subbase for a uniformity NX
on the set X . Apply Problem 110 to fix a uniformity U on the set X such that
U D .X /. It follows from U  NX that .X / D U  NX .
To prove the opposite inclusion take anySO 2 NX and a point x 2 O; there
is N 2 NX with N.x/  O. The family UX being a subbase of NX we can
choose U1 ; : : : ; Un 2 UX and a set Ui 2 Ui for each i  n in such a way that
U D U1 \ : : : \ Un  N . It follows from Problem 103 that x 2 Wi D Int.Ui .x//
for every i  n, so W D W1 \ : : : \ Wn 2 .x; X /; since also W  N.x/  O,
we proved that every point of O has a neighborhood contained in O, i.e., O is open
in X and hence NX D .X / which shows that we have verified (i).
Now fix a space Y , a continuous map f W X ! Y and a uniformity V on
Y such that V D .Y /. Consider the family V 0 D f.f  f /1 .V / W V 2 Vg.
Given W D .f  f /1 .V / 2 V 0 there is a symmetric H 2 V with H  V . It is
straightforward that the set G D .f  f /1 .H / is symmetric and G 1 D G  W .
Furthermore, there is P 2 V such that P P  V ; an immediate consequence is
that, for the set Q D .f  f /1 .P /, we have Q Q  W .
This shows that the family M D NX [V 0 satisfies the conditions of Problem 102
and hence there is a uniformity W on the set X for which M is a subbase. It is clear
that .X / D NX  M . To show that also M  .X / take any O 2 M and
fix a point x 2 O; there is a set W 2 W with W .x/  O. The family M being a
: ; Nk 2 NX and H1 ; : : : ; Hl 2 V 0 such that, for the
subbase ofTW there are N1 ; : :T
sets N D ik Ni and H D il Hi , we have N \ H  W .
It is evident that N 2 NX and H 2 V 0 ; since NX generates the topology of X ,
there is Q 2 .x; X / with Q  N.x/ (see Problem 103). Take a set G 2 V such
that H D .f  f /1 .G/; since the set G.y/ is a neighborhood of y D f .x/ (see
Problem 103), there is a set P 2 .x; X / for which f .P /  G.y/; this implies that
P  H.x/ and therefore Q \ P is an open neighborhood of the point x contained
in the set N.x/ \ H.x/  W .x/  O. This proves that O is open in X and hence
W  .X /, i.e., W D .X /.

2 Solutions of problems 001500

173

As a consequence, W 2 UX which shows that W  NX and, in particular,


V 0  NX . In other words, .f  f /1 .V / 2 NX for any V 2 V; thus the map
f W .X; NX / ! .Y; V/ is uniformly continuous and hence we settled (ii).
V.133. Let X be a Tychonoff space. Prove that the following are equivalent:
(i) there exists a complete uniformity U on the set X such that U D .X /;
(ii) the universal uniformity on the space X is complete;
(iii) the space X is Dieudonn complete.
Solution. It is evident that (ii)H)(i). Assume that there exists a complete uniformity U on the set X such that U D .X / and apply Problem 113 to find a family
F D f.Mt ; Mt / W t 2 T g of uniformly metrizable spaces such that .X; U / is a
uniform subspace of the uniform productQ
.M; M/ of the family F. By, Problem 116,
the set X is closed in the product M D t2T Mt , so X is Dieudonn complete and
hence we established that (i)H)(iii).
Fact 1. Suppose that Z is a set and V; V 0 are uniformities on Z such that V  V 0
and V D V 0 . If .Z; V/ is complete then the space .Z; V 0 / is also complete.
Proof. Fix a filter E on the set Z which isTa Cauchy family in .Z; V 0 /. It is clear that
E is also a Cauchy family in .Z; V/, so fE W E 2 Eg ; (see Problem 114; the
closure is taken in the topology D V D V 0 ). Applying Problem 114 again we
conclude that .X; V 0 / is also complete so Fact 1 is proved.
Returning to our solution assume that there is a complete uniformity U on the set
X for which U D .X /. If N is the universal uniformity on X then N D .X /
and U  N , so we can apply Fact 1 to see that .X; N / is complete as well. This
settles (i)H)(ii), so (i) (ii).
Finally, if X is Dieudonn
Q complete then we can consider that X is a closed
subspace of a product M D t2T Mt of complete metric spaces (see TFS-459). If
t is the respective complete metric on Mt then the uniformity Mt on the set Mt ,
generated by t , is complete for any t 2 T (see Problems 111 and 118). Thus the
uniform product .M; M/ of the family f.Mt ; Mt / W t 2 T g is a complete uniform
space such that .M / D M (see Problems 105 and 117). The set X being closed
in M , the uniformity V generated on X from .M; M/ is complete by Problem 115.
Since X is a subspace of M , the topology V coincides with .X /. Thus, any
Dieudonn complete space X has a complete uniformity V with V D .X /, i.e.,
we showed that (iii)H)(i); this finishes our solution.
V.134. For any linear topological space L denote by 0L its zero vector and let
G.U / D f.x; y/ 2 L  L W x  y 2 U g for any U 2 .0L ; L/. Prove that
(i) the family BL D fG.U / W U 2 .0L ; L/g forms a base for a uniformity UL on
the set L (called the linear uniformity on L).
(ii) If M is a linear subspace of L then the linear uniformity UM on the set M
coincides with the subspace uniformity induced on M from L.

174

2 Solutions of problems 001500

(iii) If L0 is a linear topological space then a map f W L ! L0 is uniformly


continuous if and only if, for any U 0 2 .0L0 ; L0 / there exists U 2 .0L ; L/
such that f .x/  f .y/ 2 U 0 for any x; y 2 L with x  y 2 U .
(iv) If L0 is a linear topological space then any linear continuous map f W
L ! L0 is uniformly continuous if L and L0 are considered with their linear
uniformities. In particular, any linear isomorphism between L and L0 is a
uniform isomorphism.
Solution. (i) Fix a set U 2 .0L ; L/; it follows from continuity of arithmetical
operations in L that U D fx W x 2 U g is an open neighborhood of 0L .
Therefore V D U \ .U / 2 .0L ; L/ and it is straightforward that G.V / D
.G.V //1  G.U /.
Apply continuity of operations in L once more to see that there exists a set W 2
.0L ; L/ such that W C W D fx C y W x; y 2 W g  U . If f.x; y/; .y; z/g 
G.W / then x  y 2 W and y  z 2 W , so x  z 2 W C W  U , i.e.,
.x; z/ 2 G.U /. This shows that G.W / G.W /  G.U /.
Now, if x; y 2 L and x y then x  y 0L , so there is U 2 .0L ; L/
such that x  y U and hence .x; y/ G.U /. This proves that the set
T
BL coincides with the diagonal of the space L. Since also G.U \ V / D
G.U / \ G.V / for any U; V 2 .0L ; L/, the family BL satisfies all premises of
Problem 101, so it indeed forms a base of a uniformity on L, i.e., (i) is proved.
(ii) Let GM .U / D f.x; y/ 2 M  M W x  y 2 U g for any U 2 .0L ; M /. It
is immediate that G.U / \ .M  M / D GM .U \ M / for any U 2 .0L ; L/;
this shows that fW \ .M  M / W W 2 BL g D BM . Given Q 2 UL there
is W 2 BL with W  Q; thus W 0 D W \ .M  M / 2 BL and hence
W 0  Q \ .M  M /. This proves that Q \ .M  M / belongs to UM for any
Q 2 UL , i.e., the uniformity induced on M from L is contained in the linear
uniformity of L.
Next take any Q 2 UM ; there is U 2 .0L ; M / such that GM .U /  Q.
Fix U 0 2 .L/ such that U 0 \ M D U : then W D G.U 0 / [ Q 2 UL and
W \ .M  M / D Q. Thus the subspace uniformity induced on M from L
coincides with UM , i.e., (ii) is proved.
(iii) Suppose that a map f W L ! L0 is uniformly continuous and fix a set U 0 2
.0L0 ; L0 /. The set W 0 D G 0 .U 0 / D f.z; t / 2 L0  L0 W z  t 2 U 0 g belongs to
UL0 , so there is W 2 UL such that .f  f /.W /  W 0 . The family BL being a
base of UL we can find U 2 .0L ; L/ for which G.U /  W . Now, if x; y 2 L
and x  y 2 U then .x; y/ 2 G.U /  W and therefore .f .x/; f .y// 2 W 0
which shows that f .x/  f .y/ 2 U 0 and hence we settled necessity.
For sufficiency assume that the second condition of (iii) is satisfied. Given any
W 0 2 UL0 there is U 0 2 .0L0 ; L0 / with G 0 .U 0 /  W 0 ; fix a set U 2 .0L ; L/
such that f .x/  f .y/ 2 U 0 whenever x  y 2 U ; it is easy to see that this
implies that .f  f /.G.U //  G 0 .U 0 /  W 0 . Since W D G.U / belongs to
UL , we established that f is uniformly continuous, so (iii) is proved.

2 Solutions of problems 001500

175

(iv) If f W L ! L0 is a continuous linear map then fix a set W 2 UL0 and any
U 0 2 .0L0 ; L0 / such that G 0 .U 0 /  W 0 . The map f being continuous at 0L
there is U 2 .0L ; L/ for which f .U /  U 0 ; let W D G.U /.
If .x; y/ 2 W then x  y 2 U and hence f .x/  f .y/ D f .x  y/ 2 f .U / 
U 0 which shows that .f .x/; f .y// 2 W 0 . Thus .f  f /.W /  W 0 and hence
the map f is uniformly continuous. This settles (iv) and makes our solution
complete.
V.135. Prove that the linear uniformity of RX coincides with the uniform product
of the respective family of real lines. Deduce from this fact that RX is the completion
of Cp .X / for any space X , so Cp .X / is complete as a uniform space if and only if
X is discrete.
Solution. Denote by L the linear uniformity on the space RX and let U be the
uniform product of the jX j-many real lines; the symbol 0 stands for the zero function
on X , i.e., 0 is the zero vector of RX . Recall that the natural projection px W RX ! R
of RX onto its factor determined by x is defined by px .f / D f .x/ for any f 2 RX .
Besides, O" D f.x; y/ 2 R2 W jx  yj < "g for all " > 0 and O D fO" W " > 0g is
the standard base of the linear uniformity on R.
Given an arbitrary set L 2 L there exists a set U 2 .0; RX / such that
G.U / D f.f; g/ 2 .Cp .X //2 W f  g 2 U g  L:
Making the set U smaller if necessary, we can assume that there are x1 ; : : : ; xn 2 X
and " > 0 suchT
that U D x1 ; : : : ; xn ; "
D ff 2 RX W jf .xi /j < " for all i  ng.
Then G.U / D f.pxi  pxi /1 .O" / W i  ng is an element of the standard base of
the uniform product (see Problem 105), so G.U / 2 U and hence L 2 U ; this proves
that L  U .
To prove the opposite inclusion take an arbitrary set W 2 U ; by the definition
of the uniform
product, there exist x1 ; : : : ; xn 2 X and "1 ; : : : ; "n > 0 such that
T
W 0 D f.pxi  pxi /1 .O"i / W i  ng  W . Now, if " D nnf"1 ; : : : ; "n g and
U D x1 ; : : : ; xn ; "
then it is straightforward that G.U /  W 0  W and hence
W 2 L, i.e., we proved that U D L.
By Problem 134, the space Cp .X / with its linear uniformity is a dense uniform
subspace of RX ; the space RX being complete by Problem 117, we can apply
Problem 121 to conclude that RX is canonically isomorphic to the completion of
the uniform space Cp .X /.
Finally, if X is discrete then Cp .X / D RX is complete by Problem 117; if,
on the other hand, the uniform space Cp .X / is complete then its closed in RX by
Problem 116, so Cp .X / D RX and hence X is discrete (see Fact 1 of S.265).
V.136. Prove that Cp .X / is  -totally bounded as a uniform space if and only if X
is pseudocompact. More formally, X is pseudocompact ifS
and only if there exists a
family fCn W n 2 !g  exp.Cp .X // such that Cp .X / D fCn W n 2 !g and each
Cn is totally bounded considered as a uniform subspace of Cp .X /. In particular, if
Cp .X / is uniformly isomorphic to Cp .Y / then the space X is pseudocompact if and
only if so is Y .

176

2 Solutions of problems 001500

Solution. Suppose first that the space X is pseudocompact and consider the set
Cn D ff 2 Cp .X / W f .x/ 2 n; S
n
g for any n 2 !. It follows from
pseudocompactness of X that Cp .X / D fCn W n 2 !g; besides, every Cn is
dense in the compact set Kn D n; n
X .
Let Un (or Vn respectively) be the subspace uniformity induced on Kn (or Cn
respectively) from RX . Then .Cn ; Vn / is a dense uniform subspace of .Kn ; Un /. The
space .Kn ; Un / is compact and hence complete by Problem 130; apply Problem 121
to see that .Kn ; Un / is the completion of .Cn ; Vn /, so .Cn ; Vn / is totally bounded
by Problem 130. It is easy to see that Vn is also the subspace uniformity induced
on Cn from Cp .X / (see Problems 134 and 135). Therefore every Cn is a totally
bounded uniform subspace of Cp .X /, so Cp .X / is  -totally bounded. This proves
sufficiency.
Fact 1. Given uniform spaces .Y; V/ and .Z; W/ suppose that f W Y ! Z is a
uniformly continuous surjective map and .Y; V/ is totally bounded. Then .Z; W/ is
totally bounded as well.
Proof. If W 2 W then there is V 2 V such that .f  f /.V /  W . The space
.Y; V/ being totally bounded there is a finite set A  Y for which V .A/ D Y . The
set B D f .A/  Z is finite; given any z 2 Z fix y 2 Y with f .y/ D z. There
exists a 2 A such that y 2 V .a/ and hence .a; y/ 2 V . Let b D f .a/; then b 2 B
and .f .a/; f .y// D .b; z/ 2 W , so z 2 W .b/. This shows that W .B/ D Z and
hence .Z; W/ is totally bounded, so Fact 1 is proved.
Fact 2. The space R! with its linear uniformity is not  -totally bounded.
Proof. For every n 2 ! the natural projection pn W R! ! R onto the n-th factor
is uniformly continuous (see Problems 106 and 135; each factor is also considered
with its linear uniformity).
S
Assume that R! D n2! Cn and every subspace Cn is totally bounded. If rn 2
Rnpn .Cn / for each n 2S! then define a point x 2 R! by letting x.n/ D rn for all
n 2 !. Then x 2 R! n. n2! Cn / which is a contradiction. Thus there exists m 2 !
such that pm .Cm / D R. The map pm jCm W Cm ! R is also uniformly continuous,
so R is totally bounded by Fact 1. Since R is also complete, it has to be compact
(see Problem 130); this contradiction shows that R! is not  -totally bounded, i.e.,
Fact 2 is proved.
Returning to our solution assume that the uniform space Cp .X / is  -totally
bounded. If X is not pseudocompact then there is a closed discrete faithfully indexed
set D D fdn W n 2 !g  X which is C -embedded in X . The restriction map
 W Cp .X / ! RD is continuous, surjective, linear and hence uniformly continuous
(see Problem 134). This, together with Fact 1, implies that RD is  -totally bounded.
The space RD is linearly (and hence uniformly) isomorphic to R! , so R! is  -totally
bounded. This contradiction with Fact 2 shows that X is pseudocompact and hence
we proved necessity.

2 Solutions of problems 001500

177

Finally note that if Cp .X / and Cp .Y / are uniformly isomorphic then Cp .X / is


 -totally bounded if and only if so is Cp .Y /; therefore the space X is pseudocompact if and only if so is Y .
u

V.137. Observe that X  Y implies X  Y . Given an example of t -equivalent


spaces X and Y which are not u-equivalent.
Solution. If ' W Cp .X / ! Cp .Y / is a uniform isomorphism then it is also a homet
omorphism by Problem 104. Therefore X  Y , i.e., we proved that u-equivalence
implies t -equivalence.
Next observe that R is t -equivalent to 0; 1
by Problem 027; every space
u-equivalent to the compact (and hence pseudocompact) space 0; 1
has to be
pseudocompact by Problem 136, so X D R and Y D 0; 1
are t -equivalent spaces
which are not u-equivalent.
V.138. Suppose that Cp .X / is uniformly isomorphic to Cp .Y /. Prove that X is
compact if and only if so is Y .
t

Solution. Assume that X is compact; we have X  Y by Problem 137, so the


space Y is  -compact by Problem 043. It follows from Problem 136 that Y is
pseudocompact and hence compact. Analogously, compactness of Y also implies
compactness of X , so X is compact if and only if so is Y .
V.139. Suppose that the spaces X and Y are u-equivalent. Prove that there exists
a homeomorphism ' W RX ! RY such that '.Cp .X // D Cp .Y /.
Solution. Let  W Cp .X / ! Cp .Y / be a uniform isomorphism. The uniform
spaces RX and RY being the completions of Cp .X / and Cp .Y / respectively
(see Problem 135) we can apply Problem 120 to conclude that there exists a
uniform isomorphism ' W RX ! RY such that 'jCp .X / D  and hence
'.Cp .X // D .Cp .X // D Cp .Y /. It follows from Problem 104 that the map '
is a homeomorphism.
V.140. Let F D fF1 ; : : : ; Fk g be a family of functionally closed subsets of a
Tychonoff space X . Suppose that fU1 ; : : : ; Uk g is a family of functionally open
subsets of X such that Fi  Ui for each i . Prove that the family F has a functionally
open swelling fW1 ; : : : ; Wk g such that Fi  Wi  W i  Ui for each i  k.
Solution. Observe first that a functionally open set is the same as a cozero set (see
Fact 1 of T.252) and the concept of a functionally closed set coincides with the
concept of a zero-set by Fact 1 of S.499.
Fact 1. Given disjoint functionally closed sets F and G in a space Z there exists
a function f 2 C.Z; 0; 1
/ such that f .F /  f0g and f .G/  f1g. In particular,
there exist functionally open sets O.F / and O.G/ and disjoint functionally closed
sets P .F /; P .G/ such that F  O.F /  P .F / and G  O.G/  P .G/.
1
Proof. Take uF ; uG 2 C.Z; 0; 1
/ such that F D u1
F .0/ and G D uG .0/. Then
uF
uF C uG is strictly positive on Z, so the function f D uF CuG is well defined and

178

2 Solutions of problems 001500

continuous on Z. It is evident that f .x/ D 1 for any x 2 G and f .x/ D 0 whenever


x 2 F . Therefore the sets O.F / D f 1 .0; 13 // and P .F / D f 1 .0; 13
/ together
with the sets O.G/ D f 1 .. 23 ; 1
/ and P .G/ D f 1 . 23 ; 1
/ constitute the promised
functionally open (closed) neighborhoods of the sets F and G respectively, so Fact 1
is proved.
Fact 2. Suppose that F is functionally closed in a space Z and O is a functionally
open subset of Z for which F  O. Then there is a functionally open set G  Z
such that F  G  G  O.
Proof. The sets F and X nO are functionally closed and disjoint; this makes it
possible to apply Fact 1 to find disjoint functionally open sets G and G 0 such
that F  G and X nO  G 0 . It follows from G \ G 0 D ; that G  O, so
F  G  G  O and hence Fact 2 is proved.
S
Returning to our solution consider the set 1 D
fP W there exist distinct
numbers i1 ; : : : ; im 2 f2; : : : ; kg such that P D Fi1 \ : : : \ Fim and P \ F1 D ;g.
The set 1 is functionally closed being a finite union of functionally closed sets (see
Fact 1 of S.499); since also 1 \ F1 D ;, we can find a function f1 2 C.X; 0; 1
/
for which f1 .F1 /  f0g and f .1 /  f1g (see Fact 1). The set K1 D f11 .0; 12
/ is
functionally closed and the family F1 D fK1 ; F2 ; : : : ; Fk g is a swelling of F.
Proceeding by induction assume that 1  n < k and we have constructed
functions f1 ; : : : ; fn 2 C.X; 0; 1
/ with the following properties:
(1) fi .Fi /  f0g for any i  n;
(2) if Ki D fi1 .0; 12
/ for each i  n then the family fK1 ; : : : ; Kn ; FnC1 ; : : : ; Fk g
is a swelling of F.
S
For the family Fn D fK1 ; : : : ; Kn ; FnC1 ; : : : ; Fk g let nC1 D fP W there exist
P1 ; : : : ; Pm 2 Fn nfFnC1 g such that P D P1 \: : :\Pm and P \FnC1 D ;g. The set
nC1 being functionally closed and disjoint from FnC1 we can apply Fact 1 again
to find fnC1 2 C.X; 0; 1
/ such that fnC1 .FnC1 /  f0g and fnC1 .nC1 /  f1g.
1
If KnC1 D fnC1
.0; 12
/ then FnC1 D fK1 ; : : : ; Kn ; KnC1 ; FnC2 ; : : : ; Fk g is
easily seen to be a swelling of Fn ; since Fn is a swelling of F, the family FnC1
is a swelling of F as well, so our inductive procedure can be continued to construct
functions f1 ; : : : ; fk 2 C.X; 0; 1
/ such that the properties (1) and (2) are fulfilled
for n D k. In particular, K D fK1 ; : : : ; Kk g is a swelling of F.
If Hi D fi1 .0; 12 // then the set Hi is functionally open for every i  k and
the family H D fH1 ; : : : ; Hn g is also a swelling of F. Apply Fact 2 to find a
functionally open set Wi for which Fi  Wi  W i  Hi \ Ui for every i  k;
then fW1 ; : : : ; Wk g is the promised swelling of F, so our solution is complete.
V.141. Let F D fF1 ; : : : ; Fk g be a family of closed subsets of a normal space X .
Suppose that fU1 ; : : : ; Uk g is a family of open subsets of X such that Fi  Ui for
each i  k. Prove that the family F has an open swelling fW1 ; : : : ; Wk g such that
Fi  Wi  W i  Ui for each i  k.

2 Solutions of problems 001500

179

S
Solution. Consider the set 1 D fP W there exist i1 ; : : : ; im 2 f2; : : : ; kg such
that P D Fi1 \ : : : \ Fim and P \ F1 D ;g. The set 1 is closed and disjoint from
F1 ; by normality of the space X we can find a function f1 2 C.X; 0; 1
/ for which
f1 .F1 /  f0g and f .1 /  f1g. The set K1 D f11 .0; 12
/ is closed and the family
F1 D fK1 ; F2 ; : : : ; Fk g is a swelling of F.
Proceeding by induction assume that 1  n < k and we have constructed
functions f1 ; : : : ; fn 2 C.X; 0; 1
/ with the following properties:
(1) fi .Fi /  f0g for any i  n;
(2) if Ki D fi1 .0; 12
/ for each i  n then the family fK1 ; : : : ; Kn ; FnC1 ; : : : ; Fk g
is a swelling of F.
S
For the family Fn D fK1 ; : : : ; Kn ; FnC1 ; : : : ; Fk g let nC1 D fP W there exist
P1 ; : : : ; Pm 2 Fn nfFnC1 g such that P D P1 \ : : : \ Pm and P \ FnC1 D ;g. The
set nC1 is closed and disjoint from FnC1 ; the space X being normal we can find a
function fnC1 2 C.X; 0; 1
/ such that fnC1 .FnC1 /  f0g and fnC1 .nC1 /  f1g.
1
If KnC1 D fnC1
.0; 12
/ then FnC1 D fK1 ; : : : ; Kn ; KnC1 ; FnC2 ; : : : ; Fk g is
easily seen to be a swelling of Fn ; since Fn is a swelling of F, the family FnC1
is a swelling of F as well, so our inductive procedure can be continued to construct
functions f1 ; : : : ; fk 2 C.X; 0; 1
/ such that the properties (1) and (2) are fulfilled
for n D k. In particular, K D fK1 ; : : : ; Kk g is a swelling of F.
If Hi D fi1 .0; 12 // then the set Hi is open for every i  k and the family
H D fH1 ; : : : ; Hn g is also a swelling of F. By normality of X we can find an open
set Wi for which Fi  Wi  W i  Hi \ Ui for all i  k; then fW1 ; : : : ; Wk g is the
promised swelling of F.
V.142. Let U D fU1 ; : : : ; Uk g be a functionally open cover of a Tychonoff space X .
Prove that U has shrinkings F D fF1 ; : : : ; Fk g and W D fW1 ; : : : ; Wk g such that
F is functionally closed, W is functionally open and Fi  Wi  W i  Ui for every
i  k.
Solution. Consider the set Pi D X nUi for each i  k. Then all elements of the
family P D fP1 ; : : : ; Pk g are functionally closed, so we can apply Problem 140 to
find a functionally open swelling O D fO1 ; : : :T
; Ok g of the family
T P. If Fi D X nOi
then Fi  Ui for all i  k; it follows from P D ; that O D ; and hence
the family F D fF1 ; : : : ; Fk g covers X , i.e., F is a functionally closed shrinking
of U . By Fact 2 of V.140 there exists a functionally open set Wi such that Fi 
Wi  W i  Ui for each i  k; it is evident that the functionally open family
W D fW1 ; : : : ; Wk g is also a shrinking of U .
V.143. Let U D fU1 ; : : : ; Uk g be an open cover of a normal space X . Prove that
U has shrinkings F D fF1 ; : : : ; Fk g and W D fW1 ; : : : ; Wk g such that F is closed,
W is open and Fi  Wi  W i  Ui for every i  k.
Solution. Consider the set Pi D X nUi for each i  k. Then all elements of the
family P D fP1 ; : : : ; Pk g are closed in X , so we can apply Problem 141 to find an
open swelling O D fO1 ; : : : ; OkT
g of the family T
P. If Fi D X nOi then Fi  Ui
for all i  k; it follows from P D ; that O D ; and hence the family

180

2 Solutions of problems 001500

F D fF1 ; : : : ; Fk g covers X , i.e., F is a closed shrinking of U . By normality of X


there exists an open set Wi such that Fi  Wi  W i  Ui for each i  k; it is
evident that the family W D fW1 ; : : : ; Wk g is also a shrinking of U .
V.144. Prove that, for any Tychonoff space X , the following conditions are
equivalent:
(i) dim X  n;
(ii) every finite functionally open cover of X has a finite functionally closed
refinement of order  n C 1;
(iii) every finite functionally open cover of X has a functionally closed shrinking of
order  n C 1;
(iv) every finite functionally open cover of X has a functionally open shrinking of
order  n C 1.
Solution. Suppose that dim X  n and take a finite functionally open cover U
of the space X . There exists a finite functionally open refinement V of the family
U of order at most n C 1; take a faithful enumeration fV1 ; : : : ; Vk g of the family
V. It follows from Problem 142, that V has a functionally closed shrinking F D
fF1 ; : : : ; Fk g. It is straightforward that the order of F is at most n C 1; the family F
being a refinement of V, it is also a refinement of U , so we proved that (i)H)(ii).
Now, if (ii) holds then take a functionally open cover U D fU1 ; : : : ; Uk g of the
space X . There exists a functionally closed refinement F of the family U of order
at mostSn C 1. For any F 2 F fix a number i.F / such that F  Ui.F / and let
Gj D fF 2 F W i.F / D j g for every j  k. The family G D fG1 ; : : : ; Gk g
is a functionally closed shrinking of U . If there are distinct j1 ; : : : ; jnC2 such that
P D Gj1 \ : : : \ GjnC2 ; then pick a point x 2 P . There are F1 ; : : : ; FnC2 2 F
such that x 2 F1 \ : : : \ FnC2 and i.Fm / D jm for every m  n C 2.
Therefore F1 ; : : : ; FnC2 are distinct elements of F with nonempty intersection. This
contradiction with ord.F/  nC1 shows that ord.G/  nC1, so G is a functionally
closed shrinking of U of order at most n C 1, i.e., we established that (ii)H)(iii).
Next assume that the condition (iii) is satisfied and take a functionally open cover
U D fU1 ; : : : ; Uk g of the space X . There exists a functionally closed shrinking
F D fF1 ; : : : ; Fk g of the family U such that ord.F/  n C 1. By Problem 140
there exists a functionally open swelling V D fV1 ; : : : ; Vn g of the family F such
that Vi  V i  Ui for each i  k. Thus V is a functionally open shrinking of U
of order at most n C 1, i.e., we settled (iii)H)(iv). The implication (iv)H)(i) is
evident, so our solution is complete.
V.145. Prove that, for any normal X , the following conditions are equivalent:
(i)
(ii)
(iii)
(iv)
(v)

dim X  n;
every finite open cover of X
every finite open cover of X
every finite open cover of X
every finite open cover of X

has a finite open refinement of order  n C 1;


has a finite closed refinement of order  n C 1;
has a closed shrinking of order  n C 1;
has an open shrinking of order  n C 1.

2 Solutions of problems 001500

181

Solution. Suppose that dim X  n and U D fU1 ; : : : ; Uk g is an open cover of X .


It follows from Problem 143 that U has a closed shrinking F D fF1 ; : : : ; Fk g. By
Fact 5 of V.100 we can choose a functionally open set Vi such that Fi  Vi  Ui for
every i  k. It is evident that V D fV1 ; : : : ; Vk g is a functionally open refinement of
U . Since dim X  n, the family V has a finite functionally open refinement W with
ord.W/  n C 1; it is clear that W is also an open refinement of U , so (i)H)(ii) is
proved.
Now assume that (ii) holds and take a finite open cover U of the space X . There
exists a finite open refinement V of the family U of order at most n C 1; take a
faithful enumeration fV1 ; : : : ; Vk g of the family V. It follows from Problem 143,
that V has a closed shrinking F D fF1 ; : : : ; Fk g. It is straightforward that the order
of F is at most n C 1; the family F being a refinement of V, it is also a refinement
of U , so we proved that (ii)H)(iii).
Next suppose that (iii) holds and take an open cover U D fU1 ; : : : ; Uk g of the
space X . There exists a closed refinement F of the family U of order at most n C 1.
For any set F 2 F fix a number i.F / such that F  Ui.F / and consider the set Gj D
S
fF 2 F W i.F / D j g for every j  k. The family G D fG1 ; : : : ; Gk g is a closed
shrinking of U . If there are distinct j1 ; : : : ; jnC2 such that P D Gj1 \: : :\GjnC2 ;
then pick a point x 2 P . There are F1 ; : : : ; FnC2 2 F such that x 2 F1 \ : : : \ FnC2
and i.Fm / D jm for every m  n C 2. Therefore F1 ; : : : ; FnC2 are distinct elements
of F with nonempty intersection. This contradiction with ord.F/  n C 1 shows
that ord.G/  n C 1, so G is a closed shrinking of U of order at most n C 1, i.e., we
established that (iii)H)(iv).
If the property (iv) holds then take an open cover U D fU1 ; : : : ; Uk g of the
space X . There exists a closed shrinking F D fF1 ; : : : ; Fk g of the family U such that
ord.F/  n C 1. By Problem 141 there exists an open swelling V D fV1 ; : : : ; Vn g of
the family F such that Vi  V i  Ui for each i  k. Thus V is an open shrinking
of U of order at most n C 1, i.e., we settled (iv)H)(v).
Finally, assume that the condition (v) is satisfied and fix a functionally open cover
U D fU1 ; : : : ; Uk g of the space X . Take an open shrinking V D fV1 ; : : : ; Vk g of the
family U such that ord.V/  n C 1; it follows from Problem 143 that we can find
a closed shrinking F D fF1 ; : : : ; Fk g of the family V. Apply Fact 5 of V.100 again
to find a functionally open set Wi such that Fi  Wi  Vi for all i  k. It is
easy to see that W D fW1 ; : : : ; Wk g is a functionally open shrinking of U with
ord.W/  n C 1, so dim X  n by Problem 144; this settles (v)H)(i) and hence
our solution is complete.
V.146. Suppose that X is a Tychonoff space and Y is a C  -embedded subset of X .
Prove that dim Y  dim X . In particular, if X is normal then dim F  dim X for
any closed F  X .
Solution. Our statement is clear if dim X D 1, so assume dim X  n 2 !.
Take a functionally open cover U D fU1 ; : : : ; Um g of the space Y ; by Problem 142
the family U has a functionally closed shrinking F D fF1 ; : : : ; Fm g. Apply Fact 1
ofV.140 to find a function fi 2 C.Y; 0; 1
/ such that fi .Fi /  f1g and fi .Y nUi / 
f0g for every i  m. The set Y being C  -embedded in X there exist g1 ; : : : ; gm 2

182

2 Solutions of problems 001500

C.X / for which gi jY D fi for every i  m. Let u.x/ D 1 for all x 2 X ; if


hi D max.u  1; nn.gi ; u// then hi 2 C.X; 0; 1
/ andThi jY D fi for each i  m.
1
Every set Oi D h1
fh1
i ..0; 1
/ is open in X ; if O D
i .0; 2 // W i  mg then
O D fO; O1 ; : : : ; Om g is a functionally open cover of X ; observe that Oi \ Y  Ui
for all i  m. By Problem 144 we can find a functionally closed shrinking P D
fP; P1 ; : : : ; Pm g of the cover O such that ord.P/  n C 1. However, O \ Y D ;
which shows that P \ Y D ; and therefore fP1 \ Y; : : : ; Pm \ Y g is a functionally
closed shrinking of U of order at most n C 1. Applying Problem 144 again we
conclude that dim.Y /  n, so dim Y  dim X . Finally, if X is normal and F is
closed in X then F is even C -embedded in X , so dim F  dim X .
V.147. Prove that dim X D dim X for any Tychonoff space X . Deduce from this
fact that dim X D dim Y for any Y with X  Y  X .
Solution. Since X is C  -embedded in X , the inequality dim X  dim X is a
consequence of Problem 146.
Fact 1. Given a Tychonoff space Z, suppose that F1 ; : : : ; Fm are functionally
closed subsets of Z such that F1 \: : :\Fm D ;. Then clZ .F1 /\: : :\clZ .Fm / D ;.
Proof. Suppose that there exists a point z 2 clZ .F1 / \ : : : \ clZ .Fm / and fix a
function fi 2 Cp .Z; 0; 1
/ such that Fi D fi1 .0/ for every i  m. It follows from
F1 \ : : : \ Fm D ; that the function f D f1 C : : : C fm is strictly positive at any
point of Z, so gi D ffi 2 Cp .Z; 0; 1
/ and gi .Fi /  f0g for each i  m. Besides,
Pm
iD1 gi .x/ D 1 for any x 2 Z.
Let u.x/ D 1 for any x 2 Z. There exists a function hi 2 Cp .Z; 0; 1
/ such
that hi jZ D gi ; it follows from gi .Fi /  f0g that
P hi .clZ .Fi //  f0g and hence
hi .z/ D P
0 for every i  m. The function h D m
iD1 hi is continuous on Z and
m
hjZ D
g
D
ujZ;
the
set
Z
being
dense
in Z we have h D u and, in
iD1 i
particular, h.z/ D 1. This contradiction with hi .z/ D 0 for every i  m shows that
clZ .F1 / \ : : : \ clZ .Fm / D ; and hence Fact 1 is proved.
To prove that dim X  dim X suppose that dim X  n; if n D 1 then there is
nothing to prove. If n 2 !, take an open cover U D fU1 ; : : : ; Um g of the space X .
By Problem 143, there exists a functionally open shrinking W D fW1 ; : : : ; Wm g of
the cover U such that W i  Ui for every i  m (the bar denotes the closure in
X ). The cover fX \ Wi W i  mg of the space X has a functionally open (in X )
shrinking fV1 ; : : : ; Vm g of order at most n C 1 (see Problem 144).
Consider the set Oi D X nX nVi ; it is straightforward that Oi is open in X and
Oi \ X D Vi for each i  m. Therefore O i D V i  W i  Ui for every i  m.
The
of functionally closed subsets of X and
T family F D fX nVi W i  mg consists T
F D ;. Apply Fact 1 to conclude that fX nVi W i  mg D ; and hence the
family O D fO1 ; : : : ; Om g is a cover of X .
Next, assume that i1 ; : : : ; inC2 are distinct elements of f1; : : : ; mg and the set
H D Oi1 \ : : : \ OinC2 is nonempty. The space X being dense in X , we have
H \ X D Vi1 \ : : : VinC2 ;, i.e., the order of the family fV1 ; : : : ; Vm g is at least

2 Solutions of problems 001500

183

n C 2 which is a contradiction. Thus O is an open shrinking of U of order  n C 1,


so dim X  n by Problem 145. Therefore dim X  dim X and hence we proved
that dim X D dim X for any space X .
Finally, observe that if X  Y  X then Y D X by Fact 1 of S.393, so
dim Y D dim Y D dim X D dim X and hence our solution is complete.
V.148. Prove that a Tychonoff space X is strongly zero-dimensional if and only if
X is normal and dim X D 0. Give an example of a Tychonoff space X such that
dim X D 0 while X is not strongly zero-dimensional.
Solution. It is evident that, dim X D 0 for any strongly zero-dimensional space
X ; besides, X has to be normal by SFFS-308. This proves necessity. Now if X is
normal and dim X D 0 then take any open cover U D fU1 ; : : : ; Um g of the space
X . By Problem 145, there exists an open shrinking V of order 1 of the cover U ; this,
evidently, implies that the family V is an open disjoint refinement of U , i.e., X is
strongly zero-dimensional.
Fact 1. Given an uncountable cardinal  let D fx 2 D W jx 1 .1/j  !g. Then
P D D n is a pseudocompact non-countably compact (and hence non-normal)
dense subspace of D .
Proof. We will need the zero element u 2 D defined by u./ D 0 for all < .
For any A   let A 2 D be the characteristic function of A, i.e., A ./ D 1 for
every 2 A and A ./ D 0 whenever A. Let 1 D fx 2 D W jx 1 .0/j  !g.
It is easy to see that 1 covers all countable faces of D , so it is pseudocompact by
Fact 2 of S.433. Furthermore, 1  P is a dense subspace of D , so P is dense in
D and pseudocompact by Fact 18 of S.351.
To see that P is not countably compact choose a disjoint family fAn W n 2 !g
of subsets of  such that jAn j D  and let xn D An for any n 2 !. The set
fxn W n 2 !g  P is closed and discrete in P because the sequence fxn W n 2 !g
converges to u P . Therefore P is not countably compact and hence not normal
by TFS-137 which shows that Fact 1 is proved.
Returning to our solution apply Fact 1 to take a pseudocompact non-normal
dense subspace X  D!1 . It follows from Fact 2 of S.309 that X D D!1 . Apply
SFFS-303 and SFFS-306 to see that the space D!1 is strongly zero-dimensional and
hence dim.D!1 / D 0. As a consequence, dim X D dim X D dim.D!1 / D 0 (see
Problem 147); however, X is not strongly zero-dimensional because it is not normal.
V.149. Prove that dim X D 0 implies that X is zero-dimensional. Give an example
of a zero-dimensional space Y such that dim Y > 0.
Solution. If dim X D 0 then dim X D dim X D 0 (see Problem 147); the space
X being compact and hence normal, we can apply Problem 148 to see that X is
strongly zero-dimensional and hence zero-dimensional by SFFS-309. Therefore X
is also zero-dimensional by SFFS-301. To give the required example observe that it
follows from SFFS-309 that there exists a normal zero-dimensional space Y which
is not strongly zero-dimensional; it follows from Problem 148 that dim Y > 0.

184

2 Solutions of problems 001500

V.150 (The countable sum theorem for normal spaces). Given n 2 !, suppose
that a normal space X has a countable closed cover F such that dim F  n for
every F 2 F. Prove that dim X  n.
Solution. Take an enumeration fFi W i 2 Ng of the family F and fix an open cover
U D fU1 ; : : : ; Um g of the space X . Let Uk0 D Uk for every k  m; then U0 D
fU10 ; : : : ; Um0 g is an open cover of X . Suppose that l 2 ! and we have constructed
open covers U0 ; : : : ; Ul of the space X with the following properties:
(1) Ui D fU1i ; : : : ; Umi g for every i  l;
(2) UkiC1  Uki for any i < l and k  m;
(3) if i 2 f1; : : : ; lg then the order of the family fU1i \ Fi ; : : : ; Umi \ Fi g does not
exceed n C 1.
Apply Problem 143 to find an open shrinking fV1 ; : : : ; Vm g of the family Ul such
that V k  Ukl for every k  m. The family V D fV1 \ FlC1 ; : : : ; Vm \ FlC1 g is an
open cover of the space FlC1 , so there exists an open shrinking W D fW1 ; : : : ; Wm g
(in the space FlC1 ) of the cover V such that ord.W/  n C 1 (see Problem 145).
It is easy to check that UklC1 D .Vk nFlC1 / [ Wk is an open subset of X for
each k  m and UlC1 D fU1lC1 ; : : : ; UmlC1 g is a cover of X such that the properties
(1)(3) hold for all i  l C 1. Therefore our inductive procedure can be continued
to construct a sequence fUi W i 2 !g of open covers of X such that the conditions
(1)(3) are satisfied for all i 2 !.
T
T
It follows from (2) that the set Hk D i2! Uki D i2! Uki is closed in X and
Hk  Uk for every k  m. An easy consequence of (3) is that the order of the
family H D fH1 ; : : : ; Hm g does not exceed n C 1.
Given any point x 2 X and i 2 ! there is ki  m such that x 2 Ukii , so we can
choose k  m and an infinite A  ! such that ki D k for all i 2 A. This, together
with the property (2), implies that x 2 Uki for all i 2 !, i.e., x 2 Hk . Thus H is a
closed shrinking of the cover U of order at most n C 1; applying Problem 145 we
conclude that dim X  n.
V.151 (General countable sum theorem). Given n 2 !, suppose that we have a
closed countable cover F of a Tychonoff space X such that
(i) every F 2 F is C  -embedded in X ;
(ii) dim F  n for each F 2 F.
Prove that dim X  n; give
S an example of a Tychonoff non-normal space Y such
that dim Y > 0 and Y D fYi W i 2 !g, where Yi is closed in Y and dim Yi D 0
for every i 2 !.
Solution. For any F 2 F let H.F / D clX .F /; since H.F / is homeomorphic to
F (see Fact 2 ofS
S.451), we have dim.H.F // D dim F  n (see Problem 147).
The space Z D fH.F / W F 2 Fg is  -compact and hence normal while the
family H D fH.F / W F 2 Fg is a countable closed cover of Z and dim H  n for
every H 2 H. Therefore we can apply Problem 150 to see that dim Z  n; since
X  Z  X , we conclude that dim X D dim Z  n (see Problem 147).

2 Solutions of problems 001500

185

Finally,Srecall that it was proved in SFFS-312 that there exists a space Y such
that Y D fYi W i 2 !g and every Yi is closed in Y and strongly zero-dimensional
while Y is not zero-dimensional and hence dim Y > 0 (see Problem 149). It follows
from Problem 148 that dim Yi D 0 for each i 2 !, so Y is our promised example.
V.152. Give an example of a compact (and hence normal) space X such that
dim X D 0 while dim Y > 0 for some Y  X .
Solution. There exists a zero-dimensional space Y such that dim Y > 0 (see
Problem 149). By SFFS-303, we can assume that Y  D for some cardinal .
If X D D then X is compact and dim X D 0 (see SFFS-303, Problem 148 and
SFFS-306), so X is as promised.
V.153. Give an example of a Tychonoff space X such that dim X D 0 and there
exists a closed set Y  X with dim Y > 0.
Solution. Fix a zero-dimensional space Y such that dim Y > 0 (see Problem 149).
By SFFS-303 and SFFS-306 we can assume that Y  D for some uncountable
cardinal . Let D fx 2 D W jx 1 .1/j  !g and take a set A   such that jAj D
jnAj D . Define a point u 2 DnA by letting u./ D 1 for every 2 nA. It is
straightforward that the set H D fug  DA is homeomorphic to D and H  D n.
Therefore we can assume that Y  H .
The set is dense in D and covers all countable faces of D , so D D
(see Fact 2 of S.433 and Fact 2 of S.309). If X D [ Y then  X  ,
so X D D D by Fact 1 of S.393. Therefore dim X D dim.D / D 0 (see
Problem 147); the set H being closed in D , the equality Y D H \ X shows that
Y is closed in X . Thus X is a space with dim X D 0 such that dim Y > 0 for some
closed subspace Y  X .
V.154. Let X be a normal space with dim X  n. Given a subspace Y  X,
suppose that, for every open U  Y , there exists an F -set P such that
Y  P  U . Prove that dim Y  n.
Solution. Fix an open cover U D fU1 ; : : : ; Uk g of the space Y and choose, for
every i 2 f1; : : : ; kg a set Vi 2 .X / such that Vi \ Y D Ui . Then Y is contained
in the set V D V1 [ : : : [ Vk , so we can find an F -set P in the space X such that
Y P V.
S
The space P is normal by Fact 1 of S.289. Besides, P D i2! Pi where every
Pi is closed in X and hence dim Pi  n (see Problem 146); therefore dim P  n
by Problem 150. Apply Problem 143 to find a closed shrinking fF1 ; : : : ; Fk g of the
open cover fV1 \ P; : : : ; Vk \ P g of the space P . There exist functionally open sets
W1 ; : : : ; Wk in the space P such that Fi  Wi  Vi \ P for every i  k (see Fact 6
of V.100).
The functionally open cover fW1 ; : : : ; Wk g of the space P has a functionally
open shrinking O D fO1 ; : : : ; Ok g such that ord.O/  n C 1 (see Problem 144). It
is evident that fO1 \ Y; : : : ; Ok \ Y g is a functionally open refinement of U of order
at most n C 1, so dim Y  n.

186

2 Solutions of problems 001500

V.155. Prove that, for any perfectly normal space X , we have dim Y  dim X for
any Y  X . In particular, dim Y  dim X for any subspace Y of a metrizable
space X .
Solution. If dim X D 1 then there is nothing to prove, so assume that n 2 ! and
dim X D n. If Y  X then any U 2 .Y; X / is an F -subset of X because X is
perfect. Therefore Problem 154 is applicable to conclude that dim Y  n D dim X .
V.156. Given n 2 ! and a Tychonoff space X , prove that dim X  n if and only
if, for any family f.A0 ; B0 /; : : : ; .An ; Bn /g of n C 1 pairs of disjoint functionally
closed sets, it is possible to choose, for each i  n, a functionally closed partition
Ci between Ai and Bi in such a way that L0 \ : : : \ Ln D ;.
Solution. Say that a space Z has .n C 1/-partition property if, for any family
f.P0 ; Q0 /; : : : ; .Pn ; Qn /g of pairs of disjoint functionally
T closed subsets of Z there
exist functionally closed sets R0 ; : : : ; Rn such that in Ri D ; while Ri is a
partition between Pi and Qi for every i  n. We must prove that dim X  n if and
only if our space X has .n C 1/-partition property.
Fact 1. Given a space Z and a natural number m we have dim Z  m if and only if
any functionally open cover U D fU0 ; : : : ; UmC1
Tg of the space Z has a functionally
open shrinking V D fV0 ; : : : ; VmC1 g such that imC1 Vi D ;.
Proof. If dim Z  m and U D fU0 ; : : : ; UmC1 g is a functionally open cover of Z
then it has a functionally open shrinking
T V D fV0 ; : : : ; VmC1 g with ord.V/  m C 1
(see Problem 144). It is evident that imC1 Vi D ;, so we proved necessity.
To establish sufficiency assume that every functionally open cover of Z of
cardinality m C 2 has a functionally open shrinking with empty intersection and
take an arbitrary functionally open cover O D fO0 ; : : : ; Ok g of the space Z.
Given a functionallyTopen cover W D fW0 ; : : : ; Wk g of Z a set B  f0; : : : ; kg
is W-irregular if i2B Wi ; but there exists
T a functionally open shrinking
W 0 D fW00 ; : : : ; Wk0 g of the family W such that i2B Wi0 D ;. Let r0 be the number
of O-irregular subsets. Proceeding inductively, assume that p  0 and we have a
sequence O0 ; : : : ; Op of functionally open covers of the space Z with the following
properties:
(1) O0 D O and OiC1 is a shrinking of Oi for all i < p;
(2) if ri is the number of Oi -irregular sets then riC1 < ri for all i < p.
If rp > 0 then take an Op -irregular set B andTa functionally open shrinking
OpC1 D fO00 ; : : : ; Ok0 g of the cover Op such that i2B Oi0 D ;. It is evident that
B is not OpC1 -irregular set and any OpC1 -irregular set is also Op -irregular. This
shows that the number rpC1 of OpC1 -irregular sets is strictly less than rp , so our
inductive procedure can be continued as long as rp > 0. Since there are only finitely
many O0 -irregular sets, at some step we will obtain a functionally open shrinking
V D fV0 ; : : : ; Vk g of the family O for which there are no V-irregular sets. In other
words, the family V is a swelling of any of its functionally open shrinkings.

2 Solutions of problems 001500

187

If ord.V/  mC1 then we already have the needed refinement of the family O. If
not, assume
without loss of generality that V0 \ : : : \ VmC1 ; and consider the set
S
G D fVi W mC1  i  kg. By our assumption about Z there exists a functionally
open shrinking H D fH0 ; : : : ; HmC1 g of the cover fV0 ; : : : ; Vm ; Gg such that
T
H D ;. Then the family H0 D fH0 ; : : : ; Hm ; HmC1 \ VmC1 ; : : : ; HmC1 \ Vk g
0
is a functionally open shrinking of V; since V is a swelling
T of H , we must have
H0 \ : : : \ Hm \ .HmC1 \ VmC1 / ; which contradicts H D ;.
Thus any finite functionally open cover of Z has a functionally open shrinking
of order at most m C 1, so dim Z  m; this settles sufficiency and shows that Fact 1
is proved.
Fact 2. Given a space Z suppose that C  Z is a functionally closed set such that
ZnC D U [ V where U; V 2 .Z/ and U \ V D ;. Then both sets U and V are
functionally open.
Proof. Fix a function f 2 C.Z/ such that C D f 1 .0/ and define a function g W
Z ! R as follows: g.x/ D f .x/ for any x 2 U and g.x/ D 0 whenever x 2 ZnU .
If f0 D gj.U [ C / and f1 D Gj.V [ C / then f0 and f1 are continuous because
f0 D f j.U [C / and f1 is identically zero on V [C . Since dom.f0 /\dom.f1 / D C
and f0 jC D f1 jC , we can apply Fact 2 of T.354 to see that g is continuous. Since
U D Zng 1 .0/, the set U is functionally open in Z. An analogous reasoning shows
that the set V is also functionally open, so Fact 2 is proved.
Fact 3. For any space Z if A and B are functionally closed disjoint subsets of Z
then there exists a functionally closed partition C between the sets A and B.
Proof. By Fact 1 of V.140 there exist disjoint functionally open sets U and V such
that A  U and B  V ; it is evident that the set C D X n.U [ V / is as promised,
so Fact 3 is proved.
Returning to our solution assume that X has .n C 1/-partition property and take
a functionally open cover U D fU0 ; : : : ; UnC1 g of the space X . Fix a functionally
closed shrinking F D fF0 ; : : : ; FnC1 g of the cover U (see Problem 142). There
exists a family fC0 ; : : : ; Cn g of functionally closed sets T
such that Ci is a partition
between the sets Fi and X nUi for every i  n and in Ci D ;. For every
i  n take disjoint sets Vi 2 .Fi ; X / and Wi 2 .X nUi ; X / such that
X n.Vi [ WiS
/ D Ci . The sets Wi and Vi are functionally open for every i  n (see
Fact 2) and Sin .Wi [ Vi / D X .
If W D in Wi then V D fV0 ; : : : ; Vn ; UnC1 S
\ W g is a functionally open
shrinking of U . Indeed, if x V0 [ : : : [ Vn then x in Fi , so x 2 FnC1 because
F is T
a cover of X ; besides, x 2 W and hence x 2 W \ FnC1  W \ UnC1 . Now, if
x 2 V then x 2 W and hence x 2 Wi for some iT n; since also x 2 Vi , we have
Vi \ Wi ; which is a contradiction. Therefore V D ; and hence dim X  n
by Fact 1; this settles sufficiency.
Now assume that dim X  n and take a family
f.A0 ; B0 /; : : : ; .An ; Bn /g of pairs
S
of disjoint functionally closed sets. If B D in Bi then B \ A0 \ : : : \ An D ;,
so the sets O0 D X nA0 ; : : : ; On D X nAn ; OnC1 D X nB constitute a functionally

188

2 Solutions of problems 001500

0
open cover of X . Choose a functionally
shrinking O0 D fO00 ; : : : ; OnC1
g of
T open
0
the cover O D fO0 ; : : : ; OnC1 g with O D ; (see Fact 1) and take a functionally
closed shrinking fP0 ; : : : ; PnC1 g of the family O (see Problem 142). Letting Ui D
X nPi for all i  n C 1 we obtain a functionally open
T cover U D fU0 ; : : : ; UnC1 g of
the space X such that Ai  Ui for all i  n and U D ;; let Vi D Ui nBi for each
i  n.
It is easy to check that V D fV0 ; : : : ; Vn ; UnC1 g is a functionally open cover of X ;
take a functionally closed shrinking F D fF0 ; : : : ; FnC1 g of the cover V. For every
i  n consider the functionally closed sets A0i D Ai [Fi and Bi0 D Bi [.FnC1 nUi /;
it is easy
that Ai  A0i ; Bi  Bi0 andTA0i \ Bi0 D ;. Given
a point x 2 X , if
S to check
T
0
x in Ai then x 2 FnC1 ; it follows from U D ; that
.
U
S in i / \ FnC1 D ;,
so x 2 FnC1 nUi  Bi0 for some i  n. This proves that in .A0i [ Bi0 / D X . Fix
a functionally closed partition Ci between the sets A0i and Bi0 (see Fact T
3); then Ci
is also
a
partition
between
the
sets
A
and
B
for
every
i

n.
Besides,
i
i
in Ci 
S
X n in .A0i [ Bi0 / D ;. Therefore X has .n C 1/-partition property, i.e., we have
established necessity and hence our solution is complete.

V.157. Given a natural n  0 and a normal space X , prove that dim X  n if and
only if, for any family f.A0 ; B0 /; : : : ; .An ; Bn /g of n C 1 pairs of disjoint closed sets,
it is possible to choose, for each i  n, a partition Ci between Ai and Bi in such a
way that L0 \ : : : \ Ln D ;.
Solution. Say that a space Z has .n C 1/-partition property if, for any family
f.P0 ; Q0 /; : : : ; .Pn ; Qn /g of
T pairs of disjoint closed subsets of Z there exist closed
sets R0 ; : : : ; Rn such that in Ri D ; while Ri is a partition between Pi and Qi
for every i  n. We must prove that dim X  n if and only if our space X has
.n C 1/-partition property.
Fact 1. Given a normal space Z and a natural number m we have dim Z  m if and
only if any open cover U D fUT
0 ; : : : ; UmC1 g of the space Z has an open shrinking
V D fV0 ; : : : ; VmC1 g such that imC1 Vi D ;.
Proof. If dim Z  m and U D fU0 ; : : : ; UmC1 g is an open cover of Z then it has an
open shrinking
T V D fV0 ; : : : ; VmC1 g with ord.V/  m C 1 (see Problem 145). It is
evident that imC1 Vi D ;, so we proved necessity.
To establish sufficiency assume that every open cover of Z of cardinality m C 2
has an open shrinking with empty intersection and take an arbitrary open cover
O D fO0 ; : : : ; Ok g of the space Z. Given an open cover
T W D fW0 ; : : : ; Wk g of Z,
but there exists
a set B  f0; : : : ; kg will be called W-irregular if i2B Wi ; T
an open shrinking W 0 D fW00 ; : : : ; Wk0 g of the family W such that i2B Wi0 D ;.
Let r0 be the number of O-irregular subsets. Proceeding inductively, assume that
p  0 and we have a sequence O0 ; : : : ; Op of open covers of the space Z with the
following properties:
(1) O0 D O and OiC1 is a shrinking of Oi for all i < p;
(2) if ri is the number of Oi -irregular sets then riC1 < ri for all i < p.

2 Solutions of problems 001500

189

If rp > 0 then we can find an Op -irregular set B  T


f0; : : : ; kg and an open
shrinking OpC1 D fO00 ; : : : ; Ok0 g of the cover Op such that i2B Oi0 D ;. It is clear
that B is not OpC1 -irregular set and any OpC1 -irregular set is also Op -irregular.
This shows that the number rpC1 of OpC1 -irregular sets is strictly less than rp ,
so our inductive procedure can be continued as long as rp > 0. Since there are
only finitely many O0 -irregular sets, at some step we will obtain an open shrinking
V D fV0 ; : : : ; Vk g of the family O for which there are no V-irregular sets. In other
words, the family V is a swelling of any of its open shrinkings.
If ord.V/  m C 1 then we already have the needed refinement of the family O.
If not, assume
S without loss of generality that V0 \ : : : \ VmC1 ; and consider the
set G D fVi W m C 1  i  kg. By our assumption about Z there exists
T an open
shrinking H D fH0 ; : : : ; HmC1 g of the cover fV0 ; : : : ; Vm ; Gg such that H D ;.
The family H0 D fH0 ; : : : ; Hm ; HmC1 \VmC1 ; : : : ; HmC1 \Vk g is an open shrinking
of V; since V is a swelling
of H0 , we have H0 \ : : : \ Hm \ .HmC1 \ VmC1 / ;
T
which contradicts H D ;.
Thus any finite open cover of Z has an open shrinking of order at most m C 1,
so dim Z  m; this settles sufficiency and shows that Fact 1 is proved.
Fact 2. For any normal space Z if A and B are closed disjoint subsets of Z then
there exists a closed partition C between the sets A and B.
Proof. By normality of Z there exist disjoint open sets U and V such that A  U
and B  V ; it is evident that the set C D X n.U [ V / is as promised, so Fact 2 is
proved.
Returning to our solution assume that X has .n C 1/-partition property and
take an open cover U D fU0 ; : : : ; UnC1 g of the space X . Fix a closed shrinking
F D fF0 ; : : : ; FnC1 g of the cover U (see Problem 143). There exists a family
fC0 ; : : : ; Cn g of closed
T sets such that Ci is a partition between the sets Fi and X nUi
for every i  n and in Ci D ;. For every i  n take disjoint
sets Vi 2 .Fi ; X /
S
and Wi 2 .X
nU
;
X
/
such
that
X
n.V
[
W
/
D
C
;
then
.W
i
i
i
i [ Vi / D X .
in
S i
If W D in Wi then V D fV0 ; : :S
: ; Vn ; UnC1 \ W g is an open shrinking of U .
Indeed, if x V0 [ : : : [ Vn then x in Fi , so x 2 FnC1 because F is aTcover of
X ; besides, x 2 W and hence x 2 W \ FnC1  W \ UnC1 . Now, if x 2 V then
x 2 W and hence x 2 Wi for some iT n; since also x 2 Vi , we have Vi \ Wi ;
which is a contradiction. Therefore V D ; and hence dim X  n by Fact 1; this
settles sufficiency.
Now assume that dim X  nS
and take a family f.A0 ; B0 /; : : : ; .An ; Bn /g of pairs
of disjoint closed sets. If B D in Bi then B \ A0 \ : : : \ An D ;, so the sets
O0 D X nA0 ; : : : ; On D X nAn ; OnC1 D X nB constitute an open cover of X .
0
0
0
Choose
T an0 open shrinking O D fO0 ; : : : ; OnC1 g of the cover O D fO0 ; : : : ; OnC1 g
with O D ; (see Fact 1) and take a closed shrinking fP0 ; : : : ; PnC1 g of the
family O (see Problem 143). Letting Ui D X nPi for all i  n C 1 we obtain an
open
T cover U D fU0 ; : : : ; UnC1 g of the space X such that Ai  Ui for all i  n and
U D ;; let Vi D Ui nBi for each i  n.

190

2 Solutions of problems 001500

It is easy to check that V D fV0 ; : : : ; Vn ; UnC1 g is an open cover of X ; take a


closed shrinking F D fF0 ; : : : ; FnC1 g of the cover V. For every i  n consider
the closed sets A0i D Ai [ Fi and Bi0 D Bi [ .FnC1 nUi /; it is easy
that
S to check
0
Ai  A0i ; Bi  Bi0 andTA0i \Bi0 D ;. Given
a
point
x
2
X
,
if
x

A
then
x
2
in i
T
FnC1 ; it follows from U D ; thatS. in Ui / \ FnC1 D ;, so x 2 FnC1 nUi  Bi0
for some i  n. This proves that in .A0i [ Bi0 / D X . Fix a closed partition Ci
between the sets A0i and Bi0 (see FactT2); then Ci is also
S a partition between the sets
Ai and Bi for every i  n. Besides, in Ci  X n in .A0i [ Bi0 / D ;. Therefore
X has .n C 1/-partition property, i.e., we have established necessity and hence our
solution is complete.
V.158. Let X be a normal space. Prove that dim X  n if and only if, for any closed
F  X and any continuous map f W F ! S n , there exists a continuous map g W
X ! S n such that gjF D f . Here S n D f.x0 ; : : : ; xn / 2 RnC1 W x02 C: : :Cxn2 D 1g
is the n-dimensional sphere with the topology inherited from RnC1 .
Solution. Given m 2 ! say that a space Z has .m C 1/-partition property if, for
any family f.P0 ; Q0 /; : : : ; .Pm ; Qm /g T
of pairs of disjoint closed subsets of Z there
exist closed sets R0 ; : : : ; Rm such that im Ri D ; while Ri is a partition between
Pi and Qi for every i  m. A space Z will be called an S m -extensor if, for every
closed F  Z and any continuous map f W F ! S m there exists a continuous
map g W Z ! S m such that gjF D f . The set Bm D fx 2 ImC1 W jx.i /j D 1 for
some i  mg is the boundary of the .m C 1/-dimensional cube ImC1 . Let Pim D
fx 2 ImC1 W x.i / D 1g and Qim D fx 2 ImC1 W x.i / D 1g be the respective
faces of ImC1 for every i  m. We must prove that, for a normal space X , we have
dim X  n if and only if X is an S n -extensor.
Fact 1. For any m 2 !, a normal space Z is an S m -extensor if and only if Z is an
S m -extensor.
Proof. Suppose first that Z is an S m -extensor and fix a closed set K  Z. If
f W K ! S m is a continuous map then there exists a set O 2 .K; Z/ and a
continuous map f0 W O ! S m such that f0 jK D f (see Fact 1 of V.093). The
space Z being normal we can find an open subset W of the space Z for which
K  W  W  O (the bar denotes the closure in Z).
The set F D W \ Z is closed in Z and the map f1 D f0 jF W F ! S m is
continuous; the space Z being an S m -extensor, we can choose g0 2 C.Z; S m / such
that g0 jF D f1 . Since the space S m is compact, there exists a continuous map
g W Z ! S m for which gjZ D g0 .
The functions g1 D gjW and h D f0 jW are continuous; besides, the set F is
dense in W and g1 jF D g0 jF D f1 D f0 jF D hjF . An immediate consequence
is that g1 D h; recalling that K  W , we conclude that gjK D g1 jK D hjK D f ,
so g is a continuous extension of f and hence Z is an S m -extensor; this proves
necessity.
Now, if Z is an S m -extensor then fix a closed set F in the space Z and a
continuous function f W F ! S m . The set F is canonically homeomorphic to
F (see Fact 2 of S.451), so there exists a continuous map f0 W F ! S m such

2 Solutions of problems 001500

191

that f0 jF D f . The space Z being an S m -extensor, there is a continuous map


g0 W Z ! S m with g0 jF D f0 . Therefore g D g0 jZ W Z ! S m is a continuous
map such that gjF D f ; this settles sufficiency and shows that Fact 1 is proved.
Fact 2. Given a continuous map f W Y ! Z suppose that A; B  Y and a set
C 0  Z is a partition between f .A/ and f .B/. Then C D f 1 .C 0 / is a partition
between A and B.
Proof. There exist disjoint sets U 0 2 .f .A/; Y / and V 0 2 .f .B/; Y / such that
Y n.U 0 [ V 0 / D C 0 . If U D f 1 .U 0 / and V D f 1 .V 0 / then the sets U and V
are disjoint while U 2 .A; X /; V 2 .B; X / and C D X n.U [ V /, so C is a
partition between A and B, i.e., Fact 2 is proved.
Returning to our solution suppose that dim X  n; then dim X  n by
Problem 147, so the space X has the .n C 1/-partition property (see Problem 157).
This makes it possible to apply Fact 7 of V.100 to see that X is an S n -extensor, so
X is also an S n -extensor by Fact 1. This proves necessity.
Now assume that X is an S n -extensor and fix a family f.A0 ; B0 /; : : : ; .An ; Bn /g
of pairs of disjoint closed subsets of X . Using normality of X it is easy to find a
continuous function 'i W X ! I such that 'i .Ai /  f1g and 'i .Bi /  f1g for
all i  n. The diagonal product ' D in 'i W X ! InC1 is continuous while
n
'.Ai /  Pin and
S '.Bi /  Qi for each i1 n.
Therefore in .Ai [ Bi /  F D ' .Bn /. The space Bn being homeomorphic
to S n (see Fact 1 of V.094) there exists a continuous map h W X ! Bn such
that hjF D 'jF . The set Ci0 D fx 2 InC1 W x.i / D 0g is, evidently, a partition
between the sets Pin and Qin , so Ci0 is also a partition between '.Ai / D h.Ai /
and '.Bi / D h.Bi /; by Fact 1, the setTCi D h1 .Ci0 / is a partition between the
sets Ai and Bi for all i  n. If x 2 in CT
i then h.x/.i / D 0 for all i  n;
this contradiction with h.x/ 2 Bn shows that in Ci D ; and hence we proved
that X has .n C 1/-partition property. This, together with Problem 157, shows that
dim X  n, so we established sufficiency and hence our solution is complete.
V.159. Prove that dim.In / D dim.Rn / D dim.S n / D n for any n 2 N. Here
S n D f.x0 ; : : : ; xn / 2 RnC1 W x02 C : : : C xn2 D 1g is the n-dimensional sphere with
the topology inherited from RnC1 .
Solution. Given m 2 ! say that a space Z has .mC1/-partition property if, for any
family f.P0 ; Q0 /; : : : ; .Pm ; Qm /g T
of pairs of disjoint closed subsets of Z there exist
closed sets R0 ; : : : ; Rm such that im Ri D ; while Ri is a partition between Pi
and Qi for every i  m. The set Bm D fx 2 ImC1 W jx.i /j D 1 for some i  mg is
the boundary of the .m C 1/-dimensional cube ImC1 .
Observe first that the set G D fx 2 InC1 W x.n/ D 1g is a subspace of Bn
homeomorphic to In , so In embeds in Bn ; the spaces S n and Bn are homeomorphic
by Fact 1 of V.094, so In embeds in S n and hence dim.In /  dim.S n / (see
Problem 155).

192

2 Solutions of problems 001500

The space .1; 1/n  In is homeomorphic to Rn , so Rn embeds in In and


therefore dim.Rn /  dim.In /. It follows from In  Rn that dim.In /  dim.Rn /, so
dim.In / D dim.Rn /  dim.S n /.
There exist zero-dimensional spaces A0 ; : : : ; An such that S n D A0 [ : : : [ An
(see Fact 8 of V.100), so the space S n has .n C 1/-partition property by Fact 4
of V.100. Now apply Problem 157 to see that dim.S n /  n. It was proved in
Problem 094 that the space In does not have n-partition property; this, together
with Problem 157, implies that dim.In / > n  1. Therefore n  1 < dim.In / D
dim.Rn /  dim.S n /  n which shows that dim.In / D dim.Rn / D dim.S n / D n.
V.160. Given n 2 N prove that, for any set X  Rn , we have dim X D n if and
only if the interior of X in Rn is nonempty.
Solution. For any number m 2 N and x; y 2 Rm such that x D .x1 ; : : : ; xm / and
y D .y1 ; : : : ; ym /, let x y D .x
q1 y1 ; : : : ; xm ym / and .x; y/ D x1 y1 C : : : C

2 and tx D .tx ; : : : ; tx / for any


xm ym ; as usual, we let jxjm D x12 C : : : C xm
1
m
m
t 2 R. Denote by 0m the point of R whose all coordinates are equal to zero. We
use the symbol P to denote the set of irrational numbers. If L is a linear space then
0L is the zero vector of L.

Fact 1. Suppose that A; B  R are countable dense subsets and f W A ! B


is a bijection such that a < b implies f .a/ < f .b/. Then there exists a unique
homeomorphism h W R ! R such that hjA D f and x < y implies h.x/ < h.y/.
Proof. If h0 ; h1 2 C.R/ and h0 jA D f D h1 jA then h0 D h1 by Fact 0 of S.351;
this proves that the promised homeomorphism is unique (if it exists).
To establish existence, for any x 2 R let h.x/ D supff .a/ W a 2 A and a < xg.
Observe first that h.x/ is well defined for if q > x and q 2 A then f .q/ is an upper
bound for the set ff .a/ W a 2 A; a < xg. It is immediate from the definitions that
h.a/  f .a/ for any a 2 A. If h.a/ < f .a/ then, by density of B D f .A/ there
is a0 2 A such that h.a/ < f .a0 / < f .a/; then a0 < a and therefore f .a0 /  h.a/
which is a contradiction. Thus h.a/ D f .a/ for every a 2 A. If x; y 2 R and x < y
then take a; b 2 A such that x < a < b < y. Then h.x/  f .a/ < f .b/  h.y/,
so we proved that
(1) hjA D f and h.x/ < h.y/ whenever x < y and, in particular, h is injective.
Now, take any point t 2 R and consider the set P D fb 2 B W b < t g. Since
f is a bijection, there is a 2 A for which f .a/ > t ; then x < a for any x 2
f 1 .P /. Therefore the point y D sup.f 1 .P // is well defined; if x < y and
x 2 A then there is z 2 f 1 .P / with x < z, so f .x/ < f .z/ < t which shows that
f .x/ < t and therefore h.y/  t . If h.y/ < t then there is c 2 A such that h.y/ <
f .c/ < t . Thus c 2 f 1 .P / and hence c  sup.f 1 .P // D y, so f .c/  h.y/,
a contradiction. This proves that h.y/ D t and hence h W R ! R is a bijection. It
follows from (1) that h..a; b// D .h.a/; h.b// and h1 ..a; b// D .h1 .a/; h1 .b//
for any a; b 2 R with a < b. Therefore h is a homeomorphism, so Fact 1 is proved.

2 Solutions of problems 001500

193

Fact 2. If k 2 N then any linear subspace of Rk is closed in Rk ; besides, if L  Rk


is a linear subspace and Int.L/ ; then L D Rk .
Proof. If L is a linear subspace of Rk then there are linearly independent vectors
w1 ; : : : ; wl 2 Rk such that L is the linear hull of fw1 ; : : : ; wl g. There exist vectors
fwlC1 ; : : : ; wk g such that W D fw1 ; : : : ; wk g is a linear basis in Rk (see Fact 1 of
S.489). For any i D 1; : : : ; k, let ei D .e1i ; : : : ; eki / 2 Rk be the point for which
eii D 1 and eji D 0 whenever i j . Take a set faji W i; j 2 f1; : : : ; kgg  R
such that ei D a1i w1 C : : : C aki wk for every i  k. It is straightforward that
P
i
L D fx D .x1 ; : : : ; xk / 2 Rk W kiD1 xi am
D 0 for all m D l C 1; : : : ; kg, so L is
closed being a finite intersection of inverse images of zero under linear (and hence
continuous) functions on Rk .
Now suppose that p 2 L belongs to the interior of L in Rk and let '.x/ D x  p
for all x 2 Rk . It is clear that ' W Rk ! Rk is a homeomorphism such that '.L/ 
L and '.p/ D 0k . Therefore 0k also belongs to the interior of L and hence there is
" > 0 such that B D fx 2 Rk W jxjk < "g  L. Thus 2" ei 2 B  L for every i  k;
the family f 2" ei W i  kg  L being a basis in Rk , we have L D Rk , so Fact 2 is
proved.
Fact 3. Suppose that m  2 is a natural number and L is an m-dimensional linear
space.
S If Lk is an .m  1/-dimensional linear subspace of L for any k 2 ! then
Ln k2! Lk ;.
Proof. Fix a linear basis fe1 ; : : : ; em g in the space L. If x D .x1 ; : : : ; xm / 2 Rm
let '.x/ D x1 e1 C : : : C xm em ; then the map ' W Rm ! L is an isomorphism, so
Mk D ' 1 .Lk / is an .m  1/-dimensional linear subspace of Rm for every k 2 !.
m
Every set Mk is nowhere dense in Rm by
the Baire
S Fact 2; since the space R hasS
k
property, we can find a point y 2 R n i2! Mi . Then x D '.y/ 2 Ln k2! Lk ,
so Fact 3 is proved.
Fact 4. Suppose that m  2 is a natural number and L is an m-dimensional linear
space. If P  Lnf0L g is a countable set then there exists an .m  1/-dimensional
linear subspace M  L such that M \ P D ;.
Proof. Fix a linear basis fe1 ; : : : ; em g in the space L. If x D .x1 ; : : : ; xm / 2 Rm
let '.x/ D x1 e1 C : : : C xm em ; then the map ' W Rm ! L is an isomorphism, so
Q D ' 1 .P / is a countable subset of Rm . The set M c
D fx 2 Rm W .x; c/ D 0g
m
is a proper linear subspace of Rm , so it is closed and nowhere
S dense in R for any
m
c 2 Q (see Fact 2). By Fact 3 there is a vector x 2 R n fM c
W c 2 Qg; then
R D M x
is an .m  1/-dimensional linear subspace of Rm with R \ Q D ;.
Therefore M D '.R/ is an .m  1/-dimensional linear subspace of L such that
M \ P D ;, so Fact 4 is proved.
Fact 5. Suppose that m 2 N and L is an m-dimensional linear space. Then, for any
countable P  Lnf0L g there exists a linear basis E D fe1 ; : : : ; em g in the space L
such that for every p 2 P all coordinates of p with respect to E are nonzero, i.e.,
there are p1 ; : : : ; pm 2 Rnf0g such that p D p1 e1 C : : : C pm em .

194

2 Solutions of problems 001500

Proof. If m D 1 then any basis will do so assume that m > 1. Apply Fact 4 to
find an .m  1/-dimensional linear subspace M  L such that M \ P D ;;
let E 0 D fe1 ; : : : ; em1 g be a linear basis in M . Denote by Hi .p/ the linear hull
of the set .E 0 nfei g/ [ fpg; then Hi .p/ is an .m  1/-dimensional linear subspace
of L for any i S
 m  1 and p 2 P . Apply Fact 3 to see that there exists a point
em 2 Ln.M [ fHi .p/ W i  m  1; p 2 P g/.
It is evident that E D fe1 ; : : : ; em g is a linear basis in L. Take any p 2 P ; then
p D p1 e1 C : : : C pm em for some p1 ; : : : ; pm 2 R. If pj D 0 then 1  j  m  1
because p does not belong to M . Therefore em D p1m .p1 e1 C : : : C pm1 em1  p/
belongs to the linear hull of the set .E 0 nfej g/ [ fpg which is a contradiction with
the choice of em . Therefore pi 0 for any i  m, i.e., the basis E is as promised,
so Fact 5 is proved.
Fact 6. Given m 2 N suppose that L is an m-dimensional linear space and P  L
is a countable set. Then there exists a linear basis E D fe1 ; : : : ; em g in the space L
such that P is in general position with
P respect to E, i.e.,
P for any distinct p; q 2 P
if p1 ; q1 ; : : : ; pm ; qm 2 R and p D im pi ei ; q D im qi ei then pi qi for
all i  m.
Proof. The set A D fp  q W p; q 2 P and p qg  Lnf0L g is countable,
so we can apply Fact 5 to find a linear basis E D fe1 ; : : : ; em g in L such that all
coordinates with respect to E of every element of A are distinct from zero. It is clear
that E is as promised, so Fact 6 is proved.
Fact 7. If n 2 N and P  Rn is a countable set then there is homeomorphism
' W Rn ! Rn such that '.P / is in general position, i.e., for any distinct p; q 2 '.P /
if p D .p1 ; : : : ; pn / and q D .q1 ; : : : ; qn / then pi qi for all i  n.
Proof. By Fact 6, there is a linear basis E D fw1 ; : : : ; wn g in the space Rn such that
P is in general position with respect to E. Let fe1 ; : : : ; en g be the standard linear
basis of Rn , i.e., ei D .e1i ; : : : ; eni / while eii D 1 and eji D 0 whenever i and j are
distinct elements of f1; : : : ; ng.
Choose a set faji W i; j 2 f1; : : : ; ngg  R such that ei D a1i w1 C : : : C ani wn
for every i  n. For each point x D .x1 ; : : : ; xn / 2 Rn let '.x/ D .y1 ; : : : ; yn /
where yj D x1 aj1 C : : : C xn ajn for all j  n. The map ' W Rn ! Rn is a linear
homeomorphism and the coordinates of every '.p/ coincide with the coordinates
of p with respect to E. Therefore '.P / is as promised, so Fact 7 is proved.
Fact 8. For any n 2 N, if A and B are countable dense subspaces of Rn then there
exists a homeomorphism h W Rn ! Rn such that h.A/ D B.
Proof. By Fact 7, there is no loss of generality to assume that both A and B are in
general position; let fak W k 2 !g and fbk W k 2 !g be some faithful enumerations of
A and B respectively. We will choose inductively new faithful enumerations fck D
.c1k ; : : : ; cnk / W k 2 !g and fdk D .d1k ; : : : ; dnk / W k 2 !g of the sets A and B
respectively in such a way that

2 Solutions of problems 001500

195

(2) fa0 ; : : : ; ak g  fc0 ; : : : ; c2k g and fb0 ; : : : ; bk g  fd0 ; : : : ; d2k g for any k 2 !;
(3) .cik  cil /.dik  dil / > 0 for any distinct k; l 2 ! and i  n.
To start off, let c0 D a0 and d0 D b0 and suppose that we have chosen points
fc0 ; : : : ; c2m g  A and fd0 ; : : : ; d2m g  B, so that the property (2) is fulfilled for
all k  m and the property (3) holds for all distinct k; l  2m.
Take the minimal j 2 N for which aj fc0 ; : : : ; c2m g and let c2mC1 D aj . For
every i  n the set Oi D ft 2 R W .ci2mC1  cik /.t  dik / > 0 for all k  2mg is a
nonempty open interval of R, so O D O1  : : :  On is a nonempty open subset of
Rn which shows that we can choose a point d2mC1 2 .Bnfd0 ; : : : ; d2m g/ \ O.
Take the minimal l 2 N for which bl fd0 ; : : : ; d2mC1 g and let d2mC2 D bl .
The set Ui D ft 2 R W .t  cik /.di2mC2  dik / > 0 for all k  2m C 1g is a
nonempty open interval of R, so U D U1  : : :  Un is a nonempty open subset of
Rn ; choose a point c2mC2 2 Anfa0 ; : : : ; a2mC1 g/ \ U . It is straightforward that the
condition (2) is fulfilled for k  m C 1 and (3) holds for all distinct k; l  2m C 2,
so our inductive procedure can be continued to construct sequences fck W k 2 !g
and fdk W k 2 !g with the properties (2) and (3). It follows from (2) and (3) that
these sequences constitute a faithful enumeration of the sets A and B respectively.
Let Ci D fcik W k 2 !g and Di D fdik W k 2 !g; letting fi .cik / D dik for any
k 2 ! we obtain a bijection fi W Ci ! Di for every i  n. An easy consequence of
(3) is that t < s implies fi .t / < fi .s/ for any i  n and t; s 2 Ci . Therefore Fact 1
is applicable to find a homeomorphism hi W R ! R such that hi is strictly increasing
and hi jCi D fi for all i  n. The diagonal product h D in hi W Rn ! Rn is a
homeomorphism and it is immediate that h.A/ D B, so Fact 8 is proved.
Fact 9. The set Z.k; m/ D fx D .x1 ; : : : ; xm / 2 Rm W jfi  m W xi 2 Pgj D kg is
zero-dimensional for all m 2 N and k 2 f0; : : : ; mg.
Proof. Let H.r1 ; : : : ; rmk ; i1 ; : : : ; imk / D f.x1 ; : : : ; xm / 2 Z.k; m/ W xij D rj
for all j  m  kg for any r1 ; : : : ; rmk 2 Q and distinct i1 ; : : : ; imk 2 f1; : : : ; mg.
Every set H.r1 ; : : : ; rmk ; i1 ; : : : ; imk / is zero-dimensional being homeomorphic
to the space Pk . Besides, the family H D fH.r1 ; : : : ; rmk ; i1 ; : : : ; imk / W ri 2 Q
for each
S i  mk and i1 ; : : : ; imk are distinct elements of f1; : : : ; mgg is countable
and H D Z.k; m/.
Now, if H D H.r1 ; : : : ; rmk ; i1 ; : : : ; imk / is any element of H then the set
F D fx 2 Rm W xij D rj for all j  m  kg is closed in Rm ; it is easy to see that
F \ Z.k; m/ D H and hence every H 2 H is closed in Z.k; m/. Finally apply
SFFS-311 and SFFS-306 to conclude that Z.k; m/ is zero-dimensional. Fact 9 is
proved.
Returning to our solution assume that the interior of X is nonempty; then there
is a point a 2 X and " > 0 such that the ball G D fx 2 Rn W jx  ajn < "g is
contained in X . The ball G is homeomorphic to the set G0 D fx 2 Rn W jxjn < "g.
"
n
n
If '.x/ D 2n
x for any x 2 Rn then
p map ' W R ! R is"a homeomorphism;
p
n
given any x 2 I we have jxjn  n and hence j'.x/jn  2n n D 2p" n < "
which shows that '.In /  G0 and hence dim G0  dim.In / D n (see Problem 155
and Problem 159). Since G0  Rn , we have dim G0  dim.Rn / D n, so dim G0 D

196

2 Solutions of problems 001500

dim G D n. It follows from G  X  Rn that we can apply Problem 155 again to


conclude that n D dim G  dim X  dim.Rn / D n and hence dim X D n. This
proves sufficiency.
For necessity observe first that Rn nQn D Z.1; n/ [ : : : [ Z.n; n/, so Fact 9
together with Fact 4 of V.100 imply that P D Rn nQn has the n-partition property
and hence dim P  n  1 by Problem 157. If X  Rn and Int.X / D ; then
Rn nX is dense in Rn ; choose a countable dense set B  Rn nX . Then B is dense
in Rn and X  Rn nB. Apply Fact 8 to find a homeomorphism h W Rn ! Rn
such that h.B/ D Qn . Then hj.Rn nB/ is a homeomorphism between Rn nB and P ;
as a consequence, dim.Rn nB/ D dim P  n  1. Finally, apply Problem 155 to
conclude that dim X  dim.Rn nB/  n  1; this settles necessity and completes
our solution.
V.161. Prove that, for any Tychonoff space X and n 2 !, we have dim X  n if and
only if, for any second countable space Y and any continuous f W X ! Y , there
exists a second countable space M and continuous maps g W X ! M; h W M ! Y
such that dim M  n and f D h g.
Solution. Given
S a space Z, a set U  Z and a family A of subsets of Z let
St.U; A/ D fA 2 A W A \ U ;g be the star of the set U with respect to
A. As usual, we write St.z; A/ instead of St.fzg; A/.
Fact 1. Given m 2 ! suppose that K is a compact space with dim K  m and
N is a second countable space. Then, for any continuous map p W K ! N there
exists a second countable space L such that dim L  m and there are continuous
maps q W K ! L and r W L ! N for which r q D p.
Proof. Fix a countable base B in the space N ; let BK D fp 1 .B/ W B 2 Bg and
denote by V0 the family of all finite covers of K with the elements of BK . For every
finite S  V0 apply TFS-230 to find a finite open cover V of the space K such
that V is a star refinement of every U 2 S. Since dim K  m, we can find a finite
open refinement VS of the cover V such that ord.VS /  m C 1. It is evident that the
family V1 D fVS W S is a finite subfamily of V0 g is countable.
Proceeding by induction assume that k 2 N and we have constructed a sequence
V0 ; : : : ; Vk of countable families of finite open covers of K such that
(1) if i 2 f1; : : : ; kg then ord.V/  m C 1 for every V 2S
Vi ;
j 1
(2) for any j 2 f1; : : : ; kg and any finite family S  iD0 Vi there is V 2 Vj
which is a star refinement of every U 2 S.
S
For every finite S  fVi W i 2 f0; : : : ; kgg apply TFS-230 to find a finite
cover V of the space K such that V is a star refinement of every U 2 S. Since
dim K  m, we can find a finite open refinement VS of the cover V such that
ord.V
S S /  m C 1. It is evident that the family VkC1 D fVS W S is a finite subfamily
of kiD0 Vi g is countable and the conditions (1)(2) are satisfied for the sequence
fV0 ; : : : ; VkC1 g. Therefore our inductive procedure can be continued to construct
a sequence
fVi W i 2 !g such that (1) and (2) are satisfied for all k 2 !; let
S
V D fVi W i 2 Ng.

2 Solutions of problems 001500

197

Given a point x 2 K let x


D fy 2 K W for every V 2 V there is V 2 V such
that fx; yg  V g; we will call the set x
the equivalence class of x. It is clear that
(3) for any x; y 2 K we have x 2 x
and x 2 y
if and only if y 2 x
.
It turns out that we also have transitivity for the equivalence classes, i.e.,
(4) for any x; y; z 2 K if y 2 x
and z 2 y
then z 2 x
.
Take any family V 2 V; it follows from the property (2) that there exists
W 2 V such that W is a star refinement of V. There exist W0 ; W1 2 W such
that fx; yg  W0 and fy; zg  W1 . There is V 2 V such that St.W0 ; W/  V ;
it is clear that fx; y; zg  W0 [ W1  St.W0 ; W/, so fx; zg  V and hence
z 2 x
, i.e., (4) is proved.
An immediate consequence of (4) is that
(5) for any x; y 2 K if y 2 x
then x
D y
; if y x
then
S x
\ y
D ;.
Let L D fx
W x 2 Kg; we will need the set O.P / D P for any P  L. If
q.x/ D x
for any x 2 K then it follows from the property (5) that the map
q W X ! L is well defined. Consider the family L D fU  L W O.U / 2
.K/g; it is evident that ; 2 L and L 2 L . If U; V 2 L then it follows from
(5) that O.U
/ D O.U / \ O.V / 2 .K/, so U \ V 2 L . Now,
S \ VS
S if U  L
then O. U / D fO.U / W U 2 U g 2 .K/ which shows that U 2 L and
hence L is a topology on L; from now on we will identify L with the space
.L; L /.
Observe that q 1 .U / D O.U / for any U  L, so the map q is quotient; since
q.K/ D
TL, the space L is compact.
T Our next step is to show that
(6) x
D fSt.x; V/ W V 2 Vg D fSt.x; W/ W W 2 Vg and, in particular, the
set x
is closed in K for any x 2 K.
T
Fix a point x 2 K and observe that the equality x
D fSt.x; V/ W V 2 Vg is
immediate from the definition. Now, if V 2 V then there is W 2 V which is a
star refinement of V. If W 2 W then, for any H 2 W we have H \ W ; if
and only if H \W ;; this implies that St.W; W/ D St.W ; W/. There exists
VW 2 V with St.W; W/  VW , so St.W ; W/
S  VW . The families V and W
being finite, we conclude that St.x; W/  fVW WT
x 2 W 2 Wg  St.x; V/.
The
cover
V
2
V
was
chosen
arbitrarily
so
x


fSt.x; W/ W W 2 Vg 
T
fSt.x; V/ W V 2 Vg D x
and hence (6) is proved.
We will also need the following property of our covers.
(7) For any x 2 K and US2 .x
; K/ there exists a cover V 2 V and V 2 .K/
such that x
 V  fy
W y 2 V g  St.V; V/  U . T
It follows from (6) that there are W1 ; : : : ; Wk such that ik St.x; Wi / 
U (see Fact 1 of S.326). Apply (2) to take a cover W 2 V which is a star
refinement of every Wi ; there exists V 2 V such that V is a star refinement
of W. Then St.x; W/  U ; let V D St.x; V/ and take any set H 2 V with
x 2 H . There exists WH 2 W such that St.H;
S V/  WH ; it follows from
that
W

St.x;
W/,
so
St.V;
V/

fSt.H; V/ W x 2 H 2 Vg 
x
2
W
H
H
S
fWH W x 2 H 2 Vg  St.x; W/  U .

198

(8)

(9)

(10)

(11)

(12)

2 Solutions of problems 001500

The inclusion x
 V D St.x; V/ is an immediate consequence of (6); the
second inclusion in (7) is evident and the third one follows from the fact that
y
 St.y; V/  St.V; V/ for any y 2 V , so (7) is proved.
For any U 2 .K/ let U  D fa 2 L W a  U g. Given any z 2 O.U  / there is
x 2 K such that z D x
.
S It follows from (7) that there exists a set V 2 .K/
such that x
 V and ft
W t 2 V g  U . Thus y 2 O.U  / for any y 2 V
which shows that every point of O.U  / is contained in O.U  / together with
some open neighborhood and hence
the set q 1 .U  / D O.U  / is open in K and hence U  is open in L for any
U 2 .K/.
If a and b are distinct points of the space L then there are x; y 2 K such that
a D x
; b D y
and x
\ y
D ;. By normality of K there exist disjoint
sets U 2 .x
; K/ and V 2 .y
; K/. It is clear that a 2 U  ; b 2 V  and
U  \ V  D ;. This proves, together with (8) and TFS-124, that
the space L is Hausdorff and hence Tychonoff.
Fix an arbitrary point x 2 X and take any z f .x/. It is easy to see that the
family B 0 D fB 2 B W jB \ ff .x/;S
zgj  1g is a cover of the space N , so there
is a finite C  B 0 such that K D ff 1 .B/ W B 2 Cg. There exists V 2 V
which is a star refinement of the cover C 0 D ff 1 .B/ W B 2 Cg. In particular,
x
 St.x; V/  f 1 .B/ for some B 2 C. Then f .x/ 2 B, so z B which,
together with f .x
/  B shows that z f .x
/. The point z f .x/ was
chosen arbitrarily and hence we proved that
f .x
/ D ff .x/g for any x 2 X .
Now, given any a 2 L take x 2 X with x
D a and let r.a/ D f .x/. The
property (10) shows that the map r W L ! N is well defined and r q D f .
The map q is quotient and r q D f is continuous, so we can apply Fact 1 of
T.268 to see that the map r is also continuous.
Now take a family V 2 V and choose W 2 V such that W is a star refinement
of V. Given a point a 2 L there is x 2 K with a D x
; take W 2 W with
x 2 W . Since there is V 2 V for which St.W; W/  V , it follows from (6)
that x
 St.x; W/  St.W; W/  V and hence a D x
2 V  . This proves
that
the family V  D fV  W V 2 Vg is an open cover of L for any V 2 V.
Now take a 2 L and H 2 .a; L/. The set U D q 1 .H / is open in K; pick a
point x 2 X with a D x
. Then x
 U , so we can apply (6) to find V 2 V
with St.x; V/  U . It follows from (11) that there is V 2 V such that a 2 V 
and hence x
 V ; since x 2 V , the inclusion V  St.x; V/  U shows that
V   U  D H and therefore a 2 V   H . As a consequence,
the family fV  W V 2 V 2 Vg is a base of L and hence L is second countable.
To finally prove that dim L  n fix a finite open cover H of the space L. Then
G D fq 1 .H / W H 2 Hg is an open cover of K, so we can apply (7) to find,
for every x 2 K, a set Vx 2 .x; K/ such that St.Vx ; Vx /  q 1 .Hx / for
some Hx 2 H and Vx 2 V.
S
There is a finite set A  K such that fVx W x 2 Ag D K; apply (2) to find
a cover V 2 V which is a star refinement of Vx for every x 2 A. To show

2 Solutions of problems 001500

199

that V refines G take any V 2 V. If V D ; then there is nothing to prove;


otherwise, there is x 2 A such that Vx \ V ;. Choose W 2 Vx such that
St.V; V/  W . Then W \ Vx ; and therefore W  St.Vx ; Vx /  q 1 .Hx /.
As a consequence, V  St.V; V/  W  q 1 .Hx /, so V refines G. Thus V  is
a refinement of H; it easily follows from ord.V/  nC1 that ord.V  /  nC1
as well so any finite open cover of L has an open refinement of order at most
nC1. Applying Problem 145 we conclude that dim L  n, so Fact 1 is proved.
Returning to our solution assume that dim X  n and take a continuous map
f W X ! Y for some second countable space Y . We can consider that Y  K for
some metrizable compact space K, so there is a continuous map f0 W X ! K such
that f0 jX D f . Since dim.X / D dim X  n (see Problem 147), we can apply
Fact 1 to find a second countable space Z and continuous maps g0 W X ! Z and
h0 W Z ! K such that dim Z  n and h0 g0 D f0 . Let M D g0 .X /; h D h0 jM
and g D g0 jX . The space M is second countable and dim M  dim Z  n (see
Problem 155); besides, g W X ! M; h W M ! Y and h g D f , so we proved
necessity.
Finally, assume that, for any second countable space Y , if f W X ! Y is a
continuous map then there exists a second countable space M and continuous maps
g W X ! M; h W M ! Y such that dim M  n and h g D f . Given
a functionally open cover U D fU0 ; : : : ; Uk g of the space X take a continuous
function fi W X ! R such that X nUi D fi1 .0/ for each i  k. The diagonal
product f D ik fi W X ! RkC1 is continuous; let Y D f .X / and consider
the natural projection i W RkC1 ! R of the space RkC1 onto its i -th factor for all
i  k. If Vi D i1 .Rnf0g/ \ Y then Vi is an open subset of Y and f 1 .Vi / D Ui
for every i  k.
By our assumption there is a second countable space M and continuous maps
g W X ! M; h W M ! Y such that dim M  n and h g D f . The family
V D fh1 .Vi / W i  kg is a functionally open cover of M , so it has a functionally
open refinement W with ord.W/  n C 1. It is straightforward that the family U 0 D
fg 1 .W / W W 2 Wg is a functionally open refinement of U and ord.U 0 /  n C 1.
Thus dim X  n, i.e., we proved sufficiency and hence our solution is complete.
V.162. Prove that, for any n 2 !, there exists a compact second countable space
Un such that dim Un  n and any second countable X with dim X  n can be
embedded in Un .
P
Solution. The space S n D fx D .x0 ; : : : ; xn / 2 RnC1 W niD0 xi2 D 1g is the usual
n-dimensional sphere with the topology inherited from RnC1 . Let A D fY W Y 
I! and dim Y  ng; choose a family fYt W t 2 T g of spaces with the following
properties:
(1) w.Yt /  ! and dim Yt  n for every t 2 T ;
(2) a homeomorphism et W Yt ! At 2 A is fixed for every t 2 T ;
(3) for any A 2 A there is t 2 T such that A D At .
L
In the space G D fYt W t 2 T g we identify every Yt with the respective clopen
subset of G. It is an easy exercise to see that G is a normal space. If F  G is

200

2 Solutions of problems 001500

closed and f W F ! S n is a continuous map then Ft D F \ Yt is closed in Yt ,


so there exists a continuous map gt W Yt ! S n such that gt jFt D f jFt for every
t 2 T (see Problem 158). Given any x 2 G there is a unique t 2 T with x 2 Yt ; let
g.x/ D gt .x/. It is straightforward that g W G ! S n is continuous and gjF D f ,
so dim G  n by Problem 158.
For every x 2 G there is a unique t 2 T with x 2 Yt ; let e.x/ D et .x/. It is
easy to check that the map e W G ! I! is continuous and ejYt D et for each t 2 T .
Take a continuous map p W G ! I! such that pjG D e; since dim.G/  n (see
Problem 147), we can apply Problem 161 to find a second countable space M such
that dim M  n and there exist continuous maps q W G ! M and r W M ! I!
with r q D p.
The space Un D q.G/ is compact; it follows from Un  M that Un is second
countable and dim Un  dim M  n (see Problem 155). If X is a second countable
compact space with dim X  n then we can consider that X  I! and hence
X 2 A; consequently, there is t 2 T such that X D At . The map pjYt D ejYt D et
is a homeomorphism of Yt onto At D X . If qt D qjYt and rt D rjq.Yt / then
et D rt qt . The map et being a homeomorphism, it is an easy exercise that both
maps rt and qt are condensations so they are homeomorphisms by Fact 2 of S.337.
Thus the space X D et .Yt / D rt .qt .Yt // is homeomorphic to the subspace qt .Yt /
of the space Un , so every second countable space X with dim X  n embeds in Un .
V.163. Suppose that X is a second countable space, Y  X and dim Y  n. Prove
that there exists a G -set Y 0 of the space X such that Y  Y 0 and dim Y 0  n.
Solution. If n D 1 then we can take Y 0 D X , so assume that n 2 !. We
can consider that X  K for some metrizable compact space K. It follows from
Problem 162 that there exists a compact metrizable space M such that dim M  n
and Y embeds in M , so fix a subspace Z  M and a homeomorphism h W Z ! Y .
Apply Fact 2 of T.333 to find a G -subset Z 0  M and a G -subset H  K such
that Z  Z 0 ; Y  H and there exists a homeomorphism g W Z 0 ! H with gjZ D
h. We have dim Z 0  dim M  n (see Problem 155), so dim H  n. It is clear that
Y 0 D H \ X is a G -subset of X such that Y  Y 0 and dim Y 0  dim H  n.
V.164. Given n 2 ! and a second countable Tychonoff space X prove that
dim X  n if and only there exist X0 ; : : : ; Xn  X such that X D X0 [ : : : [ Xn
and dim Xi  0 for each i  n.
Solution. Given spaces Y and Z we say that a map f W Y ! Z is an embedding if
f W Y ! f .Y / is a homeomorphism. If .Z; d / is a metric space and A  exp.Z/
then mesh.A/ D supfdiamd .A/ W A 2 Ag and ord.x; A/ D jfA 2 A W x 2 Agj for
any x 2 Z; if T  Z then AjT D fA \ T W A 2 Ag. Given sets A; B  Z let
d.A; B/ D inffd.a; b/ W a 2 A; b 2 Bg.
Fact 1. Suppose that M is a second countable space and F; G  M are closed and
disjoint. Then
(i) there exists a metrizable compact space K and an embedding e W M ! K such
that clK .e.F // \ clK .e.G// D ;;

2 Solutions of problems 001500

201

(ii) if, additionally, dim M  m for some m 2 ! then there exists a compact
metrizable space K and an embedding e W M ! K such that dim K  m and
clK .e.F // \ clK .e.G// D ;.
Proof. (i) By normality of M we can find a continuous function f W M ! 0; 1

such that f .F /  f0g and f .G/  f1g. There exists a metrizable compact
space K 0 such that M embeds in K 0 ; fix an embedding h W M ! K 0 . It is
easy to check that e D h f W M ! K 0  0; 1
is still an embedding; let
M 0 D e.M / and K D M 0 .
If  W K ! 0; 1
is the restriction of the natural projection of K 0  0; 1
onto
its factor 0; 1
, then .e.F // D f .F /  f0g and .e.G// D f .G/  f1g.
Therefore clK .e.F // \ clK .e.G//  clK . 1 .0// \ clK . 1 .1// D ;, so e is
the promised embedding of M .
(ii) If, additionally, we have dim M  m then apply (i) to find a metrizable
compact space K 0 for which there exists an embedding h W M ! K 0 such
that clK 0 .h.F // \ clK 0 .h.G// D ;. Take a continuous function g W M ! K 0
such that gjM D h; since dim.M / D dim M  m (see Problem 147),
we can apply Problem 161 to find a metrizable compact space K for which
dim K  m and there exist continuous maps p W M ! K and q W K ! K 0
such that g D q p.
If e D pjM and r D qjp.M / then e W M ! K and r W p.M / ! K 0 are
continuous maps such that r e D h. It is an easy exercise to prove, using Fact 2
of S.337, that both maps e and r are embeddings. It follows from q.e.F // D
h.F / and q.e.G// D h.G/ that clK .e.F // \ clK .e.G//  q 1 .clK 0 .h.F // \
q 1 .clK 0 .h.G// D ;. This, together with dim K  m, shows that we settled
(ii) and hence Fact 1 is proved.
Fact 2. If m 2 ! and .K; d / is a metric compact space then dim K  m if and only
if there is a sequence fUk W k 2 !g of finite covers of K such that ord.Uk /  m C 1
for any k 2 ! and the sequence fmesh.Uk / W k 2 !g converges to zero.
Proof. Suppose first that dim K  m and fix an arbitrary k 2 !. Since the family
B D fBd .x; 2k1 / W x 2 Kg is an open cover of the space K, we can find a finite
subcover V D fV1 ; : : : ; Vl g of the cover B; then mesh.V/  2k . Since dim K  m,
there exists a refinement Uk of the cover V such that ord.Uk /  m C 1. After
constructing Uk for every k 2 ! we obtain the promised sequence fUk W k 2 !g.
Now assume that we have a sequence fUk W k 2 !g of finite open covers of K
such that mesh.Uk / ! 0 and ord.Uk /  m C 1 for every k 2 !. If V is an open
finite cover of K then we can apply TFS-244 to find a number > 0 such that
every subset of K of diameter less than is contained in an element of V. There
exists k 2 ! with mesh.Uk / < and hence the diameter of every U 2 Uk is less
than ; this implies that every element of Uk is contained in an element of V, i.e.,
Uk is a finite open refinement of V of order  m C 1. Therefore dim K  m (see
Problem 145), i.e., we settled sufficiency and hence Fact 2 is proved.

202

2 Solutions of problems 001500

Fact 3. Given m 2 ! and a second countable space Z with dim Z  m, for any
closed disjoint sets A; B  Z there exists a partition C between the sets A and B
such that dim C  m  1.
Proof. Apply Fact 2 to find a compact space K such that dim K  m and there is an
embedding e W Z ! K with clK .e.A// \ clK .e.B// D ;. To simplify the notation
we identify Z and e.Z/; then Z  K and A \ B D ; (the bar denotes the closure
in K). Fix a metric d which generates the topology of K.
Proceeding inductively, let F0 D A; G0 D B and assume that k 2 ! and we have
a family fF0 ; G0 ; : : : ; Fk ; Gk g of closed subsets of K with the following properties:
(1) Fi  Int.FiC1 / and Gi  Int.GiC1 / for all i < k;
(2) Fi \ Gi D ; for every i  k;
S
(3) if Ci D Kn.Fi [Gi / then there is a finite Ui  .K/ such that Ci  Ui while
ord.x; Ui /  m for any x 2 Ci and mesh.Ui /  2i for every i 2 f1; : : : ; kg.
It follows from compactness of K that D d.Fk ; Gk / > 0; it is easy to construct
a finite open cover V of the space K such that mesh.V/  nnf 2 ; 2k1 g. By
dim K  m there is a finite refinement H of the cover V such that ord.H/  m C 1;
the union of the families W0 D fU 2 H W U \ FkSD ;g and W1 DSfU 2 H W
U \ F ;g is equal to H, so the open sets W0 D W0 and W1 D W1 cover
the space K.
Since the closure of any element of H cannot intersect both Fk and Gk , the set
GkC1 D KnW1 is closed and Gk  GkC1 . Besides, Gk  KnW1  GkC1 , so
Gk  Int.GkC1 /. The set FkC1 D KnW0 is also closed and Fk  KnW0  FkC1 ,
so Fk  Int.FkC1 /. It follows from W0 [ W1 D K that the sets FkC1 and GkC1 are
disjoint, so the properties (1) and (2) hold if k is replaced with k C 1.
Now, if CkC1 D Kn.FkC1 [ GkC1 / D W0 \ W1 , so both families W0 and W1
cover CkC1 . As a consequence, the family UkC1 D W0 covers the set CkC1 and it
follows from W0 \W1 D ; that ord.x; UkC1 / < mC1 for every x 2 CkC1 because,
from at most .m C 1/-many elements of V that contain x, at least one element of W1
was removed. Thus the condition (3) is also satisfied for all i 2 f1; : : : ; k C 1g and
hence our inductive procedure can be continued to construct families fFi W i 2 !g
and fGi W i 2 !g such that the conditions (1)(3)
are satisfied for
S
S every k 2 !.
It follows from (1) and (2) that OF D i2! Fi and OG D i<! Gi are disjoint
open sets such that A  OF and B  OG . If C 0 D Kn.OF [ OG / then C 0 is a
0
partition between
T the sets A and B in K. The family Vk D Uk jC is a finite open
0
cover of C D fCi W i 2 Ng and the property (3) implies that ord.Vk /  m for
each k 2 N; it is evident that the sequence fmesh.Vk / W k 2 Ng converges to zero,
so we can apply Fact 2 to conclude that dim.C 0 /  m  1. The set C D C 0 \ Z is a
partition between A and B in Z and dim C  dim.C 0 /  m  1 (see Problem 155),
so Fact 3 is proved.
Returning to our solution assume that X D X0 [ : : : [ Xn and dim Xi D 0 for
all i  n. Then every Xi is zero-dimensional by Problem 149, so we can apply
Fact 4 of V.400 to see that X has .n C 1/-partition property. This, together with
Problem 157 implies that dim X  n, so we proved sufficiency.

2 Solutions of problems 001500

203

We will prove necessity by induction on n. If n D 0 then X is zero-dimensional


by Problem 149, so X0 D X gives the desired decomposition of X into the union
of zero-dimensional subspaces. Now assume that n 2 !; dim X  n and we have
proved that every second countable space of dimension m < n is the union of at
most .m C 1/-many zero-dimensional subspaces.
Let Bd.A/ D AnA for any A  X and call a set U 2 .X / adequate if we have
the inequality dim.Bd.U //  n  1. It turns out that
(4) the adequate sets form a base in X .
Indeed, fix a point x 2 X and W 2 .x; X /. By Fact 3 there exist disjoint open
sets U and V such that x 2 U; X nW  V and dim.X n.U [ V //  n  1. Observe
that U \ V D ; and hence Bd.U /  C D X n.U [ V /; an immediate consequence
is that dim.Bd.U //  dim C  n  1 (see Problem 155), i.e., the set U is adequate.
Since also x 2 U  W , the property (4) is proved.
Next apply Claim of S.088 to conclude that there exists a countable base B in the
space X such that every set B 2 B is adequate. The family C D fBd.B/ WS
B 2 Bg
is countable while every C 2 C is closed and dim C  n  1. Let F D C and
apply Problem 150 to convince ourselves that dim F  n  1.
Let X0 D X nF ; the family B0 D BjX0 is a base in X0 . It is easy to see that
B \X0 D B \X0 for every B 2 B, so all elements of B0 are clopen in X0 and hence
X0 is zero-dimensional. By the induction hypothesis, there are zero-dimensional
spaces X1 ; : : : ; Xn such that F D X1 [ : : : [ Xn . Thus X D X0 [ : : : [ Xn is
the promised decomposition of X , so we accomplished our inductive step proving
necessity for all n 2 ! and hence our solution is complete.
V.165. Let S D fXt ; st W t; s 2 T g be an inverse
Q system of Hausdorff topological
spaces. Prove that the set lim S is closed in fXt W t 2 T g. Therefore the limit of

an inverse system of Hausdorff compact spaces is a Hausdorff compact space.
Q
Solution. Let X D t2T Xt ; observe first that the Hausdorff property is preserved
by products and subspaces, so the space Y D lim S is Hausdorff. To prove that Y


is closed in X take any point x 2 X nY . Since x is not a thread, there are s; t 2 T


such that s < t and st .x.t // x.s/. Take disjoint sets U; V 2 .Xs / such that
st .x.t // 2 U and x.s/ 2 V . By continuity of st there is a set W 2 .x.t /; Xt / for
which st .W /  U . The set G D fy 2 X W y.t / 2 W and y.s/ 2 V g is open in X .
If y 2 G then y.t / 2 W and hence st .y.t // 2 U , which, together with y.s/ 2 V
shows that st .y.t // y.s/. Thus every point x 2 X nY has an open neighborhood
G  X nY , so X nY is open in X and hence Y is a closed subset of X . If every Xt
is compact then X is also compact, so Y has to be compact as well.
V.166. Suppose that S D fXt ; st W t; s 2 T g is an inverse system in which Xt is a
nonempty compact Hausdorff space for each t . Prove that lim S ;.

Q
Solution. The space X D t2T Xt is compact. For any t 2 T consider the set
At D fs 2 T W s  t g and say that a point x 2 X is a thread on At if vu .x.u// D
x.v/ for any u; v 2 At with v  u. Denote by Ft the set of all threads on At for
each t 2 T .

204

2 Solutions of problems 001500

Fix t 2 T and take any point x 2 X nFt . Since x is not a thread on At , there
are u; v 2 At such that v < u and vu .x.u// x.v/. Take disjoint sets U; V 2
.Xv / such that vu .x.u// 2 U and x.v/ 2 V . By continuity of vu there is a set
W 2 .x.u/; Xu / for which vu .W /  U . The set G D fy 2 X W y.u/ 2 W and
y.v/ 2 V g is open in X . If y 2 G then y.u/ 2 W and hence vu .y.u// 2 U , which,
together with y.v/ 2 V shows that vu .y.u// y.v/. Thus every point x 2 X nFt
has an open neighborhood G  X nFt , so Ft is a closed subset of X .
Given t 2 T fix a point xs 2 Xs for any s At ; take a point y 2 Xt and let
xs D st .y/ for any s 2 At . Letting x.s/ D xs for any s 2 T we obtain a point
x 2 X and it follows from the definition of projections of an inverse system that
vu .x.u// D vu .ut .y// D vt .y/ D x.v/ for any u; v 2 At with v  u, so x 2 Ft .
This proves that
(1) the set Ft is nonempty and closed in X for any t 2 T .
If t1 ; : : : ; tn 2 T then there exists s 2 T such that ti Ts for each i  n. It is
straightforward that Fs  Fti for every i  n, so Fs  in Fti and therefore
T
(2)
F 0 ; for any finite family F 0  F D fFt W t 2 T g.
The properties (1) and (2) show
T that F is a centered family of closed
T subsets of
a compact space X . Therefore F ;; it is an easy exercise that F D lim S,


so lim S ;.


V.167. Let S D fXt ; st W t; s 2 T g be an inverse system. Suppose that, for a space
Y , a continuous map ft W Y ! Xt is given for every t 2 T and, besides, st ft D fs
for any s; t 2 T with s  t . Prove that the diagonal product f D t2T ft maps Y
continuously into lim S.

Q
Solution. Let X D t2T Xt ; recall that the diagonal product f W Y ! X is
defined by letting f .y/.t / D ft .y/ for any y 2 Y and t 2 T . If pt W X ! Xt is the
natural projection then pt f D ft for any t 2 T , so f is continuous by TFS-102.
To see that f .Y /  lim S fix a point y 2 Y . For any s; t 2 T with s  t we


have st .f .y/.t // D st .ft .y// D fs .y/ D f .y/.s/, so the point f .y/ is a thread
and hence f .y/ 2 lim S. Thus f is a continuous map from Y to lim S.


V.168. Let X be a topological space. Suppose that, for a nonempty directed set T ,
a subspace Xt  X is given for each t 2 T in such a way that Xt  Xs whenever
s  t . Given s; t 2 T with s  t , let st .x/ D x for each x 2 Xt . Prove that
the inverse systemTS D fXt ; st W t; s 2 T g is well defined and the limit of S is
homeomorphic to fXt W t 2 T g.
Solution. It is evident that every map st is continuous and tt D idXt . Given s 
t  u we have st .tu .x// D st .x/ D x D su .x/ for any x 2 Xu , so the inverse
system S is well
Q defined.
LetTP D t2T Xt and denote the limit of S by L; let ht .x/ D x for any x 2
Y D fXt W t 2 T g and t 2 T . Then ht W Y ! Xt is a continuous map and it is
straightforward that st ht D hs for any s; t 2 T with s  t . Consequently, the
diagonal product h D t2T ht maps Y continuously into L (see Problem 167).

2 Solutions of problems 001500

205

If x; y are distinct points of Y then ht .x/ D x y D ht .y/ for any t 2 T ,


so h.x/ h.y/ and hence h is an injection. Given a 2 L take any s; t 2 T and
choose u 2 T such that s  u and t  u. Then a.u/ D tu .a.u// D a.t / and
a.u/ D su .a.u// D a.s/ which shows that a.s/ D a.t /, i.e., there is x 2 X such
that a.t / D x for all t 2 T . Since a.t / 2 Xt , we must have x 2 Xt for all t 2 T and
hence x 2 Y . It is immediate that h.x/ D a, so h W Y ! L is an onto map whence
h is a condensation.
Finally, fix an index s 2 T and let ps W P ! Xs be the natural projection. The
map p D ps jL W L ! Xs is continuous. We already saw that, for every a 2 L, we
have a.s/ D a.t / 2 Xt for any t 2 T , so p.a/ D a.s/ 2 Y . Therefore p maps
continuously the space L in Y ; it is easy to see that p is the
T inverse of h, so h is a
homeomorphism between the spaces L D lim S and Y D t2T Xt .


V.169. Give an example of an inverse sequence S D fXn ; mn W n; m 2 !g such


that every Xn is a nonempty second countable Tychonoff space while lim S D ;.


Solution. The space Xn D .n; C1/  R is second countable, Tychonoff and


nonempty for any n 2 !. Given m; n 2 ! with m  n let mn .x/ D x for every
x 2 Xn ; this defines a continuous map mn W Xn ! Xm . Since Xn  Xm whenever
m  n, we can apply Problem 168 to see that the inverseTsequence S D fXn ; mn W
n; m 2 !g is well defined and lim S is homeomorphic to n2! Xn D ;, so the limit

of S is empty.
V.170. Given an inverse system S D fXt ; st W t; s 2 T g of topological spaces,
prove that the family B D ft1 .U / W t 2 T; U 2 .Xt /g is a base of the space
L Dlim S. Here t W L ! Xt is the limit projection for every t 2 T .


Solution.
Q It is evident that all elements of the family B are open in L. For the space
X D t2T Xt let pt W X ! Xt be the natural projection for every t 2 T . If x 2 L
and G 2 .x; L/ then there is U 2 .X / such that U \ L D G. By definition of the
product
T topology there exist t1 ; : : : ; tn 2 T and Ui 2 .Xti / for all i  n such that
x 2 in pt1
.Ui /  U .
i
The set T being directed we can find an index t 2 T such that ti  t for all
i  n. It follows from x 2 L that tti .x.t // D x.ti / 2 Ui , so we canTfind a set
Wi 2 .x.t /; Xt / such that tti .Wi /  Ui for every i  n. The set W D in Wi is
an open neighborhood of x.t / in Xt and tti .W /  Ui for each i  n.
The set O D t1 .W / D pt1 .W / \ L is an element of B and x 2 O. If
y 2 O then y.t / 2 W and y 2 L, so y.ti / D tti .y.t // 2 Ui for all i  n. As a
T
consequence, y 2 in pt1
.Ui / \ L  U \ L D G and therefore O  G. We
i
proved that, for any x 2 L and G 2 .x; L/ there is O 2 B such that x 2 O  G,
so B is a base in L.
V.171. Suppose that S D fXt ; st W t; s 2 T g is an inverse system of topological
spaces. Prove that, for any closed set F  lim S, the subspace F is the limit of the


inverse system SF D ft .F /; st jt .F / W t; s 2 T g. Here t W lim S ! Xt is the



limit projection for every t 2 T .

206

2 Solutions of problems 001500

Solution. Let L D lim S; if t 2 T and x 2 t .F / then there is y 2 F with




t .y/ D y.t / D x. For any s  t we have st .x/ D st .y.t // D y.s/ D s .y/ 2
s .F /, so st .t .F //  s .F / and hence the inverse system SF is well defined.
If LF D lim SF then every element of LF has to be a thread of S, so LF  L.


Given a point x 2 F we have t .x/ 2 t .F / for any t 2 T , so x is also a thread of


the inverse system SF and therefore x 2 LF ; this proves that F  LF .
Finally, take any point x 2 LnF ; the set F being closed in L we can apply
Problem 170 to find t 2 T and U 2 .Xt / such that x 2 t1 .U /  LnF . This
implies that U \ t .F / D ; and hence t .x/ t .F / which shows that x LF ;
therefore F D LF D lim SF .


V.172. Given an inverse system S D fXt ; st W t; s 2 T g and a cofinal set T 0  T ,


prove that the limit of the inverse system S 0 D fXt ; st W t; s 2 T 0 g is homeomorphic
to lim S.

Q
Q
Solution. Let X D t2T Xt and X 0 D t2T 0 Xt ; denote the limit of S by L and
the limit of S 0 by L0 . We will also need the natural projections pt W X ! Xt and
qs W X 0 ! Xs for any t 2 T and s 2 T 0 .
For any t 2 T 0 let w.t / D t ; if t 2 T nT 0 then fix an index w.t / 2 T 0 such that
t  w.t /. Given a point x 2 X let .x/ D xjT 0 ; the resulting map  W X ! X 0 is
continuous. It is evident that .x/ is a thread of S 0 if x is a thread of S, so .L/ 
L0 . If x and y are distinct points of L then there is t 2 T such that x.t / y.t /.
w.t/
w.t/
If x.w.t // D y.w.t // then x.t / D t .x.w.t /// D t .y.w.t /// D y.t /; this
contradiction shows that x.w.t // y.w.t // and hence .x/ .y/. Thus the map
h D jL W L ! L0 is injective.
w.t/
Take any point y 2 L0 and let '.y/.t / D t .y.w.t /// for any t 2 T . This
0
gives us a map ' W L ! X such that .'.y// D y for any y 2 L0 . It turns out
that '.L0 /  L; to see it take any y 2 L0 , let x D '.y/ and fix s; t 2 T with
s  t . There exists v 2 T 0 such that w.s/  v and w.t /  v. Since y is a thread of
v
v
S 0 , we have y.w.t // D w.t/
.y.v// and y.w.s// D w.s/
.y.v// which shows that
w.t/

st .x.t // D st .t

v
.w.t/
.y.v//// D sv .y.v//, which together with the equalities

v
x.s/ D sw.s/ .y.w.s/// D sw.s/ .w.s/
.y.v/// D sv .y.v//

shows that st .x.t // D x.s/ and hence x 2 L.


Thus we can consider that ' W L0 ! L; since '.h.x// D x for any x 2 L and
h.'.y// D y for every y 2 L0 , the maps h and ' are mutually inverse bijections.
We already saw that h is continuous; fix t 2 T and take any y 2 L0 . The equality
w.t/
w.t/
'.y/.t / D t .y.w.t /// implies that .pt '/.y/ D .t qw.t/ /.y/ for any
w.t/
y 2 L0 , i.e., the map pt ' D t qw.t/ is continuous for any t 2 T , so ' is
continuous by TFS-102. Therefore h W L ! L0 is a homeomorphism.

2 Solutions of problems 001500

207

V.173. Suppose that an inverse system S D fXt ; st W t; s 2 T g consists of compact


Hausdorff spaces Xt and all projections st are onto. Prove that all limit projections
t are also surjective maps.
Q
Solution. The space X D t2T Xt is compact; fix t 2 T and a point z 2 Xt . For
any s 2 T consider the set As D fu 2 T W u  sg and say that a point x 2 X is
a z-thread on As if x.t / D z and vu .x.u// D x.v/ for any u; v 2 As with v  u.
Denote by Fs the set of all z-threads on As for each s 2 T .
Fix an index s 2 T and take any point x 2 X nFs . If x.t / z then the set
G D fy 2 X W y.t / zg is open in X and x 2 G  X nFs . If x.t / D z
then there are u; v 2 As such that v < u and vu .x.u// x.v/. Take disjoint sets
U; V 2 .Xv / such that vu .x.u// 2 U and x.v/ 2 V . By continuity of vu there is a
set W 2 .x.u/; Xu / for which vu .W /  U . The set G D fy 2 X W y.u/ 2 W and
y.v/ 2 V g is open in X . If y 2 G then y.u/ 2 W and hence vu .y.u// 2 U , which,
together with y.v/ 2 V shows that vu .y.u// y.v/. Thus every point x 2 X nFs
has an open neighborhood G  X nFs , so Fs is a closed subset of X .
Given s 2 T choose s 0 2 T such that t  s 0 and s  s 0 . Fix a point xu 2 Xu
0
for any u As 0 ; since the map ts is surjective, there exists y 2 Xs 0 such that
0
0
ts .y/ D z. Let xu D us .y/ for any u 2 As 0 . Letting x.u/ D xu for any u 2 T we
0
obtain a point x 2 X such that x.t / D ts .y/ D z; it follows from the definition of
0
0
projections of an inverse system that vu .x.u// D vu .us .y// D vs .y/ D x.v/ for
any u; v 2 As 0 with v  u, so x 2 Fs 0 . Since also Fs 0  Fs , we proved that x 2 Fs
and therefore
(1) the set Fs is nonempty and closed in X for any s 2 T .
If s1 ; : : : ; sn 2 T then there exists s 2 T such that si Ts for each i  n. It is
straightforward
that Fs  Fsi for every i  n, so Fs  in Fsi and hence
T 0
(2)
F ; for any finite family F 0  F D fFs W s 2 T g.
The properties (1) andT(2) show that F is a centered
T family of closed subsets of a
compact space X . Thus F ;; take a point x 2 F. It is an easy exercise that
x 2 L D lim S and t .x/ D x.t / D z; the point z 2 Xt was chosen arbitrarily, so


t .L/ D Xt , i.e., the map t is surjective for any t 2 T .


V.174. Suppose that n 2 ! and S D fXt ; st W t; s 2 T g is an inverse system in
which all spaces Xt are compact and Hausdorff. Knowing also that dim Xt  n for
each t 2 T prove that dim.lim S/  n.


Solution. The space L D lim S is compact (see Problem 165) and hence normal;

let t W L ! Xt be the limit projection for any t 2 T . Take a finite open cover U of
the space L; it follows from Problem 170 that, for every x 2 L there exists t .x/ 2 T
1
and Ux 2 .Xt.x/ / such that x 2 Vx D t.x/
.Ux / and the set Vx is contained in some
element of U .
S
Take a finite set A  L for which L D fVx W x 2 Ag; the set T being
directed there exists t 2 T such that t .x/  t for any x 2 A. If x 2 A then the
t
set Ox D .t.x/
/1 .Ux / is open in Xt . It follows from the definition of the limit of

208

2 Solutions of problems 001500

t
an inverse system that t.x/
t D t.x/ and hence t1 .Ox / D Vx for all x 2 A.
S
S 1
Therefore L D ft .Ox / W x 2 Ag and hence t .L/  fOx W x 2 Ag.
The set t .L/ being closed in Xt , we have dim.t .L//  dim Xt  n (see
Problem 146), so there is a finite closed refinement F of the cover fOx \ t .L/ W
x 2 Ag of the space t .L/ such that ord.F/  n C 1. It is straightforward that
ft1 .F / W F 2 Fg is a finite closed refinement of U of order  n C 1, so we can
apply Problem 145 to conclude that dim L  n.

V.175. Suppose that n 2 ! and S D fXl ; ml W l; m 2 !g is an inverse sequence


in which every Xl is a Lindelf -space. Knowing additionally that dim Xl  n for
every l 2 ! prove that dim.lim S/  n.


Solution. The space L D lim S is Lindelf being closed in a Lindelf space



Q
m2! Xm (see Problem 165 and SFFS-256); let Kl D Xl for every l 2 !. For
l
any m; l 2 ! with m  l there exists a continuous map qm
W Kl ! Km such that
m
m
l
l
l
qm jXl D m . If k  m  l then .qk qm /jXl D k ml D kl D qkl jXl ;
l
since Xl is dense in Kl , we conclude that qkm qm
D qkl by Fact 0 of S.351.
l
Thus K D fKl ; qm ; m; l 2 !g is an inverse system of compact spaces with
dim Kl D dim Xl  n for any l 2 ! (see Problem 147). If K D lim K then

dim K  n by Problem 174.
Every thread of S is also a thread of K, so L  K. Take any U 2 .L; K/ and fix
a set Vx 2 .x; K/ such that V x  U for everySx 2 L. The space L being Lindelf
there is a countable
A  L for which L  fVx W x 2 Ag. As a consequence,
S
L  H D fV x W x 2 Ag  U which shows that H is an F -subset of K
such that L  H  U . This makes it possible to apply Problem 154 to convince
ourselves that dim L  n.
V.176. Prove that if X is second countable and A  Cp .X / is countable then there
exists a countable QS-algebra E  Cp .X / such that A  E.
Solution. Fix a countable base B in the space X and consider the family
P D f.U; V / W U; V 2 B and U  V gI
for every P D .U; V / 2 P choose a function fP 2 Cp .X / such that fP .U /  f1g
and fP .X nV /  f0g. It is evident that the set B D A [ ffP W P 2 Pg is countable,
so C D ff1 : : : fn W n 2 N; fi 2 B for all i  ng is also countable and hence
the set E D fp1 g1 C : : : C pm gm W m 2 N; pi 2 Q and gi 2 C for all i  mg
is countable as well. It is straightforward that f; g 2 E and q 2 Q imply that
f C g 2 E; f g 2 E and qf 2 E. Given a point x 2 X and a closed set F  X
with x F there exists V 2 B such that x 2 V  X nF . Pick U 2 B for which
x 2 U  U  V ; then P D .U; V / 2 P and hence f D fP 2 E. Then f .x/ D 1
and f .F /  f0g, so E is a countable QS-algebra in Cp .X / such that A  E.

2 Solutions of problems 001500

209

V.177. Let K D fx0 ; : : : ; xn g  X be a finite subset of a space X . Suppose that


U 2 .K; X / and q0 ; : : : ; qn 2 Q. Prove that, for any QS-algebra E on a space X ,
there is f 2 E such that f .X nU /  f0g and f .xi / D qi for each i  n.
Solution. The set Fi D .X nU / [ .Knfxi g/ is closed in X and xi Fi , so there
exists fi 2 E such that fi .xi / D 1 and fi .Fi /  f0g for each i  n. It is immediate
that the function f D q0 f0 C : : : C qn fn 2 E is as promised.
V.178. Given second countable Tychonoff spaces X and Y , suppose that some QSalgebras E.X / and E.Y / are chosen in Cp .X / and Cp .Y / respectively. Prove that,
if E.X / is uniformly homeomorphic to E.Y / then X is representable as a countable
union of closed subspaces each one of which embeds in Y .
Solution. Given a set A the family Fin.A/ consists of all finite subsets of A; for
each n 2 ! let A
n D fB 2 Fin.A/ W jBj D ng. If f is a function then dom.f / is
its domain; for any space Z the function 0Z 2 Cp .Z/ is defined by 0Z .z/ D 0 for
all z 2 Z.
Choose a uniformly continuous isomorphism T W E.X / ! E.Y /; it is evident
that 0X 2 E.X / and 0Y 2 E.Y /, so there is no loss of generality to assume that
T .0X / D 0Y . For every x 2 X let ax .f / D f .x/ for all f 2 E.X /; if y 2 Y
then by .f / D f .y/ for all f 2 E.Y /. For any K 2 Fin.X / and " > 0 the set
OX .K; "/ D ff 2 E.X / W f .K/  ."; "/g is an open neighborhood of 0X and
the family fOX .K; "/ W K 2 Fin.X / and " > 0g is a local base of the space E.X /
at the point 0X .
Fact 1. If x 2 X then ax W E.X / ! R is a uniformly continuous unbounded
map; therefore the map x D ax T 1 W E.Y / ! R is uniformly continuous and
unbounded. Analogously, by W E.Y / ! R is a uniformly continuous unbounded
map and hence the map y D by T W E.X / ! R is also uniformly continuous
and unbounded for every y 2 Y .
Proof. If we let '.x/.f / D f .x/ for any f 2 Cp .X / then ' W Cp .X / ! R is linear
and continuous; an evident consequence is that ' is uniformly continuous. Therefore
ax D 'jE.X / is uniformly continuous. Analogously, the map by W E.Y / ! R
is uniformly continuous. It follows from Problem 177 that, for any n 2 ! there
is a function f 2 E.X / such that f .x/ D n. This implies ax .f / D n, so
ax is unbounded and hence x is also uniformly continuous and unbounded. The
analogous statement being also true for y , Fact 1 is proved.
Fact 2. Given a point y 2 Y and a finite subset K of the space X let
u.y; K/ D supfjy .f /  y .g/j W f; g 2 E.X / and jf .x/  g.x/j
< 1 for every x 2 Kg:
Then the family E.y/ D fK 2 Fin.X / W u.y; K/ < 1g is nonempty. Analogously,
let v.x; L/ D supfjx .f /  x .g/j W f; g 2 E.Y / and jf .y/  g.y/j < 1 for all
y 2 Lg for any x 2 X and finite L  Y . Then the family D.x/ D fL 2 Fin.Y / W
v.x; L/ < 1g is nonempty.

210

2 Solutions of problems 001500

Proof. It follows from uniform continuity of y that there is a finite set K  X


and " > 0 such that f  g 2 OX .K; "/ implies jy .f /  y .g/j < 1. Fix n 2
N with n" > 1 and take any f; g 2 E.X / such that jf .x/  g.x/j < 1 for all
x 2 K. The function hi D f C ni .g  f / belongs to E.X / for every i  n and
it is straightforward that jhiC1 .x/  hi .x/j < n1 < " for all x 2 K and hence
jy .hiC1 / P
 y .hi /j < 1 for all i < n. Now, h0 D f and hn D g, so jy .g/ 
y .f /j  i<n jy .hiC1 /  y .hi /j < n which shows that u.y; K/  n and hence
K 2 E.y/. An analogous reasoning shows that D.x/ ;, so Fact 2 is proved.
Fact 3. For any point y 2 Y the empty set does not belong to E.y/; besides, if
K0 ; K1 2 E.y/ then u.y; K0 \ K1 /  u.y; K0 / C u.y; K1 / and hence K0 \ K1 2
E.y/. Analogously, if x 2 X then all elements of D.x/ are nonempty and L0 \L1 2
D.x/ whenever L0 ; L1 2 D.x/; besides, v.x; L0 \ L1 /  v.x; L0 / C v.x; L1 /.
Proof. Observe that OX .;; 1/ D E.X /; since y is unbounded on E.X / by Fact 1,
the set fjy .f /j D jy .f /  y .0X /j W f 2 E.X /g is unbounded so u.y; ;/ D 1
and hence ; E.y/. Now, if K0 ; K1 2 E.y/ then let K D K0 \ K1 and consider
any f; g 2 E.X / such that jf .x/  g.x/j < 1 for all x 2 K.
The set Ix D .f .x/  1; f .x/ C 1/ \ .g.x/  1; g.x/ C 1/ is open and nonempty,
so we can choose a rational number qx 2 Ix for any x 2 K. For every x 2 K0 nK
choose a rational number qx 2 .f .x/  1; f .x/ C 1/; if x 2 K1 nK then we can
pick qx 2 .g.x/  1; g.x/ C 1/ \ Q. Apply Problem 177 to construct a function
h 2 E.X / for which h.x/ D qx for every x 2 K0 [ K1 ; it is immediate that f  h 2
OX .K0 ; 1/ and h  g 2 OX .K1 ; 1/. Therefore jy .f /  y .h/j  u.y; K0 / and
jy .h/  y .g/j  u.y; K1 / whence jy .f /  y .g/j  u.y; K0 / C u.y; K1 /. This
shows that u.y; K/  u.y; K0 / C u.y; K1 /, so we proved all statements formulated
for E.y/. An analogous reasoning settles the case of D.x/, so Fact 3 is proved.
Fact 4. For any y 2 Y there exists a unique minimal element K.y/ in the family
E.y/ with respect to inclusion; let u.y/ D u.y; K.y//. Analogously, there is a
unique minimal element L.x/ in the family D.x/; let v.x/ D v.x; L.x// for each
x 2 X.
Proof. The elements of E.y/ are finite, so there exists a minimal element K 2 E.y/;
if K0 is another minimal element of E.y/ then K \ K0 is strictly smaller than K;
since K \ K0 2 E.y/ by Fact 3, we have a contradiction with minimality of K.
Therefore there is a unique minimal element of E.y/; the same argument proves the
relevant fact for D.x/, so Fact 4 is proved.
Fact 5. For any p; q 2 N let Y.p; q/ D fy 2 Y W there exists K 2 E.y/ such that
jKj  q and u.y; K/  pg; we will also need the set
X.p; q/ D fx 2 X W there is L 2 D.x/ such that jLj  q and v.x; L/  pg:
Given p 2 N consider the sets M.p; 1/ D Y.p; 1/ and N.p; 1/ D X.p; 1/;
for every natural q > 1 let M.p;S
q/ D Y.p; q/nY.2p; q  1/ andSN.p; q/ D
X.p; q/nX.2p; q  1/. If M.p/ D fM.p; q/ W q 2 Ng and N.p/ D fN.p; q/ W
q 2 Ng then

2 Solutions of problems 001500

211

(i) for any


Sy 2 Y and p 2 N such that u.y/  p we have y 2 M.p/; in particular,
Y D fM.p/ W p 2 Ng.
(ii) For any
Sx 2 X and p 2 N such that v.x/  p we have y 2 N.p/; in particular,
X D fN.p/ W p 2 Ng.
(iii) If we have p 2 N and distinct q0 ; q1 2 N then M.p; q0 / \ M.p; q1 / D ; and
N.p; q0 / \ N.p; q1 / D ;.
Proof. If q D jK.y/j then u.y/ D u.y; K.y//  p, so y 2 Y.p; q/. The set
K.y/ being minimal in E.y/, the point y does not belong to Y.2p; q  1/, so y 2
M.p; q/  M.p/; this proves (i). An identical argument shows that (ii) is also true.
Now assume that p 2 N and take distinct q0 ; q1 2 N; there is no loss of generality
to assume that q0 < q1 and hence q0  q1 1. It follows from the relevant definitions
that M.p; q0 /  Y.p; q0 /  Y.p; q1  1/  Y.2p; q1  1/  Y nM.p; q1 / and
therefore M.p; q0 / \ M.p; q1 / D ;. Analogously, N.p; q0 / \ N.p; q1 / D ;, so we
settled (iii) and hence Fact 5 is proved.
Fact 6. Suppose that p 2 N and y 2 M.p/; denote by q.p/ the unique natural
number such that y 2 M.p; q.p//. Then there exists a unique set Kp .y/  X such
that jKp .y/j D q.p/ and u.y; Kp .y//  p. Analogously, for any x 2 L.p/ there is
a unique set Lp .x/  Y for which jLp .x/j D q and v.x; Lp .x//  p (here q 2 N
is the unique number for which x 2 L.p; q/).
Proof. Observe that q.p/ is unique because the family fM.p; q/ W q 2 Ng is disjoint
by Fact 5. Let q D q.p/; by the definition of Y.p; q/ there exists a set K  X such
that jKj  q and a.y; K/  p. It follows from y Y.2p; q  1/  Y.p; q  1/ that
jKj D q. Now, if K 0 K; a.y; K 0 /  p and jK 0 j D q then K 00 D K \ K 0 has at
most .q1/-many elements and a.y; K 00 /  2p (see Fact 3) which is a contradiction
with y Y.2p; q  1/. This proves uniqueness of the set Kp .y/ D K. The proof of
existence and uniqueness of the set Lp .x/ is analogous, so Fact 6 is proved.
Fact 7. Given p 2 N assume that fyn W n 2 !g  Y is a sequence which converges
to a point y 2 M.p/; if Qn  X is a finite set such that u.yn ; Qn /  p for every
n 2 ! then for every U 2 .X / with U \ Kp .y/ ; there exists m 2 ! such that
U \ Qn ; for all n  m. Analogously, if fxn W n 2 !g  Y is a sequence which
converges to a point x 2 N.p/ and Rn  X is a finite set such that u.yn ; Rn /  p
for all n 2 ! then for every V 2 .Y / with V \ Lp .x/ ; there exists m 2 !
such that V \ Rn ; for all n  m.
Proof. If the set A D fn 2 ! W Qn \ U D ;g is infinite then we can pass to the
subsequence fyn W n 2 Ag to see that we can assume, without loss of generality, that
Qn \ U D ; for all n 2 !. Fix q 2 N such that y 2 M.p; q/; then jKp .y/j D q.
If K D Kp .y/nU then jKj < q, so there exist functions f; g 2 E.X / such that
jf .x/  g.x/j < 1 for all x 2 K while jy .f /  y .g/j > 2p. Using Problem 177
it is easy to find a function h 2 E.X / such that hj.X nU / D f j.X nU / and jh.x/ 
g.x/j < 1 for all x 2 U \ Kp .y/. Then jh.x/  g.x/j < 1 for all x 2 Kp .y/, so
jy .h/  y .g/j  p. An immediate consequence is that jy .h/  y .f /j > p.

212

2 Solutions of problems 001500

On the other hand, it follows from hjQn D f jQn that


. / jyn .h/  yn .f /j  p for all n 2 !.
The function T .h/ is continuous on Y , so the sequence fT .h/.yn / W n 2 !g
converges to T .h/.y/ D y .h/. Since yn .h/ D T .h/.yn / for every n 2 !, the
sequence fyn .h/ W n 2 !g converges to y .h/. Analogously, yn .f / ! y .f /,
so we can apply . / to conclude that jy .h/  y .f /j  p; this contradiction
demonstrates that only finitely many elements of the sequence fQn W n 2 !g miss
the set U . An analogous reasoning shows that only finitely many elements of the
sequence fRn W n 2 !g miss the set V , so Fact 7 is proved.
Fact 8. For any p; q 2 N the map Kp W M.p; q/ ! X
q is lower semicontinuous.
Analogously, the map Lp W N.p; q/ ! Y
q is lower semicontinuous.
Proof. Fix any U 2 .X /; if the set V D fy 2 M.p; q/ W Kp .y/ \ U ;g is
not open in M.p; q/ then there is a sequence fyn W n 2 !g  M.p; q/nV which
converges to some y 2 V . Since Kp .yn / \ U D ; and u.yn ; Kp .yn //  p for every
n 2 !, we obtained a contradiction with Fact 7. Therefore the map Kp is lower
semicontinuous. An analogous reasoning shows that Lp is lower semicontinuous as
well, so Fact 8 is proved.
Fact 9. The set Y.p; q/ is closed in Y and the set X.p; q/ is closed in X for any
p; q 2 N.
Proof. Suppose that yn 2 Y.p; q/ for all n 2 ! and yn ! y. We will pass several
times to a subsequence of the sequence S D fyn W n 2 !g; since our aim is to prove
that y 2 Y.p; q/, at each step we will identify the obtained subsequence with S
considering that all elements of S have the property we have found in a subsequence.
Fix a set Qn  X such that jQn j  q and u.yn ; Qn /  p for every n 2 !. Passing
to a subsequence of S if necessary, we can assume that jQn j D k  q for all n 2 !.
Next, use Fact 2 of U.337 to choose an infinite A  ! for which there is a
set D D fd1 ; : : : ; dr g  X such that Qn \ Qm D D for distinct n; m 2 A
(observe that it is possible that r D 0 in which case D D ;). According to
the above mentioned politics we can consider that, for any i 2 !, we have
i
Qi D fd1 ; : : : ; dr ; a1i ; : : : ; akr
g and the family fQi nD W i 2 !g is disjoint.
An evident property of metric spaces is that any sequence contains either a
convergent subsequence or an infinite closed discrete subspace. This makes it
possible to pass to a subsequence of S once more to guarantee that, for any
j 2 f1; : : : ; k  rg, the sequence Sj D faji W i 2 !g is either convergent
or constitutes a closed discrete subspace of X . If Sj is convergent then denote
by xj its limit. Renumbering every Qi if necessary we can assume that Qi D
i
i
i
fd1 ; : : : ; dr ; a1i ; : : : ; ali ; alC1
; : : : ; akr
g while the set A D falCj
W i 2 !; 1 
j  k  r  lg is closed and discrete in X and the sequence Sj converges to xj for
any j 2 f1; : : : ; lg.
Consider the set Q D fd1 ; : : : ; dr ; x1 ; : : : ; xl g; since jQj  q, it suffices to show
that u.y; Q/  p. To do this, fix f0 ; g0 2 E.X / such that jf0 .x/  g0 .x/j < 1 for

2 Solutions of problems 001500

213

every x 2 Q. Given an arbitrary " > 0 there exists a finite set E  Q and 2 .0; 1/
such that f  g 2 OX .E; / implies jy .f /  y .g/j < ". The set A being closed
and discrete in X we can find U 2 .E; X / such that U \ A is a finite set.
Apply Problem 177 to find h 2 E.X / such that h.E/  f1g and h.X nU /  f0g;
consider the functions f1 D hf0 and g1 D hg0 . It follows from f0 jE D f1 jE and
g0 jE D g1 jE that
. / jy .f0 /  y .f1 /j < " and jy .g0 /  y .g1 /j < ".
We also have f1 jQ D f0 jQ and g1 jQ D g0 jQ, so jf1 .x/  g1 .x/j < 1 for every
x 2 Q. Therefore W D fx 2 X W jf1 .x/  g1 .x/j < 1g is an open neighborhood of
the set Q and so is the set W 0 D W \ U . There is m 2 ! such that, for all i  m,
we have aji 2 W 0 for all j 2 f1; : : : ; lg and aji U for all j 2 fl C 1; : : : ; k  rg.
As a consequence, for each i  m we have jf1 .x/  g1 .x/j < 1 for every point
i
i
x 2 D [fa1i ; : : : ; ali g; besides, f1 .x/ D g1 .x/ D 0 whenever x 2 falC1
; : : : ; akr
g,
so jf1 .x/  g1 .x/j < 1 for all x 2 Qi .
By the choice of the sets Qi , we have jyi .f1 /  yi .g1 /j  p for all i  m.
We already saw that yi .f1 / ! y .f1 / and yi .g1 / ! y .g1 / as i ! 1. Passing
to the limit in the last inequality, we conclude that jy .f1 /  y .g1 /j  p. This,
together with . /, implies that jy .f0 /  y .g0 /j  p C 2". The number " > 0
was taken arbitrarily so jy .f0 /  y .g0 /j  p and hence y 2 Y.p; q/, i.e., we
established that the set Y.p; q/ is closed in X . An analogous reasoning shows that
X.p; q/ is closed in Y , so Fact 9 is proved.
Returning to our solution call a set A  Y adequate if it is closed in Y and
embeds in X ; the set A is called  -adequate if it is representable as the countable
union of adequate sets. Given any p; q 2 N, say that a set A  Y is .p; q/-small
if A  M.p; q/ and there exist continuous maps f1 ; : : : ; fq W A ! X such that
Kp .y/ D ff1 .y/; : : : ; fq .y/g for all y 2 A. Analogously, a set B  X will be
called .p; q/-small if B  N.p; q/ and there are continuous maps g1 ; : : : ; gq W
B ! Y such that Lp .x/ D fg1 .x/; : : : ; gq .x/g for all x 2 B.
Given p; q 2 N, we can apply Fact 2 of V.080 together with Fact 8 to see that
there
S exists a family fUn W n 2 !g of open subsets of M.p; q/ such that M.p; q/ D
n2! Un and every Un is .p; q/-small. By Fact 9 each M.p; q/ is an F -set in Y , so
there
S exists a family fFn .p; q/ W n 2 !g of closed subsets of Y such that M.p; q/ D
n2! Fn .p; q/ and every Fn .p; q/ is .p; q/-small. The family F D fFn .p; q/ W
n 2 !; p; q 2 Ng is a countable closed cover of Y such that every F 2 F is
.p; q/-small for some p; q 2 N; let SY .F / D ff1 ; : : : ; fq g be the respective set
of continuous functions from F to X . Analogously, there exists a countable closed
cover G of the space X such that every G 2 G is .p; q/-small for some p; q 2 N;
denote by SX .G/ the set fg1 ; : : : ; gq g which witnesses that G is .p; q/-small. Next,
observe that
S
(1) y 2 fL.x/ W x 2 K.y/g for any y 2 Y .
S
Let A D fL.x/ W x 2 K.y/g and assume that y A. Choose a number n 2 N
such that v.x/ < n for all x 2 K.y/ and apply Problem 177 to choose a function

214

2 Solutions of problems 001500

f 2 E.Y / such that f .y/ > nu.y/ and f .A/  f0g. It follows from f jL.x/ D
0Y jL.x/ that jx .f /  x .0Y /j D jx .f /j  v.x/ < n for any x 2 K.y/. If
g D T 1 .f / then jg.x/j D jx .f /j < n for every x 2 K.y/.
Consider the function hi D ni g for all i D 0; : : : ; n. Given any natural number
i < n we have jhiC1 .x/  hi .x/j D n1 jg.x/j < 1 for all points x 2 K.y/; therefore
jy .hiC1 /  y .hi /j  u.y/. Now, it follows from the equalities h0 D 0X and
P
hn D g that jy .g/j D jy .g/  y .0X /j  n1
iD0 jy .hiC1 /  y .hi /j  nu.y/.
Recalling that y .g/ D T .g/.y/ D f .y/ we conclude that jf .y/j  nu.y/; this
contradiction with our
Schoice of f shows that (1) is proved.
S
The sets SX D fSX .B/ W B 2 Gg and SY D fSY .A/ W A 2 Fg are
countable; our next step is to prove that
(2) the set Q.f; g/ D fy 2 dom.f / W f .y/ 2 dom.g/ and g.f .y// D yg is
adequate for any f 2 SY and g 2 SX .
Suppose that a sequence fyn W n 2 !g  Q.f; g/ converges to a point y 2 Y .
The set dom.f / being closed in Y , we have y 2 dom.f /; by continuity of f , the
sequence ff .yn / W n 2 !g  dom.g/ converges to the point f .y/. The set dom.g/
is closed in X , so f .y/ 2 dom.g/; the map g being continuous, the sequence
fg.f .yn // W n 2 !g D fyn W n 2 !g converges to g.f .y// whence y D g.f .y//
which proves that the set Q D Q.f; g/ is closed in Y . Note that g is the inverse of
f on the set f .Q/, so f jQ W Q ! f .Q/ is a homeomorphism and hence Q is,
indeed, an adequate subset of Y , i.e., (2) is proved.
The family F 0 D fF \ Q.f; g/ W F 2 F; f 2 SY and g 2 SX g consists of
countably many adequate sets (see (2)). Fix a point y 2 Y and take F 2 F such
that y 2 F ; the set F is .p; q/-small for some p; q 2 N, so SY .F / D ff1 ; : : : ; fq g
for some continuous functions f1 ; : : : ; fq from F to X .
The set K.y/ being the minimal element of the family E.y/, we have the
inclusion K.y/  Kp .y/ D ff1 .y/; : : : ; fq .y/g, so it follows from (1) that there is
i  q for which y 2 L.fi .y//. The family G being a cover of X we can pick G 2 G
with x D fi .y/ 2 G. By our choice of G, there are r; s 2 N such that G  N.r; s/
and SX .G/ D fg1 ; : : : ; gs g. The set L.x/ being the minimal element of D.x/, we
have L.x/  Lr .x/ D fg1 .x/; : : : ; gs .x/g. Thus y D gj .x/ D gj .fi .y// for some
j  s and therefore y 2 S
F \Q.fi ; gj / 2 F 0 . The point y 2 Y was taken arbitrarily,
so we proved that Y D F 0 is  -adequate; analogously, X is  -adequate, so our
solution is complete.
V.179. Given second countable Tychonoff spaces X and Y , suppose that some QSalgebras E.X / and E.Y / are chosen in Cp .X / and Cp .Y / respectively. Prove that,
if E.X / is uniformly homeomorphic to E.Y / then dim X D dim Y .
Solution. Let m D dim Y and apply Problem
178 to find a family F D fFn W
S
n 2 !g of closed subsets of X such that F D X and every Fn embeds in Y . If
m D 1 then dim X  m, so assume that m 2 !. It follows from Problem 155 that
dim Fn  m for all n 2 !. The space X being normal we can apply Problem 150
to see that dim X  m which proves that dim X  dim Y . Since the spaces X
and Y are in a symmetric situation, we also have dim Y  dim X and hence
dim X D dim Y .

2 Solutions of problems 001500

215

V.180. Suppose that X and Y are Tychonoff spaces such that Cp .X / is uniformly
homeomorphic to Cp .Y /. Prove that dim X D dim Y .
Solution. Given a space Z and P  Cp .Z/ let ePZ .z/.f / D f .z/ for any z 2 Z
and f 2 P . Then ZP
D fePZ .z/ W z 2 Zg  Cp .P / and ePZ W Z ! ZP
is a
Z
continuous map by TFS-166. If P  Q  Cp .Z/ then Q;P
W ZQ
! ZP
is
Z
Z
Z
the restriction map; it is evident that eP D Q;P eQ . Recall that if ' W Z ! T
is a continuous onto map then the dual map '  W Cp .T / ! Cp .Z/ is defined by
'  .f / D f ' for any f 2 Cp .T /.
If Z is a space and P  Cp .Z/ then .ePZ / W Cp .ZP
/ ! Cp .Z/ is an
embedding (see TFS-163) such that P  C Z; P
D .ePZ / .Cp .ZP
// (see
Fact 5 of U.086). Let dPZ W C Z; P
! Cp .ZP
/ be the inverse of .ePZ / , i.e.,
dPZ .f / D .ePZ /
1 .f / for every f 2 C Z; P
.
Fact 1. Given spaces Z and T suppose that ' W Z ! T is a continuous onto map
and A  Cp .T /. If and A0 D '  .A/ and  D '  jA W A ! A0 then eAT ' D   eAZ0 .
Proof. If z 2 Z then let y D '.z/ and observe that eAT .y/ is an element of Cp .A/
such that eAT .y/.f / D f .y/ for any f 2 Cp .T /. Now, it follows from the equalities
  .eAZ0 .z//.f / D .eAZ0 .z/ '  /.f / D .f '/.z/ D f .y/ D eAT .y/.f / that the
functions eAT .'.z// and   .eAZ0 .z// coincide for every z 2 Z, i.e.,   eAZ0 D eAT ',
so Fact 1 is proved.
Fact 2. Suppose that Z is a space, M is a second countable space and we have a
continuous onto map ' W Z ! M . Then there exists a countable P  Cp .Z/ and a
homeomorphism u W M 0 D ePZ .Z/ ! M such that u ePZ D '.
Proof. Let A  Cp .M / be a countable QS -algebra (which exists by Problem 176).
Then A separates the points from the closed sets of the space M , so the reflection
map eAM W M ! M0 D eAM .M / is a homeomorphism by TFS-166.
The set P D '  .A/  Cp .Z/ is countable; let  D '  jA W A ! P . The
dual map   W Cp .P / ! Cp .A/ is a homeomorphism and   .ePZ .z// D eAM .'.z//
for any z 2 Z by Fact 1. Therefore ..eAM /1 .  jM 0 // ePZ D ' and hence
u D .eAM /1 .  jM 0 / is the promised homeomorphism, i.e., Fact 2 is proved.
Fact 3. Given a space Z and a countable P  Cp .Z/ let M0 D ePZ .Z/. Suppose
that M is a second countable space and we have continuous onto maps ' W Z ! M
and r W M ! M0 such that r ' D ePZ . Then there is a countable set Q  Cp .Z/
Z
such that P  Q and M1 D eQ
.Z/ is homeomorphic to M .
Proof. Let  W Cp .M0 / ! Cp .Z/ be the dual map of ePZ , i.e., .f / D f ePZ
for any f 2 Cp .M0 /. It was proved in Fact 5 of U.086 that P  .Cp .M0 //; the
set P 0 D f1 .g/ W g 2 P g is countable and hence P 00 D r  .P 0 /  Cp .M /
is countable as well, so there is a countable QS -algebra A  Cp .M / such that
P 00  A (see Problem 176). The set Q D '  .A/  Cp .Z/ is countable; besides,
eAM W M ! eAM .M /  Cp .A/ is a homeomorphism and it follows from Fact 1 that
Z
eAM ' D   eQ
where  D '  jA.

216

2 Solutions of problems 001500

Z
Thus, for the space M1 D eQ
.Z/ the map u D .eAM /1   W M1 ! M is a
homeomorphism. To see that P  Q fix any f 2 P and let g D 1 .f /; h D
r  .g/. Then v D '  .h/ 2 Q; to see that f D v take any point z 2 Z. Then
f .z/ D g.ePZ .z//. On the other hand, it follows from h D g r and v D h ' that
v D g .r '/ D g ePZ , so v.z/ D g.ePZ .z// D f .z/. Thus f D v 2 Q, so Q  P
and hence Fact 3 is proved.

Fact 4. Suppose that Z is a space and we have a sequence fAn W n 2 !g of


subsets of Cp .Z/ such that, for every n 2 ! there is a QS -algebra Qn inS
the space
Cp .ZAn
/ such that An  Qn0 D .eAZn / .Qn /  AnC1 . Then, for A D n2! An ,
the set dAZ .A/ is a QS -algebra in Cp .ZA
/.
Proof. The maps .eAZ / and dAZ are easily seen to preserve products and linear
combinations, i.e., dAZ .u v/ D dAZ .u/ dAZ .v/ and dAZ .uCv/ D dAZ .u/CdAZ .v/
for any u; v 2 A and ; 2 R. Of course, analogous equalities hold for .eAZ / which
easily implies that f g 2 Qn0 and pf C qg 2 Qn0 for any f; g 2 Qn0 and p; q 2 Q.
If f; g 2 dAZ .A/ then f D dAZ .f0 / and g D dAZ .g0 / for some f0 ; g0 2 A.
There is n 2 ! for which f0 ; g0 2 An and hence h D f0 g0 2 Qn0  AnC1  A;
analogously, h0 D f0 Cg0 2 A. Consequently, dAZ .h/ D dAZ .f0 / dAZ .g0 / D f g 2
dAZ .A/ and dAZ .h0 / D dAZ .f0 / C dAZ .g0 / D f C g 2 dAZ .A/. Furthermore, for any
q 2 Q the function qf0 also belongs to Qn0  A, so dAZ .qf0 / D qdAZ .f0 / D qf 2
dAZ .A/.
Finally take a closed subset F  ZA
and a point y0 2 ZA
nF . There exist
f1 ; : : : ; fk 2 A and " > 0 such that the set O D fy 2 ZA
W jy.fi /  y0 .fi /j < "
for all i  kg is disjoint from F . Choose n 2 ! with ff1 ; : : : ; fk g  An . If z0 D
y0 jAn then O 0 D fz 2 ZAn
W jz.fi /  z0 .fi /j < " for all i  kg is an open
Z
subset of ZAn
such that A;A
.O/  O 0 and z0 2 O 0 . It is easy to see that the
n
Z
0
set F D A;An .F / is disjoint from O 0 , so there is a function g 2 Qn such that
Z
then f .y0 / D 1 and f .F /  f0g.
g.z0 / D 1 and g.F 0 /  f0g. If f D g A;A
n
Z
Z
eAZ / D g eAZn belongs to Qn0
Now the function h D f eA D g .A;A
n
Z
Z
and hence h 2 A. Therefore f D dA .h/ 2 dA .A/, so dAZ .A/ is a QS -algebra and
hence Fact 4 is proved.
Fact 5. Given a space Z suppose that a set Ai  Cp .Z/ is countable,
Ai  AiC1
S
and dim.ZAi
/  n for all i 2 !. Then, for the set A D i2! Ai , we have
dim.ZA
/  n.
Proof. The family fZAi
; AZj ;Ai W i < j < !g is an inverse system; denote by
Z
L its limit. If ri D A;A
then AZj ;Ai rj D ri for any i; j 2 ! with i < j , so
i
the diagonal product r of the family fri W i 2 !g maps ZA
continuously into
L by Problem 167. If y0 ; y1 2 ZA
and y0 y1 then there is f 2 A such that
y0 .f / y1 .f /; since f 2 Ai for some i 2 !, we have ri .y0 / ri .y1 / and hence
r.y0 / r.y1 /, i.e., r W ZA
! L is an injection.
Let T D r.ZA
/ and fix a point t0 2 T ; there is a unique point y0 2 ZA

such that r.y0 / D t0 . To see that the map r 1 is continuous at the point t0 take any

2 Solutions of problems 001500

217

set U 2 .y0 ; ZA
/. There are f1 ; : : : ; fk 2 A and " > 0 such that O D fy 2
ZA
W jy.fi /  y0 .fi /j < " for all i  kg  U . There exists m 2 ! for which
ff1 ; : : : ; fk g  Am . The set W D ft 2 T W jt .m/.fi /t0 .m/.fi /j < " for all i  kg
is an open neighborhood of t0 in T . If t 2 W and y D r 1 .t / then yjAm D t .m/
and y0 jAm D t0 .m/ which shows that jy.fi /y0 .fi /j D jt .m/.fi /t0 .m/.fi /j < "
for every i  k. As a consequence, y D r 1 .t / 2 O; the point y 2 W was chosen
arbitrarily, so we proved that r 1 .W /  O  U .
Thus the map r 1 is continuous at every point of T , so r W ZA
! T is a
homeomorphism, i.e., ZA
embeds in L. Finally apply Problem 175 to see that
dim L  n and hence dim.ZA
/ D dim T  dim L  n by Problem 155, so Fact 5
is proved.
Returning to our solution let u W Cp .X / ! Cp .Y / be a uniform homeomorphism. The equality dim X D dim Y will be established if we prove that
dim X  dim Y and dim Y  dim X . The spaces X and Y are in a symmetric
situation, so it suffices to show that dim Y  dim X . If dim X D 1 then there is
nothing to prove, so assume that dim X D n 2 !.
Suppose that M is a second countable space and r W Y ! M is a continuous
map; if M 0 D r.M / then r W Y ! M 0 is surjective. Apply Fact 2 to find a countable
A0  Cp .Y / and a homeomorphism h W Y A0
! M 0 such that h eAY 0 D r.
The space Y A0
being second countable, there is a countable QS -algebra Q0 
Cp .Y A0
/ with dAY0 .A0 /  Q0 ; then the set Q00 D .eAY 0 / .Q0 / is countable and
contains A0 .
The set P D u1 .Q00 /  Cp .X / is countable and the map ePX W X ! X P

is continuous, so there exists a second countable space Z such that dim Z  n


and there are maps  W X ! Z and  W Z ! X P
such that   D ePX (see
Problem 161). Passing, if necessary, to Z 0 D .Z/ and applying Problem 155
we can assume, without loss of generality, that .X / D Z. By Fact 3 there
exists a countable set B0  P such that X B0
is homeomorphic to Z and hence
dim X B0
 n.
Proceeding by induction assume that m 2 ! and we have constructed countable
sets A0 ; : : : ; Am and B0 ; : : : ; Bm with the following properties:
Ai  AiC1 and Bi  BiC1 for any i < m;
u1 .Ai /  Bi for each i  m and u.Bi /  AiC1 for every i < m;
dim.X Bi
/  n for any i  m;
there is a QS -algebra Qi  Cp .Y Ai
/ such that Ai  Qi0 D .eAY i / .Qi / and
u1 .Qi0 /  Bi for all i  m;
(5) there is a QS -algebra Ri  Cp .Y Bi
/ such that Bi  Ri0 D .eBXi / .Ri / and
u.Ri0 /  AiC1 for all i < m.

(1)
(2)
(3)
(4)

Apply Problem 176 to find a countable QS -algebra Ri  Cp .X Bi


/ with
dBXi .Bi /  Ri . Then Bi  Ri0 D .eBXi / .Ri / and the set AiC1 D u.Ri0 /  Ai
is countable, so there exists a countable QS -algebra QiC1  Cp .Y AiC1
/ such
0
D .eAY i C1 / .QiC1 / and the set
that dAYi C1 .AiC1 /  QiC1 . Then AiC1  QiC1
0
0
1
P D u .QiC1 /  Ri is countable, so there exists a second countable space Z

218

2 Solutions of problems 001500

such that dim Z  n and there are maps  W X ! Z and  W Z ! X P


such that
  D ePX (see Problem 161). Passing, if necessary, to Z 0 D .Z/ and applying
Problem 155 we can assume, without loss of generality, that .X / D Z. By Fact 3
there exists a countable set BmC1  P such that X BmC1
is homeomorphic to Z
and hence dim X BmC1
 n.
Now that we have the sets AmC1 and BmC1 is straightforward that the properties
(1)(5) are fulfilled if we substitute m by m C 1, so our inductive procedure
can be continued to construct sequences fAi W i 2 !g and fBi W Si 2 !g for
which S
the conditions (1)(5) are satisfied for all m 2 !. If A D i2! Ai and
B D i2! Bi then it follows from (1) and (2) that u.B/ D A, so A and B are
uniformly homeomorphic. The properties (4) and (5), together with Fact 4 imply
that the set A0 D dAY .A/ is a QS -algebra in Cp .Y A
/ and B 0 D dBX .B/ is a
QS -algebra in Cp .X B
/. The maps .eBX / and dAY are linear, so they are uniform
homeomorphisms; an immediate consequence is that v D dAY u .eBX / W B 0 ! A0
is a uniform homeomorphism. We have dim.X B
/  n by the property (3) and
Fact 5. Now apply Problem 179 to conclude that dim.Y A
/ D dim.X B
/  n.
Y
Finally, for the map g D h A;A
, we have g eAY D r, so we can apply
0
Problem 161 again to see that dim Y  n D dim X . Since X and Y are in a
symmetric situation, we also have dim X  dim Y , so dim X D dim Y and hence
our solution is complete.
V.181. Let X be a zero-dimensional compact space. Prove that Y is also a zerou
dimensional compact space whenever Y  X .
Solution. It follows from Problem 138 that Y is compact. Apply SFFS-306 to
see that the space X is strongly zero-dimensional and hence dim X D 0 by
Problem 148. Therefore dim Y D dim X D 0 by Problem 180, so we can apply
Problem 149 to conclude that the space Y is zero-dimensional.
u

V.182. Suppose that X is a zero-dimensional Lindelf space and Y  X . Prove that


Y is also zero-dimensional.
Solution. Apply SFFS-306 to see that the space X is strongly zero-dimensional and
hence dim X D 0 by Problem 148. Therefore dim Y D dim X D 0 by Problem 180,
so we can apply Problem 149 to conclude that the space Y is also zero-dimensional.
V.183. Given a countable ordinal   1, prove that a metrizable space X is an
absolute Borel set of multiplicative class  (i.e., X 2 M ) if and only if there exists
a completely metrizable space Z such that X is homeomorphic to some Y 2 0 .Z/.
Solution. Apply TFS-237 to find a complete metric space Z such that X is
homeomorphic to a subspace Y  Z. If X 2 M then Y 2 0 .Z/; this proves
necessity.
Assume that X is homeomorphic to a subspace Y of a complete metric space
Z such that Y 2 0 .Z/ and take an arbitrary metrizable space T which contains
a homeomorphic copy X 0 of the space X . It follows from TFS-237 that we can
assume, without loss of generality, that T  M for some complete metric space M .

2 Solutions of problems 001500

219

Fix a homeomorphism h W Y ! X 0 and apply Fact 2 of T.333 to find G -sets P


and Q in the spaces Z and M respectively such that Y  P; X 0  Q and there
exists a homeomorphism f W P ! Q with f jY D h. It follows from Y 2 0 .Z/
that Y D Y \ P 2 0 .P / and hence X 0 2 0 .Q/. By Fact 1 of T.319 there exists
a set R 2 0 .M / such that R \ Q D X 0 ; it follows from   1 and Q 2 10 .M /
that Q 2 0 .M / (see Fact 1 of T.331), so X 0 2 0 .M /. Finally apply Fact 1 of
T.319 to conclude that X 0 D X 0 \ T 2 0 .T /; this settles sufficiency.
V.184. Given a countable ordinal   2, prove that a metrizable space X is an
absolute Borel set of additive class  (i.e., X 2 A ) if and only if there exists a
completely metrizable space Z such that X is homeomorphic to some Y 2 0 .Z/.
Solution. Apply TFS-237 to find a complete metric space Z such that X is
homeomorphic to a subspace Y  Z. If X 2 A then Y 2 0 .Z/; this proves
necessity.
Now assume that the space X is homeomorphic to a subspace Y of a complete
metric space Z such that Y 2 0 .Z/ and take an arbitrary metrizable space T
which contains a homeomorphic copy X 0 of the space X . It follows from TFS-237
that we can assume, without loss of generality, that T  M for some complete
metric space M .
Fix a homeomorphism h W Y ! X 0 and apply Fact 2 of T.333 to find G -sets P
and Q in the spaces Z and M respectively such that Y  P; X 0  Q and there
exists a homeomorphism f W P ! Q with f jY D h. It follows from Y 2 0 .Z/
that Y D Y \ P 2 0 .P / and hence X 0 2 0 .Q/. By Fact 1 of T.319 there exists
a set R 2 0 .M / such that R \ Q D X 0 ; it follows from   2 and Q 2 10 .M /
that Q 2 0 .M / (see Fact 1 of T.331), so X 0 2 0 .M / by Fact 1 of T.341.
Finally apply Fact 1 of T.319 to conclude that X 0 D X 0 \ T 2 0 .T /; this settles
sufficiency.
V.185. Suppose that n 2 N and a space Xi is metrizable for every i  n. Prove
that, for any countable ordinal   2,
(i) if Xi 2 A for all i  n then X1  : : :  Xn 2 A ;
(ii) if Xi 2 M for all i  n then X1  : : :  Xn 2 M .
Solution. The following fact is crucial for this solution.
Fact 1. Given 2 !1 and m 2 N suppose that we have
; Ym and sets
Qspaces Y1 ; : : : Q
Pi 2 0 .Yi /; Qi 2 0 .Yi / for every i  m. If Y D in Yi ; P D in Pi and
Q
Q D in Qi then P 2 0 .Y / and Q 2 0 .Y /.
Proof. If D 0 then Pi is open in Yi and Qi is closed in Yi for each i  m, so
P is open and Q is closed in Y . Proceeding by induction assume that  1 is a
countable ordinal and we proved our Fact for all < .
Let i W Y ! Yi be the natural projection for each i  m. If Pi 2 0 .Yi / then
there exist families fFki W k 2 !g and fki W k 2 !g  such that Fki 2 0i .Yi /
k

220

2 Solutions of problems 001500

S
for every k 2 ! and Pi D k2! Fki for all i  m. For any k1 ; : : : ; km 2 ! if
.k1 ; : : : ; km / D maxfki i W i  mg then the set F .k1 ; : : : ; km / D Fk11  : : :  Fkmm
0
belongs to .k
.Y / by the induction hypothesis and Fact 1 of T.331. It is
1 ;:::;km /
S
straightforward that P D fF .k1 ; : : : ; km / W ki 2 ! for all i  mg; since also
.k1 ; : : : ; km / < for any k1 ; : : : ; km 2 !, we conclude that P 2 0 .Y /.
Now, if Qi 2 0 .Yi / then Yi nQi 2 0 .Yi / and hence i1 .Yi nQi / belongs
S
to 0 .Y / for all i  m (see Fact 1 of T.318). Since Y nQ D fi1 .Yi nQi / W
i  mg, we can apply Fact 1 of T.341 to see that Y nQ 2 0 .Y / and hence Q 2
0 .Y /. Thus our statement is verified for D , so our inductive procedure can be
continued to guarantee that it holds for all < !1 , i.e., Fact 1 is proved.
Returning to our solution take a completely metrizable space Mi such
Q that Xi is
homeomorphic to some Yi  Mi for all i  n (see TFS-237); let Y D in Yi and
Q
M D in Mi . If Xi 2 A then Yi 2 0 .Mi / for every i  n, so Y 2 0 .M /
by Fact 1. The space X D X1  : : :  Xn is homeomorphic to Y , so we can apply
Problem 184 to see that X 2 A . Finally, if Xi 2 M then Yi 2 0 .Mi / for every
i  n, so Y 2 0 .M / by Fact 1. The space X being homeomorphic to Y we can
apply Problem 183 to see that X 2 M .
V.186. Given ordinals ; S
2 !1 such that  2 and < suppose that X is a
metrizable space and X D fXn W n 2 !g where Xn 2 0 .X / \ M for every
n 2 !. Prove that X 2 M .
Solution. Take a complete metric space M such that X  M (see TFS-237). It
follows from Fact 1 of T.319 that there exists Yn 2 0 .M / such that Yn \ X D Xn
S
for any n 2 !; thus the set Y D n2! Yn 2 0 .M /  0 .M / (see Fact 1 of
T.331) belongs to the class 0 .M /. Now, Xn 2 M implies that Xn 2 0 .M / and
0
therefore Yn nXn 2 0 .M / for every n 2 !. The class
S .M / being  -additive (see
Fact 1 of T.341), we convince ourselves that Z D n2! .Yn nXn / also belongs to the
class 0 .M /. Consequently, X D Y nZ 2 0 .M /, so we can apply Problem 183
to conclude that X 2 M .
V.187. Prove that a metrizable space X is a Borel set of absolute additive class
  2 (i.e.,SX 2 A ) if and only if there exists a sequence fn W n 2 !g   such
that X D fXn W n 2 !g and Xn 2 Mn for every n 2 !.
Solution. Suppose that X 2 A and fix a complete metric space M such that X 
M (see TFS-237). We have X 2 0 .M /, so there exist sequences fn W n 2 !g  
S
and fXn W n 2 !g such that X D n2! Xn and Xn 2 0n .M / for all n 2 !. Let
n D n if n  1 and n D 1 if n D 0; it is clear that fn W n 2 !g   and,
besides, n  n , so Xn 2 0n .M / for every n 2 !. Apply Problem 183 to see that
Xn 2 Mn for each n 2 !; this settles necessity.
Now assume that there exist sequences fn W S
n 2 !g   and fXn W n 2 !g
such that Xn 2 Mn for every n 2 ! and X D n2! Xn . If X is a subspace of a

2 Solutions of problems 001500

221

metric space M then Xn  M and hence Xn 2 0n .M / for all n 2 !. Therefore


X 2 0 .M / and hence X is a Borel set of absolute additive class , i.e., we proved
sufficiency.
V.188. Given a countable ordinal   2, let M be the class of absolute Borel sets
of multiplicative class . Prove that the following conditions are equivalent for any
metrizable X :
(i) the space X belongs to M ;
(ii) there is a complete sequence fUn W n 2 !g of  -discrete covers of X such that,
for any n 2 !, there is n <  with Un  0n .X /;
(iii) there is a complete sequence fVn W n 2 !gSof  -discrete covers of X such that,
for any n 2 !, there is n <  with Vn  f0 .X / W < n g.
Solution. Recall that a sequence fWn W n 2 !g of covers of a space Z is called
complete
F \ Un ; for all n 2 ! we
T if, for any filter F on the set Z such thatT
have fF W F 2 Fg ;; the points of the set fF W F 2 Fg are called the
cluster points of F. Given a metric space .M; d /, a family U of subsets of M is
called uniformly discrete if there exists " > 0 such that the ball Bd .x; "/ D fy 2
M W d.x; y/ < "g meets at most one element of U for every x 2 X ; the family U
is  -uniformly discrete if it is a countable union of uniformly discrete families. For
any nonempty set A  M let diamd .A/ D supfd.x; y/ W x; y 2 Ag.
VnGiven a space Z and families A1 ; : : : ; An of subsets of Z we will need the family
iD1 Ai D A1 ^ : : : ^ An D fA1 \ : : : \ An W Ai 2 Ai for all i  ng. For any
A  exp.Z/ let AjY D fA \ Y W A 2 Ag.
Fact 1. Given a collectionwise normal space Z and 2 !1 suppose that fPt W t 2
T g is a discrete family
of subsets of Z. If Pt 2 0 .Z/ (Pt 2 0 .Z/) for each
S
t 2 T then P D t2T Pt 2 0 .Z/ (or P 2 0 .Z/ respectively).
Proof. If D 0 then every Pt is open (closed) in Z, so P is open in Z (or closed in
Z respectively, because the union of a discrete family of closed sets is closed); this
proves our statement for D 0.
Proceeding inductively, assume that > 0 and our statement is proved for all
< . If fPt W t 2 T g  0 .Z/ then, for each t 2 T , there are sequences fn .t / W
S
n 2 !g  and fPtn W n 2 !g such that Pt D n2! Ptn and Ptn 2 0n .t/ .Z/ for
every n 2 !. For every < and n 2 ! let T n D ft 2 T W n .t /  g.
Take any n 2 !; the family fPtn W t 2 T n g  0 .Z/ isSdiscrete, so we can
apply the induction hypothesis to conclude that the set Q n D t2T n Ptn belongs to

< g is countable,
0 .Z/ for each ordinal < . The family Q D fQ n W n 2 !; S
every element of Q belongs to a multiplicative class < , so Q 2 0 .Z/. It
S
is easy to see that P D Q, so P 2 .Z/, i.e., we completed the induction step
for the additive class .
Now, if fPt W t 2 T g  0 .Z/ then choose a discrete family fOt W t 2 T g such
that Pt  Ot for every t 2 T . Each set Ot nPt belongs to the family 0 .Z/, so we
S
can use what we proved for the additive class to see that W D fOt nPt W t 2 T g

222

2 Solutions of problems 001500

S
is an element of 0 .Z/. The set O D t2T Ot being open, the closed set F D
ZnO belongs to 0 .Z/ (recall that > 0); as a consequence, ZnP D W [ F 2
0 .Z/ which shows that P 2 0 .Z/ and completes our induction step. Thus our
statement is true for every < !1 , i.e., Fact 1 is proved.
Fact 2. Assume that  2 is a countable ordinal,U  0 .M / is a  -discrete cover
of a metric space .M; d / and r > 0. Then there is a  -uniformly discrete refinement
V  0 .M / of the family U such that diamd .V / < r for any V 2 V.
S
Proof. By our assumption, U D n2! Un and every Un is discrete, so we can choose
an open cover Gn of the space M such that every set G 2 Gn meets at most one
element of Un and diamd .G/ < r. Fix n 2 ! and applySFact 1 of T.373 to find an
open refinement Wn of the cover Gn such that Wn D i2! Win and every Win is
uniformly discrete. Let V.n; i / D Win ^ Un for all n; i 2 !.
Fix any n; i 2 ! and take " > 0 such that Bd .x; "/ meets at most one element
of Win for any x 2 X . Suppose that we have distinct V0 ; V1 2 V.n; i / such that
Bd .x; "/ \ Vj ; for j D 0; 1. Take W0 ; W1 2 Win and U0 ; U1 2 Un such that
Vj D Wj \ Uj and hence Bd .x; "/ \ .Wj \ Uj / ; if j 2 f0; 1g. It follows from
Bd .x; "/\W0 ; Bd .x; "/\W1 that W0 D W1 and hence U0 U1 ; thus the set
W0 2 Win meets both sets U0 and U1 . There is G 2 Gn with W0  G; an immediate
consequence is that G also meets both sets U0 and U1 which contradicts the choice
of the family Gn . This proves thatSthe family V.n; i / is uniformly discrete for any
n; i 2 !. Thus the family V D fV.n; i / W n; i 2 !g is a  -uniformly discrete
refinement of U such that diamd .V / < r for every V 2 V, so Fact 2 is proved.
Returning to our solution observe that
(1) if fUn W n 2 !g is a complete sequence of covers of a space Z and Vn is a
refinement of Un for every n 2 ! then the sequence fVn W n 2 !g is also
complete.
Apply TFS-237 to find a complete metric space .M; d / such that X  M ; we
will prove by transfinite induction that (i)H)(ii) for every   1. If  D 1 then

X 2 10 .M /, i.e., X is a G -subset of M , so X is Cech-complete


(see TFS-260 and
TFS-269) and hence X has a complete sequence fUn0 W n 2 !g of open covers by
TFS-268. Since X is metrizable, we can choose a  -discrete open refinement Un of
the cover Un0 for every n 2 !. Then Un  00 .X / for every n 2 ! and the sequence
fUn W n 2 !g is complete by (1), so we proved (i)H)(ii) for  D 1.
Now assume that > 1 is a countable ordinal and we proved (i)H)(ii) for all
 < . If X 2 M ; then X 2 0 .M /, so M nX 2 0 .M / and hence there are
sequences fn W n 2 !g  and fHn W n 2 !g
S such that 1  n  nC1 and
Hn 2 0n .M / for every n 2 ! while M nX D n2! Hn . The set Dn D M nHn
T
belongs to the class 0n .M / for each n 2 ! and X D n2! Dn .
For each n 2 ! fix sequences fin W i 2 S
!g  n and fDin W i 2 S
!g such that
n
0
Di 2 n .M / for every i 2 ! and Dn D i2! Din ; if Ein D Din n j <i Djn for
i

2 Solutions of problems 001500

223

S
all i 2 ! then the family fEin W i 2 !g is disjoint, Dn D i2! Ein and it follows
from Fact 1 of T.341 and Fact 1 of T.331 that every set Ein belongs to the family
0n .M / \ 0n .M /.
Apply Problem 183 to see that Ein 2 Mn ; by the induction hypothesis, for every
set Ein we can find a complete sequence fC.n; i; k/ W k 2 !; k  ng of  -discrete
covers of the space Ein such that C.n; i; k/  0n .Ein / for every k  n (observe
that the induction hypothesis could be used to guarantee that every family C.n; i; k/
is in the class 0k .Ein / for some k < n but we wont need that). Applying
Fact 1 of T.341 and Fact 1 of T.319 we conclude that C.n; i; k/  0n .M / for
all n; i; k 2 !; k  n. It follows from (1) and Fact 2 that we can additionally
assume, without loss of generality, that every C.n; i; k/ is  -uniformly discrete in
Ein and diamd .U /  2k for every U 2 C.n; i; k/; then every C.n; k; i / is also
 -uniformly discrete in M by Fact 2 of T.373.
For every p 2 ! consider the family Up D fF W T
F ; while there exist
p
sets U0 ; : : : ; Up and i0 ; : : : ; ip 2 ! such that F D . nD0 Un / \ X and Un 2
C.n; in ; p/ for all n  pg. It follows from 0  : : :  p that Up  0p .X / for
every p 2 ! (see Fact 1 of T.331 and Fact 1 of T.341). Fact 3 of T.373 implies that
the family E.i0 ; : : : ; ip / D C.0; i0 ; p/ ^ : : : ^ C.p;
S ip ; p/ is  -uniformly discrete for
any i0 ; : : : ; ip 2 !. Therefore the family E D fE.i0 ; : : : ; ip / W i0 ; : : : ; ip 2 !g is
also  -uniformly discrete. As a consequence, Up  EjX is  -uniformly discrete for
every p 2 !, so all is left is to prove that the sequence fUp W p 2 !g is complete
in X .
Assume that F is a filter in X such that F \ Up ; and hence we can pick a
set Up 2 Up \ F for each p 2 !. The sequence fdiamd .Up / W p 2 !g converges to
zero, so we have the following property:
(2) the filter F can have at most one cluster point in M .
Tp
By the choice of Up , we have Up D . nD0 Upn /\X where Upn 2 C.n; i.n; p/; p/
for every p 2 !. Given any n 2 ! and p; q  n it follows from Up \ Uq ; that
Upn \ Uqn ;; the family fEin W i 2 !g being disjoint we must have i.n; p/ D
i.n; q/; thus the number in D i.n; p/ does not depend on p.
Let Fn D FjEinn for every n 2 !; it follows from Up  Upn that Fn is a filter
on Einn and Upn 2 Fn \ C.n; in ; p/ for each p  n. The sequence fC.n; in ; p/ W
n  p < !g being complete in Einn , the filter Fn has a cluster point an 2 Einn . It
is clear that every an is also a cluster point of F, so we can apply the property (2)
to convince
ourselves that
T
T there is a 2 M such that an D a for all n 2 !. Now,
a 2 fEinn W n 2 !g  n2! Dn D X , so a is a cluster point of F in X . This shows
that the sequence fUp W p 2 !g is complete and hence we proved that (i)H)(ii).
Assume, toward proving (ii)H)(i), that S D fUn W n 2 !g is a complete
sequence given in (ii); we are still considering that X is a subspace of a complete
metric space .M; d /. Applying Fact 2 we can assume, without
S loss of generality,
that diamd .U /  2n for all n 2 ! and U 2 Un while Un D i2! Uin where every
Uin is uniformly discrete. For each n 2 ! and U 2 Un we can apply Fact 1 of T.319
to find a set E.U /  clM .U / such that E.U / \ X D U and E.U / 2 0n .M /.

224

2 Solutions of problems 001500

Every family Uin is uniformly discrete in M (see Fact 2 of T.373); it is easy to


see that the collection fclM .U / W U 2 Uin g is also uniformly discrete in M , so the
family Vin D fE.U / W U 2 Uin g  0n .M /  0 .M / is uniformly discrete in M
as well. If m 2 ! and we are given arbitrary k0 : : : ; km 2 ! let
Q.k0 ; : : : ; km / D fQ 2 Vk00 ^ : : : ^ Vkmm W Q \ X D ;g:
It follows from Fact 3 of T.373
that Q.k0 ; : : : ; km / is uniformly discrete and hence
S
the set Q.k0 ; : : : ; km / D Q.k0 ; : : : ; km / belongs to 0 .M / (see Fact 1) for any
S
k0 : : : ; km 2 !. Consequently, the set Q D fQ.k0 ; : : : ; km / W k0 ; : : : ; km 2 !g
also belongs to 0 .M / whence R D M nQ belongs to 0 .M / and X  R. The
set R has the following important property:
T
(3) for
T any number n 2 ! if Ui 2 Ui for all i  n then in Ui ; if and only if
. in E.Ui // \ R ;.
S n
Vi belongs to 0n .M /;
Apply Fact 1 again to see that every set Vin D
S
therefore the set Vn D i2! Vin belongs to 0n .M / for each n 2 !. This implies
T
that the set V D n2! Vn  X belongs to 0 .M /. Therefore the set X 0 D V \ R
belongs to 0 .M / and X  X 0 .
Fix any x 2 X 0 ; there is a sequence
T fUn W n 2 !g such that Un 2 Un and x 2
E.Un / for every
n
2
!.
Since
x
2
.
in E.Ui // \ R, we can apply (3) to convince
T
ourselves that in Ui ; for every n 2 !, so the family U D fUn W n 2 !g is
centered; let F be a filter which contains U . The sequence S being complete, the
filter F has a cluster point a 2 X . Since theTsequence fdiamd .clM .Ui // W i 2 !g
converges to zero, it follows from fx; ag  fclM .Un / W n 2 !g that x D a and
hence x 2 X . Therefore X 0  X , i.e., X D X 0 is an element of 0 .M /, so we can
apply Problem 183 to conclude that X 2 M ; this proves that (ii)H)(i).
Now, if (ii) holds and fUn W n 2 !g is the respective complete sequence of covers
of X then, for every
T .U / D fOn .U / W n 2 !g
S n 2 ! and U 2 Un pickSa sequence
0
such that
U
D
O
.U
/
and
O
.U
/
2
f
.X
/
W
< n g for each n 2 !. If
n

n2! n
S
Vn D fT .U / W U 2 Un g then Vn is a  -discrete refinement of UnS
for every n 2 !,
so the sequence fVn W n 2 !g is complete by (1). Since also Vn  f0 .X / W <
n g for all n 2 !, we settled (ii)H)(iii).
Finally, if (iii) holds then take the respective complete sequence fVn W n 2 !g
of  -discrete covers of X and let Un D Vn for every n 2 !. By Fact 1 of T.331
we have Un  0n .X / for each n 2 !; this shows that (iii)H)(ii) and makes our
solution complete.
V.189. Given a countable ordinal   2 prove that the following conditions are
equivalent for any second countable X :
(i) the space X belongs to M ;
(ii) there is a complete sequence fUn W n 2 !g of countable covers of X such that,
for any n 2 !, there is n <  with Un  0n .X /;

2 Solutions of problems 001500

225

(iii) there is a complete sequence fVn W n 2 !gSof countable covers of X such that,
for any n 2 !, there is n <  with Vn  f0 .X / W < n g.
Solution. If X 2 M then it follows from Problem 188 that X has a complete
sequence fUn W n 2 !g of  -discrete covers such that, for every n 2 !, we have
Un  0n .X / for some n < . The space X being second countable, every Un is
countable; this proves that (i)H)(ii).
Now, if X has a complete sequence fUn W n 2 !g of countable covers as in
(ii) then every Un is  -discrete, so we can apply Problem 188 to see that there is
a complete sequence fVn W n 2 !g of  -discrete
covers of X such that, for every
S
n 2 ! there exists n <  for which Vn  f0 .X / W < n g. Since X is second
countable, every Vn is countable, so we settled (ii)H)(iii).
Finally, if fVn W n 2 !g is a complete sequence of countable covers as in (iii)
then each Vn is  -discrete, so we can apply Problem 188 once more to conclude that
X 2 M and hence (iii)H)(i).
V.190. Prove that any analytic space has a complete sequence of countable covers.
Show that in metrizable spaces the converse is also true, i.e., a metrizable space X
is analytic if and only if there exists a complete sequence of countable covers of X .
Solution. Suppose that a space X is analytic and fix a continuous onto map ' W
! ! ! X . For any x 2 X choose a point yx 2 ' 1 .x/ and let Y D fyx W x 2 X g.
Then 'jY W Y ! X is a bijection. Given n 2 ! and k0 ; : : : ; kn 2 ! the set
O.k0 ; : : : ; kn / D ff 2 ! ! W f .i / D ki for all i  ng is open in ! ! , so the family
Un D fO.k0 ; : : : ; kn / W ki 2 ! for all i  ng is an open disjoint cover of ! ! .
The family Vn D fU \ Y W U 2 Un g is a cover of the space Y , so the family
Wn D f'.V / W V 2 Vn g is a cover of X for every n 2 !. To show that the
sequence S D fWn W n 2 !g is complete in X take a filter F on the set X such that
F \ Wn ; and hence there is Vn 2 Vn such that '.Vn / 2 F; pick a set Un 2 Un
with Un \ Y D Vn for each n 2 !.
Since 'jY W Y ! X is a bijection and f'.Vn / W n 2 !g  F is centered, the
family fVn W n 2 !g is centered too; fix a point fn 2 V0 \ : : : \ Vn  U0 \ : : : \ Un
for each n 2 !. By the definition of the family Un there are k0n ; : : : ; knn 2 ! such
that Un D O.k0n ; : : : ; knn / for every n 2 !. Take any m; n; i 2 ! with i  m  n.
It follows from ffm ; fn g  Um and fn 2 Un that kim D fm .i / D fn .i / D kin and
hence kin D kim , i.e., the number kin D ki does not depend on n.
Let f .i / D ki for every i 2 !; then f 2 ! ! . Fix any element F 2 F; if
x D '.f / F then use continuity of ' to find a set W 2 .f; ! ! / such that
'.W / \ F D ;. The family fUn W n 2 !g is easily seen to be a local base at f ,
so there is n 2 ! with Un  W . Then '.Vn /  '.Un /  '.W / and therefore
'.Vn / \ F D ;. This contradiction with f'.V
T n /; F g  F shows that x 2 F ; the
set F 2 F was chosen arbitrarily, so x 2 fF W F 2 Fg. Consequently, S is a
complete sequence in X .
Now assume that X is metrizable and S D fUn W n 2 !g is a complete sequence
of countable covers of X . If w.X / > ! then there exists a closed discrete D  X
with jDj D !1 . Since U0 is countable, we can choose U0 2 U0 such that D0 D

226

2 Solutions of problems 001500

U0 \ D is uncountable. Suppose that n 2 ! andTwe have chosen Ui 2 Ui for all


i  n in such a way that the set Di D D \ . j i Uj / is uncountable for any
i  n. The set Dn is uncountable while the family UnC1 is countable, so there is
UnC1 2 UnC1 such that set DnC1 D Dn \ UnC1 is uncountable.
Thus, our inductive procedure shows that
T we can choose a sequence fUi W i 2 !g
such that Ui 2 Ui and the set Di D D \ . j i Uj / is uncountable for every i 2 !.
This makes it possible to pick a point di 2 Di for each i 2 !, so that di dj
whenever i j . The family D D ffdi W i  ng W n 2 !g is a filterbase in X ;
let F be any filter containing D. By our choice of the set fdi W i 2 !g we have
fdi W i  ng  Un and hence
2 !. The sequence S being
T Un 2 F for every n T
complete there is a point x 2 fF W F 2 Fg, soT
x 2 D because all elements of
D  F are closed in X . This contradiction with D D ; shows that X is second
countable. The following property is evident.
(1) if Un0 is a refinement of Un for every n 2 ! then the sequence S 0 D fUn0 W n 2 !g
is also complete.
We will pass several times from the sequence S to a sequence S 0 as in (1); to
simplify the notation, we will then consider that S 0 D S and hence the sequence S
has the properties we found in S 0 .
Fix a metric  on the set X which generates .X / and let Vn be a countable
subcover of the cover fB .x; 2n1 / W x 2 X g for any n 2 !. Given any n 2 !
the cover Un0 D fU \ V W U 2 Un and V 2 Vn g is a countable refinement of Un
and diam .U /  2n for any U 2 Un0 . According to the policy described above, we
consider that diam .U /  2n for every U 2 Un and n 2 !. Since there is nothing
to prove in the case of an empty X (and neither is it interesting to decide whether the
empty space is analytic or not), we consider that X ;, so we can throw away all
empty elements of every Un obtaining its refinement with only nonempty elements.
Therefore we consider that U ; for any U 2 Un .
Take an enumeration fO.i / W i 2 !g of the family U0 (with possible repetitions)
and let O0 D fO.i / W i 2 !g. Proceeding inductively, assume that n 2 ! and we
have families O0 ; : : : ; On with the following properties:
(2) Oi is a refinement of Ui and no element of Oi is empty for all i  n;
(3) Oi D fO.k0 ; : : : ;S
ki / W kj 2 ! for all j  i g for each i  n;
(4) O.k0 ; : : : ; ki / D fO.k0 ; : : : ; ki ; kiC1 / W kiC1 2 !g for all k0 ; : : : ; ki 2 ! and
i < n;
Given any U D O.k0 ; : : : ; kn / 2 On the family W D fU \ V W V 2 UnC1 g
covers U which is nonempty so we can assume that every W 2 W is nonempty;
take an enumeration fWi W i 2 !g of the family W (with possible repetitions). Let
O.k0 ; : : : ; kn ; knC1 / D WknC1 for every knC1 2 !.
This gives us a family OnC1 D fO.k0 ; : : : ; knC1 / W ki 2 ! for all i  n C 1g
and it is straightforward that the properties (2)(4) still hold if we substitute n by
n C 1. Thus our inductive procedure provides a sequence S0 D fOn W n 2 !g of
covers of X for which the conditions (2)(4) are satisfied for all n 2 !; applying

2 Solutions of problems 001500

227

(1) once more we conclude that S0 is a complete sequence. It follows from (2) and
our choice of the sequence S that
(5) if n 2 ! then diam .O/  2n for any O 2 On .
Given any element f 2 ! ! let Un D O.f .0/; : : : ; f .n//; it follows from the
property (4) that UnC1  Un for every n 2 !. This gives us a decreasing sequence
T D fUn W n 2 !g of nonempty sets such that Un 2 On for all n 2 !. Since T is
a filterbase, we can find a filter F  TT. Then Un 2 F \ O
Tn for every n 2 !; the
sequence S0 being complete, we have fU n W n 2 !g  fF W F 2 Fg ;. It
follows from (5) that the sequence fdiam .Un / W n 2 !g converges to zero, so there
T
is x 2 X such that fU n W n 2 !g D fxg; let '.f / D x. This gives us a map
' W !! ! X .
Fix a point f 2 ! ! and an open
Tneighborhood U of the point x D '.f / in X ; by
our choice of ', we have fxg D fU n W n 2 !g where Un D O.f .0/; : : : ; f .n//
for all n 2 !. It follows from the property (5) that diam .Un / ! 0, so there is
n 2 ! such that U n  U . If g 2 V D fh 2 ! ! W h.i / D f .i / for all i  ng
then '.g/ 2 O.g.0/; : : : ; g.n// D U n  U . This proves that '.V /  U and hence
V 2 .f; ! ! / witnesses continuity of ' at the point f ; thus the map ' is continuous.
Finally, take any x 2 X ; the property (4) shows that there exists a sequence
fki W i 2 !g  ! such that x 2 Un T
D O.k0 ; : : :T
; kn / for any n 2 !.
S If f .i / D ki
for all i 2 ! then f 2 ! ! and x 2 T
U

U
.
The
set
n
n
n2!
n2!
n2! U n being
a singleton by (5) we conclude that n2! U n D fxg and hence '.f / D x; this
shows that X is analytic being a continuous image of ! ! . Thus we have proved the
converse for the class of metric spaces, i.e., our solution is complete.
V.191. For any metrizable space X and n 2 N define a map e W X n ! X
n by
e..x1 ; : : : ; xn // D fx1 ; : : : ; xn g for every .x1 ; : : : ; xn / 2 X n . Prove that there exists
an F -set G in the space X n such that e.G/ D X
n and the map ejG W G ! X
n
is a bijection.
Solution. Let T D f.x1 ; : : : ; xn / 2 X n W xi xj whenever i j g. Given any
bijection  W f1; : : : ; ng ! f1; : : : ; ng let h ..x1 ; : : : ; xn // D .x.1/ ; : : : ; x.n/ / for
any .x1 ; : : : ; xn / 2 T . It is evident that h W T ! T is a homeomorphism. Denote
by Sn the set of all bijections  W f1; : : : ; ng ! f1; : : : ; ng. Call a set Y  T
adequate if ejY is injective.
For any a D .a1 ; : : : ; an / 2 T choose disjoint sets O1 ; : : : ; On 2 .X / such that
ai 2 Oi for every i  n. It is easy to see that the set O D O1  : : :  On  T is
adequate, open and a 2 O; this shows that T has a cover which consists of open
adequate sets. The space T is metrizable and hence paracompact so we can choose
an open locally finite cover U DSfU W < g of the space T such
Sthat U is
adequate for all < . Let V D fh .V / W  2 Sn g and G D U n. fV W <
g/ for every < . The family fG W < g is disjoint and locally finite. Since
S in
the space T every open set is F , every G is an F -subset of T ; let G D fG W
< g. It is an easy exercise that the union of a locally finite family of F -sets is
an F -set, so G is an F -subset of T ; the set T being open in X n , we conclude that
G is also an F -set in X n .

228

2 Solutions of problems 001500

Take any set y D fx1 ; : : : ; xn g 2 X


n ; the point x D .x1 ; : : : ; xn / is an element
of T , so e 1 .y/ D fh .x/ W  2 Sn g. Take the minimal ordinal <  such that
U \ e 1 .y/ ; and fix  2 Sn with h .x/ 2 U . If h .x/ 2 V for some <
then there is  2 Sn and z 2 U for which h .z/ D h .x/. It is straightforward that
1
z D h1
 .h .x// D h1 .x/ 2 e .y/ \ U which is a contradiction. Therefore
a D h .x/ 2 G and hence e.a/ D y. The point y 2 X
n was chosen arbitrarily
so we proved that e.G/ D X
n .
Finally take distinct points a and b in the set G and fix ; 2  with a 2 G
and b 2 G ; we can assume, without loss of generality, that  . If <
then b V and hence b e 1 .a/ which shows that e.a/ e.b/. If D then
e.a/ e.b/ because G is an adequate set. Thus G is an F -subset of X n such that
e.G/ D X
n and ejG W G ! X
n is a bijection.
V.192. Given a metrizable space X and n 2 N consider the set X
n together with
its Vietoris topology. ProveSthat there exists a family fYm W m 2 !g of closed subsets
of X
n such that X
n D fYm W m 2 !g and every Ym is homeomorphic to some
closed subspace of X n .
Solution. For any space Z denote by K.Z/ the family of all compact subsets of
Z and let VZ be the Vietoris topology on K.Z/. For each U 2 .Z/ consider the
families OZ U
D fK 2 K.Z/ W K \ U ;g and OZ hU i D fK 2 K.Z/ W K 
U g. If S0 .Z/ D fOZ U
W U 2 .Z/g and S1 .Z/ D fOZ hU i W U 2 .Z/g then the
family S.Z/ D S0 .Z/ [ S1 .Z/ is a subbase of the space .K.Z/; VZ /.
Fact 1. For any space Z if T  Z then VT D fV \ K.T / W V 2 VZ g and hence
.K.T /; VT / is a subspace of .K.Z/; VZ /.
Proof. Since K.T /  K.Z/, we must only check that VT is induced by VZ on
K.T /. It is easy to verify that
(1) OZ U
\ K.T / D OT U 0
and OZ hU i \ K.T / D OT hU 0 i for any U 2 .Z/
and U 0 2 .T / such that U 0 D U \ T .
An immediate consequence of (1) is that G \ K.T / 2 S.T / for any G 2 S.Z/.
If G 0 2 S0 .T / then take U 0 2 .T / such that G 0 D OT U 0
and choose U 2 .Z/
with U \T D U 0 . It follows from (1) that G 0 D OZ U
\K.T /. If G 0 2 S1 .T / then
there exists U 0 2 .T / such that G 0 D OT hU 0 i; pick U 2 .Z/ with U \ T D U 0
and apply (1) to convince ourselves that OZ hU i\K.Z/ D OT hU 0 i. This proves that
S.T / D fS \ K.T / W S 2 S.Z/g. Recalling that S.Z/ is a subbase of .K.Z/; VZ /
and S.T / is a subbase of .K.T /; VT / and representing the respective open sets as
unions of finite intersections of the elements of their corresponding subbases, we
conclude that VT D fV \ K.T / W V 2 VZ g, i.e., Fact 1 is proved.
Fact 2. For any space Z the space .K.Z/; VZ / is Tychonoff.
Proof. The family F of all closed subsets of Z coincides with K.Z/, so we can
apply Fact 1 of T.372 to conclude that .K.Z/; VZ / is a compact Hausdorff (and
hence Tychonoff) space. It follows from Fact 1 that .K.Z/; VZ / is a subspace of
.K.Z/; VZ /, so .K.Z/; VZ / is a Tychonoff space, i.e., Fact 2 is proved.

2 Solutions of problems 001500

229

Fact 3. Suppose that n 2 N and f W Z ! T is an open map such that jf 1 .t /j D n


for any t 2 T . Then
(a) f is perfect;
(b) f is a local homeomorphism, i.e., for any z 2 Z there is W 2 .z; Z/ such that
f jW W W ! f .W / is a homeomorphism.
Proof. Recall that we consider that all open maps are surjective. The inverse images
of all points of T are finite and hence compact so, to prove (a), it suffices to show
that f is a closed map. Given t 2 T and U 2 .f 1 .t /; Z/, we have f 1 .t / D
fz1 ; : : : ; zn g, so we can choose disjoint open sets U1 ; : : : ; Un such that U 0 D U1 [
: : : [ Un  U and zi 2 Ui for all i  n. The set V D f .U1 / \ : : : \ f .Un / is an
open neighborhood of the point t .
For any a 2 V and i  n there is bi 2 Ui with f .bi / D a; an immediate
consequence is that B D fb1 ; : : : ; bn g  f 1 .a/. Since the set f 1 .a/ has exactly
n elements, we conclude that f 1 .a/ D B  U 0  U . The point a 2 V was
chosen arbitrarily, so we proved that f 1 .V /  U 0  U and hence the map f is
closed by Fact 2 of S.271; this settles (a).
Fix any point z 2 Z and let t D f .z/; then f 1 .t / D fz; z1 ; : : : ; zn1 g, so we
can choose disjoint sets U; U1 ; : : : ; Un1 2 .Z/ such that z 2 U and zi 2 Ui for
all i  n  1. Then V D f .U / \ f .U1 / \ : : : \ f .Un1 / 2 .t; T / and hence
W D U \ f 1 .V / is an open neighborhood of the point z.
If a 2 W then, for any i  n  1 there is ai 2 Ui such that f .ai / D f .a/, so
A D fa; a1 ; : : : ; an1 g  f 1 .f .a//. Recalling that jf 1 .f .a//j D n we conclude
that A D f 1 .f .a// and hence f 1 .f .a// \ W D fag for any a 2 W . This shows
that f jW W W ! f .W / is an injection; since f jW is an open map, we conclude
that f jW is a homeomorphism so we established (b), i.e., Fact 3 is proved.
Fact 4. Given a space Z and n 2 N let Zn
D f.z1 ; : : : ; zn / 2 Z n W zi zj
whenever i j g. For any z D .z1 ; : : : ; zn / 2 Zn
let '.z/ D fz1 ; : : : ; zn g. Then
the map ' W Zn
! Z
n is open if Z
n is considered with it Vietoris topology.
Proof. It is evident that ' is surjective. If y D fz1 ; : : : ; zn g 2 Z
n and Ui 2 .zi ; Z/
for all i  n then consider the set U1 ; : : : ; Un
D fa 2 Z
n W a \ Ui ; for all
i  n and a  U1 [ : : : [ Un g. It is straightforward that the family
Cy D fU1 ; : : : ; Un
W Ui 2 .zi ; Z/ for all i  n and Ui \Uj D ; whenever i j g
is a local base of the space Z
n at the point y.
For any point z D .z1 ; : : : ; zn / 2 Zn
the family Bz D fU1  : : :  Un W zi 2 Ui
for all i  n and Ui \ Uj D ; whenever i j g is easily seen to be a local base
of Zn
at the point z. It is an easy exercise that '.U1  : : :  Un / D U1 ; : : : ; Un
if
the family fU1 ; : : : ; Un g is disjoint. An immediate consequence is that, for the point
y D '.z/ we have Cy D f'.G/ W G 2 Bz g, so we can apply Fact 2 of S.491 to
conclude that ' is an open continuous map. Fact 4 is proved.

230

2 Solutions of problems 001500

Returning to our solution consider the map ' W X n


! X
n defined by the
formula '.x/ D fx1 ; : : : ; xn g for any x D .x1 ; : : : ; xn / 2 X n
. Apply Fact 4 to see
that ' is open; since also j' 1 .a/j D n for any a 2 X
n , we can apply Fact 3 to
convince ourselves that ' is also a closed map.
Apply Problem 191 to find a family fFm W m 2 !g of closed
S subsets of the space
X n
such that the restriction of the map ' to the set F D m2! Fn is injective and
'.F / D X
n . For every m 2 ! the set Ym D '.Fm / is closed in X
n ; the map
'jFm W F
Sm ! Ym being closed and injective, it has to be a homeomorphism, so
X
n D m2! Ym is the promised representation of X
n and hence our solution is
complete.
V.193. Suppose that there exists a uniformly continuous surjection of Cp .X / onto
Cp .Y /. Prove that if X is pseudocompact then Y is also pseudocompact. Deduce
from this fact that if X is a metrizable compact space and there exists a uniformly
continuous surjection of Cp .X / onto Cp .Y / then Y is also compact. Give an
example of a (non-metrizable!) compact space X such that there is a non-compact
space Y and a uniformly continuous surjection of Cp .X / onto Cp .Y /.
Solution. Recall that a uniform space .Z; U / is called  -totally bounded if there
exists a family
S fZn W n 2 !g of totally bounded uniform subspaces of .Z; U / such
that Z D n2! Zn .
Suppose that ' W Cp .X / ! Cp .Y / is a uniformly continuous surjection. If X
is pseudocompact then Cp .X / is  -totally bounded by Problem 136; fix a family
fC
S n W n 2 !g of totally bounded uniform subspaces of Cp .X / such that Cp .X / D
n2! Cn . By Fact 1 of V.136, the uniform space Dn D '.Cn / is
Stotally bounded
for every n 2 !. The map ' being surjective, we have Cp .Y / D n2! Dn , i.e., the
uniform space Cp .Y / is  -totally bounded, so Y is pseudocompact by Problem 136.
If X is compact and metrizable then Y is pseudocompact by what we proved
before. Besides, nw.Cp .X // D nw.X / D w.X /  !; since network weight is not
raised even by continuous maps, we have nw.Cp .Y // D ! and hence nw.Y / D !.
Therefore Y is compact (and metrizable).
Finally consider the compact space X D !1 C 1 and let Y D !1  X . Then the
restriction map  W Cp .X / ! Cp .Y / is continuous and linear, so  is uniformly
continuous. If follows from TFS-314 that every bounded continuous function on Y
can be continuously extended to X ; since the space Y is countably compact, this
implies .Cp .X // D Cp .Y /. Therefore Y is a non-compact space such that Cp .Y /
is a linear (and hence uniform) continuous image of Cp .X / for a compact space X .
V.194. Assume that X and Y are metrizable spaces and there exists either a
uniformly continuous surjection of Cp .X / onto Cp .Y / or a uniformly continuous
surjection of Cp .X / onto Cp .Y /. Prove
S that there exists a family fYn W n 2 !g of
closed subspaces of Y such that Y D n2! Yn and each Yn can be perfectly mapped
onto a closed subspace of X
kn (with the Vietoris topology) for some kn 2 N.

2 Solutions of problems 001500

231

Solution. Given a set A the family Fin.A/ consists of all finite subsets of A; for
each n 2 ! let A
n D fB 2 Fin.A/ W jBj D ng. For any space Z the function
0Z 2 Cp .Z/ is defined by 0Z .z/ D 0 for all z 2 Z. For each n 2 N the space Z
n
is assumed to carry the Vietoris topology.
Suppose that either T W Cp .X / ! Cp .Y / or T W Cp .X / ! Cp .Y / and T is
a uniformly continuous surjection. Our proof will be valid for both cases, so let us
denote by E.X / and E.Y / the respective domain and range of T . Since 0X 2 E.X /
and 0Y 2 E.Y /, there is no loss of generality to assume that T .0X / D 0Y . For every
y 2 Y let by .f / D f .y/ for all f 2 E.Y /. For any K 2 Fin.X / and " > 0 the set
OX .K; "/ D ff 2 E.X / W f .K/  ."; "/g is an open neighborhood of 0X and
the family fOX .K; "/ W K 2 Fin.X / and " > 0g is a local base of the space E.X /
at the point 0X .
It follows from Fact 1 of V.178 that
(1) by W E.Y / ! R is a uniformly continuous unbounded map and hence the map
y D by T is also uniformly continuous and unbounded for every y 2 Y .
Fact 1. Given a point y 2 Y and a finite subset K of the space X let
u.y; K/ D supfjy .f /  y .g/j W f; g 2 E.X / and jf .x/  g.x/j < 1 for every x 2 Kg:

Then the family E.y/ D fK 2 Fin.X / W u.y; K/ < 1g is nonempty.


Proof. It follows from uniform continuity of y that there is a finite set K  X
and " > 0 such that f  g 2 OX .K; "/ implies jy .f /  y .g/j < 1. Fix n 2
N with n" > 1 and take any f; g 2 E.X / such that jf .x/  g.x/j < 1 for all
x 2 K. The function hi D f C ni .g  f / belongs to E.X / for every i  n and
it is straightforward that jhiC1 .x/  hi .x/j < n1 < " for all x 2 K and hence
 y .hi /j < 1 for all i < n. Now, h0 D f and hn D g, so jy .g/ 
jy .hiC1 / P
y .f /j  i<n jy .hiC1 /  y .hi /j < n which shows that u.y; K/  n and hence
K 2 E.y/, i.e., Fact 1 is proved.
Fact 2. For any point y 2 Y the empty set does not belong to E.y/; besides, if
K0 ; K1 2 E.y/ then u.y; K0 \ K1 /  u.y; K0 / C u.y; K1 / and hence K0 \ K1 2
E.y/.
Proof. Observe that OX .;; 1/ D E.X /; since y is unbounded on E.X / by (1), the
set fjy .f /j D jy .f /  y .0X /j W f 2 E.X /g is unbounded so u.y; ;/ D 1 and
hence ; E.y/. Now, if K0 ; K1 2 E.y/ then let K D K0 \ K1 and consider any
f; g 2 E.X / such that jf .x/  g.x/j < 1 for all x 2 K.
The set Ix D .f .x/1; f .x/C1/\.g.x/1; g.x/C1/ is open and nonempty, so
choose qx 2 Ix for any x 2 K. For every x 2 K0 nK take qx 2 .f .x/1; f .x/C1/;
if x 2 K1 nK then pick qx 2 .g.x/  1; g.x/ C 1/. It is easy to find a function
h 2 E.X / for which h.x/ D qx for every x 2 K0 [ K1 ; it is immediate that f  h 2
OX .K0 ; 1/ and h  g 2 OX .K1 ; 1/. Therefore jy .f /  y .h/j  u.y; K0 / and
jy .h/  y .g/j  u.y; K1 / whence jy .f /  y .g/j  u.y; K0 / C u.y; K1 /. This
shows that u.y; K/  u.y; K0 / C u.y; K1 /, so we proved all statements formulated
for E.y/ and hence Fact 2 is proved.

232

2 Solutions of problems 001500

Fact 3. For any y 2 Y there exists a unique minimal element K.y/ in the family
E.y/ with respect to inclusion; let u.y/ D u.y; K.y//.
Proof. The elements of E.y/ are finite, so there exists a minimal element K 2 E.y/;
if K0 is another minimal element of E.y/ then K \ K0 is strictly smaller than K;
since K \ K0 2 E.y/ by Fact 2, we have a contradiction with minimality of K.
Therefore there is a unique minimal element of E.y/, so Fact 3 is proved.
Fact 4. For any p; q 2 N let Y.p; q/ D fy 2 Y W there exists K 2 E.y/ such that
jKj  q and u.y; K/  pg. Given p 2 N consider the set M.p; 1/ DSY.p; 1/; for
every natural q > 1 let M.p; q/ D Y.p; q/nY.2p; q  1/. If M.p/ D fM.p; q/ W
q 2 Ng then
(i) for any
Sy 2 Y and p 2 N such that u.y/  p we have y 2 M.p/; in particular,
Y D fM.p/ W p 2 Ng.
(ii) If we have p 2 N and distinct q0 ; q1 2 N then M.p; q0 / \ M.p; q1 / D ;.
Proof. If q D jK.y/j then u.y/ D u.y; K.y//  p, so y 2 Y.p; q/. The set
K.y/ being minimal in E.y/, the point y does not belong to Y.2p; q  1/, so y 2
M.p; q/  M.p/; this proves (i).
Now assume that p 2 N and take distinct q0 ; q1 2 N; there is no loss of generality
to assume that q0 < q1 and hence q0  q1 1. It follows from the relevant definitions
that M.p; q0 /  Y.p; q0 /  Y.p; q1  1/  Y.2p; q1  1/  Y nM.p; q1 / and
therefore M.p; q0 / \ M.p; q1 / D ;, so we settled (ii) and hence Fact 4 is proved.
Fact 5. Suppose that p 2 N and y 2 M.p/; denote by q.p/ the unique natural
number such that y 2 M.p; q.p//. Then there exists a unique set Kp .y/  X such
that jKp .y/j D q.p/ and u.y; Kp .y//  p.
Proof. Observe that q.p/ is unique because the family fM.p; q/ W q 2 Ng is disjoint
by Fact 4. Let q D q.p/; by the definition of Y.p; q/ there exists a set K  X such
that jKj  q and a.y; K/  p. It follows from y Y.2p; q  1/  Y.p; q  1/ that
jKj D q. Now, if K 0 K; a.y; K 0 /  p and jK 0 j D q then K 00 D K \ K 0 has at
most .q1/-many elements and a.y; K 00 /  2p (see Fact 2) which is a contradiction
with y Y.2p; q  1/. This establishes uniqueness of the set Kp .y/ D K, so Fact 5
is proved.
Fact 6. Given p 2 N assume that fyn W n 2 !g  Y is a sequence which converges
to a point y 2 M.p/; if Qn  X is a finite set such that u.yn ; Qn /  p for every
n 2 ! then for every U 2 .X / with U \ Kp .y/ ; there exists m 2 ! such that
U \ Qn ; for all n  m.
Proof. If the set A D fn 2 ! W Qn \ U D ;g is infinite then we can pass to the
subsequence fyn W n 2 Ag to see that we can assume, without loss of generality, that
Qn \ U D ; for all n 2 !. Fix q 2 N such that y 2 M.p; q/; then jKp .y/j D q.
If K D Kp .y/nU then jKj < q, so there exist functions f; g 2 E.X / such that
jf .x/  g.x/j < 1 for all x 2 K while jy .f /  y .g/j > 2p. Take a function

2 Solutions of problems 001500

233

h 2 E.X / such that hj.X nU / D f j.X nU / and jh.x/  g.x/j < 1 for all x 2
U \ Kp .y/. Then jh.x/  g.x/j < 1 for all x 2 Kp .y/, so jy .h/  y .g/j  p.
An immediate consequence is that jy .h/  y .f /j > p.
On the other hand, it follows from hjQn D f jQn that
. / jyn .h/  yn .f /j  p for all n 2 !.
The function T .h/ is continuous on Y , so the sequence fT .h/.yn / W n 2 !g
converges to T .h/.y/ D y .h/. Since yn .h/ D T .h/.yn / for every n 2 !, the
sequence fyn .h/ W n 2 !g converges to y .h/. Analogously, yn .f / ! y .f /,
so we can apply . / to conclude that jy .h/  y .f /j  p; this contradiction
demonstrates that only finitely many elements of the sequence fQn W n 2 !g miss
the set U , so Fact 6 is proved.
Fact 7. The set Y.p; q/ is closed in Y for any p; q 2 N.
Proof. Suppose that yn 2 Y.p; q/ for all n 2 ! and yn ! y. We will pass several
times to a subsequence of the sequence S D fyn W n 2 !g; since our aim is to prove
that y 2 Y.p; q/, at each step we will identify the obtained subsequence with S
considering that all elements of S have the property we have found in a subsequence.
Fix a set Qn  X such that jQn j  q and u.yn ; Qn /  p for every n 2 !. Passing
to a subsequence of S if necessary, we can assume that jQn j D k  q for all n 2 !.
Next, use Fact 2 of U.337 to choose an infinite A  ! for which there is a
set D D fd1 ; : : : ; dr g  X such that Qn \ Qm D D for distinct n; m 2 A
(observe that it is possible that r D 0 in which case D D ;). According to
the above mentioned politics we can consider that, for any i 2 !, we have
i
Qi D fd1 ; : : : ; dr ; a1i ; : : : ; akr
g and the family fQi nD W i 2 !g is disjoint.
An evident property of metric spaces is that any sequence contains either a
convergent subsequence or an infinite closed discrete subspace. This makes it
possible to pass to a subsequence of S once more to guarantee that, for any
j 2 f1; : : : ; k  rg, the sequence Sj D faji W i 2 !g is either convergent
or constitutes a closed discrete subspace of X . If Sj is convergent then denote
by xj its limit. Renumbering every Qi if necessary we can assume that Qi D
i
i
i
fd1 ; : : : ; dr ; a1i ; : : : ; ali ; alC1
; : : : ; akr
g while the set A D falCj
W i 2 !; 1 
j  k  r  lg is closed and discrete in X and the sequence Sj converges to xj for
any j 2 f1; : : : ; lg.
Consider the set Q D fd1 ; : : : ; dr ; x1 ; : : : ; xl g; since jQj  q, it suffices to show
that u.y; Q/  p. To do this, fix f0 ; g0 2 E.X / such that jf0 .x/  g0 .x/j < 1 for
every x 2 Q. Given an arbitrary " > 0 there exists a finite set E  Q and 2 .0; 1/
such that f  g 2 OX .E; / implies jy .f /  y .g/j < ". The set A being closed
and discrete in X we can find U 2 .E; X / such that U \ A is a finite set.
Choose h 2 E.X / such that h.E/  f1g and h.X nU /  f0g; consider the
functions f1 D hf0 and g1 D hg0 . It follows from f0 jE D f1 jE and g0 jE D g1 jE
that
. / jy .f0 /  y .f1 /j < " and jy .g0 /  y .g1 /j < ".

234

2 Solutions of problems 001500

We also have f1 jQ D f0 jQ and g1 jQ D g0 jQ, so jf1 .x/  g1 .x/j < 1 for every
x 2 Q. Therefore W D fx 2 X W jf1 .x/  g1 .x/j < 1g is an open neighborhood of
the set Q and so is the set W 0 D W \ U . There is m 2 ! such that, for all i  m,
we have aji 2 W 0 for all j 2 f1; : : : ; lg and aji U for all j 2 fl C 1; : : : ; k  rg.
As a consequence, for each i  m we have jf1 .x/  g1 .x/j < 1 for every point
i
i
x 2 D [fa1i ; : : : ; ali g; besides, f1 .x/ D g1 .x/ D 0 whenever x 2 falC1
; : : : ; akr
g,
so jf1 .x/  g1 .x/j < 1 for all x 2 Qi .
By the choice of the sets Qi , we have jyi .f1 /  yi .g1 /j  p for all i  m.
We already saw that yi .f1 / ! y .f1 / and yi .g1 / ! y .g1 / as i ! 1. Passing
to the limit in the last inequality, we conclude that jy .f1 /  y .g1 /j  p. This,
together with . /, implies that jy .f0 /  y .g0 /j  p C 2". The number " > 0
was taken arbitrarily so jy .f0 /  y .g0 /j  p and hence y 2 Y.p; q/, i.e., we
established that the set Y.p; q/ is closed in X , so Fact 7 is proved.
Fact 8. The map Kp W M.p; q/ ! X
q is continuous for any p; q 2 N.
q
Proof. Given a set U 2 .X / consider the families I.U
S / D fK 2 X
W K  U g
q
and J.U / D fK 2 X
W K \ U ;g. Then S D fI.U / [ J.U / W U 2 .X /g
is a subbase of X
q , so it suffices to show that .Kp /1 .W / is open in M.p; q/ for
any W 2 S.
Fix and open set U  X , let W D I.U / and assume that G D .Kp /1 .W / is
not open in M.p; q/. Then there is a sequence fyn W n 2 !g  M.p; q/nG which
converges to a point y 2 G. Let Kp .y/ D fx1 ; : : : ; xq g and choose disjoint sets
U1 ; : : : ; Uq 2 .X / such that xi 2 Ui for all i  q and U 0 D U1 [ : : : [ Uq  U .
We have jKp .yn /j D q and u.yn ; Kp .yn //  p for each n 2 ! (see Fact 5).
For every i  q it follows from Kp .y/ \ Ui ; that we can apply Fact 6 to find
mi 2 ! such that Ui \ Kp .yn / ; for all n  mi . If m D m1 C : : : C mq then, for
any n  m the set Kp .yn / meets Ui for all i  q. This, together with jKp .yn /j D q
implies that Kp .yn /  U 0  U for all n  m, i.e., yn 2 G which is a contradiction.
Thus .Kp /1 .I.U // is open in M.p; q/ for every U 2 .X /.
Now assume that U 2 .X / and W D J.U / while G D .Kp /1 .W / is not
open in M.p; q/. Then there is a sequence fyn W n 2 !g  M.p; q/nG which
converges to a point y 2 G; let Kp .y/ D fx1 ; : : : ; xq g. We have jKp .yn /j D q
and u.yn ; Kp .yn //  p for each n 2 ! (see Fact 5); it follows from y 2 G that
Kp .y/ \ U ;, so Fact 6 can be applied to see that there is m 2 ! such that
Kp .yn / \ U ; and hence yn 2 G for all n  m; this contradiction shows that
.Kp /1 .J.U // is open in M.p; q/ for all U 2 .X /, i.e., the map Kp W M.p; q/ !
X
q is continuous and hence Fact 8 is proved.

Fact 9. Given metrizable spaces Z and T and a continuous map f W Z ! T the


following conditions are equivalent:
(a) f is an almost perfect map, i.e., f .F / is closed in T for any closed F  Z and
f 1 .t / is compact for any t 2 T (recall that an almost perfect continuous map
is perfect if and only if it is surjective);
(b) a sequence S D fzn W n 2 !g  Z has a convergent subsequence if and only if
the sequence S 0 D ff .zn / W n 2 !g  T has a convergent subsequence.

2 Solutions of problems 001500

235

Proof. If fznk W k 2 !g is a convergent subsequence of S then ff .znk / W k 2 !g is


a convergent subsequence of S 0 by continuity of f , so (a)H)(b) (observe that we
dont need the map f to be almost perfect to obtain this implication).
Now assume that (b) holds and fix t 2 T . If f 1 .t / is not compact then there
is an infinite set D  f 1 .t / which is closed and discrete in Z; let fdn W n 2 !g
be a faithful enumeration of D. Since f .dn / D t for every n 2 !, the sequence
ff .dn / W n 2 !g is convergent while the sequence fdn W n 2 !g has no convergent
subsequences. The obtained contradiction shows that f 1 .t / is compact for any
t 2 T.
To see that f is closed take a closed subset F  Z and assume that G D f .F /
is not closed in T . Then there is a sequence ftn W n 2 !g  G which converges to a
point t 2 T nG. Pick zn 2 F with f .zn / D tn for each n 2 !. Since the property (b)
holds, there exists a subsequence fznk W k 2 !g of the sequence fzn W n 2 !g which
converges to a point z. The set F being closed in Z we have z 2 F . By continuity
of f the sequence ff .znk / W k 2 !g D ftnk W k 2 !g converges to f .z/ 2 G.
However, ftnk W k 2 !g has to converge to t f .z/ being a subsequence of the
sequence ftn W n 2 !g. This contradiction demonstrates that f .F / is closed in T for
any closed F  Z, so Fact 9 is proved.
Fact 10. Suppose that F is a closed subset of Y such that F  M.p; q/ for some
p; q 2 N. Then the map Kp W F ! X
q is almost perfect.
Proof. Fix a sequence S D fyn W n 2 !g  F ; if S has a convergent subsequence,
then the sequence S 0 D fKp .yn / W n 2 !g also has a convergent subsequence by
continuity of Kp (see Fact 8).
Now assume that S 0 has a convergent subsequence and S does not. Passing
to the relevant subsequence of S we can assume, without loss of generality, that
S 0 converges to a point K D fx1 ; : : : ; xq g 2 X
q while S has no convergent
subsequences and hence the set S is closed and discrete in F ; therefore S is also
closed and discrete in Y . There is no loss of generality to assume that yn ym if
n m.
Let A be an uncountable almost disjoint family of infinite subsets of ! (see TFS141). The space Y being metrizable, the set S is C -embedded in Y , so we can
choose, for any A 2 A, a function gA 2 E.Y / such that gA .yn / D p C 1 for all
n 2 A and gA .yn / D 0 whenever n 2 !nA. Fix a function fA 2 E.X / such that
T .fA / D gA for every A 2 A.
Let rA D .fA .x1 /; : : : ; fA .xq // 2 Rq for every A 2 A. The family A being
uncountable the set frA W A 2 Ag cannot be discrete of cardinality jAj, so there
are distinct A; B 2 A such that the distance in Rq between the points rA and rB is
strictly less than 1 and hence jfA .xi /  fB .xi /j < 1 for all i  q.
By continuity of fA  fB , the set W D fx 2 X W jfA .x/  fB .x/j < 1g is
an open neighborhood of xi for every i  q, so we can choose a disjoint family
fO1 ; : : : ; Oq g  .X / such that xi 2 Oi  W for all i  q. The sequence S 0 being
convergent to K there is m 2 ! such that Kp .yn / \ Oi ; for all i  q and

236

2 Solutions of problems 001500

S
n  m; recalling that jKp .yn /j D q, we conclude that Kp .yn /  iq Oi  W
whence jfA .x/  fB .x/j < 1 for all x 2 Kp .yn / and n  m.
As a consequence, jgA .yn /  gB .yn /j D jyn .fA /  yn .fB /j  p which shows
that fyn W n  mg  A \ B because jgA .yn /  gB .yn /j D p C 1 for all numbers
n 2 .AnB/ [ .BnA/. This contradiction with jA \ Bj < ! shows that S have
a convergent subsequence and hence we can apply Fact 9 to conclude that Kp is
almost perfect map, i.e., Fact 10 is proved.
Returning to our solution observe that it follows from Fact 7 that
S M.p; q/ is
an F -subset of Y for all p; q 2 N. Apply Fact 4 to see that Y D fM.p; q/ W
p; q 2SNg, so we can find a family fYn W n 2 !g of closed subsets of Y such that
Y D n2! Yn and every Yn is contained in some M.p; q/.
Fix n 2 ! and p; q 2 N such that Yn  M.p; q/; the map Kp W Yn ! X
q is
almost perfect by Fact 10 and, in particular, the set Fn D Kp .Yn / is closed in X
q .
If kn D q then Kp W Yn ! Fn is a perfect map of Yn onto a closed subspace Fn of
the space X
kn , so our solution is complete.
V.195. Let P be a class of metrizable spaces with the following properties:
(1) P contains all complete metrizable spaces;
(2) P is invariant under finite products andSclosed subspaces;
(3) if M is a metrizable space with M D fMn W n 2 !g, where Mn is closed in
M and Mn 2 P for each n 2 !, then M 2 P.
Suppose that X 2 P and Y is a metrizable space. Prove that, if there exists a
uniformly continuous surjection of Cp .X / onto Cp .Y / (or Cp .X / onto Cp .Y /),
then Y 2 P.
Solution. Fix a number n 2 N and consider the space X
n to carry the Vietoris
topology.SThere exists a family fFm W m 2 !g of closed subsets of X
n such that
X
n D m2! Fm and every Fm is homeomorphic to a closed subspace of X n (see
Problem 192).
It follows from X 2 P that every closed subspace of X n belongs to P, so each
Fm belongs to P and hence X
n 2 P for every n 2 N.
Apply Problem
194 to find a family fYn W n 2 !g of closed subspaces of Y such
S
that Y D n2! Yn and, for every n 2 !, there exist kn 2 N and a perfect map
fn W Yn ! Fn for some closed Fn  X
kn ; we can consider that Yn  Mn for some
complete metric space Mn (see TFS-237).
Given any n 2 ! the graph G D f.y; fn .y// W y 2 Yn g is closed in Yn  Fn and
homeomorphic to Yn (see Fact 4 of S.390). It turns out that G is closed in Mn  Fn ;
to prove this assume toward a contradiction that a point .x; y/ 2 .Mn  Fn /nG
belongs to the closure of G. Since G is closed in Yn  Fn , the point .x; y/ cannot
belong to Yn  Fn , so x Yn . There exists a set fxm W m 2 !g  Yn such that
the sequence f.xm ; fn .xm // W m 2 !g converges to .x; y/. Thus xm ! x Yn ,
so the set D D fxm W m 2 !g is closed and discrete in Yn . Since the sequence D

2 Solutions of problems 001500

237

converges to a point outside of D, the set D cannot be finite, so we can pass to a


subsequence to see that we can assume, without loss of generality, that xm xk
whenever m k.
Given any z 2 Fn the set fn1 .z/ is compact, so jD \ fn1 .z/j < !; an immediate
consequence is that the set f .D/ is infinite. By continuity of the projection onto Fn ,
the sequence S D ffn .xm / W m 2 !g must converge to y. The set S being infinite,
it is a nontrivial convergent sequence. However, S D fn .D/ must be closed and
discrete in Fn because the map fn is closed; this contradiction shows that G is
closed in Mn  Fn . Observe that Mn 2 P and Fn 2 P, so G 2 P and hence
Yn 2 P for any n 2 !. Recalling that countable unions of closed subsets preserve
the property P we conclude that Y 2 P.
V.196. Let P be a class of second countable spaces such that
(1) every compact metrizable space belongs to P;
(2) P is invariant under finite products and closed S
subspaces;
(3) if M is a second countable space with M D fMn W n 2 !g, where Mn is
closed in M and Mn 2 P for each n 2 !, then M 2 P.
Suppose that X 2 P and Y is a metrizable space. Prove that, if there exists a
uniformly continuous surjection of Cp .X / onto Cp .Y / (or Cp .X / onto Cp .Y /),
then Y 2 P.
Solution. Fix a number n 2 N and consider the space X
n to carry the Vietoris
topology.SThere exists a family fFm W m 2 !g of closed subsets of X
n such that
X
n D m2! Fm and every Fm is homeomorphic to a closed subspace of X n (see
Problem 192). It follows from X 2 P that every closed subspace of X n belongs to
P, so each Fm belongs to P and hence X
n 2 P for every n 2 N.
If there exists a (uniformly) continuous surjection T W Cp .X / ! Cp .Y / then
nw.Cp .Y //  nw.Cp .X // D nw.X / D !, so nw.Y / D nw.Cp .Y // D ! and
hence Y is second countable. If there is a continuous surjection T W Cp .X / !
Cp .Y / then nw.Cp .Y //  nw.Cp .X //  nw.X / D !. Since Cp .Y; .0; 1// 
Cp .Y / is homeomorphic to Cp .Y / we conclude that nw.Cp .Y //  nw.Cp .Y // D
!, so nw.Y / D nw.Cp .Y // D ! and hence Y is second countable. Thus Y is
second countable in all possible cases.
Apply Problem
194 to find a family fYn W n 2 !g of closed subspaces of Y such
S
that Y D n2! Yn and, for every n 2 !, there exist kn 2 N and a perfect map
fn W Yn ! Fn for some closed Fn  X
kn ; we can consider that Yn  Mn for some
compact metric space Mn (see TFS-209).
Given any n 2 ! the graph G D f.y; fn .y// W y 2 Yn g is closed in Yn  Fn and
homeomorphic to Yn (see Fact 4 of S.390). It turns out that G is closed in Mn  Fn ;
to prove this assume toward a contradiction that a point .x; y/ 2 .Mn  Fn /nG
belongs to the closure of G. Since G is closed in Yn  Fn , the point .x; y/ cannot
belong to Yn  Fn , so x Yn . There exists a set fxm W m 2 !g  Yn such that
the sequence f.xm ; fn .xm // W m 2 !g converges to .x; y/. Thus xm ! x Yn ,
so the set D D fxm W m 2 !g is closed and discrete in Yn . Since the sequence D

238

2 Solutions of problems 001500

converges to a point outside of D, the set D cannot be finite, so we can pass to a


subsequence to see that we can assume, without loss of generality, that xm xk
whenever m k.
Given any z 2 Fn the set fn1 .z/ is compact, so jD \ fn1 .z/j < !; an immediate
consequence is that the set f .D/ is infinite. By continuity of the projection onto Fn ,
the sequence S D ffn .xm / W m 2 !g must converge to y. The set S being infinite,
it is a nontrivial convergent sequence. However, S D fn .D/ must be closed and
discrete in Fn because the map fn is closed; this contradiction shows that G is
closed in Mn  Fn . Observe that Mn 2 P and Fn 2 P, so G 2 P and hence Yn 2 P
for any n 2 !. Recalling that countable unions (of closed subsets) preserve the
property P we conclude that Y 2 P.
V.197. Given a countable ordinal , let M be the class of absolute Borel sets of
multiplicative class . Suppose that X is a metrizable space such that X 2 M
for some  2. Let Y be a metrizable space such that Cp .Y / (or Cp .Y /) is a
uniformly continuous image of Cp .X / (or Cp .X / respectively). Prove that Y 2
u

M . In particular, if X  Y then X belongs to M if and only if so does Y .


Solution. If M is completely metrizable then M is closed in M , so M 2 0 .M /
which shows that we can apply Problem 183 to conclude that M 2 M ; this proves
that the class M contains all completely metrizable spaces.
Suppose that Z 2 M and F is a closed subset of Z. There exists a complete
metric space M such that Z is homeomorphic to some Z 0 2 0 .M / (see
Problem 183). Then F is homeomorphic to some closed subspace F 0 of the space Z 0
and therefore we can find a closed subset P of the space M such that P \ Z 0 D F 0 .
Since P 2 0 .M /, we can apply Fact 1 of T.341 to see that F 0 2 0 .M / and hence
F 2 M by Problem 183. Thus the class M is invariant under closed subspaces;
by Problem 185, it is also invariant under finite products.S
Now suppose that M is a metrizable space, M D n2! Mn while every Mn
is closed in M and belongs to M . If D 1 < then Mn 2 0 .M / for every
n 2 !, so we can apply Problem 186 to see that M 2 M . Therefore the class M
satisfies all conditions we need to apply Problem 195 and conclude that if X belongs
to M and there exists a uniformly continuous surjection ' W Cp .X / ! Cp .Y / or
a uniformly continuous surjection  W Cp .X / ! Cp .Y / then Y 2 M .
V.198. Given a countable ordinal , let A be the class of absolute Borel sets of
additive class . Suppose that X is a metrizable space such that X 2 A for some
 2. Let Y be a metrizable space such that Cp .Y / (or Cp .Y /) is a uniformly
continuous image of Cp .X / (or Cp .X / respectively). Prove that Y 2 A . In
u

particular, if X  Y then X belongs to A if and only if so does Y .


Solution. If M is completely metrizable then M is closed in M , so M 2 0 .M /
which shows that we can apply Problem 184 to conclude that M 2 A ; this proves
that the class A contains all completely metrizable spaces.
Suppose that Z 2 A and F is a closed subset of Z. There exists a complete
metric space M such that Z is homeomorphic to some Z 0 2 0 .M / (see
Problem 184). Then F is homeomorphic to some closed subspace F 0 of the space Z 0

2 Solutions of problems 001500

239

and therefore we can find a closed subset P of the space M such that P \ Z 0 D F 0 .
Since P 2 0 .M /, we can apply Fact 1 of T.341 to see that F 0 2 0 .M / and hence
F 2 A by Problem 184. Thus the class A is invariant under closed subspaces; by
Problem 185, it is also invariant under finite products.
S
Now suppose that M is a metrizable space and M D n2! Mn while every
Mn is closed in M and belongs to the class A . If e W M ! N is an arbitrary
embedding of M in a metrizable space N thenS
e.Mn / 2 0 .N / for every n 2 !, so
it follows from Fact 1 of T.341 that e.M / D n2! e.Mn / 2 0 .N /. This implies
that M 2 A and therefore the class A satisfies all conditions we need to apply
Problem 195 and conclude that if X belongs to A and there exists a uniformly
continuous surjection ' W Cp .X / ! Cp .Y / or a uniformly continuous surjection
 W Cp .X / ! Cp .Y / then Y 2 A .
V.199. Prove that every nonempty countable compact space X is homeomorphic to
the space C 1 D f W  g for some countable ordinal . Here, as usual, the
set C 1 is considered with the topology generated by the well-ordering on C 1.
Solution. The expression Y 'L
Z says that the spaces Y and Z are homeomorphic.
In every discrete union Z D
fZt W t 2 T g we identify every set Zt with the
respective clopen subspace of Z. If and are ordinals such that < then
.; / D f W <  < g is the usual interval of ordinals. Given two linearly
ordered spaces L and M we say that they are canonically homeomorphic if there
exists a homeomorphism f W L ! M which is also an order isomorphism between
L and M .
If K ; is a countable compact space then K is scattered (see SFFS-133);
therefore the set I.K/ D fx 2 K W x is an isolated point of Kg is nonempty.
Let K0
D K; proceeding inductively assume that < !1 and we have a family
fK
W < g of closed subspaces of K with the following properties:
(1) K C 1
D K
nI.K
/ whenever 0T
 < C 1 < ;
(2) if < is a limit ordinal then K
D fK
W  < g.
T
If is a limit ordinal, let K
D fK
W < g. If D C 1 then
let K
D K
nI.K
/. It is clear that the properties (1)(2) still hold for the
family fK
W  g, so our inductive procedure can be carried out for all < !1 .
Observe that if K
; then I.K
/ ; and hence the set K C 1
is strictly
smaller than K
. Since K is countable, there is < !1 for which K
D ;; let
be the minimal such . Observe that every K
is compact, so it follows from (2)
that is always a non-limit ordinal; let  be the predecessor of . The ordinal  is
called the dispersion index of K and we will denote it by d i.K/; it is evident that
K
is a nonempty finite set.
Fact 1. For any ordinals and there exists an ordinal number  > such that
. C 1/n. C 1/ is canonically homeomorphic to C 1 and hence  C 1 ' . C
1/ . C 1/.
Proof. Let  0 D . C 1/ C . C 1/; it follows from the definition of the sum
of ordinals that the set B D  0 n. C 1/ is canonically homeomorphic to C 1.

240

2 Solutions of problems 001500

Furthermore, C 1 D f 2  0 W  < C 1g and B D f 2  0 W  > g, so


C 1 and B are clopen disjoint subspaces of the ordinal  0 which shows that  0 is
homeomorphic to . C 1/ B ' . C 1/ . C 1/. The set B being isomorphic
to C 1 it has a maximal element  ; it is evident that  is also the maximal element
of  0 , so  0 D  C 1 and hence Fact 1 is proved.
Fact 2. If a compact space Xi is homeomorphic to an ordinal i C 1 < !1 for all
i D 0; : : : ; n then there exists an ordinal < !1 such that X D X0 : : : Xn is
homeomorphic to C 1.
Proof. Proceeding by induction observe that our statement is trivially true for
n D 0. Assume that it is true for n  k 2 ! and consider countable compact
spaces X0 ; : : : ; Xk ; XkC1 such that Xi is homeomorphic to some ordinal i C 1 for
every i  k C 1. By the induction hypothesis the space Y D X0 : : : Xk is
homeomorphic to an ordinal C 1 < !1 . Apply Fact 1 to find an ordinal (which
is, evidently, countable) such that C1 is homeomorphic to . C1/.kC1 C1/ '
Y XkC1 ' X . Therefore the space X is homeomorphic to C 1 which shows
that we completed the inductive step and hence our statement is true for all n 2 !,
i.e., Fact 2 is proved.
Fact 3. If a nonempty compact space Xi is homeomorphic to a countable ordinal
i C 1 for all i 2 ! then there L
exists an ordinal < !1 such that the one-point
compactification X of the space i2! Xi is homeomorphic to C 1.
Proof. Using Fact 1 it easy to construct by induction a sequence fi W i 2 !g of
countable ordinals with the following properties:
(3) i < iC1 for every i 2 !;
(4) 0 D 0 and BiC1 D .iC1 C 1/n.i C 1/ is homeomorphic to iC1 C 1 for all
i 2 !.
Let D supfi W i 2 !g; it follows from .i ; iC1 C 1/ \ D Bi that Bi is an
open subset of for all i 2 N. If B0 D 0 C 1 then the family fBi W i 2 !g is a
disjoint open cover of , so every Bi is a clopen subset of . RecallingL
that Bi ' Xi
for each i 2 ! we convince ourselves that is homeomorphic to i2! Xi . The
compact space C 1 is obtained from
L by adding one point; therefore C 1 is the
one-point compactification of '
i2! Xi . Thus C 1 is homeomorphic to X
and hence Fact 3 is proved.
Returning to our solution call a space X adequate if it is homeomorphic to C 1
for some countable ordinal . We will prove that every countable compact space
X is adequate by induction on the dispersion index of X . If d i.X / D 0 then X is
finite, so X is homeomorphic to the ordinal C 1 for D jX j  1. Now assume
that  D d i.X / > 0 and we proved that every nonempty compact countable space
Y is adequate whenever d i.Y / < .
Let fa1 ; : : : ; ak g be a faithful enumeration of the set X 
; the space X being
zero-dimensional, it is easy to find disjoint clopen subsets O1 ; : : : ; Ok of the space
X such that ai 2 Oi for all i  k and O1 [ : : : [ Ok D X . It is immediate that X

2 Solutions of problems 001500

241

is homeomorphic to O1 : : : Ok , so it suffices to prove that every Oi is adequate


(see Fact 2). It is straightforward that Oi 
D X 
\ Oi , so Oi 
D fai g for
every i  k. Therefore we can assume, without loss of generality, that X D O1 and
hence X 
D fag for some a 2 X .
Observe that the point a is not isolated in X because  > 0, so we can construct
a family fUn W n 2 !g of clopen neighborhoods of the point a such that U0 D X ,
a 2 UnC1  Un and Kn D Un nUnC1 ; for all n 2 !. It follows from the
inclusion Kn 
 X 
\ Kn D ; that d i.Kn / <  and hence we can apply the
induction hypothesis to see that Kn is adequate for every n 2 !. Every Kn is clopen
in X and hence in X nfag; since fKn W nL2 !g is a disjoint cover of X nfag, we
conclude that X nfag is homeomorphic to n2! L
Kn . Therefore X is homeomorphic
to the one-point compactification of the space n2! Kn , so we can apply Fact 3
to convince ourselves that X is adequate. This completes our inductive proof and
shows that every countable compact space is homeomorphic to C1 for a countable
ordinal .
u

V.200. Let X and Y be infinite countable compact spaces. Prove that X  Y , i.e.,
the spaces Cp .X / and Cp .Y / are uniformly homeomorphic.
Solution. The expression X ' Y says that the spaces X and Y are homeomorphic;
the function 0X is defined on X by letting 0X .x/ D 0 for any x 2 X . If x; y 2 R2
then x; y
D f.1  t /x C ty W t 2 0; 1
g is the segment in R2 that connects the
points x and y whereas for any > 0 we let B D f.x0 ; x1 / 2 R2 W x02 C x12 < 2 g.
Given ordinals ; with  we will need the intervals ;
D f W    g
and ; / D f W   < g. Recall that if we are given a nonempty set A then
 .A/ D fx 2 RA W jfa 2 A W jx.a/j  "gj < ! for any " > 0g.
We will often use norms on different spaces using the same symbol jj jj. This
wont lead to a confusion because we never use distinct norms on the same space.
If K is a compact space then jjf jj D maxfjf .x/j W x 2 Kg for any f 2 Cp .K/.
Given a nonempty set A let jjxjj D maxfjx.a/j W a 2 Ag for any x 2  .A/. For
any n 2 N and x D .x0 ; : : : ; xn1 / 2 Rn let jjxjj D maxfjxi j W i < ng. Now,
if we have norms on spaces M and L then jj.p; q/jj D maxfjjpjj; jjqjjg for any
.p; q/ 2 M  L.
Fact 1. If K is a countably infinite compact space then for any point a K the
space K fag is homeomorphic to K.
Proof. By Problem 199 there exists a countable ordinal such that K ' 0;
, so
there is no loss of generality to assume that K D 0;
; the space K being infinite
we have  ! and hence 0; !
is a clopen subspace of K. Thus K ' 0; !

.Kn0; !
/. The space 0; !
is a convergent sequence with its limit, so 0; !
fag
is homeomorphic to 0; !
. Thus K fag ' .0; !
fag/ .Kn0; !
/ ' 0; !

.Kn0; !
/ ' K, so Fact 1 is proved.
Fact 2. For any " > 0 there exist functions u" W R2 ! R and v" W R2 ! R with the
following properties:

242

2 Solutions of problems 001500

(a) there exists a constant C."/ > 0 such that ju" .x/  u" .y/j  C."/jjx  yjj and
jv" .x/  v" .y/j  C."/jjx  yjj for any x; y 2 R2 ;
(b) if h.x0 ; x1 / D .x0 ; u" .x0 ; x1 // for any .x0 ; x1 / 2 R2 then h W R2 ! R2 is a
uniform homeomorphism;
(c) if g.x0 ; x1 / D .x0 ; v" .x0 ; x1 // for any .x0 ; x1 / 2 R2 then g W R2 ! R2 is a
uniform homeomorphism which is the inverse of h;
(d) u" .t; t / D 0 for any t 2 R;
(e) .1 C "/1 jj.x0 ; x1 /jj  jjh.x0 ; x1 /jj  jj.x0 ; x1 /jj for any .x0 ; x1 / 2 R2 .
Proof. For any .x0 ; x1 / 2 R2 let

u" .x0 ; x1 / D

<x1 ;

if .x1 C x0 /.x1  .1 C "/x0 /  0;

 x0 /;

:.1 C 1 /.x  x /;
1
0
"
1
.x
2 1

if jx1 j  jx0 j;
if .x1  x0 /.x1  .1 C "/x0 /  0:

Observe that u" is a union of three continuous functions defined on closed


subspaces of R2 . It is easy to check that any pair of these functions coincide on
the intersections of their domains, so u" is continuous (see Fact 2 of T.354).
Now, if
8

if .x1 C x0 /.x1  .1 C "/x0 /  0;


x1 ;
<
v" .x0 ; x1 / D 2x1 C x0 ;
if .x1 C x0 /x1  0;

: " x C x ; if x .x  .1 C "/x /  0
1C" 1

for any .x0 ; x1 / 2 R2 then the same reasoning as in the case of u" shows that the
function v" W R2 ! R is continuous. It is also straightforward that the functions h
and g defined in (b) and (c) are mutually inverse, so h is a homeomorphism. The
property (d) is evident from the definition of u" .
If u" .x0 ; x1 / D x1 or u" .x0 ; x1 / D 12 .x1  x0 / then it is evident that we have the
inequality ju" .x0 ; x1 /j  jj.x0 ; x1 /jj.
To prove the same when u" .x0 ; x1 / D .1 C 1" /.x1  x0 / assume that x1  x0 ;
then 0  x0  x1  .1 C "/x0 and therefore .1 C "/.x1  x0 /  "x1 which implies
that ju" .x0 ; x1 /j D 1C"
.x1  x0 /  x1 D jj.x0 ; x1 /jj. Now if x1  x0 is the case,
"
then .1 C "/x0  x1  x0  0 and hence .1 C "/.x1  x0 /  "x1 ; an immediate
consequence is that ju" .x0 ; x1 /j D 1C"
.x0  x1 /  x1 D jj.x0 ; x1 /jj. This proves
"
that
(1) ju" .x0 ; x1 /j  jj.x0 ; x1 /jj and hence jjh.x0 ; x1 /jj  jj.x0 ; x1 /jj for any point
.x0 ; x1 / 2 R2 .
To prove the second inequality in (e) we will establish that
(2) jv" .x0 ; x1 /j  .1 C "/jj.x0 ; x1 /jj for any .x0 ; x1 / 2 R2 .
If .x0 C x1 /.x1  .1 C "/x0 /  0 then jv" .x0 ; x1 /j D jx1 j D jj.x0 ; x1 /jj, so we
have jv" .x0 ; x1 /j  .1 C "/jj.x0 ; x1 /jj.

2 Solutions of problems 001500

243

Next assume that x1 C x0  0 and x1  0. Then jj.x0 ; x1 /jj D x0 and


v" .x0 ; x1 / D 2x1 C x0  x0  .1 C "/x0 . Besides, 2x1 C x0  x0  x0 .1 C "/
which shows that .1 C "/x0  v" .x0 ; x1 /  .1 C "/x0 and hence (2) holds in this
case.
If x1 C x0  0 and x1  0 then jj.x0 ; x1 /jj D jx0 j D x0 , so we obtain the
inequality 2x1 C x0  x0  .1 C "/.x0 /. On the other hand,
2x1 C x0  x0  .1 C "/x0 D .1 C "/jx0 jI
this demonstrates that .1 C "/jx0 j  v" .x0 ; x1 /  .1 C "/jx0 j, so we proved (2)
for the case when v" .x0 ; x1 / D 2x1 C x0 .
"
Next assume that 0  x1  .1 C "/x0 ; in this case 1C"
x1  "x0 and therefore
"
jv" .x0 ; x1 /j D 1C" x1 C x0  .1 C "/x0  .1 C "/jj.x0 ; x1 /jj. Now, if .1 C "/x0 
"
"
x1  0 then 1C"
x1  "x0 and hence 1C"
x1 C x0  .1 C "/x0 , so we convince
"
ourselves that jv" .x0 ; x1 /j D . 1C" x1 C x0 /  .1 C "/x0  .1 C "/jj.x0 ; x1 /jj,
i.e., (2) is proved.
It follows from (2) that jjg.y/jj  .1 C "/jj.y/jj for any y 2 R2 . In particular,
jjh.g.x0 ; x1 //jj  .1 C "/jjh.x0 ; x1 /jj; recalling that h is the inverse of g we
conclude that jj.x0 ; x1 /jj  .1 C "/jjh.x0 ; x1 /jj for any .x0 ; x1 / 2 R2 . This, together
with (1) shows that (e) is proved.
Given a set P  R2 say that a function ' W P ! R is Lipschitz with constant
C > 0 if we have j'.x/  '.y/j  C jjx  yjj for any x; y 2 P .
Let A0 D f.x0 ; x1 / 2 R2 W .x1 C x0 /.x1  .1 C "/x0 /  0g. Given two points
x; y 2 A0 ; x D .x0 ; x1 /; y D .y0 ; y1 / we have ju" .x/  u" .y/j D jx1  y1 j 
jjx  yjj, so u" is Lipschitz with constant C0 D 1 on A0 . Let B0 D f.x0 ; x1 / 2 R2 W
jx1 j  jx0 jg; if x; y 2 B0 then ju" .x/u" .y/j D 12 j.x1 y1 /.x0 y0 /j  jjx yjj,
so the function u" is also Lipschitz on B0 with constant C1 D 1. If
x; y 2 D0 D f.x0 ; x1 / 2 R2 W .x1  x0 /.x1  .1 C "/x0 /  0g
then
1
1
ju" .x/  u" .y/j D .1 C /j.x1  y1 /  .x0  y0 /j  2.1 C /jjx  yjj
"
"
so the function u" is Lipschitz on D0 with constant C2 D 2 C 2" . Since
C0 D C1  C2 , the function u" is Lipschitz with constant C2 on each one of the
sets A0 ; B0 and D0 .
Now take two arbitrary points x; y 2 R2 ; x D .x0 ; x1 /; y D .y0 ; y1 /. We have
A0 [B0 [D0 D R2 and the boundaries of the sets A0 ; B0 and D0 are contained in the
straight lines l1 ; l2 ; l3 given respectively by the equations x1 D .1 C "/x0 ; x1 D x0
and x1 D x0 . The intersection of the segment x; y
with the lines l1 ; l2 and l3
gives at most three points on x; y
, so we can choose t1 ; t2 ; t3 2 0; 1
such that
t1 < t2 < t3 and if ai D .1  ti /x C ti y for all i 2 f1; 2; 3g then every interval
x; a1
; a1 ; a2
; a2 ; a3
and a3 ; y
is contained in one of the sets A0 ; B0 ; D0 .

244

2 Solutions of problems 001500

Letting a0 D x and a4 D y we obtain the inequality


ju" .x/  u" .y/j 

3
X

ju" .aiC1 /  u" .ai /j  C2

iD0

3
X

jjaiC1  ai jj  4C2 jjx  yjj;

iD0

which proves that u" is Lipschitz with constant C D 4C2 on R2 . An analogous


reasoning shows that v" is also Lipschitz with some constant C 0 on R2 . Letting
C."/ D maxfC; C 0 g we conclude that both functions u" and v" are Lipschitz on R2
with constant C."/, so (a) is proved.
To finally establish (b) and (c) it is sufficient to prove that both functions g
and h are uniformly continuous. We will do this simultaneously; let ' be one of
the functions u" ; v" , so it suffices to show that the function defined by the formula
2
f .x0 ; x1 / D .x0 ; '.x0 ; x1 // for any .x0 ; xq
1 / 2 R , is uniformly continuous.
For any x D .x0 ; x1 / 2 R2 let jxj2 D

x02 C x12 and fix a set O 2 ..0; 0/; R2 /;


there exists > 0 such that B  O. If  D p
then B is also an open
2
1C.C."//

neighborhood of the point .0; 0/.


Given any points x; y 2 R2 ; x D .x0 ; x1 /; y D .y0 ; y1 / suppose that x  y 2
B . Then jjx  yjj <  and hencepj'.x/  '.y/j  C."/jjx  yjj which implies
the inequality jf .x/  f .y/j2  .x0  p
y0 /2 C .C."//2 jjx  yjj2 . Observe that
2
2
.x0 y0 /  jjxyjj , so jf .x/f .y/j2  1 C .C."//2 jjxyjj < which shows
that f .x/  f .y/ 2 B  O and hence the function f is uniformly continuous.
Thus g and h are both uniform homeomorphisms, i.e., we verified (b) and (c), so
Fact 2 is proved.
Fact 3. Given a compact space K and a 2 K let I.a/ D ff 2 Cp .K/ W f .a/ D 0g.
For any " > 0 define a map '" W Cp .K/ ! I.a/ by the formula '" .f /.x/ D
u" .f .a/; f .x// for all x 2 K (see Fact 2). If h" .f / D .f .a/; '" .f // for every
function f 2 Cp .K/ then the map h" W Cp .K/ ! R  I.a/ is a uniform
homeomorphism such that .1 C "/1 jjf jj  jjh" .f /jj  jjf jj for all f 2 Cp .K/.
Proof. The function '" .f / is continuous being the composition of continuous
functions; observe also that '" .f /.a/ D u" .f .a/; f .a// D 0 for any f 2 Cp .K/,
so '" indeed, maps Cp .K/ in I.a/. For any x 2 K the map f ! '" .f /.x/ D
u" .f .a/; f .x// is continuous being the composition of continuous maps, so '" is
continuous by TFS-102; therefore h" is also a continuous map.
For any point .t; f / 2 R  I.a/ let " .t; f /.x/ D v" .t; f .x// for any x 2 K.
Then the map " W R  I.a/ ! Cp .K/ is continuous: this is proved in the same
way as for h" . It follows from (b) and (c) of Fact 2 that
(3) u" .x0 ; v" .x0 ; x1 // D x1 and v" .x0 ; u" .x0 ; x1 // D x1 for any .x0 ; x1 / 2 R2 .
Fix .t; g/ 2 R  I.a/ and let f D " .t; g/. Observe first that
f .a/ D v" .t; g.a// D v" .t; 0/ D v" .t; u" .t; t // D t
(we used (3) and (d) of Fact 2). For every x 2 K we have '" .f /.x/ D u" .t; f .x//.
Recalling that f .x/ D v" .t; g.x// and applying (3) again we convince ourselves

2 Solutions of problems 001500

245

that '" .f /.x/ D g.x/ and hence '" .f / D g. Thus h" .f / D .t; g/ which shows
that the map h" is surjective and h" " is the identity on R  I.a/.
Now if f 2 Cp .K/ and g D '" .f / then " .f .a/; g/.x/ D v" .f .a/; g.x//;
recalling that g.x/ D u" .f .a/; f .x// we can apply (3) once more to see that we
have the equality " .f .a/; g/.x/ D f .x/ for every x 2 K, i.e., " .f .a/; g/ D f .
This proves that h" and " are mutually inverse homeomorphisms.
Once more fix any function f 2 Cp .K/; then h" .f / D .f .a/; '" .f // and,
evidently, jf .a/j  jjf jj. Besides, it follows from (e) of Fact 2 that we have the
inequalities
(4) .1 C "/1 jj.f .a/; f .x//jj  jj.f .a/; u" .f .a/; f .x//jj  jj.f .a/; f .x//jj for
any point x 2 K.
An immediate consequence of (4) is that .1 C "/1 jjf jj  jjh" .f /jj  jjf jj for
any f 2 Cp .K/.
Given a finite set P  K and > 0 let P;
D ff 2 Cp .K/ W jf .x/j <
for any x 2 P g and hP; i D P;
\ I.a/. All possible sets P;
(or hP; i
respectively) constitute a local base of Cp .K/ (of I.a/ respectively) at the point 0K .
To show that the map h" is uniformly continuous take any open neighborhood W of
the point .0; 0K / in the space R  I.a/. Choose > 0 and a finite set P  K such
that .; /  hP; i  W .

If 0 D nnf; C."/
g and P 0 D P [fag then U D P 0 ; 0
is an open neighborhood
of the point 0K in Cp .K/; take any functions f; g 2 Cp .K/ such that f  g 2 U
and let h" .f / D .t; f0 /; h" .g/ D .s; g0 /. Then t D f .a/ and s D g.a/, so
jt  sj D jf .a/  g.a/j < . Given any x 2 P we have f0 .x/ D u" .t; f .x// and
g0 .x/ D u" .s; g.x//; as a consequence,
jf0 .x/  g0 .x/j  C."/jj.t  s; f .x/  g.x//jj < C."/ 0 D
(here we applied the property (a) of Fact 2), so f0  g0 2 hP; i and therefore
h" .f /  h" .g/ 2 .; /  hP; i  W . This proves that the map h" is uniformly
continuous.
Finally, take any open neighborhood W of the point 0K in the space Cp .K/;

g then
choose > 0 and a finite set P  K such that P;
 W . If 0 D nnf; C."/
0
0 0
U D hP; i is an open neighborhood of the point 0K in I.a/, so V D . ; /  U
is an open neighborhood of .0; 0K / is R  I.a/; take any points .t; f /; .s; g/ 2
R  I.a/ such that .t; f /  .s; g/ 2 V and let " .t; f / D f0 ; " .s; g/ D g0 .
Given any x 2 P we have f0 .x/ D v" .t; f .x// and g0 .x/ D v" .s; g.x//; as a
consequence, jf0 .x/g0 .x/j  C."/jj.t s; f .x/g.x//jj < C."/ 0 D (here we
applied the property (a) of Fact 2), so f0 g0 2 P;
and therefore " .f /" .g/ 2
P;
 W . This demonstrates that the map " is also uniformly continuous and
shows that Fact 3 is proved.
Returning to our solution observe that any countably infinite compact space is
homeomorphic to a space 0;
for some countable infinite ordinal (see Problem 199). Therefore it suffices to prove that Cp .0;
/ is uniformly homeomorphic

246

2 Solutions of problems 001500

to  .!/ for any countable infinite ordinal . To avoid confusion, from now on any
ordinal is considered to be a point only; if we want to see it as a set then we write
0; /. Evidently, 0;
is the set given by C 1. Applying transfinite induction we
will prove that, for any countable ordinal  ! and any " > 0,
(5) there exists is a uniform homeomorphism ' W Cp .0;
/ !  .0; !// such
that .1 C "/1 jjf jj  jj'.f /jj  jjf jj for every f 2 Cp .0;
/.
To establish (5) we will often use the following assertion; its proof is straightforward and can be left to the reader.
(6) for any .t; x/ 2 R   .0; !// let e.t; x/.0/ D t and e.t; x/.n C 1/ D
x.n/ for every n < !. Then e W R   .0; !// !  .0; !// is a
uniform homeomorphism such that jje.t; x/jj D jj.t; x/jj for any .t; x/ 2
R   .0; !//.
Our first step is to prove (5) for D !; let I D ff 2 Cp .0; !
/ W f .!/ D 0g.
The restriction map  W I ! Cp .0; !// is continuous and .I / D  .0; !//,
so we can and will consider that  W I !  .0; !//. Then  is a uniform
homeomorphism such that jj.f /jj D jjf jj for any f 2 I . Apply Fact 3 to see
that there exists a uniform homeomorphism '0 W Cp .0; !
/ ! R  I such that
.1 C "/1 jjf jj  jj'0 .f /jj  jjf jj for any f 2 Cp .0; !
/.
Let 0 .t; f / D .t; .f // for every .t; f / 2 R  I . Then 0 W R  I ! R 
 .0; !// is a uniform homeomorphism such that jj0 .z/jj D jjzjj for any z 2
R  I . It follows from (6) that ' D e 0 '0 W Cp .0; !
/ !  .0; !// is a
uniform homeomorphism. Given f 2 Cp .0; !
/ the equality jje.0 .'0 .f ///jj D
jj0 .'0 .f //jj D jj'0 .f /jj shows that .1 C "/1 jjf jj  jj'.f /jj  jjf jj for any
f 2 Cp .0; !
/, i.e., (5) is proved for D !.
Now assume that < !1 and (5) is proved for all < . If D 0 C 1 then
the space 0;
D 0; 0
[ fg is obtained from 0; 0
by adding an isolated point,
so 0;
' 0; 0
by Fact 1; the property (5) being true for 0; 0
by the induction
hypothesis, it is also true for 0;
.
If is a limit ordinal then choose a sequence fi W i < !g  0; / such that
i < iC1 for each i < ! and supfi W i < !g D ; let H0 D 0; 0
and
HiC1 D i C 1; iC1
for allS
i < !. It is easy to find a disjoint family fSi W i < !g
of subsets of 0; !/ such that i<! Si D 0; !/ and there exists a bijection between
Hi and Si (we wont need this bijection but we must assure that Hi and Si are
always of the same cardinality) for every i < !.
For any i < ! and f 2  .Si / let jjf jji D maxff .n/ W n 2 Si g. Observe that
 .Si / D RSi if Si is finite. Every space Hi is compact and countable so there is
i < such that Hi ' 0; i
(see Problem 199). Choose a positive number such
that .1 C /2 > .1 C "/1 ; by the induction hypothesis, if Hi is infinite, then there
exists a uniform homeomorphism 'i W Cp .Hi / !  .Si / such that
(7) .1 C /1 jjf jj  jj'i .f /jji  jjf jj for every f 2 Cp .Hi /.
If Hi is finite then the uniform homeomorphism 'i can even be chosen in such a
way that jj'i .f /jji D jjf jj for every f 2 Cp .Hi / D RHi , so (7) still holds for 'i .

2 Solutions of problems 001500

247

If we have a function fi 2 RSi for every i < ! then there is a unique S


f 2 R0;!/
such that f jSi D fi for all i < !; this function f will be referred to as i<! fi .
Let I D ff 2 Cp .0;
/ W f ./ D 0g; then i W I !
SCp .Hi / is the restriction
map for every i < !. For every f 2 I let .f / D i<! 'i .i .f //; then  W
I ! R0;!/ is a well-defined map. The sets Hi are eventually contained in any
given neighborhood of , so it follows from f ./ D 0 that jji .f /jj ! 0 for
every f 2 I ; it follows from (7) that jj'i .i .f //jji ! 0, so  actually maps I in
 .0; !//. Let us show that  W I !  .0; !//
S is a uniform homeomorphism.
If f; g 2 I and f g then it follows from i<! Hi D 0; / that f jHi gjHi
for some i < !. Then 'i .i .f // 'i .i .g// and hence .f / .g/, i.e.,  is
injective. Given any x 2  .0; !// let hi D xjSi ; then hi 2  .Si /, so we can
find fi 2 Cp .Hi / such that 'i .fi / D hi for each i < !. It follows from (7) that
jjfi jj  .1 C /jjhi jji for all i < !, so it follows from jjhi jji ! 0 that jjfi jj ! 0. If
we let f ./ D 0 and f jHi D fi for all i < ! then it is straightforward that f 2 I
and .f / D x; this proves that  is a bijection.
To see that  is uniformly continuous let 0 be the zero point of  .0; !// and
fix a set O 2 .0;  .0; !///. There are r > 0 and m < ! such the set O 0 D
fx 2  .0; !// W jx.i /j < r for all i  mg is contained in O. Take k < ! for
which A D f0; : : : ; mg  S0 [ : : : [ Sk . For any i  k, by uniform continuity
of 'i there is a finite set Bi  Hi and si > 0 such that if f; g 2 Cp .Hi / and
jf ./  g./j < si for all 2 Bi then
S j'i .f /.l/  'i .g/.l/j < r for all l 2 A \ Si .
Let s D nnfsi W i  kg and B D fBi W i  kg. It is immediate that if f; g 2 I
and jf ./  g./j < s for all 2 B then j.f /.i /  .g/.i /j < r for all i  m
and hence .f /  .g/ 2 O 0  O. In other words, the open set U D ff 2 I W
jf ./j < s for all 2 Bg witnesses that  is uniformly continuous.
To establish that 1 is uniformly continuous take an open neighborhood O
of the zero function of I . Pick s > 0 and a finite set B  0; / such that the
set O 0 D ff 2 I W jf ./j < s for every 2 Bg is contained in O. There is
m < ! such that B  H0 [ : : : [ Hm . For any i  k, by uniform continuity
of 'i1 there exists a finite set Ai  Si and ri > 0 such that if f; g 2  .Si /
 'i1 .g/./j < r for
and jf .l/  g.l/j < ri for all l 2 Ai then j'i1 .f /./ S
all 2 B \ Hi . Let r D nnfri W i  mg and A D fAi W i  mg. It is
immediate that if f; g 2  .0; !// and jf .l/  g.l/j < s for all l 2 A then
j1 .f /./1 .g/./j < s for all 2 B and hence 1 .f /1 .g/ 2 O 0  O.
In other words, the open set U D fx 2  .0; !// W jx.l/j < r for all l 2 Ag
witnesses that 1 is uniformly continuous.
Given f 2 I and i < ! we have jji .f /jj  jjf jj and hence jj'i .i .f //jji 
jjf jj by (7). An immediate consequence is that jj.f /jj  jjf jj. The family fHi W
i 2 !g covers 0; /, so there exists m < ! such that jjf jj D jjm .f /jj. Apply
(7) once more to see that jj'm .m .f //jjm  .1 C /1 jjm .f /jj D .1 C /1 jjf jj.
Thus jj.f /jj  jj'm .m .f //jjm  .1 C /1 jjf jj, so we proved that
(8) .1 C /1 jjf jj  jj.f /jj  jjf jj for every f 2 I .
Apply Fact 3 to find a uniform homeomorphism  W Cp .0;
/ ! RI such that
.1C/1 jjf jj  jj.f /jj  jjf jj for every f 2 Cp .0;
/. For every .t; f / 2 RI

248

2 Solutions of problems 001500

let .t; f / D .t; .f //. The reader who understood this proof up to the present
point, will have no difficulty to check, using (8), that  W R  I ! R   .0; !//
is a uniform homeomorphism such that .1 C /1 jjzjj  jj.z/jj  jjzjj for every
z 2 RI . It follows from (6) that the map ' D e  W Cp .0;
/ !  .0; !// is
a uniform homeomorphism such that .1 C /2 jjf jj  jj'.f /jj  jjf jj. Recalling
that .1 C "/1 < .1 C /2 we conclude that .1 C "/1 jjf jj  jj'.f /jj  jjf jj for
any f 2 Cp .0;
/. Thus we checked (5) for all  , so our inductive procedure
guarantees that (5) holds for all infinite ordinals < !1 .
This implies that, for any infinite ordinals ; < !1 the spaces Cp .0;
/ and
Cp .0;
/ are uniformly homeomorphic; we already saw that this is the same as
saying that Cp .X / and Cp .Y / are uniformly homeomorphic for any countably
infinite compact spaces X and Y , so our solution is complete.
V.201. Prove that the topology of any linear topological T0 -space is Tychonoff.
Solution. Fix a linear topological T0 -space L. Given any a 2 L define a map 'a W
L ! L by the formula 'a .x/ D x C a for any x 2 L. The operations in L being
continuous, the map 'a is continuous for any a 2 L. Since 'a is the inverse of
'a , every 'a is a homeomorphism. If i.x/ D x for every x 2 L then the map
i W L ! L is also a homeomorphism inverse to itself.
Now take any point a 2 Lnf0g; there exists an open set U  L such that P D
U \ fa; 0g is a singleton. If P D fag then U is a neighborhood of a missing
0. If P D f0g then let V D U \ i.U /; since i.0/ D 0, the set V is an open
neighborhood of 0. Therefore W D 'a .V / is an open neighborhood of a. If 0 2 W
then there is x 2 V with x C a D 0, i.e., x D a 2 V . However, i.V / D V , so
a D i.a/ 2 V  U which is a contradiction. Thus W is an open neighborhood of
a which does not contain 0.
We proved that every a 2 Lnf0g has a neighborhood which misses 0, so the set
f0g is closed in L. For any x 2 L the homeomorphism 'x maps f0g onto fxg, so
fxg is closed in L and hence L is a T1 -space.
For any U 2 .0; L/ let U
D f.x; y/ 2 L  L W x  y 2 U g and consider
the family B D fU
W U 2 .0; L/g. It follows from 0 C 0 D 0 and continuity of
addition in L that
(1) for any U 2 .0; L/ there exists V 2 .0; L/ such that V C V  U .
Call a set U 2 .0; L/ symmetric if U D i.U / D U . It is easy to check that
the set U \ .U / is symmetric for any U 2 .0; L/, so we have
(2) for any U 2 .0; L/ there exists a symmetric set V 2 .0; L/ with V  U .
For any A; B  L  L we will need the sets A1 D f.x; y/ W .y; x/ 2 Ag and
A B D f.x; y/ W there is z 2 L such that .x; z/ 2 B and .z; y/ 2 Ag; besides, we
let A.x/ D fy 2 L W .x; y/ 2 Ag for any x 2 L.
If x and y are distinct points of L then z D x  y 0, so U DTLnfzg is
an open neighborhood of 0 such that .x;
B. It is
Ty/ U
and hence .x;
Ty/
evident that D f.x; x/ W x 2 Lg 
B, so we proved that B D . Given
any U 2 .0; L/ use (2) to find a symmetric V 2 .0; L/ with V  U . Then

2 Solutions of problems 001500

249

V
1 D V
D V
 U
, so we proved that for any B 2 B there is C 2 B such
that C 1  B. Given U0 ; U1 2 .0; L/ if U D U0 \ U1 then U
 U0
\ U1

which shows that, for any A; B 2 B there exists C 2 B such that C  A \ B.


Now, if U 2 .0; L/ then apply (1) and (2) to find a set V 2 .0; L/ such that
V C V  U . If .x; y/ 2 V
V
then there is z 2 L such that x  z 2 V and
z  y 2 V which shows that x  y 2 V C V  U , so V
V
 U
. An immediate
consequence is that, for any B 2 B there exists C 2 B such that C C  B. We
finally checked that all conditions of Problem 101 are satisfied for the family B, so
there exists a unique uniformity U on the set L such that B is a base of U ; let be
the topology generated by the uniformity U .
Fix a set U 2 .L/ and a point x 2 U . The set U 0 D x C U D 'x .U / is an
open neighborhood of 0, so we can apply (2) to find a symmetric V 2 .0; L/ such
that V  U 0 . It is clear that x 2 x C V  U ; if y 2 V
.x/ then x  y 2 V , so
y  x 2 V because the set V is symmetric. This shows that y 2 x C V  U , so we
proved that V
.x/  U , i.e., for any x 2 U there exists B 2 B (and hence B 2 U )
with B.x/  U . Therefore the set U belongs to , i.e., .L/  .
Now take any set U 2 and x 2 U . There exists A 2 U such that A.x/  U ;
the family B being a base of U we can find V 2 .0; L/ such that V
 A. Apply
(2) to find a symmetric W 2 .0; L/ for which W  V . If y 2 x C W then
y D x C w for some w 2 W . The set W being symmetric we have w 2 W and
hence x  y D w 2 W , i.e., y 2 W
.x/  V .x/  U . Thus 'x .W / D x C W 
W
.x/  U which shows that, for every point x 2 U , there is G 2 .x; L/ with
G  U . Therefore U 2 .L/; this proves that  .L/ and hence .L/ D ,
i.e., the topology of L is generated by the uniformity U , so .L/ is Tychonoff by
Problem 110.
V.202. Let L be a linear topological Tychonoff space. Prove that, for any local base
B of the space L at 0, the following properties hold:
(1)
(2)
(3)
(4)
(5)

for any U; V 2 B, there is W 2 B such


T that W  U \ V ;
every B 2 B is an absorbing set and B D f0g;
for any U 2 B, there exists V 2 B such that V C V  U ;
for any U 2 B and x 2 U , there exists V 2 B such that x C V  U ;
for any U 2 B and " > 0 there is V 2 B such that V  U for any
2 ."; "/.

Prove that, if L is a linear space without topology and B is a family of subsets of


L which has the properties (1)(5) then there exists a unique Tychonoff topology
on L such that .L; / is a linear topological space and B is a local base of at 0.
Solution. Suppose that L is a linear topological space and .L/ is Tychonoff. The
property (1) holds because, for any U; V 2 B the set U \ VTis a neighborhood of 0,
so there is W 2 B with W  U \ V . It is clear that 0 2 B; if x 2 Lnf0g then
Lnfxg is anTopen neighborhood ofT0, so there is U 2 B such that U  Lnfxg and
hence x B. This proves that B D f0g. Consider the map m W R  L ! L
defined by m.t; x/ D tx for any t 2 R and x 2 L. If B 2 B and x 2 L then we have
m.0; x/ D 0 2 B, so, by continuity of m, there exist > 0 and W 2 .x; L/ such

250

2 Solutions of problems 001500

that m..; /; W /  B. In particular, m..; /; fxg/  B which says precisely


that tx 2 B for any t 2 .; /. Therefore the set B is absorbing and hence we
proved the property (2).
Now fix any U 2 B; since 0 C 0 D 0, it follows from continuity of the addition
in L that there exists W 2 .0; L/ with W C W  U . Take V 2 B with V  W ;
then V C V  U , so (3) is proved.
Given U 2 B and x 2 U apply again continuity of addition together with the
equality x C 0 D x to find G 2 .x; L/ and W 2 .0; L/ such that G C W  U .
Then x C W  G C W  U ; if V 2 B and V  W then x C V  U , so we
settled (4).
Next take any U 2 B and " > 0. The map m is continuous so it follows from
m.0; 0/ D 0 2 U that there exist W 2 .0; L/ and G 2 .0; R/ for which
m.G; W /  U . Choose > 0 with .; /  G and let V D " W . If x 2 V
and j j < " then there is w 2 W such that x D " w; let 0 D " . Then 0 2 .; /
and x D 0 w D m. 0 ; w/ 2 m.G; W /  U . This shows that x 2 U for any
x 2 V and 2 ."; "/, so (5) holds as well and hence we checked (1)(5) for any
linear topological Tychonoff space.
To prove the second part of our statement assume that L is a linear space and
we have a family B of subsets of L with the properties (1)(5). Consider the family
D f;g[fU W U  L and for any x 2 U there is V 2 B such that xCV  U g. It is
immediate that ; 2 and L 2 . Given U0 ; U1 2 suppose that U D U0 \ U1 ;
and take any x 2 U . There exist V0 ; V1 2 B such that x CV0  U0 and x CV1  U1 .
The property (1) enables us to find W 2 B with W  V0 \ V1 . Then x C W 
U0 \ U1 D U , so U 2 ; this shows that the intersection of any two elements of
belongs to .
S
Given any U  if G D U D ; then G 2 . If G ; then fix a point x 2 G;
there is U 2 U with x 2 U , so we can find V 2 B such that x C V  U . It follows
from U  G that x C V  G, so G 2 and hence is a topology on L.
Fix any point x 2 L; if B 2 B and y 2 x C B then y  x 2 B, so we can apply
(4) to find a set C 2 B such that y  x C C  B and hence y C C  x C B,
i.e., x C B 2 . Now, if x 2 U 2 then, by the choice of there is W 2 B with
x C W  U . This proves that
(a) for any x 2 L the family fx C B W B 2 Bg is a local base at the point x in the
space .L; /.
As before, we let m.t; x/ D tx for any t 2 R and x 2 L. To prove that the
operation m W R  L ! L is continuous fix a point z D .t0 ; x0 / 2 R  L and an
open neighborhood G of t0 x0 in the space .L; /. The property (a) shows that there
is U 2 B such that t0 x0 C U  G. Using the property (3) of the family B it is
easy to find a set W 2 B such that W C W C W  U . The set W being absorbing
there is > 0 such that M0 D m..; /; fx0 g/  W . Applying the property (5) for
" D C jt0 j C 1 we can find a set V 2 B such that M1 D m.."; "/; V /  W . The
set H D .t0  ; t0 C /  .x0 C V / is an open neighborhood of .t0 ; x0 / in R  L.
If .t; x/ 2 H then we can pick v 2 V and s 2 .; / such that t D t0 C s and
x D x0 C v.

2 Solutions of problems 001500

251

Observe that tx D t0 x0 C t0 v C sx0 C sv while t0 v 2 M1 ; sv 2 M1 and


sx0 2 M0  W . As a consequence, tx 2 t0 x0 CW CW CW  t0 x0 CU  G. This
shows that m.H /  G, so the set H witnesses continuity of the scalar multiplication
m at the point .t0 ; x0 /.
Let a.x; y/ D x C y for any x; y 2 L. To see that the addition map a W L  L !
L is continuous fix a point .x0 ; y0 / 2 L  L and G 2 .x0 C y0 ; L/. The property
(a) shows that there exists U 2 B such that x0 C y0 C U  G. Apply (3) to find a
set V 2 B such that V C V  U . The set O D .x0 C V /  .y0 C V / is an open
neighborhood of .x0 ; y0 / in L  L and
a.O/ D .x0 C V / C .y0 C V / D x0 C y0 C V C V  x0 C y0 C U  G
so the set O witnesses continuity of the addition a at the point .x0 ; y0 /.
Thus .L; / is a linear topological space and it follows from (a) that B is a local
base of .L; / at 0. If x and y are distinct points of L then z D x  y 0, so we can
apply (2) to see that there exists U 2 B with z U . Consequently, x y C U , i.e.,
every y 2 Lnfxg has a neighborhood y C U contained in Lnfxg. Therefore Lnfxg
is open in .L; /, so fxg is closed in .L; / which proves that .L; / is a T1 -space.
Applying Problem 201 we conclude that .L; / is a Tychonoff space.
Finally, assume that  is a topology on L such that .L; / is a linear topological
space for which B is a local base at 0. Given x 2 L let 'x .y/ D y Cx for any y 2 L.
Then the map 'x W L ! L is continuous both for .L; / and .L; /; since 'x is
the inverse of 'x , every 'x is a homeomorphism with respect to both topologies. An
immediate consequence is that
(b) the family fx C B W B 2 Bg is a local base at x in .L; / for every x 2 L.
Now, if U 2 and x 2 U then, by (a),
S there is B 2 B with Ux D x C B  U ;
every set Ux belongs to  by (b), so fUx W x 2 U g D U also belongs to .
This proves that  . If U 2  and x 2 U then apply (b) to find B 2 B such
that
S Ux D x C B  U . The property (a) shows that Ux 2 for every x 2 U , so
fUx W x 2 U g D U also belongs to . Therefore   and hence  D . This
proves uniqueness of the topology and makes our solution complete.
V.203. Let L be a linear topological space. Prove that

T
for any local base B of L at 0 and any A  L, we have AD fACV W V 2 Bg;
for any A; B  L, we have A C B  A C B;
if M is a linear subspace of L then M is also a linear subspace of L;
if C is a convex subset of L then the sets C and Int.C / are also convex;
if B is a balanced subset of L then B is also balanced; if, additionally, we have
0 2 Int.B/ then Int.B/ is also balanced;
(6) if E is an l-bounded subset of L then E is also l-bounded.

(1)
(2)
(3)
(4)
(5)

Solution. (1) Take any point x 2 A; if V 2 B then the set W D V \.V / is easily
seen to be symmetric, i.e., W D W . Since x C W is an open neighborhood of
x, we can choose a point a 2 .x CW /\A. There is w 2 W such that a D x Cw
and hence x D a  w 2 a C
T .W / D a C W  a C V  A C V . This gives
us the inclusion A  P D fA C V W V 2 Bg.

252

(2)

(3)

(4)

(5)

(6)

2 Solutions of problems 001500

Now, if x 2 P then take any G 2 .x; L/. It is an easy exercise that there
exists U 2 B such that x C U  G. The set W D U \ .U / is a symmetric
neighborhood of 0, so we can find V 2 B with V  W . It follows from x 2 P
that x 2 A C V , so there are a 2 A and v 2 V such that x D a C v. Then
a D x  v 2 x C .V /  x C .W / D x C W  x C U  G, i.e., a 2 G \ A
and hence G \ A ; for any G 2 .x; L/ which shows that x 2 A. As a
consequence, P D A, i.e., (1) is proved.
Let a.x; y/ D x C y for any x; y 2 L. The map a W L  L ! L is continuous,
so a.P /  a.P / for any P  LL. In particular, if A; B  L and P D AB
then P D A  B, so a.A; B/ D A C B  a.A  B/ D A C B; this settles (2).
Fix any ; 2 R and let '.x; y/ D x C y for any x; y 2 L. The map
' W L  L ! L is continuous and '.M  M /  M because M is a linear
subspace of L. It follows from continuity of ' that '.P /  '.P / for any
P  L  L. In particular, for the set P D M  M we have P D M  M ,
so '.M  M /  '.M  M /  M . In other words, x C y 2 M for any
x; y 2 M and ; 2 R which shows that M is a linear subspace of L and
hence we are done with (3).
Observe
first that, for any A  L if a set U is open in the space L then ACU D
S
fa C U W a 2 Ag is an open subset of L. Fix any t 2 0; 1
and let t .x; y/ D
tx C .1  t /y for any x; y 2 L. The map t W L  L ! L is continuous and
t .C  C /  C because C is convex. Given an open U  L at least one of the
sets t U or .1t /U is open in L because t and 1t cannot be both equal to zero.
Thus we can apply our above observation to see that t .U U / D t U C.1t /U
is an open subset of L for any U 2 .L/. In particular, t .Int.C /  Int.C // is
an open set contained in C , so t .Int.C /  Int.C //  Int.C / for any t 2 0; 1

which is a reformulation of the fact that Int.C / is a convex set.


It follows from continuity of t that t .P /  t .P / for any P  L  L. In
particular, for the set P D C  C we have P D C  C , so t .C  C / 
t .C  C /  C . In other words, tx C .1  t /y 2 C for any x; y 2 C and
t 2 0; 1
which shows that C is a convex subset of L, i.e., we finished the proof
of (4).
Fix any t 2 I and let t .x/ D tx for any x 2 L; by continuity of the operations
in L, the map t W L ! L is continuous. The set B being balanced, we have
the inclusion t .B/  B. Furthermore, t .B/  t .B/  B which shows that
t .B/  B for any t 2 I and hence B is a balanced set.
Next observe that t is a homeomorphism if t 0; therefore t .Int.B// is an
open subset of B; this, evidently, implies that t .Int.B//  Int.B/ for any t 2
Inf0g. Since 0 2 Int.B/, we also have the inclusion 0 .Int.B// D f0g  Int.B/
which shows that t .Int.B//  Int.B/ for any t 2 I, i.e., Int.B/ is balanced
and hence we showed that (5) is true.
Take any U 2 .0; L/; the space L being Tychonoff, we can find W 2 .0; L/
such that W  U . The set E is l-bounded, so there exists r > 0 such that
E  tW for any t  r. The map t W L ! L is a homeomorphism, so
E  tW D t W  t U for any t  r and hence E is l-bounded; this settles (6)
and completes our solution.

2 Solutions of problems 001500

253

V.204. Let L be a linear topological space. Prove that


(1) every neighborhood of 0 contains an open balanced neighborhood of 0;
(2) every convex neighborhood of 0 contains an open convex balanced neighborhood of 0.
Deduce from (2) that any locally convex linear topological space has a local base
B at 0 such that each U 2 B is convex and balanced.
Solution. (1) If G is a neighborhood of 0 then there is U 2 .0; L/ with U  G.
The family B D .0; L/ is a local base of L at 0, so we can apply Problem 202
to find a set V 2 .0; L/ such that tV  U for any t 2 .1; 1/. The set
StV is
open in L and 0 2 tV for every t 2 .1; 1/nf0g. Therefore the set W D ftV W
t 2 .1; 1/nf0gg is open in L. Observe also that,Sfor t D 0, the set tV D f0g
is contained in W , so we have the equality W D ftV W t 2 .1; 1/g. For any
s 2 I and x 2 W there are t 2 .1; 1/ and v 2 V such that x D t v. Then
sx D st v 2 stV  W because st 2 .1; 1/. As a consequence, sW  W for
any s 2 I, i.e., W  U  G is an open balanced neighborhood of 0, so we
proved (1).
(2) Suppose that G is a convex neighborhood of 0. Then U D Int.G/ is an open
convex neighborhood of 0 by Problem 203. It is an easy exercise to see that the
set U is also open and convex, so V D U \.U / is an open symmetric convex
neighborhood of 0 with V  G (it is again an easy exercise that any intersection
of convex subsets of L is convex). It turns out that the set V is also balanced; to
see that, take any t 2 I and x 2 V . If t  0 then tx D tx C.1t /0 2 V because
V is convex and f0; xg  V . If t < 0 then tx D jt j.x/ C .1  jt j/0 2 V ; we
used again convexity of V and the fact that fx; 0g  V . Thus tx 2 V for any
t 2 I and x 2 V , so V is the promised open balanced and convex neighborhood
of 0 contained in G; this proves (2).
Finally, if L is locally convex then it follows from (2) that the family B of all
open convex balanced neighborhoods of 0 is a local base at 0, so our solution is
complete.
V.205. Let L be a linear topological space. Given a nontrivial linear functional
f W L ! R, prove that the following properties are equivalent:
(i)
(ii)
(iii)
(iv)

f is continuous;
f 1 .0/ is closed in L;
f 1 .0/ is not dense in L;
there exists U 2 .0; L/ such that f .U / is a bounded subset of R.

Solution. The implication (i)H)(ii) holds because f 1 .0/ is the inverse image of
a closed set f0g. The functional f being nontrivial the set f 1 .0/ does not coincide
with L, so if f 1 .0/ is closed then it is not dense in L. This proves (ii)H)(iii).
Now suppose that f 1 .0/ is not dense in L and choose a nonempty open set
G  L such that G \ f 1 .0/ D ;. Pick a point z 2 G; then K D jf .z/j > 0
and V D z C G is an open neighborhood of 0. By Problem 204 there exists
an open balanced set U 2 .0; L/ such that U  V . If jf .x/j  K for some

254

2 Solutions of problems 001500

x 2 U then r D fK.x/ 2 I, so y D rx 2 U because the set U is balanced. Then


f .y/ D rf .x/ D K D f .z/ and hence f .y C z/ D 0. However, y 2 U  V ,
so t D y C z 2 z C V D G, so t 2 f 1 .0/ \ G, a contradiction. Therefore
jf .x/j < K for any x 2 U which shows that the set f .U /  .K; K/ is bounded
in R, i.e., we established (iii)H)(iv).
Finally, assume that there exists U 2 .0; L/ such that f .U / is bounded in
R, i.e., we can choose a number K > 0 such that f .U /  .K; K/ and hence
jf .y/j < K for any y 2 U . Given any open neighborhood G of f .0/ D 0 in R
there exists " > 0 with ."; "/  G. The set V D K" U is an open neighborhood of
0; if x 2 V then there is y 2 U such that x D K" y. An immediate consequence is
that jf .x/j D jf . K" y/j D K" jf .y/j < ". Thus f .V /  ."; "/  G and hence the
set V witnesses continuity of f at the point 0; applying Fact 2 of S.496 we conclude
that f is continuous. This settles (iv)H)(i) and shows that our solution is complete.
V.206. Suppose that L is a locally convex linear topological space which has a
countable local base at 0. Prove that there exists a metric d on the set L with the
following properties:
(i) d generates the topology of L;
(ii) all d -open balls are convex and all balls with the center at 0 are balanced;
(iii) the metric d is invariant, i.e., d.x C z; y C z/ D d.x; y/ for all x; y; z 2 L.
As a consequence, a locally convex space is metrizable if and only if it has
countable character.
Solution. Using Problems 202 and 204 we can find a local base fBn W n 2 Ng of
the space L at 0 such that Bn is convex, balanced and BnC1 C BnC1  Bn for every
n 2 N; let B0 D L.
Consider the set Dn D S
f 2kn W k D 0; : : : ; 2n g; it is clear that Dn  DnC1  0; 1

for every n 2 !; let D D n2! Dn . It is an easy exercise that


(1) the set D is dense in 0; 1
and, for any n 2 !, if r C s 2 Dn and r C s  1
then r C s 2 Dn and jr  sj 2 Dn .
We will define inductively a set A.r/  L for all r 2 D. To start off, let
A.0/ D f0g and A.1/ D L; this gives us a set A.r/ for every r 2 D0 . Suppose
that n 2 N and we have the set A.r/ for every r 2 Dn1 . If r 2 Dn nDn1
then r D 2kC1
for some k 2 f0; : : : ; 2n1  1g and hence r D r 0 C 21n where
2n
k
0
r D 2n1 2 Dn1 ; let A.r/ D A.r 0 / C Bn . This inductive procedure gives us a
set A.r/ for each r 2 D; if r > 0 then A.r/ is an open subset of L.
Let us prove first that
(2) if r; s 2 D and r C s  1 then A.r/ C A.s/  A.r C s/.
If r; s 2 D0 and rs D 0 then A.r/ C A.s/ D A.r C s/; if r D s D 12 then
A.r/ C A.s/  L D A.r C s/, so (2) is true. Proceeding inductively, assume that
n 2 N and (2) is proved for any r; s 2 Dn1 . Given any s; r 2 Dn with s C r  1
we have several cases to consider.

2 Solutions of problems 001500

255

(a) If r; s 2 Dn1 then (2) holds by the induction hypothesis.


(b) If r 2 Dn1 and s 2 Dn nDn1 then there is s 0 2 Dn1 with s D s 0 C 2n ,
so A.r/ C A.s/ D A.r/ C A.s 0 / C Bn  A.r C s 0 / C Bn D A.r C s 0 C
2n / D A.r C s/. The inclusion in the above formula is true by the induction
hypothesis and the equalities follow from (1) and the definition of the family
fA.t / W t 2 Dg.
(c) The case when r 2 Dn and s 2 Dn1 is considered as in (b).
(d) If r; s 2 Dn nDn1 then there are r 0 ; s 0 2 Dn1 such that r D r 0 C 2n and
s D s 0 C 2n . Then A.r/ D A.r 0 / C Bn and A.s/ D A.s 0 / C Bn , so
A.r/CA.s/ D A.r 0 /CA.s 0 /CBn CBn  A.r 0 Cs 0 /CBn1 D A.r 0 Cs 0 /CA.2nC1 /:
Since r 0 C s 0 2 Dn1 ; 2nC1 2 Dn1 and r 0 C s 0 C 2nC1 D r C s  1, we can
apply the induction hypothesis again to see that A.r 0 C s 0 / C A.2nC1 /  A.r C s/,
so A.r/ C A.s/  A.r C s/ for any r; s 2 Dn . Therefore our inductive procedure
can be continued to establish that, for any n 2 !, the condition (2) is satisfied for all
r; s 2 Dn . Since the family fDn W n 2 !g is increasing, the property (2) is proved.
An immediate consequence is that
(3) given t; s 2 D, if t  s then A.t /  A.t / C A.s  t /  A.s/.
Let .x/ D inffr 2 0; 1
W x 2 A.r/g for every x 2 L. The function  W
L ! R is well defined and .L/  0; 1
. If we let d.x; y/ D .x  y/ then it
is straightforward that we have (iii) for the function d ; besides, d.x; x/ D 0 and
d.x; y/  0 for any x; y 2 L. If z 2 Lnf0g then z Bn D A.2n / for some n 2 !,
so it follows from (3) that .z/  2n > 0. Therefore d.x; y/ > 0 whenever x y.
If P; Q are balanced (convex) subsets of L then P C Q is also balanced (or
convex respectively). Using this fact it takes an easy induction to prove that A.r/ is
balanced and convex for every r 2 D. Thus, x 2 A.r/ if and only if x 2 A.r/
and hence .x/ D .x/ for any x 2 L. This implies that d.x; y/ D d.y; x/ for
all x; y 2 L.
To prove the triangle inequality for d we will show first that
(4) .x C y/  .x/ C .y/ for any x; y 2 L.
Since (4) is trivially true if .x/C.y/  1, we can assume that .x/C.y/ < 1.
Fix an arbitrary number " > 0; there exist r; s 2 D such that .x/ < r < .x/ C 2"
and .y/ < s < .y/ C 2" while r C s < 1. It follows from (3) and the definition
of the function  that x 2 A.r/ and y 2 A.s/. Therefore x C y 2 A.r C s/ by the
property (2); as a consequence, .x C y/  r C s < .x/ C .y/ C ". The inequality
.x C y/ < .x/ C .y/ C " being true for any " > 0, the property (4) is proved.
Thus d.x; z/ D .x  z/ D .x  y C y  z/  d.x; y/ C d.y; z/ for any
x; y; z 2 L, so d is a metric on the set L; let B.x; / D fy 2 L W d.x; y/ < g for
all x 2 L and > 0. It is an easy exercise to see that

256

2 Solutions of problems 001500

(5) B.0; / D

fA.r/ W r 2 D; r < g for any > 0;

this implies that every B.0; / is balanced. Given x; y 2 B.0; / and t 2 0; 1


it
follows from (3) that there exists r 2 D; r < such that x; y 2 A.r/; since A.r/ is
convex, the point tx C .1  t /y belongs to A.r/ and hence to B.0; /. This proves
that B.0; / is a convex balanced set for any > 0.
It is a consequence of (iii) that B.x; / D x C B.0; / for all x 2 L and > 0;
this shows that all d -open balls are convex, i.e., (ii) is proved.
The property (5) shows that every B.0; / is open in L; since every d -open ball is
a shift of some B.0; /, all d -open balls are open in L. If is the topology generated
by d then the family B D fB.x; / W x 2 L; > 0g is a base of .L; /, so it follows
from B  .L/ that  .L/.
Since Bn D A.2n / it follows from (3) that A.r/  Bn for every r < 2n , so
B.0; 2n /  Bn for each n 2 N. Therefore the family fB.0; / W > 0g is a local
base at 0, so the family fB.x; / W > 0g is a local base at x for every x 2 L. This
shows that the family fB.x; / W x 2 L; > 0g  is a base of .L/, so .L/ 
and hence .L/ D . This settles (i) and demonstrates that our solution is complete.
V.207. Let p be a seminorm on a linear space L. Prove that
(1)
(2)
(3)
(4)

p.0/ D 0 and p.x/  0 for any x 2 L;


jp.x/  p.y/j  p.x  y/ for any x; y 2 L;
fx 2 L W p.x/ D 0g is a linear subspace of L;
the set B D fx W p.x/ < 1g is convex, balanced, absorbing and p D B .

Solution. It follows from p.0/ D p.0 0/ D 0 p.0/ D 0 that p.0/ D 0. Given any
x 2 L observe that 0 D p.x  x/  p.x/ C p.x/ D 2p.x/ and hence p.x/  0;
this proves (1).
Given any points x; y 2 L we have the inequalities p.x/  p.y/ C p.x  y/ and
p.y/  p.x/ C p.y  x/ which, together with the fact that p.y  x/ D p.x  y/
show that p.x  y/  p.x/  p.y/  p.x  y/, i.e., jp.x/  p.y/j  p.x  y/,
so (2) is proved.
To prove that M D fx 2 L W p.x/ D 0g is a linear subspace of L, take any
x; y 2 M and ; 2 R. Since 0  p.x C y/  jjp.x/ C jjp.y/ D 0, we
conclude that x C y 2 M ; this settles (3).
To prove (4) take any x; y 2 B and t 2 0; 1
. Then
p.tx C .1  t /y/  tp.x/ C .1  t /p.y/ < t C .1  t / D 1
so tx C .1  t /y 2 B for any x; y 2 B and t 2 0; 1
, i.e., B is convex. If 2 I and
x 2 B then p.x/ D jjp.x/  p.x/ < 1, so x 2 B and hence B is balanced.
1
Now fix any x 2 L and let D p.x/C1
. If jt j < then p.tx/ D jt jp.x/ < 1, so
tx 2 B for any t 2 .; / which shows that B is an absorbing set.
To finally prove that p coincides with the Minkowski functional of the set B take
any x 2 L. If p.x/ D 0 then p. xt / D 0 and hence xt 2 B for any t > 0; therefore
B .x/ D 0, so we have the equality p.x/ D B .x/ in this case.

2 Solutions of problems 001500

257

If p.x/ > 0 and 0 < t < p.x/ then p. xt / > 1, so xt B and hence B .x/ 
p.x/. If t > p.x/ then p. xt / < 1 which implies that xt 2 B, so B .x/  t ; the last
inequality being true for all t > p.x/ we conclude that B .x/  p.x/ and hence
B .x/ D p.x/ for every x 2 L, i.e., we completed the proof of (4).
V.208. Let A be a convex absorbing set in a linear space L. Prove that
(1)
(2)
(3)
(4)

A .x C y/  A .x/ C A .y/ for any x; y 2 L;


A .tx/ D tA .x/ for any x 2 L and t  0;
if A is balanced then A is a seminorm;
if B D fx 2 L W A .x/ < 1g and C D fx 2 L W A .x/  1g then B  A  C
and A D B D C .

Solution. (1) Fix any x; y 2 L; given t > A .x/ C A .y/ it is possible to choose
t0 > A .x/ and t1 > A .y/ such that t0 C t1 D t . There exist s0 ; s1 such that
A .x/  s0 < t0 and A .y/  s1 < t1 while sx0 2 A and sy1 2 A. The set A is
absorbing, so 0 2 A. It follows from convexity of A that
. / z 2 A implies sz C .1  s/0 D sz 2 A for any s 2 0; 1
.
y
2 A. If s D tt0 then 1  s D tt1 , so we can use convexity
t1
of A again to see that tx0 tt0 C ty1 tt1 D xCy
2 A. This shows that A .x C y/  t
t
for any t > A .x/CA .y/, so A .x Cy/  A .x/CA .y/, i.e., (1) is proved.
Since 0s D 0 2 A for any s > 0, we have A .0/ D 0 which implies that
A .0 x/ D 0 A .x/ for any x 2 L. Next, fix x 2 L and t > 0; if s > tA .x/
then A .x/ < st . This, together with . /, shows that txs 2 A and therefore
A .tx/  s. The number s > tA .x/ was chosen arbitrarily, so A .tx/ 

Therefore

(2)

x
t0

2 A and

tA .x/.
x
2 A and
Now, if s > A .tx/ then apply . / once more to see that txs D s=t
s
hence A .x/  t , i.e., tA .x/  s. As before, this implies tA .x/  A .tx/,
so A .tx/ D tA .x/ and hence we proved (2).
(3) Suppose, additionally, that A is balanced. Then xt 2 A if and only if x
2A
t
for any x 2 L and t > 0; consequently, A .x/ D A .x/ for any x 2 L. If
 0 then we can apply (2) to conclude that A .x/ D A .x/ D jjA .x/.
If < 0 then A .x/ D A .jj.x// D jjA .x/ D jjA .x/ for any
x 2 L; this, together with what was proved in (1), completes the proof of (3).
(4) If x 2 B then A .x/ < 1, so . / implies that x D x1 2 A. If x D x1 2 A
then A .x/  1, i.e., x 2 C . This proves that B  A  C ; an immediate
consequence of the definition of the Minkowski functional is that C .x/ 
A .x/  B .x/ for any x 2 L.
Now, assume that x 2 L and C .x/ < t ; there is s 2 C .x/; t / such that
x
2 C and hence 1  A . xs / D 1s A .x/, so A .x/  s < t . The number
s
t > C .x/ was chosen arbitrarily, so we proved that A .x/  C .x/ for any
x 2 L, i.e., A D C .
If x 2 L and A .x/ < t then A . xt / < 1 by (2); thus xt 2 B, so B .x/  t
for any t > A .x/. Therefore B .x/  A .x/ for any x 2 L and hence
A D B ; this settles (4) and finishes our solution.

258

2 Solutions of problems 001500

V.209. Given a locally convex linear topological space L, take any local base B at
0 such that all elements of B are convex and balanced. Prove that fV W V 2 Bg is
a separating family of continuous seminorms on L.
Solution. Observe first a local base B at the point 0 such that all elements of B
are convex and balanced exists in any locally convex space by Problem 204. All
elements of B are also absorbing by Problem 202, so we can apply Problem 208 to
see that V is a seminorm on L for any V 2 B.
Fix V 2 B and take any x 2 L; " > 0. The set W D x C 2" V is an open
z
neighborhood of x; if y 2 W then z D y  x 2 2" V and hence "=2
2 V , so V .z/ 
"
< ". Applying Problem 207 we conclude that jV .y/  V .x/j  V .z/ < " for
2
any y 2 W and therefore the set W witnesses that V is continuous at the point x.
Thus every V is continuous on L.
Finally, take any x 2 Lnf0g; the family B being a local base at 0, there exists
V 2 B with x D x1 V . If t < 1 and xt 2 V then x D t xt 2 V because the
set V is balanced. This contradiction shows that xt V for any t < 1 and hence
V .x/  1 > 0. Therefore fV W V 2 Bg is a separating family of continuous
seminorms on L.
V.210. Let P be a separating family of seminorms on a linear space L. Given
p 2 P and n 2 N, let O.p; n/ D fx 2 L W p.x/ < n1 g. Prove that the family
B D fO.p1 ; n/ \ : : : \ O.pn ; n/ W n 2 N; p1 ; : : : ; pn 2 Pg is a convex balanced
local base at 0 for some topology on L such that .L; / is a locally convex space
in which all elements of P are continuous and any E  L is l-bounded if and only
if p.E/ is bounded for any p 2 P.
Solution. Observe first that O.p; n/ D n1 O.p; 1/, so O.p; n/ is convex, balanced
and absorbing for any p 2 P and n 2 N (see Problem 207). We leave it to the
reader to verify that any finite intersection of convex, balanced and absorbing sets
is a convex, balanced and absorbing set; therefore every element of B is convex,
balanced and absorbing.
Given U; V 2 B there are k; n 2 N and p1 ; : : : ; pn ; q1 ; : : : ; qk 2 P such that
U D O.p1 ; n/ \ : : : \ O.pn ; n/ and V D O.q1 ; k/ \ : : : \ O.qk ; k/. Let ri D pi
for all i  n and ri D qin for all i D n C 1; : : : ; n C k; then, for m D n C k, the
set W D O.r1 ; m/ \ : : : \ O.rm ; m/ belongs to B and W  U \ V .
If x 2 Lnf0g then there is p 2 P such that p.x/ 0 and hence
T there is n 2 N
1
for
which
p.x/

.
This
shows
that
x

O.p;
n/
2
B,
so
x

B and therefore
n
T
B D f0g.
Given U D O.p1 ; n/ \ : : : \ O.pn ; n/ 2 B let ri D pi for all i  n and ri D p1
for all i D n C 1; : : : ; 2n. Then fr1 ; : : : ; r2n g  P and V D O.r1 ; 2n/ \ : : : \
1
and
O.r2n ; 2n/ belongs to B. If x; y 2 V and i  n then ri .x/ D pi .x/ < 2n
1
ri .y/ D pi .y/ < 2n ; an immediate consequence is that pi .x C y/  pi .x/ C
pi .y/ < n1 for all i  n and therefore x C y 2 U . This proves that V C V  U .
Now, if U D O.p1 ; n/ \ : : : \ O.pn ; n/ 2 B and x 2 U then there exists
m 2 N such that pi .x/ < n1  m1 for all i  n. For k D m C n let qi D pi
for all i  n and qi D p1 for all i D n C 1; : : : ; k. Then fq1 ; : : : ; qk g  P, so

2 Solutions of problems 001500

259

V D O.q1 ; k/ \ : : : \ O.qk ; k/ is an element of B. If y 2 V then pi .x C y/ 


pi .x/Cpi .y/ D pi .x/Cqi .y/ < n1  m1 C k1  n1 for every i  n, so x Cy 2 U for
any y 2 V . This proves that, for any x 2 U there is V 2 B such that x C V  U .
Once more, if U D O.p1 ; n/ \ : : : \ O.pn ; n/ 2 B and " > 0 then choose
1
m 2 N such that m  n and m1 < "n
. Let qi D pi for all i  n and qi D p1 for
all i D n C 1; : : : ; m. Then fq1 ; : : : ; qm g  P, so V D O.q1 ; m/ \ : : : \ O.qm ; m/
is an element of B. For any 2 ."; "/ and x 2 V we have pi . x/ D j jpi .x/ <
" m1 < n1 , so x 2 U .
Thus we checked that all conditions of Problem 202 for B are satisfied, so there
exists a unique linear space topology on L such that B is a local base at 0 for
the space .L; /. For every x 2 L the family Bx D fx C B W B 2 Bg is a local
base at x in .L; /. Since all elements of Bx are convex for each x 2 L, the family
S
fBx W x 2 Lg is a base of .L; / which consists of convex sets. Therefore .L; /
is a locally convex space.
Take any p 2 P and fix x 2 L; " > 0. Choose n 2 N with n1 < "; the set
U D O.p; n/ belongs to B, so V D x C U is an open neighborhood of x. For any
y 2 V we have p.y x/ < n1 , so jp.y/p.x/j  p.y x/ < n1 (see Problem 207);
therefore the set V witnesses continuity of p at the point x. This proves that every
p 2 P is continuous on .L; /.
Assume that E  L is l-bounded in .L; /. Given p 2 P the set U D O.p; 1/
is an open neighborhood of 0, so there is r > 0 such that E  rU . Therefore, for
any x 2 E, there is y 2 U with x D ry and hence p.x/ D rp.y/ < r; this shows
that p.E/  0; r/ is a bounded subset of R.
Finally suppose that E  L and the set p.E/ is bounded in R for any p 2 P.
Given any U 2 with 0 2 U there exist n 2 N and seminorms p1 ; : : : ; pn 2 P such
that V D O.p1 ; n/ \ : : : \ O.pn ; n/  U . Take r > 0 such that pi .E/  0; r/ for
every i  n and let s D nr. If t  s and x 2 E then pi . xt / D 1t pi .x/ < rt  rs D n1
for all i  n, so xt 2 V and therefore x 2 tV  t U . We proved that E  t U for
any t  s, so the set E is l-bounded in .L; /. Thus a set E  L is l-bounded in
.L; / if and only if p.E/ is bounded in R for all p 2 P. We have finally proved all
that was promised for .L; /, so our solution is complete.
V.211. Prove that a linear topological space is normable if and only if it has a
convex l-bounded neighborhood of zero.
Solution. Suppose that L is a normable linear topological space and fix a norm jj jj
such that the metric d W L  L ! R defined by d.x; y/ D jjx  yjj for all x; y 2 L,
generates the topology of L. Since every norm is also a seminorm, we can apply
Problem 207 to see that the set B D fx 2 L W jjxjj < 1g is convex.
We also have the equality B D fx 2 L W d.x; 0/ < 1g, so the set B is open
in L being a ball in the metric space .L; d /. To see that B is l-bounded, take any
U 2 .0; L/; there exists " > 0 such that C D fx 2 L W jjxjj < "g  U . Let
s D 1" and take any t  s. If x 2 B then jjxjj < 1, so jj xt jj D 1t jjxjj < 1t  1s D ".
Therefore y D xt 2 C , so x D ty 2 t C  t U and hence B  t U for any t  s.
This proves that B is an l-bounded convex neighborhood of 0 and hence we have
established necessity.

260

2 Solutions of problems 001500

To verify sufficiency, assume that L is a linear topological space and there exists
an l-bounded convex neighborhood B of the point 0. Apply Problem 204 to find an
open convex balanced neighborhood V of the point 0 such that V  B; it follows
from V  B that V is also l-bounded. The set V is absorbing by Problem 202
(applied to B D .0; L/), so its Minkowski functional V is a seminorm on L by
Problem 208.
If x 0 then W D Lnfxg is an open neighborhood of 0, so there is t > 0 such
x
that V  tW and hence 1t V  W which shows that x 1t V , i.e., tx D 1=t
V.
1
x
Suppose that 0 < s < t and s 2 V ; since 0 < st < 1 and x 2 sV , we have
tx 2 t sV  V the last inclusion being true because the set V is balanced. This
contradiction with tx V demonstrates that xs V for any s 2 .0; 1t / and therefore
V .x/  1t > 0.
We showed that V .x/ > 0 for any x 0, so V is a norm on L; let jjxjj D
V .x/ for every x 2 L. We must prove that the metric d W L  L ! R defined
by d.x; y/ D jjx  yjj for any x; y 2 L, generates the topology of L; denote
temporarily by the topology generated by the metric d . For any r > 0 and x 2 L
the set B.x; r/ D fy 2 L W d.x; y/ < rg is the ball of radius r centered at x.
Take any point x D 1 x 2 V ; the multiplication by scalars is continuous in L,
so we can find " > 0 and W 2 .x; L/ such that t w 2 V for any t 2 .1  "; 1 C "/
x
1
and w 2 W . In particular, .1 C 2" /x D .1C"=2/
< 1.
1 2 V , so V .x/  .1 C "=2/
This shows that V  B.0; 1/; applying Problem 208 we conclude that V D B.0; 1/
and therefore B.0; r/ D rB.0; 1/ D rV is an open subset of L for any r > 0. It
follows from the equality B.x; r/ D x C B.0; r/ that every ball B.x; r/ is open
in L. The family B D fB.x; r/ W x 2 L; r > 0g  .L/ is a base of ; an easy
consequence is that  .L/.
Take any U 2 .0; L/; the set V being l-bounded, there is t > 0 such that
V  t U and hence 1t V  U . Therefore B.0; 1t / D 1t B.0; 1/ D 1t V  U which
shows that the family fB.0; r/ W r > 0g is a local base of L at 0. Consequently,
fB.x; r/ W r > 0g is a local base of L at x for any x 2 L. This implies that the
family B  is a base of L, so .L/  and hence D .L/. We proved that
V is a norm on L which generates the topology of L; this settles sufficiency and
makes our solution complete.
V.212. Let N be a closed subspace of a linear topological space L. Prove that
(1) the quotient topology of L=N makes L=N a linear topological space;
(2) the quotient map  W L ! L=N is linear, open and continuous;
(3) If P 2 fmetrizability, local convexity, normability, complete normabilityg and
L has P then L=N also has P.
Solution. Observe that .x/ D .y/ if and only if x  y 2 N . If .x/ D .x 0 /
and .y/ D .y 0 / then x  x 0 2 N and y  y 0 2 N ; since N is a subspace
of L, the point x  x 0 C y  y 0 D .x C y/  .x 0 C y 0 / belongs to N and hence
.x Cy/ D .x 0 Cy 0 /. Therefore the addition is well defined in L=N . If 2 R and
.x/ D .x 0 / then x x 0 2 N implies that .x x 0 / 2 N and hence x x 0 2 N ,
i.e., .x/ D .x 0 /, so the definition of the scalar multiplication is also consistent.

2 Solutions of problems 001500

261

It is straightforward that a C b D b C a and .a C b/ C c D a C .b C c/ for any


a; b; c 2 L=N . For the set e D N D 0 C N 2 L=N we have a C e D a and
a C .1/a D e for any a 2 L=N , so e is the zero element of L=N . We leave
it to the reader to verify the scalar multiplication properties for L=N , i.e., that the
equalities .a/ D ./a, . C /a D a C a, .a C b/ D a C b and 1a D a
are fulfilled for all a; b 2 L=N and ; 2 R. Thus L=N is a linear space.
It follows from the definition that the map  is linear and quotient, so  is
continuous. If U is open in L then  1 .U / D U CN is open in L, so, by definition
of quotient topology, the set .U / is open in L=N . This proves that the map  is
open.
To see that the topology of L=N is compatible with its algebraic structure fix
a point a0 2 L=N and t0 2 R; there is x0 2 L such that a0 D .x0 /. If U 2
.t0 a0 ; L=N / then the set V D  1 .U / is open in L and t0 x0 2 V . By continuity
of the scalar multiplication in L we can find " > 0 and W 2 .x0 ; L/ such that
t w 2 V whenever jt  t0 j < " and w 2 W .
The set W 0 D .W / is open in L=N and contains a0 ; take any t 2 .t0  "; t0 C "/
and a 2 W 0 . There is x 2 W such that a D .x/. Then tx 2 V , so
t a D .tx/ 2 U . Thus t a 2 U for any t 2 .t0  "; t0 C "/ and a 2 W 0 ; this
proves that the scalar multiplication is continuous in L=N .
Next, fix a0 ; b0 2 L=N and U 2 .a0 Cb0 ; L=N /; there are x0 ; y0 2 L such that
.x0 / D a0 and .y0 / D b0 . By linearity of  we have .x0 C y0 / D a0 C b0 , so
the set V D  1 .U / is an open neighborhood of x0 C y0 in L. By continuity of the
addition in L we can find G 2 .x0 ; L/ and H 2 .y0 ; L/ such that G C H  V .
Recalling again that the map  is open we conclude that H 0 D .H / 2 .a0 ; L=N /
and G 0 D .G/ 2 .b0 ; L=N /. Given any a 2 H 0 and b 2 G 0 pick x 2 G and
y 2 H with .x/ D a and .y/ D b. Then a C b 2 V , so x C y D .a C b/ 2 U ;
this proves that G 0 C H 0  U and hence the addition is continuous in L=N , so
L=N is a linear topological space, i.e., we settled (1) and (2).
Fact 1. Suppose that M is a normed linear space with a norm jj jj. Given a closed
linear subspace F  M let .a/ D inffjjxjj W x 2 ag for any a 2 M=F . Then  is
a norm on the space M=F which generates the quotient topology on M=F and, if
jj jj is complete, then  is a complete norm on M=F .
Proof. Let q W M ! M=F be the quotient map and denote by zf the zero element
of M=F (which coincides with the set F ); the symbol 0 stands for the zero element
of M . For any x 2 M and r > 0 let B.x; r/ D fy 2 M W jjy  xjj < rg. Since
0 2 F D zf and jj0jj D 0, we conclude that .zf / D 0. It is evident that .a/  0
for any a 2 M=F . Now, if a zf then M na is an open neighborhood of 0, so there
is " > 0 such that B.0; "/ \ a D ;; an immediate consequence is that jjxjj  " for
any x 2 a and hence .a/  " > 0. Thus .a/ > 0 for any a 2 M=F nfzf g.
Next fix a 2 M=F and t 2 R; then a D x0 C F for some x0 2 a and t a D
tx0 C F . If t D 0 then t a D zf , so .t a/ D 0 D jt j .a/, so the second axiom
of norm is fulfilled in this case. If t 0 and " > 0 then choose a point x 2 t a such
that jjxjj < .t a/ C ". Since t a D tx0 C F D t .x0 C F /, we can pick f 2 F

262

2 Solutions of problems 001500

with t .x0 C f / D x. It follows from x0 C f 2 a that jjx0 C f jj  .a/, so


jjxjj D jt jjjx0 C f jj  jt j.a/ which shows that jt j.a/  .t a/ C " for any
" > 0. Therefore jt j.a/  .t a/.
"
Now, let us choose a point y 2 a such that jjyjj < .a/ C jtj
. Then we have
jt jjjyjj D jjtyjj < jt j.a/ C " and ty 2 t a, so .t a/  jjtyjj < jt j.a/ C ".
The inequality .t a/ < jt j.a/ C " being true for any " > 0 we conclude that
.t a/  jt j.a/ and hence .t a/ D jt j.a/ for any a 2 M=F and t 2 R.
To check the triangle inequality for  fix any a; b 2 M=F and " > 0. There
are x; y 2 M such that q.x/ D a; q.y/ D b while jjxjj  .a/ C 2" and jjyjj 
.b/ C 2" . Then jjx C yjj  jjxjj C jjyjj  .a/ C .b/ C "; it follows from
x C y 2 a C b that .a C b/  .a/ C .b/ C ". The number " > 0 was taken
arbitrarily, so .a C b/  .a/ C .b/ for any a; b 2 M=F and hence  is a norm
on M=F ; denote by the topology generated by  on the set M=F .
For any a 2 M=F and " > 0 the set C.a; "/ D fb 2 M=F W .b  a/ < "g is
the "-ball with respect to  centered at a. It turns out that
. / q.B.x; "// D C.q.x/; "/ for any x 2 M and " > 0.
Given any y 2 B.x; "/ we have jjy  xjj < "; it follows from y  x 2 q.y  x/
that .q.y  x// D .q.y/  q.x//  jjy  xjj < ". Therefore q.y/ 2 C.q.x/; "/,
so we proved that q.B.x; "//  C.q.x/; "/.
Now, if a 2 C.q.x/; "/ then .a  q.x// < ", so there exists y0 2 a  q.x/
with jjy0 jj < ". If y D y0 C x then y 2 B.x; "/ and q.y/ D q.y0 / C q.x/ D a 
q.x/ C q.x/ D a. The point a 2 C.q.x/; "/ was chosen arbitrarily, so C.q.x/; "/ 
q.B.x; "// and therefore C.q.x/; "/ D q.B.x; "//, i.e., . / is proved.
Since the open balls form a base in any metric space, we conclude that the map
q W M ! .M=F; / is open and continuous (see Fact 2 of S.491). Any open map
being quotient, the topology on M=F is the quotient one with respect to q, so
coincides with the quotient space topology on M=F .
Finally, assume that jj jj is a complete norm on M and take a Cauchy sequence
fan W n 2 !g  M=F with respect to . It is easy to construct by induction a
sequence fni W i 2 !g  ! such that ni < niC1 and .ani  ani C1 / < 2i for
each i 2 !. Pick x0 2 q 1 .an0 / arbitrarily; proceeding by induction assume that
j 2 ! and we have chosen x0 ; : : : ; xj 2 M in such a way that q.xi / D ani and
jjxi  xiC1 jj < 2i for every i < j . It follows from .anj  anj C1 / < 2j that there
exists y 2 anj C1  anj with jjyjj < 2j ; let xj C1 D xj C y. Then jjxj C1  xj jj D
jjyjj < 2j and q.xj C1 / D q.xj / C q.y/ D anj C .anj C1  anj / D anj C1 , so
we accomplished our inductive procedure and hence we can construct a sequence
fxi W i 2 !g such that q.xi / D ani and jjxi  xiC1 jj < 2i for every i 2 !.
It is easy to see that fxi W i 2 !g is a Cauchy sequence so there is x 2 M with
xi ! x. By continuity of q the sequence fq.xi / W i 2 !g D fani W i 2 !g converges
to q.x/, i.e., fani W i 2 !g is a convergent subsequence of our Cauchy sequence
S D fan W n 2 !g. Therefore S is convergent and hence we established that  is a
complete norm on M=F , so Fact 1 is proved.

2 Solutions of problems 001500

263

Returning to our solution observe that if L is (completely) normable then L=N is


also (completely) normable by Fact 1. If L is metrizable then L=N is first countable
being an open continuous image of L by (2). Therefore we can apply Problem 206
to conclude that L=N is also metrizable.
If L is locally convex then take any a 2 L=N and U 2 .a; L=N /; fix a point
x 2  1 .a/. The set U 0 D  1 .U / is an open neighborhood of x, so there exists
a convex V 2 .x; L/ with V  U 0 The set W D .V / is an open neighborhood
of a in L=N and W  U . Take any b; c 2 W; t 2 0; 1
and pick y; z 2 V such
that .y/ D b and .z/ D c. The set V being convex, the point v D ty C .1  t /z
belongs to V and therefore t b C .1  t /c D .v/ 2 W . This shows that W is a
convex open subset of L=N such that a 2 W  U , so convex open subsets of L=N
form a base in L=N , i.e., the space L=N is locally convex. Thus we have proved all
statements in (3), so our solution is complete.
V.213. Prove that any product of locally convex spaces is a locally convex space.
Solution. We will need the following general statement.

Q
Fact 1. Suppose that Lt is a linear space for anyQt 2 T and L D t2T Lt . If
Ct  Lt is a convex set for every t 2 T then C D t2T Ct is a convex subset of L.
Proof. Take any x; y 2 C; 2 0; 1
and let z D x C .1  /y; for any t 2 T ,
it
Qfollows from x.t /; y.t / 2 Ct that z.t / D x.t / C .1  /y.t / 2 Ct , so z 2
t2T Ct D C . Therefore C is convex and Fact 1 is proved.
Now assume that Lt is a locally convex space for all t 2 T and fix, for any t 2 T
t
and x 2 Lt , a local base BQ
of Bxt are convex.
x of Lt at x such that all elements
Q
Given any point a 2 L D t2T Lt the family Ua D f t2T Wt W Wt D Lt for all
t
but finitely many t and if Wt Lt then Wt 2 Ba.t/
g is a local base at a in the space
L. All elements of Ua are convex being the product of convex sets (see Fact 1).
Therefore open convex subsets of L constitute a base of L, so L is a locally convex
space.
V.214. Suppose that L and M are linear topological spaces and is an equicontinuous family of linear maps from L to M . Prove that, for any l-bounded set A  L
there is an l-bounded set B  M such that f .A/  B for all f 2 .
S
Solution. Let B D ff .A/ W f 2 g; then f .A/  B for any f 2 . Take any
open set W in the space M such that 0M 2 W . The family being equicontinuous
there is V 2 .0L ; L/ such that f .V /  W for any f 2 . Since A is l-bounded,
there exists s > 0 such that A  tV for any t  s. If y 2 B and t  s pick f 2
and x 2 A with f .x/ D y; it follows from x 2 tV that y D f .x/ 2 tf .V /  tW .
Thus B  tW for all t  s, so the set B is l-bounded in M .
V.215. Suppose that L and M are linear topological spaces and is a family of
linear continuous maps from L to M . Let .x/ D ff .x/ W f 2 g for every x 2 L
and assume that the set B D fx 2 L W .x/ is l-bounded in M g is of second
category in L. Prove that B D L and the family is equicontinuous.

264

2 Solutions of problems 001500

Solution. Take any W 2 .0M ; M / and use Problem 202 to find a set W 0 2
.0M ; M / with W 0 C W 0  W . By regularity of M and Problem 204 there exists
a balanced set V 2 .0M ; M / such that V  W 0 . Observe that the set V is also
balanced by Problem 203 and, in particular, V T
V.
All elements of are continuous, so E D ff 1 .V / W f 2 g is a closed
subset of L. If x 2 B then there is n 2 N such that .x/  nV because .x/ is
x
l-bounded in M . Therefore f .x/ 2 nV which shows
T 1that f . n /  V and hence
x
x
1
2 f .V / for any f 2 ; consequently, n 2 ff .V / W f 2 g  E. This
n
x
proves that, for any x 2 B there exists
S n 2 N such that n 2 E, i.e., x 2 nE.
We proved the inclusion B  fnE W n 2 Ng, so nE is of second category in
L for some n 2 N. The multiplication by n being a homeomorphism of L onto L, we
conclude that E is of second category in L. Recalling that E is closed we convince
ourselves that the interior of E is nonempty, so we can find a set U 2 .0L ; L/ and
a 2 E such that a C U  E.
Given any u 2 U and f 2 it follows from f .u/ D f .a C u/  f .a/ that
f .u/ 2 f .E/ C f .E/  V C .V /  V C V  W 0 C W 0  W . This shows that
f .U /  W ; the set W 2 .0M ; M / was chosen arbitrarily, so we proved that is
an equicontinuous family. Given any x 2 L the set fxg is l-bounded in L (this is an
easy consequence of the fact that every G 2 .0L ; L/ is absorbing), so the set .x/
is l-bounded in M by Problem 214. Thus every x 2 L belongs to B, i.e., B D L as
promised.
V.216 (HahnBanach theorem). Let L be a linear space (without topology).
Suppose that we are given a map p W L ! R such that p.x Cy/  p.x/Cp.y/ and
p.tx/ D tp.x/ for all x; y 2 L and t  0. Prove that, for any linear subspace M
of the linear space L and any linear functional f W M ! R such that f .x/  p.x/
for any x 2 M , there exists a linear functional F W L ! R such that F jM D f
and p.x/  F .x/  p.x/ for any x 2 L.
Solution. Consider the family L D f.N; '/ W N is a linear subspace of L with
M  N and ' W N ! R is a linear functional such that 'jM D f and '.x/  p.x/
for any x 2 N g. Given .N; '/ 2 L and .N 0 ; ' 0 / 2 L say that .N; '/ .N 0 ; ' 0 /
if N  N 0 and '  ' 0 ., i.e., ' 0 jN D '. It is easy to see that .L; / is a partially
ordered set; for any D .N; '/ 2 L let S./ D N and
S ./ D '.
Suppose that C is a chain in .L; / and let N D fS./ W 2 Cg. The family
fS./ W 2 Cg is a chain of linear subspaces of L, so N S
is a linear subspace of
L as well. We leave to the reader the checking that ' D f./ W 2 Cg is a
well-defined linear functional on N . Since ./jM D f for any 2 C, we have
'jM D f . Given x 2 N there is 2 C such that j'.x/j D j./.x/j  p.x/,
so '.x/  p.x/ for any x 2 N . This proves that .N; '/ 2 L; it is evident that
.N; '/ for any 2 C, so .N; '/ is an upper bound for the chain C.
We proved that any chain of .L; / has an upper bound which belongs to L, so
Zorns lemma is applicable to conclude that .L; / has a maximal element .N; '/.
If N D L then letting F D ' we obtain a linear functional F W L ! R such that
F jM D f and F .x/  p.x/ for any x 2 L.

2 Solutions of problems 001500

265

Assume, toward a contradiction, that N L and fix a vector v 2 LnN . It is


evident that N 0 D fx C t v W x 2 N; t 2 Rg is a linear subspace of L such that
N  N 0 and N N 0 . Observe also that, for any w 2 N 0 there are unique x 2 N
and t 2 R such that w D x C t v.
The formula '.x/ C '.y/ D '.x C y/  p.x C y/  p.x  v/ C p.v C y/
implies that
(1) '.x/  p.x  v/  '.y/ C p.v C y/ for any x; y 2 N .
It follows from (1) that, for a D supf'.x/  p.x  v/ W x 2 N g, we have
(2) '.x/  p.x  v/  a  '.y/ C p.v C y/ for all x; y 2 N .
For any x 2 N and t 2 R let .x C t v/ D '.x/ C t a; it is straightforward
that  W N 0 ! R is a linear functional such that jN D '. Fix any w 2 N 0 ; there
are unique x 2 N and t 2 R such that w D x C t v. If t D 0 then w 2 N ,
so .w/ D '.w/  p.w/. Now, assume that t > 0 and let y D xt in the second
inequality of (2); then a C '. xt /  p.v C xt / and hence '.x/ C t a  p.x C t v/,
i.e., .w/  p.w/.
x
in the first inequality of (2). This gives us the
If t < 0 then replace x with t
x
x
formula '. t /  a  p. t  v/ which can be multiplied by t to conclude that
'.x/ C t a  p.x C t v/, i.e., .w/  p.w/ in this case as well.
We have proved that .w/  p.w/ for any w 2 N 0 and hence the pair
.N 0 ; / belongs to L. We also have .N; '/ .N 0 ; / and .N; '/ .N 0 ; /;
this contradiction with maximality of .N; '/ shows that the case of N L is
impossible. Thus N D L and the linear functional F D ' is an extension of f
such that F .x/  p.x/ for any x 2 L. Multiplying the last inequality by .1/ we
convince ourselves that F .x/  p.x/; replacing x with x we conclude that
F .x/ D F .x/  p.x/, i.e., p.x/  F .x/  p.x/ for all x 2 L, so our
solution is complete.
V.217. Let L be a linear space (without topology). Suppose that we are given a
seminorm p W L ! R, a linear subspace M  L and a linear functional f W
M ! R such that jf .x/j  p.x/ for any x 2 M . Prove that there exists a linear
functional F W L ! R such that F jM D f and jF .x/j  p.x/ for any x 2 L.
Solution. We can apply Problem 216 to find a linear functional F W L ! R
such that F jM D f and p.x/  F .x/  p.x/ for every x 2 L. Since p
is a seminorm, we have p.x/ D p.x/ and hence p.x/  F .x/  p.x/, i.e.,
jF .x/j  p.x/ for every x 2 L.
V.218. Given a linear topological space L prove that any nontrivial continuous
linear functional f W L ! R is an open map.
Solution. Take any U 2 .0; L/ and apply Problem 204 to find an open balanced
set V such that 0 2 V  U . Suppose for a moment that f .V / D f0g and take any
x 2 L; the set V is absorbing by Problem 202, so there is t > 0 such that x 2 tV ,
i.e., xt 2 V and therefore f . xt / D 0 which implies that f .x/ D tf . xt / D t 0 D 0.
This shows that f .x/ D 0 for any x 2 L, i.e., f is a trivial functional which is a
contradiction. Therefore we can find a point x0 2 V with f .x0 / 0. The interval

266

2 Solutions of problems 001500

G D .jf .x0 /j; jf .x0 /j/ is an open neighborhood of 0; since V is balanced, the set
H D ftx0 W t 2 .1; 1/g is contained in V and hence G D f .H /  f .V /  f .U /.
This shows that for any open neighborhood U of the point 0 the set f .U / contains
an open neighborhood of 0, so the map f is open by Fact 3 of S.496.
V.219. Let L be a linear topological space and suppose that A and B are nonempty
disjoint convex subsets of L and A is open. Prove that there exists a continuous
linear functional f W L ! R such that, for some r 2 R, we have f .x/ < r  f .y/
for any x 2 A and y 2 B.
Solution. Pick any a 2 A; b 2 B and consider the set C D A  B C .b  a/.
We leave it to the reader to verify that 0 2 C and C is a convex open (and hence
absorbing) subset of L; let p W L ! R be the Minkowski functional of C . Observe
that x0 D b  a does not belong to C for otherwise there exist x 2 A; y 2 B such
that x  y C b  a D b  a which implies x D y and hence x 2 A \ B D ; which
is a contradiction. It follows from x0 C and Problem 208 that p.x0 /  1.
The set M D ftx0 W t 2 Rg is a linear subspace of L; let '.tx0 / D t for
any t 2 R. It is straightforward that ' W M ! R is a linear functional. If t  0
then '.tx0 / D t  tp.x0 / D p.tx0 /; if t < 0 then '.tx0 / D t < 0  p.tx0 /,
so '.y/  p.y/ for any y 2 M . By Problem 216 there is a linear functional
f W L ! R such that f jM D ' and f .x/  p.x/ for any x 2 L. In particular,
f .x/  p.x/  1 for any x 2 C (see Problem 208) and hence f .x/  1 for any
x 2 C ; thus W D C \ .C / is a neighborhood of 0 such that f .W /  1; 1
is
a bounded subset of R. Therefore the functional f is continuous by Problem 205.
Given x 2 A and y 2 B the point z D x  y C x0 belongs to C . By continuity of
z
2 C , so p.z/  1t <
scalar multiplication in L there exists t > 1 such that t z D 1=t
1. Therefore f .z/ D f .x/  f .y/ C 1  p.z/ < 1 which shows that f .x/ < f .y/
for any x 2 A and y 2 B. As a consequence, for the number r D supff .x/ W
x 2 Ag, we have f .x/  r  f .y/ whenever x 2 A and y 2 B. Observe
that f .x0 / D '.x0 / D 1, so the functional f is nontrivial and hence open by
Problem 218. This implies that f .A/ is an open subset of R, so r D sup f .A/
cannot belong to f .A/. Consequently, f .x/ < r  f .y/ for any x 2 A and y 2 B.
V.220. Let L be a locally convex linear topological space. Suppose that A and B
are disjoint convex subsets of L such that A is compact and B is closed. Prove that
there exists a continuous linear functional f W L ! R such that, for some r; s 2 R,
we have f .x/ < r < s < f .y/ for any x 2 A and y 2 B.
Solution. For any a 2 A there exists an open neighborhood Ua of the point 0
such that .a C Ua / \ B D ;; choose a convex open neighborhood Va of 0 with
Va C Va 
can find a finite set K  A such
S Ua . The set A being compact weT
that A  fa C Va W a 2 Kg. The set V D fVa W a 2 Kg is an open convex
neighborhood of 0. If x 2 .A C V / \ B then x D y C v for some y 2 A and
v 2 V ; pick a point a 2 K such that y 2 a C Va . Then x 2 y C V  y C Va 
a C Va C Va  a C Ua , so x 2 .a C Ua / \ B which is a contradiction. Therefore
W D A C V is a convex open set (it is an easy exercise that the sum of two convex
sets is a convex set) disjoint from B.

2 Solutions of problems 001500

267

Applying Problem 219 we conclude that there exists a continuous linear functional f W L ! R such that, for some t 2 R, we have f .x/ < t  f .y/ for
all x 2 W and y 2 B. The set f .A/  f .W /  .1; t / being compact, there
exists r < t for which f .A/  .1; r/; if s D rCt
then f .A/  .1; r/ and
2
f .B/  .s; C1/ which is equivalent to saying that f .x/ < r < s < f .y/ for any
x 2 A and y 2 B.
V.221. Let L be a locally convex linear topological space. Prove that L separates
the points of L.
Solution. If a and b are distinct points of L then A D fag and B D fbg are disjoint
convex compact subsets of L, so we can apply Problem 220 to see that there is a
continuous linear functional f W L ! R (i.e., f 2 L ) such that, for some r; s 2 R,
we have f .x/ < r < s < f .y/ whenever x 2 A and y 2 B. This is the same as
saying that f .a/ < r < s < f .b/, so f .a/ f .b/ and hence L separates the
points of L.
V.222. Let M be a linear subspace of a locally convex linear topological space L
and x0 M . Prove that there exists f 2 L such that f .x0 / D 1 and f .M / D f0g.
Solution. The set N D M is also a linear subspace of L by Problem 203. Every
linear subspace of L is clearly convex so A D fx0 g and N are closed disjoint convex
subsets of L and A is compact. This makes it possible to apply Problem 220 to find
a functional g 2 L such that, for some r; s 2 R, we have
(1) g.x0 / < r < s < g.y/ and, in particular, g.y/ g.x0 / for any y 2 N .
0/
If g.x/ 0 for some x 2 N then t D g.x
is well defined and y D tx 2 N ,
g.x/
so g.y/ D tg.x/ D g.x0 / which is a contradiction with (1). Therefore g.x/ D 0
for any x 2 N ; letting f D g.x1 0 / g we obtain a functional f 2 L such that
f .N / D f0g and f .x0 / D 1 as promised.

V.223. Let B be a closed convex balanced subset of a locally convex space L. Prove
that, for any x 2 LnB, there exists a continuous linear functional f W L ! R such
that f .B/  1; 1
and f .x/ > 1.
Solution. If B D ; then we can apply Problem 221 to find a functional g 2 L
2
with g.x/ 0. It is evident that, for the functional f D g.x/
g 2 L , we have
f .x/ D 2 > 1 and f .B/ D ;  1; 1
.
Now assume that B ; and fix a point b 2 B; the sets A D fxg and B
are closed, convex and disjoint; besides, A is compact. Therefore Problem 220 is
applicable to convince ourselves that there exists a functional g 2 L with the
following property:
(1) there is r 2 R such that g.x/ < r < g.y/ for any y 2 B.
Since B is balanced, the point 0 D 0 b belongs to B and therefore r < g.0/ D 0.
Take any z 2 B and suppose that g.z/ > r D jrj; the point y D z also belongs
to B and hence g.y/ D g.z/ < r which is a contraction with (1). This shows

268

2 Solutions of problems 001500

that g.B/  jrj; jrj


. The functional f D 1r g is continuous on L and we have
1
jg.y/j  1 for any y 2 B, i.e., f .B/  1; 1
. Finally, observe that
jf .y/j D jrj
g.x/ < r < 0 implies that f .x/ D 1r g.x/ > 1.
V.224. Let L be a locally convex linear topological space. Given a linear subspace
M of the linear space L and a continuous linear functional f W M ! R, prove that
there exists a functional g 2 L such that gjM D f .
Solution. If f is trivial then the trivial functional on L is the desired extension
of f . Now assume that f is nontrivial and fix a point x0 2 M such that f .x0 / 0.
If N D f 1 .0/ then N  M is a linear subspace of L; it follows from continuity
of f that x0 N , so we can apply Problem 222 to find a functional h 2 L
such that h.x0 / D 1 and h.N / D f0g. Then g D f .x0 / h is a continuous linear
functional on L such that g.N / D f0g and g.x0 / D f .x0 /; take any point x 2 M .
.x/
If y D x  ff .x
x0 then it is immediate that f .y/ D 0 and hence y 2 N . Therefore
0
g.y/ D g.x/  ff.x.x/0 / g.x0 / D 0; recalling that g.x0 / D f .x0 / we conclude that
g.x/  f .x/ D 0, i.e., g.x/ D f .x/. We proved that g.x/ D f .x/ for any x 2 M ,
so gjM D f as required.
V.225. Given a linear space L (without topology) denote by L0 the family of all
linear functionals on L. Suppose that M  L0 is a linear subspace of L0 (i.e.,
f C g 2 M whenever f; g 2 M and ; 2 R) and M separates the points of
L; let  be the topology generated by the set M . Then LM D .L; / is a locally
convex space and .LM / D M . Deduce from this fact that if L is a locally convex
space and Lw is the set L with the weak topology of the space L then Lw is a locally
convex space such that .Lw / D L .
Solution. For any f 2 M let O.f; n/ D fx 2 L W jf .x/j < n1 g for each n 2 N;
observe also that the function pf D jf j is a seminorm on L and fpf W f 2 M g is
a separating family of seminorms on L. Therefore we can apply Problem 210 to see
that there exists a unique locally convex linear space topology on the set L such
that B D fO.f1 ; n/ \ : : : \ O.fn ; n/ W n 2 N; f1 ; : : : ; fn 2 M g is a local base
at 0; it is evident that  . If f 2 M and " > 0 then pick n 2 N with n1 < ".
The set O.f; n/ 3 0 belongs to and f .O.f; n//  ."; "/; this proves that every
f 2 M is continuous at 0 and hence continuous on .L; / (see Fact 2 of S.496). An
immediate consequence is that   and hence  D is a locally convex linear
space topology on L.
It is straightforward that M  .LM / ; to prove the opposite inclusion take any
functional f 2 .LM / . By continuity of f at 0 there exist n 2 N and f1 ; : : : ; fn 2
D f11 .0/ \ : : : \
M such that f .O.f1 ; n/ \ : : : \ O.fn ; n//  .1; 1/. Let N T
fn1 .0/; we have fi1 .0/  O.fi ; n/ for every i  n, so N  niD1 O.fi ; n/ and
therefore f .N /  .1; 1/. If x 2 N and r D f .x/ 0 then y D 1r x 2 N
(because N is a linear subspace of L), so f .y/ D 1r f .x/ D 1 .1; 1/ which
is a contradiction. Thus f .N / D f0g, so we can apply Fact 14 of V.100 to find
t1 ; : : : ; tn 2 R such that f D t1 f1 C : : : C tn fn ; since M is a linear subspace of L0 ,
we proved that f 2 M and hence .LM / D M .

2 Solutions of problems 001500

269

Finally observe that if L is a locally convex space then L separates the points of
L by Problem 221; since .Lw / is generated by L , the space Lw is locally convex
and .Lw / D L .
V.226. Let E be a convex subset of a locally convex space L. Prove that the closure
of E in L coincides with the closure of E in the weak topology of L.
Solution. Denote by  the weak topology of L; given A  L the set A is the closure
of A in L and cl .A/ is the closure of A in .L; /. It follows from   .L/ that
A  cl .A/ for any A  L, so it suffices to show that cl .E/  E.
Fix a point a E; the sets A D fag and E are convex (see Problem 203), closed,
disjoint and A is compact so we can apply Problem 220 to find a functional f 2 L
and r 2 R such that f .a/ < r < f .y/ for any y 2 E. The set U D f 1 ..1; r//
belongs to  while a 2 U and U \ E D ;; an immediate consequence is that
a cl .E/ and hence cl .E/ D E.
V.227. Let V be a neighborhood of 0 in a locally convex space L. Prove that the
set P .V / D ff 2 L W f .V /  1; 1
g is compact if considered with the topology
induced from Cp .L/.
Solution. The topology induced on P .V / from Cp .L/  RL is the same as the
topology induced from RL , so it suffices to show that P .V / is closed in RL and
there is a compact K  RL such that P .V /  K.
Denote by L0 the set of all (not necessarily continuous) linear functionals on L
and take any point f 2 P .V / (the bar denotes the closure in RL ). The set L0  RL
is closed in RL (see CFS-393) and P .V /  L0 , so f 2 L0 , i.e., f is a linear
functional on L. If there is x 2 L such that f .x/ I then G D fg 2 RL W g.x/ Ig
is an open neighborhood of f in RL such that G \ P .V / D ;; this contradiction
shows that f .x/ 2 I for any x 2 V , i.e., f .V /  I and hence f .V / is a bounded
subset of R. Applying Problem 205 we conclude that f is continuous and hence
f 2 P .V /. Thus P .V / is closed in RL .
Now fix a point x 2 L; the set V is absorbing by Problem 202, so there is r.x/ 2
R such that r.x/ > 0 and x 2 r.x/V , i.e., there exists v 2 V with x D r.x/ v.
If f 2 P .V / then jf .x/j D r.x/ jf .v/j  r.x/; therefore f .x/ 2 r.x/; r.x/

for any f 2 P .V /. The pointQx 2 L was chosen arbitrarily, so we have proved


the inclusion P .V /  K D fr.x/; r.x/
W x 2 Lg. Consequently, P .V / is
compact being a closed subset of a compact set K.
V.228. Given n 2 N suppose that L is a linear topological space and M is a linear
subspace of L of linear dimension n. Prove that M is closed in L and every linear
isomorphism ' W Rn ! M is a homeomorphism.
Solution. Our first step is to prove that locally compact linear subspaces are always
closed in a linear topological space.
Fact 1. Suppose that N is a linear topological space and G  N is a locally
compact linear subspace of N . Then G is closed in N .

270

2 Solutions of problems 001500

Proof. Fix a compact set K  G which contains a neighborhood of 0 in the space


G and pick a set O 2 .0; N / such that O \ G  K. Use Problems 202 and 204 to
find a symmetric set V 2 .0; N / with V C V  O and take any point x 2 G.
The set U D x C V is an open neighborhood of the point x in N ; an easy
consequence is that x 2 U \ G. Fix a point x0 2 P D U \ G and take any
y 2 P . There exist points v0 ; v1 2 V such that x0 D x C v0 and y D x C v1 . Then
y  x0 D v1  v0 2 V C V  O; since G is a linear subspace of N , the point y  x0
belongs to G. Therefore y  x0 2 O \ G  K which shows that y 2 x0 C K for
any y 2 P and hence P  K 0 D x0 C K  G. The space K 0 being compact we
conclude that P  K 0 , so x 2 P  G and hence x 2 G. Thus G  G, i.e., G is
closed in N , so Fact 1 is proved.
Returning to our solution consider, for any n 2 N, the following statement:
.Sn / if L is a linear topological space and M is a linear subspace of L of linear
dimension n then every isomorphism ' W Rn ! M is a homeomorphism.
To prove inductively that .Sn / is true for each n 2 N assume first that n D 1
and ' W R ! M is an isomorphism. If x0 D '.1/ then '.t / D t x0 for any
t 2 R. It follows from continuity of multiplication by scalars in L that the map '
is continuous. The map  D ' 1 W M ! R is linear, i.e.,  is a nontrivial linear
functional on M . Besides,  1 .0/ D f0g is a closed subspace of M , so  is also
continuous by Problem 205. Thus ' is a homeomorphism, so .S1 / is proved.
Suppose that n 2 N; n > 1, the statement .Sk / is proved for any k < n and take
any linear isomorphism ' W Rn ! M . For each i D 1; : : : ; n let ei D .e1i ; : : : ; eni /
where eii D 1 and eji D 0 whenever i j . Then fe1 ; : : : ; en g is a linear basis in
Rn , so G D f'.e1 /; : : : '.en /g is a linear basis in M . If gi D '.ei / for all i  n
then '.t1 ; : : : ; tn / D t1 g1 C : : : C tn gn for any point .t1 ; : : : ; tn / 2 Rn . An easy
consequence of continuity of the operations in L is that the map ' is continuous.
For any point x 2 M there are uniquely determined t1 ; : : : ; tn 2 R such that
x D t1 g1 C : : : C tn gn ; let pi .x/ D ti for each i  n. It is easy to see that every
pi W M ! R is a linear functional and ' 1 D p1 : : : pn , so, to prove continuity
of ' 1 , it suffices to show that every pi is continuous. Observe that pi1 .0/ coincides
with the linear hull of the set Gnfgi g, so F D pi1 .0/ is an .n  1/-dimensional
linear subspace of M . By the induction hypothesis, the space F is homeomorphic to
Rn1 and hence locally compact. Therefore F is closed in M by Fact 1. This makes
it possible to apply Problem 205 again to conclude that every pi is continuous and
hence ' 1 is also continuous, i.e., ' is a homeomorphism.
Our inductive step having been accomplished, we have proved that .Sn / holds
for every n 2 N. It is an easy exercise that if a linear space M has linear dimension
n then there exists an isomorphism between M and Rn ; consequently, the subspace
M in .Sn / is homeomorphic to Rn which, in turn, shows that M is locally compact
and hence closed in L by Fact 1. Therefore, in the statement .Sn /, the set M is
automatically closed in L, so our solution is complete.
V.229. Given a linear topological space L prove that the following conditions are
equivalent:

2 Solutions of problems 001500

271

(i) L has a finite Hamel basis, i.e., the linear dimension of L is finite;
(ii) dim L  n for some n 2 N;
(iii) L is locally compact.
Solution. If L has linear dimension n for some n 2 N then L is locally compact
being homeomorphic to Rn (see Problem 228); this settles (i)H)(iii). Besides,
it follows from Problem 159 that dim L D n, so we also have the implication
(i)H)(ii).
Next, assume that n 2 N and dim L  n. If there are some linearly independent
vectors e1 ; : : : ; enC1 2 L then the linear hull M of the set fe1 ; : : : ; enC1 g is a linear
subspace of L of linear dimension n C 1, so it follows from Problem 228 that M
is homeomorphic to RnC1 . As a consequence, L has a subspace K homeomorphic
to InC1 . A compact subspace is C -embedded in any space (see Fact 1 of T.218), so
dim K  dim L  n by Problem 146. However, dim K D dim.InC1 / D n C 1
by Problem 159; this contradiction shows that the linear dimension of L does not
exceed n and hence (ii)H)(i) is proved.
To establish that (iii)H)(i) assume that L is locally compact and hence we can
find a balanced set O 2 .0; L/ such that K D O is compact; let Wn D 2n O
for any n 2 !. If U is an open neighborhood of 0 then pick an open balanced
neighborhood V of 0 such that V  U . Since V is an absorbing subset of L by
Problem 202, we can find, for every x 2 K a number tx > 0 such
S that x 2 tx V .
The set P
K being compact there is a finite A  K for which K  ftx V W x 2 Ag;
let t D ftx W x 2 Ag.
Since the set V is balanced, we have tx V  t V for any x 2 A and hence
K  t V . This implies O  t V and therefore 1t O  V ; pick n 2 ! with 2n < 1t .
Then Wn D 2n O  1t O  V  U . We proved that, for any U 2 .0; L/ there
is n 2 ! with Wn  U and hence the family W D fWn W n 2 !g is a local base of
L at 0.
S Apply1 once more compactness of K to find a finite set B  K such that K 
fx C 2 O W x 2 Bg. If M is the linear hull of B then M has finite linear
dimension and O  K  M C 12 O. Proceeding inductively assume that n 2 N and
O  M C 2n O. The set M being a linear subspace of L we have 2n M D M ,
so 2n O  2n .M C 12 O/ D M C 2n1 O. An immediate consequence is
that O  M C M C 2n1 O D M C 2n1 O. Thus our inductive procedure
shows that O  M C Wn for any n T
2 !. Since W is a local base of L at 0, we can
apply Problem 203 to see that O  fM C Wn W n 2 !g D M . By Problem 228,
the set M is closed in L, so M D M and hence O  M . The set O is absorbing,
so, for any x 2 L there is t 2 R with x 2 t O  t M D M ; therefore L D M
and hence linear dimension of L is finite; this settles (iii)H)(i) and completes our
solution.
V.230. Suppose that L is a finite-dimensional linear topological space. Prove that
any linear functional f W L ! R is continuous on L, i.e., L0 D L . Give an
example of an infinite-dimensional locally convex space M such that M 0 D M  .

272

2 Solutions of problems 001500

Solution. The set N D f 1 .0/  L is a finite-dimensional linear subspace of L,


so N is closed in L by Problem 228. Therefore f is continuous by Problem 205 and
hence L0 D L .
Let D be an infinite discrete space. The space M D Lp .D/ is locally convex
being a linear subspace of a locally convex space Cp .Cp .D// (see Fact 1 of T.131).
We can consider that D  Lp .D/ and D is a Hamel basis of Lp .D/ (see Fact 5
of S.489); therefore the space M is infinite-dimensional. If f W M ! R is a linear
functional then f is a linear extension of the function g D f jD; the space D being
discrete, the function g is continuous, so there exists a continuous linear functional
h W M ! R such that hjD D g. The set D is a Hamel basis of M , so there
is only one linear extension of g over M by Fact 3 of S.489; therefore f D h
is a continuous linear functional on M . Thus M is an infinite-dimensional locally
convex space such that M 0 D M  .
V.231. Let L be a locally convex space. Denote by L0  RL the set of all (not
necessarily continuous) linear functionals on L with the topology induced from RL .
Prove that L is dense in L0 .
Solution. Fix a linear functional f W L ! R and let A be a finite subset of L.
The linear span M of the set A is a finite-dimensional linear subspace of L, so the
functional f0 D f jM is continuous by Problem 230. Apply Problem 224 to find
a continuous linear functional g W L ! R such that gjM D f0 ; since A  M ,
we also have gjA D f0 jA D f jA. This shows that, for any finite A  L there
is g 2 L with gjA D f jA. An immediate consequence is that f belongs to the
closure of the set L . The point f 2 L0 was chosen arbitrarily, so L is dense in L0 .
V.232. Given a linear space L let L0  RL be the set of all linear functionals on L
with the topology induced from RL . Prove that L0 is linearly homeomorphic to RB
for some B.
Solution. Apply Fact 1 of S.489 to fix a Hamel basis B  L and consider the
restriction map  W L0 ! RB . It is clear that  is linear and continuous; Fact 3 of
S.489 implies that .L0 / D RB and  is an injection, i.e.,  is a condensation. We
will denote by 0L the zero function on L (which is also the zero element of L0 ); let
0B D .0/ be the zero function on B.
Given a finite set A  L and " > 0 let O.A; "/ D ff 2 L0 W f .A/  ."; "/g.
It is clear that the family O D fO.A; "/ W A is a finite subset of L and " > 0g
is a local base of L0 at 0L . Analogously, if C  B is a finite set and " > 0 then
W .C; "/ D ff 2 RB W f .C /  ."; "/g; the family W D fW .C; "/ W C is a finite
subset of B and " > 0g is a local base of RB at 0B .
Take an arbitrary set U 2 .0L ; L0 /; we can fix A  L and " > 0 such that
O.A; "/  U . The set B being a Hamel basis of L there exist b0 ; : : : ; bn 2 B such
that A is contained in the linear span of the set C D fb0 ; : : : ; bn g. Thus, for every
point x 2 A there exist t0x ; : : : ; tnx 2 R such
x D t0x b0 C : : : C tnx bn . We will
P that
"
x
need the number D K > 0 where K D fjti j W x 2 A; i  ng C 1.

2 Solutions of problems 001500

273

Take any f 2 W .C; / and let ' D  1 .f /. Since the functional


P ' is a linear
extension of f , for any point x 2 A we have theP
equality '.x/ D niD0 tix f .bi /, so
j'.x/j  jt0x jjf .b0 /j C : : : C jtnx jjf .bn /j  niD0 jtix j < K D " and therefore
' 2 O.A; "/. An immediate consequence is that .U /  .O.A; "//  W .C; /,
so we proved that the image under  of every U 2 .0L ; L0 / contains an open
neighborhood of 0B in RB . Applying Fact 3 of S.496 we convince ourselves that the
map  is open, i.e.,  is the desired linear homeomorphism between the spaces L0
and RB .
V.233. For any linear topological space L denote by w .L/ the weak topology of
the space L. Prove that
(1) if w .L/ D .L/ and M is a linear subspace of L then w .M / D .M /;
(2) for any space X , the topology of Cp .X / coincides with its weak topology;
(3) for any space X , the topology of Lp .X / coincides with its weak topology.
Solution. The following fact is useful for establishing coincidence of topologies on
linear spaces.
Fact 1. Given a linear space L suppose that and  are linear space topologies on
L such that, for any U 2 with 0L 2 U there exists V 2  such that 0 2 V  U .
Then  .
Proof. Let i W .L; / ! .L; / be the identity map, i.e., i.x/ D x for any x 2 L.
Our hypothesis is a reformulation of the fact that the map i is continuous at 0L . Since
i is a linear map, we can apply Fact 2 of S.496 to conclude that i is continuous.
Therefore U 2 implies that U D i 1 .U / 2 , so   and hence Fact 1 is
proved.
Returning to our solution fix a linear topological space L with .L/ D w .L/
and take any linear subspace M  L. It is evident that w .M /  .M /, so take any
U 2 .0L ; M /; there exists V 2 .0L ; L/ such that V \ M D U . It follows from
the equality
D w .L/ that there exist f0 ; : : : ; fn 2 L and " > 0 such that the
T .L/
1
set G D ffi .."; "// W i  ng is contained in V . The functional gi D fi jM is
1
continuous on M , so
T the set Wi D gi .."; "// belongs to w .M / for each i  n.
Thus the set H D fWi W i  ng also belongs to w .M /; it is straightforward that
0L 2 H  G \ M  V \ M D U . This shows that, for every U 2 .0L ; M /, there
exists H 2 w .M / such that 0L 2 H  U and hence .M /  w .M / by Fact 1.
Therefore .M / D w .M /; this settles (1).
(2) Let be the topology of Cp .X / and denote by w its weak topology; it is evident
that w  . Fix a set U 2 with 0X 2 U . There exists a finite set A  X such
that U0 D ff 2 Cp .X / W f .A/  ."; "/g  U .
For any x 2 A let ex .f / D f .x/ for any f 2 Cp .X /; then ex W Cp .X / ! R is a
continuous linear functional on CpT.X / and hence the set Vx D ex1 .."; "// belongs
to w . It is immediate that U0 D fVx W x 2 Ag, so U0 2 w . This shows that, for
every U 2 with 0X 2 U there is U0 2 w such that 0 2 U0  U . Therefore  w
(see Fact 1) and hence D w .

274

2 Solutions of problems 001500

(3) Recall that Lp .X / is a linear subspace of Cp .Cp .X //; since the weak topology
of Cp .Cp .X // coincides with its topology by (2), we can apply (1) to conclude
that the weak topology of Lp .X / coincides with the topology of Lp .X /.
V.234. Suppose that L is a locally convex space with its weak topology and X  L
is a Hamel basis in L. Prove that the following conditions are equivalent:
(i) there exists a linear homeomorphism h W L ! Lp .X / such that h.x/ D x for
all x 2 X ;
(ii) for every f 2 C.X / there exists a continuous linear functional ' W L ! R
such that 'jX D f ;
(iii) for every continuous map f W X ! M from X to a locally convex space M
with its weak topology, there exists a continuous linear map W L ! M such
that jX D f .
Solution. Assume that (i) holds and take a continuous function f W X ! R. Apply
Fact 6 of S.489 to find a continuous linear functional '0 W Lp .X / ! R such that
'0 jX D f . Then ' D '0 h is a continuous linear functional on L. If x 2 X
then '.x/ D '0 .h.x// D '0 .x/ D f .x/, so 'jX D f and hence we proved that
(i)H)(ii).
Next, assume (ii) and suppose that we have a continuous map f W X ! M for
some locally convex space M with its weak topology. It follows from Problem 225
that the set M   Cp .M / generates the topology of M ; let e.a/./ D .a/ for any
a 2 M and  2 M  . If M0 D fe.a/ W a 2 M g  Cp .M  / then M0 is a linear
subspace of Cp .M  / and e W M ! M0 is a linear homeomorphism (see TFS-166).
For every  2 M  , there exists a continuous linear functional r W L ! R such
that r jX D  f . For any point y 2 L let r.y/./ D r .y/ for any  2 M  ;

this gives us a continuous linear map r W L ! RM . Since X is a Hamel basis
of L, the set r.L/ is contained in a linear hull N of the set r.X /. For any x 2 X
and  2 M  we have the equalities r.x/./ D r .x/ D .f .x// D e.f .x//./
which show that r.x/ D e.f .x//, i.e., r D e f . Therefore r.X / D e.f .X //
is contained in M0 and hence N  M0 . Thus we can consider that r W L ! M0
and therefore the map D e 1 r W L ! M is linear and continuous. Besides,
.x/ D e 1 .r.x// D f .x/ for any x 2 X ; this shows that we obtained a continuous
linear map W L ! M such that jX D f , i.e., (ii)H)(iii) is proved.
Finally assume that (iii) is fulfilled for L and denote by 0L and 0 the zeros in L
and Lp .X / respectively. Observe that the property (ii) holds for Lp .X / by Fact 6
of S.489, so Lp .X / satisfies (iii) as well. Let id W X ! X be the identity map, i.e.,
id.x/ D x for any x 2 X ; we can consider that id W X ! Lp .X /. In the space
Lp .X / the original topology coincides with its weak topology by Problem 233, so
there exists a continuous linear map h W L ! Lp .X / such that hjX D id. The set
h.L/ is a linear subspace of Lp .X / with X  h.L/; since X is a Hamel basis in
Lp .X / (see Fact 5 of S.489), we conclude that h.L/ D Lp .X /.
To simplify the notation we denote in the same way the operations in L and
Lp .X /. Since X  L and X  Lp .X /, for any x1 ; : : : ; xn 2 X and t1 ; : : : ; tn 2 R,

2 Solutions of problems 001500

275

the linear combination z D t1 x1 C : : : C tn xn can be viewed either as an element of


L or as an element of Lp .X /; the space to which the point z belongs will be always
clear from the context.
Given distinct x1 ; : : : ; xn 2 X and t1 ; : : : ; tn 2 Rnf0g suppose that a point z D
t1 x1 C : : : C tn xn is an element of the space L and h.z/ D 0; by linearity of h, we
have h.z/ D t1 h.x1 / C : : : C tn h.xn /. Since h.xi / D id.xi / D xi for any i  n,
it turns out that h.z/ D t1 x1 C : : : C tn xn D 0 (the operations in the last equality
are carried out in Lp .X //, which is a contradiction with the fact that X is linearly
independent in Lp .X /. Therefore h is a continuous linear isomorphism.
If we consider that id W X ! L then apply the property (iii) for Lp .X / to find a
continuous linear map  W Lp .X / ! L such that jX D id. As before, it is easy to
check that, for any z D t1 x1 C : : : C tn xn 2 Lp .X / the point .z/ coincides with
w D t1 x1 C : : : C tn xn considered to be an element of L. Therefore the map h1
coincides with a continuous map  and hence h W L ! Lp .X / is the required linear
homeomorphism, i.e., settled (iii)H)(i) thus completing our solution.
V.235. Given a space X let .Lp .X // D f' 2 Cp .Lp .X // W ' is a linear
functional on Lp .X /g and consider the restriction map  W .Lp .X // ! Cp .X /.
Prove that .Lp .X // is a closed linear subspace of Cp .Lp .X // and  is a linear
homeomorphism. Deduce from this fact that the operation of extending continuous
real-valued functions on X to continuous linear functionals on Lp .X / is also a
linear homeomorphism between Cp .X / and .Lp .X // .
Solution. Let .Lp .X //0 be the set of all (not necessarily continuous) linear
functionals on Lp .X /. Then .Lp .X // D .Lp .X //0 \ Cp .Lp .X //; since .Lp .X //0
is closed in RLp .X/ by CFS-393, the set .Lp .X // is closed in Cp .Lp .X //. It is
evident that .Lp .X // is a linear subspace of Cp .Lp .X //.
To see that the restriction map  W .Lp .X // ! Cp .X / is well defined recall
that TFS-167 and Fact 5 of S.489 make it possible to consider that X  Lp .X /
identifying every x 2 X with the linear functional ex W Cp .X / ! R defined by
ex .f / D f .x/ for any f 2 Cp .X /. It follows from Fact 6 of S.489 that the map 
is a continuous linear bijection.
For any f 2 Cp .X / and  2 Lp .X / let u.f /./ D .f /; it follows from
TFS-166 that the map u W Cp .X / ! Cp .Lp .X // is continuous. Fix a function
f 2 Cp .X /; given any 1 ; 2 2 Lp .X / and ; 2 R it follows from the equalities
u.f /.1 C2 / D .1 C2 /.f / D 1 .f /C2 .f / D u.f /.1 /Cu.f /.2 /
that u.f / is a linear functional on the space Lp .X / for any f 2 Cp .X / and hence
we can consider that u W Cp .X / ! .Lp .X // . Now if f 2 Cp .X / and x 2 X then
u.f /.x/ D x.f / D f .x/ which shows that .u.f // D f , so the continuous map
u coincides with  1 . Therefore both  and u are linear isomorphisms between
the spaces Cp .X / and .Lp .X // . Since f ! u.f / is precisely the operation of
extending a function f 2 C.X / to a continuous linear functional u.f / on the
space Lp .X /, this operation is also a linear homeomorphism between Cp .X / and
.Lp .X // .

276

2 Solutions of problems 001500

V.236. Prove that any Lp .X / is homeomorphic to a dense subspace of RA for some


A. Deduce from this fact that every uncountable regular cardinal is a precaliber of
Lp .X /. In particular, c.Lp .X // D ! for any space X .
Solution. Let L D .Cp .X //0  RCp .X/ be the set of all linear functionals on
Cp .X /; apply Problem 232 to see that L is homeomorphic to RA for some set
A. The space Cp .X / is locally convex (see Problem 225 and Problem 233), so
the set .Cp .X // is dense in L by Problem 231. It follows from TFS-197 that
.Cp .X // D Lp .X /, so Lp .X / is dense in L. If  is an uncountable regular cardinal
then it is a caliber of RA (and hence of L) by SFFS-282; therefore  is a precaliber
of Lp .X / by SFFS-278.
V.237. Given spaces X and Y prove that
(i) there exists a linear continuous map of Cp .X / onto Cp .Y / if and only if Lp .Y /
is linearly homeomorphic to a linear subspace of Lp .X /;
(ii) there exists a linear continuous open map of Cp .X / onto Cp .Y / if and only if
Lp .Y / is linearly homeomorphic to a closed linear subspace of Lp .X /;
(iii) the space Cp .X / linearly condenses onto Cp .Y / if and only if Lp .Y / is
linearly homeomorphic to a dense linear subspace of Lp .X /;
(iv) Cp .X / is linearly homeomorphic to Cp .Y / if and only if Lp .Y / is linearly
homeomorphic to Lp .X /.
Solution. Given any set P  Lp .X / we denote by hP i the linear hull of P in
Lp .X /. If i 2 ! and we consider a set A D fa1 ; : : : ; ai g  Lp .X / then i D 0 says
that A D ;. By 0X we denote the element of Cp .X / which is identically zero on X ;
the zero vector of Lp .X / is denoted by 0.
We will simultaneously prove sufficiency in (i)(iv). Assume that we have a
linear homeomorphism ' W Lp .Y / ! L for some L  Lp .X / (which is
closed in Lp .X / or dense in Lp .X / or coincides with Lp .X / respectively) and
let Y 0 D '.Y /. For every function f 2 Cp .X / there exists a unique continuous
linear functional u.f / on Lp .X / such that u.f /jX D f (see Problem 234); it
follows from Problem 235 that the map u W Cp .X / ! .Lp .X // is a linear
homeomorphism. If  W .Lp .X // ! Cp .Y 0 / is the restriction map then it is
continuous and linear; besides,  is a homeomorphism (see Problem 235) if we
are proving sufficiency in (iv). In the case when L is dense in Lp .X / observe that
 D 1 0 where 0 W .Lp .X // ! L and 1 W L ! Cp .Y 0 / are the restriction
maps. The map 1 is injective by Problem 235 and 0 is injective because L is dense
in Lp .X / (see TFS-152). Therefore the map  is injective if we are dealing with the
property (iii).
As a consequence, the map  D  u W Cp .X / ! Cp .Y 0 / is continuous and
linear (and homeomorphic respectively), so the proof of sufficiency in (iv) ends
here. In the remaining three cases apply Problem 234 to see that, for every g 2
Cp .Y 0 / there is a continuous linear functional e W L ! R such that ejY 0 D g. By
Problem 224 there exists w 2 .Lp .X // such that wjL D e. Pick f 2 Cp .X / such
that w D u.f /. Then .f / D . u/.f / D .w/ D ejY 0 D g and hence  maps
Cp .X / (injectively) onto Cp .Y 0 /; the space Y being homeomorphic to Y 0 , there
exists a continuous linear (injective or homeomorphic respectively) map of Cp .X /
onto Cp .Y /, so we settled sufficiency in (i), (iii), and (iv).

2 Solutions of problems 001500

277

Let us show that the map  is open if the set L is closed in Lp .X /. Fix any open
set U  Cp .X / such that 0X 2 U . There is a finite set A  X such that W D ff 2
Cp .X / W f .A/  ."; "/g  U for some " > 0. It is easy to construct inductively a
set A0 D fa1 ; : : : ; ak g  A such that A  hL [ A0 i and aiC1 hL [ fa1 ; : : : ; ai gi
whenever 0  i < k.
Suppose that Y 0 [ A0 is not independent; since both sets Y 0 and A0 are
independent, the number k has to be positive and there exists n 2 N such that
fy1 ; : : : ; yn ; a1 ; : : : ; ak g is a dependent set for some distinct points y1 ; : : : ; yn 2 Y 0 .
If 1 y1 C : : : n yn C t1 a1 C : : : C tk ak D 0 then not all ti s are equal to zero, so let
j D maxfi W ti 0g. It follows that aj 2 hY 0 [ fa1 ; : : : ; aj 1 gi; this contradiction
with our choice of A0 shows that Y 0 [ A0 is a linearly independent set. Choose a
finite set B  Y 0 such that A  hB [ A0 i. Observe that
(1) there exists 2 .0; "/ such that, for any ' 2 .Lp .X // , if '.B [ A0 /  .; /
then '.A/  ."; "/.
0
0
Indeed, for each
a number P
y 2 R for every y 2 B [A
P x 2 AnA we can choose
0
0
such that
P x D f y y W y 2 B [ A g; let Kx D fj y j W y 2 B [ A g. If
K D fKx W x 2 AnA0 g C 1 then D K" is easily seen to be as promised.
The set V D ff 2 Cp .Y 0 / W f .B/  .; /g is open in Cp .Y 0 /; fix any f 2 V .
If f .Y / D f0g then .0X / D f , so we can assume that r D f .y/ 0 for some
y 2 Y 0.
There exists a continuous linear functional  W L ! R such that jY 0 D f . The
set Q D  1 .0/ is closed in L and hence in Lp .X /. Since Y [ A0 is independent,
the set G D hQ [ A0 i does not contain the point y. Besides, G D Q C hA0 i, so G
is closed in Lp .X / by Fact 2 of V.250. Apply Problem 222 to see that there exists a
continuous linear functional ' W Lp .X / ! R such that '.y/ D r and '.G/ D f0g.
It follows from Fact 14 of V.100 that the functional 'jL is proportional to , so
'.y/ D .y/ shows that 'jL D  and hence 'jY D f . Recalling that '.A0 /  f0g
and '.B/ D f .B/  .; / we convince ourselves that '.B [ A0 /  .; /
and hence (1) implies that '.A/  ."; "/. If g D 'jX then f D .g/ and
g 2 W  U ; since f 2 V was chosen arbitrarily, this proves that .U /  V , i.e.,
the image under  of every neighborhood of 0X contains a neighborhood of the zero
function of Cp .Y 0 /. By Fact 3 of S.496, the map  is an open surjection of Cp .X /
onto Cp .Y 0 /. Therefore there exists an open continuous linear surjection of Cp .X /
onto Cp .Y / and hence we settled sufficiency in (ii).
Suppose that  W Cp .X / ! Cp .Y / is a linear surjective map and let   .f / D
f  for any f 2 .Cp .Y // D Lp .Y / (see TFS-197). Then   W Lp .Y / !
.Cp .X // D Lp .X /; if L D   .Lp .Y // then L is a linear subspace of Lp .X /
and   W Lp .Y / ! L is a linear homeomorphism (see Fact 4 of S.489). This
proves necessity in (i). If  is an open map then   .Cp .Cp .Y /// is closed in
Cp .Cp .X // (see TFS-163); since Lp .Y / is closed in Cp .Cp .Y //, the set L is closed
in Cp .Cp .X // and hence in Lp .X /. This settles necessity in (ii).
Now, if  is a linear condensation (homeomorphism) then L is dense in Lp .X /
(or coincides with Lp .X / respectively). To see this take any functional w 2 Lp .X /
and a finite set A  Cp .X /; denote by M the linear hull of A. It is evident that

278

2 Solutions of problems 001500

w0 D w  1 is a (continuous) linear functional on Cp .Y / (and   .w0 / D w); in the


case when  is a linear homeomorphism we conclude that   .Lp .Y // D Lp .X /,
so   is a linear homeomorphism between Lp .Y / and Lp .X /, i.e., the proof of
necessity in (iv) ends here.
Since M 0 D .M / is a finite-dimensional linear subspace of Cp .Y /, the
functional w0 jM 0 is continuous on M 0 (see Problem 230). Apply Problem 224 to
find a continuous linear functional v 0 W Cp .Y / ! R such that v 0 jM 0 D w0 . Then
v D v 0  2 L; given any x 2 M we have v.x/ D v 0 ..x//. It follows from
.x/ 2 M 0 that v.x/ D v 0 ..x// D w0 ..x// D .w  1 /..x// D w.x/; an
immediate consequence is that vjM D wjM and hence vjA D wjA. Thus, for any
w 2 Lp .X / and any finite A  Cp .X / there exists v 2 L with vjA D wjA; this
implies that L is dense in Lp .X /, so we established necessity in (iii) and completed
our solution.
V.238. Given spaces X and Y prove that
(i) there is a linear continuous map of Cp .X / onto Cp .Y / if and only if Y is
homeomorphic to a subspace Y 0  Lp .X / such that every f 2 C.Y 0 / extends
to a continuous linear functional on Lp .X /;
(ii) the space Cp .X / linearly condenses onto Cp .Y / if and only if Y is homeomorphic to a subspace Y 0  Lp .X / such that every f 2 C.Y 0 / extends to a
uniquely determined continuous linear functional on Lp .X /;
(iii) Cp .X / is linearly homeomorphic to Cp .Y / if and only if Y is homeomorphic
to some Y 0  Lp .X / whose linear hull coincides with Lp .X / and every f 2
C.Y 0 / extends to a continuous linear functional on Lp .X /.
Solution. We will simultaneously prove sufficiency in (i)(iii). Assume that we
have a homeomorphism ' W Y ! Y 0  Lp .X / such that every f 2 C.Y 0 / extends
to a continuous linear functional w.f / on the space Lp .X / where the functional
w.f / is unique if we are dealing with (ii) and the linear hull of Y 0 is Lp .X / if we are
proving sufficiency in (iii). For every f 2 Cp .X / there exists a unique continuous
linear functional u.f / on Lp .X / such that u.f /jX D f (see Problem 234);
besides, the map u W Cp .X / ! .Lp .X // is a linear homeomorphism (see
Problem 235). If  W .Lp .X // ! Cp .Y 0 / is the restriction map then it is
continuous, linear and surjective; besides,  is injective in case when we deal with
(ii). Therefore  D  u is a continuous linear map of Cp .X / onto Cp .Y 0 / which
is injective if we are dealing with (ii); the space Y being homeomorphic to Y 0 there
exists a linear continuous (injective) map of Cp .X / onto Cp .Y /, so the proof of
sufficiency in (i) and (ii) ends here.
For the remaining part of the proof of sufficiency we are considering that the
linear hull of Y 0 coincides with Lp .X /. Our next step is to show that
(1) the set Y 0 is linearly independent in Lp .X /.
0
that
PnAssume that a1 ; : : : ; an 2 R and y1 ; : : : ; yn are distinct points of Y such
0
a
y
D
0.
For
every
i

n
there
exists
a
continuous
function
f
W
Y
!
R
i
i
i
iD1
such that fi .yi / D 1 and fi .yj / D 0 for all j i . If i W Lp .X / ! R is the
continuous linear functional which extends fi then

2 Solutions of problems 001500

0 D .0/ D i .

279
n
X
kD1

ak yk / D

n
X

ak i .yk / D ai :

kD1

Thus ai D 0 for each i  n and hence we proved (1).


The property (1) shows that the set Y 0 is a Hamel basis of Lp .X /, so we can
apply Problem 234 to see that Lp .Y 0 / is linearly homeomorphic to Lp .X / and
hence Lp .Y / is linearly homeomorphic to Lp .X / as well. By Problem 237, the
space Cp .X / is linearly homeomorphic to Cp .Y / and hence we completed the proof
of sufficiency in (i)(iii).
Suppose that  W Cp .X / ! Cp .Y / is a linear surjective map and let   .f / D
f  for any f 2 .Cp .Y // D Lp .Y / (see TFS-197). Then   W Lp .Y / !
.Cp .X // D Lp .X /; if L D   .Lp .Y // then L is a linear subspace of Lp .X / and
  W Lp .Y / ! L is a linear homeomorphism (see Fact 4 of S.489). If Y 0 D   .Y /
then, for every f 2 C.Y 0 / there is a continuous linear functional u W L ! R such
that ujY 0 D f (see Problem 234). Apply Problem 224 to find a continuous linear
functional w W Lp .X / ! R with wjL D u. Then wjY 0 D ujY 0 D f , so we proved
necessity in (i).
Now, if  is a linear condensation (or linear homeomorphism respectively) then
there exists a linear homeomorphism ' W Lp .Y / ! L  Lp .X / such that L is
dense in Lp .X / (or L D Lp .X / respectively) by Problem 237. If Y 0 D '.Y /
then, for every f 2 C.Y 0 / there is a continuous linear functional u W L ! R
such that ujY 0 D f . Apply Problem 224 to find a continuous linear functional
w W Lp .X / ! R with wjL D u. Then wjY 0 D ujY 0 D f ; if w0 2 .Lp .X //
and w0 jY 0 D f then w0 jL D u because the linear extension of f from Y 0 to L is
unique (see Problem 235). The set L being dense in Lp .X / it follows from w0 jL D
u D wjL that w D w0 and hence the linear extension of f over Lp .X / is unique,
i.e., we established necessity in (ii). If we deal with (iii) then Y 0 is a Hamel basis
in Lp .X / because Y is a Hamel basis in Lp .Y / and Hamel bases are preserved by
linear homeomorphisms. This proves necessity in (iii) and shows that our solution
is complete.
V.239. Let P be a class of spaces which have the following properties:
(1) if Y 2 PSand Z is a continuous image of Y then Z 2 P;
(2) if Y D fYi W i 2 !g; Yi  YiC1 ; Yi 2 P and Yi closed in Y for every i 2 !,
then Y 2 P;
(3) if Y 2 P and n 2 N then Y n  Rn 2 P;
Prove that if a space X belongs to P then Lp .X / 2 P.
n
n
Solution. For
P any n 2 N consider the map 'n W X  R ! Lp .X / defined by
'n .x; r/ D niD1 ri xi for any x D .x1 ; : : : ; xn / 2 X n and r D .r1 ; : : : ; rn / 2 Rn .
It follows from continuity of operations in Lp .X / that every 'n is continuous; if
Yn D '.X n  Rn / then Yn  YnC1 for every n 2 N. Since X is a Hamel basis of
Lp .X /, we have the equality
S
. / Lp .X / D fYn W n 2 Ng.

280

2 Solutions of problems 001500

It follows from Fact 1 of U.485 that every Yn is closed in Lp .X /. If X 2 P then


X n Rn 2 P by (3); applying (1) we convince ourselves that Yn 2 P for any n 2 N.
Finally, apply (2) and . / to conclude that Lp .X / 2 P.
V.240. Prove that i w.Lp .X // D .Lp .X // D .Lp .X // D i w.X / for any space
X ; show that also nw.X / D nw.Lp .X // and d.X / D d.Lp .X //.
Solution. It follows from X  Lp .X / that nw.X /  nw.Lp .X //. To prove the
opposite inequality observe that Lp .X /  Cp .Cp .X //, so we can apply TFS-172
to conclude that nw.Lp .X //  nw.Cp .Cp .X /// D nw.Cp .X // D nw.X / and
hence nw.X / D nw.Lp .X //.
Given an infinite cardinal  observe that the class P of spaces of density
  satisfies the conditions (1)(3) of Problem 239, so d.X /   implies
d.Lp .X //  . This proves that d.Lp .X //  d.X /.
Now assume that d.Lp .X //  . Then i w.Cp .Lp .X //   (see TFS-173);
the space Cp .X / embeds in Cp .Lp .X // (see Problem 235), so i w.Cp .X //  
and hence we can apply TFS-173 again to convince ourselves that d.X / D
i w.Cp .X //  . This shows that d.X /  d.Lp .X // and therefore d.X / D
d.Lp .X //.
To check the rest of our equalities suppose that .Lp .X //  . For any finite
A  Cp .X / and " > 0 let A; "
D f 2 Lp .X / W j.f /j < " for any f 2 Ag. The
family B D fA; "
W A is a finite subset of Cp .X / and " > 0g is a local base of the
space Lp .X / at 0, so we can find a family A D fA ; "
W < g  B such that
T
A D f0g.
S
The set A D fA W < g has cardinality at most , so its linear hull H has
density at most . Assume for a moment that H is not dense in Cp .X / and fix a
function f 2 Cp .X /nH . By Problem 222 and TFS-197, there exists
T  2 Lp .X /
such that .f / D 1 and .H / D f0g. Then  0 and  2
A which is a
contradiction. Therefore H is dense in Cp .X / and hence i w.X / D d.Cp .X // 
d.H /   which shows that i w.X /  .Lp .X //.
It follows from Lp .X /  Cp .Cp .X // that i w.Lp .X //  i w.Cp .Cp .X ///.
Furthermore, i w.Cp .Cp .X /// D d.Cp .X // D i w.X /, so i w.Lp .X //  i w.X /.
The inequalities i w.X /  .Lp .X //  .Lp .X //  i w.Lp .X //  i w.X /
imply that .Lp .X // D .Lp .X // D i w.Lp .X // D i w.X /, i.e., our solution
is complete.
V.241. Prove that, for any space X , we have the following equalities.
(i) s  .X / D s.Lp .X // D s  .Lp .X //;
(ii) hl  .X / D hl.Lp .X // D hl  .Lp .X //;
(iii) hd  .X / D hd.Lp .X // D hd  .Lp .X //.
Solution. For any n 2 N the space X n embeds in Lp .X / by TFS-337; this shows
that s  .X /  s.Lp .X //, hl  .X /  hl.Lp .X // and hd  .X /  hd.Lp .X //.
It follows from SFFS-025 that s  .X / D s  .Cp .X // D s  .Cp .Cp .X ///; since
Lp .X / is a subspace of Cp .Cp .X //, we have s  .Lp .X //  s  .Cp .Cp .X /// D
s  .X /. This gives us inequalities s  .X /  s.Lp .X //  s  .Lp .X //  s  .X /
which show that (i) is proved.

2 Solutions of problems 001500

281

Apply SFFS-026 and SFFS-027 to see that hl  .X / D hd  .Cp .X // D


hl .Cp .Cp .X ///; since Lp .X /  Cp .Cp .X //, we have hl  .Lp .X // 
hl  .Cp .Cp .X /// D hl  .X /. Therefore hl  .X /  hl.Lp .X //  hl  .Lp .X // 
hl  .X /, so we settled (ii).
Since SFFS-026 and SFFS-027 also imply hd  .X / D hl  .Cp .X // D
hd  .Cp .Cp .X ///, we can recall once more that Lp .X / is a subspace of Cp .Cp .X //
to conclude that hd  .Lp .X //  hd  .Cp .Cp .X /// D hd  .X /. An immediate
consequence is that hd  .X /  hd.Lp .X //  hd  .Lp .X //  hd  .X /, i.e., (iii)
is proved.


V.242. Given a space X prove that we have l  .X / D l.Lp .X // D l  .Lp .X // and


ext  .X / D ext .Lp .X // D ext  .Lp .X //.
Solution. For any n 2 N the space X n embeds in Lp .X / as a closed subspace (see
TFS-337); this shows that l  .X /  l.Lp .X // and ext  .X /  ext .Lp .X //.
Fact 1. If Z and T are spaces and there exists a perfect map f W Z ! T then
(1)
(2)
(3)
(4)

ext .Z/  ext .T / and ext  .Z/  ext  .T /;


ext .ZS /  ext .Z/ and ext  .ZS /  ext  .Z/ for any  -compact space S .
l.Z/  l.T / and l  .Z/  l  .T /;
l.Z  S /  l.Z/ and l  .Z  S /  l  .Z/ for any  -compact space S .

Proof. Fix an infinite cardinal  and assume that ext .T /  . If there exists a
closed discrete subspace D  Z with jDj D C then let E D f .D/. The map f
is closed, so E is a closed subspace of T . By the same reason f .G/ is closed in T
for every G  D and hence E is a discrete subspace of T . For any y 2 E the set
f 1 .y/ is compact, so S
the set Fy D fx 2 D W f .x/ D yg is finite for any y 2 E.
If jEj   then D D fFy W y 2 Eg is the union of at most -many finite sets,
i.e., jDj   which is a contradiction. Therefore jEj D C ; this contradiction with
ext .T /   shows that ext .Z/   and hence ext .Z/  ext .T /. The inequality
l.Z/  l.T / was proved in Fact 2 of T.490. It is easy to see that, for each n 2 N,
there exists a perfect map of Z n onto T n ; therefore ext .Z n /  ext .T n /  ext  .T /
and l.Z n /  l.T n /  l  .T / for every n 2 N which shows that ext  .Z/  ext  .T /
and l  .Z/  l  .T /, so we settled (1) and (3).
If K is compact then the projection map Z  K ! Z is perfect by Fact 3 of
S.288, so extS
.Z  K/  ext .Z/ and l.Z  K/  l.Z/ by (1) and (3) respectively.
Now, if S D n2! Kn where Kn is compact
and Kn  KnC1 for every n 2 ! then
S
it follows from the equality Z  S D n2! .Z  Kn / that ext .Z  S /  ext .Z/
and l.Z  S /  l.Z/, i.e., we established the first parts of (2) and (4). Apply these
inequalities to see that ext ..Z  S /n / D ext .Z n  S n /  ext .Z n /  ext  .Z/
and analogously, l..Z  S /n / D l.Z n  S n /  l.Z n /  l  .Z/ for all n 2 N; this
shows that ext  .Z  S /  ext  .Z/ and l  .Z  S /  l  .Z/, so Fact 1 is proved.
Returning to our solution fix an infinite cardinal  and consider the class P (the
class Q) of spaces Y such that l  .Y /   (or ext  .Y /   respectively). It is an
easy exercise that any continuous image of a space from P (or from Q) belongs to
P (or to Q respectively).

282

2 Solutions of problems 001500

S
Next, assume that Y D i2! Yi where every Yi is closed in Y , belongs to P
(or to Q) and the sequence fYi W i 2 !g is increasing, i.e., Yi  YiC1 for all
i 2 !. Given n 2 N the space Y n can be represented as the countable union of
its closed subspaces F .i1 ; : : : ; in / D Yi1  : : :  Yin where i1 ; : : : ; in 2 !. If j D
maxfi1 ; : : : ; in g then the set F D F .i1 ; : : : ; in / is a closed subspace of the space
Yjn , so it follows from l.Yjn /   (or ext .Yjn /   respectively) that l.F /   (or
ext .F /  ). Therefore l.Y n /   (or ext .Y n /  ) for every n 2 N, i.e., Y 2 P
(or Y 2 Q respectively).
It follows from Fact 1 that we have the equalities ext  .X n Rn / D ext  .X / and

l .X n Rn / D l  .X / which shows that X n Rn 2 P if X 2 P and X n Rn 2 Q if
X 2 Q. Therefore the classes P and Q satisfy the conditions (1)(3) of Problem 239
and hence X 2 P implies Lp .X / 2 P while X 2 Q implies Lp .X / 2 Q. This gives
us the inequalities l  .X /  l.Lp .X //  l  .Lp .X //  l  .X /; analogously, we
conclude that ext  .X /  ext .Lp .X //  ext  .Lp .X //  ext  .X /. This implies
the equalities l  .X / D l.Lp .X // D l  .Lp .X // and ext  .X / D ext .Lp .X // D
ext  .Lp .X //, i.e., our solution is complete.
V.243. Prove that an uncountable regular cardinal  is a caliber of X if and only
if  is a caliber of Lp .X /.
Solution. Consider the class P of spaces Y such that  is a caliber of Y . By
SFFS-276, SFFS-277 and SFFS-281 the class P satisfies the conditions (1)(3)
of Problem 239, so it follows from X 2 P that Lp .X / 2 P, i.e., we established
necessity.
Now, if  is a caliber of Lp .X / then the diagonal of Cp .Lp .X // is -small
(see SFFS-290). The space Cp .X / embeds in Cp .Lp .X // by Problem 235, so the
diagonal of Cp .X / is also -small. Applying SFFS-290 again we conclude that  is
a caliber of X ; this proves sufficiency.
V.244. Let L be the following collection of classes of Tychonoff spaces: fanalytic
spaces, K-analytic spaces,  -compact spaces, realcompact spaces, Lindelf spacesg. Prove that, for any class P from the list L, a space X belongs to P if
and only if Lp .X / belongs to P.
Solution. It is an easy exercise to verify that if P is one of the first four classes then
P satisfies the conditions (1)(3) of Problem 239, so if X 2 P then Lp .X / 2 P.
Besides, if P is the class of realcompact spaces then X 2 P implies Cp .Cp .X // 2
P by TFS-435; the set Lp .X / being closed in Cp .Cp .X // (see TFS-078) we also
have Lp .X / 2 P.
Now, if Lp .X / 2 P (where P is any class from the list L) then X 2 P because X
is a closed subspace of Lp .X /. Thus X belongs to P if and only if so does Lp .X /.
V.245. Given w D 1 x1 C : : : C n xn 2 Lp .X /, where x1 ; : : : ; xn 2 X and
1 ; : : : ; n 2 Rnf0g, let supp.w/ D fx1 ; : : : ; xn g. If w D 0, then supp.w/ D ;. Say
that a set B  Lp .X / is weakly bounded if .B/ is a bounded subset of R for any
continuous linear functional  W Lp .X / ! R. Observe that any bounded subset of
Lp .X /S
is weakly bounded and prove that, for any weakly bounded set B  Lp .X /,
the set fsupp.w/ W w 2 Bg is bounded in the space X .

2 Solutions of problems 001500

283

Solution. It is immediate from the definitions that any bounded subset of Lp .X / is


weakly bounded
in Lp .X /, so assume that B  Lp .X / is weakly bounded while
S
S.B/ D fsupp.w/ W w 2 Bg is not bounded in X .
Fact 1. Given a space Z, a set Y  Z is not bounded in Z if and only if there exists
a discrete family fUn W n 2 !g  .Z/ such that Un \ Y ; for any n 2 !.
Proof. If a family fUn W n 2 !g  .Z/ is discrete and every Un meets Y , pick
a point yn 2 Un \ Y for each n 2 !. Apply Fact 1 of T.217 to find a continuous
function f W Z ! R such that f .yn / D n for every n 2 !. Then f .Y / is not a
bounded subset of R, so Y is not bounded in Z and hence we proved sufficiency.
Now assume that Y is not bounded in Z and hence there exists a continuous
function f W Z ! R such that f .Y / is not bounded in R. Let g.x/ D jf .x/j
for any x 2 Z; the function g W Z ! R is continuous and it is clear that g.Y /
is not bounded in R. It is easy to choose
P by induction on n 2 ! a set D D fyn W
n 2 !g  Y such that g.yn / > n C n1
iD0 g.yi / for every n 2 !. The sequence
fg.yn / W n 2 !g  R is strictly increasing and g.yn / > n for each n 2 !.
Consequently, the set D is closed and discrete in R, so we can find a discrete family
fOn W n 2 !g  .R/ such that g.yn / 2 On for every n 2 !. Letting Un D g 1 .On /
for all n we obtain a discrete family fUn W n 2 !g  .Z/ with yn 2 Un \ Y for
every n 2 !; this settles necessity and shows that Fact 1 is proved.
Returning to our solution apply Fact 1 to find a countably infinite discrete family
U  .X / such that U \ S.B/ ; for every U 2 U . Proceeding by an evident
induction we can choose a family fUn0 W n 2 !g  U and a sequence fwn W n 2
!g  B such that Un0 \ supp.wn / ; and Un0 \ supp.wk / D ; for any k < n. For
every n 2 ! take a point xn 2 Un0 \ supp.wn / and an open set Un  Un0 such that
Un \ supp.wn / D fxn g. P
n
We have wn D n xn C kiD1
ni yin where f n ; n1 ; : : : ; nkn g  R; n 0 and
n
n
xn Pn D fy1 ; : : : ; ykn g  X ; let n D maxfjni j W i  kn g for each n 2 !.
Start off with r0 D j 0 j1 and construct inductively a sequence R D frn W n 2 !g
of positive numbers such that rn > rn1 C j 1n j .n C n kn rn1 / for every n 2 N; it
is evident that R is strictly increasing. Take a continuous function fn W X ! 0; 1

such that fn .xP


n / D 1 and fn .X nUn / D f0g for each n 2 !. By Fact 1 of T.217 the
function f D frn fn W n 2 !g is continuous, so we can apply Problem 234 to find
a continuous linear functional  W Lp .X / ! R such that
SjX D f .
Fix any n 2 !; since the set Pn does not meet Hn D fUi W i  ng, we conclude
that jf .yin /j D f .yin /  rn1 for every i  kn . An immediate consequence
Pn
is that kiD1
jni jjf .yin /j  n kn rn1 which implies, together with the inequality
Pn n
j n f .xn /j > n C n kn rn1 , that j.wn /j  j n f .xn /j  j kiD1
i f .yin /j  n.
Therefore the continuous linear functional  W Lp .X / ! R is not bounded on the
set B; this contradiction shows that S.B/ is bounded in X and makes our solution
complete.
V.246. Prove that, for any Dieudonn complete space X , if A is a bounded subset
of Lp .X / then A is compact.

284

2 Solutions of problems 001500

Solution. Our first step is to prove that closures of bounded sets are compact in
Dieudonn complete spaces.
Fact 1. If Y is a Dieudonn complete space and B is a bounded subset of Y then B
is compact.
Proof. Observe first that F D B is also bounded in Y byQFact 2 of S.398. We can
consider that Y is a closed subspace of a product M D fMt W t 2 T g in which
every Mt is a metrizable space; let t W M ! Mt be the natural projection for every
t 2 T . The set Ft D t .F / is bounded in Mt (see Fact 1 of S.399); since Pt D F t
is also bounded and C -embedded in Mt , the set Pt is pseudocompact and hence
compact (here we used again
Q Fact 2 of S.398). The set F is closed in Y and hence
in M ; since F  P D fPt W t 2 T g, the set F has to be closed in the compact
space P . This shows that F is compact, so Fact 1 is proved.
Fact 2. For any space Y , if Z is a closed subspace of Y then the linear hull H of
the set Z in Lp .Y / is closed in Lp .Y /.
Proof. Suppose that a point w D 1 y1 C : : : C n yn belongs to Lp .Y /nH and
yi yj whenever i j . Then k 0 and yk Z for some k  n. By the
Tychonoff property of Y we can find a continuous function f W Y ! 0; 1
such
that f .yk / D 1; f .yi / D 0 for all i k and f .Z/  f0g. The set U D fv 2
Lp .Y / W v.f / 0g is an open subset of Lp .Y / and w 2 U . It is immediate that
u.f / D 0 for any u 2 H , so U \ H D ;. Thus every point w 2 Lp .Y /nH has an
open neighborhood which does not meet H . This shows that H is closed in Lp .X /,
so Fact 2 is proved.
Fact 3. Given a space Y , if Z  Y is C -embedded in Y then, for the linear hull H
of the set Z in Lp .Y /, there exists a linear homeomorphism ' W Lp .Z/ ! H such
that '.z/ D z for any z 2 Z.
Proof. Given any continuous function f W Z ! R there exists g 2 C.Y / such that
gjZ D f . By Problem 234, there is a continuous linear functional  W Lp .Y / ! R
for which jY D g. Then 0 D jH W H ! R is a continuous linear functional
such that 0 jZ D jZ D gjZ D f , so we can apply Problem 234 again to conclude
that there exists a linear homeomorphism ' W Lp .Z/ ! H such that '.z/ D z for
any z 2 Z, i.e., Fact 3 is proved.
Returning to our solution recall that X is C -embedded in the Hewitt realcompactification X of the space X , so we can apply Fact 3 to identify Lp .X / with the
linear hull of the set X in Lp .X /. Assume that A is a bounded subset of Lp .X /.
For any w D 1 x1 C : : : C n xn 2 Lp .X / where i 2 Rnf0g and xi 2 X for all
i  n, let supp.w/ D fx1 ; : : : ; xn g; if w D 0 then supp.w/ D ;. The set A is still
bounded in Lp .X / by Fact 2 of S.398 (the bar denotes the closure in Lp .X /), so we
S
can apply Problem 245 to see that B D fsupp.w/ W w 2 Ag is a bounded subset
of X , so K D clX .B/ is compact by Fact 1.
Let G be the linear hull of the set K in the space Lp .X /; it is evident that
A  G  Lp .X /. By Fact 2, the set G is closed in Lp .X /; besides, A is closed

2 Solutions of problems 001500

285

in Lp .X / and hence in G, so A is closed in Lp .X /. The set A is also bounded in


Lp .X / because Lp .X /  Lp .X /; since Lp .X / is realcompact by Problem 244,
the set A is compact by Fact 1, so our solution is complete.
V.247. Suppose that a space X has a weaker metrizable topology and A is a
bounded subset of Lp .X /. Prove that A is compact and metrizable.
Solution. The space X is Dieudonn complete by TFS-461, so we can apply
Problem 246 to see that K D A is compact. Fix a condensation f W X ! M of X
onto a metrizable space M . Given a point w D 1 x1 C : : : C n xn 2S
Lp .X / with
i 2 Rnf0g for all i  n let supp.w/ D fx1 ; : : : ; xn g. The set B D fsupp.w/ W
w 2 Kg is bounded in X by Problem 245, so P D B is compact by Fact 1 of T.246.
The map f jP condenses P onto a metrizable space f .P /, so P is metrizable and
second countable.
The set P is C -embedded in X (see Fact 1 of T.218), so we can apply Fact 3 of
V.246 to convince ourselves that the linear hull H of the set P in Lp .X / is linearly
homeomorphic to Lp .P / and hence nw.H / D nw.P /  ! (see Problem 240). It
follows from K  H that w.K/ D nw.K/  ! (see Fact 4 of S.307), so K is
metrizable.
V.248. Prove that, for any infinite pseudocompact space X , there exists an infinite
closed discrete set D in the space Lp .X / which is weakly bounded in Lp .X /.
Therefore, even for a metrizable compact space X , the closure of a weakly bounded
subset of Lp .X / can fail to be compact.
Solution. The space X being infinite we can apply Fact 4 of S.382 to find a discrete
subspace A D fdn W n 2 !g  X such that dn dm for distinct m; n 2 !; it is
easy to construct a disjoint family fUn W nP
2 !g of open subsets of X such that
Un \ A D fdn g for each n 2 !. Let wn D niD0 2i di for every n 2 !; we claim
that the set D D fwn W n 2 !g  Lp .X / is as promised.
Take an arbitrary point u D 1 x1 C : : : C n xn 2 Lp .X / and choose k 2 !
such that Uk \ fx1 ; : : : ; xn g D ;. There exists a continuous function f W X ! R
with f .dk / D 2k and f .X nUk /  f0g. The set W D fw 2 Lp .X / W w.f / < 1g
is open in Lp .X / and u 2 W because u.f / D 0. Furthermore, wn .f / D 1 for any
n  k, so W \ D  fd0 ; : : : ; dk1 g. We proved that every point of Lp .X / has a
neighborhood whose intersection with D is finite, so D is a closed discrete subspace
of Lp .X /. The set X being linearly independent, we have wn wm for any distinct
m; n 2 !, so D is an infinite subset of Lp .X /.
Finally, take a continuous linear functional  W Lp .X / ! R. The space X is
pseudocompact, so the function h D jX is bounded
K > 0 such
P on X ; fix a numberP
that jh.x/j  K for all x 2 X . Then j.wn /j  niD0 2i jh.di /j  K niD0 2i 
2K for every n 2 !; this shows that .D/  2K; 2K
is a bounded subset of R
and hence D is weakly bounded.
V.249. Give an example of a space X in which all compact subspaces are
metrizable while there are non-metrizable compact subspaces in Lp .X /.

286

2 Solutions of problems 001500

Solution. Denote by X the space !1 with its order topology and consider the set
O D f 2 X W < g for every < !1 . If K is a compact subspace of X then
fO W < S
!1 g is an open cover of K, so there is a finite set A  !1 such that
K  O D fO W 2 Ag. Every set O is countable, so O is countable as well;
thus the set K is also countable and hence metrizable.
To avoid confusion with the addition of ordinals, we denote the addition in
Lp .X / by the symbol ; let w D .1/. C 1/ for every < !1 and consider
the set F D f0g [ fw W < !1 g  Lp .X /. Take any open set U in the space
Lp .X / with 0 2 U . We can find " > 0 and a finite set B  Cp .X / such that the
set V D fu 2 Lp .X / W ju.f /j < " for every f 2 Bg is contained in U . Given
any f 2 B we have w .f / D f ./  f . C 1/ for each < !1 , so there is a
finite set Pf S!1 such that jw .f /j < " for any Pf (see Fact 1 of S.334).
The set P D fPf W f 2 Bg is finite and jw .f /j < " for any f 2 B and
P . Therefore w 2 V  U for any P and hence any neighborhood of 0
in F contains all but finitely many points of F . An easy consequence is that F is
homeomorphic to A.!1 /, i.e., F is a non-metrizable compact subspace of Lp .X /.
V.250. Given spaces X and Y and a continuous map ' W X ! Y observe that there
is a unique continuous linear map u' W Lp .X / ! Lp .Y / such that u' jX D '.
Prove that the following conditions are equivalent for any continuous onto map
' W X ! Y.
(i)
(ii)
(iii)
(iv)

The map ' is R-quotient.


The map u' is R-quotient.
The map u' quotient.
The map u' is open.

Solution. By Problem 233 the topology of Lp .Y / coincides with its weak topology,
so we can apply Problem 234 to see that there is a continuous linear map u' W
Lp .X / ! Lp .Y / such that u' jX D '. Suppose that u W Lp .X / ! Lp .Y /
is a linear map with ujX D '. Given a point z 2 P
Lp .X / we can find n 2
N; 1 ; : : : ; n 2 R and x1 ; : :P
: ; xn 2 X such that z D niD1 i xP
i . By linearity of
the map u' we have u' .z/ D niD1 i '.xi /; analogously, u.z/ D niD1 i '.xi / D
u' .z/, so u D u' , i.e., our linear extension of the map ' is unique.
It is evident that (iv)H)(iii)H) (ii). Assume that u' W Lp .X / ! Lp .Y / is an
R-quotient map for some continuous onto map ' W X ! Y and take a function
f W Y ! R such that f ' is continuous. By Problem 234 there exists a continuous
linear functional  W Lp .X / ! R such that jX D f '. There exists a linear
functional  W Lp .Y / ! R such that jY D f . It is evident that  0 D  u' is
a linear functional on Lp .X /. If x 2 X then  0 .x/ D .u' .x// D .'.x//; since
'.x/ 2 Y , we have  0 .x/ D .'.x// D f .'.x// D .x/.
We proved that jX D  0 jX , so  D  0 by Fact 3 of S.489. Therefore the
functional  0 D  u' is continuous so  is continuous because u' is R-quotient.
Consequently, the map f D jY is continuous and hence ' is R-quotient, i.e., we
checked that (ii)H)(i).

2 Solutions of problems 001500

287

Fact 1. Suppose that L is a linear topological space and K is a compact subset of L.


Then the set K C A is closed in L for any closed A  L.
Proof. To prove that the set F D K C A is closed take any point x 2 LnF . For
any y 2 K the set y C A is closed in L and does not contain x, so we can find
a balanced open neighborhood Uy of 0 such that .x C Uy / \ .y C A/ D ;. Fix a
balanced open neighborhood Vy of the point 0 such that Vy C Vy  Uy . The family
fy C S
Vy W y 2 Kg is an open cover ofT
K, so there is a finite set P  K such that
K  fy C Vy W y 2 P g. Then V D fVy W y 2 P g is an open neighborhood of
0, so O D x C V 2 .x; L/.
If O \ F ; then we can pick z 2 K and a 2 A with z C a 2 x C V and
hence v D z C a  x 2 V . There is y 2 P for which z 2 y C Vy and therefore
v0 D z  y 2 Vy . It follows from z D v0 C y D v C x  a that a C y D x C v  v0 2
x C V C Vy ; since V  Vy , we conclude that a C y 2 x C Vy C Vy  x C Uy , i.e.,
.x C Uy / \ .y C A/ ; which is a contradiction. Therefore every x 2 LnF has a
neighborhood which does not meet F , so LnF is open and hence F is closed in L,
i.e., Fact 1 is proved.
Fact 2. Given a locally convex space L suppose that M is a closed linear subspace
of L and G is a finite-dimensional linear subspace of L. Then M C G is a closed
subspace of L.
Proof. Fix a Hamel basis H in the subspace M and apply Fact 1 of S.489 to find
vectors fe1 ; : : : ; en g  M C G such that H [ fe1 ; : : : ; en g is a Hamel basis in
M C G. Letting M0 D M it takes an easy induction to construct linear subspaces
M0  M1  : : :  Mn of the space L in such a way that MiC1 is a linear hull of
Mi [ eiC1 for all i 2 f0; : : : ; n  1g. It is clear that Mn D M C G, so it suffices
to show inductively that every Mi is closed in L. By our assumption the set M0 is
closed in L, so assume that 0  i < n and we have proved that Mi is closed in L.
Fix a point x MiC1 ; we claim that there exists a continuous linear functional
f W L ! R such that f .x/ D 1 and f .Mi [ feiC1 g/ D f0g. To prove this,
apply Problem 222 to find a continuous linear functional g W L ! R for which
g.x/ D 1 and g.Mi / D f0g. If r D g.eiC1 / D 0 then let f D g. If r 0 then
z D rxeiC1 Mi for otherwise x D 1r .zCeiC1 / 2 MiC1 which is a contradiction.
Thus we can apply Problem 222 once more to find a continuous linear functional
h W L ! R such that h.z/ D 1 and h.Mi / D

f0g.

1 h.x/
g.x/ h.x/

D h.eiC1  rx/ D 1

D
Since the determinant
r h.eiC1 /
g.eiC1 / h.eiC1 /
is not equal to zero, any system of linear equations with this determinant has a
solution, so we can find numbers ; 2 R such that g.x/ C h.x/ D 1 and
g.eiC1 / C h.eiC1 / D 0. The linear functional f D g C h is continuous on
L while f .x/ D 1 and f .Mi [ feiC1 g/ D 0. As a consequence, f .MiC1 / D f0g,
so x cannot belong to the closure of MiC1 . Since x MiC1 was chosen arbitrarily,
we proved that MiC1 is closed in L, so our inductive procedure shows that Mn D
G C M is closed in L, i.e., Fact 2 is proved.

288

2 Solutions of problems 001500

Returning to our solution assume that ' W X ! Y is an R-quotient map and


consider the dual map '  W Cp .Y / ! Cp .X / defined by '  .f / D f ' for
any f 2 Cp .Y /. It is straightforward that L D '  .Cp .Y // is a linear subspace of
Cp .X / and '  W Cp .Y / ! L is a linear homeomorphism; by TFS-163, the set L
is closed in Cp .X /. Apply Fact 1 of S.489 to choose a Hamel basis H in Cp .X / in
such a way that H 0 D H \ L is a Hamel basis in L.
We will first show that
(1) for any finite set A  H nH 0 and f 2 A there exists a continuous linear
functional  W Cp .X / ! R such that .f / D 1 and .L [ .Anff g// D f0g.
Let G be the linear hull of the set Anff g; since G is a finite-dimensional linear
subspace of Cp .X /, we can apply Fact 2 to see that L0 D G C L is a closed linear
subspace of Cp .X /. Since f L0 , we can apply Problem 222 to find a continuous
linear functional  W Cp .X / ! R such that .f / D 1 and .L0 / D f0g; then
.L [ .Anff g//  .L0 / D f0g, so (1) is proved. Our next step is to establish that
(2) given a finite set A  H nH 0 , for any u 2 RA there exists a continuous linear
functional  W Cp .X / ! R such that .L/ D f0g and jA D u.
Apply (1) to fix, for any f 2 A a continuous linear functional f W Cp .X / ! R
such that f .L [ .Anff
P g// D f0g and f .f / D 1. It is straightforward that the
linear functional  D f 2A u.f / f is continuous on Cp .X / while .L/ D f0g
and jA D u, so (2) is proved.
To finally prove that u' is an open map take an open neighborhood U of 0 in the
space Lp .X /. Apply Fact 2 of V.024 to find a finite set A  H and " > 0 such that
V D fz 2 Lp .X / W jz.f /j < " for all f 2 Ag  U . Let A0 D A \ L and choose
a finite set B  Cp .Y / with '  .B/ D A0 . The set W D fw 2 Lp .Y / W jw.g/j < "
for any g 2 Bg is an open neighborhood of 0 in Lp .Y /. We claim that u' .V /  W ;
to prove this, fix an arbitrary element w 2 W .
The map .'  /1 W L ! Cp .Y / being linear and continuous, w0 D w .'  /1
is a continuous linear functional on L. By Problem 224 there exists a continuous
linear functional w00 W Cp .X / ! R such that w00 jL D w0 . Apply Fact 2 to find a
continuous linear functional  W Cp .X / ! R such that j.AnA0 / D w00 j.AnA0 / and
.L/ D f0g. Then v D w00  is a continuous linear functional on the space Cp .X /
such that v.AnA0 /  f0g and vjL D w0 . Since Lp .X / coincides with .Cp .X // (see
TFS-197), the functional v belongs to Lp .X /.
If f 2 A0 then f D g ' for some g 2 B; therefore g D .'  /1 .f / and hence
jv.f /j D jw0 .f /j D jw.g//j < " by definition of the set W . If f 2 AnA0 then
v.f / D 0, so jv.f /j < " for all f 2 A which shows that v 2 V .
Given any g 2 Cp .Y / the function f D g ' belongs to L, so v.f / D w0 .f /;
it follows from g D .'  /1 .f / that
Pv.f / D w.g/. Choose n 2 N; 1 ; : : : ; n 2 R
and x1 ; : :P
: ; xn 2 X such that v D niD1 i xi ; let yi D '.xi / for every i  n. Then
u' .v/ D niD1 i yi and hence
u' .v/.g/ D

n
X
iD1

i g.yi / D

n
X
iD1

i f .xi / D v.f / D w.g/;

2 Solutions of problems 001500

289

which shows that u' .v/.g/ D w.g/ for every g 2 Cp .Y /, i.e., u' .v/ D w. We
proved that, for each w 2 W there exists v 2 V such that u' .v/ D w; as a
consequence, u' .U /  u' .V /  W . Thus, for any U 2 .0; Lp .X // there exists
W 2 .0; Lp .Y // such that W  u' .U /. Applying Fact 3 of S.496 we conclude
that the map u' is open; this proves that (i)H)(iv) and makes our solution complete.
V.251. Let f W X ! Y be an R-quotient map. Prove that, for any open U  Y ,
the map f j.f 1 .U // W f 1 .U / ! U is also R-quotient.
Solution. Let U0 D f 1 .U / and denote by f0 the map f restricted to U0 .
Fact 1. Given a space Z and a set W 2 .Z/ suppose that h W W ! R is a
bounded continuous function and g W Z ! R is a continuous function such that
g.ZnW /  f0g. Consider a function w D g h W Z ! R defined as follows:
w.z/ D g.z/h.z/ for all z 2 W and w.z/ D 0 whenever z 2 ZnW . Then the
function w is continuous.
Proof. By our hypothesis, there exists K > 0 such that jh.z/j  K for all z 2 W .
The function wjW D h .gjW / being continuous on W , it is an easy exercise that
w is continuous at every z 2 W . If z 2 ZnW and " > 0 then we can find a set
V 2 .z; Z/ for which g.V /  . K" ; K" /. If x 2 V nW then w.x/ D 0 2 ."; "/; if
x 2 W \ V then jw.x/j D jh.x/jjg.x/j < K K" D ". Therefore w.V /  ."; "/,
i.e., V witnesses continuity of w at the point z. Thus w is continuous at all points of
Z, so Fact 1 is proved.
Fact 2. A map g W Z ! T is R-quotient if and only if, for any function h W T ! I,
it follows from continuity of h g that h is continuous.
Proof. If g is R-quotient and h W T ! I is a function such that h g is continuous
then h is continuous by Fact 1 of T.268, i.e., we established necessity. To prove
sufficiency assume that our hypothesis about the functions from T to I is true
and take an arbitrary function u W T ! R such that u g is continuous. Fix a
homeomorphism  W R ! .1; 1/ and consider the function h D  u. It is evident
that h W T ! I and the function h g D  .u g/ is continuous. Therefore h is
continuous and hence u D  1 h is continuous as well. This shows that the map g
is R-quotient and settles sufficiency so Fact 2 is proved.
Returning to our solution take any function h W U ! I such that u D h f0 is
continuous. To see that h is continuous fix any point y0 2 U . Using the Tychonoff
property of the space Y and Fact 1 of S.499 it is easy to find a continuous function
v W Y ! 0; 1
such that v.Y nU /  f0g and v.W / D f1g for some open W  U
with y0 2 W . Then w D v f is a continuous function on X such that w.X nU0 / 
f0g. By Fact 1, the function q D w .h f0 / W X ! I is continuous. If p D v h
then p f D q, so the function p is continuous on Y by Fact 1 of T.268. Since
pjW D hjW , we proved that every point y0 2 U has a neighborhood W such
that hjW is continuous. Applying Fact 1 of S.472 we conclude that h is continuous.
Therefore f0 is R-quotient by Fact 2, so our solution is complete.

290

2 Solutions of problems 001500

V.252. Let X be a Tychonoff space. Prove that, for any nonempty closed set F  X ,
the R-quotient space XF is also Tychonoff and if pF W X ! XF is the contraction
map then pF j.X nF / W X nF ! XF nfF g is a homeomorphism.
Solution. If Y is a space and G ; is a closed subset of Y then a function f 2
C.Y / is called G-constant if there is c 2 R such that f .G/ D fcg. Let pG .x/ D x
for any x 2 Y nG and pG .x/ D G for each x 2 G; this gives the map pG W Y ! YG .
If f 2 C.Y / is a G-constant function with f .G/ D fcg then let uf .x/ D f .x/ for
any x 2 YG nfGg and uf .G/ D c; therefore we constructed a function uf W YG ! R
for any G-constant f 2 C.Y /.
Fact 1. Given a space Y and a closed nonempty set G  Y , for any g 2 Cp .YG /
let pG .g/ D g pG ; then pG W Cp .YG / ! Cp .Y / is a linear embedding such that
pG .Cp .YG // coincides with the set LG of all G-constant functions. Furthermore, a
function g W YG ! R is continuous if and only if there is a G-constant function
f 2 C.Y / such that g D uf .
Proof. It follows from TFS-163 that pG is an embedding; it is straightforward that it
is a linear map. If g 2 C.YG / then f D g pG is a continuous G-constant function
on X and it is immediate that g D uf ; this proves that pG .Cp .YG //  LG and, for
any g 2 C.YG / there is a G-constant function with g D uf .
If f 2 C.Y / is a G-constant function then, for the function g D uf we have
f D g pG , so fg 1 .O/ W O 2 .R/g  .YG / by definition of the topology of YG .
Therefore g is continuous on YG , i.e., uf is continuous on YG for any G-constant
f 2 C.Y / and it follows from f D uf pG D pG .uf / that LG  pG .Cp .YG //,
so Fact 1 is proved.
Returning to our solution recall first that XF is completely regular by Fact 1 of
T.139, so it suffices to show that it is Hausdorff. To distinguish the set F from the
point F 2 XF we will denote the point F in the space XF by aF .
Given distinct points a; b 2 XF we can assume that a aF and hence a F .
Consider first the case when b aF and choose f 2 C.X / such that f .a/ D
1 and f .fbg [ F / D f0g. It is clear that f is an F -constant function so g D
uf is continuous on XF by Fact 1. We have g.a/ D 1 and g.b/ D 0, so U D
g 1 ..1; 1=2// and V D g 1 ..1=2; C1// are disjoint open neighborhoods of b
and a respectively.
Now, if b D aF then there exists f 2 C.X / such that f .a/ D 1 and f .F / D
f0g. It is clear that f is F -constant so g D uf is continuous on XF . We have again
g.a/ D 1 and g.b/ D 0, so U D g 1 ..1; 1=2// and V D g 1 ..1=2; C1//
are disjoint open neighborhoods of b and a respectively. This shows that XF is
Tychonoff being a completely regular Hausdorff space.
Finally observe that the map pF is R-quotient and pF1 .XF nfaF g/ D X nF , so
we can apply Problem 251 to convince ourselves that pF j.X nF / is also R-quotient.
Since pF j.X nF / is a continuous bijection between X nF and XF nfaF g, we can
apply TFS-155 to see that pF j.X nF / is a homeomorphism.

2 Solutions of problems 001500

291

V.253. Suppose that X is a space and F is a nonempty closed subspace of X ; in


the R-quotient space XF denote by aF the point represented by the set F . Say that
F is deeply inside a set U 2 .X / if there exists a zero-set G in the space X such
that F  G  U . For the family U D fU W U is a cozero subset of X and F is
deeply inside the set U g prove that V D ffaF g [ .U nF / W U 2 U g is a local base
of the space XF at the point aF .
Solution. Let pF W X ! XF be the contraction map and fix an arbitrary set V 2 V;
by definition of V we can find a cozero set U and a zero-set G in the space X such
that F  G  U and V D .U nF / [ faF g. There exists a continuous function
' W X ! 0; 1
such that '.G/  f1g and '.X nU /  f0g (see Fact 1 of V.140).
Since ' is constant on F , we can find a function u W XF ! 0; 1
such that u pF D
'. The map pF being R-quotient, the function u is continuous; it is straightforward
that u.aF / D 1 and u.XF nV /  f0g. It follows from aF 2 u1 ..0; 1
/  V that aF
belongs to the interior of V .
Now, if x 2 V nfaF g then x 2 U nF and hence U nF D pF1 .U nF / is an open
neighborhood of x in XF with U nF  V . This shows that each point of V belongs
to the interior of V in XF , i.e., every V 2 V is open in XF and therefore V  .XF /.
Take an open set O in the space XF with aF 2 O. The space XF being Tychonoff
(see Problem 252) there exists a continuous function f W XF ! 0; 1
such that
f .aF / D 1 and f .XF nO/  f0g. Evidently, g D f pF is a continuous function
on X such that F  G D g 1 .1/; it is clear that G is a zero-set in X . Furthermore,
U D g 1 ..0; 1
/ is a cozero subset of X and it follows from G  U that F is
deeply inside the set U , i.e., U 2 U . Besides, V D .U nF / [ faF g coincides with
the set f 1 ..0; 1
/, so aF 2 V  O. Since also V 2 V, we have verified that V is a
local base of the space XF at the point aF .
V.254. Suppose that X is a normal space and F is a nonempty closed subspace
of X ; in the R-quotient space XF denote by aF the point represented by the set F .
Prove that U 2 .aF ; XF / if and only if .U nfaF g/ [ F is an open neighborhood of
F in the space X .
Solution. Let pF W X ! XF be the contraction map; apply Problem 252 to see that
the mapping pF j.X nF / W X nF ! XF nfaF g is a homeomorphism and therefore
. / the set pF .O/ is open in XF for any O 2 .X / such that O  X nF .
If U 2 .aF ; XF / then .U nfaF g/ [ F D pF1 .U / is an open subset of X
containing F ; this proves necessity.
Now, if W D .U nfaF g/ [ F 2 .F; X / then use normality of X to find a
continuous function f W X ! 0; 1
such that f .F / D f1g and f .X nW /  f0g.
Then V D f 1 ..0; 1
/ is a cozero set such that F  f 1 .1/  V , so H D
.V nF / [ faF g is open in the space XF by Problem 253. Observe that V nF 
W nF D U nfaF g and hence aF 2 H  U ; this shows that the point aF belongs to
the interior of the set U . If x 2 U nfaF g D W nF D pF .W nF / then apply . / to
convince ourselves that W nF  U is an open neighborhood of x in the space XF .
Thus every point of U belongs to the interior of U in XF , so U is open in XF and
hence we settled sufficiency.

292

2 Solutions of problems 001500

V.255. Suppose that X is a space and K is a nonempty compact subspace of X ;


in the R-quotient space XK denote by aK the point represented by the set K. Prove
that U 2 .aK ; XK / if and only if .U nfaK g/ [ K is an open neighborhood of K in
the space X .
Solution. Let pK W X ! XK be the contraction map; apply Problem 252 to see that
the mapping pK j.X nK/ W X nK ! XK nfaK g is a homeomorphism and therefore
. / the set pK .O/ is open in XK for any O 2 .X / such that O  X nK.
1
If U 2 .aK ; XK / then .U nfaK g/ [ K D pK
.U / is an open subset of X
containing K; this proves necessity.

Fact 1. Suppose that Z is a space, F  Z is closed, P  Z is compact and


P \ F D ;. Then the sets F and P are functionally separated, i.e., there exists a
continuous function f W X ! 0; 1
such that f .P /  f1g and f .F /  f0g.
Proof. The sets P and G D clZ .F / are closed in the space Z; if x 2 G \ P
then x 2 clZ .F / \ Z D clZ .F / D F which is a contradiction with P \ F D ;.
By normality of Z there exists a continuous function g W Z ! 0; 1
for which
g.P /  f1g and g.G/  f0g. The function f D gjZ W Z ! 0; 1
is continuous on
Z while f .P /  f1g and f .F /  f0g, so Fact 1 is proved.
Returning to our solution assume that W D .U nfaK g/ [ K 2 .K; X / and
use Fact 1 to find a continuous function f W X ! 0; 1
such that f .K/ D f1g and
f .X nW /  f0g. Then V D f 1 ..0; 1
/ is a cozero set such that K  f 1 .1/  V ,
so the set H D .V nK/ [ faK g is open in the space XK by Problem 253. Observe
that V nK  W nK D U nfaK g and hence aK 2 H  U ; this shows that the point
aK belongs to the interior of the set U . If x 2 U nfaK g D W nK D pK .W nK/ then
apply . / to convince ourselves that W nK  U is an open neighborhood of x in
the space XK . Thus every point of U belongs to the interior of U in XK , so U is
open in XK and hence we settled sufficiency.
V.256. Given a nonempty space X prove that closed sets P; Q  X are parallel
retracts of X is and only if there is a retraction r W X ! P such that rjQ W Q ! P
is a homeomorphism.
Solution. If P and Q are parallel retracts then there exist parallel retractions
r W X ! P and s W X ! Q; to see that the map r0 D rjQ W Q ! P is a
homeomorphism let s0 D sjP W P ! Q. By definition of parallel retractions we
have the equalities s0 .r0 .x// D s.r.x// D s.x/ D x for any x 2 Q; since also
r0 .s0 .y// D r.s.y// D r.y/ D y for any y 2 P , the maps r0 and s0 are mutually
inverse homeomorphisms and hence we proved necessity.
Now assume that there exists a retraction r W X ! P such that the mapping
r0 D rjQ W Q ! P is a homeomorphism; let s0 W P ! Q be the inverse of r0 and
consider the map s D s0 r. It is immediate that s W X ! Q is continuous; given
any x 2 Q we have s.x/ D s0 .r.x// D s0 .r0 .x// D x, so s is a retraction. Finally
observe that, for every x 2 X , we have s.r.x// D s0 .r.r.x/// D s0 .r.x// D s.x/

2 Solutions of problems 001500

293

and r.s.x// D r.s0 .r.x/// D .r0 s0 /.r.x// D r.x/. Therefore r and s are parallel
retractions, i.e., we settled sufficiency.
V.257. (Okunevs method of constructing l-equivalent spaces). Suppose that P and
Q are parallel retracts of a nonempty space X . Prove that the completely regular
quotient spaces XP and XQ are l-equivalent.
Solution. Fix some parallel retractions r W X ! P and s W X ! Q; as usual,
P W Cp .X / ! Cp .P / and Q W Cp .X / ! Cp .Q/ are the respective restriction
maps. We will also need the R-quotient map P W X ! XP defined as follows:
P .x/ D x if x 2 X nP and P .x/ D P if x 2 P . Analogously, define a map
Q W X ! XQ by Q .x/ D x whenever x 2 X nQ and Q .x/ D Q for each
x 2 Q. If we have spaces Z and T and a map ' W Z ! T then its dual map
'  W Cp .T / ! Cp .Z/ is defined by '  .f / D f ' for any f 2 Cp .T /.
Given a nonempty set Y  X say that a function f 2 C.X / is Y -constant if
there is c 2 R such that f .Y / D fcg. The dual maps P W Cp .XP / ! Cp .X /

and Q
W Cp .XQ / ! Cp .X / are linear embeddings (see TFS-163); let LP D


P .Cp .XP // and LQ D Q
.Cp .XQ //. To prove l-equivalence of XP and XQ
it suffices to show that the spaces LP and LQ are linearly homeomorphic. We
will actually construct a linear homeomorphism  W Cp .X / ! Cp .X / such that
.LP / D LQ .
Given any function f 2 Cp .X / let .f / D P .f / r C Q .f / s  f .
It follows from TFS-163 and continuity of operations in Cp .X / that the map  W
Cp .X / ! Cp .X / is linear and continuous. Fix an arbitrary function f 2 Cp .X /
and let g D .f /. For any point y 2 X we have g.y/ D f .r.y//Cf .s.y//f .y/;
now if x 2 X and y D r.x/ then g.y/ D f .r r.x// C f .s r.x//  f .r.x//;
recalling that r.r.x// D r.x/ and s.r.x// D s.x/ because the retractions r and s are
parallel, we convince ourselves that g.r.x// D f .s.x//. An analogous calculation
shows that g.s.x// D f .r.x//. Therefore
.g/.x/ D g.r.x// C g.s.x//  g.x/ D
D f .s.x// C f .r.x//  .f .r.x// C f .s.x//  f .x// D f .x/:
This proves that for any f 2 Cp .X / we have the equality ..f //.x/ D f .x/ for
every x 2 X , i.e., ..f // D f . An immediate consequence is that  W Cp .X / !
Cp .X / is a linear homeomorphism which coincides with its inverse.
Apply Fact 1 of V.252 to see that LP coincides with the set of all P -constant
functions and LQ is exactly the set of all Q-constant functions on X . Take any
function f 2 LP ; there exists c 2 R for which f .P / D fcg. Now, if x 2 Q
then .f /.x/ D f .r.x// C f .s.x//  f .x/; since s.x/ D x and r.x/ 2 P ,
we conclude that f .r.x// D c and hence .f /.x/ D c C f .x/  f .x/ D c.
Therefore .f /.x/ D c for any x 2 Q, so .f / is Q-constant for any f 2 LP ;
in other words, .LP /  LQ . An analogous proof shows that .LQ /  LP , so
jLP W LP ! LQ is a linear homeomorphism between the spaces LP and LQ . We
have already observed that Cp .XP / and Cp .XQ / are linearly homeomorphic to LP

294

2 Solutions of problems 001500

and LQ respectively. Therefore Cp .XP / is linearly homeomorphic to Cp .XQ /, i.e.,


the spaces XP and XQ are l-equivalent.
V.258. Suppose that K is a nonempty l-embedded subspace of a space X and fix a
point a X . Prove that the spaces X fag and XK K are l-equivalent. Deduce
l

from this fact that if K is a retract of the space X then X fag  XK K. Here
XK is the R-quotient space obtained by contracting K to a point.
Solution. If Z is a space and F is a nonempty closed subset of Z then ZF is the Rquotient space obtained by contracting F to a point; say that a function f W Z ! R
is F -constant if there exists c 2 R such that f .F / D fcg. Any spaces Z and T are
identified with the respective clopen subspaces of Z T ; the expression Z ' T
says that Z is homeomorphic to T . We denote by L
M the fact that the linear
topological spaces L and M are linearly homeomorphic. It is an easy exercise which
we leave to the reader that
(1) Cp .Y Z/
Cp .Y /  Cp .Z/ for any spaces Y and Z.
After Section 6.3 was finished, this solution was changed for the sake of
optimization. As a result, the following fact is not needed for our solution and is
left here only for further references.
Fact 1. Given spaces Z and T suppose that F is a nonempty closed subset of Z
and let P D Z T . Then the space PF is homeomorphic to ZF T .
Proof. To avoid confusion denote the point F in both spaces ZF and PF by zF .
Given an F -constant function f on the space PF there is c 2 R with f .F / D fcg;
let uf .x/ D f .x/ if x zF and f .zF / D c. This gives a function uf W PF ! R
for any function f W P ! R. Analogously, if f W Z ! R is an F -constant
function then f .F / D fcg for some c 2 R; let vf .x/ D f .x/ if x 2 ZF nfzF g
and vf .zF / D c. Therefore we have constructed a function vf W ZF ! R for any
F -constant function f W Z ! R.
The spaces PF and ZF T have the same underlying set so it suffices to prove
that a function g W PF ! R is continuous on PF if and only if it is continuous on
ZF T . Assume first that a function g W PF ! R is continuous. There exists a
continuous F -constant function f W P ! R such that g D uf (see Fact 1 of V.252).
The function f0 D f jZ is also F -constant and continuous, so vf0 is continuous
on ZF ; it is straightforward that vf0 D gjZF . Besides, f1 D f jT D gjT is a
continuous function on T , so fZF ; T g is an open cover of ZF T such that the
restrictions of g to its elements are continuous. Therefore g is continuous on ZF T
by Fact 1 of S.472.
Now, if g W ZF T ! R is continuous then g0 D gjZF is also continuous, so
there is an F -constant function f0 2 C.Z/ such that g0 D vf0 . Let f .z/ D f0 .z/
for any z 2 Z and f .z/ D g.z/ whenever z 2 T . It follows from continuity of f0
and gjT that f W Z T ! R is continuous. Since f is F -constant, the function
uf W PF ! R is continuous as well and it is straightforward that uf D g. This
shows that g is continuous if considered as a function on PF , so the spaces PF and
ZF T are homeomorphic, i.e., Fact 1 is proved.

2 Solutions of problems 001500

295

Let pK W X ! XK be the contraction map. Given any function f 2 Cp .XK /




let pK
.f / D f pK . Then pK
W Cp .XK / ! Cp .X / is a linear embedding and

pK .Cp .XK // coincides with the set C D ff 2 Cp .X / W f .x/ D f .y/ for any
x; y 2 Kg of K-constant functions on X (see Fact 1 of V.252).
Consider the set I D ff 2 Cp .X / W f .K/ D f0gg  C ; for any pair .f; t / 2
I  R let '.f; t / D f C t . Then ' W I  R ! C is a linear continuous map. Fix a
point z 2 K and let .f / D f  f .z/ for any f 2 C ; it is easy to see that the map
 W C ! I is linear and continuous. Therefore the map  W C ! I  R defined by
the equality .f / D ..f /; f .z// for any f 2 C , is also linear and continuous.
It is straightforward that the maps ' and  are mutually inverse, so we proved
that the space C
Cp .XK / is linearly homeomorphic to I  R. Now apply CFS448 to see that Cp .X /
Cp .K/  I which implies, together with Cp .X fag/

Cp .X /  R and the property (1) that


Cp .X fag/
Cp .K/  .I  R/
Cp .K/  Cp .XK /
Cp .XK K/;
so the spaces X fag and XK K are l-equivalent. If K is a retract of X then it is
l

l-embedded in X by CFS-465, so X fag  XK K is this case as well.


V.259. Given a space Xi and a point xi 2 Xi for any i D 1; : : : ; n consider the
space X D X1 : : : Xn and the set F D fx1 ; : : : ; xn g  X . The R-quotient
space XF is denoted by .X1 ; x1 / _ : : : _ .Xn ; xn / and called a bunch of spaces
X1 ; : : : ; Xn with respect to the points x1 ; : : : ; xn . Prove that if we choose any point
yi 2 Xi for every i D 1; : : : ; n then the spaces .X1 ; x1 / _ : : : _ .Xn ; xn / and
.X1 ; y1 / _ : : : _ .Xn ; yn / are l-equivalent.
Solution. We identify every space Xi with the respective clopen subspace of X .
Given x 2 X there is a unique i 2 f1; : : : ; ng such that x 2 Xi ; let r.x/ D xi .
The family fX1 ; : : : ; Xn g is an open cover of X while rjXi is constant and hence
continuous for every i 2 f1; : : : ; ng. Therefore the map r W X ! F is continuous
by Fact 1 of S.472. It follows from xi 2 Xi that r.xi / D xi for all i  n; in other
words, r.x/ D x for each x 2 F , so r is a retraction.
If G D fy1 ; : : : ; yn g then r.yi / D xi for all i  n, so rjG W G ! F is a
homeomorphism which shows that F and G are parallel retracts (see Problem 256).
Applying Problem 257 we conclude that the spaces XF D .X1 ; x1 / _ : : : _ .Xn ; xn /
and XG D .X1 ; y1 / _ : : : _ .Xn ; yn / are l-equivalent.
V.260. Let K be a retract of a nonempty space X and fix any point z 2 K. Denote
by aK the point of the space XK represented by the set K. Prove that the space X
is l-equivalent to the bunch .XK ; aK / _ .K; z/ of the spaces XK and K with respect
to the points aK and z.
Solution. We identify any spaces Y and Z with the respective clopen subspaces of
the space Y Z. If Y is a space and F ; is a closed subspace of Y then YF is the
R-quotient space obtained from Y by contracting F to a point and pF W Y ! YF is
the contraction map; to avoid confusion, the point represented by F in the space YF
is denoted by aF .

296

2 Solutions of problems 001500

Fact 1. Suppose that Y is a space and F is a nonempty closed subspace of Y . If


r W Y ! Q is a retraction such that the set r.F / is a singleton then the map
p D pF jQ W Q ! pF .Q/ is a homeomorphism.
Proof. If F \ Q D ; then apply Problem 252 to see that the map pF W
Y nF ! YF nfaF g is a homeomorphism; since Q  Y nF , the map p is also a
homeomorphism.
If F \ Q ; then there exists a point a 2 Q such that F \ Q D r.F / D fag. It
follows from continuity of pF that the map p is continuous. Since Qnfag  QnF ,
the map pj.Qnfag/ W Qnfag ! pF .Q/nfaF g is also a homeomorphism; besides,
p.a/ D aF , so p is a condensation. Therefore it suffices to show that the map p 1
is continuous at the point aF .
Fix a set U 2 .a; Q/; it is easy to find a cozero set V and a zero-set H in
the space Q such that a 2 H  V  U . Then H 0 D r 1 .H / is a zero-set and
V 0 D r 1 .V / is a cozero set in the space Y (see Fact 1 of S.499); the map r being
a retraction, we have V 0 \ Q D V . It follows from F  H 0  V 0 that the set
W D .V 0 nF / [ faF g is an open neighborhood of aF in YF (see Problem 253). As a
consequence, the set W 0 D W \pF .Q/ D .V nfag/[faF g is an open neighborhood
of aF in pF .Q/. If x 2 W 0 nfaF g then p 1 .x/ D x 2 V ; since p 1 .aF / D a 2 V ,
we conclude that p 1 .W 0 /  V  U and hence the map p 1 is continuous at the
point aF . Therefore p is a homeomorphism, i.e., Fact 1 is proved.
Fact 2. Suppose that Y is a space and Q is a retract of Y . If F is a nonempty closed
subset of Y with F  Q and pF W Y ! YF is a contraction map then the topology
induced on Q0 D pF .Q/ from the space YF coincides with the topology of the
space QF .
Proof. Since the map pF is an identity on the set Y nF , we will identify the sets
U; pF .U / and pF1 .U / for any U  Y nF . Note first that the sets Q0 and QF
coincide; let be the topology on Q0 induced from YF and denote by  the topology
of QF . Suppose first that U 2  and aF U ; then U  QnF , so it follows
from Problem 252 that U is open in QnF and hence there is U 0 2 .Y / such that
U 0 \ Q D U . Apply Problem 252 again to see that U 0  Y nF is open in YF and
hence U D U 0 \ Q 2 . This shows that
(1) if U 2  and aF U then U 2 .
Now, assume that U 2 and aF U ; there exists an open set U 0 in the space YF
such that U D U 0 \ Q0 . Since U 0  Y nF , it follows from Problem 252 that U 0 is
open in Y nF and hence U belongs to the topology of QnF . Applying Problem 252
to the space QF we convince ourselves that U 2 . This proves that
(2) if aF U  Q0 then U 2  if and only if U 2 .
Next assume that aF 2 U 2 ; there exist a zero-set H and a cozero set W
in the space Q such that F  H  W and U 0 D .W nF / [ faF g  U (see
Problem 253). If H 0 D r 1 .H / and W 0 D r 1 .W / then H 0 is a zero-set and W 0 is
a cozero set in the space Y ; the map r being a retraction we have W 0 \ Q D W .
Since also F  H 0  W 0 , we can apply Problem 253 again to conclude that the set

2 Solutions of problems 001500

297

V D .W 0 nF / [ faF g is open in YF , so U 0 D V \ Q0 belongs to . It follows from


(2) that U nfaF g 2 , so U D U 0 [ .U nfaF g/ also belongs to . Therefore,
(3) if aF 2 U 2  then U 2 .
Finally suppose that aF 2 U 2 and choose an open subset U 0 of the space YF
such that U 0 \ Q0 D U . There exist a zero-set H and a cozero set W in the space
Y such that F  H  W and V D .W nF / [ faF g  U 0 (see Problem 253). If
H 0 D H \ Q and W 0 D W \ Q then H 0 is a zero-set and W 0 is a cozero set in the
space Q. Since also F  H 0  W 0 , we can apply Problem 253 again to conclude
that the set V 0 D .W 0 nF / [ faF g  U is open in QF . It follows from (2) that
U nfaF g 2 , so U D V 0 [ .U nfaF g/ also belongs to . As a consequence,
(4) if aF 2 U 2 then U 2 .
It follows from (1)(4) that D , so Fact 2 is proved.
Fact 3. Given a space Y and a 2 Y suppose that Q1 ; : : : ; Qn are closed subspaces
of Y such that Y D Q1 [ : : : [ Qn and Qi \ Qj D fag for any distinct i; j 2
f1; : : : ; ng. Then Y is homeomorphic to the space .Q1 ; a/ _ : : : _ .Qn ; a/.
Proof. We will identify every Qi with the respective clopen subspace of the space
Q D Q1 : : : Qn ; the point ai 2 Qi stands for the copy of the point a in the
space Qi . Let F D fa1 ; : : : ; an g; it suffices to show that Y is homeomorphic to the
space QF . Let h.a/ D aF ; if y 2 Y nfag then there is a unique i  n with y 2 Qi ;
let h.y/ D y. This gives us a bijection h W Y ! QF ; an easy consequence of
Problem 252 is that hj.Y nfag/ W Y nfag ! QnF is a homeomorphism.
Fix a set U 2 .aF ; QF /; by Problem 253, there is an open set V in the space
Q such that F  V and .V nF / [ faF g  U . The set V nF is also open in Y ;
furthermore, Gi D Qi nV is closed in Qi and hence in Y for each i  n. Therefore
the set G D G1 [ : : : [ Gn is closed in Y , so W D Y nG is an open neighborhood
of the point a. It is immediate that h.W / D .V nF / [ faF g  U and hence h is
continuous at the point a.
Finally take any set U 2 .a; Y /; it is easy to find a zero-set H and a cozero set
W in the space Y such that a 2 H  W  U . Then Hi D H \ Qi is a zero-set
and Wi D W \ Qi is a cozero set in Qi for every i  n; consider the copy Wi0 of
Wi and the copy Hi0 of Hi in the set Qi  Q. It is evident that H 0 D H10 [ [ Hn0
is a zero-set and W 0 D W10 [ [ Wn0 is a cozero set in the space Q. It follows from
F  H 0  W 0 that O D .W 0 nF / [ faF g is an open neighborhood of aF in QF
(see Problem 253). Since h1 .O/  W  U , the set O witnesses continuity of the
map h1 at the point aF . Thus h is a homeomorphism between the spaces Y and
QF ; the space QF is homeomorphic to .Q1 ; a/ _ : : : _ .Qn ; a/, so Fact 3 is proved.
Returning to our solution fix a retraction r W X ! K and a copy K0 of the space
K with a homeomorphism h W K ! K0 . Let z0 D h.z/ and consider the space
Z D X K0 together with the set F D fz; z0 g  Z; let pF W Z ! ZF be the
contraction map.
It is trivial that there exist retractions 0 W Z ! X and 1 W Z ! K0 such that
0 .F / and 1 .F / are singletons, so we can apply Fact 1 to conclude that the maps

298

2 Solutions of problems 001500

p D pF jX W X ! pF .X / and q D pF jK0 W K0 ! pF .K0 / are homeomorphisms;


let X 0 D pF .X /; K 0 D pF .K/ and K1 D pF .K0 /. The map s1 D p r p 1 W
X 0 ! K 0 is easily seen to be a retraction while the map s2 D .pjK/ h1 q 1 W
K1 ! K 0 is a homeomorphism.
The family fX 0 ; K1 g is a closed cover of ZF such that X 0 \ K1 D faF g. Since
s1 .aF / D s2 .aF / D aF , there exists a continuous map s W ZF ! K 0 such that
sjX 0 D s1 and sjK1 D s2 (see Fact 2 of T.354). It is straightforward that s is a
retraction such that sjK1 is a homeomorphism, so K 0 and K1 are parallel retracts of
Z 0 D ZF by Problem 256.
Letting .x/ D x for any x 2 X 0 and .x/ D aF for any x 2 K1 nfaF g we obtain
a retraction  W Z 0 ! X 0 such that .K1 / is a singleton. This makes it possible to
0
apply Fact 1 again to see that the space ZK
is homeomorphic to X 0 which is in turn
1
homeomorphic to X .
Observe that p W X ! X 0 is a homeomorphism such that p.K/ D K 0 ; an
immediate consequence is that XK0 0 is homeomorphic to XK . Since X 0 is a retract
of the space Z 0 , we can apply Fact 2 to see that Q D XK0 0 is homeomorphic to
0
0
0
the respective subspace of ZK
0 ; let qK 0 W Z ! ZK 0 be the contraction map. There
0
0
exists a retraction u of Z onto K1 such that u.K / is a singleton, so we can apply
0
Fact 1 to convince ourselves that qK 0 jK1 embeds K1 in the space ZK
0 ; let P D
qK 0 .K1 /.
0
0
This shows that ZK
0 D P [ Q where P and Q are closed subsets of ZK 0 such
0
that P \ Q D faK 0 g. By Fact 3, the space ZK 0 is homeomorphic to .Q; aK 0 / _
0
0
0
.P; aK 0 /. The spaces ZK
0 and ZK1 are l-equivalent by Problem 257; besides, ZK1
0
is homeomorphic to X and ZK 0 is homeomorphic to .XK ; b/ _ .K; c/ for some
points b 2 XK and c 2 K. It follows from Problem 259 that X is l-equivalent to
.XK ; aK / _ .K; z/, i.e., our solution is complete.
V.261. Assume that K and L are retracts of a nonempty space X and there exists
a retraction r W X ! L such that r.K/ D K \ L D fag for some point a 2 L;
let M D K [ L. Prove that the space X is l-equivalent to the bunch .XM ; c0 / _
.K; c1 / _ .L; c2 / where the points c0 2 XM ; c1 2 K and c2 2 L are chosen
arbitrarily.
Solution. We identify any spaces Y and Z with the respective clopen subspaces of
the space Y Z. If Y is a space and F ; is a closed subspace of Y then YF is
the R-quotient space obtained from Y by contracting F to a point.
Denote by aK the point represented in XK by the set K and take a homeomorphic
copy K0 of the space K. Pick a point z0 2 K0 and let Q D faK ; z0 g. Define a map
on the space Y D XK K0 by letting .x/ D x for any x 2 XK and .x/ D aK
for every x 2 K0 . It is evident that W Y ! XK is a retraction such that .Q/ is a
singleton. Analogously, we can construct a retraction W Y ! K0 such that .Q/
is a singleton.
The space YQ D .XK ; aK / _ .K0 ; z0 / is l-equivalent to X by Problem 260;
denote by wQ the point of YQ represented by the set Q. Let q.x/ D x for any x 2
XK nfaK g and q.aK / D wQ ; then q is the restriction to XK of the R-quotient map

2 Solutions of problems 001500

299

that contracts Q to a point. By Fact 1 of V.260, the map q W XK ! .X nK/ [ fwQ g


is a homeomorphism so we identify XK with the subspace T D .X nK/ [ fwQ g of
the space Z D YQ . Analogously, K 0 D .K0 nfz0 g/ [ fwQ g is a homeomorphic copy
of the space K.
If pK W X ! XK is the contraction map then pK jL W L ! L0 D pK .L/ is
a homeomorphism by Fact 1 of V.260. Define a map r0 W XK ! L0 by letting
r0 .aK / D aK and r0 .x/ D pK .r.x// for any point x 2 XK nfaK g. It follows from
r0 pK D pK r that r0 pK is continuous; the map pK being R-quotient, we can
apply Fact 1 of T.268 to conclude that r0 is a continuous map.
If x 2 L0 nfaK g D Lnfag then r.x/ D x, so r0 .x/ D pK .x/ D x; this shows
that the map r0 W XK ! L0 is a retraction. Recalling that T is a copy of XK , we
can consider that L0  T is a copy of L and r0 W T ! L0 is a retraction of T onto
L0 . Let .t / D t for any t 2 T and .x/ D wQ whenever x 2 K 0 nfwQ g. It is
straightforward that  W Z ! T is a retraction so r0  is a retraction of Z onto L0 .
Denote by bL0 the point represented by the set L0 in the space ZL0 and let pL0 W
Z ! ZL0 be the contraction map. Since T is a retract of Z and L0  T , it follows
from Fact 2 of V.260 that T 0 D TL0 can be identified with the set pL0 .T /.
Apply Problem 260 again to convince ourselves that the space Z is l-equivalent
to the bunch B D .ZL0 ; bL0 / _ .L0 ; t / where the point t 2 L0 is chosen arbitrarily.
Let .x/ D wQ if x 2 T and .x/ D x whenever x 2 K 0 . It is immediate that
W Z ! K 0 is a retraction such that .L0 / is a singleton, so pL0 jK 0 W K 0 ! K1 D
pL0 .K 0 / is a homeomorphism. It is easy to see that K1 \ T 0 D fbL0 g.
Denote by uG the point of the space B represented by the set G D fbL0 ; t g. It
is easy to verify, applying Fact 1 of V.260, that the contraction map v W ZL0
L0 ! B is a homeomorphism if restricted to either ZL0 or L0 ; let K2 D v.K1 / and
L2 D v.L0 /. If T1 D v.T 0 / then the family fT1 ; K2 ; L2 g is a closed cover of the
space B and T1 \ K2 D T1 \ L2 D K2 \ L2 D fuG g. Therefore we can apply
Problem 259 and Fact 3 of V.260 to conclude that B is homeomorphic to the bunch
.T1 ; x/ _ .K2 ; y/ _ .L2 ; z/ for some x 2 T1 ; y 2 K2 and z 2 L2 .
The spaces K2 and L2 were obtained by applying several homeomorphisms to
the spaces K and L respectively. As a consequence, B is homeomorphic to the
bunch .T1 ; x/ _ .K; y/ _ .L; z/ for some x 2 Z2 ; y 2 K and z 2 L.
Consider the space E D XK and let h D pL0 jE W E ! pL0 .E/ D EL0 . Our
construction shows that the space T1 is homeomorphic to EL0 . Let sM W X ! XM
be the contraction map and denote by zM the point represented in XM by the set
M D K [ L. Let .bL0 / D zM ; if x 2 EL0 nfbL0 g then x 2 X nM , so we can
let .x/ D x. This gives us a bijection  W EL0 ! XM and it is immediate that
 .h pK / D sM . The map sM is continuous and h pK is R-quotient, so 
is a continuous map (see Fact 1 of T.268). The map  1 sM D h pK is also
continuous; since sM is R-quotient, we can apply Fact 1 of T.268 again to see that
 1 is continuous, i.e.,  is a homeomorphism. This shows that T1 is homeomorphic
to XM and hence B is homeomorphic to the bunch .XM ; x/ _ .K; y/ _ .L; z/ for
some x 2 XM ; y 2 K and z 2 L.

300

2 Solutions of problems 001500

Finally recall that X is l-equivalent to Z and Z is l-equivalent to B, so we can


apply Problem 259 to conclude that X is l-equivalent to .XM ; c0 /_.K; c1 /_.L; c2 /
for any choice of points c0 2 XM ; c1 2 K and c2 2 L.
V.262. Given spaces Y and Z consider the space X D Y  Z; choose arbitrary
points y0 2 Y; z0 2 Z and let M D .Y  fz0 g/ [ .fy0 g  Z/. Prove that, for any
x0 2 XM , the space X is l-equivalent to the bunch .XM ; x0 / _ .Y; y0 / _ .Z; z0 /.
Solution. If K D Y  fz0 g and L D fy0 g  Z then both K and L are retracts
of X and M D K [ L. Letting r.y; z/ D .y0 ; z/ for any .y; z/ 2 X we obtain a
retraction r W X ! L such that r.K/ D f.y0 ; z0 /g D K \ L. This makes it possible
to apply Problems 261 and 259 to conclude that X is l-equivalent to the bunch
.XM ; x/ _ .K; a/ _ .L; b/ for any points x 2 XM ; a 2 K and b 2 L. The space
K is homeomorphic to Y and L is homeomorphic to Z, so X is also l-equivalent
to the bunch .XM ; x0 / _ .Y; y0 / _ .Z; z0 / where the point x0 2 XM can be chosen
arbitrarily.
1
V.263. Let a D 0 and an D nC1
for all n 2 !; then S D fan W n 2 !g [ fag is
a faithfully indexed convergent sequence with limit a. Given an infinite cardinal 
consider the discrete space D./ of cardinality  and let E D D./  S . Observe
that F D D./  fag is a retract of E; as usual let EF be the R-quotient space
obtained by contracting F to a point. The space EF will be denoted by V ./; it
is often called the FrchetUrysohn -fan. The space V .!/ is called the Frchet
Urysohn fan. Prove that V ./ is l-equivalent to D./S for any infinite cardinal .
Deduce from this fact that

(i) there exist l-equivalent spaces X and Y with w.X / w.Y / and .X /
.Y /;
(ii) metrizability is not preserved by l-equivalence;

(iii) a space l-equivalent to a locally compact space, need not be Cech-complete.


Solution. Given spaces X and Y the expression X ' Y says that they are
homeomorphic. For any point .x; y/ 2 E let r.x; y/ D .x; a/; it is evident that
the map r W E ! F is a retraction, so F is, indeed, a retract of E. If D is a discrete
space with jDj   then D./ D is also a discrete space of cardinality , so we
have the following property:
(1) D./ D ' D./ for any discrete space D with jDj  .
Since a0 is an isolated point of S , the set A D D./  fa0 g is clopen and discrete
in E. The space S0 D S nfa0 g is homeomorphic to S , so G D D./  S0 is a clopen
subspace of E homeomorphic to E; it follows from E D G [A that E ' E D./.
If D is a discrete space of cardinality at most  then E D ' E .D./ D/,
so we can apply (1) to see that E D ' E D./ ' E. Therefore
(2) E D ' E for any discrete space D with jDj  .
By (2) and Fact 1 of V.258, the space EF is homeomorphic to EF D./;
the set F being a discrete subspace of E of cardinality , we convince ourselves
that EF ' EF F . Fix any point b E and apply Problem 258 to see that

2 Solutions of problems 001500

301

EF F  E fbg; it follows from (2) that E fbg ' E, so V ./ D EF is


l-equivalent to E D D./  S as promised.
Therefore the spaces X D V .!/ and Y D D.!/  S are l-equivalent. It is
straightforward that Y is a locally compact metrizable space of countable weight.
To analyze the space X let D D fdn W n 2 !g be a faithfully indexed discrete space;
we can identify the space Y with D  S . If F D D  fag then X is homeomorphic
to YF ; let Sn D fdn g  S and bn D .dn ; a/ for each n 2 !.
Denote by xF the point of X represented by the set F and take an arbitrary
family U D fUn W n 2 !g of open neighborhoods of xF in X . The space Y being
normal, every set Wn D .Un nfxF g/ [ F is an open neighborhood of F in Y (see
Problem 254). In particular, Wn 2 .bn ; Y /, so Sn nWn is finite because the sequence
Sn converges to bn ; pick a point cn 2 .Sn nfbn g/ \ Wn for every n 2 ! and let
C D fcn W n 2 !g.
The set V D Y nC is an open neighborhood of F in Y , so we can apply
Problem 254 once more to see that the set W D .V nF / [ fxF g is an open
neighborhood of xF in X . Since cn 2 Un nW for every n 2 !, no element of U is
contained in W and hence U is a not a local base at xF in the space X . Therefore X
has no countable local base at xF and hence w.X /  .X / > ! D .Y / D w.Y /,
so the property (i) holds for the spaces X and Y . The space X cannot be metrizable
being a countable space of uncountable weight; this settles (ii). It follows from

nw.X / D ! < w.X / that X cannot be Cech-complete


either (see TFS-270); this
proves (iii) and completes our solution.
V.264. Given infinite cardinals 1 ; : : : ; n prove that the space A.1 / : : : A.n /
is l-equivalent to A./ where  D maxf1 ; : : : ; n g.
Solution. To model every space A.i / fix a number i 2 f1; : : : ; ng and choose a
set Bi of a cardinality i together with a point ai Bi . Introduce the topology of
A.i / on the set Ai D fai g [ Bi declaring all points of Bi isolated in Ai and letting
.ai ; Ai / D fU  Ai W ai 2 U and jAi nU j < !g.
In the space A D A1 : : : An the set F D fa1 ; : : : ; an g is a retract of A.
Let aF be the point represented by F in the R-quotient space AF . It follows from
Problem 260 that the space A is l-equivalent to the bunch B D .AF ; aF / _ .F; z/
where the point z 2 F is chosen arbitrarily. Recall that B is defined as the R-quotient
space of the space Q D AF F in which the set G D faF ; zg is contracted to a
point bG .
Since aF is the unique non-isolated point of AF and F is finite, bG is the unique
non-isolated point of the space B D QG . The space B is a continuous image of Q
while Q is a continuous image of a compact space A F . Therefore B is a compact
space of cardinality  with a unique non-isolated point. A trivial consequence is that
l

B is homeomorphic to A./ and hence A  A./. Since A.1 / : : : A.n / is


homeomorphic to A, we conclude that the space A.1 / : : : A.n / is l-equivalent
to A./.

302

2 Solutions of problems 001500

L
V.265. Given a family of spaces fXt Q
W t 2 T g let X D
t2T Xt and prove that
Cp .X / is linearly homeomorphic to t2T Cp .Xt /. Deduce from this fact that if
L
l
l
Xt  Yt for any t 2 T then X  Y D t2T Yt .
Solution. We identify every Xt with the respective clopen subspace of X ; it is
straightforward that the restriction map t W Cp .X / ! Cp .Xt / is linear and
continuous. We omit a trivial proof that any diagonal
product of linear maps is a
Q
linear map, so  D ft W t 2 T g W Cp .X / ! t2T Cp .Xt / is a linear continuous
map; recall that .f /.t / D t .f / for any f 2 Cp .X / and t 2 T .
If f and g are distinct elements of Cp .X / then f .x/ g.x/ for some x 2 X .
There is t 2 T with x 2 Xt , so t .f / t .g/ and hence
Q .f / .g/ which
shows that  is an injective map. Given an arbitrary g 2 t2T Cp .Xt / the function
g.t / belongs to Cp .Xt / for each t 2 T . Since fXt W t 2 T g is a clopen disjoint
cover of X , there is a function f 2 Cp .X / such that t .f / D g.t / for all t 2 T . It
is immediate that .f / D g, so we established that  is a surjective map.
X
X
Recall that CQ
p .X /  R and let px .f / D f .x/ for all f 2 R and x 2 X .
Denote the set t2T Cp .Xt / by C and consider the projection map qt W C !
Cp .Xt / for each t 2 T . Fix any x 2 X and the unique t 2 T such that x 2 Xt ; if
rx .f / D f .x/ for any f 2 Cp .Xt / then the map rx W Cp .Xt / ! R is continuous.
For any g 2 C we have ' 1 .g/.x/ D g.t /.x/ D qt .g/.x/ D rx .qt .g// which
shows that px ' 1 D rx qt is a continuous map for every x 2 X . Therefore
the map ' 1 is continuous by Q
TFS-102 and hence ' is a linear homeomorphism
between the spaces Cp .X / and t2T Cp .Xt /.
Finally, assume that Xt is l-equivalent to Yt and fix a linear homeomorphism
't W Cp .Xt /Q! Cp .Yt /Qfor each t 2 T . An
Q easy proof of the fact that the product
map ' D t2T 't W t2T Cp .Xt / ! t2T Cp .Yt / is linear
left to the
Q can be
1
1
reader. It follows from Fact 1 of S.271 and the
equality
'
D
'
that ' is a
t
t2T
Q
Q
linear homeomorphism between the spaces t2T Cp .Xt / and t2T Cp .Yt /. Since
they are linearly homeomorphic to Cp .X / and Cp .Y / respectively, we conclude that
l

Cp .X / is linearly homeomorphic to Cp .Y /, i.e., X  Y .


V.266.LSuppose that a space Ji is homeomorphic to I for any i D 1; : : : ; n and let
J D fJi W 1  i  ng. Prove that the space J D is l-equivalent to I for any
finite space D. Deduce from this fact that
(i) connectedness is not preserved by l-equivalence;
(ii) for any cardinal  there exist l-equivalent spaces X and Y such that X has no
isolated points and Y has -many isolated points.
Solution. In any discrete union Z D Z1 : : :Zn we identify every space Zi with
the respective clopen subspace of Z. The expression Z ' T says that the spaces Z
and T are homeomorphic while M L denotes the fact that the linear topological
spaces M and L are linearly homeomorphic. We will first prove that
(1) I D is l-equivalent to I for any finite space D.

2 Solutions of problems 001500

303

1
Fix a finite space D and consider the subspace S D f0g [ f nC1
W n 2 !g of the
space I. Since S is a convergent sequence, the space S D is homeomorphic to S
and hence Cp .S /  RD is linearly homeomorphic to Cp .S / (see Problem 265).
Apply Fact 2 of U.216 to see that there is a linear topological subspace L of
the space Cp .I/ such that Cp .I/ is linearly homeomorphic to L  Cp .S /. Therefore
Cp .I D/ .L  Cp .S //  RD L  .Cp .S /  RD / L  Cp .S / Cp .I/,
so the property (1) is proved.

Fact 1. Given a space Z suppose that a 2 Z and Z0 ; Z1 are closed subsets of


Z such that Z0 [ Z1 D Z and Z0 \ Z1 D fag. If there exist homeomorphisms
h0 W I ! Z0 and h1 W I ! Z1 such that h0 .1/ D h1 .1/ D a then Z is
homeomorphic to I.
1
Proof. Let f0 .x/ D h1
0 .x/ for any x 2 Z0 and f1 .x/ D 2  h1 .x/ for each
x 2 Z1 . Then f0 W Z0 ! 1; 1
and f1 W Z1 ! 1; 3
are continuous functions
such that f0 .a/ D f1 .a/. By Fact 2 of T.354 there exists a continuous function
f W X ! 1; 3
such that f jZi D fi for every i 2 f0; 1g. It is straightforward that
f is a bijection; the space Z is compact, so f is a homeomorphism between Z and
1; 3
. Since 1; 3
' I, we have Z ' I and hence Fact 1 is proved.

Returning to our solution let us show that


(2) I  D is l-equivalent to I for any nonempty finite space D.
Obviously, the property (2) holds when jDj D 1. Assume that D D fd0 ; d1 g and
d0 d1 . The set F D f.1; d0 /; .1; d1 /g is a retract of Q D I  D; denote by aF
the point of the R-quotient space QF represented by F . We can apply Problem 260
to see that Q is l-equivalent to the bunch B D .QF ; aF / _ .F; d0 /. Recall that the
space B is obtained as the R-quotient space of Q F when the set G D faF ; d0 g
is contracted to a point; let bG be the point of B represented by the set G. It is easy
to see that d1 is an isolated point of B. If Zi D .1; 1/  fdi g/ [ faF g then Fact 1
of V.260 can be applied to the space QF to convince ourselves that there exists
a homeomorphism hi W I ! Zi for which hi .1/ D aF for every i 2 f0; 1g. Since
Z0 [ Z1 D QF and Z0 \ Z1 D faF g, we can apply Fact 1 to conclude that QF ' I.
Apply Fact 1 of V.260 once more to note that the set Q0 D .QF nfaF g/ [ fbG g is
homeomorphic to QF , so Q0 ' I and therefore B ' I fd1 g. The property (1)
l

shows that B  I, so Q D ID  I and hence we established (2) for all two-element
sets D.
Proceeding by induction assume that n  3 and we proved (2) for all sets D with
1  jDj < n. If jDj D n then D D D0 D1 where jD0 j D 2 and jD1 j D n  2.
l

It is evident that I  D ' .I  D0 / .I  D1 /. Since I  D0  I, it follows from


l

Problem 265 that I  D  I .I  D1 /. The space I .I  D1 / is easily seen to


be homeomorphic to I  E for some set E with jEj D jD1 j C 1 D n  1, so the
l

induction hypothesis shows that I .I  D1 /  I and hence I  D  I, i.e., (2) is


proved.

304

2 Solutions of problems 001500

Next observe that the set J D

fJi W 1  i  ng is homeomorphic to I  D0

for some finite set D0 , so J  I by the property (2). Applying (1) we conclude that
J D is l-equivalent to I and hence our first statement is proved. An immediate
consequence of (2) is that a disconnected space I I is l-equivalent to a connected
space I; this settles (i).
Finally, fix a cardinal , a homeomorphic copy I of the space I and a point
l

b I for every < . It follows from (1) that I  E D I


Lfb g for all
< , so we can L
apply Problem 265 to see that the space X D
< I is lequivalent to Y D < E while X is dense-in-itself and Y has -many isolated
points. This proves (ii) and completes our solution.
V.267. Let X be a compact space with jX j D   !. Prove that the space AD.X /
is l-equivalent to X A./. Here AD.X / is the Alexandroff double of the space X
and A./ is the one-point compactification of a discrete space of cardinality .
Solution. The expression Y ' Z says that the spaces Y and Z are homeomorphic.
Any spaces Y and Z are identified with the respective clopen subspaces of Y Z.
Recall that D D f0; 1g is the standard discrete two-point space and AD.X / D
X  D; all points of X1 D X  f1g are isolated in AD.X / and a local base at any
point y D .x; 0/ is given by the family f.U  D/nf.x; 1/g W U 2 .x; X /g. It is
evident that the subspace X0 D X  f0g of the space AD.X / is homeomorphic to
X . For any y 2 X0 let r.y/ D y; if y D .x; 1/ 2 X1 then let r.y/ D .x; 0/. It is
straightforward from the definitions that r W AD.X / ! X0 is a retraction. Let Y be
the R-quotient space obtained from AD.X / by contracting X0 to a point a0 and fix
any point z0 2 X0 .
By Problem 260, the space AD.X / is l-equivalent to the bunch B 0 D .Y; a0 / _
.X0 ; z0 /. Apply Problem 259 to see that B 0 is l-equivalent to the bunch B D .Y; a/_
.X0 ; z0 / where a 2 Y nfa0 g. Observe that Y is a compact space of cardinality  in
which only the point a0 is not isolated; consequently, Y is homeomorphic to A./.
Let G D fa; z0 g and denote by aG the point represented by the set G in the
space B. Recall that B is the R-quotient space of the space Q D Y X0 when
G is contracted to a point. It is routine to verify that we can apply Fact 1 of
V.260 to convince ourselves that the contraction map pG W Q ! QG D B
is a homeomorphism when restricted to either Y or X0 . In particular, the set
X 0 D .X0 nfz0 g/ [ faG g is homeomorphic to X0 and hence to X .
By the same reason, Y0 D .Y nfag/ [ faG g is homeomorphic to Y ; since a is an
isolated point of Y , the point aG D pG .a/ is isolated in Y0 . Therefore Y1 D Y0 nfaG g
is a clopen subset of Y0 ; a minutes thought shows that Y1 is also clopen in B and
hence B is homeomorphic to Y1 X 0 ' Y1 X . Taking away an isolated point
from the space A./ gives a subspace homeomorphic to A./, so B ' Y1 X '
l

X A./. It follows from X  B 0  B ' X A./ that X  X A./, i.e., we


proved what was promised.

2 Solutions of problems 001500

305

V.268. Let X be a compact space such that jX j D   !. Prove that AD.X /


is l-equivalent to AD.X / A./. Here AD.X / is the Alexandroff double of the
space X .
Solution. Apply Problem 267 to see that AD.X / is l-equivalent to X A./.
Therefore AD.X / A./ is l-equivalent to .X A.// A./ by Problem 265. It
l

follows from Problem 264 that A./ A./  A./, so AD.X / A./  X
l

.A./ A.//  X A./  AD.X / which shows that AD.X / A./ is lequivalent to AD.X /.
V.269. Prove that there exist l-equivalent compact spaces X and Y such that
.X / .Y /. As a consequence, pseudocharacter is not l-invariant.
Solution. As usual, D D f0; 1g is the standard two-point discrete space. Recall that
the Alexandroff double of I is the space AD.I/ D I  D in which all points of
I1 D I  f1g are isolated while the standard local base at any point y D .t; 0/ is
given by the family f.U  D/nf.t; 1/g W U 2 .t; I/g. The space X D AD.I/ is
compact by TFS-364. It clear that .y; X /  ! for any x 2 I1 ; if y D .t; 0/ for
some t 2 I then the family f..t  n1 ; t C n1 /  D/nf.t; 1/g W n 2 Ng is easily seen
to be a countable local base of X at the point y. Therefore .y; X /  ! for any
y 2 X , i.e., .X /  !.
l

It follows from Problem 267 that X  Y D I A.c/; we identify A.c/ with the
respective clopen subspace of Y . It is obvious that, for the unique non-isolated point
a of the space A.c/, we have .a; Y / D c. Therefore X and Y are l-equivalent
compact spaces such that .X / D .X / D ! < c D .Y / which shows that
pseudocharacter is not preserved by l-equivalence.
V.270. Prove that there exist l-equivalent compact spaces X and Y such that X
has nontrivial convergent sequences and Y does not have any.
Solution. As usual, D D f0; 1g is the standard two-point discrete space. Recall that
the Alexandroff double of ! is the space AD.!/ D ! D in which all points of
B1 D !  f1g are isolated while the standard local base at any point y D .x; 0/ is
given by the family f.U  D/nf.x; 1/g W U 2 .x; !/g. The space Y D AD.!/
is compact by TFS-364.
Assume that S D fsn W n 2 !g is a nontrivial convergent sequence in Y ; we can
assume that S is faithfully indexed and its limit s does not belong to S . No isolated
point can be a limit of a nontrivial convergent sequence, so s 2 B0 D !  f0g;
fix a point t 2 ! such that s D .t; 0/. The space B0 is homeomorphic to !
which does not have nontrivial convergent sequences (see Fact 2 of T.131), so we
can assume, without loss of generality, that S  B1 and .t; 1/ S . For every n 2 !
there is xn 2 ! with sn D .xn ; 1/; let wn D .xn ; 0/. It is a trivial consequence
of the definition of the topology of the space AD.!/ that the nontrivial sequence
fwn W n 2 !g  B0 nfsg converges to s, i.e., we have found a nontrivial convergent
sequence in B0 ' ! which is a contradiction. Therefore the space Y does not have
nontrivial convergent sequences.

306

2 Solutions of problems 001500


l

It follows from Problem 267 that Y  X D ! A.2c /; we identify A.2c / with


the respective clopen subspace of X . It is obvious that, for the unique non-isolated
point a of the space A.2c /, if Q is a countably infinite subset of A.2c /nfag then
Q [ fag is a nontrivial convergent sequence in X . As a consequence, X and Y are
l-equivalent compact spaces such that X has nontrivial convergent sequences and
Y does not have any.
V.271. Prove that for any uncountable regular cardinal  with its usual order
l
l
topology we have the equivalencies    A./ and . C 1/ . C 1/ A./.
Therefore there exist l-equivalent spaces X and Y such that all compact subspaces
of X are metrizable while Y has non-metrizable compact subspaces.
Solution. The expression X ' Y says that the spaces X and Y are homeomorphic.
For any uncountable regular cardinal  let L./ D f <  W is a limit ordinalg
and LC ./ D L./ [ fg. Suppose that we have a map f W  ! Z and an ordinal
< . To avoid confusion, we will consider that f ./ is the image of the point
and f
D ff ./ W < g. We will need the interval .;
D f W <   g
for any ordinals and .
Fact 1. The space L./ is homeomorphic to  and LC ./ is homeomorphic to
. C 1/ for any uncountable regular cardinal .
Proof. Fix an uncountable regular cardinal  and let f .0/ D !. If 0 <   and
we defined an ordinal f ./ 2 L./ for any < , let
(1) f ./ D nnf 2 L./ W f ./ <  for all < g.
This inductive procedure defines a point f ./ 2 L./ for any 2 ; letting
f ./ D  we obtain a function f W . C 1/ ! LC ./. It is an immediate
consequence of (1) that
(2) <   implies f ./ < f ./.
If the set A D f   W f ./ < g is nonempty then let  be the minimal
element of A. The ordinal 0 D f . / is strictly less than  and f .0 / < f . / D 0 ,
so 0 2 A which is a contradiction. Therefore the set A is empty, i.e.,
(3)  f ./ for any  .
If the set B D L./nf 
is nonempty then let  be the minimal element of
B. The set C D f <  W f ./ <  g is nonempty because 0 2 C . It follows
from (3) that f ./    for any   , so C   and hence the ordinal
0 D nnf <  W C  g is consistently defined. If < 0 then  for some
2 C and hence f ./  f ./ <  , i.e., 2 C for any < 0 which shows that
0 D C .
By minimality of  , for any 2 L./\ there is 2  such that f ./ D and
hence 2 C . Therefore f 0
D f .C /  L./ \  , so  is the minimal element of

2 Solutions of problems 001500

307

L./ such that f ./ <  for any < 0 . The definition (1) shows that f .0 / D  ;
this contradiction proves that
(4) f 
D L./ and hence f is a bijection between . C 1/ and LC ./.
The map f is clearly continuous at every 2 . C 1/nLC ./, so assume that
0   is a limit ordinal and fix an ordinal 0 < 0 D f .0 /. It follows from (2)
that the set H D ff ./ W < 0 g does not have a maximal element. If H  0
then  D sup.H /  0 < 0 . Besides,  2 LC ./ because LC ./ is closed in
 C 1. This together with (1) shows that f .0 /  0 which is a contradiction.
As a consequence, there exists an ordinal < 0 such that f ./  0 ; the set
W D .; 0
is an open neighborhood of 0 and f .W /  .0 ; 0
, i.e., W witnesses
continuity of f at the point 0 ; this proves that f is continuous at every point of
 C 1, i.e., f is a continuous map. Furthermore,  C 1 is compact and f is a
bijection, so f is a homeomorphism between  C 1 and LC ./. Since f 
D
L./, the spaces  and L./ are also homeomorphic and hence Fact 1 is proved.
Returning to our solution denote by Z the space  and let T D . C 1/. The
set F D L./ is a retract of Z and G D LC ./ is a retract of T (see Fact 5 of
U.074). Fix a point a T and observe that Z ' .! C 1/ .Zn.! C 1//; since
.! C 1/ fag ' .! C 1/, we conclude that Z fag is homeomorphic to Z. The
spaces Z fag and ZF F are l-equivalent by Problem 258, so the space Z is
also l-equivalent to ZF F ' ZF  (see Fact 1).
The space  is countably compact and hence so is ZF ; let yF be the point of
ZF represented by F . Evidently, yF is the unique non-isolated point of ZF . If
O 2 .yF ; ZF / then ZF nO is finite being discrete and countably compact. An
immediate consequence is that the space ZF is homeomorphic to A./ which shows
l

that Z D    A./.
Analogously, T ' T fag; therefore the space T is l-equivalent to TG G.
Since TG is a compact space with a unique non-isolated point, it is homeomorphic
l

to A./. It follows from Fact 1 that G ' . C 1/, so T D . C 1/ . C 1/ A./.


Finally, observe that, in the space X D !1 , all compact subspaces are metrizable
while X is l-equivalent to Y D X A.!1 / in which A.!1 / is a non-metrizable
compact subspace.
V.272. Suppose that compact spaces X and Y are l-equivalent. Prove that X  Z
is l-equivalent to Y  Z for any space Z.
Solution. If K is a compact space then Cu .K/ is the topology of uniform
convergence on K; the topology of Cu .K/ is generated by the metric K defined
by the formula K .f; g/ D supfjf .x/  g.x/j W x 2 Kg for any f; g 2 C.K/.
Fact 1. Given spaces T and W if ' W Cp .T / ! Cp .W / is a linear homeomorphism
then there exists a linear homeomorphism W RT ! RW such that jCp .T / D '.
Proof. For any w 2 W and f 2 RW let pw .f / D f .w/; then pw W RW ! R
is the projection onto the factor determined by w. The map pw ' is a continuous
linear functional on Cp .T /, so we can apply Problem 224 to find a continuous linear

308

2 Solutions of problems 001500

functional w W RT ! R such that w jCp .T / D pw ' for each w 2 W . The diagonal


product D fw W w 2 W g W RT ! RW is a continuous linear map and it is an
easy exercise that jCp .T / D '. Analogously, there exists a continuous linear map
 W RW ! RT such that  jCp .W / D ' 1 .
Let iT W RT ! RT and iW W RW ! RW be the respective identity maps. If
f 2 Cp .T / then '.f / 2 Cp .W /, so  ..f // D  .'.f // D ' 1 .'.f // D f ;
analogously, . .g// D .' 1 .g// D '.' 1 .g// D g for any g 2 Cp .W /. This
shows that . /jCp .T / D iT and .  /jCp .W / D iW , so we can apply Fact 0
of S.351 to conclude that  D iW and  D iT . Therefore and  are
mutually inverse linear homeomorphisms and hence Fact 1 is proved.
Fact 2. Given spaces T; U and W suppose that ' W RT ! RU is a linear
homeomorphism. For any function f W T  W ! R let fw .t / D f .t; w/ for
any t 2 T ; then fw 2 RT for any w 2 W . Let .f /.u; w/ D '.fw /.u/ for
any f 2 RT W and .u; w/ 2 U  W . Then W RT W ! RU W is a linear
homeomorphism.
Proof. It is immediate that is a linear map. For any function g W U  W ! R let
g w .u/ D g.u; w/ for any .u; w/ 2 U  W ; then g w 2 RU for any w 2 W and we can
let  .g/.t; w/ D ' 1 .g w /.t / for any .t; w/ 2 T  W . Then  W RU W ! RT W
and we omit a straightforward proof that and  are mutually inverse linear maps.
Fix a point .u; w/ 2 U  W and let p.u;w/ W RU W ! R be the projection
onto the factor determined by .u; w/. If .f / D fw for any function f 2 RT W
then  W RT W ! RT is the composition of the restriction map to T  fwg and
an evident homeomorphism which forgets w; therefore  is a continuous map. If
q.f / D f .u/ for any f 2 RU then the map q W RU ! R is continuous being a
natural projection. Thus p.u;w/ D q '  is a continuous map; since the point
.u; w/ 2 U  W was chosen arbitrarily, we can apply TFS-102 to conclude that
is continuous. An identical proof shows that  is also continuous, so is, indeed,
a linear homeomorphism, i.e., Fact 2 is proved.
Returning to our solution fix a linear homeomorphism ' W Cp .X / ! Cp .Y / and
a space Z; if we consider that ' W Cu .X / ! Cu .Y / then ' is also a homeomorphism
(see Fact 1 of U.481). Apply Fact 1 to find a linear homeomorphism  W RX ! RY
such that jCp .X / D '. Let fz .x/ D f .x; z/ for any f 2 RXZ and .x; z/ 2
X  Z. Then fz 2 RX whenever f 2 RXZ and z 2 Z. Letting .f /.y; z/ D
.fz /.y/ for any function f 2 RXZ and .y; z/ 2 Y  Z we obtain a linear
homeomorphism W RXZ ! RY Z (see Fact 2), so it suffices to prove that
.C.X  Z// D C.Y  Z/.
Fix a function p 2 C.X  Z/; then pz 2 C.X / for any z 2 Z. To see that
q D .p/ is a continuous function on Y  Z take any point a0 D .y0 ; z0 / 2 Y  Z
and " > 0. The map ' W Cu .X / ! Cu .Y / being continuous there exists > 0 such
that f 2 C.X / and X .f; pz0 / < implies Y .'.f /; '.pz0 // < 2" . For any x 2 X
there exist Ox 2 .x; X / and Wx 2 .z0 ; Z/ such that jp.a/  p.x; z0 /j < 2 for
any a 2 Ox  Wx . The open cover fOx W x 2 X gTof the space X has a subcover
fOx W x 2 F g for some finite set F  X ; let W D fWx W x 2 F g.

2 Solutions of problems 001500

309

Take any points x 2 X and z 2 W ; there exists x 0 2 F such that x 2 Ox 0 . Then


.x; z/ 2 Ox 0  W  Ox 0  Wx 0 which implies that jp.x; z/  p.x 0 ; z0 /j < 2 and
jp.x; z0 /  p.x 0 ; z0 /j < 2 . As an immediate consequence we obtain the inequality
jp.x; z/  p.x; z0 /j < ; the points x 2 X and z 2 W were chosen arbitrarily, so
we established that jpz .x/  pz0 .x/j < for any x 2 X and z 2 W . Recalling that
X is compact we convince ourselves that
(1) X .pz ; pz0 / < and hence Y .'.pz /; '.pz0 // <

"
2

for any z 2 W .

The function hz0 D '.pz0 / being continuous on Y there is V 2 .y0 ; Y / such


that jhz0 .y/  hz0 .y0 /j < 2" for any y 2 V . Take any point a D .y; z/ from the set
V  W and let hz D '.pz /. It follows from (1) that Y .hz ; hz0 / < 2" and therefore
jq.a/  q.a0 /j D jhz .y/  hz0 .y0 /j  jhz .y/  hz0 .y/j C jhz0 .y/  hz0 .y0 /j <
"
C 2" D ".
2
Thus the set V  W witnesses continuity of q at the point a0 . This proves that
.C.X  Z//  C.Y  Z/. Given any function g 2 RY Z let gz .y/ D g.y; z/ for
any y 2 Y ; then gz 2 RY for any z 2 Z. If we let  .g/.x; z/ D 1 .gz /.x/ for any
.x; z/ 2 X  Z and g 2 RY Z then  W RY Z ! RXZ is the inverse map of .
Since  is a symmetric version of , the above proof for can be easily modified
to establish that  .C.Y  Z//  C.X  Z/ and hence .C.X  Z// D C.Y  Z/.
This proves that the spaces Cp .X  Z/ and Cp .Y  Z/ are linearly homeomorphic,
i.e., the spaces X  Z and Y  Z are l-equivalent, so our solution is complete.
l

V.273. Given a family fX1 ; : : : ; Xn g of compact spaces assume that Xi  Yi for all
i 2 f1; : : : ; ng. Prove that the spaces X D X1  : : :  Xn and Y D Y1  : : :  Yn
are l-equivalent.
Solution. Observe first that every Yi is compact by Problem 134 and 138. For any
i 2 f1; : : : ; ng let Si be the statement which asserts that the space Pi D X1  : : : Xi
is l-equivalent to Qi D Y1  : : :  Yi . It is clear that S1 is true; proceeding by
l

induction assume that 1  k < n and Sk holds, i.e., Pk  Qk . It follows from


l

Problem 272 that Pk  XkC1 is l-equivalent to Qk  XkC1 . Since XkC1  YkC1 , we


l

can apply Problem 272 once more to conclude that Qk  XkC1  Qk  YkC1 D
QkC1 . Therefore PkC1 D Pk  XkC1 is l-equivalent to QkC1 , i.e., we established
SkC1 , so our inductive procedure shows that Si is true for all i  n. In particular Sn
is true, i.e., the spaces Pn D X1  : : :  Xn and Qn D Y1  : : :  Yn are l-equivalent
as required.
V.274. Give an example of l-equivalent spaces X and Y such that X  Z is not
t -equivalent to Y  Z for some space Z.
1
Solution. Let a D 0 and an D nC1
for any n 2 !; then S D fag [ fan W n 2 !g is
a faithfully indexed convergent sequence with its limit a. For any infinite cardinal 
we denote by D./ a discrete space of cardinality . In the space D./  S the set
F D D./  fag is a retract of D./  S ; let V ./ be the R-quotient space obtained
from D./  S by contracting F to a point. A subset A of a space T will be called
-closed in T if B  A for any B  A with jBj  . It is immediate that every

310

2 Solutions of problems 001500

closed subset of T is -closed and the intersection of every two -closed subsets
of T is a -closed subset of T . Recall that weak functional tightness tm .P / of a
space P is the minimal cardinal  such that every strictly -continuous real-valued
function on P is continuous.
Fact 1. Given spaces P; Q and an infinite cardinal  suppose that t .Q/   and
there exists a continuous closed onto map f W P ! Q such that t .f 1 .q//  
for any q 2 Q. Then t .P /  . As a consequence, if K is a compact space with
t .K/   then t .Q  K/  .
Proof. Suppose that A  P is a -closed non-closed subspace of P and fix a point
z 2 AnA. The set F D f 1 f .z/ is closed in P , so G D F \ A is -closed; since
t .F /  , we can apply Fact 1 of S.328 to conclude that G is closed in F and hence
in P . Choose an open neighborhood U of the point z in P such that U \ G D ;.
The set E D A \ U is still -closed while z 2 EnE and f 1 .f .z// \ E D ;.
If y D f .z/ then it follows from continuity of f that y 2 f .E/; by our choice
of E, the point y does not belong to f .E/, so f .E/ is not closed in Q. It is an easy
exercise that the image of a -closed set under a closed map is -closed, so f .E/
is a -closed non-closed subset of Q. This, together with Fact 1 of S.328, gives a
contradiction with t .Q/  . Therefore every -closed subset of P is closed, so we
can apply Fact 1 of S.328 once more to conclude that t .P /  .
Finally, if K is a compact space with t .K/   then the natural projection
 W Q  K ! Q is a perfect map (see Fact 3 of S.288) such that  1 .z/ is
homeomorphic to K and hence t . 1 .z//   for every z 2 Q; this implies that
t .Q  K/   and shows that Fact 1 is proved.
Fact 2. Given a space P suppose that fUa W a 2 Ag is an open cover of P such that
t .Ua /   for any a 2 A. Then t .P /  .
Proof. Let ia W Ua ! P be the identity map, i.e., ia .x/ D x forL
any x 2 Ua . If
we identify every Ua with the respective clopen subspace of U D fUa W a 2 Ag
then there is a continuous map i W U ! P such that i jUa D ia for any a 2 A. It
is clear that i.U / D P ; if O is an open subset of U then Oa DS
O \ Ua is an open
subset both of Ua and P for each a 2 A. Therefore i.O/ D fOa W a 2 Ag is
an open subset of P ; this shows that the map i is open and hence quotient. It is an
easy exercise that t .U /  , so we can apply TFS-162 to convince ourselves that
t .P /   and hence Fact 2 is proved.
Fact 3. Weak functional tightness of the space P D V .!/  V .c/ is uncountable.
Proof. To prove that tm .P / > ! it suffices to construct a strictly !-continuous
discontinuous function f W P ! R. We will need a more convenient representation
of the spaces V .!/ and V .c/. Denote by a the unique non-isolated point of V .!/ and
let b be the unique non-isolated point of V .c/. Then V .!/nfag D faji W i; j 2 !g
where the indexation is faithful and Ai D faji W j 2 !g is a copy of the subspace
fd g  S of V .!/ for each i 2 !. In this model, a set U  V .!/ is open in V .!/ if
and only if either U  A D V .!/nfag or a 2 U and Ai nU is finite for every i 2 !.

2 Solutions of problems 001500

311

Analogously, V .c/ D fbg [ fbi W i 2 !; < cg where the indexation is faithful


and the set B D fbi W i 2 !g is a sequence that converges to b for every < c.
It is easy to see that a set U  V .c/ is open in the space V .c/ if and only if either
U  B D V .c/nfbg or b 2 U and B nU is finite for each < c.
Call a countably infinite set D  A thin if jD \ Ai j  1 for all i 2 !. Choose
an enumeration fD W < cg of all thin subsets of A; let  W B ! D be a
bijection and consider the set E DSf. .y/; y/ W y 2 B g  V .!/  V .c/ for every
< c. All points of the set E D fE W < cg are isolated in P . If v D .a; y/
for some y 2 B then there is < c such that y 2 B , so the set V .!/  fyg is a
clopen neighborhood of v which contains only the point . .y/; y/ of the set E. If
v D .x; b/ for some x 2 A then, for any < c, it follows from the fact that  is
a bijection that there is a point y 2 B such that  .y/ x for all y 2 B nfy g.
Therefore the set W 0 D V .c/nfy W < cg is an open neighborhood of b in V .c/,
so W D fxg  W 0 is an open neighborhood of v in P such that W 0 \ E D ;.
This shows that every point v 2 P n.E [ f.a; b/g/ has an open neighborhood
which does not meet the set E 0 D E [ f.a; b/g; as a result, E 0 is closed in P .
We are going to prove next that the point u D .a; b/ belongs to the closure of E.
Indeed, if W is an open neighborhood of u in P then there exist G 2 .a; V .!//
and H 2 .b; V .c// such that G  H  W . It is immediate that there exists a thin
set D  G; then D D D for some < c. Since the sequence B converges to b,
we can find y 2 B \ H ; then . .y/; y/ 2 .G  H / \ E, so E \ W ; for any
W 2 .u; P / which shows that u 2 E. Let us prove that
(1) u C for any countable C  E.
Fix a countable
set C  E; there exists a sequence fn W n 2 !g  c such that
S
C  L D n2! En , so it suffices to show that u does not belong to the closure
of the set L. Let Ri D i .Bi / for all i 2 !. Since every Rk is a thin set, we can
find a number mk 2 ! such that k .bnk / Aj whenever j  k and n  mk ;
let Ri0 D Ri nfi .bji / W j < mk g for all i 2 !. An immediate consequence is
0
that Ai \ Rj0 D ; whenever i  j , so there exists a finite
S set Ai 0  Ai such that
0
0
.Ai nAi / \ Rn D ; for all i; n 2 !. The set O D fagS[ . fAi nAi W i 2 !g/ is an
open neighborhood of a in V .!/ while W D V .c/n. fbjk W k 2 !; j < mk g/ is
an open neighborhood of b in V .c/, so the set G D O  W is an open neighborhood
of u in P .
Take any point v 2 L; there are k; j 2 ! such that v D .k .y/; y/ where
y D bjk . If j < mk then y W , so v G. If j  mk then k .y/ 2 Rk0 which
shows that k .y/ O and hence v G. Thus v G for any v 2 L which shows
that G \ L D ; and hence G \ C D ;, so u C and hence (1) is proved.
Let f .v/ D 0 for any v 2 E and f .v/ D 1 whenever v 2 P nE. The function
f W P ! R is discontinuous because f .E/ D f0g while u 2 E and f .u/ D 1. To
see that f is strictly !-continuous fix any countable set C  P . If C 0 D C \ E
then u C 0 by (1), so C 0 is closed in E 0 and hence in P . Since all points of C 0 are
isolated in P , the set C 0 is clopen in P . As a consequence, the function h defined
by h.x/ D 0 for all x 2 C 0 and h.x/ D 1 whenever x 2 P nC 0 is continuous on
P . It is immediate that hjC D f jC , so f is a strictly !-continuous discontinuous
function; therefore tm .P / > !, i.e., Fact 3 is proved.

312

2 Solutions of problems 001500

Returning to our solution let X D V .!/ and Y D D.!/  S ; the spaces X and Y
are l-equivalent by Problem 263. If Z D V .c/ then tm .X  Z/ > ! by Fact 3. The
space Y Z is representable as a countable union of open subspaces homeomorphic
to V .c/  S . It is an easy exercise that the space V .c/ is FrchetUrysohn and hence
t .V .c//  !. Therefore t .S  V .c//  ! by Fact 1, so we can apply Fact 2 to
conclude that t .Y  Z/  ! and hence tm .Y  Z/  ! (see TFS-419). It follows
from tm .X  Z/ tm .Y  Z/ that the spaces X  Z and Y  Z are not t -equivalent
(see Problem 008), so our solution is complete.
V.275. Give an example of l-equivalent spaces X and Y such that X  X is not
t -equivalent to Y  Y .
1
Solution. Let a D 0 and an D nC1
for any n 2 !; then S D fag [ fan W n 2 !g is
a faithfully indexed convergent sequence with its limit a. For any infinite cardinal 
we denote by D./ a discrete space of cardinality . In the space D./  S the set
F D D./  fag is a retract of D./  S ; let V ./ be the R-quotient space obtained
from D./  S by contracting F to a point. Recall that weak functional tightness
tm .P / of a space P is the minimal cardinal  such that every strictly -continuous
real-valued function on P is continuous.
l
l
We have V .!/  D.!/  S and V .c/  D.c/  S by Problem 263, so we can
apply Problem 265 to see that the spaces X D V .!/ V .c/ and Y D .D.!/  S /
.D.c/  S / are l-equivalent. The space Y is easily seen to be metrizable, so Y  Y
is also metrizable and hence tm .Y  Y /  t .Y  Y / D ! (see TFS-419).
The space X  X contains a clopen subspace G homeomorphic to V .!/  V .c/,
so tm .G/ > ! by Fact 3 of V.274. It is an easy exercise that G is a retract of X  X ,
so fix a retraction r W X  X ! G. The map r is quotient (see Fact 11 of U.074) and
hence R-quotient, so we can apply TFS-420 to conclude that tm .X  X /  tm .G/ >
!. Now, it follows from tm .X  X / tm .Y  Y / that the spaces X  X and Y  Y
are not t -equivalent (see Problem 008).

V.276. Given infinite cardinals 1 ; : : : ; n prove that the space A.1 /  : : :  A.n /
is l-equivalent to A./ where  D maxf1 ; : : : ; n g.
Solution. Our statement is clearly true if n D 1; let us prove it for n D 2. Represent
A.1 / as D [ fag where a D is the unique non-isolated point of A.1 / and jDj D
1 . Analogously, A.2 / D fbg [ E where b E is the unique non-isolated point of
A.2 / and jEj D 2 . In the space X D A.1 /  A.2 / both sets K D fag  A.2 /
and L D A.1 /  fbg are retracts. Let r.x; y/ D .a; y/ for any .x; y/ 2 X ; it is
straightforward that r W X ! K is a retraction such that r.L/ D L \ K D f.a; b/g.
Let M D K [ L and denote by XM the R-quotient space obtained from X by
contracting M to a point aM . It is easy to see that aM is the unique non-isolated point
of the space XM . If p D .a; b/ then p 2 K \ L and p is the unique non-isolated
point of K [ L. Apply Problem 261 to see that the space X is l-equivalent to the
bunch B D .XM ; aM / _ .K; p0 / _ .L; p1 / where p0 is the copy of the point p in the
space K and p1 is the copy of p in L. It is evident that only the point represented
by the set faM ; p0 ; p1 g is non-isolated in B (recall that B is the R-quotient space

2 Solutions of problems 001500

313

of XM K L obtained by contracting the set faM ; p0 ; p1 g to a point). Since B


is compact and jBj D  D maxf1 ; 2 g, we conclude that B is homeomorphic to
A./. Therefore X D A.1 /  A.2 / is l-equivalent to A./, so we settled the case
when n D 2.
Proceeding inductively, assume that m  2 and we proved that, for any i  m
and any infinite cardinals 1 ; : : : ; i the space A. 1 /  : : :  A. i / is l-equivalent
to A. / where D maxf 1 ; : : : ; i g. Given infinite cardinals 1 ; : : : ; m ; mC1
consider the cardinals  0 D maxf1 ; : : : ; m g and  D maxf1 ; : : : ; m ; mC1 g; by
the induction hypothesis the space P D A.1 /: : :A.m / is l-equivalent to A. 0 /.
It follows from Problem 273 that P  A.mC1 / is l-equivalent to A. 0 /  A.mC1 /.
l

Again, our induction hypothesis guarantees that A. 0 /  A.mC1 /  A./, so the
space P  A.mC1 / D A.1 /  : : :  A.mC1 / is l-equivalent to A./ and hence our
inductive procedure shows that, for any n 2 N and any infinite cardinals 1 ; : : : ; n ,
the space A.1 /  : : :  A.n / is l-equivalent to A./ where  D maxf1 ; : : : ; n g.
V.277. Prove that there exist l-equivalent spaces X and Y such that X is
hereditarily paracompact while Y is not hereditarily normal.
Solution. If X D A.!1 / and Y D A.!1 /  A.!1 / then X is l-equivalent to Y
by Problem 276. It is straightforward that any subspace of X is either discrete or
compact, so X is hereditarily paracompact. If a is the unique non-isolated point of
A.!1 / then .a; A.!1 // D !1 , so fag is not a G -subset of A.!1 / (see TFS-327)
and hence A.!1 / is not perfectly normal.
Assume that Y is hereditarily normal. Since A.!1 /A.!/ embeds in Y , the space
A.!1 /  A.!/ is also hereditarily normal and hence A.!1 / is perfectly normal by
Fact 2 of S.292; this contradiction proves that Y is not hereditarily normal.
V.278. Prove that Lp .D/ is l-equivalent to Lp .D/ D for any infinite discrete
space D. Deduce from this fact that the Souslin property is not l-invariant.
Solution. The expression M
L says that the linear topological spaces M and L
are linearly homeomorphic. If I D ff 2 Cp .Lp .D// W f .D/ D f0gg then I is a
linear subspace of the space Cp .Lp .D//; since D is l-embedded in Lp .D/ by CFS467, the space Cp .Lp .D// is linearly homeomorphic to I  Cp .D/ D I  RD (see
CFS-448). Therefore Cp .Lp .D//RD
I .RD RD /
I RD
Cp .Lp .D//
l

which shows that Lp .D/  Lp .D/ D (see Problem 265).


Finally, let D be an uncountable discrete space and apply Fact 3 of V.024 to
see that c.Lp .D// D !; however, the space Lp .D/ is l-equivalent to Lp .D/ D
while c.Lp .D/ D/ D c.D/ D jDj > ! and hence the Souslin property is not
l-invariant.
V.279. Given spaces X and Y suppose that ' W Cp .X / ! Cp .Y / is a continuous
linear surjection. Prove that,
(i) for any point y 2 Y , there exist uniquely determined n D n.y/ 2 N,
distinct points x1 .y/; : : : ; xn .y/ 2 X and numbers 1 .y/; : : : ; n .y/ 2 Rnf0g
Pn.y/
such that '.f /.y/ D iD1 i .y/f .xi .y// for any f 2 Cp .X /; for further
reference denote the set fx1 .y/; : : : ; xn .y/g by supp.'; y/.

314

2 Solutions of problems 001500

(ii) if '  W Cp .Cp .Y // ! Cp .Cp .X // is the dual map of ' then '  embeds Lp .Y /
in Lp .X / and '  .y/ D 1 .y/x1 .y/ C : : : C n.y/ .y/xn.y/ .y/ for any y 2 Y .
Solution. Fix a point y 2 Y and recall that we also consider y to be a linear
functional on the space Cp .Y /. Then y ' is a continuous linear functional on
Cp .X /, i.e., y ' 2 Lp .X / (see TFS-197). It is easy to see that y ' is not
identically zero, so P
we can find n D n.y/ 2 N and distinct points x1 ; : : : ; xn 2 X
such that y ' D niD1 i xi for some 1 ; : : : ; n 2 Rnf0g. Since X is a Hamel
basis in Lp .X / (see Fact 5 of S.489), this representation of y ' is unique; let
i .y/ D i and xi .y/ D xi for all i 2 f1; : : : ; ng.
PFor any function fP2 Cp .X / we
have the equalities '.f /.y/ D .y '/.f / D . niD1 i xi /.f / D niD1 i f .xi /,
so (i) is proved. It follows from Fact 4 of S.489 that '  embeds Lp .Y / in Lp .X /.
Pn.y/
Since we have the equalities '  .y/ D y ' D iD1 i .y/xi .y/ for any y 2 Y ,
we are also done with (ii).
V.280. Suppose that ' W Cp .X / ! Cp .Y / is a continuous linear surjection and let
.y/ D supp.'; y/ for any y 2 Y . Prove that the map  W Y ! exp.X / is lower
semicontinuous.
Solution. Fix a set U 2 .X / and a point y0 2 l1 .U / D fy 2 Y W .y/\U ;g.
Pick a point x0 2 .y0 /\U and take a function f 2 C.X / such that f .x0 / D 1 and
f .x/ D 0 for any x 2 .X nU / [ ..y0 /nfx
P 0 g/. For any point x 2 .y0 / there exists
.x/ 2 Rnf0g such that '.g/.y0 / D x2.y0 / .x/g.x/ for any g 2 Cp .X /. In
particular, '.f /.y0 / D .x0 /f .x0 / D .x0 / 0, so the set V D '.f /
1 .Rnf0g/
is an open neighborhood of the point y0 .
If y 2 V and P
.y/ D fx1 ; : : : ; xn g  X nU then there exist 1 ; : : : ; n 2 R such
that '.f /.y/ D niD1 i f .xi /, so it follows from f ..y// D f0g that '.f /.y/ D 0
which is a contradiction. Therefore .y/ \ U ; for any y 2 V , so every point
y0 2 l1 .U / has an open neighborhood V contained in l1 .U /. This shows that
l1 .U / is open in Y for any U 2 .X /, i.e.,  is a lower semicontinuous map.
V.281. Given a continuous linear surjection ' W Cp .X / S
! Cp .Y / prove that, for
any bounded subset A of the space Y , the set supp.A/ D fsupp.'; y/ W y 2 Ag is
bounded in X .
Solution. Recall that if u D 1 x1 C : : : C n xn 2 Lp .X / for some x1 ; : : : ; xn 2 X
and 1 ; : : : ; n 2 Rnf0g then supp.u/ D fx1 ; : : : ; xn g; if u D 0 then supp.u/ D ;.
Any v 2 Lp .Y / is a continuous linear functional on Cp .Y /; let '  .v/ D v '.
Then '  W Lp .Y / ! Lp .X / is a linear embedding and it follows from Problem 279
that supp.'  .y// D supp.'; y/ for any y 2 Y .
If a set A  Y is bounded in Y then A is bounded in Lp .Y /, so '  .A/ is
bounded in 'S .Lp .Y // and hence in Lp .X /. Our observation above shows that
supp.A/ D fsupp.'  .y// W y 2 Ag and hence we can apply Problem 245 to
conclude that the set supp.A/ is bounded in X .

2 Solutions of problems 001500

315

V.282. Suppose that ' W Cp .X / ! Cp .Y / is a continuous linear surjection. Prove


that, for any bounded subset B  X , the set C D fy 2 Y W supp.'; y/  Bg is
bounded in Y .
Solution. Assume that the set C is not bounded in Y ; by Fact 1 of V.245, there
exists a discrete family fUn W n 2 !g of open subsets of Y such that Un \ C ;
and hence we can fix a point yn 2 Un \ C for each n 2 !.
Let supp.'; yn / D fx1n ; : : : ; xknn g and choose numbers n1 ; : : : ; nkn 2 Rnf0g such
Pn n
that '.f /.yn / D kiD1
i f .xin / for any f 2 Cp .X / and n 2 !. It follows from
Fact 5 of T.132 that we can find a function g 2 Cp .Y / such that g.yn / D n .j n1 j C
: : : C j nkn j/ for every n 2 !. Pick f 2 Cp .X / with '.f / D g; the set B being
bounded in X there is K > 0 such that jf .x/j  K for all x 2 B; it follows from
supp.'; yn /  B that jf .xin /j  K for any n 2 ! and i  kn . Take any n 2 ! with
K < n; then
jg.yn /j D j

kn
X
iD1

ni f .xin /j  K

kn
X
iD1

j ni j < n

kn
X

j ni j D jg.yn /j;

iD1

which is a contradiction. Therefore the set C is bounded in Y .


V.283. Say that X is a -space if A is compact for any bounded set A  X . Prove
that X is a -space if and only if Lp .X / is a -space. As a consequence, -property
is preserved by l-equivalence.
Solution. Suppose that Lp .X / is a -space and take any bounded subset A of the
space X . It is evident that A is also bounded in Lp .X /, so the closure K of the set
A in Lp .X / is compact. If A is the closure of A in X then A is closed in Lp .X /
because X is closed in Lp .X /. An easy consequence is that the set A D K is
compact, so X is a -space.
To prove necessity assume that X is a -space; recall first that X is C -embedded
in the Hewitt realcompactification X of the space X , so we can apply Fact 3 of
V.246 to identify Lp .X / with the linear hull of the set X in Lp .X /. Assume that
A is a bounded subset of Lp .X /. For any w D 1 x1 C : : : C n xn 2 Lp .X /
where i 2 Rnf0g and xi 2 X for all i  n, let supp.w/ D fx1 ; : : : ; xn g; if
w D 0 then supp.w/ D ;. The set A is still bounded in Lp .X / by Fact 2 of S.398
(the bar denotes the closure in Lp .X /), so we can apply Problem 245 to see that
S
B D fsupp.w/ W w 2 Ag is a bounded subset of X and hence K D clX .B/ is
compact.
Let G be the linear hull of the set K in the space Lp .X /; it is evident that
A  G  Lp .X /. By Fact 2 of V.246, the set G is closed in Lp .X /; besides, A
is closed in Lp .X / and hence in G, so A is closed in Lp .X /. Besides, A is also
bounded in Lp .X / because Lp .X /  Lp .X /; since Lp .X / is realcompact by
Problem 244, the set A is compact by Fact 1 of V.246, so Lp .X / is a -space, i.e.,
we established necessity and hence our solution is complete.

316

2 Solutions of problems 001500

V.284. Given spaces X and Y assume that X is a -space and there exists a linear
surjection ' W Cp .X / ! Cp .Y / which is an R-quotient map. Prove that Y is also
a -space. Give an example of a compact space X (which is, automatically, a space) such that there exists a continuous linear surjection of Cp .X / onto Cp .Y /
for some Y which is not a -space.
Solution. For any f 2 Cp .Cp .Y // let '  .f / D f '; this definition gives us the
map '  W Cp .Cp .Y // ! Cp .Cp .X // which embeds Cp .Cp .Y // in Cp .Cp .X // as
a closed subspace (see TFS-163). Besides, '  .Lp .Y //  Lp .X / by Problem 279;
this shows that '  .Lp .Y // is closed in Lp .X / and hence '  .Y / is closed in Lp .X /.
Thus, Y embeds in Lp .X / as a closed subspace. Apply Problem 283 to see that
Lp .X / is a -space; it is an easy exercise that -property is closed-hereditary, so Y
is a -space as well.
Now if X D !1 C 1 and Y D !1 then X is compact and Y is a countably
compact non-compact space, so Y fails to be a -space. We also have X D Y by
TFS-314 and hence the restriction map  W Cp .X / ! Cp .Y / is a continuous linear
surjection.
V.285. Given -spaces X and Y , let ' W Cp .X / ! Cp .Y / be a continuous linear
surjection. Prove that, if X is compact then Y is also compact. Observe that the
same conclusion about Y may be false if Y is not a -space.
Solution. The map ' is uniformly continuous by Problem 134, so we can apply
Problem 193 to see that the space Y is pseudocompact. Any pseudocompact space is compact, so Y is compact.
Now if X D !1 C 1 and Y D !1 then X is compact and Y is a countably
compact non-compact space, so Y fails to be a -space. We also have X D Y by
TFS-314 and hence the restriction map  W Cp .X / ! Cp .Y / is a continuous linear
surjection.
V.286. Given -spaces X and Y , let ' W Cp .X / ! Cp .Y / be a continuous linear
surjection. Prove that, if X is  -compact then Y is also  -compact. Observe that
the same conclusion about Y may be false if Y is not a -space.
S
Solution. It follows from  -compactness of X that X D n2! Xn where Xn is
compact and Xn  XnC1 for any n 2 !. For any point y 2 Y there exists a uniquely
determined nP2 N together with x1 ; : : : ; xn 2 X and 1 ; : : : ; n 2 Rnf0g such that
'.f /.y/ D niD1 i f .xi / for any f 2 Cp .X /; let supp.'; y/ D fx1 ; : : : ; xn g.
The set Yn D fy 2 Y W supp.'; y/  Xn g is bounded in Y for any n 2 !
(see Problem 282). If we let .y/ D supp.'; y/ for each y 2 Y then the map  W
Y ! exp.X / is lower semicontinuous by Problem 280. An immediate consequence
is that the set Yn is closed in Y for every n 2 !; since Y is a -space, every Yn is
compact.
S Given any y 2 Y it is clear that supp.'; y/  Xn for some n 2 !, so
Y D n2! Yn is  -compact.
Now if X D !1 C 1 and Y D !1 then X is compact and Y is a countably
compact non-compact space, so Y fails to be  -compact. We also have X D Y by
TFS-314 and hence the restriction map  W Cp .X / ! Cp .Y / is a continuous linear
surjection.

2 Solutions of problems 001500

317

V.287. For any space X let K.X / be the family of all compact subspaces of X .

Prove that a second countable space X is Cech-complete


if and only if there exists
a Polish space M and a map ' W K.M / ! K.X / such that, for any F; G 2 K.M /
the inclusion F  G implies '.F /  '.G/ and, for any P 2 K.X /, there exists
F 2 K.M / such that '.F /  P .
Solution. Let P be the space ! ! of the irrational numbers; for any s; t 2 P we
write s  t if s.i /  t .i / for all i 2 !. If K 2 K.P/ and i 2 ! then the set
fs.i / W s 2 Kg is finite being a discrete continuous image of K, so the number
sK .i / D maxfs.i / W s 2 Kg is well defined; this gives a point sK 2 P for every
K 2 K.P/. It is immediate that K  L implies sK  sL for all K; L 2 K.P/. For
any s 2 P we willQneed the set Ks D ft 2 P W t  sg which is compact being
homeomorphic to i2! f0; : : : ; s.i /g; it is clear that sKs D s and s  t implies
Ks  Kt .

Suppose first that a second countable space X is Cech-complete.


By SFFS-365,
there exists a map  W P ! K.X / such that s  t implies .s/  .t/ and 
swallows all compact subsets of X in the sense that, for each P 2 K.X / there
exists s 2 P such that P  .s/. Let '.K/ D .sK / for any K 2 K.P/; this gives
us a map ' W K.P/ ! K.X /. If F; G 2 K.P/ and F  G then sF  sG and
hence '.F / D .sF /  .sG / D '.G/. If P is a compact subset of X then there
exists s 2 P such that .s/  P , so '.Ks / D .s/  P . This shows that for the

Cech-complete
space M D P the map ' W K.M / ! K.X / preserves inclusions
and swallows all compact subsets of X , i.e., we proved necessity.

Now assume that M is a Cech-complete


second countable space and we have
a map ' W K.M / ! K.X / which preserves inclusions and swallows all compact
subsets of X . Apply SFFS-328 to find an open continuous onto map h W P ! M . For
any s 2 P let .s/ D '.h.Ks //. It is clear that  W P ! K.X /; if s  t then Ks  Kt
and hence h.Ks /  h.Kt / which in turn implies that '.h.Ks //  '.h.Kt //, i.e.,
.s/  .t/.
Now if P 2 K.X / then there exists F 2 K.M / for which '.F /  P . The map
h1 D hjh1 .F / W h1 .F / ! F is open by Fact 1 of V.077, so there is a closed
set K  h1 .F / such that h1 .K/ D h.K/ D F and h1 jK D hjK W K ! F is a
perfect map (see SFFS-326). It follows from Fact 2 of S.259 that K is compact; let
s D sK . Then K  Ks , so .s/  '.h.K// D '.F /  P and hence we proved that
the map  W P ! K.X / satisfies all premises of SFFS-365; the relevant conclusion

of SFFS-365 guarantees that X is Cech-complete.


Thus we established sufficiency
and completed our solution.
V.288. Let X and Y be second countable spaces for which there is a continuous

linear surjection of Cp .X / onto Cp .Y /. Prove that, if X is Cech-complete


then Y

is also Cech-complete. In particular, if two second countable spaces X and Y are

l-equivalent then X is Cech-complete


if and only if so is Y .
Solution. Given a space Z let K.Z/ be the family of all compact subsets of Z.
Fix a continuous linear surjection  W Cp .X / ! Cp .Y /. For any y 2 Y we can
find uniquely determined n 2 N, distinct points x1 ; : : : ; xn 2 X and 1 ; : : : ; n 2

318

2 Solutions of problems 001500

P
Rnf0g such that .f /.y/ D niD1 i f .xi / for any f 2 Cp .X /; let supp.; y/ D
fx1 ; : : : ; xn g. Letting .y/ D supp.; y/ for any point y 2 Y we obtain a lower
semicontinuous map  W Y ! K.X / (see Problem 280). Thus, for any compact
subspace F of the space X the set '.F / D fy 2 Y W .y/  F g is closed in Y ;
besides, '.F / is bounded in Y (see Problem 282) and hence compact. It is evident
that the map ' W K.X / ! K.Y / preserves inclusions, i.e., and FS G implies
'.F /  '.G/ for any F; G 2 K.X /. If K 2 K.Y / then the set A D fsupp.; y/ W
y 2 Kg is bounded in X by Problem 281. Therefore F D A is compact and it
follows from the definition of ' that '.F /  K.
We checked that all premises of Problem 287 are fulfilled for the map ', so it

follows from Cech-completeness


of X that Y is also Cech-complete.
V.289. Give an example of second countable l-equivalent spaces X and Y such
that X is pseudocomplete and Y is not Baire. As a consequence, having a dense

Cech-complete
subspace is not an l-invariant property in the class of second
countable spaces.
Solution. The expression P ' Q says that the spaces P and Q are homeomorphic.
Choose a faithful enumeration fqn W n 2 !g of the set Q and consider the set
1
T D .Q  f0g/ [ f.qn ; nC1
/ W n 2 !g  R2 with the topology induced from the
1
/ W n 2 !g is a discrete subspace of T . Indeed,
plane. Observe that D D f.qn ; nC1
1
1
; n1 / is open in R2 and Un \ D D f.qn ; nC1
/g for any n 2 N;
the set Un D R  . nC2
the point x0 D .q0 ; 1/ is also isolated in T because the set U0 D R  . 21 ; 2/ is open
in R2 and U0 \ D D fx0 g.
Take any point x D .q; 0/ 2 Q  f0g and U 2 .x; T /; there exists " > 0 such
that V \T  U where V D .q"; qC"/0; "/. The set A D Q\.q"; qC"/ being
1
1
infinite we can find n 2 ! for which qn 2 A and nC1
< ". The point y D .qn ; nC1
/
belongs to D \ U ; this proves that D \ U ; for any U 2 .x; T / and hence
x 2 D. The point x 2 Q  f0g was chosen arbitrarily, so we proved that D is dense

in T and, in particular, T has a dense Cech-complete


subspace.
For any z D .x; y/ 2 T let r.z/ D .x; 0/; we omit a simple proof that r is a
retraction of T onto the set F D Q  f0g. Let
pTFpbe the R-quotient space under the
map which contracts F to a point. If a D . 2; 2/ 2 R2 then the space TF F
is l-equivalent to the space X D T fag (see Problem 258). It is evident that

w.X /  ! and X has a dense Cech-complete


subspace, so it is pseudocomplete by
TFS-465 and TFS-467.
If Y D X Q then w.Y /  ! and we can apply Problem 265 to see that the
space Y is l-equivalent to Z D TF F Q. The space F being homeomorphic
to Q, we have F Q ' F by SFFS-349, so Z ' TF F which shows that
l

Y  Z ' TF F  X and hence Y is l-equivalent to X . The space Y is not Baire


because it has a clopen subspace homeomorphic to Q; it follows from TFS-274 and

TFS-275 that Y does not have a dense Cech-complete


subspace.

2 Solutions of problems 001500

319

V.290. Prove that there exist l-equivalent  -compact second countable spaces X
and Y such that X can be condensed onto a compact space and Y doesnt have
such a condensation.
Solution. For any space Z denote by O.Z/ the family of all clopen subsets of Z;
if O.Z/ D f;; Zg then Z is called connected. Call a subset UT of the space Z
nontrivial if U ; and U Z. For each point z 2 Z let Qz D fU W z 2 U and
U 2 O.Z/g; the
Sset Qz is called the quasi-component of the point z in the space Z.
The set Kz D fC W z 2 C  Z and C is connectedg is called the component of
the point z in Z.
Fact 1. Suppose that Z is a space and, for any points x; y 2 Z there exists a
connected subspace C  Z such that fx; yg  C . Then Z is connected.
Proof. Suppose that U 2 O.Z/ and U ;; fix a point x 2 U . If U Z then
choose a point y 2 ZnU and a connected subspace C  Z with fx; yg  C . It
follows from x 2 U and y U that U 0 D U \ C is a clopen subspace of C such
that U ; and U C which is a contradiction. Therefore U D Z and hence
O.Z/ D f;; Zg, i.e., Z is connected which shows that Fact 1 is proved.
Fact 2. Suppose that Z is T
a space and C D fCt W t 2 T g is
Sa family of connected
subspaces of Z such that C ;. Then the set C D
C is connected. As a
consequence, the component of any point of Z is a closed connected subspace of Z.
T
Proof. Fix a point x 2
C and suppose that U is a nontrivial clopen subset of
C . We can assume, without loss of generality, that x 2 U for otherwise C nU is a
nontrivial clopen subset of C which contains x. Take a point y 2 C nU and t 2 T
with y 2 Ct . Then x 2 U 0 D Ct \ U and y U 0 , so U 0 is a nontrivial clopen
subspace of a connected space Ct . This contradiction shows that C has no nontrivial
clopen subsets, i.e., C is connected.
An immediate consequence is that the component Kz of any z 2 Z is connected.
It follows from Fact 1 of T.312 that K z is also a connected set; since z 2 K z , we
must have K z  Kz , so Kz is closed in Z and hence Fact 2 is proved.
Fact 3. Given any space Z and z 2 Z we have Kz  Qz , i.e., the component of any
point is contained in its quasi-component. If Z is compact then Kz D Qz .
Proof. Take any U 2 O.Z/ such that z 2 U . The set U \ Kz is clopen in Kz and
nonempty; since Kz is connected by Fact 2, the set U \ Kz must coincide with Kz
and hence Kz  U . Therefore Kz  U for any U 2 O.Z/ such that z 2 U , i.e.,
Kz  Qz .
Now assume that Z is compact; since Kz is the maximal connected set which
contains z, to prove that Qz  Kz , it suffices to show that Qz is connected. Assume
toward a contradiction that G is a nontrivial clopen subset of Qz ; then H D Qz nG
is also a nontrivial clopen subset of Qz , so G and H are nonempty closed disjoint
subsets of Z. Assume, without loss of generality, that z 2 G. By normality of the

320

2 Solutions of problems 001500

space Z we can find disjoint sets U; V 2 .Z/ such that G  U and H  V . Since
Qz D G [ H  U [ V , we can apply Fact
T 1 of S.326 to find a finite family V of
clopen subsets of Z such that Qz  W D V  U [ V .
It is evident that W is a clopen subset of Z, so W \ U is open in Z. Besides,
U \ W and V \ W are open complementary subsets of W , so they are both closed
in W and hence in Z. This shows that W 0 D W \ U is a clopen subset of Z such
that z 2 W 0 and ; H  Qz nW which contradicts the fact that Qz is contained in
every clopen set which contains z. As a consequence, Qz is connected, so Qz D Kz ,
i.e., Fact 3 is proved.
Fact 4. Suppose that Z is a connected compact space and F is a nonempty closed
subset of Z such that F Z. Then, for any z 2 F , the component K of the point z
in the space F intersects the boundary B D F nInt.F / of the set F in Z.
Proof. If this is false then K  O D Int.F /. By Fact 3, the set K is also the quasicomponent of z in F , so we can apply Fact
T 1 of S.326 to find a finite family V of
clopen subsets of F such that K  W D V  O. It is evident that W is a clopen
subset of F , so W is closed in Z. Besides, W is open in F and hence in O  F .
Since O is open in Z, the set W is also open in Z. It follows from W  F Z
that W Z; we also have z 2 W , so W ; and hence W is a nontrivial clopen
subset of Z. The space Z being connected, this is a contradiction which shows that
Fact 4 is proved.
Fact 5. Suppose that Z is a connected compact space S
and A D fZn W n 2 !g is a
disjoint family of closed subsets of Z such that Z D n2! Zn . Then at most one
element of A is nonempty.
Proof. Say that a cover F of a space E is adequate if it is disjoint, every F 2 F is
closed in E and at least two elements of F are nonempty. We will first prove that,
for any connected compact space E, if F is an adequate cover of E then,
(1) for any H 2 F there exists a connected compact C  E such that C \ H D ;
and fF \ C W F 2 Fg is an adequate cover of C .
If H D ; then we can take C D E, so assume that H ;. The cover F being
adequate there is G 2 F such that G ; and G \ H D ;. By normality of E we
can find a set U 2 .G; E/ such that U \ H D ;; fix a point z 2 G. The component
C of the point z in the space U is connected and compact by Fact 2. The set C meets
the set U nU (see Fact 4); it follows from G  U that C nG ; and hence there is
G 0 2 F distinct from G such that G 0 \ C ;, i.e., the family fF \ C W F 2 Fg is
adequate. Furthermore, C  U and hence C \ H D ;, so (1) is proved.
Apply (1) to construct by induction a sequence fCi W i 2 !g of compact
connected subsets of Z such that CiC1  Ci ; Ci \ Zi D ; while fZn \ Ci W n 2 !g
is an adequate
family and hence Ci ; for all i 2 !. ByScompactness of Z, the set
T
C D i2! Ci is nonempty and no point of C belongs to i2! Zi ; this contradiction
shows that Fact 5 is proved.
1
Returning to our solution consider the sequence S D f nC1
W n 2 !g  R which
converges to zero and let u0 D .0; 0/ be the zero point of the plane. We will also

2 Solutions of problems 001500

321

need the point u1 D .0; 1/ and the subsets P0 D f0g  1; 2/ and P1 D f1g  1; 2/
of the plane R2 . Denote the interval 0; 1
 R by I and consider the subspace
Z D fu0 g [ P0 [ P1 [ .S  I / of the plane. For any point z D .x; y/ 2 P0 [ P1
let r.z/ D .0; y/; if w 2 Zn.P0 [ P1 / then r.w/ D u1 . It is trivial to check that
r W Z ! P0 is a retraction such that rjP1 W P1 ! P0 is a homeomorphism, so P0
and P1 are parallel retracts of Z by Problem 256.
Denote by X the R-quotient space obtained from Z by contracting the set P1 to
a point p1 and let Y be the R-quotient space obtained from Z by contracting P0 to
a point p0 . The spaces X and Y are l-equivalent by Problem 257.
A set U  X is an open neighborhood of p1 in X if and only if .U nfp1 g/ [ P1
is an open neighborhood of P1 in Z (see Problem 254). It is easy to see that this
happens if and only if .U nfp1 g/ [ f.1; 1/g is an open neighborhood of the point
.1; 1/ in the space .ZnP1 / [ f.1; 1/g. In other words, the space
.ZnP1 / [ f.1; 1/g D fu0 g [ P0 [ .S  I /
is homeomorphic to X and, in particular, w.X /  !. From now on we will identify
X with the subspace fu0 g [ P0 [ .S  I / of the plane R2 .
An analogous reasoning shows that the space Y is homeomorphic to the set
fu0 g [ fu1 g [ P1 [ .S  I / with the topology induced from the plane, so we will
identify Y with this set from now on. It follows from Y  R2 that w.Y /  !.
The space K D .S [ f0g/  I is compact and, intuitively, the space X condenses
onto K by reflecting the interval P0 symmetrically with respect to the point u1 . To
define this condensation rigorously let h.x/ D x for any x 2 Q D fu0 g [ fu1 g [
.S  I /. If x D .0; t / where t > 1 then let h.x/ D .0; 2  t /. It is evident that
h W X ! K is a bijection such that hjQ and hjP0 is continuous; applying Fact 2 of
T.354 we conclude that h is continuous and hence X condenses onto the compact
space K.
Assume toward a contradiction that Y can be condensed onto a compact space
L. We can consider that L and Y have the same underlying set and the topology of
L is contained in .Y /. The set In D f n1 g  I being compact in Y the topologies
induced on In from L and Y coincide for all n 2 N.
Suppose that U 2 O.Y / and u0 2 U . The set U is also clopen in Y , so there is
m 2 ! such that . n1 ; 0/ 2 U and hence U \ In ; for all n  m. The set In is
connected and U \ In is a nonempty clopen subset
S of In , so In  U for all n  m.
The point u1 belongs to the closure of the set fIn W n  mg  U in Y and hence
in L. Since U is closed in L, we conclude that u1 2 U which shows that u1 belongs
to the quasi-component of the point u0 in the space L.
By Fact 2 and Fact 3 the component C of the point u0 in L is a connected compact
space with fu0 ; u1 g  C . Fix U0 ; U1 2 .C / such that u0 2 U0 ; u1 2 U1 while
U 0 \ U 1 D ; and w0 D .1; 0/ U i for each i D 0; 1. The sets U 0 ; U 1 being
disjoint there exists j 2 f0; 1g such that U j does not contain the set f1g  .2  "; 2/
for any number " 2 .0; 1/. Therefore there exists a strictly increasing sequence
ftn W n 2 Ng  .1; 2/ such that supftn W n 2 !g D 2 and wn D .1; tn / U j for
all n 2 N; let t0 D 0. The set Jn D f1g  tn ; tnC1
is compact both in Y and L for

322

2 Solutions of problems 001500

each n 2 !. If Wn D f1g  .tn ; tnC1 / for all n 2 ! then the family fWn W n 2 !g is
disjoint; it follows from fwi W i 2 !g \ U j D ; that En D Jn \ U j D Wn \ U j is
a compact set in L for all n 2 ! and the family E D fEn W n 2 !g is disjoint.
Apply Fact 4 to see that the component D of the point uj in the set U j is a
compact set which meets the boundary of U j ; since the point uj is in the interior of
U j , there is a point z 2 Dnfuj g. It is easy to see that the family H D fEn \ D W n 2
!g [ fIn \ D W n 2 Ng [ fuj g is a countable disjoint cover of D and all elements
of H are compact. If H 2 H and z 2 H then H and fuj g are nonempty distinct
elements of H; this contradiction with Fact 5 shows that Y cannot be condensed
onto a compact space.
It is straightforward that Z is  -compact, so both spaces X and Y are also  compact being continuous images of Z. Thus X and Y are l-equivalent second
countable  -compact spaces such that X condenses onto a compact space and Y
does not have such a condensation, i.e., our solution is complete.
V.291. Prove that a countable second countable space is scattered if and only if

it is Cech-complete.
Deduce from this fact that if X and Y are countable second
countable l-equivalent spaces then X is scattered if and only if Y is scattered.
Solution. Given a space Z denote by I.Z/ the set of isolated points of Z. Say that

a space Z is locally Cech-complete


if, for any z 2 Z there exists a Cech-complete
set V 2 .z; Z/.

Fact 1. Any paracompact locally Cech-complete


space is Cech-complete.

Proof. Suppose that Z is a paracompact locally Cech-complete


space and choose

a Cech-complete
set Oz 2 .z; Z/ for any z 2 Z. Fix a homeomorphic copy Wz
of every space
L Oz in such a way that the family fWz W z 2 Zg is disjoint; the

space W D fWz W z 2 Zg is Cech-complete


by TFS-262. Fix a homeomorphism
'z W Wz ! Oz for each z 2 Z; then the union ' W W ! Z of the family of maps
f'z W z 2 Zg is a continuous map by Fact 2 of T.354. Given any w 2 W there is
z 2 Z such that w 2 Wz ; fix a local base B of the space W at the point w such that
S
B  Wz . The map 'z being a homeomorphism, the family f'z .B/ W B 2 Bg is a
local base of Z at the point '.w/. Since '.B/ D 'z .B/ for any B 2 B, the family
f'.B/ W B 2 Bg is also a local base of Z at the point '.w/, so we can apply Fact 2
of S.491 to see that ' W W ! Z is an open continuous onto map.

Fact 1 of S.491 shows that there exists a closed (and hence Cech-complete)
subspace F  W such that '.F / D Z and '0 D 'jF W F ! Z is a perfect

map. It follows from TFS-261 that Z is Cech-complete,


so Fact 1 is proved.
Fact 2. Given a paracompact space Z suppose that D is a closed discrete subset

of Z and .d; Z/  ! for each d 2 D. If ZnD is Cech-complete


then Z is also

Cech-complete.

Proof. The Cech-complete


open set U D ZnD witnesses that Z is locally Cechcomplete at any point z 2 U . Fix an arbitrary point d 2 D; it is immediate that
U [ fd g is an open neighborhood of d . If d is not in the closure of U then U [ fd g

2 Solutions of problems 001500

323

is Cech-complete
being homeomorphic to U fd g. Now if d is not an isolated
point of U [ fd g then K D .U [ fd g/ is a compact extension of U and therefore
U is a G -subset of K. It follows from .d; fd g [ U /  ! that .d; K/  ! (see
Fact 1 of S.158); this implies that fd g is a G -subset of K. It is easy to see that the

union of two G -sets is a G -set, so U [ fd g is Cech-complete


being a G -subset

of K. Therefore every point of Z has an open Cech-complete


neighborhood, i.e.,

Z is locally Cech-complete.
By Fact 1, the space Z is Cech-complete,
so Fact 2 is
proved.

Returning to our solution suppose that X is a countable Cech-complete


space.
If X is not scattered then there exists a subspace A  X which has no isolated

points. The set B D A is also dense-in-itself; besides, B is Cech-complete


being
closed in X . However, the S
countable family B D ffbg W b 2 Bg consists of nowhere
dense subspaces of B and B D B, i.e., B does not have the Baire property which
is a contradiction with TFS-274. Therefore X is scattered and hence we proved
sufficiency.
Now, if X is a scattered countable space then consider the set F0 D X nI.X /; it is
clear that F0 is closed in X . If < !1 and we have a closed subset F  X then let
FC1 D F nI.F /. If < !1 is a limit ordinal
and we have a family fF W < g
T
of closed subsets of X then the set F D fF W < g is also closed in X .
This inductive procedure gives us a decreasing family fF W < !1 g of closed
subsets of X . Observe that every F is scattered, so F ; implies that FC1 is
strictly smaller than F . Since the space X is countable, there exists < !1 such
that F D ;. The set X D X nF is open in X for any  and X D X , so it

suffices to prove by induction that X is Cech-complete


for every  .

Observe first that X0 is Cech-complete being a discrete space. If < and we

proved that X is Cech-complete


then XC1 D X [E where E is the set of isolated
points of F . Therefore E is closed and discrete in XC1 while XC1 nE D X is

Cech-complete.
Applying Fact 2 we conclude that XC1 is Cech-complete.

If  Sis a limit ordinal and the space X is Cech-complete for all  <
then X D fX W  < g and the family fX W  < g is an open cover of X

which witnesses that X is locally Cech-complete.


Therefore X is Cech-complete

by Fact 1 and our inductive procedure shows that X is Cech-complete for every

< !1 . In particular, X D X is Cech-complete


so we proved necessity.
Finally, assume that X and Y are countable metrizable l-equivalent spaces. If X

is scattered then it is Cech-complete,


so Y is also Cech-complete
(see Problem 288)
and hence scattered. This proves that being scattered is l-invariant in the class of
countable metrizable spaces and completes our solution.
V.292. Let X and Y be metrizable spaces such that Cp .X / is linearly homeomorphic to Cp .X /  Cp .X / and Cp .Y / is linearly homeomorphic to Cp .Y /  Cp .Y /.
Prove that if X embeds in Y as a closed subspace and Y embeds in X as a closed
subspace then X and Y are l-equivalent.
Solution. The expression L
M says that the linear topological spaces L and
M are linearly homeomorphic. Suppose first that F is a closed subspace of X

324

2 Solutions of problems 001500

homeomorphic to Y . Since F is l-embedded in X (see CFS-469), we can apply


CFS-448 to see that Cp .X /
Cp .F /  I where I D ff 2 Cp .X / W f .F /  f0gg.
The set F is a homeomorphic copy of the space Y , so we have Cp .F /
Cp .Y /
and hence Cp .X /
Cp .Y /  I ; recalling that Cp .Y / is linearly homeomorphic to
its square we conclude that Cp .X /
Cp .Y /  .Cp .Y /  I /
Cp .Y /  Cp .X /.
Now assume that G is a closed subspace of Y homeomorphic to X . Since the set
G is l-embedded in Y by CFS-469, we have Cp .Y /
Cp .G/  J (see CFS-448),
where J D fg 2 Cp .Y / W g.G/  f0gg. The set G is a homeomorphic copy of
the space X , so Cp .G/
Cp .X / and hence Cp .Y /
Cp .X /  J ; recalling that
the space Cp .X / is linearly homeomorphic to Cp .X /  Cp .X / we conclude that
Cp .Y /
Cp .X /  .Cp .X /  J /
Cp .X /  Cp .Y /. An immediate consequence is
that Cp .X /
Cp .X /  Cp .Y /
Cp .Y /, so the spaces X and Y are l-equivalent.
V.293. Let X be a countable second countable space. Prove that the following
properties are equivalent:
(i)
(ii)
(iii)
(iv)

X
X
X
X

is l-equivalent to Q;
is not scattered;
has a subspace homeomorphic to Q;
has a closed subspace homeomorphic to Q.

Solution. The expression L


M says that the linear topological spaces L and M
are linearly homeomorphic. Given spaces Y and Z say that Y ' Z if Y and Z are
homeomorphic; we will write Y  Z if Y embeds in Z as a closed subspace.
l

If X  Q then X is not scattered because Q is not scattered (see Problem 291);


this proves that (i)H)(ii). If X is not scattered then some Y  X is dense-in-itself;
it follows from SFFS-349 that the space Y is homeomorphic to Q, so (ii)H)(iii).
Next, assume that some Y  X is homeomorphic to Q and hence Y has no isolated
points. Then Z D Y has no isolated points either and therefore Z  X is a closed
homeomorphic copy of Q, so we established that (iii)H)(iv).
Now suppose that Q  X ; apply SFFS-349 again to see that Q is homeomorphic
to Q  !, so we can choose a discrete family fFi W i 2 !g in the space X such that
Fi is closed in X and Fi ' Q for every i 2 !. Apply SFFS-350 to choose a closed
set Gi  Fi such
S that Gi ' X for any i 2 !.
If G D
i2! Gi then G is l-embedded in X (see CFS-469), so Cp .X /

Cp .G/  I where I is a linear subspace of Cp .X / (seeLCFS-448). Every Gi is


a homeomorphic copy of X , so it follows from G '
i2! Gi that Cp .G/

.Cp .X //! . Consequently, Cp .X /


.Cp .X //!  I
.Cp .X //!  ..Cp .X //! 
I /
.Cp .X //!  Cp .X / which shows that Cp .X /
.Cp .X //!
.Cp .X //! 
.Cp .X //!
.Cp .X //2 , i.e., the space Cp .X / is linearly homeomorphic to its
square.
Apply SFFS-349 once more to see that Q ' Q Q, so Cp .Q/
.Cp .Q//2 .
We have already observed that X  Q by SFFS-350, so we can apply Problem 292
l

to conclude that X  Q. This settles (iv)H)(i) and makes our solution complete.

2 Solutions of problems 001500

325

V.294. Prove that, for any infinite cardinal  there exist l-equivalent spaces X and
Y such that X is dense-in-itself and Y has a dense set of -many isolated points.
Solution. Choose a faithful enumeration fqn W n 2 !g of the set Q and consider the
subspace E D .Q  f0g/ [ f.qn ; n1 / W n 2 !g of the plane R2 . It is easy to see that
D D f.qn ; n1 / W n 2 !g is a dense set of isolated points of E. Since Q embeds in E,
l

we can apply Problem 293 to see that E  Q. Let E D E; D D D and Q D Q


L
L
l
for all < . If Y D fE W < g and X D fQ W < gL
then X  Y by
Problem 265 while X is dense-in-itself and Y has a dense set Q D fD W < g
of -many isolated points.

V.295. Suppose that a second countable space X is Cech-complete


and has a
closed subspace homeomorphic to R! . Prove that X is l-equivalent to R! .
Solution. The expression Y ' Z says that the spaces Y and Z are homeomorphic.
We denote by L
M the fact that the linear topological spaces L and M are
linearly homeomorphic.
The family fR!  fng W n 2 !g is discrete in R!  R ' R! , so we can
!
find a discrete family F D fFn W n 2 !g of closed
S subsets of R such that
!
!
Fn ' R for every n 2 !. Since the set F D
n2! Fn is closed in R ,
!
we can apply CFS-469 and CFS-448 to see that Cp .R /
Cp .F /  I where
I D ff 2 Cp .R! / WL
f .F / D f0gg.
Recall that F ' n2! Fn where every space Fn is a homeomorphic copy of R! .
Therefore
Cp .R! /
.Cp .R! //!  I
.Cp .R! //!  .Cp .R! //!  I I
replacing in the last product the expression .Cp .R! //!  I with Cp .R! / we
conclude that Cp .R! /
.Cp .R! //!  Cp .R! /, i.e., Cp .R! /
.Cp .R! //! . As
an immediate consequence we obtain the formula
.Cp .R! //2
.Cp .R! //!  .Cp .R! //!
.Cp .R! //!
Cp .R! /:
Since R! embeds in X as a closed subspace, taking the respective copies of
the elements of the family F we obtain a discrete family fGn W n 2 !g of closed
subspaces of X such that every Gn is a homeomorphic copy of R! . It follows from
TFS-273 that X embeds in R! as a closed subspace, so we canSfind a closed set
Hn  Gn such that Hn ' X for each n 2 !. Since the set H D n2! Hn is closed
in X , we can apply CFS-469 and CFS-448 again to see that Cp .X /
Cp .H /  J
W f .H /  f0gg.
where J D ff 2 Cp .X /L
Q
!
Furthermore, H D
n2! Hn , so Cp .H /

n2! Cp .Hn /
.Cp .X // . It
!
!
!
follows from Cp .X /
.Cp .X //  J
.Cp .X //  ..Cp .X //  J / that the
space Cp .X / is linearly homeomorphic to .Cp .X //!  Cp .X /
.Cp .X //! . As a
consequence, Cp .X /  Cp .X /
.Cp .X //!  .Cp .X //!
.Cp .X //!
Cp .X /
which shows that the spaces X and R! satisfy all assumptions of Problem 292 and
l

therefore X  R! .

326

2 Solutions of problems 001500

V.296. Let K be an uncountable metrizable compact space. Prove that K is lequivalent to K E for any metrizable zero-dimensional compact space E.
Solution. The expression X ' Y says that the spaces X and Y are homeomorphic.
We denote by L
M the fact that the linear topological spaces L and M are
linearly homeomorphic.
Fact 1. For any infinite compact space X let .X / be the one-point compactification of the space X  !. Then
(1) w.X / D w..X //; in particular, if X is second countable then .X / is also
second countable;
(2) if X is zero-dimensional then .X / is also zero-dimensional;
(3) .X / X ' .X /.
Proof. Let  D nw.X /; since X embeds in .X /, we have nw..X //  . Let
a 2 .X / be the point which compactifies X  !, i.e., a X  !. Since network
weight is countably
additive and nw.X  fng/ D  for each n 2 !, it follows from
S
.X / D fX  fng W n 2 !g [ fag that nw..X //   and hence we have the
equality nw..X // D nw.X /. Applying Fact 4 of S.307 we convince ourselves
that w..X // D nw..X // D nw.X / D w.X /, so (1) is proved.
Assume that
S the space X is zero-dimensional and let Fn D X  fng; the set
Un D fag [ . fFi W i  ng/ is an open neighborhood of a for every n 2 !. By the
definition of the topology of .X / at the point a, the family U D fUn W n 2 !g is
a local base at a in the space .X /; it is evident that every element of U is clopen
in .X /. Given any n 2 ! and x 2 Fn choose a family Ux of clopen subsets of Fn
which forms a local base at x in the space Fn . Since the set Fn is clopen in .X /,
the family Ux is also a local clopen base of .X / at the point x. This shows that
.X / is zero-dimensional, i.e., (2) is proved.
We can consider that .X /X D .X /[G where G is a homeomorphic copy
of X while the sets .X / and G are disjoint and clopen in .X / X . Since every
Fn is a homeomorphic copy of X , we can fix a homeomorphism qn W Fn ! FnC1 for
any n 2 !. Fix a homeomorphism q W G ! F0 and define a map ' W .X / X !
.X / by the equalities '.a/ D a; 'jG D q and 'jFn D qn for all n 2 !. It is clear
that ' is a bijection which is continuous at all points of .X  !/ [ G. Observing that
'.Un /  Un for all n 2 !, we verify continuity of ' at the point a. This shows that
' is a condensation and hence homeomorphism (see TFS-123), so we established
(3) and finished the proof of Fact 1.
Returning to our solution observe first that there is a closed F  K which is
homeomorphic to the Cantor set K (see SFFS-353). The space H D .E/ is second
countable and zero-dimensional by Fact 1, so we can apply Fact 2 of U.003 to find
a closed set D  F with D ' .E/. It follows from CFS-469 and CFS-448 that
there is a linear topological space I such that Cp .K/
Cp .D/  I . Fact 1 implies
that D E ' D, so Cp .D/
Cp .D/  Cp .E/ and hence the relevant substitution
of Cp .D/ gives us the formula Cp .K/
Cp .E/.Cp .D/I /
Cp .E/Cp .K/.
l

Since Cp .K E/
Cp .K/  Cp .E/, we conclude that K  K E.

2 Solutions of problems 001500

327

V.297. Prove that a compact space X is l-equivalent to the Cantor set K if and
only if X is metrizable, zero-dimensional and uncountable. As a consequence, any
two zero-dimensional metrizable uncountable compact spaces are l-equivalent.
Solution. The expression X ' Y says that the spaces X and Y are homeomorphic.
We denote by L
M the fact that the linear topological spaces L and M are
linearly homeomorphic.
l

If X  K then w.X / D nw.X / D nw.K/ D w.K/ D ! (see Problem 001 and


Fact 4 of S.307), so X is second countable and hence metrizable. It follows from
Problem 180 that dim X D dim K D 0, so X is zero-dimensional. Since cardinality
is even t -invariant, we have jX j D jKj > !, i.e., X is uncountable; this proves
necessity.
Now assume that X is a metrizable zero-dimensional uncountable compact
space. By SFFS-353, the space K embeds in X ; it follows from SFFS-303 that X
l

also embeds in K. Next apply Problem 296 to see that X X  X and K K  K


which is the same as saying that .Cp .X //2
Cp .X / and .Cp .K//2
Cp .K/.
Therefore we can apply Problem 292 to conclude that the spaces X and K are lequivalent; this proves sufficiency and completes our solution.
V.298. Prove that a second countable space X is l-equivalent to space P of the

irrational numbers if and only if X is non- -compact, zero-dimensional and Cechcomplete.


Solution. The expression X ' Y says that the spaces X and Y are homeomorphic.
We denote by L
M the fact that the linear topological spaces L and M are
linearly homeomorphic.

Fact 1. A second countable zero-dimensional space is Cech-complete


if and only
if it embeds in P as a closed subspace.

Proof. If F is a closed subspace of the space P then it is Cech-complete


because so
is P (see TFS-260); this proves sufficiency. Now, if F is a second countable zerodimensional space then it embeds in D!  ! ! D P (see SFFS-303), so we can

consider that F  P. If F is, additionally, Cech-complete


then it isTa G -subset of
P, so fix a family fUn W n 2 !g of open subsets of P such that F D n2! Un .
It is clear that every Un is Polish and zero-dimensional. If K is a compact subset
of Un and the interior of K in Un is nonempty then the interior of F in P is nonempty
as well which is a contradiction with SFFS-347; therefore we can apply SFFS-347
to the space Un to convince ourselves
that Un ' P for any n 2 !. By Fact 7 of
Q
S.271, the space F embeds in n2! Un ' P! as a closed subspace. Finally observe
that P! ' .! ! /! ' ! !! ' ! ! ' P, so F is embeddable in P as a closed
subspace and hence Fact 1 is proved.
l

Returning to our solution assume that X is a second countable space and X  P.

Then the space X is Cech-complete


by Problem 288 and zero-dimensional by Prob-

328

2 Solutions of problems 001500

lem 180. Recall that  -compactness is even t -invariant property (see Problem 043);
since P is not  -compact, the space X cannot be  -compact either so we proved
necessity.

Now assume that X is a second countable zero-dimensional Cech-complete


space which is not  -compact. Fact 1 shows that X embeds in P as a closed
subspace. It follows from SFFS-352 that P also embeds in X as a closed subspace.
Since P  ! is homeomorphic to P, we can construct a discrete family fFn W n 2 !g
of closed subsets of X such that Fn ' P for any n 2 !. Apply Fact 1 once more
to findS
a closed set Gn  Fn such that Gn ' X for each n 2 !. Since the set
G D n2! Gn is closed in X , we can apply CFS-469 and CFS-448 to see that
Cp .X /
Cp .G/  IL
where I D ff 2 Cp .X / W f .G/ D f0gg.
Recall that G '
every space Gn is a homeomorphic copy of
n2! Gn where
Q
X ; this implies that Cp .G/
n2! Cp .Gn /
.Cp .X //! . Therefore Cp .X /

.Cp .X //!  I
.Cp .X //!  .Cp .X //!  I ; replacing in the last product the
expression .Cp .X //!  I with Cp .X / we conclude that Cp .X /
.Cp .X //! 
Cp .X /, i.e., Cp .X /
.Cp .X //! . As an immediate consequence we obtain the
formula .Cp .X //2
.Cp .X //!  .Cp .X //!
.Cp .X //!
Cp .X /. Finally
observe that P P ' P and hence Cp .P/
.Cp .P//2 ; this makes it possible
to apply Problem 292 to the spaces X and P to conclude that they are l-equivalent.
This settles sufficiency and makes our solution complete.
V.299. Given any n 2 N prove that a compact set K  Rn is l-equivalent to
l

In if and only if In embeds in K. Deduce from this fact that K  In if and only if
dim K D n.
Solution. The expression X ' Y says that the spaces X and Y are homeomorphic.
We denote by L
M the fact that the linear topological spaces L and M are
linearly homeomorphic. For any infinite compact space X let .X / be the one-point
compactification
of the space X  !. If we have a point x D .x1 ; : : : ; xn / 2 Rn then
q

jxjn D x12 C : : : C xn2 ; we denote by 0 the zero point of Rn , i.e., 0 D .u1 ; : : : ; un /


where ui D 0 for all i 2 f1; : : : ; ng.

Fact 1. For any nonempty open set U  Rn there is an open set V  U such that
V ' Rn .
Proof. Fix a point a 2 U ; then, for some " > 0, the ball B D fx 2 Rn W jx  ajn <
"g is contained in U . Let '.x/ D 1" .x a/ for any x 2 Rn . It is straightforward that
' W Rn ! Rn is a homeomorphism such that '.B/ D E D fx 2 Rn W jxjn < 1g.
If D p1n then .0; / ' R, so the set G D .0; /n is homeomorphic to Rn . If
q
x D .x1 ; : : : ; xn / 2 G then xi2 < 2 D n1 for each i  n, so jxjn < n n1 D 1
and hence x 2 E. This proves that G  E and therefore G is an open subset of
E homeomorphic to Rn ; consequently, ' 1 .G/  ' 1 .E/ D B  U is an open
subset of U homeomorphic to Rn , so Fact 1 is proved.
Fact 2. If a compact space X embeds in Rn then .X / also embeds in Rn .

2 Solutions of problems 001500

329

Proof. Let "i D 2i and Bi D fx 2 Rn W jxjn < "i g for all i 2 !. It is evident that
Ui D Bi nB iC1 is a nonempty open subset of Rn , so
Swe can use Fact 1 to find a set
Xi  Ui with Xi ' X for every i 2 !. If Y D i2! Xi then Ui \ Y D Xi for
each i 2 !; the family fUi W i 2 !g being disjoint, the collection fXi W i 2 !g is
also disjoint and consists of clopen subsets of Y . Therefore Y is homeomorphic to
L
i2! Xi ' X  !.
If G 2 .0; Rn / then there exists > 0 such that W D fx 2 Rn W jxjn < g  G.
Choose m 2 ! with 2m < ; then Ui  Bi  W  G for all i  m; consequently,
Xi  U for all i  m. This shows that any open set G 3 0 contains all but finitely
many sets Xi ; an easy consequence is that f0g [ Y is a compact space so f0g [ Y is
homeomorphic to .X / and hence Fact 2 is proved.
l

Returning to our solution assume that K  In ; then dim K D dim.In / D n (see


Problems 180 and 159). It follows from Problem 160 that the interior U of the set
K is nonempty, so Rn embeds in U (see Fact 1) and hence In embeds in U  K.
This proves necessity.
Now assume that G  K is a subset homeomorphic to In ; since the interior of
n
I in Rn is nonempty, we can apply Fact 1 again to see that Rn embeds in In and
hence in G. This, together with Fact 2, shows that we can choose a set H  G with
H ' .K/.
It follows from CFS-469 and CFS-448 that there is a linear topological space I
such that Cp .K/
Cp .H /I . Fact 1 of V.296 implies that H K ' H , so Cp .H /
is linearly homeomorphic to Cp .H /  Cp .K/ and hence the relevant substitution of
Cp .H / gives us the formula Cp .K/
Cp .K/  .Cp .H /  I /
Cp .K/  Cp .K/.
l

Note that I  I I by Problem 266, so if n D 1 then In  In In . If n > 1 then


applying Problem 272 we can see that In D IIn1 is l-equivalent to .II/In1 '
In In . As a consequence, .Cp .In //2
Cp .In /, so Problem 292 is applicable to
conclude that the spaces K and In are l-equivalent; this settles sufficiency.
Finally, if In embeds in K then n D dim.In /  dim K (see Problems 155
and 159); it follows from K  Rn that dim K  dim.Rn / D n, so dim K D n.
Now, if dim K D n then the interior U of the set K is nonempty (see Problem 160),
so we can apply Fact 1 to convince ourselves that Rn , and hence In , embeds in K.
l

Therefore the condition dim K D n is equivalent to K  In , so our solution is


complete.
V.300. Prove that a space X is l-equivalent to I! if and only if X is compact,
metrizable and I! embeds in X .
Solution. The expression Y ' Z says that the spaces Y and Z are homeomorphic.
We denote by L
M the fact that the linear topological spaces L and M are
linearly homeomorphic. For any infinite compact space K let .K/ be the onepoint compactification of the space K  !.
Suppose that X is a metrizable compact space such that a closed set G  X is
homeomorphic to I! . The space .X / being compact and metrizable (see Fact 1 of
V.296), there is a set H  G which is homeomorphic to .X /.

330

2 Solutions of problems 001500

It follows from CFS-469 and CFS-448 that there is a linear topological space I
such that Cp .X /
Cp .H /  I . Apply Fact 1 of V.296 again to see that H X
is homeomorphic to H , so Cp .H /
Cp .H /  Cp .X / and hence the relevant
substitution of Cp .H / gives us the formula Cp .X /
Cp .X /  .Cp .H /  I /

Cp .X /  Cp .X /.
l

Note that I  I I by Problem 266, so it follows from Problem 272 that I 


l

I! .I I/  I! which, together with I  I! ' I! and .I I/  I! ' I! I!


l

shows that I!  I! I! . As a consequence, .Cp .I! //2


Cp .I! /, so Problem 292
is applicable to conclude that the spaces X and I! are l-equivalent; this proves
sufficiency.
l

Next assume that X  I! ; then X is compact by Problem 138. Furthermore, we


have nw.X / D nw.I! /  ! (see Problem 001) and hence w.X /  ! by Fact 4 of
S.307 so X is metrizable.
Fix a linear homeomorphism ' W Cp .X / ! Cp .I! / and let '  .a/ D a' for any
a 2 Lp .I! /; it is an easy consequence of Fact 4 of S.489 that '  W Lp .I! / ! Lp .X /
is a linear isomorphism. The set H D '  .I! / is a Hamel basis in the space Lp .X /
such that H ' I! and every f 2 Cp .H / extends to a continuous linear functional
on Lp .X /. Given nonempty sets A1 ; : : : ; An  Lp .X / and O1 ; : : : ; On  R let
O1 A1 C : : : C On An D f 1 x1 C : : : C n xn W i 2 Oi and xi 2 Ai for all i  ng.
For any a 2 H we can find a uniquely determined n 2 N and distinct points
x1 ; : : : ; xn 2 X such that a D 1 x1 C : : : C n xn for some 1 ; : : : ; n 2 Rnf0g;
let X .a/ D fx1 ; : : : ; xn g and lX .a/ D n. Analogously, for any point x 2 X it is
possible to find a uniquely determined m 2 N and distinct points a1 ; : : : ; am 2 H
such that x D 1 a1 C : : : C m am for some 1 ; : : : ; m 2 Rnf0g; let lH .x/ D m
and H .x/ D fa1 ; : : : ; am g. It follows from Problem 279 that X W H ! exp.X /
and H W X ! exp.H / are the support maps for ' and ' 1 respectively. Applying
Problem 280 we infer that both X and H are lower semicontinuous.
The set Xn D fx 2 X W lH .x/  ng is closed in X for any n 2 N by Fact 1 of
U.485; by the same reason every set Hn D fa 2 H W lX .a/  ng is closed in H .
Let Y1 D X1 and Yn D Xn nXn1 for all n > 1; we will also need the sets G1 D H1
and Gn D Hn nH
Sn1 for all n  2.
Since H D fHn W n 2 Ng, an easy consequence of the Baire property of H
is that there exists a nonempty open set U in the space H such that U  Gn for
some n 2 N. Pick a point a 2 U ; then X .a/ D fx1 ; : : : ; xn g and we can choose
a set Wi 2 .xi ; X / for every i  n such that the family fW1 ; : : : ; Wn g is disjoint.
The set U 0 D fb 2 H W X .b/ \ Wi ; for all i  ng is open in H and a 2 U 0 .
Therefore V D U \ U 0 is an open neighborhood of a in H .
Given any b 2 V it follows from lX .b/ D n that for every i 2 f1; : : : ; ng there
is a unique element yi 2 X .b/ \ Wi ; let pi .b/ D yi . This gives us a mapping
pi W V ! Wi for each i  n.
Let O D Rnf0g; if W is a nonempty open subset of Wi then it is easy to see
that pi1 .W / D .OW1 C : : : C OWi1 C OW C OWiC1 C : : : C OWn / \ V ; it
follows from Fact 2 of U.485 that pi1 .W / is an open subset of V , so the map pi is
continuous for any i  n.

2 Solutions of problems 001500

331

Let V0 D V ; we will inductively construct nonempty open sets V0 ; : : : ; Vn of the


space H and k1 ; : : : ; kn 2 N such that
(1) Vi  Vi1 and pi .Vi /  Yki for all i D 1; : : : ; n.
Suppose that j < n and we have V0 ; : : : ; Vj 2  .H / and k1 ; : : : ; kj 2 N such
that the condition (1) is satisfied for all i D 1; : : : ; j . The set pj C1 .Vj / is compact,
S
so it has the Baire property; this, together with pj C1 .Vj /  fXn W n 2 Ng implies
that there is a nonempty open subset E of the space pj C1 .Vj / such that E  Ykj C1
for some kj C1 2 N. The set E 0 D E \pj C1 .Vj / is nonempty and open in pj C1 .Vj /,
0
so Vj C1 D pj1
C1 .E / \ Vj is open in Vj and hence in H . It is straightforward that
(1) now holds for all i  j C 1, so our inductive procedure gives us the promised
sets V0 ; : : : ; Vn and numbers k1 ; : : : ; kn 2 N. As a consequence,
(2) lH .pi .b// D ki for any b 2 Vn and i D 1; : : : ; n.
Fix a point b 2 Vn and i 2 f1; : : : ; ng. Let yi D pi .b/ and consider the set
H .yi / D fa1i ; : : : ; aki i g; choose a disjoint family fU1i ; : : : ; Ukii g   .H / such that
aji 2 Uji for all j  ki . The set Qi D fy 2 X W H .y/ \ Uji ; for all j  ki g
is open in X , so Wi0 D Qi \ Wi is open in X and nonempty because yi 2 Wi0 .
If y 2 Wi0 then, for every j  ki , there is a unique point bj 2 Uji \ H .y/; let
qji .y/ D bj . Then qji W Wi0 ! Uji is a continuous map; this is proved in the same
way as we verified
Tcontinuity of pi .
The set E D fpi1 .Wi0 / W i  ng \ Vn is open in the space H and nonempty
due to the fact that b 2 E. For any i 2 f1; : : : ; ng and j 2 f1; : : : ; ki g the set
Sji D fc 2 E W qji .pi .c// D cg is closed in E. If c 2 E and X .c/ D fz1 ; : : : ; zn g
Pn
every i  n let
then there are 1 ; : : : ; n 2 O for which c D
iD1 i zi . For P
H .zi / D fdji W j  ki g; there are i1 ; : : : ; iki 2 O such that zi D kj iD1 ij dji . The
P
P
equality c D niD1 i . kj iD1 ij dji / shows that c D dji for some i 2 f1; : : : ; ng and
j  ki . By definition of the maps pi and qji we have c D qji .pi .c//, i.e., c 2 Sji .
S
Therefore E  fSji W i  n; j  ki g; an immediate consequence is that we
can choose i  n and j  ki for which there is a set Q 2  .H / with Q  Sji .
Then qji .pi .c// D c for every c 2 Q which shows that the map pi jQ W Q ! pi .Q/
is a homeomorphism because qji jpi .Q/ is its continuous inverse.
We proved that a nonempty open set Q of the space H ' I! embeds in X . It is
an easy exercise that I! embeds in any nonempty open subspace B  I! , so I! also
embeds in X . This settles necessity and makes our solution complete.
V.301. Prove that there exist l-equivalent spaces X and Y such that X is
hereditarily paracompact while Y is not collectionwise normal.
Solution. If Z is a space and A  Z let UA D .Z/ [ ffzg W z 2 ZnAg and denote
by A the topology on Z generated by the family UA as a subbase; denote the space
.Z; A / by ZA
. The space ZA
is always Tychonoff and all points of ZnA are
isolated in ZA
; besides, a 2 A and a 2 U 2 A implies that there is V 2 .a; Z/
with V  U (see Fact 1 of S.293).

332

2 Solutions of problems 001500

Fact 1. Any space with at most one non-isolated point is hereditarily paracompact.
Proof. The property of having at most one non-isolated point is hereditary, so
it suffices to show that any space Z with at most one non-isolated point is
paracompact. Fix a point a 2 Z such that any x 2 Znfag is isolated in Z and take
an open cover U of the space Z. If U 2 U and a 2 U then fU g [ ffxg W x 2 ZnU g
is a disjoint (and hence locally finite) open refinement of U , so Z is paracompact,
i.e., Fact 1 is proved.
Fact 2. If Z is a space and A  Z is l-embedded in Z then the set A is also
l-embedded in ZA
.
Proof. It follow from .Z/  .ZA
/ that Cp .Z/  Cp .ZA
/ and it is immediate
that Cp .Z/ is a linear subspace of Cp .ZA
/. There exists a continuous linear map
 W Cp .A/ ! Cp .Z/ such that .f /jA D f for each f 2 Cp .A/. Therefore  can
be considered to be a map from Cp .A/ to Cp .ZA
/.
Let us consider that A carries the topology induced from Z and denote by A0
the set A with the topology inherited from ZA
. By Fact 1 of S.293, these two
topologies on A coincide, so we can identify Cp .A/ and Cp .A0 /; this implies that
 is also a continuous linear extender of continuous functions on A0 in the space
ZA
. Therefore A is l-embedded in ZA
, i.e., Fact 2 is proved.
Returning to our solution take a discrete space D of cardinality !1 ; then D is
closed and l-embedded in the space Z D Lp .D/ (see CFS-466). Take a point
a Z and let Y D ZD
fag. The set D is still closed and discrete in ZD
;
assume that there is a disjoint family fUd W d 2 Dg of open subsets of ZD
such
that d 2 Ud for every d 2 D. There exists Vd 2 .d; Z/ such that d 2 Vd  Ud
for each d 2 D. Therefore fVd W d 2 Dg is an uncountable disjoint family of
nonempty open subsets of Lp .D/ which contradicts Problem 236. Thus the set D
witnesses that the space ZD
is not collectionwise normal, so the space Y is not
collectionwise normal either.
Let P be the R-quotient space obtained from ZD
by contracting the set D to
a point q. Since D is l-embedded in ZD
by Fact 2, it follows from Problem 258
that Y is l-equivalent to the space X D P D. It is straightforward that q is the
unique non-isolated point of X , so X is hereditarily paracompact by Fact 1.
V.302. Prove that there exist l-equivalent spaces X and Y such that X is
collectionwise normal while Y is not normal.
Solution. Given a space Z and F  Z we denote by ZF the R-quotient space
obtained by contracting the set F to a point which will be referred to as aF .
Fact 1. In the space N D !1  .!1 C 1/ consider the set F D f.; !1 / W < !1 g.
If f 2 Cp .N / and f .F / D f0g then there exists an ordinal  < !1 such that
f.; / W  < < !1 and  <  !1 g  f 1 .0/.
Proof. If such an ordinal does not exist then it is easy to construct an !1 -sequence
fq D . ;  / W < !1 g  !1  !1 such that f .q / 0 and < nnf ;  g for
all < !1 while maxf ;  g < nnf ;  g whenever < < !1 .

2 Solutions of problems 001500

333

There exists " > 0 such that, for an uncountable A  !1 , we have jf .q /j  "
for all 2 A. It follows from TFS-314 that, for any < !1 there is < !1 such
that f .; / D 0 for all > . This makes it possible to construct, by induction on
n 2 !, sequences fn W n 2 !g  A and f.n ; n / W n 2 !g  !1  !1 such that
f .n ; n / D 0 and maxfn ; n g < n < n < nnfnC1 ; nC1 g for all n 2 !.
Let  D supfn W n 2 !g and take any O 2 ..; /; N /; pick an ordinal
 <  such that W D f.; / W  < nnf; g and maxf; g  g  O. There
exists m 2 ! such that m >  and hence .i ; i / 2 W for all i  m; since
also .i ; i / 2 W for all i  m C 1, both sequences S D f.n ; n / W n 2 !g
and T D f.n ; n / W n 2 !g converge to the point u D .; /. It follows from
f .T / D f0g that f .u/ D 0. However, f .S /  Rn."; "/, so jf .u/j  "; this
contradiction shows that the promised ordinal  exists and hence Fact 1 is proved.
Fact 2. The space T D ..!1 C 1/  .!1 C 1//nf.!1 ; !1 /g is not normal.
Proof. The sets F D f.; !1 / W < !1 g and G D f.; / W < !1 g are disjoint
and closed in T . If T is normal then there exists a continuous function f W T ! R
such that f .F / D f0g and f .G/ D f1g. If N D !1  .!1 C 1/ then f jN is
continuous and identically zero on F , so we can apply Fact 1 to find an ordinal
 < !1 such that f .; / D 0 whenever  < nnf; g and maxf; g < !1 . In
particular, f . C 1;  C 1/ D 0 which contradicts . C 1;  C 1/ 2 G  f 1 .1/.
Therefore the space T cannot be normal, i.e., Fact 2 is proved.
Fact 3. For any ordinal  let f .; / D nnf; g for every .; / 2   . Then the
map f W    !  is continuous.
Proof. Fix an arbitrary point .; / 2    and let  D f .; /. If  0 <  then
 0 <  and  0 < , so the set U D f.; / W  0 <   and  0 <  g is an
open neighborhood of .; / in  . If .; / 2 U then  0 < nnf; g  nnf; g,
i.e.,  0 < f .; /   ; this shows that f is continuous at the point .; / for any
.; / 2   , so Fact 3 is proved.
Fact 4. In the space .!1 C 1/  .!1 C 1/ consider the sets N D !1  .!1 C 1/
and F D f.; !1 / W < !1 g. Let G D .!1 C 1/  f!1 g and fix any set P  !1 .
The set F is closed in the space Z D N [ .f!1 g  P / while G is closed in the
space T D Z [ f.!1 ; !1 /g. For the R-quotient spaces ZF and TG define a map
' W ZF ! TG by letting '.aF / D aG and '.x/ D x for every x 2 ZnF . Then ' is
a homeomorphism; in particular, the spaces ZF and TG are homeomorphic.
Proof. It is immediate that ' is a bijection. It follows from Problem 252 that the
map 'j.ZF nfaF g/ W ZF nfaF g ! TG nfaG g is a homeomorphism, so it suffices to
show that ' is continuous at the point aF and ' 1 is continuous at aG .
Suppose that W 2 .aG ; TG /; by Problem 253, there is a cozero set U and a
zero-set E in the space T such that G  E  U and faG g [ .U nG/  W . Then
F  E 0 D E \ Z  U 0 D U \ Z while E 0 is a cozero set and U 0 is a zero-set in
Z, so we can apply Problem 253 again to see that W 0 D faF g [ .U 0 nF / is an open
neighborhood of the point aF in the space ZF . It is clear that '.W 0 /  W , so ' is
continuous at the point aF .

334

2 Solutions of problems 001500

Now assume that W 0 2 .aF ; ZF /; by Problem 253, we can find a cozero set U 0
and a zero-set E 0 in the space Z such that F  E 0  U 0 and faF g [ .U 0 nF /  W 0 .
By Fact 1 of V.140, there is a function f 2 Cp .Z; 0; 1
/ such that f .E 0 / D f0g
and f .ZnU 0 /  f1g. As a consequence, f .F / D f0g, so we can apply Fact 1 to find
an ordinal < !1 such that the set H D f.; / W < < !1 and < < !1 g is
contained in f 1 .0/.
Thus f .!1 ; / D 0 for every ordinal > with 2 P , so the set f 1 .0/
contains H 0 D H [ f.!1 ; / W 2 P and > g. Since H 0 [ f.!1 ; !1 /g is an
open neighborhood of the point v D .!1 ; !1 /, letting g.v/ D 0 and gjZ D f we
obtain a function g W T ! R which is identically zero on a neighborhood of v, so
g 2 Cp .T /. Besides, G  E D g 1 .0/ and E  U D g 1 .0; 1//. Since E is
a zero-set and U is a cozero set in the space T , it follows from Problem 253 that
W D .U nG/ [ faG g is an open neighborhood of aG in the space TG . It is easy to
check that ' 1 .W /  W 0 , so ' 1 is also continuous and hence Fact 4 is proved.
Returning to our solution consider the subspace Z D .!1 C 1/2 nf.!1 ; !1 /g of
the space .!1 C 1/2 . The set F D f.; !1 / W < !1 g is closed in Z; choose a point
a Z and let r.; / D .nnf; g; !1 / for every .; / 2 Z. It follows from Fact 3
that the map r W Z ! F is a retraction, so we can apply Problem 258 to convince
ourselves that Y D Z fag is l-equivalent to the space X D ZF F . Observe that
Y is not normal because Z is a closed subspace of Y and Z not normal by Fact 2.
If G D f.; !1 / W  !1 g then G is a closed subset of the space T D .!1 C 1/2 .
Applying Fact 4 to the spaces Z and T we conclude that ZF is homeomorphic to
TG ; the space T being compact, TG is also compact which shows that ZF is compact
and hence collectionwise normal. The space F is homeomorphic to the ordinal !1 ,
so it is collectionwise normal by Fact 3, Fact 4 and Fact 5 of S.232. Therefore X is
also collectionwise normal.
V.303. Prove that there exist l-equivalent spaces X and Y such that X is
hereditarily normal while Y is not normal.
Solution. For any space Z and F  Z we denote by ZF the R-quotient space
obtained by contracting the set F to a point which will be referred to as aF . Given
sets A; B  Z say that they are separated (in Z) if A \ B D ; D B \ A. Call the
sets A and B open-separated (in Z) if there exist disjoint sets U; V 2 .Z/ such
that A  U and B  V .
If Z is a space and A  Z let UA D .Z/ [ ffzg W z 2 ZnAg and consider the
topology A on Z generated by the family UA as a subbase; denote the space .Z; A /
by ZA
. The space ZA
is always Tychonoff and all points of ZnA are isolated
in ZA
; besides, a 2 A and a 2 U 2 A implies that there is V 2 .a; Z/ with
V  U (see Fact 1 of S.293).
Fact 1. Given a space Z suppose that K is a compact subspace of Z and ZnK is
normal. Then Z is also normal.
Proof. Assume that F and G are closed disjoint subsets of Z. The sets F0 D F \ K
and G0 D G \ K are compact, so they are open-separated in Z by Fact 4 of T.309;

2 Solutions of problems 001500

335

pick disjoint sets U0 ; V0 2 .Z/ such that F0  U0 and G0  V0 . The sets F0 and
GnV0 are also open-separated, so we can find disjoint U1 ; W1 2 .Z/ for which
F0  U1 and GnV0  W1 . If U D U0 \ U1 then F0  U and U \ G D ;.
The set F1 D F nU is closed in Z and disjoint from the set G0 , so we can apply
Fact 4 of T.309 again to find disjoint sets E0 ; H0 2 .Z/ such that F1  E0 and
G0  H0 . Furthermore, F1 and GnH0 are disjoint closed subsets of ZnK, so they
are open-separated in ZnK and hence in Z. Choose disjoint sets E1 ; H1 2 .Z/
for which F1  E1 and GnH0  H1 . It is immediate that E D E0 \ E1 is an
open neighborhood of F1 such that E \ G D ;. Therefore we found open subsets
U and E of the space Z such that F  U [ E and U [ E does not meet G.
An analogous reasoning shows that we can find open sets H and W in the space
Z such that G  H [ W and .H [ W / \ F D ;. It is straightforward that the
sets O D .U [ E/n.H [ W / and Q D .H [ W /n.U [ E/ are disjoint open
neighborhoods of F and G respectively, so Z is normal and hence Fact 1 is proved.
Fact 2. Suppose that Z  A./ is normal for any infinite cardinal . Then Z is
collectionwise normal.
Proof. The space Z embeds in Z  A.!/, so it is normal. Fix a discrete family
F D fFt W t 2 T g of closed subsets of Z; by normality of the space Z we can
assume, without loss of generality, that the set T is infinite. Take a point a T and
consider the topology on the set A D fag [ T such that ft g 2 for every t 2 T
and a 2 U 2 if and only if a 2 U and T nU if finite. It is easy to see that A is
homeomorphic to A./ for  D jT j. Therefore the space Z  A is normal.
TheSset G D Z  fag is, evidently, closed in Z  A and disjoint from the set
F D fFt  ft g W t 2 T g. If w D .z; t / 2 .Z  A/nF for some t 2 T then
.ZnFt /  ft g is an open neighborhood of w that does not meet F . If w D .z; a/ then
there exists O 2 .z; Z/ such that O meets at most one element of F, i.e., there
exists t 2 T such that U \ Fs D ; whenever s t . Then O  .Anft g/ is an open
neighborhood of w with O \ F D ;; this proves that the set F is closed in Z  A.
By normality of Z  A there exist disjoint open subsets U and V of the space
Z  A such that F  U and G  V . For each t 2 T choose a set Vt 2 .Ft ; Z/
such that Vt  ft g  V . Given a point z 2 Z there is a finite K  T and O 2
.z; Z/ such that O  .AnK/  U . If the set O meets infinitely many elements
of the family V D fVt W t 2 T g then there is t 2 T nK with O \ Vt ; and
hence .O  .AnK// \ .Vt  ft g/ ;; this implies U \ V ;, so we obtained a
contradiction.
Therefore V is locally finite; for every t 2 T choose a set Wt 2 .Ft ; Z/ such that
W t  Vt and W t \Fs D
S; for all s 2 T nft g. The family fWt W t 2 T g is also locally
finite, so the set Ht D fW s W s 2 T nft gg is closed in Z and does not meet Ft for
each t 2 T . It is straightforward to check that the family f.ZnHt / \ Wt W t 2 T g is
disjoint and Ft  .ZnHt / \ Wt for each t 2 T . It follows from Fact 1 of S.302 that
the space Z is collectionwise normal, so Fact 2 is proved.
Returning to our solution take a discrete space D of cardinality !1 ; then D is
closed and l-embedded in the space Z D Lp .D/ (see CFS-466). The set D is still

336

2 Solutions of problems 001500

closed and discrete in ZD


; assume that there is a disjoint family fUd W d 2 Dg of
open subsets of ZD
such that d 2 Ud for every d 2 D. There exists Vd 2 .d; Z/
such that d 2 Vd  Ud for each d 2 D. Therefore fVd W d 2 Dg is an uncountable
disjoint family of nonempty open subsets of Lp .D/ which contradicts Problem 236.
Thus the set D witnesses that the space ZD
is not collectionwise normal. By
Fact 2 there exists a space K homeomorphic to A./ for some cardinal  such that
ZD
K is not normal. Denote by a the unique non-isolated point of K and choose
a point b T D ZD
 K. Then the space Y D T fbg is not normal.
Assume that A and B are separated subsets of ZD
and choose a function f W
D ! R such that f .A \ D/  f0g and f .Dn.A \ D//  f1g. Fix a continuous
function g W Lp .D/ ! R with gjD D f ; then g is also continuous on ZD
, so
the sets U 0 D g 1 .. 13 ; 13 // and V 0 D g 1 .. 23 ; 43 // are open in ZD
and disjoint.
Therefore the sets U D U 0 nB and V D V 0 nA are closed and disjoint as well. It
is immediate that A [ U and B [ V are disjoint open neighborhoods of the sets A
and B respectively in the space ZD
. We proved that any two separated subsets of
ZD
are open-separated; thus ZD
is hereditarily normal by Fact 3 of U.193.
In the space T , the set F D Zfag is a retract of T , so we can apply Problem 258
to see that the spaces Y and X D TF F are l-equivalent. The space F is
hereditarily normal being homeomorphic to ZD
. Take an arbitrary subspace E of
the space TF ; it follows from Problem 252 that the space TF nfaF g is homeomorphic
to ZD
 .Knfag/, so it is hereditarily normal because Knfag is a discrete space.
Therefore the space EnfaF g  TF nfaF g is normal and hence we can apply Fact 1
to conclude that E is normal. This proves that TF is hereditarily normal, so X is a
hereditarily normal space which is l-equivalent to a non-normal space Y .
V.304. Prove that -weight is not preserved by l-equivalence neither in the class
of compact spaces nor in the class of countable spaces.
Solution. For an arbitrary space Z and a closed set F  Z we denote by ZF the
R-quotient space obtained by contracting the set F to a point. For any set A  Z
let UA D .Z/ [ ffzg W z 2 ZnAg and consider the topology A on Z generated by
the family UA as a subbase; denote the space .Z; A / by ZA
. The space ZA
is
always Tychonoff and all points of ZnA are isolated in ZA
; besides, a 2 A and
a 2 U 2 A implies that there is V 2 .a; Z/ with V  U (see Fact 1 of S.293).
Fact 1. Suppose that A is a retract of a space Z. Then A is also a retract of the
space ZA
.
Proof. Fix a retraction r W Z ! A. The space ZA
induces the same topology on
the set A, so the map r W ZA
! A is also continuous because the topology of
ZA
is stronger than the topology of Z. Thus r is a retraction of ZA
onto A and
hence Fact 1 is proved.
Fact 2. If A is a retract of a space Z then F D clZ .A/ is a retract of Z.
Proof. Fix a retraction r W Z ! A; the space F being compact, there exists a
continuous map s W Z ! F (see TFS-258) such that sjZ D r. Let idF W F ! F

2 Solutions of problems 001500

337

be the identity map, i.e., idF .x/ D x for all x 2 F . Observe that sjA D rjA D
idF jA; the set A being dense in F , we can apply Fact 0 of S.351 to see that sjF D
idF . This shows that s is a retraction, so Fact 2 is proved.
Returning to our solution recall that the space R!1 is separable by TFS-108; fix
a countable dense subspace A  R!1 . Since R!1 is homeomorphic to Cp .D/ for
a discrete space D with jDj D !1 , we can apply TFS-169 and Fact 1 of T.158
to see that w.R!1 / > !. Next apply Fact 1 of T.187 to convince ourselves that
w.A/ > !.
Take a space S homeomorphic to the convergent sequence ! C 1 and let a be the
unique non-isolated point of S . In the space Z D A  S , the set F D A  fag is a
retract of Z. Therefore the set F is also a retract of the space T D ZF
by Fact 1;
observe that ZnF is a dense set of isolated points of T . Choose a point b T and
let X D T fbg. It is clear that X is a countable space in which the isolated points
form a dense set; an immediate consequence is that w.X / D !.
It follows from Problem 258 that X is l-equivalent to the space Y D TF F .
The set F is homeomorphic to A, so A embeds in Y as a clopen subspace; as a
consequence, w.Y /  w.A/ > !. Therefore the spaces X and Y witness that
-weight is not l-invariant in the class of countable spaces.
The space K D T is compact and it follows from Fact 2 that G D clT .F / is
a retract of K. The space K still has a dense set of isolated points, so w.K/  !.
Take a point c K and let L D K fcg. It is clear that w.L/  !. Apply
Problem 258 again to see that L is l-equivalent to the space M D KG G. Since
F is homeomorphic to a dense subspace of G, we have w.G/ D w.F / > !
(see Fact 1 of T.187). Now, G is homeomorphic to a clopen subspace of M , so
-weight of M has to be uncountable. Therefore, L and M constitute an example
which shows that -weight is not l-invariant in the class of compact spaces.
V.305. Give an example of l-equivalent spaces X and Y with ext .X / ext .Y /.
Solution. For any space Z and F  Z we denote by ZF the R-quotient space
obtained by contracting the set F to a point.
Consider the set P D f.!1 ; C 1/ W < !1 g  .!1 C 1/  .!1 C 1/. It is
evident that P is closed and discrete in the space N D .!1  .!1 C 1// [ P . The
set F D f.; !1 / W < !1 g is closed in N and if we let r.; / D .nnf; g; !1 /
for any .; / 2 N then the map r W N ! F is a retraction (see Fact 3 of V.302).
Choose a point a N and let X D N fag. The set P is still closed and discrete
in X , so ext .X /  jP j > !.
Let w D .!1 ; !1 / and consider the space M D N [ fwg; it is clear that the set
G D .!1 C 1/  f!1 g is compact and hence closed in M . If A  !1  !1 is a
countable set then there is < !1 such that A  . C 1/  . C 1/; an immediate
consequence is that A  . C 1/  . C 1/ is compact, so Q D !1  !1 is countably
compact by Fact 1 of S.310. If U 2 .w; M / then there exists < !1 such that
.!1 ; / 2 U for all > . Therefore the set P nU  f!1 g  . C 1/ is countable.
Consequently, the space P 0 D P [ fwg is Lindelf and hence ext .P 0 / D !.

338

2 Solutions of problems 001500

Suppose that D is a closed discrete subset of M ; then the set D0 D D \ Q


is finite being closed and discrete is a countably compact space Q. The set D1 D
D \ G is finite because G is compact and it follows from ext .P 0 / D ! that the set
D2 D D \ P 0 is countable. This shows that the set D D D0 [ D1 [ D2 is countable
and hence ext .M / D !.
The R-quotient space MG also has countable extent being a continuous image
of M . By Problem 258, the space X is l-equivalent to Y D NF F . The space
F is countably compact being homeomorphic to !1 ; therefore ext .F / D !. It
follows from Fact 4 of V.302 that the spaces NF and MF are homeomorphic, so
ext .NF / D ext .MF / D !. This shows that ext .Y / D ! and hence we obtained
l-equivalent spaces X and Y such that ext .X / ext .Y /.
V.306. Prove that there exist l-equivalent spaces X and Y such that X is Frchet
Urysohn while t .Y / > ! and there is a non-closed set A  Y such that B \ A
is finite whenever B is a bounded subset of Y . As a consequence, FrchetUrysohn
property, k-property, sequentiality and countable tightness are not l-invariant.
Solution. Given a space Z and a closed set F  Z, the symbol ZF stands for the
R-quotient space obtained from Z by contracting the set F to a point which will be
denoted by aF .
Fact 1. For any space Z its diagonal D f.z; z/ W z 2 Zg is a retract of Z  Z.
Proof. Let r.x; y/ D .x; x/ for any .x; y/ 2 Z  Z. It is clear that r W Z  Z !
and r.z/ D z for any z 2 . Letting r1 .x/ D .x; x/ for every x 2 Z we obtain a
continuous map r1 W Z ! (which is, in fact, a homeomorphism); if r0 W Z  Z !
Z is the projection onto the first factor then r D r1 r0 which shows that the map r
is continuous, i.e., r W Z  Z ! is a retraction and hence Fact 1 is proved.
1
Returning to our solution let cn D nC1
for all n 2 !. Then the sequence fcn W
n 2 !g converges to the point c D 0; let C D fcn W n 2 !g [ fcg. Fix a space
D homeomorphic to the one-point compactification A.!1 / of a discrete space of
cardinality !1 and denote by d the unique non-isolated point of D. Consider the
space S D .C  D/nf.c; d /g and let X D .S  S / fag where a is a point which
does not belong to the set T D S  S . It is easy to see that T is homeomorphic to a
subspace of D 4 ; the latter space being Eberlein compact, it is FrchetUrysohn so
T is FrchetUrysohn and hence X is a FrchetUrysohn space as well.
The diagonal F D f.s; s/ W s 2 S g of the space S is a retract of T by Fact 1, so
we can apply Problem 258 to see that the space Y D TF F is l-equivalent to X .
The set E D fcg  .Dnfd g/ is closed and discrete in S , so A D .E  E/nF
is a closed discrete subset of T which does not meet F . Suppose that U is an open
subspace of T with F  U ; then the point ..cn ; d /; .cn ; d // belongs to U for each
n 2 !. Therefore there T
is a set Vn 2 .d; D/ such that .fcn g  Vn /2  U for every
n 2 !. The set V D n2! Vn is uncountable, so we can choose distinct points
d1 ; d2 2 V nfd g. If xi D .c; di / for i D 1; 2 then the point x D .x1 ; x2 / belongs
to A.

2 Solutions of problems 001500

339

Given an arbitrary set W 2 .x; T / we can find Wi 2 .xi ; S / for i D 1; 2 such


that W1 W2  W . It is easy to see that there is m 2 ! for which yi D .cm ; di / 2 Wi
for i D 1; 2; therefore the point y D .y1 ; y2 / belongs to the set W . It follows from
yi 2 fcm g  V  fcm g  Vm for every i 2 f1; 2g that y 2 .fcm g  Vm /2  U ; an
immediate consequence is that y 2 W \ U , so every neighborhood of the point x
in T meets the set U . In other words, x 2 U ; since the set U 2 .F; T / was chosen
arbitrarily, we proved that
(1) U \ A ; for any set U 2 .F; T /.
Take an arbitrary neighborhood W of the point aF in TF . There exist a zero-set
Q and a cozero set U in the space T such that F  Q  U and U nF  W nfaF g
(see Problem 253). It is possible to find a set V 2 .Q; T / such that V  U (see
Fact 2 of V.140), so (1) shows that U \ A ; and therefore W \ A ;. This
proves that aF belongs to the closure of the set A in the space TF .
It turns out that
(2) for any faithfully indexed set N D fxn W n 2 !g  E there exists a continuous
function f W S ! R such that f .xn / D 2n for all n 2 !.
Pick dn 2 Dnfd g for which xn D .c; dn / and let Rn D f.ck ; dn / W k  ng [ fxn g
n
for
S every n 2 !. For each n 2 !, if x 2 Rn then let f .x/ D 2 . If x R D
n2! Rn then let f .x/ D 0. Observe that the set Rn is an open neighborhood of
xn , so f is continuous at every xn . If x D .c; y/ 2 EnN then C  fyg is an open
neighborhood of x on which f is constant, so f is also continuous at x. Finally,
if x D .ck ; d / for some k 2 ! then V D fck g  .Dnfdi W i  kg/ is an open
neighborhood of the point x on which f is constant and hence f is continuous at
x. This shows that f is continuous on S , so (2) is proved.
All promised properties of Y will be deduced from the following fact.
(3) For any countably infinite subset N  A there is a function ' 2 Cp .T / such
that '.F / D f0g; '.N /  1; C1/ and ' is not bounded on N .
Take a faithfully indexed set M D fzn W n 2 !g  E such that N  M  M
and apply (2) to construct a function f 2 Cp .S / for which f .zn / D 2n for every
n 2 !. Letting '.x; y/ D jf .x/  f .y/j for any .x; y/ 2 T we obtain a continuous
function ' W T ! R and it is evident that '.F / D f0g. If u 2 N then there are
distinct m; n 2 ! such that u D .zn ; zm /, so '.u/ D j2m  2n j  1; this shows that
'.N /  1; C1/.
The set N being infinite there exists a sequence fuk W k 2 !g  N such that
uk D .znk ; zmk / for all k 2 ! and the sequence flk D maxfnk ; mk g W k 2 !g is
unbounded. Then f .uk / D j2nk  2mk j  j2lk  2lk 1 j D 2lk 1 , so the sequence
ff .uk / W k 2 !g is unbounded as well and hence f is not bounded on N , i.e., (3) is
proved.
Let  W T ! TF be the contraction map. We already noted that aF belongs
to the closure of the set A in TF . Suppose that N  A is a countably infinite set
and take a function ' 2 Cp .T / as in (3). It follows from Fact 1 of V.252 that
there is a function f W TF ! R such that f  D '. In particular, f .aF / D 0

340

2 Solutions of problems 001500

and f .N /  1; C1/ which shows that aF cannot belong to the closure of N , so


t .TF / > ! and hence t .Y / > !. Furthermore, the function f is not bounded on
N , so N cannot be contained in a bounded subset of TF ; the set TF being clopen in
Y no bounded subset of Y can contain N either. Therefore B \ A is finite for any
bounded subset B of the space Y .
Consequently, K \ A is finite and hence closed for any compact K  Y ; since
A is not closed in Y , the space Y is not a k-space and hence it is not sequential.
Therefore the spaces X and Y witness the fact that a FrchetUrysohn space can be
l-equivalent to a non-k-space of uncountable tightness. This shows that the Frchet
Urysohn property, k-property, sequentiality and countable tightness all fail to be
l-invariant, i.e., our solution is complete.
V.307. Show that FrchetUrysohn property is not preserved by l-equivalence in
the class of compact spaces.
Solution. Given a space Z and a set F  Z, the symbol ZF stands for the Rquotient space obtained from Z by contracting the set F to a point which will
be denoted by aF . The expression X ' Y says that the spaces X and Y are
homeomorphic. If N is a countably infinite set then a family A of infinite subsets
of N is almost disjoint if A \ A0 is finite for any distinct A; A0 2 A. Given an
almost disjoint family A of subsets of N fix a point pA N for any A 2 A in
such a way that pA pA0 for distinct A; A0 2 A. Denote by M.N; A/ the set
N [ fpA W A 2 Ag with a topology defined by declaring all points of N isolated
while BA D ffpA g [ .AnK/ W K is a finite subset of N g is a local base at every
point pA in the space M.N; A/.
It is straightforward that M.N; A/ is a first countable locally compact space;
denote by K.N; A/ its one-point compactification. For any infinite subset N 0  N
let AjN 0 D fA \ N 0 W A 2 A and A \ N 0 is infiniteg. Say that an almost disjoint
family A on the set N is FrchetUrysohn if there exists no infinite set N 0  N
such that AjN 0 is a infinite maximal almost disjoint family on N 0 .
Fact 1. For any countably infinite set N and an almost disjoint family A on N , the
space K.N; A/ fails to be FrchetUrysohn if and only if there exists a set N 0  N
such that AjN 0 is an infinite maximal almost disjoint family on N 0 . In other words,
the space K.N; A/ is FrchetUrysohn if and only if A is a FrchetUrysohn family
on the set N .
Proof. Let P D fpA W A 2 Ag and suppose that B D AjN 0 is an infinite maximal
almost disjoint family on N 0 for some N 0  N . Denote by a the unique point of
the set K.N; A/nM.N; A/ and observe that if A 2 A and A \ N 0 is infinite then
pA 2 N 0 . Therefore C D N 0 \ P is an infinite set; since P nU is finite for any
neighborhood U of the point a, we have a 2 C and hence a 2 N 0 .
If S  N 0 is a sequence which converges to a then S is infinite but S \ A
is finite for any A 2 B, so B is not a maximal almost disjoint family on N 0 . This
contradiction shows that no sequence from N 0 converges to a, so the space K.N; A/
is not FrchetUrysohn and hence we proved sufficiency.

2 Solutions of problems 001500

341

Now assume that the space K.N; A/ is not FrchetUrysohn. Recall that we
have .x; K.N; A//  ! for every point x 2 K.N; A/nfag, so there must exist
a set E  K.N; A/ such that a 2 E but no sequence from E converges to a. If
E \ P is infinite then any countably infinite subset of E \ P converges to a which
is a contradiction, so E \ P is finite and hence the set N 0 D EnP is infinite. We
still have a 2 N 0 while there is no sequence in N 0 which converges to a.
If some infinite set D  N 0 is closed and discrete in M.N; A/ then D is a
sequence which converges to a. Therefore, for any infinite D  N 0 there is A 2 A
with pA 2 D and hence D \ A is an infinite set. Assume for a moment that the
0
family A0 D
S fA 2 A W A \ N 0is infiniteg is finite. It is evident that the set U D0
K.N; A/n. ffpA g [ A W A 2 A g/ is an open neighborhood of a, so D D U \ N
is an infinite set which is closed and discrete in M.N; A/. This contradiction shows
that AjN 0 is an infinite maximal almost disjoint family on N 0 , i.e., we established
necessity and hence Fact 1 is proved.
Fact 2. Suppose that N is a countably infinite set and we are given almost disjoint
families A and B on the set N . If C D A [ B is an infinite maximal almost disjoint
family on N then the space K.N; A/  K.N; B/ is not FrchetUrysohn.
Proof. Let a be the unique point of the set K.N; C/nM.N; C/. It is immediate that
the space K0 D N [ fpA W A 2 Ag [ fag  K.N; C/ is homeomorphic to K.N; A/
and K1 D N [ fpB W B 2 Bg [ fag is homeomorphic to K.N; B/.
The space K.N; C/ is not FrchetUrysohn by Fact 1. Since the subspace
M.N; C/ is first countable, there exists a set E  K.N; C/ such that a 2 E but
no sequence from E converges to a. If E 0 D E \ fpC W C 2 Cg is infinite then it is
easy to see that E 0 contains a sequence convergent to a. Therefore a 2 EnE 0 ; since
EnE 0  N , the space N [ fag is not FrchetUrysohn.
Let D f.x; x/ W x 2 K.N; C/g be the diagonal of the space K.N; C/. The set
0 D \ .N [ fag/2 coincides with the diagonal of N [ fag, so it is not Frchet
Urysohn being homeomorphic to N [ fag. Now it follows from 0  K0  K1 that
K0  K1 is not a FrchetUrysohn space. Therefore K.N; A/  K.N; B/ ' K0  K1
is not FrchetUrysohn, i.e., Fact 2 is proved.
Fact 3. Suppose that N is a countably infinite set and A is an infinite maximal
almost disjoint family on N . Call an infinite set N 0  N nontrivial if AjN 0 is
infinite. Then, for any decreasing family N0  N1  : : :  Nk  : : : of nontrivial
subsets of N there exists a nontrivial set H  N such that H nNk is finite for every
k 2 !.
Proof. Say that a set A  N is small if AnHk is finite for any k 2 !. Observe that
S
(1) if Ak is small and Ak  Nk for every k 2 ! then A D k2! Ak is also small,
because AnNk D .A0 nNk / [ : : : [ .Ak1 nNk / is a finite set for every k 2 !.
We are going to construct by induction a family fAk W k 2 !g of disjoint small
sets such that
(2) Ak  Nk and there exists Bk 2 A such that Ak  Bk for all k 2 !;
(3) the sets Bk and Bl are distinct if k l.

342

2 Solutions of problems 001500

Since every Ni is infinite, it is easy to pick a point xi 2 Ni in such a way that


the set A D fxi W i 2 !g is faithfully indexed. The family A being maximal almost
disjoint, there exists B0 2 A such that A0 D B0 \ A is infinite. It is clear that A0 is
a small set, so (2) and (3) hold for k D 0.
Assume that n 2 ! and we have small disjoint sets A0 ; : : : ; An and sets
B0 ; : : : ; Bn 2 A such that the properties (2) and (3) are fulfilled for each k  n.
Since Nk is nontrivial, the set Nk0 D Nk n.B0 [ : : : [ Bn / has to be infinite for
every k 2 !, so we can choose a point yk 2 Nk0 in such a way that the set
A0 D fyk W k > ng is faithfully indexed. The family A being maximal almost
disjoint, there is BnC1 2 A such that AnC1 D BnC1 \ A0 is infinite. It follows from
AnC1 \ .B0 [ : : : [ Bn / D ; that the family fA0 ; : : : ; AnC1 g is disjoint and it is
evident that AnC1 is a small set. Besides, BnC1 nBi  AnC1 is infinite and hence
BnC1 Bi for any i  n. This shows that the sets A0 ; B0 ; : : : ; AnC1 ; BnC1 satisfy
(2) and (3) for each k  n C 1, so our inductive procedure can be continued to
construct the promised sequence
fAk W k 2 !g with the properties (2) and (3).
S
By (1) the set H D k2! Ak is small; the properties (2) and (3) guarantee that
fBk \ H W k 2 !g is an infinite family contained in AjH , so H is nontrivial and
hence Fact 3 is proved.
Fact 4. Suppose that N is a countably infinite set and A is a maximal almost
disjoint family on N with jAj D c. Then there exists an infinite set Q  N such that
AjQ is infinite and AjQ D F0 [ F1 for some FrchetUrysohn families F0 ; F1 on
the set Q. As a consequence, there exists an infinite maximal almost disjoint family
on a countably infinite set which is a union of two FrchetUrysohn families.
Proof. Assume the contrary; then, for any infinite set Q  N if AjQ is infinite and
AjQ D F0 [ F1 then one of the families F0 ; F1 is not FrchetUrysohn, i.e., there
exists an infinite set A  Q such that Fi jA is an infinite maximal almost disjoint
family on A.
Recall that D D f0; 1g and take a faithful enumeration fAf W f 2 D! g of the
family A; let Ain D fAf W f .n/ D i g for any n 2 ! and i 2 D. Since A00 [A10 D A,
there exists an infinite set N0  N such that Ai00 jN0 is an infinite maximal almost
disjoint family on N0 for some i0 2 D.
Proceeding by induction assume that m 2 ! and we have constructed infinite
sets N0  : : :  Nm and a sequence i0 ; : : : ; im of elements of D such that Aikk jNk is
an infinite maximal almost disjoint family on Nk for every k  m. It follows from
the equality Aimm jNm D .A0mC1 \ Aimm /jNm [ .A1mC1 \ Aimm /jNm that there exists an
imC1
infinite set NmC1  Nm and imC1 2 D such that .AmC1
\ Aimm /jNmC1 is an infinite
imC1
maximal almost disjoint family on NmC1 . It is immediate that AmC1
jNmC1 is also
an infinite maximal almost disjoint family on NmC1 , so our inductive procedure can
be continued to construct a decreasing family fNk W k 2 !g of infinite subsets of N
and a sequence fik W k 2 !g  D such that Aikk jNk is an infinite maximal almost
disjoint family on Nk for all k 2 !.
Let f .k/ D ik for every k 2 !; apply Fact 3 to find an infinite set H  N such
that H nNk is finite for all k 2 ! and AjH is an infinite family. In particular, there

2 Solutions of problems 001500

343

exists g 2 D! nff g for which Ag \H is infinite. Fix k 2 ! with ik D f .k/ g.k/;


it follows from jH nNk j < ! that A D Ag \ Nk is infinite. Since Ag Aikk , the set
A \ B is finite for any B 2 Aikk jNk . Therefore the set A witnesses that the family
Aikk jNk is not maximal; this contradiction shows that Fact 4 is proved.
Returning to our solution apply Fact 1, Fact 2, and Fact 4 to find compact spaces
K and L with the following properties:
(4) there are points p 2 K and q 2 L such that both spaces Knfpg and Lnfqg are
first countable;
(5) K and L are FrchetUrysohn spaces;
(6) the space Y D K  L is not FrchetUrysohn.
Let K W Y ! K and L W Y ! L be the natural projections. The set F D
.fpg  L/ [ .K  fqg/ is closed in the space Y . It follows from Problem 262 that
the space Y is l-equivalent to an R-quotient image X of the space YF K L.
Observe that YF nfaF g is homeomorphic to Y nF ' .Knfpg/  .Lnfqg/, so the
space YF nfaF g is first countable by (4). Therefore for any point x 2 YF nfaF g if
x 2 A then some sequence from A converges to x, i.e., YF is FrchetUrysohn at
the point x.
Suppose that A  YF nfaF g and aF 2 A. If the set G D clY .A/ does not meet
F then W D faF g [ .Y n.F [ G// is an open neighborhood of aF in YF (see
Problem 254) such that W \ A D ; which is a contradiction. Therefore clY .A/
intersects the set F D K1 .p/ [ L1 .q/; the situation being symmetric, we can
assume, without loss of generality, that clY .A/\K1 .p/ ; and hence p 2 pK .A/.
The space K being FrchetUrysohn, we can choose a set S D fan W n 2 !g  A
such that the sequence fK .an / W n 2 !g converges to p.
Take any set U 2 .aF ; YF / then V D F [.U nfaF g/ is an open neighborhood of
F and, in particular, K1 .p/  V . By Fact 1 of S.226 we can find a set H 2 .p; K/
such that K1 .H /  V . There is m 2 ! such that K .an / 2 H for all n  m,
so an 2 V and hence an 2 U for all n  m; this proves that the sequence S
converges to aF and hence the space YF is FrchetUrysohn. Therefore the space
X 0 D YF K L is FrchetUrysohn as well; if ' W X 0 ! X is the respective
R-quotient map then it is closed because the space X 0 is compact. This, together
with TFS-225, shows that X is a FrchetUrysohn space. Thus the spaces X and
Y witness that the FrchetUrysohn property is not l-invariant, i.e., our solution is
complete.
V.308. Let Y be a space in which every closed subspace has the Baire property.
Suppose that Y is l-equivalent to a space X and a nonempty set Z  X also has
the Baire property. Prove that there is a nonempty W  Z which is open in Z and
homeomorphic to a subspace of Y .
Solution. The expression P ' Q says that the spaces P and Q are homeomorphic.
Fix a linear homeomorphism ' W Cp .Y / ! Cp .X / and let '  .a/ D a ' for any
a 2 Lp .X /; it is an easy consequence of Fact 4 of S.489 that '  W Lp .X / ! Lp .Y /
is a linear isomorphism. The set X 0 D '  .X / is a Hamel basis in the space Lp .Y /

344

2 Solutions of problems 001500

such that X 0 ' X and every f 2 Cp .X 0 / extends to a continuous linear functional


on Lp .Y /. Given nonempty sets A1 ; : : : ; An  Lp .Y / and O1 ; : : : ; On  R let
O1 A1 C : : : C On An D f 1 x1 C : : : C n xn W i 2 Oi and xi 2 Ai for all i  ng.
For any x 2 X 0 we can find a uniquely determined n 2 N and distinct points
y1 ; : : : ; yn 2 Y such that x D 1 y1 C : : : C n yn for some 1 ; : : : ; n 2 Rnf0g;
let Y .x/ D fy1 ; : : : ; yn g and lY .x/ D n. Analogously, for any point y 2 Y it is
possible to find a uniquely determined m 2 N and distinct points x1 ; : : : ; xm 2 X 0
such that y D 1 x1 C : : : C m xm for some 1 ; : : : ; m 2 Rnf0g; let lX .y/ D m
and X .y/ D fx1 ; : : : ; xm g. It follows from Problem 279 that Y W X 0 ! exp.Y /
and X W Y ! exp.X 0 / are the support maps for ' and ' 1 respectively. Applying
Problem 280 we infer that both Y and X are lower semicontinuous.
The set Yn D fy 2 Y W lX .y/  ng is closed in Y for any n 2 N by Fact 1
of U.485; we will also need the sets H1 D Y1 and Hn D Yn nYn1 for all n > 1.
Let Z 0 D '  .Z/; then Z 0 ' Z and, in particular, the space Z 0 has the Baire
property. Every set Zn D fa 2 Z 0 W lY .a/  ng is closed in Z 0 . Let G1 D Z1 and
Gn D Zn nZn1S
for all n > 1.
Since Z 0 D fZn W n 2 Ng, an easy consequence of the Baire property of Z 0
is that there exists a nonempty open set U in the space Z 0 such that U  Gn for
some n 2 N. Pick a point a 2 U ; then Y .a/ D fy1 ; : : : ; yn g and we can choose
a set Wi 2 .yi ; Y / for every i  n such that the family fW1 ; : : : ; Wn g is disjoint.
The set U 0 D fb 2 Z 0 W Y .b/ \ Wi ; for all i  ng is open in Z 0 and a 2 U 0 .
Therefore V D U \ U 0 is an open neighborhood of a in Z 0 .
Given any b 2 V it follows from lY .b/ D n that for every i 2 f1; : : : ; ng there
is a unique element xi 2 X .b/ \ Wi ; let pi .b/ D xi . This gives us a mapping
pi W V ! Wi for each i  n.
Let O D Rnf0g; if W is a nonempty open subset of Wi then it is easy to see
that pi1 .W / D .OW1 C : : : C OWi1 C OW C OWiC1 C : : : C OWn / \ V ; it
follows from Fact 2 of U.485 that pi1 .W / is an open subset of V , so the map pi is
continuous for any i  n.
Let V0 D V ; we will inductively construct nonempty open sets V0 ; : : : ; Vn of the
space Z 0 and k1 ; : : : ; kn 2 N such that
(1) Vi  Vi1 and pi .Vi /  Hki for all i D 1; : : : ; n.
Suppose that j < n and we have V0 ; : : : ; Vj 2  .Z 0 / and k1 ; : : : ; kj 2 N such
that the condition (1) is satisfied for all i D 1; : : : ; j . The set pj C1 .Vj / is closed
S
in Y , so it has the Baire property; this, together with pj C1 .Vj /  fYn W n 2 Ng
implies that there is a nonempty open subset E of the space pj C1 .Vj / such that
E  Hkj C1 for some kj C1 2 N. The set E 0 D E \ pj C1 .Vj / is nonempty and
0
0
open in pj C1 .Vj /, so Vj C1 D pj1
C1 .E / \ Vj is open in Vj and hence in Z . It is
straightforward that (1) now holds for all i  j C1, so our inductive procedure gives
us the promised sets V0 ; : : : ; Vn and numbers k1 ; : : : ; kn 2 N. As a consequence,
(2) lX .pi .b// D ki for any b 2 Vn and i D 1; : : : ; n.
Fix a point b 2 Vn and i 2 f1; : : : ; ng. Let yi D pi .b/ and consider the set
X .yi / D fa1i ; : : : ; aki i g; choose a disjoint family fU1i ; : : : ; Ukii g   .X 0 / such that

2 Solutions of problems 001500

345

aji 2 Uji for all j  ki . The set Qi D fy 2 X 0 W X .y/ \ Uji ; for all j  ki g
is open in X 0 , so Wi0 D Qi \ Wi is open in X 0 and nonempty because yi 2 Wi0 .
If y 2 Wi0 then, for every j  ki , there is a unique point bj 2 Uji \ X .y/; let
qji .y/ D bj . Then every qji W Wi0 ! Uji is a continuous map; this is proved in the
same way as we verified
continuity of pi .
T
The set E D fpi1 .Wi0 / W i  ng \ Vn is open in the space Z 0 and nonempty
due to the fact that b 2 E. For any i 2 f1; : : : ; ng and j 2 f1; : : : ; ki g the set
Sji D fc 2 E W qji .pi .c// D cg is closed in E. If c 2 E and Y .c/ D fz1 ; : : : ; zn g
Pn
every i  n let
then there are 1 ; : : : ; n 2 O for which c D
iD1 i zi . For P
X .zi / D fdji W j  ki g; there are i1 ; : : : ; iki 2 O such that zi D kj iD1 ij dji . The
P
P
equality c D niD1 i . kj iD1 ij dji / shows that c D dji for some i 2 f1; : : : ; ng and
j  ki . By definition of the maps pi and qji we have c D qji .pi .c//, i.e., c 2 Sji .
S
Therefore E  fSji W i  n; j  ki g; an immediate consequence is that we
can choose i  n and j  ki for which there is a set Q 2  .Z 0 / with Q  Sji .
Then qji .pi .c// D c for every c 2 Q which shows that the map pi jQ W Q ! pi .Q/
is a homeomorphism because qji jpi .Q/ is its continuous inverse.
We proved that a nonempty open set Q of the space Z 0 ' Z embeds in X .
Therefore a nonempty subspace W of the space Z also embeds in X . This settles
necessity and makes our solution complete.

V.309. Let X and Y be Cech-complete


l-equivalent spaces. Prove that every
nonempty subspace of X has a -base whose elements are embeddable in Y .

Deduce from this fact that if X and Y are nonempty Cech-complete


l-equivalent
spaces and X is scattered then Y is also scattered.
Solution. Take a nonempty set P  X and denote by U the family of all nonempty
open subsets of P which are embeddable in Y . Given any U 2  .P / the set Z D

U is closed in X and hence Cech-complete.


Observe that every closed subspace of

Y is Cech-complete and hence has the Baire property, so we can apply Problem 308
to see that some nonempty open set W in the space Z is embeddable in Y . The set
V D W \ U is nonempty and open in U and hence in P ; besides, V is embeddable
in Y , so V 2 U . We proved that any U 2  .P / contains an element of U , so the
family U is a -base of P .
l

Finally, assume that X  Y , the spaces X and Y are nonempty, Cech-complete


and X is scattered. If Y is not scattered then there exists a dense-in-itself nonempty
set Z  Y . By what we proved in the previous paragraph, there exists a nonempty
set U 2 .Z/ which embeds in X ; this implies that U is scattered. However, no
open subspace of a dense-in-itself space can have isolated points, so U is dense-initself; this contradiction shows that Y is also scattered.

V.310. Let X and Y be Cech-complete


l-equivalent spaces such that .X /  .
Prove that Y has a dense open subspace D such that .x; Y /   for each x 2 D.
In particular, .D/  .

346

2 Solutions of problems 001500

Solution. Apply Problem 309 to find a -base B in the space Y such that every
B 2 B is embeddable in X and hence .B/  . If B 2 B then S
B is open in X ,
so we have .x; Y / D .x; B/   for any x 2 B. The set D D B is open and
dense in X ; every x 2 D belongs to some B 2 B, so .x; Y /  . Therefore D is
the promised dense subspace of Y .
V.311. Prove that X is a closed Hamel basis of L.X / for any space X .
Solution. Let L be the family of all continuous maps of X into locally convex
spaces of cardinality not exceeding jX j 2! . For every ' 2 L let L' be the locally
convex space such
Q that ' W X ! L' . If i D f' W ' 2 Lg then i maps X in
the space L D fL' W ' 2 Lg and L.X / is the linear hull of i.X / in L. Let
p' W L.X / ! L' be the natural projection for each ' 2 L.
There exists ' 2 L such that ' is the canonical embedding of X into Lp .X /.
By the definition of the diagonal map, we have .p' ji.X // i D '. It is an easy
consequence of Fact 2 of S.337 that i W X ! i.X / is a homeomorphism, i.e., i
embeds X into L.X / and therefore we can identify i.x/ with x for any x 2 X .
Observe that p' W L.X / ! Lp .X / is a linear map such that p' .x/ D x for all
x 2 X ; since X is linearly independent in Lp .X /, it has to be linearly independent
in L.X / as well, i.e., X is a Hamel basis of L.X /. As an immediate consequence,
the map p' is an isomorphism and hence p'1 .X / D X ; since X is closed in L' D
Lp .X / (see TFS-167), the set X is closed in L.X /.
V.312. Prove that, for any space X and any continuous map f W X ! L of X to a
locally convex space L, there exists a unique continuous linear map f W L.X / ! L
such that f jX D f . Observe that this makes it possible to consider that L.X /, as
a linear space, coincides with Lp .X /  C.Cp .X // while the topology of L.X /
is stronger than .Lp .X //. In all problems that follow we use this observation
identifying the underlying set of L.X / with Lp .X /.
Solution. Let L be the family of all continuous maps of X into locally convex
spaces of cardinality not exceeding jX j 2! . For every ' 2 L let L' be the locally
convex space such
Q that ' W X ! L' . If i D f' W ' 2 Lg then i maps X in
the space L D fL' W ' 2 Lg and L.X / is the linear hull of i.X / in L. Let
p' W L.X / ! L' be the natural projection for each ' 2 L. Since i W X ! i.X / is a
homeomorphism (see Problem 311), we will identify i.x/ with x for every x 2 X .
Let M be the linear hull of the set f .X / in L. Then jM j  jX j 2! and hence
there exists ' 2 L such that ' W X ! L' coincides with f W X ! M , i.e.,
M D L' and '.x/ D f .x/ for each x 2 X . The definition of the diagonal product
implies that f D p' i . Recall that we identify X with i.X /, so the map p' W
L.X / ! L' D M coincides with f on X . Since p' is linear, continuous and M is
a linear subspace of L, we can consider that p' W L.X / ! L and hence f D p'
is the promised continuous linear extension of f . This extension is unique because
two linear functionals on a linear space coincide whenever they coincide on a Hamel
basis of the space.
Finally, observe that, for the canonical embedding id W X ! Lp .X / there exists
a continuous linear map  W L.X / ! Lp .X / such that jX D id . The set X

2 Solutions of problems 001500

347

is a Hamel basis in both L.X / and Lp .X /, so  has to be a linear isomorphism.


Therefore we can consider that the underlying set of L.X / coincides with Lp .X /.
After we identify the sets L.X / and Lp .X /, it follows from continuity of  that the
identity map of L.X / onto Lp .X / is continuous, i.e., the topology of L.X / contains
the topology of Lp .X /.
V.313. Suppose that L is a locally convex space such that X is embedded as a
Hamel basis in L. Prove that the following conditions are equivalent:
(i) there exists a linear homeomorphism i W L ! L.X / such that i.x/ D x for all
x 2 X;
(ii) every continuous function f W X ! M of the space X to a locally convex
space M , can be extended to a continuous linear functional f W L ! M .
Solution. Suppose that there exists a linear homeomorphism i W L ! L.X / such
that i.x/ D x for each x 2 X . If M is a locally convex space and f W X ! M
is a continuous function then we can apply Problem 312 to find a continuous linear
functional  W L.X / ! M such that .x/ D f .x/ for every x 2 X . The map
f D  i W L ! M is continuous and linear; since also f .x/ D .i.x// D
.x/ D f .x/ for all x 2 X , we conclude that f jX D f , i.e., f is the required
extension of f and hence we proved that (i)H)(ii).
Now, if (ii) holds then let id W X ! X be the identity map. Since X  L.X /, we
can consider that id W X ! L.X /, so there exists a continuous linear map i W L !
L.X / such that i jX D id . The set X is a Hamel basis in both L and L.X / (see
Problem 311), so it follows from linearity of i that i is an algebraic isomorphism.
By Problem 312 there exists a unique linear continuous map j W L.X / ! L such
that j jX D id . As a consequence, j jX D id D id 1 D i 1 jX , so j and i 1 are
linear maps on L.X / which coincide on X . The set X being a Hamel basis of L.X /
we have the equality i 1 D j and hence the map i 1 is continuous. Therefore i is a
linear homeomorphism, i.e., we established that (ii)H)(i).
V.314. Prove that, for any space X , the set .L.X // coincides with the set
.Lp .X // . Deduce from this fact that the weak topology of the space L.X /
coincides with the topology of Lp .X /.
Solution. Recall that we consider that the spaces L.X / and Lp .X / have the same
underlying set and operations while the topology of L.X / is stronger than the
topology of Lp .X / (see Problem 312). Therefore Cp .Lp .X //  Cp .L.X // and,
in particular, .Lp .X //  .L.X // . The weak topology  of the space Lp .X / is
generated by .Lp .X // while the weak topology  of the space L.X / is generated
by the set .L.X // .
Now take an arbitrary functional ' 2 .L.X // . The function f D 'jX is
continuous on X , so there exists a continuous linear functional  W Lp .X / ! R
such that jX D f D 'jX (see Fact 6 of S.489). If two linear functionals coincide
on a Hamel basis of a linear space then they coincide on the whole space; therefore
' D  2 .Lp .X // which shows that .L.X // D .Lp .X // , i.e., the topologies

348

2 Solutions of problems 001500

 and  are generated by the same set of linear functionals and hence  D .
Recalling that  D .Lp .X // (see Problem 233), we conclude that  D .Lp .X //,
i.e., the weak topology of L.X / coincides with the topology of Lp .X /.
V.315. Given a space X let E be the weak dual of L.X /, i.e., E D .L.X // and
the topology of E is induced from Cp .L.X //. For every f 2 E let .f / D f jX ,
i.e.,  W E ! Cp .X / is a restriction map. Prove that  is a linear homeomorphism
and hence E is linearly homeomorphic to Cp .X /.
Solution. Recall that we consider that the spaces L.X / and Lp .X / have the same
underlying set and operations while the topology of L.X / is stronger than the
topology of Lp .X / (see Problem 312). Therefore Cp .Lp .X //  Cp .L.X // while
the map  W .Lp .X // ! Cp .X / is a linear homeomorphism by Problem 235.
Since .Lp .X // D E by Problem 314, we conclude that  W E ! Cp .X / is a
linear homeomorphism as well.
V.316. Observe that l-equivalence implies u-equivalence, i.e., for any spaces X
l

and Y , if X  Y then X  Y . Prove that L-equivalence implies l-equivalence.


Solution. If X is l-equivalent to Y then there exists a linear homeomorphism
' W Cp .X / ! Cp .Y /. By Problem 134, the maps ' and ' 1 are uniformly
continuous, i.e., ' is also a uniform homeomorphism between Cp .X / and Cp .Y /.
Therefore the space X is u-equivalent to Y . Now, if X is L-equivalent to Y then
L.X / is linearly homeomorphic to L.Y /, so the weak dual EX of the space L.X /
is linearly homeomorphic to the weak dual EY of the space L.Y /. By Problem 315,
EX is linearly homeomorphic to Cp .X / and EY is linearly homeomorphic to
Cp .Y /. Therefore Cp .X / is linearly homeomorphic to Cp .Y /, i.e., the spaces X
and Y are l-equivalent.
V.317. Prove that Cb .X / is complete (as a uniform space with its linear uniformity)
if and only if X is a bf -space.
Solution. For any space Z, if A  Z then A W C.Z/ ! C.A/ is the restriction
map; for each f 2 C  .A/ let jjf jjA D supfjf .x/j W x 2 Ag. If the set A is bounded
in Z then we consider that A W C.Z/ ! C  .A/.
Fact 1. If Z is a space and A  Z is bounded in Z then the set CA D A .C.Z//
is closed in Cu .A/.
Proof. Recall that the topology of the space Cu .A/ is generated by the metric given
by the formula .f; g/ D jjf  gjjA for any f; g 2 C  .A/ (see Fact 1 of T.357).
The following property is crucial for the proof.
(1) For any f 2 CA there exists g 2 C  .Z/ such that jjgjjZ D jjf jjA and
A .g/ D f .
Fix a function h 2 C.Z/ with hjA D f and let M D jjf jjA . Define a function
g 2 RZ as follows: g.x/ D h.x/ whenever jh.x/j  M ; if h.x/ > M then g.x/ D
M and if h.x/ < M then g.x/ D M . It if immediate that g 2 C  .Z/; gjA D f
and jg.z/j  M for all z 2 Z. Therefore jjgjjZ D jjf jjA , i.e., (1) is proved.

2 Solutions of problems 001500

349

Take any function f 2 C A ; there exists a sequence S D ffn W n 2 !g  CA


which converges uniformly to f . Passing to a subsequence of S if necessary we
can assume, without loss of generality, that jjfn  fnC1 jjA  2n for all n 2 !.
Apply (1) to fix an arbitrary g0 2 C  .Z/ such that A .g0 / D f0 . Proceeding by
induction assume that n 2 ! and we have functions g0 ; : : : ; gn 2 C  .Z/ such that
gi jA D fi and jjgi  giC1 jjZ  2i for all i < n. Since jjfn  fnC1 jjA  2n ,
we can apply (1) to find a function p 2 C  .Z/ such that pjA D fnC1  fn and
jjpjjZ  2n . Then gnC1 D gn C p 2 C  .Z/ and jjgn  gnC1 jjZ D jjpjjZ  2n
while gnC1 jA D fnC1 . Thus our inductive procedure can be continued to construct
a sequence fgn W n 2 !g  C  .Z/ such that gn jA D fn and jjgn  gnC1 jjZ  2n
for all n 2 !. It is easy to see that fgn W n 2 !g is a Cauchy sequence in a complete
metric space Cu .Z/, so it converges uniformly to a function g 2 C  .Z/. Therefore
the sequence S 0 D fgn jA W n 2 !g converges uniformly to the function gjA. Since
gn jA D fn for all n 2 !, the sequence S 0 coincides with S , so f D gjA 2 CA .
Therefore C A D CA , i.e., CA is closed in Cu .A/ and Fact 1 is proved.
Assume first that Cb .X / is complete and X is not a bf -space, i.e., there exists
a function p 2 RX nCb .X / such that, for any bounded set B  X we can find a
function f 2 Cb .X / with pjB D f jB. It is obvious that for any A  X the set
FA D ff 2 Cb .X / W f jA D pjAg is closed in Cb .X / and FA ; if A is bounded
and nonempty. If B D fFB W B ; is a bounded subset of X g then it follows from
FA[B D FA \ FB that B is a filterbase in Cb .X /; let F be a filter which contains B.
To see that the family F is a Cauchy filter denote by U the linear uniformity
of Cb .X / and fix any set U 2 U ; there exists W 2 .0; Cb .X // such that the set
G D f.f; g/ 2 .Cb .X //2 W f  g 2 W g is contained in U . There exists a bounded
set B  X and " > 0 for which O D ff 2 Cb .X / W f .B/  ."; "/g  W . Then
FB 2 F and f; g 2 FB implies that f jB D pjB D gjB and hence .f  g/.x/ D 0
for any x 2 B, i.e., f  g 2 O  W . Therefore .f; g/ 2 U for any f; g 2 FB ; as a
consequence, FB  FB  U and hence F is a Cauchy
Tfilter in Cb .X /. The
T uniform
space Cb .X / being complete there is a function q 2 fQ W Q 2 Fg  B. Since
fxg is a bounded subset of X , we must have q 2 Ffxg and therefore q.x/ D p.x/
for any x 2 X ; this shows that p D q 2 Cb .X /, so we obtained a contradiction
which proves necessity.
Now assume that X is a bf -space and fix a Cauchy filter F in Cb .X /. Given a
point x 2 X let PFx D ff .x/ W f 2 F g for any F 2 F. Since PFx \G  PFx \ PGx for
any F; G 2 F, the family Px D fPFx W F 2 Fg is a filterbase of closed subsets of
R and it follows from the Cauchy property
T of F that Px is a Cauchy filterbase in R.
Therefore there is a unique point c 2 Px ; let p.x/ D c. This gives us a function
p W X ! R.
Fix an arbitrary bounded subset B  X ; it is easy to see that the family fB .F / W
F 2 Fg is a Cauchy filter in CB D B .C.X //. The metric space Cu .B/ (with the
metric induced by the norm jj jjB ) is complete; the set CB is closed in Cu .B/
by Fact 1, so it is also complete and hence CB is complete as a uniform space
(see T
Problem 118). Consequently, there exists a unique function h 2 CB such that
h 2 fB .F / W F 2 Fg.

350

2 Solutions of problems 001500

If " > 0 then the set W D ff 2 CB W jjf jjB < 2" g is an open neighborhood
of 0 in CB , so, by the Cauchy property of B .F/, there exists F 2 F such that
B .f /  B .g/ 2 W , i.e., jjB .f /  B .g/jjB < 2" for any f; g 2 F . It follows
from h 2 B .F / that jjh  B .f /jjB  2" < " for all f 2 F . This shows that
(2) for any " > 0 there exists F 2 F such that jjh  B .f /jjB < " for all f 2 F .
Now take any x 2 B; given " > 0 apply (2) to find a set F 2 F such that
jjhB .f /jjB < 3" . We also have p.x/ 2 ff .x/ W f 2 F g by the definition of p.x/,
so there exists f 2 F with jp.x/  f .x/j < 3" . Therefore jp.x/  h.x/j  23 " < ";
since " > 0 was chosen arbitrarily, we proved that p.x/ D h.x/ for all x 2 B. Since
h 2 CB , there exists g 2 Cb .X / such that gjB D h D pjB. Thus the function p
is b-continuous; recalling that X is a bf -space we conclude that p is continuous.
Besides, it follows from the property (2) and the equality pjB D h that
(3) for any bounded set B  X and " > 0 there exists a set F 2 F such that
jjB .p/  B .f /jjB < " for all f 2 F .
Finally fix any set F 2 F and a neighborhood U of the function p in Cb .X /.
We can choose a bounded set B  X and a number " > 0 such that the set G D
ff 2 Cp .X / W jjB .p/  B .f /jjB < "g is contained in U . By the property (3)
there exists F 0 2 F such that jjB .p/  B .f /jjB < " for any f 2 F 0 . Take
any f 2 F 0 \ F ; then jjB .p/  B .f /jjB < " and hence f 2 U \ F . This
shows
Tthat U \ F ; for any U 2 .p; Cb .X //, so p 2 F for all F 2 F, i.e.,
p 2 fF W F 2 Fg. Therefore the uniform space Cb .X / is complete and hence we
settled sufficiency.
V.318. Given a space X call a set P  C.X / equicontinuous at a point x 2 X if,
for any " > 0 there exists U 2 .x; X / such that f .U /  .f .x/  "; f .x/ C "/
for any f 2 P . The family P is called equicontinuous if it is equicontinuous at
every point x 2 X . Say that P is pointwise bounded if the set ff .x/ W f 2 P g is
bounded in R for any x 2 X . Prove that, for any equicontinuous pointwise bounded
set P  C.X /, the closure of P in the space Cb .X / is compact. In particular, if X
is pseudocompact and P  C.X / is equicontinuous and pointwise bounded then
the closure of P in Cu .X / is compact.
Solution. For any space Z, if A  Z then A W C.Z/ ! C.A/ is the restriction
map; for each f 2 C  .A/ let jjf jjA D supfjf .x/j W x 2 Ag. If the set A is bounded
in Z then we consider that A W C.Z/ ! C  .A/. Say that a set Q  C.Z/
is adequate if it is pointwise bounded and equicontinuous. The set clp .Q/ is the
closure of Q in the space Cp .Z/; analogously, clk .Q/ and clb .Q/ are the closures
of Q in the spaces Ck .Z/ and Cb .Z/ respectively.
Fact 1. For any space Z if Q  C.Z/ is an adequate set then Q0 D A .Q/ 
C.A/ is adequate for any A  Z.
Proof. Since fA .f /.x/ W f 2 Qg D ff .x/ W f 2 Qg for any x 2 A, the set Q0
is pointwise bounded. Given any x 2 A and " > 0 take a set U 2 .x; Z/ such that

2 Solutions of problems 001500

351

f .U /  .f .x/  "; f .x/ C "/ for all f 2 Q. Then U 0 D U \ A 2 .x; A/ and


f .U 0 /  .f .x/"; f .x/C"/ for all f 2 Q which shows that Q0 is equicontinuous
and hence Fact 1 is proved.
Fact 2. For any space Z if Q  C.Z/ is a pointwise bounded set then clp .Q/
is also pointwise bounded. Therefore the sets clk .Q/ and clb .Q/ are pointwise
bounded as well.
Proof. Let F D clp .Q/ and fix a point x 2 Z. If ex .f / D f .x/ for every f 2
C.Z/ then ex W Cp .Z/ ! R is a continuous function and ex .Q/ D ff .x/ W f 2 Qg
is a bounded subset of R. Therefore ex .F /  ex .Q/ is also bounded in R. Since
ex .F / D ff .x/ W f 2 F g, we proved that the set F is pointwise bounded. It follows
from clb .Q/  clk .Q/  clp .Q/ that the sets clb .Q/ and clk .Q/ are also pointwise
bounded, so Fact 2 is proved.
Fact 3. For any space Z if Q  C.Z/ is an equicontinuous set then clp .Q/ is
equicontinuous and coincides with the closure Q0 of the set Q in RZ . In particular,
the sets clk .Q/ and clk .Q/ are also equicontinuous.
Proof. It is evident that clp .Q/  Q0 ; to prove the opposite inclusion it suffices
to show that Q0  C.Z/. Every element of an equicontinuous family must be
continuous, so it is sufficient to show that Q0 is equicontinuous. Take a point x 2 Z
and " > 0. There exists U 2 .x; Z/ such that f .U /  .f .x/  2" ; f .x/ C 2" /
for all f 2 Q. If f 2 Q0 and jf .x/  f .y/j > 2" for some y 2 U then the set
O D fg 2 RZ W jg.x/  g.y/j > 2" g is open in RZ and contains f . Therefore there
is g 2 O \ Q and hence jg.x/  g.y/j > 2" which is a contradiction with g.U / 
.g.x/  2" ; g.x/ C 2" /. Thus f .U /  f .x/  2" ; f .x/ C 2"
 .f .x/  "; f .x/ C "/
for all f 2 Q0 which shows that Q0 is equicontinuous. Consequently, Q0  C.Z/
which implies that Q0 D clp .Q/. Finally observe that clb .Q/  clk .Q/  clp .Q/,
so the sets clb .Q/ and clk .Q/ are also equicontinuous and hence Fact 3 is proved.
Fact 4. If Z is a space and Q  C.Z/ is an adequate set then clp .Q/ is compact.
Proof. For any x 2 Z fix a compact subset Kx  R such that ff .x/ W f Q2 Qg 
Kx and denote by Q0 the closure of Q in RZ . It follows from Q  K D x2Z Kx
that Q0 is compact being a closed subset of K. Now it follows from Fact 3 that
clp .Q/ D Q0 is also compact, so Fact 4 is proved.
Fact 5. If Z is a space and Q  C.Z/ is an equicontinuous set then the topology
k induced on Q from Ck .Z/ coincides with the topology p induced on Q from
Cp .Z/.
Proof. It is clear that p  k , so fix a point q 2 Q and a set U 2 k . Choose a
compact set K  Z and " > 0 such that the set V D ff 2 Q W jf .x/  q.x/j < "
for all x 2 Kg is contained in U . For every x 2 K there exists a set Ox 2 .x; Z/
"
"
such that f .Ox /  .f .x/
S  3 ; f .x/ C 3 / for all f 2 Q. Choose a finite set
A  Z for which K  fOx W x 2 Ag and observe that the set W D ff 2 Q W
jf .x/  q.x/j < 3" for all x 2 Ag belongs to p .

352

2 Solutions of problems 001500

Fix any f 2 W and y 2 K; there exists x 2 A such that y 2 Ox and hence


jf .y/  f .x/j < 3" . By the definition of Ox we also have jq.x/  q.y/j < 3" , so
it follows from the inequality jf .y/  q.y/j  jf .y/  f .x/j C jf .x/  q.x/j C
jq.x/  q.y/j that jf .y/  q.y/j < 3" C 3" C 3" D ". The point y 2 K was chosen
arbitrarily, so we proved that jf .y/  q.y/j < " for all y 2 K and hence f 2 V . As
a consequence, W  V  U , i.e., we proved that, for any q 2 Q and U 2 k with
q 2 U there is W 2 p such that q 2 W  U . Now it is easy to see that k  p ,
i.e., k D p and hence Fact 5 is proved.
Fact 6. If Z is a -space, i.e., the closure of every bounded subset of Z is compact
then the topologies of Ck .Z/ and Cb .Z/ coincide.
Proof. A set U  C.Z/ belongs to .Ck .Z// (or to Cb .Z/ respectively) if and only
if for any f 2 U there exists a compact (bounded) set K  Z and " > 0 such that
the set f; K; "
D fg 2 C.Z/ W jf .z/  g.z/j < " for all z 2 Kg is contained in U .
Since every compact subset of Z is bounded in Z, every U 2 .Ck .Z// belongs to
.Cb .Z//, i.e., .Ck .Z//  .Cb .Z//.
Now if U 2 .Cb .Z// and f 2 U then there is a bounded set B  Z and
" > 0 such that f; B; "
 U . Since Z is a -space, the set K D B is compact, so
f; K; "
 f; B; "
 U which shows that U 2 .Ck .Z// and hence we proved
that .Cb .Z//  .Ck .Z//, i.e., .Cb .Z// D .Ck .Z// and hence Fact 6 is proved.
Fact 7. Given a space Z, if p 2 C.Z/, the set B  Z is bounded (or compact)
and " > 0 then the interior of the set p; B; "
D ff 2 C.Z/ W jf .z/  p.z/j < "
for all z 2 Bg in the space Cb .Z/ (or in the space Ck .Z/ respectively) contains the
function p.
S
Proof. Let U1SD p; B; 3"
and UnC1 D ff; B; 3n1 "
W f 2 Un g for all n 2 N.
The set U D fUn W n 2 Ng is open in Cb .Z/ (or in Ck .Z/ respectively) because,
if f 2 U then f 2 Un for some n 2 N and hence f; B; 3n1 "
 UnC1  U . It is
evident that p 2 U . Let n D 3" C : : : C 3"n for all n 2 N. We will prove by induction
that
(1) Un  p; B; n
for all n 2 N.
This is clear for n D 1, so assume that n 2 N, Un  p; B; n
and take any
function f 2 UnC1 . There exists g 2 Un such that f 2 g; B; 3n1 "
. By the
induction hypothesis, we have g 2 p; B; n
which implies that
jf .x/  p.x/j  jf .x/  g.x/j C jg.x/  p.x/j < 3n1 " C n D nC1
for every x 2 B, so f 2 p; B; nC1
for all f 2 UnC1 , i.e., UnC1  p; B; nC1
and
hence our inductive procedure can be carried out for all n 2 N which shows that (1)
is true. Since n < ", it follows from the property (1) that Un  p; B; n
 p; B; "

for all n 2 N; an immediate consequence is that U  p; B; "


. The set U being
open, we have p 2 U  Int.p; B; "
/ and hence Fact 7 is proved.

2 Solutions of problems 001500

353

Fact 8. Given a continuous map ' W Z ! T let '  .f / D f ' for any f 2 C.T /.
Then the dual map '  W Cb .T / ! Cb .Z/ is continuous; besides, if we consider that
'  W Ck .T / ! Ck .Z/ then '  is continuous as well.
Proof. Fix a function q 2 C.T / and let p D '  .q/; take any set U 3 p which
is open in Cb .Z/ (or in Ck .Z/ respectively). There exists a bounded (compact) set
B  Z such that the set p; B; "
D ff 2 C.Z/ W jf .z/  p.z/j < " for all z 2 Bg
is contained in U for some " > 0.
The set B 0 D '.B/ is bounded (compact) in the space T ; consider the set W 0 D
fg 2 C.T / W jg.x/  q.x/j < " for all x 2 B 0 g. If W is the interior of W 0 in the
space Cb .T / (or in Ck .T / respectively) then q 2 W by Fact 7. If g 2 W then, for
any z 2 B, we have '  .g/.z/ D g.'.z// and x D '.z/ 2 B 0 . As a consequence,
j'  .g/.z/  p.z/j D jg.x/  q.x/j < " for every z 2 B, i.e., '  .g/ 2 p; B; "
 U
for all g 2 W , so the set W witnesses continuity of '  at the point q and hence
Fact 8 is proved.
Fact 9. If Z is a space and Q  Cb .Z/ is an adequate set then let '.z/.f / D f .z/
for any f 2 Q and z 2 Z. This defines a function '.z/ 2 C  .Q/ for any z 2 Z and
the map ' W Z ! Cu .Q/ is continuous.
Proof. Observe first that '.z/.Q/ D ff .z/ W f 2 Qg is a bounded subset of R,
so '.z/ is a bounded function on Q for every z 2 Z. Every '.z/ is continuous on
the set Q endowed with the pointwise convergence topology by TFS-166; therefore
'.z/ is continuous on Q with the topology induced from Cb .Z/. This shows that
' W Z ! Cu .Q/.
To see that ' is continuous take any y 2 Z and U 2 .Cu .Q// with '.y/ 2 U .
There is " > 0 such that the set W D fg 2 Cu .Q/ W jjg '.y/jjQ < "g is contained
in U . By equicontinuity of Q there exists V 2 .y; Z/ such that jf .z/  f .y/j < 2"
for all z 2 V and f 2 Q. This is the same as saying that j'.z/.f /  '.y/.f /j < 2"
for all f 2 Q which shows that jj'.z/  '.y/jjQ  2" < " and hence '.z/ 2 W for
every z 2 V . In other words, '.V /  W  U , i.e., the set V witnesses that ' is
continuous at the point y, so Fact 9 is proved.
Fact 10. Given a space Q and Y  Cu .Q/ let e.z/.f / D f .z/ for every f 2 Y
and z 2 Q. Then e.z/ 2 C.Y / for all z 2 Z and the set E D fe.z/ W z 2 Qg  C.Y /
is adequate.
Proof. If z 2 Q then e.z/ is continuous on Y with the topology induced from
Cp .Q/ by TFS-166; since Cu .Q/ induces a stronger topology on Y , every e.z/ is
continuous. Given any f 2 Y it follows from f 2 C  .Q/ that the set f .Q/ is
bounded in R. Therefore fe.z/.f / W z 2 Qg D f .Q/ is bounded in R and hence the
set E is pointwise bounded.
Next, fix a function f 2 Y and " > 0. The set U D fg 2 Y W jjg  f jjQ < "g is
open in Y and f 2 U . For any z 2 Q and g 2 U we have
je.z/.g/  e.z/.f /j D jg.z/  f .z/j  jjg  f jjQ < ";

354

2 Solutions of problems 001500

i.e., e.z/.U /  .e.z/.f /  "; e.z/.f / C "/ for any z 2 Q and hence U witnesses
equicontinuity of E at the point f . This shows that E is adequate, so Fact 10 is
proved.
Returning to our solution fix an adequate set P in the space Cb .X / and let
'.x/.f / D f .x/ for all x 2 X and f 2 P . Then '.x/ 2 C  .P / for every
x 2 X and the map ' W X ! Cu .P / is continuous by Fact 9; let Y D '.X /. For
each f 2 P let e.f /.y/ D y.f / for any y 2 Y ; then e.f / 2 C.Y / for every
f 2 P and the set Q D e.P /  C.Y / is adequate by Fact 10.
The closure F of the set Q in the space Cp .Y / is compact by Fact 4. Furthermore,
the set F has to be adequate by Fact 2 and Fact 3. By Fact 5, the topology induced
on F from Ck .Y / coincides with the topology induced on F from Cp .Y /. Therefore
the set F is compact as a subspace of Ck .Y /. The space Y is metrizable because
so is Cu .P /; therefore the topologies of Cb .Y / and Ck .Y / coincide by Fact 6;
consequently, F is a compact subspace of Cb .Y /.
Since the dual map '  W Cb .Y / ! Cb .X / is continuous by Fact 8, the set
G D '  .F / is a compact subspace of Cb .X /. Our last step is to prove that P  G,
so fix an arbitrary function f 2 P . Then g D e.f / 2 Q; given x 2 X we
have '  .g/.x/ D g.'.x//. If y D '.x/ then g.'.x// D e.f /.y/ D y.f / D
'.x/.f / D f .x/. Therefore g.'.x// D f .x/ for all x 2 X which shows that
'  .g/ D f and hence f 2 '  .Q/  '  .F / D G. We proved that the set P is
contained in a compact set G, so the closure of P in Cb .X / is compact, i.e., our
solution is complete.
V.319. Prove that, for any bf -space X , a set P  C.X / is equicontinuous and
pointwise bounded if and only if the closure of P in the space Cb .X / is compact.
Solution. Given a space Z and A  Z let A W C.Z/ ! C.A/ be the restriction
map. If A is bounded in Z then jjf jjA D supfjf .x/j W x 2 Ag.
Fact 1. If Z is a space and B  Z is a bounded subspace of Z then the restriction
map B W Cb .Z/ ! Cu .B/ is continuous.
Proof. Fix a function p 2 Cb .Z/, a set O 2 .B .p/; Cu .B// and let q D B .p/.
There exists " > 0 such that V D ff 2 Cu .B/ W jjf  qjjB < "g  O. Consider
the set W 0 D ff 2 Cb .Z/ W jf .x/  p.x/j < 2" for all x 2 Bg; by Fact 7 of V.318,
we have p 2 W D Int.W 0 /. If f 2 W then jjf  pjjB  2" < " which shows that
jjB .f /  qjjB < " and therefore B .f / 2 V . Thus B .W /  V  O, so the set
W witnesses continuity of B at the point p and hence Fact 1 is proved.
Returning to our solution observe that necessity was established in Problem 318,
so assume that the closure K of the set P in the space Cb .X / is compact. It suffices
to show that K pointwise bounded and equicontinuous. For any point x 2 X let
ex .f / D f .x/ for every f 2 K. Then ex W K ! R is continuous if the topology
p is induced on K from Cp .X / (see TFS-166). Since the topology b induced on
K from Cb .X / contains p (actually, b D p but we dont need that), the map
ex W .K; b / ! R is continuous. Therefore the set ex .K/ D ff .x/ W f 2 Kg is
compact and hence bounded in R, i.e., we proved that K is pointwise bounded.

2 Solutions of problems 001500

355

To see that the set K is equicontinuous fix a point a 2 X ; for any x 2 X the
function ux D jex  ea j is continuous on Cb .X /, so the set ux .K/ is compact.
Therefore the number '.x/ D sup ux .K/ is consistently defined; it is easy to see
that '.x/ D supfjf .x/  f .a/j W f 2 Kg for any x 2 X . Our next step is to prove
that the function ' W X ! R is continuous. Since X is a bf -space, it suffices to
show that 'jB can be extended to a continuous function on X for any bounded set
B  X.
So, fix a bounded set B  X ; there is no loss of generality to assume that a 2 B.
The restriction map B W Cb .X / ! Cu .B/ is continuous by Fact 1, so L D B .K/
is a compact subset of Cu .B/. Fix an arbitrary " > 0; since L is metrizable, there
exists a finite Q  K such that, for any f 2 K there is q 2 Q with jjq  f jjB < 3" .
The function  W X ! R defined by the equality .y/ D supfjr.y/r.a/j W r 2 Qg
for every y 2 X , is continuous on X .
If x 2 B then there exists f 2 K such that jf .x/  f .a/j > '.x/  3" . There is
q 2 Q with jjqf jjB < 3" and, in particular, jq.x/f .x/j < 3" and jq.a/f .a/j <
"
. Thus, jf .x/  f .a/j  jf .x/  q.x/j C jq.x/  q.a/j C jq.a/  f .a/j which
3
shows that '.x/ 3" < 23 "Cjq.x/q.a/j and hence '.x/  .x/  jq.x/q.a/j >
'.x/  ". The point x 2 B was chosen arbitrarily, so we established that
(1) for every " > 0 there is  2 C.X / such that j.x/  '.x/j < " for all x 2 B.
Applying the property (1) we can choose, for each n 2 ! a continuous function
n W X ! R such that jn .x/  '.x/j < 2n for all x 2 B. An immediate
consequence is that the sequence fB .n / W n 2 !g converges uniformly to 'jB
and hence 'jB belongs to the closure of the set B .C.X // in Cu .B/. However,
B .C.X // is closed in Cu .B/ by Fact 1 of V.317, so 'jB 2 B .C.X // for any
bounded B  X . The bf -property of X implies that the function ' is continuous
on X .
Finally, take any " > 0; by continuity of the function ' there exists a set U 2
.a; X / such that j'.x/  '.a/j D j'.x/j < " for all x 2 U . Given f 2 K we have
jf .x/  f .a/j  j'.x/j < " for every x 2 U and hence f .U /  .f .a/  "; f .a/ C
"/. This shows that K is equicontinuous and makes our solution complete.
V.320. Given a space X , let P; "
D f' 2 L.X / W '.P /  ."; C"/g for every
P  C.X / and " > 0. Prove that a set U  L.X / is open in L.X / if and only if,
for any  2 U there exists an equicontinuous pointwise bounded set P  C.X / and
" > 0 such that CP; "
 U . In other words, the topology of L.X / coincides with
the topology of uniform convergence on equicontinuous pointwise bounded subsets
of C.X /.
Solution. As before, we consider that X  L.X / and X is a Hamel basis of L.X /.
Call a set P  C.X / adequate if P is equicontinuous and pointwise bounded; let
O D fU  L.X / W for each  2 U there exists an adequate set P  C.X / and
" > 0 such that  C P; "
 U g. We have to prove that O D .L.X //.
We omit an easy checking that O is a topology on L.X /. Let us prove, however,
that L .X / D .L.X /; O/ is a locally convex space. Given a set A  L.X / we
denote by Int .A/ the interior of A in the space L .X /. Let us first prove that

356

2 Solutions of problems 001500

(1) if  2 L.X / then  2 Int . C P; "


/ for any adequate set P  C.X / and
" > 0.
S
Consider the set H1 D  CS
P; 3"
and let HnC1 D f' C P; 3n1 "
W ' 2 Hn g
for all n 2 N. The set H D fHn W n 2 Ng is open in L .X /; indeed, if  2 H
then there exists n 2 N such that  2 Hn and hence  C P; 3n1 "
 HnC1  H .
If n 2 N and n D 31 " C : : : C 3n " then it takes a straightforward induction to
show that Hn   C P; n
. Since n < ", we have Hn  P; "
for all n 2 N and
hence H   C P; "
. Therefore  2 H  Int . C P; "
/, i.e., (1) is proved.
Let B D fB 2 O W 0 2 Bg; it is evident that B0 ; B1 2 B implies B0 \ B1 2 B.
If B 2 B and  2 L.X / then we can find an adequate set P  C.X / and " > 0
with P; "
 B. There are x1 ; : : : ; xn 2 X and 1 ; : : : ; n 2 R such that  D
P
n
bounded there is qi > 0
iD1 i xi . Take any i  n; the set P being pointwise
Pn
such
that
jf
.x
/j

q
for
all
f
2
P
.
If
q
D
.j
i
i
i j C 1/qi then j.f /j D
iD1
P
j niD1 i f .xi /j  q for all f 2 P . Now if t > q" then j. 1t f /j D 1t j.f /j  qt <
" for all f 2 P which shows that 1t  2 P; "
and hence  2 t P; "
 tB, i.e., we
proved that every set B 2 B is absorbing.
Now take any  2 L.X /nf0g. There are distinct points x1 ; : : : ; xn 2 X and
1 ; : : : ; n 2 Rnf0g such that  D 1 x1 C : : : C n xn . Take a function f 2 C.X /
such that f .x1 / D 11 and f .xi / D 0 for all i D 2; : : : ; n. The set P D ff g is
adequate and  P; 1
; it followsTfrom (1) that the set B D Int .P; 1
/ belongs
to B. Since  B, we proved that B D f0g.
Given any B 2 B there is an adequate set P  C.X / such that P; "
 B for
some " > 0. Apply the property (1), to see that the set V D Int .P; 2"
/ belongs to
B; it is straightforward that V C V  P; "
 B, so, for any B 2 B there is V 2 B
with V C V  B.
Next assume that B 2 B and  2 B; choose an adequate set P  C.X / such
that  C P; "
 B for some " > 0. Apply the property (1) once more to conclude
that V D Int .P; "
/ belongs to B; since also  C V  B, we proved that, for any
B 2 B and  2 B there exists V 2 B with  C V  B.
Suppose that B 2 B and " > 0; choose an adequate set P  C.X / such that
P;
 B for some > 0. The set V D Int .P; "
/ belongs to B by (1); if
2 ."; "/ and  2 V then j .f /j  "j.f /j < " " D for each f 2 P and
hence  2 P;
 B. Therefore we can apply Problem 202 to the family B to
see that there exists a topology on the set L.X / such that .L.X /; / is a linear
topological space for which B is a local base at 0 in .L.X /; /. If U 2 and  2 U
then  C U 2 and 0 2  C U , so there is B 2 B such that B   C U and
hence  C B  U . Since B 2 O, there is an adequate set P  C.X / for which
P; "
 B for some " > 0. Consequently,  C P; "
 U ; this proves that U 2 O
and therefore  O.
If U 2 O and  2 U then it is easy to see that B D  C U 2 O; since 0 2 B,
the set B belongs to and hence U D  C B 2 . Therefore O  , i.e., O D ,
so we proved that L .X / is a linear topological space. If B 2 B then there exists an
adequate set P  C.X / such that P; "
 B for some " > 0. It is easy to see that

2 Solutions of problems 001500

357

P; "
is a convex set so W D Int .P; "
/ is also convex (see Problem 203). Since
0 2 W  B, we proved that L .X / is a locally convex space.
Let O0 D fO \ X W O 2 Og be the topology induced on X from L .X /.
If O 2 O0 and x 2 O then there exists an adequate set P  C.X / such that
.x C P; "
/ \ X  O for some " > 0. By equicontinuity of P there exists a set
U 2 .x; X / such that jf .y/  f .x/j < " for each f 2 P and y 2 U . If y 2 U
then .y  x/.f / D f .y/  f .x/ 2 ."; "/ for every f 2 P which shows that
y  x 2 P; "
and hence y 2 x C P; "
 O. Thus, for every set O 2 O0 and
x 2 O there is U 2 .x; X / such that U  O. An immediate consequence is
that O0  .X / and hence the identity map i W X ! .X; O0 / is continuous. We
can consider that i W X ! L .X /, so Problem 312 is applicable to find a linear
continuous map  W L.X / ! L .X / such that jX D i . The set X is a Hamel basis
of L.X /, so the linear extension of i is unique and hence  has to be the identity
map. It follows from continuity of  that O  .L.X //.
To prove that .L.X //  O fix a set U 2 .L.X // such that 0 2 U and
choose a convex balanced set V 2 .0; L.X // with V  U . Consider the set
Q D f 2 .L.X // W .V /  Ig  C.L.X //. Given  2 L.X / there is r > 0 such


that  2 tV for all t > r. Therefore rC1
2 V which implies that j. rC1
/j  1, i.e.,
j./j  r C 1 for all 2 Q. Thus f./ W 2 Qg  r  1; r C 1
which shows
that the set Q is pointwise bounded.
Given " > 0 and  2 L.X / the set W D 2" V is an open neighborhood of 0 in
L.X /, so CW 2 .; L.X //. If 2 Q and w 2 W then .Cw/ D ./C.w/.
There is v 2 V such that w D 2" v and hence j.w/j D 2" j.v/j  2" < ". This shows
that . C W /  ../  "; ./ C "/ for all 2 Q and hence the set Q is
equicontinuous, i.e., Q is an adequate set. The set P D fjX W 2 Qg  C.X / is
also adequate by Fact 1 of V.318. It is easy to check that
(2) for any 2 Q if f D jX then .f / D ./ for every  2 L.X /.
It follows from (1) that the set O D Int .P; 1
/ belongs to B. If there exists a
point  2 OnV then we can apply Problem 223 to the set V to find a functional
2 .L.X // such that ./ > 1 while .V /  I and hence .V /  I, i.e., 2 Q.
If f D jX then f 2 P , so we can apply (2) to see that .f / D ./ > 1 which
is a contradiction with  2 O  P; 1
. This contradiction shows that O  V and
hence O  U . Therefore every neighborhood of 0 in L.X / contains an element
of B  O, so the identity map  W L.X / ! L .X / is open by Fact 3 of S.496.
Consequently,  is a homeomorphism and hence .L.X // D O, i.e., our solution is
complete.
V.321. Given a space X and P  X let IP D ff 2 C.X / W f .P /  f0gg. Prove
that for any linear continuous functional ' W Ck .X / ! R which is not identically
zero on Ck .X /, there exists a compact subspace K  X (called the support of '
and denoted by supp.'/) such that '.IK / D f0g and '.IK 0 / f0g whenever K 0 is
a proper compact subset of K.

358

2 Solutions of problems 001500

Solution. If Z is a space and A  Z then A; "


D ff 2 C.Z/ W f .A/  ."; "/g
for any " > 0. If A ; and a function f 2 C.Z/ is bounded on the set A then
jjf jjA D supfjf .x/j W x 2 Ag.
Fact 1. Suppose that Z is a space and A is a nonempty subset of Z while some
f 2 C.Z/ is bounded on A. Then there exists a function g 2 C  .Z/ such that
gjA D f jA and jjgjjZ D jjf jjA .
Proof. Let M D jjf jjA and define a function g 2 RZ as follows: g.x/ D f .x/
whenever jf .x/j  M ; if f .x/ > M then g.x/ D M and if f .x/ < M then
g.x/ D M . It if immediate that g 2 C  .Z/; gjA D f jA and jg.z/j  M for all
z 2 Z. Therefore jjgjjZ D jjf jjA , i.e., Fact 1 is proved.
Fix a nontrivial continuous linear functional ' W Ck .X / ! R and denote by K the
family of all nonempty compact subsets of X such that '.IK / D f0g. By continuity
of ' at the point 0 there exists a compact set K  X such that '.K; "
/  .1; 1/
for some " > 0; observe that IK  K; "
. If K D ; then K; "
D C.X / and hence
we have the inclusion '.C.X //  .1; 1/. However, there exists f 2 C.X / such
that '.f / 0 and hence, for the function g D '.f1 / f , we have '.g/ D 1 which
is a contradiction. Therefore K is a nonempty set.
If f 2 IK and '.f / 0 then, for r D '.f1 / we have '.rf / D 1. Now it follows
from rf 2 IK that rf 2 K; "
which implies j'.rf /j < 1; this contradiction shows
that '.IK / D f0g, i.e., K 2 K and hence the family K is nonempty.
If K 2 K; f; g 2 C.X / and f jK D gjK then f g 2 IK and hence '.f g/ D
0 which implies that '.f / D '.g/. Therefore we proved that
(1) if K 2 K and f; g 2 C.X / then f jK D gjK implies that '.f / D '.g/; in
particular, if '.f / 0 then f .x/ 0 for some x 2 K.
Next assume that K; L 2 K and K \ L D ;. There exists a function f 2 C.X /
such that '.f / 0 and hence f jL is not identically zero by (1). Let h.x/ D 0 for
all x 2 K and h.x/ D f .x/ for all x 2 L. It is evident that h W K [ L ! R is a
continuous function, so there exists g 2 C.X / such that gj.K [ L/ D h (see Fact 1
of T.218). Since gjL D hjL D f jL, we can apply (1) to see that '.g/ D '.f / 0.
On the other hand, g 2 IK , so '.g/ D 0. This contradiction shows that
(2) K \ L ; for any K; L 2 K.
Now fix any K; L 2 K; by the property (2) the set F D K \ L is nonempty.
If F K then there exists a function f 2 IF such that '.f / 0. The property
(1) shows that f .a/ 0 for some a 2 K. Let g.x/ D f .x/ for all x 2 K and
g.x/ D 0 whenever x 2 LnK. It follows from Fact 2 of T.354 that the function g
is continuous on K [ L, so we can apply Fact 1 of T.218 again to find a function
h 2 C.X / such that hj.K [ L/ D g. Since hjK D f jK, the property (1) shows that
'.h/ D '.f / 0. However, h.L/ D f0g, so '.h/ D 0; the obtained contradiction
proves that

2 Solutions of problems 001500

359

(3) K \ L 2 K for any K; L 2 K and hence K is a centered family.


T
It follows from (3) that the set K D K is compact and nonempty. If K 0 is a
proper compact subset of K then K 0 K and hence '.IK 0 / f0g, so it suffices to
show that K 2 K. To do that we will need the following statement.
T
(4) Assume thatS
Fn  X is compact and FnC1  Fn for all n 2 !. If F D n2! Fn
then the set n2! IFn is dense in IF .
Fix a function f 2 IF and U 2 .f; Ck .X //; there exists a compact set Q  X
such that f C Q; "
 U for some " > 0. If W D f 1 .."; "// then W is an
open subset of X with F  U , so we can find n 2 ! such that Fn  W (see
Fact 1 of S.326). It follows from compactness of the set Fn that r D jjf jjFn < ".
Apply Fact 1 to find a function h 2 C.X / such that jjhjjX  r and hjFn D f jFn .
Then q D f  h 2 IFn and jjq  f jjX  r < ". In particular, q  f 2 Q; "
, so
q
S2 f C Q; "
 U . Thus every neighborhood of f in Ck .X / intersects the set
n2! IFn , so (4) is proved. Now we can show that
T
(5) if Fn 2 K for every n 2 ! then F D n2! Fn also belongs to K.
It follows from (3) that there is no loss of generality
to assume that FnC1  Fn
S
for all n 2 !. By the property (4) the set H D n2! IFn is dense in IF , so the set
f0g D '.H / is dense in '.IF / and hence '.IF / D f0g, i.e., (5) is proved.
Finally assume that K K, i.e., there exists f 2 IK such that '.f / 0. Given
n n
any n 2 ! if Wn D f 1 ..2T
; 2 // then K  Wn , so there exists FT
n 2 K such
that Fn  Wn . The set F D n2! Fn belongs to K by (5) and F  n2! Wn D
f 1 .0/ which shows that f 2 IF and hence '.f / D 0, a contradiction. Therefore
K is the minimal element of K, i.e., our solution is complete.
V.322. Recall that a set B is a barrel in a locally convex space L if B is closed,
convex, balanced and absorbing in L. The space L is barreled if any barrel in L is
a neighborhood of 0. Prove that a locally convex space L is barreled whenever it
has the Baire property.
Solution. Assume that a locally convex space L has the Baire property and fix a
barrel B  L. It is straightforward that the set rB D frx W x 2 Bg
S is also a barrel
for any r > 0. Since B is absorbing, we have the equality L D fnB W n 2 Ng,
so it follows from the Baire property of the space L that there exists m 2 N such
1
that W D Int.mB/ ;. The set V D 2m
W is open in L and V  12 B; since 12 B
is balanced, the set U D V is also contained in 12 B. It is clear that G D U C V
is an open neighborhood of 0. If x 2 G then there are u 2 U and v 2 V such
that x D u C v. There exist b; b 0 2 B with u D 12 b and v D 12 b 0 . It follows from
convexity of B that x D 12 b C 12 b 0 2 B. The point x 2 G was chosen arbitrarily, so
G  B which shows that every barrel B  L is a neighborhood of 0, i.e., the space
L is barreled.
V.323. Prove that Ck .X / is a barreled space if and only if X is a -space, i.e., the
closure of any bounded subspace of X is compact. Deduce from this fact that Ck .X /
is barreled for any realcompact space X .

360

2 Solutions of problems 001500

Solution. For any Q  X and " > 0 let Q; "


D ff 2 C.X / W f .Q/  ."; "/g
and IQ D ff 2 C.X / W f .Q/  f0gg; if a function f 2 C.X / is bounded on Q
then jjf jjQ D supfjf .x/j W x 2 Qg.
Assume that Ck .X / is barreled and take a bounded set P in the space X . It is
easy to see that the set B D ff 2 C.X / W f .P /  Ig is a barrel in Ck .X /, so we
can find a set U 2 .Ck .X // for which 0 2 U  B. There exists a compact set
K  X such that K; "
 U for some " > 0.
If P nK ; then fix a point x 2 P nK; there exists a function f 2 C.X / such
that f .x/ D 2 and f .K/  f0g. Therefore f 2 K; "
 B while f .x/ 2 f .P /
does not belong to I. This contradiction shows that P  K and hence the closure of
any bounded set P  X is compact, i.e., we proved necessity.
Now assume that X is a -space and fix a barrel B in the space Ck .X /. We omit
an easy checking that the set B 0 D B \ C  .X / is a barrel in the space Cu .X /;
the space Cu .X / is completely metrizable, so it has the Baire property. Therefore
Cu .X / is barreled by Problem 322 and hence the set B 0 is a neighborhood of 0 in
Cu .X /; this makes it possible to pick a number > 0 such that X;
 B 0  B.
Denote by the set of all continuous linear functionals  W Ck .X / ! R such
that .B/  I. For any  2 nf0g let S be the support of , i.e., S is a compact set
such that .IS / D f0g while .IF / f0g for any proper compact set F  S (see
Problem 321). We will need the following property of the supports of the functionals
from nf0g.
(1) if  2 nf0g and U \ S ; for some U 2 .X / then there exists a function
f 2 C.X / such that f .X nU /  f0g and '.f / 0.
By minimality of the set K D S there exists a function g 2 C.X / such that
.g/ 0 and g.KnU /  f0g. Since K is compact, the set g.K/ is bounded in R,
so there exists r > 0 such that, for the function h D rg, we have h.K/  I. It is
clear that h.KnU /  f0g and .h/ 0. Let us establish that
(2) there exists " > 0 such that jjf  hjjK < " implies .f / 0.
Indeed, the set O D Ck .X /n 1 .0/ is open in Ck .X /, so we can find a compact
set L  X and " > 0 such that K  L and jjf  hjjL < " implies f 2 O,
i.e., .f / 0. Now, if jjf  hjjK < " then we can apply Fact 1 of V.321 to
find a function q 2 C.X / such that qjK D .f  h/jK and jjqjjX < ". Then
jjqChhjjX D jjqjjX < " which implies jjqChhjjL < " and hence .qCh/ 0.
We also have .q C h/jK D f jK, so it follows from the definition of support that
.f / D .q C h/ 0 and hence (2) is proved.
The set M D fx 2 K W jh.x/j  2" g is compact and M  U . It is easy to see
that the closure F of the set X nU in the space X does not meet M , so there exists
a continuous function d W X ! 0; 1
such that d.F /  f0g and d.M /  f1g.
Then u D d jX 2 C.X / and u W X ! 0; 1
while u.X nU /  f0g and u.M /  f1g.
If f D uh then f .x/ D h.x/ whenever x 2 .KnU / [ M . If x 2 .KnM / \ U then
jh.x/j < 2" and hence jf .x/  h.x/j D jh.x/j ju.x/  1j  jh.x/j < 2" . This shows
that jjf  hjjK  2" < ", so we can apply (2) to conclude that .f / 0; it is clear
that f .X nU /  f0g, so (1) is proved. Our next step is to show that

2 Solutions of problems 001500

(3) the set P D

361

fS W  2 nf0gg is bounded in X .

If (3) is not true then there exists a sequence S D fn W n 2 !g  nf0g and a
discrete family U D fUn W n 2 !g of open subsets of X such that Un \ Sn ; for
all n 2 ! (see Fact 1 of V.245). We can assume without loss of generality (passing
to an appropriate subsequence of S if necessary) that Un \ Sm D ; whenever
m < n. Apply the property (1) to find a function fn 2 C.X / such that n .fn / 0
and fn .X nUn /  f0g for each n 2 !.
Let c0 D 0; then 0 .c0 f0 / D 0. Proceeding inductively, assume that n 2 !
and we have cP0 ; : : : ; cn 2 R such that i .c0 f0 C : : : C ci fi / D i for all i  n. If
P
nC1 niD0 nC1 .ci fi /
cnC1 D
then nC1 .cnC1 fnC1 / D n C 1  niD0 nC1 .ci fi / and
nC1 .fnC1 /
P
hence nC1 . nC1
iD0 ci fi / D n C 1, so our inductive procedure can be continued to
obtain a sequence fcn W n 2 !g  R such that n .c0 f0 C : : : C cn fn / D n for all
n 2 !.
P
TheP
family U being discrete, the function fSD 1
iD0 ci fi is continuous on X . If
gn D niD0 ci fi then it follows from Sn \ . m>n Um / D ; that f jSn D gn jSn
and hence n .f / D n .gn / D n for all n 2 !. The set B is absorbing, so there
exists r > 0 such that 1r f 2 B and hence jn . 1r f /j  1 which implies that
n D jn .f /j  r for all n 2 !. This contradiction shows that the set P is bounded
in X and hence (3) is proved.
Since X is a -space, the set K D P is compact; given any f 2 IK we have
f .S / D f0g and hence .f / D 0 for all  2 nf0g. If f B then we can
apply Problem 223 to find a continuous linear functional  W Ck .X / ! R such that
.B/  I and .f / > 1. Such a functional  must belong to nf0g, so .f / D 0;
this contradiction shows that f 2 B. The function f 2 IK was chosen arbitrarily,
so we proved that IK  B.
Now take any f 2 K; 3
; then 2f 2 K; 23
and we can apply Fact 1 of V.321
to find a function g 2 C.X / such that gjK D f jK and jjgjjX  jjf jjK  13 .
This implies that jj2gjjX  23 < and hence 2g 2 X;
 B. Furthermore,
2f  2g 2 IK , so again 2g  2f 2 B. By convexity of B we have f D 12 2g C
1
.2f  2g/ 2 B, i.e., we established that K; 3
 B. Now apply Fact 7 of V.318
2
to conclude that there is a set W 2 .Ck .X // such that 0 2 W  K; 3
 B. Thus
B is a neighborhood of 0, so we proved that the space Ck .X / is barreled; this settles
sufficiency.
Finally observe that if X is a realcompact space then it is a -space by Fact 1 of
V.246, so Ck .X / is barreled and hence our solution is complete.
V.324. Prove that Cp .X / is barreled if and only if all bounded subspaces of X are
finite.
Solution. For any Q  X and " > 0 let Q; "
D ff 2 C.X / W f .Q/  ."; "/g.
Assume that Cp .X / is barreled and take a bounded set P in the space X . It is easy
to see that the set B D ff 2 Cp .X / W f .P /  Ig is a barrel in Cp .X /, so we can
find a set U 2 .Cp .X // for which 0 2 U  B. There exists a finite set K  X
such that K; "
 U for some " > 0.

362

2 Solutions of problems 001500

If P nK ; then fix a point x 2 P nK; there exists a function f 2 Cp .X / such


that f .x/ D 2 and f .K/  f0g. Therefore f 2 K; "
 B while f .x/ 2 f .P /
does not belong to I. This contradiction shows that P  K and hence every bounded
set P  X is finite, i.e., we proved necessity.
Now if every bounded subset of X is finite then X is a -space, so Ck .X / is
barreled by Problem 323. However, every compact subset of X is finite, so it is
immediate from the definitions that .Ck .X // D .Cp .X //. Therefore the space
Cp .X / is also barreled and hence we settled sufficiency.
V.325. Give an example of a space X such that Cp .X / barreled but does have the
Baire property.
Solution. It was proved in TFS-285 that there exists a space X in which all bounded
subsets are finite while Cp .X / does not have the Baire property. By Problem 324,
the space Cp .X / is barreled so X is the required example.
V.326. Given a point z in a space Z, say that a family B of subsets of Z is a local
base of neighborhoods of Z at the point z, if z 2 Int.U / for any U 2 B, and
z 2 V 2 .Z/ implies U  V for some U 2 B. Prove that the family of all barrels
in Ck .X / constitutes a local base of neighborhoods of 0 in Cb .X /. Deduce from this
fact that the family of all barrels in Cp .X / is also a local base of neighborhoods of
0 in Cb .X /.
Solution. We are going to need the sets O.A; "/ D ff 2 C.X / W f .A/  ."; "/g
and Q.A; "/ D ff 2 C.X / W f .A/  "; "
g for any A  X and " > 0. Let Bp
be the family of all barrels in Cp .X / and denote by Bk the family of all barrels in
Ck .X /; it is straightforward that Bp  Bk .
Fact 1. If Z is a space and A  Z then the restriction map  W C.Z/ ! C.A/ is
continuous as a map between Ck .Z/ and Ck .A/. If we consider that  maps Cb .X /
to Cb .A/ then  is also continuous.
Proof. The map  is linear, so it suffices to show its continuity at 0 (see Fact 2
of S.496). Assume that U is a neighborhood of 0 in the space Ck .A/ (or in the
space Cb .A/ respectively). There exists a compact (bounded) set K  A such that
B D ff 2 C.A/ W f .K/  ."; "/g  U for some " > 0. Consider the set
V D ff 2 C.Z/ W f .K/  ."; "/g; if W is the interior of V in the space
Ck .Z/ (or in Cb .Z/ respectively) then 0 2 W (see Fact 7 of V.318). Since also
.W /  B  U , the set W witnesses continuity of the map  W Ck .Z/ ! Ck .A/
(or continuity of the map  W Cb .Z/ ! Cb .A/ respectively), so Fact 1 is proved.
Returning to our solution observe that
(1) Q.A; "/ is a barrel in Cp .X / and hence in Ck .X / for every bounded set A  X .
Assume first that U is a neighborhood of 0 in the space Cb .X /. There exists a
bounded set A  X such that O.A; "/  U for some " > 0. Then B D Q.A; 2" / is
a barrel in Cp .X / such that B  O.A; "/  U . Since also B 2 Bk , we proved that
both families Bp and Bk are networks at the point 0 in the space Cb .X /; therefore it
suffices to show that 0 belongs to the interior in Cb .X / of every barrel in Ck .X /.

2 Solutions of problems 001500

363

Fix any B 2 Bk ; by Fact 1, the restriction map  W Ck .X / ! Ck .X / is


continuous. Since  is also linear, the set B 0 D  1 .B/ is a barrel in Ck .X /.
Now, X is a -space by Fact 1 of V.246, so we can apply Problem 323 to see
that Ck .X / is barreled and therefore the set B 0 is a neighborhood of 0 in Ck .X /.
Consequently, we can find a compact set K  X and a number " > 0 such that
H D ff 2 C.X / W f .K/  ."; "/g  B 0 . The set A D K \ X is bounded in X ;
let us show that
(2) Q.A; 2" / is contained in the closure E of the set .H / in the space Ck .X /.
Fix a function f 2 Q.A; 2" / and a set U 2 .f; Ck .X //. There exists a compact
set M  X such that f C M;
 U for some > 0. The function f can be
continuously extended over the space X ; let g 2 C.X / be the unique extension
of f . The set F D clX .A/  K is compact and g.F /   2" ; 2"
. The space K
is compact and hence normal, so there exists a function h W K !  2" ; 2"
such that
hjF D gjF . Let d.x/ D h.x/ for each x 2 K; if x 2 M nK then d.x/ D g.x/.
Observe that P D K \ M D A \ M is compact and hjP D gjP . This, together
with Fact 2 of T.354 shows that the function d W K [ M ! R is continuous and
hence we can find a function q 2 C.X / for which qj.K [ M / D d .
It follows from qjM D f jM that .q/ 2 f C M;
 U . Besides, we have
the inclusions q.K/   2" ; 2"
 ."; "/, so q 2 H and hence .q/ 2 .H / \ U .
Thus every neighborhood in Ck .X / of each function f 2 Q.A; 2" / intersects the set
.H /, so (2) is proved.
The set B is closed in the space Ck .X / and .H /  B, so E  B and hence
O.A; 2" /  Q.A; 2" /  E  B. Finally, apply Fact 7 of V.318 to see that 0 belongs
to the interior of O.A; 2" / in the space Cb .X / and hence the interior of the set B in
Cb .X / also contains 0. The set B 2 Bk was chosen arbitrarily, so we checked that
both families Bk and Bp are local bases of neighborhoods of 0 in the space Cb .X /.
V.327. Let ' W Ck .X / ! Cp .Y / be a linear continuous map. Prove that ' is
continuous considered as a map from Cb .X / to Cb .Y /.
Solution. Take a set U 2 .0; Cb .Y // and apply Problem 326 to find a barrel B
in the space Cp .Y / such that B  U . The set B 0 D ' 1 .B/ is easily seen to be a
barrel in Ck .X /, so we can apply Problem 326 again to find a set W 2 .0; Cb .X //
such that W  B 0 . Then '.W /  B  U , so the set W witnesses continuity of the
map ' W Cb .X / ! Cb .Y / at the point 0. Applying Fact 2 of S.496 we conclude that
the map ' W Cb .X / ! Cb .Y / is continuous.
V.328. Assuming that a space Y is l-equivalent to a bf -space X prove that Y is
also a bf -space. In other words, bf -property is l-invariant.
Solution. Fix a linear homeomorphism ' W Cp .X / ! Cp .Y /. It follows from
Problem 327 that ' W Cb .X / ! Cb .Y / is also a linear homeomorphism. The space
Cb .X / is complete as a uniform space with its linear uniformity (see Problem 317),
so Cb .Y / is also complete with respect to its linear uniformity. Applying Problem 317 again we conclude that Y is a bf -space.

364

2 Solutions of problems 001500

V.329. Let ' W Lp .X / ! Lp .Y / be a linear homeomorphism. Prove that, if X is a


bf -space then ' is a linear homeomorphism of L.X / onto L.Y /.
Solution. For any space Z let CZ D f 2 Cp .Lp .Z// W is a linear functional on
Lp .Z/g. For each f 2 Cp .Z/ there exists a unique linear functional eZ .f / 2 CZ
such that eZ .f /jZ D f . The map eZ W Cp .Z/ ! CZ is a linear homeomorphism
whose inverse is the restriction map Z W CZ ! Cp .Z/ (see Problem 235). If
P  C.Z/ and " > 0 then P; "
Z D fu 2 L.Z/ W u.P /  ."; "/g.
Fact 1. If Z is a space, " > 0 and P  C.Z/ is an equicontinuous pointwise
bounded set then P; "
Z is a neighborhood of 0 in L.Z/.
S
Proof. Let W0 D P; 3"
Z and WnC1 D fuCP; 3n2 "
Z W u 2 Wn g for all n 2 !.
"
If n D 3" C : : : C 3nC1
then it takes an easy induction to show
S that Wn  P; n
Z
for every n 2 !. Since n < " for all n 2 !, the set W D n2! Wn is contained
in P; "
Z . If u 2 W then u 2 Wn for some n 2 ! and hence u C P; 3n2 "

WnC1  W which, together with Problem 320, shows that W is open in L.Z/.
Since 0 2 W  P; "
Z , the set P; "
Z is a neighborhood of 0 in L.Z/, i.e., Fact 1
is proved.
Returning to our solution observe that the spaces X and Y are l-equivalent (see
Problem 237), so Y is also a bf -space by Problem 328. For any 2 CY let '  ./ D
'; it is easy to see that '  W CY ! CX is a linear homeomorphism, so the map
 D X '  eY W Cp .Y / ! Cp .X / is a linear homeomorphism as well. Given
any u 2 Lp .X / let   .u/ D u ; this gives us a map   W Lp .X / ! Lp .Y /.
Fix any u 2 Lp .X / and f 2 Cp .Y /; then   .u/.f / D u..f // D eX ..f //.u/.
Recall that eX ..f // D eX .X .'  .eY .f //// D '  .eY .f // D eY .f / '
and therefore   .u/.f / D eY .f /.'.u// D '.u/.f /. This shows that the linear
functional   .u/ coincides with '.u/ for each u 2 Lp .X / and hence the map  
coincides with '. To sum up, we proved that
(1) there exists a linear homeomorphism  W Cp .Y / ! Cp .X / such that ' D   ,
i.e., ' is the dual map of .
It is an easy consequence of Problem 327 that
(2) the map  W Cb .Y / ! Cb .X / is also a linear homeomorphism.
Fix a neighborhood U of 0 in L.Y /; by Problem 320 there exists an equicontinuous pointwise bounded set P  C.Y / such that P; "
Y  U . If F is the closure
of P in the space Cb .Y / then F is compact (see Problem 318). Therefore the set
G D .F /  Cb .X / is also compact by (2). Since X is a bf -space, it follows from
Problem 319 that G is equicontinuous and pointwise bounded.
By Fact 1, there is a set W 2 .L.X // such that 0 2 W  G; "
X . If w 2 W
and f 2 P then g D .f / 2 G and it follows from (1) that '.w/ D   .w/ D w ;
as a consequence, '.w/.f / D w..f // D w.g/ 2 ."; "/. This shows that we have
the inclusions '.W /  P; "
Y  U , i.e., the set W witnesses continuity of the
map ' W L.X / ! L.Y / at 0. By Fact 2 of S.496 the map ' W L.X / ! L.Y / is
continuous. An analogous proof shows that the map ' 1 W L.Y / ! L.X / is also
continuous, so ' is a linear homeomorphism between the spaces L.X / and L.Y / as
promised.

2 Solutions of problems 001500

365

V.330. Let X and Y be spaces one of which is a bf -space. Prove that X is


L-equivalent to Y if and only if X and Y are l-equivalent.
l

Solution. If X is L-equivalent to Y then X  Y by Problem 316, so assume that


l

X  Y . It follows from Problem 328 that both X and Y are bf -spaces; apply
Problem 237 to see that there exists a linear homeomorphism ' W Lp .X / ! Lp .Y /.
By Problem 239, the map ' W L.X / ! L.Y / is also a linear homeomorphism, so
the spaces X and Y are L-equivalent.
V.331. Prove that there exist l-equivalent spaces which are not L-equivalent.
Solution. Let D be a discrete space of cardinality !1 ; the spaces X D Lp .D/ and
Y D Lp .D/ D are l-equivalent by Problem 278. Fix an arbitrary n 2 N; for any
x D .x1 ; : : : ; xn / 2 X n and D . 1 ; : : : ; n / 2 Rn let n .x; / D 1 x1 C : : : C
n xn . This gives us a map n W X n  Rn ! L.X /; since X  L.X / and L.X / is a
linear topological space, every map n is continuous.
The cardinal !1 is a precaliber of X by Problem 236; therefore !1 is a precaliber
of X n  Rn and hence !1 is also a precaliber
of Pn D n .X n  Rn / for all n 2 N. It
S
follows from the equality L.X / D fPn W n 2 Ng that !1 is a precaliber of L.X /
and, in particular, the space L.X / has the Souslin property.
Consider the space Cu .D/ and let jjf jj D supfjf .x/j W x 2 Dg for every
function f 2 Cu .D/. If d.f; g/ D jjf  gjj for all f; g 2 Cu .D/ then d is a
metric which generates the topology of the space Cu .D/. For each f 2 Cu .D/ and
r > 0 the set B.f; "/ D fg 2 Cu .D/ W jjg  f jj < "g is the open ball of radius "
centered at f .
Given d 2 D let fd .d / D 1 and fd .x/ D 0 for all x 2 Dnfd g. Since all
functions are continuous on D, the set E D ffd W d 2 Dg is contained in Cu .D/. It
is easy to see that the family fB.fd ; 12 / W d 2 Dg is disjoint, so if E  T  Cu .D/
then the Souslin number of T is uncountable.
There exists a continuous onto map ' W Y ! E because D is a retract of Y
which can be mapped continuously onto anything of cardinality at most !1 . We can
consider that ' W Y ! Cu .D/; since Cu .D/ is a locally convex space, we can apply
Problem 312 to find a continuous map  W L.Y / ! Cu .D/ such that jY D '
and, in particular, T D .L.Y //  E. By the above observation, c.T / > !, so
c.L.Y // > !; recalling that c.L.X // D ! we conclude that the spaces L.X / and
L.Y / are not even homeomorphic. Thus X and Y are l-equivalent spaces which fail
to be L-equivalent.
V.332. Prove that a space X has a weaker metrizable topology if and only if L.X /
has a weaker metrizable topology. In particular, if X and Y are L-equivalent and
X can be condensed onto a metrizable space then Y can also be condensed onto a
metrizable space.
Solution. Given a space Z say that a set A  C.Z/ is adequate if A is
equicontinuous and pointwise bounded. Recall that the phrases Z has a weaker
metrizable topology and Z condenses onto a metrizable space say the same
thing.

366

2 Solutions of problems 001500

Fact 1. Suppose that Z is a space and we are given a metric  on the set Z such that
.x; y/  1 for all x; y 2 Z. Assume also that  generates a topology  .Z/.
Then the set E./ D ff 2 C.Z/ W jf .x/j  1 and jf .x/  f .y/j  .x; y/ for all
x; y 2 Zg is adequate, closed in Cp .Z/ and separates the points of Z.
Proof. If g 2 Cp .Z/nE./ then there exist x; y 2 Z such that jg.x/  g.y/j >
.x; y/. The set U D ff 2 Cp .Z/ W jf .x/  f .y/j > .x; y/g is easily seen to be
open in Cp .Z/; since also g 2 U  Cp .Z/nE./, the set E./ is closed in Cp .Z/.
If x 2 Z then ff .x/ W f 2 E./g  I, so the set E./ is pointwise bounded.
Finally, fix x 2 Z and " > 0; the set B D fy 2 X W .x; y/ < "g is an open
neighborhood of x. If y 2 B and f 2 E./ then jf .y/  f .x/j  .x; y/ < ",
so f .y/  .f .x/  "; f .x/ C "/, i.e., the set B witnesses equicontinuity of E./.
Therefore the set E./ is adequate.
Given distinct points a; b 2 Z let f .x/ D .a; x/ for all x 2 Z. It is
straightforward that f 2 E./; since also f .a/ D 0 f .b/, the set E./ separates
the points of Z and hence Fact 1 is proved.
Returning to our solution observe that X embeds in L.X /, so if L.X / has a
weaker metrizable topology then X also has one, i.e., we trivially have sufficiency.
Now assume that X has a weaker metrizable topology and hence there exists a
metric  on the set X such that .x; y/  1 for all x; y 2 X and the topology
generated by  is contained in .X /. The set E./ D ff 2 C.X / W jf .x/j  1
and jf .x/  f .y/j  .x; y/ for all x; y 2 X g is adequate, closed in Cp .X /
and separates the points of X (see Fact 1). For any x 2 X and f 2 E./ let
'.x/.f / D f .x/; this gives us a map ' W X ! Cu .E.// which is continuous by
Fact 9 of V.318. The space Cu .E.// is locally convex, so there exists a continuous
linear mapping W L.X / ! Cu .E.// such that jX D ' (see Problem 312).
If u 2 L.X /nf0g then there exist distinct points x1 ; : : : ; xn 2 X and numbers
1 ; : : : ; n 2 Rnf0g such that u D 1 x1 C : : : C n xn . If n D 1 then let f .x/ D 1
for all x 2 X ; it is clear that f 2 E./ and .u/.f / D 1 f .x1 / D 1 0, so
.u/ 0. If n > 1 then let f .x/ D nnf.x; xi / W 2  i  ng for every x 2 X .
Given any points x; y 2 X there exist i; j 2 f2; : : : ; ng such that f .x/ D
.x; xi / and f .y/ D .x; xj /. The inequalities .x; xi /  .x; xj /  .x; y/ C
.y; xj / imply that f .x/  f .y/ D .x; xi /  .y; xj /  .x; y/. The points
x and y are in a symmetric situation so f .y/  f .x/  .x; y/ and hence
jf .x/  f .y/j  .x; y/ for all x; y 2 X which shows that f 2 E./. It is
easy to see that .u/.f / D 1 .x1 ; xi / for some i 2 f2; : : : ; ng, so .u/.f / 0
and hence .u/ 0. Thus .u/ 0 for all u 2 L.X /nf0g which implies that
W L.X / ! Cu .E.// is injective. The space Cu .E.// is metrizable, so L.X /
has a weaker metrizable topology, i.e., we established necessity.
Finally, if X and Y are L-equivalent spaces and X condenses onto a metrizable
space then so does L.X /. Therefore L.Y / also condenses onto a metrizable space;
being a subspace of L.Y /, the space Y must be condensable onto a metrizable space.
V.333. Suppose that X and Y are l-equivalent spaces. Prove that, if X is
metrizable, then Y can be condensed onto a metrizable space.

2 Solutions of problems 001500

367

Solution. Fix a linear homeomorphism ' W Lp .X / ! Lp .Y / (see Problem 237).


Any metrizable space is easily seen to be a bf -space so X is a bf -space and hence
we can apply Problem 329 to see that ' is a linear homeomorphism between the
spaces L.X / and L.Y /. Therefore the spaces X and Y are L-equivalent and hence
Y is condensable onto a metrizable space by Problem 332.
V.334. Say that a space X is  -metrizable if X is the countable union of its closed
metrizable subspaces. Prove that a space X is  -metrizable and paracompact if and
only if L.X / is  -metrizable and paracompact.
Solution. If Z is a space, A is a family of subsets of Z and Y  Z then we let
AjY D fA \ Y W A 2 Ag. A topology on the set Z is called a -approximation
for the space Z if  .Z/ and there
S is a countable family P of closed metrizable
subspaces of .Z; / such that Z D P and jP D .Z/jP for any P 2 P.
Let J be the interval .0; 1
 R. Given a nonempty set T let Jt D J  t for all
t 2 T . Choose a point  J  T and let H.T / D .J  T / [ fg. For any t 2 T if
x D .a; t / 2 Jt and y D .b; t / 2 Jt then we let d.x; y/ D ja bj. If t; s 2 T; t s
and x D .a; t / 2 Jt ; y D .b; s/ 2 Js then d.x; y/ D a C b. If t 2 T and
x D .a; t / 2 Jt then let d.x; / D a; finally, letting d.; / D 0 we obtain a metric
space .H.T /; d / which is called Kowalsky hedgehog (see TFS-222). Consider the
set It D Jt [ fg with the topology induced from H.T /; for any x D .a; t / 2 Jt
let ht .x/ D a and ht ./ D 0. It is straightforward that ht W It ! I D 0; 1
is a
homeomorphism for each t 2 T .
Fact 1. For any space Z, if a topology is a -approximation of Z then there
existsS
a family fPn W n 2 !g of metrizable closed subspaces of .Z; / such that
Z D n2! Pn while Pn  PnC1 and jPn D .Z/jPn for every n 2 !.
Proof. Take
S a family Q D fQn W n 2 !g of closed metrizable subspaces of .Z; /
such that Q D Z and jQn D .Z/jQn for all n 2 !. If Pn D Q0 [ : : : [ Qn
then it is easy to represent Pn as a perfect image of the space Q0 : : : Qn , so Pn
is metrizable for each n 2 ! (see TFS-226).
If A  Pn is a closed subset of Z then Ai D
S A \ Qi is closed in Qi and hence in
.Z; / for all i  n; as a consequence, A D in Ai is closed in .Z; / and hence
.Z/jPn D jPn for all n 2 !. This shows that the sequence fPn W n 2 !g is as
promised, i.e., Fact 1 is proved.
Fact 2. Given a space Z, the following properties are equivalent:
(i) Z is  -metrizable and paracompact;
(ii) Z has a metrizable -approximation;
(iii) Z has a paracompact -approximation.
Proof. Assume that Z is  -metrizable and paracompact; then we can choose a
sequence fMn W n 2 !g of
S metrizable closed subspaces of Z such that Mn  MnC1
for all n 2 ! and Z D n2! Mn . If U is an open subset of Z then U \ Mn is an
F -set in Mn and hence in Z for every n 2 !; therefore U is an F -subset of Z and
hence Z is perfectly normal.

368

2 Solutions of problems 001500

S For any n 2 ! the space Mn is metrizable, so it has a base Bn such that Bn D


fCnm W m 2 !g and every family Cnm is discrete in Mn ; discarding trivial cases we
can consider that Cnm ; and the elements of Cnm are nonempty for all n; m 2 !.
Fix n; m 2 ! and choose a faithful enumeration fCt W t 2 Tnm g of the family
Cnm . It is easy to see that the family fC t W t 2 Tnm g is discrete in Z, so we can use
paracompactness of Z to find a discrete collection fOt W t 2 Tnm g  .Z/ such
that C t  Ot for all t 2 Tnm . Pick an open set Wt  Ot such that Wt \ Mn D Ct
for all t 2 Tnm . By perfect normality of Z we can find a function ft 2 C.Z; I /
such that ft 1 .J / D Wt for all t 2
S Tnm . If x 2 W t for some t 2 Tnm then let
'nm .x/ D h1
fW t W t 2 Tnm g then let 'nm .x/ D . This
t .ft .x//; if x 2 Zn
gives us a map 'nm W Z ! H.Tnm /. Applying Fact 2 of T.354 it is easy to see that
'nm is continuous.
For each n 2 ! apply perfect normality of Z once more to pick a function
qn 2 C.Z; I / such that Mn D qn1 .0/. If F D f'nm W n; m 2 !g [ fqn W n 2 !g
then the
Qdiagonal product  of the family F maps Z continuously into the metrizable
space fH.Tnm / W n; m 2 !g  I ! ; therefore the space Y D .Z/ is metrizable.
Let nm W Y ! H.Tnm / and pn W Y ! I be the respective natural projections for
all n; m 2 !.
Given distinct x; y 2 Z there exists n 2 ! such that fx; yg  Mn . Since Bn
is a base in Mn , we can find m 2 ! and t 2 Tnm for which x 2 Ct and y Ct .
Then ft .y/ D 0 and ft .x/ 0; therefore 'nm .x/ 2 Jt while 'nm .y/ D . Thus
'nm .x/ 'nm .y/ and hence .x/ .y/; this shows that  is a condensation.
Let Mn0 D .Mn /; if y 2 Y nMn0 then y D .x/ for some x 2 ZnMn and hence
qn .x/ > 0 which implies that y 2 G D pn1 .J /, i.e., G is an open neighborhood
of y in the space Y . For every z0 2 Mn0 there is z 2 Mn with z0 D .z/; as a
consequence, pn .z0 / D qn .z/ D 0, so z0 G and hence G \ Mn0 D ;. Therefore
(1) the set Mn0 is closed in Y for every n 2 !.
Next fix n 2 !, a point y 2 Mn0 and let x D  1 .y/; then x 2 Mn . Take an
arbitrary set U 2 .x; Mn /. The family Bn being a base in Mn there exists m 2 !
and t 2 Tnm such that x 2 Ct  U . Therefore 'nm .x/ 2 Jt and hence the set
1
G D nm
.Jt / is an open neighborhood of y in Y . Suppose that z0 2 Mn0 \ G and
1 0
z D  .z /; it follows from nm .z0 / 2 Jt that 'nm .z/ 2 Jt and therefore ft .z/ 2 J ,
i.e., z 2 Wt \ Mn D Ct  U . This proves that  1 .G \ Mn0 /  U , i.e., the set
G \ Mn0 witnesses continuity of  1 jMn0 at the point y. As a consequence,
(2) the map jMn W Mn ! Mn0 is a homeomorphism for every n 2 !.
The properties (1) and (2) show that the sequence fMn W n 2 !g witnesses that
the topology D f 1 .U / W U 2 .Y /g is a metrizable -approximation for the
space Z, so we settled (i)H)(ii).
The implication (ii)H)(iii) being evident assume that Z has a paracompact
-approximation and hence we can apply Fact 1 to choose a paracompact topology
 .Z/ and a S
sequence fMn W n 2 !g of metrizable closed subsets of the space
.Z; / such that n2! Mn D Z while Mn  MnC1 and jMn D .Z/jMn for all
n 2 !. Let Bn be a  -discrete base in Mn ; observe that Fn D fB W B 2 Bn g is a

2 Solutions of problems 001500

369

S
 -discrete network in Mn for every n 2 !. Therefore F D n2! Fn is a  -discrete
network in the space .Z; / as well as in the space Z. Since .Z; / is paracompact,
we can choose, for each F 2 F, a set OF 2 such that F  OF and the family
fOF W F 2 Fg is  -discrete in .Z; / and hence in Z.
Now take an open cover U of the space Z; say that a set F 2 F is marked if
there exists UF 2 U such that F  UF . If x 2 Z then there exists U 2 U with
x 2 U ; since F is a network in Z, we can choose F 2 F such that x 2 F  U .
The set F is, evidently, marked so marked elements of F cover the space Z. It is
straightforward that the family V D fUF \ OF W F is a marked element of Fg is a
 -discrete refinement of U , so the space Z is paracompact (see TFS-230); since its
-approximation witnesses that Z is  -metrizable, we established (iii)H)(i) and
hence Fact 2 is proved.
Fact 3. Suppose that Z is a space and  .Z/ is a metrizable -approximation
of Z;Sfix a family M D fMn W n 2 !g of closed subsets of Z 0 D .Z; / such
that M D Z and jMn D .Z/jMn for all n 2 !. Denote by Ln the linear
hull of the set Mn in the linear space L.Z/ and let n and n0 be the topologies on
Ln induced from the spaces L.Z/ and L.Z 0 / respectively. The set Ln is closed in
L.Z 0 / and the identity map j W .Ln ; n / ! L.Mn / is a topological isomorphism;
if we consider that j W .Ln ; n0 / ! L.Mn / then j is a topological isomorphism as
well. In particular, n D n0 for every n 2 !.
Proof. Every set Ln is closed in Lp .Z 0 / (see Fact 2 of V.246) and hence in L.Z 0 /.
Observe that Ln and L.Mn / have the same underlying set and denote by the
topology of L.Mn / on Ln . The map j is clearly an isomorphism, so it suffices to
show that n D n0 D . It follows from  .Z/ that .L.Z 0 //  .L.Z// and
hence n0  n . Since Ln is a locally convex space, the identity map of Mn onto
Mn is extendable to a continuous linear map from L.Mn / onto .Ln ; n /. The set Mn
being a Hamel basis in Ln the mentioned linear extension is also the identity and
hence it coincides with j . Therefore the map j W L.Mn / ! .Ln ; n / is continuous
which shows that n  .
To see that the linear isomorphism j W .Ln ; n0 / ! L.Mn / is also continuous
apply SFFS-104 to find a continuous map  W Z 0 ! L.Mn / such that jMn D j .
By Problem 312 there exists a continuous linear map W L.Z 0 / ! L.Mn / such
that jZ 0 D . The set Mn being a Hamel basis in the space Ln we must have
jLn D j , so the map j W .Ln ; n0 / ! L.Mn / is, indeed, continuous. This implies
that  n0 and hence we have n D n0 D , i.e., Fact 3 is proved.
Fact 4. If Z is a metrizable space then L.Z/ is  -metrizable and paracompact.
Proof. Fix a metric  on the set Z such that .x; y/  1 for all x; y 2 Z and .Z/
is generated by . The set E D ff 2 C.X; I/ W jf .x/  f .y/j  .x; y/ for all
x; y 2 Zg is pointwise bounded, equicontinuous and separates the points of X (see
Fact 1 of V.332). Let .x/.f / D f .x/ for any f 2 E and x 2 Z; this gives us a
continuous injective map  W Z ! Cu .E/ (see Fact 9 of V.318). Let Y D .Z/
and denote by H the linear hull of Y in the space Cu .Z/. We will establish first that
E has sufficiently many functions for our purposes, namely, that

370

2 Solutions of problems 001500

(3) for any closed A  Z if dA .x/ D inff.x; a/ W a 2 Ag for every x 2 Z (if


A D ; then dA .x/ D 1 for all x 2 Z) then dA 2 E.
Indeed, this is evident if A D ;; if A ; take any points x; y 2 Z and " > 0.
There exists a point a 2 A such that dA .y/ > .y; a/  "; it follows from the
inequalities dA .x/  .x; a/  .x; y/ C .y; a/  .x; y/ C dA .y/ C " that
dA .x/  dA .y/  .x; y/ C ". Since " > 0 was chosen arbitrarily, we proved that
dA .x/  dA .y/  .x; y/. The points x and y are in a symmetric situation, so
dA .y/  dA .x/  .y; x/ D .x; y/ and hence jdA .x/  dA .y/j  .x; y/ for all
x; y 2 Z, i.e., (3) is proved.
Fix a point y 2 Y , let z D  1 .y/ and take an arbitrary set U 2 .z; Z/. The
function f D dZnU belongs to E by (3). The set V D fv 2 C  .E/ W v.f / > 0g
is open in Cp .E/ and hence in Cu .E/. Besides, y.f / D f .z/ D d.z; ZnU / > 0,
so y 2 V . If y 0 2 V \ Y and z0 D  1 .y 0 / then f .z0 / D y 0 .f / > 0 which implies
z0 2 U . This proves that  1 .V \ Y /  U , i.e., the set V \ Y witnesses continuity
of  1 at the point y. Therefore  1 W Y ! Z is continuous and hence
(4) the map  W Z ! Y is a homeomorphism.
Assume that y1 ; : : : ; yn are distinct points of Y and t1 ; : : : ; tn 2 Rnf0g; let xi D
 1 .yi / for all i  n. Consider the set A D fx1 ; : : : ; xn gnfx1 g; then the function
f D dA belongs to E by (3). If v D t1 y1 C : : : C tn yn then it is evident that
v.f / D t1 f .x1 / C : : : C tn f .xn / D t1 f .x1 / 0 and hence v 0. This proves that
(5) the set Y is linearly independent in C  .E/ and hence Y is a Hamel basis of H .
For the point 0 2 H let l.0/ D 0. If h 2 H nf0g then there exists n 2 N and
uniquely determined distinct points y1 ; : : : ; yn 2 Y such that h D t1 y1 C : : : C tn yn
for some t1 ; : : : ; tn 2 Rnf0g; let l.h/ D n. Fix n 2 ! and let Hn D fh 2 H W l.h/ 
ng. If u 2 H nHn then there is k > n such that u D t1 y1 C : : : C tk yk for some
distinct y1 ; : : : ; yk 2 Y and t1 ; : : : ; tk 2 Rnf0g.
Let xi D  1 .yi / and pick a set Ui 2 .xi ; Z/ for all i  k in such a way
that the family fU1 ; : : : ; Uk g is disjoint. Consider the function fi D dZnUi for all
i  k. The set W D fh 2 H W h.fi / 0 for all i  kg is open in H ; we have
u.fi / D ti fi .xi / 0 for all i  k, so u 2 W . If v D s1 z01 C : : : C sm z0m 2 W then
let zi D  1 .z0i / for every i  m and consider the set A D fz1 ; : : : ; zm g. It follows
from v.fi / 0 that A \ Ui ; for all i  k. Since the sets Ui are disjoint, there
are at least k-many distinct elements in A and hence in fz01 ; : : : ; z0m g, so v Hn .
Therefore u 2 W  H nHn , i.e., every u 2 H nHn has a neighborhood which does
not meet Hn ; as a consequence,
(6) the set Hn is closed in H for every n 2 !.
Fix any n 2 N and consider the set Gn D Hn nHn1 . Take a disjoint family
U D fU1 ; : : : ; Un g of nonempty open subsets of Y and sets O1 ; : : : ; On 2 .Rnf0g/.
We claim that the set W D O1 U1 C : : : C On Un is open in Gn .
To prove it take any u 2 W ; there exist y1 ; : : : ; yn 2 Z and t1 ; : : : ; tn 2 Rnf0g
such that yi 2 Ui ; ti 2 Oi for all i  n and u D t1 y1 C : : : C tn yn . The family

2 Solutions of problems 001500

371

U being disjoint the points y1 ; : : : ; yn are distinct and hence u 2 Gn . This proves
that W  Gn . Let xi D  1 .yi /; Vi D  1 .Ui / and fi D dZnVi ; then fi 2 E
while ri D fi .xi / 0 and fi .ZnVi /  f0g for all i  n. The multiplication and
taking the inverse are continuous operations in R, so it follows from the equality
ti ri ri1 D ti that
(7) for every i  n, there exist sets Pi ; Qi 2 .Rnf0g/ such that ti ri 2 Pi ; ri 2 Qi
and Pi .Qi /1  Oi .
It follows from (7) that xi 2 Di D fi1 .Qi /; if gi D dZnDi then gi 2 E and
gi .ZnDi /  f0g for all i  n. The set Ni D fv 2 H W v.fi / 2 Pi and v.gi / 0g
is open in H ; furthermore, u.fi / D ti fi .xi / D ti ri and u.gi / D ti gi .xi / 0 (we
used the
T fact that U is disjoint and Di  Vi ), so u 2 Ni for every i  n. Therefore
N D in Ni 2 .u; H /.
Suppose that v 2 N \ Gn and hence l.v/ D k  n; then v D s1 z01 C : : : C sk z0k
for distinct points z01 ; : : : ; z0k 2 Y and s1 ; : : : ; sk 2 Rnf0g. For each i  n let
zi D  1 .z0i /; we have v.fi / D s1 fi .z1 / C : : : C sk fi .zk / 0, so there is mi  k
such that fi .zmi / 0 and hence zmi 2 Vi . The family fV1 ; : : : ; Vn g being disjoint,
the points zm1 ; : : : ; zmn have to be distinct, so k D n and we can change the order
of summation if necessary to be able to assume, without loss of generality, that
v D s1 z01 C : : : C sn z0n and z0i 2 Ui for all i  n.
Now, v.gi / D si gi .zi / 0 implies that gi .zi / 0 and hence zi 2 Di ; an
immediate consequence is that fi .zi / 2 Qi . Since we also have v.fi / D si f .zi / 2
Pi , we conclude that si 2 Pi Qi1  Oi for any i  n. Thus v 2 W which shows
that any point u 2 W has a neighborhood N in H such that N \ Gn  W ; an
immediate consequence is that W is open in Gn and hence we proved that
(8) if a family fU1 ; : : : ; Un g   .Y / is disjoint and O1 ; : : : ; On 2 .Rnf0g/ then
the set O1 U1 C : : : C On Un is open in Gn .
Let i.y/ D y for every y 2 Y ; then the map i W Y ! H is continuous, so we
can apply Problem 312 to find a continuous linear map j W L.Y / ! H such that
j jY D i . It follows from (5) that j is a linear isomorphism, so we can consider that
the underlying set of L.Y / coincides with H while .H /  .L.Y //.
Fix n 2 N and let 0 D .H /jGn ; 1 D .L.Y //jGn ; it is evident that 0  1 . If
W 2 1 and u 2 W then there is a set W 0 2 .L.Y // such that W 0 \Gn D W . There
exist distinct y1 ; : : : ; yn 2 Y and t1 ; : : : ; tn 2 Rnf0g for which u D t1 y1 C: : :Ctn yn .
Choose disjoint sets U10 ; : : : ; Un0 2 .Y / such that yi 2 Ui0 for all i  n. It follows
from continuity of linear operations in the space L.Y / that, for each i  n there
exist Ui00 2 .yi ; Y / and Oi 2 .ti ; Rnf0g/ such that O1 U100 C : : : C On Un00  W 0 .
Let Ui D Ui0 \ Ui00 for all i  n. Then the family fU1 ; : : : ; Un g is disjoint and
u 2 V D O1 U1 C : : : C On Un  W 0 \ Gn D W . The property (8) shows that the
set V belongs to 0 . This proves that, for any point u 2 W there exists V 2 0 such
that u 2 V  W ; an immediate consequence is that W 2 0 . In other words,
(9) the topologies .H /jGn and .L.Y //jGn coincide for all n 2 N.
The space H is metrizable, so every Gn is an F -set in H , S
i.e., there exists a
countable family Gn of closed subsets of H such that Gn D
Gn . The family

372

2 Solutions of problems 001500

S
S
G D f0g [ . fGn W n 2 Ng/ is countable, H D
G and it follows from (9)
that .H /jP D .L.Y //jP for every P 2 G. Therefore H is a metrizable approximation of L.Y / and hence L.Y / is  -metrizable and paracompact by Fact 2.
Since Z is homeomorphic to Y by (4), the space L.Z/ is also  -metrizable and
paracompact, i.e., Fact 4 is proved.
Returning to our solution observe that if L.X / is paracompact and  -metrizable
then so is X because X is a closed subspace of L.X / (see Problem 311); this proves
sufficiency.
Now if X is  -metrizable and paracompact then we can apply Fact 1 and Fact 2 to
find a metrizable topology  .X / such that there exists a sequence
fMn W n 2 !g
S
of closed subspaces of the space X 0 D .X; / for which n2! Mn D X while
Mn  MnC1 and jMn D .X /jMn for all n 2 !. Apply Fact 4 to see that the space
L.X 0 / is paracompact and denote by Ln the linear hull of the set Mn in L.X 0 /. For
every n 2 ! apply Fact 3 to see that
(10) the set Ln is closed in L.X 0 / and .L.X 0 //jLn D .L.X //jLn and, besides,
Ln is homeomorphic to L.Mn /.
Apply Fact 4 toSfind a countable family S
Ln of closed metrizable subsets of Ln
such that Ln D
Ln . The family L D n2! LS
n is countable and consists
S of
closed metrizable subsets of L.X 0 /. It follows from n2! Ln D L.X 0 / that L D
L.X 0 /; apply (10) to conclude that .L.X //jA D .L.X 0 //jA for any A 2 L. Thus
.L.X 0 // is a paracompact -approximation of L.X /, so we can apply Fact 2 to
see that L.X / is  -metrizable and paracompact. This settles necessity and makes
our solution complete.
V.335. Suppose that a space X is l-equivalent to a metrizable space. Prove that X
is  -metrizable and paracompact.
Solution. If X is l-equivalent to a metrizable space M then X is L-equivalent to M
because M is a bf -space (see Problem 330). By Problem 334, the space L.M / is  metrizable and paracompact and hence so is L.X /. Therefore X is also  -metrizable
and paracompact being a closed subspace of L.X /.
V.336. For an arbitrary space X , prove that X is hemicompact if and only if Ck .X /
is first countable.
Solution. For any A  X and O  R let A; O
D ff 2 Ck .X / W f .A/  Og. If
X is hemicompact then
S we can find a sequence fKn W n 2 !g of compact subsets
of X such that X D n2! Kn and every compact subset of X is contained in some
Kn . The set O.n; m/ D Kn ; .2m ; 2m /
is a neighborhood of 0 for all n; m 2 !
(see Fact 7 of V.318), so it suffices to prove that O D fInt.O.n; m// W n; m 2 !g
is a local base of Ck .X / at 0. To do it take any set U 2 .0; Ck .X //; there exists
a compact set K  X such that K; ."; "/
 U . Choose n; m 2 ! such that
K  Kn and 2m < "; it is straightforward that O.n; m/  K; ."; "/
 U , so
0 2 Int.O.n; m//  U which shows that the family O is a countable local base of
Ck .X / at 0, i.e., we proved necessity.

2 Solutions of problems 001500

373

Now assume that W D fWn W n 2 !g is a local base of Ck .X / at 0; for each


n 2 ! there exists a compact set Kn  X such that Gn D Kn ; ."n ; "n /
 Wn for
some "n > 0. If K  X is compact then the set W D K; .1; 1/
is a neighborhood
of 0 by Fact 7 of V.318. Since W is a local base at 0, there exists n 2 ! such that
Wn  W and hence Gn  W . If x 2 KnKn then we can find a function f 2 Ck .X /
for which f .x/ D 1 and f .Kn /  f0g. Then f 2 Gn nW ; this contradiction shows
that KnKn D ;, i.e., K  Kn and hence the sequence fKn W n 2 !g witnesses that
the space X is hemicompact. This settles sufficiency.
V.337. Prove that hemicompactness is preserved by l-equivalence.
Solution. Suppose that a space X is hemicompact and ' W Cp .X / ! Cp .Y / is
a linear homeomorphism. The space X is  -compact, so Y is also  -compact by
Problem 043. Since the topology of Ck .X / is stronger that the topology of Cp .X /,
the map ' W Ck .X / ! Cp .Y / is also continuous. Apply Problem 327 to see that
the map ' W Cb .X / ! Cb .Y / is continuous as well. Since both X and Y are
-spaces, we have .Ck .X // D .Cb .X // and .Ck .Y // D .Cb .Y // by Fact 6
of V.318. Thus the map ' W Ck .X / ! Ck .Y / is continuous; an analogous proof
shows that ' 1 W Ck .X / ! Ck .Y / must be continuous too, so the spaces Ck .X /
and Ck .Y / are homeomorphic. Finally apply Problem 336 to see that Ck .X / is first
countable; this implies that Ck .Y / is first countable as well and hence we can apply
Problem 336 again to conclude that Y is hemicompact. Therefore hemicompactness
l

of X together with X  Y imply that Y is hemicompact, i.e., hemicompactness is


preserved by l-equivalence.
V.338. Given a space X prove that
(i) if X is a k-space then a sequence ffn W n 2 !g  Ck .X / is convergent
whenever it is linearly Cauchy;
(ii) for a hemicompact space X the converse is true, i.e., if any linearly Cauchy
sequence in Ck .X / is convergent then X is a k-space.
Solution. If A  X and " > 0 then A; "
D ff 2 Ck .X / W f .A/  ."; "/g.
Suppose first that X is a k-space and a sequence ffn W n 2 !g  Ck .X / is linearly
Cauchy in Ck .X /. If x 2 X and " > 0 then the set W D fxg; "
is a neighborhood
of 0 in Cp .X / and hence in Ck .X /, so there exists m 2 ! such that fn  fk 2 W
for all n; k  m. This is equivalent to saying that jfn .x/  fk .x/j < " for all
n; k  m and therefore the sequence ffn .x/ W n 2 !g is convergent being Cauchy;
let f .x/ D lim fn .x/ for all x 2 X .
To prove that the function f W X ! R is continuous fix a compact set K  X
and " > 0. Apply Fact 7 of V.318 to see that K; 2"
is a neighborhood of 0, so there
exists m 2 ! such that fn  fk 2 K; 2"
for all n; k  m. Given any x 2 K we
have jfn .x/  fk .x/j < 2" for all k  m. Passing to the limit we conclude that
jfn .x/  f .x/j  2" < " for all x 2 K and hence
(1) the sequence ffn jK W n 2 !g converges uniformly to f jK for every compact
set K  X .

374

2 Solutions of problems 001500

As a consequence, the function f jK is continuous for any compact K 


X ; since X is a k-space, the function f must be continuous. Now take any
neighborhood U of the function f in Ck .X /; there exists a compact K  X such
that f C K; "
 U for some " > 0. The property (1) implies that there exists
m 2 ! such that jfn .x/  f .x/j < " for all n  m and x 2 K. This is the same as
saying that fn  f 2 K; "
and hence fn 2 f C K; "
 U for all n  m. Thus
the sequence ffn W n 2 !g converges to f in Ck .X / and hence we proved (i).
Now assume that X is hemicompact and every linearly Cauchy sequence of
Ck .X / is convergent. To prove that X is a k-space assume that A  X is a nonclosed set such that A \ K is closed in X for any compact K  X ; fix a point
y 2 AnA. There exists a sequence K D fKn W n 2 !g of compact subsets of X
such that Kn  KnC1 for all n 2 ! and every compact subset of X is contained in
an element of K; there is no loss of generality to assume that y 2 K0 . By our
S choice
of A the set An D A \ Kn is compact for all n 2 ! and hence the set A D n2! An
is  -compact.
Take a function f0 2 C.X / such that f0 .y/ D 1 and f0 .A0 /  f0g. Suppose that
n 2 ! and we have functions f0 ; : : : ; fn 2 C.X / such that fi .y/ D 1; fi .Ai / 
f0g and fiC1 jKi D fi for all i < n.
Let gnC1 .x/ D fn .x/ for all x 2 Kn and gnC1 .x/ D 0 whenever x 2 AnC1 nKn .
It is easy to prove, using Fact 2 of T.354, that the function gnC1 W Kn [ AnC1 ! R
is continuous and hence there exists a continuous function fnC1 W X ! R such that
fnC1 j.Kn [ AnC1 / D gnC1 j.Kn [ AnC1 /. This shows that our inductive procedure
can be continued to construct a sequence ffn W n 2 !g  Ck .X / such that
(2) fn .y/ D 1; fn .An /  f0g and fnC1 jKn D fn jKn for all n 2 !.
If x 2 X then x 2 Km for some m 2 !; it follows from (2) that fn .x/ D fk .x/
for all n; k  m and hence there exists f .x/ D lim fn .x/ for all x 2 X . The
property (2) also implies that fn jKm D f jKm for all n  m and hence the sequence
ffn jKm W n 2 !g uniformly converges to f jKm on for every m 2 !.
Take an arbitrary set U 2 .0; Ck .X //; there exists a compact K  X such
that K; "
 U for some " > 0. Recalling that the sequence K witnesses
hemicompactness of the space X we can find m 2 ! such that K  Km . Now
if n; k  m then fn jKm D fk jKm and hence fn jK D fk jK which, in turn, shows
that fn  fk 2 K; "
 U . Thus the sequence ffn W n 2 !g is linearly Cauchy,
so fn ! f in the space Ck .X / which implies that the function f is continuous.
However, it follows from (2) that f .y/ D 1 while f .A/  f0g; this contradiction
with y 2 A shows that X is a k-space, i.e., we settled (ii) and completed our
solution.
V.339. Let X be an arbitrary space. Prove that X is a hemicompact space with
k-property if and only if Ck .X / is metrizable by a complete metric.
Solution. Suppose first that Ck .X / is metrizable by a complete metric and hence

Cech-complete
(see TFS-269). Since Ck .X / is a linear topological space, its topology is generated by its linear uniformity L (see Problem 134). Apply Problem 121

2 Solutions of problems 001500

375

to find a complete uniform space .L; U / such that .Ck .X /; L/ is a dense uniform
subspace of .L; U /. For any pair .f; g/ 2 Ck .X /  Ck .X / let s.f; g/ D f C g and
i.f / D f .
The map s W Ck .X /  Ck .X / ! Ck .X / is continuous and linear, so we can
apply Problem 134 to see that it is uniformly continuous. Since Ck .X /  Ck .X /
is a dense uniform subspace of L  L and the space L  L is complete, we can
apply Problem 119 to find a uniformly continuous map s1 W L  L ! L such that
s1 jCk .X /  Ck .X / D s. Analogously, the map i W Ck .X / ! Ck .X / is linear and
hence uniformly continuous, so there exists a uniformly continuous map i1 W L ! L
such that i1 jCk .X / D i .
Let sQ .u; v/ D s1 .v; u/ for any u; v 2 L  L. Since addition is commutative
in Ck .X /, we have the equality sQ jCk .X /  Ck .X / D s D s1 jCk .X /  Ck .X /;
the space Ck .X /  Ck .X / being dense in L  L we conclude that sQ D s1 (see
Fact 0 of S.351), i.e., the operation s1 is commutative. Analogously, consider the
operations ;  W L3 ! L defined by .u; v; w/ D s1 .s1 .u; v/; v/ and .u; v; w/ D
s1 .u; s1 .v; w// for each .u; v; w/ 2 L3 . The addition is an associative operation, so
j.Ck .X //3 D j.Ck .X //3 ; applying Fact 0 of S.351 again we conclude that  D 
and hence the operation s1 is associative.
Now let s2 .u/ D s1 .0; u/ for any u 2 L; since s2 .f / D 0 C f D f for all
f 2 Ck .X /, we can apply Fact 0 of S.351 once more to conclude that s2 .u/ D u
for all u 2 L, so s2 W L ! L is an identity map and hence 0 is a zero element in L.
Furthermore, let .u/ D s1 .u; i.u// for all u 2 L; then W L ! L is a continuous
map such that .f / D f C .f / D 0 for all f 2 Ck .X /; thus s1 .u; i.u// D 0 for
every u 2 L.
All these considerations show that the operation s1 W L  L ! L makes L
a commutative group (actually, L is a linear topological space but we wont need
that). To simplify the notation we will write uCv instead of s1 .u; v/ and u in place
of i1 .u/.
For each u 2 L let 'u .v/ D u C v for all v 2 L; then 'u W L ! L is a continuous
map. It is clear that 'u is the inverse of 'u , so 'u is a homeomorphism for all
u 2 L. Suppose that L Ck .X / and fix a point u 2 LnCk .X /. If 'u .Ck .X // \
Ck .X / ; then there exist f; g 2 Ck .X / such that g D f C u and hence u D
g C .f / 2 Ck .X / which is a contradiction. Therefore '.Ck .X // and Ck .X / are

disjoint dense Cech-complete


subspaces of L; this contradicts TFS-264 and shows
that Ck .X / D L. As a consequence,
(1) the space Ck .X / is complete with respect to its linear uniformity.
Take a sequence ffn W n 2 !g  Ck .X / which is linearly Cauchy in Ck .X / and
let Pm D ffn W n  mg for all m 2 !. The family fPm W m 2 !g is, evidently,
a Cauchy filterbase,
so it follows from completeness of Ck .X / that there exists a
T
function f 2 fP m W m 2 !g. Fix an arbitrary set U 2 .0; Ck .X // and choose
V 2 .0; Ck .X // such that V  U . There exists m 2 ! for which fn  fk 2 V for
all n; k  m.
Take any n  m and consider the set fn  Pm  V . The shift operation is a
homeomorphism, so fn  Pm D fn  P m  V  U . This, together with f 2 P m

376

2 Solutions of problems 001500

shows that fn  f 2 V  U and hence fn 2 f C U for all n  m. Therefore the


sequence ffn W n 2 !g converges to f , i.e., we proved that a sequence in Ck .X / is
convergent whenever it is linearly Cauchy. Applying Problem 338 we conclude that
X is a k-space and hence we established sufficiency.
Finally, assume that X is a hemicompact k-space; it follows from Problem 336
that Ck .X / is first countable. By Problem 206 there exists an invariant metric d on
the set Ck .X / which generates the topology of Ck .X /. Suppose that S D ffn W n 2
!g is a Cauchy sequence with respect to the metric d . Given any U 2 .0; Ck .X //
there exists " > 0 such that B D ff 2 Ck .X / W d.f; 0/ < "g  U . Take
m 2 ! such that d.fn ; fk / < " for all n; k  m; since d is invariant, we have
d.fn  fk ; 0/ < " and hence fn  fk 2 B  U for all n; k  m. This shows that
the sequence S is linearly Cauchy, so we can apply Problem 338 to see that S is
convergent. Thus the metric d has to be complete, i.e., we settled necessity.
V.340. Let X and Y be l-equivalent spaces. Prove that X is a hemicompact kspace if and only if Y is a hemicompact k-space.
Solution. Suppose that X is a hemicompact k-space and fix a linear homeomorphism ' W Cp .X / ! Cp .Y /. The map ' W Cb .X / ! Cb .Y / is also a linear
homeomorphism by Problem 327. The space Y is hemicompact by Problem 337, so
both X and Y are -spaces and hence we have the equalities .Ck .X // D .Cb .X //
and .Ck .Y // D .Cb .Y // by Fact 6 of V.318. Therefore the map ' W Ck .X / !
Ck .Y / is also a linear homeomorphism.

By Problem 339 the space Ck .X / is metrizable and Cech-complete,


so Ck .Y / is

metrizable and Cech-complete


too. Applying Problem 339 again we conclude that
Y has to be a hemicompact k-space. Since X and Y are in a symmetric situation, an
analogous proof shows that if Y is a hemicompact k-space then so is X .
V.341. Prove that any subspace of an @0 -space is an @0 -space and any countable
product of @0 -spaces is an @0 -space.
Solution. Suppose that X is an @0 -space and fix a countable network N for all
compact subsets of X . Given Y  X let NY D fN \ Y W N 2 N g; it is clear that
NY is countable. If K is a compact subset of Y and K  U 2 .Y / then take a set
U 0 2 .X / such that U 0 \ Y D U . There exists N 2 N for which K  N  U 0 ;
then N 0 D N \ Y 2 NY while K  N 0  U . This proves that NY is a network for
all compact subsets of Y and hence every Y  X is an @0 -space.
Our next step is to prove that the product of two @0 -spaces is an @0 -space, so fix
@0 -spaces X and Y and their respective networks NX and NY for the families K.X /
and K.Y /. Let X W X  Y ! X and Y W X  Y ! Y be the respective natural
projections. Call a compact set K  X  Y rectangular if K D K1  K2 for some
compact sets K1  X and K2  Y ; a set K  X  Y is standard if it is a finite
union of rectangular sets.
Let N0 D fP  Q W P 2 NX and Q 2 NY g and denote by N the family of all
finite unions of the elements of N0 . Suppose first that K D K1  K2 is a rectangular
compact subset of X  Y . If K  U 2 .X  Y / then there exist V 2 .K1 ; X /

2 Solutions of problems 001500

377

and W 2 .K2 ; Y / such that V  W  U (see Fact 3 of S.271). Pick P 2 NX and


Q 2 NY for which K1  P  V and K2  Q  W . Then R D P  Q 2 N0 and
K  R  U ; this shows that
(1) the family N0 is a network for all rectangular compact subsets of X  Y .
It is a trivial consequence of the property (1) that
(2) the family N is a network for all standard compact subsets of X  Y .
Now suppose that K  X  Y is an arbitrary compact subset of X  Y and
K  U 2 .X  Y /. For any z 2 K we can find Vz 2 .X / and Wz 2 .Y / such
that z 2 Vz  Wz  V z  W
Sz  U . By compactness of K we can pick a finite
set A  K for which K  fVz  Wz W z 2 Ag. The set Dz D .V z  W z / \ K
is compact; besides, DzS Ez D X .Dz /  Y .Dz /  V z  W z  U for every
z 2 A. Clearly, E D fEz W z 2 Ag is a standard compact subset of X  Y
and K  E  U . Apply (2) to find a set N 2 N such that E  N  U ; then
K  N  U and hence N is a network for all compact subsets of X  Y , i.e.,
X  Y is an @0 -space. A trivial induction shows that
(3) any finite product of @0 -spaces is an @0 -space.

Q
Finally, assume that Xn is an @0 -space for all n 2 ! and let X D n2! Xn . For
each n 2 ! let Yn D X0  : : :  Xn and apply (3) to find a countable network Pn in
the space Yn for all compact subsets of Yn ; we will also need the natural projection
n W X ! Yn .
S
The family N D fn1 .P / W P 2 Pn ; n 2 !g is countable, so it suffices
to show that N is a network for K.X /. Take any compact set K  X and U 2
.K; X /. For every point x 2 K there exists n.x/ 2 ! and a set Ux 2S
.Yn / such
1
that x 2 Vx D n.x/
.Ux /  U . Take a finite set A  K such that K  fVx W x 2
Ag and let n D maxfn.x/ W x 2 Ag. If Wx D n .Vx / then n1 .Wx / D Vx for all
x 2 A.
S
The set K 0 D n .K/  Yn is compact and K 0  W D fWx W x 2 Ag.
Pick P 2 Pn with K 0  P  W . Then Q D n1 .PS/ 2 N and it follows from
the inclusions K  n1 .K 0 /  Q  n1 .W / D fVx W x 2 Ag  U that
K  Q  U and hence N is, indeed, a network for all compact subsets of X , so
we proved that any countable product of @0 -spaces is an @0 -space.
V.342. Observe that a compact-covering continuous image of an @0 -space is an @0 space. Prove that a space X is an @0 -space if and only if X is a compact-covering
continuous image of a second countable space.
Solution. Suppose that X is an @0 -space and f W X ! Y is a continuous compactcovering map. Fix a countable network N for all compact subsets of X and let
N 0 D ff .N / W N 2 N g. Suppose that K 0  Y is compact and U 0 2 .K 0 ; Y /;
there exists a compact K  X such that f .K/ D K 0 . The set U D f 1 .U 0 / is open
in X and K  U , so there is N 2 N with K  N  U . Then N 0 D f .N / 2 N 0
and K 0  N 0  U 0 ; this shows that N 0 is a countable network for all compact
subsets of Y and hence Y is an @0 -space. Therefore

378

2 Solutions of problems 001500

(1) any compact-covering continuous image of an @0 -space is an @0 -space.


If M is a second countable space then take a countable base B in X and let N be
the family of all finite unions of the elements of B. It is an easy exercise that N is a
network for all compact subsets of M ; this proves that
(2) any second countable space is an @0 -space.
The properties (1) and (2) show that any compact-covering continuous image of
a second countable space is an @0 -space.
To prove the converse fix an @0 -space X and let P D fPn W n 2 !g be a network
for all compact subsets of X . It is straightforward that the family fP n W n 2 !g
is also a network for K.X /, so we can assume, without loss of generality, that all
elements of P are closed in X .
Say that a function f 2 ! ! is appropriate if there exists a point
T x 2 X such that
Qf D fPf .n/ W n 2 !g is a network for the set fxg, i.e., x 2 fPf .n/ W n 2 !g
and, for any U 2 .x; X /, there is n 2 ! with PT
f .n/  U ; observe that the point
x is uniquely determined by f because fxg D
Qf ; let '.f / D x. The space
M D ff 2 ! ! W f is appropriateg is second countable and we have defined a map
' W M ! X.
If x 2 X then the family Px D fP 2 P W x 2 P g is a network for the set fxg.
It is easy to find a function f 2 ! ! such that Px D fPf .n/ W n 2 !g and hence
'.f / D x; this proves that ' is a surjective map.
Now if f 2 M and x D '.f / 2 U 2 .X / then pick a number n 2 ! such
that Pf .n/  U ; this is possible because Qf is a network for the set fxg. The set
W D fg 2 M W g.n/ D f .n/g is open in M and
Tf 2 W . If g 2 W and y D '.g/
then it follows from Pf .n/ 2 Qg and fyg D
Qg that y 2 Pf .n/  U . Thus
'.W /  U and hence ' is a continuous map.
Next fix a compact set K  X and let fFn W n 2 !g be an enumeration of all
finite subfamilies of P which cover the set K; fix
Q a finite set D!n  ! such that Fn D
fPi W i 2 Dn g for every
n
2
!.
The
set
D
D
n2! Dn  ! is compact; suppose
T
that f 2 D and x 2 . Qf / \ K. Given any U 2 .x; X / take U 0 2 .x; K/ with
U 0  U ; there exists P 2 P such that U 0  P  U . For each y 2 Knfxg take a set
Oy 2 .y; K/ such that x O y and choose Py 2 P for which O y  Py  X nfxg.
The cover fOy W y 2 Knfxgg [ fU 0 g of the compact
space K has a finite subcover,
S
so there exists a finite A  K such that K  . fPy W y 2 Ag/ [ P . There is n 2 !
for which fP g [ fPy W y 2 Ag D Fn . Since P is the unique element of Fn which
contains x, we must have Pf .n/ D P  U . This proves that Qf is a network for
fxg, so x D '.f /, i.e., we have established that
T
(3) if f 2 D and x 2 . Qf / \ K then '.f / D x.
Given any x 2 K recall that every family Fn covers K, so there isTi 2 Dn
such that x 2 Pi ; let f .n/ D i . For the function f 2 D we have x 2 Qf , so
x D '.fT
/ by (3). Therefore K  '.M \ D/. If f 2 DnM then it follows from
(3) that . T
fPf .n/ W n 2 !g/ \ K D ; and hence there exists a finite set Q  !
such that . fPf .n/ W n 2 Qg/ \ K D ;. The set W D fg 2 ! ! W g.n/ D f .n/

2 Solutions of problems 001500

379

for all n 2 Qg is an open neighborhood of f in ! ! and it is clear that f 2 W


while W \ .M \ D/ D ;. Thus the set M 0 D M \ D is compact being closed in
D. The set K 0 D ' 1 .K/ \ M 0 is also compact and '.K 0 / D K; this shows that
' is compact-covering and hence every @0 -space is a compact-covering continuous
image of a second countable space.
V.343. Prove that a space X is an @0 -space with the k-property if and only if it is
a quotient image of a second countable space.
Solution. Let us prove a couple of useful facts about the k-property.
Fact 1. Suppose that Z is a k-space and every compact subspace of Z is sequential.
Then Z is also sequential.
Proof. If A  Z is not closed in Z then there exists a compact K  Z such that
K \ A is not closed in K. By sequentiality of K there exists a sequence S  K \ A
which converges to a point x 2 Kn.K \ A/ D KnA; therefore x A, i.e., S  A
witnesses sequentiality of Z and Fact 1 is proved.
Fact 2. If Z is a k-space and f W Z ! Y is a quotient map then Y is also a k-space.
Proof. Suppose that A  Y is not closed in Y ; then B D f 1 .A/ is not closed
in Z, so there exists a compact K 0  Z such that B 0 D K 0 \ B is not closed in
K 0 . Choose a point z 2 B 0 nB 0 ; then z 2 K 0 and hence y D f .z/ 2 K D f .K 0 /.
Besides, z 2 ZnB implies that y A; however, y 2 f .B 0 /  A \ K which shows
that y 2 A \ Kn.A \ K/ and hence the set A \ K is not closed in K, i.e., the
compact set K witnesses that Y is a k-space so Fact 2 is proved.
Returning to our solution assume that M is a second countable space and f W
M ! X is a quotient map. Since M is a k-space, it follows from Fact 2, that X is
also a k-space. Fix a countable base B in the space M which is closed under finite
unions and let N D ff .B/ W B 2 Bg; observe that N is also closed under finite
unions. Suppose that K  X is compact and U 2 .K; X /; let N 0 D fNk W k 2 !g
1
be the family of all elements of
SN 0contained in U . Since f S .U0 / is a union of a
subfamily of B, we have U D N and, in particular, K  N .
Assume
that Lk D Kn.N0 [ : : : [ Nk / ; for all k 2 !. Recalling that
S
K  k2! Nk we can choose a sequence fyk W k 2 !g in such a way that yk 2 Lk
for all k 2 ! and l k implies yl yk . The space K is easily seen to be
metrizable, so we can find a subsequence S D fyki W i 2 !g of the sequence
fyk W k 2 !g which converges to a point y 2 KnS . By our choice of S ,
(1) the set N \ S is finite for each N 2 N 0 .
Since y 2 S nS , the set S is not closed in X , so the set T D f 1 .S / is not
closed in M . Pick a sequence C D fbn W n 2 !g  T which converges to a point
b T . It is evident that C \ f 1 .yki / is finite for any i 2 !, so f .C / is an infinite
subsequence of S . Therefore f .C / converges to y and hence y D f .b/. Take a set
B 2 B such that b 2 B  f 1 .U /. Then N D f .B/ 2 N 0 ; since B contains
infinitely many points of T , the set N contains infinitely many points of S which

380

2 Solutions of problems 001500

contradicts (1) and shows that K  N 0 D N0 [ : : : [ Nk for some k 2 !. Since


N 0 2 N 0 , we have K  N 0  U and hence N is a network for all compact subsets
of X , i.e., X is an @0 -space. Thus every quotient image of a second countable space
is an @0 -space with the k-property.
Finally assume that X is an @0 -space with the k-property and apply Problem 342
to find a second countable space M such that there exists a compact-covering map
f W M ! X . Since X has a countable network, all compact subspaces of X are
metrizable, so X is sequential by Fact 2. We claim that the mapping f is quotient.
To prove this assume that A  X is not closed while B D f 1 .A/ is closed
in M . By sequentiality of X there is a sequence S D fan W n 2 !g  A which
converges to a point x 2 X nA. The set K D S [ fxg is compact, so we can find a
compact L  M such that f .L/ D K. The set B \ L is also compact being closed
in L; therefore the set f .B \ L/ D S must be compact and hence closed in X , a
contradiction. Thus the map f is quotient, so every @0 -space with the k-property is
a quotient image of a second countable space.
V.344. Prove that any @0 -space of countable character is second countable.
Solution. Suppose that X is an @0 -space and .X /  !. Fix a countable network
N for all compact subsets of X and let B D fInt.P / W P is a finite union of elements
of N g. The family B being countable it suffices to show that B is a base of X , so take
a point x 2 X and any set U 2 .x; X /; consider the family N 0 D fNk W k 2 !g
of all elements of N contained in U . Suppose that, for any k 2 !, the point x does
not belong to the interior of the set Mk D N0 [ : : : [ Nk . Let fOn W n 2 !g be a
countable local base of X at the point x such that O n  U for all n 2 !. By our
assumption about the sets Mk , we can choose a point xk 2 Ok nMk for all k 2 !.
As a consequence, the sequence S D fxk W k 2 !g converges to x and
(1) the set fk 2 ! W xk 2 N g is finite for every N 2 N 0 .
However, K D S [ fxg is a compact subset of U , so there is N 0 2 N for
which K  N 0  U ; therefore N 0 2 N 0 and S  N 0 which contradicts (1). Thus
x 2 V D Int.Mk / for some k 2 ! and we have V  U ; since also V 2 B, we
proved that B is a countable base of the space X .
V.345. Suppose that ' W X ! Y is a continuous map; recall that the dual map '  W
Ck .Y / ! Ck .X / is defined by the formula '  .f / D f ' for every f 2 Ck .Y /.
Assuming that ' is compact-covering, prove that '  is an embedding.
Solution. Given a space Z and a function f 2 C.Z/ we will need the set
f; K; "
Z D fg 2 C.Z/ W jg.z/  f .z/j < "g
for every K  Z and " > 0.
The map '  is continuous by Fact 8 of V.318; consider the set E D '  .Ck .Y //.
If f; g 2 Ck .Y / and f g then f .y/ g.y/ for some y 2 Y . Any compactcovering map is surjective, so there is a point x 2 X such that '.x/ D y. It follows

2 Solutions of problems 001500

381

from '  .f /.x/ D f .'.x// D f .y/ g.y/ D '  .g/.x/ that '  .f / '  .g/, so
the mapping '  W Ck .Y / ! E is a condensation.
To see that the map  D .'  /1 W E ! Ck .Y / is continuous fix a function f 2
E, let g D .f / and take any U 2 .g; Ck .Y //. There exists a compact set K  Y
such that g; K; "
Y  U for some " > 0. The map ' being compact-covering, we
can find a compact set L  X for which '.L/ D K. Apply Fact 7 of V.318 to
see that there is an open set W in the space Ck .X / such that f 2 W  f; L; "
X .
Thus V D W \ E is an open neighborhood of f in the space E. If h 2 V and
u D .h/ then h D '  .u/. Fix a point y 2 K and pick x 2 L with '.x/ D y.
We have ju.y/  g.y/j D jh.x/  f .x/j < ", so u 2 g; K; "
Y . This shows that
.h/ 2 g; K; "
Y  U for all h 2 V , i.e., .V /  U and hence the set V witnesses
continuity of  at the point f . Consequently, the map  D .'  /1 is continuous and
hence '  W Ck .Y / ! E is a homeomorphism.
V.346. Given a compact subspace K of a space X let v.f; x/ D f .x/ for every
f 2 Ck .X / and x 2 K. Prove that the map v W Ck .X /  K ! R is continuous.
Solution. For any function f 2 Ck .X / and L  X we will need the set
f; L; "
D fg 2 Ck .X / W jg.x/  f .x/j < " for all x 2 Lg
for every " > 0. To prove continuity of the map v fix f0 2 Ck .X /; x0 2 K and
" > 0; let r D f0 .x0 / D v.f0 ; x0 /. Apply Fact 7 of V.318 to see that there is an open
set W in the space Ck .X / such that f0 2 W  f0 ; K; 2"
. The set V D fx 2 K W
f0 .x/ 2 .r  2" ; r C 2" /g is an open neighborhood of the point x0 in the space K, so
O D W V is an open neighborhood of the point .f0 ; x0 / in Ck .X /K. If .f; x/ 2
O then x 2 V and hence jf0 .x/  f0 .x0 /j < 2" ; since also f 2 f0 ; K; 2"
, we have
jf .x/  f0 .x/j < 2" which implies that jv.f; x/  v.f0 ; x0 /j D jf .x/  f0 .x0 /j < ",
so the set O witnesses that the function v is continuous at the point .f0 ; x0 /.
V.347. Prove that the following properties are equivalent for any space X :
(i) X is an @0 -space;
(ii) Ck .X / is an @0 -space.
(iii) Ck .X / has a countable network.
Solution. Given a space Z, a set K  Z and a function f 2 C.Z/ consider the
set f; K; "
D fg 2 C.Z/ W jg.x/  f .x/j < " for all x 2 Kg for every " > 0.
Furthermore, if Q  R then hK; Qi D ff 2 C.Z/ W f .K/  Qg. Given a family
S of open subsets of Z say that a family B  exp.Z/ is an S-network for compact
subsets of Z if for any compact K  Z and S 2 S with K  S there exists B 2 B
for which K  B  S .
Fact 1. A space Z is an @0 -space if and only if it has a subbase S such that some
countable family B of subsets of Z is an S-network for compact subsets of Z.

382

2 Solutions of problems 001500

Proof. If Z is an @0 -space then it has a network B for all compact subsets of Z; the
family S D .Z/ is a subbase of Z and B is an S-network for compact subsets of
Z, so we proved necessity.
Now assume that S is a subbase of Z and B is a countable S-network for compact
subsets of Z. Let C be the family of all finite intersections of the elements of B and
denote by D the family of all finite unions of the elements of C. It is clear that
jDj  !; let us prove that D is a network for all compact subsets of Z.
Fix a compact set K  Z and U 2 .K; Z/; say that a set G 2 .K/ is adequate
if there exists C 2 C such
finite
T that G  C  U . Given any x 2 K there exists aT
S 0  S such that x 2 S 0  U . Take a set V 2 .x; Z/ for which V  S 0 ;
if S 2 S 0 then the set T
F D V \ K is contained in S , so we can find PS 2 B with
F  PS  S . If C D fPS W S 2 S 0 g then F  C  U and C 2 C; an immediate
consequence is that G D V \ K 3 x is an adequate set. Thus, for any x 2 K there
exists an adequate set G
Sx such that x 2 Gx . By compactness of K there is a finite
A  K such that K D fGx W x 2 Ag.
S
For every x 2 A pick Cx 2 C for which Gx  Cx  U ; then D D fCx W x 2
Ag is an element of D and K  D  U . This shows that D is a countable network
for all compact subsets of Z, i.e., Z is an @0 -space, so we settled sufficiency and
hence Fact 1 is proved.
Fact 2. For any space Z the family S D fhK; Oi W K is a compact subset of Z and
O 2 .R/g is a subbase of the space Ck .Z/.
Proof. Suppose that K  Z is compact, O 2 .R/ and f 2 hK; Oi. Then f .K/ 
O and it follows from compactness of the set f .K/ that there exists " > 0 such that
jf .x/  rj > " whenever x 2 K and r 2 RnO. Take any g 2 f; K; "
; if x 2 K
and r D g.x/ 2 RnO then " < jr  f .x/j D jg.x/  f .x/j < "; this contradiction
shows that g 2 hK; Oi and hence f; K; "
 hK; Oi. By definition of the topology
of Ck .Z/ every set hK; Oi is open in Ck .Z/, i.e., S  .Ck .Z//.
Now assume that U 2 .Ck .Z// and f 2 U ; there exists a compact set K  Z
such that f; K; "
 U for some " > 0. It is easy to find a finite set A  K for
which f .A/ is an 3" -net for f .K/, i.e., for every x 2 K there is a 2 A such that
"
jf .x/  f .a/j < 3" . The set
S Ka D fx 2 K W jf .x/  f .a/j " 3 g is compact
for every a 2 A and K D fKa W a 2 Ag. If Qa D .f .a/  2 ; T
f .a/ C 2" / then
Sa D hKa ; Qa i 2 S and f 2 Sa for all a 2 A; therefore f 2 S D fSa W a 2 Ag.
If g 2 S and x 2 K then take a 2 A such that jf .x/  f .a/j < 3" ; then x 2 Ka
and it follows from g 2 Sa that jg.x/  f .a/j < 2" . An immediate consequence is
that jg.x/  f .x/j  2" C 3" < " and hence g 2 f; K; "
 U for every g 2 S . Thus
finite intersections of the elements of S form a base in Ck .Z/, so S is a subbase of
Ck .Z/, i.e., Fact 2 is proved.
Returning to our solution suppose that X is an @0 -space. By Problem 342 there
exists a second countable space M and a compact-covering map ' W M ! X .
The dual map '  W Ck .X / ! Ck .M / is an embedding by Problem 345, so it
suffices to show that Ck .M / is an @0 -space (see Problem 341).

2 Solutions of problems 001500

383

Let B be a countable base of M closed under finite unions and finite intersections.
It is easy to see that
(1) the family B is a network for all compact subsets of M .
The family S D fhC; U i W C  M is compact and U is an open subset of Rg is a
subbase of Ck .M / by Fact 2. Denote by Q the family of all finite unions of intervals
with rational endpoints; it is easy to see that Q is a network for all compact subsets
of R. Consider the countable family O D fhB; Qi W B 2 B and Q 2 Qg.
Suppose that K  hC; U i 2 S and K is a compact subset of Ck .M /. For any
f 2 Ck .M / and x 2 C let v.f; x/ D f .x/; then the map v W Ck .M /  C ! R is
continuous by Problem 346. For every x 2 C the set K.x/ D v.K; x/ is compact
being a continuous image of the compact set K  fxg. We will need the following
property of the sets K.x/.
(2) if fxn W n 2 !g  C and xn ! x then for any G 2 .K.x/; R/ there exists
m 2 ! such that K.xn /  G for all n  m.
If (2) does not hold then we can find a subsequence fxni W i 2 !g of the sequence
fxn W n 2 !g and a sequence S D ffi W i 2 !g  K such that fi .xni / G
for all i 2 !. Pick an accumulation point f 2 K for the sequence S ; it follows
from f .x/ 2 K.x/  G that v.f; x/ D f .x/ 2 G, so continuity of v at the point
.f; x/ implies that we can find i 2 ! such that fi .xni / D v.fi ; xni / 2 G. This
contradiction shows that (2) is true.
Our next step is to show that
(3) there is B 2 B such that C  B and v.K; B/  Q  U for some Q 2 Q.
Let fQn W n 2 !g be an enumeration of all elements of Q contained in U . It is
easy to see that .C; M /  !; recalling that B is closed under finite unions and
intersections we can choose a countable outer base B 0 D fBn W n 2 !g  B of
the set C in M such that BnC1  Bn for all n 2 !. If (3) does not hold then
v.K; Bn /n.Q0 [ : : : [ Qn / ; and hence we can find a function gn 2 K and
yn 2 Bn such that gn .yn / Q0 [ : : : Qn for all n 2 !.
It is straightforward that the set C [ fyn W n 2 !g is compact, so the sequence
fyn W n 2 !g contains a subsequence fyni W i 2 !g which converges to a point
y 2 C . It S
follows from K  hC; U i that K.y/  U ; the set K.y/ is compact and
K.y/  n2! Qn , so there is n 2 ! such that K.y/  Q D Q0 [ : : : [ Qn
Apply (2) to see that there exists m 2 ! for which K.yni /  Q for all i  m. For
i D m C n we have K.yni /  Q  Q0 D Q0 [ : : : [ Qni and hence gni .yni / 2 Q0
which is a contradiction, i.e., (3) is proved.
An immediate consequence of (3) is that K  hB; Qi  hC; U i which proves
that for the subbase S of the space Ck .M / the family O is an S-network for compact
subsets of Ck .M /. This, together with Fact 1 shows that Ck .M / is an @0 -space and
hence Ck .X / is an @0 -space, so we established that (i)H)(ii).
The implication (ii)H)(iii) is trivial, T
so assume that Ck .X / has a countable
network N and consider the set PN D ff 1 ..0; C1// W f 2 N g for each
N 2 N . The family P D fPN W N 2 N g is countable; let us prove that P is a
network for all compact subsets of X .

384

2 Solutions of problems 001500

Observe first that Ck .X / has a stronger topology than Cp .X /, so we have the


inequalities nw.X / D nw.Cp .X //  nw.Ck .X //  ! and hence X is normal.
Given a compact set K  X and U 2 .K; X / the closed sets K and X nU are
disjoint, so there exists a continuous function f W X ! 0; 1
such that f .K/  f1g
and f .X nU /  f0g. The set O D .0; C1/ is open in R and f .K/  O, so we
have f 2 hK; Oi; the family N being a network in Ck .X / there exists N 2 N
such that f 2 N  hK; Oi (see Fact 2). If x 2 K then g.x/ 2 O and hence
x 2 g 1 .O/ for every g 2 N ; therefore x 2 PN for all x 2 K which shows that
K  PN . Finally, if x 2 PN then g.x/ > 0 for all g 2 N ; in particular, f .x/ > 0
and hence x 2 U . As a consequence, PN  U , so P is a countable network for all
compact subsets of X . Thus X is an @0 -space, i.e., we settled (iii)H)(i) and hence
our solution is complete.
V.348. Let X and Y be l-equivalent spaces. Prove that X is an @0 -space if and
only if so is Y . In particular, if some space Z is l-equivalent to a second countable
space then Z is an @0 -space. Deduce from this fact that any first countable space
l-equivalent to a second countable space must be second countable.
Solution. Fix a linear homeomorphism ' W Cp .X / ! Cp .Y / and assume that X
is an @0 -space; we have the equalities nw.Y / D nw.X / D ! by Problem 001,
so both X and Y are -spaces. The map ' W Cb .X / ! Cb .Y / is continuous by
Problem 327; since the topologies of Ck .X / and Cb .X / coincide as well as the
topologies of Ck .Y / and Cb .Y / (see Fact 6 of V.318), the map ' W Ck .X / ! Ck .Y /
is also continuous. By Problem 347 the space Ck .X / has a countable network;
network weight is invariant under continuous images so nw.Ck .Y //  ! and
therefore we can apply Problem 347 again to convince ourselves that Y is an @0 space. Since X and Y are in a symmetric situation, @0 -property of Y also implies
that X is an @0 -space.
l

Finally, observe that any second countable space is an @0 -space, so if Z  M


and w.M /  ! then Z is an @0 -space. If, additionally, .Z/  ! then w.Z/  !
because every first countable @0 -space must be second countable (see Problem 344).
V.349. Suppose that a space X has a countable network and Y is an @0 -space.
Prove that Cp .X; Y / has a countable network.
Solution. If Z and T are spaces, z1 ; : : : ; zn 2 Z and O1 ; : : : ; On 2 .T / then
we have the equality z1 ; : : : ; zn ; O1 ; : : : ; On
TZ D ff 2 Cp .Z; T / W f .zi / 2 Oi
for all i D 1; : : : ; ng. It is evident that the family fz1 ; : : : ; zn ; O1 ; : : : ; On
TZ W n 2
N; z1 ; : : : ; zn 2 Z and O1 ; : : : ; On 2 .T /g is a base of the space Cp .Z; T /.
Fact 1. Given spaces Z; Z 0 and T suppose that ' W Z ! Z 0 is a continuous onto
map. For each f 2 Cp .Z 0 ; T / let '  .f / D f '. Then '  W Cp .Z 0 ; T / !
Cp .Z; T / is an embedding.
Proof. Fix an arbitrary function h0 2 Cp .Z 0 ; T / and an open neighborhood U
of the function g0 D '  .h0 / in Cp .Z; T /. There exist point z1 ; : : : ; zn 2 Z and

2 Solutions of problems 001500

385

O1 ; : : : ; On 2 .T / such that g0 2 U 0 D z1 ; : : : ; zn ; O1 ; : : : ; On
TZ  U . If yi D
'.zi / for all i  n then W D y1 ; : : : ; yn ; O1 ; : : : ; On
TZ 0 is an open neighborhood of
the function h0 in the space Cp .Z 0 ; T /. It is straightforward that '  .W /  U 0  U ,
so the map '  is continuous at the point h0 .
To see that '  is an injection take distinct f; g 2 Cp .Z 0 ; T /. Then f .y/ g.y/
for some y 2 Z 0 . If z 2 ' 1 .y/ then '  .f /.z/ D f .y/ g.y/ D '  .g/.z/ and
hence '  .f / '  .g/.
Let E D '  .Cp .Z 0 ; T //; to see that the mapping .'  /1 W E ! Cp .Z 0 ; T / is
continuous take any f 2 E and let g D .'  /1 .f /. Fix a set U 2 .g; Cp .Z 0 ; T //;
there are y1 ; : : : ; yn 2 Y and O1 ; : : : ; On 2 .T / such that
g 2 U 0 D y1 ; : : : ; yn ; O1 ; : : : ; On
TZ 0  U:
Choose xi 2 ' 1 .yi / for all i  n; the set V D x1 ; : : : ; xn ; O1 ; : : : ; On
\ E
is an open neighborhood of f in the space E. If h 2 V and u D .'  /1 .h/ then
h.xi / D '  .u/.xi / D u.'.xi // D u.yi / 2 Oi for all i  n. Thus u D .'  /1 .h/ 2
U 0 for any h 2 V and hence .'  /1 .V /  U 0  U which proves continuity of the
map .'  /1 at an arbitrary point f . Hence the mapping '  W Cp .Z 0 ; T / ! E is a
homeomorphism, so Fact 1 is proved.
Returning to our solution fix a countable network N for all compact subsets of
Y which is closed under finite unions; there exists a second countable space M
and a continuous onto map ' W M ! X (see Fact 6 of T.250). The dual map
'  W Cp .X; Y / ! Cp .M; Y / is an embedding by Fact 1, so it suffices to show that
Cp .M; Y / has a countable network.
Let B be a countable base of M closed under finite unions. Given B 2 B and
N 2 N let hB; N i D ff 2 Cp .M; Y / W f .B/  N g. Consider the family
S 0 D fhB; N i W B 2 B and N 2 N g; we will show that the family S of all
finite intersections of the elements of S 0 is a network in Cp .M; Y /. Our main tool
will be the following property.
(1) For any f 2 Cp .M; Y / if x 2 M and U 2 .f .x/; Y / then there exists N 2 N
such that N  U and x 2 Int.f 1 .N //.
Let fNk W k 2 !g be an enumeration of all elements of N contained in U and
suppose that (1) does not hold. Since N is closed under finite unions, the point x
does not belong to the interior of the set Pk D f 1 .N0 / [ : : : f 1 .Nk / for all
k 2 !. Take a decreasing local base fWk W k 2 !g at the point x such that W0 
f 1 .U / and choose a point yk 2 Wk nPk for every k 2 !. It is clear that the set
F D fyg [ fyk W k 2 !g is compact and the set fk 2 ! W f .yk / 2 Ni g is finite for
every i 2 !. However, G D f .F / is a compact subset of U , so there is N 2 N
with G  N  U . We have N D Ni for some i 2 !, so ff .yk / W k 2 !g  Ni ;
this contradiction shows that (1) is proved.
Fix f 2 Cp .M; Y / and U 2 .f; Cp .M; Y //. There are points x1 ; : : : ; xn 2 M
and O1 ; : : : ; On 2 .Y / for which f 2 U 0 D x1 ; : : : ; xn ; O1 ; : : : ; On
YM  U .

386

2 Solutions of problems 001500

Apply (1) to find a set Ni 2 N such that Ni  Oi and xi 2 Vi D Int.f 1 .Ni // for
all i D 1; : : : ;T
n. Pick Bi 2 B such that xi 2 Bi  Vi ; then f 2 hBi ; Ni i for all
i  n. If S D fhBi ; Ni i W i  ng then it is straightforward that f 2 S  U 0  U ,
so S is a countable network of Cp .M; Y /. We already observed that this implies that
nw.Cp .X; Y //  ! and hence our solution is complete.
V.350. Given spaces X; Y and a function u W X  Y ! R let ux .y/ D u.x; y/ for
all y 2 Y ; then ux W Y ! R for every x 2 X . Analogously, let uy .x/ D u.x; y/ for
all x 2 X ; then uy W X ! R for every y 2 Y . Say that the function u is separately
continuous if the functions ux and uy are continuous (on Y and X respectively)
for all x 2 X and y 2 Y . Let Cps .X  Y / be the set of all separately continuous
functions on X  Y with the topology induced from RXY . Observe that Cps .X  Y /
is a locally convex space and let .'/.x; y/ D '.x/.y/ for any continuous function
' W X ! Cp .Y /. Prove that .'/ 2 Cps .X  Y / for every ' 2 Cp .X; Cp .Y // and
 W Cp .X; Cp .Y // ! Cps .X  Y / is a linear homeomorphism.
Solution. It is easy to see that Cps .X  Y / is a linear subspace of RXY , so
Cps .X  Y / is a locally convex space. Besides, Cp .X; Cp .Y // is a linear subspace
of Cp .Y /X , so Cp .X; Cp .Y // is also a locally convex space.
For each x 2 X and ' 2 Cp .X; Cp .Y // the function .'/x is continuous because
it coincides with '.x/ 2 Cp .Y /. If y 2 Y then let y W Cp .Y / ! R be the
projection onto the coordinate determined by y; then .'/y D y ' is a continuous
map, so .'/ is separately continuous, i.e., .'/ 2 Cps .X  Y / for every ' 2
Cp .X; Cp .Y //.
If ' and  are distinct elements of Cp .X; Cp .Y // then '.x/ .x/ for some
x 2 X ; consequently, '.x/.y/ .x/.y/ for a point y 2 Y . This shows that
.'/.x; y/ ./.x; y/ and hence .'/ ./, i.e.,  is an injective map.
Given any u 2 Cps .X  Y / let '.x/ D ux ; then '.x/ 2 Cp .Y / for every x 2 X
and therefore ' W X ! Cp .Y /. For each y 2 Y the map y ' is continuous
because it coincides with uy ; applying TFS-102 we conclude that ' W X ! Cp .Y /
is continuous. We have .'/.x; y/ D '.x/.y/ D ux .y/ D u.x; y/ for all .x; y/ 2
X  Y ; this shows that .'/ D u, so  is also surjective, i.e.,  W Cp .X; Cp .Y // !
Cps .X  Y / is a bijective map. We omit a simple verification of linearity of ; once
it is done we can see that  is a linear isomorphism.
Given .x; y/ 2 X  Y let p.x;y/ .u/ D u.x; y/ for each u 2 Cps .X  Y /; then
p.x;y/ W Cps .X  Y / ! R is the projection onto the factor determined by the point
.x; y/. For every x 2 X let qx .'/ D '.x/ for each function ' 2 Cp .X; Cp .Y //;
then qx W Cp .X; Cp .Y // ! Cp .Y / is the projection onto the factor determined
by x.
Fix any point .x; y/ 2 X  Y ; then .p.x;y/ /.'/ D '.x/.y/ for every function
' 2 Cp .X; Cp .Y //. Therefore p.x;y/  D y qx is a continuous map for
every .x; y/ 2 X  Y , so we can apply TFS-102 once again to conclude that 
is continuous.
Finally take any x 2 X and u 2 Cps .X  Y /; if ' D  1 .u/ then qx .'/ D ux ;
to see that the map qx  1 W Cps .X  Y / ! Cp .Y / is continuous observe that

2 Solutions of problems 001500

387

y qx  1 .u/ D u.x; y/ D p.x;y/ .u/ for any y 2 Y . The map y .qx  1 / D


p.x;y/ being continuous for all y 2 Y , the mapping qx  1 is continuous for all
x 2 X , so  1 is continuous (observe that we have just applied TFS-102 twice) and
hence  is, indeed, a linear homeomorphism.
V.351. Prove that the space Cp .X; Cp .X // has a countable network if and only if
X is countable. Deduce from this fact that Cp .X / is an @0 -space if and only if X is
countable.
Solution. If X is countable then w.Cp .X // D ! and hence w.Cp .X /X / D !, so it
follows from Cp .X; Cp .X //  Cp .X /X that w.Cp .X; Cp .X /// D !; in particular,
Cp .X; Cp .X // has a countable network, i.e., we proved sufficiency.
Now assume that the space Cp .X; Cp .X // has a countable network; the space
Cp .X / is a continuous image of Cp .X; Cp .X //, so nw.X / D nw.Cp .X // D !.
Thus nw.X  X / D ! and hence X  X is perfectly normal; this makes it possible
to find a continuous function u W X  X ! 0; 1
such that u1 .0/ D D f.x; x/ W
x 2 X g. For any z 2 X let fz .z; z/ D 1 and
fz .x; y/ D

2u.x; y/
whenever .x; y/ .z; z/:
u.x; y/ C u.x; z/ C u.z; y/

Then fz W X  X ! R and the function fz is continuous at all points of X 


X except .z; z/. However, fz .z; y/ D 1 and fz .x; z/ D 1 for all x; y 2 X ; this
proves that the function fz is separately continuous for every z 2 X . The space
Cps .X  X / of separately continuous functions on X  X with the topology of
pointwise convergence is linearly homeomorphic to Cp .X; Cp .X // by Problem 350,
so nw.Cps .X X // D !. We already saw that the set D D ffz W z 2 X g is contained
in Cps .X  X /. Observe that, for every z 2 X we have fz .z; z/ D 1 and fz .x; x/ D 0
for all x z. Therefore z z0 implies that fz fz0 and hence jDj D jX j.
The set Uz D ff 2 Cps .X  X / W f .z; z/ > 0g is open in Cps .X  X / and it is
immediate that Uz \ D D ffz g for every z 2 X . Therefore D is a discrete subspace
of Cps .X  X /, so it follows from nw.Cps .X  X // D ! that jDj  ! and hence
jX j D jDj  !. This proves necessity and shows that
(1) for any space X we have nw.Cp .X; Cp .X /// D ! if and only if jX j D !.
Finally, observe that if X is countable then Cp .X / is an @0 -space being second
countable. Now if Cp .X / is an @0 -space then nw.X / D nw.Cp .X //  ! and hence
the space Cp .X; Cp .X // has a countable network (see Problem 349), so (1) implies
that X is countable.
V.352. Prove that a space X is of second category in itself if and only if the player
E has no winning strategy in the BanachMazur NE-game on X .
Solution. Suppose that the player E does not have a winning strategy on X in
the BanachMazur NE-game. If X is of first category in itself then we can fix
a family K D fKn W n 2 Ng of closed nowhere dense subsets of X such that

388

2 Solutions of problems 001500

S
X D K. Now, if n 2 N and the moves V1 ; U1 ; : : : ; Vn1 ; Un1 ; Vn are made then
let Un D  .V1 ; : : : ; Vn / D Vn nKn ; it is clear that Vn is a nonempty open subset of
Un , so we defined a strategy  for the player E on the space X .
SupposeT
that P D fUn ; Vn W n 2 Ng is a play in which E applies the strategy
 ; if P D P then P \ Kn D ; for all n 2 N and hence P D ;. Therefore the
strategy  is winning for the player E; this contradiction shows that X is of second
category in itself, i.e., we proved sufficiency.
Now assume that X is of second category in itself and s is a strategy of the
player E in the NE-game on the space X . The family B D fs.V / W V 2  .X /g
is a -base of the space X , so we can choose a maximal disjoint
S (with respect to
inclusion) subfamily U .1/ of the family B. It is evident that U .1/ is dense in
X . For any U 2 U .1/ fix a set V D 1 .U / 2  .X / such that U D s.V / and
let V.1/ D f1 .U / W U 2 U .1/g. It is clear that the map 1 W U .1/ ! V.1/ is a
bijection.
Proceeding by induction assume that n 2 N and we have constructed families
U .1/; V.1/ : : : ; U .n/; V.n/ of nonempty open subsets of X with the following
properties:
S
(1) the family U .i / is disjoint and U .i / is dense in X for every i  n;
(2) for every i  n a bijection i W U .i / ! V.i / is fixed in such a way that
U  i .U / for each U 2 U .i /;
(3) if i < n and V 2 V.i C 1/ then there is U 2 U .i / with V  U ;
(4) if k  n and U1  : : :  Uk where Ui 2 U .i / for all i  k then letting
Vi D i .Ui / we obtain an initial segment V1 ; U1 ; : : : ; Vk ; Uk of a play in the
NE-game such that Ui D s.V1 ; U1 ; : : : ; Vi1 ; Ui1 ; Vi / for all i  k.
Fix a set Un 2 U .n/ and let Vn D n .Un /; if 1 < k  n and we have the sets
Vk ; Uk ; : : : ; Vn ; Un , then we can apply (3) to find a set Uk1 2 U .k  1/ (which
is evidently, unique) with Vk  Uk1 ; let Vk1 D k1 .Uk1 /. This inductive
construction gives us the sets U1  : : :  Un , so the property (4) implies that
V1 ; U1 ; : : : ; Vn ; Un is an initial segment of a play in which E applies the strategy s.
Therefore we can consider the family B D fs.V1 ; : : : ; Vn ; V / W V 2  .Un /g which
is, evidently, a -base in Un ; choose
a maximal disjoint subfamily U .n C 1/Un
of
S
the family B. It is evident that U .n C 1/ is dense in Un . For each U 2 U .n C 1/
there exists V D nC1 .U / 2  .Un / such that U D s.V1 ; : : : ; Vn ; V /; it is
immediate that the family V.n C 1/Un
D fnC1 .U / W U 2 U .n C 1/g consists of
nonempty open subsets
of Un and nC1 W U .nC1/Un
! V.nC1/Un
is a bijection.
S
If U .n C 1/ D fU .n C 1/Un
W Un 2 U .n/g and V.n C 1/ D nC1 .U .n C 1//
then the map nC1 W U .n C 1/ ! V.n C 1/ is a bijection while the family U .n C 1/
is disjoint and its union is dense in X .
It is straightforward to verify that the properties (1)(4) now hold if we replace n
with n C 1. Thus our inductive procedure can be continued to construct a sequence
of families fU .i /; V.i / W i 2 Ng with the conditions
(1)(4) satisfied for all n 2 N.
S
It follows from (1) that the set Fi D X n. U .i // is closed and nowhere dense
in X for every
S i 2 N; since X is of second
T Scategory in itself, there exists a point
x 2 X n. fFi W i 2 Ng/. Thus x 2 f U .i / W i 2 Ng, so we can choose a

2 Solutions of problems 001500

389

T
sequence fUi W i 2 Ng such that Ui 2 U .i / for each i 2 N and x 2 fUi W i 2 Ng.
For every i 2 N it is an easy consequence of (1)(3) that UiC1  Ui ; let Vi D
i .Ui /. The property (4) shows that
T P D fVi ; Ui W i 2 Ng is a play in which E
applies the strategy s. Now, x 2 P, so s is not winning for E. Since a strategy s
of the player E was taken arbitrarily, we proved that no strategy of E is winning;
this settles necessity and makes our solution complete.
V.353. Prove that a space X has the Baire property if and only if the player E has
no winning strategy in the BanachMazur E-game on X .
Solution. Suppose that the player E does not have a winning strategy on X in the
BanachMazur E-game. Fix a nonempty open set U  X and assume that E has
a winning strategy s in the NE-game on the space U . To define a strategy  of E
in the E-game on X let  .;/ D U . For any V1 2  .U / we can consider V1 to be
the first move of NE in the NE-game on V , so we can define  .V1 / D s.V1 /. It is
evident that  .U / is the family of all admissible sets for  .
Proceeding by induction assume that n 2 N and we proved that, for each
i  n, an i -tuple .V1 ; : : : ; Vi / is admissible for the strategy s if and only if it
is admissible for the strategy  and  .V1 ; : : : ; Vi / D s.V1 ; : : : ; Vi /. Observe that
fV1 ; U1 ; : : : ; Vn ; Un ; VnC1 g is an initial segment of a play in the NE-game on the
space U if and only if fU; V1 ; U1 ; V2 ; : : : ; Un1 ; Vn ; Un ; VnC1 g is an initial segment
of a play in the E-game on X . By the induction hypothesis we have the equalities
Ui D s.V1 ; : : : ; Vi / D  .V1 ; : : : ; Vi / for all i  n. The .nC1/-tuple .V1 ; : : : ; VnC1 /
is admissible for  if and only if VnC1  Un and this happens precisely when it is
admissible for s, so we can let  .V1 ; : : : ; VnC1 / D s.V1 ; : : : ; VnC1 / completing our
construction of a strategy  for the player E in the E-game on X .
The strategy s is winning, so  is also winning because, apart from the set U , all
plays where E applies  coincide with the respective plays on E where E uses the
strategy s. This contradiction shows that E does not have a winning strategy on the
NE-game on U and hence U is of second category in itself by Problem 352. Thus
every U 2  .X / is of second category, i.e., X has the Baire property which shows
that we proved sufficiency.
Now suppose that X has the Baire property and  is a strategy of the player E
in the E-game on X ; let U D  .;/. Every V1 2  .U / is admissible for  , so
we can let s.V1 / D  .V1 /. Proceeding inductively as above it is easy to see that
letting s.V1 ; : : : ; Vn / D  .V1 ; : : : ; Vn / for every admissible n-tuple .V1 ; : : : ; Vn / for
the strategy  we consistently define a strategy s for the player E in the NE-game
on the space U .
The set U is of second category in itself, so the strategy s cannot be winning by
Problem 352. Since any play where E applies  coincides (apart from the set U )
with the respective play on E where E uses the strategy s, the strategy  is not
winning either. Thus E does not have a winning strategy in the E-game on the
space X and hence we settled necessity.

390

2 Solutions of problems 001500

V.354. Prove that


(i) any pseudocompact space with the moving off property is compact.
(ii) any paracompact locally compact space has the moving off property.
Solution. If X is a pseudocompact non-compact space with the moving off property
then S D ffxg W x 2 X g is a moving off family so there is an infinite A  S
which has a discrete open expansion. However, a pseudocompact space cannot have
an infinite discrete family of nonempty open sets (see TFS-136); this contradiction
shows that (i) is proved.
To prove (ii) assume that X is a nonempty paracompact locally compact space.
Fix a locally finite open cover U of the space X such thatS
U is compact for each
U 2 U . Let U A
D fU 2 U W U \A ;g and St.A; U / D U A
for any A  X .
Suppose that K is a moving off family of compact subsets of X ; observe that K has
to be nonempty because, for any x 2 X there is K 2 K with x K.
Take any set K0 2 K; proceeding inductively assume that n 2 ! and we have
chosen sets K0 ; : : : ; Kn 2 K in such a way that
(1) KiC1 \ St.K0 [ : : : [ Ki ; U / D ; for each i < n.
The set K 0 D K0 [ : : : [ Kn is compact, so the family U K 0
is finite and hence
the set St.K 0 ; U / has compact closure F . Using the fact that K is a moving off
collection, we can find a set KnC1 2 K such that KnC1 \ F D ;. It is evident that
the property (1) still holds for all i  n, so our inductive procedure can be continued
to construct a family K0 D fKi W i 2 !g  K such that (1) is fulfilled for all n 2 !
and, in particular, the family K0 is infinite.
Fix a point x 2 X and U 2 U with x 2 U . If i < j and U \ Ki ;
then U  St.Ki ; U /, so it follows from (1) that U \ Kj D ;. Therefore, every
x 2 X has a neighborhood which meets at most one element of K0 , i.e., the family
K0 is discrete. By paracompactness of X (or, more exactly, by its collectionwise
normality), the family K0  K has a discrete open expansion. This shows that the
space X has the moving off property and hence we settled (ii).
V.355. Let X be a q-space. Prove that, if X has the moving off property then it is
locally compact.
Solution. Fix a point a 2 X and say that a family fUn W n 2 !g of open
neighborhoods of a is a q-sequence if any sequence fxn W n 2 !g  X such
that xn 2 Un for every n 2 !, has an accumulation point. Since X is a q-space,
there exists at least one q-sequence. Observe that if fUn W n 2 !g is a q-sequence,
Vn 2 .a; X / and a 2 Vn  Un for all n 2 ! then the family fVn W n 2 !g is a
q-sequence as well. Therefore we can find a q-sequence O D fOn W n 2 !g such
that O nC1  On for every n 2 !. Besides,
(1) if a sequence fni W i 2 !g  ! is strictly increasing and yi 2 Oni for all i 2 !
then the sequence S D fyi W i 2 !g has an accumulation point.
Indeed, i  ni , so Oni  Oi for every i 2 !, so S has an accumulation point
because O is a q-sequence.

2 Solutions of problems 001500

391

Suppose that X is not locally compact at the point a. Since the moving off
property is closed-hereditary, any closed neighborhood of a is a non-compact space
with the moving off property; therefore it is not countably compact by Problem 354.
Thus we can choose an infinite closed discrete set Dn  O nC1 ; observe that Dn 
On for every n 2 !. Take a faithful enumeration fxn W n 2 !g of some countably
infinite set D  D0 and consider the family D D ffxn ; d g W d 2 Dn ; n 2 !g.
If K  X is compact then D \ K is finite, so there exists n 2 ! such that
xn K. The set Dn \ K is also finite, so we can find d 2 Dn with d K. Now
fxn ; d g 2 D and fxn ; d g \ K D ;; this proves that D is a moving off collection.
Consequently, there is an infinite D0  D which has a discrete open expansion.
Observe that if fxn ; d g and fxm ; eg are distinct elements of D0 then m n, so we
can choose an increasing sequence fni W i 2 !g  ! and dni 2 Dni for all i 2 !
in such a way that ffxni ; dni g W i 2 !g  D0 and, in particular, fdni W i 2 !g is a
faithfully indexed closed discrete subset of X . However, dni 2 Oni for all i 2 !, so
we can apply (1) to conclude that the sequence fdni W i 2 !g has an accumulation
point; this contradiction shows that X is locally compact at any point a 2 X .
V.356. Prove that the following conditions are equivalent for any space X ;:
(i) X has the moving off property;
(ii) given a sequence fKi W i 2 !g of moving off collections in X , we can choose
Ki 2 Ki for each i 2 !, such that the family fKi W i 2 !g has a discrete open
expansion;
(iii) the player II has no winning strategy in the GruenhageMa game on X .
Solution. Suppose that the space X has the moving off property and we are given
a sequence fKi W i 2 !g of moving off collections in X . Since nonempty compact
spaces do not admit moving off collections, the space X is not compact and hence
not pseudocompact by Problem 354, so we can fix a faithfully indexed closed
discrete set D D fdn W n 2 !g  X .
For every n 2 ! consider
S the family Ln D ffdn g [ L0 [ : : : [ Ln W Li 2 Ki
for all i  ng and let L D n2! Ln . Given a compact set F  X , the set D \ F
is finite, so there is n 2 ! such that dn F . Pick Li 2 Ki with Li \ F D ; for
all i  n; this is possible because every Ki is a moving off collection. Then the set
L D fdn g [ L0 [ : : : [ Ln belongs to L and L \ F D ;; this shows that L is a
moving off family.
Thus there exists an infinite family L0  L which has a discrete open expansion.
If L D fdn g [ L0 [ : : : [ Ln and L0 D fdm g [ P0 [ : : : [ Pm are distinct elements of
L0 then they are disjoint, so n m. Therefore we can find an increasing sequence
fni W i 2 !g  ! and a set Qi D fdni g [ Li0 [ : : : [ Lnnii 2 Lni such that Qi 2 L0
for each i 2 !. Observe that we have i  ni , so Lni i 2 Ki ; thus, letting Ki D Lni i
for every i 2 ! we obtain the promised sequence fKi W i 2 !g which has a discrete
open expansion. This shows that (i)H)(ii).
For each k 2 N denote by SkSthe family of all finite sequences of the elements
of ! of length k and let S D fSk W k 2 Ng. If s D .n1 ; : : : ; nk / 2 S then
si
D .s1 ; : : : ; si / for every i D 1; : : : ; k and s _ n D .n1 ; : : : ; nk ; n/ for all n 2 !.

392

2 Solutions of problems 001500

Next, assume that the property (ii) holds for X and  is a strategy of the player
II in the GruenhageMa game on the space X . The family f .K/ W K 2 K.X /g is
clearly moving off, so we can choose a sequence fKn W n 2 !g  K.X / such that
f .Kn / W n 2 !g has a discrete open expansion; let K.n/ D Kn and L.n/ D  .Kn /
for every n 2 !.
Proceeding by induction assume that n 2 N and, for any nonempty finite
sequence s of elements of ! of length at most n, we have defined sets Ks and Ls is
such a way that
(1) if k  n and s 2 Sk then the family Ks1
; Ls1
; : : : ; Ksk
; Lsk
is an initial
segment of a play in which II applies the strategy  .
(2) the family fLs _ n W n 2 !g has a discrete open expansion for every sequence s
of length at most n  1.
Fix any sequence s 2 Sn ; then Ks1
; Ls1
; : : : ; Ksn
; Lsn
is an initial segment of
a play on X in which II applies the strategy  . It is clear that the family fP .K/ D
 .Ks1
; : : : ; Ksn
; K/ W K 2 K.X /g is moving off, so we can choose a faithfully
indexed sequence fKn W n 2 !g  K.X / such that fP .Kn / W n 2 !g has a discrete
open expansion. Let Ks _ n D Kn and Ls _ n D P .Kn / for all n 2 !. Having done
this construction for all s 2 Sn we obtain the sets Kt and Lt for all t 2 SnC1
and it is straightforward that the conditions (1) and (2) are satisfied if n is replaced
with n C 1. Therefore our inductive procedure can be continued to obtain a family
fKs ; Ls W s 2 Sg for which (1) and (2) are fulfilled for all n 2 N.
Any family which has a discrete open expansion is moving off so the family
Qs D fLs _ n W n 2 !g is moving off for each s 2 S [ f;g. Apply (2) to pick
Qs 2 Qs for all s 2 S [ f;g in such a way that the family P D fQs W s 2 S [ f;gg
has a discrete open expansion.
Thus there is n1 2 ! such that L.n1 / 2 P. Proceeding inductively assume that
k 2 N and we have a sequence s D .n1 ; : : : ; nk / such that Lsi
2 P for all i  k.
By our choice of P there exists nkC1 2 ! such that Ls _ nkC1 2 P. Therefore this
inductive procedure can be continued to construct a sequence fni W i 2 Ng  !
such that L.n1 ;:::;nk / 2 P for all k 2 N.
Let Pk D K.n1 ;:::;nk / and Qk D L.n1 ;:::;nk / for all k 2 N; it follows from (1)
and (2) that fPk ; Qk W k 2 Ng is a play in which II applies the strategy  . It
follows from fQk W k 2 Ng  P that the family fQk W k 2 Ng has a discrete open
expansion and hence the strategy  is not winning. Recall that a strategy  of the
second player was taken arbitrarily so we showed that II has no winning strategy
if (ii) is assumed, i.e., (ii)H)(iii).
Finally, assume that II has no winning strategy in the GruenhageMa game on
the space X while X does not have the moving off property. This means that there
exists a moving off family P in the space X such that no infinite subfamily of P
has a discrete open expansion. If moves K1 ; L1 ; : : : ; Kn1 ; Ln1 ; Kn are made in
the game of GruenhageMa
on X then let  .K1 ; : : : ; Kn / be an element of P which
S
does not meet Kn [ . fKi [ Li W 1  i < ng/; such an element exists because P
is a moving off family. In any play where II applies  the set of choices of II is
an infinite subfamily of P, so it does not have a discrete open expansion, i.e.,  is

2 Solutions of problems 001500

393

a winning strategy; this contradiction shows that we settled (iii)H)(i) and thereby
completed our solution.
V.357. Prove that, if Ck .X / has the Baire property then the space X has the moving
off property.
Solution. If K is a compact subset of the space X and O 2 .R/ then the set
hK; Oi D ff 2 Ck .X / W f .K/  Og is open in Ck .X / by Fact 2 of V.347.
Suppose that PSis a moving off family in the space X and consider, for any n 2 !,
the set Un D fhK; .n; C1/i W K 2 Pg. By our above observation the set Un is
open in Ck .X / for each n 2 !. If W is a nonempty open subset of Ck .X / then there
exists a function f 2 W and a compact set L  X such that the set
H D fg 2 Ck .X / W jg.x/  f .x/j < " for all x 2 Lg
is contained in W for some " > 0. Pick K 2 P with K \ L D ; and define a
function h W K [ L ! R by the equalities hjL D f and h.x/ D n C 1 for every
x 2 K. It is evident that h is continuous on K [ L, so there exists g 2 Ck .X / such
that gj.K [ L/ D h (see Fact 1 of T.218). It is clear that g 2 H \ Un  W \ Un ,
i.e., W \ Un ; for any nonempty open set W  Ck .X /.
This shows that every set Un is dense inT
Ck .X /, so the Baire property of Ck .X /
implies that we can find a function f 2 n2! Un . Pick a set K0 2 P such that
f 2 hK0 ; .0; C1/i and let a0 D 0; since f .K0 / is compact, we can find b0 > a0
such that f .K0 /  .a0 ; b0 /.
Proceeding inductively, assume that k 2 ! and we have chosen K0 ; : : : ; Kk 2 P
and a0 ; b0 ; : : : ; ak ; bk 2 R such that
(1) ai < bi and f .Ki /  .ai ; bi / for all i  k;
(2) bi C 1 < aiC1 for all i D 0; : : : ; k  1.
Take n 2 ! with n > bk C 1 and let akC1 D n; there exists KkC1 2 P such
that f .KkC1 /  .akC1 ; C1/. The set f .KkC1 / being compact we can find bkC1 >
akC1 for which f .KkC1 /  .akC1 ; bkC1 /. It is clear that the properties (1) and (2)
now hold if we replace k with k C 1. Thus our inductive procedure can be continued
to construct families fKi W i 2 !g  P and fai ; bi W i 2 !g  R such that the
conditions (1) and (2) are satisfied for all k 2 !.
It is immediate from (1) and (2) that f.ai ; bi / W i 2 !g is a discrete open
expansion of the family ff .Ki / W i 2 !g, so ff 1 ..ai ; bi // W i 2 !g is a discrete
open expansion of an infinite family fKi W i 2 !g  P. Therefore the space X has
the moving off property.
V.358. Prove that, for any q-space X , the following conditions are equivalent:
(i) Ck .X / has the Baire property;
(ii) X has the moving off property;
(iii) the player II has no winning strategy in GruenhageMa game on the space X .

394

2 Solutions of problems 001500

Solution. If Z is a space, A  Z; f 2 Ck .Z/ and " > 0 then, as usual, we let


f; A; "
D fg 2 Ck .Z/ W jf .x/  g.x/j < " for all x 2 Ag. Given sets A; B  Z
we write A B of B A if A  Int.B/.
Fact 1. If Z is a space then for any compact set K  Z and " > 0 the set f; K; "

is open in Ck .Z/ for every f 2 Ck .Z/.


Proof. Fix any function g 2 f; K; "
and let h D g  f ; then h.K/  ."; "/
and it follows from compactness of h.K/ that there exists 2 .0; "/ such that
h.K/  ." C ; "  /. Take any function u 2 g; K;
; for every x 2 K we
have ju.x/  f .x/j  ju.x/  g.x/j C jg.x/  f .x/j < C ."  / D ", so
u 2 f; K; "
and hence we showed that for every g 2 f; K; "
there exists > 0 for
which g; K;
 f; K; "
. Therefore the set f; K; "
is open in Ck .Z/, i.e., Fact 1
is proved.
Returning to our solution recall that it was proved in Problem 356 that (ii)
(iii) and it follows from Problem 357 that (i)H)(ii), so it suffices to establish that
(iii)H)(i). To do it, assume that the second player does not have a winning strategy
in the GruenhageMa game on the space X while Ck .X / does not have the Baire
property. It follows from Problem 355 that X is locally compact and hence, for every
compact set K  X there exists a compact L  X such that K L.
It is an easy consequence of Fact 3 of T.371 that the space Ck .X / is of the first
category in itself, so we can find aSfamily Q D fQn W n 2 Ng of closed nowhere
dense subsets of Ck .X / such that Q D Ck .X /. Given any set U 2  .Ck .X //
and n 2 N, the set n .U / D U nQn is also open and nonempty.
We are going to describe a strategy s for the player II in the GruenhageMa
game on X as follows. Suppose that the first move of I is a compact set K1  X .
Choose a compact set A1  X with K1 A1 and let g1 .x/ D 0 for all x 2 X .
The set W1 D g1 ; A1 ; 12
is open in Ck .X / (see Fact 1) and nonempty, so we can
fix a function f1 2 Ck .X / together with a compact set E1  X and "1  .0; 12 /
such that A1  E1 and f1 ; E1 ; "1
 1 .W1 /. Take a compact set D1 E1 and let
L1 D s.K1 / D D1 nInt.A1 /.
Proceeding inductively assume that n 2 N and we defined the strategy s
for all i -tuples .K1 ; : : : ; Ki / 2 .K.X //i with i  n in such a way that if
K1 ; L1 ; : : : ; Kn ; Ln is an initial segment of a play in which II applies s then we also
have compact sets fAi ; Di ; Ei W 1  i  ng, functions f1 ; g1 ; : : : ; fn ; gn 2 Ck .X /
and numbers "1 ; : : : ; "n > 0 which satisfy the following conditions:
(1) "i < 2i and Ki Ai  Ei Di for all i D 1; : : : ; n;
(2) "iC1 < "i and fiC1 ; EiC1 ; "iC1
 iC1 .giC1 ; AiC1 ; "i
/ for all i D 1; : : : ;
n  1;
(3) if i < n then Di AiC1 ; giC1 jEi D fi and giC1 .AiC1 nInt.Di //  f0g;
(4) Li D Di nInt.Ai / for all i D 1; : : : ; n.
If the first player chooses a compact set KnC1  X then we can find a compact
set AnC1 Dn [ KnC1 . It is easy to see that there exists a function gnC1 2 Ck .X /
such that gnC1 jEn D fn jEn and gnC1 .x/ D 0 for all x 2 AnC1 nInt.Dn /.

2 Solutions of problems 001500

395

Take "nC1 2 .0; 2n1 /, a compact set EnC1  AnC1 and fnC1 2 Ck .X / such
that "nC1 < "n and fnC1 ; EnC1 ; "nC1
 nC1 .gnC1 ; AnC1 ; "n
/.
Take a set DnC1 EnC1 and let LnC1 D s.K1 ; : : : ; Kn / D DnC1 nInt.AnC1 /.
This defines a strategy s for the second player on the space X and if fKi ; Li W i 2 Ng
is a play in which II applies s then we also have families fAi ; Di ; Ei W i 2 !g
together with the functions ffi ; gi W i 2 Ng and positive numbers f"i W i 2 Ng with
the properties (1)(4) fulfilled for all n 2 N.
By our assumption the strategy s cannot be winning, so we can fix a play
fKi ; Li W i 2 Ng in which II applies s and loses; we also have our overhead
consisting of the families fAi ; Di ; Ei W i 2 !g together with the functions
ffi ; gi W i 2 Ng and positive numbers f"S
i W i 2 Ng with the properties (1)(4).
It follows from (1) and (3) that A D fAn W n 2 Ng is an open subset of X . The
properties (2) and (3) imply the inequality jfiC1 .x/fi .x/j < 2i for every x 2 Ei
and i 2 N. Since the sequence fEi W i 2 Ng is increasing, we have jfnC1 .x/ 
fn .x/j < 2n for all x 2 Ei and n  i ; it is an easy exercise that this implies that the
sequence ffn W n 2 Ng converges uniformly on every set EiS
. Let f .x/ D lim fn .x/
for every x 2 A; this definition is consistent because A D fEi W i 2 Ng. Letting
f .x/ D 0 for all x 2 X nA we obtain a function f W X ! R.
Let Wn D fn ; En ; "n
for all n 2 N. It follows from (2) and (3) that WnC1  Wn ,
so ffi W i  ng  Wn for all n 2 N. As a consequence,
(5) jfi .y/  fn .y/j < "n for all i  n and therefore jf .y/  fn .y/j  "n < 2n
for every y 2 En and n 2 N.
Observe that f jEi is continuous on Ei being a uniform limit of continuous
functions. If x 2 Ei then x 2 Int.EiC1 / by the property (1), so it follows from
continuity of f jEiC1 that the function f is continuous at the point x.
Now, fix a point x 2 X nA and " > 0; pick m 2 N with 2mC1 < ". Since the
play fKi ; Li S
W i 2 Ng is a loss for II , the family fLi W i 2 Ng is discrete and hence
the set L D fLi W i 2 Ng  A is closed in X , so the set U D X n.Am [ L/ is an
open neighborhood of the point x.
If y 2 U nA then f .y/ D 0 and hence jf .y/  f .x/j D 0 < ". If y 2 A then
let p D nnfn 2 N W y 2 An g. If y 2 Dp1 then it follows from y Ap1 that
y 2 Lp1  L which is a contradiction. Therefore y 2 Ap nDp1 , so the conditions
(2) and (3) imply that jfp .y/j < 2p . The property (5) shows that jf .y/  fp .y/j 
2p and hence jf .y/j  2pC1  2mC1 < ". Thus f .U /  ."; "/ and hence U
witnesses continuity of the function f at the point x.
Thus the function f is continuous on the space X ; fix any n 2 N and a set
W 2 .f; Ck .X //. There exists a compact set P  X and > 0 such that En  P
and f; P;
 W . Since the sequence ffi W i 2 Ng converges to f uniformly on
En , there exists i  n such that jf .x/  fi .x/j < for all x 2 En . If h D fi  f
then h.En /  .; /, so it follows from compactness of En that there is 0 2 .0; /
such that h.En /   0 ; 0
. By Fact 1 of V.321 there exists a function h0 2 C.X /
such that h0 jEn D hjEn and h0 .X /   0 ; 0
.
If g D f Ch0 then jf .x/h0 .x/j  0 < for all x 2 X , so g 2 f; P;
. On the
other hand, if x 2 En then g.x/ D fi .x/, so jg.x/  fn .x/j D jfi .x/  fn .x/j < "n

396

2 Solutions of problems 001500

(here we used again the property (5)) and hence g 2 fn ; En ; "n


\ f; P;

Wn \ W . This proves that every neighborhood of f in Ck .X / intersects the set Wn ,
i.e., f 2 W n for
Tall n 2 N.
Thus, f 2 fW n W n 2 Ng; however, it follows from
T (2) and the definition of
n that W n \ Qn D ; for all n 2 N and therefore fW n W n 2 Ng D ;. This
contradiction shows that Ck .X / has the Baire property, i.e., we settled (iii)H)(i)
and hence our solution is complete.
V.359. Let X be a paracompact q-space. Prove that Ck .X / has the Baire property
if and only if X is locally compact.
Solution. If X is locally compact then X has the moving off property by Problem 354 and hence Ck .X / is a Baire space by Problem 358. If, on the other hand,
the space Ck .X / has the Baire property then X has the moving off property by
Problem 357, so X is locally compact by Problem 355.
V.360. Let X be a paracompact q-space. Prove that, if X is l-equivalent to a locally
compact paracompact space then X is also locally compact. In particular, any first
countable paracompact space l-equivalent to a locally compact paracompact space,
is locally compact. Deduce from this fact that
(i) if X and Y are metrizable l-equivalent spaces then X is locally compact if and
only if Y is locally compact;
(ii) if a first countable space X is l-equivalent to a second countable locally
compact space then X is also locally compact and second countable.
Solution. Suppose that Y is a locally compact paracompact space and we are given
a linear homeomorphism ' W Cp .X / ! Cp .Y /. Apply Problem 327 to see that
' W Cb .X / ! Cb .Y / is also a linear homeomorphism. It is straightforward that
every paracompact space is a -space, i.e., if Z is paracompact then the closure of
every bounded subset of Z is compact. Therefore X and Y are -spaces, so we
have the equalities .Cb .X // D .Ck .X // and .Cb .Y // D .Ck .Y // by Fact 6 of
V.318. Therefore ' W Ck .X / ! Ck .Y / is a linear homeomorphism as well.
It is evident that every locally compact space is a q-space so we can apply
Problem 359 to convince ourselves that Ck .Y / has the Baire property. Therefore
Ck .X / also has the Baire property and hence X has the moving off property by
Problem 357. This, together with Problem 355, implies that X is locally compact.
The statement (i) follows from the fact that every metrizable space is first countable
(and hence a q-space) and paracompact.
Now if .X / D ! and X is l-equivalent to a second countable locally compact
space then X is second countable by Problem 348, so (i) is applicable to conclude
that X is locally compact.
V.361. Suppose that a q-space X is l-equivalent to a locally compact metrizable
space. Prove that X is metrizable and locally compact.
Solution. The space X is paracompact by Problem 335, so we can apply Prob
lem 360 to see that X is locally compact and hence Cech-complete.
It follows from

2 Solutions of problems 001500

397

Problem 333 that X has a weaker metrizable topology, so the diagonal of X is a


G -set in X  X . This makes it possible to apply Fact 7 of U.421 to conclude that
X is metrizable.
V.362. Suppose that X is l-equivalent to a metrizable space, Y  X and Y is

Cech-complete.
Prove that Y is metrizable. In particular, if a Cech-complete
space
X is l-equivalent to a metrizable space then X is metrizable.
Solution. If Z is a space and A  exp.Z/ then AjP D fA \ P W A 2 Ag for any
P  Z.
Fact 1. If a space Z is  -metrizable and paracompact then every subspace of Z is
 -metrizable and paracompact.
Proof. Fix an arbitrary subspace T  Z; by Fact 2 of V.334 the space Z has a
metrizable -approximation, i.e., there exists a metrizable topology  on
S Z and a
countable family F of closed subsets of Z such that   .Z/ while F D Z
and .Z/jF D jF for every F 2 F. If F 0 D FjT and  0 D jT then F 0
is
of closed subsets of T and  0 is a metrizable topology on T such that
S a family
0
F D T and  0 jP D .T /jP for every P 2 F 0 . In other words,  0 is a metrizable
-approximation for the space T , so we can apply Fact 2 of V.334 again to conclude
that T is paracompact and  -metrizable and hence Fact 1 is proved.
Returning to our solution observe that X has a weaker metrizable topology by
Problem 333; therefore Y also has a weaker metrizable topology and, in particular,
the diagonal of Y is a G -subset of Y  Y . The space X is paracompact and  metrizable by Problem 335, so we can apply Fact 1 to see that Y is also paracompact.
Finally, it follows from Fact 7 of U.421 that Y is metrizable.
V.363. Suppose that X is l-equivalent to a metrizable space. Prove that A is an
@0 -space for any countable set A  X .
Solution. If Z is a space and B  Z then hBi is the linear hull of B in Lp .Z/.
Fact 1. If Z is a space and Y  Z then clLp .Z/ .hY i/ D hclZ .Y /i. In other words,
the closure of the linear hull of Y in Lp .Z/ coincides with the linear hull of the
closure of Y in Z.
Proof. The set P D hclZ .Y /i is closed in Lp .Z/ by Fact 2 of V.246, so it follows
from hY i  P that Q D clLp .Z/ .hY i/  P . For every n 2 N consider the map
'n W Rn  Z n ! Lp .Z/ defined by the formula 'n .t; z/ D t1 z1 C : : : C tn zn for any
t D .t1 ; : : : ; tn / 2 Rn and z D .z1 ; : : : ; zn / 2 Z n .
Let F D clZ .Y /; it follows from continuity of 'n that the set Hn D 'n .Rn  Y n /
is denseSin the set Gn D 'n .Rn  F n / for S
every n 2 N. As a consequence, the set
hY i D fHn W n 2 Ng is dense in hF i D fGn W n 2 Ng, so Q D P , i.e., Fact 1
is proved.
l

Returning to our solution fix a metrizable space M such that X  M and let
A0 D A. We can consider that M  Lp .X / is a Hamel basis of Lp .X / such that

398

2 Solutions of problems 001500

every f 2 C.M / can be extended to a continuous linear functional on Lp .X / (see


Problem 238).
Since M is a Hamel basis of the space Lp .Z/, there exists a countable set B0 
M such that A0  hB0 i. Proceeding by induction, it is easy to construct sequences
fAn W n 2 !g and fBn W n 2 !g such that An  X; Bn  M while we have the
inclusions
S An  hBn i; An 
S AnC1 and Bn  hAnC1 i; Bn  BnC1 for all n 2 !.
If P D n2! An and B D n2! Bn then it is straightforward that hP i D hBi, so
hP i D hBi by Fact 1.
By paracompactness of X (see Problem 335) and M the sets P and B are C embedded in X and in M respectively. By Fact 3 of V.246, the space hP i is linearly
homeomorphic to Lp .P / while hBi is linearly homeomorphic to Lp .B/. Therefore
the spaces P and B are l-equivalent; by metrizability of M , the space B is second
countable, so P is an @0 -space by Problem 348. It follows from A  P that A is
also an @0 -space (see Problem 341), i.e., our solution is complete.
V.364. For any space X let dc.X / D supfjU j W U   .X / and U is a discrete
familyg. Prove that if X is l-equivalent to a metrizable space then nw.X / D dc.X /.
In particular, if X is l-equivalent to a metrizable space then both the Souslin
property of X and ext .X /  ! imply that X has a countable network.
Solution. It is trivial that dc.X /  c.X /  nw.X / and dc.X /  ext .X / for
any space X . Now, if X is l-equivalent to a metrizable space then it is paracompact
and there
S exists a countable family F of metrizable closed subsets of X such that
X D F (see Problem 335).
Suppose that  is an infinite cardinal while dc.X /   and ext .X / > ; then
there is a closed discrete set D  X such that jDj D  C . By collectionwise
normality of X there is a discrete family U D fUd W d 2 Dg such that d 2 Ud
for every d 2 D. Since U   .X / and jU j > , we obtained a contradiction
which shows that ext .X /   and hence ext .X /  dc.X /. Given F 2 F we have
ext .F /  ext .X /  , so it followsSfrom metrizability of F that nw.F /   (see
TFS-214). Since F is countable and F D X , we can apply SFFS-405 to conclude
that nw.X /  . As a consequence, nw.X /  dc.X / and hence nw.X / D dc.X /
as promised.
Finally observe that if c.X /  ! then nw.X / D dc.X /  c.X /  !. If
ext .X /  ! then nw.X / D dc.X /  ext .X /  !, so both countable extent and
the Souslin property of X imply that nw.X /  !.
V.365. Given a space X and a first countable space Y assume that there exists
a continuous linear surjection ' W Cp .X / ! Cp .Y /. For any yP2 Y there exist
x1 ; : : : ; xn 2 X and 1 ; : : : ; n 2 Rnf0g such that '.f /.y/ D niD1 i f .xi / for
any f 2 Cp .X /; denote the set fx1 ; : : : ; xn g by supp.y/. Suppose that U is a locally
finite open cover of X and let T .U / D fy 2 Y W supp.y/\U ;g for every U 2 U .
Prove that the family fT .U / W U 2 U g is a locally finite open cover of Y .
Solution. Observe first that, for any y 2 Y , the definition of supp.y/ is consistent
by Problem 279.

2 Solutions of problems 001500

399

Fact 1. If Z is a space then a set A  Z is not bounded in Z if and only if there


exists an infinite locally finite family V   .Z/ such that V \ A ; for every
V 2 V.
Proof. If A is not bounded then there exists an infinite discrete family V   .Z/
such that V \ A ; for each V 2 V (see Fact 1 of V.245); it is evident that V is
locally finite, so we proved necessity.
Now assume that there is an infinite locally finite family V   .Z/ such that
V \ A ; for every V 2 V. Fix V0 2 V and a point x0 2 V0 \ A; there exists a set
W0 2 .x0 ; Z/ such that W 0  V0 and W 0 meets only finitely many elements of V.
Proceeding inductively assume that n 2 ! and we have chosen V0 ; : : : ; Vn 2 V,
points x0 ; : : : ; xn 2 A and sets W0 ; : : : ; Wn 2 .Z/ such that
(1) xi 2 Wi \ A and W i  Vi for all i  n;
(2) Vi \ W j D ; whenever 0  j < i  n;
(3) for every i  n, the set W i meets only finitely many elements of V.
By the inductive assumption, the set W D W 0 [ : : : [ W n meets only finitely
many elements of V, so there exists VnC1 2 V such that VnC1 \ W D ;. Pick a
point xnC1 2 VnC1 \ A and choose a set WnC1 2 .xnC1 ; Z/ such that W nC1 meets
only finitely many elements of V and W nC1  VnC1 . It is straightforward that the
conditions (1)(3) are now satisfied if we replace n with n C 1. Therefore we can
construct a set fxi W i 2 !g and families fVi W i 2 !g and fWi W i 2 !g such that the
properties (1)(3) hold for all n 2 !.
The family fVi W i 2 !g is locally finite, so it follows from (1) that
S the family
W D fW i W i 2 !g is also locally finite and hence the set Fi D fW j W j 2
!nfi gg is closed in Z for every i 2 !. Observe that (1) and (2) imply that the
family W is disjoint.
Fix a point x 2 Z; if x 2 W i for some i 2S! then G D S
ZnFi 2 .x; Z/
and G intersects only one element of W. If x W then X n. W/ is an open
neighborhood of x which meets no elements of W. This proves that the family W
is discrete and hence fWi W i 2 !g is discrete as well. Since xi 2 Wi \ A for all
i 2 !, we can apply Fact 1 of V.245 to conclude that A is not bounded in Z. This
settles sufficiency and shows that Fact 1 is proved.
Returning to our solution let .y/ D supp.y/ for every y 2 Y and observe that
the map  W Y ! exp.X / is lower semicontinuous by Problem 280; an immediate
consequence is that the set T .U / is open in Y for each U 2 U . If y 2 Y then .y/
is a nonempty subset of X ; since U is a cover of X , there exists U 2 U with U \
.y/ ; and hence y 2 T .U /. This shows that the family T D fT .U / W U 2 U g
is a cover of Y .
Finally fix a point y 2 Y and a local base fOn W n 2 !g of Y at the point y such
that OnC1  On for every n 2 !. If T is not locally finite at y then every set On

400

2 Solutions of problems 001500

intersects infinitely many elements of T , so we can choose a faithfully indexed (and


hence infinite) family U 0 D fUn W n 2 !g  U and a set fyn W n 2 !g  Y such
that yn 2 On \ T .Un / for all n 2 !.
The set K D fyg [ fyn W n 2 !g is easily
S seen to be a convergent sequence,
so K is compact and hence the set L D fsupp.z/ W z 2 Kg is bounded in X
by Problem 281. For every n 2 ! the set supp.yn / is contained in L, so it follows
from yn 2 T .Un / that Un \ L ;. Thus U 0 is an infinite locally finite family
and U \ L ; for every U 2 U 0 ; therefore L is not bounded by Fact 1. This
contradiction shows that the cover T is locally finite at any y 2 Y and hence our
solution is complete.
V.366. Let X and Y be metrizable spaces for which there exists a continuous linear

surjection of Cp .X / onto Cp .Y /. Prove that, if X is Cech-complete


then Y is also

Cech-complete.
Solution. IfSZ is a space and A is a family of subsets of Z then, as usual, we let
St.Y; A/ D fA 2 A W A \ Y ;g for any Y  Z.

Fact 1. Suppose that .M; / is a metric space which is not Cech-complete


and let
fUn W n 2 !g be a sequence of locally finite open covers of M with the following
properties:
(i) UnC1 is a star refinement of Un for all n 2 !;
(ii) diam .U /  2n for all n 2 ! and U 2 Un ;
(iii) if n 2 ! and U 2 UnC1 then U meets only finitely many elements of Un .
Then
T we can choose a set Un 2 Un in such a way that U nC1  Un for all n 2 !
and fUn W n 2 !g D ;.
Proof. By TFS-237, there is a complete metric space .N; 0 / such that .M; / is
isometric to a dense subspace of .N; 0 /. To simplify the notation we will consider
that .N; / is a complete metricSspace and M is a dense subspace of N . If U is an
open subset of M let O.U / D fG 2 .N / W G \ M  U g. It is immediate that
(1) if U; V 2 .M / then O.U / \ M D U and U  V implies O.U /  O.V /.
S
For every n 2 ! let Vn D fO.U / W U 2 Un g. If Vn D TVn then Vn is an
open subset of N such that M  Vn for every n 2 !. If V D n2! Vn then V is

Cech-complete,
so there exists a point z 2 V nM . Let us prove that
(2) the family Wn D fW 2 Un W z 2 O.W /g is finite for every n 2 !.
Indeed, if some Wn is infinite then take any U 2 UnC1 such that z 2 O.U /; then,
for any W 2 Wn we have z 2 O.W / \ O.U /, so O.W / \ O.U / is a nonempty
open subset of N and hence U \ W D O.U / \ O.W / \ M ;. However, this
contradicts (iii) and shows that (2) is proved.
If W 2 Un and W 0 2 UnC1 say that W W 0 if St.W 0 ; UnC1 /  W . If n 2
!; k > 1 and we are given sets W 2 Un and W 0 2 UnCk say that W W 0 if
there exist sets W1 ; : : : ; Wk1 such that Wi 2 UnCi for all i D 1; : : : ; k  1 and

2 Solutions of problems 001500

401

W W1 : : : Wk1 W 0 . It is clear that W W 0 W 00 implies W W 00 ;


besides, it follows from (1) that for every n 2 !; m > n and W 2 Un if W 0 2 Wm
and W W 0 then W 2 Wn .
The condition (i) implies that,
(3) if n 2 !; n < m and W 0 2 Wm then there is W 2 Wn such that W W 0 .
S
For every W 2 W D n2! Wn let S.W / D fW 0 2 W W W W 0 g and
A.W / D fn 2 ! W S.W / \S
Wn ;g.
S
It follows from (3) that fWn W n > 0g D fS.W / W W 2 W0 g, so we can
choose U0 2 W0 such that the set A.U0 / is infinite.
Proceeding by induction assume that k 2 ! and we have sets U0 ; : : : ; Uk such
that
(4) Ui 2 Wi and the set A.Ui / is infinite for all i  k;
(5) Ui UiC1 for all i < k.
Let H D fW 2 WkC1 W Uk W g; it follows from (3) and (4) that H is
0
nonempty. If i > k C 1 and W 2 S.Uk / \ WS
i then there exists a set W 2 H
0
such that W W .S
As a consequence,
the set fS.Uk / \ Wi W i > k C 1g is
S
contained in the set W 2H i>kC1 S.W / \ Wi . The family H being finite there
exists UkC1 2 H such that A.UkC1 / is infinite; it is clear that the statements (4) and
(5) remain true if we replace k with k C 1. Therefore our inductive procedure can
be continued to construct a family fUi W i 2 !g such that the conditions (4) and (5)
are satisfied for all k 2 !. In particular, Un 2 Un and St.UnC1 ; UnC1 /  Un ; it is an
easy exercise that this implies U nC1  Un for all n 2 !.
T
The condition (ii) shows that diam .O.U
T n // D diam .Un / ! 0, so n2! O.Un /
can contain
T at most one point; therefore n2! O.Un / D fzg and hence we have the
equality n2! Un D fzg \ M D ; which shows that Fact 1 is proved.
Fact 2. Given a metric space .Z; d / a set A  Z is totally bounded in Z if and
only if, for any " > 0, there exists a finite cover P  exp.Z/ of the set A such that
diamd .P / < " for each P 2 P.
Proof. For any r > 0 and z 2 Z let B.z; r/ D fy 2 Z W d.z; y/ < rg be the ball
of radius r centered at z. If A
Sis totally bounded and " > 0 then there exists a finite
set F  Z such that A  fB.a; 3" / W a 2 F g. If P D fB.a; 3" / W a 2 F g then
S
diamd .P /  23 " < " for every P 2 P and A  P, so we proved necessity.
Now assume that A can be covered by a finite number of sets of arbitrarily small
S
diameter and fix " > 0. There exists a finite family P  exp.Z/ such that A  P
and diamd .P / < " for each P 2 P; we can assume that all elements of P are
nonempty.
S ForSevery P 2 P choose a point xP 2 P ; then P  B.xP ; "/ and hence
A  P  fB.xP ; "/ W P 2 Pg, so the finite set F D fxP W P 2 Pg witnesses
that A is totally bounded. Fact 2 is proved.
Returning to our solution suppose that ' W Cp .X / ! Cp .Y / is a linear surjection

and X is Cech-complete;
fix a complete metric  on X which generates the topology

of X . Suppose that Y is not Cech-complete


and take any metric d on Y such that

402

2 Solutions of problems 001500

.d / D .Y /. Using paracompactness of X it is easy to construct a sequence fUn W


n 2 !g of locally finite covers of X such that, for any n 2 ! and U 2 Un we have
diam .U /  2n and UnC1 is a refinement of Un .
For every y 2 Y there
Pkexist x1 ; : : : ; xk 2 X and 1 ; : : : ; k 2 Rnf0g
such that '.f /.y/ D
2 Cp .X / (see ProbiD1 i f .xi / for each f
lem 279); let supp.y/ D fx1 ; : : : ; xk g. For every A  X consider the set
T .A/ D fy 2 Y W supp.y/ \ A ;g; then Tn D fT .U / W U 2 Un g is a locally
finite open cover of Y for each n 2 ! (see Problem 365). This makes it possible
to construct a sequence fVn W n 2 !g of locally finite open covers of Y with the
following properties:
(6)
(7)
(8)
(9)

if n 2 ! then V ; and diamd .V /  2n for any V 2 Vn ;


VnC1 is a star refinement of Vn for each n 2 !;
if n 2 ! then every V 2 VnC1 intersects only finitely many elements of Vn ;
for any n 2 ! each V 2 Vn intersects only finitely many elements of Tn .

By
TFact 1 we can choose Vn 2 Vn in such a way that V nC1  Vn for every n 2 !
and TfVn W n 2 !g D ;. Pick a point yn 2 Vn for every n 2 !; it is easy to deduce
from n2! V n D ; that
(10) the set D D fyn W n 2 !g is infinite, closed and discrete in Y .
T .U / ;g is finite by (9) and we have
The familySWn D fU 2 Un W Vn \ S
the inclusion
fsupp.y/
W
y
2
V
g

Wn for every n 2 !. Consider the set
n
S
K D fsupp.yn / W n 2 !g; given any " > 0 there exists n 2 ! such that 2n < ".
Then fyi W i  ng  Vn , so P D ffyi g W i < ng [ Wn is a finite cover of K such
that diam .P /  2n < " for every P 2 P. Since " > 0 was taken arbitrarily,
Fact 2 shows that K is totally bounded and hence K is a compact subset of X (see
TFS-212). By Problem 282, the set C D fy 2 Y W supp.y/  Kg is bounded in Y
and hence so is D  C . Since D is closed in Y , it must be compact (recall that Y

is a metric space); this contradiction shows that Y is Cech-complete


as promised.

V.367. Prove that if a metrizable space X is l-equivalent to a Cech-complete


space

then X is also Cech-complete. As a consequence, if X and Y are l-equivalent


metrizable spaces then X is metrizable by a complete metric if and only if so is Y .
l

Solution. Suppose that Y is a Cech-complete


space and X  Y . By Problem 362

the space Y is metrizable, so we can apply Problem 366 to see that X is Cechcomplete. Now, if X and Y are l-equivalent metrizable spaces and X is metrizable

by a complete metric then X is Cech-complete


by TFS-269, so Y is Cech-complete
as well. Applying TFS-269 again we conclude that Y is metrizable by a complete
metric. Since X and Y are in a symmetric situation, metrizability of Y by a complete
metric also implies metrizability of X by a complete metric.
V.368. Show that there exist first countable l-equivalent spaces X and Y such that
X is locally compact and Y is not locally compact.
Solution. In the space P D !1 .! C1/ consider the set F D !1 f!g; it is evident
that F is a retract of P . The space !1 is locally compact and ! C 1 is compact; it is

2 Solutions of problems 001500

403

an easy exercise that the product of two locally compact spaces is locally compact,
so P is locally compact and hence so is the space X D P C . It is also clear that the
space X is first countable.
Apply Problem 258 to see that X is l-equivalent to the space Y D PF F where
PF is the R-quotient space obtained from P by contracting the set F to a point. The
subspace F is homeomorphic to !1 which is first countable; besides, F is clopen in
Y , so Y is first countable at every point of F . Let aF be the point represented by
the set F in the space PF . Observe that PF nfaF g is homeomorphic to !1  ! (see
Problem 252), so PF is first countable at every point of PF nfaF g. To see that PF is
also first countable at aF , let On D !1  ..! C 1/nn/ for every n 2 ! (recall that
we identify every n 2 ! with the set f0; : : : ; n  1g). It turns out that
(1) for any U 2 .F; P / there exists n 2 ! such that On  U .
To prove (1) denote by L the set of all limit ordinals in !1 and take, for any
ordinal 2 L, an ordinal ./ and a number m./ 2 ! such that the set ../;

..! C 1/nm.// is contained in U . Apply SFFS-065 to find a stationary set A  !1
such that there exists k 2 ! for which m./ D k for all 2 A. By SFFS-067, there
exists < !1 and an uncountable set B  A such that ./ D for every 2 B.
An immediate consequence is that .; !1 /  ..! C 1/nk/  U . It is easy to deduce
from compactness of C1 that there exists l 2 ! such that . C1/..! C1/nl// 
U ; now if n D maxfk; lg then On  U , i.e., (1) is proved.
It is easy to see that F is a zero-set in P and every On is a clopen subset of P .
This, together with Problem 253, shows that the set Gn D faF g [ .On nF / is an
open neighborhood of aF in P for each n 2 !. Next, apply (1) to conclude that the
family fGn W n 2 !g is a local base at the point aF in PF , so PF is first countable at
aF and hence the space Y is first countable.
Finally, take any U 2 .aF ; Y /; by (1) there exists n 2 ! such that Gn  U and
hence Q D !1  fn C 1g  Gn is a non-compact subset of U which is closed in Y .
Therefore Q is also closed in U , so U cannot be compact. This shows that Y is not
locally compact at the point aF and hence our spaces X and Y are first countable
and l-equivalent while X is locally compact and Y is not.
V.369. Given a space Z let Z 0 be the set of non-isolated points of Z. Suppose
that X and Y are normal first countable l-equivalent spaces. Prove that if X 0 is
countably compact then Y 0 is also countably compact. Show that this statement can
be false if we omit first countability of X and Y .
Solution. It follows from Problem 263 that the space .! C 1/  ! is l-equivalent to
the FrchetUrysohn !-fan V .!/. The spaces V .!/ and .! C 1/  ! are countable
and hence normal; besides, the set of non-isolated points of .!C1/! is infinite and
discrete while V .!/ has a unique non-isolated point. Therefore even compactness of
X 0 does not need to imply that Y 0 is countably compact if we omit first countability
of X .
Now assume that X and Y are first countable l-equivalent normal spaces and X 0
is countably compact. If Y 0 is not countably compact then we can fix a faithfully

404

2 Solutions of problems 001500

indexed closed discrete subset D D fdn W n 2 !g of the subspace Y 0 . It is clear that


D is also closed in Y ; it follows from normality of Y that
(1) the set D is not bounded in Y .
Choose a disjoint family fOn W n 2 !g  .Y / such that dn 2 On for each
n 2 !. Fix a decreasing local base fBmn W m 2 !g at every point dn in such a
way that B0n  On ; pick a function fmn 2 Cp .Y; 0; 1
/ such that fmn .dn / D 1 and
fmn .Y nBmn /  f0g for every m 2 !. If n is the characteristic function of the point
dn in Y then n is discontinuous and the sequence Pn D ffmn W m 2 !g converges
to n for all n 2 !.
By Fact 1 of V.272 there exists a linear homeomorphism ' W RY ! RX such that
'.Cp .Y // D Cp .X /. If Qn D '.Pn / then Qn  Cp .X / is a sequence converging
to a function n D '. n / which is discontinuous on X . Therefore we can choose a
point an 2 X 0 such that the function n is discontinuous at an for every n 2 !. The
set X 0 being countably compact and first countable, the sequence A D fan W n 2 !g
has a convergent subsequence. Passing, if necessary, to an appropriate subsequence
of D we can assume, without loss of generality, that there is a point a 2 X 0 such
that an ! a.
Choose a decreasing local base fWn W n 2 !g at the point a in X . Passing to
the relevant subsequences, we can assure that an 2 Wn for all n 2 !. Since n is
n
discontinuous at an , we can find a sequence Sn D fxm
W m 2 !g  Wn which
converges to an while
n
(2) the sequence fn .xm
/ W m 2 !g does not converge to n .an / for every n 2 !.

The set Kn D Sn [ fan g is compact for each n 2 ! and every open neighborhood
of a contains all but S
finitely many of the sets Kn . An immediate consequence is that
the set K D fag [ . fKn W n 2 !g/ is compact.
For every x 2 X there exist distinct P
points y1 ; : : : ; yn 2 Y and numbers
1 ; : : : ; n 2 Rnf0g such that '.f /.x/ D niD1 i f .yi / for all f 2 Cp .Y / (see
Problem 279); let supp.x/ D fy1 ; : : : ; yn g. As a trivial consequence,
(3) for every x 2 X , if f 2 Cp .Y / and f .supp.x// D f0g then '.f /.x/ D 0.
S
For every set E  X let supp.E/ D fsupp.x/ W x 2 Eg; from compactness
of K it follows that the set supp.K/ is bounded in Y (see Problem 281 and Fact 2
of S.398). Apply (1) to conclude that D is not contained in supp.K/ and fix n 2 !
such that dn 2 Dnsupp.K/. There exists k 2 ! for which Bmn \ supp.K/ D ;
and hence fmn .supp.K// D f0g for all m  k. Apply (2) to see that '.fmn /.x/ D 0
for all x 2 K and m  k. The sequence f'.fmn / W m  kg converges to n , so
n
n .x/ D 0 for all x 2 K. In particular, n .xm
/ D 0 D n .an / for all m  k; this
0
contradiction with (2) shows that Y is countably compact and hence our solution is
complete.
V.370. Given a nonempty closed subspace F of a normal space X suppose that F
is a retract of some neighborhood of F . Prove that F is l-embedded in X and hence
Cp .X /
Cp .F /  I , where I D ff 2 Cp .X / W f .F / D f0gg.

2 Solutions of problems 001500

405

Solution. Fix a set O 2 .F; X / such that there exists a retraction r W O ! F .


By normality of X we can find a set G 2 .F; X / for which G  O; choose a
continuous function p W X ! 0; 1
with p.F /  f1g and p.X nG/  f0g. Given
any f 2 Cp .F / define a function '.f / 2 RX as follows: '.f /.x/ D 0 for all
x 2 X nG and '.f /.x/ D f .r.x// p.x/ for each x 2 G.
Observe first that, for any point x 2 F , we have r.x/ D x and p.x/ D 1, so
'.f /.x/ D f .x/, i.e., '.f /jF D f and hence ' W Cp .F / ! RX is an extension
operator. The function '.f / is constant on X nG, so it is continuous at all points of
the open set X nG. On the set O the function '.f / coincides with the continuous
function .f r/ p, so '.f / is continuous on O and hence '.f / 2 Cp .X / (see
Fact 1 of S.472) for every f 2 F which shows that ' W Cp .F / ! Cp .X /. It is
straightforward that ' is a linear map, so let us check that ' is continuous.
Given a point x 2 X let x .f / D f .x/ for all f 2 Cp .X /. If x 2 X nG
then .x '/.f / D '.f /.x/ D 0 for any f 2 Cp .F /, so the map x ' is
continuous. If x 2 G then .x '/.f / D p.x/ f .r.x// D p.x/r.x/ .f /, so
x ' W Cp .F / ! R coincides with the continuous function p.x/ r.x/ which
shows that x ' is continuous for all x 2 X and hence ' is continuous by TFS102. Thus ' is a continuous linear extender for F and hence F is l-embedded in X .
Applying CFS-448 we conclude that Cp .X / is linearly homeomorphic to Cp .F /I
as promised.
V.371. Given a space X assume that X0 and X1 are closed subspaces of X such
that X D X0 [ X1 and the set F D X0 \ X1 is l-embedded in X ; suppose
additionally that Cp .F /
Cp .F /Cp .F /. Prove that Cp .X /
Cp .X0 /Cp .X1 /.
Solution. If Z is a space and A  Z then IAZ D ff 2 C.Z/ W f .A/  f0gg and
A W Cp .Z/ ! Cp .A/ is the restriction map; if z 2 Z and A D fzg then we write
z instead of A .
Fact 1. Suppose that Z is a space, Z0 and Z1 are closed in Z and Z D Z0 [ Z1 . If
Y D Z0 \ Z1 then the space IYZ is linearly homeomorphic to IYZ0  IYZ1 .
Proof. For any function f 2 IYZ let '.f / D .Z0 .f /; Z1 .f //; it is immediate
that ' W IYZ ! IYZ0  IYZ1 is a continuous linear map. If f; g 2 IYZ and f g then
there is a point z 2 Z such that f .z/ g.z/. Take i 2 f0; 1g with z 2 Zi ; then
Zi .f / Zi .g/ and hence '.f / '.g/ which shows that the map ' is injective.
Given any .f0 ; f1 / 2 IYZ0  IYZ1 let f .x/ D f0 .x/ for all x 2 Z0 and f .x/ D
f1 .x/ whenever x 2 ZnZ0 . It is an easy consequence of Fact 2 of T.354 that f is
continuous on X ; since also '.f / D .f0 ; f1 /, the map ' is surjective.
Given i 2 f0; 1g and a point z 2 Zi let qzi .f / D f .z/ for each f 2 IYZi and
let i W IYZ0  IYZ1 ! IYZi be the natural projection. To show that the map ' 1 is
continuous fix a point z 2 Z. Then z 2 Zi for some i 2 f0; 1g; now, if we are
given p D .f0 ; f1 / 2 IYZ0  IYZ1 then .z ' 1 /.p/ D fi .z/ D qzi .i .p//, so the
map z ' 1 is continuous because it coincides with the continuous map qzi i .
By TFS-102 the map ' 1 is continuous, so ' is a linear homeomorphism and hence
Fact 1 is proved.

406

2 Solutions of problems 001500

Returning to our solution observe that Cp .X /


Cp .F /  IFX by CFS-448; by
Fact 1, we have IFX
IFX0  IFX1 , so Cp .X /
Cp .F /  IFX0  IFX1 . Recalling that
Cp .F /
Cp .F /  Cp .F / we conclude that
(1) Cp .X /
.Cp .F /  IFX0 /  .Cp .F /  IFX1 /.
It is easy to see that F is l-embedded in both X0 and X1 , so we can apply CFS448 again to convince ourselves that Cp .F /  I Xi
Cp .Xi / for each i 2 f0; 1g;
this, together with (1), shows that Cp .X /
Cp .X0 /  Cp .X1 / and completes our
solution.
V.372. Suppose that a space X has a nontrivial convergent sequence. Prove that
l

X is l-equivalent to X .! C 1/ and X C  X . Deduce from this fact that X C is


l-equivalent to X for every infinite metrizable space X .
Solution. Since X has a nontrivial convergent sequence, we can find a faithfully
indexed set A D fan W n 2 !g  X and a point a 2 X nA such that A converges
to a. It is clear that S D A [ fag is homeomorphic to ! C 1. Consider the set
I D ff 2 Cp .X / W f .S / D f0gg; it follows from CFS-482 that S is l-embedded in
X , so Cp .X /
I  Cp .S / by CFS-448. Apply Problem 276 to see that Cp .S / 
Cp .S /
Cp .S / and therefore Cp .X /
I  Cp .S /  Cp .S /
Cp .X /  Cp .S /

Cp .X .! C 1// which shows that the spaces X and X .! C 1/ are l-equivalent.


Next observe that the spaces S and S C are homeomorphic so it follows from
Cp .X /
I  Cp .S C / that Cp .X /
I  Cp .S /  R
Cp .X /  R
Cp .X C / and
l

hence X  X C . Finally assume that X is infinite and metrizable; if ! C 1 embeds in


l

X then X C  X by what we proved above. If X does not have nontrivial convergent


sequences then it is an infinite discrete space so X C is even homeomorphic to X
l

and hence again X C  X .


V.373. Suppose that a space X has a nontrivial convergent sequence and Y is lembedded in X . Prove that X is l-equivalent to XY Y . Consequently, for any
l

infinite metrizable space X , if Y is closed in X then X  XY Y . Here XY is the


R-quotient space obtained from X by contracting Y to a point.
Solution. Assume first that X has a nontrivial convergent sequence. It follows from
l

Problem 258 that X C is l-equivalent to XY Y ; thus, X  X C  XY Y (see


Problem 372). If X is an infinite metrizable space then X C is l-equivalent to X by
Problem 372 and any closed Y  X is l-embedded in X by CFS-469. Therefore
l

X  X C  XY Y as promised.
V.374. Suppose that X is a compact space and F is l-embedded in X . Prove that
X C is l-equivalent to F .X nF /. In particular, if X is an infinite metrizable
l

compact space then X  F .X nF / for any closed F  X .


Solution. If Z is a space and Y is a closed subset of Z then, as usual, ZY is the
R-quotient space obtained from Z by contracting Y to a point.

2 Solutions of problems 001500

407

Fact 1. Suppose that K and L are compact spaces, a is a non-isolated point of K


and b is a non-isolated point of L. If Knfag is homeomorphic to Lnfbg then there
exists a homeomorphism ' W K ! L such that '.a/ D b. In particular, K is
homeomorphic to .Knfag/.
Proof. Fix a homeomorphism h W Knfag ! Lnfbg; let '.a/ D b and '.x/ D h.x/
for any x 2 Knfag. It is clear that ' W K ! L is a bijection which is continuous
at all points of Knfag. Analogously, ' 1 is continuous at all points of Lnfbg. If
U 2 .a; K/ then P D KnU is compact so P 0 D h.P / is also compact and hence
'.U / D LnP 0 is open in L. An analogous proof shows that ' 1 .V / is open in K
for any V 2 .b; L/. Therefore ' is continuous at a and ' 1 is continuous at b, so
' is a homeomorphism and hence Fact 1 is proved.
l

Returning to our solution apply Problem 258 to see that X C  XF F . If aF


is the point of XF represented by F then XF nfaF g is homeomorphic to X nF by
Problem 252. Applying Fact 1 we conclude that XF is homeomorphic to .X nF /
l

and therefore X C  F .X nF /. If X is an infinite metrizable compact space then


the space X C is l-equivalent to X by Problem 372; besides, every closed subset of
l

X is l-embedded in X by CFS-469, so X  X C  XF F  .X nF / F . The


last equivalence takes place because XF is homeomorphic to .X nF / by Fact 1.
V.375. Let X and Y be metrizable compact spaces. Suppose that F and G
l

are closed subspaces of X and Y respectively such that F  G and X nF is


homeomorphic to Y nG. Prove that X is l-equivalent to Y .
l

Solution. If X is finite then F is finite and it follows from F  G that jF j D


jGj (see Problem 159); the spaces X nF and Y nG being homeomorphic, we have
jX nF j D jY nGj, so Y is also finite and jY j D jX j which shows that X and Y are
even homeomorphic.
l

If X is infinite then either F of X nF is infinite. In the first case F  G implies


that G is infinite (see TFS-186); if X nF is infinite then Y nG is infinite, so in both
cases the space Y has to be infinite. Now we can apply Problem 374 to see that
l

we have the equivalencies X  F .X nF / and Y  G .Y nG/. Since F  G


and .X nF / is homeomorphic (and hence l-equivalent) to .Y nG/, we can apply
l

Problem 265 to conclude that X  Y .


V.376. Let X and Y be nonempty compact metrizable spaces such that either Y 
.! C 1/ or .Y  !/ embeds in X . Prove that Cp .X /
Cp .X /  .Cp .Y //n for
every n 2 N.
Solution. As usual, we identify every m 2 ! with the set f0; : : : ; m  1g. Fix n 2 N
and let F  X be homeomorphic either to Y  .! C 1/ or to .Y  !/. It is an easy
exercise that
(1) F .Y  n/ is homeomorphic to F and hence Cp .F /
Cp .F /  .Cp .Y //n .

408

2 Solutions of problems 001500

If I D ff 2 Cp .X / W f .F / D f0gg then the space Cp .X / is linearly


homeomorphic to I  Cp .F / (see CFS-448 and CFS-482), so we can apply the
property (1) to conclude that Cp .X /
I  Cp .F /
I  Cp .F /  .Cp .Y //n

Cp .X /  .Cp .Y //n .
V.377. Say that a metrizable compact space K is universal in the dimension n 2 !
if dim K D n and any metrizable compact space of dimension at most n embeds in
K. Prove that if X and Y are metrizable compact spaces universal in the dimension
l

n, then X  Y .
Solution. If a compact space K is universal in the dimension n then the space K 0 D
K  .! C 1/ embeds in K because K 0 is compact, metrizable and dim K 0 D n
due to the fact that K 0 is the countable union of its subspaces homeomorphic to K
(see Problem 150). It follows from Problem 376 that Cp .K/
Cp .K/  Cp .K/.
In particular, Cp .X / is linearly homeomorphic to .Cp .X //2 and Cp .Y / is linearly
homeomorphic to .Cp .Y //2 . By universality of Y in the dimension n, the space X
embeds in Y and the space Y embeds in X by universality of X in the dimension n,
l

so we can apply Problem 292 to conclude that X  Y .


V.378. Suppose that a space X is l-equivalent to Y and Y is a metrizable compact
space universal in the dimension n. Prove that X is also a metrizable compact space
universal in the dimension n.
Solution. Say that a compact space K is strongly universal in the dimension n if
K is universal in the dimension n and can be embedded into any nonempty open
subspace of K.
Fact 1. If a compact space K is universal in the dimension n then it contains a
compact subspace which is strongly universal in the dimension n.
Proof. Fix an n-dimensional compact space P such that any second countable
space of dimension at most n embeds in P and, in particular, P is universal in the
dimension n; such a space exists by Problem 162. The space P ! is second countable
and P embeds in any set U 2  .P ! /. Fix a base B D fBn W n 2 !g   .P ! / in
the space P !Sand choose a set Pn  Bn homeomorphic to P for every n 2 !. The
space G D n2! Pn is second countable and dim G  n by Problem 150. By the
choice of P , there is a set G 0  P homeomorphic to G; let H D G 0 .
The space H is compact, metrizable and dim H  n; if U 2  .H / then U 0 D
U \ G 0 is homeomorphic to a nonempty open set W of the space G. Take W 0 2
.P ! / such that W 0 \ G D W and pick n 2 ! such that Bn  W 0 . Then Pn 
W 0 \ G D W and hence Pn embeds in U 0  U . Since Pn is universal in the
dimension n, the space H embeds in Pn and hence in U ; this shows that H is
strongly universal in the dimension n. Since also dim H  n, the space H embeds
in K, so K has a compact subspace which is strongly universal in the dimension n,
i.e., Fact 1 is proved.

2 Solutions of problems 001500

409

Returning to our solution observe first that the space X is compact by


Problem 138 and dim X D n by Problem 180. Apply Fact 1 to find a strongly
universal subspace K  Y . By Problem 309, there exists U 2  .K/ which is
embeddable in X . Being strongly universal in the dimension n, the space K embeds
in U and hence in X . It is trivial that every compact n-dimensional space which
contains a subspace universal in the dimension n is universal in the dimension n, so
X is universal in the dimension n as required.
V.379. Prove that, for any n1 ; : : : ; nk 2 N, the space Cp .In1 /  : : :  Cp .Ink / is
linearly homeomorphic to Cp .In / where n D maxfn1 ; : : : ; nk g.
m
Solution. For any
p points x D .x1 ; : : : ; xm / and y D .y1 ; : : : ; ym / of the space R
2
2
let dm .x; y/ D .x1  y1 / C : : : C .xm  ym / ; as usual, for each r > 0, the set
Bm .x; r/ D fy 2 Rm W dm .x; y/ < rg is the open ball of radius r centered at x.
The expression X ' Y says that the spaces X and Y are homeomorphic. It is easy
to see that Bm .x; r/ ' Bm .y; s/ for any x; y 2 Rm and r; s > 0. Let um 2 Rm be
the zero point of Rm .

Fact 1. For any m 2 N and U 2  .Rm / the space .Im  !/ embeds in U .


Proof. If V is a nonempty open subset of Rm then take a point a 2 V . There exists
r > 0 such that Bm .a; r/  V ; since Bm .a; r/ is homeomorphic to Bm .um ; 2m/,
the space Bm .um ; 2m/ embeds in V . It is straightforward that Im  Bm .um ; 2m/, so
we proved that
(1) the space Im embeds into any nonempty open subspace of Rm .
Let Wi D Bm .um ; 2i /; it is immediate that Wi nW iC1 ; for every i 2 ! and
fWi W i 2 !g is a decreasing local base at um . Take an open ball Qi  Wi nW iC1
for every i 2 !; then the family fQi W i 2 !g is disjoint and convergent to um in the
sense that any U 2 .um ; Rm / contains all but finitely many Qi s.
The property (1) shows that we can find a set Ji  Qi with Ji ' Im for each
i 2 !. It is standard that the space K D fum g [ fJi W i 2 !g is homeomorphic to
.Im  !/. Moreover, K  W0 and W0 is an open ball which is homeomorphic to
any open ball contained in U . Therefore W0 embeds in U and hence K ' .Im !/
also embeds in U , i.e., Fact 1 is proved.
Returning to our solution, let us prove, by induction on k 2 N that
(2) for any n1 ; : : : ; nk 2 N the space Cp .In1 /  : : : Cp .Ink / is linearly homeomorphic to Cp .In / for n D maxfn1 ; : : : ; nk g.
If k D 1 then (2) is clearly true, so assume that we proved (2) for all k  l and take
any n1 ; : : : ; nl ; nlC1 2 N; let n D maxfn1 ; : : : ; nlC1 g. We can assume, without loss
of generality, that nlC1 D n. It follows from Fact 1 that the space .In1  !/ embeds
in In , so we can apply Problem 376 to see that Cp .In /Cp .In1 /
Cp .In /. Therefore
Ql
Q
QlC1
ni
n
n1
ni
.In /  liD2 Cp .Ini /, so
iD1 Cp .I /
Cp .I /  Cp .I / 
iD2 Cp .I /
CpQ
we can apply the induction hypothesis to see that Cp .In /  liD2 Cp .Ini /
Cp .In /.
This completes the induction step and shows that (2) is true for all k 2 N.

410

2 Solutions of problems 001500

V.380. Given n 2 N and a closed subset F of the space In such that ; F In


prove that .In /F is l-equivalent to In . Here .In /F is the R-quotient image of In
obtained by contracting F to a point.
Solution. Denote by aF the point of the space .In /F represented by the set F and
apply Problem 373 to convince ourselves that In is l-equivalent to .In /F F and
hence Cp .In /
Cp ..In /F /  Cp .F /. Observe that .In /F nfaF g is homeomorphic to
In nF (see Problem 252); since In nF contains a nonempty open subspace of V of the
space Rn , the set .In  !/ embeds in V by Fact 1 of V.379. It follows from F  In
that the set .F !/ embeds in .In !/ and hence .F !/ embeds in .In /F . This,
together with Problem 376, shows that Cp .In /
Cp ..In /F /  Cp .F /
Cp ..In /F /,
l

i.e., .In /F  In as promised.


V.381. Prove that, for any n 2 N, if U is a nonempty open subset of the space Rn ,
then the space .U / is l-equivalent to In .
Solution. The set O D .1; 1/n is homeomorphic to Rn and open in In . Therefore
U is homeomorphic to an open subspace of O and hence there is no loss of
generality to consider that U  O. Then F D In nU is a nonempty closed subset
l

of In and F In . Thus In .In /F where .In /F is the R-quotient space obtained


from In by contracting F to a point aF (see Problem 380). If .U / D U [ fg
then U D .U /nfg is homeomorphic to .In /F nfaF g by Problem 252. Therefore
we can apply Fact 1 of V.374 to see that .In /F is homeomorphic to .U / and hence
l

.In /F  .U /. Consequently, In .In /F  .U /, so .U / is l-equivalent to In .


V.382. Given a space X with dim X D n 2 N assume that X is homeomorphic to
a finite
S union of Euclidean cubes, i.e., there is a finite family F of subsets of X such
that F D X and every F 2 F is homeomorphic to Ik for some k 2 N. Prove that
X is l-equivalent to In .
Solution. The expression Y ' Z says that the spaces Y are Z are homeomorphic.
If Z is a space and Y is a closed subset of Z then ZY is the R-quotient image of Z
obtained by contracting Y to a point; this point will be denoted by aY .
It suffices to prove, by induction on m, that
l

(1) if X D F1 [ : : : [ Fm and Fi ' Ini for all i  m then X  In .


If m D 1 then it follows from dim X D n that n1 D n (see Problem 159), so
l
X ' In and hence X  In . Assume that (1) has been proved for all m < l and
X D F1 [ : : : [ Fl where every Fi is homeomorphic to Ini for some ni 2 N.
It follows from Problem 155 that ni  n for all i  l; if ni < n for each i  l then
dim X < n by Problem 151. This contradiction shows that there exists i  l for
which ni D n; there is no loss of generality to assume that n1 D n.
l

If F1  F D F2 [ : : : [ Fl then X D F2 [ : : : [ Fl , so X  In by the induction


hypothesis. Therefore we can assume that F1 nF ;. If F \F1 D; then X ' F1F ,

2 Solutions of problems 001500

411

so Cp .X /
Cp .F1 /  Cp .F /; by the induction hypothesis, Cp .F /
Cp .Ik / for
k D dim F  n. Thus, Cp .X /
Cp .In /  Cp .Ik /
Cp .In / (see Problem 379),
l

i.e., X  In .
If G D F1 \ F ; then G is a nonempty closed subset of F1 with G F1 .
Consequently, .F1 /G is l-equivalent to In by Problem 380. The space XF is compact
and XF nfaF g is homeomorphic to X nF (see Problem 252); besides, .F1 /G nfaG g is
homeomorphic to F1 nG D X nF . Thus, Fact 1 of V.374 is applicable to conclude
l

that .F1 /G ' XF . It follows from Problem 380 that .F1 /G  In , so XF  In . Apply
l

Problem 373 to see that X  XF F  In F . The induction hypothesis shows


l

that F  In , so X  In In  In (see Problem 379); this completes the inductive


step and shows that (1) is true for any m 2 N.
V.383. For any n 2 N prove that both spaces In  .! C 1/ and .In  !/ are
l-equivalent to In .
Solution. Since the interior of In in Rn is nonempty, it follows from Fact 1 of V.379
that the space L D .In  !/ embeds in In . It is clear that In embeds in L; besides,
it follows from Problem 379 that Cp .In /  Cp .In /
Cp .In /.
Let s be the unique point of Ln.In  !/ and apply Fact 1 of V.379 once more
to see that there is a set Kn  In  fng homeomorphic
to L for each n 2 !. It is
S
immediate to verify that the space K D fsg [ . fKn W n 2 !g/ is homeomorphic
to .L  !/, i.e., .L  !/ embeds in L, so we can apply Problem 376 to convince
ourselves that Cp .L/
Cp .L/  Cp .L/. Now, it follows from Problem 292 that the
space L is l-equivalent to In .
In the space Y D In  .! C 1/ consider the set F D In  f!g; it follows from
Problem 374 that Y is l-equivalent to F .Y nF / D F .In  !/. Since F
l

is homeomorphic to In and .In  !/  In , we conclude that Y  In In  In (see


Problem 379).
V.384. Prove that In  D! is not l-equivalent to In for any n 2 N.
Solution. If the space In  D! is l-equivalent to In for some n 2 N then there is
a nonempty open set U  In  D! which embeds in In (see Problem 309). Take
any sets V 2  .In / and W 2  .D! / such that V  W  U . By Fact 1 of V.379,
the space In embeds in V , so In  W embeds in V  W and hence in In ; let Y
be a homeomorphic copy of In  W in In . Pick any w 2 W ; then In  fwg is an
n-dimensional subset of In  W which is nowhere dense in In  W because W
has no isolated points. Therefore some n-dimensional subspace K of the space Y is
nowhere dense in Y and hence in In . This, however, contradicts Problem 160 and
shows that In  D! is not l-equivalent to In .
V.385. Suppose that K is a compact space and there exists a continuous bijective
map of 0; C1/ onto K. Prove that K is l-equivalent to I. Deduce from this fact
that if there is a continuous bijection of R onto a compact space L then L is also
l-equivalent to I.

412

2 Solutions of problems 001500

Solution. Recall that a continuous bijection is called condensation and a space X


condenses onto a space Y if there exists a condensation f W X ! Y .
Fact 1. Suppose that Z is a space and fCt T
W t 2 T g is a centered family of compact
connected subsets of Z. Then the set C D t2T Ct is connected.
Proof. If C is disconnected then we can find nonempty disjoint closed sets D; E 
C such that C D D [ E. In any space disjoint compact sets can be separated by
open sets (see Fact 4 of T.309), so we can find disjoint U; V 2 .Z/ such that
D  U and E  V .
Apply Fact 1 of S.326 to see that there is t 2 T such that Ct  U [ V . It is
immediate that U 0 D U \ Ct and V 0 D V \ Ct are nonempty clopen subsets of Ct
with Ct D U 0 [ V 0 ; this contradiction with connectedness of Ct shows that Fact 1
is proved.
Returning to our solution suppose that K is a compact space and there exists
a condensation ' W 0; C1/
T ! K; consider, for every r > 0, the set Rr D
'.r; C1// and let R D fRr W r > 0g. Observe that every Rr is connected
being a continuous image of a connected space r; C1/ (see Fact 1 of T.309, Fact 1
of V.290 and Fact 2 of U.493); therefore Rr is also connected by Fact 1 of T.312,
so we can apply Fact 1 to see that R is a connected compact subset of K. Note also
that K is metrizable because it has a countable network. Let us prove first that
(1) a point x belongs to R if and only if there is a sequence ftn W n 2 !g  0; C1/
such that tn ! C1 and '.tn / ! x.
If we have a sequence S D ftn W n 2 !g as in (1) then, for every r > 0, infinitely
many terms of the sequence f'.tn / W n 2 !g belong to Rr and hence x 2 Rr ;
consequently, x 2 R. If, on the other hand, x 2 R then fix a local decreasing base
fBn W n 2 !g of the space K at the point x. It is easy to find an increasing sequence
ftn W n 2 !g  0; C1/ such that '.tn / 2 Rtn \ Bn and tn > n for each n 2 !.
Then tn ! C1 and '.tn / ! x, i.e., (1) is proved.
Our next step is to show that
(2) the set r; C1/ is not contained in ' 1 .R/ for any r > 0.
Assume for contradiction that r; C1/  ' 1 .R/ for some
S r > 0. The set
Pm D '.m; m C 1
/ is compact for every m 2 ! and R  fPm W m 2 !g. By
the Baire property of R, the set H D IntR .Pm \ R/ is nonempty for some m 2 !;
fix a point x 2 H . By (1), there is a sequence ftn W n 2 !g  0; C1/ such that
tn ! C1 and '.tn / ! x; there is no loss of generality to consider that tn > r for
every n 2 !. The set H being a neighborhood of x in R, infinitely many terms of the
sequence f'.tn / W n 2 !g  R belong to H ; since ' is a bijection, infinitely many
terms of the sequence ftn W n 2 !g belong to m; m C 1
which is a contradiction
with tn ! C1, so (2) is proved.
It turns out that the set ' 1 .R/ is even bounded, i.e.,
(3) there is a > 0 such that ' 1 .R/  0; a
.

2 Solutions of problems 001500

413

If (3) does not hold then apply (2) to find an increasing sequence ftn W n 2 Ng
of points of .0; C1/n' 1 .R/ such that tn ! C1 and tn ; tnC1
\ ' 1 .R/ ; for
all n 2 N. Letting t0 D 0 and Qi D '.ti ; ti C 1
/ \ R for all i 2 ! weS
obtain a
sequence Q D fQi W i 2 !g of disjoint closed subsets of R such that R D Q and
Qi ; for all i 2 N. By Fact 5 of V.290 at most one element of Q is nonempty;
this contradiction shows that (3) is proved.
If E D '..a; C1// then
(4) the map ' W .a; C1/ ! E is a homeomorphism.
Indeed, if the statement (4) is false then there is a point t 2 .a; C1/ and a
sequence S D ftn W n 2 !g  .a; C1/ which is does not converge to t while
the sequence f'.tn / W n 2 !g converges to the point x D '.t / 2 E. Passing to a
subsequence of S if necessary we can assume, without loss of generality, that there
is r > 0 for which jtn  t j  r for all n 2 !. An immediate consequence is that
the set S is closed and discrete in a; C1/ and hence tn ! C1. Therefore x 2 R
which implies that t 2 ' 1 .R/ \ .a; C1/ which is a contradiction with (3) and
hence (4) is proved.
Observe that F D '.0; a
/ is a compact subspace of K homeomorphic to I, so
K is l-equivalent to F .KnF / by Problem 374. It follows from (4) that KnF is
l

homeomorphic to .a; C1/, so K  I ..a; C1//  I I  I (see Problems 381


and 379). Therefore we established that
l

(5) if 0; C1/ condenses onto a compact space K then K  I.


Now assume that L is a compact space and ' W R ! L is a condensation. The
proofs of some properties of L will very similar to what we have proved for K. Let
Ar DT'.r; C1// and Br D '..1;
r
/ for every r 2 R and consider the sets
T
A D fAr W r 2 Rg and B D fB r W r 2 Rg. Mimicking the proof of (1) we
convince ourselves that
(6) A D fx 2 L W there exists a sequence ftn W n 2 !g  R such that tn ! C1
and '.tn / ! xg and B D fx 2 L W there exists a sequence ftn W n 2 !g  R
such that tn ! 1 and '.tn / ! xg.
The proof of the following property is also analogous to the case of K but we
carry it out anyway.
(7) The set r; C1/ is not contained in ' 1 .A/ and the set .1; r
is not contained
in ' 1 .B/ for any r 2 R.
Assume for contradiction that r; C1/  ' 1 .A/ (or .1; r
 ' 1 .B/) for
some r 2 R. The set Pn D '.m;
m C 1
/ is compact for every m 2 Z and we
S
have the inclusion A [ B  fPm W m 2 Zg. By the Baire property of A (or B
respectively), the set H D IntA .Pm \A/ (or the set H D IntB .Pm \B/ respectively)
is nonempty for some m 2 Z; fix a point x 2 H . By (6), there is a sequence
ftn W n 2 !g  R such that tn ! C1 (tn ! 1) and '.tn / ! x; there is no loss
of generality to consider that tn > r (tn < r) for every n 2 !. The set H being a

414

2 Solutions of problems 001500

neighborhood of x in A (or in B respectively), infinitely many terms of the sequence


f'.tn / W n 2 !g  A (f'.tn / W n 2 !g  B) belong to H ; since ' is a bijection,
infinitely many terms of the sequence ftn W n 2 !g belong to m; m C 1
which is a
contradiction with tn ! C1 (or tn ! 1 respectively), so (7) is proved.
(8) If the set .1; r
(r; C1/) is not contained in ' 1 .A/ (or in ' 1 .B/
respectively) for any r 2 R then ' 1 .A/ (' 1 .B/ respectively) is bounded
in R.
To prove (8) apply the property (7) to find a sequence S D fsn W n 2 Zg such that
m < n implies sm C 1 < sn and S \ ' 1 .A/ D ; (or S \ ' 1 .B/ D ; respectively).
The set Pn D sn ; snC1
\ A (or Pn D sn ; snC1
\ B respectively) is closed
S in A
(in B)Sfor any n 2 Z, the family P D fPn W n 2 Zg is disjoint and A D P (or
B D P respectively). By Fact 5 of V.290 at most one element of the family P is
nonempty and hence the set ' 1 .A/ (or ' 1 .B/ respectively) is bounded in R, i.e.,
(8) is proved.
(9) One of the sets ' 1 .A/ and ' 1 .B/ has to be bounded in R.
Indeed, if A and B are unbounded then it follows from (7) and (8) that there are
r; s 2 R such that .1; r
 ' 1 .A/ and s; C1/  ' 1 .B/. The set A is closed
in L, so B  B r  A and hence B  A which shows that s; C1/  ' 1 .B/ 
' 1 .A/; this contradicts (7) and shows that (9) is proved.
(10) if ' 1 .A/  .1; r/ (or ' 1 .B/  .r; C1// then 'jr; C1/ W r; C1/ !
Ar (or 'j.1; r
W .1; r
! Br respectively) is a homeomorphism.
Indeed, if this is not true then there is a point t  r (t  r) and a sequence
S D ftn W n 2 !g  r; C1/ (or S D ftn W n 2 !g  .1; r
respectively)
which is does not converge to t while the sequence f'.tn / W n 2 !g converges to
the point x D '.t /. Passing to a subsequence of S if necessary we can assume,
without loss of generality, that there is > 0 for which jtn  t j  for all n 2 !.
An immediate consequence is that the set S is closed and discrete in r; C1/ (in
.1; r
) and hence tn ! C1 (or tn ! 1 respectively). Therefore x 2 A (or
x 2 B respectively) by the property (6) which implies that t 2 ' 1 .A/ \ r; C1/
(or t 2 ' 1 .B/ \ .1; r
respectively) which is a contradiction and hence (10) is
proved.
(11) If ' 1 .A/ (or ' 1 .B/) is bounded in R then there exists r 2 R such that the
set As (or Bs respectively) is compact for any s < t (or s > t respectively).
To prove (11) assume that ' 1 .A/  .r; r/ (or ' 1 .B/  .r; r/ respectively)
and take any s < r (s > r). If x 2 As nAs (or x 2 B s nBs ) then there is a sequence
fxn W n 2 !g  As (fxn W n 2 !g  Bs ) which converges to x. Let tn D ' 1 .xn /
for each n 2 !; if the sequence ftn W n 2 !g has a bounded subsequence then it will
have an accumulation point in s; C1/ (in .1; s
) and hence fxn W n 2 !g will
have a accumulation point in As (or in Bs respectively) which is a contradiction.
Therefore tn ! C1 (tn ! 1) and hence x 2 A (x 2 B). As a consequence,
As  As [ A  As [ '.r; r
/ D As (or B s  Bs [ B  Bs [ '.r; r
/ D Bs ),
so the set As (or Bs respectively) is closed in L, i.e., (11) is proved.

2 Solutions of problems 001500

415

(12) If ' 1 .A/ (or ' 1 .B/) is bounded in R then there exists r 2 R such that
' 1 .B/  r; C1/ (or ' 1 .A/  .1; r
respectively).
If ' 1 .B/ does not have a lower bound (or ' 1 .A/ does not have an upper bound
respectively) then apply (11) and (7) to find a sequence ftn W n 2 !g  Rn' 1 .B/
(or ftn W n 2 !g  Rn' 1 .A/) such that At0 is compact (Bt0 is compact) and tnC1 <
tn  1 (tnC1 > tn C 1) while tnC1 ; tn
\ ' 1 .B/ ; (or ' 1 .A/ \ tn ; tnC1
;)
for every n 2 !. Let P0 D At0 \ B (or P0 D Bt0 \ A respectively) and consider the
set Pn D '.tnC1 ; tn
/ \ B (or Pn D '.tn ; tnC1
\ A) for all n 2 N.
Then P D fPn W n 2 !g is a disjoint family of compact subsets
of B (ofSA)
S
such that infinitely many elements of P are nonempty and B D P (or A D P
respectively). Since B is connected (A is connected) this is a contradiction with
Fact 5 of V.290, i.e., (12) is proved.
By (9) we can assume that ' 1 .A/ (or ' 1 .B/) is bounded in R. The property
(12) implies that there exists a number r 2 R such that ' 1 .B/  .r; C1/
(or ' 1 .A/  .1; r/ respectively). Apply the properties (10) and (11) to find
s 2 R such that the set As is compact (or the set Bs is compact respectively)
and 'j.1; s/ W .1; s/ ! LnAs (or 'j.s; C1/ W .s; C1/ ! LnBs )
is a homeomorphism. The set s; C1/ (.1; s
) condenses onto As (onto Bs
respectively) which is compact, so we can apply the property (5) to conclude that
l

As  I (or Bs  I). This proves that


l

(13) there is a closed set F  L such that F  I and LnF is homeomorphic to an


open subset of R.
l

Finally, apply Problem 374 to the set F from (13) to see that L  F .LnF /.
l

By Problem 381, the space .LnF / has to be l-equivalent to the space I, so L  I


l

I  I (see Problem 379) and hence our solution is complete.


V.386. Assume that X is a second countable
space and dim X D n 2 !. Let
S
O D fU 2 .X / W dim U < ng and O D O. The set K.X / D X nO is called the
dimensional kernel of X . Prove that dim O < n and dim W D n for any nonempty
open subset W of the space K.X /.
Solution. The space X S
is hereditarily Lindelf, so there exists a countable family
U  O such that O D U .S
For each U 2 U fix a countable
family FU of closed
S
subsets
of
X
such
that
U
D
F
;
the
family
F
D
fF
W
U
2 U g is countable
U
U
S
and F D O. Since dim F  n  1 for each F 2 F, we can apply Problem 150
to convince ourselves that dim O  n  1 < n.
Assume that dim W  n  1 for some nonempty open set W of the space K.X /.
Using again second countability of the
Sspace X we can find a countable family H
of closed subsets of K.X / such that H D W . The set G D O [ WSis open in
X ; besides, the elements of H are closed in X , so it follows from G D .F [ H/
that S
dim G  n  1 (see Problem 150) and hence G 2 O. Since also GnO D
Gn. O/ D W ;, we obtained a contradiction which shows that dim W D n
for any W 2  .K.X //.

416

2 Solutions of problems 001500

V.387. Prove that if n 2 N and a space X is l-equivalent to In then the dimensional


kernel K.X / of the space X is also l-equivalent to In .
Solution. The space X is compact by Problem 138 and dim X D n by Problem 180.
Since network weight is preserved even by t -equivalence, the space X is metrizable.
l

Apply Problem 273 to see that X  .! C 1/ is l-equivalent to In  .! C 1/  In  X


(see Problem 383). Therefore X  .! C 1/ is l-equivalent to X .
By Problem 309, some nonempty open subspace of In embeds in X ; since
n
I embeds in every open subspace of Rn , there exists a set F  X which is
homeomorphic to In . If U D F nK.X / ; then U is an n-dimensional open
subspace of F and U  X nK.X / while dim.X nK.X // < n (see Problem 386);
this contradiction shows that F  K.X /. Apply Problem 373 to see that there exists
a linear topological space I such that Cp .K.X //
I  Cp .F /
I  Cp .In /

I  Cp .In /  Cp .In / (see Problem 379). This shows that


(1) Cp .K.X //
Cp .K.X //  Cp .In /.
Since K.X /  .! C 1/ embeds in X  .! C 1/, it follows from Problem 373
that there exists a linear topological space E such that Cp .X  .! C 1//
E 
Cp .K.X /  .! C 1//. Observe that .K.X /  .! C 1// K.X / is homeomorphic
to K.X /  .! C 1/, so we have Cp .In /
Cp .X /
Cp .X  .! C 1//
E 
Cp .K.X /  .! C 1// and it follows from Cp .K.X /  .! C 1//
Cp .K.X /  .! C
1//  Cp .K.X // that the space Cp .In / is linearly homeomorphic to E  Cp .K.X / 
.! C 1//  Cp .K.X // which in turn is linearly homeomorphic to Cp .X  .! C 1// 
Cp .K.X //
Cp .X /  Cp .K.X //. This, together with (1), shows that Cp .In /

Cp .In /  Cp .K.X //
Cp .K.X // and hence Cp .In /
Cp .K.X //, i.e., K.X / is
l-equivalent to In as promised.
V.388. Call a second countable space Y weakly n-Euclidean if dim Y D n
and every n-dimensional subspace of Y has nonempty interior and contains a
homeomorphic copy of In . Prove that a compact space X is l-equivalent to In if
and only if its dimensional kernel K.X / has a nonempty open weakly n-Euclidean
subspace and every U 2  .K.X // contains a subset which is l-equivalent to X .
Solution. Suppose that X is l-equivalent to In and hence dim X D n. By
Problem 309, some W 2  .K.X // embeds in In ; let W 0  In be homeomorphic
to W . Since dim W 0 D n (see Problem 386), the interior V 0 of the set W 0 in
In is nonempty (see Problem 160) and hence there exists V 2  .W / which is
homeomorphic to V 0 . Thus V is a nonempty open subspace of K.X / and V is a
homeomorphic to an open subspace of In . It follows from Problem 160 and Fact 1
of V.379 that V is weakly n-Euclidean.
Now fix any U 2  .K.X //; by Problem 309 some set G 2  .U / embeds in In .
Let G 0  In be homeomorphic to G. Since dim G 0 D n (see Problem 386), the
interior of G 0 in In is nonempty by Problem 160; this, together with Fact 1 of V.379
shows that In embeds in G 0 and hence in G  U . If F  U is a homeomorphic
copy of In then F is l-equivalent to X and hence we proved necessity.

2 Solutions of problems 001500

417

Now assume that some W 2  .K.X // is weakly n-Euclidean and every


subspace U 2  .K.X // contains a subset which is l-equivalent to X . The space
In embeds in W and hence in X ; by Problem 373, there exists a linear topological
space E such that Cp .X /
E Cp .In /
E Cp .In /Cp .In /
Cp .X /Cp .In /,
i.e., we proved that
(1) Cp .X /
Cp .X /  Cp .In /.
Let F  W be a homeomorphic copy of In ; since W is weakly n-Euclidean, we
can find a set U 2  .W / such that U  F . The set U is also open in K.X /, so
l

there exists Y  U with Y  X . The space Y embeds in In ; it is an easy exercise to


see that .Y  !/ embeds in .In  !/ which, in turn, embeds in In (see Fact 1 of
V.379). Therefore .Y  !/ embeds in In and hence we can apply Problem 376 to
convince ourselves that Cp .In /
Cp .In /  Cp .Y /. This, together with the property
(1), shows that Cp .X /
Cp .In /  Cp .X /
Cp .In /  Cp .Y /
Cp .In / and hence
l

X  In , i.e., we settled sufficiency.


V.389. Given (linear) topological spaces X and Y the expression X  Y says that
they are (linearly) homeomorphic. Suppose that X !  X and there exist (linear)
topological spaces E and F such that X  Y  F and Y  X  E. Prove that
X  Y.
Solution. Observe first that the usual properties of the Tychonoff product imply that
Z  T  T  Z for any (linear) topological spaces Z and T .
It follows from Y  X  E that Y  X !  E  X  X !  E  X  Y . On the
other hand, X !  .Y  F /!  Y !  F !  Y  Y !  F !  Y  X ! . Recalling
that X !  X we conclude that X  Y  X !  Y  X  X  Y  Y , so X  Y .
V.390. Suppose that L is a linear topological space, M is a linear subspace of L
and there exists a linear retraction r W L ! M . Prove that L
M  r 1 .0/.
Deduce from this fact that for any linear topological spaces L and E there exists a
linear topological space N such that L
E  N (i.e., E is a linear topological
factor of L) if and only if there exists a linear retract E 0 of the space L such that
E 0
E.
Solution. Denote the space r 1 .0/ by N and let .u; v/ D u C v for any point
.u; v/ 2 M  N . It is evident that  W M  N ! L is a linear map. From continuity
of operations in L it follows that  is continuous. Let .x/ D .r.x/; x  r.x// for
every x 2 L; it is easy to see that  W L ! M  N is also a linear map. Besides, 
is continuous being the diagonal product of continuous maps. It is straightforward
that . /.u; v/ D .u; v/ for any .u; v/ 2 M  N and . /.x/ D x for each
x 2 L, so  and  are mutually inverse linear homeomorphisms. As a consequence,
L
M  N D M  r 1 .0/. This implies, in particular, that any linear topological
space which is linearly homeomorphic to a linear retract of L is a linear topological
factor of L.
Now assume that E is a linear topological factor of L, i.e., L is linearly
homeomorphic to E  N for some linear topological space N . There is no loss of

418

2 Solutions of problems 001500

generality to consider that L D E N . The linear subspace E 0 D f.a; 0N / W a 2 Eg


of the space L is easily seen to be linearly homeomorphic to E. Given any point
x D .a; b/ 2 L let r.x/ D .a; 0N /; then r W L ! E 0 is a linear continuous
retraction (the relevant verification can be left to the reader).
V.391. Given linear topological spaces L; M and N prove that L
M  N
if and only if there exist linear subspaces M 0 and N 0 of the space L for which
M 0
M; N 0
N and there exist linear retractions r W L ! M 0 and s W L ! N 0
such that r.x/ C s.x/ D x for any x 2 L.
Solution. Suppose first that L
M  N and hence we can identify L with M  N .
If M 0 D f.a; 0N / W a 2 M g then M 0 is a linear subspace of L which is easily seen
to be linearly homeomorphic to M . Analogously, N 0 D f.0M ; b/ W b 2 N g is a
linear subspace of L which is linearly homeomorphic N . For any x D .a; b/ 2 L
let r.x/ D .a; 0N / and s.x/ D .0M ; b/. Then r W L ! M 0 and s W L ! N 0 are
linear retractions such that r.x/Cs.x/ D x for every x 2 L, so we settled necessity.
Now assume that we have linear subspaces M 0 and N 0 of the space L for which
0
M
M; N 0
N and there exist linear retractions r W L ! M and s W L ! N
such that r.x/ C s.x/ D x for every x 2 L. Let .x/ D .r.x/; s.x// for each
x 2 L; then  W L ! M 0  N 0 is a linear continuous map. For any point x D
.a; b/ 2 M 0  N 0 let .x/ D a C b; by continuity of the operations in L the map
 W M 0 N 0 ! L is continuous. We omit a trivial verification of the fact that  and 
are mutually inverse maps and hence each one of them is a linear homeomorphism.
Therefore L
M 0  N 0
M  N , i.e., we proved sufficiency.
V.392. For any linear topological space L denote by L the set of all continuous
linear functionals on L with the topology inherited from Cp .L/. Prove that, for any
locally convex spaces M and N , we have .M  N /
M   N  . In particular,
Cp .Y / is a linear topological factor of Cp .X / if and only if Lp .Y / is a linear
topological factor of Lp .X /.
Solution. Denote the space M  N by L; by Problem 391 we can consider that M
and N are linear subspaces of L for which there exist linear retractions r W L ! M
and s W L ! N such that r.x/ C s.x/ D x for every x 2 L. Let M W Cp .L/ !
Cp .M / and N W Cp .L/ ! Cp .N / be the relevant restriction maps.
If f 2 M  then .f / D f r is a continuous linear functional on L, so the
map  W M  ! L is a linear embedding (see TFS-163) and hence the space
M 0 D .M  / is linearly homeomorphic to M  . Analogously, let .f / D f s
for each f 2 N  ; then  W N  ! L is also a linear embedding and therefore
N 0 D .N  /
N  .
Let r0 D M and s0 D N ; the restriction maps being linear and continuous,
the maps r0 W L ! L and s0 W L ! L are linear and continuous as well. It
follows from M  D M .L / and N  D N .L / (see Problem 224) that r0 .L / D
M 0 and s0 .L / D N 0 , so we can consider that r0 W L ! M 0 and s0 W L ! N 0 .
If h 2 M 0 then h D f r for some f 2 M  ; the map r being a retraction,
we have f .r.x// D f .x/ for any x 2 M and hence M .h/ D f . An immediate
consequence is that r0 .h/ D f r D h, i.e., r0 .h/ D h for any h 2 M 0 and hence
r0 is a linear retraction. Analogously, the map s0 is also a linear retraction.

2 Solutions of problems 001500

419

Take any functional f 2 L and x 2 L; since r.x/ C s.x/ D x, we conclude


that f .r.x/ C s.x// D f .x/ D f .r.x// C f .s.x//. This shows that the equality
r0 .f /.x/Cs0 .f /.x/ D f .x/ holds for any x 2 L and therefore r0 .f /Cs0 .f / D f
for every f 2 L . Applying Problem 391 to the retractions r0 and s0 we conclude
that L
M 0  N 0
M   N  as promised.
Finally, if Cp .X /
Cp .Y /L for some locally convex space L then apply TFS197 to see that we have the equalities .Cp .Y // D Lp .Y / and .Cp .X // D Lp .X /,
so Lp .X / D .Cp .X //
.Cp .Y //  L
Lp .Y /  L and hence Lp .Y / is a
linear topological factor of Cp .X /. If Lp .Y / is a linear topological factor of Lp .X /
then there is a locally convex space M such that Lp .X /
Lp .Y /  M . Recall that
.Lp .X //
Cp .X / and .Lp .Y //
Cp .Y / (see Problem 235); as a consequence,
Cp .X / D .Lp .X //
.Lp .Y //  M 
Cp .Y /  M  which shows that Cp .Y /
is a linear topological factor of Cp .X /.
V.393. Suppose that X is a second countable non-compact
S space, n 2 N and there
exists a locally finite cover I of the space X such that fInt.I / W I 2 Ig D X and
every I 2 I is homeomorphic to In . Prove that X is l-equivalent to In  !.
Solution. As usual, the expression Z ' T says that the spaces Z and T are
homeomorphic. If L and M are linear topological spaces then M is a linear
topological factor of L if there exists a linear topological space E such that
L
M  E.
Let Y D In  !; then Y  ! ' Y and hence
(1) the space .Cp .Y //! is linearly homeomorphic to Cp .Y /.
The space X is non-compact and second countable, so it is not pseudocompact;
take a discrete family fUn W n 2 !g of nonempty open subsets of X and fix a
point xn 2 Un for each n 2 !. Choose a set Pn 2 I such that xn 2 Pn for each
n 2 !. The set Pn \ Un is open in Pn and nonempty, so it contains a set Wn which
is homeomorphic to a nonempty open subset of Rn ; an immediate consequence is
that In embeds in Wn (see Fact 1 of V.379) for every n 2 !. Take a set Fn  Wn
such that S
Fn ' In for each n 2 !. The family fFn W n 2 !g being discrete, the
set F D fFn W n 2 !g is closed in X and homeomorphic to In  ! D Y . This
shows that the space Y embeds in X as a closed subspace and hence we can apply
Problem 373 to see that
(2) Cp .Y / is a linear topological factor of Cp .X /.
The family I is easily seen to be countably infinite; let fIn W n 2 !g be a faithful
enumeration of I. It is easy to find for each P
n 2 !, a function pn 2 Cp .X; 0; 1
/
such that pn1 .0/ D X nInt.In /. Let p.x/ D n2! pn .x/ for every x 2 X . Since I
is locally finite, the number of summands in the definition of p.x/ is finite, so p.x/
makes sense for every x 2 X .
Given any point x 2 X there exists a set
PG 2 .x; X / such that the set A D fn 2
! W In \ G ;g is finite. Then p.y/ D fpn .y/ W n 2 Ag for all y 2 G, so f is
continuous on the set G; applying
S Fact 1 of S.472 we convince ourselves that p is
a continuous function. Since fInt.In / W n 2 !g D X , we have p.x/ > 0 for any

420

2 Solutions of problems 001500

x 2 X , so qn D ppn is a continuous function on X for each n 2 !. Observe also that


P
n2! qn .x/ D 1 and the set B.x/ D fn 2 ! W x 2 In g is nonempty and finite for
every x 2 XS
.
Let I D fIn  fng W n 2 !g where the topology of I is inherited from X  !;
evidently, the space I is homeomorphic to Y . For any a D .x; n/ 2 I let .a/ D x.
This gives us a continuous onto map  W I ! X . Apply Problem 234 to find a
continuous linear map  W Lp .I / ! Lp .X / such that  jI D .
GivenP
x 2 X let 'n .x/ D .x; n/ for each n 2 B.x/ and consider the point
'.x/ D fqn .x/'n .x/ W n 2 B.x/g 2 Lp .I /. Our next step is to prove that the
map ' W X ! Lp .I / is continuous; let f .u/ D u.f / for all f 2 Cp .I / and
u 2 Lp .I /.
Fix any f 2 Cp .I /; since Lp .I /  Cp .Cp .I //, it suffices to show that f '
is continuous (see TFS-102). Let fn .x/ D f .x; n/ for each x 2 In ; it is immediate
that fn 2 Cp .In /, so we can find a function gn 2 Cp .X / such that gn jIn D fn
for every n 2 !. If x 2 X then there exists W 2 .x; X / such that the set A D
fn 2 ! W In P
\ W ;g is finite. For every point
Py 2 W we have the equalities
f .'.y// D fqn .y/f .'n .y// W n 2 B.y/g D fqn .y/gn .y/ W n 2 Ag because
B.y/  A and qn .y/ D 0 whenever n 2 AnB.y/.
Therefore f ' P
is continuous on W because it coincides on W with the
continuous function
n2A qn gn . By Fact 1 of S.472 the function f ' is
continuous for each f 2 Cp .I /, so ' is a continuous map.
By Problem 234 there exists a linear continuous map W Lp .X / ! Lp .I / such
that jX D '; then r D P  W Lp .I / ! Lp .I / is also
P a linear continuous map.
Observe that  .'.x// D n2B.x/ qn .x/ .'n .x// D . n2B.x/ qn .x// x D x for
any x 2 X . If X 0 D '.X / then ' W X ! X 0 and  jX 0 W X 0 ! X are mutually
inverse continuous maps, so ' is an embedding and it follows from the definition of
r that r.'.x// D '.x/ for every x 2 X . In other words,
(3) r.y/ D y for all y 2 X 0 .
If M is the linear hull of X 0 in Lp .I / then it follows from (3) and linearity of r
that r.y/ D y for all y 2 M ; observe also that r.Lp .I // D .Lp .X // D M , so
M is a linear retract of Lp .I /.
1
Take any nonempty
S finite set E  X ; the family PE D f .x/ W x 2 Eg is
disjoint so PE D PE is linearly independent in Lp .I /. If u is a nontrivial linear
combination of the set '.E/ then u is a nontrivial linear combination of PE , so
u 0 because PE is linearly independent. This shows that
(4) X 0 is linearly independent and hence it is a linear basis in M .
It follows from the property (4) that is an injection. We already saw that the
map  jX 0 W X 0 ! X is the inverse of '. Therefore  jM W M ! Lp .X / is the
inverse of W Lp .X / ! M which shows that is an embedding. In particular,
M
Lp .X /, i.e., Lp .X / is linearly homeomorphic to a linear retract of Lp .I /.
This implies that Lp .X / is a linear topological factor of Lp .I / (see Problem 390);

2 Solutions of problems 001500

421

applying Problem 392 we conclude that Cp .X / is a linear topological factor of


Cp .I /. Since I ' Y , we proved that
(5) Cp .X / is a linear topological factor of Cp .Y /.
Finally observe that the properties (1)(2) and (5) together with Problem 389
imply that Cp .X / is linearly homeomorphic to Cp .Y / D Cp .In !/, so our solution
is complete.
V.394. Prove that, for any n 2 N, every nonempty open subspace of Rn is ll

equivalent to In  !. In particular, if U 2  .Rn / then U  Rn .


Solution. The set Hn D .2n ; 2n /n is open in Rn for every n 2 ! and it is easy
to see that the family fHn W n 2 !g is a local base of Rn at 0. Consequently, B.x/ D
fx C Hn W n S
2 !g is a local base of Rn at the point x for any x 2 Rn . Therefore the
family B D fB.x/ W x 2 Rn g is a base in Rn ; if H D fB 2 B W B  U g then H
is a base in U . It is straightforward that H is homeomorphic to In for any B 2 H.
The space U is  -compact and hence Hurewicz, so we S
can apply CFS-050 to
find a family U  H such that U is locally finite in U and U D U . The family
I D fH W H 2 U g is also locally finite in U and every I 2 I is homeomorphic to
In . Observe that U is not compact (see Fact 1 of CFS-493) and the interiors of the
elements of I cover the space U , so we can apply Problem 393 to conclude that U
l

is l-equivalent to In  !. Therefore U  In  !  Rn for any U 2  .Rn /.


V.395. Given spaces X and Y assume that nw.X / D ! and Cp .Y / is a linear
topological factor of Cp .X /. Prove that dim Y  dim X .
Solution. Say that the local dimension of a space Z is at most k if, for any z 2 Z
there is a set U 2 .z; Z/ such that dim U  k. We will also need the subspace
R D Rnf0g of the space R.
Fact 1. If Z is a Lindelf space then dim Z  k if and only if the local dimension
of Z is at most k.
Proof. If dim Z  k then U D Z is the required neighborhood for any z 2 Z. Now,
suppose that for any z 2 Z there a set Uz 2 .z; Z/ such that dim Uz  k. Choose
a set Vz 2 .z; Z/ such that V z  Uz ; then dim.V z /  k for everySz 2 Z. By the
Lindelf property
of Z there is a countable A  Z such that Z D fVz W z 2 Ag;
S
then Z D fV z W z 2 Ag, so it follows from the countable sum theorem (see
Problem 150) that dim Z  k, i.e., Fact 1 is proved.
Returning to our solution observe that if dim X D 1 then there is nothing to
prove, so assume that dim X D n 2 !. Since Cp .Y / embeds in Cp .X /, we have
nw.Y / D nw.Cp .Y //  nw.Cp .X //  !, so Y also has a countable network.
By Problem 392, the space Lp .Y / has to be a linear topological factor of Lp .X /,
so we can consider that Lp .Y / is a linear subspace of Lp .X / and there exists a linear
retraction r W Lp .X / ! Lp .Y / (see Problem 390). For any point y 2 Y there exists
a unique number k 2 N such that for some uniquely determined distinct points
x1 ; : : : ; xk 2 X and numbers 1 ; : : : ; k 2 R we have y D 1 x1 C : : : C k xk ;

422

2 Solutions of problems 001500

let lX .y/ D k and X .y/ D fx1 ; : : : ; xk g. For any u 2 Lp .X / there exists a unique
number m 2 ! such that for some 1 ; : : : ; m 2 R and uniquely determined
distinct points y1 ; : : : ; ym 2 Y we have r.u/ D 1 y1 C : : : C m ym ; let lY .u/ D m
and Y .u/ D fy1 ; : : : ; ym g. Note that m D 0 means that r.u/ D 0 and Y .u/ D ;.
The set Lm D fu 2 Lp .Y / W lY .u/  mg is closed in Lp .Y / (see Fact 1 of U.485);
let L1 D ; and L0m D Lm nLm1 for every m 2 !.
The set Yk D fyS2 Y W lX .y/  kg is closed in Y for any k  ! by Fact 1 of
U.485. Since Y D k2! Yk , it suffices to show that dim Yk  n for every k 2 !
(see Problem 150). The space Y being perfect, Yk0 D Yk nYk1 is an F -set in Y ;
since also Y0 D ;, it suffices to show that dim Yk0  n for all k 2 N.
Fix any number k 2 N; for any  D .m1 ; : : : ; mk / 2 ! k consider the set
O D fy 2 Yk0 W there exists an enumeration fx1 ; : : : ; xk g of the set X .y/ such
P
that jY .xi /j  mi for all i  kg and let 
D kiD1 mi . It is clear that
S
(1) Yk0 D fO W  2 ! k g.
Given any k-tuple  D .m1 ; : : : ; mk / 2 ! k take a point y 2 Yk0 nO . Then
X .y/ D fx1 ; : : : ; xk g and jY .xi /j D ji for each i  k while j1 C : : : C jk >

. Since Lji 1 is closed in Lp .Y / for all i  k, the set Ui D fv 2 Lp .X / W
r.v/ Lji 1 g is open in Lp .X / and xi 2 Ui for all i  k. Take a disjoint family
W1 ; : : : ; Wk of open subsets of X such that xi 2 Wi  Ui for all i  k.
The set G D .R W1 C : : : C R Wk / \ Yk0 is an open neighborhood of y in Yk0
(see Fact 2 of U.485). If z 2 G then z D 1 a1 C : : : C k ak where i 2 R and
ai 2 Wi for all i  k. An immediate consequence is that lX .z/ D fa1 ; : : : ; ak g
and lY .ai /  ji for all i  k which shows that z O and hence we proved that
y 2 G  Yk0 nO , i.e., every point of Yk0 nO has a neighborhood in Yk0 contained in
Yk0 nO . Therefore
(2) the set O is closed in Yk0 for any  2 ! k .
The properties (1) and (2) show that it suffices to prove that dim O  n for every
 2 ! k . We will do that by induction on m D 
. If m D 0 and y 2 O then y is a
linear combination of the set X .y/. Therefore y D r.y/ is a linear combination of
r.X .y// D f0g, i.e., y D 0 which is a contradiction. Therefore O D ; and hence
dim O D 1  n.
Now assume that m > 0 and dim O  n whenever  2 ! k and 
< m; fix a ktuple  DS.m1 ; : : : ; mk / such that 
D m. It follows from the property (2) that the
set F D fO W 
< mg is closed in Yk0 and dim F  n (see Problem 150). The
set O nF is open in O ; since Y has a countable network, O nF is an F -subset of
O , so it suffices to prove that O 0 D O nF has dimension at most n.
By Fact 1 it is sufficient to show that O 0 has local dimension at most n, so fix
a point b 2 O 0 and a1 ; : : : ; ak 2 X such that X .b/ D fa1 ; : : : ; ak g. There is no
loss of generality to assume that jY .ai /j  mi for every i  k, so it follows from
P
k
iD1 jY .ai /j D m that jY .ai /j D mi for all i  k.
Choose a disjoint family fG1 ; : : : ; Gk g of open subsets of X such that ai 2 Gi
and r.Gi / \ Lmi 1 D ; for every i  k. The set G D .R G1 C : : : C R Gk / \ O 0 is

2 Solutions of problems 001500

423

open in O 0 by Fact 2 of U.485. If y 2 G then there is a unique point xi 2 X .y/\Gi ;


let pi .y/ D xi for every i  k.
This gives us a map pi W G ! Gi ; if W is open in Gi then it is straightforward
that pi1 .W / D .R G1 C : : : R Gi1 C R W C R GiC1 C : : : C R Gk / \ G, so
the map pi is continuous for any i  k.
Given i  k let Y .r.ai // D fbji W 1  j  mi g and choose a disjoint family
fHji W j  mi g of open subsets of Y such that bji 2 Hji for all j  mi . The set
P i
i
Hi D m
j D1 R Hj  Lmi is open in Lmi and r.ai / 2 Hi . Besides, r.pi .G// 
0
0
Lmi and hence Hi D r 1 .Hi / \ pi .G/ is open in pi .G/ for each i  k. Therefore
T
the set V D fpi1 .Hi0 / W i  kg is open in G and hence in O 0 ; it is easy to see
that b 2 V .
For any point u 2 Hi there is a unique point yji 2 Y .u/ \ Hji ; let qji .u/ D yji
for every j  mi . The map qji W Hi ! Hji is continuous for every j  mi ; this
can be proved in the same way as continuity of every map pi . By our choice of the
set V the map qji r pi is well defined on V ; it is an easy exercise that the set
Eij D fc 2 V W qji .r.pi .c/// D cg is closed in V for any i  k and j  mi .
If y 2 V then there are x1 ; : : : ; xk 2 X and 1 ; : : : ; k 2 R such that xi 2 Hi0
P
i
for each i  k and y D kiD1 i xi . Given
exist y1i ; : : : ; ym
2 Y such
i
Pimi k i there
i
i
i
that yj 2 Hj for all j  mi and r.xi / D j D1 j yj . It follows from the equalities
P
P i
P
i i
i
y D r.y/ D kiD1 i r.xi / D kiD1 i . m
j D1 j yj / that y D yj for some i  k
and i  mi ; this shows that y 2 Eij and hence
S
(3) V D fEij W 1  i  k; 1  j  mi g.
Observe that the map .qji r/jpi .Eij / is the inverse of the map pi jEij ; an
immediate consequence is that pi embeds Eij in the space X for any i  k and
j  mi . It follows from Problem 155 that dim Eij  dim X D n for any i  k and
j  mi . Applying Problem 150 and (3) we conclude that dim V  n. Therefore V
witnesses that O 0 has local dimension at most n at any point b 2 O 0 , so dim O 0  n
by Fact 1. This shows that dim O  n and completes our inductive proof. As
a consequence, dim Yk0  n for all k 2 N and hence dim Y  n D dim X as
promised.
V.396. Assuming that X is a compact space and Cp .Y / is a linear topological
factor of Cp .X / prove that dim Y  dim X .
Solution. Given spaces Z; Z 0 and a continuous onto map ' W Z ! Z 0 the dual
map '  W Cp .Z 0 / ! Cp .Z/ is defined by '  .f / D f ' for any f 2 Cp .Z 0 /.
Say that a set L  Cp .Z/ is adequate if there is a space Z 0 and a continuous onto
map ' W Z ! Z 0 such that L D '  .Cp .Z 0 //. Given n 2 !, a set L  Cp .Z/
is called dimensionally n-adequate if there exists a space Z 0 and a continuous onto
map ' W Z ! Z 0 such that dim Z 0  n and L D '  .Cp .Z 0 //.
Fact 1. Suppose that a space K is compact, Li  Cp .K/ and Li  LiC1 for every
i 2 !. Then

424

2 Solutions of problems 001500

S
(a) if every Li is adequate then L D i2! Li is adequate;
(b) if every Li is dimensionally n-adequate then L is also dimensionally nadequate.
Proof. We will give a simultaneous proof for (a) and (b). Take a space Zi (with
dim Zi  n if we are proving (b)) for which there exists a continuous onto map 'i W
K ! Zi such that Li D 'i .Cp .Zi // for each i 2 !. It follows from Li  LiC1
that there exists a continuous map hi W ZiC1 ! Zi such that hi 'iC1 D 'i for all
j
i 2 ! (see TFS-163). For any i; j 2 ! with i < j let i D hi : : : hj 1 W Zj !
j
Zi ; then S D fZi I i ; i; j 2 !g is an inverse sequence of compact spaces. If Z is
the inverse limit of S then Z is compact (and dim Z  n (see Problem 174) if we
are proving (b)). Let i W Z ! Zi be the limit projection for any i 2 !.
j
It is easy to see that i 'j D 'i for any i; j 2 ! with i < j . This, together
with Problem 167, shows that the diagonal product ' D i2! 'i maps the space K
in Z.
Suppose that a point a D .ai W i 2 !/ belongs to Z and let Fi D 'i1 .ai / for
j
j
every i 2 !. If j > i and x 2 Fj then 'i .x/ D i .'j .x// D i .aj / D ai and
hence x 2 Fi . This proves
T that fFi W i 2 !g is a decreasing family of nonempty
compact sets; if x 2 i2! Fi then 'i .x/ D ai for all i 2 !, i.e., '.x/ D a.
Consequently, '.K/ D Z; since also i ' D 'i , we can apply TFS-163 once
more to see that Li  G D '  .Cp .Z// for any i 2 !. The set G is closed in
Cp .K/, so L  G.
Q
If x and y are distinct points of the space Z then it follows from Z  i2! Zi
that there is i 2 ! such that i .x/ i .y/. Take a function f 2 Cp .Zi / with
f .i .x// f .i .y//; then gSD f i 2 i .Cp .Zi // and g.x/ g.y/.
This proves that the set A D i2! i .Cp .Zi // separates the points of Z; it is
straightforward that A is an algebra in Cp .Z/, so A is dense in Cp .Z/ by TFS-192.
As a consequence, '  .A/ is dense in '  .Cp .Z// D G.





It follows from i ' D 'i that
S'i D ' i and hence Li D ' .i .Cp .Zi ///
for any i 2 !. Therefore the set i2! Li D ' .A/ is dense in G, so L D G is an
adequate set. In the case of (b) the set G D '  .Cp .Z// is dimensionally n-adequate
because dim Z  n, so Fact 1 is proved.
Returning to our solution let n D dim X ; if n D 1 then there is nothing to
prove, so we can consider that n 2 !. Note that the space Cp .Y / embeds in Cp .X /,
so t .Cp .Y //  t .Cp .X // D ! and hence Y is Lindelf by TFS-149. There exists a
continuous linear surjection of Cp .X / onto Cp .Y / (see Problem 390), so Cp .Y / is
pseudocompact by Problem 193; as a consequence, Y is compact. It follows from
Problem 390 that we can assume that Cp .Y / is a linear subspace of Cp .X / and there
exists a linear retraction r W Cp .X / ! Cp .Y /.
Fix any continuous onto map ' W Y ! K of Y onto a second countable space K.
Then L0 D '  .Cp .K// is an adequate subspace of Cp .Y / and nw.L0 / D !; it
follows from Fact 5 of U.086 that there is an adequate subset M00 of the space Cp .X /
such that L0  M00 and nw.M00 / D !. Apply Problem 161 to find a set M0 which is
dimensionally n-adequate in Cp .X / while M00  M0 and nw.M0 /  !.

2 Solutions of problems 001500

425

Proceeding inductively assume that k 2 ! and we have sets L0 ; M0 ; : : : ; Lk ; Mk


with the following properties:
(1) the set Li is adequate in Cp .Y / and the set Mi is dimensionally n-adequate in
Cp .X / for all i  k;
(2) Li  Mi and nw.Li / D nw.Mi / D ! for all i  k;
(3) Mi  MiC1 and r.Mi /  LiC1 for every i < k.
Since nw.r.Mk //  nw.Mk /  !, we can apply Fact 5 of U.086 once more
to find an adequate subset LkC1 of the space Cp .Y / such that r.Mk /  LkC1 and
0
nw.LkC1 / D !. Analogously, we can find an adequate subset MkC1
of the space
0
0
Cp .X / such that Mk [ LkC1  MkC1 and nw.MkC1 /  !. Apply Problem 161 to
0
find a set MkC1 which is dimensionally n-adequate in Cp .X / while MkC1
 MkC1
and nw.MkC1 /  !. This completes the inductive step and shows that we can
construct families fLi W i 2 !g and fMi W i 2 !g such that the properties (1)(3)
hold for all k 2 !. Observe that it follows from (2) and (3) that Li D r.Li / 
r.Mi /  LiC1 for all i 2 S
!.
S
By Fact 1 the set L D i2! Li is adequate in Cp .Y / and the set M D i2! Mi
is dimensionally n-adequate in Cp .X /. It follows from (3)
S and continuity of r that
r.M /  L. However, the set M is closed in Cp .X / and i2! Li  M by (2); this
implies L  M and hence r.M /  r.L/ D L, i.e., r.M / D L.
There exist a compact space Z and a continuous onto map  W X ! Z such that
dim Z  n and M D   .Cp .Z//. It follows from (2) that w.Z/  !. Analogously,
there is a compact space T and a continuous onto map  W Y ! T such that
w.T /  ! and L D  .Cp .T //. Since L is a linear retract of M , the space Cp .T /
is linearly homeomorphic to a linear retract of Cp .Z/. By Problem 390, the space
Cp .T / is a topological linear factor of Cp .Z/; applying Problem 395 we obtain the
inequalities dim T  dim Z  n.
Finally observe that '  .Cp .K// D L0  L D  .Cp .T //. By TFS-163, there
exists a continuous map  W T ! K such that ' D  . Therefore, for any
continuous map ' of Y onto a second countable space K we found a space T
together with maps  W Y ! T and  W T ! K such that dim T  n and ' D .
Therefore dim Y  n D dim X by Problem 161, so our solution is complete.
V.397. (Open Mapping Theorem) Suppose that L is a Banach space, M is a
linear topological space which is of second category in itself and f W L ! M
is a surjective continuous linear map. Prove that f is open. In particular, any
continuous linear onto map between two Banach spaces is open.
Solution. There exists a norm jj jj on the space L which generates a complete
metric on L. Let B.r/ D fx 2 L W jjX jj < rg be the open r-ball centered at 0L for
any r > 0. Let us establish first that
(1) 0M belongs to the interior of f .B.r// for any r > 0.
S
By linearity of the map f we have M D fn f .B. 2r // W n 2 Ng; the space
M being of second category in itself, the set n f .B. 2r // has nonempty interior for
some n 2 N. The map ' W M ! M defined by the equality '.x/ D nx for any

426

2 Solutions of problems 001500

x 2 M is a homeomorphism so the set f .B. 2r // also has nonempty interior U .


Besides, we have the inclusions B. 2r / D B. 2r / and B. 2r / C B. 2r /  B.r/




which imply that f B. 2r /  f B. 2r /  f .B.r// by linearity of the map f .








Apply Problem 203 to see that f B. 2r /  f B. 2r /  f B. 2r /  f B. 2r / 
 r 
f .B.r//. The set U  U is an open neighborhood of 0M and U  U  f B. 2 / 


f B. 2r /  f .B.r// which shows that (1) is proved.
Our next step is to show that
(2) if rn > 0 and yn 2 f .B.rn // for any n 2 ! then rn ! 0 implies yn ! 0M .
Indeed, if U 2 .0M ; M / then there is V 2 .0M ; M / such that V  U .
By continuity of f , the exists W 2 .0L ; L/ for which f .W /  V . Choose a
number r > 0 with B.r/  W and m 2 ! such that rn < r for all n  m. Then
yn 2 f .B.rn //  f .B.r//  f .W /  V  U for all n  m which shows that
yn ! 0M , i.e., (2) is proved.
The following property is crucial for our proof.
(3) f .B.r//  f .B.3r// for any r > 0.
Fix a point y 2 f .B.r// and let rn D 2n r for all n 2 !. It follows from
(1) that P D y  f .B.r1 // is a neighborhood of y, so P \ f .B.r0 // ; and
hence we can find a point x0 2 B.r0 / in such a way that f .x0 / 2 P and therefore
y0 D y  f .x0 / 2 f .B.r1 //. Proceeding inductively, assume that m 2 ! and we
have chosen points x0 ; : : : ; xm and y0 ; : : : ; ym such that
(4) xi 2 B.ri / and yi 2 f .B.riC1 // for all i  m;
(5) yi D yi1  f .xi / for 1 < i  m.
Since ym 2 f .B.rmC1 // and the set P D ym  f .B.rmC2 // is a neighborhood
of the point ym by the property (1), we have P \ f .B.rmC1 // ; and hence we
can find a point xmC1 2 B.rmC1 / in such a way that f .xmC1 / 2 P and therefore
ymC1 D ym  f .xmC1 / 2 f .B.rmC2 //. It is easy to see that the properties (4)
and (5) now hold if m is replaced with m C 1. Thus our inductive procedure can
be continued to construct sequences fxi W i 2 !g and fyi W i 2 !g such that the
conditions (4) and (5) are satisfied for all m 2 !.
Let zi D x0 C : : : C xi forP
all i 2 !. Given P
any numbers n; m 2 ! such that
nm
m
m < n we have jjzn  zm jj  nm
jjx
jj

r which easily
mCi
iD1
iD1 rmCi  2
implies that fzi W i 2 !g is a Cauchy
sequence.
Therefore
z
!
z
for
some z 2 L;
i
P
for any i 2 ! we have jjzi jj  ij D1 jjxj jj  r0 C : : : C ri  2r, so jjzjj  2r < 3r
and hence z 2 B.3r/.
Given any number n 2 ! it follows from the property (5) that we have the
equalitiesP
f .x0 / D y  y0 ; f .x1 / D y0  y1 ; : : : ; f .xn / D yn1  yn and therefore
f .zn / D niD1 f .xi / D y  yn . The properties (2) and (4) show that yn ! 0M and
hence f .zn / ! y. The function f being continuous we have f .zn / ! f .z/ which
implies that f .z/ D y. We showed that y 2 f .B.3r// for any point y 2 f .B.r//,
i.e., f .B.r//  f .B.3r// and hence (3) is proved.

2 Solutions of problems 001500

427

Finally, take an arbitrary set U 2 .0L ; L/ and s > 0 such that B.s/  U .
It follows from (1) that V  f .B. 3s // for some V 2 .0M ; M /. Now apply (3) to
convince ourselves that V  f .B. 3s //  f .B.s//  f .U / and therefore we can
apply Fact 3 of S.496 to conclude that f is an open map.
V.398. (Closed Graph Theorem) Suppose that L and M are Banach spaces and
f W L ! M is a linear map such that its graph G D f.x; f .x// W x 2 Lg is closed
in L  M . Prove that the map f is continuous.
Solution. Say that a norm jj jj on a linear space G is complete if the metric
generated by this norm is complete.
Fact 1. Suppose that jj jjG ; jj jjH are complete norms on linear spaces G and H
respectively. For any point z D .x; y/ 2 G  H let jjzjj D jjxjjG C jjyjjH . Then
jj jj is a complete norm on G  H and the topology of the space .G  H; jj jj/
coincides with the topology of the product .G; jj jjG /  .H; jj jjH /. In particular,
the product of any two Banach spaces is a Banach space.
Proof. It is clear that jjzjj  0 for any z 2 E D G  H . If z D .x; y/ 2 E and
jjzjj D 0 then jjxjjG D 0 and jjyjjH D 0 which shows that x D 0G and y D 0H
and hence z D .0G ; 0H / D 0E . If t 2 R and z D .x; y/ 2 E then the equalities
jjt zjj D jjtxjjG C jjtyjjH D jt j.jjxjjG C jjyjjH / D jt j jjzjj show that the second
axiom of the norm also holds.
Take any a; b 2 E with a D .a1 ; a2 / and b D .b1 ; b2 /. By our definition of jj jj
we have jja C bjj D jja1 C b1 jjG C jja2 C b2 jjH  jja1 jjG C jjb1 jjG C jja2 jjH C
jjb2 jjH D jjajj C jjbjj, so the axiom of triangle is fulfilled and hence jj jj is, indeed,
a norm on E.
Let us temporarily denote by E 0 the space E with the norm jj jj. For any r > 0 let
BG .r/ D fx 2 G W jjxjjG < rg and BH .r/ D fy 2 H W jjyjjH < rg; we will also
need the r-ball B.r/ D fz 2 E W jjzjj < rg. If U 2 .0E ; E/ then there exists " > 0
such that BG ."/  BH ."/  U . It is straightforward that B."/  BG ."/  BH ."/ 
U , so the identity map i W E 0 ! E is continuous by Fact 2 of S.496.
Now, if U 2 .0E ; E 0 / then there exists r > 0 for which B.r/  U . It is
easy to see that BG . 2r /  BH . 2r /  B.r/, so i.U / is a neighborhood of 0E ; this,
together with Fact 3 of S.496, shows that the map i is open. Thus i W E 0 ! E is
a homeomorphism, which is the same as saying that the topology of E coincides
which .E 0 /.
To prove completeness of jj jj assume that zn D .xn ; yn / 2 E for all n 2 ! and
fzn W n 2 !g is a Cauchy sequence with respect to the metric generated by jj jj.
Given any " > 0 there is m 2 ! such that jjzn  zk jj < " whenever n; k  m. Then
jjxn  xk jjG  jjzn  zk jj < " and jjyn  yk jjH  jjzn  zk jj < " for all n; k  m
which shows that fxn W n 2 !g and fyn W n 2 !g are Cauchy sequences in their
respective spaces. By completeness of the norms jj jjG and jj jjH , we can find
x 2 G and y 2 H such that xn ! x and yn ! y. Therefore zn ! z D .x; y/ in
the product space E; since .E 0 / D .E/, the sequence fzn W n 2 !g converges to
z in E 0 , so jj jj is complete and hence Fact 1 is proved.

428

2 Solutions of problems 001500

Returning to our solution apply Fact 1 to see that E D L  M is a Banach space;


let L W E ! L and M W E ! M be the natural projections. It is easy to see
that any closed linear subspace of a Banach space is a Banach space; we omit a
simple proof that G is a linear subspace of L  M and hence G is a Banach space.
The map p D L jG W G ! L is continuous, linear and surjective, so it is open
by Problem 397. Since p is also an injection, it has to be a homeomorphism so
f D .M jG/ p 1 is a continuous map, i.e., our solution is complete.
V.399. Prove that, for any nonempty space X , there exists a continuous surjection
of the space Cp .X / onto Cp .X /  R.
Solution. The expression Y ' Z says that the spaces Y and Z are homeomorphic.
Fact 1. If Z is a normal space and D!  ! embeds in Z as a closed subspace then
Z can be continuously mapped onto Rn for any n 2 N.
Proof. Fix any n 2 N and observe that there exists a discrete family fFi W i 2 !g of
closed subsets of Z such that Fi ' D! for each i 2 !. The space Ki D i; i
n 
Rn is compact and metrizable for any i 2 !, so we
Scan apply TFS-128 to find a
continuous onto map 'i W Fi ! Ki . If x 2 F D i2! Fi then there is a unique
i 2 ! with x 2 Fi ; let '.x/SD 'i .x/. This gives us a continuous map ' W F ! Rn
and it is clear that '.F / D i2! Ki D Rn .
For any k 2 f1; : : : ; ng let k W Rn ! R be the projection onto the k-th factor.
The map k ' W F ! R is continuous, so there exists a continuous map k W Z !
R such that k jF D k ' for every k  n. If  is the diagonal product of the
family fk W 1  k  ng then it is straightforward that  W Z ! Rn is a continuous
map such that jF D ' and therefore .Z/  .F / D '.F / D Rn , i.e.,  is a
continuous map of Z onto Rn , so Fact 1 is proved.
Returning to our solution take any point a 2 X ; then Cp .X / ' I  R where
I D ff 2 Cp .X / W f .a/ D 0g (see Fact 1 of S.409). Since D!  ! embeds in R
as a closed subspace, we can apply Fact 1 to see that there exists a continuous onto
map ' W R ! R2 . If f 2 I and t 2 R then let .f; t / D .f; '.t //; this gives us
a continuous map  W I  R ! I  R2 and it is immediate that  is surjective.
Now it follows from Cp .X / ' I  R and Cp .X /  R ' I  R2 that there exists a
continuous surjective map of Cp .X / onto Cp .X /  R.
V.400. Prove that there exists an infinite compact space K for which there is no
linear continuous surjection of Cp .K/ onto Cp .K/  R. In particular, K C is not
l-equivalent to the space K.
Solution. If X is a space and A  X then A W Cp .X / ! Cp .A/ is the restriction
map; let Cp .AjX / D A .Cp .X //. For any nonempty D consider the set RD
 D
ff 2 RD W f .D/  r; r
for some r > 0g; in other words, RD
is
the
set
of
all

bounded functions on D. If f 2 RD
then
jjf
jj
D
supfjf
.x/j
W
x
2
Dg.
Given
D

infinite sets D and E say that a linear continuous map ' W RD ! RE is adequate if
E
D
D
E
'.RD
 / D R and the map 'jR W .R ; jj jjD / ! .R ; jj jjE / is continuous.

2 Solutions of problems 001500

429

If ' W RD ! R is a continuous nontrivial linear functional then ' 2 Lp .D/ and


hence we can apply TFS-197 to see that
(1) there exist uniquely determined
distinct d1 ; : : : ; dn 2 D and 1 ; : : : ; n 2
P
Rnf0g such that '.f / D niD1 i f .di / for any f 2 RD .
Now, if L is a dense linear subspace of RD and ' W L ! R is a continuous linear
functional then there exists a unique continuous linear extension of ' over the whole
space RD (see Problem 224), so ' has the same representation as in (1).
Given nonempty sets D and E assume that L is a dense linear subspace of RD
and M is a dense linear subspace of RE . If  W L ! M is a linear continuous
map and we are given a point y 2 E then let '.f / D f .y/ for all f 2 M ; this
defines a continuous linear functional ' W M ! R. Since '  is a continuous linear
functional on L, by our above observation there exist uniquely determined distinct
points
Pn x1 ; : : : ; xn 2 D and numbers 1 ; : : : ; n 2 Rnf0g such that '.f /.y/ D
n g then for any x 2 supp.y/
iD1 i f .xi / for any f 2 L. If supp.y/ D fx1 ; : : : ; x
P
there is a unique yx 2 Rnf0g such that '.f /.y/P
D x2supp.y/ yx f .x/ for each
f 2 L. For any A  D and y 2 E let .y; A/ D fj yx j W x 2 supp.y/ \ Ag. Of
course, .y; A/ depends on D; E; L; M and  but we omit that to simplify notation.
As usual, D D f0; 1g is the two-point discrete space. For any map f 2 R! there
exists a unique continuous function f
W ! ! R such that f
j! D f . If X
is a set and A  X then A W X ! D is the characteristic function of A, i.e.,
A .A/  f1g and A .X nA/  f0g. The function A formally depends on X but
we dont mention this because X will always be clear. If and are ordinals and
< then ; / D f W   < g.
Fact 1. Suppose that .B; jj jj/ is a normed space and L is a proper closed linear
subspace of B. For any r > 0 let S.r/ D fx 2 B W jjxjj  rg. Then for any positive
numbers r and R such that r < R the set S.R/ is not contained in S.r/ C L.
Proof. Apply Problem 222 to find a nontrivial continuous linear functional '0 W
B ! R such that '0 .L/ D f0g. There exists " > 0 such that '0 .S."//  .1; 1/; as
an immediate consequence we obtain the inclusion '0 .S.1//  . 1" ; 1" /. Therefore
the number q D supfj'0 .x/j W x 2 S.1/g is positive and consistently defined; let
' D q1 '0 .
We have '.S.1//  1; 1
and supfj'.x/j W x 2 S.1/g D 1. Take a sequence
fxn W n 2 !g  S.1/ such that j'.xn /j ! 1. Given any x 2 Bnf0g we have
x
x
j'.x/j D jjxjj '. jjxjj
/  jjxjj because jjxjj
2 S.1/. If x 2 S.r/CL then x D x 0 Cy
0
for some x 2 S.r/ and y 2 L. Then j'.x/j D j'.x 0 / C '.y/j D j'.x 0 /j  r and
hence '.S.r/ C L/  r; r
.
For any n 2 ! let yn D Rxn ; then yn 2 S.R/ and it follows from j'.xn /j ! 1
that j'.yn /j ! R. Consequently, there exists n 2 ! such that j'.yn /j > r. This
implies that yn 2 S.R/n.S.r/ C L/ and hence Fact 1 is proved.
Fact 2. Assume that .B0 ; jj jj0 / and .B1 ; jj jj1 / are normed spaces and f W B0 ! B1
is a linear map. For any positive number r let S0 .r/ D fx 2 B0 W jjxjj0  rg and

430

2 Solutions of problems 001500

S1 .r/ D fy 2 B1 W jjyjj1  rg be the closed r-balls in the respective spaces. Then


the following conditions are equivalent:
(a) the map f is continuous;
(b) there exists a number R > 0 such that f .S0 .1//  S1 .R/.
If, additionally, the map f is surjective and the spaces B1 and B2 are Banach then
the conditions (a) and (b) are also equivalent to the following statement.
(c) There exist positive numbers r and R such that S1 .r/  f .S0 .1//  S1 .R/.
Proof. If f is continuous then there exists " > 0 such that f .S0 ."//  S1 .1/. By
linearity of f we have f .S0 .1//  S1 . 1" /, so letting R D 1" we obtain the inclusion
f .S0 .1//  S1 .R/; this shows that (a)H)(b).
Now assume that f .S0 .1//  S1 .R/ for some positive number R and take any
set U 2 .01 ; B1 / where 01 is the zero element of B1 . There exists " > 0 such
that S1 ."/  U ; the interior V of the set S0 . R" / contains the zero element 00 of
x
1
2 S0 .1/, so jjxjj
f .x/ 2 S1 .R/ and
the space B0 . If x 2 V nf00 g then jjxjj
0
0

.x/jj1
 R, i.e., jjf .x/jj1  Rjjxjj0  R" R D " which shows that
hence jjfjjxjj
0
f .V /  S1 ."/  U . Therefore the map f is continuous at the point 00 , so it is
continuous by Fact 2 of S.496 and hence (b)H)(a).
Now assume that f is surjective. If f is continuous then it is open by
Problem 397; since 00 belongs to the interior of S0 .1/, the point 01 belongs to the
interior of f .S0 .1//. Therefore S1 .r/  f .S0 .1// for some r > 0 and hence we
proved that (a)H)(c). It is evident that (c)H)(b), so (a) (b) (c), i.e.,
Fact 2 is proved.

Fact 3. Let i W R ! R be the identity map, i.e., i.x/ D x for any x 2 R. Suppose
that X is a separable space and ' W Cp .X / ! Cp .X /  R is a continuous linear
surjection. Then there exists a countable dense set D  X such that the map D D
1
W Cp .DjX / ! Cp .DjX /  R is a continuous linear surjection.
.D  i / ' D
Proof. If D is dense in X then D W Cp .X / ! Cp .DjX / is a linear bijection.
Therefore D is also linear; the product of any number of surjections is also a
surjection, so D is a linear surjection for any dense D  X . To find a set D for
which D is continuous fix a point a X and identify Cp .X /  R with the space
X C D X fag. Therefore ' W Cp .X / ! Cp .X C / and we can use the notion of
support for the map '.
C
Let DS
0 be any dense subset of X ; if n 2 ! and we have a set Dn then let
DnC1 D fsupp.y/
S W y 2 Dn g. This gives us a sequence fDn W n 2 !g of subsets
of X C ; let D D . n2! Dn /nfag.
Observe first that D is dense in X because it contains D0 \ X which is dense in
X . To see that D is continuous note that Cp .DjX /  R is a subspace of the product
RD  R D RD[fag and let py W RD[fag ! R be the projection onto the factor
determined by y for any y 2 D [ fag.
Fix any y 2 D [ fag and consider the map py D W Cp .DjX / ! R.
1
Given
any f 2 Cp .DjX / let g D D
.f /. Then py .D .f // D '.g/.y/ D
P
x2supp.y/ yx g.x/. However, supp.y/  D and g.x/ D f .x/ for any x 2 D

2 Solutions of problems 001500

431

P
(recall that f is the restriction of g to D), so py .D .f // D x2supp.y/ yx f .x/.
For any x 2 D let ex .f / D f .x/ for any f 2 Cp .D/.
PThen ex is continuous on
Cp .D/ and hence on Cp .DjX /. Therefore py D D f yx ex W x 2 supp.y/g
is continuous being a linear combination of continuous functions. This shows that
py D is continuous for any y 2 D [ fag, so D is continuous by TFS-102 and
hence Fact 3 is proved.
Fact 4. Suppose that X is a separable pseudocompact space and we have a linear
continuous surjection ' W Cp .DjX / ! Cp .DjX /  R is for some countably infinite
sets D  X D D. Then there exists a unique adequate map q W RD ! RD  R
such that qjCp .DjX / D '.
Proof. It suffices to prove existence of q because its uniqueness follows from the
fact that Cp .DjX / is dense in RD . Take a point a X and let D C D D fag; we
C
will identify RD  R with RD .
C
For any y 2 D C let py W RD ! R be the projection on the factor determined
by y. The map py ' is a continuous linear functional on Cp .DjX / and hence there
exists a continuous linear functional qy W RD ! R such that qy jCp .DjX / D py '
for any y 2 D C (see Problem 224). The diagonal product q D fqy W y 2 D C g is
C
a continuous linear map and q W RD ! RD ; it is easy to see that qjCp .DjX / D '.
Let Y D X fag; the space X being pseudocompact, the norms jj jjX and jj jjY
are consistently defined on Cp .X / and Cp .Y / respectively; besides, it is easy to see
that BX D .Cp .X /; jj jjX / and BY D .Cp .Y /; jj jjY / are Banach spaces. For any
r > 0 let SX .r/ D fx 2 BX W jjxjjX  rg and SY .r/ D fy 2 BY W jjyjjY  rg;
we will also need the balls TD .r/ D fx 2 RD
 W jjxjjD  rg and TD C .r/ D fy 2
C
W
jjyjj

rg.
RD
C
D

Consider the identity maps iX W BX ! Cp .X / and iY W BY ! Cp .Y / and let
' be the graph of the map '. The maps D iX and D C iY are continuous
isomorphisms, so the map  D .D C iY /1 ' .D iX / W BX ! BY is
consistently defined, surjective and linear. If  is the graph of  then h D .D
iX /  .D C iY / is a condensation of BX  BY onto Cp .DjX /  Cp .D C jY / and
 D h1 .' / which shows that  is closed in BX BY , so the map  is continuous
by Problem 398. Apply Fact 2 to see that
() there exist positive numbers r and R such that SY .r/  .SX .1//  SY .R/.
Observe that Q D .D iX /.SX .1// is a dense subset of TD .1/, so '.Q/ is
C
a dense subset of '.TD .1// in the topology of pointwise convergence on RD
 .
Besides, '.Q/ D .D C iY /..SX .1///  .D C iY /.SY .R//  TD C .R/ by
()). The set TD C .R/ is compact under the topology of pointwise convergence on
C
C
RD because it coincides with the set R; R
D . Therefore '.TD .1//  '.Q/ 
TD C .R/ (the bar denotes the pointwise convergence closure). As a consequence,
C
we have the formula '.TD .n// D n'.TD .1//  nTD C .R/  RD
 . Since
S
C
D
RD
fTD .n/ W n 2 Ng, we conclude that '.RD
 D
 /  R ; the inclusion
D
'.TD .1//  TD C .R/ together with Fact 2 shows that 'jR is continuous with
DC
respect to the norm topologies on RD
 and R .

432

2 Solutions of problems 001500

The inclusion '.Q/ D .D C iY /..SX .1///  .D C iY /.SY .r// together with
Q  TD .1/ and compactness of TD .1/ under the topology of pointwise convergence
show that '.TD .1//  .D C iY /.SY .r// D TD C .r/. This implies the inclusion
S
C
'.TD .n//  TD C .nr/ for any n 2 N, so it follows from RD
D fTD C .nr/ W n 2

DC
D
DC
Ng that '.RD
and hence the map f is adequate, i.e.,
 /  R , so '.R / D R
Fact 4 is proved.
Fact 5. Suppose that D is a countably infinite set and ' W RD ! RD  R is an
adequate map. For an infinite set A  D let fSa W a 2 Ag be a disjoint family of
subsets
S of D. Then, for every > 0 there exists an infinite set B  A such that
.a; fSb W b 2 Bnfagg/ < for any a 2 B.
Proof. Fix any point c D and consider the set D C D D [ fcg; we will identify
C
RD  R with the set RD . For any r > 0 let S.r/ D fx 2 RD
 W jjxjjD  rg and
C
D
SC .r/ D fy 2 R
W jjyjjD C  rg. Let us prove first that
(2) for any " > 0 and for any infinite A0  A there exists b 2 A0 and an infinite
A0  A0 such that .a; Sb / < " for any a 2 A0 .
If (2) is not true then there exists " > 0 and an infinite set A0  A such that for
each b 2 A0 the set Qb D fa 2 A0 W .a; Sb / < "g is finite. By Fact 2 there exists
R > 0 such that '.S.1//  SC .R/. Pick a number j > R" and choose distinct
b1 ; : : : ; bj 2 A0 ; by our assumption there exists a point a 2 A0 n.Qb1 [ : : : [ Qbj /.
S
Pj
We have .a; D/  .a; fSbi W 1  i  j g/ D iD1 .a; Sbi /  j" > R.
for any
There exists a function f 2 1; 1
D D S.1/ such that f .d / D j ad
ad j
P
d 2 supp.a/. Then '.f /.a/ D f ad f .d / W d 2 supp.a/g D .a; D/ > R.
However, f 2 S.1/, so '.f / 2 SC .R/ and hence we must have j'.f /.b/j  R for
any b 2 D C . This contradiction shows that (2) is proved.
Apply (2) to find a point a0 2 A and an infinite set A0  A such that .a; Sa0 / <

for
any a 2 A0 . Proceeding by induction assume that m 2 ! and we have infinite
2
sets A0 ; : : : ; Am and points a0 ; : : : ; am with the following properties:
(3) aiC1 2 Ai nfa0 ; : : : ; ai g and AiC1  Ai whenever i < m;
i1
(4) if i  m and
;
S a 2 Ai then .a; Sai / < 2
(5) Sai C1 \ . fsupp.aj / W j  i g/ D ; for all i < m.
The set S
Am being infinite we can find an infinite set A0  Am nfa0 ; : : : ; am g such
that Sa \ . fsupp.ai / W i  mg/ D ; for any a 2 A0 . Apply (2) to pick an element
amC1 2 A0 and an infinite set AmC1  A0 such that .a; SamC1 / < 2m2 for any
a 2 AmC1 . It is immediate that the conditions (3)(5) are now satisfied if we replace
m with m C 1, so our inductive procedure can be continued to construct sequences
fAi W i 2 !g and fai W i 2 !g for which the properties (3)(5) hold for all m 2 !.
To check that the set B D fai W i 2 !g is as promised observe that it follows
from (3) that the indexation of B is faithful and hence B is an
Sinfinite subset of A.
Now if n 2 ! then supp.an / \ Si D ; for
all
i
>
n,
so
.a
;
fSb W b 2 Bnfan gg/
n
P
1
is equal to .an ; Sa0 [ : : : [ San1 / D n1
C : : : C 2n / <
iD1 .an ; Sai / < .2
and hence Fact 5 is proved.

2 Solutions of problems 001500

433

Fact 6. Given spaces X and Y assume that D  X D D; E  Y D E and


' W Cp .DjX / ! Cp .EjY / is a continuous linear surjection. Suppose that y 2 E
and S  supp.y/ is a nonempty set. Then for any U 2 .S; X / there exists a set
V 2 .y; Y / such that .x; U \ D/ > 12 .y; S / for any x 2 V \ E.
Proof. Let r D .y; S /; it is easy to find a set U 0 2 .X / such that U 0  U and

U 0 \supp.y/ D S . There exists a function f 2 Cp .X; I/ such that f .x/ D j yx
j for
yx
0
any x 2 S and f .X nU /  f0g; let g D f jD. It is straightforward that '.g/.y/ D
r; by continuity of the function h D '.g/ there exists a set V 2 .y; Y / such that
h.V \ E/  . 2r ; C1/.
Take any point x 2 V \ E and consider the set T D supp.x/ \ U 0 . Then
X
X
.x; U \ D/ 
fj xt j W t 2 T g 
fj xt f .t /j W t 2 T g
X
r

f xt f .t / W t 2 T g D '.f /.x/ >
2
and hence Fact 6 is proved.
Fact 7. Suppose that D D fdi W i 2 !g is a faithfully indexed set and consider the
set Dn D fdi W i  ng for any n 2 !; assume additionally that ' W RD ! RD  R
is an adequate map. Then there exists " > 0 and an infinite set A  ! such that
.dn ; DnDn / > " whenever n 2 A.
Proof. Choose a point e D and let D C D D [ feg; we will identify RD  R with
C
RD . Given any r > 0 we will need the closed balls S.r/ D fx 2 RD
 W jjxjjD  rg
DC
D
DC
is
and SC .r/ D fy 2 R W jjyjjD C  rg. The map 'jR W RD
 ! R
continuous and onto, so there exists r > 0 such that SC .r/  '.S.1// (see Fact 2).
Let " D 2r ; we claim that the set A D fn 2 ! W .dn ; DnDn / > "g is infinite.
Striving
Sfor contradiction assume that A is finite and hence there exists m 2 !
such that fsupp.dn / W n 2 Ag [ supp.e/  Dm . It turns out that
(6) if d 2 Dm [ feg then .d; DnDm /  ".
To see that (6) holds observe first that if d D dn for some number n 2 A then
supp.d /  Dm and therefore .d; DnDm / D 0. If d D dn and n A then the
inequalities .d; DnDm /  .d; DnDn /  " show that (6) is true.
If P D fx 2 S.1/ W x.DnDm / D f0gg and Q D fx 2 S.1/ W x.Dm / D f0gg
C
! RDm [feg be
then P CQ D S.1/ and hence '.P /C'.Q/  SC .r/. Let  W RD

Dm [feg
the natural projection; if T D fx 2 R
W jjxjjDm [feg  rg then T  .S C .r//
and therefore T  ..'.P // C .'.Q//. It is easy to see that the set P is contained
in an .m C 1/-dimensional linear subspace of RD
 ; since the linear dimension of
RDm [feg is equal to m C 2, the set .'.P // is contained in a proper linear subspace
L of RDm [feg (which is automatically closed byP
Problem 228).
If x 2 Q and d 2 Dm [feg
then
'.x/.d
/
D
f dt x.t / W t 2 supp.d /nDm g and
P
therefore j.'.x//.d /j  fj dt j W t 2 supp.d /nDm g D .d; DnDm /  " by

434

2 Solutions of problems 001500

the property (6). As a consequence, the set .'.Q// is contained in the "-ball T 0 D
fx 2 RDm [feg W jjxjjDm [feg  "g. Now it follows from T  .'.P // C .'.Q//
that T  L C T 0 which is a contradiction with Fact 1. Thus the set A is infinite and
hence Fact 7 is proved.
Fact 8. Suppose that X is a pseudocompact space, D is a countable dense subset of
X and there exists a continuous linear surjection ' W Cp .DjX / ! Cp .DjX /  R.
Then for any dense set E  D there exists " > 0 such that, for some infinite set
A  E and a family fSa W a 2 Ag of pairwise disjoint finite subsets of D we have
the following properties:
(a) Sa  S
supp.a/ and .a; Sa / > " for any a 2 A;
(b) A \ . a2A Sa / D ;.
(c) either ad > 0 for all a 2 A and d 2 Sa or ad < 0 for all a 2 A and d 2 Sa .
Proof. By Fact 4 we can consider that ' is defined on the whole space RD and
' W RD ! RD  R is an adequate map. Choose a faithful enumeration fdn W n 2 !g
of the set D and let Dn D fd0 ; : : : ; dn g for any n 2 !. By Fact 7 there exists an
infinite set N  ! such that, for some > 0 we have .dn ; DnDn / > for each
n 2 N.
Let n0 D nn.N / and P0 D supp.dn0 /nDn0 . Proceeding inductively we can find
an increasing sequence fni W i 2 !g  N such that
(7) for any i; j 2 ! if j < i then supp.dnj / [ fdnj g  Dni 1 .
Let Pi D supp.dni /nDni for each i 2 !; the property (7)
Simplies that the family
P D fPi W i 2 !g is disjoint and fdni W i 2 !g \ . P/ D ;. For the set
P0  supp.dn0 / take U 2 .P0 ; X / and V 0 2 .dn0 ; X / such that U \ V 0 D ; and
apply Fact 6 to find a set V 2 .dn0 ; X / such that V  V 0 and .a; U \ D/ > 2
for any a 2 V . The set E being dense in X there exists a0 2 V \ E; consider the
set Q0 D supp.a0 / \ U .
Proceeding inductively assume that k 2 ! and we have chosen distinct points
a0 ; : : : ; ak 2 E and disjoint sets Q0 ; : : : ; Qk such that
(8) Qi  supp.ai / and
S .ai ; Qi / >
(9) fa0 ; : : : ; ak g \ . ik Qi / D ;.

for all i  k;

Let H D fa0 ; : : : ; ak g [ Q0 [ : : : [ Qk and find i 2 ! such that dni [ Pi does not


meet H . Take a set U 2 .Pi ; X / such that U \.H [fdni g/ D ; and apply Fact 6 to
find a set V 2 .dni ; X / for which V \ U D ; and .a; U \ D/ > 2 for all a 2 V .
By density of E we can find a point akC1 2 E\V and a set QkC1  supp.akC1 /\U
such that .akC1 ; QkC1 / > 2 . It is clear that the properties (7) and (8) are now
fulfilled if we replace k with k C 1, so we can pursue our inductive procedure to
construct a set fai W i 2 !g  E and a family fQi W i 2 !g such that (8) and (9)
hold for all k 2 !.
For any i 2 ! let QiC D fd 2 Qi W ai d > 0g and Qi D fd 2 Qi W ai d < 0g;
it follows from .ai ; Qi / D .ai ; QiC / C .ai ; Qi / that either .ai ; QiC / > 4 or
.ai ; Qi / > 4 . This shows that there exists an infinite set N  ! such that either
.ai ; QiC / > 4 or .ai ; Qi / > 4 for all i 2 N .

2 Solutions of problems 001500

435

Finally, let " D 4 and A D fai W i 2 N g. If a 2 A then a D ai for a unique


i 2 !; let Sa D Qi . It is immediate that the conditions (a)(c) are satisfied, so
Fact 8 is proved.
Fact 9. Suppose that fSn W n 2 !g is a disjoint family of subsets of ! and we
have a set fat ; bt g  ! for every t 2 T . Assume additionally that jT j < c and
Sn
.at / D Sn
.bt / for all t 2 T and S
n 2 !. Then there exists an infinite set
A  ! such that, for any A0  A, if S D fSn W n 2 A0 g then S
.at / D S
.bt /
for each t 2 T .
Proof. Fix a family A of infinite subsets of ! such that jAj D c and A \ B is
finite for any distinct
A; BS2 A (see TFS-141). For any A  ! consider the set
S
H.A/ D cl! . n2A Sn /n. n2A cl! .Sn //. Let us prove first that
(10) H D fH.A/ W A 2 Ag is a disjoint family.
0
Indeed,Sif A; B 2 A then S
consider the sets
S C D A \ B and A D AnB. Observe
that cl! . n2A Sn / D cl! . n2A0 Sn / [ . fcl! .Sn / W n 2 C g/ because the set C
is finite; as an immediate consequence, H.A/ D H.A0 /. Analogously, if B 0 D BnA
0
then H.B/ D H.B
S/
S
The sets P D n2A0 Sn and Q D n2B 0 Sn are disjoint, so the sets cl! .P / and
cl! .Q/ are disjoint as well by Fact 1 of S.371. Since H.A/  P and H.B/  Q,
we have H.A/ \ H.B/ D ;, i.e., (9) is proved.
It follows from jT j < c and the property (9) that there exists a set A 2 A
0
such that
S H.A/ \ fat ; bt g D ; for any t 2 T ; fix any A  A; t 2 T and let
S D n2A0 SSn . If fat ; bt g \ cl! .S / D ; then S
.at / D S
.bt / D 0. The sets
S and F D
/ D ; and, in particular,
! .FS
S n2AnA0 Sn are disjoint so cl! .S / \ cl
0
H.A0 / \ . n2AnA0 cl! .S
//
D
;.
Since
also
H.A
/
\
.
n2A0 cl! .Sn // D ;, we
Sn
conclude that H.A0 / \ . n2A cl! .Sn // D ; and therefore H.A0 /  H.A/.
Finally, assume that the intersection of fat ; bt gand the set cl! .S / is nonempty.
0
However, fat ; bt g \ H.A/ D ; and
/ D ;, so it follows from
S hence fat ; bt g \ H.A
0
0
the equality cl! .S / D H.A /[. fcl! .Sn / W n 2 A g/ that fat ; bt g\cl! .Sn / ;
for some n 2 A0 . Recalling that Sn
.at / D Sn
.bt / we convince ourselves that
fat ; bt g  cl! .Sn /  cl! .S / and therefore S
.at / D S
.bt / D 1, i.e., Fact 9
is proved.

Fact 10. Suppose that ft W ! ! D for any t 2 T and jT j < c. Then, for any infinite
set A  ! there exist distinct points a; b 2 cl! .A/ such that ft .a/ D ft .b/ for any
t 2 T.
Proof. If the family F D fft W t 2 T g separates the points of F D cl! .A/ then the
diagonal product of F embeds F in the space DT and hence w.F /  w.DT / < c.
However, the space F is homeomorphic to A (see Fact 2 of S.451) which, in turn,
is homeomorphic to !. Therefore w.!/ < c; this contradiction with TFS-368
shows that the family F cannot separate the points of cl! .A/, i.e., there are distinct
a; b 2 cl! .A/ such that ft .a/ D ft .b/ for any t 2 T , so Fact 10 is proved.

436

2 Solutions of problems 001500

Returning to our solution let us introduce some notation. We will construct


inductively a set of functions fu W < cg  D! such that its diagonal product
u D fu W < cg W ! ! Dc will give us the promised space K as the closure of
the set u.!/ in Dc .
Letting un D fng for each n 2 ! we obtain the family fun W n 2 !g, so the rest
of the inductive procedure will show us how to construct the set fu W !  < cg.
For any A; B  c with A  B the map A W DB ! DA is the natural projection and
p W DB ! D is the projection onto the factor determined by , i.e., p D fg for
all 2 B. The maps A and p formally depend on B but the set B will always be
clear from the context, so we dont mention it to simplify the notation.
If A  c and the set fu W 2 Ag is already constructed then define the map
uA W ! ! DA to be the diagonal product fu W 2 Ag and let KA D uA .!/.
Say that a pair .D; '/ is admissible if the following conditions are satisfied:
(11)
(12)
(13)
(14)

there is a countable set AD  c such that !  AD and D  DAD


there is a map D W ! ! D such that D .n/j! D un for each n 2 !;
' W RD ! RD  R is an adequate map;
there exists a number " > 0 such that, for some infinite set E  ! and a family
fSn W n 2 Eg of finite pairwise disjoint subsets of D, we have
S
(14.1) D .E/ \ . fSn W n 2 Eg/ D ;;
(14.2) if the function . ; / is determined by the map ' then Sn 
supp.D .n// and .D .n/; Sn / > " for any n 2 E;
(14.2) either D .n/d > 0 for any d 2 Sn and n 2 E or D .n/d < 0 for any
d 2 Sn and n 2 E.

Take a countably infinite set Q, a point a Q and compactify the discrete


space Q with the point a, i.e., all points of Q are isolated in X D Q [ fag and
U 2 .a; X / if and only if a 2 U and X nU is finite. Fix a point q 2 Q and let
L D Cp .QjX /; in the space L consider the subspaces M D ff 2 L W f .q/ D 0g
and M 0 D ff 2 L W f .Qnfqg/ D f0gg. For any f 2 L let r.f /.x/ D f .x/
for any x 2 Qnfqg and r.f /.q/ D 0. It is easy to check that r W L ! M is a
linear retraction. If we let r 0 .f /.x/ D 0 for all x q and r 0 .f /.q/ D f .q/ then
r 0 W L ! M 0 is also a linear retraction and x D r.x/ C r 0 .x/ for any x 2 Q.
Applying Problem 391 we convince ourselves that L
M  M 0
M  R. It
is an easy exercise to see that M
L, so we proved that there exists a linear
homeomorphism of Cp .QjX / onto Cp .QjX /  R.
Now let D D fun W n 2 !g  D! and AD D !. If D .n/ D un for all n 2 !
then D W ! ! D is a bijection. Let .n/ D 0 for all n 2 !; it is straightforward
that X D D [ f g is a one-point compactification of the discrete space D. By
what we proved in the preceding paragraph there exists a linear homeomorphism
' W Cp .DjX / ! Cp .DjX /  R. By Fact 4 we can consider that ' is defined on RD
and ' W RD ! RD  R is an adequate map. Applying Fact 8 we conclude that there
exists an infinite set E  ! such for which the properties (14.1)(14.3) hold and
hence .D; '/ is an admissible pair.

2 Solutions of problems 001500

437

It is easy to see that the cardinality of the family A of all admissible pairs does
not exceed c, so we can find a set Q  cn! such that A D f.D ; ' / W 2 Qg and
AD  for any 2 Q. Letting D D D and ' D ' for all 2 .cn!/nQ we
obtain an enumeration f.D ; ' / W !  < cg of the family A such that AD 
for any 2 cn!. To simplify the notation let  D D and AD D A for any
2 cn!; observe that A! D !.
Fact 11. There exists a family of functions fu W !  < cg  D! and a set of
pairs f.a ; b / W !  < cg  !  ! such that, for any 2 !; c/ we have the
following properties:
(a) u
.a 0 / D u
.b 0 / for all ;  0 2 !; /;
(b) if uA .!/  D  KA then for any Q  Dc such that A [fg .Q/  KA [fg
and A jQ W Q ! D is a bijection, for the function g D ' .p .A jQ/1 /
we have g 
.a / g 
.b /.
Proof. Assume that !  < c and we have functions fu W < g and pairs
f.a ; b / W < g such that the conditions (a) and (b) are satisfied for all 2
!; /. If the statement uA .!/  D  KA is not true then let u D , choose
any point a 2 ! and let b D a ; it is clear that the properties (a) and (b) now
hold for all  .
Now assume that uA .!/  D  KA . The properties (11) and (12) show that
A! . .n// D un ; since the set A1! .un / \ KA 3  .n/ is clopen in K for every
n 2 !, it follows from density of uA .!/ in KA that  .n/ D uA .n/ for all n 2 !,
i.e.,  D uA .
There exist E  !; " > 0 and a family S D fSn W n 2 Eg as in the condition
(14) for the set D . Observe that all statements of (14) hold if we replace E with any
infinite subset of E. We will have to pass several times to an infinite subset of E; to
simplify notation, we will denote the respective smaller set by E again. Choose an
increasing enumeration fni W i 2 !g of the set E.
Let nQ D  .n/ for each n 2 !; recalling that KA is a metrizable compact space
we can pass to an infinite subset of E if necessary to guarantee that the sequence
fnQ i W i 2 !g converges to a point s 2 KA . The family S is disjoint, so we can
pass once more to an infinite subset of E to assure that s Sn for all n 2 E. As a
consequence,
(15) Sn \ clKA .fnQ W n 2 Eg/ D ; for any n 2 E.
Case 1. There exist a point t 2 KA and a number > 0 such that, for any U 2
.t; KA / the set fn 2 E W .n;
Q U \ Sn / > g is infinite.
Choose a decreasing local base fUk W k 2 !g of t in KA . The assumption of our
Case shows that we can find an increasing sequence fik W k 2 !g  ! such that
.nQ ik ; Uk \ Snik / > ; let mk D nik and Pmk D Uk \ Snik for any k 2 !. Passing to
a smaller infinite subset of E if necessary we can assume that E D fmk W k 2 !g.
Therefore
(16) .n;
Q Pn / > for any n 2 E.

438

2 Solutions of problems 001500

The sequence fPmk W k 2 !g being disjoint we can refine the set E once more
to be able to consider that t Pmk for all k 2 !. It is easy to find a disjoint family
fVmk W k 2 !g of clopen subsets of KA such that Pmk  Vmk  Uk nft g while
Vmk \ fnQ W n 2 Eg D ; and supp.m
Q k / \ Vmk D Pmk for each k 2 !.
Assume that t 2 supp.n/
Q for infinitely many n 2 E and hence we can pursue our
practice of refining E and consider that t 2 supp.n/
Q for all n 2 E. Let q.d / D 1

0
for all d 2 D . Then q 2 RD
 ; the map ' being adequate, the function q D ' .q/
0
is bounded on D and
Q  r for all n 2 E.
Phence there exists r > 0 such that jq .n/j
Recall that q 0 .n/
Q D f nQ d W d 2 supp.n/g;
Q since the numbers nQ d are of the same
sign, we have j nQ t j  r for all n 2 E. Therefore the sequence f nQ t W n 2 Eg is
bounded, so we can pass to smaller infinite subset of E again to consider that it is
convergent. Refining E once again we can assume that
(17) either t supp.n/
Q for all n 2 E or there exists 2 R such that t 2 supp.n/
Q
and nQ t 2 . ; C 4 / for all n 2 E.
We still need to refine the set E once more to apply Fact 5 to the family fVn \D W
n 2 !g to be able to consider that
S
(18) .n;
Q fVk \ D W k 2 Enfngg/ < 4 for any n 2 E.
Let Rn D fk 2 ! W kQ 2 Vn g for all n 2 E. It is easy to see that the family
B D fp1 .0/ \ KA ; p1 .1/ \ KA W 2 A g is a clopen subbase in the space KA .
Every set Vn is compact, so it can be represented as a finite union of some finite
intersections of the elements of B. Thus the characteristic function of Rn is a finite
sum of finite products of the elements of the family fu ; 1  u W 2 A g. This,
together with the property (a) shows that Rn
.a / D Rn
.b / for all < and
n 2 E. By Fact 9 we can refine the set E once again to consider that
(19) SfRn Wn2E 0 g
.a / D SfRn Wn2E 0 g
.b / for any E 0  E and 2 !; /.
Next, apply Fact 10 to find distinct points a ; b 2 cl! .E/ for which we have
the equality u
.a / D u
.b / for all < . Choose a set E 0  E such that
a 2 cl! .E 0 / and b cl! .E 0 /; then b 2 cl! .EnE 0 /. Let us check that for
the function u D SfRn Wn2E 0 g and the points a ; b , the conditions (a) and (b) are
satisfied.
SWe already saw that Vn \  .E/ D ; for any n 2 E; this implies the equality
. fRn W n 2 Eg/ \ E D ;, so u .E/ D f0g and hence u
.a / D u
.b / D 0.
This, together with (19), shows that (a) is fulfilled for all  .
Assume that Q  Dc , the map A jQ W Q ! D is a bijection and we have the
inclusion A [fg .Q/  KA [fg ; let h D p ..A jQ/1 /. Given any n 2 ! there
is a unique x 2 Q such that A .x/ D n;
Q since also y D A [fg .x/ 2 KA [fg
and yjA D xjA D n,
Q we conclude that yj! D un , so it follows from density of
A [fg .!/ in KA [fg that y is the unique point of KA [fg such that yjA D n.
Q
Consequently, y coincides with A [fg .n/ and hence p .x/ D p .y/ D u .n/.
This proves that h.n/
Q D u .n/ for any n 2 !.
If W is a neighborhood of t then all but finitely many elements of the family
fVn W n 2 Eg are contained in W . Therefore the function v D SfVn Wn2E 0 g is

2 Solutions of problems 001500

439

locally constant at every point of KA nft g. Given d 2 KA nft g fix a clopen set
G 2 .d; KA / such that v.G/ D fi g  D. Then the sets G 0 D G  fi g  KA [fg
and G 00 D KA [fg nG 0 are clopen in KA [fg , disjoint and G 0 [ G 00 D KA [fg .
Therefore, for every d 2 G, the point x D .d; i / is the unique element of KA [fg
such that xjA D d .
Now if d 2 D nft g and x 2 Q is the unique point such that xjA D d then
it follows from x 0 D xj.A [ fg/ 2 KA [fg that x 0 coincides with the unique
extension of d over A [ fg. This proves that
(20) h.d / D SfVn Wn2E 0 g .d / for any d 2 D nft g.
For any n 2 E let ln D nQ t h.t / if t 2 supp.n/
Q or ln D 0 otherwise. Since
h.t / 2 D, it follows from (17) that
(21) there exists l 2 R such that ln 2 .l; l C 4 / for any n 2 E.
Let g D ' .h/; observe first that
P
S
(22) g.n/
Q D f nQ d W d 2 supp.n/
Q \ . fVk W k 2 E 0 g/g C ln for any n 2 E.
By the property (14.2) all numbers
nQ d are of the same sign; assume first that
S
they are positive and let V 0 D k2E 0 Vk . If n 2 E 0 then we have the equality
P
P
0
(23) g.n/
Q D f nQ d W d 2 supp.n/\V
Q
f nQ d W d 2 supp.n/\.V
Q
nVn /gCln .
n /gC
P
Thus g.n/
Q  f nQ d W d 2 supp.n/
Q \ Vn /g C ln D .n;
Q Pn / C ln , so we can
apply (16) and (21) to see that g.n/
Q > l C . P
If n 2 EnE 0 then V 0 nVn D V 0 , so g.n/
Q D f nQ d W d 2 supp.n/
Q \ .V 0 nVn /g C

ln , so it follows from (18) that g.n/


Q < 4 C ln < l C 2 . Recalling that a 2 cl! .E 0 /
0
and b 2 cl! .EnE / we conclude that g
.a /  l C while g
.b /  l C 2
and therefore g 
.a / g 
.b /.
Now if nQ d < 0 for any n 2 E and d 2 supp.n/
Q then fix any n 2 E 0 . We still
0
have (22) and hence g.n/
Q D .n;
Q Pn /  .n;
Q V nVn / C ln  .n;
Q Pn / C ln , so
we can apply (16) and (21) to see that g.n/
Q <  C l C 4 D l  34 .
Q D .n;
Q V 0 nVn / C ln  ln  4 > l  4 .
If n 2 EnE 0 then V 0 nVn D V 0 , so g.n/
0
Recalling again that a 2 cl! .E / and b 2 cl! .EnE 0 / we obtain the inequality
g 
.a /  l  34 while g 
.b /  l  4 and therefore g 
.a /
g 
.b /.
Thus we carried out the induction step in Case 1.
Case 2. For any point t 2 KA and > 0 there exists U 2 .t; KA / such that the
set N.U; / D fn 2 E W .n;
Q U \ Sn / > g is finite.
Observe first that we can replace point by finite set in the formulation of
Case 2, i.e., for any finite set P  KA there exist U 2 .P; KA / such that the set
N.U; / is finite. To see this take an enumeration fx1 ; : : : ; xk g of the set P and pick

a set U
Si 2 .xi ; KA / such that the set N.Ui ; k / is finite for any i D 1; : : : ; k; then
U D fUi S
W 1  i  kg is an open neighborhood of the set P . It is immediate that
N.U; /  fN.Ui ; k / W 1  i  kg, so N.U; / is a finite set as promised.

440

2 Solutions of problems 001500

Take any number n0 2 E and let D 8" ; by the property (15), there exists
U 2 .supp.nQ 0 /; KA / such that N.U; 8" / is finite and U \  .E/ D ;. Let i0 D 0
and pick disjoint clopen sets Un0 and Vn0 such that Sn0  Vn0 ; supp.nQ 0 /nSn0  Un0
and Un0 [ Vn0  U .
Proceeding inductively assume that k 2 ! and we have an increasing sequence
fi0 ; : : : ; ik g  E together with a disjoint family fUnij ; Vnij W j  kg of clopen
subsets of KA with the following properties:
the set Ej D N.Unij [ Vnij ; 2j 3 "/ is finite for any j  k;
.Unij [ Vnij / \  .E/ D ; for any j  k;
S
fEl W l < j g/ for all j  k;
nij 2 En.
S
Snij n. fUnil [ Vnil W l < j g/  Vnij for all j  k;
S
supp.nQ ij /n.Snij [ . fUnil [ Vnil W l < j g//  Unij whenever j  k.
S
Take ikC1 > ik such that nikC1 2 En fEl W l 
Skg and apply (15) to find an
open neighborhood U of the set P D supp.nQ ikC1 /n. fUnij [ Vnij W j  kg/ such
S
that the set N.U; 2k4 "/ is finite and U \. .E/[. fUnij [Vnij W j  kg// D ;.
It is easy to find disjoint clopen sets UnikC1 and VnikC1 in the space KA such that
SnikC1 \P  VnikC1 ; P nSnikC1  UnikC1 and UnkC1 [VnkC1  U . It is clear that the
conditions (24)(28) are now satisfied if we replace k with k C 1, so our inductive
procedure can be continued to construct an increasing sequence fij W j 2 !g  !
and a disjoint family fUnij ; Vnij W j  kg such that the properties (24)(28) hold for
any k 2 !.
S
Let mk D nik and Pmk D Smk n. fUmjS[ Vmj W j < kg/ for any k 2 !. Observe
that .m
Q k ; Pmk / D .m
Q k ; Smk /.m
Q k ; . fU
Smj [Vmj W j < kg/\Smk /. Recalling
that .m
Q k ; Smk / > " and the number .m
Q k ; . fUmj [ Vmj W j < kg/ \ Smk / does
P
P
j 3
"  4" we
not exceed f.m
Q k ; .Umj [ Vmj / \ Smk / W j < kg 
j <k 2
conclude that

(24)
(25)
(26)
(27)
(28)

(29) .m
Q k ; Pmk / > " 

"
4

D 34 " for every k 2 !.

Observe that fmk W k 2 !g  E is an infinite set; as before, we will apply Fact 5


to the family fVk \ D W k 2 !g to find an infinite subset of fmk W k 2 !g (denoting
it by E again) to be able to consider that
S
(30) .n;
Q fVk \ D W k 2 Enfngg/ < 4" for any n 2 E.
Let Rn D fk 2 ! W kQ 2 Vn g for all n 2 E. It is easy to see that the family
B D fp1 .0/ \ KA ; p1 .1/ \ KA W 2 A g is a clopen subbase in the space KA .
Every set Vn is compact, so it can be represented as a finite union of some finite
intersections of the elements of B. Thus the characteristic function of Rn is a finite
sum of finite products of the elements of the family fu ; 1  u W 2 A g. This,
together with the property (a) shows that Rn
.a / D Rn
.b / for all < and
n 2 E. By Fact 9 we can refine the set E once again to consider that
(31) SfRn Wn2E 0 g
.a / D SfRn Wn2E 0 g
.b / for any E 0  E and 2 !; /.

2 Solutions of problems 001500

441

Next, apply Fact 10 to find distinct points a ; b 2 cl! .E/ for which we have
the equality u
.a / D u
.b / for all < . Choose a set E 0  E such that
a 2 cl! .E 0 / and b cl! .E 0 /; then b 2 cl! .EnE 0 /. Let us check that for
the function u D SfRn Wn2E 0 g and the points a ; b , the conditions (a) and (b) are
satisfied.
SWe already saw that Vn \  .E/ D ; for any n 2 E; this implies the equality
. fRn W n 2 Eg/ \ E D ;, so u .E/ D f0g and hence u
.a / D u
.b / D 0.
This, together with (31), shows that (a) is fulfilled for all  .
Assume that Q  Dc , the map A jQ W Q ! D is a bijection and we have the
inclusion A [fg .Q/  KA [fg ; let h D p ..A jQ/1 /. Given any n 2 ! there
is a unique x 2 Q such that A .x/ D n;
Q since also y D A [fg .x/ 2 KA [fg
and yjA D xjA D n,
Q we conclude that yj! D un , so it follows from density of
A [fg .!/ in KA [fg that y is the unique point of KA [fg such that yjA D n.
Q
Consequently, y coincides with A [fg .n/ and hence p .x/ D p .y/ D u .n/.
This proves that h.n/
Q D u .n/ for any n 2 !.
Observe that, for each n 2 E and d 2 supp.n/
Q the function v D SfVn Wn2E 0 g
is locally constant at d , i.e., there exists a clopen set G 2 .d; KA / such that
v.G/ D fi g  D. Indeed,
S the function v is not locally constant only at the points
of V 0 nV 0 where V 0 D fVk W k 2 E 0 g. However, supp.m
Q k /  Umk [ Vmk , so our
statement follows from the fact that .Umk [ Vmk / \ .V 0 nV 0 / D ; for any k 2 !.
Then the sets G 0 D G  fi g  KA [fg and G 00 D KA [fg nG 0 are clopen in
KA [fg , disjoint and G 0 [ G 00 D KA [fg . Therefore, for every d 2 G, the point
x D .d; i / is the unique element of KA [fg such that xjA D d .
Now if d 2 supp.n/
Q for some n 2 E and x 2 Q is the unique point such that
xjA D d then it follows from x 0 D xj.A [ fg/ 2 KA [fg that x 0 coincides with
the unique extension of d over A [ fg. This proves that
S
(32) h.d / D SfVn Wn2E 0 g .d / for any d 2 fsupp.n/
Q W n 2 Eg.
Let g D ' .h/; observe first that
P
S
(33) g.n/
Q D f nQ d W d 2 supp.n/
Q \ . fVk W k 2 E 0 g/g for any n 2 E.
By the property (14.2) all numbers nQ d are of the same sign; assume first that
they are positive. If n 2 E 0 then we have the equality
P
P
(34) g.n/
Q D f nQ d W d 2 supp.n/
Q \ Vn /g C f nQ d W d 2 supp.n/
Q \ .V 0 nVn /g.
P
Thus g.n/
Q  f nQ d W d 2 supp.n/
Q \ Vn /g DP.n;
Q Pn /  34 " by (29).
0
0
0
If n 2 EnE then V nVn D V , so g.n/
Q D f nQ d W d 2 supp.n/
Q \ .V 0 nVn /g
"
and hence it follows from (30) that g.n/
Q < 4 . Recalling that a 2 cl! .E 0 / and
0
b 2 cl! .EnE / we conclude that g 
.a /  34 " while g 
.b /  4" and
therefore we have the inequality g 
.a / > g 
.b /.
Q then fix any n 2 E 0 . We still
Now if nQ d < 0 for any n 2 E and d 2 supp.n/
0
have (34) and hence g.n/
Q D .n;
Q Pn /  .n;
Q V nVn /  .n;
Q Pn / <  34 " by the
property (29).

442

2 Solutions of problems 001500

If n 2 EnE 0 then V 0 nVn D V 0 , so g.n/


Q D .n;
Q V 0 nVn /   4" . Recalling again
0
0
that a 2 cl! .E / and b 2 cl! .EnE / we obtain the inequality g 
.a / 
 34 " while g 
.b /   4" and therefore g 
.a / < g 
.b /.
Thus our induction step is also carried out in Case 2, so in all possible cases our
inductive procedure can be continued to obtain the family fu W !  < cg and a
set f.a ; b / W !  < cg  !  ! with the properties (a) and (b), i.e., Fact 11
is proved.
Returning to our solution again take the family fu W !  < cg and the set
f.a ; b / W !  < cg  !  ! from Fact 11 and consider the infinite compact
subspace K D u.!/ of the space Dc ; recall that u D fu W < cg and the family
fun W n 2 !g was constructed when we introduced the notation for Fact 11.
Assume that there exists a continuous linear surjection ' W Cp .K/ ! Cp .K/R.
By Fact 3 there exists a countable dense subset E of the space K such that the map
 D .E  i / ' E1 W Cp .EjX / ! Cp .EjX /  R is a continuous linear
surjection; here i.r/ D r for any r 2 R, i.e., i W R ! R is the identity. By Fact 4
we can consider that  is defined on the whole space RE and  W RE ! RE  R is
an adequate map.
Observe that u.!/ is a dense set of isolated points of K, so u.!/  E. By Fact 8,
we can find " > 0, an infinite subset M  ! and a family fPn W n 2 M g of pairwise
disjoint finite subsets of E such that
S
() u.M / \ . fPn W n 2 M g/ D ;;
() Pn  supp.u.n// and .u.n/; Pn / > " for any n 2 M ;
( ) either u.n/d > 0 for any d 2 Pn and n 2 M or u.n/d < 0 for any d 2 Pn and
n 2 M.
It is easy to find a countable set A  c such that !  A and A jE is injective; let
D D A .E/, A D uA and Sn D A .Pn / for any n 2 !. Since  D A jE W E ! D
is a bijection, the map   W RD ! RE defined by   .f / D f  for any f 2 RD ,
is a linear homeomorphism such that   jRD
 is a linear homeomorphism between
E
RD
and
R
in
the
norm
topology.
Take
a
point
a D [ E and let D C D D


C
fag; E D E fag. Letting .a/ D a we can consider that  W E C ! D C and
C
 W RE ! RE .
Given any f 2 RD , let '.f / D .f /  1 . It is easy to see that ' W RD !
DC
R
and ' D .  /1    , so ' is an adequate map.
It follows from the properties ()( ) that the pair .D; '/ belongs to the family
A of all admissible pairs and hence there is 2 !; c/ such that D D D ; ' D '
and A D A . The hypothesis of the statement (b) of Fact 11 is, evidently, satisfied
so, for the function g D '.p  1 / we have g 
.a / g 
.b /.
Recalling that  D uA D A u we convince ourselves that g D gA u;
by definition of ' we have g  D .p  1 /  1 A u D .p jE/ u.
It is clear that p jE 2 Cp .EjK/, so .p jE/ 2 Cp .EjK/  R and hence there
exists a function q 2 Cp .K/ such that qjE D .p jE/jE. This gives us the formula
q u
.a / q u
.b /.

2 Solutions of problems 001500

443

Let uQ W ! ! K be the continuous extension of u. It is a consequence of the


property (a) of Fact 11 that
(35) u
.a / D u
.b / for any ;  2 !; c/.
It is an easy exercise that u
D p uQ for any 2 !; c/ and hence we have the
equality uQ D fu
W 2 !; c/g which implies, together with (35), that uQ .a / D
uQ .b / for any 2 !; c/. In particular, uQ .a / D uQ .b / and hence .q uQ /.a / D
.q uQ /.b /. Since q u
j! D q u D .q uQ /j! and ! is dense in !, the maps
q u
and q uQ coincide (see Fact 0 of S.351) and hence q u
.a / D q u
.b /;
this contradiction shows that there is no linear continuous surjection of Cp .K/ onto
Cp .K/  R, i.e., our solution is complete.
V.401. Suppose that  is an infinite cardinal and X is a Lindelf -space such that
t .X /   and  .X /  . Prove that X has a -base of order  . In particular,
if X is a Lindelf -space with t .X / D  .X / D ! then X has a point-countable
-base. Deduce from this fact that any Lindelf -space X with .X /   has
a -base of order  . In particular, any first countable Lindelf -space has a
point-countable -base.
V
Solution. Given a family A of subsets of X by A we denote the family of all
finite intersections of the elements of A. Recall that ord.A/   if every point of X
belongs to at most -many elements of A.
If J is the set of isolated points of X then the family J D ffxg W x 2 J g is a
disjoint -base at every point of J , so it suffices to construct the promised -base
in the space Y D X nJ which has no isolated points. Therefore we can assume,
without loss of generality, that Y D X , i.e., X has no isolated points and hence we
can fix, for any x 2 X , a -base Bx at the point x such that jBx j   and x U for
any U 2 Bx .
Fix a countable family F of closed subsets of X which is a network with respect
to a compact cover C of the space X ; there is no loss of generality to assume that
F is closed under finite intersections. We will show by induction over all cardinals
 d.X / that
(1) if A  X is a set with jAj D then there is a family B   .X / such that
jBj   while B is a -base at every point of A and ord.B/  .
S
If   then the family B D fBx W x 2 Ag has cardinality at most , so (1) is
true. Now assume that we are given a cardinal  >  and (1) is proved for all < ;
fix a set B  X with jBj D . Let  D f C 1 W < g be the set of all successor
ordinals smaller than  and choose an enumeration fb W 2 g of the set B. To
start our inductive construction let G0 D ; and D0 D ;. Now assume that, for some
ordinal 2 .0; 
, we have constructed a family fG W < g and a collection
fD W < g with the following properties:
(2) G is a closed subset of X and D   .X / for any < ;
(3) jD j  jj ; ord.D /   and the family D is a -base at every point of G
for any < ;

444

2 Solutions of problems 001500

(4)
(5)
(6)
(7)
(8)

 < < implies G  G and D  D ;


S
S
if < is a limit ordinal then G D fG W  < g and D D fD W  < g;
V
if  < < ; F 2 F; U 2 D and U \ F ; then G \ .U \ F / ;;
if  < < then U \ G D ; for any U 2 D nD ;
if < and 2  then b 2 G .

Suppose first that is a successor


ordinal, say D  C1 and consider the family
V
H D fH W there exists U 2 D and F 2 F such that ; H D U \F  X nG g.
For any H 2 H pick a point xH 2 H ; then the set Z D .fb g[fxH W H 2 Hg/nG
has cardinality at most jHj  jj  < , so the induction hypothesis is applicable
to fix a family D0   .X / such that D0 is a -base at every point of Z while
jD0 j  jj ; ord.D0 /   and U \ G D ; for any U 2 D0 . Letting D D
D [ D0 and G D G [ Z we obtain a family fG W  g and a collection
fD W  g such that the properties (2)(8)
are fulfilled for all S
 .
S
If is a limit ordinal then let G D fG W < g and D D fD W < g.
It is immediate that the properties (2),(4)(8) and the second part of (3) are fulfilled
for all  . To see that the first part of (3) is also fulfilled consider two cases.
a) The cofinality of does not exceed . Then it follows from (4) that D is a
union of  -many families of order  , so ord.D /  . S
W < g.
b) If cf./ >  then it follows from (4) and t .X /   that G D fG T
Assume that there is a family U  D such that jU j D  C and P D U ;.
Pick a set C 2 C such that H D C \ P ;. We claim that
(9) the family UC D fU \ C \ G W U 2 U g is centered.
T
Indeed, fix a finite family V  U and let V D V; it follows from (4) that there
is < such that V  D . If F 2 F and C  F then F \ V  H ;, so it
follows from (6) that GC1 \ F \ V ; which implies that .G \ V / \ F ;
for any F 2 F with C  F . Since the family fF 2 F W C  F g is a network at
C , the closed set G \ V has to meet C . As a consequence, ; G \ V \ C 
T
fU \ C \ G W U 2 Vg which settles (9).
Since the family UC consists
T of compact subsets of X , it follows from (9) that
there is a point x 2 G \ . fU W U 2 U g/. Consequently, there is < such
that x 2 G . The property (3) shows that the family U 0 D U \ D has cardinality at
most , so the collection W D U nU 0  D nD is nonempty. Besides, U \ G D ;
for any U 2 W by (7). This contradiction with x 2 U \ G for any U 2 U shows
that ord.D /   and hence our inductive procedure gives us a family D for which
the properties (2)(8) are fulfilled and, in particular, B D D satisfies (1).
Next, apply (1) to a dense set A  X with jAj D d.X /; the respective family B
will be a -base of X of order at most . Finally, if X is a Lindelf -space with
.X /   then t .X /   and  .X /  , so X has a -base of order at most .
V.402. Given a space X and an infinite cardinal  suppose that  .X /   and
d.X /   C . Prove that X has a -base of order  . In particular, if  .X /  !
and d.X /  !1 then X has a point-countable -base.

2 Solutions of problems 001500

445

Solution. For any x 2 X choose a -base Bx at the pointSx such that jBx j  . If
X has a dense subspace D of cardinality   then B D fBx W x 2 Dg is easily
seen to be a -base of X of cardinality at most  and hence ord.B/  . Therefore
we can assume, without loss of generality, that d.X / D  C .
Fix a set D D fd W <  C g which is dense in X and let x0 D d0 . Proceeding
inductively, assume that <  C and we have chosen points fx W  < g  D in
such a way that
(1) if 0 < < then d0 2 fx W   g;
(2) x fx W  < g for any < .
It follows from d.X / > , that Y D fx W  < g cannot be dense in X , so the
set A D f <  C W d 2 X nY g is nonempty; let  D nnA and x D d . It is clear
that the condition (2) now holds for all  .
To check (1) suppose that 0 <  . If < then d0 2 fx W   g by the
induction hypothesis. If D and there exists  < with 0 <  then we have
d0 2 fx W   g  fx W   g, so (1) holds in this case too. Finally, if D
D 0 C 1 then d 2 fx W   0 g D fx W  < g for any  < 0 which shows
that either d0 2 fx W  < g or 0 D  and hence d0 D d D x 2 fx W   g,
so we proved that the conditions (1) and (2) are satisfied for all  .
This shows that our inductive procedure can be continued to construct a set E D
fx W  <  C g such that the properties (1) and (2) hold for all <  C . It follows
from (1) that E is dense in X . It is an easy consequence of (2) that the family
C D fB 2 Bx W B \ fx W  < g D S
;g is a -base at the point x for every
<  C . By density of E, the family C D fC W <  C g is a -base in X .
C
If ord.C/
x 2 X and a family C 0  C such that jC 0 j D  C
T 0  then fix a point
0
and x 2 C . For each C 2 C there is BC 2 Bx with BC  C ; since jBx j  ,
we can find a set B 2 Bx for which the family C 00 D fC 2 C 0 W BC D Bg has
cardinality  C . The set E being dense in X there S
is <  such that x 2 B and
therefore x 2 C for all C 2 C 00 . The family B D fBx W  g has cardinality
not exceeding , so there is C 2 C 00 nB. Recalling the definition of C we convince
ourselves that C 2 C for some  > and hence x C ; this contradiction shows
that C is a -base in X with ord.C/  .
V.403. Assuming CH prove that
(i) any Lindelf first countable space has a point-countable -base;
(ii) any space X with .X / D c.X / D ! has a point-countable -base;
(iii) if !1 is a caliber of X and .X /  ! then X is separable.
Solution. Given a set A and an infinite cardinal  let exp .A/ D fB W B  A and
jBj  g.
Fact 1. For any space X we have jX j  d.X /t.X/
jX j  d.X / .X/ holds for every space X .

.X/

. In particular, the inequality

Proof. Suppose that d.X / D and t .X / .X /  ; choose a dense subset D


of the space X such that jDj D . By .X /   we can choose, for any point

446

2 Solutions of problems 001500

x 2 XT
, a family Bx of open neighborhoods of x in X such that jBx j   and
fxg D fB W B 2 Bx g; it follows from t .X /   that we can find, for any B 2 Bx ,
a set SBx  D \ B such that jSBx j   and x 2 SBx .
Let '.x/ D fSBx W B 2 Bx g for every x 2 X ; then ' W X ! exp .exp .D//.
Since jexp .exp .D//j D .jDj / D  D  , it suffices to show that ' is an
injective map. Take any distinct points x; y 2 X and pick B 2 Bx such that y B.
Then y SBx and hence SBx '.y/ because all elements of '.y/ contain y in
their closure. Therefore '.y/ '.x/ for any distinct x; y 2 X , so the map ' is
injective and hence jX j  jexp .exp .D//j D  , i.e., we established that jX j 
d.X /t.X/ .X/ for any space X ; observe that t .X / .X /  .X / and hence also
jX j  d.X / .X/ , so Fact 1 is proved.
Fact 2. If X is an arbitrary space then jX j  2l.X/
the inequality jX j  2l.X/ .X/ for any space X .

.X/t.X/

. In particular, we have

Proof. Let T
D l.X / .X / t .X / and fix a family Bx  .x; X / such that jBx j  
and fxg D
S Bx for each x 2 X . For any set A  X we will need the family
B.A/ D fBx W x 2 Ag; take any point x0 2 X and let F0 D fx0 g. Suppose
that <  C and we have a family fF W < g of closed subsets of X with the
following properties:
(1) jF j  2 for each < ;
(2) F0  F whenever 0  < ; S
S
(3) if < andSU is a subfamily of B. < F / such that jU j   and X n U
; then F n U ;.
S
Consider the
a subfamily of B. < F / such that
S collection A D fU W U is S
jU j   and SU X g. Observe first that j < F j   2 D 2 and therefore
the family B. < F / has cardinality at most 2  D 2S
. As a consequence,
jAj  .2 / D 2 . For any US
2 A choose a point x.U / 2 X n U and consider the
set P D fx.U / W U 2 Ag [ . < F /.
If F D P then d.F /  jP j  2 C 2 D 2 , so it follows from Fact 1
and t .X / .X /   that jF j  jP j  .2 / D 2 . It is straightforward that the
conditions (1)(3) are now satisfied for all  and hence our inductive procedure
can be continued to obtain a family fF W <  C g of closed subsets of X for which
C
the properties
S (1)(3) hold for all <  .
If F D fF W <  C g then jF j  2  C D 2 , so it suffices to show that
F D X . Note first that it follows from t .X /   that the set F is closed in X . If
F X then fix a point a 2 X nF and choose, for any x 2 F a set Vx 2 Bx such
that a Vx . The family V D fVx W x 2 F g is an open cover of F , so it follows S
from
l.F /  l.X /   that we can find a family U  V such that jU j   and S
F  U.
C
It is easy S
to see that there exists
S an ordinal <  such that U  B. < F /;
sinceSa
U , the set X n U is nonempty,
so
we can apply (3) to see that
S
F n. U / ;. However, F  F  U ; this contradiction shows that F D X
and hence we checked that jX j  2l.X/t.X/ .X/ for any space X ; finally it follows
from t .X / .X /  .X / that jX j  2l.X/ .X/ and hence Fact 2 is proved.

2 Solutions of problems 001500

447

Fact 3. For any space X we have the inequality jX j  2c.X/ .X/ .


Proof. Apply Fact 1 to see that jX j  d.X / .X/  w.X / .X/ ; we also have the
inequalities w.X /   .X /c.X/  .X /c.X/  .2 .X/ /c.X/ D 2 .X/c.X/ (see Fact 4
of U.083) and hence jX j  .2 .X/c.X/ / .X/ D 2 .X/c.X/ , i.e., Fact 3 is proved.
Returning to our solution observe that if X is a Lindelf first countable space
then jX j  2l.X/ .X/ D 2! by Fact 2. Recalling that we are also assuming CH, we
conclude that d.X /  jX j  !1 . If .X / D c.X / D ! then jX j  2 .X/c.X/  2!
(see Fact 3), so again d.X /  jX j  !1 . If !1 is a caliber of X then c.X /  !, so
in cases (i)(iii) we have a space X such that .X /  ! and d.X /  !1 . Therefore
we can apply Problem 402 to see that X has a point-countable -base; this settles
(i) and (ii). In the case (iii) the cardinal !1 is a caliber of X , so the relevant pointcountable -base of X has to be countable and hence d.X /  w.X / D ! which
shows that our solution is complete.
V.404. Give an example of a first countable space which has no point-countable
-base.
Solution. Let !i be the i -th uncountable ordinal with its usual well-ordering
and
Q
the discrete topology for any n 2 N and consider the set X D f!i W i 2 Ng.
Given x; y 2 X we say that x  y if x.n/  y.n/ for all n 2 N. We will also
need the set Rx D fy 2 X W x  yg for every x 2 X . If x 2 X and n 2 N
then Un .x/ D fy 2 X W y.i / D x.i / for all i  ng. It is evident that the family
S D fRx W x 2 X g [ fUn .x/ W x 2 X; n 2 Ng covers X , so there exists a unique
topology on the set X such that S is a subbase of .X; /; from this moment on we
will consider that X is the space .X; /.
It is easy to see that the family fUn .x/ W x 2 X; n 2 Ng is a base for the product
topology p on the set X . Since .X; p / is a metrizable space (being a countable
product of discrete and hence metrizable spaces), it follows from p  that X is a
Hausdorff space.
Let On .x/ D Un .x/ \ Rx for any x 2 X and n 2 N. We claim that
(1) for any x 2 X the family fOn .x/ W n 2 Ng is a base of X at the point x;
therefore the space X is first countable.
To prove (1) note first that y 2 Un .x/ implies that Un .y/  Un .x/; besides,
y 2 Rx implies Ry  Rx . Therefore On .y/  On .x/ whenever y 2 On .x/.
Now assume that x 2 U 2 . Since the family S is a subbase of the space X , we
can find points
T x1 ; : : : ; xk 2 X , a finite
T set A  X and n1 ; : : : ; nk 2 N such that
x 2 W D . fUni .xi / W i  kg/ \ . fRy W y 2 Ag/  U . If n D maxfni W i  kg
then Un .x/  Uni .x/  Uni .xi / for all i  k; besides, it follows from x 2 Ry that
Rx  Ry for all y 2 A. An immediate consequence is that x 2 On .x/  W  U
and hence (1) is proved.
Given x 2 X and n 2 N take any point y 2 X nOn .x/; if y Un .x/ then
Un .y/ \ Un .x/ D ; and hence Un .y/ \ On .x/ D ;. If y 2 Un .x/ then y Rx and

448

2 Solutions of problems 001500

hence there exists k > n such that y.k/ < x.k/. It is clear that Uk .y/ \ Rx D ; and
hence Uk .y/\On .x/ D ;. This shows that every y 2 X nOn .x/ has a neighborhood
contained in X nOn .x/, i.e., the set X nOn .x/ is open and hence
(2) the set On .x/ is clopen in X for any x 2 X and n 2 N.
An immediate consequence of (1) and (2) is that the space X is zero-dimensional
and hence Tychonoff (see Fact 1 of S.232). Let !! D supf!i W i 2 Ng; our next step
is to prove S
that d.X / > !! . Assume that D is a dense subset of X . If jDj  !!
then D D fDi W i 2 Ng where jDi j  !i for any i 2 N. For each i 2 N the
cofinality of !iC1 is strictly greater than the cardinality of Di , so there is an ordinal
i 2 !iC1 such that x.i C 1/ < i for any x 2 Di . Let x.1/ D 0 and x.i C 1/ D i
for any i 2 N.
Take any d 2 D; then d 2 Di for some i 2 N and hence d.i C 1/ < i D
x.i C 1/. This shows that d Rx for any d 2 D, i.e., D \ Rx D ;. Since Rx is a
nonempty open subset of X , this contradicts density of D and proves that
(3) w.X /  d.X / > !! .
Finally assume that X has a point-countable -base B and let  D .!! /C , i.e., 
is the cardinal that follows !! . It follows from (1) that we can assume, without loss
of generality, that
S every element of B is the set On .x/ for some x 2 X and n 2 N.
Therefore B D fBn W n 2 Ng where Bn D fOn .x/ W x 2 An g for every n 2 N.
The property (3) implies that jBj  ; since the cardinal  is regular, there is
n 2 N for which jAn j D . Since  > !n D j!1  : : :  !n j, we can apply regularity
of  once more to find a set A  An and a sequence .1 ; : : : ; n / 2 !1  : : :  !n
such that jAj D  and x.i / D i for all x 2 A.
Choose a set A0  A with jA0 j D !1 and fix any i > n. Since the cofinality of !i
is strictly greater than !1 , we can choose an ordinal i 2 !i such that x.i / < i for
all x 2 A0 . Define a point y 2 X as follows: y.i / D i for all i  n and y.i / D i
whenever i > n.TIt is immediate that y 2 Rx and hence y 2 On .x/ for any x 2 A0 .
Therefore y 2 fOn .x/ W x 2 A0 g, so the order of the family B at the point y is
at least !1 , i.e., B is not point-countable. This contradiction shows that X is a first
countable space which has no point-countable -base.
V.405. Let X be a space for which we can find a family of sets fAm W m 2 !g and
a sequence fkm W m 2 !g  Nnf1g such that supfjAm j W m 2 !g D jX j while
Am  X km n km .X / and Am is concentrated around km .X / for every m 2 !.
Prove that the space Cp .X / has a point-countable -base. In particular, the space
Cp .X / has a point-countable -base if there is a set A with jAj D jX j such that
either A  X n n n .X / and A is concentrated around n .X / or A  X and A is
concentrated around some point of X .
Solution. Given a space Z call a family U   .Z/ a -base of a family V 
 .Z/ if any element of V contains some U 2 U . It is evident that if V is a -base
in Z and U is a -base of V then U is also a -base in Z.
Denote by I the family of all nontrivial open intervals of R with rational
endpoints. The collection Q D f.Q0 ; : : : ; Qn1 / W n 2 Nnf1g; Qi 2 I for any

2 Solutions of problems 001500

449

i < n and the family fQ0 ; : : : ; Qn1 g is disjointg is countable. If x D


.x0 ; : : : ; xn1 / 2 X n and Q D .Q0 ; : : : ; Qn1 / 2 Q then let x; Q
D ff 2
Cp .X / W f .xi / 2 Qi for all i < ng. It is straightforward that the family
B D fx; Q
W there is a number n 2 Nnf1g such that x 2 X n n n .X / and
Q 2 Q \ I n g is a -base in Cp .X /. For any Q D .QS
0 ; : : : ; Qn1 / 2 Q let
BQ D fx; Q
W x 2 X n n n .X /g; it is evident that B D fBQ W Q 2 Qg. The
family Q being countable it suffices to find a point-countable -base for every BQ ,
so fix any Q D .Q0 ; : : : ; Qn1 / 2 Q.
Our assumptions about the sequence fAm W m 2 !g make it possible to choose a
family fYm W m 2 !g of subsets of X n n n .XS
/ and a sequence f'm W m 2 !g such
that every
'
W
Y
!
A
is
an
injection
and
fYm W m 2 !g D X n n n .X /. Then
m
S mm m
m
BQ D fBQ W m 2 !g where BQ D fx; Q
W x 2 Ym g for any m 2 !. Thus it
m
suffices to find a point-countable -base for every BQ
, so fix m 2 !.
m
We still have to split BQ into finitely many subfamilies. Given any point x 2 Ym
let y D 'm .x/ and Ax D fi < n W x.i / 2 fy.0/; : : : ; y.km 1/gg. For any set A  n
m
consider the family BQ
.A/
S Dmfx; Q
W x 2 Ym and Ax D Ag; it is evident that we
m
have the equality BQ D fBQ
.A/ W A  ng, so it suffices to find a point-countable
m
-base for every BQ
.A/. To do the last splitting, for any E  km consider the
m
m
family BQ
.A; E/ D fx; Q
2S
BQ
.A/ W fx.i / W i 2 Ax g D f'm .x/.j / W j 2 Egg.
m
m
It is immediate that BQ .A/ D fBQ
.A; E/ W E  km g, so it suffices to construct a
m
point-countable -base for every family BQ
.A; E/.
To do this, for r D km  jEj take an r-tuple S D .S0 ; : : : ; Sr1 / 2 Q such that
the family fS 0 ; : : : ; S r g[fQi W i < ng is disjoint and fix a bijection b W km nE ! r.
m
Given an arbitrary point x D .x0 ; : : : ; xn1 / 2 Ym such that x; Q
2 BQ
.A; E/ and
'm .x/ D .y0 ; : : : ; ykm 1 /, for any i 2 E there is a unique j.i / 2 A such that
xj.i/ D yi . Let H.x/ D ff 2 Cp .X / W f .xi / 2 Qi for any i < n and f .yi / 2 Sb.i/
if i 2 km nEg. It is evident that H.x/  x; Q
is a nonempty open set, so the family
m
m
.A; E/g is a -base for BQ
.A; E/.
H D fH.x/ W x; Q
2 BQ
m
Now, take any function f 2 Cp .X /. If x; Q
2 BQ
.A; E/ and f 2 H.x/ then
Q 1
Q 1
'm .x/ belongs to W .x/ D ff .Qj.i/ / W i 2 Eg  ff .Sb.i/ / W i 2 km nEg.
Since the family fQj.i/ W i 2 Eg [ fS b.i/ W i 2 km nEg is disjoint, we conclude that
W .x/ \ km .X / D ;. The set Am is concentrated around the km -diagonal km .X /,
m
so jW .x/ \ Am j  !. The set P D fx 2 Ym W x; Q
2 BQ
.A; E/ and f 2 H.x/g
is contained in the set R D fx 2 Ym W 'm .x/ 2 W .x/g. The set R is countable
because 'm .Ym /  Am while W .x/ \ Am is countable and 'm is injective. Thus P
is also countable, i.e., the family H is point-countable at any f 2 Cp .X /. Therefore
Cp .X / indeed, has a point-countable -base.
Observe that if there is n 2 Nnf1g and a set A  X n n n .X / such that jAj D jX j
and A is concentrated around n .X / then we can let Am D A and km D n for all
m 2 ! obtaining thus a sequence fAm W m 2 !g which guarantees that Cp .X / has
a point-countable -base.
Finally, if a 2 X and a set A  X with jAj D jX j is concentrated around the
point a then the set A0 D .Anfag/  fag  X 2 n 2 .X / has the same cardinality as
X and A0 is concentrated around the diagonal 2 .X /, i.e., in this case Cp .X / also
has a point-countable -base.

450

2 Solutions of problems 001500

V.406. Prove that


(a) Cp ./ has a point-countable -base whenever is an ordinal with its order
topology;
(b) Cp .AD.X // has a point-countable -base for any countably compact space
X . Here AD.X / is the Alexandroff double of X .
Solution. (a) Let .; 
D f W <    g for any ordinals ;  2 such
that <  . If is a countable ordinal then Cp ./ has a countable base, so we
can assume, without loss of generality, that jj > !. Let A D f.; C 1/ W
C 1 < g  .  /n 2 ./. It is clear that jAj D jj; let us prove that A is
concentrated around the set D 2 ./.
Assume toward a contradiction that A is not concentrated around and fix a set
U 2 . ;  / for which there exists an uncountable set B  such that
(1) f.; C 1/ W 2 Bg  AnU .
Let 0 D nnB and nC1 D nn.Bnf0 ; : : : ; n g/ for any n 2 !. It is immediate
from the construction of the sequence S D fn W n 2 !g that the set Pn D f 2 B W
< n g is finite for all n 2 !. Since B is uncountable, there exists  2 B such that
n <  for all n 2 !. Consequently, the ordinal  D supfn W n 2 !g belongs to .
Since .; / 2 U , there exists 0 <  such that .0 ; 
 .0 ; 
 U . Take
n 2 ! such that n > 0 ; then .n ; n C1/ 2 .0 ; 
.0 ; 
 U ; this contradicts
(1) and proves that A is concentrated around 2 ./, so we can apply Problem 405
to conclude that Cp ./ has a point-countable -base.
(b) Recall that AD.X / has the underlying set X f0; 1g and the topology generated
by the family ff.x; 1/g W x 2 X g [ f.U  f0; 1g/nK W U 2 .X / and K 
X  f1g is a finite setg. If X is finite then AD.X / is also finite and hence
there is nothing to prove, so we assume that X is infinite; denote by the set
2 .AD.X //.
For every x 2 X consider the points ax D .x; 0/ and bx D .x; 1/ of the space
AD.X / and let A D f.ax ; bx / W x 2 X g  .AD.X /  AD.X //n . It is clear that
jAj D jX j D jAD.X /j; take any open neighborhood U of the diagonal in the
space AD.X /  AD.X /. If AnU is infinite then there exists an infinite set B  X
such that A0 D f.ax ; bx / W x 2 Bg  AnU . It follows from countable compactness
of X that some z 2 X is an accumulation point for B; then w D .az ; az / 2 and
hence w 2 U , so there exists a set O 2 .z; X / and a finite K  X such that, for
the set W D .O  f0; 1g/n.K  f1g/ we have w 2 W  W  U .
Since z is an accumulation point of B, we can pick a point y 2 .OnK/\B. Then
.ay ; by / 2 W  W and hence .ay ; by / 2 U \ A0 which is a contradiction. Thus the
set AnU is finite for any U 2 . ; AD.X /  AD.X // and hence A is concentrated
around D 2 .AD.X //. Finally, apply Problem 405 to conclude that the space
Cp .AD.X // has a point-countable -base.
V.407. Prove that the following conditions are equivalent for any infinite space X
with l  .X /  !:

2 Solutions of problems 001500

451

(i) Cp .X / has a point-countable -base;


(ii) there is a family of sets fAm W m 2 !g and a sequence fkm W m 2 !g  Nnf1g
such that supfjAm j W m 2 !g D jX j while Am  X km n km .X / and Am is
concentrated around km .X / for every m 2 !.
Solution. By Problem 405, it suffices to prove the implication (i)H)(ii). Denote
by I the family of all nontrivial open intervals of R with rational endpoints. The
collection Q D f.Q0 ; : : : ; Qn1 / W n 2 N; Qi 2 I for any i < n and the family
fQ0 ; : : : ; Qn1 g is disjointg is countable. For any point x D .x0 ; : : : ; xn1 / 2
X n n n .X / and Q D .Q0 ; : : : ; Qn1 / 2 Q let x; Q
D ff 2 Cp .X / W f .xi / 2 Qi
for all i < ng.
Fix a point-countable -base B   .Cp .X //. Making the elements of B smaller
if necessary, we can assume that, for each B 2 B there is n 2 Nnf1g, a point
x 2 X n n n .X / and Q D .Q0 ; : : : ; Qn1 / 2 Q such that B D x; Q
. Choose an
enumeration fQm W m 2 !g of the family Q; we have Qm D .Q0m ; : : : ; Qkmm 1 / for
m
every m 2 !. Consider the set Am D fx 2 X km n km .X / W x;
S Q
2 Bg and let
m
Bm D fx; Q
W x 2 Am g for any m 2 !; it is clear that B D fBm W m 2 !g.
Since w.Cp .X // D w.Cp .X // D jX j (see TFS-169 and Fact 2 of T.187), we
have jBj  jX j and hence jBj D jX j because there are at most jX j-many sets of
the form x; Q
where x 2 X n and Q 2 Q \ I n for some n 2 Nnf1g. Therefore
jX j D jBj D supfjBm j W m 2 !g D supfjAm j W m 2 !g.
Fix a number m 2 Nnf1g and take a set U 2 . km .X /; X km /. For any point
x D .x0 ; : : : ; xkm 1 / 2 F D X km nU choose a function fx 2 Cp .X / such that
fx .xi / 2 Qim for all i < km . Then x 2 Wx D fx1 .Q1m /  : : :  fx1 .Qkmm 1 /.
Besides, fx 2 y; Qm
if and only if y 2 Wx ; the family B being point-countable,
the set fy 2 Am W y 2 Wx g is countable for any x 2 F .
km
The
H  F such that
Sspace F D X nU is Lindelf, so there is a countable S
F  fWx W x 2 H g. Thus the set Am nU D Am \ F  .Am \ . fWx W x 2 H g//
is countable because Am \ Wx is countable for any point x 2 H . Therefore the set
Am  X km n km .X / is concentrated around km .X / for any m 2 !.
V.408. Given a space X suppose that the cardinality of X is regular and uncountable while l  .X / D !, i.e., all finite powers of X are Lindelf. Prove that the space
Cp .X / has a point-countable -base if and only if there exists a natural number
n > 1 such that some set A  X n n n .X / is concentrated around n .X / and
jAj D jX j.
Solution. We have sufficiency by Problem 405, so assume that the space Cp .X /
has a point-countable -base. By Problem 407, we can find a family fAm W m 2 !g
and a sequence fkm W m 2 !g  Nnf1g such that supfjAm j W m 2 !g D jX j while
the set Am  X km n km .X / is concentrated around km .X / for each m 2 !. Since
jX j is a regular uncountable cardinal, there exists m 2 ! such that jAm j D jX j.
Letting A D Am and n D km we conclude the proof of necessity.
V.409. Given a metrizable space X prove that Cp .X / has a point-countable -base
if and only if X is countable.

452

2 Solutions of problems 001500

Solution. If the space X is countable then Cp .X / even has a countable base, so


we have sufficiency. To prove necessity assume that X is a metrizable space and
Cp .X / has a point-countable -base B. By SFFS-285, the cardinal !1 is a caliber
of Cp .X /, so B is countable and hence we can apply Fact 2 of T.187 to conclude
that jX j D w.Cp .X // D !.
V.410. Suppose that X is an infinite space with l  .X / D !. Prove that if Cp .X /
has a point-countable -base then jX j D .X /. Here .X / D nnf W 2 .X / is a
G -subset of X  X g is the diagonal number of the space X . Deduce from this fact
that if X is compact and Cp .X / has a point-countable -base then w.X / D jX j.
Solution. It is evident that .X /  jX j, so assume that .X / D  < jX j. We can
find a family of sets fAm W m 2 !g and a sequence fkm W m 2 !g  Nnf1g such
that supfjAm j W m 2 !g D jX j while Am  X km n km .X / and Am is concentrated
around km .X / for every m 2 ! (see Problem 407). If jAm j   for all m 2 ! then
jX j D supfjAm j W m 2 !g   which is a contradiction. Therefore there is m 2 !
for which jAkm j   C ; let n D km and A D Akm .
n
It follows from Fact 2 of T.078 that n .X / is a G -subset
T of the space X .
n
Choose a family U  . n .X /; X / such that jU j   and U D n .X /. Since
A is concentrated aroundS n .X /, the set AnU is countable for any U 2 U . It follows
from the equality A D fAnU W U 2 U g that jAj   supfjAnU j W U 2 U g 
 ! D ; this contradiction with jAj   C shows that jX j  .X / and hence
jX j D .X /.
Finally observe that if X is compact then l  .X / D ! and w.X / D .X / (see
SFFS-091), so if Cp .X / has a point-countable -base then w.X / D .X / D jX j
as promised.
V.411. Prove that if K is a scattered Corson compact space then Cp .K/ has a
point-countable -base.
Solution. If K is countable then Cp .K/ even has a countable base, so we assume
from now on that the space K is uncountable. Let  D w.K/; since K is scattered,
we have hl.K/  jKj (see SFFS-005 and SFFS-006). Besides, hl.K/  w.K/ D
nw.K/  jKj (see Fact 4 of S.307) and hence jKj D .
Apply CFS-188 to see that the diagonal D f.x; x/ W x 2 Kg  K  K
of the space K is a W -set in K  K. By CFS-186 the space P D .K  K/n
is metacompact, so there exists a point-finite open cover U of the space P such
that U \ D ; for every U 2 U (the bar denotes the closure in K S
K). Since
hl.K K/  , we can choose a subfamily V  U such that jVj   and T
V D P.
We also have .K/ D w.K/ D  by SFFS-091, so it follows from D f.K 
K/nU W U 2 Vg that the cardinality of V cannot be less than  and hence jVj D .
For any z 2 P let Vz D fV 2 V W z 2 V g and take an arbitrary point z0 2 P .
Suppose that <  and we have a set fz W < g such that
S S
(1) z f Vz W <  g for any  < .
S
Since Vz is finite for every < , the family W D fVz W < g has
cardinality at most ! jj D maxfjj; !g < . Therefore W cannot cover P because

2 Solutions of problems 001500

453

T
otherwise D f.K  K/nW W W 2 Wg and hence
. ; K  K/  jWj < 
S
which is a contradiction. Pick any point z 2 P n. W/; it is straightforward that
(1) now holds for all   and hence our inductive procedure can be continued to
construct
S a set D D fz W < g  P such that (1) holds for all < . Since
z 2 Vz for any < , it follows from (1) that z0 z whenever 0 and
hence jDj D  D jKj.
If z 2 P then there is U 2 V with z 2 U . If z 2 U then U 2 Vz and hence
(1) shows that z U for all > . Now, if < then z Vz , so U Vz ,
i.e., z U . Therefore every z 2 P has a neighborhood which contains at most one
point of D, so D is closed and discrete in P .
Now take any set W 2 . ; K  K/. The set L D .K  K/nW D P nW  P
is compact. Therefore D \ L D DnW is compact being closed in L; however,
D \ L is a discrete space, so it must be finite. This proves that DnW is finite
for any W 2 . ; K  K/ and hence D is concentrated around . Finally, apply
Problem 405 to conclude that Cp .K/ has a point-countable -base.
V.412. Prove that if Cp .X / embeds in a -product of first countable spaces then it
has a point-countable -base.
Q
Solution. Suppose that Nt is a space for every t 2 T and N D t2T Nt . Given
a point a 2 N let .N; a/ D fx 2 N W jft 2 T W x.t / a.t /gj  !g. If
x 2 .N; a/ then supp.x/ D ft 2 T W x.t / a.t /g. The spaces with a countable
network are called cosmic.
Q
Fact 1. Assume that nw.Nt / D .Nt / D ! for all t 2 T , and N D t2T Nt .
Then, for any a 2 N , every subspace of .N; a/ has a point-countable -base.
Q
Proof. For each S  T consider the set NS D t2S Nt and the natural projection
S W N ! NS ; let aS D S .a/. It is easy to see that S ..N; a// D .NS ; aS /
for any S  T . For every Y  .N; a/ let .Y / be the minimal cardinality of a
set S  T such that Y embeds in .NS ; aS /. Observe first that
(1) if Y  .N; a/ and .Y /  ! then Y has a countable -base.
Indeed, .Y /  ! implies that Y embeds in NS for some countable S  T . The
space NS is cosmic and first countable and hence so is Y . Take any countable dense
D  Y and fix a countable local baseSBd in the space Y at every point d 2 D. It is
an easy exercise that the family B D d 2D Bd is a countable -base of Y , so (1) is
proved. Our next step is to show that
(2) if Y  .N; a/ and .Y / D .V / for any nonempty open set V of the space
Y , then Y has a point-countable -base.
If (2) is not true then the family Y D fY  .N; a/ W .Y / D .V / for
any V 2  .Y / and Y does not have a point-countable -baseg is nonempty. Let
 D nnf.Y / W Y 2 Yg and choose a set Y 2 Y with .Y / D . It follows from
(1) that  > !. We can assume, without loss of generality, that Y  .NS ; aS / for
some S  T with jS j D . Choose a faithful enumeration fs W < g of the set

454

2 Solutions of problems 001500

S ; we will need the set S D fs W  g for any < . Let p .y/ D yjS for
any y 2 Y and < ; then p W Y ! Y D p .Y / is the restriction to the set Y of
the relevant natural projection.
Given any ordinal < , we have Y  .NS ; aS / and jS j < , so our
choice of  guarantees that Y has a point-countable -base B0 ; consider the set
F D fx 2 Y W x.s / D a.s /g and the family B D fB 2 B0 W B \ F D ;g. The
1

family
SE D fp .B/ W B 2 B g  .Y / is point-countable for each < ; let
E D fE W < g.
Take any U 2  .Y /; there exist 1 ; : : : ; n 2  and sets O1 ; : : : ; On such
that Oi 2 .Nsi / for all i  n while the set U 0 D fy 2 Y W y.si / 2 Oi
for each i  ng is nonempty and contained in U . Let D maxf1 ; : : : ; n g; if
y.s / D a.s / for any > and y 2 U 0 then U 0 embeds in .NS ; aS / and
hence .U 0 /  jS j <  which is a contradiction.
Therefore there exists an ordinal > such that y.s / a.s / for some point
y 2 U 0 . The set V D p .U 0 / is open in Y and U 0 D p1 .V / (see Fact 1 of S.298).
Since p .y/ 2 V nF , the open set V nF is nonempty and hence there is B 2 B0
such that B  V nF , i.e., B 2 B . Since also B 0 D p1 .B/  p1 .V / D U 0  U
and B 0 2 E, we proved that every nonempty open subset of Y contains an element
of E, i.e., the family E is a -base of Y .
T
Now assume that there exists a family E 0  E such that jE 0 j D !1 and E 0 ;.
Since E is point-countable, the family E 0 \ E is countable for every < . As a
consequence, there existsTan uncountable set A   such that E 0 \ E ; for any
2 A. Pick
point y 2 E 0 and an element B 2 B for any 2 A in such a way
T a 1
that y 2 fp .B / W 2 Ag.
It follows from the equality B \ F D ; that p .y/ F and hence y./
a./ for any 2 A. Thus A  supp.y/, so supp.y/ is uncountable which is a
contradiction with y 2 .NS ; aS /. Therefore the family E is a point-countable
-base of Y ; this contradiction with the choice of Y shows that (2) is proved.
Now take an arbitrary subspace Y of the space .N; a/ and consider the family
B D fB 2  .Y / W .B/ D .B 0 / for any B 0 2  .Y / with B 0  Bg. If
U 2  .Y / then let  D nnf.U 0 / W U 0 2  .Y / and U 0  U g and choose a set
B 2  .Y / such that B  U and .B/ D . It is immediate that B 2 B, so the
family B is S
a -base of Y . Take a maximal disjoint subfamily B 0 of the family B;
then the set B 0 is dense in the space Y . The property (2) shows that, for every set
B 2 B 0 we can
S choose a point-countable -base CB for the space B. It is easy to
see that C D fCB W B 2 B 0 g is a point-countable -base of Y , so Fact 1 is proved.
Returning to our solution assume
Q that Mt is a first countable space for any t 2 T
and we have a point a 2 M D t2T Mt such that Cp .X /  .M; a/. Let pt W
M ! Mt be the natural projection and consider the set Nt0 D pt .Cp .X // for
any t 2 T . It follows from TFS-299 that Nt0 is a cosmic
Qspace and hence Nt D
Nt0 [ fa.t /g is cosmic as well for any t 2 T ; let N D t2T Nt . It follows from
Cp .X /  .N; a/ and Fact 1 that Cp .X / has a point-countable -base, so our
solution is complete.

2 Solutions of problems 001500

455

V.413. Say that a space X is P -favorable (for the point-open game) if the player
P has a winning strategy in the point-open game on X . Prove that
(i)
(ii)
(iii)
(iv)
(v)

any countable space is P -favorable;


a continuous image of a P -favorable space is P -favorable;
the countable union of P -favorable space is P -favorable;
any nonempty closed subspace of a P -favorable space is P -favorable;
any nonempty Lindelf scattered space is P -favorable.

Solution. (i) Suppose that X is a countable space and choose an enumeration


fxn W n 2 !g of the set X . Let s.;/ D x0 and, if n 2 ! and moves
x0 ; U0 ; : : : ; xn ; Un are made then let s.U0 ; : : : ;S
Un / D xnC1 . If fxn ; Un W n 2 !g
is a play where P applies the strategy s then n2! Un  fxn W n 2 !g D X ,
so s is a winning strategy for the player P , i.e., X is P -favorable.
(ii) Assume that s is a winning strategy for the player P on a space X and f W
X ! Y is a continuous onto map. If x0 D s.;/ then let y0 D f .x0 / and
t .;/ D y0 . Suppose that moves y0 ; V0 ; : : : ; yn ; Vn are made in the point-open
game on Y and we have points x0 ; : : : ; xn 2 X such that f .xi / D yi for all
i  n and x0 ; f 1 .V0 /; : : : ; xn ; f 1 .Vn / is an initial segment of a play on X
where P applies the strategy s. If xnC1 D s.f 1 .V0 /; : : : ; f 1 .Vn // then let
ynC1 D f .xnC1 / and t .V0 ; : : : ; Vn / D ynC1 . This defines a strategy t on the
space Y .
If fyn ; Vn W n 2 !g is a play in which P applies t then there exists a sequence
fxn W n 2 !g  X such that f .xn / D yn for each n 2 ! and fxn ; f 1 .Vn / W n 2 !g
is a play onSX in which P applies the strategy
S s. Since s is a winning strategy, we
have X D n2! f 1 .Vn / and hence Y D n2! Vn , so t is a winning strategy for
the player P on Y , i.e., Y is P -favorable.
S
(iii) Assume that X D n2! Xn and there exists a winning strategy sn on the space
Xn for every n 2 !. Choose
a disjoint family fAn W n 2 !g of infinite subsets
S
of ! such that ! D n2! An . If m 2 ! and S D fx0 ; U0 ; : : : ; xn ; Un g is an
initial segment of a play on X then let S jXm D fxi0 ; Ui0 \ Xm ; : : : ; xik ; Uik \
Xm g where i0 < : : : < ik and fxi0 ; : : : ; xik g D fx0 ; : : : ; xn g \ Xm . The set
S jXm will be called the restriction of the segment S to Xm .
There is a unique m 2 ! such that 0 2 Am ; let x0 D sm .;/ and s.;/ D x0 .
Proceeding inductively assume that moves x0 ; U0 ; : : : ; xn ; Un are made in such a
way that, for any m 2 !, if S D fx0 ; U0 ; : : : ; xn ; Un g then the set S jXm is an initial
segment (maybe empty) of a play on Xm in which P applies the strategy sm . There is
a unique m 2 ! such that nC1 2 Am ; if S jXm D fxi0 ; Ui0 \Xm ; : : : ; xik ; Uik \Xm g
then the strategy sm is applicable to the segment S jXm by our induction hypothesis.
Let xnC1 D sm .Ui0 \ Xm ; : : : ; Uik \ Xm / and s.U0 ; : : : ; Un / D xnC1 . This gives a
strategy s for the player P on the space X .
Now, assume that fxn ; Un W n 2 !g is a play on X where P applies the strategy
s and fix an arbitrary m 2 !. There is a unique enumeration fij W j 2 !g of
the set Am such that ij < ik whenever j < k. By our definition of the strategy
s, the family fxij ; Uij \ Xm W j 2 !g is a play on Xm in which P applies the

456

2 Solutions of problems 001500

S
S
strategy
sm . Therefore
Xm D j 2! Uij \ Xm D n2Am Un S
\ Xm and hence
S
S
S Xm 
U

U
for
any
m
2
!.
Consequently,
X
D
X

n
n
m
n2Am S
n2!
m2!
n2! Un ,
so X D n2! Un which shows that the strategy s is winning for P , i.e., X is P favorable.
(iv) Suppose that X is P -favorable and F is a nonempty closed subspace of X . Fix
a winning strategy s for the player P on the space X . If we have an n-th move
fxn ; Un g of a play on X say that the move Un of the player O is adequate if
xn 2 X nF implies Un D X nF . In this item we will only consider the initial
segments of a play on X where P applies the strategy s and all moves of O
are adequate. A play (or an initial segment of a play) is called adequate if all
moves of O are adequate. Observe that
(1) if we have an adequate play fxn ; Un W n 2 !g and F 6 U0 [ : : : [ Uk for
some k 2 ! then there exists m > k such that xm 2 F .
S
Indeed, the strategy sS
is winning, so n2! Un D X ; if xn F then Un D X nF
for all n > k and hence n2! Un D U0 [ : : : [ Uk X which is a contradiction.
Apply the property (1) to see that there is a uniquely determined m0 2 ! and an
adequate segment fx0 ; U0 ; : : : ; xm0 1 ; Um0 1 g such that xi F for all i < m0 and
xm0 D s.U0 ; : : : ; Um0 1 / 2 F . Let y0 D xm0 and t .;/ D y0 .
Proceeding inductively assume that fy0 ; V0 ; : : : ; yk ; Vk g is an initial segment of
a play on F such that there exist natural numbers m0 < : : : < mk and an adequate
initial segment fx0 ; U0 ; : : : ; xmk ; Umk g such that yi D xmi and Vi D Umi \ F for all
i  k while fx0 ; : : : ; xmk gnfy0 ; : : : ; yk g  X nF .
If F  Wk D U0 [ : : : [ Umk then V0 [ : : : [ Vk D F , so take any point
ykC1 2 F and let t .V0 ; : : : ; Vk / D ykC1 . If F nWk ; then apply the property
(1) to see that there exists a uniquely determined mkC1 2 ! and adequate initial
segment fx0 ; U0 ; : : : ; xmk ; Umk ; xmk C1 ; Umk C1 ; : : : ; xmkC1 1 ; UmkC1 1 ; xmkC1 g such
that xmkC1 D s.U0 ; : : : ; UmkC1 1 / 2 F and xi F whenever mk < i < mkC1 .
Let ykC1 D xmkC1 and t .V0 ; : : : ; Vk / D ykC1 . This inductive procedure gives us a
strategy t for the player P on the space F .
If fyk ; Vk W k 2 !g is a play where P applies the strategy t then we have two
possibilities.
Case 1. There exists k 2 ! such that F  V0 [ : : : [ Vk . Then it is evident that the
player P is the winner.
Case 2. F 6 V0 [ : : : [ Vk for all k 2 !. Then there exists an adequate play
E D fxn ; Un W n 2 !g and an increasing sequence S D fmk W k 2 !g  ! such
that yk D xmk and Vk D UmS
k \ F for all k 2 !. Since P applied the strategy s
in the playSE, we have X D n2! Un . Besides, Un D
!nS and
S X nF for all n 2 S
therefore n2S Un  F . As a consequence, F D n2S .Un \ F / D k2! Vk
and hence the player P is the winner.
Therefore t is a winning strategy for the player P on the space F , i.e., F is
P -favorable.

2 Solutions of problems 001500

457

(v) Given a scattered space Y let I.Y / be the set of all isolated points of Y . Fix any
scattered space X and let X0 D X . Proceeding inductively assume that > 0
is an ordinal and we have a family fX W < g of closed subsets of X with
the following properties:
(2) if C 1 < then XC1 D X nI.X /;T
(3) if < is a limit ordinal then X D < X .
If D C 1 for T
some ordinal then let X D X nI.X /; if is a limit
ordinal then let X D < X . It is clear that the properties (2) and (3) still hold
for all  , so our inductive procedure can be continued indefinitely. However,
if X ; then XC1 is strictly contained in X because the space X is scattered.
Therefore the ordinal .X / D nnf W X D ;g (called the dispersion index of X )
is well defined. The family fX W < .X /g is called the canonical decomposition
of X .
Assume that X is a scattered space and fX W < .X /g is a canonical
decomposition of X . Suppose that Y  X and Y \ X D ; for some ordinal
 < .X /. Consider the canonical decomposition fY W < g of the space
Y . We have Y0 D Y  X D X0 ; proceeding inductively assume that is an
ordinalTand we proved
T that Y  X for all < . If is a limit ordinal then
Y D < Y  < X D X . If D C 1 then Y  X and hence any
non-isolated point of Y is not isolated in X , i.e., Y  X . Therefore our inductive
procedure can be continued to prove that Y  X for any ordinal . In particular,
Y  X , so it follows from Y \ X  Y \ X D ; that Y D ; and hence we
established that
(4) if fX W < .X /g is a canonical decomposition of X and Y \ X D ; for
some  < .X / and Y  X then .Y /   .
Say that a point a of a space X is central in X if X nU is P -favorable for any
U 2 .a; X /. Observe that
(5) if a space X has a central point then X is P -favorable.
Indeed, if x0 is a central point of X then let s.;/ D x0 ; if U0 is the move of
the player O then there is a winning strategy t for the player P on the space F D
X nU0 ; let x1 D t .;/ and s.U0 / D x1 . If moves x0 ; U0 ; : : : ; xn ; Un are made for
some n > 0 then let s.U0 ; : : : ; Un / D t .U1 \ F; : : : ; Un \ F /. This gives us a
strategy s on the space X . If fxn ; Un W n 2 !g is a play where P applies s then,
for the set F D X nU0 , the sequence fxnC1 ; UnC1 \SF W n 2 !g is a play in
whichSP applies a winning strategy t . ThereforeSF D n2! .UnC1 \ F / and hence
F  n2! UnC1 . This implies in turn that X D n2! Un , so s is a winning strategy,
i.e., X is P -favorable and hence (5) is proved.
Now assume that X is a nonempty scattered Lindelf space. We will prove that
X is P -favorable by induction on the dispersion index of X . Observe that X ;
implies that .X / > 0. If .X / D 1 then X is discrete and hence countable, so it
is P -favorable by (i). Now assume that > 1 is an ordinal and we proved that any
nonempty Lindelf scattered space Y with .Y / < is P -favorable.

458

2 Solutions of problems 001500

Take a Lindelf scattered space X with .X / D and let fX W < g be the
canonical decomposition of X . We have two cases.
T
Case 1. is a limit ordinal. Then < X D ;. For any point x 2 X there exists
< such that x X and hence we can choose a set Ux 2 .x; X / such
that U x \ X D ;. It follows from (4) that .U x /  < , so we can
apply the induction hypothesis to see that U x is a P -favorable space for any
x 2 XS
. The space X being Lindelf there
S exists a countable A  X such that
X D fUx W x 2 Ag and hence X D fU x W x 2 Ag. Now it follows from
(iii) that X is P -favorable.
Case 2. D C 1. Then X is countable closed discrete subset of X . Use
collectionwise normality of X to find a family fUx W x 2 X g such that
Ux 2 .x; X / for any x 2SX and the collection fU x W x 2 X g is disjoint.
Consider the set F D X n. fUx W x 2 X g/; it follows from F \ X D ; that
we can apply (4) again to convince ourselves that .F /  < and hence F
is P -favorable.
Assume that W 2 .x; U x /; then Fx D U x nW is a closed subset of X such
that Fx \ X D ; and hence we can apply the property (4) once more to see that
Fx is P -favorable by the induction hypothesis. This proves that x is a central point
of U x , so the property
S (5) guarantees that U x is P -favorable for all x 2 X . The
equality X D F [ . fU x W x 2 X g/ shows that X is the union of countably many
P -favorable spaces, so it is P -favorable by (iii). This shows that every Lindelf
scattered space is P -favorable and completes our solution.
V.414. Prove that
(a) if X is P -favorable for the point-open game and .X /  ! then X is
countable;
(b) a compact space X is P -favorable for the point-open game if and only if X is
scattered.
Solution. (a) Fix a winning strategy s for the player P on the space XT. For any
x 2 X let Ux  .x; X / be a countable family such that fxg D
Ux . Say
that an initial segment S D fx0 ; U0 ; : : : ; xn ; Un g of a play on X is s-legal if P
applies the strategy s in the segment S .
Let x0 D s.;/ and A0 D fx0 g. Proceeding inductively assume that we have a
countable set An  X and consider the set A0nC1 D fy 2 X W there exists
S k2!
and an s-legal initial segment fx0 ; U0 ; : : : ; xk ; Uk g such that Ui 2 fUx W x 2
An g for all i  k andS
y D s.U0 ; : : : ; Uk /g; let AnC1 D An [ A0nC1 and observe
that the family V D fUx W x 2 An g is countable, so we only have countably
many finite subfamilies of V. If S D fx0 ; U0 ; : : : ; xk ; Uk g is an s-legal initial
segment then the family fU0 ; : : : ; Uk g uniquely determines S because we have
the equalities x0 D s.;/; x1 D s.U0 /; : : : ; xn D s.U0 ; : : : ; Un1 /. Therefore
jA0nC1 j  jVj  ! and hence AnC1 is countable.
S
Once we haveSthe sequence fAn W n 2 !g let A D n2! An and consider the
family W D fUx W x 2 Ag. Let us show that

2 Solutions of problems 001500

459

(1) if we have an s-legal segment fx0 ; U0 ; : : : ; xn ; Un g such that U0 ; : : : ;


Un 2 W then s.U0 ; : : : ; Un / 2 A.
Indeed, it is easySto find a number m 2 ! such that fx0 ; : : : ; xn g  Am and
fU0 ; : : : ; Un g  fUx W x 2 Am g. Then s.U0 ; : : : ; Un / 2 A0mC1  A and hence
(1) is proved.
The set A is countable, so it suffices to show that X D A. Assume toward a
contradiction that X nA ; and pick a point y 2 X nA.
Since x0 2 A, we have x0 y and hence there exists U0 2 Ux0
such that y U0 . Assume that n 2 ! and we have an s-legal segment
fx0 ; U0 ; : : : ; xn ; Un g such that y U0 [ : : : [ Un and Ui 2 W for all i  n. If
xnC1 D s.U0 ; : : : ; Un / then it follows from (1) that xnC1 2 A; thus xnC1 y
and hence we can choose a set UnC1 2 UxnC1 with y UnC1 . It follows from
UxnC1  W that UnC1 2 W.
Thus we carried out the induction step, so our procedure can be continued to
construct a play fxn ; Un W n 2 !g in which PSapplies the strategy s while
y Un for each n 2 !. As a consequence, y n2! Un ; since s is a winning
strategy, we obtained a contradiction which shows that X D A and hence X is
countable as promised.
(b) If X is a compact scattered space then X must be P -favorable for the pointopen game (see Problem 413), so we have sufficiency. Now assume that X is a
compact P -favorable space. If X is not scattered then there exists a continuous
onto map f W X ! I (see SFFS-133). Any continuous image of a P -favorable
space is P -favorable by Problem 413, so I is P -favorable. Now, it follows
from (a) and .I/ D ! that I is countable. This contradiction shows that X
is scattered and settles necessity.
V.415. Given a nonempty space X define a game PO0 on the space X as follows:
at the n-th move the player P chooses a finite set Fn  X and the player O takes
a set Un 2 .X / such that Fn  Un . TheSgame ends after the moves Fn ; Un are
made for all n 2 !. The player P wins if n2! Un D X ; otherwise the victory is
assigned to O. Prove that the player P has a winning strategy in the game PO0 on
a space X if and only if X is P -favorable for the point-open game.
Solution. It is evident that every winning strategy of the player P in the point-open
game is a winning strategy of P in the game PO0 , so we have sufficiency.
Now assume that s is a winning strategy of P on a space X in the game PO0 . An
initial segment S D fF0 ; U0 ; : : : ; Fn ; Un g of a play in PO0 will be called s-legal if
P applies the strategy s in the segment S and, besides, Fi D ; implies Ui D ; for
any i  n.
There exists a number m 2 ! such that, for a uniquely determined s-legal initial
segment fF0 ; U0 ; : : : ; Fm1 ; Um1 g we have the equalities F0 D : : : D Fm1 D ;
while Fm D s.U0 ; : : : ; Um1 / ;. Choose a faithful enumeration fx0 ; : : : ; xk g of
the set Fm , let y0 D x0 and t .;/ D y0 . If 1  i  k and we have an initial segment
fy0 ; V0 ; : : : ; yi1 ; Vi1 g in the point-open game on X then let t .V0 ; : : : ; Vi1 / D xi .

460

2 Solutions of problems 001500

After the moves yi ; Vi are made for all i  k let Um D V0 [ : : : [ Vk to obtain


0
an initial segment
0 ; U0 ; : : : ; Fm ; Um g in the game PO . It is evident that S
S S D fFS
is s-legal and ik Vi D im Um .
Proceeding inductively assume that fy0 ; V0 ; : : : ; yn ; Vn g is an initial segment in
the point-open game on the space XSsuch that S
for some initial s-legal segment
fF0 ; U0 ; : : : ; Fm ; Um g, we have V D in Vi D im Ui . If V D X then choose
any ynC1 2 X and let t .V0 ; : : : ; Vn / D ynC1 . If V X then there is a uniquely
determined s-legal segment fF0 ; U0 ; : : : ; Fm ; Um ; FmC1 ; UmC1 ; : : : ; Fm0 1 ; Um0 1 g
such that Fm0 D s.U0 ; : : : ; Um0 1 / ; and Fi D ; whenever m < i < m0 . Choose
a faithful enumeration fz0 ; : : : ; zk 0 g of the set Fm0 ; if 0  i  k 0 and we have
an initial segment fy0 ; V0 ; : : : ; yn ; Vn ; ynC1 ; VnC1 ; : : : ynCi ; VnCi g in the point-open
game on X then let ynCiC1 D zi and t .V0 ; : : : ; VnCi / D ynCiC1 .
After the moves
C 1; : : : ; n C k 0 C 1 consider
S yi ; Vi are made for all i D n
0
the set Um0 D
fVi W n C 1S i  n S
C k C 1g. For the s-legal segment
fF0 ; U0 ; : : : ; Fm0 ; Um0 g we have im0 Ui D inCk 0 C1 Vi and hence our inductive
procedure can be continued to construct a strategy t for the player P on the space X
in the point-open game. Given a play fyi ; Vi W i 2 !g where P applies t , we have
two possibilities.
Case 1. There exists n 2 ! such that V0 [ : : : [ Vn D X . It clear that P wins this
play.
Case 2. V0 [ : : : ; [Vn X for any n 2 ! and hence there exists an s-legal play
fFi ; Ui W i 2 !g for which we can choose increasing
S sequences
S fmk W k 2 !g
and fnk W k 2 !g of naturalSnumbers such
that
V
D
ink i
imk Ui for all
S
k 2 !. As a consequence, i2! Vi D i2! Ui D X , so the player P wins
again and hence the strategy t is winning, i.e., the space X is P -favorable. This
settles necessity and makes our solution complete.
V.416. Given a nonempty space X define a game PO00 on the space X as follows:
at the n-th move the player P chooses a finite set Fn  X and the player O takes
a set Un 2 .X / such that Fn  Un . After all moves fFn ; Un W n 2 !g are made
let U D fUn W n 2 !g. The player P wins if lim U D X ; otherwise the victory is
assigned to O. Prove that the player P has a winning strategy in the game PO00 on
a space X if and only if X is P -favorable for the point-open game.
Solution. Recall that the moves in the game PO0 are the same as in PO00 but the
player P wins if the sets chosen by O cover the whole space. Suppose that s is a
winning strategy for the player P in the game PO00 on a space X . If fFn ; Un W n 2
!g is aSplay on X where P applies s and U D fUn W n 2 !g then lim U D X and
hence U D X which shows that s is also a winning strategy for the player P in
the game PO0 . By Problem 417 the space X is P -favorable, so we proved necessity.
Now assume that a space X is P -favorable and apply Problem 417 once more
to find a winning strategy s for the player P on the space X in the game PO0 . Say
that a sequence fW0 ; : : : ; Wn g of open subsets of X is adequate if there exist finite
sets G0 ; : : : ; Gn such that fG0 ; W0 ; : : : ; Gn ; Wn g is an initial segment of play where
P uses the strategy s. Say that a sequence fWn W n 2 !g of open subsets of X

2 Solutions of problems 001500

461

is adequate if its initial segment fW0 ; : : : ; Wn g is adequate for any n 2 !. Given


an adequate family fWn W n 2 !g the set GnC1 D s.W0 ; : : : ; Wn / is consistently
defined for any n 2 !. If G0 D s.;/ then fGn ; Wn W n 2 !g is a play in which P
applies the strategy s. The strategy s is winning, so we have the following property.
S
(1) If fWn W n 2 !g  .X / is an adequate family then n2! Wn D X .
Let F0 D s.;/; it is easy to see that
(2) for any n 2 !, a sequence fW0 ; : : : ; Wn g of open subsets of X is adequate if
and only if F0  W0 and s.W0 ; : : : ; Wi /  WiC1 for any i D 0; : : : ; n  1.
To construct a strategy t for the player P in the game PO00 on the space X let
t .;/ D F0 and t .U0 / D F0 [ s.U0 / for any U0 2 .F0 ; X /. Suppose that we have
an initial segment fF0 ; U0 ; : : : ; Fn ; Un g of a play on X such that
(3) if 0  j0 < : : : < jk  n then fUj0 ; : : : ; Ujk g is an adequate sequence;
(4) if i < n and 0  j0 < : : : < jk  i then F0 [ s.Uj0 ; : : : ; Ujk /  FiC1 .
Given any sequence fj0 ; : : : ; jk g  f0; : : : ; ng such that j0 < : : : < jk consider
the set F .j0 ; : : : ; jk / D F0 [ s.Uj0 ; : : : ; Ujk / and let
FnC1 D

fF .j0 ; : : : ; jk / W fj0 ; : : : ; jk g  f0; : : : ; ng and j0 < : : : < jk gI

let t .U0 ; : : : ; Un / D FnC1 . It is immediate from (2) that for any UnC1 2 .FnC1 ; X /
the properties (3) and (4) hold if we replace n by n C 1, so our inductive procedure
can be continued to construct a strategy t for the player P on the space X .
Now suppose that fFn ; Un W n 2 !g is a play where P applies the strategy t and
let U D fUn W n 2 !g.T
Assume for a moment that lim U X . Then there exists
x 2 X such that x fUi W i  ng for any n 2 !. Therefore we can find an
increasing
S sequence fjk W k 2 !g  ! such that x Ujk for every k 2 ! and hence
x k2! Ujk . However, it follows from (3) S
that fUjk W k 2 !g is an adequate
family, so we can apply (1) to conclude that k2! Ujk D X . This contradiction
shows that t is a winning strategy for the player P in the game PO00 , i.e., we settled
sufficiency and completed our solution.
V.417. Prove that a space X is P -favorable for the point-open game if and only if
Cp .X / is a W -space.
Solution. Suppose that X is P -favorable for the point-open game. By Problem 416
there exists a strategy s on the space X which is winning for P in the game PO00 .
Denote by u the function which is identically zero on X . For any finite set A  X
and " > 0 let A; "
D ff 2 Cp .X / W f .A/  ."; "/g.
If F0 D s.;/ then H0 D F0 ; 1
is an open neighborhood of the point u; let
t .;/ D H0 . If the second player in the W -game chooses a function f0 2 H0 then
let us consider that the player O picks the set U0 D f01 ..1; 1//; it is easy to see
that F0  U0 .
Proceeding inductively assume that moves fH0 ; f0 ; : : : ; Hn ; fn g have been made
in the Gruenhage W -game on Cp .X / at the point u, i.e., Hi 2 .u; Cp .X //;

462

2 Solutions of problems 001500

fi 2 Hi for all i  n and, besides, we have an initial segment S D


fF0 ; U0 ; : : : ; Fn ; Un g of a play in the game PO00 on X such that P applies the
strategy s in the segment S while
(1) Hi D Fi ; 2i
and Ui D fi1 ..2i ; 2i // for every i  n.
If FnC1 D s.U0 ; : : : ; Un / then the set HnC1 D FnC1 ; 2n1
is open in Cp .X /
and contains u; let t .f0 ; : : : ; fn / D HnC1 . If the answer of the second player in the
1
W -game is a function fnC1 2 HnC1 then letting UnC1 D fnC1
..2n1 ; 2n1 //
we obtain an open neighborhood UnC1 of the set FnC1 and hence the family
fF0 ; U0 ; : : : ; Fn ; Un ; FnC1 ; UnC1 g is an initial segment of a play in the game PO00
on X where P applies the strategy s. Evidently, the property (1) now holds for all
i  n C 1 and hence our inductive procedure defines a strategy t of the first player
in the Gruenhage W -game on Cp .X / at the point u for which the condition (1) is
satisfied.
Now, assume that fHn ; fn W n 2 !g is a play in the W -game in Cp .X / at the
point u in which the first player applies the strategy t . By our construction we have
a play fFn ; Un W n 2 !g in the game PO00 on X in which P applies the strategy s
and the property (1) holds for all n 2 !. Let U D fUn W n 2 !g; since the strategy s
is winning, we have X D lim U .
Given any neighborhood G of the point u in Cp .X / we can find a finite set
P  X and " > 0 such that P; "
 G. Fix m 2 ! for which 2m
P< "; for any
x 2 P there exists lx 2 ! such that x 2 Un for all n  lx . If l D mC flx W x 2 P g
then x 2 Un whenever x 2 P and n  l. Furthermore, for any point x 2 P we have
fn .x/ 2 .2n ; 2n /  .2m ; 2m /  ."; "/ which shows that fn 2 P; "
 G
for all n  l. Thus the sequence ffn W n 2 !g converges to u, so the strategy t is
winning and hence we have the W -property at the point u. The space Cp .X / being
homogeneous, every f 2 Cp .X / is a W -point, i.e., we established necessity.
To prove sufficiency suppose that u is a W -point in Cp .X / and fix a winning
strategy t for the first player in the Gruenhage game on Cp .X / at the point u. If
H0 D t .;/ then there exists a finite set F0  X such that G0 ; "0
 H0 for some
"0 > 0; let s.;/ D F0 . If the player O chooses an open set U0  F0 then we can
find a function f0 2 Cp .X / such that f0 .F0 /  f0g and f0 .X nU0 /  f1g. It is clear
that f0 2 H0 , so fH0 ; f0 g is an initial segment of a play in the W -game on Cp .X /
at the point u.
Proceeding inductively assume that n 2 ! and we have an initial segment
fF0 ; U0 ; : : : ; Fn ; Un g in the game PO0 (see Problem 415) and, besides, there is an
initial segment fH0 ; f0 ; : : : ; Hn ; fn g of a play in the W -game on Cp .X / at the point
u in which the first player applies the strategy t while
(2) fi .Fi /  f0g and fi .X nUi /  f1g for every i  n.
The set HnC1 D t .f0 ; : : : ; fn / is an open neighborhood of u in Cp .X /, so
there exists a finite set FnC1  X such that FnC1 ; "nC1
 HnC1 for some
"nC1 > 0. Let s.U0 ; : : : ; Un / D FnC1 . If UnC1  FnC1 is the move of the player
O then we can find a function fnC1 2 Cp .X / such that fnC1 .FnC1 /  f0g and
fnC1 .X nUnC1 /  f1g. Then fnC1 2 HnC1 and hence we have an initial segment

2 Solutions of problems 001500

463

fH0 ; f0 ; : : : ; HnC1 ; fnC1 g of a play in the W -game on Cp .X / at the point u in


which the first player applies the strategy t and the condition (2) is satisfied for
all i  n C 1. Therefore our inductive procedure can be continued to give us a
strategy s in the game PO0 for the player P on the space X with the condition (2)
is satisfied for all n 2 !.
Finally assume that fFn ; Un W n 2 !g is a play in which P applies the strategy s.
By our construction, there exists a play fHn ; fn W n 2 !g in the W -game on Cp .X /
at the point u in which the first player applies the strategy t and the property (2)
holds. Given any x 2 X the set fxg; 1
is an open neighborhood of u; since the
strategy t is winning, the sequence ffn W n 2 !g converges to u and hence there
exists i 2 ! such that fi 2 fxg; 1
, i.e., jfi .x/j < 1. This, together with
S (2) implies
that x 2 Ui ; the point x 2 X was taken arbitrarily, so we proved that n2! Un D X
and hence s is a winning strategy for the player P in the game PO0 . Applying
Problem 415 we conclude that X is P -favorable for the point-open game so we
settled sufficiency and completed our solution.
V.418. Assuming that 2! D !1 and 2!1 D !2 prove that there exists a scattered
Lindelf P -space X such that hd  .X / D !1 and jX j D !2 . Show that Cp .X / is a
W -space with no point-countable -base. In particular, Cp .X / is a W -space which
cannot be embedded into a -product of first countable spaces.
Solution. Given a space Z let G be the family of all G -subsets of Z. If is the
topology on Z generated by G as a base then .Z; / is called the !-modification of
Z. We denote by CHC the statement 2!1 D !2 ; given ordinals and with <
let ; / D f W   < g.
Fact 1. Given spaces Z and P , if A  Z  P and u D .z; p/ 2 Z  P then
u 2 A if and only if, for any U 2 .z; Z/ we have p 2 .A \ .U  P // where
 W Z  P ! P is the projection.
Proof. If u 2 A and U 2 .z; Z/ then u 2 A \ .U  P / because U  P is a
neighborhood of u. The projection  being continuous, it follows from .u/ D p
that p 2 .A \ .U  P // and hence we proved necessity.
Now assume that p 2 .A \ .U  P // for any U 2 .z; Z/. If u A then
there are U 2 .z; Z/ and V 2 .p; P / such that .U  V / \ A D ;. It follows
from p 2 .A \ .U  P // that V \ .A \ .U  P // ; and hence there is
a point .x; y/ 2 A \ .U  P / such that y 2 V . Since also x 2 U , we have
.x; y/ 2 A \ .U  V / which is a contradiction. Therefore u 2 A, i.e., we settled
sufficiency and hence Fact 1 is proved.
Fact 2. If Z is a Hausdorff P -space and Y is a Lindelf subspace of Z the Y is
closed in Z.
Proof. Take an arbitrary point z 2 ZnY . By the Hausdorff property of Z, for each
y 2 Y we can find Uy ; Vy 2 .Z/ such that y 2 Uy ; z 2 Vy and Uy \ Vy DS;. The
set Y being Lindelf there
T exists a countable A  Y such that Y  U D fUy W
y 2 Ag. The set V D fVy W y 2 Ag is an open neighborhood of z in Z (because

464

2 Solutions of problems 001500

Z is a P -space) and V \ U D ;. Therefore V \ Y D ; and hence any z 2 ZnY


has an open neighborhood contained in ZnY . This shows that ZnY is open in Z
whence Y is closed in Z, i.e., Fact 2 is proved.
Returning to our solution apply CHC to conclude that the family S D fS W there
is n 2 N such that S  !2n and jS j D !1 g has cardinality !2!1 D .2!1 /!1 D
2!1 D !2 . Take an enumeration fP W < !2 g of the family S; given any < !2
let n 2 N be the natural number for which P  !2n . For an arbitrary < !2 it
follows from jS j D !1 that there exists an ordinal 2 !1 ; !2 / such that s.i / <
for any s 2 P and i < n . Let  D supf W < g C 1 for any < !2 . It is
clear that P  n for each < !2 and < implies  <  . Let S D P
and n. / D n for all < !2 ; if  2 !1 ; !2 /nf W < !2 g then let S D !1
and n./ D 1. This gives us an enumeration fS W !1   < !2 g of the family S
such that
. / S  n./ for every  2 !1 ; !2 /.
Let D be a discrete space of cardinality !1 . It is clear that w.D !1 / D !1 ; fix
a base B in D !1 with jBj D !1 . Consider the family G of all G -subsets of D !1
and let 0 be the topology on D !1 generated by G as a base, i.e., E D .D !1 ; 0 / is
1 . Suppose that x 2 W 2 0 ; there exists G 2 G such that
the !-modification of D !T
!1
x 2 G  W . Since G D n2! Un for some family fUn W n 2 !g  .D
/, we can
T
pick a set Bn 2 B such that x 2 Bn  Un for every n 2 !. Then x 2 n2! Bn  G.
Therefore the family G 0 of all countable intersections of the elements of B is a base
of the space E. Observe that jG 0 j  jBj!  !1! and apply CH to see that we have
the equalities !1! D .2! /! D 2!! D 2! D !1 . Consequently, jG 0 j  !1 and hence
w.E/ D !1 .
It follows from CH and CHC that jEj D !1!1 D .2! /!1 D 2!1 D !2 , so we can
identify the underlying set of E with !2 and consider that we have a topology  on
the set !2 such that E D .!2 ; / is a P -space with w.E/ D !1 . For every < !2
and natural n > 1 we will need the natural projection n W n1 .C1/ ! .C1/.
For any < !2 , denote by  the topology on induced from E.
To start our inductive construction let D exp./ for each < !1 and assume
that, for some 2 !1 ; !2
we have a collection f W < g of topologies with the
following properties:
(1) is a topology on for all < ;
(2) for any < , the space Y D .; / is Tychonoff, locally Lindelf and every
G -subset of Y is open in Y (i.e., Y is a P -space); besides, for any n 2 N, a
base Bn with jBn j  !1 is chosen in .Y /n ;
(3) if  < < then \ exp. / D  ;
(4)   for any < ;
(5) if  < < ; n D n./ > 1 and the point belongs to the -closure of the set
n .S \ .U  . C 1/// for some U 2 Bn1 then belongs to the C1 -closure
of n .S \ .U  . C 1///;
(6) if  < < ; n D n./ D 1 and the point belongs to the -closure of S
then also belongs to the C1 -closure of S .

2 Solutions of problems 001500

465

If is a limit
Sordinal then let be the !-modification of the topology generated
by the family f W < g as a subbase. It follows from (3) that every set
is open in Y D .; /, so (2) and (3) imply that the property (3) still holds for
all 
S . If U 2  then U D U \ 2 for any < by (4). Therefore
U D fU W < g 2 , i.e., (4) also holds for all  . The properties (5)
and (6) bring no new obligations for the collection f W  g, so they are also
fulfilled for all  .
As to the property (2), observe first that Y is Hausdorff its topology being
stronger than the Tychonoff topology  . Given a point < there is a local
base C of Lindelf open neighborhoods of the point in the space YC1 . It follows
from (2) and (3) that C is also a local base in Y at the point . Besides, any Lindelf
subspace is closed in a Hausdorff P -space (see Fact 2), so C is a clopen local base in
Y at . Thus Y is zero-dimensional and hence Tychonoff. Choosing the respective
bases in all finite powers of Y , we conclude that (2) is also fulfilled for all  ,
i.e., our construction can be carried out for any limit ordinal .
Now assume that is a successor ordinal, say, D  C 1 and consider the
set M D f < : either n D n./ > 1 and  is in the -closure of the set
TU D n .S \ .U  . C 1/// for some U 2 Bn1 or n D n./ D 1 and  is in
the -closure of S g. If M D ; then let be the topology generated by  [ ffgg
as a subbase. Choosing the respective bases in all finite powers of Y we can easily
see that the conditions (1)(6) are satisfied for all  , so this case is easy.
Now, if the set M is nonempty then the family H D fS W  2 M and
n./ D 1g [ fTU W  2 M ; n./ > 1 and U 2 Bn1 g has cardinality  !1 ,
so we can choose an enumeration fH W < !1 g of the family H in which every
H 2 H occurs !1 -many times. Since E D .;  / is a P -space of weight !1 ,
we can choose a local clopen base fB W < !1 g at the point  in E such that
 < < !1 implies B  B .
The point  belongs to the -closure of every element of H , so we can pick a
point z 2 H \ B for every < !1 . It follows from . / that z  for any
< !1 . The transfinite sequence S D fz W < !1 g converges to the point  in
E , which, together with the P -property of E implies that S is a discrete subspace
of E and  is the unique cluster point of S .
Given an ordinal < !1 the set B \  is a clopen subspace of Y , so there
exists a clopenS
Lindelf subspace L of the space Y such that z 2 L  B . For
C D ffg [ . fL W   g/ W < !1 g let be the topology generated by the
family  [ C as a subbase.
An immediate consequence of the definition of is that \ exp./ D  , i.e.,
we have (1) and (3) for the collection f W  g. The family C is a local clopen
base at the point , so the space Y is Tychonoff being T1 and zero-dimensional. Any
countable intersection of the elements of C belongs to C whence  is a P -point in
Y ; thus Y is a P -space. Furthermore, every C 2 C is Lindelf because, for any
U 2 .; Y / there is < !1 such that the set C nU is closed in the Lindelf space
S
fL W  < g. After we choose the relevant bases in all finite powers of Y we
conclude that (2) also holds for the family f W  g. The property (4) is clear
and the property (5) must only be checked for D .

466

2 Solutions of problems 001500

If  <  and n./ > 1 assume that the point  is in the -closure of the set TU
for some U 2 Bn1 . Then H D TU 2 H and therefore jH \ S j D !1 which
shows that C \ H ; for any C 2 C , i.e.,  is in the -closure of H . Therefore
the condition (5) is satisfied for the family T D f W  g. The proof that (6)
is also satisfied for T is analogous, so our inductive construction is complete; let
D !2 and Y D .!2 ; /.
It follows from (2) and (3) that every Y is open in Y , so Y is right-separated
and hence scattered. The properties (2) and (3) imply that Y is locally Lindelf,
Tychonoff and every 2 Y is a P -point in Y , i.e., Y is a locally Lindelf P -space.
We will prove by induction that hd.Y n /  !1 for every n 2 N. To start the
induction let Y 0 D f;g; then hd.Y 0 / D 1 < !1 . Now assume that n  1 and we
have proved that hd.Y k /  !1 for all k < n. We will need the sets 1 D ; and
n D fx 2 Y n W there are distinct i; j < n with x.i / D x.j /g for any n 2 Nnf1g.
By Fact 0 of T.020 it suffices to show that we have the inequality hd.Y n n n /  !1 .
Consider the set Gn D fx 2 Y n W x.0/ < : : : < x.n  1/g; since any permutation
of the set n generates a homeomorphism of Y n n n onto itself, the space Y n n n is
a finite union of spaces homeomorphic to Gn . Therefore it suffices to establish that
hd.Gn /  !1 (evidently, G1 D Y ).
Take an arbitrary set A  Gn ; there is no loss of generality to consider that
jAj D !2 . We have hd.Y n1 /  !1 by the induction hypothesis; since w.E/  !1 ,
we can apply Fact 1 of V.023, to see that hd.Y n1  E/  !1 . Therefore the set
A has density at most !1 in the space Y n1  E, so we can find a set B  A such
that jBj D !1 and B is -dense in A where  D .Y n1  E/; there is an ordinal
 < !2 such that B D S . Let  W Y n1  Y ! Y be the projection.
The set A0 D fx 2 A W x.n  1/  g has cardinality at most !1 . If x 2 AnA0
then D x.n  1/ >  and x D .z; / where z 2 n1 . Given any V 2 .z; Y n1 /,
there is U 2 Bn1 such that z 2 U  V . The point x being in the -closure
of S , we can apply Fact 1 to see that the point is in the -closure of the set
TU D n .S \ .U  . C 1///. The property (5) for n > 1 (or the property (6) if
n D 1) shows that belongs to the C1 -closure of TU .
An immediate consequence of (3) is that belongs to the -closure of TU . Since
U  V , the point belongs to the -closure of the set n .S \ .V  . C 1///;
therefore belongs to the -closure of the set .S \ .V  Y //. Now, Fact 1 is
applicable to the product P  Z where Z D Y n1 and P D Y to conclude that
x D .z; / is in the n -closure of S where n D .Y n /. This proves that A0 [ B is
a n -dense subset of A of cardinality at most !1 . Therefore hd.Gn /  !1 and hence
hd.Y n /  !1 .
To finally construct the promised space X take a point a Y . To introduce a
topology X on the set X D Y [ fag consider the family L D fU  Y W Y nU is a
clopen Lindelf subspace of Y g and let X be the topology generated by the family
[ ffag [ L W L 2 Lg. It is straightforward that .X; X / is a Lindelf scattered
P -space. It follows from hd  .Y /  !1 and jX nY j D 1 that hd  .X /  !1 (see
Fact 1 of T.099).

2 Solutions of problems 001500

467

Next, apply Problem 413 to see that X is P -favorable for the point-open game
and hence Cp .X / is a W -space by Problem 417. Assume that Cp .X / has a pointcountable -base B. By Fact 2 of U.271 there existsSa family fD W < !1 g of
discrete subspaces of Cp .X / such that the set D D fD W < !1 g meets every
element of the family B; in particular, D is dense in Cp .X /.
Apply SFFS-027 to see that s.Cp .X //  hl.Cp .X //  hl  .Cp .X // D
hd  .X /  !1 and hence jD j  !1 for each < !1 ; therefore jDj  !1 . As
a consequence, .X /  i w.X / D d.Cp .X //  !1 which shows that .X /  !1 .
In particular,
T .a; X /  !1 and hence there is a family fW W < !1 g  .a; X /
such that fW W < !1 g D fag. If F D X S
nW then F is a Lindelf subspace
of Y for any < !1 . It follows from Y D fF W < !1 g that l.Y /  !1 .
However, fY W < !2 g is an open cover of Y which does not have any subcover
of cardinality !1 . This contradiction shows that Cp .X / does not have a countable
-base and hence it cannot be embedded in a -product of first countable spaces
(see Problem 412), so our solution is complete.
V.419.QProve that if Xt is a d -separable space for each t 2 T then the product
space t2T Xt is d -separable. In other words, any product of d -separable spaces
must be d -separable.
Solution. The expression X ' Y says that the spaces X and YQare homeomorphic.
Given a space Nt for each t 2 T suppose that a 2 N D t2T Nt . The space
 .N; a/ D fx 2 N W the set ft 2 T W x.t / a.t /g is finiteg is the  -product of N
with the center a. For any x 2  .N; a/ let supp.x/ D ft 2 T W x.t / a.t /g. We
will say that a topological property P is d -adequate, if
P is preserved by finite products;
if X has P and U is open in X then U has P;
P
Lis preserved by free unions, i.e., if Xt has P for every t 2 T then the space
t2T Xt also has P;
(4) If X is aSspace, Y D X and Y has P then X also has P;
(5) if X D n2! Xn and every Xn has P then X also has P.

(1)
(2)
(3)

Fact 1. SupposeQ
that a topological property P is d -adequate and Nt has P for any
t 2 T ; let N D t2T Nt . Then the space  .N; a/ has P for any a 2 N and hence
the space N also has P.
Proof. Fix any point a 2 N and let Nt D Nt nfa.t /g for any t 2 T ; given a set
S  T let aS D ajS . For every n 2 ! consider
the set Ln D fx 2  .N; a/ W
S
jsupp.x/j D ng; observe that L0 D fag and n2! Ln D  .N; a/.
Fix any n 2 N and consider the family
Q Fn D fA  T W jAj D ng. Observe
that forSevery A 2 Fn the set HA D t2A Nt  faT nA g is contained in Ln and
Ln D fHA W A 2 Fn g. If A; A0 2 Fn and A A0 then AnA0 ; or A0 nA ;. If
t 2 AnA0 then x.t / a.t / for any x 2 HA while y.t / D a.t / whenever y 2 HA0 ;
an analogous reasoning for the case of A0 nA ; shows that HA \ HA0 D ; for
distinct A; A0 2 Fn , i.e.,
(6) the family fHA W A 2 Fn g is disjoint for any n 2 N.

468

2 Solutions of problems 001500

Q
Q
Let WA D t2A Nt  t2T nA Nt for every A 2 Fn . It is clear that WA is an open
subset of N and a moments reflection shows that WA \ Ln D HA . Therefore
every
L
(6) that Ln is homeomorphic to fHA W A 2
HA is open in Ln , so it follows from Q
Fn g. It is an easy exercise that HA ' t2A Nt , so it follows from the properties (1)
and (2) that HA has P for every A 2 Fn . Applying (3) we can see that Ln has the
property P. Now the property (5) implies that  .N; a/ has P. Finally observe that
 .N; a/ is dense in N and apply (4) to conclude that N has P as well, so Fact 1 is
proved.
Returning to our solution let P stand for d -separability. Given d -separable
X
Y
spaces X and Y we can choose discrete subspaces
S XDn  X and Dn  Y for
each n 2
!
in
such
a
way
that
the
set
D
D
fD
W
n
2
!g
is
dense
in X and
X
n
S
X
DY D fDnY W n 2 !g is dense in Y . It is evident that D
D
D

DnY is a
m
Sm;n
discrete subspace of X  Y for any m; n 2 !. Since D D fDm;n W m; n 2 !g D
DX  DY , the set D is dense in X  Y and hence X  Y is d -separable. Therefore
P has the property (1).
Given a d -separable
S space X fix a family fDn W n 2 !g of discrete subspaces of
X such that D D n2! Dn is dense inSX . If U is open in X then Dn0 D Dn \ U
is a discrete subspace of U and D 0 D n2! Dn0 D D \ U is dense in U , so U is
d -separable and hence P satisfies the condition (2).
t
Now assume that Xt is d -separable
S and choose a family fDn W n 2 !g of discrete
subspaces of Xt such that Dt D n2! Dnt is dense in Xt for every
Lt 2 T . We
will identify every Xt with the respective
clopen
subspace
of
X
D
t2T Xt . It is
S t
straightforward
that
the
set
D
D
fD
W
t
2
T
g
is
discrete
for
any
n
2 !; since
n
S
S n
also D D n2! Dn D t2T Dt is dense in X , we conclude that d -separability is
preserved by free unions, i.e., P has (3).
If Y has a dense  -discrete subspace D and Y D X then D is a dense  -discrete
subspace
Finally, assume that
S of X , so X is also d -separable and hence P has (4).
n
X D n2! Xn and every Xn is d -separable;
fix
a
family
fD
W
m m 2 !g of discrete
S n
subspaces
of
X
such
that
D
D
fD
W
m
2
!g
is
dense
in Xn . Then the set
n
m
S n n
D D fDm
W n; m 2 !g is a dense  -discrete subspace of X and hence P also
has the property (5), so d -separability is a d -adequate property. Applying Fact 1 we
conclude that P is preserved by arbitrary products, i.e., any product of d -separable
spaces is d -separable and hence our solution is complete.
V.420. Given an infinite cardinal  and a space X prove that X  is d -separable
if and only if there exists a family D D fDn W n 2 !g of discrete subspaces of X 
such that supfjDn j W n 2 !g  d.X /. In particular, if X  has a discrete subspace
of cardinality d.X / then X  is d -separable. Deduce from this fact that X d.X/ is
d -separable for any space X .
Solution. If the space X  is d -separableSthen there exists a family fDn W n 2 !g
of discrete subsets of X  such that D D n2! Dn is dense in X  . Therefore jDj D
supfjDn j W n 2 !g  d.X  /  d.X /, so we have necessity.
Let D d.X / and assume that we have a family fDn W n 2 !g of discrete
subspaces of X  such that supfjDn j W n 2 !g  . Consider the cardinal n D jDn j;

2 Solutions of problems 001500

469

there is no loss of generality to assume that !  n  nC1 for all n 2 !. If X


is discrete then X  is d -separable by Problem 421, so we can assume that X is not
discrete; fix a non-isolated point p 2 X .
It is evident that there exists a set S  X nfpg such that S D X and jS j D .
It follows from supn2! n  that we can find a family fSn W S
n 2 !g of subsets
of S such that jSn j  n , Sn  SnC1 for all n 2 ! and S D n2! Sn . For each
nonempty finite set A   and n 2 ! the cardinality of the set .Sn /A does not
exceed n ; let fsA;n W  < n g be an enumeration of .Sn /A .
S
Take a family fIn W n 2 !g of subsets of  such that  D n2! In while In 
InC1 and jIn j D jnIn j D  for every n 2 !. Since X nIn is homeomorphic to X  ,
we can find a discrete subspace En  X nIn such that jEn j D n . Let fen W  < n g
be an enumeration of En and choose a set Un 2 .X nIn / such that Un \ En D fen g
for all  < n .
For any n 2 ! and k 2 N let Fkn D fA  In W jAj D kg. Given a set A 2 Fkn
and  < n define a point xA;n 2 X  as follows: xA;n ./ D sA;n ./ for all 2 A;
if 2 In nA then xA;n ./ D p and xA;n ./ D en ./ for all 2 nIn .
For arbitrary n 2 ! and k 2 N consider the set Qkn D fxA;n W  < n ; A 2 Fkn g
and take any point xA;n 2 Qkn . The set W D fx 2 X  W x./ p for all 2 A and
0

xj.nIn / 2 Un g is, evidently, open in X  and xA;n 2 W . If y D xA ;n 2 Qkn nfxA;n g
then we have two possibilities.
Case 1. If   then yj.nIn / D en Un , so y W .
Case 2. If  D  then A0 A; if y 2 W then y./ p for all 2 A. However,
A0 D f 2 In W y./ pg, so A  A0 and it follows from jAj D jA0 j D k that
A D A0 and hence y D xA;n which is a contradiction.
Therefore W \ Qkn D fxA;n g and hence we proved that
(1) Qkn is a discrete subspace of X  for any n 2 ! and k 2 N.
S
Thus the set Q D fQkn W n 2 !; k 2 Ng is a  -discrete subspace of X  . Take
an arbitrary nonempty set U 2 .X  /. There is a finite set A D f1 ; : : : ; k g  
such that for some V1 ; : : : ; Vk 2  .X / the set W D fx 2 X  W x.i / 2 Vi for all
i  kg is contained in U . The set S being dense in X we can find t1 ; : : : ; tk 2 S such
that ti 2 Vi for all i  k. There exists n 2 ! such that A  In and ft1 ; : : : ; tk g  Sn ;
it is evident that A 2 Fkn . Letting s.i / D ti for all i  n we obtain a point
s 2 .Sn /A , so there exists an ordinal  < n such that s D sA;n . Now it follows
from xA;n jA D s that xA;n 2 W \ Q  U \ Q and hence Q \ U ; for any
U 2  .X  /, i.e., Q is dense in X  , so the space X  is d -separable which shows
that we proved sufficiency.
Finally, take any space X and let  D d.X /. If X is discrete then the space
X  is d -separable by Problem 419. If X is not discrete then the two-point space D
embeds in X and hence D embeds in X  . It is an easy exercise that D has a discrete
subspace of cardinality , so X  also has a discrete subspace of cardinality  and
hence we can apply our above result to convince ourselves that X  is d -separable.

470

2 Solutions of problems 001500

V.421. Prove that


(a) if K is a compact space then K ! is d -separable;
(b) there exists a compact space K such that K n is not d -separable for any n 2 N.
Thus K ! is d -separable but no finite power of K is d -separable.
Solution. Given a space X let X D f.x; x/ W x 2 X g  X  X be the diagonal
of X . Say that a compact space X is deep if any nonempty G -subset of X has
nonempty interior.
Fact 1. For any compact space X there exists a discrete subspace D  .X X /n X
such that jDj D d.X /.
Proof. If X is finite then there is nothing to prove, so we assume from now on that
X is infinite. Call a compact space X adequate if
(1) d.U / D d.X / for any U 2  .X /.
We will show first that our Fact is true for all adequate compact spaces, so assume
that X is adequate and let  D d.X /; then w.U /  d.U / D  for any nonempty
open set U  X . Call a set U  X  X standard if there exist V; W 2  .X / such
that V \ W D ; and U D V  W . Observe that U \ X D ; for any standard set
U and the union of the family S of all standard sets coincides with .X  X /n X .
We will need the following property of S.
S
(2) If U  S and jU j <  then ..U  U /n X /n. U / ; for any U 2  .X /.
S
Assume toward a contradiction, that .U  U /n X is contained in U for some
family U  S with jU j < . Choose a set V 2  .X / such that V  U and observe
that Y D V is a compact space for which w.Y /  w.V /  . It is easy to seeSthat
.Y  Y /n Y D .Y  Y /n X  .U  U /n X and therefore .Y  Y /n X  U .
Furthermore, H \ X D ; for any H 2 U , so QH D H \ .Y  Y / is a compact
subset of .Y  Y /n Y .
S
It follows from the equality .Y  Y /n Y D fQH W H 2 U g that .Y  Y /n Y
is a union of < -many compact sets and hence Y is the intersection of < -many
open subsets of Y  Y , i.e., .Y / < . However, w.Y / D .Y / (see SFFS-091)
and hence w.Y / < ; this contradiction shows that (2) is proved.
Take an arbitrary point z0 D .x0 ; y0 / 2 .X  X /n X ; there exists a standard set
H0 such that z0 2 H0 . Proceeding inductively assume that <  and we have a set
fz D .x ; y / W < g  .X  X /n X and a family fH W < g of standard
subsets of X  X with the following properties:
(3) z 2 H
S for all < ;
(4) z fH W  < g for every < ;
(5) H \ fz W  < g D ; for all < .
The set P D fx ; y W < g has cardinality strictly less than , so P X
and hence we can find a set U 2  .X / with U \ P D ;. By the property (2) there
are distinct points x ; y 2 U such that z D .x ; y / does not belong to the set

2 Solutions of problems 001500

471

fH W < g. Choose V 2 .x ; X / and W 2 .y ; X / such that V [ W  U


and V \W D ;; then H D V W is a standard set such that fz W < g\H D ;
and hence the properties (3)(5) hold for all  .
This proves that our inductive procedure can be continued to construct a set D D
fz W < g  .X  X /n X and a family H D fH W < g of elements of S
such that the conditions (3)(5) are satisfied for all < . It follows from (3)(5)
that D \ H D fz g, so the set D is discrete; the properties (3) and (4) imply that
fz W < g is a faithful enumeration of D, so jDj D  D d.X / and hence our
Fact is proved for all adequate compact spaces X .
Now take an arbitrary compact space X . Given a set U 2  .X / consider the
cardinal  D nnfd.V / W V 2  .U /g and pick a set V 2  .U / with d.V / D . It
is evident that d.W / D  for any nonempty open set W  V . If W 2  .X / and
W  V then it is easy to see that W is an adequate compact space. Therefore
(6) the family W D fW 2  .X / W W is adequateg is a -base of the space X .
Let W 0 be
Sa maximal disjoint subfamily of W; it follows from the property (6)
that the set W 0 is dense in X and hence d.X / D supfd.W / W W 2 W 0 g. For
every W 2 W 0 take a set HW 2  .X / with H W  W ; it is easy to see that
the compact space H W is adequate and d.H W / D d.W /. By what we proved
for adequate compact spaces, we can find a discrete set DW  .H W  HSW /n X
with jDW j D d.W / for each W 2 W 0 . It is straightforward that D D fDW W
W 2 W 0 g is a discrete subset of the space .X  X /n X . Finally we can apply the
equalities jDj D supfjDW j W W 2 W 0 g D supfd.W / W W 2 W 0 g D d.X / to see
that D is the promised discrete subspace of .X  X /n X , so Fact 1 is proved.
Fact 2. If X is a deep space, then X n is also deep for any n 2 N.
Proof. Suppose that H is a G -subset of X n and x D .x
T1 ; : : : ; xn / 2 H . Fix a
family U D fUm W m 2 !g  .X n / such that H D
U . For every m 2 !
we can find sets W1m ; : : : ; Wnm 2 .X / such that xi 2 Wim for all i  n and
Wm D W1m  : : :  Wnm  Um .
T m
Given any i 2 f1; : : : ; ng observe that xi 2 fW
Ti W m 2 !g; the space X being
deep we can find a set Vi 2 .xi ; X / with Vi  fWim W m 2 !g. It is clear that
V D V1  : : :  Vn is an open neighborhood of x and V  H , so H has nonempty
interior, i.e., X n is deep and hence Fact 2 is proved.
Fact 3. If X is a deep compact space without isolated points then X cannot be
d -separable.
S
Proof. Assume that Dn is a discrete subspace of X and D D n2! Dn . Since X
is dense-in-itself, the set D n isS
nowhere dense in X for every n 2 !.SThe space X
has the Baire property, so X n. n2! D n / ; and hence H D X n. n2! D n / is a
nonempty G -subset of X . The space X being deep, we can find a set U 2  .X /
such that U  H and hence U \ D D ;. Therefore the set D is not dense in X and
hence X is not d -separable, i.e., Fact 3 is proved.

472

2 Solutions of problems 001500

Returning to our solution observe that if K is a compact space then K  K has a


discrete subspace of size d.K/ by Fact 1. Since K  K embeds in K ! , we conclude
that K ! has a discrete subspace of size d.K/, so we can apply Problem 420 to see
that K ! is d -separable; this settles (a).
To prove (b) consider the compact space K D !n!. Given any nonempty open
set U  !n! there exists an infinite set A  ! such that A \ .!n!/  U (see
Fact 2 of S.370). The set A is uncountable by Fact 1 of S.483, so A \ .!n!/  U
is uncountable as well. Therefore U is uncountable and hence K has no isolated
points. The space K is deep by TFS-370, so K n is dense-in-itself and deep by Fact 2.
Applying Fact 3 we conclude that K n is not d -separable for any n 2 !. However,
K ! is d -separable by (a), so our solution is complete.
V.422. Prove that if the space Cp .X / is d -separable then there is a discrete
subspace D  Cp .X / such that jDj D d.Cp .X //.
Solution. For any x1 ; : : : ; xn 2 X and any nontrivial open intervals O1 ; : : : ; On
of R, the set x1 ; : : : ; xn ; O1 ; : : : ; On
D ff 2 Cp .X / W f .xi / 2 Oi for all
i  ng is open in Cp .X /. Call a set U  Cp .X / standard open if there exist
points x1 ; : : : ; xn 2 X and nontrivial open intervals O1 ; : : : ; On of R such that
U D x1 ; : : : ; xn ; O1 ; : : : ; On
. It is evident that standard open sets form a base of
Cp .X /. Besides, it follows from Fact 1 of S.409 and Fact 1 of S.494 that
(1) every standard open subset of Cp .X / is homeomorphic to Cp .X /.

S
Fix a family D D fDn W n 2 !g of discrete subspaces of Cp .X / such that D
is dense in Cp .X / and hence the cardinal D supfjDn j W n 2 !g is greater than
or equal to  D d.Cp .X //; let n D jDn j for any n 2 !. Take a disjoint family
fUn W n 2 !g of nonempty open subspaces of Cp .X / and pick a standard open
set Vn  Un for each n 2 !. It follows from (1) that Cp .X / is homeomorphic to
Vn and therefore we can fix a discrete
Ssubspace En  Vn such that jEn j D n for
every n 2 !. It is evident that E D n2! En is a discrete subspace of Cp .X / and
jEj D  . Passing, if necessary, to an appropriate subspace of E we can obtain
a discrete subspace of Cp .X / of cardinality .
V.423. Given a space X and n 2 N say that a discrete subspace D  X n is
essential if D \ n .X / D ; and jDj D i w.X /. Prove that if, for some n 2 N, there
exists an essential discrete set D  X n then Cp .X / is d -separable. In particular,
if there exists a discrete subspace of X of cardinality i w.X / then Cp .X / is d separable.
Solution. For any n 2 N let on 2 Rn be the point whose all coordinates are equal
to zero; for any r > 0, the set Bnr D f.x1 ; : : : ; xn / 2 Rn W x12 C : : : C xn2  r 2 g
is the closed r-ball in Rn centered at on . It is clear that B1r D r; r
 R. Given a
space X and a set A  X let CAn .X / D ff 2 Cp .X; Rn / W f .A/  fon gg for any
n 2 N. For any points a; b 2 Rn the set a; b
D ft a C .1  t / b W t 2 0; 1
g is
the line segment in the space Rn which connects a and b. Given a space Z say that
sets P; Q  Z are functionally separated if there exists a function f 2 C.Z; 0; 1
/
such that f .P /  f0g and f .Q/  f1g.

2 Solutions of problems 001500

473

Fact 1. Given a space X and an infinite cardinal  suppose that a family F consists
of finite subsets of X , the cardinality of F is equal to  and there is m 2 N such
that jF j  m for all F 2 F. Then there exists a finite set A  X (called the core of
F) such that for any finite B  X nA there exists U 2 .B; X / such that the family
fF 2 F W F \ U D ;g has cardinality .
Proof. If the family F has no finite core then we can choose disjoint nonempty sets
A0 ; : : : ; Am  X such that for any i  m and U 2 .Ai ; X / there are strictly less
than  elements of F which miss U . Now let U0 ; : : : ; Um be disjoint open subsets
of X such that Ai  Ui for all i  m. Every family fF 2 F W F \ Ui D ;g
has cardinality strictly less than , so there is F 2 F such that F \ Ui ; for
any i  m. This implies, however, that jF j  m C 1 and hence we obtained a
contradiction which shows that Fact 1 is proved.
Fact 2. Suppose that n 2 N and X is a space for which there exists a finite set
A  X such that, for some r > 0 and some  -discrete subspace E  CAn .X /, the
set Kn D Cp .X; Bnr / \ CAn .X / is contained in the closure of E. Then the space
.Cp .X //n is d -separable.
Proof. We identify .Cp .X //n with Cp .X; Rn /. For every k 2 N let 'k .f / D k f
for any f 2 Cp .X; Rn /; it is immediate that 'k W Cp .X; Rn / ! Cp .X; Rn / is a
homeomorphism. Therefore Ek D 'k .E/ is a  -discrete subspace of Cp .X; Rn /
S
fFk W k 2 Ng
and Fk D 'k .Kn /  ES
k for any k 2 N. It is an easy exercise that
is dense in CAn .X /, so fEk W k 2 Ng is a  -discrete dense subspace of CAn .X /,
i.e., CAn .X / is d -separable. If A D ; then Cp .X; Rn / D CAn .X / is d -separable. If
A ; then Cp .X; Rn / is homeomorphic to CAn .X /  .Rn /A by Fact 1 of S.409.
Any product of d -separable spaces is d -separable (see Problem 421), so the space
.Cp .X //n ' Cp .X; Rn / is d -separable and hence Fact 2 is proved.
Fact 3. If Z is a space and P; Q  Z then P and Q are functionally
separated if and only if there exist zero-sets P 0 and Q0 in the space Z such that
P  P 0 ; Q  Q0
and P 0 \ Q0 D ;.
Proof. If P and Q are functionally separated then take a function f 2 C.Z; 0; 1
/
such that f .P /  f0g and f .Q/  f1g; then P 0 D f 1 .0/ and Q0 D f 1 .1/ are
as promised, so we have necessity.
If, on the other hand, there exist disjoint zero-sets P 0 ; Q0 such that P  P 0
and Q  Q0 then we can apply Fact 1 of V.140 to see that there exists a function
f 2 C.Z; 0; 1
/ such that f .P 0 /  f0g and f .Q0 /  f1g. It is clear that f also
separates P and Q, so Fact 3 is proved.
Fact 4. Suppose that Z is a space and we have functionally separated nonempty
sets P; Q  Z. Then, for any points a; b 2 Rn there exists a continuous function
f W Z ! a; b
such that f .P / D fag and f .Q/ D fbg.

474

2 Solutions of problems 001500

Proof. Since Rn D Cp .f1; : : : ; ng/, it follows from Fact 1 of S.301 that there exists
a homeomorphism ' W 0; 1
! a; b
such that '.0/ D a and '.1/ D b. Take a
function g 2 C.Z; 0; 1
/ such that g.P /  f0g and g.Q/  f1g. Then f D ' g
is as promised, so Fact 4 is proved.
Fact 5. Suppose that Z is a space, a set F  Z is compact and F  U 2 .Z/.
Then there exists V 2 .F; Z/ such that V is functionally separated from X nU . In
particular, F is functionally separated from ZnU .
Proof. The sets F and G D clZ .ZnU / are disjoint and closed in Z. By normality
of Z there exists a continuous function f W Z ! 0; 1
such that f .F /  f1g
and f .G/  f0g. If g D f jZ then P D g 1 . 12 ; 1
/ and Q D g 1 .0/ are zero-sets
in Z. Since ZnU  Q and V D f 1 .. 12 ; 1
/  P , we can apply Fact 3 to see
that V is functionally separated from ZnU ; besides, F  V 2 .Z/, so Fact 5 is
proved.
Fact 6. Suppose that w1 ; : : : ; wk 2 Rn and we have a disjoint family U1 ; : : : ; Uk of
open subsets of Z. If ; Fi  Ui and Fi is functionally separated from ZnUi
(in particular, if Fi is a compact subset of Ui ) for each i  k then there exists a
continuous function ' W Z ! Rn such S
that f .U i /  on ; wi
and f .Fi / D fwi g for
any i  k while f .z/ D on for all z ik Ui .
Proof. Apply Fact 4 to find a continuous function 'i W Z ! on ; wi
such that
'i .Fi / D fwi g and 'i .ZnUS
i /  fon g for every i  k. Let '.z/ D 'i .z/ if z 2 Ui
for some i  k; if z 2 Zn. ik Ui / then let '.z/ D on . It is easy to see, applying
Fact 2 of T.254, that ' is a continuous function with the required properties. Finally
observe that if Fi is compact then Fi is functionally separated from ZnUi (see
Fact 5), so Fact 6 is proved.
Returning to our solution observe that if the cardinal  D i w.X / is countable
then Cp .X / is even separable, so we can assume that  > !. For any k 2 N
denote by Mk the set f1; : : : ; kg. In this proof we will pass several times to a subset
D 0  D with jD 0 j D . To simplify the notation we will assume each time that
D 0 D D which means that all previous reasoning can be repeated for our smaller
set D 0 .
Let n D n .X / and denote by Sn the set of all bijections from the set Mn onto
itself. Every  2 Sn determines a map p W X n ! X n defined by the formula
p .x/ D .x.1/ ; : : : ; x.n/ / for any x D .x1 ; : : : ; xn / 2 X n . It is clear that every p
is a homeomorphism such that p . n / D n . The set Pd D fp .d / W  2 Sn g is
finite for all d 2 D, so we can pass, if necessary, to a subset of D of cardinality ,
to assure that D \ Pd D fd g for any d 2 D.
We can consider that n is the minimal number for which an essential discrete
subspace exists in X n . For any d D .d1 ; : : : ; dn / 2 D let Kd D fd1 ; : : : ; dn g;
if some x 2 X belongs to -many distinct elements of the family fKd W d 2
Dg then, passing to an appropriate subset of D of cardinality  we can consider
that there is j 2 Mn such that dj D x for all d D .d1 ; : : : ; dn / 2 D; if xd D
.d1 ; : : : ; dj 1 ; dj C1 ; : : : ; dn / then the set E D fxd W d 2 Dg  X n1 n n1 is also
discrete and essential which is a contradiction with the choice of n. Thus

2 Solutions of problems 001500

475

(1) jfd 2 D W x 2 Kd gj <  for every x 2 X .


It is easy to find a set O 2 . n ; X n / such that p .O/ D O for any  2 Sn and
D \ O D ;; if F D DnD then, evidently, F \ O D ;. Besides, Pd \ O D ;
for any d 2 D and the set Pd nF 3 d is nonempty; let md D jPd nF j. Passing, if
necessary, to a relevant subset of D of cardinality  we can assume, without loss of
generality, that there is m 2 N such that md D m for any d 2 D.
Choose, for any d D .d1 ; : : : ; dn / 2 D a family fU1d ; : : : ; Und g of open subsets
of X with the following properties:
(2)
(3)
(4)
(5)

the family fU1d ; : : : ; Und g is disjoint;


if U d D U1d  : : :  Und then U d \ D D fd g;
Uid1  : : :  Uidn  O whenever i1 ; : : : ; in 2 Mn are not all distinct;
d
d
 : : :  U.n/
/ \ D D ; for any d 2 D and  2 Sn such that p .d /
.U.1/
F [ fd g.

Since i w.X / D , we can choose a base B of cardinality  of some Tychonoff


topology  on X weaker than .X /; let X 0 D .X; /. There is no loss of generality
to consider that B ; for any B 2 B. From now on the bar denotes the closure in
X and all topological properties in which the space is not mentioned are meant to
hold in the space X .
Apply Fact 1 to the space X 0 to find a finite set A  X such that, for any finite
B  X nA, there is U 2 .B; X 0 / for which the set fd 2 D W Kd \ clX 0 .U / D ;g
has cardinality . It follows from (1) that only < -many elements of the family
D D fKd W d 2 Dg meet A, so, passing if necessary, to a subset of D of cardinality
, we can assume, without loss of generality, that Kd \ A D ; for any d 2 D.
Our next step is to consider, for every k 2 N, the family Wk of all 3k-tuples
.W1 ; : : : ; Wk ; V1 ; : : : ; Vk ; r1 ; : : : ; rk / 2 B 2k  Qk such that
Wi 2 B; Vi 2 B; ri 2 I \ Q for all i 2 Mk ;
V i  WS
i and V i is functionally separated from X nWi for all i 2 Mk ;
if W D i2Mk Wi then W \ A D ; and jfd 2 D W W \ Kd D ;gj D ;
the family fW i W i  kg is disjoint.
S
It is straightforward that jWk j   for any k 2 !, so if W D fWk W k 2 Ng
then jWj  . For any elementS D .W1 ; : : : ; Wk ; V1 ; : : : ; Vk ; r1 ; : : : ; rk / of the
family W let k D k; W 
D ik Wi and Ri ./ D ri for all i  k. Using the
property (8) it is easy to construct an injection ' W W ! D such that W 
\K'./ D
; for any  2 W.
Fix  2 W; if './ D d D .d1 ; : : : ; dn / then we can apply Fact 5 and Fact 6 to
choose a continuous function f W X ! 1; 2
and a set Hi 2 .di ; X / for any
i  n with the following properties:
S
(10) Hi  Uid for all i  n and . in H i / \ .W 
[ A/ D ;;
S
(11) f .di / D
S2 for all i  n and f .x/ D 0 for any x 2 X n.W 
[ . in Hi //.
(12) f .X n. in Hi //  I and f .Vi / D fRi ./g for all i  k .
(6)
(7)
(8)
(9)

We claim that the set I D ff 2 Cp .X; I/ W f .A/  f0gg is contained in the


closure in Cp .X / of the set D ff W  2 Wg. It suffices to show that, for any

476

2 Solutions of problems 001500

finite B D fx1 ; : : : ; xk g  X nA and G1 ; : : : ; Gk 2  .I/, there is f 2 such that


f .xi / 2 Gi for any i  k. First choose r1 ; : : : ; rk 2 Q such that ri 2 Gi for al i  k.
Since A is the core of the family D in the space X 0 , there exist W1 ; : : : ; Wk 2 B such
that xi 2 Wi for all i  k, the family A0 D fclX 0 .Wi / W i  kg is disjoint (and hence
the collection A D fW S
i W i  kg is disjoint as well) and there -many elements
d 2 D such that Kd \ . A/ D ;.
It is easy to choose Vi 2 .xi ; X 0 / such that Vi 2 B and the set Vi is functionally
separated from X nWi in X 0 (and hence in X ) for all i 2 Mk . An immediate
consequence is that the 3k-tuple  D .W1 ; : : : ; Wk ; V1 ; : : : ; Vk ; r1 ; : : : ; rk / belongs
to W. Thus f .xi / D ri 2 Gi for all i  k which proves that I  .
Let us show that is a discrete subspace of CA D ff 2 Cp .X / W f .A/  f0gg.
It follows from (10) and (11) that  CA . To see that is discrete, fix any  D
.W1 ; : : : ; Wk ; V1 ; : : : ; Vk ; r1 ; : : : ; rk / 2 W and let d D .d1 ; : : : ; dn / D './; the set
B D ff 2 Cp .X / W f .di / > 1 for all i 2 Mn g is open in Cp .X / and contains f ,
so it suffices to establish that B \ D ff g. Assume toward a contradiction, that
f 2 B for some  2 Wnfg; then a D .a1 ; : : : ; an / D './ d .
For any i 2 Mn we have f .di / > 1 which, together with (10)(12), implies
that di 2 Ujai for some ji 2 Mn . If jk D jl for some distinct k; l 2 Mn then
it follows from (4) that d 2 Uja1  : : :  Ujan  O which is a contradiction with
D \ O D ;. Thus the function  W Mn ! Mn defined by  .i / D ji for all
i 2 Mn , is a bijection, i.e.,  2 Sn ; let  D  1 . If b D .ak1 ; : : : ; akn / 2 Pa nF
then d.b/ D .d.k1 / ; : : : ; d.kn / / 2 Pd nF because d.b/ 2 Uka1  : : :  Ukan  X n nF
(see (5)). The property (5) also implies that d d.b/ for any b 2 Pa nF . Therefore
the set fd g [ fd.b/ W b 2 Pa nF g which has m C 1 elements, is contained in Pd nF
whereas jPd nF j D m, a contradiction. Consequently, B \ D ff g which proves
that  CA is a discrete subspace such that I  . Finally apply Fact 2 to see
that Cp .X / is d -separable.
V.424. Assume that X is a space and there exists a discrete subspace D  X  X
such that jDj D i w.X /. Prove that Cp .X / is d -separable.
Solution. If the cardinal  D i w.X / is countable then Cp .X / is even separable,
so we can assume that  > !. For any k 2 N denote by Mk the set f1; : : : ; kg.
In this proof we will pass several times to a subset D 0  D with jD 0 j D . To
simplify the notation we will assume each time that D 0 D D which means that all
previous reasoning can be repeated for our smaller set D 0 . Given a space Z say that
sets P; Q  Z are functionally separated if there exists a function f 2 C.Z; 0; 1
/
such that f .P /  f0g and f .Q/  f1g.
Let D 2 .X /; we can consider that there is no discrete subspace of X of
cardinality  because this case was already settled in Problem 423. In particular, D\
has cardinality < , so we can pass to an appropriate subset of D of cardinality
 to see that we can assume, without loss of generality, that D  X 2 n . For any
element d D .d1 ; d2 / 2 D let Kd D fd1 ; d2 g. If some x 2 X belongs to -many
elements of D D fKd W d 2 Dg then, according to our policy, we can assume that
x 2 Kd for all d 2 D and therefore D  .fxg  X / [ .X  fxg/. This shows that

2 Solutions of problems 001500

477

either jD \ .fxg  X /j D  or jD \ .X  fxg/j D . Since both sets X  fxg


and fxg  X are homeomorphic to X , a discrete space of cardinality  embeds in X
which is a contradiction. As a consequence,
(1) jfd 2 D W x 2 Kd gj <  for every x 2 X .
Choose, for any d D .d1 ; d2 / 2 D a pair fU1d ; U2d g of open subsets of X such
that U1d \ U2d D ; and, for the set U d D U1d  U2d , we have U d \ D D fd g.
Since i w.X / D , we can choose a base B of cardinality  of some Tychonoff
topology  on X weaker than .X /; let X 0 D .X; /. There is no loss of generality
to consider that B ; for any B 2 B. From now on the bar denotes the closure in
X and all topological properties in which the space is not mentioned are meant to
hold in the space X .
We can apply Fact 1 of V.423 to the space X 0 to find a finite set A  X such that,
for any finite B  X nA, there exists a set U 2 .B; X 0 / for which the cardinality
of the set fd 2 D W Kd \ clX 0 .U / D ;g is equal to . It follows from (1) that only
< -many elements of the family D meet A, so, passing if necessary, to a subset of
D of cardinality , we can assume, without loss of generality, that Kd \ A D ; for
any d 2 D.
Our next step is to consider for every k 2 N, the family Wk of all 3k-tuples
.W1 ; : : : ; Wk ; V1 ; : : : ; Vk ; r1 ; : : : ; rk / 2 B 2k  Qk such that
Wi 2 B; Vi 2 B; ri 2 I \ Q for all i 2 Mk ;
V i  WS
i and V i is functionally separated from X nWi for all i 2 Mk ;
if W D i2Mk Wi then W \ A D ; and jfd 2 D W W \ Kd D ;gj D ;
the family fW i W i  kg is disjoint.
S
It is straightforward that jWk j   for any k 2 !, so if W D fWk W k 2 Ng
then jWj  . For any elementS D .W1 ; : : : ; Wk ; V1 ; : : : ; Vk ; r1 ; : : : ; rk / of the
family W let k D k; W 
D ik Wi and Ri ./ D ri for all i  k. Using the
property (4) it is easy to construct an injection ' W W ! D such that W 
\K'./ D
; for any  2 W.
Fix  2 W; if './ D d D .d1 ; d2 / then we can apply Fact 5 and Fact 6 of V.426
to choose a continuous function f W X ! 2; 2
and a set Hi 2 .di ; X / for any
i D 1; 2 with the following properties:
(2)
(3)
(4)
(5)

(6) Hi  Uid for all i D 1; 2 and .H 1 [ H 2 / \ .W 


[ A/ D ;;
(7) f .d1 / D 2 and f .d2 / D 2 while f .x/ D 0 for any x 2 X n.W 
[
.H1 [ H2 /.
(8) f .X nH1 /  2; 1
; f .X nH2 /  1; 2
and f .Vi / D fRi ./g for all
i  k .
We claim that the set I D ff 2 Cp .X; I/ W f .A/  f0gg is contained in the
closure in Cp .X / of the set D ff W  2 Wg. It suffices to show that, for any
finite B D fx1 ; : : : ; xk g  X nA and G1 ; : : : ; Gk 2  .I/, there is f 2 such that
f .xi / 2 Gi for any i  k. First choose r1 ; : : : ; rk 2 Q such that ri 2 Gi for al i  k.
Since A is the core of the family D in the space X 0 , there exist W1 ; : : : ; Wk 2 B such

478

2 Solutions of problems 001500

that xi 2 Wi for all i  k, the family A0 D fclX 0 .Wi / W i  kg is disjoint (and hence
the collection A D fW S
i W i  kg is disjoint as well) and there -many elements
d 2 D such that Kd \ . A/ D ;.
It is easy to choose Vi 2 .xi ; X 0 / such that Vi 2 B and the set Vi is functionally
separated from X nWi in X 0 (and hence in X ) for all i 2 Mk . An immediate
consequence is that the 3k-tuple  D .W1 ; : : : ; Wk ; V1 ; : : : ; Vk ; r1 ; : : : ; rk / belongs
to W. Thus f .xi / D ri 2 Gi for all i  k which proves that I  . It is immediate
from the definition that  CA D ff 2 Cp .X / W f .A/  f0gg.
Fix any  D .W1 ; : : : ; Wk ; V1 ; : : : ; Vk ; r1 ; : : : ; rk / 2 W and consider the point
d D .d1 ; d2 / D './; the set B D ff 2 Cp .X / W f .d1 / > 1 and f .d2 / < 1g is
open in Cp .X / and contains the function f , so it suffices to establish the equality
B \ D ff g. Assume toward a contradiction, that f 2 B for some  2 Wnfg;
then a D .a1 ; a2 / D './ d .
We have f .d1 / > 1 and f .d2 / < 1 which, together with (6)(8), implies that
d1 2 U1a and d2 2 U2a ; this shows that d 2 U a contradicting U a \ D fag.
Consequently, B \ D ff g and hence  CA is a discrete subspace such that
I  . Finally apply Fact 2 of V.423 to see that Cp .X / is d -separable.
V.425. Let X be a space such that the cardinal  D i w.X / has uncountable
cofinality. Prove that the following conditions are equivalent:
(i)
(ii)
(iii)
(iv)

.Cp .X //n is d -separable for all n  2;


.Cp .X //n is d -separable for some n  2;
Cp .X /  Cp .X / is d -separable;
for some m 2 N there is a discrete set D  X m with jDj D .

Solution. For any n 2 N let Bn D f.x1 ; : : : ; xn / 2 Rn W x12 C : : : C xn2  1g be


the closed ball of radius 1 centered at the point on 2 Rn whose all
qcoordinates are

equal to zero. Given a point x D .x1 ; : : : ; xn / 2 Rn let jxjn D x12 C : : : C xn2 ;


for any k 2 N denote by Mk the set f1; : : : ; kg. For any points a; b 2 Rn the set
a; b
D ft a C .1  t / b W t 2 0; 1
g is the line segment in the space Rn which
connects a and b. If Z is a space then we say that sets P; Q  Z are functionally
separated if there exists a function f 2 C.Z; 0; 1
/ such that f .P /  f0g and
f .Q/  f1g.

Fact 1. If Z is a space such that the cardinal  D i w.Z/ has uncountable cofinality
and Cp .Z/ is d -separable then, for some n 2 N, there is a discrete E  Z n with
jEj D .
Proof. Apply Problem 422 to see that there exists a discrete subspace D  Cp .Z/
such that jDj D d.Cp .Z// D i w.Z/ D . Let B be the family of all nonempty open
intervals of R with rational endpoints. For any m 2 N if x D .x1 ; : : : ; xm / 2 Z m
and B D .B1 ; : : : ; Bm / 2 B m then the set x; B
D ff 2 Cp .Z/ W f .xi / 2 Bi for
all i  mg is open in Cp .Z/ and the family C D fx; B
W x 2 Z m and B 2 B m for
some m 2 Ng is a base in Cp .Z/.
For any f 2 D fix a set Uf 2 C such that Uf \ D D ff g and Uf D xf ; Bf

f
f
f
f
where xf D .x1 ; : : : ; xmf / 2 Z mf ; Bf D .B1 ; : : : ; Bmf / 2 B mf , the family

2 Solutions of problems 001500


f

479
f

fB1 ; : : : ; Bmf g is disjoint and f .xi / 2 Bi for all i  mf . Since B is countable


and cf./ > !, there is n 2 N such that, for some D 0  D of cardinality  and
B D .B1 ; : : : ; Bn / 2 B n , we have Bf D B for any f 2 D 0 .
Given distinct f; g 2 D 0 if xf D xg then Uf D xf ; B
D xg ; B
D Ug
and hence f 2 Ug which is a contradiction with the choice of Ug . Therefore the
correspondence f ! xf is an injection and hence the set E D fxf W f 2 D 0 g 
X n has cardinality . Fix a function f 2 D 0 ; the set W D f 1 .B1 /: : :f 1 .Bn /
g
is open in Z n and xf 2 W . If xg 2 W for some g 2 D 0 nff g then f .xi / 2 Bi
for all i  n and hence f 2 xg ; B
D Ug which is a contradiction. Therefore
W \ E D fxf g and hence E is a discrete subset of X n with jEj D , i.e., Fact 1 is
proved.
Returning to our solution observe that the implications (i)H)(ii) and (i)H)(iii)
are trivial. Suppose that, for some n 2 N, the space .Cp .X //n ' Cp .X f1; : : : ; ng/
is d -separable and let Xn D X  f1; : : : ; ng. Fact 1 shows that there is m 2 N and
a discrete subset E  .Xn /m such that jEj D . The space .Xn /m being a union
of finitely many homeomorphic copies of X m , there is a discrete D  X m with
jDj D . This shows that (ii)H)(iv) and (iii)H)(iv), so all that is left is to prove
that (iv)H)(i).
Fix any n 2 Nnf1g and let m 2 N be the minimal number for which there is a
discrete subspace in X m of cardinality . It follows from Problems 424 and 419 that
we can assume, without loss of generality, that m > 2.
Choose distinct points w1 ; : : : ; wm 2 Rn such that jwi jn D 2 for all i  m (it
is precisely at this step where we apply the fact
S that n  2). Consider the segment
Ji D on ; wi
for any i  m and let J D im Ji . For every i  m fix a set
Qi 2 .wi ; Rn / such that Qi \ .Bn [ .J nJi // D ; and the family fQ1 ; : : : ; Qm g is
disjoint.
Let D be a discrete subspace of X m with jDj D ; in this proof we will pass
several times to a subset D 0  D with jD 0 j D . To simplify the notation we
will assume each time that D 0 D D which means that all previous reasoning can
be repeated for our smaller set D 0 . Observe first that m .X / is the finite union of
spaces homeomorphic to X m1 , so if jD \ m .X /j D  then a discrete space of
cardinality  embeds in X m1 which is a contradiction. Therefore we can assume,
without loss of generality, that D  X m n m .X /.
For any point d D .d1 ; : : : ; dm / 2 D let Kd D fd1 ; : : : ; dm g; if some x 2 X
belongs to -many distinct elements of the family fKd W d 2 Dg then, passing to an
appropriate subset of D of cardinality  we can consider that there is j 2 Mm such
that dj D x for all d D .d1 ; : : : ; dm / 2 D; if xd D .d1 ; : : : ; dj 1 ; dj C1 ; : : : ; dm /
then the set E D fxd W d 2 Dg  X m1 is also discrete and jEj D  which is a
contradiction with the choice of m. Thus
(1) jfd 2 D W x 2 Kd gj <  for every x 2 X .
Choose, for any d D .d1 ; : : : ; dm / 2 D a family fU1d ; : : : ; Umd g of open subsets
of X such that the family fU1d ; : : : ; Umd g is disjoint and for the set U d D U1d  : : : 
Umd , we have U d \ D D fd g.

480

2 Solutions of problems 001500

Since i w.X / D , we can choose a base B of cardinality  of some Tychonoff


topology  on X weaker than .X /; let X 0 D .X; /. From now on the bar denotes
the closure in X and all topological properties in which the space is not mentioned
are meant to hold in the space X .
Apply Fact 1 of V.423 to the space X 0 to see that there exists a finite set A  X
such that, for any finite B  X nA, we can find U 2 .B; X 0 / for which the set fd 2
D W Kd \ clX 0 .U / D ;g has cardinality . It follows from (1) that only < -many
elements of the family D D fKd W d 2 Dg meet A, so, passing if necessary,
to a subset of D of cardinality , we can assume, without loss of generality, that
Kd \ A D ; for any d 2 D.
Our next step is to consider, for every k 2 N, the family Wk of all 3k-tuples
.W1 ; : : : ; Wk ; V1 ; : : : ; Vk ; r1 ; : : : ; rk / 2 B 2k  .Qn /k such that
Wi 2 B; Vi 2 B; ri 2 Bn \ Qn for all i 2 Mk ;
V i  WS
i and V i is functionally separated from X nWi for all i 2 Mk ;
if W D i2Mk Wi then W \ A D ; and jfd 2 D W W \ Kd D ;gj D ;
the family fW i W i  kg is disjoint.
S
It is straightforward that jWk j   for any k 2 !, so if W D fWk W k 2 Ng
then jWj  . For any element  DS.W1 ; : : : ; Wk ; V1 ; : : : ; Vk ; r1 ; : : : ; rk / of the
family W let k D k; W 
D
ik Wi and Ri ./ D ri for all i  k.
Using the property (4) it is easy to construct an injection ' W W ! D such that
W 
\ K'./ D ; for any  2 W.
Fix  2 W; if './ D d D .d1 ; : : : ; dm / then we can apply Fact 5 and Fact 6 of
V.423 to choose a continuous function f W X ! Bn [ J and a set Hi 2 .di ; X /
for any i  m with the following properties:
S
(6) Hi  Uid for all i  m and . im H i / \ .W 
[ A/ D ;;
S
(7) f .di / D wi for all i  m and f .x/ D on for any x 2 X n.W 
[. im Hi //.
(8) f1 .Qi /  Hi and f .Vi / D fRi ./g for all i  k .
(2)
(3)
(4)
(5)

The set I D ff 2 Cp .X; Bn / W f .A/  fon gg is contained in the closure


in Cp .X; Rn / of the set D ff W  2 Wg. To prove that, it suffices to show
that, for any finite B D fx1 ; : : : ; xk g  X nA and G1 ; : : : ; Gk 2  .Bn /, there
is f 2 such that f .xi / 2 Gi for any i  k. First choose r1 ; : : : ; rk 2 Qn
such that ri 2 Gi for al i  k. Since A is the core of the family D in the space
X 0 , there exist W1 ; : : : ; Wk 2 B such that xi 2 Wi for all i  k, the family
A0 D fclX 0 .Wi / W i  kg is disjoint (and hence the collection A D fW
Si W i  kg is
disjoint as well) and there -many elements d 2 D such that Kd \ . A/ D ;.
By Fact 5 of V.423 we can choose Vi 2 .xi ; X 0 / such that Vi 2 B
and the set Vi is functionally separated from X nWi in X 0 (and hence in
X ) for all i 2 Mk . An immediate consequence is that the 3k-tuple  D
.W1 ; : : : ; Wk ; V1 ; : : : ; Vk ; r1 ; : : : ; rk / belongs to W. Thus f .xi / D ri 2 Gi for
all i  k which proves that I  .
It follows from (6) and (7) that  CA D ff 2 Cp .X; Rn / W f .A/  fon gg;
to see that is discrete, fix any  D .W1 ; : : : ; Wk ; V1 ; : : : ; Vk ; r1 ; : : : ; rk / 2 W
and let d D .d1 ; : : : ; dm / D './; the set B D ff 2 Cp .X / W f .di / 2 Qi for

2 Solutions of problems 001500

481

all i 2 Mm g is open in Cp .X; Rn / and contains f , so it suffices to establish that


B \ D ff g. Assume toward a contradiction, that f 2 B for some  2 Wnfg;
then a D .a1 ; : : : ; am / D './ d .
For any i 2 Mm , it follows from f .di / 2 Qi , together with (6)(8), that di 2
Uia . An immediate consequence is that d 2 U a which contradicts the fact that
U a \ D D fag. Therefore B \ D ff g which proves that  CA is a discrete
subspace such that I  . Finally apply Fact 2 of V.423 to see that the space
.Cp .X //n ' Cp .X; Rn / is d -separable. This proves the implication (iv)H)(i) and
makes our solution complete.
V.426. Prove that
(a) if supfs.X n / W n 2 Ng > i w.X / then Cp .X / is d -separable;
(b) if K is a Corson compact space then Cp .K/ is d -separable;
(c) if X is a metrizable space then Cp .X / is d -separable.
Solution. (a) Let D i w.X / and fix a condensation ' W X ! Y of X onto a space
Y with w.Y /  . Then w.Y n /  and hence n .Y / is a G -subset of Y n for
every n 2 N. For any x D .x1 ; : : : ; xn / 2 X n let ' n .x/ D .'.x1 /; : : : ; '.xn //;
it is easy to see that ' n W X n ! Y n is a condensation such that ' n . n .X // D
n .Y /. Therefore n .X / is a G -set in X n for each n 2 N.
Take the minimal n 2 N such that there is a discrete D  X n with jDj > .
Since n D n .X / is the finite union of subspaces homeomorphic to X n1 ,
the set D \ n has cardinality at most , so, passing if necessary to the relevant
n
subset of D of cardinality > , we can consider
T that D  X n n . Choose a
family U  .X
S  X / such that jU j  and U D n . It follows form the
equality D D fDnU W U 2 U g that jDnU j > for some U 2 U . Letting
E 0 D DnU we obtain a set E 0  X n n n such that E 0 \ n D ; and jE 0 j > .
Take any set E  E 0 with jEj D ; then E  X n n n while jEj D i w.X / and
E \ n D ;. Finally, apply Problem 423 to conclude that Cp .X / is d -separable.
(b) Apply Fact 1 of V.421 to find a discrete set D  K  K such that jDj D d.K/.
It follows from CFS-121 that w.K/ D d.K/, so we have a discrete subspace
D  K  K such that jDj D w.K/ D i w.K/. Now, Problem 424 shows that
Cp .K/ is d -separable.
(c) It will be easy to complete our solution once we prove the following fact.
Fact 1. If Y is a metrizable space then there exists a disjoint family U   .Y /
such that jU j D w.Y /. Consequently, there exists a discrete subspace D  Y such
that jDj D w.Y /.
Proof. Our Fact
S is an easy exercise if w.Y /  !, so we assume that w.Y / > !. Fix
a base B D n2! Bn   .Y / of the space Y such that the family Bn is discrete for
every n 2 !. If  D w.Y / has uncountable cofinality then it follows from jBj  
that jBn j   for some n 2 !. Choosing any U  Bn with jU j D  we obtain the
promised family U .
Now assume that fn W n 2 !g is an increasing sequence of regular uncountable
cardinals such that  D supfn W n 2 !g. Say that a set U 2  .Y / is adequate

482

2 Solutions of problems 001500

if w.V / D w.U / for any V 2  .U /. For any U 2  .Y / consider the cardinal


 D nnfw.V / W V 2  .U /g and choose V 2  .U / such that w.V / D . It is
clear that V  U is an adequate set so the family W of all adequate subsets of Y
forms a -base in Y .
Suppose that U is an adequate subset of Y with w.U / D . Since U is infinite,
we can find a disjoint faithfully indexed family fUn W n 2 !g of nonempty open
subsets of U . It follows from c.Un / D w.Un / D  > n that S
there exists a disjoint
family Un   .Un / such that jUn j D n . The family U D n2! Un   .Y / is
disjoint and jU j D . Therefore we can assume that w.U / <  for any adequate set
U  Y.
S
Let V be a maximal disjoint subfamily of W. It is straightforward that G D V
is dense in Y and hence w.G/ D w.Y / D  (recall that Y is a metrizable space).
If jVj D  then letting U D V we obtain the promised family U . If not then there
exists n 2 ! such that jVj  n .
Assume first that there exists m 2 ! such that w.V /  m for all V 2 V. Then
w.G/ D nw.G/  m n <  which is a contradiction. Therefore we can choose
a faithfully indexed family fVn W n 2 !g  V such that w.Vn / > n for any n 2 !.
Since also c.Vn / D w.Vn / > n , we can choose a disjoint family Un ofSnonempty
open subsets of Vn such that jUn j D n for every n 2 !. Then U D n2! Un is
a disjoint family of nonempty open subsets of Y such that jU j D , so we settled
the first part of our Fact. If we take a point xU 2 U for every U 2 U then the set
D D fxU W U 2 U g  Y is discrete and jDj D  D w.Y /, so Fact 1 is proved.
Finally observe that if X is a metrizable space then we can apply Fact 1 to find
a discrete subspace D 0  X such that jD 0 j D w.X /  i w.X /. Therefore we can
extract a set D  D 0 such that jDj D i w.X /, so Problem 423 is applicable to
conclude that Cp .X / is d -separable.
V.427. Prove that if Cp .X / is a Lindelf -space then it is d -separable.
Solution. Say that a space is simple if it has at most one non-isolated point.
A nonempty space will be called 0-concentrated if it is countable. If n 2 N and
we have defined m-concentrated spaces for any natural number m < n say that a
space Z is n-concentrated if there is a point a 2 Z such that, for any U 2 .a; Z/,
the set ZnU is the union of a countable family of .n  1/-concentrated
S spaces. This
defines a class Cn of n-concentrated spaces for all n 2 !; let C D fCn W n 2 !g.
Let us prove that
(1) for every
S n 2 ! the class Cn is closed-hereditary and, for any space Z such that
Z D k2! Zk and Zk 2 Cn for each k 2 !, we have Z 2 CnC1 .
Let Pn be the statement that Cn is closed-hereditary and denote by Qn the
statement which says that any countable union of elements of Cn belongs to CnC1 .
Since every subspace of a countable S
space is countable, the statement P0 is true.
Now if fZk W k 2 !g  C0 and Z D k2! Zk then Z is countable and nonempty.
Take any point a 2 Z; then ZnU is countable for any U 2 .a; Z/, so Z 2 C1 .
This shows that Q0 also holds.

2 Solutions of problems 001500

483

Proceeding inductively assume that n 2 N and we proved that Pi and Qi are


fulfilled for all i < n. Suppose that Z 2 Cn and F is a closed subspace of Z. Pick
a point a 2 A such that ZnU is the countable union of elements of Cn1 for any
U 2 .a; Z/. If a F then F is the countable union of elements of Cn1 , so F 2 Cn
by the induction hypothesis.
Now if a 2 F then take any V 2 .a; F / and pick a set U 2 .a; Z/ S
such that
U \F D V . There exists a family fYk W k 2 !g  Cn1 such that ZnU D k2! Yk .
For every k 2 !Sthe set Fk0 D F \ Yk belongs to Cn1 by the induction hypothesis;
since F nV D k2! Fk0 , we conclude that the point a witnesses that F 2 Cn and
hence Pn is proved. S
Assume that Z D k2! ZS
k and Zk 2 Cn for every k 2 !. Fix any point a 2 Z;
if U 2 .a; Z/ then ZnU D k2! .Zk nU /. It follows from Pn that Zk nU 2 Cn for
all k 2 !, so the point a witnesses that Z 2 CnC1 and hence we settled Qn . Thus
our induction procedure can be continued to show that Pn and Qn are true for all
n 2 !, i.e., (1) is proved.
If Z 2 Cn then letting Zk D Z for all k 2 ! we can apply (1) to see that
Z 2 CnC1 , i.e., Cn  CnC1 for all n 2 !. Now it takes a trivial induction to establish
that
(2) Cm  Cn whenever m  n.
Let us show that every class Cn behaves properly under continuous images, i.e.,
(3) for any n 2 !, if Z 2 Cn and Y is a continuous image of Z then Y 2 Cn .
Since any image of a countable set is countable, the property (3) is true for n D 0.
Proceeding inductively assume that m 2 N and we have (3) for all n < m. If Z 2 Cm
and f W Z ! Y is a continuous onto map then fix a point a 2 Z such that ZnU is
the countable union of elements of Cm1 for any U 2 .a; Z/. Let b D f .a/ and
take any V 2 .b; Y /.
S
There exists a family fZk W k 2 !g  Cm1 such that Znf 1 .V / D k2! Zk .
By the induction hypothesis
S the set Yk D f .Zk / belongs to Cm1 for all k 2 !, so
it follows from Y nV D k2! Yk that the point b witnesses that Y 2 Cm and hence
(3) is proved.
It turns out that the family C is well behaved under products, namely,
(4) if Y 2 Cn and Z 2 Cm then Y  Z 2 CmCnC1 .
We will prove (4) by induction on k D nnfm; ng. If m D 0 then Z is countable
and hence Y  Z is the countable union of spaces homeomorphic to Y . Therefore it
follows from (1) that Y  Z 2 CnC1 D CnCmC1 . An analogous argument shows that
(4) holds for any m 2 ! whenever n D 0.
Assume that l 2 N and (4) is verified for any k < l. If m D nnfm; ng D l then
n  l and hence we can choose points a 2 Y and b 2 Z which witness that Y 2 Cn
and Z 2 Cm . Take an arbitrary open neighborhood U of the point z D .a; b/ in
the space Y  Z. Choose V 2 .a; Y / and W 2 .b; Z/ such that V  W  U ;
W p 2 !g  Cn1 and fZq W q 2 !g  Cm1 such that
there
S exist families fYp S
p2! Yp D Y nV and
q2! Zq D ZnW . By the induction hypothesis we have

484

2 Solutions of problems 001500

Yp0 D Yp Z 2 CmCn and Zq0 D Y Zq 2 CmCnSfor all p; q 2 !. If F


S D .Y Z/nU
then it follows from (1) and the equality F D . p2! .Yp0 \ F // [ . q2! .Zq0 \ F //
that .Y Z/nU is the countable union of elements of CmCn for any U 2 .z; Y Z/.
Therefore the point z witnesses that Y Z 2 CmCnC1 as promised. The case of n D l
is symmetric, so we carried out the induction step and hence (4) is proved.
It turns out that
(5) for any n 2 !, if Y 2 Cn then Y is d -separable;
To establish (5) observe that it is true for n D 0 and assume that m 2 N and we
proved it for all n < m. Take a space Z 2 Cm and fix a point a 2 Z such that ZnU
can be covered by a countable family from Cm1 for any U 2 .a; Z/. Let U be
a maximal disjoint family of open subsets of Z such that a U for any U 2 U .
Every U 2 U can be covered by a countable family of spaces from Cm1 which
are d -separable by the induction
an easy consequence is that U is also
S hypothesis;
L
d -separable and hence V D U ' fU W U 2 U g is d -separable as well. Thus
Z is d -separable because V is a dense d -separable subspace of Z; this shows that
(5) is proved.
Fact 1. If both Z and Cp .Z/ are Lindelf then there is a closed simple subspace
L  Cp .Z/ which separates the points of Z.
Proof. By CFS-285, there is a condensation ' W Z ! Y  .A/ for some infinite
set A; choosing a larger set A if necessary, we can assume, without loss of generality,
that jAj > !. For any f 2 Cp .Y / let '  .f / D f '; then '  W Cp .Y / ! Cp .Z/
is a homeomorphism (see TFS-163).
The space D .A/ being homeomorphic to Cp .L.// for  D jAj (see CFS106), the space L./ can be embedded in Cp . / in such a way that it generates
the topology of . The restriction  W Cp . / ! Cp .Y / maps L./ onto a set
M  Cp .Y / which generates the topology of Y (see Fact 1 of U.285); in particular,
M separates the points of Y and therefore the set N 0 D '  .M / separates the points
of Z. The space N 0 is a continuous image of L./, so it is concentrated around a
point, i.e.,
(6) there is a set N 0  Cp .Z/ which is concentrated around a point and separates
the points of Z.
It is easy to see that we can consider that the set N 0 is concentrated around the
function u which is identically zero on Z. Take a homeomorphism  W R ! .1; 1/
such that .0/ D 0 and let N D f f W f 2 N 0 g. Then N  Cp .Z; .1; 1//
while N is concentrated around the point u (see TFS-091) and separates the points
of Z.
Applying Fact 9 of U.285 we can find a condensation  of Cp .Z/ into some
.B/. It is evident that .N /  .B/ is concentrated around the point .u/. By
Fact 1 of U.289, there exists a family fKn W n 2 !g of simple compact spaces such
that .u/ is the unique point which can be non-isolated in some Kn and .N / D
S
fKn W n 2 !g. As a consequence, the set Kn0 D  1 .Kn / is closed in Cp .Z/ and

2 Solutions of problems 001500

485

all points of Kn0 nfug are isolated in Kn0 for S


each n 2 !. Let Ln D 2n Kn0 for every
n 2 !; it is immediate that the set L D n2! Ln still separates the points of the
space Z.
Fix a function f 2 Cp .Z/nfug and a point x 2 Z such that jf .x/j > 2n for
some n 2 !. The set U D fg 2 Cp .Z/ W jg.x/j > 2n g is an open neighborhood of
f which does not meet the set Lm for every
S m  n.
If f L then the set U 0 D Cp .Z/n. fLm W m < ng/ is also a neighborhood of
f in Cp .Z/ and hence V D U \ U 0 2 .f; Cp .Z// while V \ L D ;. This proves
that L is closed in Cp .Z/.
S
If f 2 L then U \ L is closed and discrete in U because U \ L  fLm W
m < ng while every Lm is closed and discrete
S in Cp .Z/nfug. Therefore we can find
a set U 0 2 .f; Cp .Z// for which U 0 \ . fLm W m < ng/ D ff g. Then, for the set
V D U \ U 0 we have V \ L D ff g, so all points of Lnfug are isolated in L and
hence Fact 1 is proved.
Fact 2. If Cp .Z/ is a Lindelf -space then there exists a closed simple subspace
of Cp .Z/ which separates the points of Z. In other words, Lindelf -property of
Z can be omitted in Fact 1.
Proof. Observe that Z and Cp .Z/ have to be Lindelf -spaces (see CFS-206
and CFS-234); besides, the restriction map  W Cp .Z/ ! Cp .Z/ is a condensation
such that jA W A ! .A/ is a homeomorphism for any countable set A  Cp .Z/
(TFS-437). By Fact 1 there is a simple subspace T 0  Cp .Z/ which separates the
points of Z; therefore T D .T 0 / separates the points of Z.
Any simple Lindelf space is concentrated around its non-isolated point, so T 0
is concentrated around a point f 2 T 0 . It is evident that T is concentrated around
the point g D .f /. If h 2 Cp .Z/nfgg then there is a neighborhood V of g in
Cp .Z/ such that h V . It is clear that W D Cp .Z/nV is an open neighborhood
of h which contains at most countably many elements of T ; let E D T \ W and
E 0 D  1 .E/. If h 2 Enfhg then h0 D  1 .h/ is in the closure of E 0 nfh0 g because
j.E 0 [fh0 g/ W E 0 [fh0 g ! E [fhg is a homeomorphism. However, this contradicts
the fact that T 0 nff g is closed and discrete in Cp .Z/nff g.
This contradiction shows that T nfgg is closed and discrete in Cp .Z/nfgg and
hence T is a simple closed subspace of Cp .Z/ which separates the points of Z, i.e.,
Fact 2 is proved.
Returning to our solution apply Fact 2 to find a simple closed set T  Cp .X /
which separates the points of X . Since T is Lindelf , it is concentrated around
a point f 2 T and hence T 2 C1 . If A.T / is the minimal subalgebra of
SCp .X /
that contains T then A.T / is dense in Cp .X / (see TFS-192) and A.T / D n2! An
where, for every n 2 N, there is a continuous onto map 'n W T kn  Rkn ! An for
some kn 2 N. The set Bn D 'n .T kn  Qkn / is a countable union S
of elements of C by
(3) and (4); besides, it is dense in An for any n 2 N. Thus B D n2! Bn is a dense
subspace of A.T / which can be represented as the countable union of spaces from
C. It follows from (5) that B is d -separable, so Cp .X / is also d -separable because
it contains a dense d -separable subspace.

486

2 Solutions of problems 001500

V.428. Prove that if Cp .X / is a Lindelf -space then the space X must be


hereditarily d -separable.
Solution. Recall that a family U  exp.Z/ is called weakly  -point-finite if there
exists a collection fUn W n 2 !g  exp.U / such
S that, for any point x 2 Z, if
Nx D fn 2 ! W Un is point-finite at xg then U D fUn W n 2 Nx g. Say that a space
Z is simple if Z has at most one non-isolated point.
Given a set A and a family
S s D fAn W n 2 !g  exp.A/ consider the subspace
s .A/ D fx 2 RA W A D fAn W jx 1 .Rnf0g/ \ An j < !gg of the space RA ; it
is easy to see that s .A/  .A/. We say that a space Y is a s -product of real
lines if there exists a set A and a family s D fAn W n 2 !g  exp.A/ such that
Y D s .A/. For any x 2 .A/ let supp.x/ D x 1 .Rnf0g/.
A family U  exp.Z/ is T0 -separating if, for any distinct points x; y 2 Z there
exists U 2 U such that U \ fx; yg is a singleton. If Z is a space then U  exp.Z/ is
called a Gulko family on Z if U is T0 -separating, weakly  -point-finite and consists
of cozero subsets of Z.
Fact 1. A space Z has a Gulko family if and only if Z can be condensed into some
s -product of real lines.
Proof. Suppose that U is a weakly  -point-finite T0 -separating family of cozero
subsets of Z and fix a sequence fUn W n 2 !g of subfamilies of U which witnesses
that U is weakly  -point-finite. For any U 2 U fix a function fU 2 Cp .Z/ such
that fU1 .0/ D ZnU and consider the diagonal product f D ffU W U 2 U g.
Let Y D f .Z/ and A D ffU W U 2 U g; then Y  RA and f W Z ! Y is a
condensation: this easily follows from the fact that U is a T0 -separating family.
The family U being point-countable (see Fact 2 of U.290), the set Y is contained
in .A/. Let An D ffU W U 2 Un g for every n 2 ! and consider the family
s D fAn W n 2 !g. Given any y 2 .A/ let My D fn 2 ! W the set supp.y/ \ An is
finiteg.
Take any y 2 Y and fU 2 A; there is a unique x 2 Z such that f .x/ D y. The
family U is weakly  -point-finite, so there exists n 2 ! such that Un is point-finite
at x and U 2 Un . Then fU 2 An and n 2 My because
S supp.y/ \ An coincides with
the finite set ffV W x 2 V 2 Un g. Therefore A D fAn W n 2 My g for any y 2 Y
and hence Y  s .A/, i.e., we proved necessity.
Now assume that A is a set, s D fAn W n 2 !g is a family of subsets of A
and f W Z ! Y is a condensation for some Y  s .A/. Fact 1 of U.290 implies
that there exists a Gulko family on the space s .A/. It is trivial that existence
of a Gulko family is hereditary, so Y also has a Gulko family V. Letting U D
ff 1 .V / W V 2 Vg we obtain a Gulko family on Z, so we settled sufficiency and
hence Fact 1 is proved.
Fact 2. If Cp .Z/ is a Lindelf -space then the space Z has a Gulko family and
hence Z can be condensed into a s -product of real lines.
Proof. Apply Fact 2 of V.427 to find a closed simple subspace T  Cp .Z/ which
separates the points of Z. Let e.x/.f / D f .x/ for any x 2 Z and f 2 T ; then

2 Solutions of problems 001500

487

e.x/ 2 Cp .T / for any x 2 Z and e W Z ! Cp .T / is a condensation by TFS-166.


Let Y D e.Z/; since T is a simple Lindelf -space, we can apply CFS-274 to see
that Y has a Gulko family V. Then U D fe 1 .V / W V 2 Vg is a Gulko family in
Z, so Fact 2 is proved.
Fact 3. If Z is a space and U is a weakly  -point-finite family of nonempty open
subsets of Z then there is a  -discrete subspace D  Z such that D \ U ; for
any U 2 U .
Proof. Assume that U   .X / and the collection fUn W n 2 !g  exp.U /
witnesses that U is a weakly  -point-finite family. Denote by An the set of points at
which Un has finite order
S and let Vn D fU 2 Un W U \ An ;g for each n 2 !. We
are going to show that fVn W n 2 !g D U .
Take any U 2 U and x 2 U . According to the definition of weakly  -point-finite
family, there exists n 2 ! such that U 2 Un and Un has finite order at x. Then
x 2 An and x 2 U implies U 2 Vn .
For each n 2 ! the family Wn D fU \ An W U 2 Vn g   .An / is point-finite;
by Fact 2 of U.271 there exists a  -discrete subset Bn  An such that Bn \ V ;
for every V 2 Wn . Therefore Bn \ US ; for every U 2 Vn and n 2 !; since
S
n2! Vn D U , the  -discrete set D D
n2! Bn intersects every element of U , i.e.,
Fact 3 is proved.
Fact 4. Every subspace of a s -product of real lines has a weakly  -point-finite
-base and hence the space s .A/ is hereditarily d -separable for any set A and any
family s D fAn W n 2 !g  exp.A/.
Proof. Let B D f.p; q/ W p; q 2 Q; p < q and pq > 0g; in other words, B is
the family of all nonempty rational intervals of R, which do not contain zero. It is
evident that any U 2  .R/ contains an element of B, i.e., the family B is a -base
in R. In fact, B has even stronger property: for any a 2 Rnf0g and any U 2 .a; R/,
there exists B 2 B such that a 2 B  U , i.e., B is a base in Rnf0g.
Given any points a1 ; : : : ; an 2 A, and O1 ; : : : ; On 2 .R/, consider the set
a1 ; : : : ; an I O1 ; : : : ; On
D fx 2 s .A/ W x.ai / 2 Oi for each i  ng:
It is clear that U D fa1 ; : : : ; an I O1 ; : : : ; On
W n 2 N; ai 2 A and Oi 2 .R/ for all
i  ng is a base in s .A/. If U D a1 ; : : : ; an I O1 ; : : : ; On
2 U and Bi  Oi ; Bi 2
B for all i  n then we have the inclusion V D a1 ; : : : ; an I B1 ; : : : ; Bn
 U , i.e.,
the family V D fa1 ; : : : ; an I B1 ; : : : ; Bn
W n 2 N; ai 2 A and Bi 2 B for all
i  ng is a -base of s .A/.
Denote by u the element of s .A/ for which u.a/ D 0 for all a 2 A and take any
Z  s .A/. It suffices to prove our Fact for the space Znfug. Indeed, the case when
u Z is clear; if u 2 Z and u is an isolated point of Z then for any weakly  -pointfinite -base C in the space Znfug, the family ffugg [ C is a weakly  -point-finite
-base in the space Z. If u is not isolated in Z then any -base for Znfug is also
a -base for Z, so again it suffices to find a weakly  -point-finite -base for the
space Znfug. To simplify the notation we will assume, without loss of generality,
that u Z.

488

2 Solutions of problems 001500

It turns out that


(1) the family Wn .B/ D fa1 ; : : : ; an I B1 ; : : : ; Bn
W ai 2 A for all i  ng is weakly
 -point-finite for any n 2 N and B D .B1 ; : : : ; Bn / 2 B n
Given any numbers i1 ; : : : ; in 2 ! let H.i1 ; : : : ; in / D fa1 ; : : : ; an I B1 ; : : : ; Bn
W
the set fa1 ; : : : ; an g is contained in Ai1 [ : : : [ Ain g. The collection H D
fH.i1 ; : : : ; in / W ij 2 ! for all j  ng is countable, so we can choose an
enumeration fWi W i 2 !g of the family H.
If x 2 s .A/ and W D a1 ; : : : ; an I B1 ; : : : ; Bn
2 Wn .B/ then we can choose
i1 ; : : : ; in 2 ! such that aj 2 Aij and the set Qj D supp.x/ \ Aij is finite for any
j  n. There exists m 2 ! such that H.i1 ; : : : ; in / D Wm ; then W 2 Wm and we
claim that Wm is point-finite at x.
S
Indeed, if x 2 W 0 D b1 ; : : : ; bn I B1 ; : : : ; Bn
2 Wm and bl Q D fQj W
j  ng for some l  n then there exists j  n such that bl 2 Aij nQ  Aij nQj and
therefore x.bl / D 0 Bl . As a consequence, bl 2 Q for all l  n, so .b1 ; : : : ; bn / 2
Qn . We proved that all elements b1 ; : : : ; bn I B1 ; : : : ; Bn
from Wm that can contain
x are among those for which .b1 ; : : : ; bn / 2 Qn . The set Qn being finite, we proved
that Wm is point-finite at x and hence the family fWm W m 2 !g witnesses that
Wn .B/ is weakly  -point-finite,
S i.e., (1) is proved.
It is immediate that V D fWn .B/ W n 2 N and B 2 B n g, so we can apply
Fact 1 of U.293 to convince ourselves that V is weakly  -point-finite. Therefore the
family VjZ D fV \ Z W V 2 V and V \ Z ;g is weakly  -point-finite as well.
We claim that VjZ is a -base in Z.
To prove it take any U 2  .Z/ and x 2 U . Then x./ 0 for some < ;
there exist ordinals 1 ; : : : ; n 2  and O1 ; : : : ; On 2  .R/ such that 1 D and
x 2 V D 1 ; : : : ; n I O1 ; : : : ; On
\ Z  U . If x.i / 0 then there is Bi 2 B
such that x.i / 2 Bi  Oi ; if x.i / D 0 then 0 2 Oi . This shows that we do not
lose generality if we assume that there is k 2 f1; : : : ; ng such that Oi D Bi 2 B for
all i  k and x.i / D 0 2 Oi for i D k C 1; : : : ; n.
Call a set K 0 D fi1 ; : : : ; im g  K D fkC1 ; : : : ; n g marked if there exists
a point y 2 Z such that y.i / 2 Bi for all i  k and y.ij / 2 Oij nf0g for
every j  m. Since the set K is finite, there exists a maximal marked set M D
fi1 : : : ; im g  K (which is possibly empty). This means that there is y 2 Z with
y.i / 2 Bi for all i  k and y.ij / 2 Oij nf0g for all j  m while for any z 2 Z
such that z.i / 2 Bi for all i  k and z.ij / 2 Oij nf0g for all j  m, we have
z./ D 0 for every 2 KnM .
Changing the enumeration of K if necessary, we can restrict ourselves to the case
when M D fkC1 ; : : : ; m g for some m  n. Since 0 y.i / 2 Oi for all i  m,
we can choose Bi 2 B such that y.i / 2 Bi  Oi for all i 2 fk C 1; : : : ; mg. Then
W D 1 ; : : : ; m I B1 ; : : : ; Bm
\ Z 2 VjZ because y 2 W and hence W ;;
besides, for any z 2 W we have z.i / D 0 2 Oi for all i 2 fm C 1; : : : ; ng. As a
consequence, any z 2 W belongs to V , i.e., W  V  U . This shows that, for any
U 2  .Z/ we can find a set W 2 VjZ with W  U ; therefore VjZ is a -base
in Z.

2 Solutions of problems 001500

489

Finally observe that if Z  s .A/ then there exists a weakly  -point-finite


-base B in the space Z. By Fact 3, there exists a  -discrete set D  Z such that
D \B ; for any B 2 B. It is immediate that D is dense in Z, so D is d -separable
and hence Fact 4 is proved.
Fact 5. Suppose that Z is a Lindelf -space and F is a fixed countable network
with respect to a compact cover C of the space Z. Assume additionally that F is
closed under finite intersections, and we have a condensation f W Z ! Z 0 . If
Y  Z and A  Y is a set such that f .A \ F / is dense in f .Y \ F / for any F 2 F
then Y  A.
Proof. Assume, toward a contradiction that there is a point z 2 Y nA and fix a set
C 2 C such that z 2 C . The set K D A \ C is compact, so U D Z 0 nf .K/ is an
open neighborhood of y D f .z/; take V 2 .y; Z 0 / such that V  U . Since F is
closed under finite intersections, we can choose a sequence S D fFn W n 2 !g  F
such that C  Fn and Fn  FnC1 for any n 2 ! while S is a network at C , i.e., for
any O 2 .C; Z/ there is n 2 ! with Fn  O. We have y 2 f .Y \ Fn /; the set
f .A \ Fn / being dense in f .Y \ Fn /, we can pick a point an 2 A \ Fn such that
f .an / 2 V for every n 2 !.
The sequence S D fan W n 2 !g must have an accumulation point in C . Indeed,
if every point z 2 C has an open neighborhood Oz such that the set fn 2 ! W
an 2 O
Sz g is finite then, by compactness of C , there is a finite D  C with C 
O D fOz W z 2 Dg. Then there are only finitely many n 2 ! such that an 2 O
while there exists m 2 ! with Fm  O and therefore an 2 O for all n  m. This
contradiction shows that there is an accumulation point a 2 C for the sequence S .
Then a 2 A \ C D K; since f .an / 2 V for all n 2 !, we have f .a/ 2 V by
continuity of f . Thus f .a/ 2 U \ f .K/; this contradiction shows that Fact 5 is
proved.
Fact 6. If a Lindelf -space Z condenses onto a hereditarily d -separable space
then Z is hereditarily d -separable.
Proof. Fix a condensation ' W Z ! Z 0 of the space Z onto a hereditarily d separable space Z 0 and let F be a countable network of Z with respect to a compact
cover of Z. There is no loss of generality to assume that F is closed with respect
to finite intersections. Given any Y  Z and F 2 F we can find a  -discrete set
BF  '.Y \ F / which is dense in '.Y S
\ F /. The set AF D ' 1 .BF /  Y is
 -discrete for any F 2 F and hence A D fAF W F 2 Fg is also  -discrete.
It is immediate that '.A \ F / is dense in '.Y \ F / for any F 2 F, so we
can apply Fact 5 to conclude that A is dense in Y . Therefore every Y  Z is d separable, i.e., Fact 6 is proved.
Returning to our solution observe that X is a Lindelf -space as well as
the space Cp .X / (see CFS-206 and CFS-234). By Fact 2, the space X can be
condensed onto a subset Y of a s -product of real lines. Apply Fact 4 to see the
space Y is hereditarily d -separable, so we can apply Fact 6 to see that X is also
hereditarily d -separable. This implies that X  X is hereditarily d -separable and
completes our solution.

490

2 Solutions of problems 001500

V.429. Prove that under CH,


(a) there exists a compact space K such that Cp .K/ is not d -separable;
(b) there exists a space X such that X  X is d -separable while X is not d separable.
Solution. (a) Apply SFFS-099 to see that, under CH, there exists a non-metrizable
compact space K such that hl.Cp .K// D ! and hence s.Cp .K// D !. This
implies that every  -discrete subspace of Cp .K/ is countable; if Cp .K/ is d separable then it is separable and hence K is metrizable which is a contradiction.
Thus K is a compact space such that Cp .K/ is not d -separable.
(b) By D we denote the set f0; 1g with the discrete topology. Recall that a space Z
is called Luzin if it has no isolated points and any nowhere dense subspace of
Z is countable. If A is a set then A
<! is the family of all finite subsets of A.
An open set U  D!1 is called standard if there is a finite A  !1 and a point
a 2 DA such that U D fx 2 D!1 W xjA D ag. It is clear that standard open
subsets of D!1 constitute a base in D!1 .
Fact 1. Given a countably infinite set A suppose that B  A is nonempty,
finite
S
and Hn is a nowhere dense subspace of DA for any n 2 !. If H D n2! H n then,
for any s; t 2 DB there exist hs ; ht 2 DA nH such that hs jB D s; ht jB D t and
hs j.AnB/ D ht j.AnB/.
Proof. Let  W DA ! DAnB be the projection. The sets U D fx 2 DA W xjB D sg
and V D fx 2 DA W xjB D t g are open in DA and both maps U D jU W U !
DAnB and V D jV W V ! DAnB are homeomorphisms. The sets P D U nH

and Q D V nH are dense Cech-complete


subspaces of U and V respectively, so
0
0

P D U .P / and Q D V .Q/ are dense Cech-complete


subspaces of DAnB . Any

pair of dense Cech-complete subspaces of any space have nonempty intersection


by TFS-264; if h 2 P 0 \ Q0 then it is straightforward that hs D .U /1 .h/ and
ht D .V /1 .h/ are as required, so Fact 1 is proved.
Fact 2. For any infinite cardinal  the space  D fx 2 D W jx 1 .1/j < !g is
 -discrete.
S
Proof. If n D fx 2  W jx 1 .1/j D ng for each n 2 ! then  D n2! n . Given
any point x 2 n let A D x 1 .1/. The set V D fy 2 D W yjA D xjAg is open
in D!1 . If y 2 V \ n then y./ D 1 for any 2 A and hence A  y 1 .1/.
However, jAj D jy 1 .1/j D n, so y 1 .1/ D A and hence y D x; this proves that
V \S
n D fxg and therefore every n is a discrete subspace of  . Consequently,
 D n2! n is a  -discrete space so Fact 2 is proved.
Fact 3. If CH holds then there exists a dense Luzin subspace Z of the -product
D fx 2 D!1 W jx 1 .1/j  !g of the Cantor cube D!1 such that the space Z  Z
is d -separable.

2 Solutions of problems 001500

491

Proof. For any set A  !1 let A W D!1 ! DA be the projection map defined by
A .x/ D xjA for any x 2 D!1 . Say that a set N  D!1 is canonical if there exists
a nonempty countable set A  !1 and a closed nowhere dense subset M of DA
such that N D A1 .M /. It is immediate that every canonical set is nowhere dense
in D!1 .
Fix a countable dense set P  D!1 and let fpn W n 2 !g be a faithful enumeration
of P . It is easy to findS
a family A D fAmn W m; n 2 !g of uncountable disjoint
subsets of !1 such that A D !1 . Choose a family fK W < !1 g of finite subsets
of !1 for which we have the equality fK W 2 Amn g D !1
<! for any m; n 2 !.
Suppose that N is a nowhere dense subset of D!1 and choose maximal disjoint
family U of standard openS
subsets of D!1 which are contained in D!1 nN . The family
U is countable and U D U is dense in D!1 . There exists a countable set A  !1
such that U D A1 A .U /. The set V D A .U / is open and dense in DA , so
M D DA nV is nowhere dense in DA ; the set N 0 D A1 .M / is canonical and
N  N 0 . Therefore
(1) canonical sets are cofinal in the family of all nowhere dense subsets of D!1 ,
i.e., for any nowhere dense set N  D!1 there is a canonical set N 0 such that
N  N 0 . It follows from CH that the family of all countable subsets of !1 has
cardinality !1 ; besides, the family of all closed nowhere dense subsets of DA does
not exceed !1 for any countable A  !1 . This shows that the family of all canonical
subsets of D!1 has cardinality !1 .
Therefore we can find an !1 -sequence fB W < !1 g of countable subsets of !1
such that, for any < !1 there is a closed nowhere dense N in the space DB such
that fM D B1 .N / W < !1 g is an enumeration of all canonical subsets of D!1 .
Our promised space Z will have the form fx ; y W < !1 g. To force the space
Z to be Luzin it suffices to make it dense in D!1 and guarantee that, for any < !1
the points x ; y are outside M for all > .
To obtain d -separability of the space Z  Z we will choose the points x ; y in
such a wayS
that the set Dmn D f.x ; y / W 2 Amn g is  -discrete for any m; n 2 !
and D D m;n<! Dmn is dense in Z  Z. To get  -discreteness of Dmn we will
show that the sum modulo 2 map .x ; y / ! x y D x C y  2x y takes
Dmn injectively onto a subspace of  D fx 2 D!1 W jx 1 .1/j < !g. The density
of Z in will be proved establishing density of Z  Z in  which, in turn,
will be obtained by assuring that the point .pm ; pn / belongs to the closure of the set
Dmn for all m; n 2 !.
To start carrying out the outlined program take any distinct points x0 ; y0 2 
such that K0 .x0 / D K0 .pm / and K0 .y0 / D K0 .pn / where .m; n/ 2 !  ! is the
unique pair with 0 2 Amn ; let E0 D ! [ K0 [ x01 .1/ [ y01 .1/ and fix an ordinal
0 2 E0 such that x0 .0 / y0 .0 /.
To make the inductive step assume that < !1 and we have chosen a family
fE W < g of countably infinite subsets of !1 and a set fx ; y W < g 
with the following properties:
(2) if  < < then E  E ;
(3) if < and 2 Amn then K .x / D K .pm / and K .y / D K .pn /;

492

2 Solutions of problems 001500

S
fB W  < g [ . fK W   g/  E for any < ;
x1 .1/ [ y1 .1/  E for any < ;
S
if < then there is  2 E n. fE W  < g/ such that x . / y . /;
fE .x /; E .y /g \ E .M / D ; whenever  < < ;
x y 2  for any < .
S
S
Consider the set E D fE W < g [ . fB W < g/ [ K and let
.m; n/ 2 !  ! be the unique pair such that 2 Amn . Since B  E, the set
E
H D E .M / is nowhere
S dense in D for any < , so we can apply Fact 1 to
E
find points s; t 2 D n. fH W < g/ such that sjK D pm jK ; t jK D pn jK
while sj.EnK / D t j.EnK /. Take an ordinal  2 !1 nE and let E D E [ f g;
define x ; y 2 by requiring that x jE D s; y jE D t while x . / D 0 D
1  y . / and x ./ D y ./ D 0 for any 2 !1 nE . It is straightforward
that the properties (2)(7) are fulfilled now for all  . The condition (8) is
also satisfied because EnK .x / D EnK .y / and hence the points x and y
are distinct only on finitely many coordinates. Thus our inductive procedure can be
continued to obtain a set Z D fx ; y W < !1 g  such that the properties
(2)(8) hold for all < !1 .
Given m; n 2 !, if K  ! is a finite set then there is 2 Amn such that K D K
and hence K .x / D K .pm /; K .y / D K .pn / by the property (3). This proves
that the point .pm ; pn / belongs to the closure of the set Dmn D f.x ; y / W 2 Amn g
for any m; n S
2 !. As a consequence, the set P  P is contained in the closure of
the set D D fDmn W m; n 2 !g  Z  Z which shows that Z  Z is dense in
D!1  D!1 and hence in  .
Now, if N  Z is nowhere dense then N is nowhere dense in D!1 and hence
there is < !1 such that N  M . An immediate consequence of (7) is that
x ; y M for any > and hence the set N  fx ; y W  g is countable.
This shows that Z is a Luzin space.
We have already established that D contains the set P  P in its closure, so it is
dense in ZZ. The map ' W D ! D!1 defined by '.x ; y / D x y is continuous
and '.D/   by (8). The space  is  -discrete by Fact 2, so D 0 D '.D/ is  discrete as well. If < < !1 then it follows from (5) and (6) that .x y /. / D
0 and .x y /. / D 1 which shows that '..x ; y // '..x ; y //, i.e., the map
' is a continuous bijection of D onto a  -discrete space D 0 . Thus D is a  -discrete
dense subspace of Z  Z and hence Z  Z is d -separable, so Fact 3 is proved.
(4)
(5)
(6)
(7)
(8)

Returning to our solution apply Fact 3 to see that, under CH, there exists a dense
Luzin subspace X of the set D fx 2 D!1 W jx 1 .1/j  !g S
such that X  X is
d -separable. If Y is a countable subset of then the set A D fx 1 .1/ W x 2 Y g
is countable; take any 2 !1 nA and observe that V D fx 2 W x./ D 1g
is a nonempty open subset of such that V \ Y D ;. In particular, Y is not
dense in ; this shows that d./ > !, so the space X is not separable being
dense in . Furthermore, s.X /  hl.X /  ! (see SFFS-043) and hence every
 -discrete subspace of X is countable. Therefore X is not d -separable while X  X
is d -separable.

2 Solutions of problems 001500

493

V.430. Suppose that  is an infinite cardinal, T ; is a set and Nt is a space such


that nw.N
Q t /   for all t 2 T . Assume that D is a dense subspace of the product
N D t2T Nt and f W D ! K is a continuous map of D onto a compact space K.
Prove that if .K/   then w.K/  . Deduce from this fact that if a compact
space K is a continuous image of a dense subspace of a product of cosmic spaces
then w.K/ D t .K/ D .K/.
Solution. Recall that a space is cosmic if it has a countable network. If nw.X / 
for some infinite cardinal then X is called -cosmic. If X is a space then a family
E  exp.X / separates the points of a set F  X from the points of F 0  X if for
any x 2 F and y 2 F 0 there
Qexists E 2 E such that x 2 E and y E. Q
Given a product X D a2A Xa and B  A let pB W X ! XB D a2B Xa
be the natural projection. Suppose that is an infinite cardinal and nw.Xa / 
for all a 2 A. Say that F  X is an adequate G -subset of X if there exists a set
B  A (called the support of F and denoted by supp.F /) such that jBj  and
F D pB1 .z/ for some z 2 XB . It is evident that any adequate G -subset of X is a
G -subset of X . An open subset U of the space X is called standard if there is a
finite B  A (which is also called the support
denoted by supp.U /) and
Q of U andQ
Ua 2 .Xa / for each a 2 B such that U D a2B Ua  a2AnB Xa . It is clear that
standard open subsets of X constitute a base of X .
If X is an arbitrary space and D  X is a dense subspace of X then we will
X
need an extension
operator eD
W .D/ ! .X / which is defined by the formula
S
X
eD .U / D fV 2 .X / W V \ D  U g for any U 2 .D/. It is easy to see that, for
any space X and any dense D  X , we have the following properties:
X
(1) eD
.U / \ D D U for any U 2 .D/;
X
X
(2) if U; V 2 .D/ and U  V then eD
.U /  eD
.V /;
X
X
(3) if U; V 2 .D/ and U \ V D ; then eD .U / \ eD
.V / D ;.
X
Assume that X is a space, D is a T
dense subspace of X and let e D eD
. Given a
1
map f W D ! Y , the set Q.f; y/ D fclX .e.f .U /// W U 2 .y; Y /g is a useful
extension of the set f 1 .y/ (it is an easy exercise that Q.f; y/ \ D D f 1 .y/ for
any y 2 Y ).

Fact 1.
Q Suppose that nw.Xa /  for any a 2 A. Then any G -subset of the space
X D a2A Xa is a union of adequate G -subsets of X .
Proof. Take
Tany G -set Q  X and fix a family U  .X / such that jU j 
and Q D
U . Given x 2 Q, for every U 2 U we can choose a finite
Q BU  A
U
and
a
set
W
2

.X
/
for
every
a
2
B
such
that
x
2
H
D
a
a
U
x
a2BU Wa 
S
Q
X

U
.
The
set
B
D
fB
W
U
2
U
g
has
cardinality
not
exceeding
;
a
U
a2AnBU
Q
1
let y D B .x/. If z 2 pB .y/ then pBU .z/ D yjBU D pBU .x/ 2 a2BU Wa which
T
shows that z 2 HxU for any U 2 U . Therefore z 2 U for any z 2 Hx D pB1 .y/,
i.e., HS
x is an adequate G -set such that x 2 Hx  Q for each x 2 Q. Thus
Q D fHx W x 2 Qg, so Fact 1 is proved.

494

2 Solutions of problems 001500

Fact 2. Assume that Q


nw.Xa /  for any a 2 A. If E is an arbitrary family of
G -subsets of X D a2A Xa then there exists E 0  E such that jE 0 j  and
S 0 S
E D E.
Proof. If jAj  or jEj  then
S there is nothing to prove, so we assume that
jAj > and jEj > . Let Y D E; for every set ES2 E we can choose a family
SE of adequate G -subsets
of the space X such
S
S that SE D E (see Fact 1). Then,
for the family S D fSE W E 2 Eg we have S D Y .
Take an arbitrary a0 2 A and let A0 D fa0 g; G0 D ;. Proceeding inductively,
assume that n 2 ! and we have families fAi W i  ng and fGi W i  ng with the
following properties:
(4)
(5)
(6)
(7)

Ai  AiC1  A and Gi  GiC1  S for any i < n;


jA
S i j  and jGi j  for all i  n;
fsupp.F
/ W F 2 Gi g  AiC1 whenever i < n;
S
pAi . GiC1 / is dense in pAi .Y / for all i < n.

Observe that nw.XAn /  , so nw.pAn .Y //  and hence we can pick a dense


0
set D in the space pAn .Y / withSjDj  . It is easy to find aSfamily GnC1
 S
0
0
0
with jGnC1 j  such that pAn . GnC1 /  D and henceSpAn . GnC1 / is dense in
0
pAn .Y /. Letting GnC1 D GnC1
[ Gn and AnC1 D An [ . fsupp.F / W F 2 GnC1 g/
we obtain the families fAi W i  n C 1g and fGi W i  n C 1g such that the properties
(4)(7) hold if we replace n with n C 1. Therefore our inductive procedure can
be continued to construct families fAi W i 2 !g and fGi W i 2 !g for which the
conditions (4)(7)
S are satisfied for all n 2 !.S
S
Let G D fGn W n 2 !g and A0 D n2! An . To see that G is dense in
Y take any point y 2 Y and U 2 .y; X /. There exists
Q a finite setQB  A and
Wa 2 .y.a/; Xa / for all a 2 B such that the set V D a2B Wa  a2AnB Xa is
contained in U .
Pick n 2 ! for which B 0 D B \ An D B \ A0 and observe that V 0 D pAn .V /
is an open neighborhood of pAn .y/ in XAn . The property (7) shows that there exists
F 2 GnC1 such that pAnC1 .F / \ V 0 ;. Therefore there is a point x 2 F such that
x.a/ 2 Wa for all a 2 B 0 .
Let z.a/ D x.a/ for all a 2 An.BnB 0 / and z.a/ D y.a/ whenever a 2 BnB 0 .
The points z and x are distinct only on the set BnB 0  AnA0 . It follows from (6) that
supp.F /  A0 and hence pA10 .pA0 .F // D F ; since pA0 .z/ D pA0 .x/ 2 pA0 .F /,
we conclude
that z 2 F \ V and hence ; F \ V  S
F \ U . This shows that
S
U \ . G/ ; for every U 2 .y; X / and hence y 2 G for any y 2 Y , i.e.,
S
G is dense in Y .
Finally observe that, for any F 2 G there exists EF 2 E such that F  EF . Then
0
the
S family
S E D fEF W F 2SGg 0 E has cardinality at most and the inclusions
0
E  G  Y show that E is dense in Y , so Fact 2 is proved.
Fact 3. Assume that Q
nw.Xa /  for any
S a 2 A. If E is an arbitrary family of
G -subsets of X D a2A Xa and E D
E then there exists a set B  A with
jBj  and a closed subset F of the space XB such that E D pB1 .F /.

2 Solutions of problems 001500

495

Proof. It follows easily from Fact 1 and Fact S


2 that there
S a family S of
S exists
adequate G -subsets of X such that jSj  and S  E  S which implies
S
S
that S D E. The set BSD fsupp.P /SW P 2 Sg has cardinality not exceeding
and it is easy to seeSthat S D pB1 pB . S/.
Let H D pB . S S/ and F D H . The map pB W X ! XB is open and we
have the equality S D pB1 .H /, so we can apply Fact 1 of S.424 to see that
S
E D S D pB1 .H / D pB1 .F / and hence Fact 3 is proved.
Q
Fact 4. Given a product X D a2A Xa assume that c.X /  . Then, for any
U 2 .X / there exists a set B  A such that jBj  , the set pB .U / is closed in
XB and U D pB1 pB .U /.
Proof. Take a maximal disjoint family
S V of standard open subsets of X contained
in U . It is immediate that V D
V is dense in U , S
so V D U . It follows from
c.X /  that jVj  and therefore the set B D fsupp.W / W W 2 Vg has
cardinality  . The set V 0 D pB .V / is dense in U 0 D pB .U /, so V 0 D U 0 ; it is
easy to see that V D pB1 .V 0 /. Since the map pB W X ! XB is open, we can apply
Fact 1 of S.424 to conclude that U D V D pB1 .V 0 / D pB1 .U 0 /. As a consequence,
pB .U / D U 0 is a closed subset of XB and U D pB1 pB .U /, so Fact 4 is proved.
Fact 5. Suppose that X is a compact space and F is a family of closed subsets of
X with jFj  . Then for any nonempty G -subset H of the space X there exists a
nonempty closed G -set H 0  H such that, for any F 2 F, either H 0 \ F D ; or
H0  F .
Proof. Say that a closed set P  H is small with respect to a set F 2 F if either
P  F or P \ F D ;. Therefore we must find a nonempty G -set H 0  H which
is small with respect to every F 2 F. Let fF W < g be an enumeration of F.
Fix a point x 2 H and take a closed G -set H00 such that x 2 H00  H (see Fact 2
of S.328). If H00  F0 then let H0 D H00 ; if H00 nF0 ; then H00 nF0 is a nonempty
G -subset of X , so we can apply Fact 2 of S.328 again to find a closed nonempty
G -set H0  H00 nF0 ; it is evident that in both cases H0 is small with respect to F0 .
Proceeding inductively assume that < and we have a decreasing family
fH W < g of closed nonempty G -subsets of X T
such that H is small with
respect to F for any < . The closed G -set H0 D < H is still nonempty;
if H0  F then let H D H0 . If Q D H0 nF ; then Q is a nonempty G subset of X , so we can apply Fact 2 of S.328 to see that there exists a nonempty
closed G -set H  Q.
It is evident that, in both cases, H is small with respect to F and H  H for
any  , so our inductive procedure can be continued to construct a decreasing
family fH W < g of closed nonempty G -subsets of X T
such that H is small
with respect to F for every < . It is immediate that H 0 D fH W < g  H
is a nonempty closed G -subset of X which is small with respect to every F 2 F,
so Fact 5 is proved.
Fact 6. Suppose that X is a space, D is a dense subspace of X and denote by e
X
the extension operator eD
. Assume that f W D ! Y is a continuous map and

496

2 Solutions of problems 001500

' W X ! Z is an open continuous map. For any U 2 .Y / let U  D e.f 1 .U //.


Consider the set N D fz 2 Z W ' 1 .z/ is contained in Q.f; yz / for some yz 2 Y g
and let M D clX .' 1 .N // \ D. Then
(a) '.U  / \ '.V  / \ '.M / D ; for any U; V 2 .Y / such that U \ V D ;;
(b) there exists a continuous map g W '.M / ! Y such that g .'jM / D f jM .
Proof. (a) Assume that '.U  /\'.V  /\'.M / ;. The set W D '.U  /\'.V  /
is open in Z. By continuity of ' the set '.M / is contained in clZ .N /, so it
follows from W \ '.M / ; that W \ N ;; fix a point z 2 W \ N . We have
the inclusion ' 1 .z/  Q.f; yz / and it follows from z 2 '.U  / that ' 1 .z/ \
U  ;, so U  \ Q.f; yz / ;. Analogously, it follows from z 2 '.V  / that
V  \ Q.f; yz / ;.
To obtain the desired contradiction we will prove that yz 2 U \ V . Indeed,
suppose that yz U \ V . Since U and V are in a symmetric situation, we can
assume, without loss of generality, that yz U . If G D Y nU then it follows
from (3) and the definition of U  that G  \ U  D ;. Therefore clX .G  / \
U  D ;; since yz 2 G 2 .yz ; Y /, the set Q.f; yz / is contained in clX .G  /, so
Q.f; yz / \ U  D ;; this contradiction proves that yz 2 U \ V which, in turn,
contradicts V \ U D ;, so (a) is proved.
(b) Suppose first that there exist points x; y 2 M such that '.x/ D '.y/ but
f .x/ f .y/. Take sets U 2 .f .x/; Y / and V 2 .f .y/; Y / such that
U \ V D ; and observe that x 2 U  and y 2 V  which implies '.x/ 2
'.U  / \ '.V  / \ '.M /; this contradiction with (a) shows that there exists a
map g W '.M / ! Y such that g .'jM / D f jM .
To see that g is a continuous map fix any point z 2 '.M / and U 2 .g.z/; Y /;
let y D g.z/ and choose a set V 2 .y; Y / such that V  U . Take a point
x 2 M such that '.x/ D z. Then f .x/ D g.'.x// D y. The set V  is an
open neighborhood of x in X , so W 0 D '.V  / 2 .z; Z/. Let us show that
W D W 0 \ '.M / witnesses continuity of g at the point z.
Assume that w 2 W and g.w/ V ; fix a point u 2 M such that '.u/ D w.
Choose a set H 2 .g.w/; Y / such that H \ V D ;. We have f .u/ D g.w/ 2
H and therefore u 2 H  . As a consequence, '.u/ D w 2 '.H  / \ '.V  / \
'.M / which is again a contradiction with (a). Therefore g.w/ 2 V for any
w 2 W and hence g.W /  V  U , i.e., the map g is continuous at every point
of '.M /. This settles (b) and shows that Fact 6 is proved.
Fact 7. Given a space X and an infinite cardinal suppose that F is a closed subset
of X such that .F; X /  . If G is a compact subset of X such that G  F and
.G; F /  then .G; X /  .
Proof. Fix an outer base B of the set F in X such that jBj  and let C be an
outer base of G in F such that jCj  . For any B 2 B and C 2 C the set
Bn.F nC / is an open neighborhood of G in X . Choose a set O.B; C / 2 .G; X /
with O.B; C /  Bn.F nC / (see Fact 4 of T.309). It is straightforward that the

2 Solutions of problems 001500

497

family O0 D fO.B; C / W B 2 B and C 2 Cg has cardinality not exceeding .


Therefore the family O of all finite intersections of the elements of O0 also has
cardinality at most .
T
It is evident that O  G; let us prove that O is an outer base of G in X . Given
any set U 2 .G; X / there exists a set C 2 C such that C  U \ F ; take any set
B 2 B and observe that O.B; C / \ F  .Bn.F nC // \ F D C  U . Therefore
the closed set P D O.B; C /nU is disjoint from F , so we can find a set B 0 2 B
such that B 0 \ P D ;. It is immediate that O.B; C / \ O.B 0 ; C /  U , so O is an
outer base of G in X and hence .G; X /  jOj  , so Fact 7 is proved.
Fact 8. Suppose that X is a compact space and is an infinite cardinal. Assume
that F is a G -subset of X and  .x; F /  for some x 2 F . Then there exists a
-base B of X at the point x such that jBj  and B \ F ; for any B 2 B. In
particular,  .x; X /  .
Proof. Choose a -base C of the space F at the point x with jCj  . By Fact 2
of S.328, for every C 2 C we can find a nonempty compact set PC  C such that
PC is a G -subset of F . Then .PC ; X / D .PC ; X /  (see Fact 2 of S.358),
so weScan fix an outer base BC of the set C in the space X with jBC j  . If
B D fBC W C 2 Cg then jBj  and B \ F ; for any B 2 B.
Given any set U 2 .x; X / there exists C 2 C with C  U \ F and hence
PC  U . The family BC being an outer base of C in X there is B 2 BC such
that B  U . This shows that B is a -base of X at the point x and hence Fact 8 is
proved.
Fact 9. Suppose that
Q nw.Xa /  for every a 2 A and D is a dense subspace of
the product X D a2A Xa . Assume that Y is a compact space with .Y /  and
X
f W D ! Y is a continuous onto map. Denote by e the extension operator eD
and
let P be the family of all G -subsets
P
of
the
space
X
such
that
P

Q.f;
y/
S
for some y 2 Y . If M D clX . P/ \ D then f .M / intersects every nonempty
G -subset of the space Y .
Q
Proof. Let Ka D Xa for every a 2 A and
Q consider the space K D a2A Ka .
For any B  A let pB W X ! XB D a2B Xa be the
Q natural projection; we
will also need the natural projection qB W K ! KB D a2B Ka . It follows from
d.Ka /  d.Xa /  nw.Xa /  that C is a caliber of Ka for any a 2 A and hence
C is a caliber of K (see SFFS-281); in particular, c.K/  .
For every set U 2 .Y / let U  D e.f 1 .U // and U
D clK .U  /; it is easy to
K
see that U
D clK .eD
.f 1 .U ///, so we can apply Fact 4 to see that there exists a
1
set E.U /  A such that jE.U /j  and U
D qE.U
/ qE.U / .U
/.
Assume that there exists a nonempty G -set 00 of the space Y which does not
meet f .M /. By Fact 2 of S.328 we can pass to a smaller set if necessary to be able
0
to assume, without loss of generality, that S
0 is compact.
Apply Fact 3 to the set N D clX . P/ to find B.0/  A such that
1
1
jB.0/j  and N D pB.0/
pB.0/ .N /. If O D X nN then O D pB.0/
pB.0/ .O/

498

2 Solutions of problems 001500

and f 1 .00 /  O. We have the inclusion pB.0/ .O/  XB.0/ which, together with
nw.XB.0/ /  implies that we can find a network N0 in the space pB.0/ .O/ such
that jN0 j  .
Let N00 D fclKB.0/ .N / W N 2 N0 g and denote by N000 the family of all finite
intersections of the elements of N00 . Observe that jN000 j  and the family N000
separates the points of pB.0/ .O/ from the points of KB.0/ npB.0/ .O/. Consider the
1
family F0 D fqB.0/
.F / W F 2 N000 g and let G0 D fclY .f .F \ D// W F 2 F0 g;
S
clearly,SG0 consists of compact subsets of Y . It follows from O 
F0 that
00  G0 . By Fact 5 there exists a nonempty closed G -subset 0 of the space
Y such that 0  00 and, for any G 2 G0 , either 0  G or 0 \ G D ;.
It follows from the inequality .0 /  .Y /  that there exists y0 2 0 such
that  .y0 ; 0 /  (see Fact 1 of U.086); apply Fact 8 to find a -base B0 of Y at
the point y0 such that jB0 j  and U \ 0 ; for all U 2 B0 .
Proceeding inductively assume that n 2 ! and we have constructed, for every
i  n, a nonempty compact G -subset i of the space Y , a point yi 2 i , a set
B.i /  A together with the families Fi ; Gi ; and Bi with the following properties:
(8) Fi is a family of compact subsets of K and fpB.i/ .F / W F 2 Fi g separates the
points of pB.i/ .O/ from the points of KB.i/ npB.i/ .O/ for any i  n;
1
(9) pB.i/
pB.i/ .F / D F for any F 2 Fi and i  n;
(10) Fi is closed under finite intersections for all i  n;
(11) Gi D fclY .f .F \ D// W F 2 Fi g for all i  n;
(12) maxfjB.i /j; jFi j; jGi jg  for all i  n;
(13) if i  n then i is a compact G -subset of Y such that, for any G 2 Gi either
i \ G D ; or i  G;
(14) for each i  n the family Bi is a -base of Y at the point yi such that jBi j 
and U \ i ; for all U 2 Bi ;
(15) B.i /  B.i C 1/; Fi  FiC1 and iC1  i for all i < n;
(16) for any i < n, if U 2 Bi then E.U /  B.i C 1/.
The family E D fE.U / W U 2 B0 [ : : : [ Bn g has cardinality not exceeding
, so weScan choose a set B.n C 1/  A such that jB.n C 1/j  and
1
B.n/ [ . E/  B.n C 1/. It is evident that O D pB.nC1/
pB.nC1/ .O/. We have
pB.nC1/ .O/  XB.nC1/ and nw.XB.nC1/ /  which shows that we can find a
network NnC1 in the space pB.nC1/ .O/ such that jNnC1 j  .
0
00
Let NnC1
D fclKB.nC1/ .N / W N 2 NnC1 g and denote by NnC1
the family of all
0
00
finite intersections of the elements of NnC1 . Observe that jNnC1 j  and the family
00
NnC1
separates the points of pB.nC1/ .O/ from the points of KB.nC1/ npB.nC1/ .O/.
1
00
Let FnC1 D fqB.nC1/
.F / W F 2 NnC1
g [ Fn and GnC1 D fclY .f .F \ D// W F 2
S
FnC1 g; clearly,
G
consists
of
compact
subsets of Y . It follows from O  FnC1
nC1
S
that n  GnC1 . By Fact 5 there exists a nonempty closed G -subset nC1 of the
space Y such that nC1  n and, for any set G 2 GnC1 , either nC1  G or
nC1 \ G D ;.
It follows from the inequality .nC1 /  .Y /  that there exists a point
ynC1 2 nC1 such that  .ynC1 ; nC1 /  (see Fact 1 of U.086); apply Fact 8 to
find a -base BnC1 of Y at the point ynC1 such that jBnC1 j  and U \ nC1 ;

2 Solutions of problems 001500

499

for all U 2 BnC1 . It is straightforward that the conditions (8)(16) are now satisfied
if we replace n with n C 1, so our inductive procedure can be continued to construct
for every i 2 !, a nonempty compact G -subset i of the space Y , a point yi 2 i ,
a set B.i /  A together with the families Fi ; Gi ; and Bi such that the properties
(8)(16) hold for each n 2T
!.
S
Consider
the
sets

D
n2! n and B
S
SD n2! B.n/ together with the families
S
B D n2! Bn ; F D n2! Fn and G D n2! Gn . Choose an accumulation point y
for the sequence fyn W n 2 !g; it is easy to see that y 2 and B is a -base of Y at
the point y.
Let y be the family of all open neighborhoods
S of y in Y , i.e., y D .y; Y /. For
every U T
2 y we will need the set H.U / D fV  W V 2 B and V  U g. Now
let P D fclK .H.U // W U 2 y g. It follows from (16) that qB1 qB .U
/ D U
for
any U 2 B; as an immediate consequence, we have qB1 qB .P / D P . Since P is
compact, the set qB .P / is a compact subspace of KB and hence R D qB .P / \ XB
is a closed subspace of XB . It follows from nw.XB /  that R is a G -subset of
XB and hence P 0 D P \ X D pB1 .R/ is a G -subset of X .
Given any set U 2 y it follows from the inclusion H.U /  U  that we have the
formula
//\X D clX .H.U //  clX .U  /. As an immediate consequence,
TclK .H.U
0

P  fclX .U / W U 2 y g D Q.f; y/.
The property (9) implies that qB1 qB .F / D F for any F 2 F. Besides, the family
FB D fqB .F / W F 2 Fg consists of compact subsets of KB and it follows from (8),
(9), and (15) that
(17) FB is closed under finite intersections and separates the points of pB .O/ from
the points of KB npB .O/.
Take aTpoint x 2 D with f .x/ D y; let Fx D fF 2 F W x 2 F g and consider the
set L D Fx . Then x 2 L D qB1 qB .L/ and qB .L/  qB .O/ D pB .O/; this is an
0
easy consequence of (17). Take an arbitrary U 2 y and any finite subfamily
T 0 F of
0
the family Fx . There exists n 2 ! such that F  Fn and hence F D F 2 Fn .
Therefore the set G D clY .f .F \ D// belongs to Gn ; it follows from y D f .x/ 2
G \ n that G \ n ; and hence n  G by the property (13).
There exists W 2 Bn such that W  U and it follows from the property (14)
that W \ n ;. ThusTf 1 .W / \ F ; which shows that F \ W  ;.
Consequently, H.U / \ . F 0 / ; for any finite family F 0  Fx and U 2 y . It
is easy to see, using Fact 1 of S.326, that this implies L0 D P \ L ;. Taking in
consideration that qB1 qB .L0 / D L0 we conclude that qB .L0 /  qB .P / \ qB .O/, so
qB .O/\qB .P / ;. Therefore ; L0  P \O and hence P0 D P \O D P 0 \O
0
is a nonempty G -subset of X . Besides, P
S0  P  Q.f; y/ and hence P0 2 P;
this contradiction with P0  O  X n. P/ shows that f .M / intersects every
nonempty G -subset of Y , i.e., Fact 9 is proved.
Returning to our solution assume that .K/   and denote by P the family
of all nonempty G -subsets P of the product N such that P  Q.f; y/ for some
y 2 K. It follows from Fact 1 and Fact 2 that we
Scan find a family
S A  P of
adequate
G
-subsets
of
N
such
that
jAj


and
A
is
dense
in
P. Let S D

S
fsupp.P / W P 2 Ag; then we have the equality pS1 pS .P / D P for any P 2 A

500

2 Solutions of problems 001500

S
S
S
and F D clN . P/ D clN . A/. Let A D A and take any z 2 pS .A/. There is
P 2 A with z 2 pS .P / and hence pS1 .z/  P  Q.f; y/ for some y 2 K.
Let D 0 D F \ D and apply Fact 6 to the open map ' D pS and the map f to see
that there exists a continuous map g W pS .D 0 / ! K such that g .pS jD 0 / D f jD 0 .
This implies that f .D 0 / D g.pS .D 0 // and hence nw.f .D 0 //  nw.pS .D 0 //  .
Next apply Fact 9 to see that
(18) the set E D f .D 0 / intersects every nonempty G -subset of K.
If KnE ; then pick a point y 2 KnE and take a set Oz 2 .z; K/ such that
y O z for any z 2SE. Since l.E/  nw.E/  , there is a set E 0  E such that
jE 0 j   and E  TfOz W z 2 E 0 g.
Then y 2 R D fKnO z W z 2 E 0 g  KnE, so R is a nonempty G -subset of
K contained in KnE. This contradiction with (18) shows that K D E and hence
w.K/ D nw.K/ D nw.E/   (see Fact 4 of S.307).
Finally assume that K is a continuous image of a dense subspace of a product
C
of cosmic spaces. It is trivial that  D t .K/  w.K/. Since t .I / > , it follows
C
from TFS-162 that the space K cannot be mapped onto I and hence .K/  ,
i.e., .K/  t .K/  w.K/. If .K/ D then w.K/  because K is a continuous
image of a dense subspace of a product of cosmic and hence -cosmic spaces. Thus
w.K/  .K/  t .K/  w.K/ which shows that w.K/ D t .K/ D .K/ and
hence our solution is complete.
V.431. Suppose that  is a cardinal of uncountable cofinality, T ; is a set and Nt
is a space
Qsuch that nw.Nt /  ! for all t 2 T . Assume that D is a dense subspace
of N D t2T Nt and f W D ! K is a continuous map of D onto a compact space
K with w.K/ D . Prove that K maps continuously onto I .
Solution. If X is a space then a family E  exp.X / separates the points of a set
F  X from the points of F 0  X if for any x 2 F and y 2 F 0 there exists E 2 E
such that x 2 E and y E.
Q
Q
Given a product X D a2A Xa and B  A let pB W X ! XB D a2B Xa
be the natural projection. Suppose that is an infinite cardinal and nw.Xa /  !
for all a 2 A. Say that F  X is an adequate G -subset of X if there exists a set
B  A (called the support of F and denoted by supp.F /) such that jBj  and
F D pB1 .z/ for some z 2 XB . It is evident that any adequate G -subset of X is a
G -subset of X . Say that F  X is a G< -subset of X if F is a G 0 -subset of X
for some 0 < . An open subset U of the space X is called standard if there is a
finite B  A (which is also called the support
denoted by supp.U /) and
Q of U andQ
Ua 2 .Xa / for each a 2 B such that U D a2B Ua  a2AnB Xa . It is clear that
standard open subsets of X constitute a base of X .
If X is an arbitrary space and D  X is a dense subspace of X then we will
X
need an extension
operator eD
W .D/ ! .X / which is defined by the formula
S
X
eD .U / D fV 2 .X / W V \ D  U g for any U 2 .D/. It is easy to see that, for
any space X and any dense D  X , we have the following properties:

2 Solutions of problems 001500

501

X
(1) eD
.U / \ D D U for any U 2 .D/;
X
X
(2) if U; V 2 .D/ and U  V then eD
.U /  eD
.V /;
X
X
(3) if U; V 2 .D/ and U \ V D ; then eD .U / \ eD
.V / D ;.
X
Assume that X is a space, D is a T
dense subspace of X and let e D eD
. Given a
1
map f W D ! Y , the set Q.f; y/ D fclX .e.f .U /// W U 2 .y; Y /g is a useful
extension of the set f 1 .y/ (it is an easy exercise that Q.f; y/ \ D D f 1 .y/ for
any y 2 Y ).

Fact 1. Suppose that is a cardinal of uncountable cofinality and nw.X


Q a /  ! for
any a 2 A. Then, for any family E of adequate G< -subsets of X D a2A Xa there
exists a set B  A and a familySG  ESsuch that jBj < andSjGj < S
while
S
fsupp.P / W P 2 Gg  B and E  G. In particular, E D E D G and
pB1 pB .E/ D E.
S
Proof. Let Y D
E; take an arbitrary b0 2 A and let B0 D fb0 g; G0 D ;.
Proceeding inductively, assume that n 2 ! and we have families fBi W i  ng and
fGi W i  ng with the following properties:
(4)
(5)
(6)
(7)

Bi  BiC1  A and Gi  GiC1  E for any i < n;


jB
S i j < and jGi j < for all i  n;
fsupp.P
/ W P 2 Gi g  BiC1 whenever i < n;
S
pBi . GiC1 / is dense in pBi .Y / for all i < n.

Observe that nw.XBn /  jBn j ! < , so nw.pBn .Y // < and hence we


can pick a dense set D in the space pBn .Y / such that jDj
S 0< . It is easy to find
0
0
a family
G

E
with
jG
j
<

such
that
p
.
GnC1 /  D and hence
B
n
nC1
nC1
S 0
0
/ is dense in the
set
p
.Y
/.
Consider
the
family
GnC1 D GnC1
[ Gn
pBn . GnC1
B
n
S
and the set BnC1 D Bn [ . fsupp.P / W P 2 GnC1 g/; it is clear that for the families
fBi W i  n C 1g and fGi W i  n C 1g the properties (4)(7) hold if we replace n
with nC1. Therefore our inductive procedure can be continued to construct families
fBi W i 2 !g and fGi W i 2 !g for which the conditions (4)(7) are satisfied for all
n 2 !.
S
S
Let G D fGn W n 2 !g and B D Sn2! Bn ; it follows from (5) and cf. / > !
that jGj < and jBj < . To see that G is dense in Y take any point y 2 Y and
U 2 .y; X /. There exists
Q a finite set
Q H  A and Wa 2 .y.a/; Xa / for all a 2 H
such that the set V D a2H Wa  a2AnH Xa is contained in U .
Pick n 2 ! for which H 0 D H \ Bn D H \ B and observe that V 0 D pBn .V /
is an open neighborhood of pBn .y/ in XBn . The property (7) shows that there exists
P 2 GnC1 such that pBnC1 .P / \ V 0 ;. Therefore there is a point x 2 P such that
x.a/ 2 Wa for all a 2 H 0 .
Let z.a/ D x.a/ for all a 2 An.H nH 0 / and z.a/ D y.a/ whenever a 2 H nH 0 .
The points z and x are distinct only on the set H nH 0  AnB. It follows from (6)
that supp.P /  B and hence pB1 .pB .P // D P ; since pB .z/ D pB .x/ 2 pB .P /,
we conclude
that z 2 P \ V and hence ; P \ V  S
P \ U . This shows that
S
G for any y 2 Y , i.e.,
U
\
.
G/

;
for
every
U
2

.y;
X
/
and
hence
y
2
S
S
G
is
dense
in
Y
.
It
follows
from
(6)
that
fsupp.P
/
W
P
2 Gg  B and hence
S
S
G D pB1 pB . G/.

502

2 Solutions of problems 001500

S
S
S
S
S
Finally observe that it follows from G  E  G that E D E D G.
S
S
The map pB W X ! XB being open we have the equality G D pB1 .pB . G//
(see Fact 1 of S.424). As an immediate consequence, E D pB1 pB .E/, so Fact 1 is
proved.
Fact 2. Suppose that
Q nw.Xa /  ! for every a 2 A and D is a dense subspace of
the product X D a2A Xa . Assume that is a cardinal of uncountable cofinality
and Y is a compact space which cannot be continuously mapped onto I while there
X
exists a continuous onto map f W D ! Y . Denote by e the extension operator eD
and let P be the family of all adequate G<
S-subsets P of the space X such that
P  Q.f; y/ for some y 2 Y . If M D clX . P/ \ D then f .M / intersects every
nonempty G< -subset of the space Y .
Q
Proof. Let Ka D Xa for every a 2 A and
Q consider the space K D a2A Ka .
For any B  A let pB W X ! XB D a2B Xa be the
Q natural projection; we
will also need the natural projection qB W K ! KB D a2B Ka . It follows from
d.Ka /  d.Xa /  nw.Xa /  ! that !1 is a caliber of Ka for any a 2 A and hence
!1 is a caliber of K (see SFFS-281); in particular, c.K/  !.
For every set U 2 .Y / let U  D e.f 1 .U // and U
D clK .U  /; it is easy to
K
see that U
D clK .eD
.f 1 .U ///, so we can apply Fact 4 of V.430 to see that there
1
exists a set E.U /  A such that jE.U /j  ! < and U
D qE.U
/ qE.U / .U
/.
0
Assume that there exists a nonempty G< -set 0 of the space Y which does not
meet f .M /. By Fact 2 of S.328 we can pass to a smaller set if necessary to be able
0
to assume, without loss of generality, that S
0 is compact.
Apply Fact 1 to the set N D clX . P/ to find B.0/  A such that
1
1
jB.0/j < and N D pB.0/
pB.0/ .N /. If O D X nN then O D pB.0/
pB.0/ .O/
0
1
and f .0 /  O. We have the inclusion pB.0/ .O/  XB.0/ which, together with
nw.XB.0/ / < implies that we can find a network N0 in the space pB.0/ .O/ such
that jN0 j < .
Let N00 D fclKB.0/ .N / W N 2 N0 g and denote by N000 the family of all finite
intersections of the elements of N00 . Observe that jN000 j < and the family N000
separates the points of pB.0/ .O/ from the points of KB.0/ npB.0/ .O/. Consider the
1
family F0 D fqB.0/
.F / W F 2 N000 g and let G0 D fclY .f .F \ D// W F 2 F0 g;
S
clearly,SG0 consists of compact subsets of Y . It follows from O 
F0 that
00  G0 . By Fact 5 of V.430 there exists a nonempty closed G< -subset 0 of
the space Y such that 0  00 and, for any G 2 G0 , either 0  G or 0 \ G D ;.
It follows from the fact that Y cannot be mapped onto I that 0 cannot be
continuously mapped onto I and hence there exists a point y0 2 0 such that
 .y0 ; 0 / < (see Fact 1 of U.086); apply Fact 8 of V.430 to find a -base B0 of
Y at the point y0 such that jB0 j < and U \ 0 ; for all U 2 B0 .
Proceeding inductively assume that n 2 ! and we have constructed, for every
i  n, a nonempty compact G< -subset i of the space Y , a point yi 2 i , a set
B.i /  A together with the families Fi ; Gi ; and Bi with the following properties:

2 Solutions of problems 001500

503

(8) Fi is a family of compact subsets of K and fpB.i/ .F / W F 2 Fi g separates the


points of pB.i/ .O/ from the points of KB.i/ npB.i/ .O/ for any i  n;
1
(9) pB.i/
pB.i/ .F / D F for any F 2 Fi and i  n;
(10) Fi is closed under finite intersections for all i  n;
(11) Gi D fclY .f .F \ D// W F 2 Fi g for all i  n;
(12) maxfjB.i /j; jFi j; jGi jg < for all i  n;
(13) if i  n then i is a compact G< -subset of Y such that, for any G 2 Gi either
i \ G D ; or i  G;
(14) for each i  n the family Bi is a -base of Y at the point yi such that jBi j <
and U \ i ; for all U 2 Bi ;
(15) B.i /  B.i C 1/; Fi  FiC1 and iC1  i for all i < n;
(16) for any i < n, if U 2 Bi then E.U /  B.i C 1/.
The family E D fE.U / W U 2 B0 [ : : : [ Bn g has cardinality strictly
less thanS , so we can choose B.nC1/  A such that jB.nC1/j < and
1
B.n/ [ . E/  B.nC1/. It is evident that O D pB.nC1/
pB.nC1/ .O/. We have
pB.nC1/ .O/  XB.nC1/ and nw.XB.nC1/ / < which shows that we can find a
network NnC1 in the space pB.nC1/ .O/ such that jNnC1 j < .
0
00
Let NnC1
D fclKB.nC1/ .N / W N 2 NnC1 g and denote by NnC1
the family of all
0
00
finite intersections of the elements of NnC1 . Observe that jNnC1 j < and the family
00
NnC1
separates the points of pB.nC1/ .O/ from the points of KB.nC1/ npB.nC1/ .O/.
1
00
Let FnC1 D fqB.nC1/
.F / W F 2 NnC1
g [ Fn and GnC1 D fclY .f .F \ D// W F 2
S
FnC1 g; clearly,
S GnC1 consists of compact subsets of Y . It follows from O  FnC1
that n  GnC1 . By Fact 5 of V.430 there exists a nonempty closed G< -subset
nC1 of the space Y such that nC1  n and, for any set G 2 GnC1 , either
nC1  G or nC1 \ G D ;.
It follows from the fact that Y cannot be mapped onto I that nC1 cannot be
continuously mapped onto I and hence there exists a point ynC1 2 nC1 such that
 .ynC1 ; nC1 / < (see Fact 1 of U.086); apply Fact 8 of V.430 to find a -base
BnC1 of Y at the point ynC1 such that jBnC1 j < and U \ nC1 ; for all
U 2 BnC1 . It is straightforward that the conditions (8)(16) are now satisfied if we
replace n with n C 1, so our inductive procedure can be continued to construct for
every i 2 !, a nonempty compact G< -subset i of the space Y , a point yi 2 i ,
a set B.i /  A together with the families Fi ; Gi ; and Bi such that the properties
(8)(16) hold for each n 2T
!.
S
Consider
the
sets

D
D n2! B.n/ together with the families
n2! n and BS
S
S
B D n2! Bn ; F D n2! Fn and G D n2! Gn . It follows from cf. / > ! that
jBj < and jBj < . Choose an accumulation point y for the sequence fyn W n 2
!g; it is easy to see that y 2 and B is a -base of Y at the point y.
Let y be the family of all open neighborhoods
S of y in Y , i.e., y D .y; Y /. For
every U T
2 y we will need the set H.U / D fV  W V 2 B and V  U g. Now
let P D fclK .H.U // W U 2 y g. It follows from (16) that qB1 qB .V
/ D V
for
any V 2 B; as an immediate consequence, we have qB1 qB .P / D P . Since P is
compact, the set qB .P / is a compact subspace of KB and hence R D qB .P / \ XB
is a closed subspace of XB . It follows from nw.XB / < that R is a G< -subset of
XB and hence P 0 D P \ X D pB1 .R/ is a G< -subset of X .

504

2 Solutions of problems 001500


Given any set U 2 y it follows from the inclusion H.U
T /  U that we have

0
clK .H.U // \ X D clX .H.U //  clX .U /, so P  fclX .U / W U 2 y g D
Q.f; y/.
The property (9) implies that qB1 qB .F / D F for any F 2 F. Besides, the family
FB D fqB .F / W F 2 Fg consists of compact subsets of KB and it follows from
(8),(9) and (15) that

(17) FB is closed under finite intersections and separates the points of pB .O/ from
the points of KB npB .O/.
Take aTpoint x 2 D with f .x/ D y; let Fx D fF 2 F W x 2 F g and consider the
set L D Fx . Then x 2 L D qB1 qB .L/ and qB .L/  qB .O/ D pB .O/; this is an
0
easy consequence of (17). Take an arbitrary U 2 y and any finite subfamily
T 0 F of
0
the family Fx . There exists n 2 ! such that F  Fn and hence F D F 2 Fn .
Therefore the set G D clY .f .F \ D// belongs to Gn ; it follows from y D f .x/ 2
G \ n that G \ n ; and hence n  G by the property (13).
There exists W 2 Bn such that W  U and it follows from the property (14)
that W \ n ;. ThusTf 1 .W / \ F ; which shows that F \ W  ;.
Consequently, H.U / \ . F 0 / ; for any finite family F 0  Fx and U 2 y . It
is easy to see, using Fact 1 of S.326, that this implies L0 D P \ L ;. Taking in
consideration that qB1 qB .L0 / D L0 we conclude that qB .L0 /  qB .P / \ qB .O/, so
qB .O/ \ qB .P / ;. Therefore ; L0  P \ O and hence P \ O D P 0 \ O is
a nonempty G< -subset of X .
Apply Fact 1 of V.430 to find an adequate G< -subset P0 of the space X such that
0
P0  P 0 \ O. It follows
S from P0  P  Q.f; y/ that P0 2 P; this contradiction
with P0  O  X n. P/ shows that f .M / intersects every nonempty G< -subset
of Y , i.e., Fact 2 is proved.
Returning to our solution assume that K does not map continuously onto I and
denote by P the family of all nonempty adequate G< -subsets P of the product N
such that P  Q.f; y/ for some y 2 K. It follows from Fact 1 that
S find a
S we can
family A  P and a set S  T such that jAj < ; jS j <  while P  A and
pS1 pS .P / D P for any P 2 A.
S
S
S
Observe that F D P D A; let A D A and take any z 2 pS .A/. There
is P 2 A with z 2 pS .P / and hence pS1 .z/  P  Q.f; y/ for some y 2 K.
Let D 0 D F \ D and apply Fact 6 of V.430 to the open map ' D pS and the
map f to see that there exists a continuous map g W pS .D 0 / ! K such that g
.pS jD 0 / D f jD 0 . This implies that f .D 0 / D g.pS .D 0 // and hence nw.f .D 0 // 
nw.pS .D 0 // < . Next apply Fact 2 to see that
(18) the set E D f .D 0 / intersects every nonempty G< -subset of K.
If KnE ; then pick a point y 2 KnE and take a set Oz 2 .z; K/ such that
y O z for any z 2SE. Since l.E/  nw.E/ < , there is a set E 0  E such that
jE 0 j <  and E  TfOz W z 2 E 0 g.
Then y 2 R D fKnO z W z 2 E 0 g  KnE, so R is a nonempty G< -subset
of K contained in KnE. This, however, contradicts (18) and implies that K D E,

2 Solutions of problems 001500

505

so w.K/ D nw.K/ D nw.E/ <  (see Fact 4 of S.307). This final contradiction
with w.K/ D  shows that K can be continuously mapped onto I and hence our
solution is complete.
V.432. Given
Q an infinite cardinal  suppose that nw.Nt /   for any t 2 T and
C  N D t2T Nt is a dense subspace of N . Assume additionally that we have a
continuous (not necessarily surjective) map ' W C ! L of C into a compact space
L. Prove that if y 2 C 0 D '.C / and h .y; L/   then .y; C 0 /  . Here the
cardinal h .y; L/ is the hereditary -character of the space L at the point y, i.e.,
h .y; L/ D supf .y; Z/ W y 2 Z  Lg.
Q
Q
Solution. If we have a product Z D t2T Zt and A  T then ZA D t2A Zt is
the A-face of Z and A W Z ! ZA is the natural projection. A set F  Z depends
on A  T if A1 A .F / D F ; if F depends on a set of cardinality   then we say
that F depends on at most -many coordinates. A set E  Z covers a face ZA if
A .E/ D ZA . Suppose that, for every t 2 T we have a family Nt of subsets of Zt
and let N D fNt W t 2 T g. If we have a faithfully indexed set A D ft1 ; : : : ; tn g  T
and Ni 2 Nti for each t  n then let t1 ; : : : ; tn ; N1 ; : : : ; Nn
D fx 2 Z W x.ti / 2 Ni
for all i D 1; : : : ; ng. A set H  Z is called N -standard (or standard if N is clear)
if H D t1 ; : : : ; tn ; N1 ; : : : ; Nn
for some t1 ; : : : ; tn 2 T and Ni 2 Nti for all i  n.
In this case we let supp.H / D A and r.H / D n. We also consider that H D Z
is the unique standard subset of Z such that r.H / D 0. Given any point x 2 Z
and A  T the set hx; Ai D fy 2 Z W y.t / D x.t / for any t 2 Ag is closed
in Z. If A  T then the face ZA is called -residual if jT nAj  . Say that a
nonempty closed set F  Z is -large if, for any x 2 F and any finite A  T , the
set hx; Ai \ F covers a -residual face of Z.
If Z is a space and H is a family of subsets of Z then H is called a network of Z
at a point z 2 Z if for any U 2 .z; Z/ there exists H 2 H such that x 2 H  U .
Fact 1. Q
Given an infinite cardinal  suppose that nw.Nt /   for any t 2 T and
N D
t2T Nt . Assume that C  N is dense in N , and we
Q have a compact
extension Kt of the space Nt for any t 2 T . If a set F  K D t2T Kt is -large
then there exists a G -set G in the space K such that F  G and F \ C D G \ C .
In particular, F \ C is a G -subset of C .
Proof. We can assume, without loss of generality, that KnF ;. For every t 2 T
fix a network Nt in the space Nt such that jNt j  ; we will need the family
Mt D fclKt .N / W N 2 Nt g. If M D fMt W t 2 T g then the M-standard subsets
of K will be called standard. It is easy to see that
(1) the family H of all standard subsets of K is a network in K at every x 2 C .
Given standard sets P and P 0 say that P 0  P if P D t1 ; : : : ; tn ; M1 ; : : : ; Mn

and there exists a natural k  n such that P 0 D ti1 ; : : : ; tik ; Mi1 ; : : : ; Mik
for some
distinct i1 ; : : : ; ik 2 f1; : : : ; ng; if k < n then we write P 0 P . We also include
here the case when k D 0, so P 0 D K  P for any standard set P . Say that a
standard set P is minimal if P \ F D ; but P 0 \ F ; whenever P 0 P . It
follows from (1) that

506

2 Solutions of problems 001500

(2) for any x 2 C nF there exists a minimal standard set P such that x 2 P .
It will be easy to finish our proof if we establish that
(3) the family S of minimal standard sets has cardinality not exceeding .
Assume, toward a contradiction that jSj > . Then we can choose S0  S such
that jS0 j D  C and there exists n 2 ! with r.P / D n for all P 2 S0 . Observe first
that
(4) if A  T , a set D  K covers the face KT nA and a standard set P is disjoint
from D then supp.P / \ A ;.
Indeed, if supp.P / D ft1 ; : : : ; tk g  T nA and P D t1 ; : : : ; tk ; M1 ; : : : ; Mk

then it follows from T nA .D/ D KT nA that there exists a point x 2 D such that
x.ti / 2 Mi for all i  k. Therefore x 2 D \ P which is a contradiction.
The set F being -large, there exists A1  T with jA1 j   such that F covers
the face KT nA1 . The property (4) shows that supp.P / \ A1 ; for any P 2 S0 .
There exists a point t1 2 A1 such that the family S00 D fP 2 S0 W t1 2 P g has
cardinality  C . Since jMt1 j  , we can find a family S1  S00 and M1 2 Mt1 such
that jS1 j D  C and t1 ; M1
 P for any P 2 S1 .
Proceeding by induction assume that k < n and we have a set Ak  T with
jAk j   and a family Sk such that jSk j D  C and, for some t1 ; : : : ; tk 2 Ak
and Mi 2 Mti (i D 1; : : : ; k), we have t1 ; : : : ; tk ; M1 ; : : : ; Mk
 P for every
P 2 Sk . Therefore P D t1 ; : : : ; tk ; s1 ; : : : ; snk ; M1 ; : : : ; Mk ; E1 ; : : : ; Enk
for
every P 2 Sk ; let Q.P / be the set in which s1 and E1 are omitted from the definition
of P , i.e., Q.P / D t1 ; : : : ; tk ; s2 ; : : : ; snk ; M1 ; : : : ; Mk ; E2 ; : : : ; Enk
. It is clear
that Q.P / P ; since P is minimal, the set Q.P / intersects F for each P 2 Sk .
Fix a set R 2 Sk and let F 0 D F \ Q.R/. The set F being -large, we can
find A  T with jAj   such that F 0 covers the face KT nA and hence the face
KT n.A[Ak / as well. Let AkC1 D A [ Ak and observe that every set P 2 Sk is
disjoint from F 0 ; this, together with (4) shows that supp.P / \ AkC1 ;. Suppose
for a moment that P D t1 ; : : : ; tk ; s1 ; : : : ; snk ; M1 ; : : : ; Mk ; E1 ; : : : ; Enk
2 Sk
and fs1 ; : : : ; snk g \ AkC1 D ;. Since F 0 covers the face KT nAkC1 , we can find a
point x 2 F 0 such that x.si / 2 Ei for all i  n  k; since also x.ti / 2 Mi for all
i  k because x 2 Q.R/, we conclude that x 2 F 0 \ P . This contradiction implies
that fs1 ; : : : ; snk g \ AkC1 ; and hence the set supp.P /nft1 ; : : : ; tk g intersects
the set AkC1 for any P 2 Sk nfRg.
Therefore we can find a family SkC1  Sk of cardinality  C together with a
point tkC1 2 AkC1 nft1 ; : : : ; tk g and a set MkC1 2 MtkC1 such that for any P 2
SkC1 we have t1 ; : : : ; tkC1 ; M1 ; : : : ; MkC1
 P . As a consequence, our inductive
procedure can be continued to construct a family Sn  S such that jSn j D  C
while t1 ; : : : ; tn ; M1 ; : : : ; Mn
 P for any P 2 Sn . Recalling that r.P / D n, we
conclude that we have the equality P D t1 ; : : : ; tn ; M1 ; : : : ; Mn
for each P 2 Sn ;
this contradiction shows that jSj  , i.e.,
S (3) is proved.
It is straightforward that G D Kn. S/ is a G -subset of K such that F  G
and F \ C D G \ C , i.e., Fact 1 is proved.

2 Solutions of problems 001500

507

Returning to our solution observe that there is no loss of generality to assume that
C 0 is denseQin L. Choose a compact extension Kt of the space Nt for any t 2 T ;
then K D t2T Kt is a compact extension of both N and C . There exist continuous
maps W C ! L and  W C ! K such that jC D ' and .x/ D x for any
x 2 C . It is clear that both and  are surjective.
For every t 2 T fix a network Nt in the space Nt such that jNt j   and let
Mt D fclKt .N / W N 2 Nt g. If M D fMt W t 2 T g then the M-standard subsets
of K will be called standard. Our first step is to prove that
(5) the set Fy D . 1 .y// is -large.
Fix a point x 2 Fy , a finite A  T and consider the set P D hx; Ai D fx 0 2 K W
x .t / D x.t / for all t 2 Ag. It follows from P \ Fy ; that  1 .P / \ 1 .y/
; and hence y 2 Q D . 1 .P //. The set Q is compact and it follows from
h .y; L/   that we can choose a -base B of the space Q at the point y such
that jBj  . For every B 2 B pick a set OB 2 .L/ such that ; OB \ Q 
O B \ Q  B. It follows from c.K/   and Fact 4 of V.430 that
0

(6) for any U 2  .L/, the set clK .' 1 .U // depends on at most -many
coordinates and coincides with the set .clC . 1 .U ///.
Apply (6) to find a set S  T of cardinality at most  for which A  S and
the set DB D .clC . 1 .OB /// depends on S for any B 2 B. The face KT nS is
residual; to show that P \ Fy covers KT nS fix any point w 2 KT nS and consider
the set E D fz 2 K W T nS .z/ D w and S .z/ 2 S .P /g. Clearly, E is a nonempty
compact subset of P .
Fix any B 2 B; it follows from OB \ Q ; that there is a point u 2  1 .P /
such that .u/ 2 OB ; thus u 2 1 .OB / which shows that .u/ 2 DB \ P . Define
a point u0 2 K by the equalities T nS .u0 / D w and S .u0 / D S ..u//. Since the
sets DB and P depend on S , we conclude that u0 2 DB \ P . On the other hand,
S .u0 / 2 S .P /, so u0 2 E, and therefore E \ DB ;.
As a consequence, . 1 .E// \ O B ; and hence . 1 .E// \ B ; for
any B 2 B; since . 1 .E// is a closed subset of Q and B is a -base of Q at
y, we must have y 2 . 1 .E// which implies that  1 .E/ \ 1 .y/ ; and
hence E \ Fy ;. If v 2 E \ Fy then w D T nS .v/ 2 T nS .P \ Fy /; the point
w 2 KT nS was chosen arbitrarily, so P \ Fy covers KT nS and hence (5) is proved.
By Fact 1 there exists a G -set G in the space K such that Fy  G and G \
C D Fy \ C D ' 1 .y/. Therefore we can
S choose a family F of compact subsets
of K such that jFj   and C nFy 
F  KnFy . For any F 2 F the set
1
WF D Ln.
.F
//
is
an
open
neighborhood
of y in L and it is straightforward
T
that H D fWF W F 2 Fg is a G -subset of L such that H \ C 0 D fyg. This
shows that .y; C 0 /   and hence our solution is complete.
Q
V.433. Suppose that C is a dense subspace of a product N D t2T Nt such that
nw.Nt /   for each t 2 T . Assume that K is a compact space with t .K/   and
' W C ! K is a continuous (not necessarily surjective) map; let C 0 D '.C /. Prove
that every closed subspace of C 0 is a G -set; in particular, .C 0 /  .

508

2 Solutions of problems 001500

Solution. Fix a nonempty closed set F 0 in the space C 0 and let F D clK .F 0 /.
Consider the quotient map p W K ! KF obtained by contracting the set F to
a point and let q D pjC 0 . It is easy to see, applying TFS-162, that we have the
inequalities t .KF /  t .K/  ; denote by y the point of the space KF represented
by F and let C 00 D p.C 0 /.
Fact 1. If X is a compact space then h .x; X /  t .X / for any x 2 X .
Proof. Fix any set Z  X such that x 2 Z and let P D Z; it is an easy exercise that
 .x; Z/ D  .x; P /. The space P is compact and t .P /  t .X /, so we can apply
TFS-331 to see that  .P /  t .P /  t .X / and hence  .x; Z/ D  .x; P / 
t .X / which shows that Fact 1 is proved.
Returning to our solution observe that h .y; KF /   by Fact 1; therefore
we can apply Problem 432 to the map p ' to see that .y; C 00 /  . Since
F 0 D q 1 .y/, we conclude that F 0 is a G -subset of C 0 .
Q
V.434. Suppose that C is a dense subspace of a product N D t2T Nt such that
nw.Nt /   for each t 2 T . Assume additionally that l.C /   and K is a compact
space with t .K/   such that there exists a continuous (not necessarily surjective)
map ' W C ! K. Prove that if C 0 D '.C / then hl.C 0 /  .
Solution. We have l.C 0 /   while every closed subspace of the space C 0 is a
G -set by Problem 433. Now, SFFS-001 shows that hl.C 0 /  .
V.435. Prove that if C is a dense subspace of a product of cosmic spaces and
K is a compact space then, for any continuous map ' W C ! K, we have
 .'.C //  t .K/.
Solution. Let  D t .K/ and Y D '.C /. The space C is dense in the product of
cosmic and hence -cosmic spaces, so we can apply Problem 433 to see that every
closed subspace of Y is a G -set. Therefore  .Y /   as promised.
V.436. Suppose that C is a dense subspace of a product of cosmic spaces and ' W
C ! K is a continuous (not necessarily surjective) map into a compact space K of
countable tightness; let Y D '.C /. Prove that Y is a perfect space and w.Y /  !.
In particular, if ' W Cp .X / ! K is a continuous map of Cp .X / into a compact
space K with t .K/  ! then '.Cp .X // is a perfect space of countable -weight.
Solution. There is no loss of generality to assume that Y is dense in K. Apply
Problem 435 to see that  .Y /  t .K/  !, so Y is a perfect space.
The cardinal !1 is a precaliber of C , so it also a precaliber of Y (see SFFS-277,
SFFS-278 and SFFS-280). Therefore !1 is a caliber of K (see SFFS-279). By TFS332, we can find a point-countable -base B in the space K; this, together with !1
being a caliber of K implies that B is countable. Therefore the family BY D fB \Y W
B 2 Bg is a countable -base in the space Y and hence w.Y /  !. Finally observe
that our solution is also applicable to any space C D Cp .X / because Cp .X / is a
dense subspace of RX .

2 Solutions of problems 001500

509

V.437. Suppose that C is a dense Lindelf -subspace of a product of cosmic


spaces and ' W C ! K is a continuous (not necessarily surjective) map into a
compact space K of countable tightness. Prove that nw.'.C //  !. In particular,
if Cp .X / is a Lindelf -space and ' W Cp .X / ! K is a continuous map of Cp .X /
into a compact space K of countable tightness then '.Cp .X // is cosmic.
Solution. Let Y D '.C /; observe that C  C is a Lindelf -space and the space
Y  Y  K  K is a continuous image of C  C . It is clear that C  C also embeds
as a dense subspace in a product of cosmic spaces; besides, t .K  K/  ! (see
Fact 1 of V.274), so we can apply Problem 434 to convince ourselves that Y  Y
is hereditarily Lindelf and hence .Y /  !. Since Y is a Lindelf -space, we
can apply SFFS-300 to conclude that nw.Y /  !. Finally observe that our solution
is also applicable to any space C D Cp .X / because Cp .X / is a dense subspace
of RX .
V.438. Given an infinite cardinal  observe that the property P of being strongly
-cosmic is stronger than being -cosmic. Besides, P is preserved by subspaces
and continuous images.
Prove that if  < cf./ and X is strongly -cosmic for any
Q
<  then X D < X is also strongly -cosmic.
S
Solution. Suppose that Z D fZ W < g and nw.Z / <  for every
S < . Fix
a network N in the space Z for each <  and observe that N D < N is a
network in Z with jN j  . Therefore every strongly -cosmic space S
is -cosmic.
If Y  Z then let Y D Y \ Z for every < . Then Y D < Y and
nw.Y /  nw.Z / <  for each < . This shows that every subspace of a
strongly -cosmic space is strongly -cosmic.
Now assume that f W Z !ST is a continuous onto map and let T D f .Z /
for all < . Then T D
< T and nw.T /  nw.Z / <  for each
< . Therefore every continuous image of a strongly -cosmic space is strongly
-cosmic.
Fact 1. If Z is a strongly -cosmic space and D cf./ then there exists a family
fZ W < g of subspaces of Z such that < < implies that Z  Z while
S
< Z D Z and nw.Z / <  for every < .
S
Proof. Choose a family fY W < g of subspaces
of Z such that Z D < Y
S
and nw.Y / <  for each < . If Y0 D fY W   and nw.Y /  jjg
then nw.Y0 /  jj jj  jj ! <  for any < . It is immediate that <
implies Y0  Y0 . Given a point z 2 Z there is an ordinal <  such that z 2 Y ; if
S
D maxf; nw.Y /g then <  and z 2 Y  Y0 . This shows that Z D < Y0 .
Observe that it follows from cf./ D that there is a function  W !  such
that < < implies ./ < ./ and for any  <  there is < such that
0
./ >  . Letting Z D Y./
for any < it is easy to see that we obtain the
promised family fZ W < g, so Fact 1 is proved.

510

2 Solutions of problems 001500

Returning to our solution assume that Q< D cf./ and the space X is
strongly -cosmic for any < ; let X D < X . By Fact 1 we can choose a
family fZ W < g such that < 0 implies Z  Z0 while nw.Z / <  for
S
each < and < Z D X for every < .
Q
Letting Y D < Z we obtain a family fY W < g of subsets of X . Given
any < note that nw.Y /  supfnw.Z / W < g <  because  < D cf./
and nw.Z / <  for each < . Thus nw.Y / <  for each < .
For any x 2 X and <  there exists < such that x./ 2 Z . Take any

<
S with f W < g  ; it is immediate that x 2 Y , so we proved that
< Y D X and hence X is strongly -cosmic, i.e., our solution is complete.
V.439. Given an infinite cardinal  prove that D is not strongly -cosmic and
hence no strongly -cosmic space can be continuously mapped onto I .
Solution. It is easy to find a disjoint
family fA W < g of subsets of  such that
S
jA j D  for any <  and < A D . If D is strongly -cosmic
then fix a
S
family fY W < g such that nw.Y / <  for every <  and < Y D D .
Let  W D ! DA be the natural projection; since nw. .Y // < , it is
impossible that  .Y / D DA , so we can choose a point x 2 DA n .Y / for
each < . For every <  there is a unique <  such that 2 A ; let
x./ D x ./. This gives us a point x 2 D . For each
S <  it follows from
 .x/ D x  .Y / that x Y . Consequently, x < Y ; this contradiction
shows that D is not strongly -cosmic.
Finally observe that if a strongly -cosmic space X maps continuously onto
I then I and D  I must be strongly -cosmic by Problem 438. This final
contradiction proves that no strongly -cosmic space can be continuously mapped
onto I .
V.440. Assume that  is an infinite cardinal and K is a strongly -cosmic compact
space. Prove that there exists x 2 K such that .x; K/ < .
Solution. Say that F  K is a G< -set if F is a G -subset of K for some < .
Assume, toward a contradiction, that .x; K/   for any x 2 K and the space K
is strongly -cosmic. If  D cf./ then there
S exists a -sequence fY W < g
such that < implies Y  Y and K D fY W < g while nw.Y / <  for
every <  (see Fact 1 of V.438). It follows from TFS-327 and Fact 7 of V.430
that
(1) For any < , if F is a closed G -subset of K then .x; F /   for any
x 2 F.
Since nw.Y0 / < , it is impossible that Y0 D K, so we can pick a point x 2
KnY0 . Take a set Oz 2 .z; K/ such that x O z for any z 2 YS
0 . Since l.Y0 / 
nw.Y0 / < , there
is
a
set
E

Y
such
that
jE
j
<

and
Y

fOz W z 2 E0 g.
0
0
0
0
T
Then x 2 R0 D fKnO z W z 2 E0 g  KnY0 ; since R0 is a G< -subset of K, we
can apply Fact 2 of S.328 to find a closed G< -subset F0 of the space K such that
x 2 F0  R0  KnY0 .
Proceeding inductively assume that <  and we have constructed a family
fF W < g of nonempty closed G< -subsets of K with the following properties:

2 Solutions of problems 001500

511

(2) <  < implies F  F ;


(3) F \ Y D ; for any < .

T
It follows from <  that F D < F is a nonempty closed G< -subset of
K, so we can derive from the property (1) that .x; F /   for any x 2 F . As a
consequence, w.F / D nw.F /   > nw.Y / and therefore Y cannot cover F .
Pick any point x 2 F nY ; there exists a set Oz 2 .z; K/ such that x O z for
any z 2 Y .S
Since l.Y /  nw.Y / < , there is aT
set E  Y such that jE j < 
and Y  fOz W z 2 E g. Then x 2 R D fKnO z W z 2 E g  KnY ;
since R is a G< -subset of K, we can apply Fact 2 of S.328 to find a closed G< subset H of the space K such that x 2 H  R  KnY . It is evident that if
F D H \ F then (2) and (3) are fulfilled for all  , so we can construct a
family fF W < g with the properties
T (2) and (3) satisfied for all < . The
space K being compact,
the
set
P
D
fF W < g is nonempty; it follows form
S
(3) that P  Kn. fY W < g/, so we obtained a contradiction which shows
that K must have a point x such that .x; K/ < .
V.441. Suppose that  is an uncountable cardinal and X is a space such that
w.X /   and l.X / < cf./. Prove that Cp .X / is strongly -cosmic. In particular,
if cf./ > ! and X is a Lindelf space with w.X / D  then Cp .X / is strongly
-cosmic.
Solution. It follows from w.X /   that we can assume, without loss of generality,
that X  I . For any A   let pA W I ! IA be the natural projection. If < 
and A D f W < g then p D pA jX and Y D p .X /.
Let p .g/ D g p for any g 2 Cp .Y /; then the map p W Cp .Y / ! Cp .X /
is an embedding for any <  and for Z D p .Cp .Y // we have nw.Z / D
nw.Y / < . It follows from TFS-298 that any continuous function f W X ! R
factorizes through a  l.X /-face in I , i.e., there is a set A   such that jAj  l.X /
for which there is a continuous map g W pA .X / ! R such that g .pA jX / D f .
It follows from jAj < cf./ that we can find <  with A  . It is easy to see
that there exists a continuous map h W Y !
S R for which h p D f and hence
f D p .h/, i.e., f 2 Z . This proves that fZ W < g D Cp .X /, so Cp .X / is
strongly -cosmic.
V.442. Let  be a cardinal of uncountable cofinality. Prove that if X is a Lindelf
-space with nw.X /   then Cp .X / is strongly -cosmic.
Solution. We have d.Cp .X //  nw.Cp .X // D nw.X /  , so there exists a set
fg W < g which is dense in Cp .X /; let P D fg W < g for every < .
Any Lindelf -space is stable (see SFFS-266), so Cp .X / is monolithic and hence
jj <  for any < . Besides, it
the space F D P is jj-cosmic, i.e., nw.F /  S
follows from t .Cp .X // D ! and cf./ > ! that fF W < g D Cp .X /, so the
space Cp .X / is strongly -cosmic.
V.443. Given a cardinal  > ! suppose that a space X is strongly -monolithic,
i.e., w.A/   for every A  X with jAj   and, additionally, l.X / < cf./.
Prove that w.K/ <  for any compact continuous image K of the space Cp .X /.

512

2 Solutions of problems 001500

Deduce from this fact that if X is a Lindelf strongly !1 -monolithic space (in
particular, if l.X / D ! and w.X /  !1 ) then every compact continuous image
of Cp .X / is metrizable.
Solution. For any A  X let A W Cp .X / ! Cp .A/  RA be the restriction map.
Suppose that K is a compact space, ' W Cp .X / ! K is a continuous onto map and
w.K/  . Since Cp .X / is a dense subspace of RX , we can apply Problem 430 to
see that .K/ D w.K/. If w.K/ D  then cf./ > ! together with Problem 431
show that K can be continuously mapped onto I . On the other hand, if w.K/ > 
C
then it follows from .K/ D w.K/ that K can be continuously mapped onto I
and hence K also maps continuously onto I .
Therefore, in all possible cases there is a continuous onto map q W K ! I .
Since Cp .X / is a dense subspace of RX , it follows from TFS-299 that we can find
a set A  X with jAj   and a continuous map g W A .Cp .X // ! I for which
g A D q '. If B D A then it is easy to see that there exists a continuous map
h W B .Cp .X // ! K such that h B D q '.
By strong -monolithity of X we have w.B/  ; since B is closed in X , we
conclude that l.B/  l.X / < cf./. This makes it possible to apply Problem 441
to see that the space Cp .B/ is strongly -cosmic and hence so is B .Cp .X // 
Cp .B/. Since the space I is a continuous image of B .Cp .X //, we obtained a
contradiction with Problem 439 and hence w.K/ < .
Finally, observe that the above result is applicable in the case when  D !1 and
X is a Lindelf strongly !1 -monolithic space. Thus, for every compact continuous
image K of the space Cp .X / we have w.K/ < !1 , i.e., w.K/  ! and hence K is
metrizable.
V.444. Given a cardinal  with cf./ > ! assume that X is a -monolithic Lindelf
-space. Prove that if K is a compact continuous image of Cp .X / then w.K/ <
. In particular, if X is an !1 -monolithic Lindelf -space then every compact
continuous image of Cp .X / is metrizable.
Solution. For any A  X let A W Cp .X / ! Cp .A/  RA be the restriction map.
Suppose that K is a compact space, ' W Cp .X / ! K is a continuous onto map and
w.K/  . Since Cp .X / is a dense subspace of RX , we can apply Problem 430 to
see that .K/ D w.K/. If w.K/ D  then cf./ > ! together with Problem 431
show that K can be continuously mapped onto I . On the other hand, if w.K/ > 
C
then it follows from .K/ D w.K/ that K can be continuously mapped onto I

and hence K also maps continuously onto I . Therefore, in all possible cases there
is a continuous onto map q W K ! I .
Since Cp .X / is a dense subspace of RX , it follows from TFS-299 that we can
find a set A  X with jAj   and a continuous map g W A .Cp .X // ! I for
which g A D q '. If B D A then it is easy to see that there exists a continuous
map h W B .Cp .X // ! K such that h B D q '.
In particular, the space Cp .B/ D B .Cp .X // maps continuously onto I . By
-monolithity of the space X we have nw.B/  ; since B is closed in X , it is
a Lindelf -space, so we can apply Problem 442 to see that Cp .B/ is strongly
-cosmic; this contradiction with Problem 439 shows that w.K/ < .

2 Solutions of problems 001500

513

Finally, observe that the above result is applicable in the case when  D !1
and X is an !1 -monolithic Lindelf -space. Thus, for every compact continuous
image K of the space Cp .X / we have w.K/ < !1 , i.e., w.K/  ! and hence K is
metrizable.
V.445. Let  be a cardinal and denote by D a discrete space of cardinality . For
the compact space K D D prove that Cp .K/ maps continuously onto I .
Solution. There exists a continuous surjective map ' W Cp .K/ ! Cp .K; I/ (see
TFS-092). If  W Cp .K; I/ ! ID is the restriction map then we have the equality
.Cp .K; I// D ID ; therefore  ' maps Cp .K/ continuously onto the space ID .
It is easy to find a homeomorphism  W ID ! I ; then   ' maps Cp .K/
continuously onto I .
V.446. Prove that every one of the following statements is equivalent to Luzins
axiom (2!1 > c/:
(i) for every separable compact space K any compact continuous image of Cp .K/
is metrizable;
(ii) any compact continuous image of Cp .!/ is metrizable;
(iii) any compact continuous image of Cp .Ic / is metrizable;
(iv) for every compact space K with w.K/  c, any compact continuous image of
Cp .K/ is metrizable.
Solution. A space X is called extremally disconnected if U is open for any
U 2 .X /. It is an easy exercise that a space X is extremally disconnected if and
only if for any disjoint sets U; V 2 .X / we have U \ V D ;. For any infinite
cardinal  let D be a discrete space of cardinality .
Fact 1. Suppose that f W X ! Y is a closed irreducible onto map. If Y is
extremally disconnected then f is a homeomorphism.
Proof. For any U 2 .X / let f ] .U / D Y nf .X nU /; it is evident that f ] .U / 
f .U / and f ] .U / is open in Y while f 1 .f ] .U //  U ; besides, f ] .U / is dense
in f .U / by Fact 1 of S.383.
Suppose that x; y 2 X and x y. Take disjoint sets U; V 2 .X / such that
x 2 U; y 2 V and U \ V D ;. It is straightforward that f ] .U / \ f ] .V / D ;. The
space Y being extremally disconnected, we have f ] .U /\f ] .V / D ;. Furthermore,
f .U / D f ] .U / and f .V / D f ] .V / which shows that f .U / \ f .V / D ;. Since
x 2 f .U / and y 2 f .V /, we conclude that f .x/ f .y/ whenever x and y
are distinct points of X . Therefore f is a bijection, so it is a homeomorphism by
TFS-155 and hence Fact 1 is proved.
Fact 2. For any cardinal  the space D is extremally disconnected.
Proof. If  is finite then D D D is a finite space, so it is discrete and hence
extremally disconnected. Now if   ! then take any set U 2 .D / and let
V D U \ D . Then U D V because D is dense in D . Besides, V is clopen in

514

2 Solutions of problems 001500

D by Fact 2 of T.371, so U is clopen in D for any U 2 .D / and hence the


space D is extremally disconnected, i.e., Fact 2 is proved.
Fact 3. If 2!1 D c and D is a discrete space of cardinality !1 then the space D
embeds in !.
Proof. Since d..D//  jDj D !1 , we can apply Fact 2 of S.368 to see that
w.D/  2!1 D c and hence we can choose a set F  Ic which is homeomorphic
to D. Take a countable set E  Ic which is dense in Ic and choose a surjection
' W ! ! E. The map ' is continuous because ! is discrete, so there exists a
continuous map W ! ! Ic such that j! D ' (see TFS-257). The set .!/
contains the dense set E of the space Ic , so .!/ D Ic .
Consider the set G D 1 .F / and let  D jG; then  W G ! F is a perfect
map. Therefore there exists a closed set G 0  G such that .G 0 / D F and jG 0
is irreducible (see TFS-366). Therefore jG 0 W G 0 ! F is a homeomorphism by
Fact 1, so F embeds in ! and hence D embeds in !, i.e., Fact 3 is proved.
Returning to our solution observe that (iv)H)(iii) because w.Ic /  c. If a
compact space K has weight at most c then K embeds in Ic and hence we can
consider that K  Ic . The restriction K W Cp .Ic / ! Cp .K/ is a continuous
onto map and hence any continuous image of Cp .K/ is also a continuous image of
Cp .Ic /; thus (iii)H)(iv), i.e., the statements (iii) and (iv) are equivalent.
It is evident that (i)H)(ii); since Ic is separable, we also have (i)H)(iii). Any
separable Tychonoff space has weight at most c (see Fact 2 of S.368), so (iv)H)(i)
which shows that the properties (i),(iii) and (iv) are all equivalent and each one of
them implies (ii).
Now, if Luzins Axiom holds then jCp .Ic /j D c < 2!1 D jI!1 j, so the space
Cp .Ic / cannot be mapped (even discontinuously) onto I!1 . If K is a compact
continuous image of Cp .Ic / then K cannot be mapped onto I!1 either so w.K/ D
.K/  ! (see Problem 430) and hence K is metrizable.
Thus, Luzin Axiom implies (iii); since (iii) implies all the statements (i)(iv),
Luzins Axiom implies (i)(iv).
Now, assume that 2!1 D c (i.e., Luzins Axiom does not hold) and let D be a
discrete space of cardinality !1 . By Fact 3 we can consider that D is a subspace
of !. The restriction D W Cp .!/ ! Cp .D/ is a continuous onto map and
the space Cp .D/ can be mapped on I!1 by Problem 445, so Cp .!/ also maps
continuously onto I!1 . Therefore (ii) does not hold and hence (ii) implies Luzins
Axiom together with all statements (i)(iv). Thus Luzins Axiom is equivalent to
every one of the statements (i)(iv), i.e., our solution is complete.
V.447. Under Luzins Axiom prove that
(a) if X is Lindelf and w.X /  c then every compact continuous image of Cp .X /
is metrizable.
(b) if X is Lindelf and first countable then every compact continuous image of
Cp .X / is metrizable.

2 Solutions of problems 001500

515

Solution. (a) We have jCp .X /j  w.X /l.X/  2! (see Fact 7 of U.074), so it


follows from Luzins Axiom that jCp .X /j < 2!1 D jI!1 j and hence Cp .X /
cannot be (continuously) mapped onto I!1 . If K is a compact continuous image
of Cp .X / then K does not map onto I!1 either and hence .K/  !. Applying
Problem 430 we conclude that w.K/ D .K/  ! and hence K is metrizable.
(b) It follows from Fact 2 of V.403 that jX j  2l.X/ .X/
 2! . Take a countable
S
local base Bx at every point x 2 X . Then B D fBx W x 2 X g is a base in X
such that jBj  c ! D c. Therefore w.X /  c and hence we can apply (a) to
conclude that every compact continuous image of Cp .X / is metrizable.
V.448. Prove that, for every hereditarily Lindelf first countable space X , any
compact continuous image of Cp .X / is metrizable. In particular, for each perfectly
normal compact space X , any compact continuous image of Cp .X / is metrizable.
Solution. For any A  X let A W Cp .X / ! Cp .A/ be the restriction map. If some
compact continuous image K of the space Cp .X / is not metrizable then we have
.K/ D w.K/ > ! (see Problem 430) and hence K maps continuously onto I!1 .
Therefore there exists a continuous surjective map ' W Cp .X / ! I!1 . By TFS299 we can find a set A  X such that jAj  !1 and there is a continuous map
q W A .Cp .X // ! I!1 for which q A D '.
Fix S
a countable local base Bx at every point x 2 X and consider the family
B D fBx W x 2 Ag; it is clear that jBj  !1 and fB \ A W B 2 Bg is a
base in A. Thus w.A/  !1 and hence A is a Lindelf space of weight  !1 which
implies that Cp .A/ is strongly !1 -cosmic by Problem 441. The space A .Cp .X // 
Cp .A/ is also strongly !1 -cosmic and hence so is I!1 (see Problem 438) which is a
contradiction with Problem 439.
Finally observe that if X is a perfectly normal compact space then .X /  !
and X is hereditarily Lindelf by SFFS-001. Therefore every compact continuous
image of Cp .X / is metrizable.
V.449. Suppose that X is a space such that l.Cp .X // D t .Cp .X // D !. Prove
that every compact continuous image of Cp .X / is metrizable.
Solution. Given a space Z recall that a set S D fx W < g  Z is called a free
sequence in Z if fx W < g \ fx W  g D ; for any < . The cardinal  is
called the length of the free sequence S .
Fact 1. Suppose that  is an infinite cardinal and Z is a space such that l.Z/  
and t .Z/  . Then Z has no free sequences of length  C .
Proof. Assume that S D fx W <  C g is a free sequence in Z and consider the
sets F D fx W Tg and G D fx W < g for any <  C . Let U D ZnF
for each <  C ; if fF W <  C g D ; then the family fU W <  C g is an open
0
cover of the space Z such that < 0 implies
S U  U . It follows from l.Z/  
that there exists <  C such that U D fU W < g D Z. However,
T x U
which is a contradiction. Consequently, there exists a point x 2 F D < C F .

516

2 Solutions of problems 001500

It follows from x 2 fx W <  C g D F0 that we can choose a set A   C such


that jAj   and x 2 fx W 2 Ag; pick an ordinal <  C with A  ; then
x 2 fx W 2 Ag  G . It follows from x 2 F that x 2 F and hence x 2 F \ G
which is a contradiction with the definition of free sequence. Thus Z has no free
sequences of length  C , i.e., Fact 1 is proved.
Returning to our solution suppose that ' W Cp .X / ! K is a continuous onto map
of Cp .X / onto a compact space K. If fx W < !1 g is a free sequence in K then
pick a point f 2 ' 1 .x / for every < !1 . It is straightforward that ff W < !1 g
is a free sequence in Cp .X / which contradicts t .Cp .X // D l.Cp .X // D ! (see
Fact 1).
Therefore the space K has no uncountable free sequences and hence t .K/  !
by TFS-328. Applying Problem 430 we conclude that w.K/ D t .K/  !, so K is
metrizable and hence our solution is complete.
V.450. Prove that if X is a space such that Cp .X / is a Lindelf -space then every
compact continuous image of Cp .X / is metrizable.
Solution. Denote by X the Hewitt realcompactification of the space X . It is
evident that Cp .X / is a continuous image of the space Cp .X /. Besides, Y D X
is a Lindelf -space (see CFS-206) and hence t .Cp .Y // D !; since Cp .Y / is
also a Lindelf -space (see CFS-234), we can apply Problem 449 to see that
every compact continuous image of Cp .Y / is metrizable. Therefore every compact
continuous image of Cp .X / is also metrizable.
V.451. For the double arrow space K prove that the space Cp .K/ can be
condensed onto I! .
Solution. Suppose that Z is a space, f W Z ! R and z 2 Z. Then the number
osc.f; z/ D inffdiam.f .U // W U 2 .z; Z/g is called the oscillation of f at
the point z. Recall that f is continuous at the point z if and only if osc.f; z/ D 0
(Fact U.347).
As usual, D D f0; 1g; given a; b 2 R we will have to distinguish the point
.a; b/ 2 R2 from the interval .a; b/ D ft 2 R W a < t < bg  R. To avoid
confusion, the point in the plane whose coordinates are a and b will be denoted
by ha; bi. Recall that K D K0 [ K1  R2 where K0 D .0; 1
 f0g and K1 D
0; 1/  f1g. For any x D ht; i i 2 K let p.x/ D t ; thus p W K ! 0; 1
is the
restriction to K of the respective projection of the plane.
If x D ht; 0i 2 K0 and 0 < a < t then Oa .x/ D fxg [ ..a; t /  D/. The family
fOa .x/ W 0 < a < p.x/g is a local base of K at the point x. If x D ht; 1i 2 K1 and
t < a < 1 then Oa .x/ D fxg [ ..t; a/  D/. The family fOa .x/ W p.x/ < a < 1g is
a local base of K at the point x.
We will also need the lexicographical order on the set K. Given two points
x D .t; i / and y D .s; j / of the set K say that x y if either t < s or t D s and
i < j . It is an easy exercise that .K; / is a linearly ordered set and the topology of
K is generated by the order .

2 Solutions of problems 001500

517

The space K is first countable (TFS-384), so we can fix, for any x 2 K, a local
base fUn .x/ W n 2 !g of the space K at the point x such that UnC1  Un for any
n 2 !. For any A  K let A W Cp .K/ ! Cp .A/ be the restriction map and let
Cp .AjK/ D A .Cp .K//.
Fact 1. If A is a countable dense subset of the double arrow space K then Cp .AjK/
is an F -subset of RA and, in particular, Cp .AjK/ is a Borel set.
Proof. Given any points x; y 2 A and m 2 ! it is easy to see that the set
D.x; y; m/ D ff 2 RA W jf .x/  f .y/j  2m g is closed in RA . The oscillation
of a function f 2 RA at a point a 2 A is equal to zero if and only if for any m 2 !
there exists n 2 !Tsuch that jf .x/  f .y/j  2m for all x; y 2 Un .a/ \ A. The
set E.m; n; a/ D fD.x; y; m/ W x; y 2 Un .a/ \ Ag is also closed in RA for any
m; n 2 ! andT
a 2 A;Sit is immediate that a function f 2 RA is continuous at a if and
only if f 2 m2! n2! E.m;
a function f 2 RA is continuous
TTherefore
S
Tn; a/.
on A if and only if f 2 E D a2A m2! n2! E.m; n; a/, i.e., Cp .A/ D E. It is
clear that E is an F -subset of RA , so we proved that
(1) Cp .A/ is an F -subset of RA .
However, the set Cp .AjK/ is strictly smaller than Cp .A/, so we need something
else to characterize Cp .AjK/. Consider the following property of a function
f 2 RA .
. / for any sequence S D fan W n 2 !g  A if S ! x and x 2 KnS then the
sequence ff .an / W n 2 !g is convergent.
Let P be the set of all functions from RA which have the property . /; it is
clear that Cp .AjK/  P \ Cp .A/. To prove the reverse inclusion take any function
f 2 Cp .A/ which has the property . /. For any point x 2 K there is a sequence
fan W n 2 !g  Anfxg which converges to x; let g.x/ be the limit of the sequence
ff .an / W n 2 !g which exists by . /.
If fbn W n 2 !g  Anfxg is any other sequence that converges to x and ff .bn / W
n 2 !g does not converge to g.x/ then let z2n D an and z2nC1 D bn for any n 2 !.
The sequence fzn W n 2 !g  Anfxg still converges to x but ff .zn / W n 2 !g is
not convergent; this contradiction with . / shows that we have consistently defined
a function g W K ! R such that gjA D f .
Assume that the function g is discontinuous at some point x 2 K and take
" > 0 such that diam.g.U // > " for any U 2 .x; K/. For every n 2 ! fix points
yn ; zn 2 Un .x/ such that jg.yn /  g.zn /j > ". It is easy to find sequences fakn W k 2
!g  .A \ Un .x//nfx; yn ; zn g and fbkn W k 2 !g  .A \ Un .x//nfx; yn ; zn g such
that akn ! yn and bkn ! zn (when k ! 1). By our construction of the function
g we have g.akn / ! g.yn / and g.bkn / ! g.zn / when k ! 1. Therefore we can
choose kn ; ln 2 ! such that jg.aknn /  g.yn /j < 3" and jg.blnn /  g.zn /j < 3" .
It is easy to see that faknn ; blnn W n 2 !g  Anfxg is a sequence which converges
to x. Therefore the sequence S D fg.aknn /; g.blnn / W n 2 !g must be convergent. If,
for some n 2 ! we have jg.aknn /  g.blnn /j < 3" then

518

2 Solutions of problems 001500

jg.yn /  g.zn /j  jg.yn /  g.aknn /j C jg.aknn /  g.blnn /j C jg.blnn /  g.zn /j < ";
which is a contradiction. Therefore jg.aknn /  g.blnn /j  3" for all n 2 ! and hence
the sequence S cannot be convergent. This contradiction shows that g is continuous
and hence we proved that Cp .AjK/ consists of the functions from Cp .A/ which
satisfy the condition . /.
Let us also consider the following condition for a function f 2 RA .
./ For each m 2 ! we can find n 2 ! such that for any a0 ; : : : ; an 2 A with
a0 : : : an , there exists i < n for which jf .ai /  f .aiC1 /j  2m .
Denote by Q the set of functions from RA which satisfy the condition ./.
To show that Cp .AjK/  Q take any f 2 Cp .AjK/; then f D gjA for some
g 2 Cp .K/. Given any m 2 ! it follows from compactness of K that there exists
an open cover fU0 ; : : : ; Uk g of the space K such that diam.g.Ui // < 2m for every
i  k. Making the sets Ui smaller if necessary we can assume that every Ui is an
interval with respect to the order .
Let n D k C 1 and take any points a0 ; : : : ; an 2 A such that a0 : : : an .
It follows from n > k that there are i; j  n such that i < j and ai ; aj 2 Ul for
some l  k. Since the set Ul is an interval, it follows from ai aiC1  aj that
aiC1 2 Ul , so jf .ai /  f .aiC1 /j D jg.ai /  g.aiC1 /j  diam.Ul /  2m which
shows that f has the property ./ and hence we established that Cp .AjK/  Q.
It turns out that Q  P or, equivalently, the property ./ for a function f 2 RA
implies that f satisfies . /. To see this, assume that a function f 2 RA has ./
and take any sequence S D fai W i 2 !g  A which converges to some point
x S . If the sequence ff .ai / W i 2 !g is not convergent then it is not a Cauchy
sequence and hence there exists " > 0 and a sequence fik ; jk W k 2 !g  ! such
that ik < jk < ikC1 for all k 2 ! while jf .aik /  f .ajk /j > " for all k 2 !.
From now on we assume that x D ht; 0i for some t 2 .0; 1
; it is an exercise for
the reader to carry out the respective adjustments to make the proof work in the case
when x 2 K1 . It follows from S ! x that the set fi 2 ! W ai xg is finite and the
set fi 2 ! W ai yg is finite for any y x.
Let b0 D ai0 ; proceeding inductively assume that n 2 ! and we have points
b0 ; : : : ; bn 2 S such that b0 : : : bn and jf .bi /  f .biC1 /j > 2" for any
i < n. Since only finitely many elements of S are below bn under the order ,
we can find k 2 ! such that aik > bn and ajk > bn . If jf .aik /  f .bn /j  2" and
jf .ajk /f .bn /j  2" then jf .aik /f .ajk /j  " which is a contradiction. Therefore
we can find a point bnC1 2 faik ; ajk g such that jf .bn /  f .bnC1 /j > 2" .
Therefore our inductive procedure can be continued to construct a sequence S 0 D
fbi W i 2 !g  S such that bi biC1 and jf .bi /  f .biC1 /j > 2" for all i 2 !. Take
m 2 ! such that 2m < 2" and observe that the sequence S 0 is a counterexample
to the existence of the number n promised in ./. This contradiction shows that
Q  P and hence Cp .AjK/ consists of functions from Cp .A/ with the property
./, i.e., Cp .AjK/ D Q \ Cp .A/.
For arbitrary points a0 ; : : : ; an 2 A and any number m 2 ! consider the set
H.a0 ; : : : ; an / D ff 2 RA W jf .ai /  f .aiC1 /j  2m for some i < ng; it follows

2 Solutions of problems 001500

519

S
from the equality H.a0 ; : : : ; an / D fD.ai ; aiC1T
; m/ W iS< ngT
that H.a0 ; : : : ; an /
is closed in RA . It is straightforward that Q D m2! n2! fH.a0 ; : : : ; an / W
a0 ; : : : ; an 2 A and a0 : : : an g, so Q is and F -subset of RA . Applying (1) we
conclude that Cp .AjK/ D Q \ Cp .A/ is an F -subset of RA and hence Fact 1 is
proved.
Returning to our solution take a countable dense subspace A of the space K. It is
clear that A W Cp .K/ ! Cp .AjK/ is a condensation. By Fact 1 the space Cp .AjK/
is a Borel set. If Cp .AjK/ is  -compact then A is a P -space (see Fact 4 of S.186);
any countable P -space is discrete while A has no isolated points because K has no
isolated points. This contradiction shows that Cp .AjK/ is not  -compact, so we can
apply SFFS-354 to see that Cp .AjK/ can be condensed onto I! . Therefore Cp .K/
also condenses onto I! and hence our solution is complete.
V.452. Prove that Cp .Dc / condenses onto I! .
Solution. Suppose that Z is a space and A is a dense subspace of Z. Given a
continuous function ' W A ! R let osc.'; z/ D inffdiam.'.U \A// W U 2 .z; Z/g
for any z 2 Z; the number osc.'; z/ is called the oscillation of ' at the point z. If
Z is a space and D  Z then D W Cp .Z/ ! Cp .D/ is the restriction map and
Cp .DjZ/ D D .Cp .Z//.
Fact 1. Suppose that M is a compact space and D is a countable subset of Cp .M /
such that the closure K of the set D in the space RM is compact. Then Cp .DjK/ is
an F -subset of RD .
Proof. To characterize the set Cp .DjK/ we must describe what functions from
Cp .D/ can be extended over K. Given any k 2 N and t D .t1 ; : : : ; tk / 2 M k
let coord.t / D ft1 ; : : : ; tk g. For an arbitrary finite set A  M and f; g 2 D let
A .f; g/ D supfjf .a/g.a/j W a 2 Ag; clearly, OA .f; / D fg 2 RM W F .f; g/ <
g is an open subset of RM for any > 0; f 2 RM and finite A  M .
The following condition on a function ' 2 RD turns out to be a characterization
of functions from Cp .DjK/.
. / for any m 2 ! there exists a number n 2 ! and a finite set A  M such that
j'.f /  '.g/j  2m whenever f; g 2 D and A .f; g/ < 2n .
Denote by P the set of all functions from RD which have the property . / and fix
a function ' 2 P . Given any h 2 K and " > 0 take m 2 ! with 2m < " and apply
. / to find a number n 2 ! and a finite set A  M such that A .f; g/ < 2n implies
j'.f /  '.g/j < 2m for any f; g 2 D. The set U D ff 2 K W A .f; h/ < 2n1 g
is an open neighborhood of h in K. If f; g 2 U \ D then it is easy to see that
A .f; g/  A .f; h/ C A .h; g/ < 2n and hence j'.f /  '.g/j  2m < "; this
shows that diam.'.U \ D//  ". Since " > 0 was chosen arbitrarily, we proved
that, for any " > 0, there exists a set U 2 .h; K/ such that diam.'.U \ D//  ";
therefore osc.'; h/ D 0 for any h 2 K. Applying Fact 3 of T.368 we conclude
that the function ' can be continuously extended over K, i.e., ' 2 Cp .DjK/ and
therefore P  Cp .DjK/.

520

2 Solutions of problems 001500

To prove the inclusion Cp .DjK/  P take any function  2 Cp .K/ and let
' D jD. Fix any m 2 !; by continuity of , for any h 2 K we can choose a finite
set Ah  M and h > 0 such that the diameter of the set .OAh .h; 2h / \ K/ does
not exceed 2m .
The space K being compact we can extract a finite S
subcover of the cover
fOAh .h; S
fOAh .h; h / W h 2 F g.
h / W h 2 Kg, so fix a finite F  K such that K 
Let A D fAh W h 2 F g; D nnfh W h 2 F g and pick a number n 2 ! such that
2n < .
Given any f; g 2 D with A .f; g/ < 2n take a function h 2 F such that
f 2 OAh .h; h /. We have jf .x/  g.x/j < 2n <  h for any x 2 A and
hence these inequalities also hold for any x 2 Ah . It is easy to see that this implies
g 2 OAh .h; 2h /, so j'.f /  '.g/j  diam..OAh .h; 2h / \ K//  2m . This
proves that any function from Cp .DjK/ has . / and hence Cp .DjK/  P , so we
established that P D Cp .DjK/.
For any m; n 2 ! and k 2 N consider the set F .m; n; k/ D f.'; t / 2 RD  M k W
for any f; g 2 D either coord.t/ .f; g/  2n or j'.f /  '.g/j  2m g.
Take any .; s/ 2 .RD  M k /nF .m; n; k/; there exist functions f; g 2 D such
that coord.s/ .f; g/ < 2n and j.f /  .g/j > 2m . Let s D .s1 ; : : : ; sk /; the
functions f and g being continuous on M , the set
W D ft D .t1 ; : : : ; tk / 2 M k W jf .ti /  g.ti /j < 2n for all i  kg
is open in M k and s 2 W . It is clear that G D f' 2 RD W j'.f /  '.g/j > 2m g
is open in RD and  2 G. We have .; s/ 2 G  W and .G  W / \ F .m; n; k/ D
;, so every point of .RD  M k /nF .m; n; k/ has a neighborhood contained in the
set .RD  M k /nF .m; n; k/. Therefore the complement of F .m; n; k/ is open in
RD  M k and hence F .m; n; k/ is closed in RD  M k for any m; n 2 ! and k 2 N.
Let  W RD M k ! RD be the natural projection. The space M k being compact,
the map  is closed (see Fact 3 of S.288), so the set E.m; n; k/ D .F .m; n; k//
is closed in RD for all m; n 2 ! and
S it is easy to see that a function
T k 2SN. Now
' 2 RD has . / if and only if ' 2 m2!
S k2N
S E.m; n; k/.
T n2!
Thus we have the equality P D m2! n2! k2N E.m; n; k/ D Cp .DjK/
which shows that Cp .DjK/ is an F -subset of RD and hence Fact 1 is proved.
Returning to our solution consider the compact space M D D! ; then the space
K D DM is compact. The space D D Cp .M; D/  Cp .M / is countable and dense
in K (see Fact 1 of U.077 and Fact 1 of S.390). Therefore Fact 1 can be applied to see
that C D Cp .DjK/ is an F -subset of RD , so C is a Borel set. If C is  -compact
then D has to be a P -space by Fact 4 of S.186. Any countable P -space is discrete;
since D has no isolated points, we obtained a contradiction which shows that C is
not  -compact. Thus we can apply SFFS-354 to see that C can be condensed onto
I! . The space Cp .K/ condenses onto C , so Cp .K/ can also be condensed onto I! .
Finally observe that K is homeomorphic to Dc and hence Cp .Dc / condenses onto
the space I! .

2 Solutions of problems 001500

521

V.453. Suppose that X is a nonempty  -compact second countable space. Prove


that Cp .X / condenses onto a compact space.
Solution. Observe first that R is a locally compact space, so it condenses onto a
compact space K by Fact 3 of T.357. Therefore RX condenses onto the compact
space K X . This shows that if X is discrete then Cp .X / D RX condenses onto a
compact space.
Now assume that X is not discrete and fix a non-isolated point a 2 X . It is easy
to find a countable dense set A  X such that a 2 A and hence the point a is not
isolated in A, which implies that the space A is not discrete. The restriction map
A W Cp .X / ! Cp .A/ is a condensation of Cp .X / onto Cp .AjX / D A .Cp .X //.
It follows from SFFS-368 that Cp .AjX / is a Borel set in RA . If the space
Cp .AjX / is  -compact then A has to be a P -space by Fact 4 of S.186; since any
countable P -space must be discrete, A is discrete which is a contradiction. Therefore Cp .AjX / is not  -compact so there exists a condensation ' W Cp .AjX / ! I!
by SFFS-354. The map ' A is a condensation of Cp .X / onto I! , so Cp .X /
condenses onto a compact space in all possible cases.
V.454. Prove that there exists a set X  R such that Cp .X / does not condense
onto an analytic space.
Solution. Given a space Z and A  Z let A W Cp .Z/ ! Cp .A/ be the restriction
map and Cp .AjZ/ D A .Cp .Z//.
Fact 1. Suppose that Z is a metrizable space and D  Z is a dense subspace
of Z. If D  Y  Z and Y Z then Cp .DjZ/  Cp .DjY / and Cp .DjZ/
Cp .DjY /.
Proof. It is evident that Cp .DjZ/  Cp .DjY /; pick a point y 2 ZnY and fix a
1
metric d which generates the topology of Z. Letting f .x/ D d.x;y/
for any x 2 Y
we obtain a function f 2 Cp .Y / and hence D .f / 2 Cp .DjY /.
Assume that there exists a function h 2 Cp .Z/ such that hjD D f jD and take a
sequence fan W n 2 !g  D which converges to y. Then S D fh.an / W n 2 !g must
converge to h.y/. However h.an / D f .an / D d.a1n ;y/ ! 1 and hence S is not
convergent. This contradiction shows that D .f / 2 Cp .DjY /nCp .DjZ/, so Fact 1
is proved.
Returning to our solution denote by K the Cantor set and fix a countable dense
set Q  K; we consider that K  R. Say that a set E  K has the Bernstein
property if E \ F ; for any uncountable closed set F  K. Observe that A D
ffxg  K W x 2 Kg is a disjoint family of uncountable closed subsets of K  K
such that jAj D c. Since K  K is homeomorphic to K by SFFS-348, there exists
a disjoint family A0 of uncountable closed subsets of K such that jA0 j D c. As an
immediate consequence,
(1) jEj D c for any set E  K with the Bernstein property.
Apply Fact 5 of S.151 to fix a set E  K such that Q  E and both sets E and
KnE have the Bernstein property. Consider the family F of all continuous functions

522

2 Solutions of problems 001500

' W B ! R! where B is an analytic subset of RD for some countable D  K with


Q  D. It is easy to see that jFj D c, so we can take an enumeration f' W < cg of
the family F. Thus, for every < c we have a countable set D with Q  D  K
and an analytic set B  RD such that ' W B ! R! .
Denote by G the family of all G -subsets of K which contain E. Observe first
that
.2/ KnG is countable for any G 2 G.
S
Indeed, KnG D fKn W n 2 !g where every Kn is closed in K. If KnG is
uncountable, then the set Kn is uncountable for some n 2 !; since Kn \ E D ;,
we obtain a contradiction with the Bernstein property of E. It follows from (1) that
jKnEj D c, so the following condition is an immediate consequence of .2/:
T
(3) if G 0  G and jG 0 j < c, then the set . G 0 /nE has cardinality c.
Let u.t / D 0 for every t 2 K and denote by X0 the set E. We have to consider 4
cases:
Case 1. D0 nX0 ;. Pick y0 2 D0 nX0 and fix an arbitrary point x0 2 KnE
distinct from y0 . Take A0 D K and f0 D g0 D u.
Case 2. D0  X0 and Cp .D0 jX0 /nB0 ;. Take any h0 2 Cp .X0 / such that
h0 jD0 B0 and apply Fact 3 of T.368 to find a set A0 2 G and a continuous
function f0 W A0 ! R such that X0  A0 and f0 jX0 D h0 . Let g0 D f0 and
apply (3) to pick x0 2 A0 nX0 and y0 2 Kn.X0 [ fx0 g/.
Case 3. D0  X0 , Cp .D0 jX0 /  B0 and '0 jCp .D0 jX0 / is not injective. Take
distinct functions p0 ; q0 2 Cp .X0 / such that '0 .p0 jD0 / D '0 .q0 jD0 / and apply
Fact 3 of T.368 once more to find a set A0 2 G and continuous functions f0 ; g0 W
A0 ! R such that X0  A0 while f0 jX0 D p0 and g0 jX0 D q0 . Pick a point
x0 2 A0 nX0 and take any point y0 2 Kn.X0 [ fx0 g/.
Case 4. D0  X0 , Cp .D0 jX0 /  B0 and '0 jCp .D0 jX0 / is injective. Then choose
a point x0 2 KnX0 and take any point y0 2 Kn.X0 [ fx0 g/. Finally, let A0 D K
and f0 D g0 D u.
Proceeding inductively assume that 0 < < c and we have constructed a set
fx ; y W < g  KnE, a set of functions ff ; g W < g and a family of sets
fA W < g  G with the following properties:
ff ; g g  Cp .A / for any < ;
if <  < then x x ;
x y for any ;  < ;
if <T, X D E [ fx W  < g and D nX ; then y 2 D nX ;
x 2 fA W   g for any < ;
for each < , if D  X and Cp .D jX /nB ;, then X  A and
f jD B ;
(10) if < ; D  X ; Cp .D jX /  B and ' jCp .D jX / is not injective,
then X  A ; f g and ' .f jD / D ' .g jD /.
(4)
(5)
(6)
(7)
(8)
(9)

Let X D E [ fx W < g. If D nX T; then pick any point y 2 D nX ;


let A D K and f D g D u. The set A D . fA W < g/nE is has cardinality

2 Solutions of problems 001500

523

c by (3), so we can choose a point x 2 An.X [ fy W  g/. It is evident that


the conditions (4)(10) are still satisfied for all  .
Now if D  X and Cp .D jX /nB ; then take any h 2 Cp .X / such
that h jD B and apply Fact 3 of T.368 to find a set A 2 G and a continuous
function f W A ! R T
such that X  A and f jX D h . Let g D f and
apply (3) to pick x 2 . fA W  g/n.X [ fy W < g/. Take any point
y 2 Kn.X [fx g/ and observe that the properties (4)(10) now hold for all  .
Suppose that we have the case when D  X and Cp .D jX /  B while
the map ' jCp .D jX / is not injective. Take distinct functions p ; q 2 Cp .X /
such that ' .p jD / D ' .q jD / and apply Fact 3 of T.368 once more to find a
set A 2 G and continuous functions f ; gT
W A ! R such that X  A while
f jX D p and g jX D q . Pick x 2 . fA W  g/n.X [ fy W < g/
and take any y 2 Kn.X [ fx g/; observe that the properties (4)(10) still hold for
all  .
The remaining case is when D  X and Cp .D jX /  B while the map
' jCp .D
T jX / is injective. Then apply the property (3) once more to choose a point
x 2 . fA W  g/n.X [ fy W < g/ and take any point y 2 Kn.X [
fx g/. Let A D K; f D g D u and observe that the properties (4)(10) keep
holding for all  .
Therefore we can carry out the c-many steps in our inductive procedure to obtain
a set fx ; y W < cg  KnE, a set of functions ff ; g W < cg and a family
fA W < cg  G such that the properties (4)(10) hold for all < c. Our
promised set will be X D E [ fx W < cg. Suppose that Cp .X / condenses onto
an analytic space Y . Then nw.Y /  nw.Cp .X //  ! and hence Y is condensable
onto a second countable space Z; it is clear that Z has to be analytic. Thus Cp .X /
condenses onto a second countable analytic space Z and we can consider that
Z  R! .
Let  W Cp .X / ! Z be a condensation and apply TFS-299 to see that we can
find a countable set D  X such that Q  D and there is a continuous map  W
Cp .DjX / ! Z for which  D D ; it is clear that  is a condensation. By Fact 3
of T.368 there exists a G -subset H of the space RD such that Cp .DjX /  H and
there is a continuous map  W H ! R! with jCp .DjX / D . If B D  1 .Z/ and
' D jB then B is an analytic subset of RD (see SFFS-338) such that Cp .DjX / 
B while ' W B ! Z is a continuous map and 'jCp .DjX / is injective.
The map ' belongs to the family F, so there exists an ordinal < c such that
' D ' ; B D B and D D D . If D nX ; then it follows from (7) that
y 2 D. However, the property (6) shows that y X while y 2 D  X which is
a contradiction.
Therefore D  X ; assume for a moment that Cp .D jX /nB ;. Then we
can apply the property (9) to see that X  A and f jD B . Since x 2 A for
all  by (8), we conclude that X  A and hence g D f jX is a function from
Cp .X / such that gjD D f jD B ; this is a contradiction with Cp .DjX /  B .
Thus D  X and Cp .D jX /  B . Assume that the map 'jCp .D jX /
is not injective. Then the property (10) is applicable and hence X  A while

524

2 Solutions of problems 001500

f g and '.f jD/ D '.g jD/. Since x 2 A for all  by (8), we


conclude that X  A and hence f D f jX and g D g jX are distinct functions
from Cp .X /, so the functions f jD and gjD are distinct as well by Fact 0 of S.351.
However, '.f jD/ D '.f jD/ D '.g jD/ D '.gjD/ which contradicts the fact
that 'jCp .DjX / is injective.
Finally assume that we have D  X while Cp .D jX /  B and the map
'jCp .DjX / is injective. It follows from Cp .DjX /  B that '.Cp .DjX //  Z.
By Fact 1, there exists a function h 2 Cp .DjX /nCp .DjX /. By injectivity of the
map 'jCp .DjX / we have h 2 Zn'.Cp .DjX //; this final contradiction with the
equality '.Cp .DjX // D Z shows that Cp .X / cannot be condensed onto an analytic
space, so our solution is complete.
V.455. Given a nonempty space X such that jCp .X /j < 2d.X/ prove that Cp .X /
cannot be condensed onto a compact space. Deduce from this fact that neither one
of the spaces Cp .!n!/ and Cp ..Dc // condenses onto a compact space. Here
.Dc / D fx 2 Dc W jx 1 .1/j  !g is the -product of Dc .
Solution. It is an easy consequence of the inequality jCp .X /j < 2d.X/ that X is an
infinite space. Suppose that ' W Cp .X / ! K is a condensation of Cp .X / onto a
compact space K and let  D d.X /. If .x; K/   for any x 2 K then jKj  2
(see TFS-330). However, jKj  jCp .X /j < 2 which is a contradiction. Therefore
we can find a point x 2 K such that .x; K/  .x; K/ < . T
Fix a family U  .x; K/ such that jU
U . If '.f / D x
Tj <  and fxg D
then ff g D ' 1 .x/ and hence ff g D f' 1 .U / W U 2 U g. This proves that
.f; Cp .X //  jU j < . The space Cp .X / being homogeneous, we must have
.Cp .X // <  and hence d.X / D .Cp .X // <  D d.X / (see TFS-173) which
is a contradiction. Thus Cp .X / cannot be condensed onto a compact space.
Now let X D !n!; it follows from separability of ! that jCp .!/j  c. Since
Cp .!/ maps continuously onto Cp .X / (see TFS-380), we have jCp .X /j  c. Next
apply TFS-371 to see that d.X /  c.X / D c and hence jCp .X /j D c < 2c  2d.X/ ,
so Cp .X / cannot be condensed onto a compact space.
Finally assume that X D .Dc /. The space Dc is separable, so jCp .Dc /j  c.
For any A  c let pA W Dc ! DA be the natural projection. It is easy to see that
pA .X / D DA for any countable A  c, so we can apply Fact 2 of S.433 to see that
the space X is pseudocompact (in fact, it is countably compact) and hence Dc D X
(see Fact 2 of S.309). This implies that the restriction map X W Cp .Dc / ! Cp .X /
is surjective
hence jCp .X /j  jCp .Dc /j  c. If D  X and jDj < c then the
S and
1
set A D fx .1/ W x 2 Dg has cardinality not exceeding jDj ! < c, so we
can choose an ordinal 2 cnA. Then W D fx 2 X W x./ D 1g is nonempty
and open in X while W \ D D ;. Therefore D is not dense in X ; this proves that
d.X /  c and hence jCp .X /j  c < 2c  2d.X/ which shows that Cp .X / cannot be
condensed onto a compact space.
V.456. Prove that if 2! D !1 and 2!1 > !2 then Cp .D!2 / does not condense onto
a compact space.

2 Solutions of problems 001500

525

Solution. The cardinal !2 is not !-cofinal; thus, for any function f W ! ! !2


there exists
that f .!/  and hence f 2 ! . Therefore
S an! ordinal < !2 such
!
!
!2 D j f W < !2 gj  !1 !2 . Besides, !1!  .2! /! D 2! D !1 . As a
consequence, !2!  !1 !2 D !2 .
Apply Fact 7 of U.074 to see that jCp .D!2 /j  w.D!2 /! D !2! D !2 . If the
!
space D!2 is separable then w.D!2 /  2d.D 2 /  2! D !1 (see Fact 2 of S.368)
which is a contradiction. Therefore d.D!2 /  !1 and hence we can conclude that
!
jCp .D!2 /j D !2 < 2!1  2d.D 2 / , so it follows from Problem 455 that Cp .D!2 /
does not condense onto a compact space.
V.457. Suppose that 2! < 2!1 and Cp .X / condenses onto a compact space. Prove
that X is separable if and only if jCp .X /j  c.
Solution. If X is separable then fix a countable dense set D  X ; the restriction
map condenses Cp .X / onto a subspace of Cp .D/. This implies the inequalities
jCp .X /j  jCp .D/j  jRD j  c! D c, so jCp .X /j  c. Observe that we need
neither Luzins Axiom nor condensations of Cp .X / onto a compact space.
Now assume that Cp .X / condenses onto a compact space K while jCp .X /j  c
and Luzins Axiom holds. Fix a condensation ' W Cp .X / ! K. If .x; K/  !1 for
any x 2 K then jKj  2!1 (see TFS-330) which, together with jKj D jCp .X /j 
c < 2!1 gives a contradiction. Therefore .x; K/  ! for
T some x 2 K, so we can
fix a countable family U  .x; K/ such that fxg D
U. T
Take a function f 2
Cp .X / such that '.f / D x. It is straightforward that ff g D f' 1 .U / W U 2 U g
and hence .f; Cp .X //  !. The space Cp .X / is homogeneous and therefore
.Cp .X // D !, so we can apply TFS-173 to see that d.X / D .Cp .X // D !,
i.e., X is separable.
V.458. Suppose that l.X n / D ! for all n 2 N and Cp .X / is Lindelf. Observe
that if Cp .X / condenses onto a compact space K then K is metrizable and X is
separable. Prove that if Cp .X / condenses onto a  -compact space Y then X is
separable and .Y / D !. Deduce from this fact that if X is a non-metrizable
Corson compact space then Cp .X / does not condense onto a  -compact space.
Solution. Suppose first that Cp .X / condenses onto a compact space K. It follows
from l  .X / D ! that t .Cp .X // D ! and hence we can apply Problem 449 to
convince ourselves that K is metrizable. The existence of a condensation onto K
together with w.K/  ! shows that i w.Cp .X //  ! and hence d.X /  ! (see
TFS-173).
Now assume that Cp .X / condenses onto a  -compact space and fix a condensation ' W Cp .XS/ ! Y and a family fKn W n 2 !g of compact subsets of Y
such that Y D n2! Kn . The set Fn D ' 1 .Kn / is closed in Cp .X / for every
n 2 !. If n 2 ! and S is an uncountable free sequence in Kn then the set
S 0 D ' 1 .S / is an uncountable free sequence in Fn which is impossible because
l.Fn /  l.Cp .X // D ! and t .Fn /  t .Cp .X // D ! (see Fact 1 of V.449).
This contradiction shows that the space Kn has no uncountable free sequences and
therefore t .Kn /  ! for any n 2 ! (see TFS-328).

526

2 Solutions of problems 001500

Apply SFFS-432 to see that there exists n 2 ! such that the space Cp .X / is
homeomorphic to C for some C  Fn . Since 'jC maps C into Kn , it follows from
Problem 433 that .'.C //  !. Since the map 'jC is a condensation, we must
have .Cp .X // D .C /  .'.C // D ! and hence d.X / D .Cp .X // D !,
i.e., X is separable as promised.
It is a consequence of .Cp .X // D ! that Cp .X /nff g is an F -set for any
f 2 Cp .X /. The space Cp .X / being Lindelf, Cp .X /nff g is Lindelf as well.
Therefore Y nfyg is Lindelf for any y 2 Y ; this implies that .Y /  !.
Finally note that l.Cp .X // D t .Cp .X // D ! for any Corson compact space X
(see CFS-150), so if Cp .X / condenses onto a  -compact space then X is separable
and hence metrizable by CFS-121.
V.459. Given a space X prove that the space Cp .X / condenses onto a space
embeddable in a compact space of countable tightness if and only if Cp .X /
condenses onto a second countable space.
Solution. If Cp .X / condenses onto a second countable space M then M is
embeddable in a metrizable compact space K (see TFS-209); of course then t .K/ 
! and hence we proved sufficiency.
Now if ' W Cp .X / ! Y is a condensation of Cp .X / onto a space Y such that
Y  K for some compact space K of countable tightness then .Y /  ! by
Problem 433. It is an easy exercise that .Cp .X //  .Y /  !, so i w.Cp .X // D
! (see TFS-173), i.e., Cp .X / condenses onto a second countable space.
V.460. Assuming MAC:CH prove that if K is a compact space such that Cp .K/
is Lindelf and condensable onto a  -compact space then K is metrizable.
Solution. Apply Problem 458 to see that K is separable; it follows from CFS-080
that K is !-monolithic, so w.K/ D nw.K/  ! by Fact 4 of S.307.
V.461. Assume that Cp .X / is a Lindelf -space and there exists a condensation
of Cp .X / onto a  -compact space Y . Prove that nw.X /  ! and nw.Y /  !.
Solution. Denote by X the Hewitt realcompactification of the space X . Then
Z D X is a Lindelf -space by CFS-206; since Cp .Z/ is also a Lindelf
-space (see CFS-234), we conclude that t .Cp .Z// D l.Cp .Z// D !. The space
Cp .Z/ condenses onto Cp .X / and hence Cp .Z/ is condensable onto Y ; this shows
that the space Z is separable (see Problem 458). It follows from stability of Cp .Z/
(see SFFS-266) that Z is monolithic and hence nw.Z/  !. Since X  Z, we have
nw.X /  nw.Z/ D !. Finally observe that nw.Y /  nw.Cp .X // D nw.X /  !.
V.462. Prove that, for any compact space X such that w.X /  !1 , the space
Cp .X / is hereditarily metalindelf.
Solution. Recall that a space Z is called metalindelf if every open cover of Z
has a point-countable refinement. If A is a family of subsets of Z and Y  Z then
AjY D fA \ Y W A 2 Ag. If d is a metric on a set Z then .d / is the topology
generated by d . Given a continuous map ' W Z ! T let '  .f / D f ' for any
f 2 C.T /; then '  W C.T / ! C.Z/ is the dual map of the map '.

2 Solutions of problems 001500

527

Fact 1. A space Z is hereditarily metalindelf if and only if every open subspace


of Z is metalindelf.
Proof. Since necessity is evident, assume that every open subspace of the space
Z is metalindelf and fix an arbitrary set Y  Z. If U  .Y / is a cover of Y
then choose
a set OU 2 .Z/ such that OU \ Y D U for every U 2 U . The set
S
G D fOU W U 2 U g is open in Z and V D fOU W U 2 U g is an open cover
of G. By our assumption G is metalindelf, so there exists a point-countable open
refinement W of the family V. It is straightforward that the family W 0 D WjY is
a point-countable open refinement of U , so Y is metalindelf and hence Fact 1 is
proved.
Fact 2. Suppose that .Z; d / is a metric space and  is a topology on Z with  
.d /. Assume additionally that we have a family fF W < !1 g of closed subsets
of .Z; / with the following properties:
(a) S
< < !1 implies F  F ;
(b)
fF W < !1 g D Z;
(c) for any < !1 the set F is second countable if considered with the topology
induced from .Z; d /.
S
(d) if < !1 is a limit ordinal then the set fF W < g is dense in F in the
topology induced from .Z; d /.
Then the space .Z; / is hereditarily metalindelf.
Proof. Take any set U 2  and let F0 D F \ U for any < !1 . If we consider the
metric d 0 D d j.U  U / and the topology 0 D jU then the space .U; d 0 / together
with the family fF0 W < !1 g trivially satisfies the conditions (a)(c).
Now if < !1 is a limit ordinal then take any nonempty open subset W of the
space F0 . The set F0 is open in F , so W is also open in F and hence it follows
from the property (d) that W \ F ; for some < . SinceSW  U , we have
W \ F0 D W \ F \ U D W \ F ; and hence the set fF0 W < g is
dense in F0 , i.e., the space U also has the property (d). Thus the properties (a)(d)
are open-hereditary, so we can apply Fact 1 to see that it suffices to establish that
.Z; / is metalindelf.
Let B.x; r/ D fy 2 Z W d.y; x/ < rg be the r-ball centered at x for any x 2 Z.
Take an arbitrary open cover O of the space .Z; / and let rx D supfr W there exists
O 2 O such that B.x; r/  Og; every element of O is open in .Z; d /, so rx > 0
for any x 2 Z. For each x 2 Z fix a set Ox 2 O with B.x; 34 rx /  Ox .
Let F1 D ; and fix, for each 2 !1 [ f1g a countable dense subset C of
the set FC1 nF with the topology induced from .Z; d /. Consider the family U D
fOz nF W z 2 C g for each 2 !1 [f1g. It is clear that every
S element of U is open
in .Z; / and contained in some element of O; let U D fU W 2 !1 [ f1gg.
Given a point x 2 S
Z there is < !1 such thatSx 2 F , so it follows from the
property (a) that x U for any  . Since fU W < g is countable, we
proved that every x 2 Z belongs to at most countably many elements of U , i.e., the
family U is point-countable.

528

2 Solutions of problems 001500

To see that U covers Z fix any point x 2 Z and denote by x the least ordinal
 < !1 such that x 2 F . It turns out that
(1) there exists < x such that .FC1 nF / \ B.x; 14 rx / ;.
Indeed, if x D C1 for some 2 !1 [f1g then x 2 .FC1 nF /\B.x; 14 rx /. If
x is a limit ordinal then it follows from the property (d) that we can find a minimal
2 !1 [ f1g such that FC1 \ B.x; 14 rx / ;; it easily follows from minimality
of that .FC1 nF / \ B.x; 14 rx / ;, so (1) is proved.
The set C being dense in FC1 nF there exists a point z 2 C \ B.x; 14 rx /.
It follows form the inclusions B.z; 12 rx /  B.x; 34 rx /  Ox that rz  12 rx .
Furthermore, B.z; 38 rx /  B.z; 34 rz /  Oz . We have d.z; x/ < 14 rx < 38 rx which
shows that x 2 B.z; 38 rx /  Oz . It follows from < x that x F , so
x 2 Oz nF 2 U and therefore the family U covers Z which shows that U is a
point-countable refinement of the cover O. Thus .Z; / is metalindelf; we already
saw that this implies that every subspace of Z is metalindelf and hence Fact 2 is
proved.
Returning to our solution fix any compact space X with w.X /  !1 ; we can
consider that X  I!1 . For every countable ordinal > 0 let  W X ! I be
the restriction to X of the natural projection; if X D  .X / then we can consider
that  W X ! X . If 0 < < < !1 then we will also need the projection
p W X ! X .
It is immediate that the set F D  .C.X // is an algebra in C.X / for every
> 0; let F0 D fug where u.x/ D 0 for all x 2 X andSobserve that F is closed in
Cp .X / for each < !1 . It follows from TFS-298 that fF W < !1 g D C.X /. If
0 < < < !1 then it is an easy consequence of the equality  D p  that
F  F ; clearly F0  F for every < !1 .
Given any 2 !1 nf0g the space Cu .X / is second countable by Fact 2 of T.357;
since the map  W Cu .X / ! Cu .X / is continuous by Fact 8 of V.318, the set F
is separable and hence second countable with the topology induced from Cu .X /.
Next assume that < !1 is a limit ordinal and let G D .p / .C.X // for every
< . The family of maps fp W 0 < < g separates the points of X and hence
S
the algebra G D fG W < g separates the points of X . Applying TFS-191 we
conclude that G is dense in X . Note that F D  .G / for every < and hence
S
fF W < g D  .G/ is dense in  .C.X // D F in the topology induced
from Cu .X /.
Let d.f; g/ D supfjf .x/  g.x/j W x 2 X g for any f; g 2 C.X /; then d is
a metric on C.X / which generates the topology of Cu .X /. If  is the topology of
Cp .X / then the space Z D C.X / with the metric d and the family fF W <
!1 g satisfies the conditions (a)(d) of Fact 2 which shows that the space .Z; / D
Cp .X / is hereditarily metalindelf and hence our solution is complete.
V.463. Given an uncountable discrete space X prove that the space Cp .X; D/ is
not metalindelf; therefore Cp .X / is not metalindelf either.

2 Solutions of problems 001500

529

Solution. Given a space Z and a set A  Z suppose that Ua 2 .a; Z/ for every
a 2 A; then the family U D fUa W a 2 Ag is called an open expansion of the set A.
The expansion U is called point-countable if the set fa 2 A W x 2 Ua g is countable
for any x 2 Z.
Fact 1. If Z is a metalindelf space then every closed discrete subspace of Z has a
point-countable open expansion.
Proof. Let D be a closed discrete subspace of Z. For every x 2 Z take a set
Ux 2 .x; Z/ such that jUx \Dj  1 and choose a point-countable open refinement
V of the open cover U D fUx W x 2 Zg of the space Z. For every d 2 D pick a set
Od 2 V with d 2 Od . Then O D fOd W d 2 Dg is an open expansion of D. If d
and d 0 are distinct points of D and Od 3 d 0 then pick U 2 U with Od  U and
observe that fd; d 0 g  U which is a contradiction with the choice of U . Therefore
Od \ D D fd g for all d 2 D and hence
(1) Od Od 0 for distinct d; d 0 2 D.
point-countable then there is an uncountable D 0  D such that
T Now if O is not
0
fOd W d 2 D g ;. It follows from (1) that the family fOd W d 2 D 0 g  V
is uncountable, so V is not point-countable which is a contradiction. Therefore O is
an open point-countable expansion of D and hence Fact 1 is proved.
Fact 2. Suppose that  is an infinite regular cardinal, and E is a set with jEj D .
Assume that A is a family of subsets of E such that jAj D  and jAj D  for any
A 2 A. If, additionally, jA \ A0 j <  for any distinct A; A0 2 A then there exists a
set B   such that jBj D  and jB \ Aj <  for any A 2 A.
Proof. Take a faithful enumeration fA W < g of the family A. Choose an
arbitrary point x0 2 A0 ; proceeding inductively, assume that <  and we have a
set fx W < g such that
(2) x x0 for any distinct ; 0 and x A whenever  < < .
The set A has cardinality  while jA \ A j <  for
S any < . By regularity
of  we can choose a point x 2 A n.fx W < g [ fA W < g/. It is clear
that (2) now holds for all  , so we can construct a set B D fx W < g such
that (2) is satisfied for all <  and, in particular, jBj D . Now, if <  then
it follows from (2) that B \ A  fx W  g and hence jB \ A j <  for all
< , i.e., Fact 2 is proved.
Fact 3. Suppose that  is an infinite regular cardinal and E is a set with jEj D .
Then there exists a family A of subsets of E such that jAj D  C while jAj D  for
any A 2 A and jA \ A0 j <  for any distinct A; A0 2 A.
Proof. Say that a family B of subsets of E is almost disjoint if jBj   while
jBj D  for any B 2 B and jB \ B 0 j <  for any distinct B; B 0 2 B. It is easy
to find a disjoint family A D fA W < g of subsets of E such that jA j D 

530

2 Solutions of problems 001500

for any < ; of course, A is almost disjoint. Proceeding inductively assume that
  <  C and we have a family fA W < g such that A D fA W   g is
almost disjoint for any  < .
It is an easy exercise that the family fA W < g is also almost disjoint. By
Fact 2 there exists a set A  E such that jA j D  and jA \ A j <  for any
< , i.e., the family A D fA W  g is almost disjoint. Therefore our
inductive procedure can be continued to construct a family A D fA W <  C g
such that A is almost disjoint for every <  C . If A; A0 are distinct elements of
A then there is <  C such that A; A0 2 A and hence jA \ A0 j <  because A
is almost disjoint. This shows that A is almost disjoint and hence Fact 3 is proved.
Fact 4. Let Z be a discrete space of cardinality !1 . Then there exists a closed
discrete subset D  Cp .Z; D/ such that jDj D !1 and D has no point-countable
open expansion.
Proof. Recall that for any A  Z the set A is clopen in Z by Fact 2 of T.371
(the bar denotes the closure in Z). Let fz W < !1 g be a faithful enumeration
of Z and denote by Z the set fz W < g for all < !1 . Given any f 2 DZ
there exists a unique function e.f / 2 Cp .Z; D/ such that e.f /jZ D f . For any
< !1 let f .z / D 1 for all < and f .z / D 0 whenever  , i.e.,
f is the characteristic function of the set Z ; let g D e.f /. We will show that
D D fg W < !1 g is the promised set.
Given any function f 2 Cp .Z; D/ and a finite subset K of the space Z let
f; K
D fg 2 Cp .Z; D/ W gjK D f jKg; it is clear that the family ff; K
W K is
a finite subset of Zg is a local base of the space Cp .Z; D/ at the point f .
If D C 1 < !1 and O D g ; fz ; z g
then g .z / D 0 and hence g O
for any  < . If  > then g .z / D 1 and hence g O. This shows that
O \ D D fg g.
Next, assume that  !1 is a limit ordinal. It is straightforward that the family
E D fZ nZ W < g is centered and consists of nonempty compact subsets of
T
E ;. Furthermore Z \ Z nZ D ; (see Fact 1 of
Z. Therefore E D
S.382) and hence Z \ E D ; for any < ; an immediate consequence is that
(3) for any limit ordinal  !1 the set E is nonempty and g .E / D f0g for each
< .
Now if < !1 is a limit ordinal then pick a point x 2 E and consider the set
O D g ; fx; z g
. If  > then g .z / D 1 g .z / and hence g O. If
 < then g .x/ D 0 by (3). However, x 2 Z and g .Z / D f1g; this shows that
g .x/ D 1 and hence g O for all  < . Thus O \ D D fg g and hence D is a
discrete subset of Cp .Z; D/.
To prove that D is closed in Cp .Z; D/ take any function f 2 Cp .Z; D/nD
and assume that f .z / < f .z / for some ordinals ; 2 !1 with < .
It is immediate that U D f; fz ; z g
is an open neighborhood of f such that
U \ D D ;. Therefore, to prove that f D, we can consider that
(4) f .z /  f .z / whenever < < !1 .

2 Solutions of problems 001500

531

The function f cannot be identically zero on Z because otherwise it would


coincide with the function g0 2 D. This shows that the set f 1 .1/ is nonempty
and hence f 1 .1/ \ Z ;. Consider the ordinal D supf W z 2 f 1 .1/g.
If < !1 and z 2 f 1 .1/ then it follows from (4) that f 1 .1/ \ Z D ZC1
and hence f D gC1 which is a contradiction. If < !1 and z f 1 .1/ then
must be a limit ordinal and it is easy to deduce from (4) that f jZ D f and hence
f D g which is again a contradiction.
Finally, assume that D !1 and hence we can apply (4) once more to see that
f .z / D 1 for all < !1 . Thus f .z/ D 1 for all z 2 Z. Apply (3) to take a point
x 2 E ; then g .x/ D 0 for all < !1 while f .x/ D 1 and hence U D f; fxg

is an open neighborhood of f such that U \ D D ;. We established that any


f 2 Cp .Z; D/nD has an open neighborhood which does not meet D and hence
D is closed in Cp .Z; D/.
Suppose that U 2 .g ; Cp .Z; D// for any < !1 and fU W < !1 g is a
point-countable extension of D. For any < !1 there exists a finite set K  Z
such that V D g ; K
 U ; it is evident that the family fV W < !1 g is also a
point-countable open
S extension of the set D.
The set Y D fZ W < !1 g is open in Z; let L D K \ Y for all < !1 .
Given any x 2 Y let .x/ D nnf W x 2 Z g and q.x/ D z.x/ . Since M D q.L /
is a finite subset of Z for every < !1 , we can apply SFFS-038 to find a finite
set R  Z and an uncountable A  !1 such that M \ M D R for any distinct
; 2 A. Let M0 D M nR and L0 D fx 2 L W q.x/ Rg for each 2 A.
Given any finite set P  Z consider the ordinals m.P / D nnf W z 2 P g and
j.P / D maxf W z 2 P g if P ;; if P D ; then let j.P / D m.P / D 0. The
family M D fM0 W 2 Ag is disjoint and hence, for any < !1 only countably
many elements of M intersect Z . Therefore we can pass to an appropriate
uncountable subset of A to be able to consider, without loss of generality, that for
the ordinal  D j.R/ we have m.M0 / >  and >  for any 2 A. Let
L00 D L nL0 for any 2 A; then q.L00 / D R  Z ; since g .Z / D f1g and
L00  Z  , we convince ourselves that
(5) g .L00 /  f1g for any ; 2 A.
If E D ZnY then it turns out that
(6) for any subsets P and Q of the set Z if jP \ Qj < !1 then P \ Q \ E D ;.
To prove (6) choose < !1 such that P \ Q  Z and assume that z 2
P \ Q \ E. Consider the family Cz D fU \ Z W U 2 .z; Z/g and apply Fact 2
of T.371 to see that P 2 Cz and Q 2 Cz . The family Cz is a filter, so P \ Q 2 Cz and
hence z 2 P \ Q  Z ; this contradiction shows that (6) is proved.
Furthermore, g .ZnZ / D f0g; since E \ Z D ;, we have the equality
(7) g .E/ D f0g for any < !1 .
Given a countable set P  Z, only countably many elements of M intersect P ,
so it is easy to construct by transfinite induction !1 -sequences f W < !1 g  A
and f W < !1 g  !1 with the following properties:

532

2 Solutions of problems 001500

(8) 0 D  and  C 1 < m.M0 / for any < !1 ;


(9) j.M0 /  C1 for every < !1 ;
(10)  > supf W < g for any < !1 .
Consider the set H D ZC1 nZ C1 ; it is straightforward from the definitions
that L0  H for every < !1 .
Apply Fact 3 to find a family of subsets of !1 such that jj D !2 while
jS j D !1 and nn.S / >  for any S 2 and S \ S 0 is countable whenever S and
S 0 are distinct
elements of . The family fH W < !1 g is disjoint, so if we let
S
QS D fH W 2 S g for any S 2 then the family Q D fQS W S 2 g will be
almost disjoint in the sense that QS \ QS 0 is countable for any distinct S; S 0 2 .
It follows
from (6) that QS \ QS 0 \ E D ; for any distinct S; S 0 2 . The set
S
K D fK \E W < !1 g has cardinality  !1 while the family fQS \E W S 2 g
is disjoint and has cardinality !2 . Therefore we can find a set S 2 such that
QS \ K D ;.

To define a function h W Z
S ! D let h.z / D 1 for all  and h.z / D 0

whenever > and z fH W 2 S g. Given any 2 S observe that H is
an open set which contains L0 , so we can choose a disjoint family fOd W d 2 L0 g

of open subsets of Z in such a way that d 2 Od  H for every d 2 L0 . The


S
family O D fOd \ Z W d 2 L0 g is disjoint; let h.z/ D 0 if z 2 H n. O/ and
h.z/ D g .d / for any z 2 Od \ Z and d 2 L0 .
This gives us a function h W Z ! D; let w D e.h/. Take any 2 S and
observe that it follows from d 2 Od \ Z that w.d / D g .d / for each d 2 L0 .

Furthermore, if d 2 L00 then d 2 Z  C1 and hence w.d / D 1, so it follows from


(5) that w.d / D g .d /. If d 2 K \ E then observe that h1 .1/  Z C1 [ QS
while Z  C1 \ E D ; and QS \ K D ;. Therefore h1 .1/ \ K D ; and hence
we can apply (7) to see that w.x/ D 0 D g .x/ for any x 2 K \ E.
We proved that wjK D g jK which shows that w 2 V  U for any
2 S and hence w witnesses that the family fU W < !1 g is not point-countable.
Thus D is a closed discrete subset of Cp .Z; D/ which has no point-countable open
expansion, i.e., Fact 4 is proved.

Returning to our solution assume that X is an uncountable discrete space and


hence there exists a surjective map ' W X ! Z of X onto a discrete space Z
such that jZj D !1 ; clearly, ' is continuous. There exists a continuous map  W
X ! Z such that jX D '; let   .f / D f  for any f 2 Cp .Z/. The
dual map   W Cp .Z/ ! Cp .X / is a closed embedding and it is straightforward
that   .Cp .Z; D//  Cp .X; D/. Apply Fact 4 to find a closed discrete subspace
D  Cp .Z; D/ which has no point-countable open expansion in Cp .Z; D/. It is
immediate that   .D/ is a closed discrete subset of the space Cp .X; D/ which has
no point-countable open expansion in Cp .X; D/ and hence the space Cp .X; D/
is not metalindelf by Fact 1. The property of being metalindelf is easily seen to
be closed-hereditary; since Cp .X; D/ is a closed subspace of Cp .X /, the space
Cp .X / is not metalindelf either, i.e., our solution is complete.

2 Solutions of problems 001500

533

V.464. Prove that


(i) every strong -space is a D-space. In particular, any Lindelf -space is a
D-space;
(ii) every space with a point-countable base is a D-space.
Solution. (i) Fix a strong -space X and take itsS
compact cover K for which there
exists a network F modulo K suchS
that F D i2! Fi and every Fi is discrete.
For each i 2 ! the family Gi D fFj W j  i g is locally finite; choose an
ordinal i for which there exists a faithful enumeration fGi W < i g of the
family Gi .
Take an arbitrary neighborhood assignment N on the space X . Given a set U 2
.X / say that an element F 2 F is U -special if there exists a set K 2 K such that
K  F; KnU ; and we can find a finite set E  KnU for which N.E/  F nU ;
such a set E will be called a U -kernel of the set F .
If the set G00 is ;-special then choose a ;-kernel E00 for G00 ; if G00 is not ;-special
then let E00 D ;. Proceeding inductively assume that < 0 and we have chosen a
set E0  G0 for any < in such a way that
S
S
(1) if < and G0 is N. < E0 /-special then E0 is an N. < E0 /-kernel of
G0 ; otherwise, E0 D ;.
S
S
If G0 is N. < E0 /-special then let E0 be an N. < E0 /-kernel of G0 ;
otherwise, let E0 D ;. It is trivial that the property (1) now holds for all  , so
we can construct a family E0 D fE0 W < 0 g such that (1) holds for all < 0 .
Proceeding inductively assume that n 2 ! and we have constructed a family
Ei D fEi W < i g for each i  n in such a way that
S j
(2) if i < n; < iC1 ; A.i; / D fE W j  i and < j g [ fEiC1 W < g
and the set GiC1 is N.A.i; //-special then EiC1 is an N.A.i; //-kernel
of GiC1 .
S j
Let A.n; 0/ D fE W j  n and < j g; if G0nC1 is N.A.n; 0//-special
then choose an N.A.n; 0//-kernel E0nC1 of the set G0nC1 ; otherwise, let E0nC1 D ;.
Assume that < nC1 and we have a family fEnC1 W < g such that
S
(3) for any ordinal < , if A.n; / D A.n; 0/ [ < EnC1 and the set GnC1 is
N.A.n; //-special then EnC1 is an N.A.n; //-kernel of GnC1 .
S
Let A.n; / D A.n; 0/ [ < EnC1 ; if GnC1 is N.A.n; //-special then take
any N.A.n; //-kernel EnC1 for the set GnC1 ; otherwise let EnC1 D ;. This shows
that our construction can be continued to obtain a family fEnC1 W < nC1 g such
that the condition (3) is satisfied for every < nC1 . Letting EnC1 D fEnC1 W <
nC1 g we obtain a family EnC1 for which the property (2) holds for i D n and all
< nC1 and hence we can construct the family Ei D fEi W < i g for each
i 2 ! in such a way that the condition (2) is satisfied for all n 2 ! and < nC1 .

534

2 Solutions of problems 001500

Observe first that it follows from (1) and (2) that Ei  Gi for any < i , so
every family Ei is locallyS
finite. Any locally finite union of finite sets is closed and
D
Ei is closed and discrete in X for every i 2 !.
discrete, so the
set
D
i
S
Let D D i2! Di and assume that N.D/ X ; then there exists a set K 2 K
such that K 0 D KnN.D/ ;. By compactness of K 0 we can find a finite set Q 
is covered by the
K 0 such that K 0  N.Q/. The compact space K 00 D KnN.Q/ S
00
family fN.Di / W i 2 !g, so there exists
n
2
!
such
that
K

N.
in Di /. Take a
S
set F 2 F for which K  F  N. in Di / [ N.Q/; there exists m > n such that
F D GmC1 for some < mC1 . It is easy to see that the sets K and Q witness that
GmC1 is N.A.m; //-special and hence GmC1 nN.A.m; //  N.EmC1 /. As an
immediate consequence, F nN.D/  GmC1 nN.A.m; //  N.EmC1 /  N.D/;
this contradiction proves that N.D/ D X .
Finally take any point x 2 X ; since N.D/ D X , we can choose a number n 2 !
such that x 2 N.Dn /. The property
(2) implies that N.Dn / is an open neighborhood
S
of xSwhich does not meet i>n Di . Since every Di is closed and discrete, the
set in Di is also closed and S
discrete, so there exists a set V 2 .x; X / which
contains at most one element of in Di . Thus the set V \ N.Dn / contains at most
one element of D. The point x 2 X was chosen arbitrarily, so we proved that every
x 2 X has a neighborhood which contains at most one element of D, i.e., D is
closed an discrete. Since also N.D/ D X , we established that X is a D-space.
(ii) Take a space X with a point-countable base B; this means that the family Bx D
fB 2 B W x 2 Bg must be countable for any x 2 X . It is easy to find infinite
disjoint subsets fAn W n 2 !g
S of the set ! such that f0; : : : ; n C 1g  A0 [
: : :[An for every n 2 ! and n2! An D !. Given a neighborhood assignment
N on the space X observe first that we can assume, without loss of generality,
that N.x/ 2 B for any x 2 X . Indeed, if we proved (ii) for all neighborhood
assignments N such that fN.x/ W x 2 X g  B, assume that M is an arbitrary
neighborhood assignment on X . For each x 2 X choose a set Bx 2 B such
that x 2 Bx  M.x/ and let N.x/ D Bx . For the assignment N , there exists a
closed discrete set D  X with N.D/ D X ; since M.D/  N.D/ D X , we
have M.D/ D X , i.e., D witnesses the fact that X is a D-space.
We will construct by transfinite induction countable sets D whose union will
be a closed discrete set D such that N.D/ D X . Let D0 D
S ; and assume that, for
some ordinal we have the family fD W < g. If N. fD S
W < g/ D X ,
then our construction stops. If not, pick a point x0 2 Q D X nN. fD W < g/
and let H0 D fx0 g. Choose an enumeration fUn W n 2 A0 g of the family Bx0 . If we
have a finite set Hn D fx0 ; : : : ; xn g  Q and an enumeration fUn W n 2 Ai g of the
family Bxi for any i  n, observe that there is a unique k  n such that n C 1 2 Ak
and consider the following condition
E.; n/: the set R.; n/ D fx 2 QnN.Hn / W N.x/ 2 fUm W m 2 Ak gg is
nonempty.
If E.; n/ holds, then choose the minimal m 2 Ak such that N.x/ D Um for some
x 2 QnN.Hn / and take xnC1 to be any point in QnN.Hn / such that N.xnC1 / D
Um ; let HnC1 D Hn [ fxnC1 g. If R.; n/ D ;, then
S let xnC1 D x0 and HnC1 D Hn .
After we have the sets fHn W n 2 !g let D D n2! Hn .

2 Solutions of problems 001500

535

Note that the sets D are disjoint and D ; for any ordinal > 0
C
if D is defined. Therefore, there exists
S an ordinal  < jX j at which our
inductive construction stops; let D D
fD W < g. By definition of our
inductive procedure we must have N.D/ D X , so the only thing left is to prove
that D is a closed discrete subset of X .
Fix any x 2 X and let be the minimal ordinal such that x 2 N.D /. If > ,
thenSD  X nN.D /, so N.D / is a neighborhood of x that does not meet the
set fD W < < g. There exists n 2 ! such that x 2 N.Hn / where
fHn W n 2 !g is the family used for the construction of D . The condition E.; n/
shows that W D N.Hn / is a neighborhood of x that does not meet D nHn and
hence W \ D is finite.
Finally assume that N.x/ \ D ; for some ordinal < and pick a point
y 2 D \ N.x/. Let fGn W n 2 !g be the family used for the construction of D .
Then y 2 Gn for some n 2 ! and there is k 2 ! such that By D fUn W n 2 Ak g;
observe that N.x/ 2 By and therefore N.x/ D Um for some m 2 !. Besides,
the point x witnesses the fact that E.; n/ holds for any n 2 Ak . Therefore at the
inductive step n a point xn 2 D will be chosen in such a way that N.xn / D Up for
some p  m. If l > n and l 2 Ak , then xl N.xn / D Up and hence N.xl / Up .
This shows that there will be infinitely many steps of the induction procedure at
which distinct sets are chosen from the finite family U D fUn W n 2 Ak ; n  mg;
this contradiction shows that N.x/ \ D D ; for any < . Consequently, the set
N.x/ \ N.D / \ W is a neighborhood of x that meets finitely many elements of D.
Thus D is a closed discrete set that witnesses the D-property of X .
V.465. Prove that,
(i) for any infinite cardinal , if a space is monotonically -monolithic then it is
-monolithic;
(ii) for any infinite cardinal , if X is a monotonically -monolithic space and
Y  X then Y is also monotonically -monolithic;
(iii) if  is an infinite cardinal, then any countable product of monotonically
-monolithic spaces is monotonically -monolithic;
(iv) for any infinite cardinal , every closed continuous image of a monotonically
-monolithic space is monotonically -monolithic;
(v) a space X is monotonically monolithic if and only if X is monotonically
-monolithic for any infinite cardinal ;
(vi) for any infinite cardinal , a space X is monotonically -monolithic if and
only if, to any finite set F  X we can assign a countable family O.F / 
exp.X
/ in such a way that for any set A  X with jAj  , the family
S
fO.F / W F 2 A
<! g is an external network at all points of A;
(vii) a space X is monotonically monolithic if and only if, to any finite set F  X
we can assign a countable
family O.F /  exp.X / in such a way that for any
S
A  X , the family fO.F / W F 2 A
<! g is an external network at all points
of A;

536

2 Solutions of problems 001500

(viii) if X is a space and the set X 0 of non-isolated points of X has a countable


network then X is monotonically monolithic. In particular, if a space X
is cosmic or has a unique non-isolated point then X is monotonically
monolithic;
(ix) given an infinite cardinal , if a space X is monotonically -monolithic and
t .X /  , then X is monotonically monolithic;
(x) for any infinite cardinal , every  -product of monotonically -monolithic
spaces is monotonically -monolithic. In particular, any  -product of monotonically monolithic spaces is monotonically monolithic.
Solution. (i) Suppose that a space X is monotonically -monolithic and take the
respective operator O on X . If A  X and jAj   then N D fP \ A W
P 2 O.A/g is a network in A and jN j  jO.A/j  maxfjAj; !g  , so X is
-monolithic.
(ii) Suppose that X is monotonically -monolithic and take the respective operator
O on the space X . If Y  X and A  Y then consider the family OY .A/ D
fP \Y W P 2 O.A/g. It is straightforward that OY is an operator that witnesses
monotone -monolithity of Y .
(iii) Let Xi be a monotonically
-monolithic space for any i 2 ! and consider
Q
the space X D i2! Xi . Fix an operator Oi which witnesses monotone monolithity of Xi and consider the projection i W X ! Xi for each i 2 !.
Given a collection A D fAi W i 2 Q
!g of familiesQsuch that Ai  exp.Xi /
for all i 2 !, the family W.A/ D f. i2Q Ai /  . i2!nQ Xi / W Q is a finite
subset of ! and Ai 2 Ai for each i 2 Qg consists of subsets of X .
Given any set A  X with jAj   let Ai D i .A/ and Ai D Oi .Ai / for
every i 2 !. If A D fAi W i 2 !g then it is easy to check that the family
O.A/ D W.A/ is an external network for the set A in X . We omit a simple
verification that the operator O witnesses monotone -monolithity of X .
(iv) Assume that X is monotonically -monolithic and f W X ! Y is a closed
map. Fix a point xy 2 f 1 .y/ for any y 2 Y and take an operator O which
witnesses monotonic -monolithity of X . Given a set B  Y with jBj  
let A D fxy W y 2 Bg and N .B/ D ff .P / W P 2 O.A/g. To see that N is
an external network of B in Y take any y 2 B and U 2 .y; Y /. The map f
being closed we have f .A/ D B, so there is a point x 2 A with f .x/ D y. If
P 2 O.A/ and x 2 P  f 1 .U / then f .P / 2 N .B/ and y 2 f .P /  U . It
is easy to see that N satisfies all other conditions of the definition of monotone
-monolithity.
(v) If X is monotonically monolithic and an operator O witnesses this, then O
witnesses that X is monotonically -monolithic for any infinite cardinal ,
i.e., we proved necessity.
Now if X is monotonically -monolithic for any infinite cardinal  then take
an operator O that witnesses this for  D maxfjX j; !g. Since jAj   for
any A  X , the operator O satisfies all conditions of the definition of (strong)
monotone monolithity and hence we settled sufficiency.
(vi) We will need the following fact.

2 Solutions of problems 001500

537

Fact 1. If  is an infinite cardinal and Z is a monotonically -monolithic space with


a respective
S operator O, then for any set A  Z with jAj  , we have the equality
O.A/ D fO.F / W F 2 A
<! g.
Proof. If A is finite, then there is nothing to prove, so assume, proceeding by
induction, that    is a cardinal and we proved our equality for any set B  Z
with jBj < . If A  ZSand jAj D , then it is easy to find an increasing family
fB W < g such that fB W < g D
any < . By the
SA and jB j <  for<!
induction hypothesis, we have O.B / D fO.F / W F 2
B

S g for any < .


Since the operator O is -monotone,
we
have
O.A/
D
so it
S
S S fO.B / W < g,<!
follows from
the
equalities
fO.B
/
W

<
g
D
f
fO.F
/
W
F
2
B

gW

S
S
< g D fO.F / W F 2 A
<! g that O.A/ D fO.F / W F 2 A
<! g, i.e., Fact 1
is proved.
Returning to our solution assume that in a space
S X we have an operator O on all
finite subsets of X as in (vi). Letting O0 .A/ D fO.F / W F 2 A
<! g for every
set A  X with jAj  , it is immediate that we obtain a monotone -monolithity
operator O0 on X . This proves sufficiency.
Now, if O is a monotone -monolithity operator on X , then let O0 .F / D O.F /
for any finite subset of X . If A  X and jAj  , then
[
[
fO0 .F / W F 2 A
<! g D
fO.F / W F 2 A
<! g D O.A/
S
by Fact 1, so the family fO0 .F / W F 2 A
<! g is an external network at all points
of A, i.e., we proved necessity.
(vii) Assume that in a space X weS
have an operator O on all finite subsets of X
as in (vii). Letting O0 .A/ D fO.F / W F 2 A
<! g for any A  X , it is
immediate that we obtain a monotone monolithity operator O0 on X . This
proves sufficiency.
Now, if O is a monotone monolithity operator on X , then let O0 .F / D O.F /
for any finite subset of X . If A  X , then
[
[
fO0 .F / W F 2 A
<! g D
fO.F / W F 2 A
<! g D O.A/
S
by Fact 1, so the family fO0 .F / W F 2 A
<! g is an external network at all
points of A, i.e., we proved necessity.
(viii) Let N be a countable network in the space X 0 . Given an arbitrary set A  X
let O.A/ D ffag W a 2 AnX 0 g [ N . It is trivial that the operator O witnesses
monotonic monolithity of X .
(ix) Let O0 be an operator of monotonic -monolithity on X . Consider the family
O.F / D O0 .F / for any finite set F  X . Given an arbitrary set A  X ,
take any point x 2 A and U 2 .x; X /. It follows from t .X /   that we
can find a set B  A such that jBj   and x 2 B. The operator O0 is
applicable to B, so there exists N 2 O0 .B/ such that x 2 N  U . By Fact 1,
there is a finite set F  B such that N 2 O0 .F / D O.F /. Therefore the

538

2 Solutions of problems 001500

S
family fO.F / W F 2 A
<! g contains an external network at every point of
A which, together with (vi), implies that X is monotonically monolithic.
(x) Suppose that Xt is monotonically -monolithic
Qand fix the respective operator
Ot on the space Xt for any t 2 T . Let X D t2T Xt and fix a point a 2 X .
We must prove that the space  .X; a/ D fx 2 X W jft 2 T W x.t / a.t /gj <
!g is Q
monotonically -monolithic. For any S  T we will need the point
aS 2 t2S Xt such that aS .t / D a.t / for any t 2 S . Let pt W X ! Xt be the
projection for each t 2 T .
For every point x 2  .X; a/ let supp.x/ D ft 2 T W x.t / a.t /g. Suppose
that we have a finite set K  T and a family At of
Qsubsets of the space Xt for any
t 2 K. Let A D fAt W t 2 Kg and H.K; A/ D f t2K Pt  faT nK g W Pt 2 At for
every t 2 Kg. Now we are ready to construct a monotonic -monolithity operator
on  .X; a/.
To do so, fix a set A   .X; a/Swith jAj   and let D maxfjAj; !g. It
is evident that the family E.A/ D fsupp.x/ W x 2 Ag has cardinality at most
; for every finite K  E.A/ let AK .A/ D fOt .pt .A// W t 2 Kg. Then the
cardinalitySof the family H.K; AK .A// does not exceed . Therefore the family
O.A/ D fH.K; AK .A// W K is a finite subset of E.A/g also has cardinality at
most .
To check that the operator O is as promised, observe that if A  B   .X; a/
then pt .A/  pt .B/ for any t 2 T . Therefore H.K; AK .A//  H.K; AK .B//
for every finite K  E.A/ which in turn implies that O.A/  O.B/. To see that
the increasing union property holds for O observe that O.A/ is defined by using
finitary operations with families which have this property. To finally see that O.A/
is an outer network at all points of A take any x 2 A and any U 2 .x;  .X; a//.
There exists a finite set K  T and a family fWt W t 2 Kg such that supp.x/  K
and Wt 2 .x.t /; Xt / for any t 2 K while W D fy 2  .X; a/ W y.t / 2 Wt for each
t 2 Kg  U .
Since x.t / 2 pt .A/, there
Q exists Pt 2 O.pt .A// such that x.t / 2 Pt  Wt for
every t 2 K. The set P D t2K Pt  faT nK g belongs to O.A/ and x 2 P  U , so
O is, indeed, a monotone -monolithity operator on  .X; a/.
V.466. Prove that,
(i) a space X is strongly monotonically monolithic if and only if X is strongly
monotonically !-monolithic;
(ii) every space with a point-countable base must be strongly monotonically
monolithic;
(iii) if a space X is strongly monotonically monolithic, then it must be strongly
monolithic and monotonically monolithic;
(iv) every subspace of a strongly monotonically monolithic space is strongly
monotonically monolithic;
(v) any countable product of strongly monotonically monolithic spaces must be
strongly monotonically monolithic;

2 Solutions of problems 001500

539

(vi) if X is strongly monotonically monolithic and f W X ! Y is an open map


such that d.f 1 .y//  ! for any y 2 Y then Y is strongly monotonically
monolithic;
(vii) every countably compact strongly monotonically monolithic space is compact
and metrizable.
Solution. (i) We must only prove sufficiency, so assume that a space X is
strongly monotonically !-monolithic and fix a respective operator O on X .
Take any set A  X , a point x 2 A and U 2 .x; X /. Note first that O.fxg/
contains a countable local base at the point x, so there is a countable set
B  A such thatSx 2 B. Take an increasing family fBn W n 2 !g of finite sets
such that B D fBn W n 2 !g. The operator O is applicable to B, so there
exists N 2 O.B/ for which x 2 N  U . By !-monotonicity of O, we can
find n 2 ! such S
that N 2 O.Bn /. This proves that for any set A  X , the
family O0 .A/ D fO.F / W F 2 A
<! g contains a local base at every point
of A. It is immediate that the operator O0 is monotone, so it witnesses that X
strongly monotonically monolithic.
(ii) Assume that a space X has a point-countable base B. For any A  X let
O.A/ D fB 2 B W B \ A ;g. If x 2 A and U 2 .x; X / then there is
B 2 B such that x 2 B  U ; we also have B \ A ;, so B 2 O.A/ and
hence the family O.A/ is an external base of A in X . It is evident that O.A/
has all other properties required by the definition of the operator of strong
monotone -monolithity.
(iii) Since any external base of a set is also an external network of the same set,
any operator of strong monotone monolithity is also an operator of monotone
monolithity. If X is strongly monotonically monolithic and A  X , then there
is an external base B for the set A such that jBj  maxfjAj; !g. It is clear that
the family B 0 D fB \ A W B 2 Bg is a base in A such that jB 0 j  maxfjAj; !g;
this proves that X is strongly monolithic.
(iv) Suppose that X is strongly monotonically monolithic and take the respective
operator O on the space X . If Y  X and A  Y then consider the family
OY .A/ D fP \ Y W P 2 O.A/g. It is straightforward that OY is an operator
that witnesses strong monotone monolithity of Y .
(v) Let Xi be a strongly monotonically
-monolithic space for any i 2 ! and
Q
consider the space X D i2! Xi . Fix an operator Oi which witnesses strong
monotone -monolithity of Xi and consider the projection i W X ! Xi
for each i 2 !. Given a collection A D fAi W i 2 !g of families such
that
i / for all i 2 ! it is easy to see that the family W.A/ D
Q Ai  .XQ
f. i2Q Ai /  . i2!nQ Xi / W Q is a finite subset of ! and Ai 2 Ai for each
i 2 Qg consists of open subsets of X .
Given any set A  X with jAj   let Ai D i .A/ and Ai D Oi .Ai / for
every i 2 !. If A D fAi W i 2 !g then it is easy to check that the family
O.A/ D W.A/ is an external base for the set A in X . We omit a simple
verification that the operator O witnesses strong monotone -monolithity
of X .

540

2 Solutions of problems 001500

(vi) Take a countable dense set Ay in the space f 1 .y/ for any y 2 Y and let
O be the
S strong monotone monolithity operator in X . If B  Y then let
QB D fAy W y 2 Bg and N .B/ D ff .E/ W E 2 O.QB /g.
To see that N .B/ is an external base for B take a point y 2 B and a set
U 2 .y; Y /; observe that f 1 .B/ \ f 1 .y/ ; because the map f is
open. Since QB is dense in f 1 .B/, there exists a point x 2 f 1 .y/ with
x 2 QB . Take a set V 2 O.QB / such that x 2 V  f 1 .U /; then W D
f .V / 2 N .B/ and y 2 W  U , i.e., N .B/ is an external base for B. An easy
proof that N satisfies all other conditions of the definition of strong monotone
monolithity can be left to the reader.
(vii) Given a strongly monotonically !-monolithic countably compact space X
take an operator O which witnesses strong monotone !-monolithity of X and
fix a point a 2 X ; let A0 D fag. Proceeding inductively assume that n 2 !
and we have countable subsets A0 ; : : : ; An of the space X with the following
properties:
(1) Ai  AiC1 for any i < n;
S
(2) if i < n and U is a finite subfamily
of O.Ai / such that U X then
S
there is a point x 2 AiC1 n. U /.
The set An being countable the family V of all finite unions of the elements
of O.An / is also countable. Let V 0 D fV 2 V W V X g and choose a point
xV 2 X nV for any V 2 V 0 . The set AnC1 D An [ fxV W V 2 V 0 g is countable and
it is immediate that the properties (1) and (2) still hold if we replace n with n C 1.
Therefore our inductive procedure can be continued to construct
S a sequence fAn W
n 2 !g such that (1) and (2) are true for all n 2 !. If A D n2! An is not dense
in X then pick a set W 2  .X / such that W \ A D ;. The family O.A/ being an
external base for A we can choose a set Vx 2 O.A/ such that x 2 Vx  X nW for
any x 2 A.
The space A is countably compact and second countable because O.A/ is a
countable external base of A. Thus
S the set A is compact and hence there exists a
finite Q  A such that A  fVx W x 2 Qg. It follows from the definition of
strong monotone !-monolithity that V D fVx S
W x 2 Qg  O.An / for some number
S
n 2 !. The property (2) implies that AnC1 n. V/ ; while AnC1  A  V;
this contradiction shows that A is dense in X , so X is compact and metrizable
because O.A/ is a countable base in X .
V.467. For any infinite cardinal  prove that
(a) a space X is monotonically -stable if and only if Cp .X / is monotonically
-monolithic.
(b) a space X is monotonically -monolithic if and only if Cp .X / is monotonically
-stable.
Solution. Fix a countable base B in the space R; given a family A of subsets of a
space Z let W.A/ D fA1 ; : : : ; An I B1 ; : : : ; Bn
W n 2 N; Ai 2 A and Bi 2 B for

2 Solutions of problems 001500

541

every i  ng; here, as usual, A1 ; : : : ; An I B1 ; : : : ; Bn


D ff 2 Cp .Z/ W f .Ai / 
Bi for any i  ng. If ' W Z ! Z 0 is an onto map, then its dual map '  W Cp .Z 0 / !
Cp .Z/ is defined by the formula '.f / D f ' for any f 2 Cp .Z 0 /.
Fact 1. If Z is a space, then the assignment A ! W.A/ is -monotone on exp.Z/
for any infinite cardinal .
Proof. The family B being countable, we have jW.A/j  maxfjAj; !g for any
set A  Z. It is evident that A  A0 implies W.A/  W.A0 /; if   and
fA W < g is an increasing family
of subsets of Z, then take any set Q D
S
A1 ; : : : ; An I B1 ; : : : ; Bn
2 A D fA W < g. There exists an ordinal
<
S
such that fA1 ; : : : ; An g  A and hence Q 2 W.A /. Therefore W. < A / D
W.A/, i.e., Fact 1 is proved.
Fact 2. Suppose that Z is a space and ' W Z ! Z 0 is a continuous onto map. If
N is a network for the map ', then W.N / is an external network of '  .Cp .Z 0 // in
Cp .Z/.
Proof. Take a function f 2 '  .Cp .Z 0 // and a set W 2 .f; Cp .Z//. There exist
points x1 ; : : : ; xn 2 Z and sets B1 ; : : : ; Bn 2 B such that f 2 U D fg 2 Cp .Z/ W
g.xi / 2 Bi for any i  ng  W . Fix a function h 2 Cp .Z 0 / such that f D
h '. Let yi D '.xi /; it follows from f .xi / D h.yi / 2 Bi that yi 2 Gi D
h1 .Bi / for any i  n. The family N being a network for ', we can find sets
N1 ; : : : ; Nn 2 N such that xi 2 Ni and '.Ni /  Gi for each i  n. Then Q D
N1 ; : : : ; Nn I B1 ; : : : ; Bn
2 W.N / and f 2 Q  U  W , i.e., Fact 2 is proved.
Fact 3. If Z is a space and Y  Z, then let Y W Cp .Z/ ! Cp .Y / be the restriction
map. If N is an external network of the set Y in Z, then W.N / is a network for the
map Y .
Proof. Take any f 2 Cp .Z/ and U 2 .Y .f /; Cp .Y //. Choose y1 ; : : : ; yn 2 Y
and B1 ; : : : ; Bn 2 B such that Y .f / 2 V D fg 2 Cp .Y / W g.yi / 2 Bi for all
i  ng  U . There exists Ni 2 N such that yi 2 Ni  f 1 .Bi / for every i  n.
The set H D N1 ; : : : ; Nn I B1 ; : : : ; Bn
belongs to W.N / and f 2 H . Now, if
h 2 H , then we have the inclusions h.yi / 2 h.Ni /  Bi for each i  n; since
Y .h/.yi / D h.yi /, we conclude that Y .h/.yi / 2 Bi for every i  n which shows
that Y .h/ 2 V  U for any h 2 H , i.e., Y .H /  U and hence Fact 3 is proved.
Fact 4. If a space Z is monotonically -monolithic for some infinite cardinal ,
then Cp .Cp .Z// is also monotonically -monolithic.
Proof. Recall that for any space Y , if A  Y , then A W Cp .Y / ! Cp .A/ is the
restriction map and Cp .AjY / D ff jA W f 2 Cp .Y /g D A .Cp .Y //. Apply TFS300 to fix, for any f 2 Cp .Cp .Z// a countable set S.f /  Z and a continuous
function p.f / W Cp .S.f /jZ/ ! R such that p.f / S.f / D f . If B  Z and
B
S.f /  B, then let qS.f
/ W Cp .BjZ/ ! Cp .S.f /jZ/ be the restriction map.
B
It is clear that f D p.f / S.f / D p.f / qS.f
/ B which shows that f 2

B .Cp .Cp .BjZ///. This proves that

542

2 Solutions of problems 001500

(1) if f 2 Cp .Cp .Z// and S.f /  B  Z, then f 2 B .Cp .Cp .BjZ///.
Now assume that Z is monotonically -monolithic and fix the respective
operator O on the
S space Z. For any set A  Cp .Cp .Z// with jAj   consider
the set S.A/ D fS.f / W f 2 Ag. It is easy to check that
(2) the assignment A ! S.A/ is -monotone.
Now let N .A/ D W.W.O.S.A/// for any set A  Cp .Cp .Z// with jAj  .
Apply Fact 1 and the property (2) to see that N is the composition of four monotone maps, so it is -monotone and hence we only have to prove that N .A/ is
an external network for A.
Let B D S.A/ and Y D Cp .BjZ/. The map B W Cp .Z/ ! Y is open by TFS152, so the set E D B .Cp .Y // is closed in Cp .Cp .Z// by TFS-163. The property
(1) shows that A  E and hence A  E. By the choice of the operator O, the family
O.S.A// is an external network for B, so W.O.S.A/// is a network for the map B
by Fact 3. Now, apply Fact 2 to see that N .A/ D W.W.O.S.A/// is an external
network for the set E; since A  E, the family N .A/ is also an external network
for A, i.e., the operator N witnesses that Cp .Cp .Z// is monotonically -monolithic
and hence Fact 4 is proved.
Returning to our solution assume that X is monotonically -stable and fix the
respective operator O. Given a set A  Cp .X / with jAj  , denote by ' the
reflection map with respect to A, i.e., ' W X ! Cp .A/ and '.x/.f / D f .x/ for
any x 2 X and f 2 A. The family O.A/ is a network for the map '; let N .A/ D
W.O.A//. The assignment A ! N .A/ is -monotone being the composition of
two -monotone mappings. By Fact 2, the family N .A/ is an external network in
Cp .X / for the set '  .Cp .Y // where Y D '.X /. Observe that A  '  .Cp .Y // by
Fact 5 of U.086 and therefore N .A/ is an external network in Cp .X / for the set A.
Thus, the operator N witnesses that Cp .X / is monotonically -monolithic, i.e., we
proved necessity in (a).
Now assume that Cp .X / is monotonically -monolithic and let N be the
respective operator on Cp .X /. Let e W X ! Cp .Cp .X // be the natural embedding
defined by e.x/.f / D f .x/ for any x 2 X and f 2 Cp .X / (see TFS167). Take a set A  Cp .X / with jAj   and apply Fact 3 to the restriction
map  W Cp .Cp .X // ! Cp .A/ to note that W.N .A// is a network for  in
Cp .Cp .X //. Letting O.A/ D fe 1 .H \ e.X // W H 2 W.N .A//g we obtain a
-monotone operator, so it suffices to show that O witnesses monotone -stability
of X , i.e., O.A/ is a network for the reflexion map ' W X ! Cp .A/ defined by
'.x/.f / D f .x/ for any x 2 X and f 2 A.
To do this, observe first that we have the equality ' D  e; take any point
x 2 X and a set U 2 .'.x/; Cp .A//. Since W.N .A// is a network for the map 
and '.x/ D .e.x// 2 U , we can find a set Q 2 W.N .A// such that e.x/ 2 Q and
.Q/  U . Then the set P D e 1 .Q \ e.X // belongs to O.A/ and x 2 P . Since
also '.P / D .e.P //  .Q/  U , the set P witnesses the fact that O.A/ is a
network for ' and hence the space X is monotonically -stable, i.e., we completed
the proof of (a).

2 Solutions of problems 001500

543

(b) If X is monotonically -monolithic, then so is Cp .Cp .X // by Fact 4 and


hence Cp .X / is monotonically -stable by (a). If Cp .X / is monotonically stable, then Cp .Cp .X // is monotonically -monolithic by (a) and therefore X
is monotonically -monolithic being embeddable en Cp .Cp .X //. This settles
(b) and makes our solution complete.
V.468. Prove that if X is a Lindelf -space then the space Cp .X / must be
monotonically monolithic. In particular, if Cp .Y / is a Lindelf -space then Y
is monotonically monolithic.
Solution. For any E1 ; : : : ; En  X and U1 ; : : : ; Un 2 .R/ we will need the set
E1 ; : : : ; En ; U1 ; : : : Un
D ff 2 Cp .X / W f .Ei /  Ui for every i  ng. Fix a
countable base B in R and take a compact cover K of the space X such that there
exists a countable network N modulo K. For every family E of subsets of X let
C.E/ D fN nE W E 2 E; N 2 N g and consider the family F.E/ of all finite unions
of the elements of E; besides, let W.E/ D fE1 ; : : : ; En ; U1 ; : : : ; Un
W Ei 2 E and
Ui 2 B for all i  ng.
Given a set A  Cp .X / let E D ff 1 .B/ W f 2 A and B 2 Bg; if H D C.F.E//
and O.A/ D W.H/ then O.A/ is an external network for A. To see it take any
function f 2 A; we claim that
(1) for any x 2 X and B 2 B such that f .x/ 2 B there exists P 2 H for which
f 2 P; B
 fxg; B
.
To prove (1), take a set K 2 K with x 2 K. For any y 2 X nf 1 .B/ pick a set
By 2 B such that f .y/ 2 By and f .x/ B y ; there exists a function ay 2 A such
that ay .x/ 2 BnB y and ay .y/ 2 By . The family A D fay1 .By / W y 2 X nf 1 .B/g
.B/ and hence we can choose a finite set Q  X nf 1 .B/
covers the set X nf 1S
such that K  G D fay1 .By / W y 2 Qg [ f 1 .B/. There exists a set N 2 N
such that K  N  G; observe that we have x ay1 .By / for any point y 2 Q, so
S
x 2 P D N n. fay1 .By / W y 2 Qg/  f 1 .B/. This shows that f .P /  B and
hence f 2 P; B
and it follows from x 2 P that P; B
 fxg; B
; since P 2 H,
the property (1) is proved.
Now, take any V 2 .f; Cp .X //; there exist x1 ; : : : ; xn 2 X and sets
B1 ; : : : ; Bn 2 B such that T
V 0 D fx1 g; : : : ; fxn g; B1 ; : : : ; Bn
 V . Observe that
we have the equality V 0 D ffxi g; Bi
W i  ng and apply the property (1) to find
sets P1 ; : : : ; Pn 2 H such that f 2 Pi ; Bi
 fxi g; Bi
for each i  n. Then the
set L D P1 ; : : : ; Pn ; B1 ; : : : ; Bn
belongs to O.A/ and f 2 L  V 0  V , i.e.,
O.A/ is, indeed an external network for A. It is straightforward that the operator O
satisfies all other conditions of the definition of monotonic monolithity, so Cp .X /
is monotonically monolithic.
Finally observe that if Cp .Y / is a Lindelf -space then Cp .Cp .Y // is monotonically monolithic; the space Y embeds in Cp .Cp .Y //, so it must be monotonically
monolithic by Problem 465.

544

2 Solutions of problems 001500

V.469. Prove that if X is a Lindelf -space then the space Cp .X / must be


monotonically !-monolithic. In particular, if .Cp .X // is a Lindelf -space
then Cp .X / is monotonically !-monolithic. Deduce from these facts that if X is
pseudocompact then Cp .X / is monotonically !-monolithic.
Solution. Let us first formulate the following easy fact for future references.
Fact 1. Given an infinite cardinal , suppose that X is a monotonically
-monolithic space and f W X ! Y is a continuous bijection such that f jA is
a homeomorphism for any set A  X with jAj  . Then Y is monotonically
-monolithic.
Proof. It is easy to see that f .A/ D f .A/ and f jA is a homeomorphism for
any set A  X with jAj  . Let O be an operator which witnesses monotone
-monolithity of X . For any set B  Y with
 jBj   let A.B/ D ff .P / W P 2
1
1
O.f .B//g; it follows from f f .B/ D B that A.B/ is an external network
for B. The map f being a bijection, the rest of conditions for A to be a monotone
-monolithity operator on Y are trivially satisfied, so Fact 1 is proved.
Returning to our solution note that the set X is dense in X and the restriction
map  W Cp .X / ! Cp .X / is a continuous bijection such that jA is a
homeomorphism for any A  Cp .X / with jAj  ! (see TFS-436). The space
Cp .X / is monotonically monolithic by Problem 468, so Fact 1 is applicable
to convince ourselves that Cp .X / is monotonically !-monolithic. Furthermore, if
.Cp .X // is a Lindelf -space then apply CFS-206 to see that X is Lindelf
and therefore Cp .X / is monotonically !-monolithic. Finally, if X is pseudocompact
then X D X (see TFS-417 and Fact 3 of S.309), so Cp .X / is monotonically
!-monolithic.
V.470. Let X be a monotonically -monolithic space with ext .X / D  . Prove
that
(i) if <  then l.X /  ;
(ii) if D  and t .X /   then l.X /  .
Solution. Assume that Z is a monotonically -monolithic space and fix an operator
O which witnesses that; let N be a neighborhood assignment on Z. For every
P  Z say that x 2 P is a central point of P if P  N.x/; denote by K.P /
the set of all central points of P . For an open set U  Z say that a set A  Z is
U -saturated if jAj   and K.P /  N.A/ [ U for any P 2 O.A/. If U D ; then
U -saturated sets will be called saturated.
Fact 1. Assume that Z is a monotonically -monolithic space with the respective
operator O; let N be a neighborhood assignment on Z and fix a set U 2 .Z/.
Suppose that   is an ordinal and we are given a -sequenceSfA W < g of
U -saturated sets such that < 0 implies A  A0 . Then A D fA W < g is
U -saturated.

2 Solutions of problems 001500

545

Proof. Take any P 2 O.A/; there exists < such that P 2 O.A /. The set A
being U -saturated, we have K.P /  N.A / [ U  N.A/ [ U which shows that
A is U -saturated, i.e., Fact 1 is proved.
Fact 2. Suppose that Z is a monotonically -monolithic space; fix the respective
operator O and let N be a neighborhood assignment on Z. Then for any A  Z
with jAj   and U 2 .Z/ such that N.A/  U there exists a closed discrete set
D  ZnU such that jDj  maxfjAj; !g and the set A [ D is U -saturated.
Proof. We will use induction on the cardinal  D jAj. To start off, assume that
jAj
S  ! and take a family fn W n 2 !g of infinite disjoint subsets of ! such that
fn W n 2 !g D ! and f0; : : : ; ng  0 [ : : : [ n for all n 2 !. Enumerate the
family O.A/ as fPn W n 2 0 g and let B0 D A. Proceeding inductively, assume that
k 2 ! and we have sets B0 ; : : : ; Bk with the following properties:
(1) Bi  BiC1 and jBiC1 nBi j  1 for any i < k;
(2) an enumeration fPn W n 2Sj g for the family O.Bj / is chosen for each j  k;
(3) if i < k; i 2 j and fK.Pn / W n 2 j gn.N.Bi / [ U / ; then, for
the number m D nnfn 2 j W K.Pn /n.N.Bi / [ U / ;g we have BiC1 D
Bi [ fd g for some d 2 K.Pm /n.N.Bi / [ U /.
There is
Sa unique j 2 ! such that k 2 j and hence j  k. If we have the
inclusion fK.Pn / W n 2 j g  N.Bk / [ U then let BkC1 D Bk ; if not, then
consider the number m D nnfn 2 j W K.Pn /n.N.Bk / [ U / ;g, choose a
point d 2 K.Pm /n.N.Bk / [ U / and let BkC1 D Bk [ fd g. Choose an enumeration
fPn W n 2 kC1 g of the family O.BkC1 / and observe that now the conditions (1)
(3) are satisfied if we replace k with k C 1. Therefore our inductive procedure can
be continued to construct a family fBi W i 2 !g such that the properties (1)(3) hold
for all k 2 !. S
Let B D i2! Bi and consider the set D D BnA. It follows from (1) that
jDj  !; the condition (3) shows that D  ZnU . To see that the set A [ D D B
is U -saturated take any P 2 O.B/; there exists j 2 ! such that P 2 O.Bj / and
hence P D Pn for some n 2 j . If K.P /n.N.B/ [ U / ; then K.Pn /n.N.Bi / [
U / ; for all i 2 j which is impossible because the condition (3) implies that
after at most n inductive steps a point d 2 K.Pn /n.N.Bi / [ U / has to be chosen for
some i 2 j and hence Pn  N.d /  N.B/; this contradiction shows that A [ D
is U -saturated.
Now fix any point x 2 Z; if x 2 U then U is a neighborhood of x with U \
D D ;. If x N.B/ [ U and x 2 D then there exists P 2 O.D/ such that
x 2 P  N.x/ and hence x 2 K.P /n.N.B/ [ U /. We have O.D/  O.B/, so
P 2 O.B/ which contradicts the fact that B is U -saturated. Thus x D for any
x N.B/ [ U .
Finally, if x 2 N.B/nU then let m be the minimal i for which x 2 N.Bi /. It
follows from (3) that N.Bi / is an open neighborhood of x which does not meet the
set DnBi ; this, together with (1) shows that N.Bi / \ D is finite. Thus every x 2 Z
has a neighborhood whose intersection with D is finite and hence D is closed and
discrete. This completes our proof for a countable A.

546

2 Solutions of problems 001500

Now assume that    is an uncountable cardinal and our lemma is proved


whenever A  Z; jAj <  and U 2 .N.A/; Z/. Take an arbitrary set A  Z
such that jAj D , let fa W < g be an enumeration of A and take any U 2
.N.A/; Z/; we will also need the set A D fa W < g for all < .
By the induction hypothesis there is a countable closed discrete set D!  ZnU
such that the set A! [ D! is U -saturated. Proceeding inductively assume that ! <
<  and we have a family fD W !  < g with the following properties:
S
(4) if U D U [ N. !< D / then D is a closed discrete subspace of ZnU
and jD j  jjSfor all 2 !; /;
(5) if B D A [ !< D then B [ D is U -saturated whenever !  < ;
S
Let U DSU [ N. !< D /; it follows from (4) that the cardinality of the set
B D A [ !< D does not exceed jj; besides, N.B /  U , so we can apply
our induction hypothesis to find a closed discrete subset D of the set ZnU such
that B [ D is U -saturated. The equality U [ N.B [ D / D U [ N.B [ D /
shows that B [ D is also U -saturated.
It is clear that the conditions (4) and (5) are satisfied for all 2 !;
, so our
induction procedure can be continued to construct a family fD W !  < g for
which the conditions
(4) and (5) are satisfied whenever ! < < .
S
If D D !< D then it follows from (4) that D  ZnU . Take any set
P 2 O.A [ D/. There exists <  such that P 2 O.B /, so the condition (5)
implies that K.P /  U [ N.B [ D /  U [ N.D/; this proves that A [ D is
U -saturated.
Take any x 2 Z; if x 2 U then U is a neighborhood of x which does not meet D.
If x U [ N.D/ and x 2 D then there exists P 2 O.D/  O.A [ D/ such that
x 2 P  N.x/ and hence x 2 K.P /n.U [ N.D// which is a contradiction with
the fact that A [ D is U -saturated. Therefore x D. If x 2 N.D/nU then let be
the minimal
S ordinal such that x 2 N.D /. If D  C 1 for some ordinal  then the
set E D !< D is contained in B [ D . If x 2 E then x 2 P  N.x/ for
some P 2 O.E /  O.B [ D /. The set B [ D
S being U -saturated, we have
x 2 K.P /  U [ N.B [ D / and hence x 2 N. ! D / which contradicts
the choice of . This proves that x ES
.
If is a limit ordinal then B D !< .B [ D /, so it follows from (5)
and Fact 1 that B is U -saturated. If x 2 E then x 2 P  N.x/ and hence
x 2 K.P / for some P 2 O.E /  O.B /. The set B being U -saturated, we
have K.P /  U [ N.B /, since x U , we conclude that x 2 N.E / which
is again a contradiction with the choice of . Therefore x E in all cases. The
property (4) shows that N.D / is a neighborhood of x which does not meet the set
S
D . Consequently, the point x has a neighborhood which does not meet
<<S
the set fD W 2 !; /nfgg. The set D is closed and discrete, so x has a
neighborhood which contains at most one point of D. Therefore the set D is closed
and discrete in Z and it follows from the property (4) that jDj  ; thus, we carried
out the inductive step and hence Fact 2 is proved

2 Solutions of problems 001500

547

Returning to our solution observe that there is no loss of generality to assume


that  !. Fix an operator O which witnesses monotone -monolithity of X .
Since in both cases we have to show that l.X /  , we are going to give a common
reasoning for (i) and (ii) indicating the places in which the proofs differ.
Assume that l.X / > and take an arbitrary neighborhood assignment N on the
space X such that N.A/ X for any set A  X with jAj  .
Fix a point a 2 X and apply Fact 2 to the sets fag and U D N.a/ to find a closed
discrete set D00  X nU such that the set D0 D fag [ D00 is U -saturated; it is easy
to see that D0 is closed, discrete and saturated.
Proceeding by induction assume that 0 < < C and we have chosen a
family fD W < g of closed nonempty discrete subsets of X with the following
properties:
S
(6) the set  S
D is saturated for any < ;
(7) D  X nN. < D / for each < .
S
Observe that jD jS
 ext .X /  for any < and therefore j S
< D j 
which shows that N. < D / X by our choice of N . Let Q D < D and
choose a point b 2 X nN.Q/. Apply Fact 2 to the sets Q[fbg and V D N.Q[fbg/
to find a closed discrete set D0  X nN.fbg [ Q/ such that Q [ fbgS[ D0 is
V -saturated and let D D fbg[D0 . It is easy to see that the set D [Q D  D
is saturated, so (6) and (7) still hold for all  .
Therefore this inductive construction can be continued to construct a family D D
C
fDS
W < g with the properties (6) and (7). It follows from (7) that the sets
N. < D / are strictly increasing and the family D is disjoint, so the cardinality
S
of the set D D < C D is equal to C .
Fix any point x 2 X ; we are going to prove that x has a neighborhood which
contains at most one point of D. If x D then there is nothing to prove, so we
assume from now on that x 2 D.
If we are proving (i), observe that jDj D C   which shows that operator O
is applicable to D. If x 2 DnN.D/ then x 2 P  N.x/ for some P 2 O.D/ and
hence the set D is not saturated which is a contradiction with Fact 1.
In the case (ii) if x 2 DnN.D/ then x 2 H nN.D/ for some set H  D with
jH j  . The operator O is applicable to H , so thereSis P 2 O.H / such that
x 2SP  N.x/. There exists < C such that H  S  D and hence P 2
O.  D / which is a contradiction with the fact that  D is a saturated set
by (6).
Therefore in both cases (i) and (ii) we must have x 2 N.D/. Denote by  the
least ordinal such that x 2 N.D /.S
It follows from (7) that N.D / is a neighborhood
of x which does not meet the set << C D . Apply the property (6) and Fact 1
S
to see that the set E D < D is saturated. If x 2 E then x 2 P  N.x/ for
some P 2 O.E/. Therefore x 2 K.P /  N.E/ which is a contradiction with the
choice of  . Thus x E and hence the point x has a neighborhood which meets
only the set D . Since D is closed and discrete, the point x has a neighborhood
which contains at most one point of D. As a consequence, D is a closed discrete
subset of X with jDj D C ; this contradiction makes our solution complete.

548

2 Solutions of problems 001500

V.471. Prove that every subspace of a monotonically monolithic space is a Dspace. As a consequence,
(i) if X is a Lindelf -space then every subspace of Cp .X / is a D-space.
(ii) Observe that ext .Y / D l.Y / for any D-space Y . Therefore (i) generalizes
Baturovs theorem (SFFS-269).
Solution. Assume that X is a monotonically monolithic space and fix an operator O
which witnesses that; let N be a neighborhood assignment on X . For every P  X
say that x 2 P is a central point of P if P  N.x/; denote by K.P / the set of all
central points of P . For an open set U  X say that a set A  X is U -saturated
if K.P /  N.A/ [ U for any P 2 O.A/. If U D ; then U -saturated sets will be
called saturated.
Observe first that monotone monolithity is a hereditary property by Problem 465,
so it suffices to show that every monotonically monolithic space X is a D-space. Let
 D jX j; there is no loss of generality to assume that   !. Fix an operator O
which witnesses monotone monolithity of X . Take an arbitrary point a 2 X and
apply Fact 2 of V.470 to the sets fag and U D N.a/ to find a closed discrete set
D00  X nU such that the set D0 D fag [ D00 is U -saturated; it is easy to see that
D0 is closed, discrete and saturated.
Proceeding by induction assume that 0 < <  C and we have chosen a
family fD W < g of closed nonempty discrete subsets of X with the following
properties:
S
(1) the set  S
D is saturated for any < ;
(2) D  X nN. < D / for each < .
S
S
If N. < D / X then let Q D < D and choose a point b 2 X nN.Q/.
Apply Fact 2 of V.470 to the sets Q [ fbg and V D N.Q [ fbg/ to find a closed
0
discrete set D0  X nN.fbg [ Q/ such that Q [ fbg [ D
S is V -saturated and let
0
D D fbg [ D . It is easy to see that the set D [ Q D  D is saturated, so
the properties (1) and (2) still hold for all  .
ThereforeSthis inductive construction can be continued to construct
S a set D as
soon as N. < D / X . It follows from (2) that the sets N. < D / are
strictly increasing, so it is impossible
W <  C g and hence
S to obtain the family fDS
C
there exists <  such that N. < D / D X . Let D D < D and fix any
point x 2 X .
Denote by  the least ordinal such that x 2 N.D /. ItSfollows from (2) that
N.D / is a neighborhood of x which does not meet the setS << D . Apply the
property (1) and Fact 1 of V.470 to see that the set E D < D is saturated. If
x 2 E then x 2 P  N.x/ for some P 2 O.E/. Therefore x 2 K.P /  N.E/
which is a contradiction with the choice of  . Thus x E and hence the point x
has a neighborhood which meets only the set D . Since D is closed and discrete,
the point x has a neighborhood which contains at most one point of D. This proves
that D is a closed discrete subset of X and hence X is a D-space.

2 Solutions of problems 001500

549

Finally apply Problem 468 to see that if X is a Lindelf -space then Cp .X /


is monotonically monolithic and hence every subspace of Cp .X / is a D-space; this
proves (i).
Now, if Y is a D-space and ext .Y /  , take any open cover U of the space Y .
For any x 2 Y chooseS
a set Ux 2 U such that x 2 Ux . There exists a closed discrete
set D  X such that fUx W x 2 Dg D Y . Therefore fUx W x 2 Dg is a subcover
of U of cardinality  ext .Y /  , i.e., we proved that l.Y /  ext .Y / which
implies l.Y / D ext .Y /. In particular, if X is a Lindelf -space and Y  Cp .X /,
then Y is a D-space by (i) and hence ext .Y / D l.Y /. This settles (ii) and makes
our solution complete.
V.472. Given an infinite cardinal , suppose that a space X is monotonically monolithic and nw.X /   C . Prove that every subspace of X is a D-space.
Deduce from this fact that for any pseudocompact space Z with nw.Z/  !1 ,
every subspace of the space Cp .Z/ is a D-space.
Solution. Observe first that both monotone -monolithity and network weight
not exceeding  C are hereditary properties, so it suffices to show that every
monotonically -monolithic space of network weight at most  C is a D-space.
Fix an operator O of monotone -monolithity in X and let E D fE W <  C g
be a network in X . Take an arbitrary neighborhood assignment N on X . For every
P  X say that x 2 P is a central point of P if P  N.x/; denote by K.P /
the set of all central points of P . For an open set U  X say that a set A  X of
cardinality   is U -saturated if K.P /  N.A/ [ U for any P 2 O.A/. If U D ;
then U -saturated sets will be called saturated.
Take an arbitrary point a 2 X and apply Fact 2 of V.470 to the sets fag and
U D N.a/ to find a countable closed discrete set D00  X nU such that the set
D0 D fag [ D00 is U -saturated; it is easy to see that D0 is closed, discrete and
saturated.
Proceeding by induction assume that 0 < <  C and we have chosen a
family fD W < g of closed nonempty discrete subsets of X with the following
properties:
S
(1) the set  D is saturated
Sfor any < ;
(2) jD j   and D  X nN. < D / for each < .
S
(3) if < and M D f <  C W K.E /nN. < D / ;g then, for the ordinal
 D nnM we have D \ K.E / ;.
S
Let Q D < D ; if N.Q/ D X then our inductive construction ends. If
S
N.Q/ X then consider the set M D f <  C W K.E /nN. < D / ;g. If
M D ; then choose a point b 2 X nN.Q/ arbitrarily. If M ; then let  D nnM
and take a point b 2 K.E /nN.Q/. Apply Fact 2 of V.470 to the sets Q [ fbg
and V D N.Q [ fbg/ to find a closed discrete set D0  X nN.fbg [ Q/ such that
jD0 j   and Q [ fbgS
[ D0 is V -saturated; let D D fbg [ D0 . It is easy to see
that the set D [ Q D  D is saturated, so the properties (1)(3) still hold for
all  .

550

2 Solutions of problems 001500

Therefore this inductiveSconstruction can be continued to construct a set D


whenever <  C and N. < D / X . Suppose that our construction did not
S
stopSat any step <  C . If fN.D / W <  C g X then pick a point x 2
X n fN.D / W <  C g and E 2 E such that x 2 ES N.X /. There is  <  C
such that E D E ; since x 2 K.E /, we have K.E /n fN.D / W < g ; for
any <  C . However, this is impossible becauseS
the ordinal  must be the minimal
element of the set M D f <  C W S
K.E /nN. < D / ;g for some  
and hence K.E
fD W   C 1g which in turn implies that
S  / has to meet the set S
x 2 E  fN.D / W   C 1g  fN.D / W <  C g, a contradiction.
S
C
This proves
S that there exists an ordinal   such that N. < D / D X .
Let D D < D and fix any point x 2 X . Denote by  the least ordinal such
that x 2 N.D /. It
Sfollows from (2) that N.D / is a neighborhood of x which does
not meet the set << D . Apply the property (1) and Fact 1 of V.470 to see
S
that the set E D < D is saturated. If x 2 E then x 2 P  N.x/ for some
P 2 O.E/ (the operator O is applicable to E because jEj   by (2)). Therefore
x 2 K.P /  N.E/ which is a contradiction with the choice of  . Thus x E
and hence the point x has a neighborhood which meets only the set D . Since D
is closed and discrete, the point x has a neighborhood which contains at most one
point of D. This proves that D is a closed discrete subset of X and hence X is a
D-space.
Finally observe that if Z is pseudocompact and nw.Z/  !1 then Cp .Z/ is
monotonically !-monolithic (see Problem 469) and nw.Cp .Z// D nw.Z/  !1 ,
so Cp .X / is a hereditary D-space.
V.473. Prove that any countably compact monotonically !-monolithic space must
be Corson compact.
S
Solution. Let Z be a space; given any A  Z let A
! D fB W B  A and jBj 
!g and A
! D fx 2 X W if H is a G -subset of Z and x 2 H then H \ A ;g.
Recall that the Gruenhages W -game is played on a nonempty closed subset F of
a space Z as follows: at the n-th move the first player called O chooses an open
set Un 2 .F; Z/ and the player P responds by taking a point xn 2 Un . After !many moves have been made, the player O wins if the sequence S D fxn W n 2 !g
converges to F , i.e., the set fn 2 ! W xn V g is finite for any V 2 .F; Z/;
otherwise P is the winner.
Fact 1. Suppose that X is a monotonically !-monolithic space
S and there exists a
family fX W < !1 g of cosmic subsets of X such that X D <!1 X . Then X is
monotonically monolithic.
Proof. It is easy to see that d.X /  !1 ; if d.X /  !, then the space X is cosmic
and hence monotonically monolithic. Now assume that we have d.X / D !1 , take a
countable dense subspace M in the space XS
for each < !1 and let fd W < !1 g
be a faithful enumeration of the set D D <!1 M . We will also need the sets
D D fd W < g and F D D for every < !1 .

2 Solutions of problems 001500

551

Given any x 2 X , there is < !1 such that x 2 X and hence x 2 M . There


exists < !1 such that M  D and hence x 2 D D F . This proves that
S
<!1 F D X . Fix an operator A which witnesses monotone !-monolithity of X
and call a set Y  X small if Y  F for some < !1 .
For every small set Y  X denote by .Y / the least ordinal < !1 such
that Y SF and let O.Y / D A.D.Y / /. If a set Y  X is not small then let
O.Y / D fA.D / W < !1 g. It is immediate that O.Y / is an external network of
Y in X such that jO.Y /j  maxfjY j; !g for any Y  X and Y  Z  X implies
O.Y /  O.Z/.
S
Next, assume that Y D fY W  < g  X for an ordinal and Y  Y
whenever  <  < . If some Y is not small then O.Y / D O.X / D O.Y /. If
everyS
Y is small thenS
let  D .Y / and
S observe that if D supf W  < g D !1
then < O.Y / D < A.D / D <!1 A.D / D O.X / D O.Y /.
 D for every S< and hence Y  D which
Finally, if < !1 then Y S
shows that O.Y /  A.D / D < A.D / D < O.Y /, so Fact 1 is proved.
Fact 2. Suppose that X is countably compact and F is a closed subset of XTfor
which there exists a family U D fUn W n 2 !g  .F; X / such that F D
U
and U nC1  Un for every n 2 !. Then U is an outer base of F in X and hence
.F; X /  !.
Proof. Take any set U 2
T the set Gn D U n \ .X nU / for each
T .F; X / and consider
n 2 !. It follows from n2! U n D F that n2! Gn D ;. Since fGn W n 2 !g is a
decreasing family of closed subsets of a countably compact space X , it is impossible
that Gn ; for all n 2 !, so there exists n 2 ! with Gn D ; and hence the
inclusions F  Un  U n  U show that U is an outer base of F in X , i.e., Fact 2
is proved.
Fact 3. Let Z be a countably compact space. Then A
!
! D A for any A  Z.
Proof. Since it is evident that A
!
!  A, let us prove that A  D D A
!
! . If
x 2 AnD then it is easy to find a sequence
T U D fUn W n 2 !g  .x; Z/ such that
U nC1  Un for every n 2 ! and F D n2! Un  ZnA
! .
By Fact 2, the family U is an outer base for the set F . Since x 2 F , for every
n 2 ! we can choose xn 2 Un \ A because x 2 A and Un a neighborhood
of x. The
T
set B D fxn W n 2 !g  A has cardinality  !, so F \ B  . U / \ A
! D ;.
Since U is an outer base of F in Z, there is n 2 ! with Un \ B D ; which is a
contradiction with the fact that xn 2 Un \ B, so Fact 3 is proved.
Fact 4. If X is a countably compact space and t .X / > ! then X has an uncountable
free sequence.
Proof. There exists an !-closed non-closed set A  X by Fact 1 of S.328. Then
A
! D A and hence A D A
! by Fact 3. Fix any x 2 AnA; then x 2 A
! , so
. / H \ A ; for any G -set H 3 x.

552

2 Solutions of problems 001500

Take a0 2 A arbitrarily and let H0 D X . Suppose that < !1 and we have


constructed points fa W < g  A together with closed G -sets fH W < g
with the following properties:
(1) fx; a g  H for all < ;
(2) H  H0 if 0 < < ;
(3) fa W  < g \ H D ; for all < .
Since x P D fa W  < g, there
T exists a closed G -set H 3 x such that
H \ P D ;. If we let H D H \ . fH W < g/ and take any a 2 H \ A
(this choice is possible because of . /), then the same conditions are fulfilled for
all  and hence the inductive construction can be continued to provide a set
S D fa W < !1 g  A.
We claim that S is a free sequence. Indeed, if < !1 then fa W  < g \ H D
; while fa W   g  H by (1) and (2). The set H being closed, we have
fa W  < g \ fa W   g  fa W  < g \ H D ;. As a consequence, S is a
free sequence of length !1 , so Fact 4 is proved.
Fact 5. If X is a monotonically !-monolithic countably compact space, then X is
compact.
If tightness of X is uncountable then we can apply Fact 4 to find a free sequence
S D fs W < !1 g  X . Let S D fs W < g for any < !1 ; it is
S
straightforward that the set F D <!1 S is countably compact. Since every set
F D S is cosmic, it follows from Fact 1 that F is monotonically monolithic.
Therefore F is compact being a countably compact D-space (see Problem 471).
For the compact space F we have F D S ; since F can be continuously mapped
onto !1 C 1 (see Fact 8 of T.298), we can apply Problem 465 to see that !1 C 1 is
monotonically !-monolithic and hence the space !1 is monotonically !-monolithic
as well. However, ext .!1 / D t .!1 / D !, so !1 must be Lindelf by Problem 470;
this contradiction shows that t .X /  !. By Problem 465(ix), the space X is
monotonically monolithic, so it has D-property by Problem 471. As a consequence,
X is compact being a countably compact D-space (see Problem 471(ii)), i.e., Fact 5
is proved.
Fact 6. If X is a countably compact monotonically !-monolithic space, then for
every nonempty closed set F  X , the player O has a winning strategy at F .
Proof. The space X is compact by Fact 5; fix an operator N that witnesses !-monolithity of X and let  .;/ D X . Suppose that n 2 ! and
U0 ; : : : ; Un ; p0 ; : : : ; pn are the moves of the players O and P respectively
such that U0 D  .;/ and UiC1  Ui for any i < n. Choose an enumeration
fNij W j 2 !g for the family N .fp0 ; : : : ; pi g/ for any i D 0; : : : ; n. The family
Fn D fNij W i  n; j  n and N ij \ F D ;g is finite, so we can find
S
an open set UnC1  Un such that F  UnC1 and U nC1 \ . Fn / D ;; let
 .p0 ; : : : ; pn / D UnC1 .

2 Solutions of problems 001500

553

To see that the strategy  is winning for the player O suppose that a family
fUn ; pn W n 2 !g is a play in which O applies  . If the sequence S D fpn W n 2 !g
does not converge to the set F , then, by compactness of X , there exists a cluster
point x for the sequence S such that x F . Take a set V 2 .x; X / such that V \
F D ;; since N .S / is an external network for S 3 x, weS
can find a set N 2 N .S /
such that x 2 N  V . It follows from the equality S D ffp0 ; : : : ; pn g W n 2 !g
that N 2 N .fp0 ; : : : ; pn g/ for some n 2 ! and hence N D Nij for some i  n and
j 2 !.
By our choice of the strategy  , the set U k does not meet N for any k > m D
maxfn; j g, so for the neighborhood W D X nU mC1 of the point x we have W \S 
fp0 ; : : : ; pm g, i.e., x is not a cluster point of S which is a contradiction. Therefore
 is a winning strategy for the player O and hence Fact 6 is proved.
Returning to our solution suppose that X is a countably compact !-monolithic
space. The space X is compact by Fact 5; apply Problem 465(iii) to see that X  X
is also monotonically !-monolithic, so we can apply Fact 6 to conclude that for
every nonempty closed subset F of X  X , the player O has a winning strategy in
the Gruenhages game on F . Therefore X is Corson compact by Problem CFS-188
and hence our solution is complete.
V.474. Give an example of a Corson compact space that is not monotonically !monolithic.
Solution. We will consider several trees identifying any tree .T; / with the
underlying set T . Recall that a set C  T is a chain if .C; / is a linearly ordered
set. Maximal chains are called branches. To simplify the notation, we denote the
order on !1 and all trees we use by the same letter; the set on which the order
is taken will be always clear from the context. Given a point x 2 T , the set
Lx D fy 2 T W y < xg is well-ordered; its order type will be denoted by ht.x/. It
is an ordinal, then the set fx 2 T W ht.x/ D g is called the -th level of T .
A tree T is continuous if for any x 2 T such that ht.x/ is a limit ordinal, if z 2 T
and y < z for any y 2 Lx , then x  z. We consider that zero is a limit ordinal,
so continuity of a tree T implies that there is only one element x 2 T such that
ht.x/ D 0. We say that a tree T is complete if every branch of T has a maximal
element. For any x 2 T let V .x/ D fy 2 T W x  yg and xO D fy 2 T W y  xg.
A set A  T is dense in T if V .x/ \ A ; for any x 2 T .
Two elements x; y of a tree T are called comparable if either x  y or y  x; if
x and y are not comparable, they are incomparable. Say that x 2 T is an immediate
successor of y 2 T if y is the maximal element of Lx . The point y is called the
immediate predecessor of x. A point x 2 T is an immediate successor if it has an
immediate predecessor. It is an easy exercise that x 2 T is an immediate successor
if and only if ht.x/ is a successor ordinal.
Apply SFFS-065 and SFFS-066 to fix a stationary set S  !1 such that !1 nS
is also stationary and denote by T0 the family of all subsets of S closed in !1 . It is
immediate that all elements of T0 are countable. Given x; y 2 T0 , say that x  y
if x  y and < whenever 2 x and 2 ynx. The set T0 considered with

554

2 Solutions of problems 001500

the above order is a tree: this was proved in Fact 1 of U.176. Every x 2 T0 nf;g is
compact, so it has a maximal element which will denoted by max.x/. For notational
purposes it is convenient to consider that max.;/ D 0.
The set D is the doubleton f0; 1g with the discrete topology. We let D0 D f;g
and Dn is the set of all functions
f W f0; : : : ; n  1g ! D for any n 2 N. We will
S
also need the set D<! D fDn W n 2 !g. The set D<! is easily seen to be a tree if
we define its order by the condition that f  g if and only if dom.f /  dom.g/
and gjdom.f / D f . The expression Lim.!1 / stands for the set of all limit ordinals
in !1 .
Fact 1. Given a tree T suppose that D is a dense subset of T and C  D is a
maximal antichain in D. Then C is a maximal antichain in T .
Proof. Take any x 2 T ; by density of D there is d 2 D such that x  d . It follows
from maximality of C in D that there exists y 2 C which is comparable with d . If
y  d then y is comparable with x because fx; yg  dO and the set dO is linearly
ordered. If d  y, then x  d  y implies that x  y, so any point of T is
comparable with an element of C , i.e., C is a maximal antichain in T and hence
Fact 1 is proved.
Fact 2. Given a complete continuous tree T consider the set T 0 D fx 2 T W ht.x/
is a successor ordinalg and denote by T the topology generated by the family S D
fV .x/ W x 2 T 0 g [ fT nV .x/ W x 2 T 0 g as a subbase. Then
(a) the family S0 D fV .x/ W x 2 T 0 g is T0 -separating, i.e., for any distinct points
x; y 2 T , there exists z 2 T 0 such that V .z/ \ fx; yg is a singleton;
(b) .T; T / is a compact Hausdorff space.
Proof. (a) Take any distinct points x; y 2 T . If x and y are comparable say, x < y,
then y 2 V .y/ and x V .y/. If x and y are incomparable and the ordinal ht.x/
is a successor, then x 2 V .x/ and y V .x/. Now, if both ht.x/ and ht.y/ are
limit ordinals, then by continuity of T there exists z 2 T such that z < x and
y V .z/. Since x 2 V .z/, we proved
that the family S0 is T0 -separating.
T
(b) It follows from (a) that fxg D fS 2 S W x 2 S g; since every element of S is
clopen in .T; T /, the set fxg is closed in .T; T / for any x 2 T , i.e., .T; T / is
Tychonoff being a zero-dimensional T1 -space (see Fact 1 of S.232).
To prove that X D .T; T / is compact it suffices to show that any cover U  S
of the space X has a finite subcover (see TFS-118). So take any U  S such that
S
U D T and let U0 D fU W U 2 U \ S0 g. It is easy to see that
(1) if x; y 2 T are incomparable, then .T nV .x// [ .T nV .y// D T ,
and therefore we can assume, without loss of generality, that the set C D fx 2
T W T nV .x/ 2 U g is a chain. It is an easy exercise to see that there exists a branch
B  T such that C  B; let a be the maximal element of B. Observe that if x B,
then a V .x/ and a .T nV .y// for any y 2 B. Therefore there exists a minimal
b 2 B such that V .b/ 2 U ; since the ordinal ht.b/ is a successor, there exists c 2 B
which is an immediate predecessor of b, i.e., b is the minimal element of B greater

2 Solutions of problems 001500

555

than c. If x B, then c V .x/; besides, c V .x/ for any x 2 B such that


V .x/ 2 U . As a consequence, there exists x 2 B for which c 2 .T nV .x//; this
implies that b  x and therefore V .b/ [ .T nV .x// D T , i.e., fV .b/; T nV .x/g is a
finite subcover of U , so .T; T / is compact and hence Fact 2 is proved.
Fact 3. Given a continuous tree T denote by B the family of all branches of T
which have no maximal element. Take a faithfully indexed set Q D ftb W b 2 Bg
such that Q \ T D ; and let TQ D T [ Q. For each b 2 B, declare that x < tb
for any x 2 b and tb is incomparable with all elements of .T nb/ [ .Qnftb g/. Then
TQ with the original order on T together with the order for the elements of Q is a
complete continuous tree called the completion of T .
Proof. It is straightforward that  is a partial order on TQ . If b 2 B, then no element
of TQ is strictly greater than tb ; this shows that fy 2 TQ W y < xg D fy 2 T W y < xg
is a well-ordered set for any x 2 T . Now, if b 2 B, then fy 2 TQ W y < tb g D b is
also a well-ordered set, so TQ is, indeed, a tree.
To see that TQ is continuous take any x 2 TQ such that ht.x/ is a limit ordinal. We
will assume first that x 2 T . Suppose that z 2 TQ and y < z whenever y < x. If
z 2 T then x  z by continuity of T . If b 2 B and z D tb , then observe that set
Lx D fy 2 T W y < xg is not a branch, so Lx  b and Lx b; take any t 2 bnLx .
Then y < t for any y 2 Lx , so x  t by continuity of T ; it follows from x  t < tb
than x < tb D z.
Finally take any tb 2 Q and assume that z 2 TQ and y < tb for any y 2 b D Ltb .
It follows from maximality of b that z T . If z D tb 0 for some b 0 b, then the
branches b and b 0 are distinct, so bnb 0 ; by maximality of b. If y 2 bnb 0 , then
tb 0 is not comparable with y; this contradiction shows that z D tb , so the tree TQ is
continuous and hence Fact 3 is proved.
S
Fact 4. Assume that An is a maximal antichain
in T0 and let On D fV .a/ W a 2
T
An g for every n 2 !. Then the set O D n2! On is dense in T0 , i.e., for any x 2 T0
there exists y 2 O such that x  y.
Proof. Fix an arbitrary point x 2 T0 ; it follows from maximality of An that
(2) for any t 2 T0 and n 2 ! there exist u 2 An and s 2 T0 such that fu; t g  sO .
We are going to construct by transfinite induction a family fE W < !1 g and an
!1 -sequence f W < !1 g  !1 with the following properties:
(3)
(4)
(5)
(6)

0 D max.x/ and E0 D fxg;


E  T0 is countable and max.p/  for any p 2 ES
and < !1 ;
if 2 Lim.!1 / then D supf W < g and E D fE W < g;
if < < !1 then  < ; E  E and, for any p 2 E and n 2 !
there is u 2 An and q 2 E such that fp; ug  qO and max.q/ > .

To satisfy the condition (3) we must start with 0 D max.x/; E0 D fxg; if <
!1 is a limit ordinal and we have the set f
S W < g and the family fE W < g
then let D supf W < g and E D fE W < g. This guarantees (5).

556

2 Solutions of problems 001500

Now, suppose that, for some ordinal  < !1 , we have the set f W  g
and the family fE W  g with the properties (3)(6) fulfilled for all ;  .
For every element t 2 E and n 2 ! fix u.t; n/ 2 An and s.t; n/ 2 T0 such
that t  s.t; n/; u.t; n/  s.t; n/ and max.s.t; n// > maxf;  g (this is possible
by (2)).
S
0
0
Let EC1
D E [ fs.t; n/ W t 2 E ; n 2 !g; EC1 D fpO W p 2 EC1
g and
C1 D supfmax.p/ W p 2 EC1 g. It is straightforward that the properties (3)(6)
hold for the set f W   C 1g and the family fE W   C 1g, so our inductive
procedure can be continued to construct the promised !1 -sequence f W < !1 g
and the family fE W < !1 g with the properties (3)(6).
Observe that
(7) the set H D f < !1 W D g is closed and unbounded in !1 .
Indeed, if fn gn2!  H is an increasing sequence and n ! then it follows
from (5) that D supfn W n 2 !g D supfn W n 2 !g D because n D n
for any n 2 !. This proves that the set H is closed. Given any < !1 , let 0 D
and nC1 D n C 1 for any n 2 !. A consequence of (6) is that n < nC1 for any
n 2 !; if D supn2! n then 2 H and > which shows that H is cofinal in
!1 and (7) is proved.
Our set S being stationary, it follows from the property (7) that there exists a
limit ordinal 2 H \ S ; fix an increasing sequence fn W n 2 !g such that
supn2! n D . Applying the property (7) once more we conclude that supfn W
n 2 !g D .
Observe that E0  E0 , so we can take p0 D x 2 E0 . Suppose that n 2 !
and we have sets fpi W i  ng  T0 and fui W i < ng  T0 with the following
properties:
(8) pi 2 Ei for all i  n;
(9) pi  piC1 and max.piC1 / > i for any i < n;
(10) ui 2 Ai and ui  piC1 for every i < n.
The property (6) implies that we can choose pnC1 2 EnC1 and un 2 An such that
un  pnC1 ; pn  pnC1 and max.pnC1 / > n  n . It is immediate that (8)(10)
are fulfilled for the sets fpi W i  n C 1g and fui W i < n C 1g, so our inductive
procedure gives us sequences fpn W n 2 !g and fun W n 2 !g with the properties
(8)(10).
It follows from
S (4), (6) and (8) that pn for any n 2 !. It is easy to see that
the set p D . n2! pn / [ fg belongs to T0 and un  pnC1 < p; in particular,
p 2 V .un /  On for any n 2 !. Therefore p 2 O and x D p0 < p, so O is dense
in T0 and hence Fact 4 is proved.
Fact 5. An element x 2 T0 is an immediate successor if and only if D max.x/ is
an isolated point of the set x.
Proof. Suppose that y is an immediate predecessor of x and let D max.y/. All
points of xny are greater than . If  D nn.xny/, then y < y [ f g  x which
shows that y [ f g D x and hence  D , so the set fg D .; !1 / \ x is open in
x; this proves necessity.

2 Solutions of problems 001500

557

If the point is isolated in x, then fg is an open subset of x, so y D xnfg 2 T0


is an immediate predecessor of x and hence Fact 5 is proved.
Fact 6. The tree T0 is continuous.
Proof. Suppose that x 2 T0 and ht.x/ is a limit ordinal. Then x has no immediate
predecessor and hence D max.x/ is not isolated in x by Fact 5. If z < y for any
z with z < x, then 0;
\ x  y for any < and therefore xnfg  y. Since
xnfg is dense in x, the set x is contained in y.
If  2 ynx and  < , then pick an ordinal 2 .; / \ x and observe that the
ordinal  2 yn.0;
\ x/ witnesses the fact that y is not a successor of 0;
\ x <
x; this contradiction shows that x  y and hence Fact 6 is proved.
Returning to our solution consider the set T1 D T0  D<! . For every t 2 T0
choose a faithfully indexed set fbt; W 2 S; > max.t /g of branches in the tree
D<! such that jf W s 2 bt; gj D !1 for any s 2 D<! . Given x; y 2 T1 such that
x D .t; s/ and y D .t 0 ; s 0 / we declare that x < y if and only if
(11) t D t 0 and s < s 0 ; or
(12) there exists > max.t / such that t [ fg  t 0 and s 2 bt; .
Let us check that  is a partial order on T1 . It is evident that the inequalities
x < y and y < x cannot hold together, so we only have to check transitivity.
Suppose that x; y; z 2 T1 , x D .t; s/; y D .t 0 ; s 0 /; z D .t 00 ; s 00 / and x < y < z.
It follows from (11) and (12) that t  t 0  t 00 ; assume first that t D t 0 and hence
s < s 0 . If t 0 D t 00 , then s < s 0 < s 00 , so x D .t; s/ < z D .t; s 00 /. If t 0 < t 00 , then
there exists > max.t 0 / such that t 0 [ fg  t 00 and s 0 2 bt 0 ; . The set bt 0 ; being a
branch, it follows from s < s 0 that s 2 bt 0 ; D bt; ; this, together with the fact that
t [ fg D t 0 [ fg  t 00 shows that x < z by (12). Finally, assume that t < t 0 and
fix > max.t / such that t [ fg  t 0 and s 2 bt; ; since also t [ fg  t 0  t 00 , the
property (12) implies that x < z.
To see that T1 is a tree, take any x D .t; s/ 2 T1 . We will first prove that the set
Lx D fy 2 T1 W y < xg is linearly ordered by <, i.e., any two elements of Lx are
comparable. Take any y D .t 0 ; s 0 / 2 Lx and z D .t 00 ; s 00 / 2 Lx . Observe that T0 is
a tree and t 0  t and t 00  t which implies that t 0 and t 00 are comparable, so we can
assume, without loss of generality, that t 0  t 00 . If t 0 D t 00 then there exist ordinals
and greater than max.t 0 / D max.t 00 / such that t 0 [ fg  t and t 00 [ fg  t ; it
is easy to see that this implies D . Therefore s 0 2 bt 0 ; and s 00 2 bt 0 ; ; since bt 0 ;
is a chain, the elements s 0 and s 00 must be comparable, so the property (11) shows
that y D .t 0 ; s 0 / and z D .t 0 ; s 00 / are comparable.
Next, assume that t 0 < t 00 and let D nn.t 00 nt 0 /; then t 0 [ fg  t 00 . There exists
< !1 such that t 0 [ fg  t and s 0 2 bt 0 ; ; it follows from t 0 [ fg  t 00  t that
t 0 [ fg  t and therefore D . Finally, note that s 0 2 bt 0 ; D bt 0 ; and therefore
.t 0 ; s 0 / < .t 00 ; s 00 /, i.e., y and z are comparable by (12).
To see that the set Lx is well-ordered, take any nonempty set A  Lx and
consider the element t0 D nnft 0 W .t 0 ; s 0 / 2 A for some s 0 2 D<! g. If A0 D
f.t 0 ; s 0 / 2 A W t 0 D t0 g, then it immediate from (11) and (12) that y < z whenever

558

2 Solutions of problems 001500

y 2 A0 and z 2 AnA0 , so it suffices to find the minimal element in A0 . We already


know that all elements of A0 are comparable and A0 D f.t0 ; s 0 / W S 0 2 D0 g for some
D0  D<! , so it follows from (11) that all elements of D0 are comparable; since
D<! is a tree, there is a minimal element d 2 D0 . It is immediate that .t0 ; d / is a
minimal element of A0 and hence by our above observation, .t0 ; d / is the minimal
element of A, i.e., we proved that T1 is, indeed, a tree.
Let us establish the following relationship between the trees T0 and T1 :
(13) for any elements t; t 0 2 T0 we have t  t 0 if and only if .t; ;/  .t 0 ; ;/.
Note that if .t; ;/  .t 0 ; ;/ then t  t 0 : this is an immediate consequence of
(11) and (12), so we have sufficiency in (13). Now, assume that t < t 0 and let
D nn.t 0 nt /. Then t [ fg  t 0 ; the set bt; being a branch, we have ; 2 bt; and
hence .t; ;/ < .t 0 ; ;/ by (12). This settles necessity and shows that (13) is proved.
The following property characterizes the elements of T1 that have no immediate
successor.
(14) given any x D .t; s/ 2 T1 its height ht.x/ is a limit ordinal if and only if
s D ;.
If s ; then it has an immediate predecessor s 0 in the tree D<! . It is
straightforward that .t; s 0 / is an immediate predecessor of x, i.e., we proved
necessity. Now, if y D .t 0 ; s 0 / < x D .t; ;/, then there is an ordinal < !1 such
that t 0 [ fg  x and s 0 2 bt 0 ; . If s 00 2 bt 0 ; and s 0 < s 00 , then .t 0 ; s 0 / < .t 0 ; s 00 / < x,
i.e., .t 0 ; s 0 / is not an immediate predecessor of x and hence (14) is proved.
Our next step is to show that T1 is continuous, so fix any element x 2 T1 such
that ht.x/ is a limit ordinal and assume that y D .t0 ; s0 / is greater than any element
of Lx D fy 2 T1 W y < xg. It follows from (14) that x D .t; ;/ for some t 2 T0 .
Consider first the case when the element t has an immediate predecessor t 0 . Then
t D t 0 [ fg for some > max.t 0 /; for any s 2 bt 0 ; we can apply (12) to see
that .t 0 ; s/ < x and hence .t 0 ; s/ < y by our choice of y. There exists an ordinal
.s/ > max.t 0 / such that t 0 [f.s/g  t0 and s 2 bt 0 ;.s/ . Observe that .s/ .s 0 /
implies that the elements t 0 [f.s/g and t 0 [f.s 0 /g are incomparable in the tree T0 ;
this contradiction with the inclusion ft 0 [ f.s 0 /g; t 0 [ f.s/gg  tO0 shows that there
exists < !1 such that .s/ D for any s 2 bt 0 ; and hence bt 0 ;  bt 0 ; . Since
bt 0 ; and bt 0 ; are branches of the tree D<! , we have bt 0 ; D bt 0 ; and hence D
by faithfulness of the indexation of the family fbt 0 ; W  > max.t 0 /g. Consequently,
t D t 0 [ fg D t 0 [ fg  t0 and therefore x D .t; ;/  .t0 ; s0 / D y as promised.
Finally assume that ht.t / is s limit ordinal. If t 0 < t , then it follows from (13)
that .t 0 ; ;/ < .t; ;/ D x and hence .t 0 ; ;/ < .t0 ; s0 / which in turn implies that
t 0 < t0 . By continuity of the tree T0 (see Fact 5) we have t  t0 and therefore
x D .t; ;/  .t0 ; ;/  .t0 ; s0 / D y and hence we proved that
(15) the tree T1 is continuous.
Let T 0 D f.t; s/ 2 T1 W s ;g; it follows from (14) that T 0 is precisely the set
of the elements of T that have an immediate predecessor. Consider the family B of
the branches of T1 . Given any .t; s/ 2 T1 it is easy to find s 0 2 D<! such that s < s 0

2 Solutions of problems 001500

559

and hence .t; s/ < .t; s 0 /. This proves that every element of T1 has a successor and
hence B coincides with the set of the branches of T1 with no maximal element.
Take a faithfully indexed set Q D ftb W b 2 Bg such that Q \ T D ; and
let T D T1 [ Q. For each b 2 B, declare that x < tb for any x 2 b and tb
is incomparable with all elements of .T1 nb/ [ .Qnftb g/. Then the set T with the
respective order is a complete continuous tree by Fact 3. Since no element of Q has
an immediate predecessor, also for the tree T, the set T 0 coincides with the set of all
elements that have an immediate predecessor. Denote by the topology on the tree
T generated by the family S D fV .x/ W x 2 T 0 g [ fTnV .x/ W x 2 T 0 g as a subbase.
By Fact 2, the space K D .T; / is compact and Hausdorff.
Suppose that C  T is an infinite chain. Since Q is an antichain, the set C has
at most one element of Q and hence the chain C 0 D C nQ  T1 has the same
cardinality as C . Given any elements .t; s/ 2 C 0 and .t 0 ; s 0 / 2 C 0 , it follows from
their comparability, that t and t 0 are comparable in T0 by (11) and (12). Therefore
the set E D ft 2 T0 W there exists s 2 D<! such that .t; s/ 2 C 0 g is a chain in T0 .
Since all chains in T0 are countable by Fact 1 of U.176, we conclude that jEj  !.
As a consequence, jC j D jC 0 j  jE  D<! j  ! and hence we proved that
(16) every chain in the tree T is countable.
Observe that U D fV .x/ W x 2 T 0 g is a T0 -separating family of clopen subsets
of K by Fact 2. If x 2 T, then it follows from (16) that the set fy 2 T 0 W x 2
V .y/g  xO is countable because xO is a chain in T. This shows that K has a T0 separating point-countable family of clopen sets and therefore K is Corson compact
(see CFS-118).
Since monotone !-monolithity is a hereditary property, to show that K is not
monotonically !-monolithic, it suffices to disprove monotone monolithity of T1
with the topology inherited from K.
Fact 7. Suppose that An is an antichain in T1 for each n 2 !. Then
S there exists a
maximal antichain B in T1 such that B \ xO D ; for any x 2 A D n2! An .
Proof. For any a D .t; s/ 2 A consider the set C.a/ D ft [ fg W 2 S; >
max.t / and s 2 bt; g. We will prove first that
S
(17) En D fC.a/ W a 2 An g is an antichain in T0 .
Take distinct elements a D .t; s/ and a0 D .t 0 ; s 0 / of the set An and assume that
some set t [ fg 2 C.a/ is comparable with a set t 0 [ fg 2 C.a0 /. There is no
loss of generality to assume that t [ fg  t 0 [ fg. If t [ fg t 0 [ fg, then
D max.t 0 [ fg/ > D max.t [ fg/ which implies that t [ fg  t 0 and hence
.t; s/  .t 0 ; s 0 / by (12), i.e., we obtained a contradiction. Therefore t [fg D t 0 [fg
and hence D max.t 0 [ fg/ D max.t [ fg/ D which shows that t D t 0 . We
also have s 2 bt; and s 0 2 bt; ; the set bt; being a branch, the elements s and s 0
are comparable and hence .t; s/ is comparable with .t; s 0 / D .t 0 ; s 0 / by (11). This
contradiction shows that no element of C.a/ is comparable with an element of C.a0 /
if a a0 . Since it is evident that every C.a/ is an antichain, the set En is also an
antichain, so (17) is proved.

560

2 Solutions of problems 001500

Take a maximal antichain Dn in the tree T0 such that En  Dn ; it turns out that
(18) for any a D .t; s/ 2 An , the set tO does not meet Dn .
To see that (18) is true, assume that d  t and .t; s/ 2 An for some s 2 D<! and
d 2 Dn . If d 2 Dn nEn , then take > max.t / such that s 2 bt; and observe that
d  t  t [fg 2 En which is a contradiction with the fact that d and t [fg belong
to the same antichain. Now, if d 2 En , then d D t 0 [ fg where .t 0 ; s 0 / 2 An and
s 0 2 bt 0 ; for some s 0 2 D<! . This, together with (12) implies that .t 0 ; s 0 /  .t; s/,
so t 0 D t and hence
S t [ fg  t which is again a contradiction, i.e.,
T we proved (18).
Let Hn D fV .t / W t 2 Dn g for every n 2 !; the set H D n2! Hn is dense
in T0 by Fact 4. Take a maximal antichain B 0 in the set H ; then B 0 is a maximal
antichain in T0 by Fact 1. To see that the set B D f.t; ;/ W t 2 B 0 g is as promised
observe first that distinct elements of B are not comparable by (13), i.e., B is an
antichain. Now, if .t 0 ; s 0 / 2 T1 , then take > max.t 0 / such that s 0 2 bt 0 ; . The set
t 0 [ fg is comparable with some t 2 B 0 by maximality of B 0 . If t 0 [ fg  t , then
.t 0 ; s 0 /  .t; ;/ by (12). If t < t 0 [ fg then D max.t 0 [ fg/ does belong to t
and hence t  t 0 . Then .t; ;/  .t 0 ; s 0 /, so in all possible cases .t 0 ; s 0 / is comparable
with an element of B; this proves that B is a maximal antichain in T1 .
Finally assume that .t 0 ; ;/ 2 B and there exists n 2 ! such that .t 0 ; ;/  .t; s/
for some x D .t; s/ 2 An ; this evidently implies that t 0  t . However, t 0 2 Hn and
hence there is d 2 Dn such that d  t 0 , so it follows from d  t 0  t that d  t
which is a contradiction with (18). Consequently, xO \ B D ; for any x 2 A, i.e.,
Fact 7 is proved.
Finally, assume that an operator N witnesses that the space T1 is monotonically
!-monolithic. For every x 2 T1 let N  .x/ D N .x/;
O observe that the operator N is
applicable to the set xO because xO is countable by (16). Given any N 2 N  .x/ let
M.N; x/ D fy 2 N \ V .x/ W V .y/ \ N D fygg and observe that the set M.N; x/
is an antichain for any N 2 N  .x/.
Let A0 be a maximal antichain in the tree T1 ; proceeding by induction assume
that n 2 ! and we have maximal antichains A0 ; : : : ; An in the tree T1 with the
following properties:
S
(20) if i < n, then AiC1 D fAiC1 .a/ W a 2 Ai g where AiC1 .a/  V .a/nfag is a
maximal antichain in V .a/ for any a 2 Ai ;
(21) if i < n and a 2 Ai , then AiC1 .a/ \ uO D ; whenever u 2 M.N; a/ and
N 2 N  .a/.
For every a 2 An let Ma D ffagg [ fM.N; a/ W N 2 N  .a/g; since Ma
is a countable family of antichains of T1 , we can apply Fact 7 to find
S a maximal
antichain Ba in the tree T1 such that xO \ Ba D ; whenever x 2
Ma . Then
AnC1 .a/ D Ba \ V .a/ is a maximal antichain in V .a/ and AnC1 .a/  V .a/nfag.
To see it, take any x 2 V .a/; by maximality of Ba there exists y 2 Ba comparable
with x. If x  y, then a  x  y implies that y 2 V .a/. If y < x, then it follows
from a  x, that y is comparable with a. Since the inequality y  a is impossible

2 Solutions of problems 001500

561

S
because of the choice of Ba and the fact that a 2
Ma , we have a < y, i.e.,
y 2 V .a/ \ Ba D AnC1 .a/ and hence AnC1 .a/  V .a/nfag is a maximal antichain
in V .a/.
S
Let AnC1 D fAnC1 .a/ W a 2 An g. Take any distinct x; y 2 AnC1 . If x; y 2
AnC1 .a/ for some a 2 An , then x and y are incomparable because AnC1 .a/ is an
antichain. Now, if there are distinct a; a0 2 An such that x 2 AnC1 .a/ and y 2
AnC1 .a0 /, then V .a/ \ V .a0 / D ; because An is an antichain; since V .x/  V .a/
and V .y/  V .a0 /, we have the inclusion V .x/ \ V .y/  V .a/ \ V .a0 / D ;
and therefore the elements x and y are incompatible. This proves that AnC1 is an
antichain and hence the property (20) holds for all i  n.
Take any x 2 T1 . Since the antichain An is maximal, there exists a 2 An
which is comparable with x. If x  a, then any element of the nonempty set
AnC1 .a/ is comparable with x. If x  a, then we can apply maximality of AnC1 .a/
to find an element y 2 AnC1 .a/ comparable with x. This shows that AnC1 is a
maximal antichain in T1 . Finally, observe that the property (21) holds by our choice
of AnC1 .a/ for any a 2 An and hence our inductive procedure can be continued
to obtain a sequence fAn W n 2 !g of maximal antichains in T1 that satisfy the
conditions (20) and (21) for any n 2 !.
Apply
S Fact 7 once more to find an element x 2 T1 such that x aO for any point
a 2 n2! An . By maximality of An we can find an 2 An such that an < x for any
n 2 !. The set Q D fan W n 2 !g  xO is linearly ordered, so it follows from (20)
that an < anC1 for any n 2 !. Let y be the minimal element of xO such that an < y
for any n 2 !.
It is easy to see that y has no immediate predecessor and hence ht.y/ is a limit
ordinal. Take any open set U in the space T1 such that y 2 U . There exist points
p1 ; : : : ; pm ; q1 ; : : : ; qr 2 T1 such that
y 2 W D .TnV .p1 // \ : : : \ .T1 nV .pm // \ V .q1 / \ : : : \ V .qr /  U:
Observe first that the inequality an < y implies that an 2 T1 nV .pi / for any
i  m and n 2 ! and hence Q  W0 D .TnV .p1 // \ : : : \ .T1 nV .pm //. Every
element qi has an immediate predecessor, so qi < y and hence we can pick a point
an.i/ > qi for any i D 1; : : : ; r. If k > maxfn.1/; : : : ; n.r/g, then ak 2 W1 D
V .q1 / \ : : : \ V .qr / and therefore ak 2 W0 \ W1 D W  U . This proves that
U \ Q ; for any set U 2 .y; T1 /, i.e., y 2 Q.
Take t 2 T0 and s 2 D<! such that y D .t; s/. There is n 2 ! such that s 2 Dn ;
let si jn D s and si .n/ D i for each i 2 D. We leave it to the reader to check that s0
and s1 are the only immediate successors of s, i.e.,
(22) for any s 0 2 D<! , if s < s 0 , then si  s 0 for some i 2 D.
Let y0 D .t; s0 / and y1 D .t; s1 /; it is clear that y < yi for every i 2 D. We will
prove that y0 and y1 are the only immediate successors of y, i.e.,
(23) for any z 2 T1 , if y < z, then yi  z for some i 2 D.

562

2 Solutions of problems 001500

To see that (23) holds take any z D .t 0 ; s 0 / 2 T1 such that y < z. If t 0 D t , then
s < s 0 and hence si  s 0 for some i 2 D by (22). Then yi  z, so the property (23)
is true for this case. If t 0 t , then there exists > max.t / such that t [ fg  t 0
and s 2 bt; . Since bt; has no maximal element, we can pick s 00 2 bt; with s < s 00 .
Apply (22) once more to see that si  s 00 for some i 2 D and hence si 2 bt; . This
shows that yi D .t; si /  .t 0 ; s 0 / D z by (12), so (23) is proved.
It is an immediate consequence of (23) that V .y/nfyg D V .y0 / [ V .y1 /, so
the set G D .T1 nV .y0 // \ .T1 nV .y1 // is an open neighborhood of the point y.
Therefore we can find N 2 N .Q/ such that y 2 N  G. Since N is !-monotone,
there exists n 2 ! such that N 2 N .fa0 ; : : : ; an g/  N  .an /. By our choice of
the set G we have y 2 M.N; an /, so the property (21) shows that AnC1 .an / \
yO D ; which is a contradiction with anC1 2 AnC1 .an / \ y.
O Therefore T1 is not
monotonically !-monolithic and hence our Corson compact space K  T1 is not
monotonically monolithic either, i.e., our solution is complete.
V.475. Observe that if X is a monotonically !-monolithic compact space and !1
is a caliber of X , then the space X is metrizable. Prove that there exists a monotonically !1 -monolithic (and hence monotonically !-monolithic) non-compact
pseudocompact space X such that !1 is a caliber of X .
Solution. If X is a monotonically !-monolithic compact space, then X is Corson
compact by Problem 473, so it is metrizable if !1 is a caliber of X by CFS-132.
The following statement gives a method of construction of monotonically
-monolithic spaces.
Fact 1. If X is a space and every subspace of X of cardinality   is closed in X
then X is monotonically -monolithic.
Proof. If X 0 is the set X with the discrete topology and f W X 0 ! X is the identity
map then f jA is homeomorphism for any A  X 0 with jAj  . The space X 0 being
monotonically monolithic, Fact 1 of V.469 guarantees that X is monotonically monolithic, so Fact 1 is proved.
Fact 2. Given an uncountable regular cardinal  assume that  is a caliber of a space
X and Y  X is G -dense in X , i.e., every nonempty G -subset of X intersects Y .
Then  is also a caliber of Y .
Proof. Take any family U   .Y / with jU j D . For each U 2 U choose a set
OU 2 .X / such that OU \ Y D U ; then U V implies OU OV , so the family
O D fOU W U 2 U g   .X / has cardinality . Since
T  is a caliber of X , there
exists a family V  U such that jVj D

and
G
D
fOU W U 2 Vg ;. Now,
T
G is a nonempty G -subset of X , so V D G \ Y ; and hence the family V
witnesses that  is a caliber of Y , so Fact 2 is proved.
Returning to our solution let  D 2!1 and take a set A of S
cardinality . Choose a
disjoint family fA W < g of subsets of A such that A D < A and jA j D 
for every < . If B  A and jBj  !1 then jIB j  .2! /!1 D . Since the family

2 Solutions of problems 001500

563

of all subsets of A of cardinality !1 ofSthe set A has cardinality , we can choose an


enumeration ff W < g of the set fIB W B  A and jBj  !1 g. For any < 
there is a unique B  A such that jB j  !1 and f 2 IB .
Fix <  and define a point x 2 IA as follows: x .a/ D f .a/ for any a 2 B ;
if a 2 A nB then x .a/ D 1 and x .a/ D 0 for all a 2 An.A [ B /.
Let us show that X D fx W < g is the promised space. If B  A; jBj  !1
and y 2 IB then y D f and B D B for some ordinal < . As a consequence,
x jB D f jB D y and hence X covers all !1 -faces of IA . Therefore X is
pseudocompact by Fact 2 of S.433. It is easy to see that X is G!1 -dense in IA ;
since  D !1 is a caliber of IA (see SFFS-282), we can apply Fact 2 to convince
ourselves that !1 is a caliber of X .
If Y  X and jY j  !1 then fix any point xS2 X . Choose a set E   such that
jEj  !1 and Y  fx W 2 Eg. The set G D fB W 2 E[fgg has cardinality
at most !1 , so A nG ;; pick a point a 2 A nG and note that x .a/ D 1 while
x .a/ D 0 for any 2 Enfg. This shows that the set fy 2 X W y.a/ > 0g is
an open neighborhood of x whose intersection with Y has at most one point. This
proves that every set Y  X of cardinality at most !1 is closed and discrete in X , so
X is monotonically !1 -monolithic by Fact 1. The space X is not compact because it
is dense in IA and the point whose all coordinates are equal to zero does not belong
to X .
V.476. Suppose that X is a monotonically Sokolov space. Prove that
(a) X is monotonically !-monolithic;
(b) any F -subset of X is monotonically Sokolov.
Solution. (a) For any countable family F of closed subsets of the space X fix a
retraction rF W X ! X and a countable family N .F/  exp.X / that witness
the monotone Sokolov property of X . Given a countable set A  X let F.A/ D
ffxg W x 2 Ag and O.A/ D N .F.A//. It is immediate that the assignment A !
F.A/ is !-monotone, so the operator O is also !-monotone. Since rF .A/ .F / 
F for any F 2 F.A/, we have the equality rF .A/ .x/ D x for any x 2 A.
The continuity of the map rF .A/ implies that rF .A/ .x/ D x for any x 2 A
and therefore A  rF .A/ .X /. The family N .F.A// being a countable external
network for rF .A/ .X / in X , it must be a countable external network for A in X ,
so X is monotonically !-monolithic.
(b) For any countable family G of closed subsets of X fix a continuous retraction
rG W X ! X and a countable collection N .G/  exp.X / that witness the
monotone Sokolov property of X . Take any F -subset Y ofSthe space X and let
H be a countable family of closed subsets of X with Y D H.
Given a countable family G of closed subsets of the space Y it is easy to see
that F.G/ D H [ fG \ H W G 2 G and H 2 Hg is a family of closed subsets
of X and the assignment G ! F.G/ is !-monotone. We will prove that the map
sG D rF .G/ jY and the family O.G/ D fN \ Y W N 2 N .F.G//g witness the
monotone Sokolov property of the space Y .

564

2 Solutions of problems 001500

If y 2 Y , then y 2 H for some H 2 H. Since H 2 H  F.G/, by the


choice of rF .G/ , we have rF .G/ .H /  H . Hence sG .y/ D rF .G/ .y/ 2 H  Y ,
so sG W Y ! Y is a continuous retraction. The family N .F.G// being an external
network for rF .G/ .X / in X , it follows that O.G/ is an external network for sG .Y /
in Y . The assignments F ! F.G/ and F.G/ ! N .F.G// are !-monotone, so the
operator O must also be !-monotone. Finally, we have the inclusion sG .G/
S G
for each G 2 G. Indeed, take any y 2 G for some G 2 G. Since y 2 Y D H, we
can find H 2 H with y 2 H . Then y 2 G \ H 2 F.G/. It follows from the choice
of rF .G/ that rF .G/ .G \ H /  G \ H . Thus sG .y/ D rF .G/ .y/ 2 G \ H  G.
V.477. Suppose that a space X is monotonically retractable and fix, for any
countable set A  X , a retraction rA W X ! X and a network N .A/ for the
map rA that witness monotone retractability of X . Prove that, for any countable
family G of closed subsets of X we can find a countable set P .G/  X such that
rP .G/ .G/  G for any G 2 G and the assignment G ! P .G/ is !-monotone.
S
Solution. Let N .X / D fN .A/ W A is a countable subset of X g. For any closed
set G  X and H 2 N .X /, if G \ H 6D ; then pick a point pG;H 2 G \ H . Given
a countable family G of closed subsets of X and a countable subfamily H of N .X /,
let E.G; H/ D fpG;H W G 2 G; H 2 H; and G \ H 6D ;g. If G is a nonempty
closed subset of X , then pick a point pG 2 G.
Fact 1. Suppose that we have families A  exp.X /; B  exp.Y / and C  exp.Z/
for some sets X; Y and Z.
(a) If the maps p W A ! B and q W B ! C are !-monotone then the map q p W
A ! C is also !-monotone;
(b) if B is invariant under countable unions, jT j  ! and pt W A ! B is !monotone
for any t 2 T then the map p W A ! B defined by p.A/ D
S
fpt .A/ W t 2 T g for any A 2 A is also !-monotone.
Proof. (a) If A 2 A is countable, then B D p.A/ is countable and hence q.B/ D
.q p/.A/ is countable. If A  A0 , then B D p.A/  p.A0 / D B 0 because the
0
0
map p is !-monotone. Therefore
S .q p/.A/ D q.B/  q.B / D .q p/.A /.
Finally, if we have a set A D n2!
S An such that An  AnC1 for any n 2 !,
then .q p/.A/ D q.p.A// D q. n2! p.An // by !-monotonicity of the map
p. Since also
S p.An /  p.AnC1
S / for any n 2 !, by monotonicity of the map q,
we have q. n2! p.An // D n2! q.p.An //; this settles (a).
(b) If A 2 A is a countable, then p.A/ is countable being a countable union of
countable sets. If A  A0 , then ptS
.A/  pt .A0 / for every t 2 T , so p.A/ 
0
p.A /. Finally assume that A D S
n2! An and An  AnC1 for each n 2 !.
We only have to show that p.A/  n2! p.An /, so take any point x 2 p.A/.
There exists t 2 T such that x 2 pt .A/; by !-monotonicity of pt we can
find k 2 ! forSwhich x 2 pt .Ak /. Now it follows from pt .Ak /  p.Ak / that
x 2 p.Ak /  n2! p.An /; this settles (b) and shows that Fact 1 is proved.
Returning to our solution, for any countable family G of closed subsets of X let
Q.G/ D fpG W G 2 G n f;gg. The assignment G ! Q.G/ is easily seen to be

2 Solutions of problems 001500

565

!-monotone. Define P0 .G/ D Q.G/


S and PnC1 .G/ D Pn .G/ [ E.G; N .Pn .G/// for
any n 2 !. Then the set P .G/ D fPn .G/ W n 2 !g is as promised.
It takes an easy induction and Fact 1 to show that each of the assignments Pn is
!-monotone and hence P is also !-monotone. Let us show that rP .G/ .G/  G for
each G 2 G.
If this is not true, then rP .G/ .x/ 62 G for some G 2 G and x 2 G. Then U D
X nG is a neighborhood of rP .G/ .x/ which does not intersect G. Since N .P .G// is a
network for rP .G/ , there exists a setS
H 2 N .P .G// such thatS
x 2 H and rP .G/ .H / 
U . The equality N .P .G// D N . fPn .G/ W n 2 !g/ D fN .Pn .G// W n 2 !g
implies that H 2 N .Pn .G// for some n 2 !. It follows from x 2 G \ H that
G \ H 6D ;, so a point pG;H 2 G \ H has been chosen and
pG;H 2 E.G; N .Pn .G///  PnC1 .G/  P .G/  rP .G/ .X /:
Therefore pG;H D rP .G/ .pG;H / 2 rP .G/ .H /  U  X n G. This contradiction
shows that rP .G/ .G/  G for any G 2 G.
V.478. Given aS
space Y denote by CL.Y / the family of all closed subsets of Y and
let CL .Y / D fCL.Y n / W n 2 Ng. Prove that, for any space X , the following
conditions are equivalent:
(a) X is monotonically retractable:
(b) for any countable family G of closed subsets of X , there exists a retraction
sG W X ! X and a countable network O.G/ for the map sG such that s.G/  G
for any G 2 G and the assignment O is !-monotone;
(c) for any countable family G  CL .X / there exists a retraction tG W X ! X
and a countable network P.G/ for the map tG such that, for any n 2 N, we have
the inclusion tGn .G/  G for any G 2 G \ CL.X n / and the assignment P is
!-monotone.
Solution. Recall that if Y is a space and f W Y ! Y , then for any n 2 N, the
function f n W Y n ! Y n is defined by the equality f n .y/ D .f .y1 /; : : : ; f .yn // for
any y D .y1 ; : : : ; yn / 2 Y .
It is immediate that (c)H)(b). If (b) holds and A is a countable subset of X
then consider the family G.A/ D ffxg W x 2 Ag, the retraction rA D sG.A/ and the
family N .A/ D O.G.A//. It is easy to check that rA and N .A/ witness that X is
monotonically retractable, so we have (b)H)(a).
Next assume that (a) holds and fix, for any countable B  X , a continuous
retraction sB W X ! X and a countable network O.B/ for the map sB such that
B  sB .X / and the assignment O is !-monotone.
Take an arbitrary point a 2 Y D X N and let pn W Y ! X be the
S projection onto
the n-th factor for any n 2 N. Observe that the set E.A/ D fpn .A [ fag/ W
n 2 Ng  X is countable for any countable A  Y . For each y 2 Y let
.sE.A/ /N .y/.n/ D sE.A/ .y.n// for any n 2 N. We leave it to the reader to check that
the map rA D .sE.A/ /N W Y ! Y is a retraction such that A  rA .Y /. Denote by

566

2 Solutions of problems 001500

Q
N .A/ the family of all sets U  Y such that U D fNn W n 2 Ng where, for some
finite set F  N, we have Nn 2 O.E.A// for all n 2 F and Nn D X if n 62 F .
It is straightforward to verify that the assignment A ! N .A/ is !-monotone. To
see that the family N .A/ is a network for the map rA take any point y 2 Y and a set
W 2 .rA .y/; Y /. There exists a number n 2 N and sets V1 ; : : : ; Vn 2 .X / such
that rA .y/ 2 V  W where V D V1  : : :  Vn  X Nnf1;:::;ng . The family O.E.A//
being a network for the map sE.A/ , it follows from rA .y/.i / D sE.A/ .y.i // 2 Vi that
we can find Fi 2 O.E.A// such that y.i / 2 Fi and sE.A/ .Fi /  Vi for each i  n.
Then y 2 F D F1  : : :  Fn  X Nnf1;:::;ng 2 N .A/ and rA .F /  V  W which
shows that N .A/ is a network for the map rA and hence rA and N .A/ witness that
the space X N is monotonically retractable.
Now we can apply Problem 477 to convince ourselves that for any countable
family H of closed subsets of Y we can choose a countable set P .H/  Y such that
rP .H/ .H /  H for any H 2 H and the assignment P is !-monotone.
Observe that the set Zn D fy 2 Y W fk 2 N W y.k/ 6D a.k/g  f1; : : : ; ngg  Y
is closed in Y and the map 'n D pn jZn W Zn ! X n is a homeomorphism for which
n
we have the equality sE.A/
D 'n rA 'n1 for any n 2 N. If G is a countable
subfamily of CL .X /, then fix n.G/ 2 N such that G  X n.G/ for every G 2 G and
1
consider the family G 0 D f'n.G/
.G/ W G 2 Gg  CL.X N /.
Let tG D sE.P .G 0 // W X ! X and P.G/ D O.E.P .G 0 ///. Then tG is a retraction
and P.G/ is a network for tG . Since the assignments P , E, O and G ! G 0 are !monotone, the assignment P is also !-monotone. Finally, for any G 2 G \ CL.X n /
we have n.G/ D n and therefore
.tG /n .G/ D .sE.P .G// /n .G/ D 'n ..sE.P .G// /N .'n1 .G///  'n .'n1 .G// D G;
i.e., the map tG and the family P.G/ witness that (c) holds, so we proved that
(a)H)(c) and hence our solution is complete.
V.479. Given a monotonically retractable space X , prove that
(a) every F -subset of X is monotonically retractable;
(b) the space X is Sokolov; in particular, X is !-stable, collectionwise normal and
ext .X /  !;
(c) Cp .X / is a Lindelf D-space.
Solution. (a) Suppose that Y is an F -subset
Sof X and let F be a countable family
of closed subsets of X such that Y D
F. Apply Problem 478 to assign to
any countable family G of closed subsets of X a retraction sG W X ! X and a
countable network O.G/ for sG such that sG .G/  G for any G 2 G and the
assignment O is !-monotone.
For any countable set A  Y let G.A/ D F [ ffxg W x 2 Ag. It is clear
that the assignment A ! G.A/ is !-monotone; observe first that sG.A/ .Y /  Y .
Indeed, if y 2 Y then y 2 F for some F 2 F  G.A/, so sG.A/ .F /  F which
implies that sG.A/ .y/ 2 F  Y . Therefore rA D sG.A/ jY W Y ! Y is a retraction.

2 Solutions of problems 001500

567

Since O.G.A// is a network for sG.A/ , the family N .A/ D fP \ Y W P 2 O.G.A//g


is a network for rA and it is immediate that the assignment N is !-monotone.
For any x 2 A it follows from rA .fxg/  fxg that rA .x/ D x; this shows that we
have the inclusion A  rA .Y /, so the map rA and the family N .A/ witness that Y
is monotonically retractable.
(b) Suppose that Fn is a closed subset of X n for any n 2 N. Apply Problem 478(c)
to find a retraction r W X ! X and a countable network N for the map r
such that r n .Fn /  Fn for any n 2 N. It is straightforward that the family
ff .N / W N 2 N g is a countable network in r.X /, so the map r witnesses that
X is a Sokolov space. The rest of the statements of (b) follow from the fact
that every Sokolov space is !-stable, collectionwise normal and has countable
extent by CFS-161 and CFS-163.
(c) Fix a countable base B in the space R; given a family A of subsets of a space Z
let W.A/ D fA1 ; : : : ; An I B1 ; : : : ; Bn
W n 2 N; Ai 2 A and Bi 2 B for every
i  ng; here, as usual, A1 ; : : : ; An I B1 ; : : : ; Bn
D ff 2 Cp .Z/ W f .Ai /  Bi
for any i  ng. If ' W Z ! Z 0 is an onto map, then its dual map '  W Cp .Z 0 / !
Cp .Z/ is defined by the formula '.f / D f ' for any f 2 Cp .Z 0 /.
For any countable set A  X fix a retraction rA W X ! X and a network N .A/
for the map rA that witness monotone retractability of X . Given a set S  X , let
S W Cp .X / ! Cp .S / be the restriction map. Let O.A/ D W.N .A// for any
countable set A  X . By Fact 1 of V.467 and Fact 1 of V.477, the assignment O is
!-monotone.
We will prove the following statement which will imply everything we need:
(1) if N W Cp .X / ! .Cp .X // is a neighborhood assignmentSon Cp .X /, then
there exists a countable closed discrete D  X such that fN.f / W f 2
Dg D Cp .X /.
Take an arbitrary neighborhood assignment N on Cp .X /; there is no loss
of generality to assume that N.f / is a standard open subset of Cp .X / and, in
particular, there exists a finite set Sf  X such that N.f / D S1
Sf .N.f //
f
for any f 2 Cp .X /. For any set B  Cp .X / say that f 2 B is a central point of
B if B  N.f /; denote by Z.B/ the set of all central points of B. It is easy to find
disjoint infinite sets
S fi W i 2 !g such that f0; : : : ; ng  0 [ : : : [ n for every
n 2 ! and ! D n2! n .
Pick a function f0 2 Cp .X / arbitrarily; let A0 D Sf0 and take an enumeration
fPk W k 2 0 g of the family O.A0 /. Proceeding inductively, assume that we have
functions f0 ; : : : ; fn 2 Cp .X / and countable subsets A0 ; : : : ; An of the space X
with the following properties:
(2)
(3)
(4)
(5)

Ai  AiC1 for all i < n;


S
Si g is an enumeration of O.Ai / for every i  n;
Sfi  Ai and fPk W k 2
i g  fN.fj / W j  i g for all i  n;
fZ.Pj / W j <S
fiC1 2 Cp .X /n fN.fj / W j  i g or fiC1 D fi for each i < n.

568

2 Solutions of problems 001500

S
S To construct fnC1 let Un D fN.fi / W i  ng. It follows from (3) and n 2
in i that the set Pn is defined; if Z.Pn /  Un then let fnC1 D fn . This ensures
that (4) holds for all i  n C 1. If Z.Pn /nUn ; then choose fnC1 2 Z.Pn /nUn .
This implies that Z.Pn /  Pn  N.fnC1 /, so (4) now holds for all i  n C 1.
If AnC1 D An [ SfnC1 and fPk W k 2 nC1 g is an enumeration of O.AnC1 /, then
the properties (2)(5) still hold if we replace n by n C 1, so we can construct a set
D D ffi W i 2 !g and a family A S
D fAi W i 2 !g such that the conditions (2)(5)
are satisfied for all n 2
!;
let
A
D
i2! Ai .
S
We will show that fN.fi / W i 2 !g D Cp .X /, so fix any function f 2 Cp .X /.
Observe that g D f rA 2 rA .Cp .Y // where Y D rA .X /. Since N .A/ is a network
for rA , we can apply Fact 2 of V.467 to see that O.A/ is an external network for
rA .Cp .Y // and hence we can find a set P 2 O.A/ such that g 2 P  N.g/ and in
particular, g 2 Z.P /. By (2) and !-monotonicity of O, there exists a number n 2 !
such that P 2 O.AS
n / and hence P D Pk for some k 2 n . The property (4) shows
that g 2 Z.Pk /  fN.fi / W i 2 !g, so there exists m 2 ! such that g 2 N.fm /.
It follows from (2) and (3) that Sfm  Am  A  Y . Since rA .x/ D x for any
x 2 A, we have the equality f jSfm S
D gjSfm , so it follows from g 2 N.fm / that we
have the inclusions f 2 N.fm /  S fN.fi / W i 2 !g. The point f 2 Cp .X / was
taken arbitrarily, so we proved that fN.fi / W i 2 !g D Cp .X /.
Given any f 2 Cp .X / take n 2 ! such that f 2 N.fn /; by the property (5) we
have N.fn /\D  ff0 ; : : : ; fn g and hence every point of Cp .X / has a neighborhood
whose intersection with D is finite. This shows that D is a closed discrete subset of
Cp .X / and finishes the proof of (1).
It is clear that (1) implies that Cp .X / is a D-space. Given any open cover U
of the space Cp .X / take a set Uf 2 U such that f 2 Uf for any f 2 Cp .X /.
Apply (1) to the neighborhood
S assignment fUf W f 2 Cp .X /g to find a countable
set D  Cp .X / such that fUf W f 2 Dg D Cp .X /. Since fUf W f 2 Dg is
a countable subcover of U , we proved that Cp .X / is Lindelf. This settles (c) and
makes our solution complete.
V.480. Prove that
(a) any  -product of monotonically retractable spaces is monotonically
retractable;
(b) any -product of monotonically retractable spaces is monotonically
retractable and hence every countable product of monotonically retractable
spaces is monotonically retractable;
(c) any closed subspace of a -product of cosmic spaces is monotonically
retractable.
Solution. If Z is a set, then Z
! is the family of all countable subsets of Z.
Suppose that in a space Xt , for any countable set A  Xt we have a retraction rAt W
Xt ! Xt and a network Nt .A/ for the mapQrAt that witness monotone retractability
of Xt for any t 2 T . Fix a point a 2 X D t2T Xt . We must prove that the spaces

2 Solutions of problems 001500

569

.X; a/ D fx 2 X W the set supp.x/ D ft 2 T W x.t / a.t /g is countableg and


 .X; a/ D fx 2 X W supp.x/ is finiteg are both monotonically retractable.
Fact 1. Suppose that ht W Xt ! Xt is a continuous retraction such that ht .a.t // D
a.t / for every t 2 T and we have a set E  T . For any x 2 X let s.x/.t / D
ht .x.t // for each t 2 E and s.x/.t / D a.t / whenever t 2 T nE. Then
(i) s W X ! X is a continuous retraction;
(ii) s..X; a//  .X; a/;
(iii) s. .X; a//   .X; a/.
Proof. Let pt W X ! Xt be the natural projection for every t 2 T . Assume first
that t 2 T nE; then pt .s.x// D s.x/.t / D a.t / for any x 2 X and hence pt s is
continuous being a constant function. If t 2 E, then pt .s.x// D s.x/.t / D ht .x.t //
for any x 2 X , so pt s D ht pt is also a continuous function. This proves
that the map s is continuous (see TFS-102). To see that s is a retraction, take any
y 2 s.X / and fix x 2 X such that y D s.x/. If t 2 E, then s.y/.t / D ht .y.t // D
ht .ht .x.t /// D ht .x.t // D y.t /. If t 2 T nE, then s.y/.t / D a.t / D s.x/.t / D
y.t /. Therefore s.y/ D y for any y 2 s.X / and hence s is a continuous retraction;
this settles (i).
Now take any point x 2 X and t 2 T nsupp.x/. If t 2 E, then the equalities
s.x/.t / D ht .x.t // D ht .a.t // D a.t / show that t supp.s.x//. If t E, then
s.x/.t / D a.t / and again t supp.s.x//. This shows that supp.s.x//  supp.x/
for any x 2 X ; this immediately implies (ii) and (iii) and shows that Fact 1 is proved.
Suppose that A  X is a countable set and E  T . Define a map rA; E
W
X ! X by the formula rA; E
.x/.t / D rAt t .x.t // if t 2 E; here At D pt .A [
fag/. If t 2 T nE, then let rA; E
.x/.t / D a.t /. Observe that a.t / 2 At and
hence rAt t .a.t // D a.t / for any t 2 T , so it follows from Fact 1 that rA; E
is
a continuous retraction such that rA; E
..X; a//  .X; a/. Call a set Y 
.X; a/ monotonically
invariant if we can assign to any countable set A  Y a
S
subset E.A/  fsupp.x/ W x 2 Ag of the set T such that rA; E.A/
.Y / 
Y and the assignment E is !-monotone. Observe that it follows from Fact 1 that
rA; E.A/
jY W Y ! Y is a continuous retraction whenever Y is a monotonically
invariant subset of .X; a/ and A is a countable subset of Y.
Fact 2. If Y is a monotonically invariant subset of .X; a/, then Y is monotonically retractable.
Proof. Fix a map E W Y
! ! T
! that witnesses monotonical invariance of Y .
Fix a countable set A  Y and let sA D rA; E.A/
jY ; the map sA W Y ! Y is
a continuous retraction by Fact 1. If x 2 A, then x.t / 2 At and hence sA .x/.t / D
rAt t .x.t // D x.t / for any t 2 E.A/. If t 2 T nE.A/ then x.t / D a.t / because
supp.x/  E.A/. This implies that sA .x/.t / D a.t / D x.t /, so sA .x/.t / D x.t /
for any t 2 T , i.e., sA .x/ D x for any x 2 A.

570

2 Solutions of problems 001500

Q
Say that a set Q  X is A-standard if Q D t2T Qt and there is a finite
F  E.A/ such that Qt 2 Nt .At / for any t 2 F and Qt D Xt for all t 2 T nF .
It is easy to see that the family N .A/ D fQ \ Y W Q is an A-standard setg is
countable; we will show that N .A/ is a network for sA .
Q Suppose that x 2 Y and y D sA .x/ 2 U 2 .Y /. There exists a set V D
t2T Vt such that Vt 2 .Xt / for all t 2 T while Vt D Xt if t 2 T nF for some
finite set F  T and y 2 V \ Y  U . If t 2 K D F \ E.A/, then it follows
from y.t / D rAt t .x.t // 2 Vt that we can find Qt 2 Nt .At / such that x.t / 2 Qt
and rAtQ
.Qt /  Vt . Letting Qt D Xt for any t 2 T nK, we obtain an A-standard set
t
Q D t2T Qt . It follows from x 2 Q that x 2 Q0 D Q \ Y 2 N .A/.
Take any z 2 Q0 ; if t 2 K, then sA .z/.t / D rAt t .z.t //  rAt t .Qt /  Vt . If
t 2 F nK, then sA .z/.t / D a.t / D sA .x/.t / D y.t / 2 Vt . Therefore sA .z/.t / 2 Vt
for all t 2 F which implies sA .z/ 2 V \ Y  U for each z 2 Q0 , i.e., sA .Q0 /  U .
This proves that N .A/ is a network for the map sA .
Let us show that the operator N is !-monotone. It is evident that the family
N .A/ is countable for any countable A  Y . If A  A0 , then At  A0t and hence
Nt .At /  Nt .A0t / for any t 2 T . Since also E.A/  E.A0 /, every A-standard set
is A0 -standard, so N .A/  N .A0 /. Finally,S
if fAn W n 2 !g is a non-decreasing
family of countable
subsets
of
Y
and
A
D
n2! An , then it suffices to show that
S
N .A/  Q n2! N .An /, so take any set Q0 2 N .A/. There exists an A-standard
set Q D t2T Qt such that Q0 D Q \ Y . Let F  T be a finite set such that
Qt 2 Nt .At / for any t 2 F and Qt D Xt whenever t 2 T nF . By monotonicity of
every Q
Nt , we can find n 2 ! such that Qt 2 Nt ..An /t / for every t 2 F and hence
Q D t2T Qt is an An -standard set, i.e., Q0 2 N .An /. Therefore we have verified
that the retractions sA and the families N .A/ witness monotone retractability of the
space Y and hence Fact 2 is proved.
Returning to our solution
S let Y be one of the spaces .X; a/ or  .X; a/. Observe
that if we let E.A/ D fsupp.x/ W x 2 Ag for any countable set A  Y , then the
assignment E is !-monotone. Since also rA; E.A/
.Y /  Y by Fact 1, we proved
that both .X; a/ and  .X; a/ are monotonically invariant in .X; a/, so they are
monotonically retractable by Fact 2. This settles (a) and (b).
(c) Assume that nw.Xt / D ! for all t 2 T and let Y be a closed subset of .X; a/.
Fix a countable network Et in the space Xt for every t 2 T . If, for any countable
set A  X , we let Nt .A/ D Et and rAt .x/ D x for all x 2 Xt , then the maps
rAt and the families Nt .A/ witness monotone retractability of the space Xt for
every t 2 T .
Taking in consideration the structure of the maps rAt , we can see that for any
A  Y and B  T , the map rA; B
W X ! X acts as follows: rA; B
.x/.t / D
x.t / if x 2 B and rA;S
B
.x/.t / D a.t / whenever t 2 T nB. For every countable
set A  Y let S.A/ D fsupp.x/ W x 2 Ag. It is immediate that the assignment S
is !-monotone.
Q
Say that a set Q D t2T Qt is canonical, if Q \ Y ;, the set H.Q/ D ft 2
T W Qt Xt g is finite and Qt 2 Et for every t 2 H.Q/. For each canonical set

2 Solutions of problems 001500

571

Q pick a point xQ 2 Q \ Y and let D.B/ D fxQ W Q is a canonical set and


H.Q/  Bg for any countable B  T . Again, it is an easy exercise to prove that
the operator D is !-monotone.
For any countable set A  Y , consider the sets E0S
.A/ D S.A/ and EnC1 .A/ D
En .A/ [ S.D.En .A/// for any n 2 !; let E.A/ D n2! En .A/. Using Fact 1(a)
of V.477 it takes a simple induction to prove that every operator En is !-monotone.
Therefore the assignment A ! E.A/ is also !-monotone by Fact 1(b) of V.477.
Fix
S any countable set A  Y and let B D E.A/. If y 2 D.B/ D
D. n2! En .A//, then y 2 D.En .A// for some n 2 ! by monotonicity of the
operator D. This implies that supp.y/  S.D.En .A///  EnC1 .A/  E.A/ D B;
by definition of the map rA; B
we have rA; B
.y/ D y and hence we proved that
(1) rA; B
.y/ D y for any y 2 D.B/.
Now take any point y 2 rA; B
.Y / and aQset U 2 .y; X /; pick x 2 Y such
that y D rA; B
.x/. There exists a set V D t2T Vt such that Vt 2 .Xt / for all
t 2 T while Vt D Xt if t 2 T nF for some finite set F  T and y 2 V  U . Let
K D F \ B; the family Et being a network in Xt , we can find a set Qt 2 Et such
that y.t / 2 Q
Qt  Vt for any t 2 K. Let Qt D Xt for all t 2 T nK and consider
the set Q D t2T Qt . Since K  B, it follows from the definition of rA; B
that
x.t / D y.t / 2 Qt for every t 2 K; therefore x 2 Q \YSand hence Q is a canonical
set. The sequence fEn .A/ W n 2 !g is increasing and n2! En .A/ D B  K, so
there exists a number n 2 ! such that H.Q/  K  En .A/. As an immediate
consequence, xQ 2 D.En .A//  D.B/. Since also xQ 2 Q  V  U , we
proved that D.B/ \ U ; for any set U 2 .y; X / and hence y 2 D.B/ for
any point y 2 rA; B
.Y /. Therefore we can apply the property (1) to see that
rA; B
.D.B// D D.B/  Y is dense in rA; B
.Y /. Since the set Y is closed
in .X; a/ and we have the inclusion rA; B
.Y /  .X; a/ by Fact 1(ii), we
conclude that rA; B
.Y /  cl.X;a/ .D.B//  Y and hence we proved that Y is
a monotonically invariant subset of .X; a/. Finally apply Fact 2 to conclude that
Y is monotonically retractable; this finishes the proof of (c) and makes our solution
complete.
V.481. For any space X prove that
(a) X is monotonically retractable if and only if Cp .X / is monotonically Sokolov.
(b) X is monotonically Sokolov if and only if Cp .X / is monotonically retractable.
Solution. Fix a countable base B in the space R; given a family A of subsets of a
space Z, let W.A/ D fA1 ; : : : ; An I B1 ; : : : ; Bn
W n 2 N; Ai 2 A and Bi 2 B for
every i  ng; here, as usual, A1 ; : : : ; An I B1 ; : : : ; Bn
D ff 2 Cp .Z/ W f .Ai / 
Bi for any i  ng. If ' W Z ! Z 0 is an onto map, then its dual map '  W Cp .Z 0 / !
Cp .Z/ is defined by the formula '.f / D f ' for any f 2 Cp .Z 0 /. For any
space
Z denote by CL.Z/ the family of all closed subsets of Z and let CL .Z/ D
S
fCL.Z n / W n 2 Ng.
Recall that a subset Y of a space Z is a retract of Z if there exists a continuous
onto map r W Z ! Y (called a retraction) such that r.z/ D z for any z 2 Y .

572

2 Solutions of problems 001500

However, it is equivalent to say that a continuous map r W Z ! Z is a retraction if


r r D r, so we will often consider that a retraction on Z is a map from Z to Z. To
make the perception easier for the readers used to another definitions of retraction,
we will often say that a retraction is continuous neglecting the fact that in this book,
continuity is a part of the definition of retraction.
Fact 1. Suppose that Z is a space and F is a retract of Z. If r W Z ! F is a
continuous retraction, then s D r  F W Cp .Z/ ! r  .Cp .F // is also a continuous
retraction.
Proof. It is clear that the map s is continuous. The set F must be C -embedded in Z
by Fact 1 of S.398, so F .Cp .Z// D Cp .F / and hence s.Cp .Z// D r  .Cp .F //.
Take any function f 2 s.Cp .Z//; there exists g 2 Cp .F / such that f D g r. It is
easy to see that f jF D g and hence s.f / D .f jF / r D g r D f , so s.f / D f
for any f 2 r  .Cp .F //, i.e., s is a retraction and hence Fact 1 is proved.
Fact 2. Suppose that f W Z ! Y is a continuous onto map and U is a base of Y .
Assume that, for the family F D ff 1 .U / W U 2 U g, there exists a continuous map
r W Z ! Z such that r.F /  F for any F 2 F. Then f D .f jr.Z// r.
Proof. Take any point x 2 Z. Given a set U 2 U such that f .x/ 2 U observe that
x 2 F D f 1 .U / 2 F. By our hypothesis we have r.x/ 2 F , so f .r.x// 2 U .
Thus f .r.x// 2 U for any U 2 U with f .x/ 2 U and hence f .x/ D f .r.x// for
any x 2 X , i.e., Fact 2 is proved.
Fact 3. If a space Z is monotonically retractable, then Cp .Z/ is monotonically
Sokolov.
Proof. Apply Problem 478 to assign to any countable family G  CL .Z/ a
continuous retraction tG W Z ! Z and a countable network P.G/ for the map
tG such that, for any n 2 N and G 2 G \ CL.Z n / we have .tG /n .G/  G and the
assignment P is !-monotone.
Let F be a countable family of closed subsets of Cp .Z/. Given any F 2 F, for
any n 2 N and B1 ; : : : ; Bn 2 B consider the subset
G.F; B1 ; : : : ; Bn / D f.x1 ; : : : ; xn / 2 Z n W x1 ; : : : ; xn I B1 ; : : : ; Bn
\ F D ;g
of the space Z n . Take any z D .z1 ; : : : ; zn / 2 Z n nG.F; B1 ; : : : ; Bn /. There exists
a function f 2 z1 ; : : : ; zn I B1 ; : : : ; Bn
\ F ; since Wi D f 1 .Bi / is an open
neighborhood of zi for any i  n, the set W D W1  : : :  Wn is an open
neighborhood of z such that W \ G.F; B1 ; : : : ; Bn / D ;. This proves that every
set G.F; B1 ; : : : ; Bn / is closed in Z n and therefore G.F/ D fG.F; B1 ; : : : ; Bn / W
F 2 F; B1 ; : : : ; Bn 2 B; and n 2 Ng is a countable subfamily of CL .Z/.
The assignment F ! G.F/ is easily seen to be !-monotone. Consider the set
M.F/ D tG.F / .Z/; we will prove that the retraction rF D .tG.F / / M.F / and the
family N .F/ D W.P.G.F/// witness that Cp .Z/ is monotonically Sokolov.
By Fact 1 the map rF W Cp .Z/ ! Cp .Z/ is a retraction. The family P.G.F//
being a network for the map tG.F / , we can apply Fact 2 of V.467 to see that the

2 Solutions of problems 001500

573

family W.P.G.F/// is an external network for .tG.F / / .Cp .M.F/// in Cp .Z/.


This is the same as saying that the family W.P.G.F/// is an external network for
the set .tG.F / / M.F / .Cp .Z// in Cp .Z/, i.e., N .F/ is an external network for
rF .Cp .Z// in Cp .Z/. Since the assignments W, P and G are !-monotone (see
Fact 1 of V. 467), the assignment N is !-monotone by Fact 1 of V.477. Now, to
finish the proof, we only need to show that rF .F /  F for each F 2 F.
Suppose that rF .f / 62 F for some set F 2 F and f 2 F . Since F is closed in
the space Cp .Z/, we can choose x1 ; : : : ; xn 2 Z and B1 ; : : : ; Bn 2 B such that the
set U D x1 ; : : : ; xn I B1 ; : : : ; Bn
contains rF .f / and U \ F D ;. This shows that
.x1 ; : : : ; xn / 2 G.F; B1 ; : : : ; Bn / 2 G.F/ \ CL.Z n /. Since .tG.F / /n .G/  G for
any G 2 G.F / \ CL.Z n /, we have .tG.F / /n .x1 ; : : : ; xn / 2 G.F; B1 ; : : : ; Bn /. As
a consequence, V \ F D ; if V D tG.F / .x1 /; : : : ; tG.F / .xn /I B1 ; : : : ; Bn
. On the
other hand, rF .f / 2 U implies rF .f /.xk / D f .tG.F / .xk // 2 Bk for k D 1; : : : ; n,
i.e., f 2 V \ F ; this contradiction shows that rF .F /  F for each F 2 F and
hence Fact 3 is proved.
Fact 4. If a space Z is monotonically Sokolov, then Cp .Z/ is monotonically
retractable
Proof. We can assign to any countable F  CL.Z/ a retraction rF W Z ! Z and
a countable external network N .F/ for rF .Z/, such that rF .F /  F for every
F 2 F and the assignment N is !-monotone. For any countable set E  Cp .Z/
consider the family F.E/ D ff 1 .B/ W f 2 E and B 2 Bg. The assignment
E ! F.E/ is easily seen to be !-monotone; denote by K.E/ the set rF .E/ .Z/. It
turns out that the map sE D .rF .E/ / K.E/ and the family O.E/ D W.N .F.E///
witness that Cp .Z/ is monotonically retractable.
It follows from Fact 1 that sE D .rF .E/ / K.E/ W Cp .Z/ ! Cp .Z/ is a
retraction. Since the family N .F.E// is an external network for K.E/ in Z, Fact 3
of V.467 shows that the family W.N .F.E/// is a network for the map K.E/ . It
follows from continuity of .rF .E/ / that W.N .F.E/// is a network of .rF .E/ /
K.E/ , i.e., the family O.E/ is a network for the map sE . All assignments W, N ,
and F being !-monotone, the assignment O is !-monotone as well.
To convince ourselves that E  sE .Cp .Z// take any f 2 E and observe that
ff 1 .B/ W B 2 Bg  F.E/. By the choice of rF .E/ , we have rF .E/ .F /  F for
any F 2 F.E/. Therefore we can apply Fact 2 to conclude that
f D .f j K.E// rF .E/ D .rF .E/ / K.E/ .f / D sE .f / 2 sE .Cp .Z//;
i.e., the space Cp .Z/ is monotonically retractable and hence Fact 4 is proved.
Returning to our solution observe that if X is a monotonically retractable space,
then Cp .X / is monotonically Sokolov by Fact 3. Now, if Cp .X / is monotonically
Sokolov, then Cp .Cp .X // is monotonically retractable by Fact 4. Being embeddable
in Cp .Cp .X // as a closed subspace (see TFS-167), the space X must be monotonically retractable by Problem 479, so we proved (a).

574

2 Solutions of problems 001500

If X is monotonically Sokolov, then Cp .X / is monotonically retractable by


Fact 4. If, on the other hand, the space Cp .X / is monotonically retractable, then
Cp .Cp .X // is monotonically Sokolov by Fact 3. By TFS-167 the space X embeds
in Cp .Cp .X // as a closed subspace; applying Problem 476 we can see that X is
monotonically Sokolov. This settles (b) and makes our solution complete.
V.482. Prove that
(a) any monotonically Sokolov space is Lindelf and Sokolov;
(b) any countable product of monotonically Sokolov spaces is monotonically
Sokolov;
(c) every R-quotient image of a monotonically Sokolov space is monotonically
Sokolov;
(d) every R-quotient image of a monotonically retractable space X must be
monotonically retractable;
(e) A monotonically retractable space must be !-monolithic but it can fail to be
monotonically !-monolithic.
Solution. (a) If X is monotonically Sokolov, then Cp .X / is monotonically
retractable by Problem 481, so it is Sokolov by Problem 479. Therefore X
is a Sokolov space by CFS-156. Besides, Cp .Cp .X // must be Lindelf by
Problem 479, so X is Lindelf being homeomorphic to a closed subset of
Cp .Cp .X //.
(b) We will first prove that X ! is monotonically Sokolov if X is monotonically
Sokolov; it is clear that we can assume that X is infinite. In this case the space
Cp .X / is monotonically retractable and hence Cp .Cp .X // is monotonically
Sokolov by Problem 481. It follows from TFS-177 that the space Y D
Cp .Cp .X // is homeomorphic to Y ! . The space X is homeomorphic to a closed
subspace of Y and therefore X ! is also homeomorphic to a closed subspace of
Y ' Y ! , so it follows from Problem 476 that X ! is monotonically Sokolov.
Now assume L
that Xi is monotonically Sokolov for any i 2 !. Then the
space X D
fXi W i 2 !g is also monotonically Sokolov because every
.X
/
is
monotonically
retractable by Problem 481 and hence Cp .X / '
C
Qp i
fCp .Xi / W i 2 !g is also monotonically retractable
by Problem 480. Thus X !
Q
is monotonically Sokolov and hence so is fXi W i 2 !g being homeomorphic
to a closed subspace of X ! .
(c) If Y is an R-quotient image of a monotonically Sokolov space X then
Cp .Y / embeds in Cp .X / as a closed subspace by TFS-163. The space
Cp .X / is monotonically retractable by Problem 481 and hence so is Cp .Y /
by Problem 479. Applying Problem 481 once more we conclude that Y is
monotonically Sokolov.
(d) If Y is an R-quotient image of a monotonically retractable space X then Cp .Y /
embeds in Cp .X / as a closed subspace by TFS-163. The space Cp .X / is
monotonically Sokolov by Problem 481 and hence so is Cp .Y / by Problem 476.
Applying Problem 481 again we conclude that Y is monotonically retractable.

2 Solutions of problems 001500

575

(e) If X is monotonically retractable, then it is Sokolov by Problem 479. Every


Sokolov space is !-monolithic by CFS-163 so X must be !-monolithic. Now,
take a Corson compact space K which is not monotonically !-monolithic.
Such a space exists by Problem 474. Since K embeds in a -product of real
lines, it is monotonically retractable by Problem 480. Therefore K is a compact
monotonically retractable space which is not monotonically !-monolithic.
V.483. Prove that a compact space X is monotonically retractable if and only if X
is Corson compact.
Solution. For any space Z we denote by CL.Z/ the family of all closed subsets of
Z. Recall that the Gruenhages W -game is played on a nonempty closed subset F
of a space Z as follows: at the n-th move the first player, called O, chooses an open
set Un 2 .F; Z/ and the player P responds by taking a point xn 2 Un . After !many moves have been made, the player O wins if the sequence S D fxn W n 2 !g
converges to F , i.e., the set fn 2 ! W xn V g is finite for any V 2 .F; Z/;
otherwise P is the winner.
Fact 1. If Z is a compact monotonically retractable space and F ; is a closed
subset of Z, then the player O has a winning strategy in the W -game on F .
Proof. Apply Problem 478 to assign to any countable family F  CL.Z/ a
continuous retraction rF W Z ! Z and a network N .F/ for the map rF such
that rF .G/  G for any G 2 F and the assignment N is !-monotone. Observe
first that the family N 0 .F/ D fP W P 2 N .F/g is still a network for the map rF
and the assignment N 0 is also !-monotone so, passing to N 0 if necessary, we can
consider, without loss of generality, that every element of N .F/ is closed in Z.
To start defining a strategy s for the player O in the W -game on F let s.;/ D Z
and F0 D fF g. Proceeding by induction assume that n 2 ! and we have moves
U0 ; a0 ; : : : ; Un1 ; an1 ; Un in the W -game on F and countable families F0 ; : : : ; Fn
of closed subsets of Z with the following properties:
(1)
(2)
(3)
(4)
(5)

an enumeration fQi;k W k 2 !g is chosen for the family Fi for each i  n;


U0 D s.;/ D Z and ai 2 Ui for all i < n;
U iC1  Ui for all i < n;
FiC1 D Fi [ N .Fi [ ffa0 g; : : : ; fai gg/ whenever i < n;
if max.j; k/  i < n and Qj;k \ F D ;, then U iC1 \ Qj;k D ;.

After the player P picks a point an 2 Un , define a family FnC1 by the formula
FnC1 D Fn [ N .Fn [ ffa0 g; : : : ; fan gg/ and choose an enumeration fQnC1;k W
k 2 !g of FnC1 .SThe family G D fQj;k W max.j; k/  n and Qj;k \ F D ;g is
finite, so the set G is closed and does not meet F . By normality of the
S space Z
we can find a set UnC1 2 .F; Z/ such that U nC1  Un and U nC1 \ . G/ D ;.
Let s.a0 ; : : : ; an / D UnC1 and observe that the conditions (1)(5) still hold if we
replace n with n C 1, so we completed the definition of a strategy s such that (1)(5)
hold for all n 2 ! whenever the player O applies s.
Suppose that P D fUn ; an W n 2 !g is a play on F where O applies s and hence
we have a collection fFn W n 2 !g of families with the properties (1)(5). To see

576

2 Solutions of problems 001500

that the O wins in the play P assume that the sequence S D fan W n 2 !g does
not converge to F ; by compactness of Z there exists a cluster point p F
S for the
sequence S . Observe that the family ffan g W n 2 !g is contained in F D n2! Fn
by (4), and hence rF .an / D an for each n 2 !. It follows from p 2 S and continuity
of the retraction rF that rF .p/ D p 2 ZnF . Since N .F/ is a network for rF , there
exists N 2 N .F/ such that p 2 N and rF .N / \ F D ;. The operator N being
!-monotone, we can find n 2 ! such that N 2 Fn and hence N D Qn;k for
some k 2 !. Besides, N 2 FnC1 by (4) and hence N 2 F which implies that
rF .N /  N . Since also F 2 F, we have rF .F /  F ; if x 2 F \ N , then
rF .x/ 2 rF .N / \ F D ; which is a contradiction. Therefore N \ F D ;.
Now, if we consider the number m D max.n; k/ C 1, then (5) can be applied to
the set N D Qn;k to see that N \ U m D ;. By the properties (2) and (3) we have
fai W i  mg  Um and therefore G D .ZnU m / is an open neighborhood of the
point p 2 N such that G \ S  fa0 ; : : : ; am1 g which implies that p is not a cluster
point of S ; this contradiction shows that the sequence S converges to F , so s is a
winning strategy for the player O and hence Fact 1 is proved.
Returning to our solution observe that if X is Corson compact, then it embeds in
a -product of real lines, so it is monotonically retractable by Problem 480(c); this
proves sufficiency.
Finally, assume that X is a compact monotonically retractable space. By
Problem 480(b) the space X  X is also monotonically retractable, so we can apply
Fact 1 to conclude that for every nonempty closed subset F of X  X , the player O
has a winning strategy in the Gruenhages W -game on F . Therefore X is Corson
compact by Problem CFS-188 and hence our solution is complete.
V.484. Prove that a monotonically retractable space X is monotonically Sokolov
if and only if X is monotonically !-monolithic. In particular, a compact space is
monotonically Sokolov if and only if it is monotonically !-monolithic.
Solution. For any space Z we denote by CL.Z/ the family of all closed subsets
of Z. Any monotonically Sokolov space is monotonically !-monolithic by Problem 476, so necessity holds trivially.
To prove sufficiency assume that X is a monotonically retractable monotonically
!-monolithic space and apply Problem 478 to assign to any countable family F of
closed subsets of X a continuous retraction rF W X ! X and a network N .F/
for the map rF such that rF .G/  G for any G 2 F and the assignment N is
!-monotone. Observe first that the family N 0 .F/ D fP W P 2 N .F/g is still
a network for the map rF and the assignment N 0 is also !-monotone so, passing
to N 0 if necessary, we can consider, without loss of generality, that every element
of N .F/ is closed in X . By monotone !-monolithity of X , to any countable set
A  X we can assign an external network M.A/ for the set A in such a way that
the operator M is !-monotone. For any nonempty closed subset F of the space X
fix a point xF 2 F .

2 Solutions of problems 001500

577

Given a countable G  CL.X / consider the family P.G/ D ffxG g W G 2


Gnf;gg; it is straightforward that the operator P is !-monotone. For any countable
family F  CL.X / let O0 .F/ D F [ P.F/ and
(1) OnC1 .F/ D On .F/ [ P.On .F// [ N .On .F/ [ P.On .F///
for any n 2 !. It takes a trivial induction together S
with Fact 1(a) of V.477 to see that
every operator On is !-monotone. Let O.F/ D n2! On .F/ and apply Fact 1(b)
of V.477 to see that the operator O is also !-monotone.
Letting M.F/ D fxF W F 2 O.F/nf;gg we also obtain an !-monotone operator
which assigns a countable set to any countable family of closed subsets of X . Given
a nonempty set F 2 O.F/, there exists a number n 2 ! such that F 2 On .F/ and
hence we have the inclusions fxF g 2 OnC1 .F/  O.F/. This proves that
(2) fxF g 2 O.F/ and hence rO.F / .xF / D xF for any F 2 O.F/nf;g.
Now, if F 2 N .O.F// then F 2 N .On .F// by !-monotonicity of N . Then
F 2 OnC1 .F/  O.F/ which implies that
(3) N .O.F//  O.F/ and hence M.F/ is dense in rO.F / .X /.
Indeed, we must only prove the second part of the statement (3), so observe that
the family N .O.F// is a network for the map rO.F / and hence for any set Q  X
such that Q \ N ; for any nonempty set N 2 N .O.F//, the set rO.F / .Q/ is
dense in rO.F / .X /. Now it follows from (2) and (3) that M.F/ is dense in rO.F / .X /.
Therefore the family E.F/ D M.M.F// is an external network of the set rO.F / .X /
and it follows from F  O.F/ that rO.F / .F /  F for any F 2 F. This shows that
the !-monotone operator E and the retraction sF D rO.F / witness the monotone
Sokolov property of X , so we proved sufficiency.
Finally observe that if X is an !-monolithic compact space, then X is Corson
compact by Problem 473. Therefore X is monotonically retractable by Problem 483
and hence X is monotonically Sokolov, i.e., our solution is complete.
V.485. Suppose that X is a first countable countably compact subspace of an
ordinal with its order topology. Prove that X is monotonically retractable and hence
Cp .X / is a Lindelf D-space.
Solution. Suppose that is an ordinal and Y is a nonempty subset of . Say that
a set A  Y is saturated in Y if nnY 2 A and every isolated point of A is also
isolated in Y . If Y is clear, the set A will be called saturated.
Fact 1. Suppose that is an ordinal and Y  . If a set A  Y is saturated in Y ,
then B D A is also saturated in Y .
Proof. It follows from nn.Y / 2 A  B that nn.Y / 2 B. Now, if y 2 B is isolated
in B, then fyg is open in B, so density of A in B implies that fyg \ A ;, i.e.,
y 2 A and hence y is isolated in A. The set A being saturated, the point y must be
isolated in Y , so Fact 1 is proved.
Fact 2. Suppose that is an ordinal and A  is countable. Then A is also
countable.

578

2 Solutions of problems 001500

Proof. If A is uncountable, then the set P D AnA is also uncountable; consider


the ordinal 0 D nnP . Proceeding by induction assume that  < !1 and we have
chosen ordinals f W <  g  P such that
(1) < 0 <  implies < 0 and the set P \ is countable for any <  .
If  D supf W <  g then P \ . C 1/ is also countable so, for the ordinal
 D nn.P n. C 1//, the property (1) still holds for all   . Therefore our
construction can be continued to construct a set f W < !1 g  P for which the
condition (1) is satisfied for all  < !1 . The set O D . ; C1
is open in for
any < !1 and the family fO W < !1 g is disjoint. It follows from C1 2 A that
O \ A ; for any < !1 , so the set A cannot be countable; this contradiction
shows that Fact 2 is proved.
Fact 3. Suppose that is an ordinal and Y is a countably compact subset of .
Then Y is !-closed in , i.e., A  Y for any countable A  Y (the bar denotes the
closure in ). As a consequence, clY .A/ is compact and countable for any countable
set A  Y .
Proof. The set B D A is countable by Fact 2; since B is the closure of a countably
compact set B \ Y , it has to be pseudocompact, so B is second countable and hence
compact. If x 2 BnY then it follows from x 2 AnA that there exists a nontrivial
sequence S  A which converges to x. It is straightforward that S is an infinite
closed discrete subset of Y ; this contradiction with countable compactness of Y
shows that B  Y , i.e., clY .A/ D A is a countable compact subset of Y and hence
Fact 3 is proved.
Fact 4. Suppose that is an ordinal, Y is a countably compact subset of and F is
a compact saturated subset of Y . For every x 2 Y let rF .x/D maxfy 2 F W y  xg.
Then rF W Y ! F is a continuous retraction.
Proof. Observe first that rF .x/ is consistently defined for every x 2 Y because the
set fy 2 F W y  xg is nonempty, closed in F and hence compact. It is clear from
the definition that rF .x/ D x for any x 2 F , so we only need to prove that the map
rF is continuous.
To do that, fix a point x 2 Y and an open neighborhood U of the point y D rF .x/
in F . Assume first that x F ; then y < x and, for the set V D .y; x
\ Y , we have
rF .V / D fyg  U , so the set V 2 .x; Y / witnesses that rF is continuous at the
point x. Now, if x 2 F then y D x. If x is isolated in F then it is also isolated in Y ,
so the map rF is trivially continuous at x. If x is not isolated in F then there exists
a < y such that .a; y
\ F  U and hence we can find a point z 2 .a; x/ \ F . The
set V D .z; x
\ Y is an open neighborhood of x in Y and rF .V /  z; x
\ F  U ,
so again rF is continuous at x and hence Fact 4 is proved.
Returning to our solution fix an ordinal  and a first countable countably compact
subspace X  ; there is no loss of generality to assume that X is infinite.
From now on, any interval is considered only for the points of X ; in particular,

2 Solutions of problems 001500

579

x; y
D fz 2 X W x  z  yg while x; y/ D fz 2 X W x  z < yg and
x; !/ D fz 2 X W x  zg whenever x; y 2 X and x < y. Let I.X / be the
set of isolated points of X and z0 D nn.X /.
For any countable compact saturated set K  X , the map rK W X ! K is the
retraction defined in Fact 4. If x 2 X nI.X /, then fix a set Sx  I.X / such that
sup.Sx / D x and Sx n.z; x
is finite for any z < x. In other words, Sx  . ; x/ is a
sequence of isolated points
S of X that converges to x. Given a countable set A  X ,
let E.A/ D fz0 g [ A [ fSx W x 2 AnI.X /g. It turns out that
(2) The set E.A/ is saturated for any countable A  X .
To prove the property (2) we must only check that every isolated point of the set
E.A/ isSisolated in X , so take any y 2 E.A/ which is isolated in E.A/. If y D z0
or y 2 fSx W x 2 AnI.X /g  I.X /, then y is certainly isolated in X . The same
is true if y 2 A \ I.X /. Since it follows from x 2 Sx nfxg for every x 2 AnI.X /,
that the points of AnI.X / are not isolated in E.A/, we have no other possibilities
for y and hence (2) is proved.
We leave it to the reader to verify that
(3) the assignment A ! E.A/ is !-monotone.
For any countable set A  X the family
O.A/ D ffxg W x 2 Ag [ fx; !/ W x 2 Ag [ fx; y/ W x; y 2 A and x < yg
is countable and it is again easy to check that the assignment O is !-monotone. This
shows that letting N .A/ D O.E.A// for any countable set A  X , we obtain an
!-monotone assignment A ! N .A/ by Fact 1(a) of V.477. The set K.A/ D
E.A/ is compact and saturated by the property (2), Fact 1 and Fact 3, so
rK.A/ W X ! K.A/ is a continuous retraction by Fact 4. Since A  K.A/, it suffices
to show that
(4) the family N .A/ is a network for the map rK.A/ for any countable set A  X .
To prove (4) take any point x 2 X and a set U 2 .rK.A/ .x/; X /. By density of
E.A/ in K.A/, there exists a point y 2 E.A/ such that y; rK.A/ .x/
 U .
Case 1. We have z  x for any z 2 E.A/. Then x 2 Q D y; !/ 2 N .A/ and
the inclusion rK.A/ .Q/  y; rK.A/ .x/
 U shows that Q 2 N .A/ witnesses that
N .A/ is a network for rK.A/ .
Case 2. There exists z 2 E.A/ such that x < z. Then the point u D nnft 2
E.A/ W x < t g belongs to E.A/ and x 2 Q D y; u/ 2 N .A/; it is easy to check
that again rK.A/ .Q/  y; rK.A/ .x/
 U ; this finishes the proof of (4) and shows
that our space X is indeed, monotonically retractable.
Finally, observe that Cp .X / must be a Lindelf D-space by Problem 479, so our
solution is complete.
V.486. Prove that
(a) the set E of all ordinals < !2 of uncountable cofinality is monotonically
!-monolithic but is neither a D-space nor monotonically !1 -monolithic;

580

2 Solutions of problems 001500

(b) for set Y of all ordinals < !2 of countable cofinality, the space X D Y [
f!2 g is countably compact and ext .Cp .X // D ! while l.Cp .X // D !2 . In
particular, X is a countably compact space such that Cp .X / is not a D-space.
Solution. (a) It is easy to see that if A  E and jAj D !1 then A has an
accumulation point; as a consequence, ext .X /  !.
Fact 1. If Z is a space and every subspace of Z of cardinality   is closed in Z
then Z is monotonically -monolithic.
Proof. If Z 0 is the set Z with the discrete topology and f W Z 0 ! Z is the identity
map then f jA is homeomorphism for any A  Z 0 with jAj  . The space Z 0 being
monotonically monolithic, Fact 1 of V.469 guarantees that Z is monotonically monolithic, so Fact 1 is proved.
Returning to our solution observe that every countable subset of E is closed in
E, so E is monotonically !-monolithic by Fact 1. The family f \ E W < !2 g is
an open cover of E which has no countable subcover, so E is not Lindelf; applying
Problem 470 we conclude that E is not monotonically !1 -monolithic.
Finally let N./ D f 2 E W  g for any 2 E; then N is a neighborhood
assignment on E. If D is a closed discrete subset of E then jDj  ! and hence
the set Z D N.D/ is not cofinal in !2 . The set E being cofinal in !2 , we can find
2 E such that Z, so N.D/ E for any closed discrete D  E which shows
that E is not a D-space.
(b) If A  Y is a countably infinite set then we can choose a strictly increasing
sequence fn W n 2 !g  A. The ordinal D supfn W n 2 !g is countably
cofinal and hence 2 Y ; it is easy to check that is an accumulation point of
A. This shows that X is a countably compact space.
By the definition of the order topology on !2 C 1, the point !2 cannot belong to
the closure of any subset of Y of cardinality less than !2 . Therefore t .X /  !2 and
hence l.Cp .X //  !2 by TFS-189; besides, l.Cp .X //  nw.Cp .X // D nw.X / D
!2 , so l.Cp .X // D !2 .
Fact 2. For any f 2 Cp .Y / there exists 2 Y such that f ./ D f ./ for all
2 Y with  .
Proof. Take any continuous function f W Y ! R. Assume that, for any 2 Y ,
there is an ordinal D ./ > such that 2 Y and f ./ f ./; let us also fix
n D n./ 2 N with jf ./  f ./j  n1 . There exists m 2 N and a set A  Y such
that jAj D !2 and n./ D m for each 2 A. Take any 0 2 A and 0 D .0 /; if
we have i ; i for all i  n, find nC1 2 A with nC1 > maxfi ; i W i  ng and
let nC1 D .nC1 /. This inductive construction gives us sequences fi W i 2 !g
and fi W i 2 !g such that i D .i / and i < iC1 for all i 2 !.
The ordinal D supfi W i 2 !g belongs to Y ; the function f being continuous
1
at the point , there is < such that jf . /  f ./j < 2m
for all  2 .; / \ Y .
Choose n 2 ! with < n < n ; then

2 Solutions of problems 001500

jf .n /  f .n /j  jf .n /  f ./j C jf .n /  f ./j <

581

1
1
1
C
D :
2m
2m
m

This contradiction with jf .n /f .n /j  m1 proves that, for some 2 Y , there will
be no > ; 2 Y with f ./ f ./, i.e., f ./ D f ./ for all  ; 2 Y
and hence Fact 2 is proved.
Returning to our solution let I D ff 2 Cp .X / W f .!2 / D 0g; then Cp .X / '
I  R, so it suffices to show that ext .I /  ! (see Fact 1 of V.242). To do it,
fix any 2 Y and consider the set F D fy 2 X W y > g. Observe that both
sets F and Y D X nF are clopen in X and let P D ff 2 Cp .X / W f .F / D
f0gg; if  W Cp .X / ! Cp .Y / is the restriction map then it is easy to see that
p D  jP W P ! Cp .Y / is an embedding. It is evident that  .P / D Cp .Y /,
so P is homeomorphic to Cp .Y /. Since Y  !2 is a countably compact first
countable subset of !2 , we can apply Problem 485 to convince ourselves that Cp .Y /
is Lindelf. This proves that
(1) the space P is Lindelf for any 2 Y .
Finally, assume that a set D  I is closed, discrete and jDj D !1 . By Fact 2,
for any f 2 D there exists f 2 Y such that f ./ D f .f / D 0 for any 2 Y
with  f . The set Y is cofinal in !2 , so there exists an ordinal 2 Y such that
f < for all f 2 D. As a consequence, D  P and hence D is an uncountable
closed discrete subset of P ; this contradiction with the property (1) shows that
ext .Cp .X // D ext .I / D !. Finally observe that ext .Z/ D l.Z/ for any D-space
Z, so it follows from ext .Cp .X // D ! < l.Cp .X // that Cp .X / is not a D-space.
V.487. Suppose that X and Cp .X / are Lindelf -spaces. Prove that both X and
Cp .X / must be monotonically retractable and monotonically Sokolov. In particular,
X and Cp .X / have the Sokolov property
Solution. Let B be a fixed countable base in R. If Z is a space and x1 ; : : : ; xk 2
Z then as usual, x1 ; : : : ; xk I B1 ; : : : ; Bk
D ff 2 Cp .Z/ W f .xi / 2 Bi for all
i D 1; : : : ; kg whenever B1 ; : : : ; Bk 2 B. If U D x1 ; : : : ; xk I B1 ; : : : ; Bk
, then
supp.U / D fx1 ; : : : ; xk g.
If r W Z ! Y is a continuous map then it dual map r  W Cp .Y / ! Cp .Z/ is
defined by the equality r  .f / D f r for any f 2 Cp .Y /. Given a space Z recall
that M W Cp .Z/ ! Cp .M / is the restriction map for any M  Z; if L  Cp .Z/,
then eL W Z ! Cp .L/ is the reflection map defined by eL .x/.f / D f .x/ for any
f 2 L. If we have a fixed set Y  Cp .Z/ suppose that P  Z and Q  Y ; we
will say that sets M  P and L  Q are .P; Q/-conjugate if M .L/ D M .Q/
and eL .M / D eL .P /; the sets M and L are .P; Q/-preconjugate if M .L/ is dense
in M .Q/ and eL .M / is dense in eL .P /. If no confusion is possible, then .Z; Y /conjugate sets will be called conjugate and .Z; Y /-preconjugate sets will be simply
called preconjugate. The following statement was proved as Fact 2 in U.285.
Lemma 1. Suppose that Z is a space and a set Y  Cp .Z/ generates the topology
of Z. Assume also that M  X and L  Y are conjugate sets. Then the mappings

582

2 Solutions of problems 001500

u D eL jM W M ! eL .M / and v D M jL W L ! M .L/ are homeomorphisms;


besides, r D u1 eL W X ! M and q D v 1 M W Y ! L are continuous
retractions such that q D r  .M jY /. The maps r and q are called the pair of
retractions corresponding to the conjugate pair .M; L/.
We will also need the lemma given below; it was proved as Fact 7 in U. 285.
Lemma 2. Suppose that X is a Lindelf -space and a Lindelf -space Y 
Cp .X / generates the topology of X . Assume additionally that some countable
families P  exp.X / and Q  exp.Y / are closed under finite intersections and
finite unions and there exist compact covers K and C of the spaces X and Y
respectively such that P is a network with respect to K and Q is a network with
respect to C. Assume that, for some sets M  X and L  Y the pair .M \P; L\Q/
is .P; Q/-preconjugate for any .P; Q/ 2 P  Q. Then the pair .clX .M /; clY .L//
is conjugate.
The following simple fact is often useful when we consider networks for
mappings.
Fact 1. Suppose that f W Z ! Y is a continuous map and N  exp.Z/ is a
network for f . Then, for any continuous map g W Y ! T , the family N is also a
network for g f .
Proof. Take any x 2 Z and U 2 .g.f .x//; T /; by continuity of g there exists a
set V 2 .f .x/; Y / such that g.V /  U . Since N is a network for f , we can find
a set N 2 N such that x 2 N and f .N /  V . Then .g f /.N /  g.V /  U , so
N is a network for g f and hence Fact 1 is proved.
Our next step is to prove that any countable subset of a space Z can be extended
to a countable set that generates many preconjugate sets.
Fact 2. Suppose that Z is a space and a set Y  Cp .Z/ generates the topology of
Z. Assume also that P  exp.Z/ and Q  exp.Y / are countable families. Then
for any countable set A  Z, it is possible to find countable sets M.A/  Z and
L.A/  Y such that A  M.A/, the pair .M.A/ \ P; L.A/ \ Q/ is .P; Q/preconjugate whenever .P; Q/ 2 P  Q and the assignments A ! M.A/ and
A ! L.A/ are !-monotone.
Proof. Choose first an enumeration f.Pn ; Qn / W n 2 !g of the set P  Q in
which every pair .P; Q/ 2 P  Q occurs infinitely many times. For any n 2 !,
denote by Un the family of all nonempty sets x1 ; : : : ; xk I B1 ; : : : ; Bk
\ Qn , where
x1 ; : : : ; xk 2 Z, B1 ; : : : ; Bk 2 B and k 2 N; we will also need the family Vn
of all nonempty sets f1 ; : : : ; fk I B1 ; : : : ; Bk
\ eY .Pn /, where f1 ; : : : ; fk 2 Y ,
B1 ; : : : ; Bk 2 B and k 2 N. For any n 2 ! and U 2 Un pick a function fU 2 U ;
for each V 2 Vn fix a point xV 2 Pn such that eY .xV / 2 V .
Take a countable set A  Z. We will recursively construct M.A/ and L.A/.
Let M0 .A/ D A and L0 .A/ D ;. Assume that n 2 ! and we have countable sets
M0 .A/; : : : ; Mn .A/  Z and L0 .A/; : : : ; Ln .A/  Y . Let

2 Solutions of problems 001500

583

LnC1 .A/ D Ln .A/ [ ffU W U 2 Un and supp.U /  Mn .A/g; and


MnC1 .A/ D Mn .A/ [ fxV W V 2 Vn and supp.V /  LnC1 .A/g:
Note that MnC1 .A/  Z and LnC1 .A/  Y are countable sets because Mn .A/
and Ln .A/ are countable. So our inductive procedure can be continued to construct
sequencesSfMi .A/ W i 2 !g and fLi .A/ S
W i 2 !g. We will prove that the sets
M.A/ D fMn .A/ W n 2 !g and L.A/ D fLn .A/ W n 2 !g are as promised. Let
us show first that
(1) the assignments A ! M.A/ and A ! L.A/ are !-monotone.
It takes a straightforward induction to see that the sets M.A/ and L.A/ are
countable for any countable set A  X and the assignments A ! Mn .A/ and
A ! Ln .A/ are !-monotone for every n 2 !. Applying
Fact 1(b) of V.477
S
we can see that
the
assignments
A
!
M.A/
D
fM
.A/
W n 2 !g and
n
S
A ! L.A/ D fLn .A/ W n 2 !g are !-monotone and hence we proved (1).
Our last step is to establish that
(2) the pair .M.A/ \ P; L.A/ \ Q/ is .P; Q/-preconjugate for any P 2 P and
Q 2 Q.
To prove (2), take any pair .P; Q/ 2 P Q and consider the sets M D M.A/\P
and L D L.A/ \ Q; we have to establish that the pair .M; L/ is .P; Q/preconjugate. Take any function f 2 Q and W 2 .M .f /; M .Q//. By the
definition of the topology of pointwise convergence there are points x1 ; : : : ; xk 2
M  M.A/ and sets B1 ; : : : ; Bk 2 B such that f 2 U D x1 ; : : : ; xk I B1 ; : : : ; Bk
\
Q  Y and we have M .f / 2 M .U /  W .
It follows from the choice of our enumeration of P  Q and the fact that the
sequence fMi .A/ W i 2 !g is increasing, that there exists a number n 2 !
such that .Pn ; Qn / D .P; Q/ and fx1 ; : : : ; xk g  Mn .A/. Since U 2 Un , it
easily follows from the inclusions fU 2 LnC1 .A/ \ U  L.A/ \ Q D L that
M .fU / 2 M .U /  W . Therefore M .L/ is dense in M .Q/.
To show that the set eL .M / is dense in the space eL .P / fix any point x 2 P
and a set W 2 .eL .x/; eL .P //. There exist functions f1 ; : : : ; fk 2 L  L.A/ and
sets B1 ; : : : ; Bk 2 B such that eY .x/ 2 V D f1 ; : : : ; fk I B1 ; : : : ; Bk
\ eY .P / and
we have the inclusion eL .x/ 2 L .V /  W . It follows from the choice of our
enumeration of P  Q and the fact that the sequence fLi .A/ W i 2 !g is increasing,
that there is n 2 ! for which .Pn ; Qn / D .P; Q/ and ff1 ; : : : ; fk g  Ln .A/.
Then V 2 Vn and it follows from xV 2 MnC1 .A/ \ Pn  M.A/ \ P D M that
eL .xV / D L .eY .xV // 2 L .V /  W . Therefore eL .M / is dense in the space
eL .P /, i.e., we settled (2) and hence Fact 2 is proved.
Fact 3. For any Lindelf -space Z, if Cp .Z/ is also Lindelf , then Z is
monotonically retractable.

584

2 Solutions of problems 001500

Proof. Observe that the set Y D Cp .Z/ generates the topology of Z; choose
countable families P  exp.Z/ and Q  exp.Y / closed under finite intersections
and finite unions which are networks with respect to a compact cover of X and Y
respectively.
It follows from Fact 2 that for any countable A  Z, there exist countable sets
M.A/  Z and L.A/  Y such that A  M.A/ and the pair .M.A/\P; L.A/\Q/
is .P; Q/-preconjugate for any .P; Q/ 2 P  Q while the assignment A ! L.A/ is
!-monotone. If F D clZ .M.A// and G D clY .L.A// then the pair .F; G/ has to be
conjugate by Lemma 2. Applying Lemma 1 we can convince ourselves that the map
uA D eG jF W F ! eG .F / is a homeomorphism and the map rA D .uA /1 eG W
Z ! F is a continuous retraction.
Since Z is a Lindelf -space, the space Cp .Z/ must be monotonically monolithic by Problem 468; applying Problem 467 we conclude that Z is monotonically
stable and, in particular, it is monotonically !-stable. Therefore, to each countable
set L  Y we can assign a network O.L/  exp.Z/ for the map eL in such a way
that the operator O is !-monotone. If A  Z is a countable set, then .uA /1 is a
homeomorphism, so the family O.L.A// is a network for rA by Fact 1. Finally,
let K.A/ D clZ .M.A// and N .A/ D O.L.A//. Then A  K.A/  Z and
rA W Z ! K.A/ is a continuous retraction. Besides, N .A/ is a countable network
for rA and the assignment N is !-monotone; this shows that Z is monotonically
retractable, so Fact 3 is proved.
Returning to our solution, assume that X and Cp .X / are Lindelf -spaces. By
Fact 3, the space X is monotonically retractable and hence Cp .X / is monotonically
Sokolov by Problem 481. Since Cp .Cp .X // is also a Lindelf -space by CFS219, we can apply Fact 3 to the space Z D Cp .X / to conclude that Cp .X / is
monotonically retractable and hence X is monotonically Sokolov by Problem 481.
Finally, apply Problem 479 to see that both spaces X and Cp .X / are Sokolov.
V.488. Prove that
(a) any Eberlein compact space X has a  -closure-preserving local base at every
point, S
i.e., for any x 2 X there exists a local base Bx at the point x such that
Bx D n2! Bxn and every Bxn is closure-preserving;
(b) there exists an Eberlein compact space K that does not have a closurepreserving local base at some point;
(c) there exists a non-metrizable compact space Y such that Cp .Y / has a closurepreserving local base at every point.
Solution. (a) Given a space Z and a closed set F  Z recall that a family U 
.F; Z/ is an outer base of F in Z if for any V 2 .F; X / there is U S
2 U such
that U  V . A family A  exp.Z/ is  -closure-preserving if A D n2! An
and every An is closure-preserving.
Fact 1. If Z is a zero-dimensional Eberlein compact space and F is a closed subset
of Z, then F has a  -closure-preserving clopen outer base in Z.

2 Solutions of problems 001500

585

Proof. The space ZnF must be  -metacompact, i.e., every open cover of ZnF
has a  -point-finite refinement (see CFS-363). For any x 2 ZnF choose a set Ux 2
.x; Z/ with FS
\U x D ;. There exists a refinement U of the cover fUx W x 2 ZnF g
such that U D n2! Un while Un  UnC1 and Un is point-finite for any n 2 !.
Observe that U is compact for any U 2 U , so the cover U can be shrunk, i.e., we
can find, for any U 2 U , a compact set QU  U such that fQU W U 2 U g is still a
cover of ZnF (see Fact 1 of U.188). Using normality of Z it is easy to find a clopen
set GU such that QU  GU  U S
for any U 2 U . Given a compact Q  ZnF there
is a finite V  U such that Q  fGV W V 2 Vg. Consequently, the family W of
all finite intersections of the family fZnGU W U 2 U g is an outer base for the set F
in Z; observe that all elements of W are clopen in Z.
To see that W is  -closure-preserving denote by Wn the family of all finite
intersections of the elements of the family fZnGU W U 2 Un g. UsingSthe fact that
the sequence fUn W n 2 !g is increasing, it is easy to prove that W D n2! Wn , so
it suffices to show that each Wn is closure-preserving.
Suppose that n 2 ! and
S thereSexists a point x 2 Z together with a family
W 0  Wn such that xT2 W 0 n W 0 . The family fGU W U 2 Un g being pointfinite, the set H D fGU W x 2 GU g is open in X . If W 2 W 0 and W D
T
fX nGUi W 1  i  ng then it follows from
S x W that x 2 GUi for some i  n
and hence W \ H D ;. Consequently, . W 0 / \ H D ;; this contradiction with
S
x 2 W 0 shows that every Wn is closure-preserving and hence Fact 1 is proved.
Now assume X is an arbitrary Eberlein compact space and x 2 X . Take a zerodimensional Eberlein compact space Z for which there exists a continuous onto
map f W Z ! X (see CFS-336). Passing to an appropriate closed subset of the
space Z if necessary we can assume, without loss of generality, that the map f is
irreducible, i.e., f .E/ X for any proper closed set E  Z (see TFS-366).
Apply Fact 1 toS
find a clopen outer base U of the set G D f 1 .x/ in the space
Z such that U D n2! Un and each Un is closure-preserving in Z. For every set
U 2 Un let H.U / D X nf .ZnU /; observe that the set H.U / is open in X and
x 2 H.U /. The map f being irreducible, the set H.U / must be dense in f .U / for
any U 2 Un by Fact 1 of S.383.
Since every U 2 Un is compact, the family Hn D ff .U / W U 2 Un g consists
of compact subsets of X ; the fact that the map f is closed easily implies that Hn is
closure-preserving in X . Therefore the family Vn D fH.U / W U 2 Un g 2 .x; X /
is also closure-preserving in X for every n 2 !. It is an easy exercise
that the family
S
V D fH.U / W U 2 U g is a local base at x in X ; since V D n2! Vn , the family V
is  -closure-preserving and hence we settled (a).
(b) Recall that for any set D, the expression D
<! stands for the family of all finite
subsets of D.
Fact 2. Assume that we are given a map  W !1
<! ! !1
<! such that A\.A/ D
<!
; for any A 2 !1
<! . Then thereSexists a disjoint
S family fAn W n 2 !g  !1

such that jAn j ! 1 and the sets n2! An and n2! .An / are disjoint.

586

2 Solutions of problems 001500

Proof. It is easy to find a sequence


fUn W n 2 !g of disjoint families of finite subsets
S
of !1 such that the family n2! Un is disjoint and, for any n 2 ! we have jUn j D !1
and jU j > n for each U 2 Un .
Passing to a smaller family Un if necessary, we can assume, without loss of
generality, that, for every n 2 !, the family f.U / W U 2 Un g is a -system with a
root Dn , i.e., we S
have U \ V D Dn for any distinct U; V 2 Un (see SFFS-038).
The set D D S
n2! Dn being countable we can pass again to a smaller family Un
to guarantee that . Un / \ D D ; for any n 2 !. Take an arbitrary A0 2 U0 and
assume, proceeding inductively, that n 2 ! and we have chosen a set Ai 2 Ui for
every i  n in such a way that the sets Bn D A0 [ : : : [ An and Sn D .A0 / [ : : : [
.An / are disjoint.
The set Bn [ .Sn nD/  !1 nD is finite while both families f.U /nD W U 2
UnC1 g and UnC1 are disjoint, so we can find AnC1 2 UnC1 such that the sets AnC1 [
.AnC1 / and Bn [ .Sn nD/ are disjoint. This shows that our inductive
S procedure
can
be
continued
to
construct
a
family
fA
W
n
2
!g
such
that
the
sets
n
n2! An and
S
.A
/
are
disjoint
and
A
2
U
for
every
n
2
!.
This
implies
that
jA
n
n
n
n j ! 1,
n2!
so fAn W n 2 !g is the promised family, i.e., Fact 2 is proved.
Returning to our solution, recall that A.!1 / is the one-point compactification of
a discrete set of cardinality !1 . Therefore A.!1 / D D [ fag where jDj D !1
and a D is the unique non-isolated point of A.!1 /. The space K D A.!1 /! is
Eberlein compact (see CFS-107, CFS-301 and CFS-307). Denote by p the point of
K all coordinates of which are equal to a; we will prove that K does not have a local
closure-preserving base at p.
To do it, take an arbitrary local base B at the point p. Given any finite E  D
let n D jEj and consider the set O.E/ D fx 2 K W x.i / E for any i  ng. It is
clear that every O.E/ is an open subset of K and, besides, the family fO.E/ W E 2
D
<! g is a local base of K at p. As a consequence, for any E 2 D
<! there exist
sets B.E/ 2 B and .E/ 2 D
<! such that .E/ \ E D ; and O.E [ .E// 
B.E/  O.E/.
Apply Fact 2 to find a disjoint family
fEn W n 2 !gSof finite subsets of D such
S
that jEn j ! 1 while the sets E D n2! En and S D n2! .En / are disjoint. We
can consider that jEn j > n for any n 2 !.
For each n 2 ! pick a point zn 2 En and define a point q 2 K by q.n/ D zn for
any n 2 !. We will also need the point qn 2 K such that qn .i / D zi for all i  n
and qn .i / D a whenever i > n. It is straightforward that qn ! q. It follows from
q.n/ D zn 2 En that q O.En / and hence q B.En / for all n 2 !.
However, zi En [ .En / for all i < n which shows that qn1
S2 O.En [ .En //
and hence qn1 2 B.En / for every n 2 !nf0g. Therefore q 2 n2! B.En / which
shows that the family B is not closure-preserving and finishes the proof of (b).
(c) Given a set D say that a family A  D
<! is cofinal in D
<! if for any
F 2 D
<! there is a set A 2 A such that F  A.

2 Solutions of problems 001500

587

Fact 3. Given spaces Z and Z 0 suppose that Z has a closure-preserving local base
B at a point z and Z 0 has a closure-preserving local base B 0 at a point z0 . Then the
family C D fB  B 0 W B 2 B and B 0 2 B 0 g is a closure-preserving local base at the
point .z; z0 / in the space Z  Z 0 .
0
Proof. We leave it to the reader to verify
S that
S C is a local base at .z; z /. Take any
family D  C and assume that u 2 Dn fC W C 2 Dg for some u D .x; y/ 2
Z  Z 0 . For every C D B  B 0 2 D let C0 D B and C1 D B 0 ; observe that
C D C 0  C 1 . Consider the families D0 D fC 2 D W x C 0 g and D1 D fC 2 D W
y C 1 g.
S
The family B being closure-preserving, it follows from x fC 0 W C 2 D0 g
that we can find a set U 2 .x; Z/ such that

(1) U \ C0 D ; for any C 2 D0 .


Analogously, there exists a set V 2 .y; Z 0 / such that
(2) V \ C1 D ; for any C 2 D1 .
The set W D U  V is an open neighborhood of the point u in Z  Z 0 . If C 2 D,
then u C 0  C 1 and hence we have two cases:
Case 1. x C 0 . Then C
.U  V / \ .C0  C1 / D ;.
Case 2. y C 1 . Then C
.U  V / \ .C0  C1 / D ;.

D0 , so U \ C0

; and therefore

D1 , so V \ C1

; and therefore

S
Thus, in all possible cases, we have W \ C D ; for any C 2 D, i.e., W \ . D/ D
;; this contradiction shows that C is closure-preserving and hence Fact 3 is proved.
Fact 4. Suppose that Z is a space with a countable dense set D of isolated points.
Assume that there exists a point a 2 ZnD and a family F  Z
<! nf;g cofinal in
Z
<! such that for any closed neighborhood C of the point a, we can find a finite
set E  ZnC such that E \ F ; whenever F 2 F and F nC ;. Then Cp .Z/
has a closure-preserving local base at every point.
Proof. Let u 2 Cp .Z/ be the function equal to zero at all points of the space Z.
If I D ff 2 Cp .Z/ W f .a/ D 0g, then Cp .Z/ is homeomorphic to I  R (see
Fact 1 of S.409). It is trivial that R has a closure-preserving local base at every
point, so if we prove that I has a closure-preserving local base at u, then Fact 3 can
be applied to see that Cp .Z/ has a closure-preserving local base at some point. This
implies that Cp .Z/ has a closure-preserving base at every point (see e.g., TFS-062,
TFS-115, TFS-116 and Fact 1 of S.496). Therefore it suffices to show that I has a
closure-preserving local base at u.
Given a function f 2 I , a finite set A  Z and a number " > 0 consider the set
M.f; A; "/ D fg 2 I W jg.x/  f .x/j < " for all x 2 Ag. It is clear that the family
fM.f; A; "/ W A 2 Z
<! and " > 0g is a local base at f in I for any f 2 I .
Observe first that if D is finite then Z is finite and hence our Fact is trivially
true, so we can assume, without loss of generality, that D is infinite and hence we
can fix a faithful enumeration fdn W n 2 Ng of the set D. Let Dn D fd1 ; : : : ; dn g

588

2 Solutions of problems 001500

for any n 2 N. For every F 2 F let mF D jF j and A.F / D F [ DmF ; then


W .F / D M.u; A.F /; 2m1 F / is an open neighborhood of f . Let us establish first that
(3) the family W D fW .F / W F 2 Fg is a local base at u.
Take any U 2 .u; I /; there exists a finite set K  Z and " > 0 such that
M.u; K; "/  U . Since F is cofinal in Z
<! , we can find F 2 F such that K  F
and mF > 1" . It is clear that u 2 W .F /  M.u; K; "/  U , so (3) is proved.
To finish the proof of our Fact, it suffices to establish
S that W is closurepreserving, so take any family G  F and assume that f fW .F / W F 2 Gg. The
set H D f 1 .Rnf0g/ is open and nonempty because f u. Therefore H \D ;,
1
1
so we can find k; n 2 N such that jf .dn /j > 2k
and hence r D jf .dn /j  2k
> 0.
Consider the set U D M.f; fdn g; r/. If F 2 G and jF j  l D maxfk; ng, then
1
the point dn belongs to A.F / and jg.dn /j < 2m1 F  2l1  2k
for any g 2 W .F /.
On the other hand, if h 2 U , then jh.dn /  f .dn /j < r and hence jh.dn /j >
1
jf .dn /j  r D 2k
. This shows that h g and hence
(4) U \ W .F / D ; for any F 2 Gl D fF 2 G W mF  lg.
For any i < l let Gi D fF 2 G W mF D i g; the set Ci D fx 2 Z W jf .x/j  2i1 g
is a closed neighborhood of the point a, so we can find a finite set Ei  ZnCi such
that Ei \ F ; whenever F 2 F and F nCi ;. There is no loss of generality to
assume that Di nCi  Ei . Observe that we have the inequality jf .x/j  2i1 > 0 for
any x 2 Ei , so the number "i D nnfjf .x/j  2i1 W x 2 Ei g is positive and hence the
set Ui D M.f; Ei ; "i / is an open neighborhood of f for any i < l.
Fix any F 2 Gi and g 2 W .F /; it follows from f W .F /, that jf .x/j > 2i1 for
some x 2 F [ Di . Given any function h 2 Ui we have two cases.
Case 1. F  Ci ; then x 2 Di nCi  Ei , so it follows from the inequalities
jh.x/j > jf .x/j  "i  jf .x/j  .jf .x/j 

1
1
/D
2i
2i

and the fact that jg.x/j < 2i1 that h g.


Case 2. F nCi ;; by the choice of Ei we can find a point y 2 Ei \ F , so the
inequalities jh.y/j > jf .y/j  "i  jf .y/j  .jf .y/j  2i1 / D 2i1 together with
the fact that jg.y/j < 2i1 show that h g. This proves that
(5) if 1  i < l, then Ui \ W .F / D ; for all F 2 Gi .
As an immediate consequence of (4) and (5), the set V D U \U
S 1 \: : :\Ul1 is an
open neighborhood of the function f that does not meet the set fW .F / W F 2 Gg,
S
so f fW .F / W F 2 Gg and hence the family W is a closure-preserving local
base at u, so Fact 4 is proved.
Returning to our solution let Y be the compact space M0 described in TFS-387.
Recall that Y D D [ A [ fpg where the sets D; A and fpg are disjoint, D is a
countably infinite dense set of isolated points of Y , the set A is uncountable and
A [ fpg is homeomorphic to the one-point compactification of a discrete space

2 Solutions of problems 001500

589

while p is the unique non-isolated point of A [ fpg. Let fdn W n 2 Ng be a faithful


enumeration of the set D. For any F 2 A
<! nf;g let mF D jAj and QF D
F [ fp; d1 ; : : : ; dmF g. We claim that the family F D fQF W F 2 A
<! nf;gg and
the point p satisfy the hypotheses of Fact 4.
It is immediate that F is cofinal in Y
<! , so take any closed neighborhood C of
the point p. There is nothing to prove if C D Y ; if C Y then .Y nC / \ D ;
because D is dense in Y . Therefore dn 2 DnC for some n 2 N; we claim that the
set E D .A [ fd1 ; : : : ; dn g/nC witnesses that the hypothesis of Fact 4 holds for
Y . It is easy to see that E is finite; assume that QF 2 F and QF nC ;. Since
AnC  E, there is no loss of generality to consider that F  C . If jF j  n,
then dn 2 QF \ E. If jF j < n, then ; QF nC  fd1 ; : : : ; dn gnC  E, so
di 2 QF \ E for some i  n.
We proved that E \ G ; whenever G 2 F and GnC ;, so Fact 4 can be
applied to convince ourselves that the space Cp .Y / has a closure-preserving local
base at every point. Finally observe that the compact space Y is not metrizable
because A is an uncountable discrete subset of Y ; this settles (c) and completes our
solution.
V.489. Given a compact space K let jjf jj D supfjf .x/j W x 2 Kg for any f 2
C.K/; if f; g 2 C.K/ then K .f; g/ D jjf  gjj. As usual, Cu .K/ is the set C.K/
with the topology generated by the metric K . If A  Cp .K/ and A ; then
diam.A/ D supfjjf  gjj W f; g 2 Ag. For each " > 0 say that a family A of subsets
of C.K/ is "-small if diam.A/ < " for any A 2 A. The space Cp .K/ is said to have
the property JNR if for every " > 0 weScan find a family fMn W n 2 !g of closed
subsets of Cp .K/ such that Cp .K/ D n2!
SMn and, for every n 2 ! there exists
an "-small family Un  .Mn / such that Un D Mn . Prove that the following
conditions are equivalent:
(i) the space Cp .K/ has the property JNR;
(ii) there exists a  -discrete family N in the space Cp .K/ such that N is a network
in both spaces Cp .K/ and Cu .K/.
In particular, if Cp .K/ has the property JNR then it has a  -discrete network.
Solution. Assume that Cp .K/ has the property JNR. For any f 2 C.K/ and r > 0
the set B.f; r/ D fg 2 C.K/ W K .f; g/ < rg is the r-ball centered at f in the
space Cu .K/. Given > 0 say that a family A of subsets of C.K/ is -uniformly
discrete if, for any f 2 C.K/ the set B.f; / intersects at most one element of A.
For each n 2 ! consider the open cover Un D fB.f; 2n / W f 2 C.K/g of
the space Cu .K/; by Fact 1 ofST.373 we can find a refinement Vn  .Cu .K// of
the cover Un such that Vn D m2! Bnm and, for every m 2 !, the family Bnm is
nm -uniformly discrete
S for some nm > 0. Applying Fact 1 of U.050 it is easy to see
that the family B D fBnm W n; m 2 !g is a base in the space Cu .K/.
For every pair .n; m/ 2 !  ! apply the property JNR to obtain
S a family fEnmk W
k 2 !g of closed subspaces of Cp .K/ such that Cp .K/ D k2! Enmk and, for
every k 2 !, there exists a nm -small cover Gnmk  .Enmk / of the space Enmk . Let
Nnmk D fB \ Enmk W B 2 Bnm g for any n; m; k 2 !. Given any point f 2 Enmk

590

2 Solutions of problems 001500

take G 2 Gnmk with f 2 G and observe that it follows from diam.G/ < nm that
G  B.f; nm / and hence G intersects at most one element of Bnm . Therefore G
intersects at most one element of Nnmk ; since f 2 Enmk was chosen arbitrarily
and G is open in Enmk , we conclude that the family Nnmk is discrete in Enmk ; the
set Enmk being closed in Cp .K/, the family Nnmk is discrete in Cp .K/ for any
n; m; k 2 !. S
Thus N D fNnmk W n; m; k 2 !g is a  -discrete family in Cp .K/. Take an
arbitrary f 2 Cu .K/ and O 2 .f; Cu .K//. Since B is a base in Cu .K/, we can
find n; m 2 ! such that f 2 B  O for some B 2 Bnm . Choose k 2 ! for which
f 2 Enmk and observe that B 0 D B \ Enmk 2 N . Since also f 2 B 0  O, we
proved that N is a network in Cu .K/; it is evident that this implies that N is also a
network in Cp .K/ and hence we established that (i)H)(ii).
Fact 1. Suppose that Z is a compact space; let Z .f; g/ D supfjf .x/  g.x/j W x 2
Zg for any f; g 2 C.Z/. As usual, diamZ .B/ D supfZ .f; g/ W f; g 2 Bg for any
nonempty set B  C.Z/. Given a set A  C.Z/ let A be the closure of A in the
space Cp .Z/. Then diamZ .A/ D diamZ .A/ for any nonempty set A  C.Z/.
Proof. Let r D diamZ .A/; it suffices to show that diamZ .A/  r. If this is false
then we can find f; g 2 A for which Z .f; g/ > r C " for some " > 0. Therefore
there exists a point x 2 Z such that jf .x/  g.x/j > r C ". Take f0 ; g0 2 A for
which jf0 .x/  f .x/j < 3" and jg.x/  g0 .x/j < 3" . Then
jf0 .x/  g0 .x/j D jf0 .x/  f .x/ C f .x/  g.x/ C g.x/  g0 .x/j 
"
2
jf .x/  g.x/j  jf0 .x/  f .x/j  jg0 .x/  g.x/j  r C "  " D r C > r;
3
3
which shows that diamZ .A/  Z .f0 ; g0 / > r which is a contradiction. Therefore
diamZ .A/ D diamZ .A/, i.e., Fact 1 is proved.
Returning
S to our solution assume that N is a network in the space Cu .K/ such
that N D n2! Nn and every family Nn is discrete in the space Cp .K/. The family
Mn D fN W N 2 Nn g is still discrete in Cp .K/ for every n 2 !; let M D
S
n2! Mn . Take a point f 2 Cu .K/ and U 2 .f; Cu .K//; there exists r > 0 such
that B.f; r/  U . The family N being a network of Cu .K/ we can find N 2 N
with f 2 N  B.f; 3r /. Apply Fact 1 to see that if M D N then diam.M / D
diam.N /  23 r. Furthermore, f 2 M , so M  B.f; 34 r/  B.f; r/  U . This
shows that M is also a network S
in the space Cu .K/.
/ < "g for each n 2S!. It
Given any " > 0 let Mn D fM 2 Mn W diam.M
S
is easy to see that every set M 2 Mn is open in Mn (in the topology on Mn
induced from Cp .K/); therefore the family Un D fM 2 Mn W diam.M / < "g is an
"-small open cover of the set Mn . Take any point f 2 Cp .K/; since M is a network
in Cu .K/, we can find n 2 ! and M 2 Mn such that f 2 M  B.f; 3" /. This
implies that diam.M /  diam.B.f; 3" //  23 " < ", so M 2 Un and hence f 2 Mn .

2 Solutions of problems 001500

591

S
Thus Cp .K/ D Mn which shows that Cp .K/ has the property JNR; this settles
(ii)H)(i) and makes our solution complete.
V.490. Prove that
(a) if L is a linearly ordered separable compact space then Cp .L/ is a  -space,
i.e., it has a  -discrete network;
(b) for any separable dyadic compact space K, the space Cp .K/ has a  -discrete
network.
Solution. (a) For any set A  R let dm.A/ D supfjt  sj W s; t 2 Ag, i.e., dm.A/
is the diameter of A in R. Given a space X and a continuous function f on the
space X let jjf jjY D supfjf .x/j W x 2 Y g for any Y  X . If P  Cp .X / then
let diamX .P / D supfjjf  gjjX W f; g 2 P g. A family P of subsets of Cp .X /
is "-small if diamX .P / < " for any P 2 P.
Denote by < the order that generates the topology of L. Let a be the smallest
element of L and denote by b its largest element (see TFS-305). For any x; y 2 L
the expression x  y says that either x < y or x D y. For any x; y 2 L let
x; y
D fz 2 L W x  z  yg and .x; y/ D fz 2 L W x < z < yg. We will also need
the intervals x; y/ D fz 2 L W x  z < yg and .x; y
D fz 2 L W x < z  yg. Fix a
countable dense set S  L such that fa; bg  S .
Fact 1. Suppose
that X is a space, " > 0 and A is a family of subsets of X such
S
that X D A; let F D ff 2 C.X / W dm.f .A//  " for any A 2 Ag. Assume
additionally, that D  X and D \ A ; for any A 2 A. Then jjf  gjjX  3" for
any pair of functions f; g 2 F such that jjf  gjjD  ".
Proof. Take any t 2 X and choose A 2 A such that t 2 A. There exists d 2 D \ A;
it follows from jjf  gjjD  " that jf .d /  g.d /j  ". Besides, we have the
inequalities jf .t /  f .d /j  dm.f .A//  " and jg.t /  g.d /j  dm.g.A//  ".
As an immediate consequence, jf .t /  g.t /j  jf .t /  f .d /j C jf .d /  g.d /j C
jg.d /  g.t /j  3" for any t 2 X , i.e., jjf  gjjX  3" and hence Fact 1 is proved.
Returning to our solution consider, for any x; y 2 L with x  y and any " > 0
the set A.x; y; "/ D ff 2 Cp .L/ W there exist p; q 2 x; y
such that p  q and
x; y
D x; p
[ q; y
while dm.f .x; p
//  " and dm.f .q; y
//  "g. Take any
function f 2 Cp .X /nA.x; y; "/; to construct a neighborhood of f which does not
meet A.x; y; "/ we will need the points u D supft 2 x; y
W dm.f .x; t
//  "g
and v D infft 2 x; y
W dm.f .t; y
//  "g.
It is easy to see that dm.f .x; u
//  " and dm.f .v; y
//  "; this, together
with f A.x; y; "/ implies that x; y
x; u
[ v; y
and hence there exists a
point z 2 .u; v/. By our choice of u and v there exist r; s 2 x; z
and t; w 2 z; y

such that jf .r/  f .s/j > " and jf .t /  f .w/j > ".
The set W D fg 2 Cp .L/ W jg.r/  g.s/j > " and jg.t /  g.w/j > "g is
open in Cp .L/ and f 2 W . If g 2 W \ A.x; y; "/ then there exist p; q 2 L
such that p  q and x; y
D x; p
[ q; y
while dm.g.x; p
//  " and
dm.g.q; y
//  ". Therefore the segment x; p
cannot contain the points r and s
and hence p < z. Analogously, the set ft; wg cannot be contained in q; y
, so z < q.

592

2 Solutions of problems 001500

Thus z 2 x; y
n.x; p
[ q; y
/ which is a contradiction. Therefore W \
A.x; y; "/ D ;, so we proved that every f 2 Cp .L/nA.x; y; "/ has a neighborhood
contained in Cp .L/nA.x; y; "/. As a consequence,
(1) for every " > 0 the set A.x; y; "/ is closed in Cp .L/ for any x; y 2 L with
x  y.
Our next step is to prove that
(2) for any " > 0 and x; y 2 L with x  y, if f 2 A.x; y; "/ then there exists
Uf 2 .f; Cp .L// such that jjg  f jjxy
 3" for each g 2 Uf \ A.x; y; "/.
Fix points p; q 2 x; y
such that p  q and x; y
D x; p
[q; y
while we have
the inequalities dm.f .x; p
//  " and dm.f .q; y
//  ". Let D D fx; y; p; qg
and observe that the set Uf D ff 2 Cp .L/ W jg.t /  f .t /j < " for all t 2 Dg is an
open neighborhood of f in Cp .L/ such that jjf  gjjD  " for any g 2 Uf . Take
any function g 2 Uf \ A.x; y; "/ and choose points r; s 2 x; y
such that r  s
and x; y
D x; r
[ s; y
while dm.g.x; r
//  " and dm.g.s; y
//  ". There
are several cases to consider.
Case 1. r D p D q and hence q  s. Then the family A D fx; r
; s; y
g in
the space X D x; y
and the set D together with the functions f jX and gjX
satisfy the assumptions of Fact 1, so jjf  gjjX  3".
Case 2. r D p < q; it is an easy exercise that this implies s  q. Then the family
A D fx; p
; q; y
g in the space X D x; y
and the set D together with the
functions f jX and gjX satisfy the assumptions of Fact 1, so jjf  gjjX  3".
Case 3. r < p; it is easy to see that this implies s  p. Then the family A D
fx; r
; s; p
; q; y
g and the set D together with the functions f jX and gjX
satisfy the assumptions of Fact 1, so jjf  gjjX  3".
Case 4. r > p; it is straightforward that this implies r  q. Then the family A D
fx; p
; q; r
; s; y
g and the set D together with the functions f jX and gjX
satisfy the assumptions of Fact 1, so jjf  gjjX  3".
Thus, in all possible cases we have jjf  gjjX  3", i.e., (2) is proved. Now let
us establish that
(3) for any f 2 Cp .L/ and " > 0 there exists a sequence fs0 ; : : : ; sn g  S such
that s0 D a < s1 < : : : < sn D b and f 2 A.si ; siC1 ; "/ for any i < n.
Recall that a set P  L is called convex if x; y
 P for any x; y 2 P . It
is easy to see that the family C of convex open subsets of L is a base of L. By
continuity of f for any x 2 L thereS
exists Cx 2 C such that dm.f .Cx //  ".
Choose a finite K  L such that L D fCx W x 2 Kg; it is easy to find a sequence
Q D fs0 ; : : : ; sn g  S such that a D s0 < : : : < sn D b and Q \ Cx ; for any
x 2 K.
Fix any i < n and consider the families E0 D fCx W x 2 K and S
si 2 CxS
g and
g.
Observe
first
that
s
;
s


E
[
E1 .
E1 D fCx W x 2 K and sS
iC1 2 C
x
i
iC1
0
S
Indeed, if t 2 si ; siC1
n. E0 [ E1 / then t 2 Cx for some x 2 K; by convexity

2 Solutions of problems 001500

593

of Cx , it follows from Cx \ fsi ; siC1 g D ; that Cx  .si ; siC1 /. There exists a


point q 2 QS\ Cx ;S
then q 2 .si ; siC1 / which is a contradiction. This proves that
si ; siC1
 E0 [ E1 .
If E; E 0 2 E0 then it is easy to deduce from convexity of E and E 0 that either
E \ si ; siC1
 E 0 \ si ; siC1
or E 0 \ si ; siC1
S
 E \ si ; siC1
. Consequently,
there exists E0 2 E0 such that E0 \ si ; siC1
D . ES0 / \ si ; siC1
. Analogously,
we can find E1 2 E1 such that E1 \ si ; siC1
D . E1 / \ si ; siC1
. Therefore
si ; siC1
 E0 [ E1 .
If x 2 E0 \ E1 \ si ; siC1
then it is easy to see that the points p D q D x
witness that f 2 A.si ; siC1 ; "/. If E0 \ E1 \ si ; siC1
D ; then, for the points
p D sup.E0 \ si ; siC1
/ and q D inf.E1 \ si ; siC1
/, we have p  q and it is
straightforward that p and q witness that f 2 A.si ; siC1 ; "/, so (3) is proved.
Fix an arbitrary " > 0 and observe that for any T
points s0 ; : : : ; sn 2 S such that
a D s0 < : : : < sn D b, the set M.s1 ; : : : ; sn / D fA.si ; siC1 ; 9" / W 0  i < ng
is closed in Cp .L/. The family M D fM.s0 ; : : : ; sn / W n 2 N; s0 ; : : : ; sn 2 S and
a D s0 < : : : < sn D bg is countable and it follows from
S (1) that M consists
of closed subsets of Cp .L/. The property (3) shows that M D Cp .L/. Now
fix any s0 ; : : : ; sn 2 S such that a D s0 < : : : < sn D b and take any function
f 2 H D M.s0 ; : : : ; sn /.
f
f
The property (2) makes it possible to find sets U0 ; : : : ; Un1 2 .f; Cp .L//
T
f
f
such that jjg  f jjsi ;si C1
 3" for any g 2 Ui \ H ; if U f D i<n Ui then
jjg  f jjL  3" for any g 2 U f \ H and hence Gf D U f \ H is an open subset
< ". Therefore fG f W f 2 H g is an "-small open
of H such that diamL .G f /  2"
3
cover of H ; this demonstrates that Cp .L/ has the property JNR and hence we can
apply Problem 489 to conclude that Cp .L/ has a  -discrete network.
(b) We will first prove that the space Cp .Dc / has a  -discrete network. To that
end we will identify Dc with DS where S D D! is the Cantor set. For every
x 2 DS let x; E
D fy 2 DS W yjE D xjEg for any finite set E  S . If
f 2 Cp .DS / then jjf jj D supfjf .x/j W x 2 DS g. For any bounded set B  R
let dm.B/ D supfjt  sj W s; t 2 Bg, i.e., dm.B/ is the diameter of B in R.
Given a set N  Cp .DS / let diam.N / D supfjjf  gjj W f; g 2 N g. A family
N of subsets of Cp .DS / is called "-small if diam.N / < " for any N 2 N .
Say that a family F of subsets of S is adequate if F is nonempty, disjoint, finite
and F ; for any F 2 F. Given an adequate family F say that L  S is
selection set for F if jLj D jFj and L \ F ; for any F 2 F; besides, let
k.F / D jfF 2 F W jF j > 1gj.
For any adequate family F and " > 0 let A.F; "/ D ff 2 Cp .DS / W there exists
a selection set L for F such that jf .x/  f .y/j  " whenever x; y 2 DS and
xjL D yjLg. If F is an adequate family, " > 0 and f 2 Cp .DS /nA.F; "/ then a set
Q  DS is representative for the function f if for any selection set L for the family
F there exist x; y 2 Q such that xjL D yjL and jf .x/  f .y/j > ". It turns out
that

594

2 Solutions of problems 001500

(4) given an adequate family F and " > 0, if f 2 Cp .DS /nA.F; "/ then there
exists a finite representative set Q  DS for the function f .
We will prove (4) by induction on k.F/. If k.F/ D 0 then all elements of F are
singletons and hence there exists a unique selection set L for F. Since f A.F; "/,
there must exist x; y 2 DS such that xjL D yjL and jf .x/  f .y/j > ". It is clear
that Q D fx; yg is a representative set for f .
Now assume that n 2 N and we proved (4) for all families F such that k.F/ < n.
If F is an adequate family and k.F/ D n then take any selection set M for the
family F and fix a; b 2 DS such that ajM D bjM and jf .a/  f .b/j > ". The set
O D f.x; y/ 2 DS  DS W jf .x/  f .y/j > "g is an open neighborhood of the point
.a; b/, so there exist G 2 .a; DS / and H 2 .b; DS / such that G  H  O. We
can find a finite set E  S such that M  E while a; E
 G and b; E
 H . As
a consequence,
(5) for any x; y 2 DS , if xjE D ajE and yjE D bjE then jf .x/  f .y/j > ".
Pick u; v 2 DS such that ujE D ajE; vjE D bjE and uj.S nE/ D vj.S nE/.
For each F 2 F let sF be the unique point of M \ F and denote the set E \ F
by EF . For any s 2 EF nfsF g consider the family D.F; s/ D ffsgg [ .FnfF g/;
it is evident that D.F; s/ is adequate and k.D.F; s// D n  1. Since also
f 2 Cp .DS /nA.D.F; s/; "/, the induction hypothesis is applicable to find a finite
representative set Q.F; s/ for
S the family D.F; s/.
The set Q D fu; vg [ fQ.F; s/ W F 2 F and s 2 EF nfsF gg is finite; we
claim that it is representative for f . Indeed, take any selection set L for the family
F and denote by tF the unique point of L \ F for any F 2 F. If tF 2 EF nfsF g for
some F 2 F then L is a selection set for the family D.F; tF /; since Q.F; tF / is a
representative set for f we can find x; y 2 Q.F; tF /  Q such that xjL D yjL and
jf .x/  f .y/j > ". If tF EF nfsF g then it is easy to see that u.tF / D v.tF / for
each F 2 F and hence ujL D vjL; since jf .u/  f .v/j > " by (5), the functions
u; v 2 Q witness that Q is representative for f . This completes our inductive step
so (4) is proved.
Now take any adequate family F and " > 0; if f 2 Cp .DS /nA.F; "/ then
apply (4) to find a finite representative set Q for the function f and consider the set
W D fg 2 Cp .DS / W jg.x/g.y/j > " whenever x; y 2 Q and jf .x/f .y/j > "g;
it is immediate that W 2 .f; Cp .DS //. If g 2 W and L is a selection set for F
then there exist x; y 2 Q such that xjL D yjL and jf .x/  f .y/j > "; by the
definition of W we have jg.x/  g.y/j > " and hence g A.F; "/. This proves that
every f 2 Cp .DS /nA.F; "/ has a neighborhood contained in Cp .DS /nA.F; "/ and
therefore
(6) the set A.F; "/ is closed in Cp .DS / for any adequate family F and " > 0.
Fix " > 0 and an adequate family F of subsets of S . Given f 2 A.F; "/ take a
selection set L for the family F such that jf .x/
S f .y/j  " whenever xjL D yjL.
The set R D fx 2 DS W x is constant on S n F and on F nL for any F 2 Fg is
easily seen to be finite. The set V D fg 2 Cp .DS / W jg.x/  f .x/j < " for any

2 Solutions of problems 001500

595

x 2 Rg is open in Cp .DS /; take any g 2 V \ A.F; "/. There exists a selection set
M for F such that jg.x/  g.y/j  " for all x; y 2 DS such that xjM D yjM . Take
an arbitrary x 2 DS ; it is easy to find y 2 R such that xj.M [ L/ D yj.M [ L/.
The choice of y guarantees that jf .x/  f .y/j  " and jg.x/  g.y/j  "; it
follows from y 2 R that jf .y/  g.y/j < ". Since x 2 DS was chosen arbitrarily, it
follows from jf .x/  g.x/j  jf .x/  f .y/j C jf .y/  g.y/j C jg.y/  g.x/j  3"
that jf .x/  g.x/j  3" for any x 2 DS , i.e., jjf  gjj  3". As a consequence,
(7) if F is an adequate family and " > 0 then for any f 2 A.F; "/ there exists a
set V 2 .f; Cp .DS // such that jjf  gjj  3" for any g 2 V \ A.F; "/.
Take any function f 2 Cp .DS / and " > 0. By continuity of f , for every x 2 DS
there is a finite set Ex  S such that dm.f .x; Ex
// <
S ". By compactness of
DS weScan choose a finite set P  DS such that DS D fx; Ex
W x 2 P g; let
E D fEx W x 2 P g. Now if x; y 2 DS and xjE D yjE then pick a 2 P for
which x 2 a; Ea
. Then yjEa D xjEa D ajEa which shows that y 2 a; Ea
and
hence jf .x/  f .y/j  dm.f .a; Ea
// < ". This proves that
(8) for any f 2 DS and " > 0 there is a finite set E  S such that
jf .x/  f .y/j < " whenever xjE D yjE.
Denote by B the family of all nonempty clopen subsets of S (recall that S D D!
is the Cantor set). Then B is countable, so the collection F of all adequate families of
elements of B is countable. Fix an arbitrary " > 0 and consider the countable family
A D fA.F; 9" / W F 2 Fg. It follows from (6) that all elements of A are closed
in Cp .DS /. Given any A.F; 9" / 2 A apply (7) to find a set Vf 2 .f; Cp .DS //
such that jjf  gjj  3" for any f 2 A.F; "/ and g 2 Gf D Vf \ A.F; 9" /. An
immediate consequence is that diam.Gf /  23 " < " for any f 2 A.F; 9" / and hence
fGf W f 2 A.F; 9" /g is an open "-small cover of A.F; 9" /. If f 2 Cp .DS / then we
can apply (8) to find a set E D fs1 ; : : : ; sn g  S such that jf .x/  f .y/j < 9"
whenever xjE D yjE. Take disjoint sets B1 ; : : : ; Bn 2 B such that si 2 Bi for
each i  n. Then F D fB1 ; : : : ; Bn g is an adequate
S family which, together with
the set E, witnesses that f 2 A.F; 9" /. Therefore A D Cp .DS /, so we proved
that Cp .DS / has the property JNR and hence Cp .DS / ' Cp .Dc / has a  -discrete
network by Problem 489.
Finally assume that K is a separable dyadic compact space; by Fact 2 of S.368,
we have w.K/  2d.K/  c. Suppose that A is a set and ' W DA ! K is a continuous
onto map. It follows from TFS-299 that we can find a set B  A such that jBj  c
and there exists a continuous map  W DB ! K for which   D ' where
 W DA ! DB is the natural projection. In particular, DB maps continuously onto K
and hence Dc also maps continuously onto K. Therefore Cp .K/ embeds in Cp .Dc /,
so Cp .K/ has a  -discrete network, i.e., we proved (b) and hence our solution is
complete.
V.491. Prove that there exists a scattered compact space K with a countable dense
set of isolated points such that Cp .K; D/ is not perfect. Deduce from this fact that
Cp .!; D/ is not perfect and, in particular, the space Cp .!/ does not have a
 -discrete network.

596

2 Solutions of problems 001500

Solution. Recall that a space Z is perfect if every U 2 .Z/ is an FS -subset of Z.


Let S be the set of all finite sequences of natural numbers, i.e., S D f! n W n 2 !g
where ! 0 D f;g. We consider that the set ! ! carries the usual product topology.
For any s 2 S let s
D fx 2 ! ! W xjdom.s/ D sg; it is clear that fs
W s 2 S g is a
base in ! ! . If s 2 S; dom.s/ D n and i 2 ! then t D s _ i 2 ! nC1 is defined by
the conditions t jn D s and t .n/ D i .
If x 2 ! ! then let Bxm D fxjn W n  mg for any m 2 !. To introduce a
topology on the set L D S [ ! ! declare all points of S to be isolated; if x 2 ! !
then let the family Bx D ffxg [ Bxm W m 2 !g be a local base at the point x. It
is straightforward that L is a locally compact Tychonoff space; denote by K the
one-point compactification of L and let  be the unique point of KnL.
It is immediate that K is a scattered separable compact space and S is a countable
dense set of isolated points of K. To show that Cp .K; D/ is not a perfect space
consider the set F D ff 2 Cp .K; D/ W f 1 .1/  S g. It follows from the equality
F D ff 2 Cp .K; D/ W f .! ! [ fg/ D f0gg that the set F is closed in Cp .K; D/.
If Cp .K; D/ is perfect then we can find
S a family fPn W n 2 !g of closed subsets of
Cp .K; D/ such that Cp .K; D/nF D n2! Pn .
Given any x 2 ! ! the set Bx D fxg [ Bx0 is easily seen to be clopen in K; let gx
be the characteristic function of Bx , i.e., gx .y/ S
D 1 for all y 2 Bx and gx .y/ D 0
whenever y 2 KnBx . Then fgx W x 2 ! ! g  n2! Pn ; the Baire property of ! !
implies that there exists n 2 ! such that the set Q D fx 2 ! ! W gx 2 Pn g is dense
in s
for some s 2 S ; let m D dom.s/. Choose a point xi 2 Q such that xi 2 s _ i

for every i 2 !.
For the set A D fsji W i  mg  K let g 2 Cp .K; D/ be the characteristic
function of A; then A D g 1 .1/ and hence g 2 F . It is straightforward that Bxi \
Bxj D A for any distinct i; j 2 ! which easily implies that the sequence fgxi W
i 2 !g  Pn converges to g. However, Pn is closed in Cp .K; D/ and g Pn ; this
contradiction shows that Cp .K; D/nF is not an F -subset of Cp .K; D/ and hence
Cp .K; D/ is not a perfect space.
Finally, observe that there exists a continuous surjective map ' W ! ! K
because ! can be continuously mapped onto any separable compact space. Let
'  .f / D f ' for any f 2 Cp .K/; then '  W Cp .K/ ! Cp .!/ is an embedding
such that '  .Cp .K; D//  Cp .!; D/. The property of being perfect is easily seen
to be hereditary so Cp .!; D/ is not perfect. Any space with a  -discrete network
is perfect and hence Cp .!/  Cp .!; D/ has no  -discrete network.
V.492. Prove that, for any ultrafilter  2 !n!, the following conditions are
equivalent:
(i)  is not a P -point in !n!;
(ii)  .R! / is homeomorphic to a closed subspace of Cp .! /;
(iii) Cp .! / is not hereditarily Baire.
S
Solution. Let ! 0 D f;g and consider the set D f! n W n 2 !g of all finite
sequences of natural numbers. If s 2 then let l.s/ D jdom.s/j be the length
of s. We follow the usual practice of identifying any ordinal with the set of its

2 Solutions of problems 001500

597

predecessors. In particular, 0 D ; and i D f0; : : : ; i  1g for any i 2 N. Thus,


for any s 2 , if l.s/ D n  i 2 ! then sji is the restriction of s to the set
f0; : : : ; i  1g.
For any s 2 and n 2 ! the sequence t D s _ n is defined by the equalities
t .i / D s.i / for any i < k D dom.s/ and t .k/ D n. Given s; t 2 the expression
s t says that t extends s, i.e., dom.s/  dom.t / and t .i / D s.i / for any i 2
dom.s/. Analogously, u 2 ! ! extends s (we also denote this by s u) if u.i / D
s.i / for any i 2 dom.s/. If a metric  is considered on the space X then, for any
point x 2 X and r > 0 the set B.x; r/ D fy 2 X W .x; y/ < rg is the open ball of
radius r centered at x and diam.A/ D supf.x; y/ W x; y 2 Ag for any A  X .
A family fV .s/ W s 2 g  .X / is an A-system on X if
S
(A1) V .;/ D X and V .s/ D fV .s _ i / W i 2 !g for any s 2 ;
1
(A2) diam.V .s// < l.s/
for any s 2 nf;g;
T
(A3) fV .uji / W i 2 !g ; for any u 2 ! ! .
For linear spaces L and M the expression L
M says that L is linearly
homeomorphic to M . Given an ultrafilter  2 !n!, the GulkoSokolov game
on the space ! is played as follows: at the n-th move player N U takes a set
Sn 2 exp.!/n and player U replies by choosing a finite set Tn  ! in such a
way that the family fSi W i  ng [ fTi W i  ng is disjoint. The
S game ends after
the moves Sn ; Tn are made for all n 2 ! and player U wins if fTn W n 2 !g 2 .
Otherwise, the player N U is the winner.
Fact 1. A separable metrizable space X is hereditarily Baire if and only if there is
no closed subset of X homeomorphic to Q.
Proof. If some closed set F  X is homeomorphic to Q then F is not Baire, so
X is not hereditarily Baire; this proves necessity. Now if X is not hereditarily Baire
then we can find a closed set F  X which is not Baire. Therefore there exists a
set U 2  .F / which is of first category in F . The space X being second countable
we canSfind a family fPn W n 2 !g of closed nowhere dense subsets of F such that
U D n2! Pn . It is easy to see that every Pn is nowhere dense in U and hence in
U . Since P D U nU is also closed and nowhere dense in U , the set U D U [ P
is of first category in itself. Observe that no space of first category in itself can have
isolated points, so we can apply Fact 4 of T.351 to conclude that some closed subset
Q of the space U is homeomorphic to Q. The set Q is also closed in X , so we
settled sufficiency, i.e., Fact 1 is proved.
Fact 2. If X is a separable metrizable space, then player E has a winning strategy
in the Choquet game on X if and only if X is not a hereditarily Baire space.
Proof. Suppose that X is not a hereditarily Baire space and hence we can find a
closed set F  X which is homeomorphic to Q; let fqn W n 2 !g be a faithful
enumeration of the set F and fix a metric  which generates the topology of X . For
every number i 2 ! let Qi D fq0 : : : ; qi g;
Choose a point x0 2 F nfq0 g and r0 2 .0; 1/ such that the closure
of the set U0 D B.x0 ; r0 / does not contain q0 and let  .;/ D .x0 ; U0 /.

598

2 Solutions of problems 001500

Proceeding inductively assume that n 2 ! and we have moves .x0 ; U0 /; V0 ; : : : ;


.xn1 ; Un1 /; Vn1 ; .xn ; Un / in the Choquet game on X such that
(1)
(2)
(3)
(4)

xi 2 F for all i  n;
Ui D B.xi ; ri / where 0 < ri < 2i for all i  n;
U i \ Qi D ; for each i  n;
U iC1  Vi for all i < n.

If the player NE makes a move Vn then we can choose a point xnC1 2 Vn nQnC1
because the space F has no isolated points. Pick a number rnC1 2 .0; 2n1 / such
that the closure of the set UnC1 D B.xnC1 ; rnC1 / is contained in Vn nQnC1 and let
.xnC1 ; UnC1 / D  .V0 ; : : : ; Vn /. It is easy to see that this defines a strategy  of
the player E on the space X . To see that  is winning assume that we have a play
f.xi ; Ui /; Vi W i 2 !g in which
T the strategy  is applied. The property (2) shows
that
diam.U
/
!
0,
so
if
i
i2! Ui ; then this intersection is a singleton, i.e.,
T
U
D
fxg
for
some
x
2
X.
i
i2!
Furthermore, it follows from x 2 Ui and (2) that .xi ; x/ < 2i for every i 2 !
and hence xi ! x. The set F being closed in X , the point x has to belong to F and
hence x D qn for some n 2 !. The property (3) implies that qn U n ; however,
S
T D fxi W i  ng  U n and hence qn D x 2 S  U n ; this contradiction shows that
i2! Ui D ; and hence  is a winning strategy for E, i.e., we proved sufficiency.
Now assume that  is a winning strategy for the player E; for any U 2 .X /
and x 2 U take a number rUx > 0 such that B.x; rUx /  U . If .x0 ; U0 / D  .;/
then let A.;/ D f.x0 ; U0 /g and q.;/ D x0 . For any s 2 with l.s/ D n > 0 let
A.s/ D fM0 ; V0 ; : : : ; Mn1 ; Vn1 ; Mn g if the following conditions are satisfied:
(5) A.s/ is an initial segment of a play in the Choquet game on X , i.e., Mi D
.xi ; Ui / while xi 2 Vi  Ui for all i < n and Ui  Vi1 whenever 0 < i  n;
(6) the moves of the player E are made applying the strategy  , i.e., M0 D  .;/
and MiC1 D  .V0 ; : : : ; Vi / for any i < n;
(7) Vi D B.xi ; 2s.i/ rUxii / for each i < n.
It is evident that the family A.s/ is uniquely determined by the sequence s; let
q.s/ D xn . We are going to show that the set F D fq.s/ W s 2 g witnesses that
the space X is not hereditarily Baire.
It is clear that the set F is countable; fix an arbitrary s 2 and " > 0. If
l.s/ D n then A.s/ D fM0 ; V0 ; : : : ; Mn1 ; Vn1 ; Mn g where Mi D .xi ; Ui / for
every i  n. Then q.s/ D xn and we can choose k 2 ! such that 2k rUxnn < ".
Let t D s _ k and fix any u 2 ! ! such that t u. The function u defines a play
P D fNi ; Wi W i 2 !g such that A.ujm/ D fN0 ; W0 ; : : : ; Nm1 ; Wm1 ; Nm g for
each m 2 !. As an immediate consequence,TE played in P according to the strategy
 , so the play P is a win for E and hence i2! Wi D ;. It follows from t u that
Ni D Mi and Wi D Vi for all i < n while Mn D Nn and Wn D B.xn ; 2k rUxnn /.
If the set
Tfxj W j 2 !g is finite then some xj belongs to infinitely many Wi s
and hence i2! Wi ; which is a contradiction. Therefore there exists m > n C 1
such that xm xn . The point xm has to belong to the set Wn and hence we have the

2 Solutions of problems 001500

599

inequality .xm ; xn / < 2k rUxnn < " which shows that xm D q.ujm/ 2 F nq.s/ is
an element of F nfq.s/g which belongs to the set B.q.s/; "/. Therefore every "-ball
centered at q.s/ contains a point of F nfq.s/g and hence q.s/ is not isolated in F
for any s 2 , i.e., the set F has no isolated points. By SFFS-349, the space F is
homeomorphic to Q and hence F is of first category in itself.
To see that F is a closed subset of the space X assume that there exists a point
z 2 F nF and choose a faithfully enumerated sequence Z D fzn W n 2 !g  F
which converges to z. For every s 2 consider the set H.s/ D fq.t / W t 2 nfsg
_
and s t g and
S let Hi .s/ D fq.t / W t 2 and s i t g for any i 2 !; it is evident
that H.s/ D i2! Hi .s/.
We claim that
(8) given any s 2 and D  ! the set ZD D fzn W n 2 Dg intersects only finitely
many sets Hi .s/.
To prove (8) assume that the set Q D fi 2 ! W ZD \ Hi .s/ ;g is infinite and
fix " > 0. We have A.s/ D fM0 ; V0 ; : : : ; Mm1 ; Vm1 ; Mm g for some m 2 ! where
Mi D .xi ; Ui / for all i  m.
There exists i 2 Q such that 2i rUxmm < "; take n 2 D such that zn 2 Hi .s/
and hence zn D q.t / for some t 2 such that t jm D s and t .m/ D i . This
implies that zn 2 B.q.s/; 2i rUxmm / and hence .zn ; q.s// < ". Since " > 0 was
taken arbitrarily, we established that q.s/ is an accumulation point of ZD which is
impossible because ZD is a nontrivial subsequence of Z, so it has to converge to
z q.s/. This contradiction shows that (8) is proved. It is easy to deduce from (8)
that
(9) for any s 2 if the set D D fn 2 ! W zn 2 H.s/g is infinite then there exists
m 2 ! such that D 0 D fn 2 ! W zn 2 Hm .s/g is also infinite.
It is easy to construct by induction, using the property (9), a function u 2 ! ! such
that the set Dn D fi 2 ! W zi 2 H.ujn/g is infinite for any n 2 !. There exists a play
M D fMi ; Vi W i 2 !g in which E applies the strategy  and, for every n 2 !, we
have A.ujn/ D fM0 ; V0 ; : : : ; Mn1 ; Vn1 ; Mn g where Mi D .xi ; Ui / for all i  n.
Fix any n 2 !; then the set J D fi 2 ! W zi D q.ti / for some ti ujng is infinite. If
i 2 J and l.ti / D k > n then A.ti / D fM0 ; V0 ; : : : ; Mn ; Wn ; : : : ; Nk1 ; Wk1 ; Nk g;
since A.ti / is an initial segment of a play, we must have zi D q.ti / 2 Wk1  Un
and hence fzi W i 2 J g  Un T
. As a consequence,
z 2 fzi W i 2 J g  U n for each
T
n 2 ! which shows that z 2 n2! U n D n2! Un which
T is a contradiction with
the fact that the play M is favorable for E and hence n2! Un D ;. Therefore F
is a closed subspace of X which is of first category in itself so X is not hereditarily
Baire, i.e., we settled necessity and hence Fact 2 is proved.
Fact 3. Given a separable metrizable space X , the player NE has a winning

strategy in the Choquet game on X if and only if X is Cech-complete.

Proof. Assume that X is Cech-complete


and hence we can find a complete metric 
on the set X with .X / D ./. If n 2 ! and the player E chooses a pair .xn ; Un /
where xn 2 Un 2 .X / then the player NE takes a number r 2 .0; 1/ such that

600

2 Solutions of problems 001500

B.xn ; r/  Un and declares his move to be the set Vn D B.x; 2n r/. This gives a
strategy  for the player NE in the Choquet game on X . If P D f.xi ; Ui /; Vi W i 2
!g is a play in which NE applies  then we have a decreasing family fV i W i 2 !g
of closed subsets of X such that diam.V i /  2iC1 for every i 2 ! and hence
diam.V i / ! 0. The space
T
T.X; / being
T complete we can apply TFS-236 to see that
V

;
and
hence
U
D
i
i2!
i2! i
i2! V i ; which shows that  is a winning
strategy of NE and hence we proved sufficiency.
Now suppose that the player NE has a winning strategy  in the Choquet game
on the space X . Take a metric  which generates the topology of X . For every
x 2 X let the move of the player E be the pair .x; B.x; 13 //; the strategy  gives
us a set Vx D  ..x; B.x; 13 ///. Choose a countable subcover fVxn W n 2 !g of the
open cover fVx W x 2 X g of the space X ; let q.s/ D xs.0/ ; U.s/ D B.q.s/; 13 /
and V .s/ D Vq.s/ D  ..q.s/; U.s/// for any s 2 ! 1 . We will also need the set
V .;/ D X .
Suppose that, for some n 2 N we have defined sets U.s/; V .s/ 2 .X / and a
point q.s/ 2 X for any s 2 nf;g with l.s/  n in such a way that
S
(10) if 0  l.s/ < n then V .s/ D fV .s _ i / W i 2 !g;
(11) if 1  l.s/  n then diam.U.s//  2 3l.s/ ;
(12) if 1  l.s/  n then f.q.sji /; U.sji //; V .sji / W 1  i  l.s/g is an initial
segment of a play on X in which E applies the strategy 
Fix any s 2 with l.s/ D n and consider an open cover fB.y; ry / W y 2
V .s/g such that ry 2 .0; 3n1 / and
SB.y; ry /  V .s/ for any y 2 V .s/. Pick a
set fyn W n 2 !g  V .s/ such that i2! B.yi ; ryi / D V .s/; let q.s _ i / D yi and
U.s _ i / D B.yi ; ryi / for any i 2 !. If i 2 ! then the sequence
.q.sj1/; U.sj1//; V .sj1/; : : : ; .q.s/; U.s//; V .s/; .q.s _ i /; U.s _ i //
is an initial segment of a play in X is which E applies the strategy  , so this strategy
gives us the set V .s _ i / D  ..q.sj1/; U.sj1//; : : : ; .q.s/; U.s//; .q.s _ i /; U.s _ i ///.
After we construct the point q.s _ i / and sets U.s _ i /; V .s _ i / for all s 2 ! n and
i 2 !, we will have a point q.s/ and sets U.s/; V .s/ for all s 2 nf;g with
l.s/  n C 1. It is straightforward that the conditions (10)(12) are satisfied if we
replace n with n C 1, so our inductive procedure can be continued to construct a
point q.s/ and sets U.s/; V .s/ for all s 2 nf;g in such a way that (10)(12) hold
for all n 2 N.
It follows from the properties (11) and (12) that if s 2 and l.s/ D n > 0
then diam.V .s//  diam.U.s//  2 3n < n1 . Given any u 2 ! ! , it follows from
(12) that f.q.uji /; U.uji //; V .uji / W i 2 Ng isTa play in the Choquet
game on X
T
in which E applies the strategy  and hence i2N V .uji / D i2N U.uji / ;.
This, together with the property (10) shows that the family V D fV .s/ W s 2 g is

an A-system on X and hence X is Cech-complete


by Fact 3 of S.491. This settles
necessity and finishes the proof of Fact 3.
To simplify the notation, let !  D !n!.

2 Solutions of problems 001500

601

Fact 4. The following conditions are equivalent for any ultrafilter  2 !  :


(i)  is a P -point in !  ;
(ii) for any countable family E  exp.!/n, there exists A 2  such that A \ E is
finite for any E 2 E;
(iii) for any countable disjoint family E  exp.!/n, there exists A 2  such that
A \ E is finite for any E 2 E.
Proof. It is evident that (ii)H)(iii). Assume that (iii) holds and  is not a P -point
in !  ; then there exists a  -compact set G  !  nfg such
S that  2 G. Choose a
family fKn W n 2 !g of compact subsets of !  such that n2! Kn D G. It S
is easy to
find a clopen set Un in the space !  such that Kn  S
Un  !  nfg. If U D n2! Un
then  2 U nU . Let V0 D U0 and VnC1 D UnC1 n. fUi W i  ng/ for any n 2 !.
It is S
easy to see that fVn W n 2 !g is a disjoint family of clopen subsets of !  such
that n2! Vn D U .
It is an easy exercise that there exists a clopen subset Wn of the space ! such
that Wn \ !  D Vn ; if Hn D Wn \ ! then cl! .Hn / \ !  D Vn for every n 2 !.
Assume for a moment that n m and the set B D Hn \ Hm is infinite. Then
F D cl! .B/ \ !  ; and F  Vn \ Vm D ; which is a contradiction. Therefore
the set Hn \ Hm is finite for any distinct n; m 2 ! and hence we can find a set
En  Hn such that jHn nEn j < ! for every n 2 ! and the family fEn W n 2 !g
is disjoint. It follows from Fact 1 of S.370 that cl! .En / \ !  D Vn ; in particular,
 cl! .En / and hence En  for each n 2 !.
Thus the disjoint family E D fEn W n 2 !g  exp.!/ contains no elements
of , so we can apply (iii) to find a set A 2  such that A \ En is finite for any
n 2 !. Apply Fact 1 of S.371 to see that the set Q D cl! .A/ \ !  does not meet
cl! .En / \ !  D Vn for any n 2 !. However, Q 2 .; !  / by Fact 1 of S.370 so
it follows from Q \ U D ; that  U ; this contradiction shows that (iii)H)(i).
To prove the implication (i)H)(ii) assume that the ultrafilter
 is a P -point in
S
!  and fix a countable
family
E

exp.!/n.
Then


fcl
.E/
W E 2 Eg and
!
S
therefore G D fcl! .E/ \ !  W E 2 Eg is an F -subset of !  such that  G.
The ultrafilter  being a P -point in !  we can find a set U 2 .; !  / such that
U \ G D ;. By Fact 2 of S.370 there exists A  ! with  2 cl! .A/ such that
cl! .A/ \ !   U . Then A 2  and it follows from cl! .A/ \ cl! .E/ \ !  D ;
that A \ E is finite for any E 2 E (see Fact 1 of S.371); this settles (i)H)(ii) and
shows that Fact 4 is proved.
Given an ultrafilter  2 !n!, the GulkoSokolov game on ! is played as
follows: at the n-th move player N U takes a set Sn 2 exp.!/n and player U
replies by choosing a finite set Tn  ! in such a way that the family fSi W i 
ng [ fTi W i  ng is disjoint. S
The game ends after the moves Sn ; Tn are made for all
n 2 ! and player U wins if fTn W n 2 !g 2 . Otherwise, the player N U is the
winner.
Fact 5. Player N U has a winning strategy in the GulkoSokolov game on ! if
and only if  is not a P -point in !n!.

602

2 Solutions of problems 001500

Proof. Assume first that  is not a P -point in !  and apply Fact 1 to find a disjoint
family E D fEn W n 2 !g  exp.!/n such that no element of  has a finite
intersection with all elements of E. Let S0 D E0 and  .;/ D S0 . If n 2 ! and
moves S0 ; TS
0 ; : : : ; Sn1 ; Tn1 are made in the GulkoSokolov game at ! then let
Sn D En n. fTi W i < ng/ and  .T0 ; : : : ; Tn1 / D Sn . This defines a strategy  of
the player N U on the space ! . If fSn ; Tn W n 2 !g is a play in whichS
N U applies
the strategy  then Sn  En and En nSn is finite for any n 2 !. If T D n2! Tn 2 
then, for some n 2 !, the set T \ En is infinite by our choice of the family E. This
implies that T \ Sn ; which is a contradiction. Therefore T , i.e., N U wins
every play in which he applies  , so  is a winning strategy of N U and hence we
proved sufficiency.
To establish necessity, assume that  is a winning strategy of the player N U on
the space ! and the ultrafilter  is a P -point of the space !  . Consider the family
E of all possible moves of the player N U made according to the strategy  . In other
words, E D f .;/g [ f .T0 ; : : : ; Tn / W n 2 ! and the .n C 1/-tuple .T0 ; : : : ; Tn /
constitutes the moves of U in some play fSi ; Ti W i 2 !g in which N U applies the
strategy  g. Since we only have countably many finite sequences of finite subsets of
!, the family E is countable. Furthermore, E  exp.!/n, so we can apply Fact 1
to see that there exists an element A 2  such that A \ E is finite for every E 2 E.
Let fai W i 2 !g be a faithful enumeration of the set A.
0
Let S0 D S00 D  .;/; we will also need the set T1
D ;. Passing to the set
An .;/ if necessary, we can assume, without loss of generality, that S0 \ A D ;.
Proceeding inductively assume that n 2 ! and we have families fSi W 0  i  ng
and fSi0 W 0  i  ng together with the families fTi W 0  i < ng and fTi0 W 1 
i < ng such that
(13) Sn D fS0 ; T0 ; : : : ; Sn1 ; Tn1 ; Sn g is an initial segment of a play in which N U
applies the strategy  ;
0
0
(14) Sn0 D fS00 ; T00 ; : : : ; Sn1
; Tn1
; Sn0 g is also an initial segment of a play in which
N U applies the strategy  ;
0
0
(15) if 0  i < n then fa0 ; : : : ; ai g  Mi D .T0 [ TS
0 / [ : : : [ .Ti [ Ti /;
0
(16) if 0  i  n then .S0 [ : : : [ Si / \ A  Li D SfTj W 1  j < i g;
(17) if 0  i < n then .S00 [ : : : [ Si0 / \ A  Ki D fTj W 0  j  i g.
The finite set F D Sn0 \ A is disjoint from the set Ln and we have the inclusion
.S0 [ : : : [ Sn / \ A  Ln by the property (16), so F is disjoint from S0 [ : : : [ Sn .
If an Mn1 then let Tn D fan g [ .F nKn1 /; if a 2 Mn1 then define the set
Tn to be equal to F nKn1 . The above observations show that, in both cases, Tn is
an admissible move for U to continue the play started in Sn . Besides, the condition
(17) is now satisfied for all i 2 f0; : : : ; ng.
The strategy  gives us the set SnC1 D  .T0 ; : : : ; Tn /; observe that SnC1 is
disjoint from Kn D T0 [: : :[Tn . Besides, .S00 [: : :[Sn0 /\A  Kn , so G D SnC1 \A
is disjoint from .S00 [: : :[Sn0 /. Therefore the set Tn0 D GnLn is a valid move for U to
continue the play started in Sn0 . The property (16) now holds for all i 2 f0; : : : ; nC1g
0
and it is evident that we also have (15) for i D n. Letting SnC1
D  .T00 ; : : : ; Tn0 /
0
0
0
we obtain families SnC1 D Sn [ fTn ; SnC1 g and SnC1 D Sn [ fTn0 ; SnC1
g together

2 Solutions of problems 001500

603

with the families fTi W 0  i  ng and fTi0 W 1  i  ng for which the properties
(13)(17) hold if we replace n with n C 1.
Therefore our inductive procedure can be carried out for every n 2 ! to construct
families S D fSn ; Tn W n 2 !g and S 0 D fSn0 ; Tn0 W n 2 !g such that the conditions
(13)(17) are satisfied for all n 2 !. It is clear that SSand S 0 are plays in
S which
N U applies the strategy  , so neither of the sets T D n2! Tn and T 0 D n2! Tn0
belongs to the ultrafilter . Therefore  cl! .T / and  cl! .T 0 / which shows
that  cl! .T [ T 0 / and hence T [ T 0 . However, it follows from (15) that
A  T [ T 0 ; since A 2 , we have T [ T 0 2 . This contradiction shows that 
cannot be a P -point, so we settled necessity and hence Fact 5 is proved.
Fact 6. Suppose that X is a space and F is closed subspace of X . Consider the
linear subspace I D ff 2 Cp .X / W f .F /  f0gg of the space Cp .X / and let
 W Cp .X / ! Cp .X nF / be the restriction map. Then the map p D jI is a linear
embedding of I in Cp .X nF /.
Proof. Let J D p.I /; we must prove that p W I ! J is a homeomorphism. The
map p is linear and continuous because so is the map . If f; g 2 I and f g
then there exists x 2 X nF such that f .x/ g.x/; therefore the point x witnesses
that p.f / p.g/ and hence p is a condensation.
To see that the map q D p 1 W J ! I is continuous, fix a function g0 2 J
and let f0 D q.g0 /; then g0 D p.f0 /. If U 2 .f0 ; I / then we can find a finite set
A  X and " > 0 such that the set V D ff 2 I W jf .x/  f0 .x/j < " for all x 2 Ag
is contained in U . If B D AnF then the set W D fg 2 J W jg.x/  g0 .x/j < " for
all x 2 Bg is an open neighborhood of g0 in J .
Given any function g 2 W let f D q.g/; then f j.X nF / D g and, in particular,
jf .x/  f0 .x/j D jg.x/  g0 .x/j < " for any x 2 B. Besides, f .x/ D f0 .x/ D 0
for any x 2 AnB which shows that jf .x/  f0 .x/j < " for any x 2 A, i.e., f 2 V .
This proves that q.W /  V  U , so the set W witnesses continuity of the map q
at the point g0 . Thus p is a homeomorphism and hence Fact 6 is proved.
Fact 7. Suppose that  2 !n! and consider the set I D ff 2 Cp .! / W f ./ D
0g; let  W Cp .! / ! Cp .!/ D R! be the restriction map. Then Cp .! /
.I /.
Proof. By Fact 1 it suffices to show that Cp .! / is linearly homeomorphic to I .
Take an infinite set A  ! such that !nA is also infinite; then exactly one of the
sets A and !nA belongs to . Renaming the relevant set if necessary, we can assume,
without loss of generality, that A .
It is clear that F D ff 2 I W f .! nA/ D f0gg is a linear subspace of I ; for any
f 2 I let r.f /.x/ D 0 if x 2 ! nA and r.f /.x/ D f .x/ for every x 2 A. Then
r W I ! F is a linear continuous map such that r.f / D f for any f 2 F , i.e., r is a
retraction. It follows from Problem 390 that I
F  M for some linear topological
space M . Since also, F
RA
R! , we conclude that I
R!  M and therefore
R  I
.R  R! /  M
R!  M
I . Recalling that R  I
Cp .! / (see
Fact 1 of S.409), we conclude that I
Cp .! / and hence Fact 7 is proved.

604

2 Solutions of problems 001500

Returning to our solution let Fn D fx 2 R! W jx 1 .Rnf0g/j  ng for any n 2 !.


We omit a simple proof that Fn is a S
closed nowhere dense subspace of  .R! /; it
!
follows from the equality  .R / D n2! Fn that the space  .R! / is of the first
category in itself. This shows that if  .R! / is homeomorphic to a closed subspace
of Cp .! / then Cp .! / is not hereditarily Baire, i.e., we proved that (ii)H)(iii).
Now assume that  is not a P -point in !n! and apply Fact 4 to find a disjoint
family fAn W n 2 !g of infinite subsets of ! such that An  for any n 2 !
and, for every U 2 , the set U \ An is infinite for some n 2 !. ChooseS
a faithful
enumeration fakn W k 2 !g of the set An for each n 2 !. SinceS!n. n2! An /
cannot belong to , there is no loss of generality to assume that n2! An D !.
For any f 2 R! let e.f / be the function from R! such that e.f /j! D f and
e.f /./ D 0. It follows from Fact 7 that the set C D ff 2 R! W e.f / 2 R! is
continuousg isS
homeomorphic to Cp .! /.
Let B D fAn W n 2 Ng. For any function f 2 RA0 consider a function
'.f / 2 RB defined as follows: '.f /.akn / D 0 whenever k 2 !; n 2 N and k < n;
if k  n then
P
(18) '.f /.akn / D jf .ak0 /j fjf .ai01 / : : : f .ai0n /j W 0  i1 < : : : < in  kg.
This gives us a map ' W RA0 ! RB and it is easy to see, using TFS-102, that ' is
continuous. Given a function f 2 RB let
P
(19) .f /.akn / D k fjf .ain /j W i  kg
for any n 2 N and k 2 !; this gives us a map  W RB ! RB and again it is
straightforward that  is continuous. Therefore the map  D  ' W RA0 ! RB is
continuous.
For each f 2 RA0 there exists a unique function uf 2 R! such that uf jA0 D f
and uf jB D .f /. An evident homeomorphism between RA0  RB and R! shows
that we can identify the set  D fuf W f 2 RA0 g with the graph of the map ,
so  is closed in R! . Furthermore, if we let q.f / D uf for any f 2 RA0 then
q W RA0 !  is a homeomorphism (see Fact 4 of S.390).
The set E D  \ C is closed in C ; we claim that E D q. .RA0 //. Indeed, take
any f 2  .RA0 /; there exists m 2 ! such that f .ak0 / D 0 for any k  m. Now,
if n > m and k  n then every summand in the expression for '.f /.akn / involves
a product of values of f at .m C 1/ distinct coordinates. Since one of those values
is bound to be zero, we conclude that '.f /.akn / D 0 for all n > m and k 2 !, i.e.,
'.f /jAn is identically zero for all n > m. As an immediate consequence, .f /jAn
is identically zero for all n > m.
S
Therefore the function uf is identically zero on the set A0 D fAi W i > mg;
since Ai  for all i  m, the set A0 belongs to  and hence the function e.uf / is
continuous on ! , i.e., q.f / 2 E. This shows that q. .RA0 //  E.
Now, if f 2 RA0 n .RA0 / then, for any n 2 N we can find positive integers
k1 < : : : < kn such that f .ak0i / 0 for all i  n. Then it follows from (18) that
'.f /.aknn /  r D jf .ak0n /j jf .ak01 /j : : : jf .ak0n /j > 0.

2 Solutions of problems 001500

605

Apply (19) to see that .'.f //.akn /  kr for any k > kn ; this proves that,
(20) for any n 2 N, we have .f /.akn / ! C1 as k ! 1.
If uf 2 C then there exists U 2  such that uf .U /  .1; 1/. Choose a number
n 2 N such that U \ An is infinite; then j.f /.akn /j < 1 for infinitely many k.
This, however, gives a contradiction with (20) and proves that q.RA0 n .RA0 // \
C D ; and hence q. .RA0 // D E which shows that E is a closed subspace of
C homeomorphic to  .RA0 /. Since C
Cp .! /, the space  .RA0 / embeds in
Cp .! / as a closed subspace, so we settled (i)H)(ii).
To show that (iii)H)(i) assume that Cp .! / is not hereditarily Baire and hence
C is not hereditarily Baire either. If f 2 C ; " > 0 and A is a finite subset of !
then let f; A; "
D fg 2 C W jf .n/  g.n/j < " for each n 2 Ag. It is clear that the
family ff; A; "
W A is a finite subset of ! and " > 0g is a local base of C at f .
Apply Fact 2 to see that we can choose a winning strategy  for the player E in
the Choquet game on the space C . Our plan is to construct a winning strategy s for
the player N U on the space ! .
If .f0 ; U0 / D  .;/ then let S0 D fn 2 ! W jf0 .n/j  1g. Since e.f0 / is
continuous and equals zero at , the set S0 does not belong to , so letting s.;/ D S0
we obtain a strategy s for the first move of the player N U .
If the player U chooses a finite set T0  !nS0 then jf0 .n/j < 1 for any n 2 T0
and therefore we can find "0 2 .0; 1/ together with a finite set A0  ! such that
T0  A0 and f0 ; A0 ; "0
 U while f0 .n/  "0 ; f0 .n/ C "0
 .1; 1/ for any
n 2 A0 nS0 . If V0 D f0 ; A0 ; "0
then the strategy  gives us a pair .f1 ; U1 / D  .V0 /,
so we can define the set S1 D fn 2 ! W jf1 .n/j  21 gn.S0 [ T0 /; let S1 D s.T0 /.
Proceeding inductively, assume that m 2 N and we have an initial segment Sm D
fS0 ; T0 ; : : : ; Sm1 ; Tm1 ; Sm g of a play in the GulkoSokolov game on ! and an
initial segment Um D f.f0 ; U0 /; V0 ; : : : ; .fm1 ; Um1 /; Vm1 ; .fm ; Um /g of a play in
the Choquet game on C with the following properties:
the player E applies the strategy  in Um ;
the player N U applies the strategy
S s in Sm ;
Si D fn 2 ! W jfi .n/j  2i gn j <i .Sj [ Tj / for any i 2 f1; : : : ; mg;
Vi D fi ; Ai ; "i
and "i 2 .0; 2i / for any i < m;
Ti [ Ai1  Ai for any i 2 f1; : : : ; m  1g;
fi .n/  "i ; fi .n/ C "i
 .fi1 .n/  "i1 ; fi1 .n/ C "i1 / for any n 2 Ai1
and i 2 f1; : : : ; m  1g;
(27) fi .n/  "i ; fi .n/ C "i
 .2i ; 2i / for any n 2 Ti and i < m.

(21)
(22)
(23)
(24)
(25)
(26)

The properties (22) and (24) show that .fm ; Um / D  .V0 ; : : : ; Vm1 / and, in
particular, fm .n/ 2 .fm1 .n/  "m1 ; fm1 .n/ C "m1 / for any n 2 Am1 . If Tm
is the move of the player U in the GulkoSokolov game then it follows from
(23) that we have the inequality jfm .n/j < 2m for any n 2 Tm . This makes
it possible to find a number "m 2 .0; 2m / and a finite set Am  ! such that
Am1 [ Tm  Am and fm ; Am ; "m
 Um while the properties (26) and (27) are

606

2 Solutions of problems 001500

fulfilled for i D m. Letting Vm D fm ; Am ; "m


we can apply the strategy  to
obtain the move .fmC1 ; UmC1 / D  .V0 ; : :S
: ; Vm / of the player E. Consider the set
SmC1 D fn 2 ! W jfmC1 .n/j  2m1 gn j m .Sj [ Tj /; it is easy to see that if
we let s.T0 ; : : : ; Tm / D SmC1 then the properties (21)(27) are now fulfilled for m
replaced with m C 1.
Therefore our inductive procedure can be continued to define a strategy s for
the player N U on the space ! . Assume that S D f.Sm ; Tm / W m 2 !g is a play
on ! in which N U applies s. The properties (21)(27) show that there is a play
f.fm ; Um /; Vm W m 2 !g on the space C in which E applies the strategy  and the
conditions (21)-(27)
are satisfied for any m 2 !.
S
S
Let A D i2! Ai ; it follows from (25) that T D i2! Ti  A. Given any
n 2 A the family E.n/ D ffi .n/  "i ; fi .n/ C "i
W i 2 !g consists of compact
decreasing
T sets whose diameters tend to zero; therefore there is a unique number
f .n/ 2
E.n/. Letting f .n/ D 1 for any n 2 T
!nA we obtain
T a function f W
! ! R. If f 2 C then it is easy to see that f 2 n2! Vn D n2! Un which is a
contradiction with the fact that  is a winning strategy for the player E. Therefore
f C , i.e., the function e.f / is discontinuous on ! .
It follows from (27) that the function f tends to zero on T ; this easily implies
that if T 2  then e.f / is continuous on ! which is a contradiction. Therefore
T  and hence N U is the winner in the play S. Thus s is a winning strategy for
N U on ! , so we can apply Fact 5 to see that  is not a P -point in !n!. This
proves that (iii)H)(i) and makes our solution complete.
V.493. Let ;  2 !n!. Prove that the spaces ! and ! are l-equivalent if and
only if there exists a bijection b W ! ! ! such that b./ D fb.U / W U 2 g D . In
particular, ! and ! are l-equivalent if and only if they are homeomorphic.
Solution. We will need the following set-theoretic fact known as Katetovs three
sets lemma.
Fact 1. Suppose that D is a nonempty set and f W D ! D is a map such that
f .x/ x for any x 2 D. Then there exist disjoint subsets A0 ; A1 ; A2 of the set D
such that D D A0 [ A1 [ A2 and f .Ai / \ Ai D ; for any i D 0; 1; 2.
Proof. Say that a set U  D is invariant if f .U /  U ; let us call U strongly
invariant if it is invariant and f 1 .U /  U . A set U  D will be called adequate if
there exist disjoint sets U0 ; U1 and U2 such that U D U0 [U1 [U2 and f .Ui /\Ui D
; for each i  2; we will call the family fU0 ; U1 ; U2 g an adequate decomposition
of U .
Suppose that U is an adequate set and fU0 ; U1 ; U2 g is its adequate decomposition.
Say that an adequate set V properly contains U if there exists an adequate
decomposition fV0 ; V1 ; V2 g of the set V such that Ui  Vi for all i  2.
Given any point x 2 D let x0 D x and, if xi is defined for some i 2 ! then let
xiC1 D f .xi /. The set F .x/ D fxi W i 2 !g is easily seen to be invariant; besides,
jF .x/j  2 for any x 2 D.
Fix a point x 2 D and assume first that F .x/ is finite, say F .x/ D fx0 ; : : : ; xk g.
Let U0 D fx0 g; U1 D fxi W i  k and i is oddg and U2 D fxi W 0 < i  k and i is

2 Solutions of problems 001500

607

eveng and observe that f .U0 /  U1 while f .U1 /  U0 [ U2 and f .U2 /  U0 [ U1 ,


i.e., the sets U0 ; U1 and U2 witness that F .x/ is an adequate set.
If F .x/ is infinite and hence fxi W i 2 !g is a faithful enumeration of the set
F .x/ then letting U0 D ;; U1 D fx2i W i 2 !g and U2 D fx2iC1 W i 2 !g we
convince ourselves that F .x/ is also an adequate set. This proves that
(1) for any point x 2 D there exists an adequate invariant set F  D such that
x 2 F.
Suppose that U is an invariant adequate set and fU0 ; U1 ; U2 g is its adequate
decomposition. Consider the map s W f0; 1; 2g ! f0; 1; 2g defined by s.0/ D
1; s.1/ D 2 and s.2/ D 0. If Vi D Ui [ .f 1 .Us.i/ /nU / then Ui  Vi for any
i  2. Given distinct i; j  2 we have Ui \ Uj D ; and Ui \ .f 1 .Us.j / /nU / D ;.
This, together with the equalities Uj \ .f 1 .Us.i/ /nU / D ; and f 1 .Us.i/ / \
f 1 .Us.j / / D ; implies that Vi \ Vj D ; for any distinct i; j  2.
It is immediate from the definition that the set Vi is contained in f 1 .U / for
every i  2, so V D V0 [ V1 [ V2  f 1 .U /. It is evident that U  V ; if x 2
f 1 .U /nU then f .x/ 2 Us.i/ and hence x 2 Vi for some i  2. This proves that
V D f 1 .U / and, in particular, V is invariant. Furthermore, f .Vi /  f .Ui / [ Us.i/
which, together with the equalities f .Ui / \ Ui D ; and Us.i/ \ Ui D ; shows that
f .Vi / \ Vi D ; for every i  2 and hence the set V is adequate. Therefore,
(2) for any invariant adequate set U the set V D f 1 .U / is also invariant and
adequate and U is properly contained in V .
Again, fix an invariant adequate set U  D and its adequate decomposition
fU0 ; U1 ; U2 g. Apply the property (2) inductively to obtain a sequence fVin W n 2 !g
for any i 2 f0; 1; 2g in such a way that
(3) V n D V0n [V1n [V2n is an invariant adequate set and fV0n ; V1n ; V2n g is its adequate
decomposition for any n 2 !;
(4) f 1 .U / D V0 and f 1 .V n / D V nC1 for any n 2 !;
(5) Ui  Vi0 and Vin  VinC1 for any n 2 ! and i 2 f0; 1; 2g.
S
n
LetSVi D
n2! Vi for each i 2 f0; 1; 2g; it is straightforward to see that
n
V D n2! V is a strongly invariant adequate set and fV0 ; V1 ; V2 g is its adequate
decomposition. Therefore it follows from (1) that
(6) for any x 2 D there exists a strongly invariant adequate set U  D such that
x 2 U.
Apply the property (6) to construct a maximal disjoint family U of nonempty
S
strongly invariant adequate subsets of D. We omit an easy proof that U D U is
a strongly invariant adequate subset of D. If D 0 D DnU ; then the property (6)
applied to the set D 0 and the map f jD 0 W D 0 ! D 0 gives us a nonempty set V  D 0
which is strongly invariant and adequate with respect to the map f jD 0 . Since D 0
is strongly invariant, the set V is strongly invariant with respect to the map f ; this
implies that U [ fV g is a disjoint family of strongly invariant
Sadequate subsets of D
which is a contradiction with maximality of U . Therefore U D D and hence D
is an adequate set, i.e., Fact 1 is proved.

608

2 Solutions of problems 001500

Returning to our solution say that ultrafilters  and  are equivalent if there exists
a bijection b W ! ! ! such that b./ D . Observe first that we have the equalities
.; ! / D fA [ fg W A 2 g and .; ! / D fB [ fg W B 2 g. Therefore, for any
bijection b W ! ! ! such that b./ D  we can define a map h W ! ! ! letting
h./ D  and h.n/ D b.n/ for any n 2 !; then h. .; ! // D .; ! / and hence h
is a homeomorphism.
l

On the other hand, if ! is homeomorphic to ! then !  ! ; take any


homeomorphism h W ! ! ! and observe that we must have h./ D , so b D hj!
is easily seen to be a bijection which takes the elements of  onto the elements of .
Therefore the ultrafilters  and  are equivalent if and only if ! is homeomorphic
to ! .
From now on we assume that ! is l-equivalent to ! ; we already saw that it
suffices to prove that the ultrafilters  and  are equivalent. For any function f 2 R!
let e .f /.n/ D f .n/ for each n 2 ! and e .f /./ D 0. It follows from Fact 2
of V.489 that the set C D ff 2 R! W e .f / is continuous on ! g is linearly
homeomorphic to Cp .! /. Analogously, let e .f /.n/ D f .n/ for every n 2 ! and
e .f /./ D 0 for any f 2 R! . Applying Fact 2 of V.489 again we conclude that
the space C D ff 2 R! W e .f / is continuous on ! g is linearly homeomorphic to
Cp .! /.
Given any n 2 ! let n .f / D f .n/ for any f 2 R! ; then n W R! ! R
is a continuous linear functional on R! . Now assume that ' W C ! R is a
nontrivial continuous linear functional. By Problem 224, there exists a continuous
linear functional W R! ! R with jC D '. We can apply TFS-197 to see
that D r1 k1 C : : : C rn kn for some n 2 N, r1 ; : : : ; rn 2 Rnf0g and distinct
k1 ; : : : ; kn 2 !. The set C being dense in R! , the functional i is the unique
extension of every i jC over R! . This makes it possible to identify i jC with i , so,
to simplify our notation, we will write that ' D r1 k1 C : : : C rn kn . Analogously, if
 W C ! R is a nontrivial continuous linear functional then there exists m 2 N such
that  D s1 l1 C: : :Csm lm for some s1 ; : : : ; sm 2 Rnf0g and distinct l1 ; : : : ; lm 2 !
(here we identify li jC with li for any i  m).
Fix a linear homeomorphism T W C ! C . For any n 2 ! the map n T is a
nontrivial continuous linear functional
on C , so there exists a nonempty finite set
P
P .n/  ! such that n T D m2P .n/ rmn m where rmn 0 for any m 2 P .n/.
Analogously, there exists a nonempty finite set Q.n/  ! such that n T 1 D
P
m2Q.n/ smn m where smn 0 for all m 2 Q.n/. The following property is a key
to our proof.
(7) Suppose that A 2 ; B  ! and P .n/ \ B ; for any n 2 A; if, additionally,
RK D fn 2 A W P .n/ \ B  Kg is finite for any finite K  B then B 2 .
Assume that B ; we are going to construct a function f 2 C . Let f .n/ D 0
for any n 2 !nB; it follows from !nB 2  that f will belong to C no matter
what values it will take on B. It follows from (7) that B is infinite, so we can choose
a faithful enumeration fni W i 2 !g of the set B; let Bi D fn0 ; : : : ; ni g for every
i 2 !.

2 Solutions of problems 001500

609

If RB0 D ; then let f .n0 / D 0; otherwise choose a number f .n0 / such that
jrn0 n f .n0 /j  1 for any n 2 RB0 . Observe that it is possible because the set RB0 is
finite by (7) and rn0 n 0 for each n 2 RB0 .
Proceeding inductively assume that j 2 ! and we have defined f .ni / for any
number i  j . If the set RBj C1 nRBj is empty then let f .nj C1 / D 0. Otherwise,
choose f .nj C1 / in such a way that
P
. / j m2P .n/\B rmn f .m/j  1 for all n 2 RBj C1 nRBj .
This is possible because nj C1 2 P .n/ \ B while there are only finitely many
elements in RBj C1 nRBj and rnj C1 n 0 for any n 2 RBj C1 nRBj which shows that
f .nj C1 / can be chosen sufficiently big to guarantee that the absolute value of all
the sums in . / is greater than or equal to 1.
Therefore our inductive procedure can be continued to construct a function f 2
R! such that f .!nB/ D f0g and the property . / holds for all j 2 !. We already
saw that f 2 C and hence g D T .f / 2 C .
P Take any n 2 A; we have g.n/ D n .g/ D .n T /.f / D
m2P .n/ rmn
Pm .f /. However, the function f is identically zero on !nB, so
g.n/ D
m2P .n/\B rmn f .m/. If j D maxfk W nk 2 P .n/ \ Bg then
n 2 RBj nRBj 1 if j > 0 and n 2 RB0 if j D 0. Therefore it follows from
. / or the choice of f .n0 / respectively, that jg.n/j  1; since we checked this for
any n 2 A, the function g cannot belong to C . This contradiction shows that (7)
is proved. It is important to see that an analogous proof shows that we have the
counterpart of (7) if we interchange  and , namely,
.70 / suppose that B 2 ; A  ! and Q.n/ \ A ; for any n 2 B; if, additionally,
the set fn 2 B W Q.n/ \ A  Kg is finite for any finite K  A then A 2 .
Next assume that A 2  and we have a nonempty set P0 .n/  P .n/ for each
n 2 A. Given a set B  ! with P0 .n/ \ B ; for every n 2 A, it is an immediate
consequence of (7) that
(8) if the set fn 2 A W m 2 P0 .n/g is finite for any m 2 B then B 2 .
We will also need the version of (8) obtained by interchanging  and . That is,
assume that B 2  and we have a nonempty set Q0 .n/  Q.n/ for each n 2 B.
Given a set A  ! with Q0 .n/ \ A ; for every n 2 B, it is an immediate
consequence of .70 / that
.80 / if the set fn 2 A W m 2 Q0 .n/g is finite for any m 2 A then A 2 .
We will apply the property (8) to show that our task can be reduced to establishing
the following statement.
(9) If there exists a set E 2  for which we can find an injection ' W E ! ! such
that '.n/ 2 P .n/ for any n 2 E then the ultrafilters  and  are equivalent.
Choosing a smaller element of  if necessary, we can assume, without loss of
generality, that the sets !nE and !n'.E/ are infinite. Take an arbitrary bijection
'0 W !nE ! !n'.E/ and define a map h W ! ! ! by the equalities h.n/ D '.n/
if n 2 E and h.n/ D '0 .n/ for any n 2 !nE.

610

2 Solutions of problems 001500

It is clear that the map h is a bijection; fix an arbitrary element C 2  and let
P0 .n/ D f'.n/g for any n 2 A D C \ E. The map ' being injective it is easy to
see that (8) is applicable to the sets A and B D '.A/ D h.A/, so B 2 . It follows
from h.C /  B that h.C / 2 . This shows that h.C / 2  for any C 2 , i.e.,
h./  . To see that h./ D  take any element B 2 ; if A D h1 .B/  then
A0 D !nA 2  and hence !nB D h.A0 / 2  which is a contradiction. Therefore
A 2  which shows that B D h.A/ 2 h./. This proves that h./ D  and hence
the ultrafilters  and  are equivalent, i.e., (9) is proved.
For each n 2 ! consider the sets P0 .n/ D fm 2 P .n/ W n 2 Q.m/g and
Q0 .n/ D fm
2 P .m/g. It follows from the equality n D n T T 1
P2 Q.n/ W n P
that n D
m2P .n/ rmn
k2Q.m/ skm k ; the family fi W i 2 !g being linearly
independent, there exists m 2 P .n/ such that n 2 Q.m/ and hence m 2 P0 .n/.
Therefore the set P0 .n/ is nonempty for any n 2 !. Analogously, Q0 .n/ ; for
every n 2 !.
Next observe that the family P D fP0 .n/ W n 2 !g is point-finite because for
any m 2 ! if m 2 P0 .n/ then n 2 Q.m/ and the set Q.m/ is finite. An analogous
reasoning shows that the family Q D fQ0 .m/ W m 2 !g is also point-finite.
It will be easy to finish our proof after we establish the following property of the
families P and Q.
(10) The set E D fn 2 ! W Q0 .m/ D fng for any m 2 P0 .n/g belongs to the
ultrafilter .
Note first that it is straightforward from the definition that n 2 Q0 .m/ for any
m 2 P0 .n/. If the property (10) does not hold then there exists an element A 2 
such that, for any n 2 A we can choose a number ln 2 P0 .n/ such that there exists
.n/ 2 Q0 .ln /nfng. For every n 2 !nA choose a number .n/ n arbitrarily.
This gives a map W ! ! ! such that .n/ n for all n 2 !.
By Fact 1 we can find disjoint sets A0 ; A1 ; A2 such that ! D A0 [ A1 [ A2
and .Ai / \ Ai D ; for any i 2 f0; 1; 2g. It is easy to see that one of the sets
A0 \ E; A1 \ E; A2 \ E belongs to ; denote this set by W . It follows from the
choice of the sets Ai that .W / \ W D ;.
Consider the set W 0 D fln W n 2 W g; then ln 2 P0 .n/ \ W 0 , so P0 .n/ \ W 0 ;
for any n 2 W and hence we can apply the property (8) to see that W 0 2 . If
W 00 D .W / then .n/ 2 Q0 .ln / \ W 00 and hence Q0 .ln / \ W 00 ; for any
n 2 !, so we can apply .80 / to conclude that W 00 2 . Thus, W 2 ; W 00 2  and
W \ W 00 D ; which is a contradiction, so the property (10) is proved.
Finally, choose an element '.n/ 2 P0 .n/ for any n 2 E. Given distinct n; m 2 E
if '.n/ D '.m/ D k then fm; ng  Q0 .k/ which is a contradiction with the choice
of E. Therefore '.n/ '.m/ for any distinct m; n 2 E, i.e., the map ' W E ! !
is an injection, so we can apply (9) to see that the ultrafilters  and  are equivalent
and hence our solution is complete.
V.494. Suppose that and are ordinals such that !   < !1 . Prove that
. C 1/ is l-equivalent to . C 1/ if and only if < ! . Deduce from this fact that
there exist countable compact u-equivalent spaces which are not l-equivalent.

2 Solutions of problems 001500

611

Solution. The expression Y ' Z says that the spaces Y and Z are homeomorphic.
If L and M are linear topological spaces then we write L
M if L and M are
linearly homeomorphic. Given a space Z let I.Z/ be the set of isolated points
of Z and D0 .Z/ D Z. If  is an ordinal and we have D .Z/ then DC1 .Z/ D
D .Z/nI.DT
 .Z//; if  is a limit ordinal and we have D .Z/ for any  <  then
D .Z/ D < D .Z/. The set D .Z/ will be called the -th derivative of the
space Z. If Y is a space and A  Y is a closed subspace of Y then YA is the space
obtained from Y by contracting the set A to a point. If ' W X ! Y is a continuous
map then let '  .f / D f ' for any f 2 C.Y /; then '  W C.Y / ! C.X / is called
the dual map of the map '.
Given two linearly ordered spaces L and M we say that they are canonically
homeomorphic if there exists a homeomorphism f W L ! M which is also an
order isomorphism between L and M . If <L and <M are the orders on L and M
respectively then the lexicographic order < on L  M is defined as follows: for any
a; b 2 L  M with a D .a1 ; a2 / and b D .b1 ; b2 / we declare that a < b if either
a1 <L a2 or a1 D a2 and b1 <M b2 . It is an easy exercise that the lexicographic
order on the product of two well-ordered sets is also a well-order.
If K is a nonempty countable compact space then K is scattered. Therefore, if 
is an ordinal and D .K/ ; then DC1 .K/ is a proper closed subset of D .K/.
Therefore D .K/ has to be empty for some countable ordinal ; it is easy to see that
 D nnf < !1 W D .K/ D ;g is a successor ordinal; the predecessor of  is called
the dispersion index of K and is denoted by d i.K/. In other words, an ordinal is
the dispersion index of a countable compact space K if D nnf < !1 W D .K/ is
finiteg. An easy proof by transfinite induction shows that, for any countable scattered
compact space K,
(1) if F is a closed subset of K then D .F /  D .K/ \ F ; if the set F is clopen
in K then D .F / D D .K/ \ F for any ordinal .
Say that an infinite ordinal  is a prime component if  C  <  for any ordinals
 and  such that  <  and  < . Given ordinals  and  with    let
; 
D f W     g; all other ordinal intervals are defined analogously. In
particular, for an ordinal  we will often denote the space  C 1 by 0; 
.
Fact 1. Suppose that a well-ordered set A is isomorphic to an ordinal  and a wellordered set B is isomorphic to an ordinal . Consider the set
C D C.A; B/ D .f0g  A/ [ .f1g  B/
with the following order: if c1 ; c2 2 C and ci D .ai ; bi / for each i 2 D then say that
c1 < c2 if either a1 < a2 (which actually means that a1 D 0 and a2 D 1) or a1 D a2
and b1 < b2 (observe that b1 < b2 makes sense because a1 D a2 implies that either
fb1 ; b2 g  A or fb1 ; b2 g  B). Then C with the described order is isomorphic to
 C . In other words, to obtain the sum of two well-ordered sets A and B we must
place all elements of B after all elements of A.

612

2 Solutions of problems 001500

Proof. There is no loss of generality to consider that A D  and B D . Therefore


C D C.; / D .f0g  / [ .f1g  /. We will prove our Fact by induction on . If
 D 0 then C D f0g   is isomorphic to  D  C .
Assume that  is an ordinal and we proved that C.; / is isomorphic to  C 
for all  < . If  D  C 1 for some ordinal  then C.; / D C.; / [ f.1; /g
and .1; / is the greatest element of C.; /. By the induction hypothesis, C.; / is
isomorphic to  C, so C.; / has the same order type as . C/C1 D  C.C1/,
so C.; / is isomorphic to  C .
If  is a S
limit ordinal then we have the equalities  C D supf C W  < g and
C.; / D fC.; / W  < g. By the induction hypothesis, C.; / is isomorphic
to  C  and hence C.; / <  C  for any  < . As a consequence, the order type
of C.; / does not exceed  C . Besides, if the order type of C.; / is strictly
less than  C  then it is strictly less than  C  for some  < . However,  C 
is isomorphic to C.; /  C.; /, so the order type of  C  does not exceed the
order type of C.; /; this contradiction shows that C.; / is isomorphic to  C ,
so we carried out the induction step and hence Fact 1 is proved.
Fact 2. If  is a prime component then  is a limit ordinal; besides, if  <  then
 C  D  and hence the space 0; 
0; 
is homeomorphic to 0; 
.
Proof. If  D  C 1 then the ordinals  and  D 1 witness that  is not a prime
component; this contradiction shows that  has to be a limit ordinal. Therefore  
 C  D supf C  W  < g   because  C  <  for any  < . Consequently,
 C  D ; it is easy to deduce from Fact 1 that 0;  C 
is homeomorphic to
0; 
1; 
' 0; 
0; 
, so 0; 
' 0; 
0; 
, i.e., Fact 2 is proved.
Fact 3. The ordinal !  is a prime component for any countable ordinal  > 0.
Proof. If  D 1 then !  D ! is a prime component because m C n < ! whenever
m < ! and n < !. Suppose that  > 1 is an ordinal and we proved that !  is
a prime component for any  < . Assume first that  is a limit ordinal and we
have ordinals ; 0 such that D maxf; 0 g < !  . There exists  <  for which
< !  ; by the induction hypothesis,  C 0 < !   !  , so  C 0 < !  .
Now, if  D  C 1 then take any ordinals ; 0 such that D maxf; 0 g < ! 
and observe that !  D !  ! D supf!  n W n 2 !g. Pick n 2 ! such that < !  n;
then  C 0  !  n C !  n D !  .2n/ < !  . Thus !  is also a prime component,
so our inductive procedure shows that !  is a prime component for any  2 1; !1 /,
i.e., Fact 3 is proved.
Fact 4. For any countably infinite ordinal  < !1 there exists a unique ordinal
 > 0 such that !    < ! C1 .
Proof. It is easy to prove by transfinite induction that !    for any ordinal . In
particular, ! C1   C 1 > , so we can define the ordinal  0 D nnf W !  > g.
0
0
If  0 is a limit ordinal then !  D supf!  W  <  0 g, so it follows from  < !  that
 < !  for some  <  0 which is a contradiction. Therefore  0 D  C 1 for some
ordinal , so, by our choice of  0 , we have !    < ! C1 as promised.

2 Solutions of problems 001500

613

Suppose that !   < ! C1 and ; if >  then  C1, so !  ! C1 >


 which is a contradiction. If <  then C 1   and therefore ! C1  !   ;
this contradiction shows that D , so  is uniquely determined and hence Fact 4 is
proved.
Fact 5. If  is a countable ordinal then D .1; ! 
/ D f!  g.
Proof. If  D 0 then !  D 1, so D0 .1; ! 
/ D 1; 1
D f!  g. Proceeding by
induction assume that 0 <  S
< !1 and D .1; ! 
/ D f!  g for any  < . If  is a

limit ordinal then 1; !
D f1; ! 
W  < g [ f!  g. Given any  < , the set
1; ! 
is clopen in 1; ! 
, so D .1; ! 
/ \ 1; ! 
D D .1; ! 
/ D f!  g by the
induction hypothesis. As a consequence, D .1; ! 
/ \ 1; ! 
D D .1; ! 
/ D ;


for any  <  and hence
T D .1; ! 
/  f! g. On the other hand, we have the

equality D .1; !
/ D fD .1; !
/ W  < g; the family D D fD .1; !
/ W
 < g is decreasing and we saw that !  2 D .1; ! 
/  D .1; ! 
/ and hence
T all

elements of D are nonempty.
By
compactness
of
the
space
1;
!

we
have
D
T
;, so D .1; ! 
/ D D D f!  g.
If  D  C 1 for some
S countable ordinal  then it follows from the equalities
1; ! 
D 1; !  !
D f1; !  n
W n 2 !g [ f!  g that there is a family
fBn W n 2
S
!g of disjoint clopen subsets of 1; ! 
such that 1; ! 
D f!  g [ fBn W n 2 !g
and Bn ' 1; ! 
for every n 2 !. The set D .1; ! 
/\Bn D D .Bn / is a singleton
by the induction hypothesis; fix a point yn such that D .1; ! 
/ \ Bn D fyn g for
every n 2 !. As a consequence, the set D .1; ! 
/ D f!  g [ fyn W n 2 !g is a
convergent sequence with limit !  . Therefore D .1; ! 
/ D D1 .D .1; ! 
// D
f!  g, so our inductive procedure shows that D .1; ! 
/ D f!  g for any  < !1
and hence Fact 5 is proved.
Fact 6. Given a countable ordinal , a countable compact space K is homeomorphic to 1; ! 
if and only if jD .K/j D 1.
Proof. Since necessity is immediate from Fact 5, let us prove sufficiency by
induction on . If  D 0 and D .K/ D K is a singleton then !  D 1 and hence K
is homeomorphic to f1g D 1; ! 
.
Assume that  > 0 is a countable ordinal and we proved that, for any countable
compact space K, if  <  and jD .K/j D 1 then K ' 1; ! 
. Suppose first that 
is a limit ordinal and fix a countable compact space K such that D .K/ D fag for
some point a 2 K.
Choose a family fOn W n 2 !gTof clopen subsets of K such that O0 D K while
UnC1  Un for each n 2 ! and n2! Un D fag. If Pn D On nOnC1 then Pn is a
clopen compact subset of Knfag for any n 2 !.
Apply (1) to see that D .Pn / D D .K/ \ Pn D ; and hence n D d i.Pn / < 
for each n 2 !. Given any n 2 ! the set An D Dn .Pn / is finite, so it is easy to
find a clopen disjoint finite cover On of the space Pn such that jDn .O/j D 1 for
every O 2 On . By the induction hypothesis, every O 2 On is homeomorphic
to
S
1; ! n
; if we choose an enumeration fUk W k 2 !g of the family n2! On then
Uk ' 1; ! k
for every k 2 !.

614

2 Solutions of problems 001500

Using Fact 1 of V.199 it is easy to construct a sequence fk W k 2 !g of countable


ordinals with the following properties:
(2) i < iC1 for each i 2 !;
(3) 0 D ! 0 and the interval BiC1 D i C 1; iC1
is canonically homeomorphic
to 1; ! i C1
for every i 2 !.
Let B0 D 1; ! 0
; the ordinal i being a finite sum of ordinals which are all
strictly smaller than !  , it follows from Fact 2 and Fact 3 that i < !  for all i 2 !.
Therefore the ordinal  D supfi W i 2 !g does not exceed !  . On the other hand,
if  <  then DC1 .K/ is an infinite set, so DC1 .K/ \ Uk ; for some k 2 !.
Consequently, d i.Uk / D k   C 1 >  which shows that the order type of
1; kC1
is greater than or equal to ! k  ! C1 > !  . Thus   !  for any  < ;
since !  D supf!  W  < g, we conclude that   !  and hence  D !  .
It follows from (2) and (3) that fBi W i 2 !g is a clopen compact cover of the


space
L 1; ! / and therefore 1; !
is the one-point compactification of the space
i2! Bi . The family fUi W i 2 !g is a clopen
L compactLcover of Knfag, so K is
a one-point compactification of the space i2! Ui '
i2! Bi . This shows that
K ' 1; ! 
and hence we carried out the induction step for any limit ordinal .
Now assume that  is a successor and fix an ordinal  such that  D C1. For the
space Y D D .K/ the set D1 .Y / D D .K/ D fag is a singleton, so Y is a compact
space with a unique non-isolated point a, i.e., Y is a sequence which converges to
a. It is easy to find a decreasing
sequence O D fOn W n 2 !g of clopen subsets of K
T
such that O0 D K while O D fag and .On nOnC1 /\.Y nfag/ ; for any n 2 !.
Splitting every On nOnC1 into a finitely many disjoint clopen subsets if necessary,
we obtain a disjoint clopen compact cover fWn W n 2 !g of the space Y nfag such
that Wn \ Y is a singleton; let an 2 Y nfag be the point for which Wn \ Y D fan g
for each n 2 !.
For each n 2 ! we have the equalities D .Wn / D D .K/ \ Wn D fan g, so
Wn ' 1; ! 
by theL
induction hypothesis. Therefore the space K is the one-point
compactification
of
homeomorphic to 1; ! 
 !. Observe that
n2! Wn which is S
S


1; ! / D f1; ! n
W n 2 !g D n2! !  n C 1; !  .n C 1/
and the set
!  n C 1; !  .n C 1/
is clopen in 1; ! 
and homeomorphic to 1; ! 
, so 1; !  /
is homeomorphic to 1; ! 
 ! and hence 1; ! 
is the one-point compactification
of 1; ! 
 !. Therefore K is homeomorphic to 1; ! 
, i.e., we completed the
induction step and hence Fact 6 is proved.
Fact 7. Suppose that K is a countable compact space and A D D .K/ ; for
some countable ordinal . If a is the point represented by A in the space KA then
D .KA / D fag.
Proof. If x 2 KnA then take a clopen set U 2 .x; K/ such that U \ A D ;. It
follows from (1) and D .K/\U D ; that D .U / D ; and hence D .KA /\U D ;;
in particular, x D .KA /. Therefore D .KA /  fag; to see that a 2 D .KA /
observe that K is scattered, so we can find an isolated point b in the space A and a

2 Solutions of problems 001500

615

clopen set V such that V \ A D fbg. The set V 0 D .V nfAg/ [ fag  KA is the
one-point compactification of the space V nA D V nfbg which shows that V is also
the one-point compactification of V nA.
Consequently, V ' V 0 ; since b 2 D .K/\V D D .V / we must have D .V 0 /
; and hence D .KA / ;. Therefore D .KA / D fag, i.e., Fact 7 is proved.
Fact 8. If K and L are countable compact spaces such that d i.L/ < d i.K/ then
K L ' K. In particular, if K is infinite and L is finite then K L ' K.
Proof. Let  D d i.K/; since D .K/ ;, we can find n 2 N and distinct points
a1 ; : : : ; an 2 K such that D .K/ D fa1 ; : : : ; an g. There exist disjoint clopen sets
U1 ; : : : ; Un for which K D U1 [ : : : [ Un and ai 2 Ui for every i  n. We have the
equality D .Ui / D fai g and hence Ui ' 1; ! 
for each i  n by Fact 6. Observe
that K ' U1 : : : Un and D .U1 L/ D D .U1 / D .L/ D D .U1 / D fa1 g
which shows that U1 L ' 1; ! 
' U1 by Fact 6. As an immediate consequence,
K L ' .U1 L/ : : : Un ' U1 : : : Un ' K, i.e., Fact 8 is proved.
Fact 9. If K and L are countably infinite compact spaces and d i.K/ D d i.L/ then
l

K  L.
Proof. If  D d i.K/ D d i.L/ then D .K/ and D .L/ are nonempty finite sets. We
can find n 2 N and distinct points a1 ; : : : ; an 2 K such that D .K/ D fa1 ; : : : ; an g.
There exist disjoint clopen sets U1 ; : : : ; Un for which K D U1 [: : :[Un and ai 2 Ui
for every i  n. Then D .Ui / D fai g and hence Ui ' 1; ! 
for each i  n by
Fact 6.
The set F D fa1 ; : : : ; an g is easily seen to be a retract of K, so K is l-equivalent
to the space KF F (see Problem 373). Since KF is infinite and F is finite, we can
apply Fact 8 to convince ourselves that KF F ' KF . It follows from Fact 6 and
Fact 7 that D .KF / is a singleton and therefore KF ' 1; ! 
. This shows that the
space K is l-equivalent to 1; ! 
. Since we can repeat the same reasoning for L, we
l

conclude that K 1; ! 


 L, so K  L and hence Fact 9 is proved.
Fact 10. If !   < !1 and  is a prime component then 1; 
is l-equivalent to
1; 
for any ordinal  such that    < ! .
Proof. Take the ordinal  > 0 such that  D !  and let K D 1; 
. The set
F D D .K/ is nonempty because D .1; ! 
/ D fg  F (see Fact 5). Since
! D supfn W n 2 !g, there exists a number n 2 N such that n   < nC1 . We
will prove our Fact by induction on n.
If n D 1 then !    < .!  /2 D ! 2 . Observe that K is a clopen subspace
of the space 1; ! 2
and hence we can apply (1) together with Fact 5 to see that
D2 .K/ D D2 .1; ! 2
/ \ K D f! 2 g \ K D ;. Therefore D .F / D D2 .K/ D
;; this shows that d i.F / <  D d i.1; ! 
/ and hence 1; ! 
F ' 1; ! 
by
Fact 8.
The space KF F is l-equivalent to K by Problem 373; besides, KF ' 1; ! 

by Fact 7 and Fact 6. Therefore K  KF F 1; ! 


F ' 1; ! 
and hence we
settled the case of n D 1.

616

2 Solutions of problems 001500

Now assume that n   < nC1 for some number n > 1 and we have proved
l

that 1;
1; 
for any ordinal such that k  < kC1 for some k < n. Since
K is a clopen subspace of 1; ! .nC1/
, we can apply the property (1) together with
Fact 5 to see that D.nC1/ .K/ D D.nC1/ .1; ! .nC1/
/ \ K D f! .nC1/ g \ K D ;.
Therefore Dn .F / D D.nC1/ .K/ D ;; this shows that d i.F / <  n.
The space KF F is l-equivalent to K by Problem 373; besides, KF ' 1; ! 

by Fact 7 and Fact 6. There exists an ordinal such that F ' 0;


(see
Problem 199). If <  then d i.F / <  and hence F 1; ! 
' 1; ! 
by
l

Fact 8. Therefore K  F 1; ! 
' 1; ! 
as promised.
If d i.F /   then   and it follows from d i.F / <  n that < ! n .
Observe also that 1;
' 1;
f0g ' 0;
' F by Fact 8. Since < ! n , there
exists k < n such that ! k  < ! .kC1/ , i.e., k  < kC1 , so our induction
l

hypothesis can be applied to see that F ' 1;


1; 
. As a consequence,
l

K  KF F ' 1; 
1;
1; 
1; 
. Besides, d i.1; 
1; 
/ D
l

 D d i.1; 
/, so we can apply Fact 9 to conclude that K 1; 
1; 
1; 
;
this completes our induction step and shows that Fact 10 is proved.
Fact 11. If .B1 ; jj jj1 / and .B2 ; jj jj2 / are normed spaces then B1
B2 if and
only if there exists a surjective linear map ' W B1 ! B2 such that, for some number
K > 0, we have jjxjj1  jj'.x/jj2  Kjjxjj1 for any x 2 B1 .
Proof. Let 01 and 02 be the zero vectors in the spaces B1 and B2 respectively.
Suppose first that ' W B1 ! B2 is a surjective linear map as in the hypothesis
of our Fact. If x 2 B1 and '.x/ D 01 then jjxjj1  jj'.x/jj2 D 0, so jjxjj1 D 0
and hence ' is injective. For any r > 0 let S1 .r/ D fx 2 B1 W jjxjj1  rg and
S2 .r/ D fx 2 B2 W jjxjj2  rg.
The second inequality of our hypothesis implies that '.S1 .1//  S2 .K/, so the
map ' is continuous by Fact 2 of V.400. If  W B2 ! B1 is the inverse of ' then the
first inequality of our hypothesis shows that jj.y/jj1  jjyjj2 because, for x D .y/
we have y D '.x/ for any y 2 B2 . Therefore .S2 .1//  S1 .1/ and hence  is also
continuous by Fact 2 of V.400. It turns out that ' is a homeomorphism, so we settled
sufficiency.
Now assume that  W B1 ! B2 is a linear homeomorphism and let  W B2 ! B1
be its inverse. By Fact 2 of V.400 there exists R > 0 such that .S2 .1//  S1 .R/.
Let '.x/ D R .x/ for any x 2 B1 . It is immediate that ' W B1 ! B2 is a surjective
1
linear map. If x 2 B1 nf01 g and y D '.x/ D R .x/ then jjyjj
y belongs to S2 .1/
2
1
and therefore jjyjj2 jj.y/jj1  R. Observe that .y/ D .R .x// D R x, so we
1
have jjyjj
R jjxjj1  R, so jjxjj1  jjyjj2 D jj'.x/jj2 and hence we have the first
2
of the promised inequalities for '.
The map ' being continuous, we can apply Fact 2 of V.400 once more to see
1
that '.S1 .1//  S2 .K/ for some K > 0. If x 2 B1 nf01 g then y D jjxjj
x 2 S1 .1/
1

2 Solutions of problems 001500

617

1
and hence jj'.y/jj2  K, i.e., jjxjj
jj'.x/jj2  K which shows that we have the
1
inequality jj'.x/jj2  Kjjxjj1 for every x 2 B1 and hence we established necessity,
so Fact 11 is proved.

Fact 12. Suppose that .B1 ; jj jj1 / and .B2 ; jj jj2 / are normed spaces and a linear
continuous map ' W B1 ! B2 is not a topological embedding (i.e., ' is not a
homeomorphism between B1 and '.B1 /). Then, for any " > 0 there exists x 2 B1
such that jjxjj1 D 1 and jj'.x/jj2 < ".
Proof. If ' is not injective then there exists x 0 2 B1 nf01 g such that '.x 0 / D 02 .
Then x D jjx10 jj1 x 0 is as promised. If ' is injective then let C D '.B1 / and consider
the inverse map  W C ! B1 for the map '. The map  cannot be continuous, so
there exists a sequence fyn W n 2 !g  C nf02 g such that yn ! 02 and f.yn / W n 2
!g does not converge to 01 .
Passing to a subsequence if necessary, we can assume that there is > 0 such that
jj.yn /jj1  ; let xn D jj.y1n /jj1 .yn / for all n 2 !. Then, for every n 2 ! we have
jjxn jj1 D 1, so it follows from jj'.xn /jj2  1 jjyn jj2 that the sequence fjj'.xn /jj2 W
n 2 !g converges to zero. Thus there exists n 2 ! for which jj'.xn /jj2 < ", i.e.,
x D xn is as promised and hence Fact 12 is proved.
Fact 13. Suppose that K be an infinite countable compact space and a 2 K is a
non-isolated point of K. Let I D ff 2 Cu .K/ W f .a/ D 0g. Then the Banach space
I (with the norm inherited from Cu .K/) is linearly homeomorphic to Cu .K/.
Proof. Recall that jjf jj D supfjf .x/j W x 2 Kg for any f 2 Cu .K/ and the
topology of Cu .K/ is generated by the norm jj jj. Given a point b K, the space
fbg K is homeomorphic to K by Fact 8. Therefore we can find an isolated point
z 2 K such that Q D Knfzg ' K. To simplify the notation we denote the norms in
I and Cu .Q/ by the same symbol jj jj.
For every f 2 I let .f /.x/ D f .x/ C f .z/ for any x 2 Q. This defines a linear
map W I ! Cu .Q/. If g 2 Cu .Q/ then let f .x/ D g.x/  g.a/ for all x 2 Q and
f .z/ D g.a/. It is straightforward that .f / D g, so the map is surjective. It is
immediate that jj.f /jj  2jjf jj for any f 2 I . We claim that also
(4) jjf jj  2jj.f /jj for every f 2 I .
Indeed, fix a function f 2 I and let r D jjf jj. If f D 0 then the promised
inequality clearly holds, so assume that f 0 and hence r > 0. If jf .z/j  2r
then j.f /.a/j D jf .z/j  2r and hence jj.f /jj  jf .z/j  2r as required. If
jf .z/j < 2r then there is a point x 2 Q such that jf .x/j D r; since j.f /.x/j D
jf .x/Cf .z/j > 2r , we conclude that jj.f /jj  jf .x/Cf .z/j > 2r , so (4) is proved.
Finally, let '.f / D 2.f / for any f 2 I . Then ' W I ! Cu .Q/ is a surjective
linear map and it follows from (4) that jjf jj  jj'.f /jj  4jjf jj for any f 2 I , so
we can apply Fact 11 to convince ourselves that I
Cu .Q/
Cu .K/ and hence
Fact 13 is proved.
Fact 14. Suppose that K is a compact space and F is a closed subset of K. Assume
additionally that there exists a retraction r W K ! F . For every function f 2 Cu .F /

618

2 Solutions of problems 001500

let '.f / D f r. Then ' W Cu .F / ! Cu .K/ is a linear isometric embedding, i.e.,


' W Cu .F / ! L D '.Cu .F // is a linear homeomorphism and jjf jj D jj'.f /jj for
any f 2 Cu .F /.
Proof. It is evident that ' is a linear map; recall that jjf jj D supfjf .x/j W x 2 F g
for any function f 2 Cu .F /. If g 2 Cu .K/ then jjgjj D supfjg.x/j W x 2 Kg. It
follows from '.f /jF D f that jj'.f /jj  jjf jj for any f 2 Cu .F /. We also have
j'.f /.x/j D jf .r.x//j  jjf jj for any x 2 K, so jj'.f /jj  jjf jj which shows
that jjf jj D jj'.f /jj for all f 2 Cu .F /, so Fact 11 is applicable to conclude that
' W Cu .F / ! L is a linear homeomorphism and hence Fact 14 is proved.
Fact 15. For any ordinals  and  the order type of   coincides with the order
type of the set    with the lexicographic order.
Proof. If  D 0 then   D 0 is the empty set as well as   , so the order types
of   and    coincide for  D 0. Proceeding by induction assume that  > 0 is
an ordinal and our Fact is proved for all ordinals  < .
Assume first that  is a successor ordinal, i.e.,  D  C 1 for some ordinal .
Then   D  C. Observe that   D A[B where A D   and B D  fg;
we have x < y whenever x 2 A and y 2 B, so    has the order type of A C B
(see Fact 1). Furthermore, A is isomorphic to   by the induction hypothesis and
B is isomorphic to . Thus    has the order type of   C  as promised.
Now if  is a limit
S ordinal then   D supf  W  < g. It follows from the
equality    D f   W  < g and the induction hypothesis that the order type
of   is the same as the order type of   , so it does not exceed the order type
of    for any  < . This proves that the order type of   does not exceed the
order type of   . If, on the other hand, the order type of    is strictly less than
  then the order type of    is strictly less than   for some  <  which is
impossible because the ordinal   has the same type as the subset    of the set
  . This contradiction shows that the order type of    coincides with  , so
we completed our induction step and hence Fact 15 is proved.
Fact 16. Suppose that  and  are infinite ordinals. Then there exists a closed set
A  1;  
with the following properties:
(a) A is canonically homeomorphic to 1; 
;
(b) there is a retraction r W 1;  
! A and a set B  A such that r 1 ./ is
canonically homeomorphic to 1; 
for each  2 B and r 1 .B/ is dense in
1;  
.
Proof. By Fact 15 we can identify 0;  / with the set    with the lexicographic
order. Take a point s    and let Z D ..  /nf.0; 0/g/ [ fsg; the order in Z
is the same as in    for the points of    and s > t for any t 2 Znfsg. Then Z
is canonically homeomorphic to 1;  
.
Consider the set A D f.; 0/ W 1   < g [ fsg; let '.s/ D  and '..; 0// D 
for each  2 1; /. This gives us a bijection ' W A ! 1; 
and it is straightforward
that ' is a canonical homeomorphism.

2 Solutions of problems 001500

619

Let r.a/ D a for any a 2 A. If z 2 ZnA then z D .; / for some  <  and
> 0; let r.z/ D . C 1; 0/. For any  <  the set H D .fg  1; // [ f. C 1; 0/g
is easily seen to be clopen in Z and canonically homeomorphic to 1; 
; it follows
from the equality H D r 1 .. C 1; 0// that the map r is continuous at all points
of H S
. The family H D fH W  < g is disjoint, so r is continuous at all points of
H D H. Furthermore, ZnA  H and hence H is dense in Z. Therefore, for the
set B D f. C 1; 0/ W  < g we have r 1 .B/ D H , so r 1 .B/ is dense in Z.
If  <  is a limit ordinal then r 1 ..; 0// D f.; 0/g. Given any ordinal  < 
the set W D fz 2 Z W . C 1; 0/ z .; 1/g is an open neighborhood of .; 0/
such that r.W /  .; 
. This shows that r is also continuous at all points of ZnH ,
so r has all promised properties, i.e., Fact 16 is proved.
Returning to our solution take any countable infinite ordinals and such that
 < ! . By Fact 4 there exists an ordinal  > 0 such that !   < ! C1 .
Since ! C1 D !  ! D supf!  n W n 2 !g, we can find a number n 2 ! for which
!  n  < !  .n C 1/. It is easy to see that 1; !  .n C 1/
' Y0 : : : Yn where
Yi ' 1; ! 
for every i  n. Therefore d i.1; !  .n C 1/
/ D  D d i.1; ! 
/,
so it follows from 1; ! 
 1;
 1; !  .n C 1/
that d i.1;
/ D  and hence
l

1;
1; ! 
by Fact 9.
The ordinal  D !  is a prime component and    < ! ; it follows
from ! D supf m W m 2 !g that there exists m 2 ! such that < m  . !/m .
The ordinal  is infinite, so  !    and therefore  .2 /m D 2m < ! ;
l

thus   < ! , so we can apply Fact 10 to convince ourselves that 1;


1; 

and hence 1;
1;
. The ordinals and being infinite, we can apply Fact 8
to see that C 1 D 0;
' 1;
and C 1 D 0;
' 1;
. This implies that
l

. C 1/ . C 1/ and hence we proved sufficiency.


Now suppose that and are infinite ordinals such that  and Cp . C 1/ is
linearly homeomorphic to Cp .C1/. Observe that 1;
' C1 and 1;
' C1
by Fact 8. It follows from Problem 327 that the spaces Cu .1;
/ and Cu .1;
/
are linearly homeomorphic. Let  D nnf W there exists a linear homeomorphic
embedding of Cu .1;
/ into Cu .1; 
/g. It is clear that  is an infinite ordinal; if
 is a successor then  D 0 C n for some limit ordinal 0 and n 2 N. Apply
Fact 8 once more to see that 1; 
' 1; 0
1; n
' 1; 0
and hence Cu .1;
/
also linearly embeds in Cu .1; 0
/ which is a contradiction with the choice of .
Therefore  is a limit ordinal.
Assume that  !  ! . Then the set 1; !
 1;
is a retract of 1;
(see
SFFS-316), so Cu .1; !
/ linearly embeds in Cu .1;
/
Cu .1;
/ (see Fact 14)
and hence in Cu .1; 
/. The space I D ff 2 Cu .1; 
/ W f ./ D 0g is linearly
homeomorphic to Cu .1; 
/ by Fact 13 so Cu .1; !
/ linearly embeds in I .
Fix a linear subspace L of the space I which is linearly homeomorphic to
Cu .1; !
/. We denote the norm in I and in Cu .1; !
/ by the same symbol jj jj.
Let ' W L ! Cu .1; !
/ be a linear homeomorphism; by Fact 11, there exists

620

2 Solutions of problems 001500

K > 0 such that jjf jj  jj'.f /jj  Kjjf jj for any f 2 L. Pick a number n 2 !
with n > 4K; the space 1; n
is a retract of 1; !
by SFFS-316 so Cu .1; n
/ is
isometric to a subspace of Cu .1; !
/ by Fact 14.
Therefore there exists a linear subspace M of the space L and a surjective map
 W M ! Cu .1; n
/ such that jjf jj  jj.f /jj  Kjjf jj for any f 2 M . Let
g0 2 Cu .1; n
/ be the function which is identically equal to 1. The map  being a
linear homeomorphism, it has an inverse  W Cu .1; n
/ ! M . Let f0 D .g0 /; it
1
follows from f0 ./ D 0 that we can find an ordinal 0 <  such that jf0 . /j < nC1
for all   0 .
Apply Fact 16 to find a closed set Y0  1; n
canonically homeomorphic to
1; 
and a retraction r0 W 1; n
! Y0 such that, for some B0  Y0 , the set
r01 .B0 / is dense in 1; n
and r01 .x/ is canonically homeomorphic to 1; n1

for all x 2 B0 . The dual map r0 W Cu .Y0 / ! Cu .1; n


/ is a linear embedding
by Fact 14. Let 0 W I ! Cu .1; 0
/ be the restriction map. Since Y0 ' 1; 
,
the linear map 0  r0 cannot be an embedding by the choice of , so we
can apply Fact 12 to find a function g1 2 r0 .Cu .Y0 // such that jjg1 jj D 1 and
1
j.g1 /. /j < nC1
for all   0 . Let f1 D .g1 /; it follows from f1 ./ D 0 that
1
for all   1 .
we can find 1 > 0 such that jf1 . /j < nC1
1
The map g1 is constant on every set r0 .x/; since r01 .B0 / is dense in 1; n
and
jjg1 jj D 1 we can choose x 2 B0 such that jg1 .z/j  12 for any z 2 Z1 D r01 .x/.
Proceeding inductively assume that 1  k < n and we have constructed
functions f0 ; g0 ; f1 ; g1 ; : : : ; fk ; gk , ordinals 0 < 1 < : : : < k <  and sets
Z0 D 1; n
 Z1  : : :  Zk with the following properties:
(5)
(6)
(7)
(8)
(9)

fi 2 I and gi D .fi / for all i  k;


1
jjgi jj D 1 and jfi . /j < nC1
for all   i1 and i 2 f1; : : : ; kg;
1
jfi . /j < nC1 for all   i and i  k;
gi .Zi / D fsi g and jsi j  12 for all i  k;
Zi is canonically homeomorphic to 1; ni
for each i  k.

Apply Fact 16 to find a closed set Yk  Zk canonically homeomorphic to 1; 

and a retraction rk W Zk ! Yk such that, for some Bk  Yk , the set rk1 .Bk / is dense
in Zk and rk1 .x/ is canonically homeomorphic to 1; nk1
for all x 2 Bk . The
dual map rk W Cu .Yk / ! Cu .Zk / is a linear embedding by Fact 14. Since Zk is a
retract of 1; n
, the space Cu .Zk / embeds in Cu .1; n
/ as a linear subspace (see
Fact 14); to simplify the notation, we consider that rk .Cu .Yk // is a linear subspace
of Cu .1; n
/.
Let k W I ! Cu .1; k
/ be the restriction map. Since Yk ' 1; 
, the linear
map k rk cannot be an embedding by the choice of , so we can apply Fact 12 to
1
find a function gkC1 2 rk .Cu .Yk // such that jjgkC1 jj D 1 and j.gkC1 /. /j < nC1
for all   k . Let fkC1 D .gkC1 /; it follows from fkC1 ./ D 0 that we can find
1
kC1 > k such that jfkC1 . /j < nC1
for all   kC1 .
The map gkC1 is constant on every set rk1 .x/; since rk1 .Bk / is dense in 1; nk

and jjgkC1 jj D 1, we can choose x 2 Bk such that jgkC1 .z/j  12 for any z 2
ZkC1 D rk1 .x/. It is clear that the conditions (5)(9) are now satisfied if we replace

2 Solutions of problems 001500

621

k with k C 1, so our inductive procedure can be continued to construct functions


ffi ; gi W i  ng, ordinals 0 < : : : < n and sets Z0  : : :  Zn such that
the properties (5)(9) hold for all k  n. Choose a number "i P
2 f1; 1g such
n
that "i si > 0P
for every i  n and consider the function f D
iD0 "i fi ; then
n
g D .f / D iD0 "i gi .
Take a point z 2 Zn ; it follows from the property (8) that gi .z/ D si for all
i  n. RecallPthat jjfi jj  jj.f
Pi /jj D jjgi jj DP1 for all i  n and observe that
jj.f /jj D jj niD0 "i .fi /jj  niD0 "i fi .z/ D niD0 jsi j  nC1
.
2
Let E0 D 1; 0 /; EnC1 D n ; 
and EiC1 D i ; iC1 / for every i < n. Given
any g 2 1; 
, there is a unique j  n C 1 such that  2 Ej . It follows from (6)
1
and (7) that jfi . /j < nC1
for all i j ; since jjfj jj  jj.fj /jj D jjgj jj D 1, we
1
< 2 for any  2 1; 
and hence
have jfj . /j  1. Therefore jf . /j  1 C n nC1
jjf jj D maxfjf . /j W  2 1; 
g < 2.
We must have the inequalities jj.f /jj  Kjjf jj < 2K. However, jj.f /jj 
nC1
which shows that 2K > nC1
, i.e., n C 1 < 4K; this contradiction with the
2
2
choice of n shows that the inequality  ! is impossible, so we settled necessity.
Finally observe that if K D 1; !
and K 0 D 1; ! !
then the countable compact
spaces K and K 0 are not l-equivalent by our above results; however K is uequivalent to K 0 by Problem 200, so our solution is complete.
V.495. Suppose that X is a metrizable zero-dimensional compact space and
there exists a continuous linear surjection of Cp .X / onto Cp .Y /. Prove that Y
is a metrizable compact zero-dimensional space. In particular, there exists no
continuous linear surjection of Cp .K/ onto Cp .I/. Here, as usual, K is the Cantor
set and I D 1; 1
 R.
Solution. Recall that closed map means continuous closed onto map. A map
' W T ! Z is finite-to-one if ' 1 .z/ is a finite set for any z 2 Z.
Fact 1. Suppose that Z is a zero-dimensional space and T is a space for which there
exists a closed finite-to-one map ' W T ! Z. Then T is also zero-dimensional.
Proof. Fix an arbitrary point x 2 T and a set U 2 .x; T /. Choose a faithful
enumeration fx1 ; : : : ; xn g of the set ' 1 '.x/; there is no loss of generality to assume
that x D x1 . Take a disjoint family fV1 ; : : : ; Vn g  .T / such that V1  U and
xi 2 Vi for all i 2 f1; : : : ; ng. If V D V1 [ : : : [ Vn then ' 1 '.x/  V , so we can
apply Fact 1 of S.226 to find a set W 2 .'.x/; Z/ such that ' 1 .W /  V .
The space Z being zero-dimensional, there exists a clopen set G such that '.x/ 2
G  W and hence H D ' 1 .G/  V . The set H is clopen in T ; since V1 is clopen
in V , the set H 0 D H \ V1 is clopen in H and hence in T . We have x 2 H 0 
V1  U , so clopen subsets of T constitute a base in T , i.e., T is zero-dimensional
and hence Fact 1 is proved.
Returning to our solution observe that nw.Cp .X // D nw.X / D w.X /  !; this
implies that nw.Cp .Y //  ! and hence nw.Y /  !, so Y is a -space. Therefore
we can apply Problem 285 to see that Y is compact; since w.Y / D nw.Y /  !, the
space Y must be metrizable.

622

2 Solutions of problems 001500

Apply Problem 237 to see that Lp .Y / embeds in Lp .X / as a linear subspace;


in particular, we can consider that Y is a closed linearly independent subspace of
Lp .X /. For any n 2 N; a D .a1 ; : : : ; an / 2 Rn and x D .x1 ; : : : ; xn / 2 X n let
n .a; x/ D a1 x1 C : : : C an xn ; this gives a continuous map n W Rn  X n ! Lp .X /
for any n 2 N. Let M0 D f0g where 0 is the zero vector of Lp .X /; for any n 2 N
the set Mn D n .Rn  X n / is closed in Lp .X / by Fact 1 of U.485. Therefore the set
An D Mn nMn1 is open in Mn ; it follows from nw.Lp .X // D ! that An is an F subsetSof Lp .X / for every n 2 N. The set Y is linearly independent in Lp .X /, so
Y  fAn W n 2 Ng. Therefore we can find a family fYn W n
S2 !g of closed subsets
of Y and a sequence fkn W n 2 !g  N such that Y D n2! Yn and Yn  Akn
for each n 2 N. For every point u 2 Lp .X /nf0g we can find a uniquely determined
n 2 N and a1 ; : : : ; an 2 Rnf0g such that u D a1 x1 C : : : C an xn for some distinct
x1 ; : : : ; xn 2 X ; let supp.u/ D fx1 ; : : : ; xn g.
Say that a set G  Lp .X / is adequate if there exists n 2 N such that, for some
disjoint family fU1 ; : : : ; Un g  .X / and a family fO1 ; : : : ; On g  .Rnf0g/ we
have G D O1 U1 C : : : C On Un D n .O1  : : :  On ; U1  : : :  Un /. Adequate
sets form a base in every An by Fact 3 of U.485. Therefore every Yn can be covered
by finitely many adequate subsets of Akn .
Consequently, there exists a family fPn WS
n 2 !g of compact subsets of Y and a
sequence fmn W n 2 !g  N such that Y D n2! Pn and the set Pn is contained in
an adequate subset Gn of Amn for every n 2 !.
Fix any n 2 ! and choose the sets O1 ; : : : ; Omn 2 .Rnf0g/ and disjoint sets
U1 ; : : : ; Umn 2 .X / such that Gn D O1 U1 C: : :COmn Umn ; let O D O1 : : :Omn
and U D U1 : : :Umn . The map mn j.O U / W O U ! Gn is a homeomorphism
(see Fact 4 of U.485), so we can find a set Pn0  O  U such that Pn0 ' Pn and
mn .Pn0 / D Pn .
Let n W Pn0 ! X mn be the natural projection and denote by K the compact set
n .Pn0 /. Fix any point x D .x1 ; : : : ; xmn / 2 K; if u 2 n1 .x/ then u D .a; x/ 2 Pn0
for some a D .a1 ; : : : ; amn / 2 O and hence n .u/ D a1 x1 C : : : C amn xmn 2
Y . If the set n1 .x/ is infinite, then there exists an infinite set Q  Y such that
S
fsupp.w/ W w 2 Qg  fx1 ; : : : ; xmn g. Therefore Q is contained in the linear
span of the finite set fx1 ; : : : ; xmn g; this contradiction with linear independence of
Q shows that n1 .x/ is finite for any x 2 K.
The spaces Pn0 and K being compact, the map n W Pn0 ! K is closed; the
space K  X mn is zero-dimensional and we already saw that n is finite-to-one, so
the space Pn0 is zero-dimensional by Fact 1. It follows from Pn0 ' Pn and SFFS306 that dim.Pn / D 0 for all n 2 !, so we can apply the countable sum theorem
(see Problem 150) to conclude that dim.Y / D 0 and hence Y is zero-dimensional.
Finally, observe that the Cantor set K is zero-dimensional and I isnt, so there exists
no continuous linear surjection of Cp .K/ onto Cp .I/.
V.496. Suppose that a space X is metrizable, compact and  -zero-dimensional.
Prove that if there exists a continuous linear surjection of Cp .X / onto Cp .Y /
then Y is also a compact metrizable  -zero-dimensional space. In particular, if
X is compact, metrizable and finite-dimensional then there is no continuous linear
surjection of Cp .X / onto Cp .I! /.

2 Solutions of problems 001500

623

Solution. Recall that closed map means continuous closed onto map. A map
' W T ! Z is finite-to-one if ' 1 .z/ is a finite set for any z 2 Z. Observe first that
nw.Cp .X // D nw.X / D w.X /  !; this implies that nw.Cp .Y //  ! and hence
nw.Y /  !, so Y is a -space. Therefore we can apply Problem 285 to see that Y
is compact; since w.Y / D nw.Y /  !, the space Y must be metrizable.
Apply Problem 237 to see that Lp .Y / embeds in Lp .X / as a linear subspace;
in particular, we can consider that Y is a closed linearly independent subspace of
Lp .X /. For any n 2 N; a D .a1 ; : : : ; an / 2 Rn and x D .x1 ; : : : ; xn / 2 X n let
n .a; x/ D a1 x1 C : : : C an xn ; this gives a continuous map n W Rn  X n ! Lp .X /
for any n 2 N. Let M0 D f0g where 0 is the zero vector of Lp .X /; for any n 2 N
the set Mn D n .Rn  X n / is closed in Lp .X / by Fact 1 of U.485. Therefore the set
An D Mn nMn1 is open in Mn ; it follows from nw.Lp .X // D ! that An is an F subsetSof Lp .X / for every n 2 N. The set Y is linearly independent in Lp .X /, so
Y  fAn W n 2 Ng. Therefore there exists a family fYn W n
S2 !g of closed subsets
of Y and a sequence fkn W n 2 !g  N such that Y D n2! Yn and Yn  Akn
for each n 2 N. For every point u 2 Lp .X /nf0g we can find a uniquely determined
n 2 N and a1 ; : : : ; an 2 Rnf0g such that u D a1 x1 C : : : C an xn for some distinct
x1 ; : : : ; xn 2 X ; let supp.u/ D fx1 ; : : : ; xn g.
Say that a set G  Lp .X / is adequate if there exists n 2 N such that, for some
disjoint family fU1 ; : : : ; Un g  .X / and a family fO1 ; : : : ; On g  .Rnf0g/ we
have G D O1 U1 C : : : C On Un D n .O1  : : :  On ; U1  : : :  Un /. Adequate
sets form a base in every An by Fact 3 of U.485. Therefore every Yn can be covered
by finitely many adequate subsets of Akn .
Consequently, there exists a family fPn WS
n 2 !g of compact subsets of Y and a
sequence fmn W n 2 !g  N such that Y D n2! Pn and the set Pn is contained in
an adequate subset Gn of Amn for every n 2 !.
Fix any n 2 ! and choose the sets O1 ; : : : ; Omn 2 .Rnf0g/ and disjoint sets
U1 ; : : : ; Umn 2 .X / such that Gn D O1 U1 C: : :COmn Umn ; let O D O1 : : :Omn
and U D U1 : : :Umn . The map mn j.O U / W O U ! Gn is a homeomorphism
(see Fact 4 of U.485), so we can find a set Pn0  O  U such that Pn0 ' Pn and
mn .Pn0 / D Pn .
Let n W Pn0 ! X mn be the natural projection and denote by K the compact set
n .Pn0 /. Fix any point x D .x1 ; : : : ; xmn / 2 K; if u 2 n1 .x/ then u D .a; x/ 2 Pn0
for some a D .a1 ; : : : ; amn / 2 O and hence n .u/ D a1 x1 C : : : C amn xmn 2
Y . If the set n1 .x/ is infinite, then there exists an infinite set Q  Y such that
S
fsupp.w/ W w 2 Qg  fx1 ; : : : ; xmn g. Therefore Q is contained in the linear
span of the finite set fx1 ; : : : ; xmn g; this contradiction with linear independence of
Q shows that n1 .x/ is finite for any x 2 K.
The spaces Pn0 and K being compact, the map n W Pn0 ! K is closed. The space
mn
X is easily seen to be  -zero-dimensional, so the space K  X mn is also  -zerodimensional.
S Take a family fEi W i 2 !g of zero-dimensional subspaces of K such
that K D i2! Ei .
If Ei0 D n1 .Ei / then the map n jEi0 W Ei0 ! Ei is closed for every i 2 ! by
Fact 1 of S.261; it is clearly finite-to-one
so the space Ei0 is zero-dimensional by
S
0
0
Fact 1 of V.495. We have Pn D i2! Ei , so the space Pn0 is  -zero-dimensional.

624

2 Solutions of problems 001500

It follows from S
Pn0 ' Pn that the space Pn is  -zero-dimensional for all n 2 !, so
the space Y D n2! Pn is  -zero-dimensional as well.
Finally, observe that if X is finite-dimensional then it representable as the finite
union of its zero-dimensional subspaces (see Problem 164); in particular, the space
X must be  -zero-dimensional. If there exists a continuous linear surjection of
Cp .X / onto Cp .I! / then I! must be  -zero-dimensional; this contradiction with
Problem 098 shows that there exists no continuous linear surjection of Cp .X / onto
Cp .I! /, so our solution is complete.
V.497. Prove that a space K is l-equivalent to the Cantor set K if and only if there
exists a continuous linear surjection of Cp .K/ onto Cp .K/ as well as a continuous
linear surjection of Cp .K/ onto Cp .K/.
Solution. Necessity being trivial, assume that there exists a continuous linear
surjection of Cp .K/ onto Cp .K/ and vice versa. We can apply Problem 495 to see
that K is a metrizable zero-dimensional compact space. It follows from existence
of a continuous linear surjection of Cp .K/ onto Cp .K/ that Lp .K/ embeds in
Lp .K/ as a linear subspace (see Problem 237). In particular, there exists a linearly
independent set F  Lp .K/ which is homeomorphic to K.
For any u 2 F there exists a nonempty finite set '.u/  K such that u belongs
to the linear hull Lu of the set '.u/. The linear space Lu is finite-dimensional; since
F is linearly independent, the set Lu \ F has to finite. Therefore we have a map
' W F ! K
<! D fA  K W jAj < !g such that ' 1 .A/ is finite for any
A 2 S
K
<! . If the space K is countable then K
<! is also countable and hence
F D f' 1 .A/ W A 2 K
<! g is countable as well; this contradiction shows that K
l

is uncountable and hence K  K by Problem 297.


V.498. Let I be the closed interval 0; 1
 R. Given any n 2 ! suppose that K
is a metrizable compact space and dim K  n. Prove that there exists a compact
subspace K 0  I 2nC1 with the following properties:
(i) K 0 is homeomorphic to K;
(ii) for any function ' 2 C.K 0 /, we can choose ' 1 ; : : : ; ' 2nC1 2 C.I / such that
'.y/ D ' 1 .y1 / C : : : C ' 2nC1 .y2nC1 / for any y D .y1 : : : ; y2nC1 / 2 K 0 .
Deduce from this fact that if X is a second countable space and dim X  n then
X embeds in I2nC1 .
Solution. Recall that an open cover U of a spaceS
Z is shrinkable if, for each U 2 U ,
we can choose a closed set FU  U such that fFU W U 2 U g D Z. Say that a
family V of subsets of Z is inscribed in U if, for any V 2 V there exists U 2 U
such that V  U . We will often use the following simple observation:
(1) if Z is a space then a finite family A of subsets of Z is discrete if and only if
the family fA W A 2 Ag is disjoint.
If Z is a space and A; B  Z then the sets A and B are called separated if
A \ B D B \ A D ;; say that A and B are open-separated if there exist disjoint
sets U; V 2 .Z/ such that A  U and B  V .

2 Solutions of problems 001500

625

Fact 1. Suppose that Z is a hereditarily normal space and Y  Z. If G  .Y / is a


finite disjoint family then there exists a disjoint family fUG W G 2 Gg  .Z/ such
that G  UG for any G 2 G.
Proof. Take a faithful enumeration G D fG1 ; : : : ; Gm g of the family G. Our Fact
is trivially true if m D 1. Proceeding by induction assume that we proved it for
m D k and G D fG1 ; : : : ; GkC1 g is a disjoint family of open subsets of Y . It is
immediate that the sets G 0 D G1 [ : : : [ Gk and GkC1 are separated in Z, so we
can apply Fact 1 of S.291 to see that G 0 and GkC1 are open-separated. Fix disjoint
sets U 0 ; UkC1 2 .Z/ such that G 0  U 0 and GkC1  UkC1 .
Our induction hypothesis is now applicable to the family fG1 ; : : : ; Gk g and the
space U 0 , so we can find disjoint sets U1 ; : : : ; Uk 2 .U 0 / such that Gi  Ui for
all i  k. Then fU1 ; : : : ; UkC1 g is a disjoint family of open subsets of Z such that
Gi  Ui for every i  k C 1. This completes the induction step and shows that
Fact 1 is proved.
Fact 2. If Z is a metrizable compact space and m 2 ! then dim Z  m if and
only if, for any open cover U of the space Z and any k  m C 1 there exists a
collection C D fV1 ; : : : ; Vk g of discrete families of open subsets of Z such that
every VS
i is inscribed in U and for any set A  f1; : : : kg with jAj D m C 1, the
family fVi W i 2 Ag is a cover of Z.
Proof. Given a number k  m C 1 let E.k/ be the statement which says that for any
open cover U of the space Z there exists a collection C D fV1 ; : : : ; Vk g of discrete
families of open subsets of Z such that every VS
i is inscribed in U and for any set
A  f1; : : : kg with jAj D m C 1, the family fVi W i 2 Ag is a cover of Z. If
E.m C 1/ holds then every open cover U of the space Z has a refinement V which
is a union of .m C 1/-many discrete subfamilies. This implies ord.V/  m C 1 and
hence dim Z  m, i.e., we proved sufficiency.
Now assume that dim Z  m; we will prove, by induction on k that E.k/ holds
for each k  mC1. Apply Problem 164 to find sets Z0 ; : : : ; Zm  Z such that Z D
Z0 [: : :[Zm and dim Zi D 0 for all i  m. Let U be an open cover of Z; since Z is
compact, we can assume, without loss of generality, that U D fU1 ; : : : ; Ul g for some
l 2 N. For any j  m, apply Fact 2 of T.311 to the open cover fUi \ Zj W i  lg of
j
j
the space Zj , to find a disjoint family Vj D fV1 ; : : : ; Vl g of clopen subsets of Zj
S
j
such that Vi  Ui for every i  l and Vj D Zj .
j
j
By Fact 1 there exists a disjoint family Wj D fW1 ; : : : ; Wl g  .Z/ such
j
j
j
j
that Vi  Wi for all i  l. If Hi D Wi \ Ui for every i  l then the family
S
j
j
inscribed in U and Zj  Hj for each j  m.
Hj D fH1 ; : : : ; Hl g is disjoint,
S
Therefore the family H D fHj W 0  j  mg is an open cover of the space Z.
By Fact 1 of U.188 the cover H is shrinkable and hence we can choose a closed set
j
j
j
Fi in the space Z such that Fi  Hi for every i  l and j  m in such a way
j
that the family fFi W i  l and j  mg covers Z.
By normality of the space Z we can choose, for every j  m and i  l,
j
j
j
j
j
a set Gi 2 .Z/ such that Fi  Gi  clZ .Gi /  Hi . Since the family

626

2 Solutions of problems 001500


j

fclZ .Gi / W i  lg is disjoint, we can apply (1) to see that the family Vj D fGi W
i  lg is discrete for each j  m. It is clear that every Vj is inscribed in U and
S
fVj W 0  j  mg is a cover of Z, so we proved E.m C 1/.
Now assume that we established the statement E.k/ for some k  m C 1 and fix
an open cover U of the space Z. By the induction hypothesis there exists a collection
C D fV1 ; : : : ; Vk g of
S discrete families of open subsets of Z such that every Vi is
inscribed in U and fVi W i 2 Ag is a cover of Z for any set A  f1; : : : ; kg of
cardinality m C 1.
S S
Let A D fA  f1; : : : ; kg W jAj D mg; consider the set PA D Zn f Vi W i 2
Ag for every
SA 2 A. IfSA and B are distinct elements of A then jA [ Bj  m C 1
and hence i2A Vi [ i2B Vi is a cover of X ; as a consequence, PA \ PB D ;.
Therefore fPA W A 2 Ag is a disjoint family of closed subsets of Z, so we can find
a discrete family fOA W A 2 Ag  .Z/ such that PA  OA for each S
A 2 A.
For every A 2 A fix an index jA 2 f1; : : : ; kgnA; since the family i2A Vi [ VjA
is a cover of Z, the family VjA covers PA . Thus the family DA D fV \ OA W V 2
VjA g is discrete, inscribedSin VjA (and hence in U ) and covers PA . This easily implies
that the family VkC1 D fDA W A 2 Ag is discrete and inscribed in U . To see that
the family fV1 ; : : : ; VkC1 g is as promisedS
take any set B  f1; : : : ; k; k C 1g of
cardinality m C 1. If B  f1; : : : ; kg then i2B Vi is a cover of Z by the induction
hypothesis. If kSC 1 S
2 B then A D B \ f1;S
: : : ; kg 2 A and hence VkC1 covers the
set PA D Zn i2A Vi . This shows that fVi W i 2 Bg is a cover of Z, so we
carried out the induction step and hence E.k/ holds for any k  m C 1, i.e., Fact 2
is proved.
Returning to our solution, let Q D f1; : : : ; 2n C 1g and fix a metric  on the
space K which generates the topology of K. For any k 2 ! we will need the set
Ik D 0; 1  2k1
 I . Let diam.Z/ D supf.x; y/ W x; y 2 Zg for any Z  K.
Apply Fact 2 to construct, for any q 2 Q,Sa discrete family Cq;0 of open subsets of
K such that diam.U /  1 forS
every U 2 fCq;0 W q 2 Qg and, for any set A  Q
with jAj D n C 1, the family q2A Cq;0 is a cover of K.
Let E.r/ D fi r 1 W i 2 f0; : : : ; rgg for any prime number r 2 N. Take distinct
q
prime numbers r01 ; : : : ; r02nC1 2 N such that r0 > 2jCq;0 j for each q 2 Q and choose
1
1 1
a number "0 > 0 such that "0 < nnf 2 ; .r0 / ; : : : ; .r02nC1 /1 g. Using normality of
q
the space K it is easy to construct, for any q 2 Q, a function f0 2 C.K; I0 / such
q
q
that, for each U 2 Cq;0 we have f0 .U / D faU g for some number aU 2 E.r0 / \ I0
and aU aV for distinct U; V 2 Cq;0 .
Proceeding inductively assume that m 2 ! and we have constructed, for any
k  m, a collection fCq;k W q 2 Qg of discrete families of open subsets of K, prime
q
numbers rk1 ; : : : ; rk2nC1 2 N, a set of functions ffk W q 2 Qg  C.K; Ik / and a
number "k > 0 with the following properties:
(2)
(3)
(4)
(5)

if 1  k  m then "k < 12 "k1 ;


"k < nnf2k1 ; .rk1 /1 ; : : : ; .rk2nC1 /1 g for any
S k  m;
if k  m then diam.U /  2k for any U 2 fCq;k W q 2 Qg;
q
p
if p; q 2 Q and k; l  m then rk D rl if and only if p D q and k D l;

2 Solutions of problems 001500

627

S
(6) if k  m then for any set A  Q with jAj D n C 1, the family q2A Cq;k is a
cover of K;
q
q
(7) if k  m and U 2 Cq;k then fk .U / D faU g for some number aU 2 E.rk / \ Ik
and aU aV for distinct U; V 2 Cq;k ;
q
q
q
(8) if j < k  m and q 2 Q then fj .x/  fk .x/  fj .x/ C "j  "k for all
x 2 K.
q

There exists a number > 0 such that jfm .x/  fm .y/j < 14 "m for any q 2 Q
whenever .x; y/  . Take a collection fCq;mC1 W q 2 Qg of discrete
S families of
m1
open subsets
of
K
such
that
diam.U
/
<
nnf;
2
g
for
any
U
2
fCq;mC1 W q 2
S
Qg and fCq;mC1 W q 2 Ag is a cover of K for any A  Q with jAj D n C 1.
q
2nC1
1
Choose distinct prime numbers rmC1
; : : : ; rmC1
2 Nnfrk W k  m and q 2 Qg
q
q
such that rmC1 > .jCmC1 j C 2/ "4m for every q 2 Q. Fix a number q 2 Q and U 2
q
Cq;mC1 ; observe that the diameter of the set fm .U / does not exceed 14 "m . If dU D
q
q
inf fm .U / then fm .U /  dU ; dU C 14 "m
and the interval dU C 14 "m ; dU C 12 "m

q
q
contains at least jCmC1 j-many points of the set E.rmC1 /. Therefore it is possible
q
to choose a number aU 2 dU C 14 "m ; dU C 12 "m
\ E.rmC1 / in such a way that
q
aU aV for any distinct U; V 2 CmC1 . It is immediate that aU 2 ImC1 for all
U 2 Cq;mC1 .
q
q
Given any set U 2 CmC1 let gq .x/ D aU  fm .x/ for every x 2 U . Then
S
gq is a continuous function on the closed set F D fU W U 2 Cq;mC1 g and,
besides, 0  gq .x/  12 "m for any point x 2 F . Therefore we can find a continuous
q
q
function hq W K ! 0; 12 "m
such that hq jF D gq . Letting fmC1 D hq C fm
q
q
we obtain a function fmC1 2 C.K; ImC1 / such that fmC1 .U / D faU g and aU 2
q
E.rmC1 / \ ImC1 for any set U 2 Cq;mC1 . Choose a number "mC1 > 0 such that
2nC1 1
1
/1 ; : : : ; .rmC1
/ g; it is straightforward
"mC1 < 12 "m and "mC1 < nnf2m1 ; .rmC1
that the conditions (2)(7) are now satisfied for all k  m C 1.
To prove (8), fix any index q 2 Q and observe that we have the inequalities
q
q
q
fm .x/  fmC1 .x/  fm .x/ C 12 "m < "m  "mC1 for every x 2 K, i.e., the property
q
q
(8) holds for j D m and k D m C 1. Now, if j < m then fm .x/  fj .x/ C "j  "m
q
q
q
for all x 2 K and therefore fmC1 .x/  fj .x/C"j "m C 12 "m < fj .x/C"j "mC1
for each x 2 K; this shows that (8) is true for k D m C 1 and any j < k. If k  m
and j < k then (8) is true by the induction hypothesis, so we checked that (8) holds
if we replace m with m C 1.
Thus we carried out the induction step, so our procedure guarantees that we can
construct, for any k 2 !, a collection fCq;k W q 2 Qg of discrete families of open
q
subsets of the space K, prime numbers rk1 ; : : : ; rk2nC1 2 N, a set of functions ffk W
q 2 Qg  C.K; Ik / and a number "k > 0 such that the conditions (2)(8) are
satisfied for all m 2 !.
q
The properties (3) and (8) show that the sequence ffk .x/ W k 2 !g is increasing
q
and bounded by 1; therefore there exists f q .x/ D lim fk .x/ for any x 2 K and
hence we have a function f q W K ! I for every q 2 Q. The property (8) implies
q
that jf q .x/  fj .x/j  "j for all x 2 K and j 2 !; therefore the sequence

628

2 Solutions of problems 001500

ffk W k 2 !g converges uniformly to the function f q and hence f q 2 C.K; I / for


each q 2 Q.
Consider the family Dq;k D ff q .U / W U 2 Cq;k g for any numbers q 2 Q and k 2
q
q
!. Recall that fk .U / D faU g for some aU 2 E.rk /. The property (8) implies that
q
q
q
fk .x/  f .x/  fk .x/ C "k for all x 2 K and therefore f q .U /  aU ; aU C "k

for every U 2 Cq;k . The property (7) shows that if U and V are distinct elements
q
of Cq;k then aU aV and hence jaU  aV j  .rk /1 ; by the property (3) we have
aU ; aU C "k
\ aV ; aV C "k
D ;, so the family faU ; aU C "k
W U 2 Cq;k g is
disjoint and hence Dq;k is discrete for any q 2 Q and k 2 !.
q
Fix an arbitrary continuous function h W K ! R; let k0 D 0 and 0 .t / D 0 for
all q 2 Q and t 2 R. Proceeding by induction assume that m 2 ! and we have a set
q
q
of numbers fki W i  mg  ! and functions 0 ; : : : ; m 2 C.R/ for every q 2 Q
with the following properties:
(9) ki < kiC1
m;
P i<
P whenever
q
(10) if hi D q2Q ij D0 j f q and Mi D supfjh.x/  hi .x/j W x 2 Kg for each
q
i < m, then j iC1 .t /j  .n C 1/1 Mi for any q 2 Q and t 2 R;
q
(11) for every i < m and U 2 Cq;ki C1 if D D f q .U / 2 Dq;ki C1 then iC1 .D/ D
1
fsD g where sD D .n C 1/ .h.yD /  hi .yD // for some yD 2 U .
(12) if i < m then j.h  hi /.x/  .h  hi /.y/j < .2n C 2/1 Mi for any x; y 2 K
such that .x; y/  2ki C1 .
P
P
q
q
Let hm D q2Q m
j D0 j f ; if Mm D supfj.h  hm /.x/j W x 2 Kg D 0
then h D hm and our construction stops. If Mm > 0 then, by uniform continuity of
h  hm , there exists > 0 such that j.h  hm /.x/  .h  hm /.y/j < .2n C 2/1 Mm
for any x; y 2 K with .x; y/ < . Pick a number kmC1 2 ! such that kmC1 > km
and 2kmC1 < .
Fix an arbitrary q 2 Q; for every U 2 Cq;kmC1 take a point zU 2 U and let
.t / D .n C 1/1S
.h.zU /  hm .zU // for any t 2 f q .U /. This gives a continuous
function W Z D fD W D 2 Dq;kmC1 g ! .n C 1/1 Mm ; .n C 1/1 Mm
. The
q
q
family Dq;kmC1 is discrete, so there exists mC1 2 C.R/ such that mC1 jZ D and
q
1
j mC1 .t /j  .n C 1/ Mm for all t 2 R.
It is straightforward that the conditions (9)(12) are now satisfied if we replace
m by m C 1, so our inductive procedure can be continued. If our induction stops at
q
a step m then wePobtain
of functions f i W iP
 m; q 2 Qg  C.R/ such
Pamfamily
q
q
q
q
that h D hm D q2Q j D0 j f . Letting ' D P m
j D0 j for every q 2 Q we
q
obtain a family f' W q 2 Qg  C.R/ such that h D q2Q ' q f q .
Now, if our inductive construction did not stop at any finite step then we have a
q
sequence fki W i 2 !g  ! and a family of functions f i W q 2 Q; i 2 !g  C.R/
such that the conditions (9)(12) are satisfied for
S all m 2 !.
Fix any i 2 !; q 2 Q and a point x 2
Cq;ki C1 ; if x 2 U 2 Cq;ki C1 then
q
q
there is y 2 U such that iC1 .f q .x// D iC1 .f q .y// D .n C 1/1 .h  hi /.y/
for any x 2 U . Furthermore, diam.U /  2ki C1 , so we can apply (12) to see that
j.h  hi /.x/  .h  hi /.y/j < .2n C 2/1 Mi and therefore

2 Solutions of problems 001500

629

(13) j.n CS1/1 .h  hi /.x/  iC1 .f q .x//j < .n C 1/1 .2n C 2/1 Mi for any point
x 2 Cq;ki C1
S
Take an arbitrary point x 2 X . If the set A D fq
Cq;ki C1 g has
S2 Q W x 2
cardinality less than n C 1 then jQnAj  n C 1 and i2QnA Cq;ki C1 does not cover
the point x which is a contradiction with (6). Thus jAj  n C 1, so we can choose a
set B  A with jBj D n C 1. It follows from (10) and (13) that
P
q
j.h  hiC1 /.x/j D j.h  hi /.x/  q2Q iC1 .f q .x//j 
P
P
q
q
(14) j.h  hi /.x/  q2B iC1 .f q .x//j C j q2QnB iC1 .f q .x//j <
1
n
2nC1
Mi C nC1 Mi D 2nC2 Mi :
2nC2
q

It follows from (14) that MiC1 <

2nC1
Mi
2nC2

for every i 2 ! and hence

/i M0 for all i 2 !,
(15) Mi < . 2nC1
2nC2
/i M0 for any i 2 ! and x 2 K. As a consequence, for
so j.h  hi /.x/j < . 2nC1
2nC2
every point x 2 K the sequence fhi .x/ W i 2 !g converges to h.x/.
1
Fix an index q 2 Q; if di D nC1
. 2nC1
/i M0 then it follows from (10) and
2nC2
P
q
(15) that j i .t /j  di for any i 2 ! and t 2 R. It is evident that the series 1
iD0 di
q
q
converges, so we can apply TFS-030 to see that the sequence f 0 C: : :C i W i 2 !g
converges uniformly to a function ' q 2 C.R/.
q
q
q
Let i D 0 C : : : C i for every q 2 Q and i 2 !. Given any point x 2 K
q
q
q
q
the sequence fi .f .x//
P W i q 2 q!g converges to the point ' .fP .x//. qTherefore
the sequence S D f q2Q i .f .x// W i 2 !g converges to q2Q ' .f q .x//;
recalling that S D fhi .x/ W i P
2 !g, we conclude that S also converges to h.x/, so
we have the equality h.x/ D Pq2Q ' q .f q .x// for each x 2 K. Thus, in all cases,
we obtained the equality h D q2Q ' q f q which shows that
1
2nC1
(16) for any continuous
2 C.R/
P function h W K ! R there exist ' ; : : : ; '
such that h D q2Q ' q f q .

If x and y are distinct points of K and f q .x/ D f q .y/ for all q 2 Q then fix a
function h 2 C.K/ such that h.x/ D 0 and h.y/ D 1. If f' q W q 2 Qg  C.R/ is a
family as in (16) then ' q .f q .x// D ' q .f q .y// for all q 2 Q which, together with
(16), implies that h.x/ D h.y/ which is a contradiction.
Therefore the family ff q W q 2 Qg separates the points of K. Define a map
f W K ! I 2nC1 by the equality f .x/ D .f 1 .x/; : : : ; f 2nC1 .x// for any x 2 K. It
is immediate that f is an embedding; let K 0 D f .K/.
Given any continuous function ' W K 0 ! R the function h D ' f is continuous
and hence
(16) to find a family f' q W q 2 Qg  C.R/ such that
P weq can apply
q
h D q2Q ' f . Since f q .K/  I , we can restrict every function ' q to I to
obtain the same equality. Therefore we can consider, without loss of generality, that
' q 2 C.I / for every q 2 Q.
If y D .y1 ; : : : ; y2nC1 / 2 K 0 then there is aP
unique x 2 K with yq D f q .x/
for any q 2 Q. We have '.y/ D '.f .x// D q2Q ' q .f q .x//; in other words,
'.y/ D ' 1 .y1 / C : : : C ' 2nC1 .y2nC1 / for any point y D .y1 ; : : : ; y2nC1 / 2 K 0 , so
the set K 0  I 2nC1 is as promised.

630

2 Solutions of problems 001500

Finally observe that we proved, in particular, that any compact space K with
dim K  n embeds in I 2nC1 and hence in I2nC1 . If X is a second countable space
and dim X  n then there exists a second countable compact space K such that
X  K and dim K  n (see Problem 162). If  W K ! I2nC1 is an embedding then
jX embeds X in I2nC1 and hence our solution is complete.
V.499. Prove that, for any finite-dimensional metrizable compact space K, there
exists a surjective continuous linear mapping of Cp .I/ onto Cp .K/.
Solution. The expression L
M says that the linear topological spaces L and M
are linearly homeomorphic. Let I be the closed interval 0; 1
 R and fix n 2 !
such that dim K  n. By Problem 498, we can consider that K  I 2nC1 and,
(1) for any continuous
P function f W K ! R there exist h1 ; : : : ; h2nC1 2 C.I / such
that f .x/ D 2nC1
iD1 hi .xi / for every point x D .x1 ; : : : ; x2nC1 / 2 K.
Let i W I 2nC1 ! I be the natural projection of I 2nC1 onto its i -th factor; the
function pi D i jK W K ! I is continuous for each i  2n C 1. The dual map
pi W Cp .I / ! Cp .K/ is a linear embedding by TFS-163; thus Zi D pi .Cp .I // is
a linear subspace of Cp .K/ homeomorphic to Cp .I / for every i  2nC1. Therefore
the space Z D Z1  : : :  Z2nC1 is linearly homeomorphic to .Cp .I //2nC1 .
For any point g D .g1 ; : : : ; g2nC1 / 2 Z let .g/ D g1 C : : : C g2nC1 ; then
 W Z ! Cp .K/ is a linear continuous map. Given any f 2 Cp .K/ apply (1)
P
to find functions h1 ; : : : ; h2nC1 2 C.I / such that f .x/ D 2nC1
iD1 hi .xi / for every
point x D .x1 ; : : : ; x2nC1
/
2
K.
P
Therefore f .x/ D 2nC1
iD1 hi .pi .x// for every x 2 K and hence we have the
P
equality f D 2nC1
h

pi . Letting gi D pi .hi / D hi pi for all i  2n C 1 we


i
iD1
obtain a point g D .g1 ; : : : ; g2nC1 / 2 Z such that .g/ D f . Therefore  W Z !
Cp .K/ is a linear continuous surjective map. It follows from Z
.Cp .I //2nC1

.Cp .I//2nC1 that Z is linearly homeomorphic to .Cp .I//2nC1 . Take a space Ji


homeomorphic to I for every i D 1; : : : ; 2nQ
C 1 and let J D J1 : : : J2nC1 ;
2nC1
it follows from Problem 265 that Cp .J /
2nC1
. Apply
iD1 Cp .Ji /
.Cp .I//
Problem 266 to conclude that Cp .J /
Cp .I/ and hence Cp .I/
Z, so there exists
a continuous linear surjection of Cp .I/ onto Cp .K/.
V.500. Prove that l.X / D l.Y / for any l-equivalent spaces X and Y . In particular,
l

if X  Y then X is Lindelf if and only if so is Y.


Solution. Given a set A we denote by Fin.A/ the family of all finite subsets of A.
A family fUn W n 2 !g of subsets of a space Z is called increasing if Un  UnC1
for each n 2 !. Suppose that we have spaces Z and T and a map ' W Z ! exp.T /;
let 'l1 .U / D fz 2 Z W '.z/ \ U ;g and 'u1 .U / D fz 2 Z W '.z/  U g for any
U  T . A map ' W Z ! exp.T / is called lower semicontinuous if 'l1 .U / is open
in Z for any U 2 .T /.
If Z is a set and A D fAn W n
T is a sequence of subsets of Z then the
S2 !g
expression
A
!
Z
says
that
Z
D
n2!
kn Ak . For any A  exp.Z/ we denote
V
by A the family of all finite intersections of elements of A.

2 Solutions of problems 001500

631

If Z and T are spaces and ' W Z ! Fin.T / say that a map e W .T / ! exp.Z/
is an extractor for ' (or '-extractor) if the following conditions are satisfied:
(1) 'u1 .U /  e.U / for any U 2 .T /;
(2) if U; V 2 .T / and U  V then '.x/\.V nU / ; whenever x 2 e.V /ne.U /;
(3) if U D fU
S n W n 2 !g  .T / is an increasing sequence and fe.Un / W n 2 !g !
Z then U D T .
Given an extractor e W .T / ! exp.Z/ let e  .U / D Zne.U / for any U 2 .T /.
If Z is a spaceVand A is a family of subsets of Z then we write l.A/ S if l.A/  
for any A 2 A. For every family U of subsets of Z let U
 D f V W V  U
and jVj  g. Say that a cover U of the space Z is -trivial if Z 2 U
 ; if U is not
-trivial, we call it -nontrivial.
If ' W Z ! Fin.T / say that an extractor e W .T / ! exp.Z/ is -synchronous
if there exists a base B in the space T and a family L of subsets of Z such that
l.L/   and, for any -nontrivial cover U  B of the space T , if U 2 U
 then
there exists V 2 U
 such that U  V and e  .V / 2 L.
Fact 1. Given spaces Z and T , let ' W Z ! exp.T / be a lower semicontinuous
map. Then, for any U 2 .T / and n 2 N, the set V D fx 2 Z W j'.x/ \ U j  ng is
open in Z.
Proof. Fix a point x 2 V and choose distinct points t1 ; : : : ; tn 2 '.x/ \ U . There
exist disjoint
sets W1 ; : : : ; Wn 2 .T / such that ti 2 Wi  U for any i  n. The set
T
G D in 'l1 .Wi / is open in Z and it is straightforward that x 2 G  V , so every
point of V is in the interior of V , i.e., V is open in Z and hence Fact 1 is proved.
Fact 2. Suppose that Z and T are spaces and ' W Z ! Fin.T / is a lower
semicontinuous map. If e W .T / ! exp.Z/ is an extractor for ' and fUn W
n 2 !g  .T / is an increasing sequence then, for any point x 2 Z, either
I D fn 2 ! W x 2 e.Un /g or !nI is finite.
Proof. If both sets I and !nI are infinite then it is easy to see that we can find a
sequence fkn W n 2 !g  ! such that knC1 > kn C1 while kn 2 !nI and kn C1 2 I
for any n 2 !. Therefore x 2 e.Ukn C1 /ne.Ukn / and hence '.x/ \ .Ukn C1 nUkn / ;
for any n 2 ! by the property (2). The family fUkn C1 nUkn W n 2 !g being disjoint,
the set '.x/ has to be infinite; this contradiction shows that Fact 2 is proved.
Fact 3. Suppose that Z and T are spaces, ' W Z ! Fin.T / is a lower
semicontinuous map and e W .T / ! exp.Z/ is an extractor for '. Assume
additionally that fUn W n 2 !g  .T / is an increasing sequence
that for
Tn such

e
.U
any k; n 2 ! with k  n we have j'.x/j

n

k
for
each
x
2
i /. Then
iDk
S
fe.Un / W n 2 !g ! Z and hence T D n2! Un .
Proof. Fix an arbitrary point x 2 Z; we must prove that there exists m 2 ! such
that x 2 e.Un / for any n  m. By Fact 2, it suffices to show that the set I D fn 2
! W xT2 e.Un /g is infinite. However, if I is finite then there exists k 2 ! such that
x 2 niDk e  .Ui / and hence j'.x/j  n  k for any n  k. This implies that '.x/
is infinite, so we obtained a contradiction which shows that Fact 3 is proved.

632

2 Solutions of problems 001500

Fact 4. Suppose that Z and T are spaces, ' W Z ! Fin.T /nf;g is a lower
semicontinuous map and  is an infinite cardinal. If e W .T / ! exp.Z/ is a synchronous extractor for ' and l.Z/   then l.T /  .
Proof. There exists a base B in the space T and a family L of subsets of Z such
that l.L/   and, for any -nontrivial cover U  B of the space T , if U 2 U

then there exists V 2 U
 such that U  V and e  .V / 2 L.
Assume that l.T / > ; then we can find a -nontrivial open cover U  B of the
space T . For any x 2 Z there exists a set Ux 2 U such that '.x/\Ux ; and hence
x 2 'l1 .Ux /. The cover f'l1 .Ux / W x 2 Zg has a subcover
S of cardinality   and
hence there exists A  Z such that jAj   and ZSD f'l1 .Ux / W x 2 Ag. If
U0 D fUx W x 2 Ag  U then jU0 j   and '.x/ \ . U0 / ; forSany x 2 Z.
By the choice of B and L there exists U0 2 U
 such that U0  U0 and
e  .U0 / 2 L. If x 2 e  .U0 / then the property (1) implies that '.x/ is not contained
in U0 , so it follows from '.x/ \ U0 ; that j'.x/j  2.
Proceeding inductively, assume that n 2 ! and we have sets U0 ; : : : ; Un 2 U

such that
e  .Ui / 2 L for any i  n;
T

j'.x/j  m  k C 2 for any x 2 Fk;m D m
iDk e .Ui / and k  m  n;
j'.x/ \ UmC1 j  m  k C 2 for any x 2 Fk;m and k  m < n;
Um  UmC1 for any m < n.
V
Fix any k  n; the set Fk;n belongs to L, so l.Fk;n /  . By (5),
S we can
choose, for any x 2 Fk;n a finite family Wx 
U
such
that
j'.x/
\
Wx j 
S
n  k C 2. The set Hx D fy 2 Z W j'.y/ \ Wx j  n  k C 2g is an open
neighborhood of x by Fact 1, so the cover fHx W x 2 Fk;n g of the set Fk;n has a
subcover of cardinality not exceeding .
Therefore
S there exists a family UnC1  U such that jUnC1 j   and we have
j'.x/ \ UnC1 j  n  k C 2 for any x 2SFk;n and k  n. By the choice of B we
can find a set UnC1 2 U
 such that Un [ UnC1  UnC1 and e  .UnC1 / 2 L. It is
immediate that the conditions (4), (6) and (7) are satisfied if we replaceTn with nC1.
nC1 
To see that (5) is also true it suffices to check it for all sets Fk;nC1 D iDk
e .Ui /,
so fix k  n C 1 and a point x 2 Fk;nC1 .
If k D nC1 then it follows from the inclusion U0  UnC1 that '.x/\UnC1 ;.
Since x e.UnC1 /, by the property (1) it is impossible that '.x/  UnC1 , so
'.x/nUnC1 ; and hence j'.x/j  2 D .n C 1/  .n C 1/ C 2.
If k  n then Fk;nC1  Fk;n , so x 2 Fk;n . Applying (6) for m D n we can
see that j'.x/ \ UnC1 j  n  k C 2; since also x 2 e  .UnC1 /, we must have
'.x/nUnC1 ; by the property (1), so j'.x/j  n  k C 2 C 1 D .n C 1/  k C 2
and hence (5) also holds if we replace n with n C 1.
Therefore our induction procedure can be continued to construct an increasing
sequence fUn W n 2 !g  U
 for which the conditions (4)(7) are satisfied for all
n 2 !.
(4)
(5)
(6)
(7)

2 Solutions of problems 001500

633

It follows
from the property (5) and Fact 3 that fe.Un / W n 2 !g ! Z and hence
S
T D n2! Un . As a consequence, T 2 U
 , i.e., U is -trivial; this contradiction
shows that l.T /   and hence Fact 4 is proved.
Returning to our solution observe that the spaces X and Y are in a symmetrical
situation, so it is sufficient to prove that l.Y /  l.X /. There is no loss of generality
to assume that l.X / is an infinite cardinal; let  W Cp .Y / ! Cp .X / be a linear
homeomorphism. For any point x 2 X and f 2 Cp .X / let x .f / D f .x/. Then
x W Cp .X / ! R is a nontrivial continuous linear functional and hence the map
x  W Cp .Y / ! R is a nontrivial continuous linear functional on Cp .Y /. By TFS197, there exists a nonempty
finite set Ax  Y and a set f y .x/ W y 2 Ax g  Rnf0g
P
such that .g/.x/ D y2Ax y .x/ g.y/ for any g 2 Cp .Y /. Since the point x
will always be clear from the context, in the expression for .g/.x/ we will write
y instead of y .x/. For technical reasons, we will consider that any sum with the
empty set of summands is equal to zero.
Letting '.x/ D Ax for any point x 2 X we obtain a finite-valued lower
semicontinuous map ' W X ! Fin.Y /nf;g (see Problem 280). Fix an arbitrary
point y 2 Y and consider a continuous nontrivial linear functional  W Cp .Y / ! R
defined by .g/ D g.y/ for every g 2 Cp .Y /. Then   1 is a continuous
nontrivial linear functional on Cp .X /, so there exists a nonempty
finite set B  X
P
and a set ftx W x 2 Bg  Rnf0g such that  1 .f /.y/ D x2B tx f .x/ for any
f 2 Cp .X /.
S
If y does not belong to the finite set F D fAx W x 2 Bg then pick a function
g 2 Cp .Y / such
P that g.y/ D 1 and g.F / D f0g. For the function f D .g/ we
have f .x/ D
/ D f0g. Therefore
y2Ax y g.y/ D 0 because g.Ax /  g.FP
1
f .x/ D 0 for any x 2 B and hence g.y/ D  .f /.y/ D x2B tx f .x/ D 0
which is a contradiction. Thus y 2 F , i.e., there exists x 2 X such that y 2 Ax .
Since the point y 2 Y was chosen arbitrarily, we proved that
S
(8) the map ' is surjective, i.e., f'.x/ W x 2 X g D Y .
P
Given an open set V  Y and x 2 X let rV .x/ D f y W y 2 '.x/nV g and
consider the set e.V / D fx 2 X W rV .x/ D 0g. It turns out that
(9) the map e W .Y / ! exp.X / is a '-extractor.
If V 2 .Y / and '.x/  V then '.x/nV D ; and hence rV .x/ D 0 by our
agreement on sums with the empty set of summands; therefore 'u1 .V /  e.V /,
i.e., the property (1) holds for e.
Now assume that U; V 2 .Y / and U  V ; if x 2P
e.V /ne.U / then rV .x/ D 0
and rP
f y W y 2 '.x/ \ .V nU /g
U .x/ 0, so it follows from rU .x/ D rV .x/ C
that f y W y 2 '.x/ \ .V nU /g 0 and hence '.x/ \ .V nU / ;, i.e., we
proved that e satisfies the condition (2).
To prove that the map e has (3), fix an increasing
sequence fUn W n 2 !g  .Y /
S
such that fe.Un / W n 2 !g ! X . If U D n2! Un Y then fix a point y 2 Y nU
and consider the set G D fg 2 Cp .Y / W jg.y/j < 1g. The map  being open, the
set .G/ is an open neighborhood of the zero function of Cp .X /; in particular, there

634

2 Solutions of problems 001500

exists a finite set F  X such


S that .G/ contains the set P D ff 2 Cp .X / W
f .F /  f0gg. The set Q D f'.x/ W x 2 F g is finite, so we can find a number
k 2 ! such that Q \ U D Q \ Uk and F  e.Uk /.
Choose g 2 Cp .Y / such that g.Q \ Uk /  f0g and g.fyg [ .QnUk // D f1g.
For each x 2 F we have the equality
.g/.x/ D

X
z2'.x/\Uk

z g.z/ C

z g.z/ D rUk .x/ D 0;

z2'.x/nUk

and hence .g/ 2 .G/. Now,  is a bijection, so g 2 G whence jg.y/j < 1; this
contradiction shows that Y D U and hence e has the property (3), i.e., (9) is proved.
Recall that a set U  Y is called functionally open if we can find a set O 2
.R/ and g 2 Cp .Y / such that U D g 1 .O/. Functionally open sets are also
called cozero sets. The complements of functionally open sets are called functionally
closed. The family B of all functionally open subsets of Y is a base in Y (see Fact 1
of T.252). Say that a set U 2 B is adequate if there exists an increasing
S sequence
fFn W n 2 !g of functionally closed subsets of Y such that U D n2! Fn and
'u1 .U /n'u1 .Fn / ; for any n 2 !. The following lemma shows that there are
sufficiently many adequate sets for our purposes.
Lemma 1. Suppose that  is an infinite cardinal and U  B is a -nontrivial cover
of Y . Then, for every V  U with jVj   there exists a family W  U
! such that
(a) W is closed under finite unionsS
and jWj
S ;
(b) every W 2 W is adequate and V  W.
S
Proof. Let V D
V; the cover U being -nontrivial, the set Y nV is nonempty.
The map ' is surjective by (8), so there exists x0 2 X such that '.x0 /nV ;. Pick
U0 2 U
! with '.x0 /  U0 . Proceeding inductively, assume that n 2 ! and we
have points x0 ; : : : ; xn 2 X and sets U0 ; : : : ; Un 2 U
! such that
(10) '.xi /  Ui for all i  n;
(11) '.x0 /nV ; and '.xiC1 /n.V [ U0 [ : : : [ Ui / ; for any i < n.
Since V 0 D V [ U0 [ : : : [ Un 2 U
 , we can pick a point xnC1 2 X such that
'.xnC1 /nV 0 ;. Taking a set UnC1 2 U
! with '.xnC1 /  UnC1 we obtain points
x0 ; : : : ; xn ; xnC1 2 X and sets U0 ; : : : ; Un ; UnC1 2 U
! such that the conditions
(10) and (11) are satisfied if we replace n with n C 1. Therefore our inductive
procedure can be continued to obtain a sequence fxn W n 2 !g  X and a family
fUn W n 2 !g  U
! such
Sthat (10) and (11) are true for every n 2 !.
It is clear that U D n2! Un 2 U
! . Denote by Vf u the family of all finite
unions of elements of V. For every H 2 Vf u let OH D H [ U and consider the
family W D fOH W H 2 Vf u g. It is easySto see that the family W is closed under
finite unions; besides, jWj   and V  W.
To see that every W 2 W is adequate observe first that a countable union of
functionally open sets is a functionally open set (see Fact 1 of T.252), so we have the
inclusion W  B. If H 2 Vf u and W D OH then choose an
S increasing sequence
fFn W n 2 !g of functionally closed sets such that H D n2! Fn . We can also

2 Solutions of problems 001500

635

find, for every S


n 2 !, an increasing sequence fFnk W k 2 !g of functionally closed
sets such that k2! Fnk D Un . Let Gn D Fn [ Fn0 [ : : : [ Fnn for every n 2 !.
It is immediate that fGn W nS2 !g is an increasing sequence of functionally closed
subsets of Y and OH D n2! Gn . Since also fxn W n 2 !g  'u1 .OH / but
xnC1 'u1 .Gn / for every n 2 !, we conclude that W D OH is an adequate set
and hence Lemma 1 is proved.
Lemma 2. Given an infinite cardinal  suppose that V  .Y / is a family
S of
adequate sets with jVj   which is closed under finite unions. Then e  . V/ is
an F -subset of X .
S
Proof. Let V D V and choose an indexation fVs W s 2 S g of the family V. For
every s 2 S there exists an increasing
sequence fFns W n 2 !g of functionally closed
S
s
subsets of Y such that Vs D n2! Fn and 'u1 .Vs /n'u1 .Fns / ; for each n 2 !.
For any s 2 S and n 2 ! the set Fns is functionally separated from Y nVs (see
Fact 1 of V.140), so we can find a function gns 2 Cp .Y / such that gns .Fns /  f0g and
gns .Y nVs /  fng; let fns D .gns /. For any s 2 S and x 2 Qs D 'u1 .Vs / denote
by nsx the first integer m 2 ! such that '.x/  Fms . It follows from the inclusion
s
s
'.x/  FiCn
s that we have fiCns .x/ D 0 for any i 2 ! and therefore the set
x
x
T
s
Uks .x/ D ik fy 2 X W jfiCn
s .y/j < 1g is an open neighborhood of the point x
x
for any k 2 !.
S
T
As a consequence, As D k2! x2Qs Uks .x/ is a G -subset of X for each s 2 S .
T
The map ' being lower semicontinuous, Bs D 'l1 .VTnVs / D n2! 'l1 .V nFns / is
also a G -subset of X for any s 2 S . Therefore A D s2S .As [ Bs / is a G -subset
of X , so it suffices to show that e  .V / D X nA.
Take any y 2 e  .V /; the family V being invariant under finite unions, we can
find s 2 S such that '.y/ \ V D '.y/ \ Vs and hence y Bs . Pick k 2 ! such
that '.y/ \ V  Fks and
V .y/j  1; this is possible because rV .y/ 0.
S kjr
To see that y
fUks .x/ W x 2 Qs g take any x 2 Qs and observe that
s
s
s
fkCn
s .y/ D .k C nx /rV .y/, so it follows from .k C nx /jrV .y/j  kjrV .y/j  1
x
S s
s
that y Uk .x/. Thus y fUk .x/ W x 2 Qs g, so y .As [ Bs / and hence y A.
This proves that e  .V /  X nA.
To prove that X nA  e  .V /, take any y 2 X nA; there exists s 2 S such that
y As [ Bs . It follows from y Bs that '.y/ \ V D '.y/ \ Vs , so we can
choose
p 2 ! such that '.y/ \ V  Fps . There exists k 2 ! such that
S a number
s
y fUk .x/ W x 2 Qs g; besides we can find x 2 Qs for which '.x/nFps ;
s
and hence nsx > p. It follows from y Uks .x/ that jfiCn
s .y/j  1 for some i  k.
x
s
s
s
We have the inclusions '.y/ \ V  Fp  FiCnsx which imply that fiCn
s .y/ D
x
s
s
.i C nx /rV .y/. Therefore .i C nx /jrV .y/j  1; in particular, rV .y/ 0 and hence
y 2 e  .V /.
This proves that X nA  e  .V /, so X nA D e  .V / and hence e  .V / is, indeed,
an F -set in X , so Lemma 2 is proved.
Finally, let  DV
l.X / and consider the family L of all F -subsets of X . We have
l.L/   because L D L. Therefore Lemma 1 and Lemma 2 say that the families
L and B witness that e is a -synchronous extractor for ', so l.Y /   by Fact 4
and hence our solution is complete.

Chapter 3

Bonus results: Some Hidden Statements

The reader has, evidently, noticed that an essential percentage of the problems of
the main text is formed by purely topological statements some of which are quite
famous and difficult theorems. A common saying among Cp -theorists is that any
result on Cp -theory contains only 20% of Cp -theory; the rest is general topology.
It is evident that the author could not foresee all topology which would be needed
for the development of Cp -theory; so a lot of material had to be dealt with in the
form of auxiliary assertions. After accumulating more than seven hundred such
assertions, the author decided that some deserve to be formulated together to give a
big picture of the additional material that can be found in solutions of problems.
This section presents 80 topological statements which were proved in the solutions of problems without being formulated in the main text. In these formulations
the main principle is to make them clear for an average topologist. A student could
lack the knowledge of some concepts of the formulation; so the index of this book
can be used to find the definitions of the necessary notions. After every statement
we indicate the exact place (in this book) where it was proved.
The author considers that most of the results that follow are very useful and have
many applications in topology. Some of them are folkloric statements and quite a
few are published theorems, sometimes famous ones. For example, Fact 5 of V.290
is a classical theorem of Sierpinski that can be found in practically all textbooks
and, in particular, in Engelking (1977). Fact 1 of V.274 is a well-known result of
Arhangelskii published in Arhangelskii (1978b) and Fact 3 of V.403 is a celebrated
theorem of Hajnal and Juhsz that has an infinity of applications (see the survey of
Hodel (1984)).
To help the reader find a result he/she might need, we have classified the material
of this section according to the following topics: standard spaces, compact spaces
and their generalizations, properties of continuous maps, cardinal invariants and

Springer International Publishing Switzerland 2016


V.V. Tkachuk, A Cp-Theory Problem Book, Problem Books in Mathematics,
DOI 10.1007/978-3-319-24385-6_3

637

638

3 Bonus results: Some Hidden Statements

set theory, locally convex spaces and homotopies, zero-dimensional spaces, and
connected spaces. The last section is entitled raznoie which in Russian means
miscellaneous and contains unclassified results. The author hopes that once we
understand in which subsection a result should be, then it will be easy to find it.

3.1 Standard spaces

639

3.1 Standard spaces


By standard spaces we mean the real line, its subspaces and it powers, Tychonoff

and Cantor cubes as well as ordinals together with the Alexandroff and StoneCech
compactifications of discrete spaces.
V.022. Fact 4. The space R! is not  -totally bounded (as a linear topological space).
V.148. Fact 1. Given a cardinal  > ! let D fx 2 D W jx 1 .1/j  !g. Then
the set P D D n is a pseudocompact non-countably compact (and hence nonnormal) dense subspace of D .
V.160. Fact 1. Suppose that A; B  R are countable dense subsets and a function
f W A ! B is a bijection such that a < b implies f .a/ < f .b/. Then there exists
a unique homeomorphism h W R ! R such that hjA D f and x < y implies
h.x/ < h.y/.
V.160. Fact 8. For any n 2 N, if A and B are countable dense subspaces of Rn then
there exists a homeomorphism h W Rn ! Rn such that h.A/ D B.
V.266. Fact 1. Given a space Z suppose that a 2 Z and Z0 ; Z1 are closed subsets
of Z such that Z0 [Z1 D Z and Z0 \Z1 D fag. If there exist homeomorphisms h0 W
I ! Z0 and h1 W I ! Z1 such that h0 .1/ D h1 .1/ D a then Z is homeomorphic
to I.
V.299. Fact 1. For any nonempty open set U  Rn there is an open set V  U such
that V ' Rn .
V.302. Fact 1. For the space N D !1  .!1 C 1/ let F D f.; !1 / W < !1 g. For
any f 2 Cp .N / with f .F / D f0g there exists an ordinal  < !1 for which we have
the inclusion f.; / W  < < !1 and  <  !1 g  f 1 .0/.
V.302. Fact 2. The space T D ..!1 C 1/  .!1 C 1//nf.!1 ; !1 /g is not normal.
V.302. Fact 3. For any ordinal  let f .; / D nnf; g for every .; / 2   .
Then the map f W    !  is continuous.
V.434. Fact 2. For any infinite cardinal  the space  D fx 2 D W jx 1 .1/j < !g
is  -discrete.
V.434. Fact 3. If the Continuum Hypothesis holds then there exists a dense Luzin
subspace Z of the -product D fx 2 D!1 W jx 1 .1/j  !g of the Cantor cube
D!1 such that the space Z  Z is d -separable.
V.452. Fact 2. For any cardinal  the space D is extremally disconnected.
V.452. Fact 3. If 2!1 D c and D is a discrete space of cardinality !1 then the space
D embeds in !.

640

3 Bonus results: Some Hidden Statements

V.488. Fact 1. The following conditions are equivalent for any ultrafilter  2 !  :
(i)  is a P -point in !  ;
(ii) for any countable family E  exp !n, there exists A 2  such that A \ E is
finite for any E 2 E;
(iii) for any countable disjoint family E  exp !n, there exists A 2  such that
A \ E is finite for any E 2 E.

3.2 Compact spaces and their generalizations

641

3.2 Compact spaces and their generalizations


This Section contains some statements about compact and countably compact
spaces.
V.072. Fact 1. Any sequentially compact space is countably compact; if Z is
countably compact and sequential then Z is sequentially compact.
V.072. Fact 2. Any finite product of sequentially compact spaces is sequentially
compact and hence countably compact.
V.073. Fact 1. Suppose that Z is a space and is a cardinal such that l.Z/ 
and A is compact for any A  Z with jAj  . Then Z is compact.
V.161. Fact 1. Given m 2 ! suppose that K is a compact space with dim K  m
and N is a second countable space. Then, for any continuous map p W K ! N there
exists a second countable space L such that dim L  m and there are continuous
maps q W K ! L and r W L ! N for which r q D p.
V.200. Fact 1. If K is a countably infinite compact space then for any point a K
the space K fag is homeomorphic to K.
V.290. Fact 5. Suppose that Z is a connected compact spaceS
and A D fZn W n 2 !g
is a disjoint family of closed subsets of Z such that Z D n2! Zn . Then at most
one element of A is nonempty.
V.296. Fact 1. For any infinite compact space X let .X / be the one-point
compactification of the space X  !. Then
(1) w.X / D w..X //; in particular, if X is second countable then .X / is also
second countable;
(2) if X is zero-dimensional then .X / is also zero-dimensional;
(3) .X / X ' .X /.
V.423. Fact 1. For any compact space X there exists a discrete D  .X  X /n X
such that jDj D d.X /.
V.480. Fact 5. If X is a countably compact space and t .X / > ! then X has an
uncountable free sequence.

642

3 Bonus results: Some Hidden Statements

3.3 Properties of continuous maps


We consider the most common classes of continuous maps: open, closed, perfect,
and quotient. The respective results basically deal with preservation of topological
properties by direct and inverse images.
V.077. Fact 1. Given spaces Z; T and an open continuous onto map f W Z ! T ,
the map fA D f jf 1 .A/ W f 1 .A/ ! A is open for any A  T .
V.192. Fact 3. Assume that n 2 N and f W Z ! T is an open map with
jf 1 .t /j D n for any t 2 T . Then
(a) f is perfect;
(b) f is a local homeomorphism, i.e., for any z 2 Z there is W 2 .z; Z/ such that
f jW W W ! f .W / is a homeomorphism.
V.194. Fact 9. Given metrizable spaces Z and T and a continuous map f W Z ! T
the following conditions are equivalent:
(a) f is an almost perfect map, i.e., f .F / is closed in T for any closed F  Z and
f 1 .t / is compact for any t 2 T (recall that an almost perfect continuous map
is perfect if and only if it is surjective);
(b) a sequence S D fzn W n 2 !g  Z has a convergent subsequence if and only if
the sequence S 0 D ff .zn / W n 2 !g  T has a convergent subsequence.
V.251. Fact 2. For any spaces Z and T , a map g W Z ! T is R-quotient if and
only if, for any function h W T ! I, it follows from continuity of h g that h is
continuous.
V.272. Fact 1. Given spaces T and W if ' W Cp .T / ! Cp .W / is a linear
homeomorphism then there exists a linear homeomorphism W RT ! RW such
that jCp .T / D '.
V.274. Fact 1. Given spaces P; Q and an infinite cardinal  suppose that t .Q/  
and there exists a continuous closed onto map f W P ! Q such that t .f 1 .q//  
for any q 2 Q. Then t .P /  . As a consequence, if K is a compact space with
t .K/   then t .Q  K/  .
V.304. Fact 2. If A is a retract of a space Z then the set F D clZ .A/ is a retract of
Z.
V.306. Fact 1. For any space Z its diagonal D f.z; z/ W z 2 Zg is a retract of
Z  Z.
V.343. Fact 2. If Z is a k-space and f W Z ! Y is a quotient map then Y is also a
k-space.
V.452. Fact 1. Suppose that f W X ! Y is a closed irreducible onto map. If Y is
extremally disconnected then f is a homeomorphism.

3.4 Cardinal invariants and set theory

643

3.4 Cardinal invariants and set theory


To classify function spaces, using cardinal invariants often gives crucial information.
This Section includes both basic, simple results on the topic and very difficult
classical theorems.
V.023. Fact 1. Given an infinite cardinal , if hd.Y /   and w.Z/   then
hd.Y  Z/  . In particular, the product of a second countable space and a
hereditarily separable space is hereditarily separable.
V.274. Fact 1. Given spaces P; Q and an infinite cardinal  assume that t .Q/  
and there exists a continuous closed onto map f W P ! Q such that t .f 1 .q//  
for any q 2 Q. Then t .P /  . As a consequence, if K is a compact space with
t .K/   then t .Q  K/  .
V.274. Fact 2. Given a space P suppose that fUa W a 2 Ag is an open cover of P
such that t .Ua /   for any a 2 A. Then t .P /  .
V.274. Fact 3. The weak functional tightness of the space P D V .!/  V .c/ is
uncountable.
V.403. Fact 1. For any space X we have jX j  d.X /t.X/
inequality jX j  d.X / .X/ holds for every space X .
V.403. Fact 2. If X is an arbitrary space then jX j  2l.X/
have the inequality jX j  2l.X/ .X/ for any space X .

.X/

. In particular, the

.X/t.X/

. In particular, we

V.403. Fact 3. For any space X we have the inequality jX j  2c.X/ .X/ .
V.413. FactQ1. Assume that Nt is a space with nw.Nt / D .Nt / D ! for all t 2 T ,
and N D t2T Nt . Then, for any a 2 N , every subspace of .N; a/ has a pointcountable -base.
V.439. Fact 1. If X is a compact space then h .x; X /  t .X / for any x 2 X .
V.456. Fact 1. Suppose that  is an infinite cardinal and Z is a space such that
l.Z/   and t .Z/  . Then Z has no free sequences of length  C .
V.490. Fact 1. Suppose that D is a nonempty set and f W D ! D is a map such
that f .x/ x for any x 2 D. Then there exist disjoint subsets A0 ; A1 ; A2 of the set
D such that D D A0 [ A1 [ A2 and f .Ai / \ Ai D ; for any i D 0; 1; 2.

644

3 Bonus results: Some Hidden Statements

3.5 Locally Convex Spaces and Homotopies


The methods of the homotopy theory were used by Gulko to establish that Cp .!1 /
is not homeomorphic to its square. Here we present the results he used to prove this
famous theorem.
V.024. Fact 3. The space Lp .Z/ has the Souslin property for any space Z.
V.100. Fact 1. Given spaces X; Y; Z and maps f; g; h such that f; g 2 C.X; Y / and
h 2 C.Y; Z/, if f is homotopic to g then h f is homotopic to h g.
V.100. Fact 2. Given spaces X; Y; Z and maps f; g; h such that f; g 2 C.Y; Z/ and
h 2 C.X; Y /, if f is homotopic to g then f h is homotopic to g h.
V.100. Fact 3. Homotopical equivalence is an equivalence relation on the class of
topological spaces.
V.100. Fact 9. If n < m then any continuous function f W S n ! S m is homotopic
to a constant map.
V.100. Fact 10. For any n 2 ! and p 2 RnC1 the spaces RnC1 nfpg and S n are
homotopically equivalent.
V.100. Fact 11. For any distinct n; m 2 N the spaces S n and S m are not
homotopically equivalent.
V.100. Fact 15. Suppose that L is a linear topological space and K  L is a linear
subspace of L of codimension n. Then LnK is homotopically equivalent to S n1 .
V.228. Fact 1. Suppose that N is a linear topological space and G  N is a locally
compact linear subspace of N . Then G is closed in N .
V.233. Fact 1. Given a linear space L suppose that and  are linear space
topologies on L such that, for any U 2 with 0L 2 U , there exists V 2  such that
0 2 V  U . Then  .
V.246. Fact 2. For any space Y , if Z is a closed subspace of Y then the linear hull
H of the set Z in Lp .Y / is closed in Lp .Y /.
V.246. Fact 3. Given a space Y , if Z  Y is C -embedded in Y then, for
the linear hull H of the set Z in Lp .Y /, there exists a linear homeomorphism
' W Lp .Z/ ! H such that '.z/ D z for any z 2 Z.
V.250. Fact 1. Suppose that L is a linear topological space and K is a compact
subset of L. Then the set K C A is closed in L for any closed A  L.
V.250. Fact 2. Given a locally convex space L suppose that M is a closed linear
subspace of L and G is a finite-dimensional linear subspace of L. Then M C G is
a closed subspace of L.

3.6 Zero-dimensional Spaces and Connected Spaces

645

3.6 Zero-dimensional Spaces and Connected Spaces


A selection of results on connected spaces and zero-dimensional spaces is presented here. Even though every space Cp .X / is connected and hence never
zero-dimensional, each of the above-mentioned properties turns out to be important
in Cp -theory.
V.097. Fact 1. Suppose that X is a cosmic space such that X nfag is zerodimensional for some point a 2 X . Then X is zero-dimensional. In other words,
adding a point to a cosmic zero-dimensional space gives a zero-dimensional space.
V.097. Fact 2. Suppose that X is a second countable space and F; G  X are
disjoint closed subsets of X . Then, for any zero-dimensional Z  X , there is a
partition C between the sets F and G such that C \ Z D ;.
V.290. Fact 1. Suppose that Z is a space and, for any points x; y 2 Z, there exists
a connected subspace C  Z such that fx; yg  C . Then Z is connected.
V.290. Fact 2. Suppose that Z is a T
space and C D fCt W t 2 S
T g is a family of
connected subspaces of Z such that C ;. Then the set C D C is connected.
As a consequence, the component of any point of Z is a closed connected subspace
of Z.
V.290. Fact 3. Given any space Z and z 2 Z we have Kz  Qz , i.e., the component
of any point is contained in its quasi-component. If Z is compact then Kz D Qz .
V.290. Fact 4. Suppose that Z is a connected compact space and F is a nonempty
closed subset of Z such that F Z. Then, for any z 2 F , the component K of the
point z in the space F intersects the boundary B D F nInt.F / of the set F in Z.
V.290. Fact 5. Suppose that Z is a connected compact spaceS
and A D fZn W n 2 !g
is a disjoint family of closed subsets of Z such that Z D n2! Zn . Then at most
one element of A is nonempty.

V.298. Fact 1. A second countable zero-dimensional space is Cech-complete


if and
only if it embeds in P as a closed subspace.
V.495. Fact 1. Suppose that Z is a zero-dimensional space and T is a space for
which there exists a closed finite-to-one map ' W T ! Z. Then T is also zerodimensional.

646

3 Bonus results: Some Hidden Statements

3.7 Raznoie (Unclassified results)


Last but not least, we placed here some interesting results which do not fit in any of
the earlier sections.
V.023. Fact 3. Suppose that X is a space such that Cp .X / is Lindelf. Given a set
A  X assume that every countable subset of A is C -embedded in X . Then A is
C -embedded in X .
V.140. Fact 1. Given disjoint functionally closed sets F and G in a space Z there
exists a function f 2 C.Z; 0; 1
/ such that f .F /  f0g and f .G/  f1g.
In particular, there exist functionally open sets O.F / and O.G/ and disjoint
functionally closed sets P .F /; P .G/ such that F  O.F /  P .F / and G 
O.G/  P .G/.
V.140. Fact 2. Suppose that F is functionally closed in a space Z and O is a
functionally open subset of Z for which F  O. Then there is a functionally open
set G  Z such that F  G  G  O.
V.147. Fact 1. Given a Tychonoff space Z, suppose that F1 ; : : : ; Fm are functionally
closed subsets of Z such that F1 \: : :\Fm D ;. Then clZ .F1 /\: : :\clZ .Fm / D ;.
V.156. Fact 1. Given a space Z and a natural number m we have dim Z  m if
and only if any functionally open cover U D fU0 ; : : : ; UmC1
Tg of the space Z has a
functionally open shrinking V D fV0 ; : : : ; VmC1 g such that imC1 Vi D ;.
V.245. Fact 1. Given a space Z, a set Y  Z is not bounded in Z if and only if
there exists a discrete family fUn W n 2 !g  .Z/ such that Un \ Y ; for any
n 2 !.
V.246. Fact 1. If Y is a Dieudonn complete space and B is a bounded subset of Y
then B is compact.

V.291. Fact 1. Any paracompact locally Cech-complete


space is Cech-complete.
V.291. Fact 2. Given a paracompact space Z suppose that D is a closed discrete

subset of Z and .d; Z/  ! for each d 2 D. If ZnD is Cech-complete


then Z is

also Cech-complete.
V.301. Fact 1. Any space with at most one non-isolated point is hereditarily
paracompact.
V.433. Fact 3. If Z is a space and U is a weakly  -point-finite family of nonempty
open subsets of Z then there is a  -discrete subspace D  Z such that D \ U ;
for any U 2 U .

3.7 Raznoie (Unclassified results)

647

V.433. Fact 5. Suppose that Z is a Lindelf -space and F is a fixed countable


network with respect to a compact cover C of the space Z. Assume additionally that
F is closed under finite intersections, and we have a condensation f W Z ! Z 0 .
If Y  Z and A  Y is a set such that f .A \ F / is dense in f .Y \ F / for any
F 2 F then Y  A.
V.433. Fact 6. If a Lindelf -space Z condenses onto a hereditarily d -separable
space then Z is hereditarily d -separable.

Chapter 4

Open problems

This Chapter contains 100 unsolved problems which are classified by topics
presented in seven sections, the names of which outline what the given group of
problems is about. At the beginning of each section we define the notions which
are not defined in the main text. Each published problem has a reference to the
respective paper or book. If it is unpublished, then my opinion on who is the author is
expressed. The last part of each problem is a very brief explanation of its motivation
and/or comments referring to the problems of the main text or some papers for
additional information. If the paper is published and the background material is
presented in the main text, we mention the respective exercises. If the main text
contains no background, we refer the reader to the original paper. If no paper is
mentioned in the motivation part, then the reader must consult the paper/book in
which the unsolved problem was published.
To do my best to assign the right author to every problem, I implemented the
following simple principles:
1. If the unsolved problem is published, then I cite the publication and consider
myself not to be involved in the decision about who is the author. Some
problems are published many times and I have generally preferred to cite the
articles in journals/books which are more available for the Western reader. Thus
it may happen that I do not cite the earliest paper where the problem was
formulated. Of course, I mention it explicitly, if the author of the publication
attributes the problem to someone else.
2. If, to the best of my knowledge, the problem is unpublished then I mention the
author according to my personal records. The information I have is based upon
my personal acquaintance and communication with practically all specialists
in Cp -theory. I am aware that it is a weak point and it might happen that the
problem I attributed to someone was published (or invented) by another person.
However, I did an extensive work ploughing through the literature to make sure
that this does not happen.

Springer International Publishing Switzerland 2016


V.V. Tkachuk, A Cp-Theory Problem Book, Problem Books in Mathematics,
DOI 10.1007/978-3-319-24385-6_4

649

650

4 Open problems

4.1 Mappings which involve Cp -spaces


The existence of an algebraic structure compatible with the topology of Cp .X /
radically improves its topological properties. In particular, if Cp .X / is an open
continuous image of some nice space Z then Cp .X / might have even better
properties than Z. For example, if Cp .X / is an open image of a metrizable space
then it is second countable. Also, if a space Z is a continuous image of Cp .X / then
we can expect very strong restrictions on Z if, say, Z is compact. A lot of research
has been done in this area and this Section contains a compilation of the respective
open questions.
4.1.1 Suppose that Cp .Y / is an image of Cp .X / under a continuous (an open
continuous) mapping. Is it true that s.Y /  s.X /?
Published in Okunev (1997a)
Motivated by the fact that spread is t -invariant (Problem 068)
4.1.2 Suppose that Cp .Y / is an image of Cp .X / under a continuous (an open
continuous) mapping. Is it true that hd.Y /  hd.X /?
Published in Okunev (1997a)
Motivated by the fact that hereditary density is t -invariant (Problem 069)
4.1.3 Suppose that Cp .Y / is an image of Cp .X / under a continuous (an open
continuous) mapping. Is it true that hl.Y /  hl.X /?
Published in Okunev (1997a)
Motivated by t -invariance of hereditary Lindelf number (Problem 070)
4.1.4 Assume that X is Eberlein compact and S is a dense subspace of Cp .X /.
Suppose that Y is a continuous image of S and B  Y is compact. Is it true
that B has countable tightness (is sequential or monolithic)?
Published in Tkachenko (1985)
Motivated by the fact that t .B/  !1 .
4.1.5 Let K be a compact space with jKj  c. Is it true in ZFC that every compact
continuous image of Cp .K/ is metrizable?
Published in Tkachenko and Tkachuk (2005)
Motivated by the fact that this is true under Luzins Axiom (Problem 452)
4.1.6 Let K be a first countable compact space. Is it true in ZFC that every
compact continuous image of Cp .K/ is metrizable?
Published in Tkachenko and Tkachuk (2005)
Motivated by the fact that this is true under Luzins Axiom (Problem 452)
4.1.7 Let K be a FrchetUrysohn compact space. Is it true that every compact
continuous image of Cp .K/ is metrizable?

4.1 Mappings which involve Cp -spaces

651

Published in Tkachenko and Tkachuk (2005)


Comment this is true under Luzins Axiom if K has countable character
(Problem 452)
4.1.8 Let K be a compact space of countable tightness. Is it true that every
compact continuous image of Cp .K/ is metrizable?
Published in Tkachenko and Tkachuk (2005)
Comment this is true under Luzins Axiom if K has countable character
(Problem 452)
4.1.9 Let K be a hereditarily separable compact space. Is it true in ZFC that every
compact continuous image of Cp .K/ is metrizable?
Published in Tkachenko and Tkachuk (2005)
Motivated by the fact that this is true under Luzins Axiom (Problem 452)
4.1.10 Suppose that K is a compact space and Cp .K/ maps continuously onto
I for some uncountable cardinal . Is it true that the space K contains a
discrete C  -embedded subset of cardinality ?
Published in Tkachenko and Tkachuk (2005)
Related to Problem 451
4.1.11 Let K be a scattered (or left-separated) compact space. Is it true that every
compact continuous image of Cp .K/ is metrizable?
Published in Tkachenko and Tkachuk (2005)
Comment this is true if K is Corson compact (Problem 456)
4.1.12 Let X be a space such that Cp .X / is Lindelf. Is it true that every compact
continuous image of Cp .X / is metrizable?
Published in Tkachenko and Tkachuk (2005)
Motivated by the fact that it is true if t .Cp .X // D ! (Problem 456)
4.1.13 Let X be a Lindelf P -space. Is it true that every compact continuous image
of Cp .X / is metrizable?
Published in Tkachenko and Tkachuk (2005)
4.1.14 Suppose that K is an Eberlein compact space and ' W Cp .K/ ! Y is a
continuous surjective map of Cp .K/ onto a  -compact space Y . Must Y be
cosmic?
Published in Tkachuk (2009b)
Motivated by the fact that this is true if ' is a condensation
4.1.15 Suppose that K is a Corson compact space and ' W Cp .K/ ! Y is a
continuous surjective map of Cp .K/ onto a  -compact space Y . Must Y
be cosmic?
Published in Tkachuk (2009b)
Motivated by the fact that this is true if ' is a condensation

652

4 Open problems

4.2 Properties preserved by t-equivalence


The main line of study in this area is to find out which common properties the
spaces X and Y must have if Cp .X / is homeomorphic to Cp .Y /. In many cases the
respective open question reflects an attempt to check whether an l-invariant property
is also t -invariant.
4.2.1 Suppose that X and Y are compact scattered t -equivalent spaces. Must X
and Y be u-equivalent?
Published in Arhangelskii (1992b)
Comment the answer is yes if X and Y are countable (Problem 200).
4.2.2 Let X and Y be t -equivalent spaces. Is it true that dim X D dim Y ?
Published in Arhangelskii (1990a)
Motivated by the fact that this is true if the spaces X and Y are uequivalent (Problems 180182).
4.2.3 Is Cp .I/ homeomorphic to Cp .K/?
Published in Arhangelskii (1998b)
Motivated by the fact that a positive answer would show that the dimension
dim is not t -invariant. It is also an open question whether Cp .I/ is
homeomorphic to Cp .I  I/ (Arhangelskii (1992b))
4.2.4 Let X be an infinite (compact) space. Is it true that Cp .X / is homeomorphic
to Cp .X /  R?
Published in Arhangelskii (1998b)
Motivated by the fact that, for any n 2 N, there is a space Y such that
Cp .X / is homeomorphic to Y  Rn .
Related to Problems 399 and 400
4.2.5 Suppose that there exists a homeomorphism ' W RX ! RY for which we
have the equality '.Cp .X // D Cp .Y /. Is it true that dim X D dim Y ?
Published in Arhangelskii (1992b)
Related to Problems 180182, 054
4.2.6 Let X and Y be t -equivalent spaces. Is it true that l.X / D l.Y /?
Published in Arhangelskii (1992b)
Motivated by the fact that this is true if the spaces X and Y are l-equivalent
(Problem 500)
4.2.7 Suppose that X is t -equivalent to Y and X is a k-separable space. Must Y
be k-separable?
Published in Arhangelskii (1989a)
Motivated by the fact that the answer is yes if X D Cp .X 0 / and Y D
Cp .Y 0 / for some spaces X 0 and Y 0 (Problem CFS-315).

4.2 Properties preserved by t-equivalence

653

4.2.8 Let X be a metrizable  -compact space. Does there exist a compact space
Y such that X is t -equivalent to Y ?
Published in Arhangelskii (1992b)
Related to Problem 043
4.2.9 Let X be a first countable space which is t -equivalent to a second countable
space. Must X be second countable?
Published in Arhangelskii (1992b)
Related to Problem 348
4.2.10 Suppose that X is t -equivalent to Y and Y is  -pseudocompact. Must X be
 -pseudocompact?
Published in Arhangelskii (1989a)
Related to Problems 043 and 045
4.2.11 Suppose that X is t -equivalent to Y and Y is  -countably compact. Must X
be  -countably compact?
Published in Arhangelskii (1992b)
Related to Problems 043 and 045
4.2.12 Suppose that X is t -equivalent to Y and Y is a Hurewicz space. Must X be
a Hurewicz space?
Published in Arhangelskii (1989a)
Related to Problems 014 and 043
4.2.13 Does there exist an infinite compact space X such that any space, which is
t -equivalent to X , is compact (homeomorphic to X )?
Published in Arhangelskii (1992b)
Motivated by the fact that there exists a space Y such that Cp .X / '
Cp .Y / implies that X is pseudocompact (Problem TFS-400).
4.2.14 Is it true that every infinite compact space is t -equivalent to a compact space
containing a nontrivial convergent sequence?
Published in Arhangelskii (1990a)
Motivated by the fact that there are compact spaces without nontrivial
convergent sequences which are l-equivalent to compact spaces with
nontrivial convergent sequences (Problem 270)
4.2.15 Suppose that X is t -equivalent to Y and all countable subsets of X are
closed. Is it true that all countable subsets of Y must be closed?
Published in Arhangelskii (1983a)
Motivated by the fact that if all countable subsets of X are closed and
C -embedded then all countable subsets of the space Y also have this
property (Problem TFS-485).

654

4 Open problems

4.2.16 Given a cardinal , a space X is called -scattered, if jAj   for any


A  X with jAj  . Suppose that X is a -scattered space and Cp Cp .X /
is homeomorphic to Cp Cp .Y /. Must the space Y be -scattered?
Published in Arhangelskii (1989a)
t
Motivated by the fact that Y  X implies Y is -scattered (Problem
SFFS-187).
4.2.17 Say that a space X is a k! -space if it is a hemicompact k-space. Is it true
that the k! -property is preserved by t -equivalence?
Published in Arhangelskii (1988a)
Related to Problem 340
4.2.18 Suppose that a space X is t -equivalent to a compact space. Must X have the
k! -property? Must X be k-space or hemicompact space?
Published in Arhangelskii (1988a)
Related to Problem 340
4.2.19 Suppose that a space X is t -equivalent to R. Must X be k! -space?
Published in Arhangelskii (1988a)
Related to Problem 340
4.2.20 Suppose that X is t -equivalent to a K -space Y . Must X be a K -space?
Published in Arhangelskii (1988a)
Motivated by the fact that K-analyticity is t -invariant (Problem 043).
4.2.21 Assume that X and Y are t -equivalent spaces. Is it true that the space X  Z
is t -equivalent to Y  Z for any compact space Z?
Published in Okunev (1995b)
l

Motivated by the fact that it is true if X  Y .


4.2.22 Suppose that Cp .X / is homeomorphic to Cp .Y /. Must the space Lp .X / be
homeomorphic to Lp .Y /?
Published in Arhangelskii and Choban (1989)
l

Motivated by the fact that this is true if X  Y (Problem 237).


4.2.23 Suppose that Lp .X / is homeomorphic to Lp .Y /. Must Cp .X / be homeomorphic to Cp .Y /?
Published in Arhangelskii and Choban (1989)
Motivated by the fact that this is true if Lp .X / is linearly homeomorphic
to Lp .Y / (Problem 237).

4.3 Properties preserved by u-equivalence

655

4.3 Properties preserved by u-equivalence


The main line of study in this area is to find out which common properties the spaces
X and Y must have if Cp .X / is uniformly homeomorphic to Cp .Y /. In many cases
the respective open question reflects an attempt to check whether an l-invariant
property is also u-invariant.
4.3.1 Say that a space X is a k! -space if it is a hemicompact k-space. Is it true that
the k! -property is preserved by u-equivalence?
Published in Arhangelskii (1992b)
Related to Problem 340
4.3.2 Suppose that a space X is u-equivalent to R. Must X be k! -space?
Published in Arhangelskii (1988a)
Related to Problem 340
4.3.3 Is the class of @0 -spaces u-invariant?
Published in Arhangelskii (1992b)
Related to Problem 348
4.3.4 Is the class of perfect spaces u-invariant?
Published in Arhangelskii (1992b)
Related to Problem 070
4.3.5 Suppose that there exists a homeomorphism ' W RX ! RY for which we
have the equality '.Cp .X // D Cp .Y /. Is it true that X is u-equivalent to Y ?
Published in Arhangelskii (1992b)
Related to Problem 139

4.3.6 Let X and Y be u-equivalent metrizable spaces. Is it true that X is Cechcomplete if and only if so is Y ?
Published in Marciszewski and Pelant (1997)
Motivated by the fact that this is true if the spaces X and Y are l-equivalent
(Problems 366367).

656

4 Open problems

4.4 Properties preserved by l -equivalence


The topology of a space X is completely determined by the structure of topological
algebra on Cp .X /; so it is natural to ask what is implied by linear topological
properties of Cp .X /. In other words, the main line of research is to find out
what common properties the spaces X and Y must have if Cp .X / is linearly
l

homeomorphic to Cp .Y /. Recall that a space X is l-stable if X  X A for any


l-embedded set A  X . The space X is S -stable if X  .! C 1/ is l-equivalent
to X .
4.4.1 Suppose that X is a countably compact space and X is l-equivalent to a
space Y . Must Y be countably compact?
Published in Arhangelskii (1988a)
Motivated by by the fact that pseudocompactness is even u-invariant and
hence l-invariant (Problems 134 and 193)
4.4.2 Say that a space X is -pseudocompact for some cardinal , if f .X / is
compact for any continuous f W X ! Y such that w.Y /  . Is it true that
-pseudocompactness is preserved by l-equivalence?
Published in Arhangelskii (1998a)
Motivated by the fact that !-pseudocompactness is l-invariant because it
coincides with pseudocompactness (Problems 134 and 193).
4.4.3 Suppose that X is a k-separable space and X is l-equivalent to a space Y .
Must Y be k-separable?
Published in Arhangelskii (1988a)
Motivated by the fact that the answer is positive if X D Cp .X 0 / and Y D
Cp .Y 0 / for some spaces X 0 and Y 0 (Problem CFS-315).
4.4.4 Is the class of perfect spaces l-invariant?
Published in Arhangelskii (1992b)
Motivated by the fact that the hereditary Lindelf property is known to be
even t -invariant (Problem 070).
4.4.5 Suppose that X and Y are metrizable spaces and each one embeds into the
l

other as a closed subspace. Is it true that X  Y ?


Published in Arhangelskii (1992b)
Motivated by the fact that this is true if one of them is l-stable
4.4.6 Suppose that X and Y are (compact, or compact metrizable) spaces and, for
l

each one, there exists an l-embedding into the other. Is it true that X  Y ?
Published in Arhangelskii (1991)
Motivated by the fact that this is true if one of them is l-stable

4.4 Properties preserved by l-equivalence

657

4.4.7 Is every infinite metrizable compact space l-stable (or S -stable)?


Published in Arhangelskii (1991)
Motivated by the fact that X  .! C 1/ is S -stable for any space X .
4.4.8 Is every infinite metrizable space l-stable (or S -stable)?
Published in Arhangelskii (1991)
Motivated by the fact that X  .! C 1/ is S -stable for any space X .
4.4.9 Is it true that every infinite l-stable (compact) space is S -stable?
Published in Arhangelskii (1991)
Comment for metrizable spaces, S -stability implies l-stability.
l

4.4.10 Suppose that Y is a nonempty space with Y Y  Y . Is it true that Cp .Y /


is linearly homeomorphic to Cp .Y /  R?
Published in Arhangelskii (1991)
Motivated by the fact that the conclusion is true for metrizable spaces.
4.4.11 Suppose that X is a K -space which is l-equivalent to a space Y . Must Y
be a K -space?
Published in Arhangelskii (1988a)
Motivated by the fact that K-analyticity is t -invariant (Problem 043).
4.4.12 Is it true that every infinite compact space is l-equivalent to a compact space
containing a nontrivial convergent sequence?
Published in Arhangelskii (1990a)
Motivated by the fact that there are compact spaces without nontrivial
convergent sequences which are l-equivalent to compact spaces with
nontrivial convergent sequences (Problem 270).
4.4.13 Is it true that every infinite compact space K is l-equivalent to a compact
space containing a point of countable character?
Published in Arhangelskii (1990a)
4.4.14 Suppose that X and Y are (perfectly) normal l-equivalent spaces. Is it true
that ext .X / D ext .Y /?
Published in Baars and Gladdines (1996)
Motivated by l-invariance of the Lindelf property (Problem 500).

658

4 Open problems

4.5 Generalizations of functional equivalences


Considering condensations or linear embeddings instead of linear homeomorphisms
gives a natural weakening of l-equivalence; so there were numerous attempts to
strengthen respectively the positive results on l-invariance of topological properties.
To obtain the relevant results, sometimes, instead of l-equivalence of X and Y
l

it suffices to assume that Cp .X /  Cp .Y / or even Cp .Cp .X //  Cp .Cp .Y //. The


problems of this Section constitute a sample of what we could expect if we try
to prove the counterparts of some well-known results for a weakened notion of
l-equivalence.
4.5.1 Given spaces X and Y , suppose that X is compact and there exists a linear
condensation of Cp .X / onto Cp .Y / as well as a linear condensation of
Cp .Y / onto Cp .X /. Must Y be compact?
Published in Arhangelskii (1992b)
Motivated by the fact that compactness is l-invariant (Problem 193).
4.5.2 Given spaces X and Y , suppose that X is Lindelf and there exists a (linear)
condensation of Cp .X / onto Cp .Y / as well as a (linear) condensation of
Cp .Y / onto Cp .X /. Must Y be Lindelf?
Published in Arhangelskii (1992b)
Motivated by l-invariance of the Lindelf property (Problem 500).
4.5.3 Given spaces X and Y , suppose that X is  -compact and there exists a (linear) condensation of Cp .X / onto Cp .Y / as well as a (linear) condensation
of Cp .Y / onto Cp .X /. Must Y be  -compact?
Published in Arhangelskii (1992b)
Motivated by the fact that  -compactness is t -invariant (Problem 043).
4.5.4 Given spaces X and Y , suppose that X is a Lindelf -space and there
exists a (linear) condensation of Cp .X / onto Cp .Y / as well as a (linear)
condensation of Cp .Y / onto Cp .X /. Must Y be a Lindelf -space?
Published in Arhangelskii (1992b)
Motivated by t -invariance of the Lindelf -property (Problem 043).
4.5.5 Given spaces X and Y , suppose that X is analytic and there exists a (linear)
condensation of Cp .X / onto Cp .Y / as well as a (linear) condensation of
Cp .Y / onto Cp .X /. Must Y be analytic?
Published in Arhangelskii (1992b)
Motivated by the fact that analyticity is l-invariant (Problem 044).
4.5.6 Suppose that X is Lindelf and Cp .Y / (linearly) embeds in Cp .X /. Must Y
be Lindelf?
Published in Arhangelskii (1992b)
Motivated by l-invariance of the Lindelf property (Problem 500).

4.5 Generalizations of functional equivalences

659

4.5.7 Suppose that X is Lindelf and each one of the spaces Cp .X / and Cp .Y /
(linearly) embeds in the other. Must Y be Lindelf?
Published in Arhangelskii (1992b)
Motivated by l-invariance of the Lindelf property (Problem 500).
4.5.8 Suppose that Cp .Cp .X // is homeomorphic to Cp .Cp .Y // and the space Y
is ( -)compact. Must X be  -compact?
Published in Arhangelskii (1988a, 1989a)
Motivated by the fact that  -compactness is t -invariant (Problem 043).
4.5.9 Suppose that Cp .Cp .X // is homeomorphic to Cp .Cp .Y // and Y is a
Lindelf -space. Must X be Lindelf -space?
Published in Arhangelskii (1988a)
Motivated by t -invariance of the Lindelf -property (Problem 043).
4.5.10 Suppose that Cp .Cp .X // is homeomorphic to Cp .Cp .Y // and Y is discrete.
Must X be discrete?
Published in Arhangelskii (1989a)
Motivated by the fact that discreteness is t -invariant (Problem TFS-487).
4.5.11 Suppose that Cp .Cp .X // is homeomorphic to Cp .Cp .Y // and Y is countable (has cardinality ). Must X be countable (have cardinality )?
Published in Arhangelskii (1988a)
Motivated by the fact that cardinality is t -invariant (Problem 001).
4.5.12 Suppose that X is finite and Cp .Cp .Cp .X /// is (linearly) homeomorphic to
Cp .Cp .Cp .Y ///. Must Y be finite?
Published in Arhangelskii (1988a)
Motivated by the fact that discreteness is t -invariant (Problem TFS-487).
4.5.13 Suppose that X is a dyadic compact space such that Cp .X / embeds in
Cp .Y / for some Eberlein compact Y . Must X be metrizable?
Published in Arhangelskii (1992b)
Motivated by the fact that the Eberlein property is t -invariant in compact
spaces (Problem 033).
4.5.14 Suppose that X is a compact space such that Cp .X / embeds in Cp .Y / for
some Eberlein compact Y . Must X be Eberlein compact?
Published in Arhangelskii (1992b)
Motivated by the fact that the Eberlein property is t -invariant in compact
spaces (Problem 033).

660

4 Open problems

4.5.15 Suppose that X is a compact space such that Cp .X / embeds in Cp .Y / for


some Corson compact Y . Must X be Corson compact?
Published in Arhangelskii (1992b)
Motivated by the fact that the Corson property is t -invariant in compact
spaces (Problem 033).
4.5.16 Suppose that X and Y are compact spaces such that Cp;n .X / is homeomorphic to Cp;n .Y / for some n 2 N. Is it true that Cp .X / is K-analytic if and
only if Cp .Y / is K-analytic?
Published in Arhangelskii (1988a)
Motivated by the fact that the space X is Gulko compact if and only if so
is Y (Problem CFS-243).
4.5.17 Say that spaces X and Y are at -equivalent if there exists a homeomorphism
' W RX ! RY for which we have the equality '.Cp .X // D Cp .Y /.
Suppose that X and Y are at -equivalent and X is an @0 -space. Is it true
that Y is also @0 -space?
Published in Arhangelskii (1992b)
Motivated by the fact that @0 -property is l-invariant (Problem 348).
4.5.18 Suppose that Cp .X / is linearly homeomorphic to Cp .Y /L for some linear
topological space L. Is it true that dim Y  dim X ?
Published in Arhangelskii (1990a)
Motivated by the fact that this is true for compact spaces (Problem 396).

4.6 Fuzzy questions

661

4.6 Fuzzy questions


A question belongs to this Section if it does not say exactly what its author wants
us to prove or find out but rather expresses an intuitive idea of what should be
done. Such questions can have many different solutions and their inherent difficulty
consists in impossibility to be sure whether a given solution is satisfactory.
4.6.1 Given n 2 !, find a characterization of the property dim X  n in terms
of the linear topological structure of Cp .X /.
Published in Arhangelskii (1992b)
Related to Problems 180182
4.6.2 Given n 2 !, find a characterization of the property dim X  n in terms
of the uniform structure of Cp .X /.
Published in Arhangelskii (1992b)
Related to Problems 180182
4.6.3 Find a method of constructing t -equivalent but not l-equivalent spaces.
Published in Arhangelskii (1989a)
4.6.4 Does there exist a natural (linear) topological property of the space Cp .X /
which characterizes Lindelf spaces X ?
Published in Arhangelskii (1989a)
Motivated by the fact that Lindelf property is l-invariant (Problem 500)
4.6.5 Characterize hereditary Lindelf degree of X by a well-formulated topological property of Cp .X /.
Published in Arhangelskii (1998b)
Motivated by the fact that hereditary Lindelf degree is t -invariant (Problem 070)
4.6.6 Characterize spread of X by a topological property of Cp .X /.
Published in Arhangelskii (1998b)
Motivated by the fact that spread is t -invariant (Problem 068)
4.6.7 Characterize hereditary density of a space X by a topological property of
Cp .X /.
Published in Arhangelskii (1998b)
Motivated by the fact that hereditary density is t -invariant (Problem 069)
4.6.8 Characterize the spaces X such that Cp .X / embeds into some Lindelf
space Cp .Y /.
Published in Arhangelskii (1998b)
Comment R!1 is not embeddable in a Lindelf space Cp .Y / (Problem 023)

662

4 Open problems

4.6.9 For any n 2 N, characterize compact metrizable spaces l-equivalent to the


cube In .
Published in Arhangelskii (1992b)
Related to Problem 299
4.6.10 Characterize analyticity of X by a topological property of Cp .X /.
Published in Arhangelskii and Calbrix (1999)
Motivated by the fact that analyticity is t -invariant (Problem 044)
4.6.11 In which classes of spaces is the extent l-invariant?
Published in Baars and Gladdines (1996)

4.7 Raznoie (unclassified questions)

663

4.7 Raznoie (unclassified questions)


It is usually impossible to completely classify a complex data set such as the open
problems in Cp -theory. This last group of problems contains the open questions
which do not fit into any previous Section.
4.7.1 Let X be a perfectly normal compact space. Is it true in ZFC that Cp .X / is
(hereditarily) metalindelf?
Published in Arhangelskii (1998b)
Comment This is true under CH (Arhangelskii (1998b))
Related to Problems 469 and 470
4.7.2 Is it true that Cp .X / is submetalindelf for any compact space X ?
Published in Arhangelskii (1997)
Motivated by the fact that there exist compact spaces X such that Cp .X /
is not metalindelf (Problem 470)
4.7.3 Let X be a scattered compact space. Must Cp .X / have a point-countable
-base?
Published in Tkachuk (2005d)
Motivated by the fact that this is true if X is Corson compact (Problem 412).
4.7.4 Is Cp .!/ homeomorphic to Cp .!/  R! ?
Author V.V. Tkachuk
Related to Problem 025
4.7.5 Suppose that Cp .X / has a uniformly dense monotonically !-monolithic
subspace. Must Cp .X / be monotonically !-monolithic?
Author V.V. Tkachuk
4.7.6 Suppose that X is a monotonically monolithic compact space. Must X have
a dense metrizable subspace?
Published in Tkachuk (2013b)
Motivated by the fact that this is true for Gulko compact spaces (Problem
TFS-293)
4.7.7 Suppose that X is a monotonically monolithic compact space such that
c.X /  !. Must X be metrizable?
Published in Tkachuk (2013b)
Motivated by the fact that this is true for Gulko compact spaces (Problem
CFS-294)

664

4 Open problems

4.7.8 Is it true that, for any compact X , the space Cp .X / can be embedded in a
Lindelf Cp .Y /?
Published in Arhangelskii (1998b)
Motivated by the fact that if X is a discrete uncountable space then Cp .X /
cannot be embedded in a Lindelf Cp .Y / (Problem 023)
4.7.9 Suppose that X is a compact space such that Cp .X / can be embedded in a
Lindelf Cp .Y /. Is it true that t .X / D !?
Published in Arhangelskii (1998b)
Motivated by the fact that if X is a discrete uncountable space then Cp .X /
cannot be embedded in a Lindelf Cp .Y / (Problem 023)
4.7.10 Suppose that D has countable pseudocharacter for any discrete subspace
D  Cp .X /. Must the space X be separable, or, equivalently, is it true that
.Cp .X // D !?
Published in Tkachuk (2015a)
4.7.11 Suppose that D is Lindelf for any discrete set D  Cp .X /  Cp .X /. Must
Cp .X / be Lindelf?
Published in Tkachuk (2015a)
4.7.12 Suppose that the subspace D has the FrchetUrysohn property for any
discrete set D  Cp .X /. Must the space Cp .X / be FrchetUrysohn?
Published in Tkachuk (2015a)
4.7.13 Assume that the subspace D is realcompact for any discrete set D  Cp .X /.
Must the space Cp .X / be realcompact?
Published in Tkachuk (2015a)

Bibliography

My original intention was to reference all research publications on Cp -theory.


However, once this work was started, the impossibility of this task became evident.
Not only are there hundreds of journals and institutions where the results are being
published, but it is too hard to even imagine how many papers on Cp -theory are in
the process of publication at the present time. This book has been in preparation
for more than ten years; so I reluctantly decided to be content with a reasonably
complete bibliography. The selection below has 800 items; it covers the material of
this book and can claim to reflect the state of the art of modern Cp -theory.
AFANAS, D.N., CHOBAN, M.M.
[2000] Topological properties of function spaces, Bul. Acad. Stiinte Repub.
Mold. Mat. 3(2000), 2852.
AHARONI, I., LINDENSTRAUSS, J.
[1978] Uniform equivalence between Banach spaces, Bull. Amer. Math. Soc.,
84(1978), 281283.
ALAS, O.T.
[1978] Normal and function spaces, Topology, Vol. II, Colloq. Math. Soc.
Jnos Bolyai, 23, North-Holland, Amsterdam, 1980, 2933.
ALAS, O.T., GARCIA-FERREIRA, S., TOMITA, A.H.
[1999] The extraresolvability of some function spaces, Glas. Mat. Ser. III
34(54):1(1999), 2335.
ALAS, O.T., TAMARIZ-MASCARA, A.

[2006] On the Cech


number of Cp .X / II, Questions Answers Gen. Topology
24:1(2006), 3149.

Springer International Publishing Switzerland 2016


V.V. Tkachuk, A Cp-Theory Problem Book, Problem Books in Mathematics,
DOI 10.1007/978-3-319-24385-6

665

666

Bibliography

ALAS, O.T., TKACHUK, V.V., WILSON R.G.,


[2009] A broader context for monotonically monolithic spaces, Acta Math.
Hungarica, 125:4(2009), 369385.
ALSTER, K.
[1979] Some remarks on Eberlein compacta, Fund. Math., 104:1(1979),
4346.
ALSTER, K., POL, R.
[1980] On function spaces of compact subspaces of -products of the real
line, Fund. Math., 107:2(1980), 135143.
AMIR, D., LINDENSTRAUSS, J.
[1968] The structure of weakly compact sets in Banach spaces, Annals Math.,
88:1(1968), 3546.
ANDERSON, B.D.
[1973] Projections and extension maps in C.T /, Illinois J. Math., 17(1973),
513517.
ANGOA, J., TAMARIZ-MASCARA, A.
[2006] Spaces of continuous functions, -products and box topology,
Comment. Math. Univ. Carolin. 47:1(2006), 6994.
ARENS, R.
[1946] A topology of spaces of transformations, Annals Math., 47(1946),
480495.
[1952] Extensions of functions on fully normal spaces, Pacific J. Math.,
2(1952), 1122.
ARENS, R., DUGUNDJI, J.
[1951] Topologies for function spaces, Pacific J. Math., 1:1(1951), 531.
ARGYROS, S., FARMAKI, V.
[1985] On the structure of weakly compact subsets of Hilbert spaces and
applications to the geometry of Banach spaces, Trans. Amer. Math.
Soc., 289(1985), 409427.
ARGYROS, S., ARVANITAKIS, A.D., MERCOURAKIS, S.
[2008] Talagrands K problem Topology Appl. 155:15(2008), 17371755.
ARGYROS, S.A., DODOS, P., KANELLOPOULOS, V.
[2008] A classification of separable Rosenthal compacta and its applications,
Dissertationes Math. (Rozprawy Mat.) 449(2008).
ARGYROS, S.A., MANOUSSAKIS, A., PETRAKIS, M.
[2003] Function spaces not containing l1 , Israel J. Math. 135(2003), 2981.

Bibliography

667

ARGYROS, S., MERCOURAKIS, S.


[1993] On weakly Lindelf Banach spaces, Rocky Mountain J. Math.,
23(1993), 395446.
ARGYROS, S., MERCOURAKIS, S., NEGREPONTIS, S.
[1983] Analytic properties of Corson compact spaces, General Topology and
Its Relations to Modern Analysis and Algebra, 5. Berlin, 1983, 1224.
ARGYROS, S., NEGREPONTIS, S.
[1983] On weakly K-countably determined spaces of continuous functions,
Proc. Amer. Math. Soc., 87:4(1983), 731736.
ARHANGELSKII, A.V.
[1959] An addition theorem for the weight of sets lying in bicompacta
(in Russian), DAN SSSR, 126(1959), 239241.
[1976] On some topological spaces occurring in functional analysis (in Russian), Uspehi Mat. Nauk, 31:5(1976), 1732.
[1978a] On spaces of continuous functions with the topology of pointwise
convergence (in Russian), Doklady AN SSSR, 240:3(1978), 506508.
[1978b] The structure and classification of topological spaces and cardinal
invariants (in Russian), Uspehi Mat. Nauk, 33:6(1978), 2984.
[1980] On the relationship between the invariants of topological groups and
their subspaces (in Russian), Uspehi Mat. Nauk, 35:3(1980), 322.
[1981] Classes of topological groups (in Russian), Uspehi Mat. Nauk,
36:3(1981), 127146.
[1982a] On linear homeomorphisms of function spaces (in Russian), Doklady
AN SSSR, 264:6 (1982), 12891292.
[1982b] A theorem on -approximation and functional duality (in Russian),
Matem. Zametki, 31:3(1982), 421432.
[1982c] Factorization theorems and function spaces: stability and monolithity
(in Russian), Doklady AN SSSR, 265:5(1982), 10391043.
[1982d] On relationship between topological properties of X and Cp .X /,
General Topology and Its Relations to Modern Analysis and Algebra,
5. Berlin, 1982, 2436.
[1983a] Function spaces and completeness-like conditions (in Russian), Vestnik Mosk. Univ., Math., Mech., 38:6(1983), 49.
[1983b] Topological properties of function spaces: duality theorems
(in Russian), Doklady AN SSSR, 269:6(1983), 12891292.
[1983c] Functional tightness, Q-spaces and -embeddings, Comment. Math.
Univ. Carolinae, 24:1(1983), 105120.
[1984a] Function spaces with the topology of pointwise convergence and
compact spaces (in Russian), Uspehi Mat. Nauk, 39:5(1984), 1150.
[1984b] Continuous mappings, factorization theorems and function spaces
(in Russian), Trudy Mosk. Mat. Obsch., 47(1984), 321.

668

Bibliography

[1985] Function spaces with the topology of pointwise convergence


(in Russian), General Topology: Function Spaces and Dimension
(in Russian), Moscow University P.H., 1985, 366.
[1986] Hurewicz spaces, analytic sets and fan tightness of function spaces (in
Russian), Doklady AN SSSR, 287:3(1986), 525528.
[1987] A survey of Cp -theory, Questions and Answers in General Topology,
5(1987), 1109.
[1988a] Some results and problems in Cp .X /-theory, General Topology and Its
Relations to Modern Analysis and Algebra, VI, Heldermann Verlag,
Berlin, 1988, 1131.
[1988b] Some problems and lines of investigation in general topology, Comment. Math. Univ. Carolinae, 29:4(1988), 611629.
[1989a] Topological Function Spaces (in Russian), Moscow University P.H.,
Moscow, 1989.
[1989b] Hereditarily Lindelf spaces of continuous functions, Moscow University Math. Bull., 44:3(1989), 6769.
[1989c] On iterated function spaces, Bull. Acad. Sci. Georgian SSR,
134:3(1989), 481483.
[1989d] On linear topological and topological classification of spaces Cp .X /,
Zbornik radova Filoz. fakult. u Niu, Mat., 3(1989), 312.
[1990a] Problems in Cp -theory, in: Open Problems in Topology, North
Holland, Amsterdam, 1990, 603615.
[1990b] On the Lindelf degree of topological spaces of functions, and on
embeddings into Cp .X /, Moscow University Math. Bull., 45:5(1990),
4345.
[1991] On linear topological classification of spaces of continuous functions
in the topology of pointwise convergence, Math., USSRSbornik,
70:1(1991), 129142.
[1992a] Topological Function Spaces (translated from Russian), Kluwer
Academic Publishers, Dordrecht, 1992.
[1992b] Cp -theory, in: Recent Progress in General Topology, North Holland,
Amsterdam, 1992, 156.
[1995a] Spaces of mappings and rings of continuous functions, in: General
Topology, 3, Encyclopedia Math. Sci., Springer, Berlin, 51(1995),
73156.
[1995b] A generic theorem in the theory of cardinal invariants of topological
spaces, Comment. Math. Univ. Carolinae, 36:2(1995), 303325.
[1995c] General Topology III, Encycl. Math. Sciences, 51, Springer-Verlag,
Berlin Heidelberg, New York, 1995.
[1996a] On Lindelf property and spread in Cp -theory, Topol. and Its Appl.,
74:(1-3)(1996), 8390.
[1996b] On spread and condensations, Proc. Amer. Math. Soc., 124:11(1996),
35193527.
[1997] On a theorem of Grothendieck in Cp -theory, Topology Appl. 80(1997),
2141.

Bibliography

669

[1998a] Some observations on Cp -theory and bibliography, Topology Appl.,


89(1998), 203221.
[1998b] Embeddings in Cp -spaces, Topology Appl., 85(1998), 933.
[2000a] On condensations of Cp -spaces onto compacta, Proc. Amer. Math.
Soc., 128:6(2000), 18811883.
[2000b] Projective  -compactness, !1 -caliber and Cp -spaces, Topology
Appl., 104(2000), 1326.
[2002a] Relative normality and dense subspaces, Topology Appl., 123(2002),
2736.
[2002b] Topological invariants in algebraic environment, Recent Progress in
Gen. Top. II, North-Holland, Amsterdam, (2002), 157.
[2002c] Erratum to: On condensations of Cp -spaces onto compacta [Proc.
Amer. Amer. Math. Soc. 128:6(2000), 18811883], Proc. Amer. Math.
Soc. 130:6(2002), 1875
[2005] D-spaces and covering properties, Topology Appl., 146147(2005),
437449.
ARHANGELSKII, A.V., BUZYAKOVA, R.Z.
[1999] On linearly Lindelf and strongly discretely Lindelf spaces, Proc.
Amer. Math. Soc., 127:8(1999), 24492458.
[2002] Addition theorems and D-spaces, Comment. Math. Univ. Carolinae,
43:4(2002), 653663.
ARHANGELSKII, A.V., CALBRIX, J.
[1999] A characterization of  -compactness of a cosmic space X by means of
subspaces of RX , Proc. Amer. Math. Soc., 127:8(1999), 24972504.
ARHANGELSKII, A.V., CHOBAN M.M.
[1988] The extension property of Tychonoff spaces and generalized retracts,
Comptes Rendus Acad. Bulg. Sci., 41:12(1988), 57.
[1989] Cp .X / and some other functors in general topology. Continuous
extenders, in: Categorical Topology, World Scientific, London, 1989,
432445.
[1990] On the position of a subspace in the whole space, Comptes Rendus
Acad. Bulg. Sci., 43:4(1990), 1315.
[1992] Extenders of Kuratowskivan Douwen and classes of spaces, Comptes
Rendus Acad. Bulg. Sci., 45:1(1992), 57.
[1996] On continuous mappings of Cp -spaces and extenders, Proc. Steklov
Institute Math., 212(1996), 2831.
ARHANGELSKII, A.V., OKUNEV, O.G.
[1985] Characterization of properties of spaces by properties of their continuous images (in Russian) Vestnik Mosk. Univ., Math., Mech.,
40:5(1985), 2830.

670

Bibliography

ARHANGELSKII, A.V., PAVLOV, O.I.


[2002] A note on condensations of Cp .X / onto compacta, Comment. Math.
Univ. Carolinae, 43:3(2002), 485492.
ARHANGELSKII, A.V., PONOMAREV, V.I.
[1974] Basics of General Topology in Problems and Exercises (in Russian),
Nauka, Moscow, 1974.
ARHANGELSKII, A.V., SHAKHMATOV, D.B.
[1988] On pointwise approximation of arbitrary functions by countable families of continuous functions (in Russian), Trudy Sem. Petrovsky,
13(1988), 206227.
ARHANGELSKII, A.V., SZEPTYCKI, P.J.
[1997] Tightness in compact subspaces of Cp -spaces, Houston J. Math.,
23:1(1997), 17.
ARHANGELSKII, A.V., TKACHUK, V.V.
[1985] Function Spaces and Topological Invariants (in Russian), Moscow
University P.H., Moscow, 1985.
[1986] Calibers and point-finite cellularity of the spaces Cp .X / and some
questions of S. Gulko and M. Huek, Topology Appl., 23:1(1986),
6574.
ARHANGELSKII, A.V., USPENSKIJ, V.V.
[1986] On the cardinality of Lindelf subspaces of function spaces, Comment.
Math. Univ. Carolinae, 27:4(1986), 673676.
ASANOV, M.O.
[1979] On cardinal invariants of spaces of continuous functions (in Russian),
Modern Topology and Set Theory (in Russian), Izhevsk, 1979, N 2,
812.
[1980] On spaces of continuous maps, Izvestiia Vuzov, Mat., 1980, N 4, 610.
[1983] About the space of continuous functions, Colloq. Math. Soc. Janos
Bolyai, 41(1983), 3134.
ASANOV, M.O., SHAMGUNOV, N.K.
[1981] The topological proof of the NachbinShirotas theorem, Comment.
Math. Univ. Carolinae, 24:4(1983), 693699.
ASANOV, M.O., VELICHKO, M.V.
[1981] Compact sets in Cp .X / (in Russian), Comment. Math. Univ. Carolinae, 22:2(1981), 255266.
BAARS, J.
[1992] Equivalence of certain free topological groups, Commentationes
Math. Univ. Carolin. 33:1(1992), 125130.
[1993a] Topological equivalence of function spaces, Ann. N.Y. Acad. Sci.,
704(1993), 351352.

Bibliography

671

[1993b] A note on linear mappings between function spaces, Comment. Math.


Univ. Carolinae, 34:4(1993), 711715.
[1993c] On the lp -equivalence of certain locally compact spaces, Topology
Appl. 52:1(1993), 4357.
[1994] Function spaces of first countable paracompact spaces, Bull. Polon.
Acad. Sci., Math., 42(1994), 2935.
BAARS, J., DE GROOT, J.
[1988] An isomorphical classification of function spaces of zero-dimensional
locally compact separable metric spaces, Comment. Math. Univ.
Carolinae, 29(1988), 577595.
[1991] On the l-equivalence of metric spaces, Fund. Math., 137(1991), 2543.
[1992] On Topological and Linear Equivalence of Certain Function Spaces,
Center for Mathematics and Computer Sciences, Amsterdam, CWI
Tract 86(1992).
BAARS, J., DE GROOT, J., MILL, J.
[1989] A theorem on function spaces, Proc. Amer. Math. Soc., 105:4(1989),
10201024.
BAARS, J., DE GROOT, J., MILL, J. VAN, PELANT, J.
[1989] On topological and linear homeomorphisms of certain function
spaces, Topology Appl., 32(1989), 267277.
[1993] An example of lp -equivalent spaces which are not lp -equivalent, Proc.
Amer. Math. Soc., 119(1993), 963969.
BAARS, J., DE GROOT, J., PELANT, J.
[1993] Function spaces of completely metrizable spaces, Trans. Amer.
Mathem. Soc., 340:2(1993), 871879.
BAARS, J., GLADDINES, H.
[1996] On linear invariance of Lindelf numbers, Canadian Math. Bull.,
39:2(1996), 129137.
BAARS, J., GLADDINES, H., MILL, J. VAN
[1993] Absorbing systems in infinite-dimesional manifolds, Topology Appl.,
50:2(1993), 147182.
BANACH, T., CAUTY, R.
[1997] Universalit forte pour les sous-ensembles totalement borns. Applications aux espaces Cp .X /, Colloq. Math., 73(1997), 2533.
BANDLOW, I.
[1991] A characterization of Corson compact spaces, Commentationes Math.
Univ. Carolinae, 32:3(1991), 545550.
[1994] On function spaces of Corsoncompact spaces, Comment. Math. Univ.
Carolinae, 35:2(1994), 347356.

672

Bibliography

BARTLE, R.G.
[1955] On compactness in functional analysis, Trans. Amer. Math. Soc.,
79(1955), 3557.
BARTLE, R.G., GRAVES, L.M.
[1952] Mappings between function spaces, Trans. Amer. Math. Soc.,
72(1952), 400413.
BASHKIROV, A.I.
[1972] Normality and bicompactness of spaces of mappings (in Russian),
Vestnik Mosk. Univ., Math., Mech., 1972, N 3, 7779.
BATUROV D.P.
[1987] On subspaces of function spaces (in Russian), Vestnik Moskovsk.
Univ., Math., Mech., 42:4(1987), 6669.
[1988] Normality of dense subsets of function spaces, Vestnik Moskovsk.
Univ., Math., Mech., 43:4(1988), 6365.
[1990] Normality in dense subspaces of products, Topology Appl., 36(1990),
111116.
[1990] Some properties of the weak topology of Banach spaces, Vestnik Mosk.
Univ., Math., Mech., 45:6(1990), 6870.
BELL, M., MARCISZEWSKI, W.
[2004] Function spaces on t -Corson compacta and tightness of polyadic
spaces, Czech. Math. J., 54(129)(2004), 899914.
[2007] On scattered Eberlein compact spaces, Israel J. Math. 158(2007),
217224.
BELLA, A., YASCHENKO, I.V.
[1999] On AP and WAP spaces, Comment. Math. Univ. Carolinae,
40:3(1999), 531536.
BENYAMINI, Y., RUDIN, M.E., WAGE, M.
[1977] Continuous images of weakly compact subsets of Banach spaces,
Pacific J. Mathematics, 70:2(1977), 309324.
BENYAMINI, Y., STARBIRD, T.
[1976] Embedding weakly compact sets into Hilbert space, Israel J. Math.,
23(1976), 137141.
BESSAGA, C., PELCZINSKI, A.
[1960] Spaces of continuous functions, 4, Studia Math., 19(1960), 5362.
[1975] Selected Topics in Infinite-Dimensional Topology, PWN, Warszawa,
1975.
BLASKO, J.L.
[1977] On -spaces and kR -spaces, Proc. Amer. Math. Soc., 67:1(1977),
179186.
[1990] The G -topology and K-analytic spaces without perfect compact sets,
Colloq. Math., 58(1990), 189199.

Bibliography

673

BORGES, C.J.
[1966a] On stratifiable spaces, Pacific J. Math., 17:1(1966), 116.
[1966b] On function spaces of stratifiable spaces and compact spaces, Proc.
Amer. Math. Soc., 17(1966), 10741078.
BOURGAIN, J.
[1977] Compact sets of the first Baire class, Bull. Soc. Math. Belg.,
29:2(1977), 135143.
[1978] Some remarks on compact sets of first Baire class, Bull. Soc. Math.
Belg., 30(1978), 310.
BOURGAIN, J., FREMLIN, D.H., TALAGRAND, M.
[1978] Pointwise compact sets of Baire-measurable functions, Amer. J. Math.,
100(1978), 845886.
BOURGAIN, J., TALAGRAND, M.
[1980] Compacit extremal, Proc. Amer. Math. Soc., 80(1980), 6870.
BOUZIAD, A.
[1990a] Jeux topologiques et points de continuit dune application separement
continue, Comptes Rendus de la Acadmie des Sciences, Paris, Ser. I,
310(1990), 359362.
[1990b] Une classe despaces co-Namioka, Comptes Rendus de la Acadmie
des Sciences, Paris, Ser. I, 310(1990), 779782.
[1994] Notes sur la proprit de Namioka, Trans. Amer. Math. Soc.,
344:2(1994), 873883.
[1996] The class of co-Namioka spaces is stable under product, Proc. Amer.
Math. Soc., 123(1996), 983986.
[2001] Le degr de Lindelf est l-invariant, Proc. Amer. Math. Soc.,
129:3(2001), 913919.
BOUZIAD, A., CALBRIX, J.

[1995] Images usco-compactes des espaces Cech-complets


de Lindelf, C. R.
Acad. Sci. Paris Sr. I Math. 320:7(1995), 839842.
BROWN, R.
[1964] Function spaces and product topologies, Oxford J. Math., 15:2(1964),
238250.
BUCHWALTER, H., SCHMETS, J.
[1973] Sur quelques proprits de lespace Cs .T /, J. Math. Pures Appl.,
52:3(1973), 337352.
BURKE, D.K.
[1984] Covering properties, Handbook of Set-Theoretic Topology, ed. by
K. Kunen and J.E. Vaughan, Elsevier Science Publishers B.V., 1984,
347422.
[2007] Weak-bases and D-spaces, Comment. Math. Univ. Carolin.
48:2(2007), 281289.

674

Bibliography

BURKE, D.K., LUTZER, D.J.


[1976] Recent advances in the theory of generalized metric spaces, in:
Topology: Proc. Memphis State University Conference, Lecture Notes
in Pure and Applied Math., Marcel Dekker, New York, 1976, 170.
BURKE, D.K., POL, R.
[2003] On Borel sets in function spaces with the weak topology, Journal
London Math. Soc. (2)68:3(2003), 725738.
[2005] Note on separate continuity and the Namioka property, Topology
Appl. 152:3(2005), 258268.
BURKE, M.R., TODORCEVIC, S.
[1996] Bounded sets in topological vector spaces, Math. Ann. 305:1(1996),
103125.
BUROV, YU.A.
[1984] On mutual decompositions of weak topological bases of a topological
vector space (in Russian), Uspehi Mat. Nauk, 39:5(1984), 237238.
[1986a] On the properties of (weakly) l-equivalent spaces (in Russian). General Topology. Mappings of Topological Spaces (in Russian), Moscow
University P.H., 1986, 1319.
[1986b] On l- and M -equivalences, Topics in Geometry and Topology,
Petrozavodsk University P.H., Petrozavodsk, 1986, 313.
BUZYAKOVA, R.Z.
[1993] On splittable spaces, Vestnik Moskov. Univ. Mat., Mekh., 48:6(1993),
8384.
[1996] Splittability of compacta over linearly ordered spaces and over the
class of all linearly ordered spaces, Vestnik Moskov. Univ. Mat.,
Mekh., 51:3(1996), 8184.
[2002] On D-property of strong -spaces, Comment. Math. Univer. Carolinae, 43:3(2002), 493495.
[2004a] In search for Lindelf Cp s, Comment. Math. Univ. Carolinae,
45:1(2004), 145151.
[2004b] On cleavability of continua over LOTS, Proc. Amer. Math. Soc.,
132:7(2004), 21712181.
[2004c] Hereditary D-property of function spaces over compacta, Proc. Amer.
Math. Soc., 132:11(2004), 34333439.
[2005] Cleavability of compacta over the two arrows, Topology Appl., 150:
1-3(2005), 144156.
[2006a] Spaces of continuous step functions over LOTS, Fund. Math.,
192(2006), 2535.
[2006b] Spaces of continuous characteristic functions, Comment. Math. Universitatis Carolinae, 47:4(2006), 599608.
[2007] Function spaces over GO spaces, Topology Appl., 154:4(2007),
917924.

Bibliography

675

[2008] How sensitive is Cp .X; Y / to changes in X and/or Y ? Comment.


Math. Univ. Carolin., 49:4(2008), 657665.
[2010] Injections into function spaces over ordinals, Topology Appl.,
157(2010), 28442849.
[2012] More on injections into function spaces over ordinals, Topology Appl.,
159(2012), 15731577.
CALBRIX, J.
[1985a] Classes de Baire et espaces dapplications continues, Comptes Rendus
Acad. Sci. Paris, Ser I, 301:16(1985), 759762.
[1985b] Espaces K et espaces des applications continues, Bulletin Soc. Math.
France, 113(1985), 183203.
[1987] Filtres sur les entiers et ensembles analytiques, Comptes Rendus Acad.
Sci. Paris, Ser I, 305(1987), 109111.
[1988] Filtres Boreliens sur lensemble des entiers et espaces des applications
continues, Rev. Roumaine Math. Pures et Appl., 33(1988), 655661.
[1996] k-spaces and Borel filters on the set of integers (in French), Trans.
Amer. Math. Soc., 348(1996), 20852090.
CALBRIX, J., TROALLIC, J.
[1979] Applications separement continues, C. R. Acad. Sci. Paris, Ser I,
288(1979), 647648.
CASARRUBIAS-SEGURA, F.
[1999] Realcompactness and monolithity are finitely additive in Cp .X /,
Topology Proc., 24(1999), 89102.
[2001] On compact weaker topologies in function spaces, Topology Appl.,
115(2001), 291298.
CASARRUBIAS-SEGURA, F., OKUNEV, O., PANIAGUA RAMREZ, C.G.
[2008] Some results on L./-spaces, Comment. Math. Univer. Carolinae,
49:4(2008), 667675.
CASARRUBIAS-SEGURA, F., ROJAS-HERNNDEZ, R.
[2015] On some monotone properties, Topology Appl., 182(2015), 3652.
CASCALES, B.
[1987] On K-analytic locally convex spaces, Arch. Math., 49(1987),
232244.
CASCALES, B., KA KOL, J., SAXON, S.A.
[2002] Weight of precompact subsets and tightness, J. Math. Anal. Appl.,
269(2002), 500518.
CASCALES, B., MANJABACAS, G, VERA, G.
[1998] Fragmentability and compactness in C.K/-spaces, Studia Math.,
131:1(1998), 7387.

676

Bibliography

CASCALES, B., NAMIOKA, I.


[2003] The Lindelf property and  -fragmentability, Fund. Math., 180(2003),
161183.
CASCALES, B., NAMIOKA, I., ORIHUELA, J.
[2003] The Lindelf property in Banach spaces, Studia Math., 154:2(2003),
165192.
CASCALES, B., NAMIOKA, I., VERA, G.
[2000] The Lindelf property and fragmentability, Proceedings American
Math. Society., 128:11(2000), 33013309.
CASCALES, B., ONCINA, L.
[2003] Compactoid filters and USCO maps, J. Math. Anal., Appl., 282(2003),
826845.
CASCALES, B., ORIHUELA, J.
[1987] On compactness in locally convex spaces, Math. Z., 195(1987),
365381
[1988] On pointwise and weak compactness in spaces of continuous functions,
Bull. Soc. Math. Belg., 40:2(1988), 331352.
[1991a] A sequential property of set-valued maps, J. Math. Analysis Appl.,
156:1(1991), 86100.
[1991b] Countably determined locally convex spaces, Portugal. Math.
48:1(1991), 7589.
CASCALES, B., RAJA, M.
[2004] Bounded tightness for weak topologies, Arch. Math., 82(2004),
324334.
CASCALES, B., VERA, G.
[1994] Topologies weaker than the weak topology of a Banach space, J. Math.
Analysis Appl., 182:1(1994), 4168.
CAUTY, R.
[1974] Rtractions dans les espaces stratifiables, Bull. Soc. Math. France
102(1974), 129149.
[1991] Lespace de fonctions continues dun espace metrique denombrable,
Proc. Amer. Math. Soc., 113(1991), 493501.
[1998] La classe borlienne ne dtermine pas le type topologique de Cp .X /,
Serdica Math. J. 24:3-4(1998), 307318.
CAUTY, R., DOBROWOLSKI, T., MARCISZEWSKI, W.
[1993] A contribution to the topological classification of the spaces Cp .X /,
Fundam. Math., 142(1993), 269301.

Bibliography

677

CHOBAN, M.M.
[1998a] General theorems on functional equivalence of topological spaces,
Topol. Appl., 89(1998), 223239.
[1998b] Isomorphism problems for the Baire function spaces of topological
spaces, Serdica Math. J. 24:1(1998), 520.
[1998c] Isomorphism of functional spaces, Math. Balkanica (N.S.) 12:
1-2(1998), 5991.
[2001] Functional equivalence of topological spaces, Topology Appl.,
111(2001), 105134.
[2005] On some problems of descriptive set theory in topological spaces,
Russian Math. Surveys 60:4(2005), 699719.
CHRISTENSEN, J.P.R.
[1974] Topology and Borel Structure, North Holland P.C., Amsterdam, 1974.
[1981] Joint continuity of separably continuous functions, Proceedings of the
Amer. Math. Soc., 82:3(1981), 455461.
CHRISTENSEN, J.P.R., KENDEROV, P.S.
[1984] Dense strong continuity of mappings and the RadonNykodym property, Math. Scand., 54:1(1984), 7078.
CIESIELSKI, K.
[1993] Linear subspace of R without dense totally disconnected subsets,
Fund. Math. 142 (1993), 8588.
CIESIELSKI, K., POL, R.
[1984] A weakly Lindelf function space C.K/ without any continuous
injection into c0 . /, Bull. Acad. Polon. Sci., Ser. Math., Astron. y
Phys., 32:(11-12)(1984), 681688.
COMFORT, W.W.
[1968] On the Hewitt realcompactification of a product space, Transactions
Amer. Math. Soc. 131(1968), 107118.
[1971] A survey of cardinal invariants, General Topology and Appl.
1:2(1971), 163199.
[1977] Ultrafilters: some old and some new results, Bull. Amer. Math. Soc.
83:4(1977), 417455.
[1988] Cofinal families in certain function spaces, Commentationes Math.
Univ. Carolin. 29:4(1988), 665675.
COMFORT, W.W., FENG, L.
[1993] The union of resolvable spaces is resolvable, Math. Japon. 38:3(1993),
413414.
COMFORT, W.W., HAGER, A.W.
[1970a] Estimates for the number of real-valued continuous functions, Trans.
Amer. Math. Soc., 150(1970), 619631.

678

Bibliography

[1970b] Dense subspaces of some spaces of continuous functions, Math. Z.


114(1970), 373389.
[1970c] Estimates for the number of real-valued continuous functions, Trans.
Amer. Math. Soc. 150(1970), 619631.
COMFORT, W.W., NEGREPONTIS, S.A.
[1965] The ring C.X / determines the category of X , Proceedings Amer.
Math. Soc. 16(1965), 10411045.
[1966] Extending continuous functions on X Y to subsets of X Y , Fund.
Math. 59(1966), 112.
[1974] The theory of ultrafilters. Die Grundl. Math. Wiss., 211, Springer, New
York, 1974.
[1982] Chain Conditions in Topology, Cambridge Tracts in Mathematics, 79,
New York, 1982.
CONTRERAS-CARRETO, A., TAMARIZ-MASCARA, A.
[2003] On some generalizations of compactness in spaces Cp .X; 2/ and
Cp .X; Z/, Bol. Soc. Mat. Mexicana (3)9:2(2003), 291308.
CORSON, H.H.
[1959] Normality in subsets of product spaces, American J. Math.,
81:3(1959), 785796.
[1961] The weak topology of a Banach space, Trans. Amer. Math. Soc.,
101:1(1961), 115.
CORSON, H.H., LINDENSTRAUSS, J.
[1966a] On function spaces which are Lindelf spaces, Trans. Amer. Math.
Soc., 121:2(1966), 476491.
[1966b] On weakly compact subsets of Banach spaces, Proc. Amer. Math. Soc.,
17:2(1966), 407412.
CTH, M.
[2014] Characterization of compact monotonically .!/-monolithic spaces
using system of retractions, Topology Appl., 171(2014), 8790.
DEBS, G.
[1985] Espaces K-analytiques et espaces de Baire de fonctions continues,
Mathematika, 32(1985), 218228.
[1986] Pointwise and uniform convergence on a Corson compact space,
Topol. Appl., 23:3(1986), 299303.
DEVILLE, R.
[1989] Convergence ponctuelle et uniforme sur un espace compact, Bull.
Acad. Polon. Sci., Math., 37(1989), 712.

Bibliography

679

DEVILLE, R., GODEFROY, G.


[1993] Some applications of projectional resolutions of identity, Proc. London
Math. Soc., Ser. (3), 67:1(1993), 183199.
DIESTEL, J.
[1975] Geometry of Banach spaces selected topics, Lecture Notes Math.,
11:485(1975).
DIJKSTRA, J.
[2005] On homeomorphism groups and the compact-open topology, Amer.
Math. Monthly 112:10(2005), 910912.
DIJKSTRA, J., DOBROWOLSKI, T., MARCISZEWSKI, W., MILL, J. VAN,
MOGILSKI, J.
[1990] Recent classification and characterization results in geometric topology, Bull. Amer. Math. Soc., 22:2(1990), 277283.
DIJKSTRA, J., GRILLOT, T., LUTZER, D., VAN MILL, J.
[1985] Function spaces of low Borel complexity, Proc. Amer. Math. Soc.,
94:4(1985), 703710.
DIJKSTRA, J., MOGILSKI, J.
[1996] Cp .X /-representation of certain Borel absorbers, Topology Proc.,
16(1991), 2939.
[1996] The ambient homeomorphy of certain function spaces and sequence
spaces, Comment. Math. Univ. Carolinae, 37:3(1996), 595611.
DI MAIO, G., HOL, L., HOL, D., MCCOY, R.A.
[1998] Topologies on the space of continuous functions, Topology Appl.
86:2(1998), 105122.
DIMOV, G.
[1987] Espaces dEberlein et espaces de type voisins, Comptes Rendus Acad.
Sci., Paris, Ser. I, 304:9(1987), 233235.
[1988] Baire subspaces of c0 . / have dense G metrizable subsets, Rend.
Circ. Mat. Palermo (2) Suppl. 18(1988), 275285.
[2014] An internal topological characterization of the subspaces of Eberlein
compacta and related compacta-I, Topology Appl., 169(2014), 7186.
DIMOV, G., TIRONI, G.
[1987] Some remarks on almost radiality in function spaces, Acta Univ.
Carolin. Math. Phys. 28:2(1987), 4958.
DOBROWOLSKI, T., GULKO, S.P., MOGILSKI, J.
[1990] Function spaces homeomorphic to the countable product of `2f , Topology Appl., 34(1990), 153160.

680

Bibliography

DOBROWOLSKI, T., MARCISZEWSKI, W.


[1995] Classification of function spaces with the pointwise topology determined by a countable dense set, Fund. Math., 148(1995), 3562.
[2002] Failure of the factor theorem for Borel pre-Hilbert spaces, Fund.
Math., 175(2002), 5368.
[2004] Infinite-dimensional topology, in: Encycl. General Topology, Elsevier
Sci. Publ., Amsterdam, 2004, 497502.
DOBROWOLSKI, T., MARCISZEWSKI, W., MOGILSKI, J.
[1991] Topological classification of function spaces Cp .X / of low Borel
complexity, Trans. Amer. Math. Soc., 328(1991), 307324.
DOBROWOLSKI, T., MOGILSKI, J.
[1992] Certain sequence and function spaces homeomorphic to the countable
product of `2f , J. London Math. Soc., 45:2(1992), 566576.
DOUWEN, E.K. VAN
[1975a] Simultaneous extension of continuous functions, PhD Dissertation,
99(1975), Amsterdam, Free University.
[1975b] Simultaneous linear extensions of continuous functions, General
Topology Appl., 5(1975), 297319.
[1984] The integers and topology, Handbook of SetTheoretic Topology,
K. Kunen and J.E. Vaughan, editors, Elsevier Science Publishers B.V.,
1984, 111167.
DOUWEN, E.K. VAN, LUTZER, D.J., PRZYMUSINSKI, T.C.
[1977] Some extensions of the TietzeUrysohn theorem, American Math.
Monthly, 84(1977), 435441
DOUWEN, E.K. POL, R.
[1977] Countable spaces without extension properties, Bull. Polon. Acad.
Sci., Math., 25(1977), 987991.
DOW, A.
[2005a] Closures of discrete sets in compact spaces, Studia Sci. Math. Hungar.
42:2(2005), 227234.
[2005b] Property D and pseudonormality in first countable spaces, Comment.
Math. Univ. Carolin. 46:2(2005), 369372.
DOW, A., JUNNILA H., PELANT, J.
[1997] Weak covering properties of weak topologies, Proceedings of the
London Math. Soc., (3)75:2(1997), 349368.
[2006] Coverings, networks and weak topologies, Mathematika 53:2(2006),
287320.

Bibliography

681

DOW, A., PAVLOV, O.


[2006] More about spaces with a small diagonal, Fund. Math. 191:1(2006),
6780.
[2007] Perfect preimages and small diagonal, Topology Proc. 31:1(2007),
8995.
DOW, A., SIMON, P.
[2006] Spaces of continuous functions over a  -space, Topology Appl.
153:13(2006), 22602271.
DRANISHNIKOV, A.N.
[1986] Absolute F -valued retracts and function spaces with the topology of
pointwise convergence (in Russian), Sibirsk. Mat. Zhurnal, 27(1986),
7486.
DUGUNDJI, J.
[1951] An extension of Tietzes theorem, Pacific J. Math., 1(1951), 353367.
EBERLEIN, W.F.
[1947] Weak compactness in Banach spaces, I, Proc. Nat. Acad. Sci. (USA),
33(1947), 5153.
EFIMOV, B.A.
[1977] Mappings and imbeddings of dyadic spaces, I, Math. USSR Sbornik,
32:1(1977), 4557.
ELEKES, M., KUNEN, K.
[2002] Transfinite sequences of continuous and Baire class 1 functions, Proc.
Amer. Math. Soc., 131:8(2002), 24532457.
ENGELKING, R.
[1977] General Topology, PWN, Warszawa, 1977.
[1978] Dimension Theory, PWN, Warszawa, 1978.
ESENINVOLPIN, A.S.
[1949] On the existence of a universal bicompactum of arbitrary weight (in
Russian), Dokl. Acad. Nauk SSSR, 68(1949), 649652.
FENG, Z., GARTSIDE, P.
[2007] More stratifiable function spaces, Topology Appl. 154:12(2007),
24572461.
[2013] Function spaces and local properties, Fund. Math., 223(2013),
207223.
FERRANDO, J.C.
[2009] Some characterizations for X to be Lindelof or K-analytic in
terms of Cp .X /, Topology Appl., 156(2009), 823830.

682

Bibliography

FERRANDO, J.C., KA KOL, J.


[2008] A note on spaces Cp .X / K-analytic-framed in RX , Bull. Aust. Math.
Soc., 78(2008), 141146.
FLORET, K.
[1980] Weakly Compact Sets, Lecture Notes in Math., 801(1980), Springer,
Berlin.
FORT, M.K.
[1951] A note on pointwise convergence, Proc. Amer. Math. Soc., 2(1951),
3435.
FOX, R.H.
[1945] On topologies for function spaces, Bull. Amer. Math. Soc., 51(1945),
429432.
FREMLIN, D.H.
[1977] K-analytic spaces with metrizable compacta, Mathematika, 24(1977),
257261.
[1994] Sequential convergence in Cp .X /, Comment. Math. Univ. Carolin.,
35:2(1994), 371382.
FULLER, R.V.
[1972] Condition for a function space to be locally compact, Proc. Amer.
Math. Soc., 36(1972), 615617.
GALE, D.
[1950] Compact sets of functions and function rings, Proc. Amer. Math. Soc.,
1(1950), 303308.
GARCA, F., ONCINA, L., ORIHUELA, J.
[2004] Network characterization of Gulko compact spaces and their relatives, J. Math. Anal. Appl. 297:2(2004), 791811.
GARCAFERREIRA, S., TAMARIZMASCARA, A.
[1994] p-FrchetUrysohn property of function spaces, Topology Appl.,
58:2(1994), 157172.
[1994] p-sequential like properties in function spaces, Comment. Math.
Universitatis Carolinae, 35:4(1994), 753771.
GARTSIDE, P.
[1997] Cardinal invariants of monotonically normal spaces, Topology Appl.
77:3(1997), 303314.
[1998] Nonstratifiability of topological vector spaces, Topol. Appl.
86:2(1998), 133140.

Bibliography

683

GARTSIDE, P., GLYN, A.


[2005] Closure preserving properties of Ck (metric fan), Topology Appl.
151:1-3(2005), 120131.
GARTSIDE, P., LO, J.T.H., MARSH, A.
[2003] Sequential density, Topology Appl., 130(2003), 7586.
GARTSIDE, P.M., REZNICHENKO, E.A.
[2000] Near metric properties of function spaces, Fund. Math. 164:2(2000),
97114.
GERLITS, J.
[1974] On the depth of topological spaces, Proc. Amer. Math. Soc. 44(1974),
507508.
[1980] Continuous functions on products of topological spaces, Fund. Math.
106:1(1980), 6775.
[1983] Some properties of C.X /, II, Topology Appl., 15:3(1983), 255262.
GERLITS, J., NAGY, ZS.
[1982] Some properties of C.X /, I, Topology Appl., 14:2(1982), 151161.
GERLITS, J., JUHSZ, I., SZENTMIKLSSY, Z.
[2005] Two improvements on Tkacenkos addition theorem, Comment. Math.
Univ. Carolin. 46:4(2005), 705710.
GERLITS, J., NAGY, ZS., SZENTMIKLOSSY, Z.
[1988] Some convergence properties in function spaces, in: General Topology
and Its Relation to Modern Analysis and Algebra, Heldermann, Berlin,
1988, 211222.
GILLESPIE, D.C, HURWITZ, W.A.
[1930] On sequences of continuous functions having continuous limits, Trans.
Amer. Math. Soc., 32(1930), 527543.
GILLMAN, L., JERISON, M.
[1960] Rings of Continuous Functions, D. van Nostrand Company Inc.,
Princeton, 1960.
GODEFROY, D.
[1980] Compacts de Rosenthal, Pacific J. Math., 91:2(1980), 293306.
GODEFROY, D., TALAGRAND, M.
[1982] Espaces de Banach representables, Israel J. Math., 41(1982), 321330.
GORK, R.
[1977] Function spaces on ordinals, Comment. Math. Univ. Carolinae,
46:1(2005), 93103.

684

Bibliography

GORDIENKO, I.YU.
[1990] Two theorems on relative cardinal invariants in Cp -theory, Zb. Rad.
Filozofskogo Fak. Niu, Ser. Mat., 4(1990), 57.
GRAEV, M.I.
[1950] Theory of topological groups, I (in Russian), Uspehi Mat. Nauk,
5:2(1950), 356.
GRANADO, M., GRUENHAGE, G.
[2006] Baireness of Ck .X / for ordered X , Comment. Math. Univ. Carolin.
47:1(2006), 103111.
GROTHENDIECK, A.
[1952] Critres de compacit dans les espaces fonctionnels gnereaux, Amer.
J. Math., 74(1952), 168186.
[1953] Sur les applications linaires faiblement compactes despaces du type
C.K/, Canadian J. Math., 5:2(1953), 129173.
GRUENHAGE, G.
[1976] Infinite games and generalizations of first-countable spaces, General
Topology and Appl. 6:3(1976), 339352.
[1984a] Covering properties of X 2 n , W -sets and compact subspaces of
-products, Topology Appl., 17:3(1984), 287304.
[1984b] Generalized metric spaces, Handbook of Set-Theoretic Topology,
North-Holland, Amsterdam, 1984, 423501.
[1986a] Games, covering properties and Eberlein compacta, Topol. Appl.,
23:3(1986), 291298.
[1986b] On a Corson compact space of Todorcevic, Fund. Math., 126(1986),
261268.
[1987] A note on Gulko compact spaces, Proc. Amer. Math. Soc., 100(1987),
371376.
[1997] A non-metrizable space whose countable power is  -metrizable, Proc.
Amer. Math. Soc. 125:6(1997), 18811883.
[1998] Dugundji extenders and retracts of generalized ordered spaces,
Fundam. Math., 158(1998), 147164.
[2002] Spaces having a small diagonal, Topology Appl., 122(2002), 183200.
[2006a] A note on D-spaces, Topology Appl., 153(2006), 22292240.
[2006b] The story of a topological game, Rocky Mountain J. Math. 36:6(2006),
18851914.
[2008] Monotonically compact and monotonically Lindelf spaces, Questions
and Answers Gen. Topology 26:2(2008), 121130.
[2011] A survey of D-spaces, Contemporary Math. 533(2011), 1328.
[2012] Monotonically monolithic spaces, Corson compacts, and D-spaces,
Topology Appl., 159(2012), 15591564.

Bibliography

685

GRUENHAGE, G., MA, D.K.


[1997] Bairness of Ck .X / for locally compact X , Topology Appl., 80(1997),
131139.
GRUENHAGE, G., MICHAEL, E.
[1983] A result on shrinkable open covers, Topology Proc., 8:1(1983), 3743.
GRUENHAGE, G., TAMANO, K.
[2005] If X is  -compact Polish, then Ck .X / has a  -closure-preserving base,
Topology Appl. 151:1-3(2005), 99106.
GRUENHAGE, G., TSABAN, B., ZDOMSKYY, L.
[2011] Sequential properties of function spaces with the compact-open topology, Topology Appl., 158(2011), 387391.
GUERRERO SNCHEZ, D.
[2010] Closure-preserving covers in function spaces, Comment. Math. Univ.
Carolinae, 51:4(2010), 693703.
GUERRERO SNCHEZ, D., TKACHUK, V.V.
[2012] Dense subspaces vs closure-preserving covers of function spaces,
Topology Proc., 39(2012), 219234.
GULKO, S.P.
[1977] On properties of subsets lying in -products (in Russian), Doklady
AN SSSR, 237:3(1977), 505507.
[1978] On properties of some function spaces (in Russian), Doklady AN
SSSR, 243:4(1978), 839842.
[1979] On the structure of spaces of continuous functions and their hereditary
paracompactness (in Russian), Uspehi Matem. Nauk, 34:6(1979),
3340.
[1981] On properties of function spaces (in Russian), Seminar Gen. Topol.,
Moscow University P.H., Moscow, 1981, 841.
[1988] The space Cp .X / for countable infinite compact X is uniformly homeomorphic to c0 , Bull. Acad. Polon. Sci., 36:(5-6)(1988), 391396.
[1990] Spaces of continuous functions on ordinals and ultrafilters
(in Russian), Matem. Zametki, 47:4(1990), 2634.
[1993] Uniformly homeomorphic spaces of functions, Proc. Steklov Inst.
Math., 3(1993), 8793.
GULKO, S.P., KHMYLEVA, T.E.
[1986] The compactness is not preserved by the relation of t -equivalence
(in Russian), Matem. Zametki, 39:6(1986), 895903.

686

Bibliography

GULKO, S.P., OKUNEV, O.G.


[1986] Local compactness and M -equivalence (in Russian), in: Topics in
Geometry and Topology, Petrozavodsk University P.H., Petrozavodsk,
1986, 1423.
GULKO, S.P., OSKIN, A.V.
[1975] Isomorphic classification of spaces of continuous functions on wellordered compact spaces (in Russian), Functional Analysis Appl.,
9:1(1975), 6162.
GULKO, S.P., SOKOLOV, G.A.
[1998] P -points in N and the spaces of continuous functions, Topology
Appl., 85(1998), 137142.
[2000] Compact spaces of separately continuous functions in two variables,
Topology and Its Appl. 107:1-2(2000), 8996.
GUTHRIE, J.A.
[1973] Mapping spaces and cs-networks, Pacific J. Math., 47(1973),
465471.
[1974] Ascoli theorems and pseudocharacter of mapping spaces, Bull.
Austral. Math. Soc., 10(1974), 403408.
HAGER, A.W.
[1969] Approximation of real continuous functions on Lindelf spaces, Proc.
Amer. Math. Soc., 22(1969), 156163.
HAGLER, J.
[1975] On the structure of S and C.S / for S dyadic, Trans. Amer. Math. Soc.,
214(1975), 415428.
HANSARD, J.D.
[1970] Function space topologies, Pacific J. Math., 35(1970), 381388.
HANSEL, G., TROALLIC, J.P.
[1992] Quasicontinuity and Namiokas theorem, Topology Appl., 46(1992),
135149.
HAO-XUAN, Z.
[1982] On the small diagonals, Topology Appl., 13:3(1982), 283293.
HART, J.E., KUNEN, K.
[2002] Bohr topologies and compact function spaces, Topology Appl.,
125(2002), 183198.
[2005] Limits in function spaces and compact groups, Topology Appl. 151:
1-2(2005), 157168.
[2006] Inverse limits and function algebras, Topology Proc. 30:2(2006),
501521.

Bibliography

687

HAYDON, R.G.
[1972] Compactness in Cs .T / and applications, Publ. Dp. Math. Lyon,
9:1(1972), 105113.
[1978] On dual L1 -spaces and injective bidual Banach spaces, Israel J. Math.,
31(1978), 142152.
[1990] A counterexample to several questions about scattered compact
spaces, Bull. London Math. Soc., 22(1990), 261268.
[1994] Countable unions of compact sets with the Namioka property, Mathemat., 41(1994), 141144.
[1995] Baire trees, bad norms and the Namioka property, Mathematika,
42(1995), 3042.
HAYDON, R.G., ROGERS, C.A.
[1990] A locally uniformly convex renorming for certain C.K/, Mathematika,
37(1990), 18.
HEATH, R.W.
[1973a] Monotonically normal spaces, Trans. Amer. Math. Soc., 178(1973),
481493.
[1973b] Some comments on simultaneous extensions of mappings on closed
subsets, in: Topology Conference, Springer, Berlin, VPI 375(1973),
114119.
[1977] Extension properties of generalized metric spaces, Univ. North
Carolina at Greensboro, 1977, 146.
HEATH, R.W., LUTZER, D.J.
[1974a] The Dugundji extension theorem and collectionwise normality, Bull.
Acad. Polon. Sci., Ser. Math., 22(1974), 827830.
[1974b] Dugundji extension theorem for linearly ordered spaces, Pacific
J. Math., 55(1974), 419425.
HEATH, R.W., LUTZER, D.J., ZENOR, P.L.
[1975] On continuous extenders, Studies in Topology, Academic Press,
New York, 1975, 203213.
HELMER, D.
[1981] Criteria for Eberlein compactness in spaces of continuous functions,
Manuscripta Math., 35(1981), 2751.
HERNNDEZ RENDN, A., OKUNEV, O.
[2010] On L.n/-spaces: G -points and tightness, Topology Appl.,
157(2010), 14911494.
HEWITT, E.
[1948] Rings of real-valued continuous functions, I, Trans. Amer. Math. Soc.,
64:1(1948), 4599.

688

Bibliography

HEWITT, E., ROSS, K.


[1963] Abstract Harmonic Analysis, Volume 1, SpringerVerlag, Berlin, 1963.
HODEL, R.
[1984] Cardinal Functions I, in: Handbook of Set-Theoretic Topology, Ed. by
K. Kunen and J.E. Vaughan, Elsevier Science Publishers B.V., 1984,
161.
HRUAK, M., SZEPTYCKI, P.J., TAMARIZ-MASCARA, A.
[2005] Spaces of continuous functions defined on Mrwka spaces, Topol.
Appl., 148(2005), 239252.
HUEK, M.
[1972] Realcompactness of function spaces and .P  Q/, General Topology
and Appl. 2(1972), 165179.
[1977] Topological spaces without -accessible diagonal, Comment. Math.
Univ. Carolinae, 18:4(1977), 777788.
[1979] Mappings from products. Topological structures, II, Math. Centre
Tracts, Amsterdam, 115(1979), 131145.
[1997a] Productivity of some classes of topological linear spaces, Topology
Appl. 80:1-2(1997), 141154.
[1997b] Mazur-like topological linear spaces and their products, Comment.
Math. Univ. Carolin. 38:1(1997), 157164.
[2000] Counterexample on products of bornological spaces, Arch. Math.
(Basel) 75:3(2000), 217219.
[2005] Cp .X / in coreflective classes of locally convex spaces, Topol. Appl.
146/147(2005), 267278.
IVANOV, A.V.
[1978] On bicompacta all finite powers of which are hereditarily separable,
Soviet Math., Doklady, 19:6(1978), 14701473.
JARDN, D.
[2002] Weakly Eberlein compact spaces, Topology Proc., 26(20012002),
695707.
JARDN, D., TKACHUK, V.V.
[2002] Ultracompleteness in EberleinGrothendieck spaces, Boletn de la
Sociedad Mat. Mex., 10:3(2004), 209218.
[2015] Splittability over some classes of Corson compact spaces, Topol.
Appl., 184(2015), 4149.
JAYNE, J.E.
[1974] Spaces of Baire functions, 1, Ann. Inst. Fourier, 24:4(1974), 4776.

Bibliography

689

JAYNE, J.E., NAMIOKA, I., ROGERS, C.A.


[1990] Norm fragmented weak-star compact sets, Collect. Math., 41(1990),
133163.
[1992]  -fragmentable Banach spaces, Mathematika, 39(1992), 161188.
JOHDAI, N.
[1989] Note on equivalent properties relating with Lp .X /, Kobe J. Math.,
6(1989), 201206.
JUHSZ, I.
[1971] Cardinal functions in topology, Mathematical Centre Tracts, 34,
Amsterdam, 1971.
[1980] Cardinal Functions in TopologyTen Years Later, Mathematical
Centre Tracts, North Holland P.C., Amsterdam, 1980.
[1991] Cardinal functions, Recent Progress in General Topology, NorthHolland, Amsterdam, 1992, 417441.
[1992] The cardinality and weight-spectrum of a compact space, Recent
Devel. Gen. Topol. and Appl., Math. Research Berlin 67(1992),
170175.
JUHSZ, I., MILL, J. VAN
[1981] Countably compact spaces all countable subsets of which are scattered, Comment. Math. Univ. Carolin. 22:4(1981), 851855.
JUHSZ, I., SOUKUP, L., SZENTMIKLSSY, Z.
[2007] First countable spaces without point-countable -bases, Fund. Math.
196:2(2007), 139149.
JUHSZ, I., SZENTMIKLSSY, Z.
[1992] Convergent free sequences in compact spaces, Proc. Amer. Math. Soc.,
116:4(1992), 11531160.
[1995] Spaces with no smaller normal or compact topologies, 1993), Bolyai
Soc. Math. Stud., 4(1995), 267274.
[2002] Calibers, free sequences and density, Topology Appl. 119:3(2002),
315324.
[2008] On d -separability of powers and Cp .X /, Topology Appl. 155:4(2008),
277281.
JUHSZ, I., SZENTMIKLOSSY, Z., SZYMANSKI, A.
[2007] Eberlein spaces of finite metrizability number, Comment. Mathem.
Univ. Carolinae 48:2(2007), 291301.
JUST, W., SIPACHEVA, O.V., SZEPTYCKI, P.J.
[1996] Non-normal spaces Cp .X / with countable extent, Proc. American
Math. Society, 124:4(1996), 12271235.

690

Bibliography

KADEC, M.I.
[1967] A proof of topological equivalence of all separable infinitedimensional Banach spaces (in Russian), Functional Analysis Appl.,
1(1967), 5362.
KA KOL, J.
[2000] Remarks on linear homeomorphisms of function spaces, Acta Math.
Hungarica, 89(2000), 315319.
KA KOL, J., KUBI S , W., LPEZ-PELLICER, M.
[2011] Descriptive Topology in Selected Topics of Functional Analysis, Developments in Mathematics, 24, Springer, New York, 2011.
KA KOL, J., LPEZ-PELLICER, M.
[2003] On countable bounded tightness for spaces Cp .X /, J. Math. Anal.
Appl., 280(2003), 155162.
KA KOL, J., LPEZ-PELLICER, M., OKUNEV, O.
[2014] Compact covers and function spaces, J. Math. Anal. Appl., 411(2014),
372380.
KALAMIDAS, N.D.
[1985] Functional properties of C.X / and chain conditions on X , Bull. Soc.
Math. Grce 26(1985), 5364.
[1992] Chain condition and continuous mappings on Cp .X /, Rendiconti Sem.
Mat. Univ. Padova, 87(1992), 1927.
KALAMIDAS, N.D., SPILIOPOULOS, G.D.
[1992] Compact sets in Cp .X / and calibers, Canadian Math. Bull.,
35:4(1992), 497502.
KALENDA, F.K.O.
[2004] Note on countable unions of Corson countably compact spaces,
Comment. Math. Univ. Carolin., 45:3(2004), 499507.
KAUL, S.K.
[1969] Compact subsets in function spaces, Canadian Math. Bull., 12(1969),
461466.
KAWAMURA, K., MORISHITA, K.
[1996] Linear topological classification of certain function spaces on manifolds and CW, Topology Appl., 69:3(1996), 265282.
KENDEROV, P.S.
[1980] Dense strong continuity of pointwise continuous mappings, Pacific
Journal Math., 89(1980), 111130.
[1987] C.T / is weak Asplund for every Gulko compact T , Comptes Rendus
Acad. Bulg. Sci., 40:2(1987), 1719.

Bibliography

691

KHAN, L.A., MORISHITA, K.


[1996] The minimal support for a functional on C b .X /, Topology Appl.,
73(1996), 285294.
KHMYLEVA, T.E.
[1979] Classification of function spaces on segments of ordinals (in Russian),
Siberian Math. Journal, 20:3(1979), 624631.
KHURANA, S.S.
[1981] Pointwise compactness on extreme points, Proc. Amer. Math. Soc.,
83(1981), 347348.
KISLYAKOV, S.V.
[1975] Isomorphic classification of spaces of continuous functions on ordinals
(in Russian), Sibirsk. Mat. Zhurnal, 16:3(1975), 293300.

KO CINAC
, L.D., SCHEEPERS, M.
[1999] Function spaces and strong measure zero sets, Acta Math. Hungar.
82:4(1999), 341351.
[2002] Function spaces and a property of Reznichenko, Topology Appl.
123:1(2002), 135143.

KOLMOGOROV, A.N.
[1957] On representation of continuous functions of several variables as a
superposition of continuous functions of one variable and summing
(in Russian), DAN SSSR, 114:5(1957), 953956.
KOROVIN, A.V.
[1992] Continuous actions of pseudocompact groups and axioms of topological groups, Comment. Math. Univ. Carolinae, 33(1992), 335343.
KOYAMA, A., OKADA, T.
[1987] On compacta which are l-equivalent to I n , Tsukuba J. Math.,
11:1(1987), 147156.
KRAWCZYK, A., MARCISZEWSKI, W., MICHALEWSKI, H.
[2009] Remarks on the set of G -points in Eberlein and Corson compact
spaces, Topology Appl., 156(2009), 17461748.
KRIVORUCHKO, A.I.
[1972] On the cardinality of the set of continuous functions, Soviet Math.,
Dokl., 13(1972), 13641367.
[1973] On cardinal invariants of spaces and mappings, Soviet Math.,
Doklady, 14(1973), 16421647.
[1975] The cardinality and density of spaces of mappings, Soviet Math.,
Doklady, 16(1975), 281285.

692

Bibliography

KRUPSKI, M.
[2015] A note on condensations of function spaces onto  -compact and
analytic spaces, Proc. Amer. Math. Soc., 143(2015), 22632268.
KUBIS, W., LEIDERMAN, A.
[2004] Semi-Eberlein spaces, Topology Proc. 28:2(2004), 603616.
KUBIS, W., OKUNEV, O., SZEPTYCKI, P.J.
[2006] On some classes of Lindelof -spaces, Topology Appl., 153(2006),
25742590.
KUNDU, S., MCCOY, R.A.
[1993] Topologies between compact and uniform convergence on function
spaces, Internat. J. Math. Math. Sci. 16(1993), 101110.
[1995] Weak and support-open topologies on C.X /, Rocky Mountain J. Math.
25:2(1995), 715732.
KUNDU, S., MCCOY, R.A., OKUYAMA, A.
[1989] Spaces of continuous linear functionals on Ck .X /, Math. Japon.
34:5(1989), 775787.
KUNDU, S., MCCOY, R.A., RAHA, A.B.
[1993] Topologies between compact and uniform convergence on function
spaces II, Real Anal. Exchange 18:1(1992/93), 176189.
KUNDU, S., OKUYAMA, A.
[1993] Complete duals of C  .X /, Math. Scand. 72:1(1993), 3346.
KUNEN, K.
[1980] Set Theory. An Introduction to Independence Proofs, Studies Logic
Found. Mathematics, 102(1980), North Holland P.C., Amsterdam,
1980
[1981] A compact L-space under CH, Topology Appl., 12(1981), 283287.
[1998] Bohr topologies and partition theorems for vector spaces, Topology
Appl. 90:1-3(1998), 97107.
KUNEN, K., MILL, J. VAN
[1995] Measures on Corson compact spaces, Fund. Math., 147(1995), 6172.
KUNEN, K., DE LA VEGA, R.
[2004] A compact homogeneous S -space, Topology Appl., 136(2004),
123127.
KURATOWSKI, C.
[1966] Topology, vol. 1, Academic Press Inc., London, 1966.

Bibliography

693

LAMBRINOS, P.
[1981] The bounded-open topology on function spaces, Manuscripta Math.,
36(1981), 4766.
LEE, J.P., PIOTROWSKI, Z.
[1985] A note on spaces related to Namioka spaces, Bull. Austral. Math. Soc.,
31(1985), 285292.
LEIDERMAN, A.G.
[1984] On properties of spaces of continuous functions (in Russian), Cardinal
Invariants and Mappings of Topological Spaces (in Russian), Izhevsk,
1984, 5054.
[1985] On dense metrizable subspaces of Corson compact spaces
(in Russian), Matem. Zametki, 38:3(1985), 440449.
[1988] Adequate families of sets and function spaces, Commentationes Math.
Univ. Carolinae, 29:1(1988), 3139.
LEIDERMAN, A., LEVIN, M., PESTOV, V.
[1997] On linear continuous open surjections of the spaces Cp .X /, Topol.
Appl., 81(1997), 269279.
LEIDERMAN, A.G., MALYKHIN, V.I.
[1988] On nonpreservation of final compactness by products of spaces Cp .X /
(in Russian), Sibirsk. Mat. Zhurnal, 29:1(1988), 8493.
LEIDERMAN, A.G., MORRIS, S.A., PESTOV, V.G.
[1997] The free Abelian topological group and the free locally convex space
on the unit interval, J. London Math. Soc., 56(1997), 529538.
LEIDERMAN, A.G., SOKOLOV, G.A.
[1984] Adequate families of sets and Corson compacts, Commentat. Math.
Univ. Carolinae, 25:2(1984), 233246.
LELEK, A.
[1969] Some cover properties of spaces, Fund. Math., 64:2(1969), 209218.
LINDENSTRAUSS, J.
[1972] Weakly compact sets their topological properties and the Banach
spaces they generate, Annals Math. Studies, 69(1972), 235273.
LINDENSTRAUSS, J., STEGALL, C.
[1975] Examples of separable spaces which do not contain `1 and whose
duals are nonseparable, Studia Math., 54(1975), 81105.
LINDENSTRAUSS, J., TZAFRIRI, L.
[1977] Classical Banach Spaces I, Springer, Berlin, 1977.

694

Bibliography

LUTZER, D.J., MCCOY, R.A.


[1980] Category in function spaces I, Pacific J. Math., 90:1(1980), 145168.
LUTZER, D.J., MILL, J. VAN, POL, R.
[1985] Descriptive complexity of function spaces, Trans. Amer. Math. Soc.,
291(1985), 121128.
LUTZER, D. J., MILL, J. VAN, TKACHUK, V.V.
[2008] Amsterdam properties of Cp .X / imply discreteness of X , Canadian
Math. Bull. 51:4(2008), 570578.
MA, D.K.
[1993] The Cantor tree, the  -property, and Baire function spaces, Proc.
Amer. Math. Soc., 119:3(1993), 903913.
MALYKHIN, V.I.
[1987] Spaces of continuous functions in simplest generic extensions, Math.
Notes, 41(1987), 301304.
[1994] A non-hereditarily separable space with separable closed subspaces,
Q & A in General Topology, 12(1994), 209214.
[1998] On subspaces of sequential spaces (in Russian), Matem. Zametki,
64:3(1998), 407413.
[1999] Cp .I / is not subsequential, Comment. Math. Univ. Carolinae,
40:4(1999), 785788.
MALYKHIN, V.I., SHAKHMATOV, D.B.
[1992] Cartesian products of Frchet topological groups and function spaces,
Acta Math. Hungarica, 60(1992), 207215.
MARCISZEWSKI, W.
[1983] A pre-Hilbert space without any continuous map onto its own square,
Bull. Acad. Polon. Sci., 31:(9-12)(1983), 393397.
[1987] Lindelf property in function spaces and a related selection theorem,
Proc. Amer. Math. Soc., 101(1987), 545550.
[1988a] A remark on the space of functions of first Baire class, Bull. Polish
Acad. Sci., Math., 36:(1-2)(1997), 6567.
[1988b] A function space C.K/ not weakly homeomorphic to C.K/  C.K/,
Studia Mathematica, 88(1988), 129137.
[1989] On classification of pointwise compact sets of the first Baire class
functions, Fund. Math., 133(1989), 195209.
[1991] Order types, calibers and spread of Corson compacta, Topology Appl.,
42(1991), 291299.
[1992] On properties of Rosenthal compacta, Proc. Amer. Math. Soc.,
115(1992), 797805.
[1993] On analytic and coanalytic function spaces Cp .X /, Topology and Its
Appl., 50(1993), 241248.

Bibliography

695

[1995a] On universal Borel and projective filters, Bull. Acad. Polon. Sci.,
Math., 43:1(1995), 4145.
[1995b] A countable X having a closed subspace A with Cp .A/ not a factor of
Cp .X /, Topology Appl., 64(1995), 141147.
[1995c] On sequential convergence in weakly compact subsets of Banach
spaces, Studia Math., 112:2(1995), 189194.
[1997a] A function space Cp .X / not linearly homeomorphic to Cp .X /  R,
Fundamenta Math., 153:2(1997), 125140.
[1997b] On hereditary Baire products, Bull. Polish Acad. Sci., Math.,
45:3(1997), 247250.
[1997c] On topological embeddings of linear metric spaces, Math. Ann.,
308:1(1997), 2130.
[1998a] P -filters and hereditary Baire function spaces, Topology Appl.,
89(1998), 241247.
[1998b] Some recent results on function spaces Cp .X /, Recent Progress in
Function Spaces., Quad. Mat. 3(1998), Aracne, Rome, 221239.
[1998c] On van Mills example of a normed X with X 6
X  R, Proc. Amer.
Math. Society, 126:1(1998), 319321.
[2000] On properties of metrizable spaces X preserved by t -equivalence,
Mathematika, 47(2000), 273279.
[2002] Function Spaces, in: Recent Progress in General Topology II, Ed.
by M. Huek and J. van Mill, Elsevier Sci. B.V., Amsterdam, 2002,
345369.
[2003a] A function space Cp .X / without a condensation onto a  -compact
space, Proc. Amer. Math. Society, 131:6(2003), 19651969.
[2003b] On Banach spaces C.K/ isomorphic to c0 . /, Studia Math.,
156:3(2003), 295302.
[2004] Rosenthal compacta, in: Encycl. General Topology, Elsevier Sci. Publ.,
Amsterdam, 2004, 142144.
MARCISZEWSKI, W., MILL, J. VAN
[1998] An example of tp -equivalent spaces which are not tp -equivalent,
Topology Appl., 85(1998), 281285.
MARCISZEWSKI, W., PELANT, J.
[1997] Absolute Borel sets and function spaces, Trans. Amer. Math. Soc.,
349:9(1997), 35853596.
MARCISZEWSKI, W., POL, R.
[2009] On Banach spaces whose norm-open sets are F -sets in the weak
topology, J. Math. Anal. Appl., 350:2(2009), 708722.
[2010] On some problems concerning Borel structures in function spaces,
Revista de la Real Acad. Cien. (RACSAM), 104:2(2010), 327335.

696

Bibliography

MTRAI, T.
[2004] A characterization of spaces l-equivalent to the unit interval, Topology
Appl., 138(2004), 299314.
MCCOY, R.A.
[1975] First category function spaces under the topology of pointwise convergence, Proc. Amer. Math. Soc., 50(1975), 431434.
[1978a] Characterization of pseudocompactness by the topology of uniform
convergence on function spaces, J. Austral. Math. Soc., 26(1978),
251256.
[1978b] Submetrizable spaces and almost  -compact function spaces, Proc.
Amer. Math. Soc., 71(1978), 138142.
[1978c] Second countable and separable function spaces, Amer. Math.
Monthly, 85:6(1978), 487489.
[1980a] Countability properties of function spaces, Rocky Mountain J. Math.,
10(1980), 717730.
[1980b] A K-space function space, Int. J. Math. Sci., 3(1980), 701711.
[1980c] Necessary conditions for function spaces to be Lindelf, Glas. Mat.,
15(1980), 163168.
[1980d] Function spaces which are k-spaces, Topology Proc., 5(1980),
139146.
[1983] Complete function spaces, Internat. J. Math. Math. Sci. 6:2(1983),
271277.
[1986] Fine topology on function spaces, Internat. J. Math. Math. Sci.
9:3(1986), 417424.
MCCOY, R.A., NTANTU, I.
[1985] Countability properties of function spaces with set-open topologies,
Topology Proceedings 10:2(1985), 329345.
[1986] Completeness properties of function spaces, Topology Appl.,
22:2(1986), 191206.
[1988] Topological Properties of Spaces of Continuous Functions, Lecture
Notes in Math., 1315, Springer, Berlin, 1988.
[1992] Properties of C.X / with the epi-topology, Boll. Un. Mat. Ital. B
(7)6:3(1992), 507532.
[1995] Properties of C.X / with the weak epi-topology, Q&A General Topol.
13:2(1995), 139152.
MERKCOURAKIS, S., NEGREPONTIS, S.
[1992] Banach spaces and Topology, 2,
Elsevier, N.Y., 1992.

Recent Progress in Topology,

MEYER, P.R.
[1970] Function spaces and the AlexandroffUrysohn conjecture, Estratto
degli Annali di Matematica Pura ed Aplicata, Ser. 4, 86(1970), 2529.

Bibliography

697

MICHAEL, E.
[1961] On a theorem of Rudin and Klee, Proc. Amer. Math. Soc., 12(1961),
921.
[1966] @0 -spaces, J. Math. and Mech., 15:6(1966), 9831002.
[1971] Paracompactness and the Lindelf property in finite and countable
Cartesian products, Compositio Math., 23(1971), 179214.
[1973] On k-spaces, kR -spaces and k.X /, Pacific J. Math., 47:2(1973),
487498.
[1977] @0 -spaces and a function space theorem of R. Pol, Indiana Univ. Math.
J., 26(1977), 299306.
MICHAEL, E., RUDIN, M.E.
[1977a] A note on Eberlein compacta, Pacific J. Math., 72:2(1977), 487495.
[1977b] Another note on Eberlein compacta, Pacific J. Math., 72:2(1977),
497499.
MICHALEWSKI, H.
[2001] An answer to a question of Arhangelskii, Commentat. Math. Univ.
Carolinae, 42:3(2001), 545550.
MILL, J. VAN
[1982] A homogeneous Eberlein compact space which is not metrizable,
Pacific J. Mathematics, 101:1(1982), 141146.
[1984] An introduction to !, Handbook of Set-Theoretic Topology, NorthHolland, Amsterdam, 1984, 503567.
[1987] Topological equivalence of certain function spaces, Compositio Math.,
63(1987), 159188.
[1989] Infinite-Dimensional Topology. Prerequisites and Introduction, North
Holland, Amsterdam, 1989.
[1999] Cp .X / is not G : a simple proof, Bull. Polon. Acad. Sci., Ser. Math.,
47(1999), 319323.
[2002] The Infinite-Dimensional Topology of Function Spaces, North Holland
Math. Library 64, Elsevier, Amsterdam, 2002.
MILL, J. VAN, PELANT, J., POL, R.
[2003] Note on function spaces with the topology of pointwise convergence,
Arch. Math., 80(2003), 655663.
MILL, J. VAN, POL, R.
[1986] The Baire Category Theorem in products of linear spaces and topological groups, Topology Appl., 22(1986), 267282.
[1993] A countable space with a closed subspace without measurable extender, Bull. Acad. Polon. Sci., Math., 41(1993), 279283.
MISRA, P.R.
[1982] On isomorphism theorems for C.X /, Acta Mathematica Hungarica,
39:4(1982), 389390.

698

Bibliography

MOLINA LARA, I., OKUNEV, O.


[2010] L. !/-spaces and spaces of continuous functions, Cent. Eur. J.
Math., 8:4(2010), 754762.
MORISHITA, K.
[xxxx] The kR -property of function spaces, Preprint.
[1992a] The minimal support for a continuous functional on a function space,
Proc. Amer. Math. Soc. 114:2(1992), 585587.
[1992b] The minimal support for a continuous functional on a function space.
II, Tsukuba J. Math. 16:2(1992), 495501.
[1999] On spaces that are l-equivalent to a disk, Topology Appl., 99(1999),
111116.
MORITA, K.
[1956] Note on mapping spaces, Proc. Japan Acad., 32(1956), 671675.
MYCIELSKI, J.
[1973] Almost every function is independent, Fund. Math., 81(1973), 4348.
MYKHAYLYUK V.V.
[2006] Namioka spaces and topological games, Bull. Austral. Math. Soc.
73:2(2006), 263272.
[2007] Metrizable compacta in the space of continuous functions with the
topology of pointwise convergence, Acta Math. Hungar. 117:4(2007),
315323.
NACHBIN, L.
[1954] Topological vector spaces of continuous functions, Proceedings Nat.
Acad. Sci., (USA), 40:6(1954), 471474.
NAGAMI, K.
[1969] -spaces, Fund. Math., 65:2(1969), 169192.
NAGATA J.
[1949] On lattices of functions on topological spaces and of functions on
uniform spaces, Osaka Math. J., 1:2(1949), 166181.
NAKHMANSON, L. B.
[1982] On continuous images of  -products (in Russian), Topology and Set
Theory, Udmurtia Universty P.H., Izhevsk, 1982, 1115.
[1984] The Souslin number and calibers of the ring of continuous functions
(in Russian), Izv. Vuzov, Matematika, 1984, N 3, 4955.
[1985] On Lindelf property of function spaces (in Russian), Mappings and
Extensions of Topological Spaces, Udmurtia University P.H., Ustinov,
1985, 812.

Bibliography

699

NAKHMANSON, L. B., YAKOVLEV, N.N.


[1982] On compacta, lying in  -products (in Russian), Comment. Math. Univ.
Carolinae, 22:4(1981), 705719.
NAMIOKA, I.
[1974] Separate continuity and joint continuity, Pacific J. Math., 51:2(1974),
515531.
[2002] On generalizations of Radon-Nikodym compact spaces, Topol. Proc.
26:2(2001/02), 741750.
NAMIOKA, I., PHELPS, R.R.
[1975] Banach spaces which are Asplund spaces, Duke Math. J., 42(1975),
735750.
NAMIOKA, I., POL, R.
[1992] Mappings of Baire spaces into function spaces and Kadec renorming,
Israel J. Math., 78(1992), 120.
[1998]  -fragmentability of mappings into Cp .K/, Topology Appl., 89(1998),
249263.
[1996]  -fragmentability and analyticity, Mathematika 43:1(1996), 172181.
NEGREPONTIS, S.
[1984] Banach spaces and topology, Handbook of Set-Theoretic Topology,
North Holland, Amsterdam, 1984, 10011142.
NOBLE, N.
[1969a] Products with closed projections, Trans Amer. Math. Soc., 140(1969),
381391.
[1969b] Ascoli theorems and the exponential map, Trans Amer. Math. Soc.,
143(1969), 393411.
[1970] The continuity of functions on Cartesian products, Transactions Amer.
Math. Soc., 149(1970), 187198.
[1971] Products with closed projections II, Trans Amer. Math. Soc.,
160(1971), 169183.
[1974] The density character of function spaces, Proc. Amer. Math. Soc.,
42:1(1974), 228233.
NOBLE, N., ULMER, M.
[1972] Factoring functions on Cartesian products, Trans Amer. Math. Soc.,
163(1972), 329339.
NYIKOS, P.
[1981] Metrizability and the FrchetUrysohn property in topological groups,
Proc. Amer. Math. Soc., 83:4(1981), 793801.
[1989] Classes of compact sequential spaces, Set Theory Appl., Lecture Notes
in Math., 1401, Springer, Berlin, 1989, 135159.

700

Bibliography

OHTA, H., YAMADA, K.


[1998] On spaces with linearly homeomorphic function spaces in the compact
open topology, Tsukuba Math. J., 22(1998), 3948.
OKADA, T.
[1985] A remark on Eberlein compacta, Kobe J. Math., 2(1985), 6566.
OKADA, T., OKUYAMA, A.
[1987] Note on topologies of linear continuous functionals and real measure
functions, Kobe J. Math. 4:1(1987), 7377.
OKUNEV, O.G.
[1984] Hewitt extensions and function spaces (in Russian), Cardinal Invariants
and Mappings of Topological Spaces (in Russian), Izhvsk, 1984,
7778.
[1985] Spaces of functions in the topology of pointwise convergence: Hewitt
extension and -continuous functions, Moscow Univ. Math. Bull.,
40:4(1985), 8487.
[1986] On non-preserving a property by the relation of M -equivalence
(in Russian), Continuous Functions on Topological Spaces (in Russian),
Riga, 1986, 123125.
[1989] On compact-like properties of spaces of continuous functions in the
topology of pointwise convergence, Bull. Acad. Sci. Georgian SSR,
134:3(1989), 473476.
[1990] Weak topology of an associated space, and t -equivalence, Math.
Notes, 46:(1-2)(1990), 534538.
[1993a] On Lindelf -spaces of functions in the pointwise topology, Topol.
Appl., 49(1993), 149166.
[1993b] On analyticity in cosmic spaces, Comment. Math. Univ. Carolinae,
34(1993), 185190.
[1995a] On Lindelf sets of continuous functions, Topology Appl., 63(1995),
9196.
[1995b] M -equivalence of products, Trans. Moscow Math. Soc., 56(1995),
149158.
[1996] A remark on the tightness of products, Comment. Math. Univ. Carolinae, 37:2(1996), 397399.
[1997a] Homeomorphisms of function spaces and hereditary cardinal invariants, Topology Appl., 80(1997), 177188.
[1997b] On the Lindelf property and tightness of products, Topology Proc.,
22(1997), 363371.
[2002] Tightness of compact spaces is preserved by the t -equivalence relation,
Comment. Math. Univ. Carolin. 43:2(2002), 335342.
[2005a] Frchet property in compact spaces is not preserved by M -equivalence,
Comment. Math. Univ. Carolin. 46:4(2005), 747749.
[2005b] A  -compact space without uncountable free sequences can have
arbitrary tightness, Questions Answers Gen. Topology 23:2(2005),
107108.

Bibliography

701

[2009] On the Lindelf property of spaces of continuous functions over a


Tychonoff space and its subspaces, Comment. Math. Univ. Carolin.
50:4(2009), 629635.
[2011a] A relation between spaces implied by their t -equivalence, Topology
Appl. 158(2011), 21582164.
[2011b] The Lindelf number of Cp .X /Cp .X / for strongly zero-dimensional
X , Cent. Eur. J. Math., 9:5(2011), 978983.
[2013] The one-point Lindelfication of an uncountable discrete space can be
surlindelf, Cent. Eur. J. Math., 11:10(2013), 17501754.
OKUNEV, O.G., RAMREZ, H.D.
[2013] Linear homeomorphisms of function spaces and properties close to
countable compactness, Topology Proc., 40(2013), 18.
OKUNEV, O., REZNICHENKO, E.
[2013] A note on surlindelf spaces, Topology Proc., 31(2007), 667675.
OKUNEV, O.G., SHAKHMATOV, D.B.
[1987] The Baire property and linear isomorphisms of continuous function
spaces (in Russian), Topological Structures and Their Maps, Latvian
State University P.H., Riga, 1987, 8992.
OKUNEV, O.G., TAMANO, K.
[1996] Lindelf powers and products of function spaces, Proceedings Amer.
Math. Soc., 124:9(1996), 29052916.
OKUNEV, O.G., TAMARIZ-MASCARA, A.

[2004] On the Cech


number of Cp .X /, Topology Appl., 137(2004), 237249.
OKUNEV, O.G., TKACHUK, V.V.
[2001] Lindelf -property in Cp .X / and p.Cp .X // D ! do not imply
countable network weight in X , Acta Mathematica Hungarica,
90:(12)(2001), 119132.
[2002] Density properties and points of uncountable order for families of open
sets in function spaces, Topology Appl., 122(2002), 397406.
[2014] Calibers, !-continuous maps and function spaces, RACSAM,
108:2(2014), 419430.
OKUYAMA, A.
[1981] Some relationships between function spaces and hyperspaces of compact sets, General Topology and Its Relations to Modern Analysis and
Algebra, V, Fifth Prague Topolol. Symposium, 1981, 527535.
[1983] Questions on angelic and strictly angelic spaces, Questions Answ.
Gen. Topol. 1:1(1983), 5153.
[1987a] On a topology of the set of real measure functions, Kobe J. Math.
3:2(1987), 237242.

702

Bibliography

[1987b] On a topology of the set of linear continuous functionals, Kobe Journal


Math. 3:2(1987), 213217.
[1990] Note on spaces of continuous functionals on C  .X /, Questions
Answers Gen. Topology 8:1(1990), 7982.
OKUYAMA, A., TERADA, T.
[1989] Function spaces, Topics in General Topology, 1989, 411458.
OMEARA, P.
[1971] On paracompactness in function spaces with the compact-open topology, Trans. Amer. Math. Soc., 29:1(1971), 183189.
ORIHUELA, J.
[1987] Pointwise compactness in spaces of continuous functions, Journal
London Math. Soc., 36:2(1987), 143152.
[1992] On weakly Lindelf Banach spaces, Progress in Functional Analysis,
Elsevier S.P., 1992, 279291.
ORIHUELA, J., SCHACHERMAYER, W., VALDIVIA, M.
[1991] Every RadonNykodym Corson compact space is Eberlein compact,
Studia Mathematica, 98:2(1991), 157174.
OSTRAND, PH. A.
[1965] Dimension of metric spaces and Hilberts problem 13, Bull. Amer.
Math. Soc., 71:4(1965), 619622.
OXTOBY, J.C.
[1961] Cartesian products of Baire spaces, Fund. Math., 49:2(1961),
157166.
PASYNKOV, B.A.
[1967] On open mappings, Soviet Math. Dokl., 8(1967), 853856.
PAVLOVSKII, D.S.
[1979] The spaces of open sets and spaces of continuous functions (in
Russian), Doklady AN SSSR, 246:4(1979), 815818.
[1980] On spaces of continuous functions (in Russian), Doklady AN SSSR,
253:1(1979), 3841.
[1982] On spaces which have linearly homeomorphic spaces of continuous
functions endowed with the topology of pointwise convergence (in
Russian), Uspehi Matem. Nauk, 37:2(1982), 185186.
PELANT, J.
[1988] A remark on spaces of bounded continuous functions, Indag. Math.,
91(1988), 335338.

Bibliography

703

PELCZI NSKI
, A.
[1960] Projections in certain Banach spaces, Studia Math., 19(1960),
209228.
[1968] Linear extensions, linear averaging, and their applications to linear
topological classification of spaces of continuous functions, Dissertationes Math., (Rozprawy Mat.), 58(1968).

PELCZI NSKI
, A., SEMADENI, Z.
[1959] Spaces of continuous functions (3) (spaces C./ for ) without
perfect subsets, Studia Math., 18(1959), 211222.

PESTOV, V.G.
[1982] Coincidence of the dimension dim of l-equivalent topological spaces
(in Russian), Doklady AN SSSR, 266:3(1982), 553556.
[1984] Some topological properties preserved by the relation of M -equivalence
(in Russian), Uspehi Mat. Nauk, 39:6(1984), 203204.
PIOTROWSKI, Z.
[1985] Separate and joint continuity, Real Analysis Exchange, 11(1985/86),
293322.
PLEBANEK, G.
[1995] Compact spaces that result from adequate families of sets, Topol.
Appl., 65:3(1995), 257270.
POL, R.
[1972] On the position of the set of monotone mappings in function spaces,
Fund. Math., 75(1972), 7584.
[1974] Normality in function spaces, Fund. Math., 84:2(1974), 145155.
[1977] Concerning function spaces on separable compact spaces, Bull. Acad.
Polon. Sci., Sr. Math., Astron. et Phys., 25:10(1977), 993997.
[1978] The Lindelf property and its analogue in function spaces with weak
topology, Topology. 4-th Colloq. Budapest, 2(1978), Amsterdam,
1980, 965969.
[1979] A function space C.X / which is weakly Lindelf but not weakly
compactly generated, Studia Math., 64:3(1979), 279285.
[1980a] A theorem on the weak topology of C.X / for compact scattered X ,
Fundam. Math., 106:2(1980), 135140.
[1980b] On a question of H.H. Corson and some related matters, Fund. Math.,
109:2(1980), 143154.
[1982] Note on the spaces P .S / of regular probability measures whose topology is determined by countable subsets, Pacific J. Math., 100(1982),
185201.
[1984a] An infinite-dimensional pre-Hilbert space not homeomorphic to its
own square, Proc. Amer. Math. Soc., 90:3(1984), 450454.

704

Bibliography

[1984b] On pointwise and weak topology in function spaces, Univ. Warszawski, Inst. Matematiki, Preprint 4/84, Warszawa, 1984.
[1986] Note on compact sets of first Baire class functions, Proceedings Amer.
Math. Soc., 96:1(1986), 152154.
[1989] Note on pointwise convergence of sequences of analytic sets, Mathem.,
36(1989), 290300.
[1995] For a metrizable X , Cp .X / need not be linearly homeomorphic to
Cp .X /  Cp .X /, Mathematika, 42(1995), 4955.
PONTRIAGIN, L.S.
[1984] Continuous Groups (in Russian), Nauka, Moscow, 1984.
POPOV, V.V.
[xxxx] The space Cp .N/ has not a projectively Baire property, Preprint.
PREISS, D., SIMON, P.
[1974] A weakly pseudocompact subspace of Banach space is weakly compact, Comment. Math. Univ. Carolinae, 15:4(1974), 603609.
PRYCE, J.D.
[1971] A device of R.J. Whitleys applied to pointwise compactness in
spaces of continuous functions, Proc. Amer. Math. Soc., 23:3(1971),
532546.
PTK, V.
[1954a] On a theorem of W.F. Eberlein, Studia Math., 14:2(1954), 276284.
[1954b] Weak compactness in locally convex topological vector spaces,
Czechoslovak Math. J., 79(1954), 175186.
[1955] Two remarks on weak compactness, Czechoslovak Math. J., 80(1955),
532545.
[1963] A combinatorial lemma on the existence of convex means and its
application to weak compactness, Proc. Sympos. Pure Math., 7,
Convexity, 1963, 437450.
PYTKEEV, E.G.
[1976] Upper bounds of topologies, Math. Notes, 20:4(1976), 831837.
[1982a] On the tightness of spaces of continuous functions (in Russian), Uspehi
Mat. Nauk, 37:1(1982), 157158.
[1982b] Sequentiality of spaces of continuous functions (in Russian), Uspehi
Matematicheskih Nauk, 37:5(1982), 197198.
[1985] The Baire property of spaces of continuous functions, (in Russian),
Matem. Zametki, 38:5(1985), 726740.
[1990] A note on Baire isomorphism, Comment. Math. Univ. Carolin.
31:1(1990), 109112.
[1992a] On FrchetUrysohn property of spaces of continuous functions,
(in Russian), Trudy Math. Inst. RAN, 193(1992), 156161.

Bibliography

705

[1992b] Spaces of functions of the first Baire class over K-analytic spaces (in
Russian), Mat. Zametki 52:3(1992), 108116.
[2003] Baire functions and spaces of Baire functions, Journal Math. Sci.
(N.Y.) 136:5(2006), 41314155
PYTKEEV, E.G., YAKOVLEV, N.N.
[1980] On bicompacta which are unions of spaces defined by means of
coverings, Comment. Math. Univ. Carolinae, 21:2(1980), 247261.
RAJAGOPALAN, M., WHEELER, R.F.
[1976] Sequential compactness of X implies a completeness property for
C.X /, Canadian J. Math., 28(1976), 207210.
REYNOV, O.I.
[1981] On a class of compacts and GCG Banach spaces, Studia Math.,
71(1981), 113126.
REZNICHENKO, E.A.
[1987] Functional and weak functional tightness (in Russian), Topological
Structures and Their Maps, Latvian State University P.H., Riga, 1987,
105110.
[1989a] Paracompactness is not preserved by l-equivalence, General Topology, Spaces and Mappings, Mosk. Univ., Moscow, 1989, 155156.
[1989b] A pseudocompact space in which only the subsets of not full cardinality
are not closed and not discrete, Moscow Univ. Math. Bull. 44:6(1989),
7071.
[1990a] Normality and collectionwise normality in function spaces, Moscow
Univ. Math. Bull., 45:6(1990), 2526.
[1990b] Convex compact spaces and their maps, Topology Appl., 36(1990),
117141.
[1994] Extension of functions defined on products of pseudocompact spaces
and continuity of the inverse in pseudocompact groups, Topology
Appl., 59:3(1994), 233244.
[2008] Stratifiability of Ck .X / for a class of separable metrizable X , Topology Appl. 155:17-18(2008), 20602062.
[xxxxa] Groups with separately continuous and jointly continuous multiplication, Preprint.

[xxxxb] Talagrand compact spaces: -points and StoneCech


extensions,
Preprint.
REZNICHENKO, E. A., SHKARIN, S.A.
[2002] On linear extension operators, Russ. J. Math. Phys. 9:2(2002),
188197.
RIBARSKA, N.K.
[1987] Internal characterization of fragmentable spaces, Mathematika,
34(1987), 243257.

706

Bibliography

ROJAS-HERNNDEZ, R.
[2014a] On monotone stability, Tooplogy Appl., 165(2014), 5057.
[2014b] Function spaces and D-property, Topology Proc., 43(2014), 301317.
ROJAS-HERNNDEZ, R., TAMARIZ-MASCARA, A.,
[2012] D-property, monotone monolithicity and function spaces, Topology
Appl., 159(2012), 33793391.
ROJAS-HERNNDEZ, R., TKACHUK, V.V.
[2014] A monotone version of the Sokolov property and monotone retractability in function spaces, J. Math. Anal. Appl., 412:1(2014), 125137.
ROSENTHAL, H.P.
[1974] The heredity problem for weakly compactly generated Banach spaces,
Compositio Math., 28:1(1974), 88111.
[1977] Pointwise compact subsets of the first Baire class, Amer. J. Math.,
99:2(1977), 362378.
[1978] Some recent discoveries in the isomorphic theory of Banach spaces,
Bull. Amer. Math. Soc., 84(1978), 803831.
RUDIN, M.E.
[1956] A note on certain function spaces, Arch. Math., 7(1956), 469470.
RUDIN, W.
[1973] Functional Analysis, McGraw-Hill Book Company, New York, 1973.
SAINT RAYMOND, J.
[1976] Fonctions boreliennes sur quotients, Bull. Sci. Math., 100:2(1976),
141147.
[1984] Jeux topologiques et espaces de Namioka, Proc. Amer. Math. Soc.,
87(1984), 499504.
SAKAI, M.
[1988a] On supertightness and function spaces, Comment. Math. Univ.
Carolin., 29:2(1988), 249251.
[1988b] Property C 00 and function spaces, Proc. Amer. Math. Soc.,
104:3(1988), 917919.
[1992] On embeddings into Cp .X / where X is Lindelf, Comment. Math.
Univ. Carolinae, 33:1(1992), 165171.
[1995] Embeddings of -metrizable spaces into function spaces, Topol. Appl.,
65(1995), 155165.
[2000] Variations on tightness in function spaces, Topology Appl., 101(2000),
273280.
[2003] The Pytkeev property and the Reznichenko property in function spaces,
Note di Matem. 22:2(2003/04), 4352

Bibliography

707

[2006] Special subsets of reals characterizing local properties of function


spaces, Selection Principles and Covering Properties in Topology,
Dept. Math., Seconda Univ. Napoli, Caserta, Quad. Mat., 18(2006),
195225,
[2007] The sequence selection properties of Cp .X /, Topology Appl.,
154(2007), 552560.
[2008] Function spaces with a countable cs  -network at a point, Topol. Appl.,
156:1(2008), 117123.
[xxxx] On calibers of function spaces, Preprint.
SCHACHENMAYER, W.
[1977] Eberlein compacts et espaces de Radon, C. R. Acad. Sci. Paris,
284(1977), 405477.
SCHEEPERS, M.
[1997a] Combinatorics of open covers III. Games, Cp .X /, Fund. Math.
152:3(1997), 231254.
[1997b] A sequential property of Cp .X / and a covering property of Hurewicz,
Proc. Amer. Math. Soc., 125:9(1997), 27892795.
[1998a] Cp .X / and Arhangelskiis i -spaces, Topology Appl., 89(1998),
265275.
[1998b] Strong measure zero sets, filters, and pointwise convergence, EastWest Journal Math. 1:1(1998), 109116.
[1999a] Sequential convergence in Cp .X / and a covering property, East-West
J. Math., 1(1999), 207214.
[1999b] Combinatorics of open covers. VI. Selectors for sequences of dense
sets, Quaest. Math. 22:1(1999), 109130.
[2008] Rothbergers property in all finite powers, Topology Appl.
156:1(2008), 93103.
SCHEPIN, E.V.
[1979] On -metrizable spaces (in Russian), Izvestiya Acad. Nauk SSSR, Ser.
Math., 43:2(1979), 442478.
SCHMETS, J.
[1976] Espaces de fonctions continues, Lecture Notes in Math., 519(1976),
N XII.
SEMADENI, Z.
[1960] Banach spaces non-isomorphic to their Cartesian squares, Bull. Acad.
Polon. Sci., Math., 8(1960), 8184.
[1971] Banach spaces of continuous functions, Monog. Mat., 1:55(1971).
SHAKHMATOV, D.B.
[1986] A pseudocompact Tychonoff space all countable subsets of which are
closed and C  -embedded, Topology Appl., 22:2(1986), 139144.

708

Bibliography

SHAPIROVSKY, B.E.
[1974] Canonical sets and character. Density and weight in compact spaces,
Soviet Math. Dokl., 15(1974), 12821287.
[1978] Special types of embeddings in Tychonoff cubes, subspaces of
-products and cardinal invariants, Colloquia Mathematica Soc.
Janos Bolyai, 23(1978), 10551086.
[1981] Cardinal invariants in bicompacta (in Russian), Seminar on General
Topology, Moscow University P.H., Moscow, 1981, 162187.
SHIROTA, T.
[1954] On locally convex vector spaces of continuous functions, Proceedings
Japan Acad., 30:4(1954), 294298.
SIMON, P.
[1976] On continuous images of Eberlein compacts, Commentationes Math.
Univ. Carolinae, 17:1(1976), 179194.
SIPACHEVA, O.V.
[1986] Description of the topology of free topological groups without using
structures of universal uniformities (in Russian), General Topology.
Mappings of Topological Spaces, Moscow University P.H., Moscow,
1986, 122130.
[1988] Lindelf -spaces of functions and the Souslin number, Moscow Univ.
Math. Bull., 43:3(1988), 2124.
[1989] On Lindelf subspaces of function spaces over linearly ordered separable compacta, General Topology, Spaces and Mappings, Mosk.
Univ., Moscow, 1989, 143148.
[1990] The structure of iterated function spaces in the topology of pointwise
convergence for Eberlein compacta (in Russian), Matem. Zametki,
47:3(1990), 9199.
[1992] On surlindelf compacta (in Russian), General Topology. Spaces,
Mappings and Functors, Moscow University P.H., Moscow, 1992,
132140.
SOKOLOV, G.A.
[1984] On some classes of compact spaces lying in -products, Comment.
Math. Univ. Carolinae, 25:2(1984), 219231.
[1986] On Lindelf spaces of continuous functions (in Russian), Mat.
Zametki, 39:6(1986), 887894.
[1993] Lindelf property and the iterated continuous function spaces,
Fundam. Math., 143(1993), 8795.
STEGALL, C.
[1975] The RadonNikodym property in conjugate Banach spaces, Trans.
Amer. Math. Soc., 206(1975), 213223.

Bibliography

709

[1987a] Applications of the Descriptive Topology in Functional Analysis, Linz


Univ., 19851987.
[1987b] Topological spaces with dense subspaces that are homeomorphic to
complete metric spaces and the classification of C.K/ Banach spaces,
Mathematika, 34(1987), 101107.
[1988a] Generalizations of a theorem of Namioka, Proc. Amer. Math. Soc.,
102(1988), 559564.
[1988b] The topology of certain spaces of measures, Univ. de Murcia, Notas
Mat.,3(1988).
STOKKE, R.
[1997] Closed ideals in C.X / and -algebras, Topology Proc., 22(1997),
501528.
STONE, A.H.
[1963] A note on paracompactness and normality of mapping spaces, Proc.
Amer. Math. Soc., 14(1963), 8183.
TALAGRAND, M.
[1975] Sur une conjeture de H.H. Corson, Bull. Sci. Math., 99:4(1975),
211212.
[1977] Espaces de Banach faiblement K-analytiques, Comptes Rendus de la
Academie de Sciences, Paris, 284:13(1977), 745748.
[1978] Comparaison des boreliens dun espace de Banach pour les topologies
fortes et faibles, Indiana Univ. Math. J., 27(1978), 10011004.
[1979a] Espaces de Banach faiblement K-analytiques, Annals Math.,
110:3(1979), 407438.
[1979b] Sur la K-analyticit des certains espaces doperadeurs, Israel J.
Math., 32(1979), 124130.
[1979c] Deux gnralisations dun thorme de I. Namioka, Pacific J. Math.,
81(1979), 239251.
[1980] Compacts de fonctions mesurables et filtres non mesurables, Studia
Math., 67(1980), 1343.
[1984] A new countably determined Banach space, Israel J. Math.,
47:1(1984), 7580.
[1985] Espaces de Baire et espaces de Namioka, Math. Ann., 270(1985),
159164.
TAMANO, K., TODORCEVIC, S.
[2005] Cosmic spaces which are not -spaces among function spaces with the
topology of pointwise convergence, Topology Appl. 146/147(2005),
611616.
TAMARIZMASCARA, A.
[1996] Countable product of function spaces having p-Frechet-Urysohn like
properties, Tsukuba J. Math. 20:2(1996), 291319.

710

Bibliography

[1998] -pseudocompactness in Cp -spaces, Topology Proc., 23(1998),


349362.
[2006] Continuous selections on spaces of continuous functions, Comment.
Math. Univ. Carolin. 47:4(2006), 641660.
TANI, T.
[1979] Perfectly finally compact spaces are hard, Math. Japonica, 24(1979),
323326.
[1986] On the tightness of Cp .X /, Memoirs Numazu College Technology,
21(1986), 217220.
TKACHENKO, M.G.
[1978] On the behaviour of cardinal invariants under the union of chains of
spaces (in Russian), Vestnik Mosk. Univ., Math., Mech., 33:4(1978),
5058.
[1979] On continuous images of dense subspaces of topological products (in
Russian), Uspehi Mat. Nauk, 34:6(1979), 199202.
[1981a] On a result of E. Michael and M.E. Rudin (in Russian), Vestnik Mosk.
Univ., Math., Mech., 36:5(1981), 4750.
[1981b] On one property of bicompacta (in Russian), Seminar on General
Topology, Moscow University P.H., Moscow, 1981, 149156.
[1982] On continuous images of dense subspaces of -products of compacta
(in Russian), Sibirsk. Math. J., 23:3(1982), 198207.
[1985] On continuous images of spaces of functions, Sibirsk. Mat. Zhurnal,
26:5(1985), 159167.
TKACHENKO, M.G., TKACHUK, V.V.
[2005] Dyadicity index and metrizability of compact continuous images of
function spaces, Topol. Appl., 149:(13)(2005), 243257.
TKACHUK, V.V.
[1983a] On a method of constructing examples of M -equivalent spaces (in
Russian), Uspehi Mat. Nauk, 38:6(1983), 127128.
[1983b] On cardinal functions of Souslin number type (in Russian), Dokl.
Akad. Nauk SSSR, 270:4(1983), 795798.
[1984a] On multiplicativity of some properties of function spaces with the
topology of pointwise convergence (in Russian), Vestnik Mosk. Univ.,
Math., Mech., 39:6(1984), 3638.
[1984b] Characterization of the Baire property in Cp .X / in terms of the properties of the space X (in Russian), Cardinal Invariants and Mappings
of Topological Spaces (in Russian), Izhevsk, 1984, 7677.
[1984c] On a supertopological cardinal invariant, Vestn. Mosk. Univ., Matem.,
Mech., 39:4(1984), 2629.
[1985a] Duality with respect to the Cp -functor and cardinal invariants of
Souslin number type (in Russian), Matem. Zametki, 37:3(1985),
441451.

Bibliography

711

[1985b] A characterization of the Baire property of Cp .X / by a property of X ,


(in Russian) Mappings and Extensions of Topological Spaces, Ustinov,
1985, 2127.
[1986a] The spaces Cp .X /: decomposition into a countable union of bounded
subspaces and completeness properties, Topology Appl., 22:3(1986),
241254.
[1986b] Approximation of RX with countable subsets of Cp .X / and calibers
of the space Cp .X /, Comment. Math. Univ. Carolinae, 27:2(1986),
267276.
[1987a] The smallest subring of the ring Cp .Cp .X // which contains X [ f1g is
dense in Cp .Cp .X // (in Russian), Vestnik Mosk. Univ., Math., Mech.,
42:1(1987), 2022.
[1987b] Spaces that are projective with respect to classes of mappings, Trans.
Moscow Math. Soc., 50(1987), 139156.
[1988] Calibers of spaces of functions and metrization problem for compact
subsets of Cp .X / (in Russian), Vestnik Mosk. Univ., Matem., Mech.,
43:3(1988), 2124.
[1991] Methods of the theory of cardinal invariants and the theory of mappings applied to the spaces of functions (in Russian), Sibirsk. Mat.
Zhurnal, 32:1(1991), 116130.
[1992] A note on splittable spaces, Comment. Math. Univ. Carolinae,
33:3(1992), 551555.
[1994] Decomposition of Cp .X / into a countable union of subspaces with
good properties implies good properties of Cp .X /, Trans.
Moscow Math. Soc., 55(1994), 239248.
[1995] What if Cp .X / is perfectly normal? Topology Appl., 65(1995), 5767.
[1997] Some non-multiplicative properties are l-invariant, Comment. Math.
Univ. Carolinae, 38:1(1997), 169175.
[1998] Mapping metric spaces and their products onto Cp .X /, New Zealand
J. Math., 27:1(1998), 113122.
[2000] Behaviour of the Lindelf -property in iterated function spaces,
Topology Appl., 107:3-4(2000), 297305.
[2001] Lindelf -property in Cp .X / together with countable spread of X
implies X is cosmic, New Zealand J. Math., 30(2001), 93101.
[2003] Properties of function spaces reflected by uniformly dense subspaces,
Topology Appl., 132(2003), 183193.
[2005a] A space Cp .X / is dominated by irrationals if and only if it is
K-analytic, Acta Math. Hungarica, 107:4(2005), 253265.
[2005b] Function spaces and d -separability, Quaestiones Math., 28:4(2005),
409424.
[2005c] A nice class extracted from Cp -theory, Comment. Math. Univ. Carolinae, 46:3(2005), 503513.
[2005d] Point-countable -bases in first countable and similar spaces,
Fundam. Math., 186(2005), 5569.

712

Bibliography

[2007a] Condensing function spaces into -products of real lines, Houston


Journ. Math., 33:1(2007), 209228.
[2007b] Twenty questions on metacompactness in function spaces, Open Problems in Topology II, ed. by E. Pearl, Elsevier B.V., Amsterdam, 2007,
595598.
[2007c] A selection of recent results and problems in Cp -theory, Topol. Appl.,
154:12(2007), 24652493.
[2009a] Monolithic spaces and D-spaces revisited, Topology Appl.,
156:4(2009), 840846.
[2009b] Condensations of Cp .X / onto  -compact spaces, Appl. Gen. Topology, 10:1(2009), 3948.
[2009c] Embeddings and frames of function spaces, J. Math. Anal. Appl.,
357(2009), 237243.
[2010] Lindelf -spaces: an omnipresent class, RACSAM, 104:2(2010),
221244.
[2011] Countably compact first countable subspaces of ordinals have the
Sokolov property, Quaestiones Math., 34:2(2011), 225234.
[2012] The Collins-Roscoe property and its applications in the theory of
function spaces, Topology Appl., 159:6(2012), 15291535.
[2013a] Some criteria for Cp .X / to be an L. !/-space, Rocky Mountain
J. Math., 43:1(2013), 373384.
[2013b] Lifting the Collins-Roscoe property by condensations, Topol. Proc.,
42(2013), 115.
[2015a] Discrete reflexivity in function spaces, Bull. Belgian Math. Soc.,
21:5(2014), 114.
[2015b] Lindelf P -spaces need not be Sokolov, Math. Slovaca, 65(2015), to
appear.
TKACHUK, V.V., SHAKHMATOV, D.B.
[1986] When the space Cp .X / is  -countably compact? (in Russian), Vestnik
Mosk. Univ., Math., Mech., 41:1(1986), 7072.
TKACHUK, V.V., YASCHENKO, I.V.
[2001] Almost closed sets and topologies they determine, Comment. Math.
Univ. Carolinae, 42:2(2001), 395405.
TODD, A.R.
[1991] Pseudocompact sets, absolutely Warner bounded sets and continuous
function spaces, Arch. Math., 56(1991), 474481.
TODORCEVIC, S.
[1989] Partition Problems in Topology, Contemporary Mathematics, American Mathematical Society, 84(1989). Providence, Rhode Island, 1989.
[1993] Some applications of S and L combinatorics, Ann. New York Acad.
Sci., 705(1993), 130167.

Bibliography

713

[1999] Compact sets of the first Baire class, J. Amer. Math. Soc., 12:4(1999),
11791212.
[2000] Chain-condition methods in topology, Topology Appl. 101:1(2000),
4582.
TROALLIC, J.P.
[1979] Espaces fonctionnels et thormes de Namioka, Bull. Soc. Math.
France, 107(1979), 127137.
[1981] Une approche nouvelle de la presque-priodicit faible, Semigr.
Forum, 22(1981), 247255.
[1990] Quasi-continuit, continuit separe et topologie extremale, Proc.
Amer. Math. Soc., 110(1990), 819827.
TULCEA, A.I.
[1974] On pointwise convergence, compactness and equicontinuity, Adv.
Math., 12(1974), 171177.
TYCHONOFF A.N.
[1935] ber einer Funktionenraum, Math. Ann., 111(1935), 762766.
USPENSKIJ, V.V.
[1978] On embeddings in functional spaces (in Russian), Dokl. AN SSSR,
242:3(1978), 545546.
[1982a] On frequency spectrum of functional spaces (in Russian), Vestnik
Mosk. Univ., Math., Mech., 37:1(1982), 3135.
[1982b] A characterization of compactness in terms of the uniform structure in
function space (in Russian), Uspehi Mat. Nauk, 37:4(1982), 183184.
[1983a] On the topology of free locally convex space (in Russian), Doklady AN
SSSR, 270:6(1983), 13341337.
[1983b] A characterization of realcompactness in terms of the topology of
pointwise convergence on the function space, Comment. Math. Univ.
Carolinae, 24:1(1983), 121126.
VALOV, V.M.
[1986] Some properties of Cp .X /, Comment. Math. Univ. Carolinae,
27:4(1986), 665672.
[1991a] Linear topological classification of certain function spaces, Trans.
Amer. Math. Soc., 327:2(1991), 583600.
[1991b] Linear topological classifications of certain function spaces, Trans.
Amer. Math. Soc. 327:2(1991), 583600.
[1993] Linear topological classifications of certain function spaces II,
Mathem. Pannon. 4:1(1993), 137144.
[1997a] Function spaces, Topology Appl. 81:1(1997), 122.
[1997b] Spaces of bounded functions with the compact-open topology, Bull.
Polish Acad. Sci. Math. 45:2(1997), 171179.
[1999] Spaces of bounded functions, Houston J. Math. 25:3(1999), 501521.

714

Bibliography

VALOV, V., VUMA, D,


[1996] Lindelf degree and function spaces, Papers in honour of Bernhard
Banaschewski, Kluwer Acad. Publ., Dordrecht, 2000, 475483.
[1998] Function spaces and Dieudonn completeness, Quaest. Math. 21:
3-4(1998), 303309.
VAAK, L.
[1981] On one generalization of weakly compactly generated Banach spaces,
Studia Mathematica, 70:1(1981), 1119.
VEGA, R., KUNEN, K.
[2004] A compact homogeneous S -space, Topology Appl., 136(2004), 123
127.

DE LA

VELICHKO, N.V.
[1981] On weak topology of spaces of continuous functions (in Russian),
Matematich. Zametki, 30:5(1981), 703712.
[1982] Regarding the theory of spaces of continuous functions (in Russian),
Uspehi Matem. Nauk, 37:4(1982), 149150.
[1985] Networks in spaces of mappings (in Russian), Mappings and Extensions of Topological Spaces, Udmurtia University P.H., Ustinov, 1985,
36.
[1993] On the theory of spaces of continuous functions, Proc. Stekl. Inst.
Math. 193:3(1993), 5763.
[1995a] C.X / in the weak topology, Georgian Math. J. 2:6(1995), 653657.
[1995b] On normality in function spaces, Math. Notes, 56:5-6(1995),
11161124.
[1996] On the sequential completeness of Cp .X / (in Russian), Proc. Institute
Math. Mech. 4(1996), 118123.
[1998a] The Lindelf property is l-invariant, Topology Appl., 89(1998),
277283.
[1998b] On the sequential completeness of C .X / (in Russian), Fund. Prikl.
Mat. 4:1(1998), 3947.
[1998c] On the family of -topologies in a space of functions, Siber. Math. J.,
39:3(1998), 422430.
[2000] On spaces C .X /, Topology Appl. 107:1-2(2000), 191195.
[2001] On subspaces of functional spaces, Proc. Steklov Inst. Math. 2(2001),
234240.
[2002] Remarks on Cp -theory, Proc. Steklov Inst. Math., 2(2002), 190192.
[2004] Cardinal invariants of -topologies, Siberian Math. J., 45:2(2004),
241247.
VIDOSSICH, G.
[1969a] On topological spaces whose function space is of second category,
Invent. Math., 8:2(1969), 111113.

Bibliography

715

[1969b] A remark on the density character of function spaces, Proc. Amer.


Math. Soc., 22(1969), 618619.
[1970] Characterizing separability of function spaces, Invent. Math.,
10:3(1970), 205208.
[1971] On a theorem of Corson and Lindenstrauss on Lindelf function
spaces, Israel J. Math., 9(1971), 271278.
[1972] On compactness in function spaces, Proc. Amer. Math. Soc., 33(1972),
594598.
WAGE, M.L.
[1980] Weakly compact subsets of Banach spaces, Surveys in General Topology, New York, Academic Press, 1980, 479494.
WARNER, S.
[1959] The topology of compact convergence on continuous function spaces,
Duke Math. J., 25:2(1959), 265282.
WHITE, H.E., JR.
[1978] First countable spaces that have special pseudo-bases, Canadian
Mathem. Bull., 21:1(1978), 103112.
WILDE, M. DE
[1974] Pointwise compactness in space of functions and R.C. James theorem,
Math. Ann., 208(1974), 3347.
YAKOVLEV, N.N.
[1980] On bicompacta in -products and related spaces, Comment. Math.
Univ. Carolinae, 21:2(1980), 263283.
YASCHENKO, I.V.
[1989] Baire functions as restrictions of continuous ones (in Russian), Vestnik
Mosk. Univ., Math., Mech., 44:6(1989), 8082.
[1991] On rings of Baire functions (in Russian), Vestnik Mosk. Univ., Math.,
46:2(1991), 88.
[1992a] On the extent of function spaces, Comptes Rendus Acad. Bulg. Sci.,
45:1(1992), 910.
[1992b] Embeddings in Rn and in R! and splittability, Vestnik Mosk. Univ.,
Math., Mech., 47:2(1992), 107.
[1992c] On fixed points of mappings of topological spaces, Vestnik Mosk.
Univ., Math., Mech., 47:5(1992), 93.
[1992d] Cardinality of discrete families of open sets and one-to-one continuous
mappings, Questions and Answers in General Topology, 2(1992),
2426.
[1994] On the monotone normality of functional spaces, Moscow University
Math. Bull., 49:3(1994), 6263.

716

Bibliography

YOSHIOKA, I.
[1980] Note on topologies for function spaces, Math. Japonica, 25(1980),
373377.
YOUNG, N.J.
[1973] Compactness in function spaces; another proof of a theorem of
J.D. Pryce, J. London Math. Soc., 6(1973), 739740.
ZENOR, PH.
[1980] Hereditary m-separability and hereditary m-Lindelf property in
product spaces and function spaces, Fund. Math., 106(1980),
175180.

List of special symbols

For every symbol of this list we refer the reader to a place where it was defined.
There could be many such places but we only mention one here. Note that a symbol
is often defined in one of the previous volumes of this book. The first volume entitled
Topology and Function Spaces is denoted by TFS. The second volume has the title
Special Features of Function Spaces; it is denoted by SFFS, and the third volume
whose title is Compactness in Function Spaces is referred to as CFS. We never
use page numbers; instead, we have the following types of references:
(a) A reference to an introductory part of a section.
For example,
A 1.1
says that A is defined in the Introductory Part of Section 1.1 of this volume.
Of course,
Cp .X / TFS-1.1
shows that Cp .X / is defined in the Introductory Part of Section 1.1 of the book TFS.
Analogously,
K SFFS-1.4
says that K was defined in the Introductory Part of Section 1.4 of the book SFFS
and
Cp;n .X / CFS-1.2
means that Cp;n .X / was defined in the Introductory Part of Section 1.2 of the book
CFS.
(b) A reference to a problem.
For example,
h .y; L/ 432
says that h .y; L/ was defined in Problem 432 of this volume, while
Cu .X / TFS-084
indicates that the expression Cu .X / is defined in Problem 084 of the book TFS and,
naturally,

Springer International Publishing Switzerland 2016


V.V. Tkachuk, A Cp-Theory Problem Book, Problem Books in Mathematics,
DOI 10.1007/978-3-319-24385-6

717

718

List of special symbols

X A
CFS-090
shows that X A
was defined in Problem 090 of the book CFS.
(c) A reference to a solution.
For example,
O.f; K; "/ S.321
says that the definition of O.f; K; "/ can be found in the Solution of Problem 321
of the book TFS.
Analogously, we can infer from
n .Z/ T.019
that the definition of n .Z/ can be found in the Solution of Problem 019 of the
book SFFS.
From the expression
B1 .X / U.463
we can understand that the definition of B1 .X / can be found in the Solution of
Problem 463 of the book CFS and, finally,
.K/ V.300
says that the definition of .K/ can be found in the Solution of Problem 300 of this
volume.
Every problem is short; so it wont be difficult to find a reference in it. An
introductory part is never longer than two pages; so, hopefully, it is not hard to
find a reference in it either. Please, keep in mind that a solution of a problem can be
pretty long but its definitions are always given in the beginning.
The symbols are arranged in alphabetical order; this makes it easy to find the
expressions
B.x; r/ and X but it is not immediate what to do if we are looking
L
for t2T Xt . I hope that the placement of the expressions which start with Greek
letters or mathematical symbols is intuitive enough to be of help to the reader. Even
if it is not, then there are only three pages to plough through. The alphabetic order is
by line and not by column. For example, the first three lines contain symbols which
start with A or something similar and lines 35 are for the expressions beginning
with B, , or B.
A./ TFS-1.2
a.X / TFS-1.5
AD.X
/













TFS-1.4
.X / 1.4
V
W
A T.300
A U.031
AjY T.092
a0 ; : : : ; am
1.1
A 1.1
C 1.5
1.5
1.5
B.x; r/ TFS-1.3
Bd .x; r/ TFS-1.3
B1 .X / U.463
(B1)(B2) TFS-006
X TFS-1.3
B.X / SFFS-1.4
cl .A/ TFS-1.1
clX .A/ TFS-1.1
C.X / TFS-1.1
C  .X / TFS-1.1
C.X; Y / TFS-1.1
Cp .X; Y / TFS-1.1
Cu .X / TFS-084
Ck .X / 1.3
Cp .X / TFS-1.1
Cp .X / TFS-1.1

List of special symbols

719

Cp .Y jX / TFS-1.5
Cp;n .X / CFS-1.2
c.X / TFS-1.2
.A; X / TFS-1.2
.X / TFS-1.2
D./ TFS-1.2
dom.f / SFFS-1.4
dim X 1.2
X TFS-1.2
n .Z/ T.019
t2T ft TFS-1.5
} SFFS-1.1
exp X TFS-1.1
f < g SFFS-1.1
fn !
!f TFS-1.1
jjf jj CFS-346
F TFS-1.3
'  .X / for a card. inv. ' SFFS-1.1
G TFS-1.3
G TFS-1.5
h'.X / for a card. inv. ' 2.1
h# .U / S.228
Int.A/ TFS-1.1
i w.X / TFS-1.2
J./ TFS-1.5
K SFFS-1.4
L.X / 1.3
l.X / TFS-1.2
lim S for S D fAn W n 2 !gSFFS-1.5
lim S 1.2

Cb .X /
conv.A/
CH
.x; X /
A
d.X /
diam.A/
D
.X /
ij
n .Z/
F
}C
ext .X /
f g
f jY
Fin.A/
F
F
G
GCH
h .y; L/
HFD
IntX .A/
I
J
K
Lp .X /
L./
(LB1)(LB3)
L0 ; L

1.3
SFFS-1.2
SFFS-1.1
TFS-1.2
CFS-1.2
TFS-1.2
TFS-1.3
SFFS-1.4
TFS-1.2
T.019
TFS-1.5
SFFS-1.1
TFS-1.2
SFFS-1.4
TFS-022
S.326
TFS-1.3
SFFS-1.4
TFS-1.3
SFFS-1.1
432
T.040
TFS-1.1
TFS-1.1
U.093
SFFS-1.4
1.3
TFS-1.2
TFS-007
1.3

max.f; g/
(MS1)(MS3)
MA./
M./
nw.X /
O.f; K; "/
ord.U /
L
t2T Xt
p.X /Q Q

p
W
X
!
S
t
t2T
t2S Xt
Q
X









t
t2T
Q
t2T gt
.A; X /
.X /

nn.f; g/
m.X /
MA
M
N
O.f; x1 ; : : : ; xn ; "/
0;
L 0L
fXt W t 2 T g
(PO1)(PO3)

Q
p
W
X
! Xt
t
s
s2T
Q
fXt W t 2 T g
0 .X /
.x; X /
 .x; X /

TFS-1.1
TFS-1.5
SFFS-1.1
1.1
TFS-1.1
TFS-1.2
1.3
TFS-1.4
TFS-1.4
TFS-1.2
TFS-1.2
SFFS-1.4
TFS-1.2
TFS-1.4

TFS-1.1
TFS-1.3
SFFS-1.1
CFS-1.2
TFS-1.2
S.321
1.2
TFS-1.4
TFS-1.2
TFS-1.4
TFS-1.2
TFS-1.5
TFS-1.2
TFS-1.2

720

List of special symbols

w.X /
Y W Cp .X / ! Cp .Y /
P
P
q.X /
r  for r W X ! Y
s.X /
St.x; U /
hS i
.A/
 ./ Q
.X; x/ for x 2 t2T Xt
 ./
t0 .X /
t .X /
.X /
 .X /
X
(U1)(U2)
V ./
!
! <!
x1 ; : : : ; xn I O1 ; : : : ; On

X A

L

X Y
t
X Y
X

X

X 'Y
XT
.Z/

TFS-1.4
TFS-1.5
CFS-1.1
CFS-1.1
TFS-1.5
TFS-1.5
TFS-1.2
TFS-1.3
S.489
CFS-1.4
TFS-1.5
CFS-1.4
TFS-1.5
TFS-1.5
TFS-1.2
TFS-1.1
TFS-1.2
TFS-1.5
1.2
CFS-1.2
TFS-370
SFFS-1.4
TFS-1.1
CFS-090

 .X /
P
P
P
Q
R
SA
St.A; U /
0 .X /
 .A/
./ Q

 .X; x/ for x 2 t2T Xt
 .A/
tm .X /
.x; X /
.A; X /
.d /
S
ffi W i 2 I g
U1 ^ : : : ^ U n
vet .X /  
w.X /
!
.K/
X1  : : :  X n

X Y
u
X Y
X
<
X
<!
X
Y
X `P
.Z/!

1.3
1.1
1.1
1.1
1.1
TFS-1.2
S.493

TFS-1.4
SFFS-1.1
CFS-1.1
SFFS-1.4
TFS-1.1
TFS-1.1
SFFS-1.1
TFS-1.3
SFFS-1.4
CFS-1.4
TFS-1.5
CFS-1.4
CFS-1.4
TFS-1.5
TFS-1.1
TFS-1.1
TFS-1.3
SFFS-1.4
S.144
CFS-1.1
TFS-1.1
1.5
V.300
TFS-1.2

1.3
1.2
1.1
1.1
1.4
CFS-1.1
S.493

Index

A
Absolute Borel set of class , 22, 218, 219
Alexandroff double of a space, 34, 304, 305
Almost disjoint family of sets, 235,
340342
Almost lower semicontinuous map, 5, 9, 10,
95, 96, 100, 102104
Almost perfect map, 234, 236, 642
Analytic space, 8, 23, 31, 55, 90, 91, 225, 282,
521, 523, 524
@0 -space, 37, 4143, 376, 377384, 387, 397,
398, 655, 660

B
Baire property, 39, 40, 42, 43, 48, 77, 193, 323,
330, 331, 343345, 359, 360, 362, 389,
393, 394, 396, 412, 413, 471, 596
BanachMazur game, 3738, 42, 387, 389
Banach space, 37, 46, 425, 427, 428,
431, 617
Barrel, 40, 41, 359363
Barrelled space, 26, 40, 340, 359, 361363
Barycenter, 5, 11, 122, 125, 126
Barycentric subdivision
coordinates, 5, 11, 122124, 126, 127, 132
b-continuous function, 26
Borel set, 5, 22, 24, 61, 218220, 221, 238,
517, 519521
Browers fixed-point theorem, 11, 132
bf -space, 26, 40, 41, 348350, 354, 355,
363365, 367, 372
Bunch of spaces, 33, 295

C
Caliber, 6, 30, 31, 50, 57, 65, 276, 282, 365,
445, 447, 452, 497, 502, 508, 562,
563
Canonical homeomorphism, 618
Cantor cube
set, 490
Cardinal invariant, 4, 25, 637, 643
Category of a set in a space, 388, 425
Cauchy filter, 14, 16, 162164, 165, 349, 375

Cech-complete
space, 2, 11, 35, 36, 39, 43, 44,
62, 121, 122, 222, 300, 317, 318, 322,
323, 325, 327, 328, 345, 375, 396, 400,
402, 490, 645, 646

CechStone
compactification, 639
C*-embedded subspace, 19, 20, 81, 181, 182,
184, 651
C-embedded subspace, 66, 81, 82, 182, 235,
271, 284, 285, 315, 572, 653
Centered family, 162, 163, 170, 204, 207,
359, 412
Character at a point
of a set, 500
of a space, 310, 312
Choquet game, 47, 597599, 600, 605
Closed Graph theorem
discrete set, 427
map, 46, 427
Cofinal set, 21, 206
Collectionwise normality, 17, 39, 58, 167, 168,
221, 331, 332, 334336, 390, 398, 458,
566, 567
Compact-covering map, 380382

Springer International Publishing Switzerland 2016


V.V. Tkachuk, A Cp-Theory Problem Book, Problem Books in Mathematics,
DOI 10.1007/978-3-319-24385-6

721

722
Compactification, 34, 37, 240, 241, 284, 304,
326, 328, 329, 338, 340, 436, 516, 586,
588, 596, 614, 615, 639, 641
Compact-open topology, 26
Compact-valued map, 10, 107
Complete class of spaces
family of covers, 4
uniformity, 17, 18
uniform space, 90
Completely metrizable space. See also Banach
space
normable space, 26, 38, 259, 263
Completely regular space, 25, 32, 290, 293
Completion of a uniform space, 170
Component of a point in a space, 319, 645
Concentrated set, 47, 50, 448, 450, 451
Condensation, 35, 55, 64, 66, 87, 148, 200,
205, 272, 277, 279, 285, 296, 319, 321,
322, 326, 368, 381, 412, 413, 431, 481,
484486, 487, 489, 519, 521, 523525,
526, 603, 647, 651, 658
Connected space, 304, 319, 412, 638, 645
Consistency with ZFC, 650, 651, 663
Continuous function, 11, 39, 40, 42, 70, 72, 81,
84, 94, 116, 120, 122, 132, 134136,
138, 141143, 190, 191, 193, 199, 201,
213, 214, 230, 242, 244, 274, 278,
283285, 289, 291, 292, 294, 303, 308,
332, 333, 336, 339, 347, 351, 355, 357,
358, 360, 374, 384, 386, 387, 395, 405,
419, 420, 429, 431, 473475, 477, 480,
511, 519, 521523, 541, 569, 580, 591,
627629, 630, 644
Continuous linear functional, 2931, 39, 46,
83, 140, 147, 265269, 272278, 279,
282288, 307, 314, 330, 344, 347, 358,
360, 361, 398, 418, 429, 431, 608, 633
Continuous map, 4, 1012, 14, 15, 17, 18, 20,
21, 3032, 39, 41, 48, 52, 53, 6971,
75, 92, 108, 110, 112, 134136, 140,
142145, 147, 154, 155, 165, 168, 172,
174, 190, 191, 196, 199, 200, 204, 205,
208, 215, 217, 229, 234, 244, 274, 275,
278, 295, 298, 299, 302, 307, 308, 310,
322, 331, 336, 338, 345347, 353, 357,
363, 365, 369, 375, 386, 418, 420, 428,
429, 493, 496, 500, 508, 512, 514, 515,
523, 572, 582, 642
Continuum Hypothesis (CH), 50, 52, 55,
107, 445, 447, 464, 490492, 526,
639, 663
Convex hull
set, 252, 259, 263, 266, 357

Index
Corson compact space, 51, 55, 57, 62, 452,
481, 525, 526, 550, 553, 562, 575,
651, 660
Cosmic space, 53, 58, 138, 454, 493, 500,
508510, 568, 645
Countable sum theorem for covering
dimension, 19, 184, 421, 622
Countably compact space
paracompact space, 135, 142
Covering dimension (dim), 2, 61, 202
Cozero set, 141, 142, 177, 291, 292, 296,
297, 333, 334, 339, 634. See also
Functionally open set
Crowded space. See Dense-in-itself, space

D
Dense-in-itself, space, 36, 304, 323325, 345,
471, 472
Dense subspace, 17, 30, 52, 53, 165, 166, 168,
183, 194, 276, 323, 337, 346, 400, 445,
473, 485, 490, 492, 493, 495, 497, 500,
502, 505, 507509, 512, 519, 521, 550,
604, 639, 650
Density degree, 2, 6, 61, 64, 100, 103, 192,
280, 434, 437, 438, 441, 445, 448, 466,
491, 554, 577, 579, 650, 661
Diagonal of a space
number, 51
Diameter of a set, 520, 593, 627
Dieudonn complete space, 18, 31, 173,
283285, 646
Dimension. See Covering dimension
Directed set, 14, 21, 204
Disconnected space, 304
Discrete family of sets
open expansion, 37, 78, 315, 390
Discrete space, 33, 34, 54, 55, 6567, 84, 88,
272, 300, 301, 304, 305, 309, 312, 313,
323, 332, 335337, 338, 365, 406, 429,
436, 453, 464, 477, 479, 490, 492, 513,
514, 528, 530, 532, 588, 639
Discrete subspace
topology, 490, 562
union, 239
Double arrow space, 54, 516, 517
d -separable space, 52, 467, 468, 473, 489, 647
D-separating family of functions, 94
D-space, 47, 55, 5759, 533, 534, 548550,
552, 566, 568, 577, 579581
Dual map
of a linear topological space, 94, 364
Dyadicity index, 47

Index
E
Eberlein compact, 59, 88, 338, 584586, 650,
651, 659
"-essential point, 9, 10, 101103
"-inessential point, 9, 101
Embedding, 5, 7, 9, 10, 14, 25, 42, 88, 89, 94,
101103, 138, 200202, 215, 239, 290,
293, 295, 314, 346, 380, 382, 384, 385,
418, 420, 511, 532, 542, 581, 596, 603,
617620, 629, 630, 658
Equicontinuous family of maps
at a point, 28, 263, 264
"-small family, 59, 589
Essential subspace, 52, 472, 474
Extension of a mapping, 137, 144, 286
Extent, 2, 78, 338, 398, 567, 662
External base
network, 539
Extractor of a multi-valued map, 4
Extremal disconnectedness, 513, 514, 639,
642

F
Face of a product, 129
Face of a simplex, 5, 124, 128, 130
Filter, 162165, 170, 173, 221, 223227,
349, 531
Finite-dimensional space, 3, 30, 49, 60,
62, 271, 272, 287, 288, 622, 624,
630, 644
Finite intersection property, 385
Finite-to-one map, 621623, 645
Finite-valued map, 4, 5, 810, 95, 96, 100,
102104, 108, 110, 633
First countable space, 4143, 44, 50, 51, 54,
58, 263, 340, 341, 372, 384, 396, 398,
402, 403, 443, 445, 447, 448, 453, 454,
463, 514, 517, 577, 578, 581, 650, 653
Fixed point of a map, 154, 297
FrchetUrysohn fan
-fan, 33
space, 300
Free sequence
locally convex space, 25
union (see Discrete subspace)
F -set, 20, 23, 185, 213, 227, 367, 371,
422, 526
Functionally closed set. See Zero-set
Functionally open set. See also Cozero set
perfect space, 53, 508, 596, 655
separated sets, 178, 187

723
G
G -set, 11, 119121, 200, 219, 222, 313, 323,
327, 397, 464, 470, 471, 522, 523, 550,
552, 635
General countable sum theorem, 19, 184
Generating topology by a base
a closure operator, 13, 29, 202, 269, 580
a family of maps, 528
a linear order, 2, 14, 18, 2538, 45, 46,
62, 76, 77, 140, 147, 148, 173, 174,
249, 251, 253, 254, 258260, 261, 263,
265271, 273, 287, 294, 302, 313,
323329, 330, 356, 365, 374, 375,
416418, 419, 425, 603, 611, 630, 639,
644, 660
a metric, 26, 27, 38, 59, 202, 254, 259, 260,
307, 365, 366, 521, 526528, 589, 597,
600, 626
a norm, 26, 38, 259261, 262, 422,
427, 431
a subbase, 4, 25, 26, 87, 336, 554
a uniformity, 13, 14, 16, 159, 374
G -set, 51, 53
Graph of a map, 431, 604
GruenhageMa game, 37, 43, 391393, 394
Gulko compact space, 88, 663
Gulko family, 486, 487, 597, 601, 602, 605
GulkoSokolov game, 597, 601, 602, 605
Gulko space, 486, 597, 663

H
HahnBanach theorem, 28, 264
Hamel basis, 26, 29, 30, 39, 83, 84, 271, 272,
274, 279, 287, 288, 314, 330, 343, 346,
347, 355, 357, 369, 370, 397, 398
Hausdorff space, 21, 203, 207, 290, 447, 554
Hedgehog space, 367
Hemicompact separable space, 37, 41,
372374, 376, 654, 655
Hemicompact space, 41, 373, 374, 654
Hereditarily Baire space
d -separable space, 48
metalindelf space, 55, 526, 527
Hereditarily normal space
paracompact space, 12, 135
Hereditary density
Lindelf number, 2, 61, 100
-character, 661
HewittNachbin number, 81
Hewitt realcompactification, 284, 315,
516, 526

724
Homeomorphic spaces, 162
Homeomorphism, 162, 6875, 81, 82, 86,
133, 136, 146, 246248, 252, 269, 270,
273279, 288293, 296299, 302304,
307, 308, 322, 326, 328, 343, 345348,
363365, 375, 384387, 396, 413415,
436, 473, 484, 485, 513, 608, 616620,
622, 639, 642, 652
Homotopic equivalence, 147
Homotopic maps, 12, 135, 144
Homotopy, 4, 644
Hurewicz space, 6, 67, 653

I
Identity map, 137, 155, 166, 273, 274, 308,
310, 337, 347, 357, 369, 375, 427, 430,
431, 562, 580
Independent set of points in Rn , 5, 11, 122
Induced topology, 5, 25, 26, 29, 30, 40, 42,
88, 89, 269, 272, 296, 318, 321, 351,
353, 354, 357, 369, 386, 464, 527,
528
Invariance under an operation, 569
Inverse sequence
system, 14, 205
Inverse sequence
system, 14, 205
Irrationals, 5, 36, 192, 317, 327
Irreducible map, 513, 585, 642
Isometric spaces, 400, 620
Isomorphism of linearly ordered sets, 18, 29,
174, 193, 239, 270, 275, 330, 343, 347,
369, 371, 386, 611
i -weight, 6, 64

J
JNR property, 59, 589, 591, 593, 595

K
-analytic space, 8, 31, 90, 282
-cosmic space, 53, 508510
-monolithic space, 5557, 535537, 539, 544,
549, 562
Kowalsky hedgehog, 367
k-space, 41, 340, 373, 374, 376, 379, 642,
654, 655

Index
L
l-bounded set, 28, 263
Left-separated space, 97
l-embedded set, 656
L-equivalence, 40, 348
l-equivalence, 2, 3, 2562, 293, 300, 302, 305,
315, 336, 340, 348, 373, 416, 656658
Lexicographic order, 516, 611, 618
Limit of a sequence
of an inverse system, 21, 203, 204, 207
of a family of sets, 50, 448, 451, 452, 522
Lindelfnumber (degree), 2, 3, 61, 62, 78, 100,
101, 104, 650
Lindelf space, 22, 54, 208, 218, 421, 457,
465, 485, 511, 515, 529, 661
Lindelf -space, 8, 21, 31, 50, 52, 5355,
57, 59, 88, 90, 208, 282, 443, 444, 482,
485487, 489, 509, 511513, 516, 526,
533, 543, 544, 548, 549, 581584, 647,
659
Linear functional
homeomorphism, 1, 30, 3942, 149,
275279, 284, 308, 364, 386
map, 29, 30, 265, 270, 275, 286, 288,
307308, 364
(topological) space, 2, 14, 18, 2538, 45,
46, 62, 76, 77, 122, 140, 147, 148,
173, 174, 249, 251, 253254, 258263,
265273, 327330, 416, 417419,
425427, 644
span (hull) of a set in a linear space, 194,
270, 271, 284, 288, 315, 369, 372
uniformity, 14, 18, 40, 173176, 348, 349,
363, 374375
Linearly bounded set in a linear topological
space. See also l-bounded set
Cauchy sequence, 373, 374, 376
homeomorphic linear topological spaces,
2, 14, 18, 2537, 45, 294, 302, 313,
323329, 416, 417, 419
Linearly ordered space, 239, 611
l-invariant property, 35, 318, 652, 655
Local base
dimension, 15
Locally Cech-complete space
compact space, 33, 42, 43, 300, 340, 390,
396, 403, 521
convex space, 25, 2730, 39, 40, 254,
258, 263, 267269, 271272, 274275,
346347, 359, 386, 418, 419, 644
finite family, 227, 399, 400

Index
Lower semicontinuousmap, 9, 10, 95, 96, 100,
103, 112, 314, 318, 631633
Lower semicontinuous with respect to a map,
5, 8, 9, 95102
Luzins Axiom, 10, 54, 106, 107, 513515,
525, 650, 651
Luzin space, 492

M
-approximation, 367369, 372, 397
Martins Axiom (MA), 10, 11, 106, 107, 110,
111
Maximal almost disjoint family, 340342
Metalindelf space, 55, 526529, 663
Metric space, 17, 111, 112, 117, 161, 162,
164, 166, 169, 173, 200, 212, 218223,
227, 233, 236238, 259, 262, 349, 367,
400402, 527
Metrizable space, 2, 5, 11, 16, 20, 2224,
3746, 52, 62, 111, 112, 116, 143,
161, 166, 173, 218220, 225, 230239,
285, 323324, 365369, 372, 396398,
400402, 406, 451452, 481, 597, 642,
655657
Minkowski functional, 26, 256, 257, 260, 266
Monotonically -monolithic space
k-stable space, 48, 56, 540, 542, 543
monolithic space, 5557, 535537, 539,
544, 549, 562
retractable space, 58, 566, 568, 573576
Sokolov space, 57, 58, 563, 574, 576
Moving off collection
property, 37, 43, 390, 391
-space, 35, 37, 40, 315, 316, 352, 361363,
373, 376, 396, 621, 623
Multiplicative class of absolute Borel sets, 22,
24, 218, 221, 238
Multi-valuedmap, 4
Mushroomlemma, 12, 135137, 143, 144

N
Natural projection, 14, 15, 73, 74, 86, 105,
107, 108, 154156, 162, 164, 204206,
284, 310, 343, 346, 377, 428, 433, 436,
453, 454, 464, 497, 500, 502, 510, 520,
524, 569, 622, 623, 630
Neighborhood assignment, 47, 533, 534, 544,
545, 547549, 567, 568, 580
Network, 37, 42, 48, 56, 59, 64, 369, 376378,
380387, 489, 505, 536, 537, 541544,
564568, 570573, 582, 590

725
Network weight
with respect to a cover, 6, 64, 91, 230, 326,
384, 416, 549
Normable linear topological space, 259
Normal space, 19, 20, 32, 44, 135, 141, 178,
179, 184186, 188190, 199, 221, 291,
336, 403, 404405, 428, 625
Normed linear space, 261
Norm in a linear space, 38, 261, 427
Nowhere dense set, 491
O
!-modification of a space, 463, 464, 465
Okunevs method of construction of pairs of
l-equivalent spaces, 62
One-point compactification, 34, 37, 240, 241,
304, 326, 328, 329, 338, 340, 436, 586,
588, 596, 614, 615, 641
Open mapping, 29, 30, 56, 229, 262, 265, 276,
277, 288, 427, 500, 504, 539, 642
Open Mapping Theorem, 45, 425
Open-separated sets in a topological space,
334, 335, 624, 625
Order of a family of sets, 182184
Ordinal space, 22, 59, 145, 168, 248,
579580, 611
Outer base, 383, 496, 497, 551, 584, 585
P
Paracompact space, 12, 17, 43, 135, 142, 168,
322, 396, 646
Parallel retractions, 25, 292, 293
Partially ordered set, 264
Partition between two sets, 5, 12, 136, 137,
139, 142, 186191, 202
-base, 39, 50, 51, 345, 346, 388, 443454,
463, 467, 471, 482, 487489, 497499,
502, 503, 507, 508, 643
-character, 53, 505
Perfect image/preimage
map, 368
normal space, 367
Player in a game, 461, 462
Play of a game, 38
Point-countable family of sets, 448449
Point-finite cellularity, 6, 64
Point-open game, 47, 51, 455, 458461,
463, 467
Pointwise bounded subset of function space,
40, 355
Polish space, 35, 317
P -point, 47, 60, 465, 466, 596, 601604,
606, 640

726
Precaliber, 30, 276, 365, 508
Prime component of an ordinal, 612, 615, 619
Product space, 52, 427, 467
Pseudocharacter of space, 34, 305, 664
Pseudocompact space, 6, 32, 42, 57, 65, 73, 75,
177, 285, 390, 431, 434, 549, 562
Pseudocomplete space, 35, 66, 67, 318
Pseudometric, 13, 16, 156162
P -space, 6, 51, 66, 67, 463465, 466, 519521,
651
-weight, 39, 53, 336, 337, 508

Q
QS-algebra, 14, 21, 22, 208, 209, 215218
q-space, 37, 42, 43, 390, 393, 396
Quasi-component, 319, 321, 645
Quotient image
map, 379, 380
space of a linear space, 58, 343, 379, 380,
410, 574
topology, 25, 26, 28, 260, 261

R
Rational numbers (space), 81, 210
Realcompact space, 31, 40, 282, 359, 361
Real line, 18, 68, 175, 486, 487, 489, 575, 576,
639
Restrictionmap, 30, 40, 81, 82, 8587, 89, 110,
145, 148, 176, 215, 230, 246, 247, 272,
275, 276, 278, 293, 302, 308, 316, 348,
350, 354, 355, 362364, 405, 418, 428,
485, 512, 513, 515, 517, 519, 521, 524,
525, 541, 542, 544, 567, 581, 603, 620
Retract, 32, 33, 44, 45, 294296, 298301,
303, 307, 309, 312, 336338, 365, 402,
404, 417, 420, 425, 571, 572, 615, 619,
620, 642
Retraction, 25, 32, 33, 45, 48, 57, 58, 132134,
137, 144, 292, 293, 295300, 304, 312,
318, 321, 334, 336338, 405, 417419,
421, 424, 436, 563565, 566573,
575579, 582, 584, 603, 617, 618, 620
Right-separated space (this coincides with the
concept of scattered space), 99, 466
R-quotient map (or image)
topology (or space), 25, 286

S
-bounded space, 91, 252, 259, 260
Scattered space, 51, 455, 457459, 654

Index
-compact space, 6, 7, 31, 46, 55, 67, 90, 91,
94, 281, 282, 316, 322, 390, 525, 526,
651, 653
-discrete base, 368369
-discrete family, 48, 59, 589, 590
-discrete network, 3, 48, 59, 62, 368369,
589, 591, 593, 595, 596
-discrete space, 490, 492
Second category set
countable space, 20, 24, 35, 36, 39, 4143,
45, 50, 51, 54, 55, 60, 66, 78, 138, 141,
145, 196, 199203, 215, 217, 237, 240,
301, 317319, 322325, 327328, 336,
337, 377380, 382, 384, 385, 396, 398,
408, 415, 416, 424, 425, 445, 447, 448,
453455, 463, 467, 482, 515, 521, 523,
526, 624, 630, 641, 642, 645, 653, 664
Seminorm, 2526, 28, 29, 256260, 265, 268
Separable space, 52, 78, 430, 467, 468, 473,
489, 643, 647, 652, 656
metrizable space (in this book this is the
same as second countable space), 2,
56, 11, 16, 20, 2224, 35, 3746, 52,
62, 111, 112, 116, 143, 161, 166, 173,
186, 200, 201, 218220, 225, 227, 228,
230, 234, 236239, 284, 285, 301, 323,
365369, 372, 396398, 400, 402, 406,
407, 447, 451, 452, 481, 482, 521, 597,
599, 642, 650, 655657, 662
Separated sets in a space, 78, 98, 99, 334,
412
Separately continuous function, 42, 386, 387
Separating points by a family of maps, 559
Sequentially compact space, 105, 641
Sequential space, 10, 11, 104, 108, 110
Shrinking of a cover, 19, 178, 184, 189
Simple space, 484
Simplex, 5, 11, 122134
Simplicial subdivision, 5, 11, 123125, 128
-metacompact space, 458, 584585
-metrizable space, 41, 367, 369, 372,
397
Souslin number
property, 34, 43, 84, 313, 365, 398, 644
Space A./
!, 306
D./ (see Discrete space)
D  (see Cantor cube)
I , 67, 305
K (see Cantor set)
P (see Irrationals)
Q (see Rational numbers)
R (see real line)

Index
Space R , 199
 (!), 6, 7
V ./, 33
!1 , 12
!1C1 , 12
Sperners lemma, 11, 126
Sphere S n in Rn , 12
Spread, 2, 61, 100, 103, 650, 661
-product, 51, 55, 58, 453, 463, 467, 486, 490,
524, 568, 575, 576, 639
-space, 48
Star refinement, 167, 196198, 400, 402
-totally bounded uniform space, 18, 175,
176, 230
Strategy of a player in a topologicalgame,
37, 38
Stronger topology, 353, 384
Strongly -cosmic space
monotonically -monolithic space, 5557,
535539, 544, 547549, 562
monotonically monolithic space, 56, 57,
536, 538, 539, 548
zero-dimensional space, 8, 12, 19,
60, 9193, 121, 138, 139, 143,
183, 185, 192, 203, 327, 621, 622,
637638, 645
Strong -space, 533
Subbase of a uniformity, 13, 15, 17, 151, 152,
154, 172
Sub metrizable space, 41, 367, 663
Sum theorem for the dimension dim, 61
Support of a linear functional, 40
Swelling of a family of sets, 178, 179
Symmetric subset of a square of a set, 159,
160

T
t -equivalence, 2, 4, 6, 7, 1012, 18, 34, 61,
64, 65, 8688, 103, 110, 111, 121,
140, 177, 309, 312, 416, 652654,
660, 661
Thread (for an inverse system), 14
Tightness of a space, 53, 55, 310, 312, 340,
509, 526, 552, 643, 651
t -invariance, 61, 650, 658, 659
Topological game, 2
Totally bounded subset, 78
Totally bounded uniform space, 17, 169,
176, 230
T1 -separating family, 37
T0 -space, 27, 248
Two arrows space. See Double arrow
space

727
Tychonoff cube
space, 14, 1722, 25, 27, 31, 32, 159, 166,
167, 171173, 177, 179186, 196, 200,
205, 209, 214, 215, 228, 249251, 282,
290, 514, 596, 646
U
u-equivalence, 2, 1324, 40, 60, 61, 177, 348,
610, 652, 655
Ultrafilter, 16, 17, 47, 60, 162, 163, 169, 170,
596, 597, 601603, 608610, 640
Uniform convergence topology
homeomorphism, 26, 40
isomorphism, 154
Uniform product of uniform spaces, 13, 15
Uniformity
induced by a metric, 16, 17, 160, 161, 164,
166, 169
Uniformly continuous map
metrizable space, 161
pseudometric, 13, 16, 156, 158, 159, 161,
162
Uniform product
space, 13, 1517, 154, 155, 164, 166, 169,
170, 173, 175
Universal space
uniformity, 17, 18, 173
Upper semicontinuous map, 5, 10, 107110
V
Vietoris topology, 4, 23, 228231, 236, 237
W
Weaker topology, 31, 41, 285, 365, 366, 397,
475, 477, 480
Weak functional tightness
dual of a linear topological space, 313
topology of a linear topological space, 310
Weakly -point-finite family of sets, 487, 646
Weight, 6, 39, 53, 64, 91, 230, 301, 326, 384,
416, 465, 514, 515, 549
Winning strategy, 42, 43, 51, 387, 389,
392394, 455463, 552, 553, 575, 576,
597602, 605, 606
W -space, 186, 458, 459
Z
Zero-dimensional space, 8, 11, 12, 19, 22, 36,
60, 9193, 121, 137141, 143, 183,
185, 192, 195, 203, 218, 240, 326328,
448, 554, 584, 621624, 638, 645
Zero-set, 32, 142, 177, 291, 296, 297, 333,
334, 339, 403, 473, 474

También podría gustarte